Vous êtes sur la page 1sur 451

Physique 3

Ondes, optique et physique moderne


Traduction de •t-undamentats of Physics, Sixth Edition,
de David Halliday et coli., © 2001 John Wiley & Sons, Inc.
(ISB N 0-471 3323S-6; a 471-36037-6).
© 2004 Les Éditions de la Chenelière me.
Édition: Michel Poulin
Coordination Annie Fortier
Révision linguistique : Sylvain Archambault
Correction d'épreuves Renée Bédard et Lucie Lefebvre
Infographie: Alphatek
Couverture ' Michel Bérard
Maguetto intorioure: Lee Goldstein
Illustrations: Radlant/Precision Graphics
Catalogage avant publication
d e la B ibliothèque nationale du C anada
Halliday, David, 1916-

P*^ysique
Traduction de la 6" od. de ; Fundamentals of physics.
Comprend dos Index.
Sommaire ■ I Mécanique - 2. Électricité et magnétisme -
.3 Ondes, optique et physique morierne.
Pour tes étudiants du niveau collégial.
ISBN 2-89461-851-4 (vol. 1)
ISBN 2-89461-852-2 (vol. 2)
ISBN 2-89461-853-0 (vol. 3)
1 Physique. 2 Mécanique. 3 Flectricité 4 Magnétisme
5. Ondes. 6. Optique (ihysique. I Resnick, Robert, 1923-. Il
Walker, Jeari, 1945-. III. Carbonneau, Yves. 1964-, IV. Titre.
QC21 3.H3514 2003 530 C2003-940770-5

r
Chenelière McGraw-Hill
7001, boul. Saint-Laurent
Montréal (Québec)
Canada H2S 3E3
Téléphone (514) 273-1066
Télécopieur (514) 276-0324
McGraw-Hilt chcne@dlcmcgrawhill.ca

Tous droits réservés.

Toute reproduction, en tout ou en partie, sous quelque forme et par


r|iielt|ue prf)cédé que ce soil, est inlerriite sans l'autorisation écrite
prcaloblc de l'Éditeur.
ISBN 2 B9461 853 0
Dé) lût légal l ”'trimestre 2004
Bibliothèque nationale du Québec
Pitiiiothôqi le nationale du Canada

Imprimé au Canada

1 2 .3 4 5 ITIR 07 06 05 04 03

A certains endroits, le masculin a été utilisé dans le but d’alléger le


texte. La lectrice et te 'octeur verront a interpréter selon le contexte.

Noue reconnaissons l'aide financière du gouvernement du Canada


par l’entremise du Programme d’aide au d^eloppem ont de i’industne
de l'édition (PAUlfc) pour nos activités d’édition.

Gouvernement du O uéliec Program me de crédit d’impôt pour


l’Cdition de livres Gestion S O D E C

L Éditeur a fait tout ce qui était en son pouvoir pour retrouver les
fiopyrightfi O n peut lui signaler tout renseignement menant à la
correction d’erreurs ou d’omissions.
OPTIQUE ET PHYSIQUE MODERNE
David fialUday
liriiver uty o f FitVihurgh

Robert ResnicK
Rensselaer Potylechiiii frisiiiiiie

Jean walker
Cleveland Stare Universirv

Adaptation
Yves Carbon neau
( allege dc Rosemont

R e né Lafrance
Cégep de Saint-Hyadnlhe

Jean Parent
WkBBÊBÊÊÊÊÊi Collège de t'Ouiuouuis
№ SRV H BH R

avec ta coUaboiation de
André D e BeUefeuiUe
Collège Édouard Montpetit

Martin Saint-Michel
Collège de Sherbrooke

C la u d e Th ib a u d e a u
Collège de Sherbrooke

Traduit de l'anglais par


Michel Arsenault
Jean Blaquière

Chenelière/McGraw-Hiil
MONTRÉAL• TORONTO
iËCES DES PHOTOS
CHAPIlRt I CHAPITRES
Page 1. T Ctunpkui'Sygina. Page 21 : OraLicusetc de la NASA. Page 209: Jeffrey Zaruba/Tony Stone Images/New' York. Inc.
Page 210: Gracieuseté de Albert Einstein Archive.s, The .Icwish
CHAPIfRi 2
National & University Library, The Hebrcv\ University of Jerusalem.
Page 25 ; John Vi.vser/Rnice Coleman. Inc. Page 45 : Richard
Israël. Page 230: Gracieuseté de la NASA.
Megna/Fundamental Photographs. Page 46; Gracieuseté de
l.L>. Kossxng. Northem Illinois University. CHAPITRE 9
Page 245 : Lawrence Berkeley Lahoratory/Scieiice Photo Libnu-y/
CHAPITRE 3
Photo Researchers. Page 259 : Gracieuseté de A. lonomura. J. Undo.
Page 55: Stephen Daltoii/Animals Animais. Page 54; Howard
T. Matsuda et T. Kavva.saki/Advanced Research Laboratory. Hitachi.
SoeluiidT/The Stock Market. Pape 61 : Ben Rose/1 he Image Bank.
Ltd.. Kokubinju, Tokyo; H. E /aw a, Department of Physics,
Page 64 Boh Oruen/btar Pile. Page 65: John Ea.stcott/Vva
Ciakushuin University. Mejiro. Tokyo. Page 260 (A gauche):
Momativk/DKK Photo. Page 73 : U.S. Navy, photo de IVnseignc Gracieuseté cic Ribcr Division o f liistruinenis, Inc. Page 260
John Gray. (à droite) ; de PSSC film « Matter Waves », gracieuseté de Education
CHAPITRE4 Development Center, Newton, Massachusetts. Page 266 : © IBMKl^
Page 80 ; John Chuinack/Photo Reseatchens. Page 93 : PSSC Phvsici, Visuals Unlimited.
¿'édition: © 1975 17. Г Heath and <'о avec Htlucation Development CHAPITRE 10
renter. Newton, MA Reproduit avec l'autorisation de l’Education
Page 274. Gracieuseté de International Business Machines
Development Center. Page 95: Gracieuseté de Baiisch & l.onib.
Corporation, Almaden Re.search Center, CA, Page 285 : De ¡¡cientific
Page 9fi Hiirbara Filei/Tony Stone Jmages/New York. Inc. Page 98 :
American, janvier 1993, page 122. Reproduction autorisée par
Greg Pease/^Гопу Stone Imagcs/Ncw York. Inc. Page 101 G ra­
Michael Stcigcrwald, Bell Labs-lAicem Technologies. Page 286: Dc
cieuseté de i'ornell University.
Scientific American, septembre 1995, page 67. Reproduction
CHAPITRES autorisée par H. Teinkin, Texas Tech University. Page 290 Dc
Page 106 : ( iracienseté de Courtauld Institute Gallerie.s. Londres. W. Finkelnburg, Structure o f Matter. Springer-Vcriag. 19M
Page 115: Dr Paul A. '/ahl/Photo Researchers. Page 118: Gra Reprtxiuciion autorisée.
cieusetc de Matthew ,1. Wlicde.r. Page 124- Light and Vision. Conrad G. CHAPITRE 11
Mullet, Mae Rudolph et les éditeurs de Science Library, 1966 Times
Page .302 ; Larry Mulvehill/Photo Researchers. Page 303 : Gra­
Inc Page 139. Piergiorgio Scharandis/ Black Star.
cieuseté de Warren Nagourney. Page 312: David Job/Tony Stone
THAPlIRl 6 Imagcs/Nevv York, Inc. Page 322 ; Michael Roseiifeld/Tony Stone
Page 145- David Jiilian/Phototake. Page L50; Riink Schoenberger/ Images/New York. Inc.
Graiil Heilman Photography. Page 152; De \Wtla.\ o f Optical CHAPITRE 12
Phpnotnena par M. Cagnet et al.. Springer-Verlag. Prentice Hall. Page 332: Elscint/Scicnce Photo Library/Photo Researchers
1962. Page 163: Richard Megna/P'andamental Pliotographs. Page 349 (au bas); R. Perry/CYirbis Sygma Page 349 (en haut!:
Page 170: Gracieuseté de Bausch & Lomb. George RtKkw in/Bruce Coleman. Inc.
(HAPITRF? CHAPITRE 13
Page 174 : Georges Sourat, Lirt dimanche aprè.^-midi à l'île de lu Pages 362 ct 371 : Gracieuseté du U.S. Department of Energy.
Cnmdp Julie, 1884; huile sur canevas ( 107,5 X .708 cm). Helen Page 368 ; V. Ivclv,V.Magnum Photos. Inc. Page 372. Gary Sheehan,
Birch Bartlett Memorial Collection, 1926; photographie© 19%. The
Hirth o f du Atomic Age, 1957. (iraeicusetc dc Chicago Historical
Art Ingtiture ot Chicago. Tous droits ré.servé.s. Page. 17.5: Ken Society. Page 377; Giacieusclé de Anglo .Australian Telescope
Kay/Pundamcntal Photographs. Pages 176. 182. 183 et 186: De Board. Page 379 (en haut): Gracieuseté de Princeton University
l'Atlas o f Optical l'hcmmiam de C agncl. Francon. Thieri. Springer Physics L.aboratory. Page .379 (au centre); Gracieuseté de Los
Verlag, Berlin. 1962. Reproduction autorisée. Page 184: Warren Alamos .National Laboratory. Nouveau-.Mexique. Page 382 ; Gra
Rosenberg/ BPS/Toiiy Stone linages/ New York. Inc Page 190: cicusetc de Martin Marietta Energy Systems/U.S. Department o f
Depanincm o f Physics. Imperial College/ .Science Photo Library/
Energy.
Photo Researchers. Page IVl : Kristen Brochin.snn/Piindamcntal
Phourgraphs. Page 198: Diane bchiumo/l 'undamental Photographs. CHAPITRE 14
Page 3<M (à gauche) : Kjcll B. Sandvcd/Bnice Coleman. Inc Page 204 Page 385 ; Ciracie.uscté de la NA.SA. Page 387. David Parker' Photo
(à droite) Prkka Parviarten/Plioro Researchers. Page 208 : AP/Wide Researchers. Page, 388 : Gracieuseté de Michael Mathews. Page 389 ;
Wri ld P1k <os Gracieuseté de I.awrcnce Berkeley I..aboraiory.
h fv*
Li

P H Y S IQ U E 3

ONDES. OPTIQUE E T PHYSIQUE MODERN&r-^

Les oscillations 1
Pourquoi les ondes sismiques causèrent-elles beaucoup de dégâts
à Mexico et relativement peu le long de leur trajet ?
1.1 Les oscillations 2
1.2 Le mouvement harmonique simple 2
1.3 La deuxième loi de Newton appliquée à un mouvement
harmonique simple 5
1.4 L’énergie dans le mouvement harmonique simple 7
1.5 L'oscillateur harmonique simple angulaire 9
1.6 Les pendules 10
1.7 Le mouvement harmonique simple et le mouvement
circulaire uniforme 13
1.8 Le mouvement harmonique simple amorti 15
1.9 Les oscillations forcées et la résonance 17
Révision et résumé 18 Questions 18 Exercices et problèmes 20

Les ondes 25
Comment un scorpion fait-il pour localiser très précisément sa proie
sans la voir ni l ’entendre ?
21 Les ondes et les particules 26
2.2 Les types d ’ondes 26
2.3 Les ondes transversales et les ondes longitudinales 27
viii Table des matières

2.4 La longueur d ’onde et la fréquence 28


2.5 La vitesse d ’une onde sinusoïdale progressive 30
2.6 La vitesse d ’une onde dans une corde tendue 33
2.7 L’énergie et la puissance d ’une onde sinusoïdale
dans une corde 35
2.8 Le principe de superposition des ondes 37
29 L’interférence des ondes 38
2.10 Les vecteurs de Fresnel 41
2.11 Les ondes stationnaires 42
2.12 Les ondes stationnaires et la résonance 44
Révision et résumé 47 Questions 47 Exercices et problèmes 49

L e s oncles s o n o re s 53
Cotmnent le système de détection de la chauve-souris fonctionne-t-il,
et comment un papillon peut-il neutraliser ce système, ou le rendre
moins efficace ?
3.1 Les ondes sonores 54
3.2 La vitesse du son 54
3.3 Les ondes sonores progressives 57
34 L’interférence des ondes sonores 59
3.5 L’intensité sonore et le niveau sonore 61
3.6 Les ondes stationnaires dans les colonnes d ’air 65
3.7 Les txittements 67
3.8 L’effet Doppler 69
3 9 Les vitesses supersoniques et les ondes de choc 73
Révision et résumé 74 Questions 75 Exercices et problèmes 76

La réflexion et la réfraction de la lumière 80


Pourquoi la queue de poussière d'une comète est-elle courbée ?
4.1 Le spectre électromagnétique 81
42 L'aspect qualitatif des ondes électromagnétiques
progressives 82
4 3 L’aspect quantitatif des ondes électromagnétiques
progressives 85
4.4 Le transport de l’énergie et le vecteur de Poynting 88
4.5 La pression de radiation 90
4.6 La reflexion et la réfraction 93
4.7 La réflexion totale interne 97
Révision et résumé 98 Questions 99 Exercices et problèmes 100
Table aes matières ix

fr ‘»s'
Les miroirs et les lentilles 106
Quelles sont ces subtiles distorsions de let réalité qit’Édoiuird Manet
a intégrées dans son tableau Bar aux Folies-Bergère?
5.1 Définitions 107
5.2 Les miroirs plans 108
5.3 Les miroirs sphériques 110
5.4 Les images produites par les miroirs sphériques 111
5.5 Les dioptres sphériques 114
5.6 Les lentilles minces 116
5.7 L’œ il 123
5.8 Les instruments d ’optique 130
5.9 Trois démonstrations 135
Révision et résumé 137 Questions 139 Exercices et problèmes l'iO

_ _ ....
L'interférence de la lumière 145
Q u’a de particulier la surface extérieure de ra d e d'un papillon
dont les couleurs varient lorsqu 'on se déplace ?
6.1 L’interférence 146
6.2 La nature ondulatoire de la lumière 146
63 La diffraction 150
6.4 L’expérience de Young sur l'interférence 150
6.5 La cohérence 154
6.6 L’intensité de la figure d’interférence produite
par deux fentes 155
6.7 L’interférence produite par les pellicules minces 158
6.8 L'interféromètre de Michelson 154
Révision et résumé 165 Questions 166 Exercices et problèmes 167

La diffraction et la polarisation 174


Qu 'est-ce qui cause le changement de couleur, dans les tableaux
utilisant la technique du pointillisme, lorsqu’on s ’en éloigne ?
7.1 La diffraction et la théorie ondulatoire de la lumière 175
72 La diffraction produite par une seule fente : localiser
les minima 176
7.3 L’intensité lumineuse dans une figure de diffraction
produite par une seule fente ; approche qualitative 178
7.4 L’intensité lumineuse dans une figure de diffraction
produite par une fente simple : approche quantitative 180
7.5 La diffraction produite par une ouverture circulaire 182
7.6 La diffraction produite par deux fentes 185
7.7 Les réseaux de diffraction 187
7 8 Les réseaux ; le pouvoir dispersif et le pouvoir
de résolution 191
Table des matières

7.9 La diffraction des rayons X 193


7.10 La polarisation 195
7.11 La polarisation par réflexion 199
fïévlsion et résumé 200 Questions 201 Exercices et problèmes 202

La relativité restreinte 209


Conimeni se fait-il qu'une théorie aussi abstraite que la relativité restreinte
d ’Eirvitein soit nécessaire ii une chose aussi pratique que ki navigation ?
8.1 Ou’est-ce que la relativité? 210
82 Les postulats 211
8.3 Mesurer un événement 212
84 La relativité de la simultanéité 213
85 La relativité du temps 215
86 La relativité de la longueur 220
8.7 La transformation de Lorentz 222
8.8 Quelques conséquences de la transformation de Lorentz 224
8.9 La transformation relativiste des vitesses 226
8.10 L effet Doppler relativiste 227
8.11 La quantité de mouvement en relativité 230
8.12 L'énergie en relativité 231
8.13 Trois démonstrations 236
Révision et résumé 239 Questions 240 Exercices et problèmes 241

Les photons et les ondes de matière 245


Une particule peut-elle être une onde ?
91 Une nouvelle direction 246
9.2 Le photon, quantum de lumière 246
93 Le rayonnement du corps noir 248
94 L’effet photoélectrique 250
9.5 Leffet Compton 253
9.6 La lumière en tant qu'onde de probabilité 256
97 Les électrons et les ondes de matière 258
9.8 L'équation de Schrôdinger 261
9.9 Le principe d ’incertitude de Heisenberg 263
9.10 L effet tunnel 265
Révision et résumé 267 Questions 268 Exercices et problèmes 269

La mécanique ondulatoire 274


C om nent ces atomes ont-ils pu form er un cercle, et que sont
les ondulations piégées dans le corail ? *
10.1 La structure de l atome 275
10.2 Les ondes dans les cordes et les ondes de matière 275
10.3 Les énergies d'un électron piégé 276
Table des matières xi

10.4 Les fonctions d ’onde d ’un électron piégé 280


10.5 Un électron dans un puits fini 283
106 D’autres pièges à électron 285
10.7 Les pièges à électron bidimensionnels et tridimensionnels 286
10.8 L’atome d ’hydrogène 288
Révision et résumé 297 Questions 298 Exercices et problèmes 299

__________

Les atomes 302


Pourquoi la lumière d ’un laser est elle si différente de la lumière naturelle ?
11.1 Les atomes et le monde qui nous entoure 303
11 2 Quelques propriétés des atomes 303
11.3 Le spin de l’électron 305
114 Le moment cinétique et les moments dipolaires
magnétiques 306
115 L’expérience de Stern et Gerlach 309
116 La résonance magnétique 311
11.7 Le principe d ’exclusion de Pauli 313
11.8 Plusieurs électrons dans des pièges rectangulaires 313
11.9 La construction du tableau périodique 316
11.10 Les rayons X et la numérotation des éléments 317
11.11 Les lasers et la lumière d ’un laser 321
11.12 Le fonctionnement des lasers 322
Révision et résumé 325 Questions 326 Exercices et problèmes 327

La physique nucléaire 332


Qu ’orrive-t-il exactement aux noyaux radioactifs qui subissent
une désintégration, et que signifie le terme «■désintégration » ?
12.1 La découverte du noyau 333
12.2 Certaines propriétés nucléaires 335
12.3 La désintégration radioactive 340
124 La désintégration alpha 344
12 5 La désintégration bèta 346
126 La datation par radioactivité 349
12.7 Mesurer la dose de rayonnements ionisants 350
12.8 Les modèles nucléaires 351
Révision et résumé 353 Questions 354 Exercices et problèmes 355

L'énergie nucléaire 362


Quelles sont les notions de physique sous-jacentes et l'image
du champignon atomique qui a tant horrifié les gens 'è
131 L’atome et son noyau 363
132 La fission nucléaire : principe de base 363
13.3 Un modèle de fission nucléaire 366
13.4 Le réacteur nucléaire 368
xii Table des matières

13.5 Un réacteur nucléaire naturel 372


13.6 La fusion thermonucléaire: principe de base 374
13.7 La fusion thermonucléaire dans le Soleil
et dans les autres étoiles 376
13.8 La fusion thermonucléaire contrôlée 378
Révision et résumé 380 Questions 381 Exercices et problèmes 381

Les quarks, les leptons et le Big Bang 385


Comment une photographie représentant l'Univers âgé de seulement
379 (HH) uns a-t-elle pu être prise ‘7
141 La vie sur une corde raide 386
142 Une «m énagerie» de particules 386
14.3 Un interlude 389
14.4 Les leptons 392
14.5 Les hadrons 394
14.6 Un autre principe de conservation 395
14.7 La voie octuple 396
14.8 Le modèle des quarks 398
149 Les interactions fondamentales et les particules
messagères 400
1410 Une pause pour réfléchir 402
14.11 L’Univers est en expansion 403
14.12 Le rayonnement cosmique fossile 404
14.13 La matière sombre 404
14.14 Le 6ig Bang 405
14.15 Une récapitulation 406
Révision et résumé ^07 Questions 407 Exercices et problèmes 408

ANNEXES
A. Le système international d unités (Si) A-1
B. Constantes fondamentales de la physique A-3
C. Quelques données physiques A 4
D. Formules mathématiques A-6
F Propriétés des éléments A 9
F. Tableau périodique des éléments A-12

R-1

INDEX M
Cel ouvrage est la traduciitMi de la sixièm e édition de Fundci w w w .dlcm cgniw hill.ca/physiqiie Les élèves Irouveroiif aussi
tm n ta h o f Physics, un tex te d é so rm a is c lassiq u e d ’ab o rd su r ce site de nom breux élém cnis interactifs q ui les aide ront
rétligé par D avid H alliday et R obert Resnick, puis m aintenant à co m p re n d re le con ten u d e leiii.s c o u rs Mien q u ’ils soient
enrichi par Jearl Walker. pré.seiités uniquem ent en langue an glaise, ces clém en ts sont
A u fil des ans, cet o u v rag e s ’e.st tra n sfo rm é g râce aux fac iles à co n su lte r et d 'u n e g ran d e u tilité p o u r q u ic o n q u e
com m entaires et aux suggestions des enseignants et de leurs c h e rch e à v isu a lise r les n o tio n s p a rfo is ab stin itc s d e In
élèv es, et il est devenu ce q u ’il est a u jo u rd ’hui. F x am in o n s physique.
qiickiucs-uncs d e ses caractéristiques.
B onne étude !

Caractéristiques des chapitres


> D e s énig m es a u d éb u t des chapitres. C h a q u e ch a p itre Remerciements
com m ence par la description d 'u n e énigm e dont la solutioti se
L’adaptation française d ’un o u v rag e soulève inév itablem ent
tro u v e à l'in té rie u r du c h a p itre , ce qui .stim ule l ’in té rê t de
de nom breu.ses q u estio n s. L’ÎÎd iteu r tie n t donc ù re m e rcie r
l ’élève quant à la découverte de la m atière.
to u s ceux et celles qui nous o n t aid és à y ic p o n d rc en nous
> Vérifiez vos connaissances. R égulièrem ent, les élèves sont faisant p art de leurs com m entaires et de leurs suggestions.
invites à répondre à une question en utili.sant un raisonnem ent
fo n d é su r le te x te ou s u r l ’ex e m p le q u ’ils v ie n n en t de lire. G illes A rcham bault Jean I éonard
S ’ils en sont incapables, ils d ev raien t rev en ir su r la m atière C égep de Rim ouski C égep Saint Jérôm e
étudiée avant de poursuivre leur lecture. L es réponses à toutes
Louis Assclin G illes L esage
les .sections « V érifiez vos cotinais.sances » se trouvent à la fin
C égep H cauce-A ppalaches C égep de T A biiibl
du m anuel.
T ém iscam ingue
R ichaid B eaucham p
>-Le.v exem ples. C h a q u e c h a p itre c o n tie n t de nom breux
C ollège Bois-dc-B oulogne Sohcil M anoli
p roblèm es résolus qui ont été choisis pour aid er les élèv es à
C ham plain R egional C ollege
organiser logiquem ent les civnccpts de base présentés dans le A ndré B ordeleau
tex te et à développer leurs habiletés en m atière de résolution É cole de T echnologie B ru n o M ai tel
d e p ro b lèm es. C h a q u e pro b lèm e type est résolu étap e p ar Supérieure C égep de Saint-Félicien
étape à l ’aide d ’un ou de plusieurs concepts clés clairem ent
Jacques Bridet P ierre N oël de Tilly
énoncés.
C ollège 1.innel-G ronIx C égep du V ieux M ontreal
> R ésolution de problèm es. D ans cette rubrique, les élèves
R ichard Borland G illes Parizeau
qui com m encent l’étude de la phy sicjiie trouvercxil des directives
C ollège Jcan-dc-B rchcitf C egep de Saint-Jérôm e
utiles sur les façons de résoudre des problèm es et d ’éviter des
erreurs courantes. G eneviève C aron D onald Pelletier
C ollège M ontm oiency C ollège de V allcyticld
> Révision et résum é. C ette section constitue un som m aire
du contenu du chapitre. Elle présente les concepts essentiels C laude C ham pagne Y ves Pelletier
m ais ne peu! rem placer la lecture du chapitre. C égep de Trois-R ivières C ollège <k- l’O ulaounis

)►Questions. A l’instar des problèm es des sections « V érifiez M artin Charest l'Jm isc Provençal
vos connais.sanccs ». les questions exigent des élèves un elfo rt C ollège A huntsic C ollège G éiald-G odin
de raisonnem ent et de com préhension plutôt que des calculs,
M arc Filiatreault D aniel R(x;hettc
l es réponses sont fournies à la fin du m anuel.
C égep de G ranby C ollège François-X avier-
Exercices et problèmes. Sous cette rubricjuc. les exercices H aute-Y am aska O arneau
et les problèm es sont plus ou m oins ordonnés selon leur com ­
Y von F o itin G illes Sirois
p le x ité , e t ils sont reg ro u p és sous les titre s des sectio n s du
C ollège F iançois-X avier- C égep de R im ouski
texte. C eux m arqués d ’un astérisque f" ) représentent un défi
G arneau
p articu liè rem e n t ardu. L es rép o n se s à tous les ex e rcic es et M arlène Tonsignaiil
problèm es SC trouvent à la fin du m anuel. A ndré G irard C ollège 1 fançois-X avier-
C égep de Sainte-Foy G arneau
site Web. C ertains problèm es sont accom pagnés de la
m ention « vwvw» : elle indique que les élèves auront accès à Réjean l_aurier Jean-M arc V iens
des solutions détaillées de ces problèm es s ’ils visitent le site ilo llèg e Shavvmigan C'egep régional do 1 anandièro
1 Les oscillations

Le 19 septembre 1985, des ondes sismiques provenant d'un tremblement de terre sur la côte ouest du Mexique causèrent
d'importants dommages
dans la ville de Mexico,
pourtant située à environ
400 km de l'épicentre
du séisme.

Pourquoi les ondes sismiques


causèrent elles beaucoup
de dégâts à Mexico
et relativement peu le long
de leur trajet?
La répome % trouve dons ce chopilre.
2 Chapitre I Les oscillations

1.1 Les oscillations


Les gens sont entouras d 'o s c illa tio n s - des m ouvem ents sim ilaires qui se léiTètcnt. Il j
a dans les lustres suspendus, dans les bateaux ancrés qui se balanceni et dans les pisi
qui m ontent e t descendent dans les m oteurs des autom obiles. Il y a aiis.si des oscillati’
qui agitent les cordes de guiiaies. les instruments de percussion, les clcKhcs. les diapliragi
des léicpliones ei des haut-parleurs, et les cristaux de quart/, des m ontres. Il existe ai
des oscillations m oins apparentes, com m e celles des m olécules d 'a ir qui iransm ett
le son, celles d es atom es des solides qui iransm cilcnt la tcinpéraiurc, celles en fin i
élec tro n s circ u la n t d a n s les an te n n e s d e rad io e t d e té lév isio n qui iran sn iettem i
inldrm ations.
I a^s oscillations sont habiUiclIerncnt amorties, c'est-à-d ire q ue ra in p lilu d c de l
m ouvem ent diminue graduellem ent, et que leur énergie mécanique est convertie en ener
llicrmique sou.s l’effet des forces de frottement. M ême si on ne peut complètement élimi
les p e rte s d ’én e rg ie m é ca n iq u e, on peut les co m p en se r p ar une én erg ie proven;
d ’une autre source. P ar exem ple, vous savez q u ’en balançant vo.s Jam bes ou votre tor
vous pouvez accentuer le m ouvem ent de façon à m aintenir ou à augm enter les o.scillatio
Hn agissant ainsi, vous tran sfo rm ez votre énergie b io chim ique en én erg ie m écaniq
dans le systèm e oscillant.

1.2 Le mouvement harmonique simple


La figure 1.1 a) m o n tre une su c ce ssio n d ’« in sta n ta n é s » d 'u n sy stèm e o sc illa r
une particule a un m ouvem ent répété vers l’avant et l ’arrière, au tour d e l’origine d '
axe des x. D ans cette section, on .sc contentera d e décrire le nu Hivernent On v e n a par
suite com m ent on peut produire un tel m ouvem ent.
Tout m ouvem ent oscillatoire possède une propriété im portante, sa frequence, qui i
le nom bre d ’oscillations effectuées chaque seconde. Le sym bole de la fréquence est
cl l’imité SI qui la désigne est le hertz (dont J'abrévialion e.st Hz) :

1 hertz = 1 H z = 1 oscillation p ar seconde = 1 s (I-

On trouve associée à cette fréquence la période 7 du mouvcnient, qui est le tem ps reqii
pour effectuer une oscillation com plète (ou cycle) ; donc.

0 .2 )
^ r

T out m o u v em en t qui se ré p è te à in te rv a lle s ré g u lie rs se nom m e mouvcmci


périodique, ou mouvement harmonique. Le cas qui est étudié ici est celui d 'i
inouvcmeni qui se répète d’une façon particulière, com m e dans la figure L ia ) . Dans un t

Temps \t)

figure 1.1 ai Cette succession il’« insiantané« » (pris à des intervalles


de temps égaux) illustre la position il'unc particule qui wcille
vers l’avant et l’arrière p;ir rapport à roriginc d’un axe des ,v entre
les position-s extrêmes et —x„. Les fièches ont clé tracées
à l'échelle afin fl indiqiier le niixliile de la vitesse de la particule
L.e inoilule de la viies.se niaximal lorsque la particule se trouve
à l'origine, et est nul lorsque la particule sc trouve à
/- r .Si la particule sc trouve à f à / 0, clic rcUmine à + v„, à f • /.
oil / est la période du mouvement Le mouvement sc répète
ai
par la suite b) Le graphique de x en fonction du temps relativement
au mouvement illustré en a)
1.2 Le mouvement harmonique simple 3

Piwilioii m ouverncni, la p o sitio n x d e la p a rtic u le p a r rap p o rt à sa p o sitio n d ’é q u ilib re est


à riiisUmt / donnée par
P h ase - ,
\

Æ) ■V c o s (< ü / + 0 ) jc(f) = V cos((yr > 4’) (la position), (1.3)

Ainpiitiule fmijis '


où jc„, ù j c l 4 sont des constantes. Ce m ouvem ent se nom m e in o u v e in cn t liariiiü n iiiiie
riiiiiuii<.c CoiisiaiiU' s im p le , ce qui sig n ifie q u e le m o u v em en t p é rio d iq u e est une fo n ctio n sin u so ïd ale
angulaire tic pliast^
(111 ou angle du tem ps. L’éq u a tio n I J , où la fo n ctio n sin u so ïd ale e s t une fo n ctio n eo siu tis, est
pulsation (lejiliase représentée dans la figure 1 .1 bl. (Vous o h iie n d re / le m êm e gi'a|ibi(|iie eu tout liant la
figure 1.1 al de 90'' en sens antihoraire ei en reliant par une emirlie les l'Hisiiion.s su cccs“
Figure 1.2 Lne illiisirdlion pratique
sives de la particule.) Les quantités déterm inani la form e de ce graphique .sont indiquées
des quaiuitcs de l’iiquation 1.3,
par leurs nom s dans la figure 1.2. C.cs quantités sont défîm es ci-dessous.
(jui se rapportent au mouvement
harmonique simple La quantité nom m ée a m p litu d e du m onvem ent. est une constante positive dont
la valciii dépend de la façon dont le mouvemenl a com mencé. L ’indice m signifie inaximai,
p u isq u ’il rep rése n te le m o d u le du d ép lac em e n t m axim al de la p a rtic u le d an s l'u n e
ou l’autre direction pttr rapport à sa position d ’équilibre. La fonction cosinus de l'équû
tion 1.3 varie entre les valeurs 1. et la position r ( n varie entre les valeurs ± a„ autour
de sa position d ’équilibre.
L a q uantité (col + 4 ) de l ’équation 1.3 .se nom m e la p h a s e du m ouvem ent, et la
constante 4 se nom m e la co n sta n te d e p h ase (ou an g le d e phase). La valeur de 4 dépend
de la position et de la vitesse de la p articu le à |■ins^am t — 0. hn ce qui co n cern e les
valeurs x(t) représentées dans la figure 1.3 a), la constante de phase 4 l'sl égale à rérn.
P o u r in te rp ré te r la co n stan te a>, n o m m ée f r é q u e n c e a n g u la ir e du m o u v em en t
(ou p u ls a tio n ), on n o te d ’abord que la position r(/) doit reto u rn er à sa v aleu r initiale
ap rès u n e pério d e T du m o u v em en t, c 'e s t à-d ire qu e x(i) d o it être ég a l à x(t 3 T)
à tout instant l. P o u r sim p lifier cette analyse, on pose que ^ — 0 dan.s l'éq u a tio n 1 3.
E n fonction de cette équation, on peut m aintenant écrire :

A„ c o s COt = A,„ CCS [0)(t + T)]. (1.4)

L a fonction cosinus sc répète la prem ière fois lorsque son argum ent (la phase) a aug
m enté de 2-t rad. et l’équation 1.4 donne alors

eu(/ 4 T) — c»t f 2.T

ou wT — In .

D onc, d ’après l’équation I 2, la fri'quence angulaire est

2n
Cü = - I-Tf. (1.5)

a) b) — r

Figure 1.3 Hans chaque cas. la courbe bleue est obtenue à l'aide
de l'équation 1.3. oii 4 0. a) La couific rouge ne diffère de la courbe
bleue qitr pai une plus grande amplitude, .tj,, (les positions'extrêmes
dr la courbe rouge sont plus hautes et plus basses'), b) La coui-bc rouge
ne diffère de la ctxirbc bleue que par sa période. 7" = T/2 (la courbe
rouge est comprimée horizontalement), c) La courbe rouge ne diffèm
de la courbe bleue que par la valeur de 4 qui est de —,i/4 rad. au lieu
de zéro (la valeur négative de 4 décale la courbe rouge vers la droite).
4 Chapitre 1 Les oscillations

L 'u n ité S i d e la fré q u en c e an g u laire est le rad ian par se co n d e (p ar souci de


cohcrcncc, <f>doit donc s ’ex p rim er en radians). La figure 1.3 représente les \ aleurs de
xit) dans lieux m ouvem ents harm oniques sim ples qui different soil par leu r am plitude,
soit p ar leur période (donc daas leur fréquence et leur fréquence angulaire), soit par leur
eonsm nte de phase.

/VÉRIFIEZ VOS CONNAISSANCES! ; Une particule décrivant un mouvement harmonique


simple de période T (comitK' celui de la (Ipurc I I ) est à —.v„, à r = 0. Scra-t-ellc à —.Vm. +JC,„.
à 0. entre r„ ei 0 on entre 0 et à a) t = 2,(K)7i b) / = 3,507"et c) f = 5.257'.’

La vitesse d'une particule décrivant un mouvement


harmonique simple
En d é riv a n t p ar rap p o rt au tem p s l'é q u a iio n 1.3. on peut tro u v er l ’ex p re ssio n d e la
vitesse d ’une particule décrivant un m ouvem ent harm onique sim p le; cela donne
d x (t) d
Kri) = —;— ^ - r |x,„ cos(w / + <f>)\
dt dt

ou IV(/) = ~o)X^ sin(fu/ f <f>) (la vite.sse) ( 1.6)

La fig u re 1.4 a) est un e re p ré se n ta tio n g rap h iq u e d c l ’éq u a tio n 1.3, oii <i> = 0
La figure 1.4 b) représente l ’équation 1.6, avec la m êm e valeur dc 4* — 0 Par analogie
av e c l ’am p litu d e dc l'c q iia lio n 1.3. la q u an tité p o sitiv e i-it,,, d e l ’éq u a tio n 1.6
représente le module de la vitcs.se maximale Vp,. C om m e vous pouvez le voir dans 1a
figure 1.4 b), la vitesse d e la particule o.seillante varie entre les lim ites ± i„ , — ±o>x^.
N otez égalem ent, dans cette figure, que la courbe de i;(r) est décalée (vers la gauche)
d 'u n quart de péricide p ar rappoiT à la courbe de x{t) ; lorsque la position d e la particule
attein t sa v aleu r m a x im a le ( c ’e s t-à -d ire xf/) = jc„ ), le m o d u le d e la v itesse e st nul
(c ’esl-à-d trc r,(/) = 0). L orsque la po.sition d e la particu le est nulle, le m o d u le de la
vitesse atteint sa valeur m axim ale (donc, v„, — cdx^).

L’accélération d’une particule décrivant un mouvement


harmonique simple
En connaissant la vitesse v,(f) d 'u n m ouvem ent harm onique sim ple, on peut la dériver
par rap p o rt au tem p s p o u r d éterm in e r l’ex p re ssio n de l'a c c é lé ra tio n d 'u n e p articu le
FigurGl.4 a) La positum x(/) cTunc oscillante. A insi, on a, d ’après l’équation 1.6,
pjriiaile oscillant dans un mouvement d\^(t) d
harmonique sitnple, S an angle a j t ) = — — = — I -cu.Vn, sin(iL>f + <i>)l
di dt
tie phase th = 0 La jidritKle 7
rcprcscntc unc oscillation complete,
h) La Vitesse v,(r) dc ia particulc ou a^U) = — œaUot f A) (l’accélération). (1.7)
e) i.'aiTeldmtion (¡¿t) de In particulo
La fig u re 1 4 c) est une rep rése n tatio n g rap h iq u e de l’éq u atiim 1.7 loi.sque = 0.
La q u an tité p o sitiv e w-Xm d e l'é q u a tio n 1.7 rep rése n te le m txlulc dc 1 accvicratinn
maximale ; ain si, l’ac cé lé ra tio n d e la p a rtic u le v arie en tre les v aleu rs lim ites
IzJn, — ±ft>-Xn,. com m e l’illustre la figure 1.4. N otez égalem ent que la courbe d e a^{t)
est décalée (vers la gauche) de un quart dc période par iapport à la courbe de i>,.(f).
■Si on com bine les équations 1.3 et 1.7. on obtient

« ,(/) = —o r ï(f). ( 1. 8 )

sint IV xprrssion qui décrit un m oiivcnient harm onique sim ple.

^ Dans un mouvcnient hannoniquc .simple, l'accélération esr proportionnelle à la position,


mais de signt- opposé, et les deux quantités sont reliées piu le carré dc la rréqticncf angulaire

I tone, com m e on le v o it d an s la figure 1.4. lorsque la p osition attein t In p lu s grande


v aleu r p o sitiv e, l’ac cé lé ra tio n a la pins g ran d e v aleu r n ég ativ e, et récip ro q u em en t
l orsqiio la position est nulle, l’accélération est égalem ent nulle
1.3 La deuxième loi de Newton appliquée à un mouvement harmonique simple 5

R É S O L U T IO N D € P R O e L t M E S - ;•

1" stratégie ; Les constantes de phase une différence de phase : chacun est alors déphasé par rapport à
Notez l’effet de la constante de pha.se d> sur un graphique de .v(f). I' autre. Par exemple, les œurbes de la figure J .3 c) ont une différence
Lxtrsque d) = Ü, le graphique de xlf) est semblable à celui de la de phase de tt/4 rad.
figure 1.4 a), une courbe cosinusoidale ty pique. Loisque ta valeur de tl> Puisqu'un mouvement harmonique simple se répète après chaque
augmente, la courbe est décalée vers la gauche sur l’axe des r. période T et que la foiicnon cosinu.v sc répète aprè.v chaque 2tt rad.
(Vous pouvez utiliser comme aide-mémoire le symbole où la flèche une période '/'représente une différence de phase de i.Tr.ad. Dans In
pointant vers le haut indique une augmentation de la valeur de à figure 1 4. x(t) est déphasé tl’un quart de période vers la droite pat
et la flèche pointant vers la gauche indique le décalage résultant rapport à v^(f). ou de —n/2 rad ; il est déphasé d’une demi-période
de la courbe.) Lorsque la valeur de d>diminue, la courbe est dtcalée vers la droite ou de - t rad par rapport à «,(/). l ’ti déphasage
vers la droite, comme on le voit dans la figure 1.3 c), où d> = —rr/4. de 27t rad fait coïncider la courbe du inouvcnicnt iiannoiiiquc siiniilc
Ixirsquc deux graphiques représentant des moiivement.s harmoni­ avec elle même ; clic semble donc inchangée.
ques simples oni des «instantes de phase différentes, on dit qu’ils ont

1.3 La deuxième loi de Newton appliquée


à un mouvement harmonique simple
Lorsqu’on sait que l’accclcration d ’une particule varie en Ibnctioii du temps, on [tetil utiliser
la deuxième loi de Newton pour déterminer le module de la force devant agir .sur la particule
pour lui donner cette accélération. St on com bine la dcu.xième loi de N ew ton et l’équa­
tion 1.8, on arrive à cette expression, qui s'ap p liq u e au m ouvem eiu harm onique siiiqtle :

^rài.x ~ ~ —(ГГКО^)Х. (1 9 )

C e résultat une force de rappel prop<irtionnc11e à la p osition m ais de signe opposé


vous sem ble sans doute fam ilier. C 'e s t la loi de H ooke,

f-Kx = - k x (1.10)

s'appliquant à un ressort, la constante de rapjxil étant ici

к - tno>^. ( 1. 11)

En fait, l’éq u atio n 1.10 fxiut se rv ir à d é fin ir de faço n d iffé re n te le m o u v em en t


harm onique sim ple. C elte définition serait la suivante.

Un mouvement harmonique simple est un mouvement effectué par une particule de masse tn
soumise à une fort« proponioiuielle à sa position, mais de signe opposé.

Le systèm e bloc-ressort d e la figure 1.S form e un oscillateur harmonique simple


linéaire (ou, sim p lem en t, un o sc illa te u r lin é aire ), où « lin é a ire » in d iq u e q u e la
com iw sante / ’ est proportionnelle à л, cl non à une aiitrr puissance d e л. La fréciiicncc
an g u laire w du m o uvem ent h arm o n iq u e sim p le du b lo c est reliée à la con.staiitc de
rappel к et à la m asse ni du bloc par l ’équation 1. 11, ce qui donne

Fk
(O — \l — (la trcqiiencr angulaire). (I 12)
Vm

En com binant les équations I “i et l . 12. on peut écrire ainsi la p ério d e de l’oscülaiem
Figuie 1.5 l.hi oscillateur harnionique linéaire de la figure 1.5 :
linéaire. 11 n’y a aucun frottement
entre le bloc et la surface. À l'instar (1.13)
(la péricxle).
de la particule de la figure 1.2, V к
le bkx. décrit un moiivcinenl
harmonique simple aprc.s avoir été tiré Les éq u a tio n s 1.12 ei 1.13 in d iq u en t q u ’une fréq u en ce an g u laire élev ée (d o n c, une
vers la droite puis relâché. Sa position courte période) co rresp o n d à un ressort rig id e (la co n stan te k est élevée) et à un bloc
esi alors donnée par l'équation 13, léger (la m asse m est faible).
é Chapitre I Les oscillations

D ans tout systèm e oscillant, q u ’il s ’agisse de l’o scillateur linéaire de la ligure
d ’un plonget)ir ou d ’une corde de violon, on trouve un élém ent d ’élasticité et un éléi
d ’inertie ou de m asse, com m e dans un oscillateur linéaire. D ans l’oscillateur lincair
la figure 1.5, ces élém ents sc situent dans des parties distinctes du sy stèm e: l’cla.st
se trouve entièrem ent dans le ressort, que l’on supprrse de m asse négligeable, et l’iiu
se trouve entièrem ent dans le bloc, que Гоп suppose rigide. D ans une corde de vio
toutefois, ces dcu.x clém ents se trouueiit dans la corde elle-m êm e, com m e vous le vc
dans le chapitre 2.

✓ v é r if ie z VOS CONNAISSANCES 2 ; Laquelle des rdaium s .suivantes entre la c o m p o sa n ic F,


I de la force exercée sur uik particule et la position ,v de la particule implique une o.scillatiuii
I harmonique simple de la particule : a) f , = - 5 .г .Ь ) Г ,= -4<Юл^. c) é ’, = lOv di é , “ 3.»^'?

Exemple 1.1
l III Moc de masse m ~ 6HU p est aliadié un resson doni la constante Ce module dc viiessc est aiicint lorsquc le bloc o.stillaiii passe
de iap|irl est P = 6.V ,'J/m. Le bloc c.st hré h une disiance de r = M cm I'origine. Coniparez Ics figures 1.4 a) ct 1.4 b). w i vous jx iu v cz x
(le sa position (Téquilibre ,v ~ 0 sur une surface horizontale sans qiie Ic mtxlule dc la viiessc est maximal lorsquc ( “■0.
fronciiieiii. tsi reliiohé û / “ 0. d) Quel esi le module a„ de raccelcraiion luaximale du bitx
a) Duclles somI la riéijiieiiee angulaire, la tréqucncc et la période du SOLUTION D.ins ce cas. le tontepl de cm que Ic module a„, dc I'acccli:
monvemrni réjuiliarl ? lion maxim.ilc correspond ii I'ainpliiude de I'cquatior I
SOUIIIOK IX- comepi dé utilisé ici est le suivant : le système bloe-rcssurt c’esi-^-dirc <)
foiiiic iiii oselllatcur linéaire, et le bloc décrit un mouvcmeni
li:<i inomque simple. La fréquenee angulaire c9t alors donnée = (tr i„ (9,78 rad/.si'((l.l 1 m)
p ari equation 1.12. = 11 m/s^ (répoD!

jT _ iés NVm Ce mtidulc de l’accéléraiioii maximiilc se pnidiiii loisque le Мое atte


— 9 .7 8 ra d /s ^ 9 .8 ra d /s. (réponse)
V w “ V Ö .08 Kg le.s limites de son mouvemciu. À ces poini.s. la toree agi.ssani .sur
bItK' atteim .son inrxlule maximal. Ctmiparez les figures 1.4 a) ct 1.4 1
On trouve lu fréquence n l’aide de l'équaiion 1.5. ce qui donne:
où vous poussez voir que le mixlule de la position et le module •
9,78 rad/s I’accelciutioii sont maximaux au même instant
J - — 1.56 H? 1.6 Н/ (réponse)
2 :r rad c) Quelle est la consianie dc phase </>dc ce mouvement ?
Lû période csi foui nie par l'équation 1.2. ce qui donne : SOIÜIION; Le (oiKtpt de utilisé ici est que l'équaiion I .4 donne la positk
I I du bloc en fonction du temps. On .sait qu’à i = 0. le hliv se irom
Г = - = O.W s. (réponse) à .V = .ï|„. Eu insérant ces condiiiom initutles dans l’équaiion 1.3 ■
/ 1T 6H Z
en simplifiant .t^. on fibiicm
h) (Quelle est ramiiliiudc de roscillaiion 7
I •= CfIS ф. I 1. 1“
SOllIilÛN Ix" comeptdé e u le suivant; en rabsencc de, fiotiemciii.
I cncrgic mécanique du système bloc-ressort est conservée. Le bloc En ргепшн l’arccosinus de chaque c6té, on obtient alors
esi relâché à 11 cm de .su ptisitioii d’équilibie. son énergie cinclique
esi nulle et reiiorgic potentielle élastique du système est maximale. <f>= 0rad. (réponse
DniH'. le blot aura une énergie cinétique égale à zéro lorsqu'il
SC troiivwii (le iimiveaii a 11 cm de sa position d ’cqiiilihrc, ce qui (Tout angle qui c.si un multiple dc 2rr md peut être aussi utilisé dan
signifie qu’il ne sc innivera jam ais plus loin q u 'à 11 cm tic celle l'équation 1. 14 : on a choisi ici le plus petit angle )
position. Sa posiiiiiii iiia.xiniale (con umplitiide) est de 11 cm f) Quelle est l'équation décrivant la position xit) du bloc dans ci
.t„ - 11 cm (répon.sc) système bk4.-ress(in ?

SOLUTION: Le comtpt dé cm que .r(M est donné pai l'équation I 4


e) y ucl est le module de la vitesse iiiaxiinalc v,,, du bloc o.scillaiit. cl En insérant les quantités connues dans cette équation, on tibiicnt
till le bloc se trouve t-il lorsque te module est atteint ?

SOlüTION I jp (ontspt dé, dans le prc.scnt cas. cm i|He le module de la x{t) = cosftur T d>)
vitesse maximale correspond a l’amplitude air,„ de l'équation 1.6. = (0.11 m)eos[(9.8 rad/s')l •» 0)|
c'est a dire à
= 0 .11 ctji(9.8/), (réponse)
1*^ — - (9.78 rod’s)(0.11 ni)
- 1.1 m/s. (réponse) ‘W' Vest en mètres ei / esi en secondes.
1.4 L’énergie dans (e mouvement harmonique simple 7

Exemple 1.2
À f = Ü, la position xiO) du Woc dans un oscillateur linéaire conmic SOLUTION: Le concept clé utilisé dans la partie a) est également pertinent
celui de la figure 1.5 est de —8.30 ciii. (.x(0) signifie «.r à r — 0»). ici, tout comme les équations 1.15 à 1.17. Daas le présem tas, lould'oi s.
La vitesse v,(0) du bloc est alors de —0,920 m/s, et son acceleration on connaît 01 cl on veut trouver les valeurs de cpet de Ji„,. St ou divise
«,(()) c.st de - 47,0 m/s^. l’équation 1.16 par l'équatiou 1.15, on détermine que

n) Quelle est la fréquence angulaire w de cc système ‘i i\(0 ) —tu sin ip


>tan <!>.
Jt(0) rniCOS0
SOlUTlON: Dans ce cas, le concept dé est que. pour un bloc décrivant un
mouvement harmonique simple, les équations I 5, 1.6 et 1.7 donnent Kn résolvant celte équation pour irouvcr tan é. on obiicm
re.spectivemeni la po.siiion, la vitcs.se et raccéicration du bloc, et que
(t> SC trouve dans chacune des équations. On peut insérer r = 0 i\(0» -0 .9 2 0 iii/s
tan < /) = - - = -0 ,4 6 1 ,
dans chaque équation pour voir si on peut les résoudre cl trouver eu. w r(0) ~ (2.5,5 rad/.s)(-n.0R50 m)
On a alors:
Cette équation a deux solutions :
x{0) = cos <jl. 0.15)
rf> = —0.432 rad et d> = ,Trad I ( 0.432 rail) ” 2,71 nul.
1^(0 ) = sir (f). (1.16) (Normalement, seule b première solution esi affii’liée, pai une calai-
et fl,(0) = —f«^f„,cos Ó. 0 .1 7 ) latncc.) Pour choisir la bonne soliilion. on vérifie les deux .soliiiions
en les iiiilisam pour i alculcr les valeurs de l'amplitude A l'oidc
Dans l’équation 1.15, ru n'apparaît pas. Dans les équations 1.16 cl 1.17, de I’cquation 1.15, on peut déterminer que si d» — Ü,4J2 rad,
on connaît les valeurs des termes à gauche, mais x^et <f>sont inconnus. alors
Toutefois, si on divise l'équation 1.17 par l’équation 1.15, on élimine
.r(0) - 0 0850 m
à la fois .iTn, et d>et on peut alors, de cette tai;on, trouver eu; t = ------ — ------ — —0,093 6 m .
^ cos cos(-0,432)

^ /_ 4 7 .0 m /s 2 _ I>c la même façon, on détermine que si <5 = 2,71 rad, il s'ensuit que
V r(0) V -0,085 Ora — 0,093 6 rn. Puisque l'amplitude du mouvement harmonique
simple doit être une constante positive, la constante de phase ei
= 25.5 rad/s. (réponse)
l’amplitude correctes sont ici de

b) Quelles sont la constante de phase d>ct l'amplitude <f>- 2,71 rad et Xrr, = 9,36 cm. (re^Kiusej

RC OI .UTION DE PROBLéMCS

stratégie ; Recotmainv un mouvement harmonique simple {MHS) Dans certains problèmes, vous aurez peut être à déiiver une
Dans un MHS linéaire, l'accélération cç et la position x du système expression de la composante F, de la force résuliantc en fonction de la
son! reliées par une équation de cette forme : position X. Si le mouvement est un MHS linéaire, la cunipusaiite de
la force rc,sultanie et la posilion sont reliées par
n, = - (constante positive).v,
f , = —(corLsiante [losittveU.
qui indique que Taccéléralion est proportionnelle à la position par
rapport à la position d’équilibre, mais dans la direction opposée. ce qui indique que la composante x de la force résultante est pmpor-
I/)rsqiie vous avez trouve une expression semblable pour un système tionnellc à la |X)silion, mais dans la direclioii oppo.séc. Une fois que
oscillant, vous iKiiive/ immédiatement la comparer à l'équalion 1.8, vous aurez trouvé une telle expression, vous fwmvez imméiliafcmcin
vérifier que la constante positive est égale à oi^ et obtenir rapidement la comparer à l’équation l.lfl et vérifiei si la consulate itositive
une expression pour la fréquence angulaire du mouvcmeni. À l’aide correspond bien à k. Si vou.s connaissez la masse, vous pouvez utiliser
de l'équation 1.5, v'Ous pouvez ensuite trouver la période T et la les équations 1 1 2 ,1. 13 cl 1.15 pour trouver la fréquence angulaire «a,
fréquence J. la péi iodc T et la fréquence f

1.4 L’énergie dans le mouvement


harmonique simple
Dans le chapitre 8 du volume I , on a vu que l’cncrgie d ’un osctllntrur linéaire sc convertit
alternativem ent en énergie cinétique et en énergie fxuentielle aloi-s tpie la som m e tie.s
deux l'énerg ie m ceanique E de l’oscillateur - dem eure eonsianre On analysera m ain­
tenant celte .situation d 'u n point de vue quaiiiitatif
L 'é n e rg ie p o te n tie lle d 'u n o sc illa te u r lin é a ire co m m e celu i d e la fig u re 1 5 est
entièrem ent asstxriéc au ressort. Sa valeur dépend de l étirenienl ou de la com pression
du ressort - c'est-à-d ire de x(r). On peut utiliser réq u a tio n 8 .11 du volum e I ei l équa
tint! I ..3 pour trouver

l'U) - eos^fmt + </>) (I 18)


8 Chapitre 1 Les oscillations

N otez bien q u ’une fonction c.Kprimée par eos- A (com m e dans le présent cas) signifi
(cos A)- et / l 'est pas identique à celle exprim ée par cos A-, qui signifie cos (A-).
L’énergie cinétique du systèm e de la figure 1.5 csl en tièrem en t associée au bloc
S a valeur d épen d de la vitesse du bloc - qui est v,(i). O n peut u tilise r l’éq u atio n l.i
pour trouver

K(t) = siir(cot -t <(>}. ( i.iy

Si on utilise I cciuaiion 1.12 pour substituer k/ni à co\ on peut écrire l’équation 1.19 sou
celte form e ;

K(t) = 5»iv- = |A,v,-n sin^tfwf + r/>). ( 1.20)

L 'énergie m écanique correspond à l'ad d itio n des équations 1.18 et 1.20, et est

¡■: = u + K
* 2 '^in 'eos-(itj; H </>) + siii^iwr + <f>)

= jA.vii, |eos-(a)f + d>) + sin-(wr i <#>)).

Peu im porte l'an g le a.


Rgme l.é ;() L’énerpi«? poieniicllc U{t).
c o s * « f sin^of = 1.
l énerpio cintnitlue /¿(M et l'cncrpic
mccanique L en forK;lM>ii du temps t
flans (III nsfilliilciii liiK^aini. Nolir/, D onc, la quantité entre parenthèses ci-dcssiis vaut sim plem ent 1. et on a
que liuiles les énerpies will une vulcui’
posiiive et que l’éncrpic cinétique E=V y U 'i ,- ( 1. 21)
atteint su valeur max ininic deux fiiiv
au courN de chaque période. b> L cncrgic En effet, l'é n e rg ie m écanique d ’un o scillateu r linéaire est co n stan te et ne d épend pas
potentielle f/(x). !’éiiorj;,ic. l iix'iiqiic du tem ps L 'é n e rg ie p o te n tie lle et l'é n e rg ie c in éliq u e d 'u n o sc illa te u r lin é aire sont
A't'.i) el l'énergie mécunique t représentées en fonction du tem ps t dans la figure 1 6 a), et en fonction de la position x
en fonction de la positiiiii « dans dans la figure 1.6 b).
un os( illaieur linéaire d'amplitude x,,. V ous d e v rie z m a in te n an t co m p re n d re p o u rq u o i un sy stèm e o sc illa n t co m p o rte
Lorsque X ” 0, l’énergie csl cniicvenient norm alcm enl un clém ent d ’élasticité et un élém ent d ’in ertie: le prem ier reiifen n e son
cinélique, el lorsque x = ±.Vm. énergie potentielle alors que le second renferm e son énergie cinélique.
clic est entièrement potentielle
^ V É R I F IE Z VOS CONNAISSANCES 3 : fa n s la fig u re 1 ..‘i, lorsque le b lo c est à r = + 2,0 cm .
Il a une ciu'.rgie cinétique de J. el le ressort a une énergie [Xitcnticllc élastique de 2 J
a) Quelle est l'énergie cinéti(|ue lorsque le bloc est à.» - 0 ? Quelle est l’énergie potentielle
élastique lor.si|Ue le bl<K: est à b) v = - 2,0 cm et c) x = ' Vo,'.’

Exemple 1.3
al Quelle est réiicrgie iiircanii|iic F. de l'oscillaleui linéaire de SOLUTION. Ici. on utilise le concept dé suivant: puisqu'on connaît la
1 exemple 1.1 7 (Initialement, la position du bloc cm i * 11 cm position du bl(K. on peut facilement trouver l’énergie potentielle
et sa vitesse est v,. —0. La constante de rappel est k = 65 N/in j du ressort à l’aide de la formule U ~ jLx'. D an.s le cas où .v “
nn a
SOILHIOH ! e lontept dé. dans le présent cas, est que l'énergie méciuiique t
(la simiiiK' de l’ém rgie cinélique A = j»m~ du bloc et de l'énergie U = J Al - — J A ( j x„)" — (j)(j)A r^ .
poirniiclle du ressorr — ~kx~) esi consianie dans l'ensem ble
du iiiiaivrnu-iil ik- I'oM ilfaieur. Donc, on peut év.aluer A en tout point On peut insérer les valeurs numériques de k et .r,„, ou on peut utiliser
l'énergie mécanique loialc. donnée dans la panic a), qui est
du muiivcinciii Piiis(iu’f»n sait que les comliiions initiales de l’oscil-
Cela permet d'ccrire, à l’aide de l'équalion ci-dessus,
latcui M7iil X — 11 cm Cl i. -^0. on peut évaluer A'en fonction de ces
conditions. ( / = 3(4A v;„) = \F. = itO.193 J) = 0.098 J (réponse)
Qii liouvc alors que
Ensuite, en utilisant le même concept clÉqu'en a) ( t — A' + U), on peul
A= n= -k- - O a- ^(b.S N/m)(U.l I m)- écrire
— Ü.J93 J 0,39 J. (rciKiiise)
A' =» A - (/ = 0.303 J - 0.008 J = 0.20S ,t (réponse)

b) Quelles sont l'éiiciKic potentielle U cl l’énergie cinélique K Si on reprenait ces calculs relaii’vemeni à .t —4 Vm- on arrivenui
de ro,scillaicur loisque le hloe esi 0 » = î.iu,“’ Quelles sont-elles aux memes réponses, ce qui concxirderail avec la symétne des parties
Idi sqiip le bloc est à .v — gaucho el droite de la ligure 1.6 h).
1 5 L'oscillateur harmonique simple angulaire 9

l^ lE x lié m ité fixe


1.5 L’oscillateur harmonique simple angulaire
La figure 1.7 rep rése n te une vension an g u laire d 'u n o sc illa te u r h arm o n iq u e sim p le ;
l’élém ent d ’élasticité est associé à la torsion d ’un fil de suspension plutôt q u ’à l'étirem ent
' Kil lie tiHS|Kinsion ou à la co m p re ssio n d ’un re.ssort, co m m e d a n s les cas p réc éd en ts. C e d isp o s itif se
nom m e p e n d u le d e to rsio n .
r Ligne Si on tourne le di.sque tic la figure 1.7 en lui faisant faire une- rotation ou un déplacement
i de léféri'nce angulaire в par rapport à sa position d’équilibre (où la ligne de référence est à ft = 0)
et q u ’on le relâ ch e , il o sc ille ra au to u r d e c e tte p o sitio n cl d é c rira un mouvement
harmonique simple an};ulaire. En tournant le disque d 'u n angle (7dans l'u n e ou l'au tre
direction, on produit un m om ent de force de rappel donné par

T --------кв. n.2'21
Rgiire 1 7 l'n oscillateur harmonique
Dans ce cas. k {k est la lettre grec kupixi) est une constante nom m ée constante de torsion,
simple angulaire, ou pendule de torsion,
qui dépend de la loiigueui, du diam ètre et du m atériau com posant le fil de su.spen.siun,
est une version angulaire de roscillaleur
L orstju 'o n com pare l’équation 1.22 à l’équation 1 10, on peut sou[)i;onnei qitc la
hantK'nique simple linéaite de la fi
gurc 1.5. Le disque oscille dans un prem ière est la form e angulaire de la loi de Ilo n k e, et q u 'o n peut tran.sformcr l’cqua
plan horizontal ; la ligne de référence tton 1.1-^, qui donne la périixle d ’un M HS linéaire, en une équation donnant la période
oscille avec une amplitude angulaire 6,,,. d 'u n M H S a n g u la ire : on rem p lace la c o n stan te de rap p el k de l’éq u a tio n 1 .J3 par
r.a torsion du fil de suspension son équivalent, la constante de torsion k de l’équation 1.22, et on rem place la m asse m
emmagasine de l’cncrgic potentielle de l ’éq u atio n 1.13 p a r son é q u iv ale n t, le m o m e n t d ’in e rtie I du d isq u e o scillan t.
comme le ferait un ressort, et produit Ces substitutions m ènent à
un nximent de force de rappel
/
7 == 2 . ^ / ^ (le pendule de torsion). (1.23)

qui est l’équation exprim ant la période d ’un oscillateur harm onique sim ple angulaire,
ou pendule de torsion

RESOLUTION Dfc PR O eL f MTS

3' sirolégie. RevonmUre. un MHS tmf’ulaiie Vous pouvez également reconnaître un MH.S angulaire si vous
Lorsqu'un système a un mouvement harmonique simple angulaire, voyez une expression qui relie le moment de force t à la position
son aecéléraiion angulaire a et sa position angulaire 0 sont reliées angulaire; cette expression doit avoir la forme de l'équation 1 ??
par une équation de Lu forme ( t — —k€), où
T — -(constante positive)©.
a - -(constante positive)^
Cette éfiuation esi réquivalcnt angulaire de l’équation l.l()(7'nr kx).
Cette équation est réquivaicnl angulaire de l’cquati<«i 1.8 (<i, — Llle indique que le niomeiu de force i est proportionnel à la position
Elle indique que l’accélération angulaire a est proportionnelle à angulaire 6 mesurée par rapport à la position d’équilibre, mais tend
la position angulaire B mesurée par rappon à la position d'équilibre, à faire tourner le système dans la direction opposée an déplacement.
mais tend à faire tourner le système dans la direction opposée Si vous voyez une expression de cette forme, vous pi.uivez conclure
au déplacement. Si vous voyez une expression de cette forme, vous que la constante positive est la constante de torsion k du système.
pouvez conclure que la constante positive est ur, et vous pouvez Si vous connaissez le moment d'inertie / du système, vous pouvez
ensuite déterminer w, f et T. ensuite déterminer T à l'aide de l’équation 1.23.

Exemple 1 4
Fil
I a figure l 8 a) montre une tige mince de longueur L = 12.4 cm fie
et de masse m = I T'i g. attachée en son |ximt milieu à un long fil SllS|K*n4nn
de suspension. Son MHS angulaire a um- période 7",, de 2..S3 s.
Un objet de forme irrégulière, qu’on nommera objet X. est ensuite Tige
suspendu au même fil. comme on le voit dans la figure 1.8 b), et
sa périmle 1), est de 4,76 s. Quel est le moment d'inertie de l'objet X
par rapixirt à son axe de suspt'nsion

SOlUnON: Dans le présent cas, le rontepf tlé qu’on utilise est le suivant :
le moincni d'inertie de la tige, ef'nime celui de l'objet X, est relié à la
période mesurée p,u' l’équauon 1.23. Dans la case e) du tableau 11.2 Figure 1.8 Exemple 1.4 Deux pendules de torsion ; a) une tige et iiii fil.
du volume l, Icmomcni d’inertie d’une tige niincc parrapixiri k un axe et b) le mêmi' fil et iin objet de lorme irrégulière
10 Chapitre 1 Ues oscillations

pcrpciuliciilaire passant en son poini tnilieii est donné par -pm/,*. constante x. qui est une propriété du fil. e.st la même poui les deux
On a donc, daas le cas de la tige de la figure 1.8 a), expressiims ; .seuls les périodes et les moments d'inertie ditlèrcnt.
On élève ensuite chacune de ces équations au carré, on divise la
/,. = ,'y n /i = (y',KO,135 kg)(0,l24 ml- seconde par la première cl on résout l'éiiuation résultante pour
trouver II,. Le ré.sultai est
= 1,73 X lO- 'k g ni^

On écrit m.iintcnanl deux fois l’équation 1.23. une fois pour lu lige
et une fois pour rf)hjei X

T., - 2,T, cl ¡'h — 2 n \! —. = 6.I2x lO“H g m’ (réixtnsc)

1.6 Les pendules


On étudiera m ainlenani un type d 'o scillateu r harm onique sim ple dans lequel l'é-lémcnt
tl'élasticilé csl a.ssocié à la force graviialionnelle. et non aux propriétés élastiques d ’un
fil tordu ou d 'u n ressort eom prim é <su étire.

Le pendule simple
S i vous atta c h e z u n e p o m m e à un long fil fix é ù son ex trc m itc su p é rie u re, e t que
vous donnez ensuite à la pom m e une légère im pulsion pour générer un m ouvem ent de
halaneem ent, vous co n staterez facilem ent que le m ouvem ent est périodique S 'ag ii-il
d o n c d 'u n m ou v em en t h a rm o n iq u e s im p le ? Si o u i. q u elle en est la p ério d e 7 ?
Pour répondre à cette question, on doit exam iner un p en d u le sim p le, qui consiste en une
p arik u lc de m asse m siis()cndue à une exiiém ilc d ’une corde non éiirablc de longueur/,
et de m asse négligeable, fixée à l'autre extrémité, de la façon illustrée dans la figure 1.9 a).
Le pendule peut se balancer librem ent dans le plan de la page, vers la gauche et la droite
par rapport à une ligne verticale traversant le point de pivot du pendule.
L es forces agissant sur le pendule sont la force T de la corde cl la force gravitation­
nelle F j. com m e on le vint dans la figure 1.9 b), où la corde form e un angle 6 avec la
verticale. On décom pose en .sa com posante radiale etis 0 et une eom posanic Fg sin $
qui e.st tangente à la trajectoire du j)cnduic. C ette com posante tangentielle produit un
m om ent de force de rappel par rap|x>n au pivot du pendule. Puisque ce m om ent de force
agit toujours en sens opposé au d éplacem ent du pendule, il tend à ram en er le (xmdule
à son point d ’équilibre. C e point sc nom m e la position {¡'équilibre (b - 0), puisque le
pendule .serait au repos ti cet endroit s'il ne balançait pas.
À l’aide de l’équation 11.33 du vtilum e 1 ( r = r j ' ) , on peut écrire ce m om ent de
force de rappel sous la form e

r = sin 6), (1.24)

où le signe négatif indique que le m om ent de force s’oppose au déplacem ent du pendule
pat rap p o rt il sa p o sitio n d 'é q u ilib re , et o ù L est le b ras d e le v ier d e la eo in p u san ie
la n g en iielle de la fo rce, Fg sin f). En in sé ra n t l ’éq u a tio n 1.24 d an s l ’é q u atio n 11.36
du vo lu m e 1 ( r — la) et en in séran t en.siiile tng p o u r e x p rim e r le m o d u le de 7^,
on obtient

—lim g sin B) — la. (1.2.5)

•') où / csl le m oincni d ’inertie du pendule par rapport au pivot, e t où crest son accélération
angulaire aiitoui d e ce point.
Ijfiurt 1.9 u> Un iKiidnlt simple
b I Le.s torces agissant sur le p^ndulc Ün peut .simplifier l'éq u atio n 1.25 si on suppose que l’angle B est petit, et on peut
soni la foti'f graviiaiionnelle alors faire l'appro x im atio n sin 6 »= 6 (exprim é en radians). (Par exem ple, ai B —
ot la toree 1 evcrccc par la corde. - 0.087 3 rad. a lo rs sin B = 0 ,0 8 7 2, un e d iffé re n c e d e seu lem en t 0.1 'H). A l ’aide
La v v in p o s u n u la n g c m ic llc F,, sin 6 de cette approxim ation, et en réarrangeant les termes,<on obtient alix s
de la fo n <■g ra v iiH ilo m ie lle evl
une fo rre de m ppet q u i tend á ram ener m g l.
a — (1.26)
le p u id u lt: vcr.s .Sil p o s itio n d ‘éi|UÍ1ihrc.
>6 L es p e n d u le s II

Celle équation est réquivalent angulaire de l’équation 1.S, qui décrit un MHS. Elle indique
que l'a c c é lé ra tio n an g u laire a du p en d u le est p ro p o rtio n n elle m ais de signe op p o sé
à la position angulaire ft. Dcinc, lorsque le pendule se déplace vers la droite com m e dans
la fig u re 1.9 a), son ac cé lératio n o rien tée vers la gauche augm eiUe ju s q u ’à c e q u 'il
s ’im m obilise et en trep ren n e son m ouvem ent vers la gauche. Par la suite, lo rsq u ’il est
à gauche de sa p o sitio n d ’éq u ilib re, son accélératio n o rien tée vers la d ro ite ten d à le
ram ener vers ce côté, et ainsi de suite, alors q u ’il se balance dans un M HS. Plus prccisc-
meni, on peut dire que le in o u \ cm ent d ’un pendule simple qui se hatam e en/nrnuint de
petits angles est approxim ativem ent un M H S. En d ’autres term es, l'u m p litu d c d e la
p osition a n g u la ire 0,„ du m o u \cm en t ( l’angle m axim al du balancem ent) doit être faible
pour que son m ouvem ent d écrive approxim ativem ent un M HS.
En co m p aran t les éq u a tio n s 1.26 et 1.8. on voit q u e la fré q u en c e an g u laire
du pendule est co = J m g Ü J l. Si on insère ensuite cette expression tic dans l'é q u a
lion 1.5 (<w = 2nlT). on voit que la période du pendule peut être écrite sous la form e

(I 27)

T oute la m a sse d ’un p en d u le sim p le e s t co n c en tré e d an s la m asse m d e la p articu le


attachée au bout de la corde. t|ui est située à un rayon L du pivot. D onc, o n peut utili.ser
l'é q u a tio n 11.26 du v o lu m e 1 ( / = mr^) p o u r éc rire I = mÜ-, e t e x p iiin e r ain si le
m om ent d 'in ertie du pendule. En in.sérant cette valeur dans l ’équation 1.27, ou obtient

T — In J - (le pendule simple, faible amplitude), (1 2S)


V i

ce qui est une expression sim plifiée de la période d 'u n pendule sim ple dont la po,sition
angulaire par rapport à la position d ’équilibie forme de petits angles (D ans les problèm es
de ce chapitre, on supposera que tes positions angulaires form ent toutes de petits angles.)

Le pendule composé
Un pendule réel, habituellem ent nom m é pendule composé, peui avoir une disii ibulion
de m asse com plexe, assez différente de celle d ’un pendule sim ple. Un pendule com posé
effertu c t-il égalem ent un M HS ? Si oui. quelle en est la péiitKle t
La figure I 10 rep résen te un pejidule com posé arb itraire, in clin é d 'u n cô té selon
un angle ft. La force gravitationnelle agit sur son rentre de masse C. à une distance h du
pivot O. MalgiX’ leurs form es différentes, les pendules des ligures 1.10 et 1.9 b) i-évèlrni,
si on les com pare, une seule, différence im portante entre un pendule com posé arbitraire
et un pendule sim ple. D ans un pen d u le com posé, la co m p o san te lan g cn ticllc /p sin ft
de la forée gra\ itationncllc (com posante qui exerce une force de rappel vers la position
d ’équilibre du pendule) a un brus d e levier h par rapport au p it ot plutôt q u ’une longueur
/. (longueur d e la corde). Sous tous les autres as|X'cts, une anuly.se du pendule com posé
serait la m êm e que Eanaly.sc du pendule sim ple, ju s q u ’à l’équation 1.27. U ne lo ts de
plus, on découvrirait (dans le cas d 'u n petit ft„,) que le inouv cment est approximativtmwni
Figure 1,10 I n pendule composé. un MHS.
Le moment de force de rappel eo Si on rem place L par h dans l'éq u a tio n 1.27. on |setii écrire ainsi la p ério d e d 'u n
Bin ft Lorsque ft — 11. le centre pendille com posé :
de masse C se trouve directement
sous le pivot O.
r — 2iT, , île pendule coinposé. tailile amplitude). (1.29)
y mgh

Com m e dans le cas du pendule simple, I est le moment d'inertie du pendule par rapport à O
T o utefois, / n ’éq u iv au t pas sim p lem en t à m l.' (il d ép e n d de la form e du p en d u le
com posé), m ais est encore proportionnel à m.
Un pendule com posé ne balancera pas s 'il pivote aiitoui de son centre d e m assé.
De façon form elle, cela équivaut à m ciiie h ■ 0 dans l’équation 1.29. C ctle equation
perm et alors de prédire q u e 1 —> ce qui im plique q u ’un tel pendule n e com plétera
jam ais une oscillation.
12 Chapitre I L^s oscillations

T üui p en d u le co m p o se q u i o sc ille p a r rap p o rt à un p iv o t O d o n n é e t avec une


période T’corrcsiw nd à un pendule sim ple de longueur i,) qui a la m êm e périexlc T
O n p eu t tro u v e r la v aleu r d e L,j à l’aid e d e l ’éq u a tio n 1.28. Le p o in t du {pen­
dule com posé situé à une distance Ц du point O se nom m e семге d'osdllusion du pendule
coinpo.sé pour un point de suspension donné.

La mesure de g
Le pendule coraptisé peut servir à m esurer l'accélératio n en chute libre à un endroit
particulier à la surface de la Terre. (D es m illiers de iiieMires sem blables ont été prises
lors de pro.s|Teciions geophysiques.)
Pour analyser un cas particulier, on suppose que le pendule est une tige uniform e
de longueur L su sp en d u e à une de ses ex trém ités. D ans un tel pendule, la d istan ce h
(lie l’équation 1.29) entre le pi v(4 et le centre de ma.sse est de case e) du tableau 11.2
du volum e 1 indique que le m om ent d ’inertie de ce pendule par rappoi t à un axe perpen
diculaire passant par .чш centre d e m asse est d e À l’aide du théorèm e des axes
parallèles donné par l’équation 11.29 du v o lu m e 1 ( / = /f-v, i Mh^), on p eu t alors
détem uner que le m om ent d’inertie p ar rapport à un axe perpendiculaire passant par une
extrém ité de la tige est

/ = ^CM Mh- = Â w L - + = [mL~. (1.30)

Si on pose h = \ L ei I = jinL^ dans l’équation 1.29 et q u ’on la résout pour trouver g,


on üblient

8я7,
(1.31)
3 ^

Doue, en mesurant L et la période T, on peut trouver la valeur de g là oit sc trouve le pendule.


(S i une grande p récisio n est req u ise, q u elq u es m esures su p p lém en taires d o iven t être
prises ; on peut, p ar e.xempic, faire balancer le penilulc dans une cham bre sous vide.)

Exemple 1.5
Dan.s la figure 1.11 a», un mètre se balance par rapport à un pivot Figure 1.11 Exemple 1.5 a) Un mètre
situé à une de scs exlréniités, à une distance h de son centre de masse. suspendu à une de ses extrémités
et constituant un pendule aimposé
a) Quelle est sa période d'oscillation T?
b) Un pendule simple dont
SOIUTIOK: Le lomept clé utilisé ici est le suivant ; le mètre n'est pas un la longueur ¿o a été déterminée
Ijcndulc simple, puisque sa ma.sse n’e.si pas concentrée dans une de façon à ce que les périodes
pimiciile ponctuelle située à l’extrémité opposée à celle du pivot - des deux pendules .soient égales.
donc, le mètre est un pendule com()osé Sa période est altirs donnée Le point P sur le pendule illustré
par l'équation 1.29, dont la résolution requiert qu’on «innaissc le en a) indique le centre d’oscillation.
moment d’inertie /d u inètic par rapport au pivot On peut considérer
le metre comme une tige homogène de longueur L et de masse m.
EV>nc, l'équation I..30 indique que / “ et que la distance h de En égalisant les équations 1.28 et 1.32. on obtient
l’éiitialiiMi I ?9est jjL. Én insérant ces quantités dans l’équation 1.29,
on détermine que
T — 2;Г4 / — ^ 2n
V 3if
r r
— 2тг. IL ( 1..32t On trouve alors
meh Ъу
/.(I = ■ { /.= (^К Н Ю ст) 66,7 cm. (réponse)
(2)(1.(XJ m)
- 1,64 .4. (réponse)
= 4 i(3)(9.1! m /sO Dans la figure 1.11 a), le point P est alors à 66,7 cm du pivot D.
Donc, le point P est le centre d’oscillation de ce point de suspension
Notez que ce résultat ne dépend pas de la masse m du pendule. donné.
h) Ouclle est la tlisiance h. entre le pivot O de la lige et son centre
fl osoilluiion t VERIFIEZ VOS CONNAISSANCES 4: Troi.s pendules com|x>sés,
di'nt les nias.ses sont respcctti'ctnent m„. hti^ex hn„, ont la même
SOIUTIOH. Dans ce ras. le tontepi tlé est le suivant: on veut trouver la I fforme et la même taille et sont suspendu' au même point. C’la.s.sc7,
longueur /.pdu pendule simple (illustré dans la figure 1.11 b|) qui a les ma.\ses en ('rdre décroissant selon les périodes d'oscillation
la même péi itxlc que le pendule conir«vsé (mètre) de la figure 1. 11 a). des pendules.
1.7 Le mouvement harmonique simple et le mouvement circulaire unifórme 13

Exemple 1.6
Diuis la figure 1.12, un тапсЫ я (apparemment doué pour les sports En substituant ces expressions à / cl à F dans l’équation 1.34,
nautiques) plonge d ’un tremplin homogène. Le tremplin est fixé à on obtient
une charnière du côté gauche, et attaché à un ressort du côté droit. mira
Il a une longueur L - 2,0 m et une masse ni= 12 k g , la constante -L k x = - ^ . (1.35)
de rappel к est I 300 N/ra. l.x)rsque le manchot plonge, il provttque
On a donc une relation entre la po.sition linéaire x (verticale) et
une oscillation de faible amplitude dans le tremplin et le ressort.
l’accélération ttngulaire a (par rapport il la charnière). On peut rem­
En supposant que le tremplin est suffisamment rigide pour ne pas
placer o dans l’équation 1.35 par l’accélération (linéaire) le long
plier, trouvez la période T des oscillations.
de l’axe des .ren suh.stiiuaiu l'c(|iiaiion 11.22 du volume 1 Ut« “ wr)
SOLUTION: Puisque le sy.stème еощ|юПе un ies.sort, on ptiurraii penser il l’accélération tangentielle. Dans ce cas. l'accélération langentielle
que les oscillations prcxluisent un MHS, mais on évitera de faire cette est U, et le rayon r est L, de sorte que or — a JL. Avec cette suhstitii
hypotltèsc. On appliquera plutôt le concept clé suivant : si le tremplin fion, l’équation 1.3.1 devient
effectue un MHS, l’accélération et la position de l’cxtrémilé («cillante ml}a,
-Ik x =
du itcmplin doivent être reliées par une expression ayaiii la forme de 3/.
l’équalion 1.8 (o = Si c’est le cas, on devrait |X)uvoir trouver ce qui donne
ft) et T. que l’on cherche à déterminer, à l’aide de celle expression.
3k
On peut vérifier la pertinence de cei énoncé en trouvant la relation (I .16)
entre l’accélération ci la ixisition de l'extrémilé droite du Ivemplin.
Puisque le tremplin pivote autour de la charnière lorsque L’équation 1.36 a la même forme que l’éqiiarion 1.8 (rq —
l’cxtiémité droite oscille, il existe un moment de force r par rapport Donc, le tremplin pnxluir effectivement un MHS, et la comparaison
à la charnière. Ce moment de force est causé par la force F exercée des équations 1.36 et 1.8 montre que
par le ressort sur le tremplin. Puisque F varie dans le temps, т doit 3k
également varier De ce fait, on peut, pour tout instant donné, relier
les modules de т cl de F à l'aide de l’équation 11.31 du volume I
( t = »'F sin Ф). Dans ce cas. on a ce qui donne (o = .J îk /m . Alors, en iiiilisani ré»|iiaiion 1.5
(ft) IrrfT) pour trouver F, on obtient
LF sin W “ (1.33)

où L est le hra.s de levier du moment de force de F , ci où ce bras de T = 2jtJ — = ^ 0.31 s (réponse)


V 3k V 3(13(X )N /m ) ^
levier et la direction de la force forment un angle de 9(F. En amibinani
l’équalion 1.33 et l’équittion 11.36 du volume 1 ( t = lu), on a Cela peut sembler surprenant, mais la période ne dépend pas de la

L F = Ia , (1.34)

où / est le moment d’inertie du tremplin par rapport à la charnière,


et a est son accélération angulaire par rapport à ce point On peut
considérer le tremplin comme une tige mince qui pivote autour d ’une
extrémité. On déduit donc de l'équation 1.30 que le moment d’inertie /
du tremplin est ml.'.
On peut aussi imaginer un axe vertical x {vassam par rextiémité
oscillante du tremplin, la direction positive étant vers le haut. La force
exercée par le ressort sur l’extrémité droite du tremplin est alors figure l.)2 Exemple 1,6 Le plongeon du manchot fait osciller
F — -kx, où X est la position verticale de l’extrcniité droite mesurée le tremplin et le ressort ; le tremplin pivote autour de la charnière
par rapport à sa position d’équilibre. sihiée à l’extrémité gauche.

1.7 Le mouvement harmonique simple


et le mouvement circulaire uniforme
En 1610. G alilée, en utilisani son nouveau télescope, découvrii les quatie lunes principales
de Jupiter. Au cours de scs observations, qui s ’étendirent sur dc.s sem aines, il lui sem bla
que chaque lune se déplaçait d ’avant en arrière p ar rapixirl à la planète, dans ce q u ’on
appellerait aujourd'hui un m ouvem ent harm onique sim p le , le disque de la planète était
le (voint m ilieu du m ouvem ent. Le rapport des observations de G alilée, éerii d e sa main
existe toujours. A. P. French, du MIT, utilisa les d<vnnées de G alilée ¡xiiit vérifier la |rosi-
tion de la lune C allisto par rapport à Jupiter. D ans les résultats illustrés dans la figure 1.13,
les cercle-s scmi basc.s sur les observations de G alilée, et la courbe trouvée crî rajaslem en i
optim al des données, ('e tte courbe rappelle gr.andemcnt l’équiition 1.3, ejui exprim e la
fonction de la p o sitio n d an s un M H S L e g rap h iq u e perm et de m esu rer une p ério d e
d ’environ 16.8 jours.
14 Chapitre 1 Les osciUations

Rgurel.13 L’angle, vu de la Terre,


séparant .lupitcr et sa lune Callistt).
Les cercles leprésentent les nxsures
prises par Galilée en 1610.
La courbe est rajustcn'ient
o)-)tiniuI, et suggère fortciiieiit
un mouvement harmonique
simple. Par rapport à la distance
inoycmie entre la Terre et Jupiter,
10 minutes d'arc correspondent
à environ 2X 10*’ km. (Adapte
de A. P. French. Nt'w-tnninn
Mediauirs. New- York,
W W. Norton & Ctanpany,
1971. p. 288.)

En rcalité, Callisto .sc déplace à une vitesse de m odule coastant sur une orbite essentiel­
lement circulaire autour de Jupiter. Son véritable m ouvem ent - loin d 'être un m ouvem ent
ham iorüquc sim ple - est un m ouvem ent circulaire uniftirm e. Ce que G alilée a vu - et
ce que vous pouvez voir avec une bonne paire d e ju m elles et un peu d e patience - est
la p ro je c tio n de ce m o u v em en t c irc u la ire u n ifo rm e .sur une lig n e d an s le p lan du
m ouvem ent. Les observations rem arquables d e G alilée m ènent à la conclusion que le
m o u v em en t h arm o n iq u e sim p le est un m o u v em en t c irc u la ire u n ifo rm e, vu de cô té.
Ln term es plus clairs, on {lourrait dire ce qui suit.

Un niouventcnt harrminiquc simple est une projection d'un mouvement circulaire uniforme
■sur le diamètre du cercle dans Ier]ue1 ce mouvement se produit.

L a figure 1.14 a) peut servir d ’exem ple. O n y v o it une particule de référence / ’'
qui SC déplace en décrivant un m ouvem ent circulaire uniform e à une \ ites.se angulaire
a) (constante) (11, sur un cercle de référence. Le rayon du cercle est le m odule du vecteur
position de la particule. À to u t instant t, la position angulaire de la particule est ait + <f>,
où <f>est sa position angulaire à r = 0.
La projection d e la particule P' su r l ’axe d es jr corrc.spond au p o in t P, q u ’on peut
c o n s id é re r com m e une seco n d e p articu le . L a p ro jec tio n du v e c te u r p o sitio n de la
particule P ' sur l’axe des x indique l’endroit xU) où sc situe P. D onc, on déterm ine que

X(t) = ATn, co.s(tóf + 4>)y

ce qui correspond précisém ent à l'éq u atio n 1.3. La conclusion précédem m ent ém ise est
donc \ niable : si la particule de rélérence P' décrit un m ouvem ent circulaire uniform e,
.sa particule de projection P décrit un m ouvem ent harm onique sim ple le long du diam ètre
du cercle
La figure 1.14 b) m ontre la vitc.sse r de la particule de a'fcren ce. D 'a p rè s I’etjua-
tion 11.18 du volum e 1 fv = air), le m odule du vecteur vites.se c.stoix^; sa projection sut
l’axe des x est

V, (/) — —oix^ sin(w r J (fl),

qui correspond exactem ent à l ’éq u atio n 1.6. L e .signe est n ég atif parce que la ctm ipo
sanie de la vitesse dti P est orientée vers la gauche dans la figure 1.14 b), donc dan.s la
direction négative de l’axe des r.

Figure 1.11 a) Unc panicule dc rdference P’ d«*crivant un monvcmeni circulairc unilorme


sui tin ccicle lie rdfdrence de rayon Sa projection P su/ I’axc dcs » pnaliiii iiii mouvement
harmonique simple, h) 1 projection dc la vitessc v de la panicule de rdference cst la vitesse du
MUS. c) La projection de I’accdldpatinn radiale n de la panicule dc refiircnce cst I'acccldraiion
du MILS.
18 Le mouvement harmonique Simple amorti 15

L a fig u re 1.14 c) illu stre l'a c c é lé ra tio n a d e lu p articu le de référen ce. C ette
ac cé lé ra tio n n 'a q u 'u n e c o m p o san te ra d ia le , c a r la p a rtic u le d éc rit un m o u v em en t
circ u la ire u n ifo rm e. D 'a p rè s l'é q u a tio n 11.23 du v o lu m e I ( « , = a>'r), le m o d u le
de l’accéléralion radiale est <у ’ап,; sa projection su r l'ax e des дг est

д,(/) = costruf 4 Ф),

qui corresp o n d ex actem en t à l’éq u atio n 1.7. D onc, qu e l'o n co n sid ère la p o sitio n , la
vites.se ou l’accélératio n , la p ro jectio n du m o u v em en t circ u laire u n ifo rm e co n stitu e
effectivem ent un m ouvem ent han iio m q u c sim ple.

1.8 Le mouvement harmonique simple amorti


Un t>cndule placé sous l’eau n ’o.scillern que très brièvem ent, parce que l’eau ex erce une
fo rce d e résistan c e qui élim in e rap id e m en t le m o u v em en t du p en d u le. Un (lendtilc
oscille m ieux à l’air libre, m ais le m ouvem ent finit tout de m êm e par .s'esiom iKr. car
l’air exerce aussi une force de résistance sur le pendule (ei le froiicnicni ajoute à cette
résistance) et convertit l'én erg ie m écanique du [leiuliilc en énergie thci mique.
L orsque l’am p liu id c du m o uvem eiu d ’uii oscillaiciir dim inue parce q u ’une force
externe de résistance est exercée sur celui-ci, ou dil que l’ose illnicur et son m ouvem ent
sont a m o rtis . L 'e x em p le idéal d e l'am ortis.scnicnt d 'u n o sc illa teu r e st présen té dans
la figure 1. l-S, où un bloc de m asse «i, attaché à un ressort dont la constante de rappel
e.st k, oscille v erticalem en t. U ne tige fixée au bloc sc te rm in e p a r u n e ailette q ui esi
subm ergée d an s un liquide (la tige et l'a ile tie ont des mas.sc.s n égligeables). L orsque
ra ile iie sc déplace verticalem ent, le liquide exerce une force de résistance sur clic et. par
le fait m êm e, sur l'en sem b le du systèm e oscillant. A vec le tem ps, l'én e rg ie m écanique
du systèm e bloc-ressort dim inue. C ette énergie m écanique est convertie en énergie ther­
m ique duns le liquide et l'ailelle.
O n peut supposer ici que le liquide exerce une fo rce d 'a in o riis s c m c n t jiropor-
tinnncllc à la vitesse v de l'ailette et du bloc (cette supposition est précise dans la me.snrc
où l'a ilc ttc .se d éplace lentem ent). La com posaiilc d e le long de l ’nxc des x de lu
figuiv 1.1.5 est alors

= -K . (1 3 7 )

où h e.st la c o n s ta n te d ’a m o rtis s e m e n t qui d épend des caractéristiq u es de l'a ile u e el


du liquide, e t dont l’u nité SI est le kilogram m e par seconde \jc signe ncgniif indique
Support rigiilf que In direction de /•), est opposée à la vitesse v.
L a force exercée sur le bloc par le ressort e«i F r . = -fcv. On suppose que la fome
g ra v ita tio n n e lle su r le b lo c est n ég lig ea b le , c o m p arativ em e n t à è;,, e t f . O n peut
OoDMamc de r.ip|K-l, к alors écrire la deuxièm e loi d e N ew ton d éen v an t les com posantes le long de l'a x e des r
sous la form e suivante :

—hv^ — kx ~ m i f , (U 8 )

M.-i4se m En substituant dxiüt à i\ et d 'x id i- à a , cl en réarrangeant les term es, on o b tien t cette
équation différentielle :
d X ilx
ni — T -4 /) —- -4 Ar = 0. (I iQ)
d i' dt
.\ileiir
La solulirrn d e cette équation est
А тоО кадчпепП A

x(f) = Xn,f cosUo't t- é). (I 40)


Fiflurel.15 Représentation idéale
d ’un oscillateur harmonique simple où est l'a m p litu d e et m ’ est la fréq u en ce an g u laire de l ’o sc illa te u r am o rti. C ette
amorti. Une ailette inintergée frét)iicncc angulaire est donnée par
clans un liquide exerce une toree
de résistance sur le bloc lorsque к h'
ce dernier oscille parallclemcm
à r«xe des x.
Ч. w Arn ’
(I 41)
16 Chapitre I Les oscillations

Si b = 0 (a bsence d ’am o rtissem en t), l'é q u a tio n 1.41 se réd u it alo rs à l'éq u a tio n I
(w = v/i/rn) quant à la fréquence angulaire d ’un oscillateur non am oiti, ei rét|iiiuion 1
se réduit à l’équation 1.3 quant à la position en fonction du tem ps d ’un oscillateur r
amorti. Si la constante d ’am ortissem ent est petite sans cire nulle (de stnic que b ^/ki
alors Q)' ~ w.
On peut considérer l’équation 1.4Ü com m e une fonction cosinus d ont l’amplitui
qui est dim inue gniduellcm ent avec le tem ps, com m e le suggère la figure 1.
D ans un oscillateur non am orti, l’énergie m écanique dem eure constante et est d o n t
par l’équation 1.21 ( £ ~ 't t j , ) . Si l'o scillateu r est am orti, l’énergie m écanique n ’e.st j
con.stante m ais dim inue avec le tem ps. Si l’am ortissem ent est faible, on peut trouver £
en rem plaçant ,v„ dans l’équation 1.21 par l’am plitude des oscillations am<
tics. A insi, on déterm ine que

EU) « hîim (1.42

ce qui in d iq u e q u ’à l'in s ta r d e l ’am p litu d e , l’é n e rg ie m écan iq u e d im in u e d e façi


exponentielle avec le temps.

^V ÉR IFIEZ VOS CONNAISSANCES 5: Voici Iroi.s groupes de valeurs representam la constante


de rappel, la constante d'amortissement et la masse de l'oseillMeur amorti de la figure l.l.*).
Classe/, en ordre décroissant les groupes de voleurs selon le temps que prendra l’énergie
mécanique pour diminuer d'un quart de sa valeur initiale.

Groupe l 2kf, bu m„
Croupe 2 bho
(jToupe 3 3^0 3fco

Exemple 1.7
Hans rosi'illaieur harmonique amorti de la figure 1.15, m = 250 g,
A =” 85 N/ni cl h = 70 g/s.

û) Quelle est la période du mouvement ?

SOLUTION Le tontepl de utilisé ici est le suivant : puisque h ^ y/km


= 4,6 kg/s. la période est approxiniaiivenieni celle d’un oscillateur
non amoiti À l’aide de réquation 1.13, on obtient alors

= 2 r r ^ /^ = 0 ..3 4 s . (réponse) figure 1.16 La fonction donnant la position .»(r) de l’oscilJateur


de la ligure 1.15, avec les valeurs indiquées dans l'exemple 1.7.
b) Combien de temps faut-il pour que I'anipliliide des oscillation.s L'amplitude, qui es! de diminue de façon exponentielle
unioitics diniiime jusqu’à la moitié de sa valeur initiale avec le temps.

SOUTTION: Le tontept tl* à utili.ser dans le présent cas est celui-ci;


c) Combien de temps faut-il pour que Péiicrgic mécanique diminuí
l’anipliliide à l'in.s'tani i, donnée par l’équation 1.40. est
jusqu’à la moitié de sa valeur initiale'.'
Kllc a la valeur Xn, à r = 0. Donc, on doit trouver la valeur de r
correspondant a SOlUnON; Ici, on exploite le tonreptdé suivant ■selon l’équation I 42.
y g! -- t y l’énergie mccnniqiie à l’instant t est Sa valeur est
de iltv¿ à r = 0. Donc, on doit trouver la valeur de t correspondant à
F.n siinplifiani et en prenant le logarithme naturel de chaque membre
de réqualion résultante. iMi H In ^ (Kiiir le membre de droite et
Si on divise les deux membres de cetie équatioti par et qu’on lu
I n f e - * " ^ ) - -ht/2nj
résout pour trouver / comme on l’a fait précédemment, on peut
pour le meirihro de gauche. Doiit’, déterminer que

—m In r -(0 .2 5 kg)(ln J)
—7.Ж In i -(2K0.25 kg) (In i ) = 2.5 s. (réponse)
I = -"J. - = 5 0 s. (rcpon.se) h ~ 0T070 kg/s '
0-070 kg/s
Cela équivaut exactement à la moitié du temps calculé en bi. ou
Puisque T —0..T4 s. la diminution de l’amplitude correspond à environ à envinm 7,5 périodes d ’oscillation. La figure 1 lf> illustre cet
13 périrwies d ’oscillation. exemple.
1.9 Les oscillations forcées et la résonance 17

1.9 Les oscillations forcées et la résonance


Une personne qui se balance sans q u ’aucune autre personne ne la pt)ussc esi un exem ple
d'oscillation libre. T outefois, si q u e lq u ’un im p rim e p ério d iq u em en t une p o u ssée au
balancem ent, ce dern ier a des oscillations forcées, ou dirigées. Deux fréquences angu­
laires .sont a s sœ ié e s à un .système produisant d es oscillations fo icées : 1 ) la fréquence
angulaire naturelle œ du systèm e, q ui e st la fréquence angulaire à laquelle il o seillenut
après av o ir etc m om en tan ém en t p ertu rb é, el 2) la fréq u en ce an g u laire en,, de la force
d ’entraînem ent externe causant les oscillations forcées.
On peut utiliser l’équation 1.15 |xnimepréNcnter un oscillaicui harmonique simple itlcal.
.si on suppose que le support rigide peut se déplacer vers le haut et le ba.s à une fréquence
angulaire variable (o^. U n tel oscillateur forcé oscille à la fréquence angulaire de la
force d ’entraînem ent, et sa position x(t) est donnée par

xfrl = x „ cosfeu,,« + (¡>). (1.43)

où est l’am plitude des oscillations.


L ’am p litu d e du m o u v em en t x,„ d ép en d d ’une fonction co m p lex e d e eu,, c l d e to.
Le moflule de la vitesse m axim ale v,„ des oscillations est plus facile à deenre : il atteint
sa valeur m axim ale lorsque

II). - (0 (la résonance). (1.44)

une co n d itio n n o m m ée r é s o n a n c e . L’éq u atio n 1.44 ex p rim e clic au ssi la c o n d itio n


approximative o ù l ’am plitude du m ouvem ent ¡r,„ des oscillations est m axim ale D onc, ,si
vous effectuez une poussée de balanc-cment à sa fréquence angulaire naturelle, ram p liu id e
du m o u v em en t et le m o d u le d e la v itesse aiig m cn ieu m t p o u r a ttc m d rc des v aleu rs
élevées ; il s’agit là d ’une chose que les enfants apprennent rapidem ent à force d'essayer.
Si vous effec tu ez une p o u ssée à d ’autres fréq u en ces an g u laires, q u ’e lles .soient pki.s
élevées ou plus bits.ses. l’am plitude du m ouvem ent et le mrxliile de la vitesse seront plus
faibles.
La figure 1.17 m ontre com m ent l’am plitude du m ouvem ent d ’un oscillateur dépend
de la fréquence angulaire co^ de la force d ’entraînem ent, en fonction de trois valeurs du
coefficient d ’am ortissem ent b. N otez que, dans les Iroi.s cas, l’am plitude est m axim ale
to u t près de la v a le u r (Oe/m ~ 1. c ’e s t-à -d ire lo rsq u e la c o n d itio n de réso n a n ce est
rem plie. L es courbes d e la figure 1.17 m ontrent q u ’un am ortissem ent m oindre produit
un p ic de résonance plus haut et plus étroit.
T o u tes les stru c tu re s m éca n iq u es ont une ou p lu sie u rs fré q u en c es angulail'cs
naturelles, et si une structure est soum ise à une grande force d ’entiaînem cnl externe qui
correspond à une de ses fréquences angulaires, les oscillations résultantes de la structure

Figure 1.17 [.’amplitude du nx>uvcment


d'un ascillateur forcé varie lorsque
la fréquence angulaire w, de la force
exercée est modifiée. L’amplitude
est maximale appmsinuitivement
à mJ i» = 1, ce qui est la condition
de résonance. I ,e\ courbe.s
coriie,sporKJcnt à trois valeurs
de la constante d’amortissement h.
18 Chapitre I Les oscillations

peuvent causer sa défaillance. Par exem ple, les concepteurs d ’avions doivent .s’assure
q u 'a u c u n e d e s fréq u en ces a n g u la ire s n a tu re lle s à la q u elle un e a ile p eu t o sc ille r n<
correspond à la fréquence angulaire des m ineurs durant les vols. U ne aile qui vibrerai
v io le m m e n t lo rsq u e le m o teu r nlieini c e rta in e s v ite sse s se ra it év id em m en t trè
dangereuse.
I .e trem blement de terre qui survint à M exico en .septembre 19X5 constituait un .séisme
m a jeu r (8.1 su r l'é c h e lle d e R ich ter), m ais les o n d es sism iq u es ré.suliantes auraien
norm alem ent été trop faibles pour eau.ser des dom m ages étendus en atteignant la vilk
de M exico, située à en v iro n 4(M) km de l’épicentre. T outefois, une bonne partie de Is
ville a été édifiée sur un ancien lit de lac, où le sol est m euble et hum ide Si ram plitiuU
des ondes sism iques est d em eu rée faible d an s le sol as.se/ stable .séparant l’épiccntrc
de M exico, elle a, p a r co n tre, au g m en té de m anière sig n illc aliv e dans le sol m euble
soutenant la cité. Le m odule de l’accéléiation des ondes atteignit ().20g, et la fréquence
angulaire é tait su rto u t co ncentrée au to u r d e 3 rad /s, ce qui en éto n n a plus d 'u n N on
.seulement le sol ircn ib la-i-il fortem ent, m ais p lu sieu rs éd ifices de h au teu r m oyenne
cu re n t des fréq u en c es an g u laires d e réso n a n ce d 'e n v iro n 3 rad /s. L a p lu p art de ces
édifices s'effon d rèren t lors du trem blem ent de terre, alors (]uc les édifices m oins élevés
(dont les fréquences an g u laires de résonance étaient plus élev ées) et les éd ifices plus
hauts (dont les fréquerK es angulaires de résonance étaient m oins élevées) lésisièrent.

RÉVISION E T R ÉSUM É . ^^

! m j'rvtiui’nce l.a friiitiencef à'\.m mouvciiKnt périodique ou 'm


o.scillaluire est le noiuhre d'oscillations eWeetué par seconde. Dans le et V— (la penode). (1.13)
sysiiîine SI. elle est mesurée en hertz ;

1 hertz = I Hz = 1 oscillation par seconde = 1 .s~‘. (Il) Un tel système se nomme o.seillatcur harmonique simple, ou oscilla­
teur linéaire.
La période La ¡i^rintlf T est le temps requis pour effectuer une L'énergie Une particule qui décrit un mou veinent harmonique
ascitlAlion compK'te, on cycle. File est reliée à la fréquence par .simple a. en tout temps, une énergie cinétique K ^ et iiiic
énergie potentielle U — En l’ab.senee tie frottement, l’énergie
( 1.2 ) mécanique E = K + U demeure constante, même si A' et V varient.

Les pendules Le pendule de torsion de la figure 1.7. le pendule


L e m o u vem en t h a rm o n iq u e sim ple Dans un mouvement simple de la figure 1.9 et le pendule composé de la figure I. lü sont
tuirmonû/iie simple (MHS), la position .r(/) d'une particule par rapport des exemples de dispositifs effcciuani un iiKiuvement h.armonique
à sa position d'ét|tiilibre est décrite par l'équation simple. En présence de petites oscillations, leurs périodes d'oscillation
sont respectivement
X{t) r„| COS(<<>f + <(>) (la ( 1 .1 )
T - iTTy/yjs, (1.23)
OÙ jr„ est ra m p litu d e du mouvement, la quantité {wt -t <^) est la
phase du inouveiiicnl cl é est la eunstmile (le phase. Lu fréqucnci'
r - 2 7 r v //7 j? . (1.28)
angiilnirc O) est relicc à la période cl à la Ircqiieiice tlu nioiivcment
pal et T = TjnyJ! ¡mgh . (1.29)
2jt
(O = — = IT lf (la Ireqtionce .-uigutairo). ( 1.5 )
m ouvem ent harmonique simple et le m ouvem ent circulaire
La dérivée par rapport au temps de l'équation 1.3 mène aux cqua- uniform e Un mouvemeni hurmonique simple est la projection
lion.s de la vites.se et île l'accélération d'une particule en MHS en d’un mouvemeni circulaire uniforme sur le diamètre du cercle où il
tonction du le.nip.s SC produit. La figure I I 4 meintre la projection de tous les paramétres
du nKXJverncni ciaulaire (position, vitesse et acccléi ation) pour donner
l'j =-----sin(w( <t) dii»itesse) (1 M les valeurs correspondantes du mouvement harmonitjiic simple.
et t(, ^ - o /.x „ C in ( m l t (/<) (raei'<lér.inon) (1.7) I a; m ouvem ent harm onique simple amorti Daas un syucmc
Dans réqnaiion 1 6. la quantité po.silive est le module de viiessc d'ostàllalions réel, l'énergie mécanique f,'diminue durant les oscillntioas
iiiaxiiiiale 1,^ du luoiivement. Dans I équation 1.7. la quantité positive sous fcffet de forces externes, comme une foa-c de résisianee, ce qui
est le module de riterélorution mu.\iinale du nKXJvcment. ralentit les oscillations ci converiit rénergic niéoaniqtitr en énergie
thermique. On dit alors que l'o.scillateur cl son imnivemeni sont
/ 'ovciUateur linéaire Une particule de masse m qui se tiéplat e am ortis. Si la force d'am ortissem ent est domice par = hr. où
en étant soumise à une force de rappel découlant de la lot de I looke, r est la vitesse de I'oscillatcur éi h est une constante d'amortissement,
donnée pur Th. ” slét rii tin moiiventenl harmonkjue simple oit la position de rosi-illafeur est alors donnée par

V
ÎE (la Irequenee angulaire) (M 2 ) .r(/) - Jt,„e cos(w'f + </>), (I 40)
Ouestlons 19

où O)', la fr64Uence angulaire de l’oscillateur amorti, est donnée par Les oscillations forcées et la résonance Si une force d’cnirai
nement externe dont la fréquence angulaire est agit .sur un système
(1.41) oscillant ayant une fréquence angulaire naturelle tu. le système oscillera
Vm 4m^ à la fréquence angulaire tWj. Le mtxlule de la vitesse r„ du système
Si h» constante d’amonissenicnt est petite (h <g alors îu' = (o, atteint une valeur maximale
où O) e.st la fréquence angulaire de l’ascillateur non amorti. l,orsque h CO- — tu, (\M)
est petit, l'énergie mécanique E de l’oscillateur est donnée par
une condition nommée résonance. L'amplitude du mouvement
£(/)*= iikx^„e ( 1.42) est (approximativement) maximale dans les memes conditions.

1. 1 aquellc des relations suivantes entre l’accclcration n, et la position


Xd’une particule corresjxmd à un MHS ; a) o, = O.S.r. b) tj, = 400.t-,
c) ü, — —20x. d) Qj, = — ?
î. Si vous deviez trouver la vitesse ù / = 2 s dans un MHS où
*■= (2.0 m) co.s(5/), feriez-vous la substitution de t avant de dériver
par rapport à t, ou l’inverse ?
3. L'accélération a,{t) d’une particule en MHS est représentée dans
la figure 1. IS. a) Lequel des points indiqués correspond à la position
de la particule quand elle est à —x„? b) Au point 4, la vitesse de la par­
ticule est-elle positive, négative ou nulle? c) Au point 5, la particule
est-elle à — à -t-Xm, à 0, entre et 0 ou entre 0 et -I .ïm ?

/
K ./ 7. Les figures 1.21 a) et b) illustrent, soils forme d’instantanés pri.s au
même mooKait les posittoas de quatre oscillateurs ikaii les masses et les
constantes de rappel sont identiques. Quelle e.st la différence de phase
Figure 1.18 Question .3 entre les deux oscillateurs de a) la figure. 1.21 a) b) entre ceux de la
figure 1.21 b) c) Quel le est la différence de phiLse entre l’oscillateur
4. Laquelle ou lesquelles des propositions suivantes correspond à d> isHige de la figure 1.21 a) et l’oscillateur vert de la figure 1.21 b) ’
dans le MHS de la figure 1.19 a)? a) n < 4> < Trll.h)n < <j><
3 rr/2, c) —3 jt/2 < <f>< -rr? .000000000

é'

\J V /
.•1 »■=
a) b)
.»■= n

a) b)
figure 1.21 Q u e s t i o n ?
Figure 1.19 Questions 4 et 5
8. a) Dans la figure 1 22 a), quelle courbe correspond à I’acailcra
5. La vitesse v,(f) d’une particule en MHS est représentée à la lion a f t ) en fonction de la position xU) d'un oscillateur harmonique
figure 1.19 b), l.a particule cst-elle momentanément immobile, ou sim ple? b) Dans la figure 1.22 bï. quelle courbe corrcsiwnd ù la
se dirigc-t-clle vers —x„, ou ver.s : a) au point A du graphique. vitcvsc i',(i) en fonction de x(t)
b) au point B du graphique? I..a particule est-elle à x,„, à + ï „,.
à 0. entre —x,„ et 0 ou entre 0 et ‘ lorsque sa vitesse correspond n(9
c) au point A et d) au p<nnl B7 Le module de la vitesse de la particule
augmenle-i-il e) au [Kiinf A et 0 au point B ?
6. l-a figure 1.20 indique, en fonction de trois situations, les posi­ xU)
tions rit) d’une paire d’oscillateurs harmoniques simples (A et B \ qui
sont identiques sauf en ce qui concerne la phase. Dans chaque cas, quel
déphasage (en radians et en degrés) est retiiiis pour décaler la courbe
A et la faire coïncider avec la couibe B ? Parmi les réponses passibles,
choisissez le déphasage dont la valeur absolue est la plus faible.
20 Chapitre i Les oscillations

9. Dans la figure 1.23, un bloc A de petite taille repose sur un plus suspension O. Classez les pendules en ordre décroissant selon leur
j>roK bloc S et il y a un coellicient de Irottemenl statique non nul entre péritxle d'oscillation.
les (leux blues. I.c bloc B. t|iii icposc sur une surface lisse, est initiale
ment à .V« ü, et le ressort e„st à sa longueur naturelle ; on tire ensuite
0
le blue Mti une distance d vers la droite, ¡niLs on le relâche. Lorsque
le .système bloc rc.s.surl ciitrcprcm] un MHS d ’une ainpliludc Jtnj.
le bloc A est sur le point de gli.sser sur le bloc B. a) L'accélération » 0 0
du bloc A est-elle constante ou vanable '? b) Le module de la force
de fmiieinent qui fait accélérer A est-il eonstani ou variable?
ù Q 0
ci A ilcvrail-il eiimmcnccr â gli.sser îi v = O, ou â r =
a) h) c)
(I) cSi le (Icplnceiiieiil initial du MHS avail été plus grand que d,
Ir glis.staïuait aiiiail-il alors été plus probable ou moins probable? Figure 1.25 Q u e s t io n 11
(Pirparalion en vue du prolilf-mc 16)
12. (Suite de l’cxcrcicc 36 de la section «Exercices ei problètnes»)
Si la vitesse du (>roja:tile était plus grande, les composantes suivaïues
Surtaco du MHS résultant auraient-elles une plus grande valeur, une valeur
J' frottciiiciit moindre ou la même valeur ? a) l'iunplitude, b) la péiiode. c) l ’énergie
potentielle maximale.
13. Vous devez construire un disposilit de iransferl d ’oscillations
hgurg V23 Q ue^luH i 9
comme celui illustré dans la figure 1.26.1,e dispositif consiste en deux
systèmes bloc-ressort suspendus à une lige flexible Ixirsque le ressort
10. Dans la figure 1.24, un système bloc-ressort est soumis à un MHS
du système 1 est étiré puis relâché, le MHS résultanl du système 1
lots de deux expériences. Dans la première, le bloc est tiré à une distance
fait osciller la tige à la fréquence/,. La tige exerce ensuite une Force
d, de su position d'équilibre, puis relâché. Dans la deuxième, il est tiré
d'entraînement sur le système 2, avec la meme fréquence/,. Vous
il une plus grande distance, soit d ,, de sa position d’équilibre avant
avez le choix entre quatre ressorts dont les constantes de rappel k
d 'etre relâché, a) L'amplitude est elle plus grande, moins grande
sont de 1 600 N/in. 1 500 N/m. I 4(K) K/m cl 1 2(X) N/in. et entre
ou lit meme dans la deuxieme expenence, par rapport à la première ?
quatre blocs dont les masses ni sont de 800 kg. 500 kg. 400 kg
h) et la période? c) la fréquence? d) l’énergie cinétique maximale?
et 200 kg. Déterminez lesquels des 4 ressoits et des 4 blocs devraient
e) l'énergie potentielle maximale ?
être assortis dans chacun des deux systèmes afin de maximi.scr
l’amplitude diîs oscillations dans le système 2.
-.r

t W,-»l
Figure 1.24 Uucstion 10

11. La figure 1.25 présente troi.s pendules composés constitués de


sphères unilormes identiques de même ma.sse, solidement reliées
S)stcinc 1 ' Sysièiiie 2
pur (les liges idenliques de masse négligeable. Chaque pendule est
on position verticale, et est libre de pivoter autour du point de Figure 1.26 Question 13

dl la constante de rappel c) le module de la vitesse maximale du


ynnv/ La solution se trouve sur le site Wef), à l’adrosc ci-dcs.sou.s :
nioiivemcnf et fl le module de la force maximale exercée .sur le bloc
•www.dlcnicgrawliill.ca/physiqnc par le ressort.
41. Quel est le module de l’accélération maximale d’une plateforme
SEfTION 1.3 La deux'iènie loi de Newton oppii()uée 6 un mouvement
(jiii o.seille avec une amplitude de 2.20 cm â une fréquence de 6.6fl Hz "
horiponique simple
St Les oscillations d 'un diaphragme produisent un son musical
IL. 1 In obirt .soiinii-s h im monveiiirnl harmonuiiie simple prend 0.25 s dans un haut parleur. Si l’amphlude des oscillations est limitée à
pour se déplacer entre un point de vitesse nulle et le prochain point 1,0 X lO "' mm, quelles fréquences entraîneront une accélération
oil la vitesse est nulle I a distance entre ces deux points est de 16 cm. ayant un module excédant g dans le diaphragiiK
Ciilaili Z a) la iiriiixlc h) la (’rcrjucrici: cl i ) r<implitudc du mouvement. 6E. L'échelle graduée d'une balance à ressori doni la lecture va
?F f In système blor-ressort os( illaiii mei 0 71s avani de recommencer de 0 a 1.1,0 kg mesure 12,0 cm de longueur, l )n obiet suspendu à la
une nouvelle oscillation. Trouvez a) la période, b) la fréquence cii balance oscille verticalement à une fréquence de 2,(M.' Hz. a) Quelle
hei iz cl c I kl fiétiuciKr angnkiirc en radians par seconde du tnoiivemeni est la consianic de lappcl b) Quel e.sl le poids de l’objet?
d'owilhitinn 7[ Une particule de ma.sse m = 1.00 x lO“” ’ kg oscille en détrivaiit
•If Hn iiv illairin se compose d un hliv de masse m = 0.100 kg un mouvement harmonique simple dont la périixlc est 1.00 x 10 ’ s
relié à un ressort. Lorsijui' les oscillations ont une amplitude et â une vitesse maximale ayant un module de 1,00 ' ' 10-' m/s.
de 35,0 cm. l’oscillateur répète son inoiivciiienl à chatiiie 0..100 s. falciilez a) la fréquence angulaire et h) l’amplitude du mouvement
Trouvez al la jiériode, b) la fréquence, c) la fréquence angulaire. de la particule.
Exercices et problèmes 21

8 t Un petit objet d ’une niasse de 0,12 kg est soumis à un mouvement où T est la pénrxle d’oscillaaon et k est la constante de rappel, b) Lors
harmonique simple d’une amplitude de 8.5 cm et dont la période de la mission Skylab 2, la constante de rappel du dispositif était
est 0,20 s. a) Quel est le module de la force maximale qui s’exerce k — 605,6 N fm ; la période d'oscillation du siège non occupé était
sur lui ? b) Si les oscillations sont causées par un ressort, quelle en est de U.9Ü1 49 s. Calculez la masse effective du siège, c) Li'rsqu’un
la constante de rappel astronaute prend place sur le siège, la période d ’oscillation est
9L La lame d’un rasoir électrique se déplace vers l’avant et l’arrière de 2,088 32 s. Calculez, la ma.s.se de Tastronaute.
en décrivant un mouvement hannonique simple et sur une distance 15P. Dans un port, les vagues soulèvent et abaissent la surface de
maximale de 2.0 mm. La fréquence du mouvement est de 120 Hz. l’océan d'une distance ti (du plus liant au plus bas niveau) en décrivant
Trouvez a) l'amplitude du mouvement, b) le module de la \itesse un mouvement harmonique simple, dont la période est de 12.5 h.
maximale de la lame et c) le module de l’accélération maximale de Combien de temps laut-il pour que l’eau baisse d’iine distance r//4
la lame. par rapport à son plus haut niveau ?
IDE. I.C diaphragme d'un haut parieur oscille en décrivanl un mouve­ I6P. Dans la figure 1.28, deux bloc.s (>« — 1,0 kg et W — 10 kg) et un
ment harmonique simple à une fréquence de 440 Hz et en effectuant ressort {k — 200 N/m) sonl disposés sur une surface horizontale sans
un mouvement d’une amplimde de 0.7.5 mm. Quels sont a) la fréquence frottement. Le coefficient de frnllenient statique entre les deux hlm:.s csi
angulaire, b) le module de la vitesse maximale et c) le module de de 0.40. Quelle amplitude devrait as'oir le mouvement harmonique
l’accélération maximale'* simple du système bkx-resson (Xiin inic le pciii bhx; .soit sur le jxiint
11E. Le châssis d’une automobile esi monté sur quatre ressorts iden­ de glixser sur le gros bloc ?
tiques. Pans une automobile, les ressorts sont ajustés afin que les
oscillations verticales aient une fréquence de 3,IX) Hz. a) Quelle est
la con.stantc de rappel de chaque ressort si la masse de l'automobile
est de I 4.50 kg et est également répartie entre les ressorts ? b) Quelle
' Suiface
c-st la fréquence des oscillations si 5 passagers, ayant chacun une mas.se sans Irottcmenl
de 73,0 kg, se retrouvent à bord de l’automobile (nous considérons encore
une fois que la masse totale est également répartie entre les res.sorts)
Rgutel.28 Pioblème 16
12E. L'n objet oscille en décrivant un rrKXivemcnt haimonique simple
donné par l’équation 17P Un bl(K- SCtnxivc sur une surface horizontale (une table vibrante)
qui oscille d’avant en arrière en décrivant un mouvement ImriiKniique
x(f) = (6,0 m ) cos[(37T rad/s)/ ■+•tt/3 rad],
simple et à une fréquence de 2,0 Hz. Le coefficient de fmnnneni sta-
À ( = 2,0 s, quelles sont a) la position, b) la viies.se, c) l’accéléraiion titiiie entre le blix: cl la surface e.st de 0.50. Quelle amplitude le MHS
et d) la phase du mouvement'.’ Quelles .sont e) la fréquence et peut-il atteindre sans que le bloc glisse sur la siii face'.* www
f) la période du mouvemeni ? 18P. Un bloc repose sur un piston qui sc déplace vcriu Hicincnl en
13L Dans un moteur de loconurtive, le piston de la ciilas.se a une course décrivant un mouvement harmonique simple, a) Si la période du
(le double de l’amplitude) de 0,76 ni. .Si le piston effectue un nxiuvement MH.S est de 1.0 s, à quelle amplitude le bloc et le piston ccs.scront-lls
hannonique simple dont la fréquence angulaire est de 180 tr/min, quel d’être en contact ? b) Si le piston a une amplinide de 5 cm, quelle est
est le module de sa vites.se maximale? la fréquence maximale qui pemicUra au bloc et au piston de demeurer
MP. La figure 1.27 montre un astronaute utilisant un engin de mesure coniiniicllenienl en coniaci ?
lie la masse corporelle. Conçu pour les véhicules spatiaux en orbite, 19P. Un oscillateur se compo.se d ’un bloc attaché à un ressort
cet in.struinent pcnixit aux astronautes de mesurer leur rna.s.sc dans les {k — 400 ^7Itl). À l’instant /. la position (mesurée à paitir du point
coixlitions d’apesanteur qui existent quand ils .sont en orbite autour d’équilibre du système), la vitesse et l'accélération du bloc sont
de la Terre. Le dispositif se compose d’un siège installe sur un ressort : V= 0 ,1(X) m, V, = —13.6 m/s et rt, —123 m/s’. Calculez a) la fré­
l’astronaute mesure sa période d'oscillation sur le siège ; il déduit la quence des oscillations, b) la masse du bloc et c) l’amplitude du mou­
masse à l’aide de la formule décrivant la péritxie d’un système oscil­ vement. WVOA/
lateur bloc-ressort, a) Si M est la niasse rie l’astronaute et ni est la 20P. Un oscillateur linéaire sc compose d’un bloc de masse m ~ 2,0t) kg
masse effective de la partie du disposiiil qui oscille, tlémoniiez que attache à un re.ssort (X - lOü N/in). K t - 1,00 s, la position
M = (k/4rr-)T- — m. et la vitesse du bloc sont v - 0,129 m ei i, * .3,4 15 in/s. a) Quellc
est Tamplitude des oscillations? Quelles éiaiciii b') la position ci
c) la vitesse du bloc it f — 0 s ?
2IP l'n rcsstirt de masse négligeable est suspendu au plafond et un
petit objet est attaché à son extrémité intérieure. L'objet est initialement
maintenu au repos à une position V|, de sorte que le ressort est à sa
longueiirnaturellc. L'objet e.sl ensiiile rclâi.hé et rescille verticalement,
sa pttsilion la plus basse étant 10 cm sous V|. a) Quelle est la frequence
des oscillations ? b) Quel est le module de la vitesse de l’objet
lorsqu’il ,se trouve à 8,0 cm soirs sa position initiale'* c) l n objet de
masse m 300 g est attaché au premier objet, et le système oscille
pal la suite il la moitié de la fréquence initiale. Quelle est la masse du
iwcrnier objet ? <l) Par rapport à v,, quelle esi la nouvelle poMtion
d'équilihre avec les deux objets attachés au ressort'*
22P. Deux particules effectuent un mouvemeni harmonique simple
de même amplitude et de même fréijuence. en suivant deux lignes
Figure 1.27 P ro b lè m e 14 parallèles rappris hées Files sc croisent en allanl dans des directions
22 Chapitre 1 Les oscillations

opprirrccs diiKiuc fois que leur position curresponcl à la moitié de leur b) Si le bloc est tiré légèrement vers le bas à partir de sa position
amplitude. Quelle est leur dilTcrciicc ik phase? d'équilibre puis relâché, quelle est lu période des oscillations
23P. Deux panieules oscillent en décrivant un mouvement harmonique résultantes ?
simple le long d ’un segment d'une droite de longueur A. Les deux 29P Un ressort uniforme, qui a une longueur 1. lorsqu'il n'e.st pas
particules ont une pérunle de t,5 s, mais présentent une drf'fcrencc étiré et une constante de rappel k. est coupé en deux section.s de lon­
de pliasv de ;t/6 rad. al À quelle di.stancc sont-elles l'une de l'autre gueurs (non étirées) L, et Lj, où Ц = nL., Quelles sont les constantes
(p:rr rapport u /\t Ü.5Ü0 s apres que la particule en retard a atteint de rappel correspondantes a) Aq cl b) A; en fonction de n et k">
re.ruidmiie de sa trajeetoire? b) Se déplacent’clics alors dans la Si un bloc est attaché au ressort original, comme on le voit dans
même dircx-iirtn. Tune vers l’autre, chi en s’éloignant l’une de l’autre la figure 1.5. il oscille à la fréquence/ Si le ressort est remplacé par
24P Dans la figure 1.2‘J, deux ressorts identiques dont la t iNi.stanic de la section L, ou Lj, la fréquence correspondame est / , o u /,. Irouvez
ra|i^iel est k sortt attachés à un bloc de masse m, et sont fixés à des c) / | et d ) /, en fonction d e/. www
supports Démontrez que la fréquence des oscillations du bloc sur la 30P. Dans la figure I..12, trois wagons comenant 10000kg de charbon
surface sans frottement est .sont maintenus immobiles par un câble parallèle à la pente, sur une
voie ferrée inclinée de .10° dcs.servant une galerie minière. Le câble
I [n s’étire de 15 cm juste avant que l 'attclagc cède entre les deux derniers
~ Jn\ m ' wagcHts. pcrmcilani ainsi à celui du bas de se détacher. En supposant
que le câble obéit à la loi de Hooke. trouvez a) la fréquence et b) l’am-
?5P Supposez que les deux ressorts de la figure 1.29 ont des plitude des oscillations résultantes dans les deux premiers wagons.
constantes de rappel différentes <r, et k , . Démontrez que la fréquence
de> oscilliiiions du hlix e.st alors donnée par

f - \ l n + f\-
oti/i et}\ sont les fréquences auxquelles le bloc oscillerait s’il n'claii
relié qu’au l’SKsort 1 ou au ressort 2. www

|^«'J■^750í5D5^ f
------------1 —

figure l 29 Problèmes 24 et 25
figute 1.32 Problème .K)
2iP. la: bout d’une tk's pointes d’un diapa.son soumis à un rmnivemenl
harmonique simple d'une fréquence de 1 (K)ü Hz a une amplitude SECTION 1 4 L'énergie dons le mouvement harmonique simple
de 0.40 mni. Trouvez a) le mrxlule de l’accéléralion maximale et
b) le module de la vitesse maximale de la pointe du diapason. Trouvez 311 Trouvez la valeur de l'cnergie mécanique d’un système bloc-
c) le module de l'accélération et d) le module de la vitesse de la pointe ressort ayant une constante de rappel de 1..1 N/cm cl une ampliliKlc
du diapason lorsque la pointe est à la position Jt = 0,20 mm. d'oscillations de 2.4 cm.
27P, Dans lu figure L.'IO. deux ressorts stnii reliés bout à bout et 32L L'n système oscillant bloc-ressort a une énergie méetmique
aiiachés à un hlix- rie niasse m I .a siirlaee est lisse et sans frottement, de 1.00 J, une amplitude de 10,0 cm et une vitesse maximale de
S!i les deux rc.ssorts ont une constante de rappel k, démontrez que 1.2Ü m/s. Trouvez, al la conslanlc de rappel, b) la masse du bloc et
c) la fréquence des oscillatioas.
^ L i l 33E Un objet de 5,(K) kg sur une surface sans frottement c.st attaché à
1tt\ 2m un ressort dotil la constante de rappel e.st de 1 000 N/m. À t ' 0. l’objet
donne la fréquence des oscillations du bloc. se déplace à une distance hori/ontale de 50,0 cm tic son point irrqiii-
librc. et a une vitesse ayant un module de 10.0 m/s en direction du
point d'équilibre a) Quelle e.st la fréquence du mouvement ? Quelles
sont h) l'énergie potentielle initiale du système bloc ressort, c) l'énergie
cinétique initiale et d) l'anipiituilc des oscillations ? www
34f. Un lance-pierres (ficliO de grande taille e.st étiré de l.SO m pour
figure 1.30 РгоЫспк;27 lancer un projectile de 1.10 g à une \ ile.ssc ayant un imxlulc suffisant
pour qu’il échap(TC à raliraction terrestre ( 11.2 km/s) Suppose* que
2BP Dans lu figure 1.11. un bloc les bandes élastiques du lance-pierres obéissent â la loi de Hooke.
jirvani 14.0 ?q plisse sans froiiemeni a) Quelle est la constante de rappel du dispositif si toute l'énergie
sur une pente iireliiiéo de -W)'’. II c.si potentielle élastique est convertie en énergie cinétique ? b) Supposez
relié mi sommet de la pente par un qu'une personne peut exercer oni; force moyenne de 220 N tàimbien
tesson tir ni/tsv- iH?gIigrahle m e s u ­ de |x;r.sonnes faudrait il poui étirer les bandes élu.siique,s
rant O 4X() m lorsqu’il n ’est pas
35E. Un ressort vertical s’étire de 9,6 cm pour attciiivlrc la position
élire, et dont lu Cuttsiuntc de rappel d'équilibre lorsqu'on attachc*un bloc de I..1 kg à stm extrémité,
CM 120 N/ni a) À quelle distance a) Calculez la conslanlc de rappel. bloc est ensuite déplacé
du sommet de la ponte le bloc
de 5.0 cm vers le bas, puis relâché. Trouvez b) la pcriocle. c) la fréquence,
atleint-il sa position d ’équilibre?
d) l'amplitude et e) le module de la vitesse maximale du MHS rt'-sult.mt.
Exercices et problèmes 23

36E. L'n bloc de masse M. au |■ep^>^ sur une table horizontale sans
frottement, est attaché à un support rigide par un ressort de constante k.
Un projectile lie masse m et ayant une vitesse v frappe le bloc, coinnK
cela est illustre dans la figure 1.33. Le projectile s'encastre dans le
bloc. Déterminez a) le module de la vitesse du hliK immédiatement
après fuTipact et b) l’amplitude du mouvement harmonique simiile
qui en résulte.

’ k
m

Figirel.33 Exercice 36

37L Dans un MHS. lorsque la position correspond à la moitié


de ramplitiide quelle fraction de l’énergie mécanique totale est
Figure 1.34 Exercice 42
a) de l'énergie cinétique et h) de l’énergie potentielle? c) À quelle
position l’énergie mécanique totale du système sera-t-elle composée
quelle sera la nouvelle période du .système '? Considérez Irufièzc
en parts égale.s d’énergie cinétique et d’énergie potentielle?
et l ’athlète comme un pendule simple.
38P. Une particule de 10 g effectue un mouvement harmonique
4SE. Un pendule composé est lait d’un mètre rigide qui pivote autour
simple ayant une amplitude de 2.0 x I0 ~^ m. et une accélération
d’un petit trou percé a une distance rf de la marque de 50 cm. 1-a péiimle
maximale de modt île 8,0 x 10 *m/s-. I .a constante de phase est —rr/3 rad.
d’oscillation est de 2,3 s. Trouvez d. www
a) Écrive/, l’équation exprimant la force exercée sur la particule
en fonction du temps, b) Quelle est la période du mouvement? 46E. Dans la figure 1.35. un pendule compose est forme d’un disque
c) Quel e.st le nxxlulc de la vitesse maximale de la particule '' d) Quelle rigide uniforme (tic masse M cl de rayon H) supporté dans un pian
est l'énergie mécanique totale de cet oscillateur harmonique simple ? vertical par un pivot situe à une distance d de .son centre. île disque
est déplacé d’un petit angle, puis relâché. Trouvez l’exiMc.ssion de la
39P*. Un b lix ; de 4,0 kg est suspendu à un ressort dont la constante de
péritxle du mouvement harmonique simple résultant
rappe.l est de 500 N/in. Un projectile de .50 g est lancé dans le bltx:
directement du dessous, à une vitesse ayant un module 150 m/s,
et s’y encastre, a) Trouvez l'amplitude du mouvement harmonique
Pivot*
simple qui résulte de l'impact, b) Quel pourcentage île l’énergie ciné­
tique initiale du projectile est transforme en énergie mécanique de
foscillateuT liarmoiiique ? www

SECTION 1.5 l'o « illa te u r hormonique s'imple ongtiloire

40E. Un disque uniforme, plat et circulaire, a une masse de 3.00 kg Figure 1.35 E x e rc ic e 4 6
et un rayon de 70.0 cm. Tl est dans un plan hoiiaontai et suspendu à
un fil vertical fixé à .son centre Si on impose au disque une rotation 47L Un pendule sc compose d ’une longue tige mince hninngène
de 2.50 rad par rapport au fil, un moment de force de 0,060 0 N ■m de longueur L cl de masse m, qui pivote autour d’un point sur cette
est requis pour garder cette orientation. Calculez a) le moment d’inertie tige situé a une distance d au-dessus du centre de la tige, a) Trouvez
du disque par rapport au fil, b) la constante de torsion et c) In fre- la péritxle de ce pendule en fonction de d, de L de m et de g, en sup­
queiKe angulaire de ce pendule de torsion lorsqu'on le laisse osciller. posant que l'oscillation est de faible amplitude. Uhi’am vc-t-il à la
41P I.a roue de balancier d'une montre oscille avec une ainplilude péritxle b) si d diminue, c) si l. augmente ou d) si m augmente ?
angulaire de n rad et une période de 0,5(X) s. Trouvez, a) le module de 48E. Un disque uniforme de rayon R - 12,5 cm e.st suspt'ndu. comme
la vitesse angulaire maximale de la rmie, b) la vitesse angulaire de la un pendule composé, par un point siiué sur sa ciuconfcrcnt c ai Quelle
roue lorsque sa position angulaire est de n il rad et c-) le module de esi sa période'* b) À quelle di.stance radiale r < /7 y a-t-il un pivol qui
l’accélération angulaire de la roue lorsque sa position angulaire tltx in eraii la même péritxle?
est de ;r/4 rad.

SECTION 1.6 les pendules


42E. Dan.s la figure 1.34, un boulet de démolition de 2 5(X) kg se
balance à l’extrémité du câble d’une grue I æ section du câble
qui se balance, mesure 17 m. a) Trous'ez la [sénode de l’oscillation,
en eonsidcranl ce système comme un pendule simple, bj La période
dépend-elle de la masse du IxHilet?
43E CHielIc est la longueur d’un pendule simple qui indique les secondes
en complétant un balancement complet gauche droite-gauche toutes
les 2.0 s ?
44L Une athlète assise sur un trapèze se balame de rarrièie vers l’avant
en décrivant un moiivemoni d'une période de 8,RS s. Ri elle se lève,
et hausse ainsi le centre de masse du trapèze et le sien de 35,0 cm. fiÿire 1,36 Exercice 49
24 Chapitre 1 tes oscillations

49t Le i>cndiile de la tigure I se compiise d’un disque uniforme S7P. Un pendule simple de longueur R se déplace en torniunt un an
d ’un rayon de 10,0 cm el d'une masse de 500 g atiaché à une de cercle, a) t n considérant que l’accélération radiale du |iendule
lige unitorme dont la longueur est 500 mm et la masse est 270 g. lorsqu’il passe par sa position d’équilibre, correspond à celle d'ui
al Cálenle/, le moment d ’inertie du pendule par rapport au pivot, mouvement circulaire uniforme {r/R), démontrez que la tension dt
b) Quelle est la distance entre le pivot et le centre de masse du la corde est ilc mg{ 1 + ()•„) à celle position si l’angle *=^l faibli
Iteiidulc? cl Calcule/, la péritxlc d’oscillation. amplitude. (Voir « Dévek^pemem des fonctions irigonomélriqucs>
5tl£. al Dan.s rcxcinplc 1 5. si le pendule composé est inversé et sus­ dans l'annexe D.) b) Dans le cas d ’autres positions du }>cndulc
la tension est elle plus grande, plus petite ou la même ?
pendu au point P, quelle est .sa période d’o.scillalion ’’ bl La période
csi-elle plus grande, plus petite ou opale à la période initiale ’? 58P Une roue peut tourner librement autour de son essieu fixe
Un resson est attaché à un de .ses rayons à une distance r de I’essieii.
51E. Dans 1 exemple 1.5, on a ' u que le centre d ’oscillaiion d'un
comme le montre la figure 1.39. a) En considérant la roue comme
pendule composé est situé il une distance 2L/3 de son point de
un cerceau mince de mas.se m et de rayon R. trouvez la fréquence
suspension Démontre/ que la distance entre le point de suspension
angulaire des laibles oscillations de ce système en fonction de m. de R,
Cl le centre d'oscillaiioii d’un pendule composé de n’importe quelle
de r. et de la constante de rappel k. En quoi le résultat change-t-il
fui inc est Hmh. où / et h représentent les valeurs indiquées dans
si b) r = /? cl c) r = 0 ?
l’ciiiiatinii 1.29. cl ni esi la masse du pendule.
52f l'n bâton de longueur L oscille comme un pendule composé, pivo-
Imii miumi du iniini O. comme le montre la figure 1.37. a) [Jérivez Í
l'expression de la pciiodc du |x:niliilc par rap[XHi à f. et â r. qui est la
distance entre le pivot et le centre de niasse du pendule, b) À quelle
vidrin de r//, coi respond la période minimale ? cl Démontrez que si
L ~ l.(K) III cl g — 9.80 ni/s’, celle péiiiKlc minimale est de I ..5.1 s.

Figure 1.39 Pi oblcinc SX

SfCTION 1.8 Le mouvement harmonique simple amorti


59E. Dans l'exem ple 1.7, quel est le rapport entre l’amplitude des
oscillations amorties el l’amplitude initiale, q^rcs 20 oscillations
complètes?
60E L’amplitude d 'un oscillateur légèrement amorti diminue de
3% à chaque cycle. Quelle fraction de l’énergie mécanique de
l’oscillateur est periliic après chaque o.scillalion complète
61E, Dans le système illustré dans la figure 1.15. le bloc a une masse
de 1.50 kg et la constante de rap(iel est de 8.00 N/m La force
d’aniortis.siuiieni est donnée par bçdxldt). où h = 230 g/s. Supposez
que le bloc est initialement tiré vers le bas- à pariir de sa position
53P. Dans la figure I . IK. une longue tige uniforme de longueur L et
d’équilibre sur une distance de 12,0 cm. puis relâché, a) Calcule/ le
de masse m peut pivoter librement dans un plan horizontal, autour
temps requis pour que ranipliiudc des oscillations résultantes
d’un axe vertical qui pas.se pai- son centre. Un res.sort, dont la constante
diminue jusqu'à un tiers de sa valeur initiale, b) Combien troscilla-
de rappel est k, est relié liorizoïilalement à une extrémité de la lige et
tions le bliK a-t-il alor.s complétées
à un mur Пхе. Loixque la lige esi au poini d’équilibre, elle est parallèle
au mur. Quelle est la période des petites oscillations qui se prtxliiiscnl 62P. Vous examinez les propriétés oscillatoires du système de suspen­
lorsqu'un fait pivoter légèrement la tige el qu’on la relâche? wv/w sion d'une autoniobilc de 2 (XK) kg. loi suspcn.sion s’alTaissc île 10 cm
lorsqu’elle supporte cntièienienl rautomobilc. D’autre part l'amplitude
Mur des oscillations diminue de .50% durant une oscillation complète.
Estimez les vuleurs a) de la constante de rap|'x:l k el bl de la constante
d ’amortissement h dans le système amortisseur-ressort d ’une roue,
en supposant que chaque roue supporte 5(X) kg.
Axe d e to ta tio n

SECTION 1.9 les osiillDtions forcées el b résononce


Figute 1 38 FVnhlème S3
63E Dans l'équaiioti 1.43. supposez que l'ainplitudc A'n,esi donnée par
54P. l Tn pendule composé de longueur L et de masse m est suspendu
<l!ins une automobile qui sc déplace à une vitesse de module ci'iistanl __________ f n ______
1 en formant d'un cercle de rayon R. Si le pendule oscille faiblement lwj-(it)j —oA)~ + ’
dans la dirccliuii radiale amour de sa position d'équilibre, quelle est où csr le moduli: (crsnsiante) de la force extci iie exercée p-ar le siippon
VI Iréiiiicnce d'nu-illaiinn ? rigiile sur le res,sorl dans la figure 1 15. À la resonance, quels sont
5SP. Quelle est la fréquente d un ixndiilc coni|xisé d’une longueur a) l’amplinidc et b) le nuxliile de la vite.sse maximale de l’objet oscillant ?
lie 2.0 ni al dans tiiw* chambre, bi dans un a.scpnsour dont l'accéléra- 64P. Une automobile de I (KXI kg traasportaiit quatre personnes de 82 kg
iinn dp 2.(1 m/c- vers le haut est el c) en chute libre ? se déplace sur une roule cahoteuse parsemée de nids de poule à tons
56P Rdiiiivcmcni â un pendule simple, trouvez l'amplitude angulaire les 4,0 m, ce qui la lait rebondir sur ses ressorts de su.spension.
à laquelle le moment de force de rappel requis pour un numvenicnt L’automobile rebondit av-ec une amphtude maximale lorsque sa vitesse
harmonique simple dévie de 1.0 '^ par rapport au moment de force est de 16 km/h. Elle s’arrête et les quatre passagers en dcsccndeni.
de rappel réel du pendule. (Voir l'annexe D, « Dévcloppeineiil des De quelle h.-uiteui l’aiiiomobilc s’élève-t-elle .sur ses re.s.sorts de
fondions trigononictriqucs ».) suspension à la suite de cette réduction de la masse ?
■'sar'jz^ 2 Les ondes

Lorsqu'un insecte se déplace à quelques dizaines de centimètres du scoq)ion des sables, celui-ci se retourne
immédiatement et bondit ■“
vers lui (pour s'en délecter).
Le scorpion agit ainsi sons
voir ni entendre l'insecte
(il s'agit d'un animal
nocturne).

Comment le scorpion peut-il


localiser sa proie aussi
précisément?
La réponse se trouve dons re chcpirc.
26 Chapitre 2 Les ondes

2.1 Les ondes et les particules


P our entrer en contact avec un am i éloigne, on p eu t lui écrire une lettre ou lui téléphoner.
Le prem ier choix (la lettre) im plique le co n cep t de « p articu les » : un objet m atériel
.se déplace d ’un endroit à un autre, tran,sportani d e l’inform ation et de l’énergie. D ans la
p lu p a rt d es c h a p itre s p réc éd en ts, il é ta it q u estio n d e particule.s ou d e sy stèm e s de
particules.
L e second ch o ix (l'a p p el téléphonique) im plique le co n cep t d ’« o n d e s » . qui fera
l’objet de.s prochains chapitres. D ans une onde, l'in fo rm atio n et l’énergie se déplacent
d 'u n point à un autre sans q u ’aucun objet m atériel so it transm is du point de départ au
point d’arrivée de l'o n d e. D urant votre appel téléphonique, une onde sonore transporte
d ’abord votre m essage de vos cordes vocales au com biné de l’appareil. À partir de là,
une onde électrom agnétique prend le relais et circule dans im fil de cuivre, dans une fibre
optique ou dans l’atm osphcrc (et possiblem ent dans un systèm e de télécom m unication par
satellite). A l’arrivée, une autre onde sonore intervient entre le téléphone et l oreille de
votre am i Le m essag e a été transm is, m êm e si aucun o b je t en contact avec vous n ’a
atteint votre am i. Léonard de V inci com prenait bien la nature des ondes. Il avail observé
que, souvent, la vague s'élo ig n e du lieu de sa création, bien que l’eau ne se déplace pas,
et q u 'il en était d e m êm e ties onde.s produites p ar le vent dans un cham p de gram inées,
où on v oit les ondulations traverser le cham p alors que les épis restent en place.
Les particules et les ondes sont les deux grands concepts de la physique classUpte,
car on peut avaxiicr presque tous les phénom ènes que nous pouvons observer avec nos sens
à l’un ou à l'au tre de ces concepts. Il y a une grande différence entre les deux concepts.
Ix; moi particule suggère une infim e concentration de m atière capable de transm ettre
de l ’énergie. Le mot onde suggère exactem ent l’opposé, soit une transm ission d ’cncrgic
qui iilili.se un espace relativem ent grand. On laissera m aintenant le concept de particules
de côté afin de se concentrer su r celui des ondes.

2.2 Les types d ’ondes


Il existe trois principaux types d ’ondes :

1. l^.s o ndes m écaniques. C es o n d es vous so n t fam ilière s c a r vous les ren c o n tre z
so u v e n t; il s ’ag it, n o ta m m e n t, d e s o n d u la tio n s à la su rfa c e d e l’eau , d es o n d es
so n o res cl des o n d es sism iq u es. T outes ces o n d es p o ssè d en t certain e s p ro p riétés
essentielles ; elles obéissent aux lois de N ew ton et ne peuvent exister q u ’en présence
d 'u n milieu m atériel com m e l’eau, l’air ou les roches.
2. Les ondes électromagnétiques. C es ondes vous sont m oins fam ilières, bien qu e voii.s
les utilisiez presque con.stammenl ; elles com prennent la lum ière visible et la lum ière
ultraviolette, Ic.s ondes de la radio et de la télévision, les m icro-ondes, les rayons X
et les onde.s d e s rad ars. C es o n d es n ’o nt pas b eso in d ’un m ilieu m a tériel p o u r
SC propager. L es ondes lum ineuses des étoiles, p ar exem ple, voyagent d an s le v ide
interstellaire avant d ’atteindre la Terre. Toutes les ondes électrom agnétiques voyagent
dans le vide à une vitesse ayant le m êm e m odule c. dont la valeur est

C = 299 792 4.5X m/s (le module de la vitesse de la lumière). (2 1 )

X Les ondes de matière. M êm e si on utili.se couram m ent ces ondes dans la technologie
m oderne, vous les connaissez probablem ent fo rt peu Ces ondes sont a.<wociécs aux
électrons, aux protons et aux autres particules fondaiiK-ntale.s, et m êm e aux atom es et
aux m olécules. C es o n d es .se n o m m en t les o n d es de m atière, ca r on cro it q u ’elles
constituent la m atière.

O n traitera dans ce chapitre de sujets relatifs à tous les types d ’ondes. O n se servira
toutefois des ondes m écaniques p o u r donner des exem ples particuliers.
2.3 Les ondes transversales et les ondes longitudinales 27

2.3 Les ondes transversales


Impulsion
lî et les ondes longitudinales
b-x U ne o nde qui se propage dans une cordc tendue constitue le plus sim ple exem ple d ’une
onde m écanique. Si vous secouez d e h au t en bas l ’extrém ité d ’une co rd e tendue, une
K
onde SC propagera dans la co rd e sous la form e d ’une im p u lsio n , com m e le m o n tre la
figure 2.1 a). C ette im pulsion et son dépiacem eni ne [leiivent se pnitluire que si la conle
est tendue. L orsque vous tirez l’extrém iié de la cordc vers le haut, le segm ent adjacent
est soulevé sous l’effet de la tension en tre les deux segm ents. Ce segm ent se soulève
et com m ence à so u lev er le segm ent suivant, et ainsi de suite. Pendant ce tem ps, vous
av e z co m m en c é à tire r v o tre e x tré m ité d e la co rd e v ers le bas. t a n d is qu e c e rta in s
segm ents se soulèvent succvjssivement, d ’autres com nx;nceni à être tirés vers le bas [lar
des seg m en ts vo isin s. L e résu ltat n et est q u ’une d isto rsio n ( l’im p u lsio n ) se d ép lace
le long de la corde, à une vitesse v.
Si vous d ép lac ez v o tre m ain v ers le h au t e t le bas en effccMuam u n m o u v em en t
— .t harm onique sim ple continu, une onde continue se propage tlan.s la corde à une v iie.v.se v
Puisque le m ouvem ent de votre main est une fonction sinastiidalc pai rapport au tem ps,
fo n d e a une form e sinusoïdale à tout instant donne, com m e fillu s lre la figure 2.1 b) ;
on rem arque que fo n d e a la form e d ’une fonction sinus ou cosinus.
D ans cet ex e m p le , on c o n sid è re se u lem en t un e co rd c « id é a le » , d an s laq u elle
aucune force (de frottem ent, par exem ple) ne vient atténuer ou déform er f o n d e pendant
b)
q u ’elle se propage. De plu.s, on suppo.se que la cordc a une longueur intim e alin de ne
Figure 2.1 a) Une impulsion esi transmise pas tenir com pte de la léflexion dt' f o n d e à l'a u tre extrém ité.
le long d’une corde tendue. Un segment O n peu t étu d ier les ondes de la ligure 2.1 en ex a m in a n t les p ro fils d ’o n d e s (les
de corde (marqué d’un point) se soulève fo n n e s prise.s p ar les ondes) pendant q u 'e lle s se propagent vers la droite. On [loiiitaii
et revient à sa position d’équilibre
aussi exam iner le niouvcm eiu d ’un élém ent de la cotdc qui oscille verticalem ent plaidant
après le passive de l’impulsion.
q u ’une onde le traverse. O n constaterai! qu e le d éplacem ent de tout clém ent d e corde
Le niOLivernent du segment est
oscillant est perjwndicidaire à la direction de la propagation de fo n d e , com m e l ’indique
perpendiculaire à la direction
de propagation de l'onde ; l’impulsion la figure 2.1. Il s’agil dans ce cas d ’un m ouvem ent tr a n s v e r s a l, e t ce type d ’o n d e sc
est donc une onde tramvenale. nom m e alors o n d e tra n sv e rsa le . L a figure 2.2 m ontre com m ent une m ule .smiorc peut
b) Une onde sinasoïdale esi transmise être produite par un piston dans un long lube rem pli d ’air Si vous déplacez rapidcm cnl
par la corde, t Chaque segment de corde le piston vers la d m ite puis vers la gauche, vous pouvez envoyer une im pulsion sonore
se soulève et s’abais.se plusieurs fois dans le tube. Le pi.ston, en se déplaçant vers la droite, déplace les m olécules d 'a ir dans
pendiml le passage de l’onde. L s’agit cette direction et m odifie la pression d ’air à cei endroit. C ette augm entation de pression
également d'une onde transversale. pousse ensuite vers la droite les m olécules d ’air qui sont un p<;ti plus loin dans le tube.
L orsque le piston se déplace s ers la gauche, la pression d ’air dim inue devant lui. Après
s ’clrc déplacées vers la droite, les m olécules les plus proches, puis celles qui sont plus
éloignées, reviennent vers la gauche. I x m ouvem ent de l’air et la variation de pression
se dirigent donc vers la droite dans le tube, à la m anière d 'u n e im pulsion.
Si v o u s p o u sse z et tirez sur le p isto n en effec tu an t un m o u v em en t h arm o n iq u e
sim ple, com m e on le voit à la figure 2 .2 , une onde sinu.sdidalc sc propage dans le tube.
C om m e le m o u v em en t d es m o lé cu les d ’air e.st p ara llèle au d cp lac cm e n l de f o n d e ,
ce m ouvem ent est lo n g itu d in a l, cl il s ’agit alors d ’une o n d e lo n g itu d in a le . D ans ce
chapitre, on sc concentrera sur les ondes transversales et, en jiai m uliei, sur les ondes qui
se propagent dans une c o rd e , dans le chapitre 3, on étudiera les ondes longitudinales,
et plus particulièrem ent les ondes sonores.
I .es ondes transversales et longitudinales sont toutes les deux des o n d es progressives,
parce* q u ’elles ne déplacent d ’un point à un autre, com m e d 'u n e extrém ité à l’autre de la
corde dans la figure 2.1, ou d 'u n e extrém ité à l’autre du tube dans la figure 2.2 N otez
Figure 22 Une onde sonore est produite que c’est fo n d e qui se déplace à la vitesse v', et non la nuatièie (corde ou air) dans laquelle
dans un tube rempli d'air lorsqu'on
elle se propage. C ela est égalem ent vrai pour les petites vagues que f on pntduil lorsque,
déplace un piston vers l’avani
pai exem ple, on jette un caillou dans un étang calm e. Pour vou.s en convaincre, observez
ei l’arricrc. Puisque les oscillations
d’iinc molécule d ’air (représentée le m ouvem ent d ’une feuille m orte déposée à la surface de f e a u ; celle ci effectuera un
par le point noir) sont ]taiallèles m ouvem ent de haut en bas dans les vaguelettes et non le long de la sut lace de fe a u .
à la direction de la pixrpagation Le .scorpion des sables illu stré à la p rem ière p ag e du ch a p itre sc sert des ondes
de l’onde, il s’agit d ’une onde traasversalc-s et longitudinales pour lix aliser .sa pioic. L o rsq u ’un insecte rem ue le .sable,
lonnitudinale. m êm e faiblem ent, il transm et des im pulsions à la surface du sable (voir la figure 2.3).
28 Chapitre 2 Les ondes

U n groupe d ’im pulsions longitudinales se déplace à une vitesse d tuu le m odule est
v'i = 150 mJs. U n autre g am p e d ’ondes transversales se déplace à une vitesse ayant un
m odule V, = 50 m/s.
Le scorpion, avec ses 8 pattes écartées form anl un cercle d ’environ 5 cm de diam ètre,
intercepte les intpulsions longitudinales plus rapides, et déterm ine la direction de l’in.secte ;
cette direction lui est indiquée par la patte qui a été perturbée en prem ier par les impulsions.
Le sc<»pion analyse ensuite l’intervalle d e tem ps A/ entre cetic prem ière ina'rccpiion et
l’interception des im pulsions transversales p ías lentes, et peut déterm iner la distance d
qui le sépare de l’in.secte. C elle distance est donnée par

A / = -------------,
Vi V|

HgWi 2.3 Un mouvement de l'in.sccte ce qui donne


fitivoie des impulsions longitudinales
M[)ides et dev iinptiNions transversales (I — (75 m/s) Ar.
plu* Iciilcft ù la surfaec du sable.
I.e scorpion dos sables intercepte
P ar exem ple, si A/ = 4 .0 m s, alors d = 30 cm , ci cela indique au scorpion la position
d'abord les iinpidsious longitudinales;
exacte de l’inscctc.
dan* lu pié.seni ua.s. c'esi la patte
aiTiOro droite qui détecte en premier
1rs iiiipiilsions
2 A La longueur d'onde et la fréquence
Pour décrire de façon com plète une onde .se propageant dans une corde (et le m ouvem ent
de tous les petits segm ents de la corde), on doit avoir une fonction qui donne la forme
de l ’o nde. C ela sig n ifie q u 'o n a besoin d ’u ne relation d e la fo rm e y — h(x, t), où on
exprim e le déplacem ent transversal de tout segm ent de co n le p ar rapport à sa position
d ’équilibre, com m e une fo nction h du tem ps / e t de la p osition x du petit seg m en t de
corde. En général, une form e sinusoïdale com m e celle de l ’onde de la figure 2.1 b) peut
être décrite, avec h, com m e une fonction sinus ou cosinus ; les deux donnent la m êm e
form e générale à l’onde. On utilisera, dans ce chapitre, la fonction sinus.
Im ag in ez une o n d e sin u so ïd a le co m m e c e lle d e la fig u re 2.1 b) se p ro p ag e an t
dans la direction positive de l’axe des x. Q uand l’onde traverse les élém ents successifs
(.soit de très c o u n s segm ents) d e la cordc, ces élém ents oscillent p arallèlem ent à l ’axe
des y. O n p o se q u e la co rd e ten d u e est .sur l’axe d es x lo rsq u ’e lle est en éq u ilib re
(c’est-à-dire lorsqu’il n’y a pas d ’onde qui s’y propage) ; donc y = 0 pour tous tes élém ents
de la corde à leur position d ’équilibre. À l’instant t, le déplacem ent y (par rapport à la
position d ’équilibre) de l’élém ent situé à la position x est donne par

y(X-, t) = .Vn, Sin(fcx — (01 + (j)). ( 2 . 2)

où Vn,. k, (O et </> sont des constantes ; leurs significations c l la façon de d éterm iner leurs
valeuiN font l’objet des paragraphes suivants. Puisque celte équation est écrite en fonction
de la position x, clic peut se rv ir à trouver les d éplacem ents de tous les élém ents de la
corde en fonction du tem ps /. D onc, elle peut indiquer la form e de l’onde à tout instant
donné, de m êm e que les ch an g em en ts d e form e à m esure qu e f o n d e se d éplace dans
'\ itim r la co rd c. L es n o m s et les d e sc rip tio n s d es q u an tité s d e l’éq u atio n 2.2 so n t in d iq u es
IV lii \ ».•HÎîlum à la figure 2,4.
l PlM't. A vant d ’en d iscu ter, to u tefo is, on d o it e x a m in e r la fig u re 2,5. q ui p résen te cinq
3 « Instantanés» d ’une onde sinu.soïdale se propageant dans la direction positive de l’axe
des i. Le m ouvem ent de fo n d e e st indiqué par la progression vers la droite de la petite
MrimKv // ^
/ (.onstantc flèche orientée vers une crête de l’onde. U ’un instantané à l’autre, la petite llèche se déplace
/ cU' jilift«* vers la tlroiie avec la fo rm e d e f o n d e , m ais la co rd e ne se d ép lace qu e parallèlem ent
Ptidiiion à l'axe des y. Pour vérifier cette observation, on .suit le m ouvem ent de l’élém ent de corde
FircjiKm i rouge situe à V = 0 . D an s le p rem ier instantané (v o ir la figure 2.5 a)), il est à v 0,
•iimiiUiirt' D ans le deuxièm e, il est extrêm em ent bas parce q u 'u n creux (ou une vallée) de fo n d e le
hgur* 2.4 Les noms de* guantitc.s traverse. Tl passe ensuite de nouveau par v = 0. D ans le quatrièm e instantané, il est à x m
de réqualion 1. 7. ilaiis le cas d une (>nde plus haut point parce q u ’une crête (ou un somnu t) de fo n d e le traverse. D ans le dernier
sinusoïdale transversale instantané, il se trouve de nouveau à v = 0 et a elfèelu é une oscillation com plète
2A La (ongueur d'onde et (o fréquence 29

i --------- ► L’ampUtude et la phase


.A js .
A- A, l. a m p l i t u d e d ’une onde, com m e celle de la fig u re 2.5, co rresp o n d au m o d u le du
dcplaccm cnl m axim al des élém ents de la co rd e à p artir de leurs positions d ’équilibre
\J V l------ A-------i
pendant le passage de l’ondc. (L ’indice m signifie « m axim um ».) P u isq u ’il s 'a g it d 'u n e
am plitude, v„, est toujours une quantité positive, m ême lorsqu'elle est tne,surée vers le bas
et non ver.s le haul, com m e l'illu stre la figure 2.5 a).
La phase d ’une onde est Varmunmt kx - ojt t d>de la fonction sinas de l ’cquation 2.2.

A ’ . A A Au m om ent où une onde traver.se un élém ent de cordc à une txisition x donnée, la phase
change linéairem ent avec le tem ps t. C ela signifie que la valeur du sinus change égale
r A i \J l>)
\ J \ ment, oscillant entre + 1 e t — 1. Sa valeur positive extrêm e (sin(Lv — cot + 4>) ~ ■•■1)
correspond à une crête de l'o n d e qui tiaverse l’éléincnt ; donc, la valeur m axim ale do v
à une position x donnée est Sa valeur négative ex trêm e (.sin(Lt — ou -r tf>) — —
l
correspond à un creux de fo n d e qui traverse félé m en i ; donc, l.i valeur m inim ale do y

A A P
à une po sitio n x d o n n ée est —yy,. L a fo nction sinus d 'u n e o n d e, dont la v aleu r d e la
phase varie dans le tem p s, co rresp o n d donc à l ’o scillatio n d ’un élém en t de la cordc.
■J 1M c)
V et l’am plitude de fo n d e déterm ine les déplacem ents extrêm es de f c lé m e n t

^ i La longueur d'onde et le nombre d'onde


L a lo n g u e u r d 'o n d e / est la d istan ce (parallèlem ent k la direction d e p iopavation de
\ V //1 V/ v / fo n d e ) entre deux élém ents successifs de la corde qui se com portent de façon identique
d ans le tem ps (en tre deux crêtes succes.sives, p ar ex e m p le ). U ne lo n g u e u r d o n d e A
<i) typique est indiquée à la figure 2.5 a), qui représente un instantané d ’une o n d r à f in.stant
i i = 0 . À eel instant, l ’équation 2.2 donne

y(x. ü) = >'n, siniLr + (f)). (2.3)


A -A
V V V. e)
P ar définition, le déplacem ent y est le m êm e pour deux élém ents de la corde distants
d ’une longueur (fo n d e, soit à x x, et à a: = Xj + A (voir la figure 2.5 aj). l>onc. l’équa­
tion 2.3 donne
Figure 2.5 Cinq « instantanés »
d'une unde sinusoïdale se déplaçant Vn, sinikxi + 4>) = >V sinf/rl-ïi A) f c&l
dans une corde dans la direction = ,V„, sin(Lï, + A’A + (&■). (2.4)
positive de l'axe des x. L’amplitude
>•„, e.sl indiquée, de même qu’une P u isq u ’un c y c le c o m p let d ’une fo n ctio n sin u s co m m en ce à se ré p é te r lo rsq u e sa
longueur d'onde À, mesurée à partir pha.se (ou son arg u m en t) est au g m en té e de 2 n rad , on d o it avoii AA = 2:t afin qu e
d’une position arbitraire r , . sin(Lt, + (f)) soit égal à sinfLci + AA + </>), donc

, I tt
k =— (le nombre d’onde). ( 2 .5 )
A

On nom m e A le n o m b re d ’o n d e rie f onde ; en unités SI, il est e.xprimé en rad/m (radians


par m ètre). (N o tez qu e le sy m b o le A ne d ésig n e pas ici la co n stan te d ’é lasticité |d e
rappel], com m e prêt édem m ent.)
On rem arque que fo n d e de la figure 2.5 se déplace de ^A entre deux instantanés.
D onc, au cinquièm e instantané, elle s'e st déplacée de 1A vers la dioite.

La période, la fréquence et la fréquence angulaire


La figure 2.6 e.st un graphe du déplacem ent y (de f éejuation 2.2) en fonciion du tem ps t
à une prvsition d<mnée de la corde, soit.v — 0 . ,51i vous o b serv e/ la corde, vous v erre/ que
l'é lé m e n t de la co rd e à c e tte p o sitio n se d ép lac e v erticalem e n t en un m o u v em en t
Figure 2.6 Rcprcscniaiion graphique harm onique sim ple donné par l’équation 2.2 avec .x 0 ;
du déplacement de l'élément de corde
situé à A' 0 en fonction du temps, y(0 , /) — y,^ sin( (Ot </>)
au moment où fonde .sinusoïdale -------V,„ sin(o)/ - (f>). ( 2.6 )
de la figure 2.5 le traverse.
L'amplitude v„ est indiquée, où on a utilisé le fail que sin( cr) - sin (o'), où a e.st un angle queletm que L a figure
de même qu’une périixlc T. mesurée 2.6 est un graphe de cette é q u a tio n , elle ne représente pas la form e do f o n d e , m ais le
à partir d’un instant donné t,. déplacem ent par rapport au tem ps de l'élém en t de corde situe à a = 0 .
30 Chapitre 2 Le& ondes

On diSHnit la p é rio d e d ’oscillation 7’d ’une onde com m e le tem ps q u ’il faut à tout
clém ent de cordc pour com pléter une oscillation. Une période typique est indiquée dans
le graphe de la figure 2.6. En appliquant Teejuation 2.6 aux deux valeurs extrêm es de cet
interv alle de tem ps, soit à r = fj e t i = f| + T (voir la fig u re 2 6 ), et sachant q u ’à ces
deux temps, le déplacem ent y est le m êm e, on obtient

-> •„ s i n ( w / , - </>) = -> •„ s in lf f j{ r ,+ T) - (f>]


s in (t/j/, — 4>) = s in ( w f | + cdT — 4>). (2.7)

P u isq u 'u n cy c le c o m p le t d ’une fo n ctio n sin u s co m m en c e à .sc ré p é te r lo rsq u e


sa phase (ou son argum ent) est augm entée de 27t rad, on doit avoir (oT = 27t , ou

2n
£t» — — (la fréquence angulaire). ( 2.8)

O n n o m m e o> la f r é q u e n c e a n g u la ir e d ’un e o n d e ; en u n ités S I, e lle est ex p rim ée


en rad /s (ra d ian s p ar seco n d e). O b serv ez d e n o uveau les cin q in stan ta n és d e l ’o nde
p ro g re ssiv e d an s la fig u re 2..‘i. 11 s ’é c o u le un tem p s e n tre d eu x in stan ta n és Au
cinquièm e instantané, chacun des élém ents de la corde a donc effectué une oscillation
com plète.
L a f r é q u e n c e /d ’une onde corrcsponil à 1/7 et est reliée à la fréquecKe angulaire m par

1 < i>

/ = - = -- (la fréquence). (2.9)

C on u n c c ’était le cas pour le m ouvem ent harm onique sim ple, cette fréquence./ désigne
le nom bre d 'o s c illa tio n s p a r u n ité d e tem p s ; d a n s le ca s p résen t, e lle rep rése n te le
nom bre d ’oscillations par unité de tem ps d ’un clém ent de corde traversé par une onde.
D e la m êm e façon que dans le chapitre 1, / c s l habituellem ent exprim ée en hertz (Hz)
ou en m ultiples du hertz, tels que le kilohertz (kHz).

La constante de phase
La c o n s ta n te d e p h a s e </> p o ssè d e une sig n ific atio n an alo g u e à celle du c h a p itre 1.
Sa v aleu r d épen d u n iq u em en t des co n d itio n s in itiales. P ar ex em p le, si on co n sid ère
la form e de la corde à ; = O, l’équation 2.2 donne y( v. 0) = .v„, sin(Lx + </i). Si la form e
de la cordc à cet instant csl représentée par la figure 2..5 a), </>est nulle ca r la foiK tion
sin u s n 'c s l pas d ép h asée (elle a la fo rm e y = v'n, sin (L r), so it y = 0 lo rsq u e x = 0
et elle est croissan te pour les faibles valeurs positives de .x). P ar contre, si la form e de
la corde à r = 0 est repré.scntée par la figure 2.5 d), on rem arque, en observant l’élém ent
de co rd e situé à .r = 0 , q u e la fo n ctio n sin u s est d ép h a sé e de -f nl2 ou d e —3nf2
(faites la com paraison avec la figure 2.5 aj), donc </> = n!2 ou </> = —3Tr/2.

.i.v ^ VÉRIFIEZ VOS CONNAISSANCES 1 : cen e figua- es.


une combinaisofi de teois insl.mlanés, chacun représentam
une onde qui se propage dans une corde. Les pha.scs
tics ondes sont données par a) Zr 4i. b) 4v — 8; et
c) 8.r - 16f. Jumelez chaque plia.se à fonde correspondante.

/ '^O m u a / A/
Onde à 1 - O
Figure 2.7 Deux instantanés de l’onde
2.5 La vitesse d ’une onde sinusoïdale
de la figure 2.5, à l’instant / • 0 progressive
et, ciisuilc, à r instant t St. Pcndiint
qtir rm idc m*déplace vers la droite La figure 2.7 présente deux instantanés de l'o n d e de l’équation 2.2, pris à deux instants
il la viiesee ?, la conrhe entière séparés par un co u n inierv.illc de tem ps Ar. L’onde se propage dans la direction positive
SC déplace d'une distance Ai duianl de l’axe des x (vers la d ro ite dans la figure 2 .7 ); le profil e n tie r de l ’o nde se déplace
l'imci vallc de temps St la? point A tloiie d 'u n e distance A» dans cette tlirection du ran t rin te rv a llc de tem ps S t Le rapport
«accompagne»' l'une des crêtes S x !S t (ou, à la lim ite si S t tend vers zéro, d\/dt) représente la com posante t de la vitc.ssc
de l’onde, mais les elements de la torde de l ’ontlc V,. qui est égale au m odule de la v itesse d e l'o n d e v, puisque A \ et S i sont
ne sc iléplaceiii que venicaletnent. positifs. C om m ent peut-on déterm iner la valeur de v ?
2.5 La vitesse d’une onde sinusoïdale progressive 31

L orsque l ’o n d e d e la fig u re 2.7 se d ép lace, c h a q u e p o in t du p ro fil de l ’o iidc en


m o u v e m en t, co m m e le p o in t A in d iq u é su r une c rê te , c o n se rv e so n d ép lac em e n t y.
(Les points sur la corde n ’ont pas un déplacem ent y constant, contrairem ent aux points
du profil de l ’o n d e.) P u isq u e le point A p o ssèd e un d é p lac em e n t y q u i est co n stan t,
la phase de l ’équation 2.2 générant ce déplacem ent doit dem eurer constante :

kx - cùt + ф ~ une constante. ( 2 . 10)

N otez que. m êm e si cet argum ent (phase) est co n stan t, .t et t changeni tous les deux.
En fait, lorsque l augm ente, x augm ente ég alem ent, d e sorte que I’argum cnt dem eure
constant. C ela confirm e que l’onde se déplace dans la direction positive d e 1 axe des x.
Pour trouver le m odule de la vitesse de l’onde »; on ixem lia dérivée de l’équation 2.10
par rapport au tem ps et on obtient

dx
к - — eu 0
dt

dx Ю
ou ( 2 . 11)
dt к

En utilisant l’équation 2..‘ï (k — 2n/X) et l’équation 2.S (eu — 2jt/T ), on peut écrire
le m odule de la vitesse de l ’onde sous la form e suivante :

(le inotlule de la vitcs.se d’une onde sinusoï'dalc) (2. 12)


k J f

L ’é q u a tio n v = X !T m o n tre que le m o d u le d e la v itesse d e l ’o n d e c o rre sp o n d à une


longueur d ’onde p ar jiériode ; l’onde se déplace d 'u n e distance d 'u n e longueur d ’onde
en une période d ’oscillation.
I .’équation 2.2 décrit une onde se propageant dans la direction positive de T axe des x.
On peut tro u v e r l ’éq u a tio n d ’u n e o n d e se p ro p ag e an t d an s la d irec tio n o p p o sée en
rem plaçant / par —f dans l’éxiuation 2.2. C^cla correspond à la condition

kx + eut + <J>— une eoii.stante. (7 13)

ce qui im plique (si on com pare cette condition avec l’équation 2. 10) que x d i m m e avec
le temps. D onc, une onde voyageant dans la direction négative dt; Taxe dc.s a e.st décrite
par l ’équation

t) = .Vn, sin(Lï + wt + Ф). (2.14)

Si vous analysez l’onde d éen te par l’équatiMi 2.14 a ira m e on vient de le faire pour celle
de l’équation 2 .2, vous découvrirez que la com posante x de sa vitesse est

dx CO
(2 15)
dt T '

L e sig n e n é g a tif (co m p a re z av ec l ’éq u a tio n 2 .1 1) in d iq u e q u e l’o n d e se d ép lac e


e ffe c tiv e m e n t d an s la d ire c tio n n ég a tiv e de l ’axe d es л. et ju stifie q u e l’on ch a n g e
le signe devant le facteur eut.
C onsidérez m aintenant une onde de form e arbitraire, tloiinéc par

\<л. t) - h(kx * u>l). (2.16)

o ù h rep rése n te n ’im porte quelle fo n ctio n , la fo n ctio n sin u s é ta n t un e pc>ssibiHié.


L ’an a ly se p récéd en te d ém o n tre q u e to u te s les o n d es où le s v aria b les x et t snnt
com binées .sous la form e kx ± eut sont d es ondes progressives. D e plus, toutes le^i ondc.s
p ro g re ssiv e s doivent a v o ir la fo rm e de l ’c q u a tio n 2 .1 6 . D o n c, v( a, /) ~ y/ax 4- ht
rep résen te p o ssib lem en t une o nde p ro g ressiv e (bien que cela sem ble un peu bizarre,
du p o in t de v ue de la p h y siq u e). La fo n ctio n y ( x f) = sin (or^ ht), par co n tre ,
ne représente pas une onde progressive.
32 Chapitre 2 Les ondes

Exemple 2.1
Ifiic iMiilc m: iMOpagcani ilans une cnrdc est décrite par et on a, selon l'équation 2.0,
>tJi, t) = 0,IK)3 27 siiH72,1A- - 2,72/ ^ 1,05), (2.17) 1
/ J = 0.433 Hz. (réponse)
2.31 s
tHi les consuintcs numériques sont en unités SI {0,003 27 ni. 7Zl rad/m,
2,72 rail/s et 1,05 raü).
t) Quel est le module de la vitevse de ccHc onde'.'
a) Quelle est l’aniplitudc de cctic itntlc?
SOLUTION: Ix module de la vites.se de fonde est donne pai l’équation 2.12 •
501UT10H On utilise le contepl tié suivant : l’équation 2 .17 a la même
fomie que l'équaiinn 2.2, = = 0,0377 m /s
h 72.1 rad/m
y " Vm sini/iï - wt 1 (2.18) = 3,77 cm /s. (réponse)
on U dont: fltf’airc ü une onde .siiiuMii'dale Kn coniparanl les deux Dans l’équation 2 .17, la phase comprend la variable de position a. ce
équations, on voit que l’uniplitudc est qui indique que fonde se déplace le long de l’axe des X. De plus,
0.003 27 lit - 3.27 mm. (réponse) puisque l'équation d’onde est écrite dans la forme de féqiiation 2.2,
le signe ncgiifi/prcccdam le terme (¡>1 indique que fonde se déplace
b) truelles sont la longueur d onde, la période et la fréquence de cette dans la direction positive de f axe dc.s a . (Note/, que les quantiié.s
onde calculées en b) et en e) sont itidépcndantcs de l’amplitude de fonde
et de la constante de phase. )
30LUIIÜN: En comparant les équations 2.17 et 2.18, on voit que le
nomlMTO d ’onde et la frequence angulaire sont d) Quel est le déplacement y à a = 22,5 cm et / — 18,9 s ?

k — 72.1 i.id/ni cl w = 2.72 ratl/.s. SOLUTION; On utili.se le concept de siiivani ; l'équation 2.17 donne le
déplacemeiu en fonction de la position x et du temps l. En leur
(3n relie ensuite la longueur d’onde A. à A: à l’aide de l'équation 2.5 : substituant les valcun données dans l’équation, on ohiicni
27T 27t rati
y = U.003 27 sin(72.1 x 0.225 - 2,72 x 18.9 + 1,05)
A 7 2 .1 rad/m
= (0.003 27 m) sin (-3 4 .1 35 5 rad)
= 0,0X7 I m = 8,71 cm. (réponse)
= (0.(X)3 27 niK-0 .4 0 9 60)
Ensuite, On relie 7 à eu avec l’cquation 2.8 : = —0,001 34 m = —1.34 mm. (réponse)
2:r rad Le déplacement est donc négatif. (Assurer-vous de mettre votre
2.31 s (répoii.se)
(û 2.72 rad/s calculatrice en mode radian avant d ’évaluer le sinus.)

Exemple 2.2
Dans l’exemple 2 .1 d). on a démontre i|ii’à f - 18.9 s. le déplacement Donc, à / ■ 18,9 s, fcléincnl de la corde situé à x = 22.5 cm se
transversal y de f element de la corde situe à r = 0.255 m. pour déplace dans la direction positive de l’axe des y. et à une vitesse
fonde décrite par l’équation 2.17, était de - 1,34 mm. ayant un module de K. 11 mm/s.

a) Quelle est y,., la compo.sanie y de la vitesse transversale du même b) Quelle est, au meme instant, la composante y de l’accélération
clément de la cordc. à ccl instant'? (Cette vitesse, qui est assiKiée à transversale a,, du même élément ?
l'oscillation tran.sversale d’un élément de la cordc. est dans la direction
SOLUTION: On uti lise ici ce fontepi dé : la composante y de l’accélération
de l’axe des- v. Ne la confondez pas avec v, le mcxlule de la vitcs.se
transversale ri, est le taux de variation de la compo.santc v de
constante auquel le profil île l ’oiulc voyage le long de l'axe des x.)
la vitesse transversale r,. de l'élément. À l'aide de l’équation 2.20,
SOlUTlOH On iiiili.se le toncept dé suivant : la composante y de kt vitesse en prenant toujours x comme une constante mais en permettant à /
transversale est le taux de variation du déplacement v de l’clcmcnt. de varier, on délernnnc que
Ce déplacement cm généralement donné par lia ,
fl, = = — a rx - sinOLr — m l + i}>).
y(x, t) = Vmsin(Ax - wt ■* d>). (2.19) ’ Ht
Pour un element situé h une position x donnée, on trouve le taux de Puisc)iie y„ sin(A.r —(ot -I- t|)) est égal au déplacement y; selon f éciiia-
vanaiion du déplacement transversal V en prenant la dcnvcc de l’équa­ tinn 2.19, on peui écrire que
tion 2.19 par rapjxirt à /, tout en considérani .v comme turc constante. fl, = —m'y.
Line dérivée effectuée alors qii' une ou plusieurs variables sont consi-
dérce.s cotTuiK con.stantcs se nomme une dérivée partielle, et elle est On voit que la coniptisanlc y de l’accélération transversale d'un element
rcprésriiiéc par k- symbole fl/il r plutôt que iUd\ On a ici de corde oscillant est proportionnelle à son déplacement tran.svci'sal.
mais de signe opposé. O l a est tout à fait cohérent avec le typ»> de
4v
V,. ' ----- wVn, cos(At — ai/ •-<i>). (2.?('l mouvcineni qii'effectue f élcrnenl lui même : il se déplace de fayun
dl transversale en un mouvement harmonique simple. En insérant
Disuile. en üisdi aiil Ic.s valeur> numénque.s de l'exemple 2.1. on obtient les valeurs numériques, t.in obtient

y, = -(2 .7 2 rad/sX3,27 mm) cos( 34,135 5 rad) </,. = —(2,72 md/s)- ( —1,34 mm)
= 8.11 mni/s (réponse) = 9,91 mm/s^. (réponse)
2.6 Lx) vitesse d’une onde dans une corde tendue 33

Donc, à f = 18,9 s. l’dlément de corde situé à x - 22,5 cm s’est ✓ v é r if ie z VOS CONNAISSANCES 2 ; Voici les équations de tro is
déplacé, de sa position d ’équilibre, de 1,34 mm dans la direction ondes : I ) v(x. f) = 2 sin(4x 2f) ; 2) j(x , t) = sin(3x —4r + 2) ;
négative de l’axe des y. et il a une accélération de 9,91 irmi/s^ dans 3) v(x, f) = 2 sin(3x — 3r — 5). Classez les ondes en ordre
la direction positive de l’axe des y. > décroissant selon a) le module de leur vitesse de propagation
et b) le module de leur vitesse tninsversale maximale.

«eSOLUTIOM OC PROBuèM€S

1" stratégie: L'évaluation des fonctions trigonométriques avec des les valeurs des fonctions trigonométriques. Par exemple, .si l'angle ust
angles relotivejnent grands de -35.185 5 rad. fin additionnant (6)(2Tt rad) à cet angle, on obtient
Il arrive parfbi.s. comme dans les exemples 2 .1 d) et 2.2, qu'un angle
—35,185 5 rad I (6K2/r rad) = 2.513 61 rad.
beaucoup plus grand que 2jt rad (ou 360‘ ) .se présente et qu’on vous
demande de calculer son sinus ou son cosinus. Additionner ou soit un angle plus petit que 2;r rad et dont les valeurs des tonenons
soasiraire un multiple entier de Zt rad à un tel angle ne change en nen trigonométriques sont les mêmes que pour l'angle de —35,185 .5 rad
(voir la figure 2.8), Ainsi, le sinus de 2,513 61 est 0,588, soit le même
que pour —.35,185 5.
Votre calculatrice effectuera elle-même la réduction de cor
angles relativement grands. Attention : n'arromli.sse/ pas de grands
angles si vous devez prendre leur sinus ou leur cosinu-s. 1 orsqu'on
effectue les calculs en radians, par exemple, les décimales sont
importantes car la calculatnce doit réduire l'angle à une valeur com
prise entre 0 et 2n lad (environ 6,283 2 rad). Si, par exemple, vous
arrondissiez —35,185 5 rad à 35 rad (une différence de 0 5%
qui semble raisonnable), vous changeriez le sinus de l'angle de 27 % !
De plus, si vous convertissez en radians un grand angle expnm c
-3.5,185 5 rad
en degrés, assurez-vous d ’utiliscr un facteur de conversion exact
figure 2.8 Ces deux angles .sont differents, mais les valeurs de leurs (comme 180“ = tr rad), et non un facteur approximatif (cimime.
fonctions trigoniiinétriques sont toutes les mêtnes. 57,3'^ = 1 rad).

2.6 La vitesse d'une onde


dans une corde tendue
Le m odule de la vitesse d 'u n e onde est relié à sa longueur d 'o n d e c i à sa fréquence pat
l'éq u atio n 2 . 12, m ais dépend également des propriétés du milieu. Q uand une onde voyage
dans un m ilieu com m e l'eau . Pair, l ’acier ou u n cco rd e tendue, elle fail osciller les particules
de ce m ilieu lors de son passage. P our que cela puis.se se produire, le m ilieu doit av o ir à la
fois une m asse (présence d’une énergie cinétique) et une élasticité (présence d ’une énergie
potentielle). D onc, les propriétés reliées à la m asse et à l ’élasticité du m ilieu d éten n in en t
la vites.se à laquelle l'o n d e se propage dans ce milieu, inversem ent , il devrait être possible
de calcu ler le m odule la vites.se d e l’o nde dans un m ilieu en fonction d e ces propriétés.
C ’est ce q u ’on exam inera m aintenant, de deux façons différentes, dans le cas d ’une corde
tcnrlne.

L'analyse dimensionnelle
D ans une analyse dim ensionnelle, on analyse attcntivem ciu les dim ensions de toutes les
quantités physiques (jui font partie d ’une situation donnée afin de d éterm in er les autres
quantités physitjues avec lesquelles elles ptturraicnl être reliccs. D aas le présent exem ple,
on exam ine la mas.se et l ’élasticité afin d e tro u v er une expression jxnir le mtxJule de la
vitesse V, qui a la dim ension d’une longueur d ivisée par un tem ps, ou LT '.
P our ce i|iii est de la m asse, on la représentera par la mas.se rn de la corde divisée par
la longueur I de la corde. On appelle ce rapport la masse linéique p (ou m asse par unité de
lo n g u eu r) de la co rd e . D o n c, p = m /l, sa d im e n sio n étant une m asse d iv isée p a r une
longueur. M l, '.
Vous p o u v ez e n v o y e r une o n d e d an s une co rd e à co n d itio n q u e ce lle co rd e soir
tendue, ce qui signifie q u ’elle a été tendue par ries forces extérieures appliquées à scs deux
extrém ités. I a tension dans la cordc r. qu e l'o n .suppose co n stan te puisque l.r m asse de
la cordc est très faible, est égale au m odule de ces dc'ux forces extérieures. L o rsq u ’une
onde se p ropag e d an s la co rd c, elle d ép lace des élém en ts de co rd e en caus’ani une très
34 Chapitre 2 bes ondes

petite tension additionnelle, ca r on co n sid ère q u e ia déform ation de la corde n’est pas
trop im portante, e t les sectio n s ad jacen tes de ia co rd e tirent les unes su r les au tres à
ca u se de ce tte ten sio n . O n p eu t d o n c asso cie r la ten sio n d an s la co rd e à l ’étircn ien t
(l’élasticité) d e cette corde. La tension et les forces exicrieures appliquées ont la dim en­
sion d 'u n e force, c ’est-à-dire MLT"^ (selon = mFi).
L e but, ici, est de com biner ¡à (dim ension M l ‘ ) et r (dim ension M LT ~) en vue
d e g é n é re r r (d im en sio n LT '). En jo n g la n t avec d iffé re n te s co m b in aiso n s (il faut,
entre autres, d iv iser l'u n e des deux q u an tités p ar l'a u tre afin qu e M .st‘ sim plifie), on
obtient

V= Г (2.21)

où C e st une co n stan te san s d im en sio n qui ne p eu t être d éterm in é e p ar une an aly se


dim ensionnelle. D ans la deuxièm e аррг(ч;Нс q u 'o n utilisera p o u r déterm iner le m odule
de la vitesse de l'o n d e, on verra que l’équation 2.21 est en effet correcte et que C = 1.

La dérivation à l’aide de la deuxième loi de Newton


Au lieu de Tonde sinusoïdale de la figum 2.1 b), on considère une im pulsion sym étrique
com m e celle de la figure 2.9, q u i se déplace de gauche à droite dans une cordc et à une
vites.se ayant un m tnlule v. P ar com m o d ité, on ch o isit un .système d e ccKudonnées où
T im pulsion est im m obile, c'est-à-d ire un systèm e de coordonnées où on sc déplace avec
l'im p u ls io n , e t où cette im p u lsio n e st co n stam m en t v isib le. D an s ce sy stèm e
de co o rd o n n é e s, la co rd e d e la fig u re 2 .9 n o u s ap p a raît co m m e si e lle se d ép laçait
derrière nous, de droite à gauche à une vitesse v.
O n c o n sid è re un p etit élém e n t de co rd e d an s c e tte im p u lsio n , de lo n g u e u r Д/.
form ant un aie d e cercle de rayon R e t un an g le 26 par rapport au cercle d e rayon R.
Puisque la m asse de la corde est très faible et que Ton considère que la déform ation de
la cordc pixxluite par le passage de l’im pulsion n ’est pas trop im portante, la tension dans
la corde est la m cinc partout le long de la cordc. U ne fcx'ce. d ont le mcxlule est égal à la
tension dans la corde, tire de façon taiigenticllc sur chaque extrém ité de cci clém ent. Les
com posantes horizontales de ces forces s ’annulent, alors que les com posantes verticales
s ’addilionncnt pour pixxluire une force résultante orientée vers le centre du cercle,
( ü n néglige le poids de T élém ent de cordc, qui est beaucoup plus p etit qu e la tension
d ans la c o rd c .) F^s est la fo rce de rap p el a g issa n t su r T élém e n t d e co rd e et d o n t le
m odule est donné par

Al
/çjs 2 ( r sin 0) = r (20) = r (le miKlule de la liHce réNiiItante), (2.22)
R

où o n a utili.sé l'approxim ation des petits angles, ,sin 6, p u isq u ’on considère un très
court segm ent de la corde. D ’après l'illustration, o n a egalem ent utilisé 26 = A!/R.
L.a ma.ssc de Télém ent est donnée par

1Д / ---- Am = fi A i (la ma.ssc). (2.23)

où e.st la m asse linéique de la corde.


À T instant rep rése n té à la fig u re 2.9 , T élém ent d e co rd e A i sc d ép lac e su r une
trajectoire en form e d ’arc de cercle. 11 a donc une accélération centripète orientée vers
le centre de ce cercle, dont le m odule est donné par

«c •— (le module de l’accélération). (2.24)


A
Figure 2.9 Tue impulsion symétriijiie,
dans un sysfiime de coordonnées on Lc.s équations 2.22. 2.23 et 2.24 com portent tous les élém ents de la deuxièm e loi
Tiiiipul&ioii est iiiiinobik et où de N ew ton ( = nui). P uisque les vecteurs et o nt la m êm e orientation (vers le
la uirdc .sciiiblr SC clèplarer de la droite centre d u cercle), la deuxièm e loi de N ew ton peut être récrite sous la form e — ma^.
ver« lu giuiché. ü une vitessé v. Si on rem p lace m et de la d eu x ièm e lo j d e N ew to n pat les e x p re ssio n s
O n determ ine le module de lu vitesse v des equations 2.22, 2.23 et 2.24, on obtient
en upiiliquflin 1(1dciisiémr loi dr Newton
à u n élément de cord«' de longueur àl r Ai V-
situé sur la crctc de Timpulsion. R
2.7 L 'én erg ie e t la p u iss a n c e d 'u n e o n d e sin u so ïd ale d a n s u n e c o rd e 35

E n résolvant cette équation en fonction du m odule de la vitesse v, un obtient

V= • / — (le module de la vitesse d'une onde dan.s une corde tendue), (2.25)
VM
ce qui concorde parfaitem en t avec l'éq u atio n 2.21. .si la constante C de cctlc équation
est égale à 1. L’équaiitai 2.25 donne le m odule de la vitesse de 1 impulsion de la figure 2.y,
et le m odule de la vitesse de mure autre onde dans la m êm e corde tendue.
L’équation 2.25 indique ce qui suit.

► Ivc module de la vitesse de propagation d'une onde ilans une corde idéale (endue dé|Xitd
.seulement de la tension dans la crade cl de la iiia.s.sc linéique de la cordc. et nr>n de lu frequence
de l’onde.

L a fréquence de l’onde est entièrem ent déterm inée par ce qui génère l'o n d e (par exem ple
la personne illu strée à la figure 2.1 b]). L a longueur d'onde est alo rs d éterm in ée par
l’équation 2.12 , soit X — v /f

VÉRIFIEZ VOS CONNAISSANCES 3: Vou.s produi.se/ une onde dans une œ rdc en faisant
osciller une de ses extrcniitcs. Si vihis augmentez la fréquence des oscillations, a) la vitesse
de l’onde et b) la longueur d’onde augmentent elles, dimmucm-elles ou restent-elles les mêmes''
Si vous augmentez plutôt la tension dans la corde, e) la vitesse de l'onde et d) la longueur d'onde
augmentent-elles, diminuent-elles ou restent-elles les mêmes ?

Exemple 2.3
Dan.s la figure 2.10, deux cordes sont nouées et étirées entre 3. Puisque les deux cordes sont attachées ensemble, elles doivent
deux supports rigides. Les cordes ont les masses linéiques être soumises à la même tension dans la corde (T -l(X) N ).
/t, — 1.40 X 10“"* kg/m et //i = 2.80 x 10 “*kgAn. Leurs longueurs
En combinant ces trois concepts, on trouve le temps qu'il faut pour
sont L, ~ 3,00 m et - 2.00 m. et la cordc I est stximise à une tension
que l'impulsion atteigne le nœud de la cnrde 1
ayant un module de 4(X) N. Une impulsion est envoyée simultanément
dans les deux cordes à partir des supports rigides, en direction du
/P i _ /1 ,4 0 X 10 '* kg/m
nœud. Quelle impulsion atteindra le nœud en premier?
SOlUIlON: Plusieurs concepts clés s’appliquent à cette situation.
- = t u / -
V| ^
(3,00 m)
V 400 N
= 1,77 X UV'-’ s.
1. Le temps t pris par une impulsion pour parcourir une longueur L est
f = IJv, où Vest le module de la vitesse constante de l’impulsion. f)e In même façon, les données pour riiiipiilsion de la corde 2 donnent
2. Le mcxliile de la vitesse d’une impulsion dans une corde tendue
dépend de la tension dans la coixie r et dc_ sa masse linéique t2 = = 1.67 X 10 ’ s.
et est donné par l'cquaiion 2.25 (v = y jr lq ) .

'H*------^2-------J l ’impulsion de la corde 2 atteint donc le nœud en prcniici.


.Maintenant, considérez de nouveau le tontcpf dé 2. La masse
T " '" '- ”! linéique de la corde 2 est plus grande que celle de la corde I. de sorte
' Nœud B que l’impulsion de la corde 2 doii être plus Icnic que celle de
la corde I. Auriez-vous pu deviner la n'ponsc d’après ce seul fail ?
figure 2.10 Exemple 2.3 Picux cordes, de longueurs L| et L -., Non. car le premier concept clé indique que la distance pan ouruc par
sont attachées par un na-ud et tendues entre deux supports rigides. une impulsion intervient également

2.7 L’énergie et la puissance d'une onde


sinusoïdale dans une corde
Q uand vous p ro d u isez une o nde sin u so ïd ale dans une co rd e ten d u e, vous fournissez
eJe l'én erg ie à la eorde afin que celle-ci se m ette en nxm vem eiu. L’onde, en s'rlo ig n n n l
de vous, transporte cette énergie sous form e d ’énergie cinétique et d ’énergie poleniielle
éla.stiquc. E.xaminons chacune de ces forrnes d ’énergie.
36 Chopitre 2 Les ondes

L’énergie cinétique
Un elem e n t de c o rd e d e mas.se dm. q ui o se ille tra n sv e rsa le m e n t en un rnouvem en
harm onique sim ple pendant que l’onde le traverse, a une énergie cinétique asscKiée à st
vitesse transversale. L orsque l’élém ent est à sa position d 'éq u ilib re y = 0 n 'é le m e n t l
B . de la figure 2 . 11). sa vitesse transversale - donc, son énergie cinétique - est m aximale
i!ù / '\
fia vitesse est m ax im ale à la p o sitio n d ’é q u ilib re d an s un m o u v e m en t h arm o n iq u e
sim ple). Lorsque l ’élém ent se trouve à sa position extrêm e y - y,„ (com m e l ’élém ent ci U
sa vitesse transversale - et son énergie cinétique - est nulle (la vitesse est nulle à y = Vn.
dans un inoiivem eni harm onique sim ple).
\ y '
J.v' |(<X L'énergie potentielle élastique
Figure ? 11 T'n insraniané d'une onde Pour produire une onde .sinuso'idale le long d ’une corde initialem ent droite, l ’onde doit
sinu.nMiliilc dans iiiic torde 5i f —0. nécessairem ent étirer la corde. L orsqu’un élém ent de corde ayant une longueur dx avant
L'élément de eoide a |Hfs.sùlc im le pa.s,sage de l’o nde oscille transversalem ent, sa longueur doit augm enter et dim inuer
dép]a(»nienl v = y,,,, et l’élément de façon péricKlique afin qu e l’élém en t de co rd e suive le profil sinusoïdal d e T onde.
lie eoide h siibii un déplHcenient L’énergie potentielle élastique est associée à ces variations de longueur, conune c ’est le
V ■” 0. L’énergie eiiiétiquc des dens cas p o u r un ressort.
elements depend de leur vitesse l^orsque l’élém ent de corde est à sa position y = v„, (Tclcm ent a de la figure 2.11 ).
irunsvcrsalc i.'énergic ixiieniielle sa lo n g u e u r a une v aleu r n o rm ale e t non p ertu rb ée du. c a r il est ap p ro x im ativ em en t
dépend de i ’étueiTKnt vie l’éléincnl parallèle h Taxe des x, et son énergie potentielle élastique est nulle. Toutefois, lorsque
ch* corde lorsejue l'onde le traverse. T clém cnt atteint sa position y = 0. il e st étiré à son m axim um , car son inclinaison par
rapport à Taxe des x est maximale, cl son énergie polcmtielle élastique est alors maximale.

Le transport de l’énergie
L ’é n e rg ie cin étiq u e et l'é n e rg ie p o te n tie lle é lastiq u e so n t d o n c m ax im ales lor.squc
l ’élém ent de corde oscillant se trouve à y = 0. D ans la figure 2. I l , les segm ents où le
déplacem ent est m axim al iTont aucune énergie, et ceux où le déplacem ent est m inim al
ont une énergie m axim ale. Pendant que Tonde se propage le long de la corde, les forces
de tension effec tu en t co n tin u ellem en t du trav ail afin q ue T cn erg ie des seg m en ts où
i énergie est présente soit transférée vers les .segments o ù T éncrgie est nulle.
Suppo.sez q u e vous p ro d u isez une onde d an s u ne co rd c ten d u e le long d ’un axe
des X et que le déplacem ent des élém ents de la corde est décrit p a r l’équation 2.2. Vous
pouvez tran sm ettre une o n d e le lo n g de la co rd e en faisa n t o sc ille r co n tin u ellem en t
une do scs e x tré m ité s, co m m e à la fig u re 2.1 b). En a g issan t ain si, vo u s fo u rn issez
co n tin u ellem en t de T én c rg ie serv ant à é tire r e t à m e u re en m o u v e m en t la co rd e ;
p u isq u e les seg m en ts d e co rd e o sc illen l p e rp e n d ic u la ire m e n t à T ax e des x, ils o nt
de T éncrgie cinétique e t de l’én erg ie potentielle élastique. L orsque Tonde sc propage
dans les segm ents initialem ent an repos, de l’énergie est transférée dans ces segm ents.
O n peut donc dire que l’onde inm.'.porte de l’énergie le long d e la corde.

Le toux de transmission de l’énergie


I.a petite quantité d 'én erg ie cinétique dK associée à un élém ent de corde de ma.s.se dm
est donnée par

dK dm v~. (2.26)

où V,. est la eomjXisanie y de la vitesse transversale d e Télém ent o siillan l Pour trouver
on dérive T equation 2.2 par rapport au tem ps, en considérant x com m e une constante ;
dv
V,. — = - (oy,„ co s(/a — col i- d>). (2.27)
ni
Fn utilisant cette relation et en rem plaçant dm par /z dx, on récrit l ’équation 2.26 sous
la form e

dK ~ Up. dx)( a>y^Ÿ cos^(L d— cot F d»). (2.28)

En divisant T equation 2.28 par dt, on trouve le taux de variation de l'én erg ie cinétique
d ’un clém ent de corde, ce qui nous donne le taux de transm ission de T éneigie cinétique
2.8 Le principe de super|x>sitlon des ondes 37

tran sp o rté e p a r l ’onde. Le ra p p o rt dxldt qui a p p a ra ît alo rs d an s le m em b re d ro it de


l’équation 2.28 est le m odule de la vitesse v de l’onde, et on obtient
dK ,
—— = co s^fct — üjt i <j>). (2.29)
dt ^

Le taux moyen auquel l’énergie cinétique est transportée c.st

,^ i’i^^.Vmtco.s^(7n: - toi +

(2.30)

où on a e ffec tu é la m oyen n e en co n sid éran t un n o m bre en tier de lo n g u eu rs d ’o n d e,


e t on a u tilisé le fait qu e la v aleu r m o y en n e d 'u n e fo nction co sin u s ¡tu c a rré d an s
un nom bre entier de péritxlcs est \ .
L’énergie potentielle élastique est égalem ent transportée par l ’onde, et ce, au meme
taux moyen donné par l'éq u atio n 2.30. M em e si on n ’en fera pas ici la dém onstration,
on sait que, dans un systèm e oscillant, com m e l’est un pendule ou un systèm e bloc-ressort,
l’énergie cinétique m oyenne et l ’énergie potentielle m oyenne .sont effeelivcinent égales.
La puissaiK e m oyenne, qui est le taux moyen de transm ission de ces deux types d énergie
par l'onde, est alors

ou. encore, selon l’équation 2.30,

^mciy ~ Oa puissance moyenne transmise jiar une onde siiiasoïdalc). (2..32)

D ans cette équation, les facteurs p e t v dépendent du type de m atériau d ont la corde est
co n stitu é e e t de sa ten sio n . L es facteu rs co et y„ d ép e n d en t du p ro a ts s u s q ui gén ère
l'o n d e. C e lien de proportionnalité entre la puissance m oyenne d ’une onde et le carré
de son am plitude et, aussi, le c a rré d e sa fréquence an g u laire, est un résu ltat g énéral
qui s ’applique à tous les types d ’ondes

Exemple 2.4
Une corde tendue a une masse linéique (j. ~ 52.') g/m. et e.st .soumise D ’après l’équation 2.25,
à une tension ayant un module r ~ 45.0 N. On lui transmet par une
de scs extrémités une onde sinusoïdale de fréquence f = 120 Hz et ¡T I 45.0 N
, I — = , / --------------- = 9.26 m/s.
d’amplitude y,„ = 8,3 mm. À quel taux moyen l'onde transportc-t-elle ^ Y 0,525 kg/in
l'énergie?
L'équation 2 32 donne alors
50HITI0N. Le coricepl dé est que le taux moyen de transport d’énergie est
la puissance moyenne donnée par l'équation 2.32. Toutefois,
pour utiliser cette équation, on doit d’abord calculer la fréquence ' n,oy —iP'tO-yt
•.

angulaire«»et le iiKxluIc de la vites.se vde l’onrle. D’après l'équation 2.9. - (j -)(0,525 kg/tn)(9.26 iii/s)(754rad/s)’(0.008 .3 m)--
a> - 2 n f (2n-K120 Hz) - 7.54 rad/s. ==95W. (réponse)

^VÉRIREZ VOS CONNAISSANCES 4 : Dans cet exemple, il est possible d’ajuster trois paramètres : la tension dans la corde, la fréquence de l'onde
et l’amplitude de l’onde. Le taux moyen de traasport d’énergie par l’onde augmente t il. dimitiuc-t-il ou reste-t-il le metne si f>n augmente
a) la tension, h) la fréquence et c) l’amplitude ?

2.8 Le principe de superposition des ondes


Il arrive souvent que deux ou plusieurs ondes traversent sim ultaném ent la m êm e région
P ar exemple, lorsque vous assistez à un concert, les ondes sonores provenant de différents
instrum ents atteignent sim ultaném ent vos tym pans. D ans les antennes des radios et des
téléi'isions, les électro n s sont m is en m ouvem ent p ar l'e ffe t nei de nom breuses ondes
électrom agnétiques provenant de differents centres de diffusion l 'ea u il’iin lac ou d 'u n
port pt'ut être agitée p a r des vagues provenant de plusieurs bateaux.
3B Chapitre 2 Lies ondes

Siippxise/ que deux onde.s se p ro p ag en t .sim ultaném ent le long d e la m êm e corde


\ tendue. S oit > ,( a. î ) et v^îa , /), les dépIacem cnLs q u 'e ffe c tu e ra ie n t les élém en ts d e la
corde si chaque onde se propageait .séparément. Le déplacem ent résultant des élém ents
de la corde y'{x, t) lorsque les ondes se chevauchent e.st donné par

y'(.v. /) - vii.v, t) + >s(x /). (2..^3)

C elle soinm c algébrique de déplacem ents le long de la cordc signifie q u e:

)► des ondes qui sc superposent s’addiiinnnenl ulgcbriqueineni pour produire


une onde résullanle (<xi oncle nette).

C e la con.stitne un an tre ex em ple du p r in c ip e d e s u p e rp o s itio n , selon lequel lorsque


plusieurs effets sc produi.senl sim ultaném ent, leur effet net est la som m e des effets indi­
viduels.
La figure 2.12 présente une série d ’instantanés de deux impulsions se propageant dans
des directions oppo.sécs ckins la même corde tendue. l,orsque les impulsions se chevauchent,
l’impulsion Résultante correspond à leur somme. De plus, chaque impulsion passe à iiavers
ra iitie , com m e si celle d ernière n'exi.siait pas.

Des ondes qui se chevauchent ne s’affectent pas mutuellement. elles conservent leur torme
Hgurt 2.]2 Une sene d’instaiituncs
et leur direcliou de propagation.
repesen! ;«it deux ontle>. se propageant
iliiii's des diiwiioiis opitosicN le long
d’une corde tendue. Le principe
de superposition s apphrjue lorsque 2.9 L’interférence des ondes
les impulsions .SCclicvaucliciil.
Suppixsez (¡u'on produit deux ondes sinusoïdales le long d ’une corde tendue et que leur
lo n g u e u r e ïo n d e . le u r am p litu d e et leu r d irec tio n d e p ro p ag a tio n so n t les m êm es.
Le principe de su p erp o sitio n s ’applique. Q u elle onde résultante y au ra-t-il alors dans
la corde ?
L’onde résultante dépend de la différetu e de phase entre les deux o n d e s . autrem ent
dit. de com bien le pnrfil d ’un e onde est déphasé par rap p o n à celui de l'au tre . St Ic.s
ondes sont parfaitem ent en phase (de sorte qu e leurs crêtes et leurs creux coïncident),
elles se eom bincni p o u r d oubler le d éplacem ent qui serait produit par une seule onde.
.Si e lle s sont p a rfa ite m e n t d ép h a sé es (les crêtc.s de l'u n e c o ïn cid e n t avec les creu x
de r a u tr e ) , leu r co m b in aiso n p ro v o q u e leu r an n u latio n , et la co rd e reste d ro ite.
C e phénom ène où les ondes se com binent se nom m e in te rfé re n c e des ondes, puisque
les ondes in te rfè re n t l'u n e avec l'autre.
Soit une onde se propageant seule le long d ’une corde tendue, donnée par

>’i(x f) = y„, sin(AjT — wt), (2.34)

et une antre onde, déphasée d e d>par rapport il la prem ière, donnée par

v’2( r. t) = >',n sin(fcr - a>t + (P). (2.35)

C es onde.s ont la m êm e fié<iticnte angulaire o> (donc, la m êm e fréquence f) , le m êm e


nom bre d ’onde A (donc, la m êm e longueur d ’onde À) et la m êm e am plitude Hiles se
propagent toutes les deux dans la dinectinn positive de l’axe des x, à la m êm e vitesse,
tlont le m odule est donné par l’équation 2.25. E lles diffèrent .seulement pat la constante
de phase d> On dit que ces ondes sont déphasées de 4>. ont une différeiwe de phase tj>,
ou q u 'u n e des ondes présente un déphasage d e </>par rapport à l’autre.
Selon le princi|x; de superposition (équation 2.33), l’onde résultante y'(A. t) est la
som m e algébrique des deux ondes qui interfèrent, donc

yï.V. I) = V|ÎX t) L y,(A, t)


— y„, sin(A.t - (üt) + >n, sin(AA - (lit -f (ft). (2.36)

C om m e on l'in d iq u e dans l'a n n e x e D . on p eu t réc rire la som m e d es sin u s de d eux


angles a et fi sous la form e suivante ;

sin a *- sin f i = 2 sinfjlu + ^ ) | eos[^j(û' - /l)j. (2.37)


2.9 L Interférence des ondes 39

Di'jiirttciufni En appliquant celle relation à ré q u a tio n 2.36, on obtient


f) ^ r^Yni cvisiit 2)1 sb\(lr\ -« / ç> 2)
*- « . — ^
\m}>lUudi- ' \ nv.c Of.cill;yU y'{x, t) = [2.VV1co s(0 / 2 j] sin(Lx — tuf 3 ф/2) (2.38)

Figure 2.13 L’onde résultante


de l’équation 2.38, cau.sée L a quantité exprim ée entre le.s crochets est l’am plitude de l’onde résultante puisqu’elle ne
par r interférence de deux ondes contient que des constantes ; clic est indépendante de la position x de l'élém en t de corde
sinusoïdales de même amplitude, et du tem ps r. Le term e osciikun, identifie à la figure 2.13, indique que l ’otuit: résultante
de meme longueur d'onde est ég a le m en t un e o n d e sin u so ïd ale ay an t la wê/we lo n g u e u r d ’o n d e e t la meme
et SC propageant dans ta même trequence que les ondes qui la constituent et sc propageant dans la direction positive do
dircetion le long d’une corde temliie, l'a x e des x. C ’est la seule onde que vous verriez réellem ent sur la corde (vous ne veiriez
e.st également une onde sinusoïdale pas les deux ondes qui interfèrent, représentées par les équations 2.34 cl 2.3.'ï).
comportant utx: amplitude
et un terme oscillant.
Si deux ondes sinusoïdales de même amplitude et de même longueur d'onde se pupageni
dans la même direction le long d'une corde tendue, clics interfèrent pour pruduiic une onde
sinusoïdale rcsulumte se propageant dans cette direction.

L’onde résultante diffère des deux ondes qui intcricrent de deux m anières : 1) sa constante
de phase est <¿/2 , et 2 ) son am plitude >•„ est la q uantité ex prim ée en tre cro ch ets dans
l’équation 2.38 ;

= 2y^ соч(ф/2) (l’amplinide). (2.39)

Si <i!) = 0 rad (ou 0 °). le s deux o n d es qui in te rfè re n t sont eu p h ase, co n iin e à la
figure 2.14 a). L ’équation 2.38 sc réduit alors à

v '(r, t) — 2v„, sin(/c.r — cot) (Ф = 0 ). (2.40)

Cette onde résultante est représentée graphiquem ent à la ligure 2 14 d) N otez que, d ’apres
cette illustration cl l’équation 2.40, l’am plitude de fo n d e résultante esi le double de celle
de chaque onde qui la constitue. C ’est la plus grande am plitude que l'o n d e rcsultantc
peiil avoir, puisejue le term e cosinus de.s équations 2.38 et 2.39 a sa plus grande valeur
(l’unité) lorsque ф = 0. O n nom m e in te rfé re n c e con.structivc fin le rféren c c qui produit
la plus grande am plitude ptvssible.
Si Ф = iT rad (on 180'’). les o n d es qui in te rfè re n t .sont p arfaitem e n t d ép h asées,
com m e le m ontre la figure 2 .14 h). D ans ce cas. cos (d>/2) d evient cos (я/7) = 0, et
J’amplitiide de fo n d e résultante esl nulle, com m e l’indique fé q u a tio n 2.39.
O n a alors, prjur toutes les valeurs possibles d e x et de t.

y'(x, r) = 0 (ф — JT lad). (2.41)

Rgure 214 Deux otnlcs


sinusoïdales, V|(jr. I) et yjix, t).
de même amplitude et do même
longueur d'onde se propagent
le long d’une corde dans
la direction positive de l’axe
des r Flics inferfèreni pour
i
pnxluire une. i>nde ré.sult.'uite
y'(x. t). L’onde résultante
est celle qu'on voit réellement
sur la corde. La difference
de phase é entre les deux ondes , y'ijr. fl
qui interfèrent est a) 0 rad
ou (L. h) n r.id ou 1Kff'
et c) ^7T rad ou 120' 1 es ondes
résultantes correspondantes
sont rcprc.sentécs en d). e) et fi. c)
40 Chopítre 2 Les ondes

[ACliAU 2.1 Différences de phase et types d'interférences'


DiffcreiKes de phase exprimées en Amplitude
degrés radians longueurs d’onde de l’onde résultante d’interférence

0 0 0 2y„, Constructive
120 ¥ 0,33 .Vm Intermédiaire
180 TT 0,50 0 Destructive
240 ¥ 0.67 Дт Intermédiaire
360 2тг l.tX) 2уш Coastructive
865 15,1 2,40 0,60y„ Intermédiaire
i-a (iirrercnoe fle phase entre deux oikIcn de meme amplitiide, de meme longueur d onde
cl se dépla^anl dans la même direction fiui, sans celle différence, scraicm idc-nliques

L’onde résultante e.st représentée graphiquem ent à la figure 2 14 e). M em e si on produit


deu x onde.s le long d e la co rd e , on ne v o it aucun m o u v e m en t de la co rd e. C e ty p e
d'in terféren ce .se nom m e in te rfé re n c e d e stru c tiv e .
P u isq u ’une o nde sinusoïdale répète sa form e à tous les 2n rad. un e d ifférence de
phase <j> 2ji rad (ou .^60'^) correspond à un décalage entre deux ondes é g u o alanl à une
dislance d ’une lo n g u eu r d ’onde. D onc, les d ifféren ces d e p hase peu v en t être décrites
Kous fo rm e de lo n g u e u rs d 'o n d e , aussi bien q u e .sous fo rm e d ’angle.s. Par ex em p le,
à la fig u re 2.1 4 b), on pciii dire q u e les o n d es so n t d ép h a sé es d ’un e dem i lo n g u e u r
d 'o n d e (A/2). Le tableau 2.1 présente d ’autres exem ples d e différences de phase et des
interferences q u ’elles produisent. Notez que, lorsqu’une interférence n ’est ni constructive
ni destructive, on dit q u ’elle est une in te rfé re n c e in te rm é d ia ire . L 'am p litu d e de l’onde
résultante est alors interm édiaire, soit entre 0 et 2v„,. Par exem ple, d 'ap rès le tableau 2.1,
si les ondes qui in terfèren t ont une différence d e phase d e 120'^ { é ~ 2jr/3 rad, ce qui
corre.spond à un tiers de longueur d ’onde, kfJ), fo n d e résultante aura alors une am plitude
deVn,. soit la m êm e que celle des ondes qui interfèrent (voir les figures 2.14 c] et 1]).
D eux o n d es ayant la m êm e lo n g u e u r d ’onde so n t en p h ase si leu r d iffé ren ce de
phase est nulle ou .si elle est un nom bre imtier quelconque d e longueurs d ’ondc. D onc,
la partie entière d e toute d ifféren ce d e p h ase exprimée en longueurs d ’onde p eu t être
retirée. P ar exem ple, une différence de phase de 0,4 0 longueur d ’onde équivaut en tous
points à une dilTcrcnce de 2,40 longueurs d ’onde, de .sorte que le plus sim ple des deux
nom bres peut être utili.sé dans les calculs.

Exemple 2.5
Deux ondes sinusoidales ayant la même longueur d’onde, se propageant On a maintenant
dans la même direction le long d’une atrde tendue, interfèrent l’une avec
l'autre. I .'amplitude y„, de chaque onde est de 9,g nim, et la difference 4,9 mm = (2)(9.8 mm) cos(<V2),
(le phase ó entre elles est de lÜO'’. ce qui donne (avec la calculatrice en mode radians)
a) Quelle e.st famplilude >'ñ, de fonde résultante produite par l'intór-
lérence de ces deux ondes, et de quel type d’interférence s’agil-il .’ 4.9 mm
ф ~ 2 cos
(2)(9,8 mm)
SOLUTION. Le concept clé utilisé ici est le suivant; lorsque des ondes
sinusoïdales de même longueur d'onde et de même amplitude se ±2.636 rad Ä; i 2,6 rad. (réponse)
propagent dans la même direction le long d’uiK corde, leur interférence
produit une onde sinusoïdale progressive donnée par l'équation 2..T8, Deux réponses .sont po.ssihles, ca ró n peut obtenir la même onde
Donc, r!unpIitii(Jc de l’onde résultante est donnée par l’équation 2..W : résultante avec la premièie onde en avance ou en retard de 2,6 rad
sur la deuxième onde. E.xprimée en longueurs d’onde, la différence
y'„, = 2y„ cos(d»/2) = (2)(9.g mm) cos(100"/2) de phase est
= П mm (réponse)
Ф ±2.636 rad
( )n peut dire que cette mierlérenee est intermédiaire de deux façons
2;r rad/longueur d'onde 2я rad/longueur d’onde
D’une pan, la différence de phase se situe entre 0 et 180" et, d’autre
part, l'amplitude v,'„ se situe entre 0 et 2Vm ( = I nun). = ±0,42 longueur d'onde. (répomse)
h) Quelle difference de phase, en radians et en longueurs d ’ondc.
fonde résullante une amplitude de 4.9 mm '
d o n n e ra à ✓ vérifiez VOS CONNAISSANCES 5- Voici, exprimées en longiieuis
SOLUTION <)n applique ici le même concept clé q u ’en a), bien qu’on d’rindc, quatre autres dificrcnces de pha.se possibles entre les deux
connaisse maintenant et qu’on chenrhe ф. D’après l’équalinn 2..^9. ondes de cet exemple ; 0.20. 0.45,0,60 et 0.80. Classez-les en ordre
décrois-sant (la plus grande valeur en premier) scion l’amplitude
V’m = 2y,„ cosïdi/2 ). de fonde nSultante.
2 10 Les vecteurs de fresnei 41

2.10 Les vecteurs de Fresnel


Un v e c te u r d e F re sn e l perm et d e faire une rcprésentalion vectorielle d 'u n e onde dans
une co rd e (ou de to u t autre type d ’o nde) Un vecteur de Fresnel est, essen tiellem en t,
un v ecteu r d o n t le m o d u le e st ég al à l'a m p litu d e d e f o n d e e t d o n t f o rien tatio n , par
nippon à un axe hori/ontal. est égale à la phase de l’on Je. F iii«|ue la phase de fo n d e v o n t
dans le tem ps, le vecteur de I-rcsncl tourne pni rapport à ftir ig in e ; la viiosse angulaire
du vecteur d e frc sn c i est égale à la fréquence angulaire m tic fo n d e On jx-iii considérer,
par exem ple, fo n d e

V|(.ï, t) = y„,i *in(A.r - cüt). (2.42)

qui est rcpréseinéc par le vecteur d e Frcsiicl de la figure 2 . 1i a) L e m odule du vecteur


de Frcsnci correspond à f am plitude de l’onde >„1 ci mmi orientation, p a r n ip p tm à I axe
h o ri/o n ta l, co rresp o n d à la p hase d e f o n d e L \ — ojt. L e v ecteu r Uuirnc d an s le sens
horaire à cau.se du signe « m oins » devant le facteu r wi ; en effet, pour une valeur de »
dtinnéc, la p h ase, ilonc l’angle t|u ’e ffec n ie le v ec te u r p a r rap p o rt à f û x c h o rizo n tal,
dim inue lorsque le tem p s augm ente. Q u an d le vecteur d e F resnel to u rn e p a r rap p o rt
à l’o rig in e à la vitesse an g u laire (o. sa p ro jectio n >, su r l’axe v c n ita l v a n e d e façon
sinustVidale, d 'u n e valeur m axim ale Vn,, à une v aleu r m inim ale -y „ ,i en imssuih par zéro,
pour retoiim er ensuite à la v a l e u r y ,C e t t e variation cxincsjHmd ii la vaiialion .sinusoïdale
du déplacem ent V| tic tout point parcouru par fo n d e , le long de la cordc.
L orsque d eux ondes se propagent dans la m em e corde et d an s la lucnic directiiMi,
on peut les représenter, de m êm e que leur onde résultante, au m oyen d 'u n Juif;ramntc de
Fresnel. Les vecteurs de Fresnel de la figure 2.5 b l représentent fo n d e de fétiiiation 2.42
et une deuxièm e onde donnée |wr

v4.r, r) = y„2 sinlfci: — w f + ti>). (2.43)

C elte deuxièm e onde est déphasée p ar rapport à In première d ’une constuuic de phase é.
Le v ec te u r de F resn el d e f o n d e 2 a donc un m o d u le de et u n e o rien tatio n de
kx — lot (f>par rap p o n à l’axe horizontal. L’angle entre les deux vecteurs de Fresnel
[(Lr — (üt •*- 4>) - {kx - ù>t)\ co rresp o n d donc au déphasage tf>en tre les deux ondes.
P uis(|ue les v ecteurs de Frcsnci tou rn en t à la m êm e vitesse angulaire o). l'a n g le entre
les deux vecteurs de Fresnel csi to u jo u rs <f>. Si </> est une iniantité ¡uisith e. le vecteur
de F rcsnci de f o n d e 2 est en reum l su r celui de f o n d e 1 dans le u r ro tatio n , com m e
l’illustre la figure 2.15 b). Si d> est une quantité négative, le vecteur de Fresnel d e fo n d e 2
est en m a n t e su r le vecteur tic Fresnel de fo n d e I.
P u isqu e les o n d es y, et y^ ont le m êm e n o m bre d ’o n d e k et la m êm e fré q u en c e
angulaire tu, on sait, d 'ap rès l’équation 2.38. que leur onde rcsulianie prcml la fi« me
figure 2.15 a) ü n vcticur de Fresnel
de module tournant par rapport
l'origine a une vitesse angulaire aj, y'(.r. r) = >■;„ sin(L ï - tuf + j8). (2.44)
représente une onde sinusoïdale.
La projection >■, du vecteur de Fresnel où yj„ est l’am plitude de f o n d e résultante et )3 est .sa con.slanle de phase. P our trouver
sur ra.\e vcnieal représente les valeurs de et de fi, il faudrait faire la soniiiH - algébrique des deux ondes y, et y -,
le (tfplaeemem d'un point traversé com m e on l'a fait pour obtenir l ’équation 2.38.
par l’onde, b) Un deuxième vecteur P o u r e ffec tu er cette o p ératio n d an s un d iag ram m e d e F rcsnci, on fait la som m e
de Fresnel, de même vitesse angulaire v ec lo rielic d es deux vecteu rs de F resn el à un instant arb itra ire d u ran t leur rotation,
u>mai* de module cl tournant avec com m e à la fig u re 2.15 e). où on a fait coïncidoi l’o rig in e du vecteur de F resnel y ^
un angle constant é par rapport au avec fc x lrc m itc du vecteur de Frcsnci
premier veelciirde Fresnel, représc-nte
Le m odule d e la som m e v ectorielle est égal à fa m p lilu d c y ^ d e I éq u atio n 2 44
une deuxième onde dè|)iiascc de <b
L‘angle entre la som m e vectorielle et le vecteur de Fresnel représentant v'i est égal à la
c) L’onde résultante des deux ondes est
constante de phase fi de l éqiiaiion 2.44.
rcpréscniée par la somme vectorielle
des deux vecteurs de Fresnel. N otez que. coniraircm eni à la iiiéihode utilisée dans la section 2 .9 ;
La projection y' sur Taxe vertical
représoiiic le déplacement d’un point On peui utiliser les vccietirs de Frcsnci pour comFancr des ondes, meme si leurs iimi>liniites
au moment où l’onde résultante sont différentes
le traverse.
42 Chupitre 2 Les ondes

Exemple 2.6
Deux ondes sinusoïdiilcs, t) et ViOt, /), ont la monte longueur
{l'niitlc ei se propagent dans la même direction le long d’une corde.
L o u is ampliiudc.s sont v„i — 4,0 mm et = 3.0 mm, et leurs
constantes do phase sont respectivement 0 et jr/3 rad. Déterminez
l'am]iliiiidc cl la coiiManie de phase /} de Tonde résultante.
-tii.i
Lorivcz I’ oikIc résultante .sou.s la IVirnic de Téquatioii 2.44.
a) b)
SfllUÎION On jK-m appliquer le contept tlé suivant : les deux ondes ont
Figure 2.16 Exemple 2.6 a) Deux vecteurs de Fre.snel d'amplitudes
certaines propriéle» en coiniiiun; puisqu’elles se propagent dans
.'■|iii et ,v„i2et présentant une différence de phase de jt/3 h) l.a somme
l.a mémo tlirection le l<mg de la même corde, elles doivent avoir
vectorielle de ces vecteurs de Fresnel à tout instant pcndiuii
la même vitesse ë, dont le module est déterminé par la tension ci la
leur rotation donne l’amplitude y;i, et la con.stantc de pha.se fi
tuasse linéique de la corde, d'après Téquation 2.2,‘t. Uc plus, clics
de Tonde résultante.
ont la m im e longueur (Tmide A, et doivent donc avoir le même
nombre d'onde k f Z'f/2,). Cnfin. avec le même nombre d’onde k et
de n B rad, eu on peut additionner les vecteurs â l'aiile de la méthode
le même module de vitesse v, elles doivent avoir la même rréqucncc
des composantes. Pour les composantes horizontales, on a;
angulaire tu ( - kv).
Ün peut appliquer uii .sceiaid concept dé; les ondes (on les nommera = .v,„i cos 0 f y„i2 cos nr/3
onde I et onde 2) peuvent être représentées par des vecteurs de Fresnel
= 4.0 mm + (3.0 mm) cos nB = 5,50 mm.
iiHii nam à la même vitesse angulaire w par rappon à Torigine. Puisque
la constante de phase de Tonde 2 est plus /grande que celle de Tonde D'autre part, pour les composantes verticales, on a :
I de .t/3. le veeieiir de FreMiel 2 doii être en reuml de rr/'t par rapport
au vecteur de Fresnel 1 dans leur rotation en .sens horaire, comme y.'n,v = >ml Sin 0 + y,jj Sin 7T/3
Tillustrc la figure 2,16 a). L'onde résultante pioduiie pai l'interférence = 0 + (3,0 mm) sin nB = 2,60 mm.
des ondes I et 2 peut alors être représentée par un vecteur de Fresnel
Donc. Tonde résultante a une amplitude de
qui est la somme vectorielle des vecteurs de Fre.sncl I et 2.
Pour .simplifier la somme des vecteurs, on a dessiné les vecteurs JV, - \ / (5,.50 nim)'^ + (2,W) mm)-
de Ficsiici I et 2 dans la figure 2.16 a) à l’instant où le vecteur de
Fresnel I coi'ncidc avec Taxe horizontal. On a ensuite dessiné le = 6.1 mm. (réponse)
vecteur Uc Frcsncl 2 (en retard) à un angle positif jr/3 rad par rapport et sa constante de phase est
à J’axe horizontal. Dans la figure 2 16 b), on a déplacé le vecteur
de Fresnel 2 par translation de façon que son origine coïncide avec , 2.60 mm
fi = tan ' — 7------ = 0,44 rad. (réponse)
l’extrémité du vecteur de Fresnel I On peut ensuite tracer le vecteur 5.50 mm
de Fre.snel y'„ reprc.scntani Tonde résultante, de l’origine du vecteur
D’après la figure 2.16 b), la constante de phase fi est entre 0
de Fresnel 1 à Textrémité du vecteur de Fresnel 2. l.a constante de
et t /2 rad ( 1.57 rad). Donc, l’onde résultante est en retard par rapport
pha.se e,st Tangle qu'il forme avec k vecteur de Frcsncl 1.
à Tonde I dans leur parcours, ce retard équivalant à une coastante de
Pour trouver les valeurs de et de /S. on peut faire la somme
phase fi = ‘ 0,44 rad. À Taidc de Téquation 2.44, on peut écrire
des vecteurs de Fresnel 1 et 2 avec une calculatrice munie de fonctions
Tonde résultante ainsi ;
vectorielles en additionnant un vecteur ayant un module de 4,ü et un
angle de 0 rad avec un vecteur ayant un module de 3,0 et un angle y'TV, /) = (6,1 mm) sin(L\ — wf + 0.44 rad). (réponse)

2.11 Les ondes stationnaires


D;uis les deux .sections ptccédenics. on a étudié deux ondes .sinusiVùlales de m êm e longueur
d ’onde et de m em e am plitude, se propageant dans la même direction le long tl’ une cordc
tendue. Q ue sc produit-il si elles se propagent dans des directions opp<isécs ? On peut
encore trouver Tonde résultante en appliquant le princip«^ de superposilitm .
La figure 2.17 est une représentation graphique de la situation. E lle illustre les deux
ondes qui intcifcren t, une se p ro p ag ean t vers la gauche d an s la figure 2.17 a), Tautre
vers la droite dans la figure 2 .17 b). La figure 2 .17 c) représente leur som m e, obtenue en
appliquant graphiquem ent le principe de superposition. L ’o nde résultante possède une
caraciéristiquc intéressante: il y a des endroits le long de la corde, nom m és des n œ u d s,
où la corde n 'a aucun m ouvem ent, cl où les élém ents de la corde ont loujinirs un dépla
cernent nul, v' ~ 0 Q u atre nteuds se m b la b les sont in d iq u é s p ar d es p o in ts d an s la
fig u re 2.17 c) On ap e rço it, à m i-ch em in en tre deux n œ uds v o isin s, les v e n tre s , où
l'am plitude d’o.scillation des élém ents de la corde e^ttmaximale. U ne onde com m e celle
de la figure 2 17 c) se nom m e une o n d e s ta tio n n a ire , car le profil d 'o n d e ne se déplace
ni vers la g au c h e ni vers la d r o ite , la p o sitio n des nteiids et d es v en tres d em eu re
inchangée.
2.11 Les ondes stationnaires 43

■ > r x T

К У К - H d T

гА-Г
U U
—-• X

I- ir / =r

Figure 2.17 a) Cinq insiantatiés


Si deux ondes sinusoïdales de même amplitude ei de même longueur d’nnde se propagent
d ’une onde se propageant vers
dans des directions oppti.wes dans un même milieu, leur interférence prtxluit une onde
la gauche, aux instants f indiqués
stationnaire.
SOU.S la partie c) (7"est la période
d'ciscillfilion.l b) Cinq instamané.s
d’une onde Identique à celle de a), Afin d ’analyser une onde stationnaire, on représente les deux ondes qui interfèrent
mais .se propageant vers la droite, avec les équations
aux mêmes instants / c) Instantanés
correspondant à la superposition }'i(x t) - >n, sin(7c.v - c<;/) (-Z.dS)
des deux ondes sur la même corde.
À f = 0, r/2 et T, une interférence
cl >s (a, t) = sin(fcv + dUf). (2 46)
ctMistructive se produit à cause
de raligncinent des crêtes et des creux.
k f — r/4 et 37/4, une interférence L e principe de .superposition donne, ptnir l'o n d e ré.sullantc,
desu uetive se ptxKluii îi cause
de l'alignement des crêtes avec y'{X, f) - V|(a, /) + ,V2(a. î ) = y „ Sinffcv - ÙJt) + v„ sinffcif + lOt).
les creux. Certains points (les nœuds.
it)diqué,s par des ptrints) n’oscillent L’application de la relation trigonom étrique de l'éq u atio n 2.37 donne
jamais ; certains points (les ventres)
ont la plus forte oscillation.
y'ÎA. t) = r2vm sin(/ü;)| cos(w r), (2.47)

qui est représenté à la figure 2.18. C ette équation ne décrit pas une onde progressive,
puisqu’elle n ’a pas la form e de l’cqualion 2.16. Elle décrit plutôt une onde stationnaire qui
po.ssède la meme fréquence et la même longueur d ’onde que les ondes qui la constituent
P uisque seul le term e cos(ft^r) d épend du tem ps t, il représente le term e oscillant,
celui qui fait oscillei' dans le tem ps un élém ent de corde situé à une position x donnée.
La q u an tité in d iq u ée e n tre cro c h ets d an s l’éq u a tio n 2.47 peut d o n c ê tre vu e co m m e
l ’am p litu d e d e l ’o sc illa tio n d e l’élém e n t d e co rd e situ é à la p o sitio n ». Touiefoi.s.
puisqu’une am plitude est toujours positive et que sin(A») peut être ncgaiif, on pose que
I2y,„ sin(ib)l est l’am plitude de l’oscillation à la position x.
D ans une o nde sim iso’i’dale pro g ressiv e, l’am plitude d e l’o scillatio n est la m êm e
pour tous les élém ents de la cordc. C ela ne s’applique pas dan.s le cas d ’une onde sta­
tionnaire, où l'am p litu d e d 'o scillatio n varie selon la position i Гош l’onde .siniioniiaiic
décrite par l’éq u atio n 2.47. par ex em p le, l ’am p litu d e d ’o scillatio n e s t nulle p o u r les
Diplataanciit valeurs de Lx qui donnent sin(Lr) U. C es valeurs sont
.ï ‘(x. <) (2>iii sinifcqjrostdiifl
kx = П71. où n = 0. -!: 1, ± 2 ,... (2.48)
A iiip lit iu ic e n ii .
à la »>M:iUiinl
pfjsitioii X En substituant к = 2гг/л dans cette équation et en la réarrangeant, on obtient
Figure 2.18 L onde résultante
de l’équation 2.47 e.si iimr onde A" = n —. où n — O, ± 1 , ± 2 , .. . (les positions des nœuds).
stationnaire produite pai f interférence (2.4Q)
de deux ondes sinusoïdales de même
amplitude et de morne longueui d'onde
SC propageant dans des directions qui sont des positions où l’am plitude de l’onde stationnaire de l’cquation 2.47 est nulle,
opposées. faisant en .sorte que y ' = 0 p o u r tout instant t ce (|iii cnrresptind ù ries m ends N o te/ que
44 Chapitre 2 Les ortdes

les noeuds adjacents sont distants de À/2. soit la m oiiié d une longueur d ’onde.
- L 'a m p litu d e d e l’o n d e sta tio n n a ire d e r é q u a tio n 2.47 a un e v aleu r m a x im a le
de 2vni. qui se vérifie pour les valeurs d e kx qui donnent sin(^jr) = ± i. C es valeurs sont

kx = ± \n , ± \ n , ...
= (n + i);r, où « = 0, 1, ± 2 , .. . (2.50)

» Hn substituant k = I n l k dans l’équation 2.50 et en la réarrangount. on obtient

"V où H = 0 ,1 ,2 .... (les positions des ventres). (2.51)


■ ( ■ • » I -
soit les positions où l’am plitude de l’onde stationnaire de l’équation 2.47 c.st m axim ale,
“V fai.saiit en sorte que l’am plitude d ’o.scillation des élém ents de corde est m axim ale à ees
positions, ce qui correspond à des ventres. Les ventres sont distants de Â/2, et sont situés
à m i-cbem in entre les mruds.

Les réflexions à une extrémité


On peut p ro d u ire un e o n d e sta tio n n a ire dans une co rd e len d u e en lai.ssani u ne onde
progressive .se réfléchir à l’extrém ité éloignée de la cordc. de façon à ce q u ’elle revienne
dans la direction opposée. L’onde incidente (initiale) et Tonde rélléchic (qui se propage
dans la direction op p o sée) peu v en t alors être d écrites p a r les éq u atio n s 2.4.5 et 2.46,
et peuvent se superposer |ю иг form er une onde stationnaire.
h) D ans la figure 2.19. on utilise une seule im pulsion pour m ontrer com m ent de telles
réflexions se produi.sent. D ans la figure 2.19 a), la corde est fixée à son extrém ité gauche.
figure 2.19 aj L’no impulsiotr incidente
Q uand l'im p u lsio n arriv e à ce lle ex tré m ité , elle ex erce une fo rce vers le haut su r le
partant de la droite e.si réfléchie
support (le mur). D ’après la troisièm e loi de N ew ton, le support exerce une force dans
à rextrémiié gauche de la eortlc,
la direction opposée (vers le bas) ci de m êm e m odule sur la ctKde. C ette deuxièm e force
qui est attachée à un mur. Notez que génère une im pulsion dans la corde, com m e si on tirait la co rd e vers le bas. et qui se
l’impulsion réfléchie est inversée
propage le long de la corde dans la direction opposée à celle de l’im pulsion incidente.
par rappoit à l’inipulsuMi incidente, D ans une réflexion d e ce type, que Ton nom m e réflexion dure, il doit y avoir un meud
b) L’extrémité gauche de la corde e.st au support, puisque la co rd e y est fixée. L ’im pulsion incidente el Tiinpulsion réfléchie
attachée à un anneau qui peut glisser doivent donc produire une interférence destructive afin q u 'elles s ’annulent à ce point ; par
verticalement et sans frottement
conséquent, l'im p u lsio n réfléchie doit être inversée par rapport à l ’im pulsion incidente.
sur la tige. Notez que. dun.s ce cas. D an s la fig u re 2 .1 9 b), T ex irc m ité g au ch e d e la c o rd e est attac h ée à un an n eau
ritnpulsion n’est pas inversée qui peut glisser sans frottem ent le long d ’une lige. Q uand Tanneau .sc iléplace au passage
par la réflexion. de l’im pulsion, il lire sur la corde, Tétire et poxluit une im pulsion réfléchie non inversée
et de m êm e am plitude que l'im p u lsio n initiale. D onc, dans une telle réflexion, que Ton
nom m e réflexion molle, l'im p u lsio n in cid en te et l ’im p u lsio n réfléch ie se ren fo rcen t
m utuellem ent, créant un ventre à T extréinité de la corde ; le déplacem ent m axim al de la
corde est le double de T am plitudc d e Tune ou l’autre im pulsion.

^V ÉR IFIEZ VOS CONNAISSANCES 6: X’ux ondes de même amplitude et de meme longueur


d'onde interlèient dans trois situations diftérenies cl prochilsem des ondes ré.sultantes décrites
par les équaiioas suivantes :
I ) y'(x /) = 4 sin(5.» — 4/)
2) y'Cv. /1 = 4 sin(5.r) cos(4f)
.l) y'(.ï, 0 = 4 sin(5i + 40.
Dans quelle situation les deux ondes qui interfèrent se propagent-elles a) dans la direction
positive fk: Гяхе des r. b) dans la direction négative de Taxe des r et c) dans des directions
irpposées ?

2.12 Les ondes stationnaires et la résonance


C o n sid érez une c o rd e , un e co rd e d e g u itare par ex em p le, q ui est ten d u e en tre deux
pin ces. S up p o sez q u 'o n en v o ie un e o n d e sin u so ïd ale p ro g re ssiv e d ’un e c e rtain e
fréquence le long de la co rd e, vers la droite. L orsque Tonde atteint T cxtiéniité droite,
elle est réfléchie e t reto u rn e vers la gauche. C ette o n d e ch ev au ch e alors celle qui se
propage toujours vers la droite. Ixtrsquc Tonde réfléchie attemc l'ex trém ité gauche, elle
SC réfléchit do nouveau et une nouvelle onde réfléchie am orce son trajet vers la droite,
chevauchant les o n d es qui se propagent vers la droite cl celles qui se propagent vers la
gauche. En réstimé, on aura bientôt de nom breuses ondes progressives qui Se chevauchent
et interfèrent les unes avec les autres.
2.12 Les ondes stationnaires et la résonance 45

•Ч I .

figure 2.20 Des photographies À certaines fréquences, l’interférence produit une onde stationnai]c com im rtant des
stroboscopiqucs révèlent (de manière nœ uds cl des ventres com m e ceux de la figure 2.20. On dit alors que ces ondes stationnaires
imparfaite) des profils d’ondes se produisent lo rsq u ’il y a ré so n a n c e , et que la corde résonru: à cc.s fréquences données,
stationnaires sur une eorde qu’une tige nom m ées fré q u e n c e s d e ré so n a n c e . Si on fait o sciller la co rd e à une fréquence autre
vibrante fait osciller à l'extrémité qu’une fréquence de résonance, il n’y a pa.s d’onde stationnaire. L’inici fércncc de.s ondes
gauche. Ces profils s'observent se propageant vers la droite et de.s ondes se propageant vers la gauche ne cause que de
à certaines fréquences d'oscillation. faibles (parfo is m êm e im p ercep tib les) o.scillations de la corde. Il y a donc réso n an ce
seulem ent si l’agent extérieur, com m e la tige vibrante située à gauche clans la tlgiire 2.20,
effectue un m ouvem ent à la m êm e fréquence que l’une des fréquences de résonance de
la co rd e, créan t ain si d an s la co rd e un m o u v em en t de fo rte am p litu d e q ui p o u rra it,
éventuellem ent, la casser. Il n 'y a pa.s que le.s cordes qui résonnent ; en fait, tout systèm e
peut résonner. Par exem ple, vous voulez pous.ser une personne qui sc balance su r une
balançoire afin cTaugmenier l'am plitude de son m ouvem ent. Si la balançoire effectue un
cycle p ar seconde (fréquence d e 1 H z), vous a ile / sy nchroniser vo.s ptui.sscc.s avec le
m ouvem ent de la balançoire, soit une poussée p ar seconde (fréquence de I H /) Votre
fréquence doit donc être la m eme que celle de la balançoire, qui représente l’une de sc.s
fréquences de résonance, sinon vos poussées risquent d ’etre très inefficaces (m ê m e avec
b eaucoup d 'a rd e u r) et l'am p litu d e du m ouvem ent de la balan ço ire sera faible. D onc,
un agent extérieur à la balançoire (la personne qui pousse), en effectuant un m ouvem ent
à la m êm e fréquence que l ’une des fréquences de résonance de la balançoire, p ro d u it
une fo rte au g m en ta tio n d e l ’am p litu d e du m o u v em en t de la b alan ço ire, à sa v o ir le
phénom ène d e résonance.
S o it une co rd e ten d u e e n tre deux p in ces, sép arées p a r une d istan ce fixe L. Pour
trouver l’expression des fréquences de résonance de la corde, on note q u ’il doit y avciir
un nœ ud à chacune d e scs ex trém ités, puisque ces extrém ités sont fixes et ne p eu \ eni
osciller. La situ atio n la plus sim ple q ui p erm e t d 'a v o ir des nœ uds aux extrém ités est
celle de la figure 2.21 a), qui illastrc les deux déplacem ents extrêm es d e la corde (une
ligne contin u e et une lig n e p o in tillé e), et que l’on n o m m e le p rem ier m ode 11 n ’y a
q u 'u n seul ventre, soit au centre de la corde. N otez que la m oitié d ’une longueur d ’onde
co uvre la lo n g u eu r L . qui est ici la lo n g u eu r de la corde. D onc, d an s celle viinaiion,
K X/2 = L, c e qui in d iq u e qu e si les o n d es p ro g re ssiv e s se p ro p ag e an t vers la gau ch e

K > 1
et celles sc p ro p ag ean t vers la d ro ite p ro d u isen t ce profil p ar leur in te rfé re n ce , clics
doivent avoir une longueur d ’onde Л, = 2L.
La figure 2.21 b) représente une autre situation sim ple, le deuxièm e mode, où il y a
ég alem en t d es n œ u d s aux e x tré m ité s fixes. R em arq u ez q u ’il y a un nœ ud au centre
de la corde ; c e profil com porte donc trois nœ uds et deux ventrœs. Pour produire un tel
pm fil. les ondes voyageant dans les deux directions doivent avoir une longueur d ’onde
'*-2 = L - 2IJ2. Une troisièm e situation, le troisièm e mode, est illusuée à la figure 2.21 c).
C e profil co m p o rte q u atre n œ uds et tro is ventres ; la lo n g u eu r d 'o n d e cs( Ai = 2Z73.
f - -Âî ==2 A O n pourrait poursuivre cette dém onstration en traçant les m odes suivants, qui ont des
profils de plus en plus com plexes. C liaque nouveau mode aurait un m eud et un ventre de
plus que le m ode précédent et la longueur de la t:ordc L contiendrait A/2 de plas.
Line onde stationnaire peut donc être produite su r une corde de longueur L p ar une
onde dont la longueur d ’o nde est égale à une de ces valeurs :
„ lO X r X D I
2L
A„ = - , ou fl = 1 , 2 ,3 . .. . (2.52)
rt
Figur* 2.21 Une corde tendue entre deux
pinces est soumise à des oscillations n représente donc le m o d e d ’u seiilatio n . L es frécjuences de résonance i otTespnndaui ii
formant des profils d'ondes stationnaires, ces longueurs d ’onde découlent de l’équation 2 .1 2 .
a) Le profil le plus simple est celui ayant
un seul ventre. Les lignes continue et
pointillée représentent les déplacements
^ A = « , ou n - 1 .2 . .L ... (2.33)
A, 21.
extrêmes do la corde, b) Le deuxieme
plus simple profil comporte deux V représente ici le m odule de la vitesse des ondes sc propageant sur la cordc. (Jn appelle
ventres, c) Le profil suivant comporte aussi les fré q u en c es d e réso n an ce d o n n ées p a r l'é q u a tio n 2 53 fré q u e n c e s p r o p r e s
trois ventres. ou fré q u e n c e s h a rm o n iq u e s.
46 Chapitre 2 l-«s crades

Figura 2.22 Une des nombreuses ondes stationnaires pouvant être produites sur une peau de iitnbale.
rendue visible grâce au saupuudrage de poudre m ire sur la peau. Lxirsqtie la peau se met à asciller
à une de ses fréquences de résonance sous reffei du vibreur mécanique qu’on voit dans lu partie
.supérieure gaiiehc de la phiiio. la poudio s’accumule sur les noeuds, qui prennent la ftrrme
de cercles et de lignes droites dans cette démonstration d’oscillations bidimensionnelles.

L’équation 2.53 m ontre que les fréquences de résonance sont des m ultiples entiers
de la p lu s basse fréq u en ce de ré so n a n c e ./) = vUI., qui corres|H )nd à « = I . A cette
fréquence, le m ode d ’oscillalion se nom ine le mode fondmiental. ou premier hurmunique.
Le deuxieme hurrnonupte est le m ode d ’o scillation où n = 2. le troisiènu hm num iquc
est celui où n = 3. et ainsi de suite. Les fréquences associées à ces m odes sont souvent
noice.s/ , . / j . etc. A insi, p o u r le cn iéiu e h arm o n iq u e d ’une co rd e tendue fixée aux
deux cx trc iu ité s. on a / „ — «v/2L = »»/,. L’en sem b le de tous les m o d es d ’o sc illa tio n
possibles SC nom m e lu .série h a m u m iq iic , et n se nom m e aussi le n tim b re h a rm o n iq u e
du énièinc harm onique.
Le phénom ène de résonance est com m un à tous les syslcnies oscillants, et peut se
produire en deux et en trois dim ensions. Par excm iilc, la figure 2.22 présente une onde
.stationnaire hidim ensioiinellc observée sur une peau de tim bale.

' VÉRIFIEZ VOS CONNAISSANCES 7: Une fréquence de résonance (inférieure à 4(X) I izl a été

r
omise duns la séquence suivante ; ! 50. 225. 300, 375 H/, a) Quelle est la fréquence niaiU|Uanie ?
b | Quelle est la iréquence du septième harmonique ?

Exemple 2.7
Duns lu figure 2.23. une corile est aliaehée au point P à un vibreur Vihi (ui
produisani des ondes sinusoïdales dans la corde. La cordc passe
sur un support au point Q et est tendue par un bloc de masse m.
La distance L entre P et Q est de 1,2 m. ki masse linéique de la corde
est 1.6 g/ni. et la fréquence/'du vibreiii est à 120 Hz. Au point P,
l’ampliiude du mouvement est assez petite pour que ce point soit
considéré comme un meud. Il y a également un noeud au point Q. figure 2.23 Exemple 2.7 Une arrdc tendue de longueur L reliée
à un vibreur A une fréquciiec rie vibrations donnée, une onde
a) Quelle masse m permettrait au vibreur de produire le quatrième
stationnaire sera produite, à certaines valeurs de tension dans la corde.
harmonique sur la corde ?
SOLUTION Le (onceftf de utilisé ici est le suivant : la corde ne résonnera En remplaçant »■ de l'équation 2.54 par .son expression donnée
qu’à certaines fréquences, déterminées par le naodtile de la vitesse v
par l’équation 2.55. en substituant n par 4 pour le quatrième har­
de fo n d e et la longueur L de la corde. D ’après l’équation 2.53.
monique et en isolant m, on obtient :
ces fréquences de résonance sont
M .Y-p (2.56)
L Il = 1.2. 3.... (2 54)
21'
Pour produire le quatrième harmonique (pour lequel n = 4), on doit _ (4)(1,2 m)-(12(l Hzr^(),(K)l 6 kg/m )
ajuster le membre droit de cette équation en mettant « — 4, de sorte (4)^(q.8nVs2)
que le membre gauche soit égal à lu fréquence du vibreur ( 120 Hz).
= 0.R46 kg as ü.«5 kg. (réponse)
Puisque L est donnée, on ne |xxit donc fiijii.ster dans ré(|iialion 2 .54.
l'outefoi.s. un autre contepl (le peut être utilisé : on peut ajuster v. car b) Quel m ixlr d ’oscillation est produit lorsque w = 1,00 kg ?
il dépend de ht nia.ssc m qu'on suspend à la cordc. D'après l’équation
2.25, le module de la vitesse de fonde est donné par »’ “ \ / t/p. SOlUîlON: Si on insère celte valeur de m dans l’éqimtion 2 56 et qii’on
Puisque la masse suspendue est immobile, la tension dans la cindc r résout cette équation pour trouver n, on détermine que « ** 3.7.
doit être égale au poids mg du bloc. Donc. Le (ortepl dé. dons ce cas. est que n doit être un nombre entier, et une
valeur de n = 3,7 est donc impossible. Pai conséquent, si m ~ 1.(X) kg.
ÍL - (255) le vibreur ne peut produire une onde stationnaire dans la corde,
\ /« \ P cl toute oscillation de la corde sera faible, sinon iniixTccplible.

RESOUUT'ON PC PROecèMES

2*strotégii : Le.v litimwnique.f dtm.s une rorde 5IÀ/2) = !.. et / 2/,/5. Vous poui ivz ensuite utiliser l'équation 2.12
Si vous devez trouver des iiifiinnaiions eoneernani un hariiUHiique sur (/ = l'/À) pour trouver la fréquence de cet harmonique
une corde tendue de longueur !.. commencez jutr faire un schéma ilc cet Souvenez-vous que la longueur d'onde d’un harmonique n’osi
harmonique (comme celui de lu figure 2.21). Si on vous demande, déterminée que par la longueur de la corde, mais que la fréquence
par exemple, le cinquième harnuinique. vous devrez tracer cinq ventres dépend également du module de la vitesse i de fonde, qui. elle, est
entre les points fixes des deux supports Cela signifiera que cinq ventres, déicrminéc par la tension cl la masse linéique itc lu corde, à l'aide de
chacun de longueur À/2, couvriront la longueur L de la corde. Donc. l’équation 2.25.
Questions 47

RÉVISION É T R É S U H Ç \¿^J

Les ondes transversales et longitudinales Les ondes incca- ÎM superposition d ’ondes Lorsque deux ou plusieurs ondes
niques ne peuvetit exister sans support maiénel et obéissent aux lois traversent le même milieu, le déplacement de toute particule
de Newton. Les ondes mécaniques transversales, comme celles du milieu est la somme des déplacements ipie les ondes individuelles
qu'on trouve dans une cirrdc tendue, sont des ondes dans lesquelles produiraient.
les particules du milieu oscillent perpendiculairement à la direction
de propagulion des ondes. Les ondes dans le.squelles les particules du L ’interférence des ondes Deu.s ondes sinusoïdales se propageant
milieu oscilicni parallèlement à la direction de propagation des ondes dans la même corde priKliii.seiit de rin terréreiii e. eu s’additionnant
selon le principe de superposition Si les deux ondes se propagent
sont dc.s ondes longitudinales.
dans la même direction, ont la même amplitude >■„ et la meme
J^S ondes sinusoïdales Lne onde sinusoïdale se déplaçant fréquence (donc, la même longueur d’onde), mai.s diffticiii quant à leur
dans la direction positive de l’axe des x a la forme mathématique pha.se d ’une con-stante de phase à>. elles produisent alors une seule
onde ayant cette même fréquence ;
y(s. n = ym sin(Lï - wt + d)). (2.2)
v’iJr. r) ■ [2_v,„ co.s(d>/2)] simLr - œi t jé ). (2,.1fi)
où est l'um plitude de l’onde, k est le nom bre d'o n d e, co est la
fréquence angulaire, é est la consbante de phase et kx — lot + é Si <5 = 0 (ou un inuliiple entier de 2tr), les ondes sont parfaitement
est la phase, l a longueur d ’onde À est rcltée à k par en phase et leur intciféreiicc est constructive; si n rad (ou un
multiple entier impair de rr). elles sont par%iicment dé|>tia.\éc.s et leur
2;r
=— . (2.5) interférence est destructive.

La période 7 et la fréqiicnce/de l’onde sont reliées à a» par Les vecteurs de Fresnel Une onde v(.r. r) peut être représentée
par un vecteur de Fresnel. L'n vecteur île Fresnel est un vcucur dont
1 le module est égal à l’amplitude y^ de Fonde, et d o it l’orientation,
(2.9)
T' par rapport à un axe horizontal, est égale à In phase do Fonde.
Il tourne par rapport A l'origine à une vitcs.se angulaire égale à la
Enfin. le module de la vitesse r de l’onde est relié à ces paramètres p;u
fréquence angulaire o) de Fonde. La projection du vecteur de Fresnel
en rotation sur un axe vertical donne le déplacement y d ’un point
i' = - = - = Xf ( 2 . 12) situé sur le trajet de Fonde.
*’ k i ^■
Les ondes stationnaires L’interference de deux ondes sinusoï­
L 'éq u a tio n d 'u n e on d e progressive Toute fonction de la
dales de même longueur d'omlc. île nicme iimpliuide et sc déplaçant
forme
dans des direction.s opposées produit de.s ondes stationnaires. Dans
W.ï, /) —Mkx — lot) (2.16) une corde dont les extrémités sont fixes. Fonde stationnaire est
donnée par
peut représenter une tinde progressive dont le module de la vitesse
est donné par l’cquatioii 2.12, et dont le profd d'onde est donné par y'(r. f) = sin(Lr)] cost'tnf). (2.47)
la forme niathénialiqiie de la fonction h. Si le signe devant w est Les ondes stationnaires soni caractérisées par dc.s endroits qui
positti. l’onde SC propage dans la direction négative de l’axe des x. n’effectuent aucun mouvenieiii, les nw uds, ci des endroits où
et si le signe devant a>est négatif, l’onde sc propage dans la direction Fampliiiide du niouveineni est maximale, les ventres.
positive île l’axe de.s .r.
L a résonance Des ondes stationnaires peuvent être produites
La vitesse d ’u n e onde d a n s u n e corde ten d u e l a vitesse dans une cordc par la réflexion des ondes progressives venant des
d’une onde dans une ciirik; tendue est déterminée par les propriétés extrémités de la conie. .Si une extrémité esi fixe, on doit y trouver un
de la corde. Dans une corde dont le module de la tension est r et la noeud. Cela limite les fréquences des ondes stationnaires dans une cordc
inas.se linéique est ju. le module de ki vites.se est donnée. Chaque fréquence possible est une fréquence de résonance,
et Fonde stationnaire correspondante est un m ode d'oscillation.
(2.25) Dans une corde tendue de longueur l. dont les cxnéiniics sont fixc.s.
les fréquentes de ré.sonancc .sont

ÎM puissance La puissance moyenne, ou taux moyen auquel fn = 1.2, .i,... (2.5 V)


l'énergie est transmise par une onde sinusoïdale daiw une corde tendue, K 2C
est donnée par Lx' m o d e d ’ o s c illa tio n c o r r e s p iin d a iil à n — I .se n o m m e le mode
fondamental, ou p ir m i e r hamwnique; le m ode ciMTCspondant àn — 2
Ftntvf spsxô~y^. (2.V2)
se n o m m e le deuxième mode ou deuxième Iwmt inique, cl ainsi de suite.

O U C S T fO r^ S

I Quelle est la longueur d ’onde de Fonde de la figure 2.24. dont 2. La figure 2.25 a) est un instantané d'une onde .se propagcani dans
chaque segment a une longueur d ’ la direction positive de Fa.xc des \ le long d’une corde sous tension,
t^ a tre éléments de corde y sont indiqués par les points o. h e et d.
Fiéterminez, dans chaque cas, si l'clénicnl se déplttcc vers le haut ou
vers le bas, ou s’il est momeiilanéiiienl au re|V)s Un/Iice: visiiali.vc/
le mouvement de Fonde à ccl instant.)

figure 2.24 (^lesiion


46 Chapitre 2 Les ondes

l-a ligure 2.25 b) illu<:tre le déplacement d’un élément de corde 6. Si vou.s produisez dans une cordc deux ondes sinusoïdales de
situe à Ji = 0. en fonciiuii du icnips. Aux instants indiqués par les même amplitude, de même longueur d ’onde et en phase, et que
lettres, l 'élément se déplace t il vers le haut ou vers le bas, t»i cst-il vous décalez ensuite l’une d’elles de 5,4 longueurs d’ondc. quel ty pe
Muuiictuancmcni au repo.s'' d’interférence se produira-t-il dans la corde si les onde.s se propagent
>' dans la même direction ?
7. I.es lunplitudcs et les différences de jrhase (déphasages) de 4 paires
d ’ondes de même longueur d ’onde sont a) 2 mm. 6 mm et n lad.
b) 3 mm. 5 mm et rr rad, c) 7 mm, V mm et ?r rad, et d) 2 mm, 2 mm
et 0 rad. Les paires d’ondes sc propagent toutes dans la même direction
et dans la même corde. San.s calculs écrits, classez les quatre paires
en ordre décroissant .selon l’amplitude de leur onde résultante.
a) [indice: trace/, des diagrammes de Frcsncl.)
Figure ? 25 Question 2 8. Si vous produisez le septicnic harmonique dans une corde,
a) amibien y a-i-il de noeuds ? h) Que irouve-t on au centre de la corde ;
3. La figure 2.26 c.sl un instantané à r - 0 d'uiic omle sinusoïdale sc un nœud, un ventre ou un état intermédiaire ? Si vous produisez ensuite
propageant vers la droite, et oii cinq points sont indiqués. Quelle est le sixième harmonique, c) la longueur d’onde e,st-elle plus longue ou
la ditlérenee de phase entre le point 1 et a) le point 2, b) le point 3. plus courte que celle du .septième liai nionique ? d) Sa frctiuenee de
«) le point 4 et d) le intiiii 5 ? liAprimcz votre réfionsc en radians, puis résonance est-elle plus élevée ou plus basse
sous forme de Itmgtieur d'onde. I.a figure indique que le dc^ilaeement
9. Les cordes A et B oui la même longueur et la iiiêine nia.sse linéique,
est nui à jf = U. Lxpnmcs en fonction de la péritxic T de l’onde, à quel
mais la corde B est soumise à une plus grande tcn.sion que la corde A,
premier instant c) une eróle el f) le prochain déplatcnicra nul seront-ils
La figure 2.26 présente quatre situations, a) à d), où il y a des modes
observés a jt = Ü?
d’ondes stationnaires dans les deux cordes. Dans quelle situation est-
il possible que les cordes A et B oscillent à la même fréquence ?
A C^jrdt* B

4. Les quatre ondes suivantes sont priKluiies dans des cordes ayant
la même masse linéique (.v est en mètres et t est en .secondes). Classez
les ondes en ordre décroissant scion a) le module de leur vitesse et b)
la tension dans la corde où elles se propagent :

I ) >'i = (3 mm) sin(.r — 3r), 3) y? = ( 1 mm) sin(4x - t).


2) V2 = Í6 mit)) sin(2v — t). 4) >1, — (2 mm) sin(.v 2l).
figure 2.28 Question h
5. À la figure 2.27, l'onde I a une crête rectangulaire d'une hauteur
de 4 imités et de laigeur d. et un creux rectangulaire d’une profondeur
10. a) Si une onde stationnaire se propageant dans une corde est
de 2 unités et rie largeur d. L’onde se propage ver.s la droite le long de
donnée par
l'axe des ,v. L’onde 2 a la meme forme que l’onde I, alors que les
ondes 3 et 4 .sont inversées par rapport à l’onde 1 (dnùtc <-> gauche y'(x, t) (3 mm) sin(.5x) cos(4r).
pour l’onde 3 et haut bas pour l'onde 4). Les ondes 2, 3 cl 4
y a-t-il un nœud ou un ventre à x 0 ? b) Si l'onde stationnaire est
SC propagent vers la gauche le Umg de Taxe des x et traversent l'onde 1.
donnée par
Avec laquelle de ces trois ondes rinicrfércncc donnera-t-elle, à un
instant donné, a) le creux le plus profond, b) une ligne droite y'(ï, /) = (3 mm) sin(5.ï + nl2) cos(4f),
et cl une crête dont la largeur sera 2<7?
y a-i-il un nœud ou un ventre à v = 0

_r 11 a) Dans la figure 2.23 de l’exemple 2.7, si on augmente graduel­


lement la niasse du bloc (la frét)uencc restant la même), de nouveaux
modes de résonance apparaissent. Les nombres harmoniques des
nouveaux modes de résonance vont-ils en augmentant ou en dimi­
1) 2)
nuant? b) Ixi passage d’un mode de résonance à un autre cst-il
giaduel, ou siirvicnt-ii quand l’un di.sparaît complètement avant que
le .suivant n'apparaisse ?
1 _

5) 4)
figure 2.27 Question 5
Exercices et problèmes 49

EX ER CICES .

y en fonction de la position des éléments de corde x (la portion


www I m solution se trouve sur le site Web, à l’adresse ci-dessous;
comprise entre .v = 0 et z- = 20 cm) à l’instant f = 0 bi Quel est le
www.dlciiKgrawhill.ca/physiquc
module de la vitesse de Fonde? c) Écrivez l’équation de Fonde,
en évaluant toutes les constantes, d) Quelle esl la vitessi' tran.svci'sale
SECTION 2.5 La vitesse d'une onde sinusoïdale pogressive de l'élément à x ~ 0 et à l’instant / — 5,0 s ?
1E. Une onde a une fréquence angulaire de 110 rad/s et une longueur 9P. Lne onde sinusoïdale a une fréquence de 5(K) Hz ei une vitesse
d’onde de 1,80 m. Calcule/ a) le nombre d ’onde et b) le module ayant un mcxlulc de .8.50 m/s. a) Quelle distance y a l-il entre deux
de la vitesse de l'onde. points sur la corde cjui présentent une différenc'e de phase de tt/ 8 rad ?
2 t Le module de la vitesse des ondes électromagnétiques (qui compren­ b) Quelle esl la différence de phase entre deux déplacements d'un
nent la lumière visible, les ondes radio et les rayons X) est de certain point s’il y a un délai de 1,00 ms enire les deux ? www
8,0 X 10" m/s dans le vide, a) Les ondes de la lumière visible ont
des longueurs d ’onde allant de 400 nm dans le violet à 700 nm SECTION 2,6 La vitesse d'une onde dans une torde fendue
dans le rouge. Quelle est la gamme de fréquences de ces ondes? lOE. La masse linéique de la plus grosse corde d’un violon esl de
b) Les fréquences des ondes courtes radio O'adio FM et television VHf-, .8,0 g/m. et celle de la plus petite corde est de 0,20 g/m. Si les deux
par exemple) vont de 1.5 MHz à 800 MHz. Quelle est la gamme cordes sont faites du nicinc matériau, quel esl le rap|x>it entre
correspondante de longueurs d’onde? c) Les longueurs d ’onde le diamètre de la pins grosse corde el¡ et celui de la plus petite corde
des rayons X vont de 5,0 nm à environ 1,0 x 10 nm. Quelle est la 2 Sf)it i/|/if-.?
(¡ ,
gamme de fréquences des rayons X
IIE. Quel est le module de la vitesse d ’une onde tr.'insversalc kc
3E. Une onde sinusoïdale progressive se propage dans une corde. propageant dans une corde ayant une longueur de 2,00 ni et une niasse
Pour un point donné, le temps qui s’écoule entre un déplacement de 60,(1 g, et qui est soumise à une tensifin dont le module e.si de ,500 N ’’
maximal et le pnxthain déplacement nul est de 0.170 s. Quelles sont
12E. On double la tension d'un fil fixé à scs deux extrémités, sans
a) lu période et b) la fréquence de cette onde ? c) La longueur d’onde
modifier sensibiemem la longueur du fil entre les fixations, Quel esL
est 1.40 ni ; quel est le mixiule de la vites.se de Fonde?
pour les ondes transversales circulant dans cc tïl. le rapp<rrt entre
4E. Écrivez l’équation d’onde d'une onde sinusoïdale se ¡iropageam le module de la nouvelle vitesse des ondes et le module de la
dans la direction négative de l’axe des x et ayant une amplitude de vitesse initiale soit
0,010 m, une fréquence de 550 Hz. utic constante de phase de n/4.
13E. La masse linéique d’une coidc est de 1.6 x 10“’’ Lg/m. l ne onde
et une vitesse demi le module est de 830 m/s.
üansvcrsalc parcfuirani eette corde est décrite par I équation
5L Démontrez, que
y - (0,021 m )sin[(2,0m 'Xi f (.10s-')rl.
y = >■„ sinf^(x - V’Î)] y= sin iTi
< : - ^')]- Quels sont a) le module tic la vitcs.se de Fonde et h) la tension dans
la corde ?
y-y„sin w > =>raS'n 2TT
'G - f ) ] 14E. Une onde transversale sc propageant dans une corde est décrite
par cette équation ;
sont tous équivalents à y = Vm sin(/cr - wt).
y = (2,0 mm) sin[(20 m “ ')j: — (600 s“ ' )/].
éP. L’équation d’onde d ’une onde transversale sc déplaçant dans une
très longue cordc est y = 6,00 sin(0,020 OTr.r + 4.00^/ — 1,2‘in), La corde est soumise à une tension ayant un module de 15 N. a) Quel
où y et y sont exprimés en centimètres, et i est exprimé en secondes. esl le module de la vitesse de Fonde? b) Trouvez la mas.se linéique
Déterminez a) l'amplitude, b) la longueur d’onde, c) la fréquence, de cette corde, en grammes par mètre.
d) le module de la vitesse de propagation de Fonde et c) le module de ISP Ifne corde tendue a une masse par unité de longueur de 5.00 g/cm.
la vitesse transversale maximale d’un élément de la corde, f) (^lellc est et est soumise à une tension ayant un module de 10.0 N. Lne onde
la direction de la propagation de Fonde? g) Quel est le déplacement sinusoïdale ,se propage dans cette corde dans la direction négative de
transver.sal de l’élément situé à x .8.50 cm à l’instant t = 0.260 s ? Faxe des x : cette onde a une ainpliuide de 0 .120 inin et une fréquence
If. a) Éenvez une équation décnvani une onde transversale sinusoïdale de HX) Hz. Si à / = Ole point de la corde situé à.v = 0 a undcplaccmcnl
se propageant tians une corde dans la direction positive de l’axe de.s x. de 0 ,1(X) mm et une vitesse transversale oncnlée vers la partie négartve
Cl ayant une longueur d’onde de 10.0 cm. une fréquence de 400 Hz, (le Faxe des y. écrivez une éfjiiation pour décriie cette onde
une constante de phase fie 4.00 ratl et une amplitude de 2.00 cm. 16P. Quel esl le module de la vitesse maximale d'une onde transversale
b) Quel est le module de la vitesse maximale d'un point donne de la pouvant voyager dans un fil d ’.tcier? Pour des taisons de sécurité, la
corde c) Quel est le module de la vitesse de Fonde ? contrainte de traction maximale à laquelle les fils d’acicr peuvent
8P. Une onde sinusoïdale iransvcr- y (<-,n) être soumis est de 7,0 x 10" N/m‘. L’acier a une masse volumique
sale, dont la longueur d onde est de 7 XOO fcg/m'^ Démontrez que votre résultat ne dépend pits du
de 20 cm, .se fléplace dans une corde diamètre du ill.
dans la direction positive de l’axe 17P. line onde sinuso’idale transversale d ’amplitude y„, ayant une
des X. Le déplacement transversal longueur (Fonde A se propage dans une corde tendue a) Trouvez le
- 4 .0
d’un élément de corde situé à .r = 0, rapport entre le mixlule de la vitesse maximale d un élément
en fonction du temps, est intliqué (la vitesse à laquelle un élément de corde se dépla(.c perpendiculaire­
à la figure 2,2è. a) Faites un gra- figure 2.79 PioblèmeS ment par rapport à Fonde) et le module de la vitesse de 1 (>ndc,
phitjue du iléplacemcnt de la corde soit v,T,/v, b) Si une onde d’une amplitude et d'une longueur d’onde
50 Chapitre 2 Les ondes

données CM priHliiiic dans la cnrdc, ce lappori de vitesse dcpend-il de temps de 30.0 ms entre chaque impulsion, à quel endroit Ic's impul­
du mntenau constituant la cordc. qui peut être, par exemple, de l'acier sions se rencontreront-elles ? vrwv^
ou du nylon 22P. La petite bande de caotilehouc utilisée à riniéricur de certaines
I8P. L'nc onde sinusoïdale sc propage dans une cordc à une vitesse halles de golf et de baseball obcii à la loi de Hooke dans une gamme
ayant un moiliile de 40 cm/s. À r = 10 cm, le déplacement de étendue d'éiiremcnts. Un segment de cette bande a une longueur non
rélciuciil de la corde varie en fonction du temps selon l'équatuin tendue f et une masse rn. Lorsqu’une force F est appliquée, la bande
r = ^ ^ .llcm ;sin|l,0 — (4,0 s ')r|. La masse linéique de la corde est s’étire d’une longueur additionnelle A f. a) Quel est le module de la
4.0 g/cm. Quelles vont al la fréquence et b) ta longueur d’onde vites.se (en fonction de m. de Af et de la constante d'élicsticitc k) des
de l’onde ? e) Cerivert rcqiiatioii générale donnant le déplacement ondes transversales sc prop.igeani dans celte bande éla.siiqiie étirée'.'
tranfivorsal des éléments de la eoide en foiiclion de la position et du b) A l’aide de votre réponse fournie en b), démontrez que le temps
temps, tl) Calcule/ la tension dans la corde. requis par une impulsion transversale pour pareouiii ki longueur
IVP. Une onde sinusoïdale transversale sc propage dans une corde de la bande de caoutchouc est proportionnel h 1 / v ^ .si Aï <K é cl est
dans la direction negative de l'axe des x. La figure 2..10 est une constant si Af » t.
a-préseiuaiioii graphique du déplacement en fonction de la position 23P. l.inc corde uniforme de masse ni et de longucui /. e.vl suspendue
sur la cordc à i'insiaiil i — 0 , rordoim éc à l'origine est de 4.0 cm à un plafond, a) Démontrez que le mixlulc de la vitesse d'une onde
t Mcorde est stiumise à une tension dont le module est de 3.6 N et sa transversale se propageant dans la corde dépend de la distance y
masse liuckjiir est île 2.5 g/m. Trouve/ a) ramplitude. b) la longueur à partir du bas de la corde, et est donne par v = .^/jo • Démontiez
d ’onde, c) le module de la vitesse et d) la période de I'oikIc. e) Tïouvez que le temps requis par une tinde transversale pour parcourir la
le module de la viics.vc iransversalc maximale d' un élément de lu longueur de la cordc tist donné par t — (AttentUm : kt variable
ciMile. t) Écrive/ une équation décrivant l’onde progressive, vrwv y spétrifiée ici n ’est pas le déplacement transversal des éléments de
la cordc.)

SFCTiON 2.7 L'énergie et lo puissonce d'une onde sinusdi'dale dam une torde

24f Une corde dans laquelle des ondes peuvent se propager a une
longueur de 2.70 m et une masse de 260 g. Le module de la tension
dans la cordc est de 36.0 .N. Quelle doit être la frequence des ondes
sintisoïdales progressives d ’une amplitude de 7,70 mm pour que leur
puis-sancc moyenne .soit de 85.0 W ''
25P. Une onde .siiuisoidale transversale est générée à l’extrémitc
d'une longue corde horizontale par une tige qui oscille de bas en haut
sur une distance de 1,00 cm. Ix; mouvement c*st continu et se répète
avec régularité 120 fois par seconde. La corde a une mas.se linéique
de 120 g/in et est soumise à une tension ayant un module de 90.0 N.
Trouvez la valeur maximale a) du module de la vitesse transversale
du point de la c«)rdc et b) de la compo.sante transversale de la tension t .
20P. Diurs lu figure 2.31 a), la corde I a une masse linéique de 3,(X) g/m, (IntUce: cette composante est t sin 0. où (/est l'angle que la corde
et la corde 2 a une masse linéique de 5.00 g/m. Le bloc suspendu de forme avec l'horizontale. Vous devrez relier l ’angle 6 à r/y/r/.ï.)
masse M — .500 g permet aux cordes d'être tendues. Calculez le e) Dénumtrez que les deux valeurs maximales calculées ci -dessus
module de la vitesse d ’une onde se propageant dans af la corde 1 et sont atteintes au cours de phases identiques de l’onde. Quel est le
h) la cmdc 2. (Indice: lorsqu'une corde est enroulée sur la moitié de déplacement transversal y de la corde durant ces phases ? d) Quel
la jante d’une poulie, elle tire sur la poulie avec une force résultante est le taux maximal de transfert d ’énergie le long de la corde .’
égale au double de la tension de la eorde.) Le blix; est ensuite divisé e) Quel est le déplacement traasversai y lorsque ce transfert maximal
en deux blocs (où M\ H Мл = M). et le dispositif est réarrangé .se produit ? D Quel est le taux minimal de tran.sfen d'énergie le long
comme le montre la figure 2.31 b). Déterminez c) Af, et d) M. afin de la corde"' g) Quel est le <lcplacement transversal y lorsque ce
que les modules des vitesses des ondes se propageant dans le,s cordes transfert minimal se produit ? vrww
.soient égaux.

( orde 1 SECTION 2.9 L'inlerférente des otides


Corde 2
24E Si deux ondes sinuso'i'dates progressives xc propageant dans la
meme direction dans une corde tendue sont identiques sauf en ce qui
concerne leur phase, quelle différence de phase produira une onde
résultante ayant une amplitude 1,50 fois plus élevée que celle des
deux ondes initiales '? É ctIvcz votre réponse en a) degrés, b) radians
et c) longueurs d’tmdc.
27t Deux ondes sinusoïdales progressives de même amplitude et
de même longueur d’onde, .se propageant dnns la même direction
a) h) dans une corde, .sont dépha.sécs de .t /2 rad. Quelle est l'amplitude
de fonde résultante, par rapport à l'amplitude initiale y„. des deux
figure Z31 l’roblè.mc 20
ondes qui interfèrent 7
2iP. Un fil d’une longueur de 10,0 m et d'une niasse de 100 g est 28P. Deux ondes sinusoïdales identiques, sauf en ce qui concerne
.soumis à une tension ayant un module de 250 N. Si on transmet deux leur pha.se, se propagent dans la même dircciiim «tons une corde,
impulsions, une a chaque extrémité du fil, en laissant un intervalle e! leur interférence produit une onde résultante donnée par
Exercices et problèmes SI

y'(x. r) = (3.0 mm) stn(20.v —4,0f + 0.820 rad). où x est en mètres 38E. Une corde A est tendue entre deux pinces situées à une distance L
et f est en secondes. Quelles «>nt a) la longueur d'onde X des deux ondes, l’une de l’autre. Une corde B. qui a la même masse linéique et est
h) la différence de phase entre ces ondes et c) leur amplitude y„ ? soumise à la même tension que la corde A. est tendue entre deux
f pinces situées ù une distance 4L l’une de l’autre. Considérez les huit
SECTION 2.10 Les vedeurs de Fresnel premiers harmoniques de la corde B. Le cas échéant, lesquels de ces
harmoniques ont une fréquence de résonance égale à une des
Wf. Déterminez l’amplitude de l’onde résultante lorsque deux ondes fréquences de résonance de la «rorde A ?
sinusoïdales de même fréquence et ,se propageant dans la même
39P. l ine corde tendue entre deux supptins fi xes situés ù 75,0 cm l'un
direction SC superposent dans une corde, et ont des amplitudes
de l’autre a des fréquences de résrin.incc de 420 Hz et de .H 5 Hz,
respectives de 3,0 cm et de 4.0 cm. et des constantes de phase de
sans fréquence de résonance intermédiaire. Quels sont a) Ui plus has.se
Oct de -t /2 rad
fréquence de ré.sonance et h) le nxidule do la vitesse des ontles I wu/w
3CP. Deux ondes sinuso'i'dalcs ont la même période, des amplitudes
40P Dans la ligure 2.33. deux impulsions de 2.0 cm de largeur se
respectives de 5.0 mm et de 7.t) rnm. et se propagent dans la même
propagent dans des directions opposées dans une corde. Le module
direction dans une corde tendue ; elles produisent une oixle résultante
de la vite.sse v des ondes est de 2.0 mis cl il y a une di.stancc de
dont l'am plitude est de 9.0 mm. L’onde qui a une amplitude de
6,0 cm entre les impulsions à ; = 0. a) Faites des schémas qiialitalil.s
5,0 mm possède une constante de phase de 0. Quelle est la constante
de la forme de la corde aux instants t — 5.0 ms. 10 nvs. 15 ms, 20 ms
de phase de l’onde dont l'amplitude est de 7,0 mm?
et 25 ms. b) Quel type d ’énergie (ou forme d'énergie) les impulsions
31P. Trois ondes sinuso'idalcs de meme fréquence se propagent dans ont-elles à t = 15 ms?
une corde dans la direction positive de l’axe des x. Files ont, respec­
tivement. une amplitude de y,, de y,/2 et de y|/3. cl une constante ---- -
de pha.se de 0. de ,il2 et de n. a) Quelle est l’amplitude et b) quelle est
la constante de phase de l’onde résultante ? e) Faites un graphique de
l’onde résultante à / = 0. et expliquez son comportement lorsque r I -V
I
augmente, wvrw
Figure 2.33 Problème 40
SECTION 2.T2 Les ondes slolionnoires et lo résonante
41 P. L’équation » 110.11110 tiécril l’oscillation d’une cordc;
32E. Une corde soumise à une tension ayant un nuidulc (xsciIIc au
troisième harmonique à la frcqucnec,/i. cl les ondes se propageant dans y' = (0.50cm ) cm“ ' ^ x j cosl(40)Ts ’ ir|.
la corde ont une longueur d'onde X-,. Si la tension dans la corde est aug­
mentée à tf —4 t, et que la corde oscille encore au troisième harmonique,
Quels sont a) l’amplitude et b) le iividule de la vitesse des ik'iix ondes
quelles .sont alors a) In fréquence de l’oscillation exprimée en fonction
dont la superposition produit cette oscillation (les oixles sont identiques
d e /, et b) la longueur d’onde des ondes exprimée en fonction de X, ?
sauf en ce qui concerne leur direction) ? c) Quelle est la distance
33E line corde de guitare en nylon a une mas.se linéique de 7,20 g/m entre deux nœuds adjacents ? d) Quelle est la vitcs.se d'un élément de
et est soumise â une tension ayant un module de 150 N. Les supports corde à la position x = 1,5 cru X/ — 9/8 s
fixes de la corde sont distants de 90,0 cm. La corde oscille et prtxluil
42P Lîne onde stationiiaiie' est la somme des deux ondes progres.sive«
l'onde stationnaire illustrée à la ligure 2.32. Calculez a) le module
transversales suivantes.
de la vitesse, b) la longueur d’onde et c) la fréquence des ondes pro-
gressivc.s dont la superposition produit celte onde statitmnaire www V, = 0.05Î) cosfTTr —4TTt)

90,0 rm et \'2 = 0,050 cos(7Tji + 4 t /),

où .V, y, et Vj sont exprimés en mètres, et I en secondes, a) Quelle est la


plus petite valeur positive de x qui correspond à un nœud?
figure 2.32 E x e rc ic e 33 b) Dans l'intervalle 0 S r £ 0.50 s, à quels instants l’élément situé
à x = 0 aura t il uik vitesse nulle?
34E. Deux ondes sinusoïdales de mémiî longueur d ’onde et de meme 43p. Une corde de 3.0 nr de longueur oscille sous l’effet d ’une onde
amplitude se pnipagent dans des directions opposées dans une corde, stationnaire qui comporte trois ventres d'une amplitude de 1 0 cm
à une vitesse ayant un mixlulc de 10 em/s. S’il s’écoule un temps de Le module de la vitesse des ondes est de 100 m/s. a) Quelle en est
0.50 s entre deux Instants successifs où la corde est droite, quelle est la fréquence? b) Écrivez les équations des deux ondes qui. en se
la longueur d’onde des ondes ’’ combinant, produisent cette onde stationnairp
351 Une corde fixée à ses deux extrémités mesure 8,40 m de longueur 44P Lors d’une expérience sur les ondes stationnaires, une corde
et a une masse de 0 .120 kg. Elle oscille cl est soumise à une tension d ’une longucui de 90 cm est attachée à la pointe d ’un diapason qui
ayant un module île 96,0 N. a) Quel est le nKxliile de la vitesse des ondes oscille perpendiculairement à la corde et à une fréquence de 60 Hz
>c propageant dans ta corde? b) Quelle e.sl la plus graiKlc longueur I a corde a une masse de 0.044 kg. Quelle doit être la tension dans la
d'onde qu'une onde stationDairc peut avoir dans cette corde ? c) Déter­ corde (des mas.ses sont attachées il raiiiie extrémité et susperKliie« dans
minez la héqucnce de cette onde ayant la plus grande longueur d’onde. les airs) poui produire iinr oiulc stationnaire ayant quatre ventres?
3éf Une eorilc longiKi de 125 cm a une masse de 2.00 g. Elle est étirée 45p. Une oscillation de 600 Hz provenant d ’un diapason produit
entre des supports fixes produisant une tension »tans la corde de des ondes stationnuircs dans une corde fixée à ses deux extrémités.
7.tKI N. a) Quel est le module de la vitesse des ondes .se propageant Le module de la vitesse des ondes sc pnipageant dans la corde est
dans celle corde h) Quelle est la plus basse fréquence de résonance ? de 4CK) m/s. L'tinde stationnaire a quatre vonites et une amplitude de
37F Qiietli5> sont lc‘s trois plus basses fréquences des ondes stationnaires 2.0 mm. a) Quelle est la longueui de la corde ? h) Éi rivez une ét|ualion
ve pnipagrant dans un fil d'une longiiem de 10,0 in et d’une masse de décrivant le déplacement de la corde en fonction de la position et
100 g qui est tendu de façon à produire une tension dans le fil de 250 N du temps.
52 Chopitre 2 Les ondes

4éP U ne o in le. soumise à une tension ayant un т«ч1и1е de 200 N composé, qui supporte un bkx de mas.se m — 10.0 kg, est placé de Caçoii
et iÏACe à acô deuA cAtréinites, oscille uu deuxième harmonique. à ce que la distance io entre le joint et la poulie de .support .soit de
Le déplacement de la corde est donne par 86,6 cm. Des ondes transversales stint produites dans le fil sous l’ettet
y = (0,10 nij .sintJT.r/2) sin(12Ttj, du mtHivcmenl d’une source externe de fréquence variable, un nœud
est .situé à la [xxilie. a) Trouvez la plus basse fréquence d’cxciiatiun
ou ir = 0 à une cvtrémité de la corde ci est exprime en mètres, alors à laquelle fonde stationiiairc résullante a un nteud qui coïncide avec
que t est exprimé en sixihkIcs. Quelles sont a) la longueur de la corde, la jonction des fils, b) Combien de nœuds y a-t-il à cette trcqucncc ?
h) le module de In vitc-ssc des ondes dans la corde et c) la masse de
la Kirdc'' di Si la corde oscille au troisième liarinoniquc, quelle .sera
Probicfiie supplémentaire
lu perivaie d’oscillation
47P. Л une cxtrcm ilé d 'u n e 1res longue corde, un générateur prtxJuit 52P. Une veste parc-halles. Lorsqu'un projectile i|iii voyage à hante
une oncle donnée par vitesse, comme une balle ou un échu de bombe, frappe une veste
pare-balles de modèle récent, le tis.su de la veste stoppe le pnijectilc
y — (6.0 cm) tüs 1(2.0 iad/ni),r (8,0 rad/s)/| et empêche sa pénétration par une diffusion l apide de son énergie sur
une grande .surface. Cette diffusion s’effectue au nxiycn d’mipulsions
alors qu’à l’autre extrenute de lu corde, un autre générateur produil longitudinales et transversales qui sc déplacent de fa^on nidialt à
Гишк' partir du point d’impact, où le projectile produit un creux de forme
conique dans la trame du tissu. L'impulsion longitudinale, qui sc
V = (fi.O cnij cos ^ [(2.0 rad/m).v + (8.0 rad/s)t] prttpage dans les fibres du tissu à une vitesse ayant un nxidtile v, vers
le creux, provoque un étirement et un amincisscnwnt des libres, et le
Calculer, a) la fréquence, b) la longucui d'onde, cl c) le mrtdule
tissu s’étire d’une façon radiale vers l'intérieur du creux, Une fibre
de la vitesse de chaque onde À quelles valeurs de к sont situés
radiale de ce type est illustrée dans la figure 2.36 a). Une partie de
d) les nœuds et c) les ventres '!
l’énergie du projectile sert à ce nxHivement et à cet étirement. L’oixle
4ЙР l ine onde stationnaire .se propageant dans une corde est décrite par transversale, sc déplaçant à une plus petite vitesse v,, est produite
v(,v. i) 0,040 .sin(5^A) со»(40я0, par la formation du creux «inique. Coninx: le projectile aiiginenu.' la
profondeur du creux, il en augmente aussi le rayon, ce qui force
nh Дet >■som exprimés en mètres, et / en secondes, a) Détermine/ la les fibres à sc déplacer dans la même direction que le projectile
po.sition de tous les noeuds lorsque 0 ^ .r « 0,40 m b) (.luelle est la (perpendiculairement à la direction de l'im pulsion iransversalc).
période du mouvement oscillant de n’importe quel point, à l’exception I x rc.stc de l'énergie du projectile sert à ce niouvcmenl. L’énergie
des nœuds, sur la corde ? Quels .sont c) le module de la vitesse et résiduelle qui n’esl pas utilisée pour produire des déformations
d) rampliiude des deux ondes progressives dont l’intertérence produit permanentes d;uis les fibres devient de la chaleur.
cette onde ? e) 1orsque 0 £ t fi 0,050 s. à quels instants tous les La figure 2.36 b) est un graphe du module de la vitesse v en
points de la corde aurout-ils une vites.se transversale nulle? fonction du temps t dans le cas d’une balle de masse m = 10,2 g
49P. Démontrez que l’énergie cinétique maximale contenue entre tirée à bout ptirtani dans la veste à l’aide d’un revoher Spécial .38.
deux nœuds adjacents d ’une onde stationnaire produite par deux I ci , l’i = 2 (XX) m/s, cl supptiscz que le creux «inique a un demi-angle
ondes progressives de même amplitude est ((de 60° À la suite de l’impacl. quel est le rayon de a) la zone étirée
et b) du creux (si on suppose que la personne qui porte la veste
50P. Soit une onde stationnaire est immobile) ?
produite dan.s une longue corde;
elle pos.sede un ventre à r = Ü,
et un nœud à л = 0,10 m. Il n 'y a I (s)
aucun ventre ou nœud entre v = 0
et V—0.10 m. Le déplacement v4t)
de l'élém ent de corde situé à
A = 0 est illustré à la figure 2.34.
À f = 0.50 s. quels sont le.s
déplacements des éléments de Fifluie 2.34 Pioblème 50
coidc .situé à a) .X “ 0.20 ni et b) t — 0,30 m ? Quelle est la com­
posante Vde la vites.se transversale de l’élément de corde situé à
.V—0,2(1 m .à l’instant c) t 0.50 s et d) » ~ 1.0 s ? e) Faites le schéma
de l’onde .stationnaire à f “ 0..50 ,s entre les limilr.s x 0 e t.« —0.40 m.
51P. Dan.s la figure 2..35, un fil d’aluminium, de longucui L; -6 0 ,0 cm
cl ayant une st’ciioii tran.svcr.salc de 1.П0 x 10 ^ cm^ et une masse
volimiiqtie de ? 60 g/cm-*, est joint à un lïl d ’acier d’une masse
volumique de 7,80 g/cni’ qui a la mctnc .section iransver.siilc. Le fil

b)

figure 2.36 Problème 52


3 Les ondes
sonores

Cette (houve-souris peut non seulement détecter un papillon nocturne dons l'obscurité totale, mais elle peut
aussi déterminer sa vitesse
relative ovont de foncer
sur lui.
и Chapitre 3 Les onde^ s^neie^

3.1 Les ondes sonores


C o m m e o n l ’a v u d a n s le c h a p itre 2 . les o n d e s m é c a n iq u e s o n t b e s o in d ’u n su p p tirl
m a té rie l, d 'u n m ilie u , p o u r ex iste r. Il y a d e u x ty p e s d ’o n d e s m é c a n iq u e s : les ondes
transversales, d o n t les o sc illa tio n s d u m ilieu so n t p e rp e n d ic u la ire s à la d ire c tio n d e la
p ro p a g a tio n d e l’o n d e, et les ondes longitudinales, d o n t les o sc illa tio n s d u m ilie u so n t
p a ra llè le s à la d ire c tio n d e la p ro p a g a tio n de T o n d e.
L e s o n d e s s o n o r e s so n t d e s o n d e s m é c a n iq u e s lo n g itu d in a le s . L es é q u ip e s d e
p ro sp e c tio n u tilise n t ce.s o n d e s p o u r so n d e r la c ro û te te rre stre à la rec h e rc h e d e p étro le.
L es nav ires utilisent d es sy stèm es d e rep érag e p ar le .son (sonai ) |x x ir d étecter des o b stacles
so u s-m arin s, l i s é q u ip a g e s d e s so u s-m arin s u tilisen t d es o n d e s so n o re s |ю иг traqtiei les
a u tre s b â tim e n ts, p iin c ip a lc n ie n i en é c o u la n t les b ru its c a ra c té ri.stiq iies p ro d u its p a r
leurs sy.stcnics d e p ro p u lsio n . L a fig u re 3.1 p résen te u n e é c h o g ra p h ie , o ù l'o n v o it une
im ag e nu m érisée d e la tête et d u bras d ’un foetus, qui am .stitu c un e x em p le d e l'u tilisa tio n
d e s o n d e s so n o res d a n s T étu d e d e s tissu s d é lic a ts d u c o rp s h u m ain D an s c e ch a p itre , o n
m e ttra l'a rc e n t su r les o n d e s so n o res au d ib le s q ui se jiro p ag en l d a n s l’air,
L a fig u re 3 .2 illu stre d iffé re n ts c o n c e p ts q iT o n u tilise ra ici. l e p o in t 5 re p ré se n te
Rçute 3.1 U ne 6dK»j;riipliit' d'iin firtiis une très p e tite so u rc e so n o re , q u e T o n n o m m e source p o n ctu elle, q u i é m e t d e s o n d e s
(]ui suce son ponce •, l’imiige est produite
pur Jvfl ullrusons fdoiit la fréquente so n o res d an s to u te s les d ire c tio n s. fro n ts d 'o n d e et les niyn n s in d iq u e n t la d ire c tio n
Vf situe aii-ilelà de votre limite de la p ro p ag atio n et d e la diffu sio n d es o n d es sonores. I x s l'ro n ls d ’<H0ide sont d e s sut faces
d ’audihilito). o u les o sc illa tio n s d e T air p ro d u ite s p ar T o n d e so n o re sont id en tiq u e s (m em e a m p litu d e
et m êm e p h a s e ); d a n s u n d e ssin b id im e n sio n n e l, c e s su rfa c e s so n t rcpré.senté& s p a r des
cerc le s c o m p le ts o u p artie ls ay an t p o u r c e n tre la so u rce p o n ctu elle. Lc.s ra y o n s so n t des
lig n e s p e rp e n d ic u la ire s au x fro n ts d 'o n d e q ui in d iq u e n t la d ire c tio n d e p ro p a g a tio n de
CCS fro n ts d 'o n d e . L es p etites llè c h c s d o u b le s su p e rp o sé e s su r les ray o n s d e la fig u re 3.2
in d iq u e n t q u e les o s c illa tio n s lo n g itu d in a le s d e T a ir .sont p a ra llè le s au x ra y o n s, d o n c
p arallèles à la d irectio n d e p ro p a g a tio n d e T onde.
C o m m e on le v o it à la fig u re 3 .2 . les fio n ts d 'o n d e so n t sp h é riq u e s d an s une régitin
ra p p ro c h é e d 'u n e .source p o n c tu e lle c l se d é p lo ie n t e n tro is d im e n sio n s ; o n dit q u e les
o n d es so n t sp h éru p u s à de tels e n d ro its relativ em en t p rès de la so u rce. A m esu re q u e les
fro n ts d 'o n d e s ’é lo ig n e n t d e la s o u rc e e t q u e le u r ra y o n a u g m e n te , le u r c o u rb u re
d im in u e . À un p o in t é lo ig n é d e la so u rc e , le s fro n ts d ’o n d e so n t a p p ro x im a tiv e m e n t
plats (o u b id im e n sio n n e ls) ; o n dit alo rs q u e les cm des so n t planes.

3.2 La vitesse du son


L a v ite sse d e to u te o n d e m ccanicjue, q u ’e lle .soit tra n sv e rsa le o u lo n g itu d in ale, d é p e n d à
la fo is d ’u n e p n ip rié ié d 'in c riic (e m m a g a sin e r d e l ’én e rg ie c in étiq u e) e t d ’u n e p ro p rié té
d 'é la s tic ité (e m m a g a sin e r d e T én e rg ic p o te n tie lle ) d u m ilieu . O n p eu t d o n c g é n é ra lise r
l'é q u a tio n 2 .2 5 , qui d o n n e k m o d u le d e la v ite sse d ’u n e o n d e tra n sv e rsa le le lo n g d 'u n e
c o rd e ten d u e, en é criv an t
_ fr _ j p ro p riété d ’élasticité
(3 .1 )
V d y p ro p riété d 'in e rtie

où (p o u r les o n d es tran sv ersales) r est la tension d an s la c o rd e et // est sa m asse lineitjue.


D a n s le c a s d ’ u n e o n d e lo n g itu d in a le sc p ro p a g e a n t d a n s T a ir, o n p eiil s ’a tte n d re à
e e q u e la p ro p rié té d ’in c rtic c o rre s p o n d a n t à p s o it la m a sse v o lu m iq u e p d e l’a ir
M ais q u ’en c st-il d e la p ro p rié té d ’é la stic ité '
D an s u n e c o rd c ten d u e, u n e én e rg ie p o ten tielle est a.ssociéc à T éiirem en i pt-rindique
d e s c lé m e n ts d e c o rd e p a rc o u ru s p a r u n e o n tlc. L o rs q u 'u n e o n d e so n o re sc p ro p a g e
Figme 3 ? Une onde sonore se propage d a n s P a ir, u n e é n e rg ie p o te n tie lle e st a s s o c ié e a u x c o m p ic s s io n s el aux ra ré fa c tio n s
a paifir d 'une source ponctuelle S
dans un inilirii nidiinensionnel. p ério d iq u e s d e s (Hilils é lé m e n ts d u v o lu m e d 'a ir. I p ro p rié té qui d é te rm in e la v ariatio n
l es t'ronls d Onfic form ent de.c sphères d e v o lu m e d ’u n é lé m e n t d u m ilieu lo rsq u e la p ressio n p l p — ¡'IA, m iK liile d e la fo rc e
centrées sur S ; les rayons ont a p p liq u é e p ai u n ité d e s u rfa c e ) y e s t m o d ifié e e st le m m iu le d e c o m p re s s ib ilité B.
une direction radiale par l'apport à S. C elu i-ci a é té d éfin i à la se c tio n 13.6 d u v o lu m e I p a r (v o ir T éq u aiio n 1 3 .2 7 );
I .es petites flèdtes doubles indiquent
que les élém ents du milieu oscillent Ap
(la définition du m odule de compies.sibiiiié).
parallèlem ent à la direction des rayons. «= - (3.2)
3.2 La vitesse du son 55

IA6LlA'd 3 ! Le module de la vitesse du son* où AV/V e st la v aria tio n relativ e d u v o lu m e p ro d u ite p a r u n e flu ctu aiitin d e p ressio n Ap.
Milieu Vitesse (m/s) L ’u n ité S I d e la p re ssio n e s t le n ew to n p a r m ètre c a rré (N /m ^), q u e l ’o n n o m m e pascal
Gaz (P a ). D ’a p rè s l’é q u a tio n .3.2, o n v o it q u e I’u n itc d e B e s t é g a le m e n t le p a sc a l.
Air (0 '■C) 3.31
L e s sig n e s d e Ap e t d e AV so n t to u jo u rs o p p ttsés : q u a n d o n a u g m e n te la p ressio n su r un
élém en t (Ap est positive), son volum e d im in u e (AV^est n égative). O n m et un signe n é g a n t
Air (20 X ) 343 d a n s l’é q u a tio n 3 .2 d e .sorte q u e B e s t to u jo u rs u n e q u a n tité p o s itiv e . E n s u b s titu a n t
Hélium %5 e n su ite B à T e t P à /Li d an s l’é q u a tio n 3 .1. on a
Hydrogène 1 284
Uquides [b
Eau (0 X )
V = «/ — (le inodulc de la vitesse du son), (3 .3 )
1 402
V fi
Eau (20 X ) 1 482
Eau de mer** 1 522 q u i e st le m o d u le d e la v itesse d u son d an s un m ilieu d o n t le m o d u le d e co m p re ssib ilité
Solides e s t B e t la m as.se v o lu m iq u e e s t p . C ’e s t e ffe c tiv e m e n t l ’é q u a tio n o b te n u e p a r u n e
p re u v e p lu s fo rm e lle , p ré s e n té e à la fin d e c e tte s e c tio n . L e ta b le a u 3.1 in d iq u e le
Aluminium 6 420
m o d u le d e la v itesse d u so n d a n s d iffé re n ts m ilieu x .
Acier 5 941 L a m asse v o lu m iq u e de l’ea u e st p resq u e 1 0 0 0 fois p lu s g ran d e q u e celle d e l ’air.
Granité 6 000 .Si c ’était le seu l facteu r im p o rtan t, o n d e v ra it s ’a tten d re, d ’a p rè s l’é q u atio n 3 .3 , à c e q u e
la vitesse d u son d an s l’eau soit con sid érab lem en t réduite p a r rapport à sa vitesse d an s l’air.
* À 0 C cl à une pression de I altii, sauf
indication conttairt. P o u rta n t, le tab leau 3 .1 m o n tre le co n traire. O n e n d éd u it (en c o re d ’ap rè s l'é q u a tio n 3.3 )
** À 20 °C et avec une salinité de 3.5 %.
q u e le m o d u le d e c o m p re s s ib ilité d e l’e a u d o it ê tre a u m o in s I 0 0 0 fo is p lu s g ra n d
q u e ce lu i d e l ’air. C ’e st e ffe c tiv e m e n t le cas. L ’e a u e st b e a u c o u p p lu s in c o m p re ssib le
q u e l ’air, c e q u i e s t u n e a u tre fa ç o n d e d ire (v o ir l’é q u a tio n 3 .2 ) q u e so n m o d u le d e
c o m p re s s ib ilité e s t b e a u c o u p p lu s g ra n d . E n e ffe t, p o u r u n m ê m e v o lu m e V d ’a ir
e t d ’ea u so u m is à la m em e v ariatio n d e p ressio n Ap, la v ariatio n d e v o lu m e AV d e l ’eau
e st b e a u c o u p p lu s faib le q u e c e lle d e l’air.
Preuve formelle de l’équation 3.3
O n p m u v e ra m ain ten an t l’éq u atio n 3.3 en u tilisan t les lois d e N ew ton. S o it u n e im p u lsio n
o ù l’a ir e s t c o m p rim é , et q u i se p ro p a g e (de la droite vers la Kauche) d a n s l’a ir à la
v itesse —V d an s un lo n g tu b e, c o m m e ce lu i d e la fig u re 2 .2 . O n su it l’im p u lsio n à c e tte
v ite sse , d e so rte q u e l ’im p u lsio n sem b le im m o b ile d a n s c e réfé re n tie l. L a fig u re 3.3 a)
rep résen te la siiiialion telle q u ’elle ap p araît dans c e réfcrenticl. L ’im p u lsio n e st im m o b ile,
e t l’air pa.s,sc à tra v e rs e lle à la v itesse 4 v, de la f^aïu he vers la droite.
O n po.se q u e la p ressio n d e l'a ir n o n p ertu rb e e st p, e t q u e la p ressio n d an s l’im p u l
sio n est p + Ap, o ù A p est p o sitiv e à cau.se d e la co m p re ssio n . O n éta b lit, à la fig u re 3.3,
un ax e d e s x p o sitifs o rie n te v ers la d ro ite. C o n sid é re z un é lé m e n t d ’a ir d ’épaL sseur A r
e t d e s u rfa c e A se d é p la ç a n t v ers l’im p u lsio n à la v ite sse v. L o rsq u e c e t é lé m e n t d ’a ir
tra v e rse l’im p u lsio n , sa fa c e a v a n t re n c o n tre u n e rég io n o ù la p ressio n e st p lu s g ran d e,
c e q u i le r a le n tit, la c o m p o sa n te x d e sa v ite sse d e v ie n t r;, i A v,, o ù A v, e s t n é g a tiv e .
C e rale n tisse m e n t est c o m p let lo rsq u e la fa c e a rriè re a ttein t l ’im p u lsio n , ce q u i d em an d e
un in ic rv a llc d e tem p s
Ax
A/ = — . (3.4)

Air e n m ouvem ent


(élém ent d'air)
f) V&/>. r , *- Av,
Figute33 Line impulsion de compression
est transini.se tl;uis un long tube rempli
d’ail. Dans le rclcrentiel de cette
illustration, l’impulsion est au reptis / I.
,,.7 HAS’- Im pulsion
et l’air SC déplace de la gauche vers
la droite, a) Un élément d’air d'épaisseur a)
Ax se déplace vers l’impulsion
à la vitesse v. b) La face avant
de l'élcmcnt d’air atteint l’impulsion.
Les forces (accompagnées de leur
tnodidc) agissant sur les faces avant
et arrière (sous l’elfet (le la pression Aï b '
de l’air) sont indiquées. b)
56 Chapitre 3 te s ondes sonores

ü n p e u t a p p liq u e r la d eu x iè m e loi d e N e w to n = m a ) à l'é lé m e n t d 'a ir. D u ran


rin te rv a lle d e te m p s M u n e fo rce m o y e n n e , d o n t la c o m p o sa n te x e st pA, s 'e x e rc e v e r
la d ro ite su r la fa c e a rriè re d e l'é lé m e n t, et u n e fo tcc m o y en n e, a y a n t u n e c o m p o sa n te j
d e —(j> -t- s 'e x e rc e v e rs la g a u c h e su r sa face av a n t (fig u re 3 .3 b). L a co m p o san te
X d e la fo rce m o y e n n e résu lta n te su r l'é lé m e n t d u ra n t l'in te rv a lle A / e st d o n c

in » .. = p A - { p \ l p ) A
— —A/>A (la eompt>sante.vüc la force rcsuluuitc). (3 .5 ;
L e sig n e n é g a tif in d iq u e q u e la fo rce ré su lta n te su r l ’é lé m e n t d ’a ir e s t o n e n te e v ers la
g a u ch e d a n s la lig u re 3.3 b). L e v o lu m e d e l’é lé m e n t e s t A (A .r). c l, à l'a id e d e l'e q u a iio n
3 .4 , o n p eu t é crire sa m asse so u s la fo rm e
A/n = pA {A \) = pA \\(A i) (la masse). (3.6)

L a c o m p o sa n te x d e l'a c c é lé ra tio n m o y e n n e d e l'é lé m e n t d 'a ir d u ra n t A r e st

(la composante .r de l'accclération). (3 .7 )


ÀT

S e lo n la d e u x iè m e lo i d e N e w to n e x p rim é e e n fo n c tio n d e s c o m p o s a n te s
X (f|rt , = m a, ), o n a. à l'a id e d es é q u atio n 3 .5 , 3 .6 et 3 .7 ,

- A p lA ) = p A r , ( A / ) |^ 'j ,

q u ’o n p eu t é c rire so u s la fo rm e

pv^ = - (3.S)
A f,/li
L 'a ir q u i o e c n |te u n v o lu m e V = /li',( A t) à re x ic rie u i de rü n p u ls io n e st c o m p n m é d 'u n e
q u a n tité AV = A (A v,)(A /) lo rs q u 'il p én è tre l'im p u ls io n . D o n c.
A (A v.)(A O ^ (3 .9 )
V A ti(A /)

E n in sén u il l'é q u a tio n 3 .9 p u is l'é q u a tio n 3.2 d an s l'é q u a tio n 3 .8 . o n o b tie n t


Ap Ap
PVx~ - = B.
A \\/\\ AV/V

E n iso lan t V,. qui e st égal h e, o n o b tien t l'é q u a tio n 3.3, q u i d o n n e le im x lu le de la v itesse
d e l’é lé m e n t d ’a ir v e rs la d ro ite d a n s la fig u re 3 .3 , d o n c le m o d u le d e la v ite s s e d e
l'im p u lsio n v ers la g a u c h e .

Exemple 3.1
Afin de déicrminer I'originc d'une source sonore, voire cerveau
analyse le ilélai At entre l’arrivée du son à l’oreille la plus rapprochée
de la source et l'arrivcc du niciiie son à l'aune oreiUe. Supposez que
la distance vous séparant de la source est telle qu'un de scs fronts
d'onde est approximativement plan lorsqu'il vous atteint, cl po.sez
que P C.SI la distance entre vo.s oreilles
a) Trouvez l'expression du délai At exprimée en l'onciion de D et
de la ilireeiioii (tde la source par rapport à on axe dirigé vers l’avant
de la personne. Figure 3.4 Exemple 3 .1 lin front d'onde parcourt une distance
rf(= f> sin W) supplémentaire pour atteindre roreillc gauche
SOIUTION: La situation e,si rcpré.scméc (vue en plongée) à la figure 3.4. aptes avoir atteint l'oreille droite.
où k s fronts d'onde s'approchent de vous depuis la source située
tievani vous et à votre droite Le tootepJ tl». dons ce cas, est celui scion où !• est le motlule de la viicsiic du .son dans l'air. En se basant sur
lequel le délai Al est causé par la distance il que chaque front d’onde toute une vie d'expériences, votre cerveau fait la corrélation entre
doit parcourir p<uir atteindre votre oreille gmiclie après avoir atteint iiHiie valeur de At (de zéro à fa valeur maximale) et une valeur de fi
votre oreille droilc. D'après lu figure 3 4, on détermine que (de zéro à 90") pour déierniincr la direction de la source stmorc.
P .sin B h) Suppose/ que vous éte.s dans l'eau 6 20 °C lorsqu'un front d'onde
= ^ = (réponse) (3 10)
r provenant dircctcmenl de la droite vous atteint. En t ous basant
3.3 Les ondes sonores progressives 57

sur r indication du temps de délai, quelle direction 0 la source semble donc être à un angle ô plus petit que 90". Pour trouver cet
semble-t-elle tonner par rapport à un axe dirige en avant de vous ? angle apparent, on substitue le délai D/v^ de l’équation 3.11 à Ai
de l’équation 3.10, et on obtient
SOLUTION Dans ce cas, le contepî dé à appliquer est celui selon lequel
la vitesse du son dan.s l’air v est maintenant la vitesse du son dans D fl sin ^
l’eau q.. On substitue donc à v et 90'" à 6 dans l’équation 3.10, (3.12)
Ve V
ce qui donne
Ensuite, afin de trouver S, on insère v = 343 m/s et v, = 1 482 m/s
D sin 90 D
Af. = (3.11) (d’après le tableau 3.1) dans l'équation 3.12, et on determine que
l’e Vc"
Puisque la valeur de v,, est environ quatre fois supérieure à celle de v, v 343 m /s
sin 0 = — _ — -— h - - 0.231,
la valeur du délai A/^ est environ le quart de celle du délai maximal Vc 1482 m /s
dans l'air. En se ba.sant sur son vécu, votre cerveau analysera le délai
dans l'eau comme s’il se produisait dans l’air. La source sonore donc. fi - 13,4°. (réponse)

3.3 Les ondes sonores progressives


Or. peut m aintenant ex am in er les déplacem ents et les v ariatio n s de pression associés
à une onde sonore sinusoïdale se propageant dans l'air. La figure 3.5 a) représente une
onde se déplaçant vers la droite dans un long tube rem pli d ’air. R appelez-vous, com m e
on l ’a vu dans le chapitre 2 , q u ’on peut produire une onde sem blable à l’aide d ’un piston
e ffe c tu a n t un m o u v em en t h a rm o n iq u e sim p le, situ é à l ’e x tré m ité g au ch e du tube
(v o ir la fig u re 2.2 ). Le d é p lac em e n t du p isto n v ers la d ro ite d é p la c e et co m p rim e
l ’élém ent de volum e d 'a ir devant lui, produisant une région près du piston où la pression
est lé g èrem e n t p lu s é le v é e ; o n ap p e lle cette rég io n une z o n e d e c o m p re s s io n .
Le déplacem ent du piston vers la gauche perm et à cet clém ent d ’air de revenir vers la
gauche et entraîne une dim inution de pression, créant ainsi une région où la pression est
lég èrem en t p lu s faible qu e l’on nom m e z o n e d e r a r é f a c tio n . C h aq u e élém e n t d ’air
pousse .successivement sur l’élcm cnt suivant, déplaçant ainsi les zones de com pression
et de raréfaction le long du tube. Le m ouvem ent d 'o scillatio n (droite-gauche) de l ’air et
les variations de pression se déplacent donc dans le tube, produisant une onde sonore.
Considérez un mince élém ent de volum e d ' ^ d 'u n e épaisseur à x situé à la jxxsition x
dans le tube. L orsque l’o nde le traverse, l’élém ent d 'a ir oscille de d ro ite a gauche cn
un m o u v em en t h arm o n iq u e sim p le au to u r d e sa p o sitio n d ’éq u ilib re (fig u re 3.5 b).
L es oscillations de chaque élém en t d ’a ir causées p ar le passage d e l ’onde sonore sont
donc co m parab les à celles d ’un élém en t de co rd e cau sées p a r une o nde transversale,
sauf que l’élém ent d ’air oscille longitudinalement et non transversalement. P uisque les
élém ents d e cordc oscillent parallèlem ent à Taxe des v, on écrit leurs déplacem ents sous
la form e y(x, t). De la m em e façon, puisque les clém ents d 'a ir oscillent parallèlem ent à
l ’axe des x, on po u rrait éc rire leurs d ép lacem en ts sous la form e x(x. t). Afin d 'é v ite r
toute confusion dans la notation, puisqu’ il y a deux vanabies x dont la signification diffère,
ligure 3.5 a) Une onde sonore qui on utilisera plutôt la fo rm e s{x, t). L a variable v représente donc le déplacem ent d 'iin
SC prttpagc à la vitesse v dans un long élém ent de volum e d ’air par rapport à sa position d ’équilibre à l'instanl t.
tube rempli d’air forme un motif
périodique de zones de compression Z o i» '< U 'conip iexsioii I ].
A '
et de zone.s de raréfaction de l'air. ______\
L'onde est représentée à un instant
arbitraire, b) Une vue agrandie
horizontalement d'une petite .section
du tube. 1ors du passage de fonde,
un clément de volume d’îiii d épaissciu
A« ost illc de droite à gauche en un
mouvement harmonique simple
autour de sa position d'équilibre.
.À l'instant illustré en bX l’élément
se trouve déplacé d’une distance s
à la droite de sa position d'équilibre.
La grandeur maximale de son
déplacement, vers la droite ou
vers la gauche, est
58 Chapitre 3 bes ondes sonores

^ r)r|(l,i<« iiti tii O n peut u tiliser une fonction sinus ou co sin u s pour ex p riiiicr m athém atiquem ent
î (a',
/), puisqu’elles peuvent décrire des déplacem ents qui v arient de façon sinusoïdale
a) Mx.f) - i;„ cacifct r>lj
en fonction de r e t dc t. Dans ce chapitre, on utilisera une fonction cosinus, en écrivant
V/ — -V

/ r»:i-ill:>in
v( a , f) = C0S(/CÏ - ù J t). (3.13)
ll) Aj&,„sin(/u ill)

^ \ .i:i.uiijii ill iiicvii111 La figure 3.6 a) représente les principales parties de cette équation, où .v„, est l’ain p litu d c
ngure3.6 ii) Lit ibnulioii ik'plact^nieni dc d é p la c e m e n t - c ’cst-îi-dire la grandeur m axim ale du déplacem eni de rélé m en l d 'a ir
ct la tonction dc variation d 'u n c ô té ou dc r a tu r e dc sa p o sitio n d ’é q u ilib rc (v o ir la fig u re 3..5 b). Le nom bre
lie pic-ssinii il'unc Uiidf sonorc d ’o n d e k, la fré q u en c e an g u laire a>, la fréq u en ce f, la lo n g u eu r d ’o n d e À, le m odule
l omportem unc aniphtiide dc la vitesse v et la période / ’d ’une onde sonorc (longituiiinale) sont d efinis et reliés
cl un terme DMcilliint. exactem ent dc la m êm e façon que dans le cas d ’une onde transversale, sau f que À est ici
la distan ce (p arallèle à la d irection de p ro p ag atio n de l’o nde) à laq u elle le m otil des
zones de com pression et dc raréfaclion causé par l ’onde com m ence à .se répéter (voir la
figure 3.5 a). (On suppose que est beaucoup p las petite que A.)
Q uand l'o n d e se déplace, la pression dc l’air à toute position x dans la figure 3.5 a)
v arie dc façon sin u so ïd ale , co m m e on le d é m o n trera à la fin d c ce tte se ctio n . P our
décrire cette variation, on écrit

Ap(x. t) — Д/>„, sin(Lx - cot). (3.14)

La figure 3.6 b) ic|iréscntc les principales parties de cette équation. D ans l’équation 3.14.
une valeur négative dc Ap correspond à unc raréfaction d c l'air, et unc valeur po.sitive
correspond à une com pression. Ici, Ap,„ est l'a m p litu d e d e p ressio n , qui est la grandeur
de l’augm entation ou d e la d im in u tio n m axim ale d c pression causée par l ’onde ; Ap,„
(de l’ordre dc I Pa) est norm alem ent beaucoup plus petite qu e la pression d ’équilibre p,
observée lorsqu’il n 'y a pas d ’o nde (la pression atm o sp h ériq u e est d ’environ 10^ Pa).
C o m m e on le d ém o n trera, l ’am p litu d e d e p ressio n A/j „, est re lié e à l'a m p lttu d e dc
déplacem ent .îm de l’équation 3.13 par

^Pm = (»/0W).Vn (.3.1.5)

La figure 3.7 représente graphiquem ent les équations 3.13 et 3.14 à f = 0 ; avec le
tem ps, les deux courbes se déplacent vers la droite le long des axes horizontaux. Notez
que le déplacem ent e t la variation de pression sont déphasés dc n t l rad (ou 9 ü' ). A insi,
la variation de pression A/t en tout point le long de l'o n d e est nulle lorsque la grandeur
du déplacem ent est m axim ale.

✓ vérifiez VOS CONNAISSANCES!; Lorsque rélémenl de volume d’air osiillant de la figure 3.5 b)
a) se déplace vers la droite à partir du point dc déplacement /.cro, la pression dans l’élément
I est elle à sa valeur d’étiuilibre, commence-t-elle à augmenter ou commence t-elle à diminuer?

La preuve des équations 3.14 et 3 15


La figure 3.5 b) m ontre un élém ent de voltinie d ’air o.scillant de section tninsversalc A
et d ’une épais.scur Av, et dont le centre est déplacé d une distance .vpar rapport à .sa position
d ’équilibre.
D 'a p rè s l'é q u a tio n 3.2 . on p eu t é c rire la v aria tio n d e p ressio n d an s l ’élém e n t
déplacé sous la form e

AV'
Ftgure 37 a) Le graphique de la fonciion A p = —B — . (3.16)
' V
déplacement à t = 0 (équation .3.1.3).
b) I e graphique équivalent dc la lonction La quantité V'dans l’équation 3.16 est le volum e de l’élém ent, donné par
de varia(i(>n de pression (equation 3.14).
Les deux graphii|ues illustrent unc onde V '- A A v , ' (3.17)
sonore dc I 000 Н/ dont l’ampliuide
de pre.s.ston correspond au seuil L a quantité AV de l’équation 3.16 csi la variation de volum e qui se produit lorsque Télé
de la douleur (voir l'exemple 3.2). m ent e st déplacé. C ette variation se ptivliiii parce que les déplacem ents des deux faces
3.4 L'interférence des ondes sonores 59

de l’élém ent d ’air ne sont pas tout à fait les m êm es, et difièrenl d ’iinc quantité A,v. On
peut donc écrire sous cette form e la variation d e volum e :

AV = A l s . (3.18J

En insériuit les équations 3.17 et 3.18 dans l’cquation 3.16, on obtient, à la lim ite
lorsque l’épaisseur de l’élém ent X i tend vers zéro :
A.s ils
a 19)
^ = - * s ; = - V
Le sym bole 9 indique que la dérivée dttns l’équation 3.19 est une dérivée partielle, qui
m ontre com m ent s varie avec x lorsque le tem ps t est constant. D ’après l’équation 3.13,
on a alors, en considérant t с о т п к : une constante,

^ co s(/u — = — fon, %in(kx — wt).


dx Sx
En substituant cette quantité à la dérivée partielle de l’équation 3.19. on obtient

A/> - Bkx„ sin(Aa' — tu/).


Si on p o se que = Bks„„ ce la no u s d o n n e l ’éq u a tio n 3 .1 4 , q u e l ’o n c h e rc h a it à
dém ontrer.
En utilisant l ’équation 3.3, on peut m aintenant écrire:

= (Bk)s^ = b^pk)s,y,.

Si on rem p lace к p ar tu/v (selo n l ’é q u a tio n 2 .1 2 ), d an s ce tte d e rn iè re éq u a tio n ,


on obtient l'éq u atio n 3.15 que l’on voulait démontrer.

Exemple 3.2
L’amplitude de pression maximale A/>,„ que l'oreille humaine peut En insérant les données connues, on a
tolérer dans les «ides .sonores est d’environ 28 Pa (ce qui est beaucoup
28 Pa
moins que lapre.ssion normale de l’air, qui est d’environ lO’ Pa). Quelle
(343 m/s)(1.21 к е /т Ъ (2 я )(1 000 Hz)
est l’amplitude de déplacement .v„ d’un tel stwi émis dans l’air dont la
mas.se volumique e.st p = 1.21 kg/m \ à une fréquence de I (XK) Hz. = 1.1 X 1(1 m = 11p m . (réponse)
et à une vitesse ayant un nnxlule de 343 m/s ? Cela représente environ le septième de l’épaisseur de celte pape.
SOU/TIOM: On utilise le concept de suivant : l’ampliiude de déplacement s„ Donc, l'ampliiudc de déplaeenieni du son, incluant le son le jilu.s fort
d ’une onde sonore est reliée à l’amplitude de pression A/>,„ de l’onde, que l’oreille puisse tolérer, est très petite.
d’après l'équation 3.1.5. En résolvant cette équation pour trouver . L.’amplitude de prcs.sion Ap„, du plu.sfaible .son detectable й
on obtient I 000 Hz est 2.8 x 10" ' Pa. En pmeédani comme on vient de le faire,
on obtient .v„ 1.1 X 10 " m. ou 11 pm. cc qui cquivaul à environ
le dixième du rayon d’un atome moyen. On peut en déduire que l oreiHe
ipo) \р{2жП ' est effectivement un déicciciir très sensible aux ondes sonores.

З Л L'interférence des ondes sonores


À l’instar des ondes transversalc.s, les ondes sonores peuvent produire des interférences.
O n peut considérer, par exem ple, l’intci l'érencc entre deux ondc.s sonores identiques se
propageant dans la m êm e direction. La figure 3.8 illustre de quelle façon on peut créer
une telle situation : deux sources ponctuelles. S, et ém ettent des ondes sonores qui
sont en phase et qui ont la темпе loiigueui d ’onde À. O n dit que les .stiurccs ponctuelles
.sont elles-m êm es en phase, c ’est-à-dire que les zones de com pression et de raréfaction
(que l’un peut représenter par des fronts d ’onde) sont émises par ees sources siniulianériiciii.
Figure 3.8 Deux sources ponctuelles. O n s ’intéresse aux ondes qui ,se propagent en pa.ssani par le point P dans la figure 3.8.
Si et S,, énicltenl des ondes sonores O n suppose que la d istance entre les sources ponctuelles et le point P est plus gronde
sphériques en phase. Les rayons. que la distance entre les deux sources, cc qui perm et de dire que les ondes se propagent
/.) et L;, indiquent les parcours approxim ativem ent dans la m êm e direction au (xiinl Г.
qu’effectuent les ondes pas.sant Si les ondes suivaient des parcours de longiieuis identiques |xnii allcindrc le iwiiil P,
par le point P. e lle s se ra ien t en p h ase p a rv e n u e s à ce p o in t. Et co m m e c 'e s t le cas p o u r les o n d es
60 Chapitre 3 Les ondes sonores

transversales, cela signifierait q u ’elles produiraient alors un e inlerl'ércncc e o n stru a iv c .


lo u ie lo is, dans la figure 3.8, le parcours Li effectué par l’onde ém ise par Si est plus long
que le parcours L, effccluc p ar l’onde provenant de S ,. C ette difl'crcncc dans la hm gucur
des parcours signifie que les ondes ne sont peut-être pas en phase au pirini P. En d ’autres
mots, leur dificrcncc de phase, à , h P dépend de la d ifféren ce d e p a rc o u rs L 2 — L,.
Pour relier la différence de pha.se ^ à la différence de parctiurs A I . on se rappelle
q u ’u n e d ilfc rc n c c de p h ase d e 27t rad c o rresp o n d à un e lo n g u e u r d ’o n d e, co m m e
on l'a vu à la .section 2.9. Si </> = 2Tr rad, cela im plique q u e AC = Â ; on peut donc ccria-
le rapport suivant (règle de trois) :

AL
JÈ. (3.20)
2n
d ’où

AC
0 = - - 2.T. (3.21)

U ne in te rfe re n ce c o n stru c tiv e ,se p ro d u it lo rsq u e </> = ü, 2n, o u tout m u ltip le en tier
positif ou négatif de 2Tr. C ette condition peut s’écrire sous la forme

<f> ~ ni(2Tt), o ù m — 0. —l, A 2, ... (rinterfcrcncec<Histnjcdvc). (3.22)

Si on rem place la valeur de <j) dans l’équation 3.22 par celle de l’équation 3.21, on obtient
A L /k = m ; la condition pour l'o b ten tio n d ’une interférence constructive est donc :

AI. = m k, o ù /« = 0. ± 1, ± 2 , .. . (l’interférence consüuctive) (3.23)

Si la v aleu r de m est n ég ativ e, c e la in d iq u e q u e Cj est p lu s grand que C i , ce q u i est


plausible. Le point P de la figure 3.8 serait alors situé sous la droite horizontale (en pointillé
dans la figure), équidistante par rapport aux sources.
P ar ex em p le, si la d iffé re n c e de p a rc o u rs A l = C, - L^ est ég a le à 2A, alors
A C — 2 k - rnk. et les o n d es p ro d u isen t un e in te rfé re n c e c o n s tru c tiv e au po in t P.
L ’in te rfé re n ce est co n stru ctiv e p arce que l’o nde de So au p o in t P e st déphasée de 2 k
par ( apport à l’onde de S , , de sorte que les deux ondes sont en phase à P.
fin e in te rfé re n ce d estru c tiv e se p ro d u it lorsque é est un m u ltip le im p air p o sitif
ou négatif de tt, ce qui peut s ’écrire ainsi ;

= (2m -t- 1);r, où m = 0. ± 1 , —2 ,... (l’intctfcrence desnuctivc). (.3.24)

vSi on rem place la valeur de çfc dans l’équation 3.24 par celle de l’équation 3.21, on obtient
A U k = (>« + :( ), et la condition pour l'obtention d 'u n e interférence destructive est donc ;

A/. = (m t- L)k, où w — 0, ± 1, ± 2 . ... (l’interférence destructive). (3.2.5)

Par exemple, si la différence de parcours AL = L~_ — C, de la figure 3.8 est égale à lA k ,


alors A L — 1.5 k = (2 + j )à - (/n )- j)?., et les o n d es p ro d u isen t une in terféren ce
destructive au point P. L ’interférence est d estructive parce que fo n d e de au point P
e.st déphasée de 2„5 longueurs d ’onde par rapport à fo n d e de .5,. de sorte que les deux
ondes sont en opposition de phase à P.
Bien sût. deux ondes peuvent produire une interférence interm édiaire lorsque, jiar
exem ple. A l. ~ \,2 k . C ela .se rapprocherait davantage d une interférence constructive
(AL = l.OX) que (l'une interférence destructive (AL = l,5X),

Exemple 3.3
Dans la figure 3.V ¡1), deux soiircixs iK'nciiielIcs. i', ei Si, sont en 5j et p arS j qui se rendent au point ()uel type d'interférence
phase Cl séparées par une distanee D — 1.“iX ; elles cnicttent des se produit à Pi
ondes sonores idcnricjiies de longueur d'<Hide X.
SOUinOH: On utilise ici le concept dé suivant' puisque les ondes parcou­
a) On considère le fX)int / ’, situé sut la droite passant perptindieulai- rent des distances identiques pour atteindre P ,, L, est égal à Lj
reincnt au tenirc du segment de droite reliant les deux sources. et leur différence de jrarcouts L., — L, est
La (li.stanee de P,, par rapport aux sources, est plu.s grande que I).
(Quelle est la différeniv de parcours entre les ondes émises par AL 0. (répoiLse)
3.5 L'intensité sonore et le niveau sonore 61

-s, SOLUTION: Le concept clé ici est que l’onde émise par S, parcourt une
distance supplémentaire D (= 1,5à ) pour atteindre P,. La différence
de parcours, si on considère que AL ~ L^ — L^, est donc
D/2Í AL = -1,5L. (réponse)
%
•V,
D'après l’équation 3-25, cela signifie que les oiKks sont en opposition
de phase à Pj. et produisent à cet endroit une interférence dcstructiw.

c) La figure 3.9 b) représente un cercle doni le rayon est beaucoup


K plus grand que D, centré par rapport au point mitoyen entre les
a)
sources S, et Sj- Q u d est le nombre de points N sur ce cercle oit
r interférence est constructive 7
SOLUTION: Imaginez qu’à partir du point a, identifié sur la ligure on se
déplace en sens horaire sur le cercle ju.squ’au point d. Appliquez alors
ce concept clé: à mesure qu’on se déplace vers le point d, la différence
de parcours AL devient de plus en plus négative, ce qui fait changer
le ty pe d’interférence. On a vu dans la partie a) que la différence de
parcours est AL OX au point n. On sait aussi, d’apiès la partie b),
b) que AL = —1,5X au point d. Il doit dotic y avoir un point entre a e id
figure 3.9 Exemple 3.3 a) Deux sources ponctuelles. A, et sur le cercle où AL = —1,()À, tel que cela est indiqué clans la
séparées par une distance P émettent en phase des txides sonores figure .3.9 b). D'après l’équation 3.23. une interférence ctmsiructivc
sphériques. Les ondes parcourent des di.slaiKes égales pcHir aneindre se produit à ce point. De plus, il ne peut y avoir d'autre point entre les
le point P,. Le point est situé sur le piolongemcnt de la ligne points a et <f où une interférence constructive se produit, puisqu’il
traversant Sj et Si. b) La différence de parcours (exprimée sous n’y a pas d’autre nombre entier négatif entie 0 et —1,5.
forme de longueurs d’onde) entre les ondes émanant de Sj et de S2 La symétrie, un autre cotKepf clé, peut aussi vous aider à localiser
pour huit points .situés sur un cercle entourant les sources. les autres pcânts d’interférence conslmclive dans le re,sie du cercle.
Par rapport à la ligne cd. la symétrie vous donne le point h où AL = OÀ.
De plus, il y a trois autres points où AL = ± 1,0A. Au total, on a dotic

N = t. (réponse)

t "" yVÉlilFIK vos CONKAISSANCES 2 : d» ,,


si la distance D entre les sources 5, et 5, était égale à 4.0X,
b) Quelle est la différence de parcours, et quel type d'interférence sc quelle serait la différeiKe de parcours et quel type d'interférence
produit au point P> dans la figure 3.9 a) ? aurait-on a) au point P| et b) au point Pj 7

3.5 L'Intensité sonore et le niveau sonore


Si vous avez déjà essayé de d o rm ir alors que votre voisin faisail jo u e r de la mu.sique à
un volume élevé, vous savez 1res bien que le son com porfe autre chose q u ’une fréquence,
une longueur d 'o n d e et une viies.se. Il com porte égalem ent une intensité. L’in te n sité /
d ’une onde sonore su r une sut face est le taux m oyen par unité de surface de l’énergie de
fo n d e passant à travers la surface. O n peut l’écrire sous la form e
P
I —,
A
où A est f a ir e de la surface qui iiilerrcptc le .son et P est le taux d 'cn e rg ic (la puissance)
de f o n d e sonore p assan t à trav ers A. L ’unité SI de l ’in ten sité / est le W/m^. C om m e
on le verra dans la preuve à la fin de cctlc section, l’intensité / est reliée à l'am plitude
de déplacem ent s„ de f o n d e sonore par

F I •) t
l - . (3.27)

Le son peut faire osciller les parois


d'un verre. .Si le son produit une onde La variation de l’intensité avec la distance
stationnaire dans le verre et qu’il alieint
l,a variation en fonction de la d istan ce de l'in te n sité d ’une source sonore est p arfo is
une assez grande intensité, le verre
com plexe. C ertaines sources (des haut parleurs, par exem ple) peuvent iransnieiirc le son
éclatera.
d ans c e rta in e s d irec tio n s seu lem en t, et le m ilieu a m b ian t p ro d u it h ab itu ellem en t
62 Chapitre 3 Les ondes sonores

des échos (onde.s sonores réfléchies) qui chevauchent les ondes .sonores directes. D ans
certaines situations, on peut umicfois ignorer les échos et supposer que la source sonore est
une source ponctuelle qui ém et le son de façon isoirope, c ’csi-à-dire de façon uniform e
dans toutes les directions, figure 3.10 illustre des fronts d ’onde qui s ’éloignent d ’une
telle source ponctuelle isotrope 5 à un instant donné.
S upposez que l’énergie m écanique des ondes sonores est conservée pendant que les
ondes SC dispensent à partir de cette source. Im aginez m aintenant une .sphère de rayon r
cenüée par ntpjKiri à la source, com m e à la figure 3.10. Toute l’énergie ém ise par la source
doit traverser la surface de la sphère. I x taux d ’énergie des ondes sonores passant à travers
la surface de la sphère doit être égal au taux d 'ém issio n de l'énergie par la source (c ’esi
à dire la puissance F, de la source). D ’après l'équation 3,26. l ’inteasité / à la .surface de
la splière doit alors être de
Rgufe 3.10 Une -icpuat pontiiicllc 5
itw t deq ondes sonores de façon
uiiit'ornic daii:s toutes les dirceliutis. / = (une source i.sotnipe). (3.28)
Les ondes sonores passent à travers
line sphère imaginaire rie rayon r.
centrée p a r ra p p o rt à S. oü4.Tr^ est f a ir e de la sphère. L 'équation 3.28 indique que l’intensité du son provenant
d ’u n e source p o n ctu elle iso tro p e d im in u e .selon le ca rré de la d istan c e r à la so u rce
ponctuelle.

✓ v é r if ie z VOS CONNAISSANCES 3 ; ^’ illu s tra tio n c i-c o n tre


montre trois petites plaques, 1, 2 et 3, situées sur la surface
de deux sphères imaginaires ; ces sphères sont centrées par rapport
à une source ponctuelle S qui émet un son de façon isotrope.
l,es taux d’énergie des otules sonores passant à travers les trois
plaques sont égaux. Classez les plaques en ordre décroissant
scion a) l'intensité du son sur chacune d ’elles et b) leur aire.

L’échelle des décibels


Dan.s l'exem ple 3.2, vous avez v\i que l’am plitude de déplacem ent dans fo rc illc hum aine
variait, approxim ativem ent, d e 10" ' m, pttur les sons se situant au seuil de la douleur,
à 10 m, pour les plrcs faible.s sons détectés, soit un rapport de 10*’. Par l'équation 3.27,
on v o it q u e l’in te n sité d ’un son v arie selon le carré d e son am p litu d e , ce q ui fait
que le rapport d'in ten sité entre ces deux valeurs extrêm es du systèm e au d itif hum ain est
de 10'-. I .’être hum ain peut entendre une gam m e très étendue d ’intensités.
P our tra ite r un e g am m e de v aleu rs aussi éten d u e , on iiiili.se les lo g arith m es.
C onsidérez la relation

y = log(A-).

où X et y so n t des v ariab les. E n sc serv an t d 'u n e p ro p rié té des lo g a rith m es, on peut
d é m o n tre r q u e si on m ultiplie a par 10, y au g m en te d e 1. En e ffe t, on p eu t réc rire
y ' = log( K k ) .scHis la form e

v' = log(H)A) = lo g (lü ) -b log(A) - 1 + y.

De la m êm e façon, si on m ultiplie a par 1 0 '\ y augm ente seulem ent de 12.


D onc, au lieu de p arlei d e l’in ten sité / d ’une onde sonore, il e s t plus in d iq u é de
parler de son n iv e a u s o n o re fi, q u 'o n définit ainsi ;

fi - ( l O d B l l o g —. (3.29)

T ^ns cette équation, dB est l’abréviation de décibel, l'unité associée au niveau .sonore, un
nom qui a été choisi pour rendre Ы итпарс à A lcxiuxlerG raham Bell. Dans l’équation 3.29,
/о est une référen ce standard d ’inlen.sité qui esl ég a le à 10" '- W /m ^ q u 'o n a ch o isie
parce q u ’elle se rap p ro ch e de la lim ite in férie u re d ç fin tc n .sité so n o re qui p eu t être
perçue chez l ’hum ain. On appelle /„ le seu il d ’a u d ib ilité . P our / = /ц. l'éq u atio n 3.29
donne /I — 10 log( 1) = 0 , de sorte que le seuil d 'au d ib ilité correspond й zéro décibel.
fi augm ente ensuite de 10 dB chaque fois que l'in ten sité du son augm ente d 'u n facteur
3,5 L'intensité sonore et le niveau sonore 63

TABLbAU3.2 Cueiques niveoux sonores (dB) d e 10. A in si, ^ = 4 0 dB c o rre s p o n d à u n e in te n silé d e 10^ fo is le se u il d ’a u d ib ilité .
Seuil d’audibilité 0 L e ta b le a u 3 .2 pré-scnic les n iv eau x so n o re s d e q u e lq u e s en v iro n n e m e n ts.
Bruissement de feuilles 10
La preuve de l’équation 3 27
Conversation 60
Concert rocit 110
C o n sid é re z , d a n s la lig u re 3.5 a), u n m in c e élé m e n t d e v o lu m e d ’a ir d ’u n e é p a isse u r dx,
d ’a ire A et d e m a s s e dm , q u i o s c ille v e rs l’a v a n l et l’a rriè re lo rs q u e l ’o n d e s o n o re
Seuil de la douleur 120 de l’é q u a tio n 3 .1 3 le trav erse. L ’é n e rg ie c in é tiq u e d K d e l’élé m e n t d e v o lu m e d ’a ir est
Réacteur d’avion (à 20 m) 130
d K = ^dni if . (3 .3 0 )
Ici, V, n ’e st p a s le m o d u le d e la v ite sse d e l'o n d e , m a is b ien ce lu i d e l’é lé m e n t o scillan t.
E n d é riv a n t l’é q u a tio n 3 .1 3 p a r ra p p o rt au tem p s, u n o b tie n t la c o m p o sa n te v ,,i
hx
V, , = — = tüSn, SiniL r - ûJt).
l)t “
P u isq u e l'é lé m e n t d ’a ir effe c tu e u n m o u v em en t en une d im en sio n , le lo n g d e l’ax e d es x,
l’j-est é g a l à v ^ ,. E n u tilisan t c e ré su lta t et en rem p laçan t dm p a r p A dx, o n p e u t récrire
l’é q u a tio n 3 .3 0 so u s la fo rm e
d K - i(/oA </.v)(ty.v„)’ s in ’fL r - wt). (3.311
E n d iv isa n t l'é q u a tio n 3.31 p a r dj. o n d é te rm in e le tau x a u q u e l l’é n e rg ie c in é tiq u e e st
tra n sp o rté e a v ec l ’o n d e. C o im n e o n l'a v u d a n s le c h a p itre 2 , e n ce q u i a trait au x o n d es
Iran sv ersale.s. d x id t e st la c o m p o s a n te x d e la v ite s s e d e l ’o n d e q u i, ic i, e st é g a le au
iikk Iu Ic d e la v ite sse d e l’o n d e v ; o n a d o n c

= sin^(L< — (Dt). (3.3?)


dt

L e tau x nioven a u q u el l’é n e rg ie c in é tiq u e e st tra n sp o rté e est

( ^ ) m o y "" lsin -(L r -


= \p A vio h i,. (3 .3 3 )
Pt)ur o b te n ir c e tte d e rn iè re é q u a tio n , o n a ten u c o m p te d u fait tju e la v a le u r m o y eu n c
d 'u n e fo n ctio n sin u s (t)u c o sin u s) au caiT é lo rs d ’une p é n o d e c o m p lè te c.st | .
O n su p p o se q u e l’é n e rg ie p o ten tielle est tra n sp o rté e a v e c l’o n d e à c e m ê m e tau x
m o y en . L ’in ten sité i d e l’o n d e, q u i e st le taux m o y e n p ai u n ité d e su rfa c e deji d eu x types
d ’én e rg ie d e l’o n d e p assa n t à trav ers u n e su rfa c e , a st alo rs, d 'a p rè s l'équaL ion 3 .3 3 ,
2{dK /dt) moy I 1,
/ = = jp v r./.v -,

c e q u i c o rre sp o n d à l'é q u a tio n 3 .2 7 . q u e l’o n v o u la it d ém o n trer.

Exemple 3.4
Une étincelle électrique se déplace le long d’une ligne droite de TrajfH't(»ire
longueur £ = 10 ni en émettant du son. Dans une telle situation,
tm peut considérer que la source sonore est une .source linéaire, de
/ d r I t'iincelle

lungucur £_ qui émet une impulsion sonore et dont les fronts d’onde
sont des surfaces cylindriques ayant la source comme axe de
symétrie La puissance de rémission e.st I’, — 1.6 x 10^ W.

al Quelle est rinlensité £du sou lorM)u'il atleint une distance r = 12 tn


de la source ■’

SOUITIOK' Imaginez un cylindre de rayon r = 12 ni et de longueur


1,— 10 m (ouvert aux deux extrémités), centré par rapport à la source
sonore linéaire, comme à la figure 3.(1. l.e corxepl tié est celui-ci : Fgute 3.11 Exemple 3.4 Une étincelle se déplaçant le long d'une ligne
l'intensité / sut la surface cylindrique est le rapport P/A entre le taux P droite de 1c>ngueur L émet des ondes sonores On considère la trajectoire
au(|uel l’énergie son<w traverse la surface et l'aire de la surface A. de l’clincclle comme une source linéaire, de longueur /., émettant
Suivant un autre cotKegl tlê. on suppose que le principe dt' cnascrvalion line impulsion sonore Les ondes tiaver.seni un eviindre imaginaire
de l’énergie s’applique îi l’énergie sonore. Cela signifie que le taux P de ray on r et de longueur L centre pai rapport à la source.
M Chapitre 3 Les ondes sonores

auijuel l’énci^ie pa<isc à travers le cylindre doit être égal au taux F, SOLUTION: Si on applique le prem iertonteplclé utilisé en a), on sait que
auquel l'cncrgic est émise par la source. En ciinibinant ces idées et l'intensité du son au détecteur équivaut au rapport entre le taux
en notant que l’aire ilc la surface cylimirique e.st A = iTtrL, on a d’énergie sonore passant à travers la surface du détecteur et l’aiic
P du détecteui ;
(3.34)
Im L
Cela indique que rinlensitc du .son provenant d ’une source linéaire Pi
(3.35)
diminue avec la distance r (et non scion le carré de la distance r. Ai
comme dans le cas d'une source ponctuelle). En iirsérant les données
On peut supposer que le détecteur repose sur la surface cylin­
connues, (Ht détermine que
drique de a) puisque sa stirlacc est faible par rapport à celle du
E é x lO ^ W -, , cylindre. L'intensité du son à la surface du détecteur est l’intensité
1 = = 21.2 W/m‘ ^ 21 W/iii". (réponse)
2,7(12 in)(lf) m) / ( = 21.2 W /nr) à la surface du cylindre. En isolant P,y daii.s l’équa­
tion 3.35, on obtient
b) À quel taux Pÿ l'énergie sonore est-elle inlerccjMcc par un détecteur
acoustique d'aiic - 2.0 cm’ orienté vers la source sonore et situé
ü tiiR‘ distance r = 12 m de la WHirce 7 P^ = (21,2 W/ni’)(2,() y. 10“'' m^) " 4.2 mW. (réponse)

Exemple 3.5
Fn 197fi. le groupe rock The Who a ét.ihli le record du plus haut
niveau sonore jam ais atteint dans un concert; le niveau sonore à
4fï m en face des hout-pnrieuis était de fij = 120 dB. Quel est le rap­
port de l’intcnsiic It du groupe à cet endroit sur l'intensité /, d ’un
manraii-iijqneiii privinisani un niveau sonore /?, - 92.0 dB ?

SOLUIION. Ix coixeptilé utilisé ici est que ptiui le gnuipe The Who comme
pour le marteau piqueur, le niveau sonore fi est relié à l’intensité
par la dcfimiion du niveau sonore que donne l’équation 3.29 Pour
The Who. on a

/32 = (10dB) l o g ^ .
/0
et, pour le marteau-piqueur, on a

fl, = ( lO c lB )lo g ^ .
lo
La différence entre les niveaux sonores est

= ( 1 0 d B )^ lo g j= - l o g j i j . (3..36)

En utilisant l'identité

a c ad
lo g - - lo g - = lo e -,

on peut récrire l'équaiinn 3.36 .mhis la forme

(10 dB) l o g ^ . (3.37)


/1
figure 3.12 Exemple 3.5 Pete Townshend, du groupe The WTio. jouant
En réarrangeant les fermes cl en insérant les niveaux sonores connu.s.
devant un ensemble de haut-parleurs. Il a subi une réduction
on obtient
pci manentc de ses capacités auditives, causée par son exposition
, h /*2 - h 120dH - 92.0dB à des sons de haute intensité, pas tellement durant ses performances
= 2.80. sur scène que lorsqu'il portait des écouteurs dans les studios
/i 10dB lOdB
d’enregistrement et à la maison.
En prenant le logarithme inverse des membres gauche et droit de
cene éqii.ation (la fonction antilogarithme est 10'). on trouve
The Who produit une réduction temporaire des capacités auditives,
^ = 10^» = 631. (rcpon.se) l-ne exposition prolongée ou répétée peut résulter en une réduction
/|
permanente des capacités auditives (voir la hgurc 3.12). I,a perte de
Effectivement. The Who jouait très foit. l’nuïc est un nsque évident poûr la personne qui. par e.xemple. écoule
Kiic exposition temporaire à des intensités sonores comme continuellement de la musique heavy metal à un volume élevé,
celles d ’un marteau-piqueur nu du concert de 1976 du groupe particulièrement si elle utilise des écouteurs.
3.6 Les ondes stationnaires dans les colonnes d'air 65

3.6 Les ondes stationnaires


dons les colonnes d ’air
D e s so n s m u sicau x p cu v c n i ê tre p ro d u its e n faisan t o sc ille r d e s c o rd es (g u ita re , p ian ti.
vio lo n ), d es m em b ran es (tim b ale, caisse claire), d e s co lo n n es d ’a ir (Ilûtc, h au tb o is, o rgue,
fu ja ra d e la fig u re 3.1 3 ). d e s b lo c s d e b o is o u d e s b a rre s d ’a c ic r (m a rim b a , x y lo p h o n e,
v ib rap h o n e) et d e n o m b reu x au tres o bjets. L a p lu p art d es in stru m c n ls ne c o m p o rte n t pas
q u ’u n e sim p le p a rtie o sc illa n te . D a n s un v io lo n , p a r e x e m p le , les c o rd e s e t le c o rp s de
l ’in stru m e n t p artic ip e n t en se m b le à la p ro d u c tio n d e m u siq u e.
D a n s le c h a p itre 2 . o n a vu q u e d e s o n d e s s ta tio n n a ire s p o u v a ie n t ê tre p io d u ilc s
d a n s u n e c o rd e te n d u e e t fix ée à ses d eu x c x tic m ité s. E lle s se p ro d u ise n t p arc e q u e les
o n d e s c irc u la n t d a n s la c o rd c so n t ré flé c h ie s d a n s la e ttrd e à c h a q u e c x irc m iié . S i les
o n d e s o n t u n e lo n g u e u r d 'o n d e c o n v e n a b le m e n t as.so rtie à la lo n g u e u r d e la c o rd e ,
la superpr> sition d e s o n d e s ,sc p ro p a g e a n t d a n s d e s d in ectu in s o p p o sé e s prvHluil su r la
c o rd e u n p ro fil d 'o n d e s ta tio n n a ire (o u in ix le d ’o sc illa tio n ). L a lo n g u e u r d ’o n d e d o it
a lo rs c o rre s p o n d re à m t fré q u e n c e d e réso n a n ce d e la c o rd e L 'a v a n ta g e d e s o n d e s
s ta tio n n a ire s e s t q u e la c o rd e o s c ille a lo rs a v e c u n e a m p litu d e fo rte e t s o u te n u e , q u i
p o u sse l’a ir en v iro n n a n t v ers l'a v a n t e t l’a rriè re et g é n è re ain si u n e o n d e so tx irc aiuliblc
q u i a la m êm e fré q u e n c e q u e les o scillât ¡»ms d e la c o rd e. C e tte p ro d u c tio n d e so n p ren d
to u te son im p o rta n c e p o u r un g u ita riste , p ar ex em p le.
O n p e u t p ro d u ire d e s o n d e s so n o re s sta tio n n a ire s d e la m ê m e façim d a n s u n tu b e
re m p li d 'a ir. I,e s o n d e s .sonores q ui sc p ro p a g e n t d a n s f a i r d u tu b e so n t ré n ë c h ie s à
c h a q u e e x tré m ité e t re to u rn e n t e n se n s in v e rse d a n s le tu b e (I.a ré fle x io n se pn>duil
m em e lo rs q u ’u n e extrém itt'i est o u v e rte , m ais c lic n c sl p a s aus.si c o m p lè te q u ’av ec u n e
e x tré m ité fe rm é e .) Si la l»»ngue.ur d ’oncle d e s o n d i's so n o re s e st a sso rtie à la lo n g u e u r
d u tu b e , la su p erp o sitio n d es o n d e s c irc u la n t d a n s d e s d ire c tio n s o p p o sé e s d an s le tulie
p rtx lu it u n e o n d e statio n n aire. L a lo n g u e u r d ’o n d e d e s o n d e s so n o re s d o it alo rs c o tre s
p o n d re à u n e fré q u e n c e d e ré so n a n c e d u tu b e. L ’av an ta g e d 'u n e telle o n d e statio n n aire
e st q u e l’a ir d a n s le tu b e o sc ille a lo rs a v e c u n e a m p litu d e fo rte e t so u te n u e , et é m e t à
to u te e x tré m ité o u v e rte u n e o n d e so n o re q u i a la m êm e f'réi)iiencc q u e les o.scillations
d a n s le tu b e . P o u r u n flû tis te , p a r e x e m p le , e e tte é m is s io n s o n o re re v ê t u n e grande-
Figure 3.13 Des oscilhuions se produisenl im p o rtan ce.
dans la colonne d’iiii d ’un/w/ora
L e s d iffé re n ts m o d e s d ’o n d e s ta tio n n a ire d a n s un tu b e p ré.scn len t d e n o m b re u x
lor«|ii’on jonc de CCI insirumcnt
a u tre s a sp e c ts se m b lab les à c e u x d es o n d e s statio n n aires d an s u n e eo rtlc l’e x lrcm ilc
tradiiionnel slovaque.
fe rm é e d ’un tu b e c o m m e à l’e x tré m ité fix e d ’u n e c o rd e , il d o it y a v o ir un lu eu d (P a ir
ne p e u t o s c ille r lo n g itu d in a le m e n t, p a ra llè le m e n t au tu b e , à c a u s e d e la p a ro i), e t à
l’e x tré m ité o u v e rte d ’un tu b e c o m m e à l'e x iré m iié d 'u n e c o rd e a tta c h é e à un a n n e a u
c o u lissa n t lib re m e n t, c o m m e d a n s la fig u re 2 IQ b), il d o it y a v o ir u n v e n tre d ’a ir e st
libre d ’o sciller). (L n réalité, le v e n tre à l'e x tré m ité m iv ertc d ’un tu b e sc situ e u n p eu en
d e h o rs de cette e x tré m ité , m ais o n ne sc so u c ie ra p a s d e c e d étail ici.)
Al = 2L
L a fig u re 3 .1 4 a) p résen te les d é p la c e m e n ts lo n g itu d in a u x (fo lle m e n t e x a g é ré s) des
é lé m e n ts d ’a ir p o u r fo n d e statio n n aire la p lu s sim p le p o u v a n t être p ro d u ite d an s u n uilie
d o n t le s d e u x e x tré m ité s s o n t o u v e rte s . C o m m e il sc d o it, il y a u n v e n tre à c h a q u e
e x tré m ité o u v erte. Il y a é g alcm eiil u n neeud au m ilieu du tu b e. U ne faço n p lu s facile d e
rep résen ter cette o n d e so n o re statio n n aire lo n g itu d in ale est illustrée à la figuiv 3.14 b), où
elle est d essin ée sous la fo rm e d ’une o n d e tran sv ersale siaiioniinirc co m m e d an s une cordc.
L ’o n d e statio n n a ire d e la fig u re 3 .1 4 a) rep ré se n te le p re m ie r m o d e ; <in le n o m m e
le m ode fondam ental o u le p rem ier harmonique. Potii k- p ro d u ire. le.s o n d es so n o res dan.s
figure 3.11 a) L'onde stmionnaire la plus
un tu b e d e U ingtieiir L iloiveni av o ir une lo n g u e u r d 'o n d e d o n n é e p a r L = A ,/2 , d e so rte
»impie qui peut êire générée par des ondes
q u e A, — 2 / . D e n o m b re u x a u tre s m o d e s d 'o s c illa tio n d a n s un tu b e au x e x tré m ité s
sonores (longitudinales), dans iin tuyau
o u v e rte s so n t p ré se n té s à la fig u re 3. L'i a) so u s la fo rm e d ’o n d es tra n sv e rsa le s co m m e
dont les deux extrémités srrnt ovivctlcs,
dan.s u n e co rd e. L e deuxièm e h a rm onique o u d eu xièm e m ode d o it a v o ir u n e lo n g u e u r
comporte un ventre (V) à chaque
d ’o n d e A 2 = A = 2 U 2 . le tro isiè m e h a rm o n iq u e o u tro isiè m e m o d e d o it a v o ir u n e
cxlréinité et un nreud (N) au milieu,
lo n g u e u r d ’tvndc A j = 2U '^, et ain si di^ suite.
l.es déplacements longitudinaux
D e faço n p lu s g én érale, u n e o n d e statio n n aire p eu t donc ê tre p ro d u ite d a n s u n tu y au
des éléments d'air représentes
dti lo n g u eu r L aux d eu x ex trém ités o u v ertes .si la lo n g u eu r d ’o n d e d e l’cmdc so n o re c o rres­
par les (lèches doubles .sont
p o n d à l'u n e d e s v a le u rs su iv a n te s :
considérablement exagérés dans 2A
r illustration b) On représente cette o ù n =■ 1 .2 ,3 ,... (3 .3 8 )
onde sonore stationnaire longitudinale
sou.s la forme d’une onde transversale et <vù n re p ré se n te le m o d e d 'o scilla tio n . L e s fré q u e n te s d e ré.sonancc d a n s u n tu y a u
stationnaire comme daas une corde. d o n t le s d eu x e x tré m ité s so n t o u v e rte s so n t d o n n é e s p a r
66 cnapi»re 3 Les ondes sonores

V nv
и = ‘2 = ‘- ÎV 2 - L où и = 1 ,2 ,3 , ... (un tuyau, deux cxtréniilés ouveriesj, (3.39)
-.СУ
et où l’est le m odule de la vitcs.se du son.
n= 3 Aj = 2/./3
L 'équation 3.39 m ontre que les fréquences de résonance sont des m ultiples entiers
de v /(2L ). q u i c o rre sp o n d à la fréq u en ce fondam entaie f (la p lu s b asse fré q u en c e
t iX y 3 , = 2/./4 - U 2 de résonance obtenue lorsque n = I ). c'est-à-d ire que f„ = nvHL = rt/'j. Puisque la série
^ - harmonique
numique (l’enscinble
ire n s c in n ie d e tous les m odes d ’o.scillation
o.scillafion po.ssihlc.s) d ’un tuyau ouvert
aux deux ex tré m ité s c o n tie n t tous les m u ltip le s e n tie rs d e / , . to u s les h arm o n iq u es
( / t . / i , / } . etc.) peuvent e x iste r; donc, n rcpré.scntc ég alem en t le nombre harmonique
'"J.n - 1 = 1 4L du énièm e harm onique.
l . ./-л
La figure 3.1 .“i b) m ontre (sous form e d ’ondes transversales dans une corde) certains
m odes d ’ondes sonores stationnaires pouvant être pixxluits dans un tuyau à une seule
277 - ■Ч
=C extrém ité tHivcrtc. C om m e il se doit, il y a un ventre à l’extrém ité ouverte et un nreud à
l’cxtrcm ité ferm ée. Pour l’onde stationnaire la plus sim ple, le prem ier mode, les ondes
2« - 1- R A..” 1Л..-1 sonores ont une longueur d ’onde donnée p ar L = À,/4 , de sorte que à , = 4L. D ans le
deuxièm e m ode, la longueur d 'o n d e est donnée par L = 3X,/4, de sorte que Aj = 4L/3,
et ainsi de suite.
2n - 1= 7 A, = 4/77 D e façon plus générale, une onde stationnaire peut donc cire produite dans un tuyau
o e z
de lo n g u e u r L d ont u n e seu le ex tré m ité est o u v erte si la lo n g u e u r d 'o n d e d e l'tm d c
b)
sonore correspond à l'u n e des valeurs suivantes :
Rgure 3.15 Des profils d'ondes
4L
.staiit »maires' transversales comme Лг„-1 = - -------- -, où « = 1. 2. 3 ,... (3.40)
dans une cordc sont superposc.s 2/1 — 1
sur des tuymix poui illii.siier leurs Com m e pour toutes les autres situations, n représente le mode d ’oseillution. Les fréquciKes
dilfcrents mottes d’oscill.ation, de résonance sont alors données par
a) Dans les tuyaux dont les deux
cxlrcmités sont ouvertes, tous (2n - l)v-
les harnK>ni<|iies peuvent être prtxluits. fin - I “ où /1 = 1 .2 , 3 , ... (uii tuyau, une extrémité «iverte). (3.41)
^ 2«-l 4L "*
b) Lorsqu’une seule extrémité est
ouverte, .seuls les harmoniques L 'équation 3.41 m ontre que les fréquences de résonance sont des m ultiples impairs de
impairs peuvent être produits. v/(4L). qui correspond à la fréquence fondam entale /,. c ’est à dire que -i = (2n — 1) /|.
Puisque la série harm onique d ’un tuyau ou v ert à une ex trém ité contient seulem ent les
m ultiples im pairs d e / |, seuls tes harmoniques impairs etc.) peuvent exister;
donc, 2/1 —I rep rése n te le nombre harm onique d u én ièm e h a rm o n iq u e alo rs q u e n
représente le m ode d ’oscillation. Par exem ple, le troisièm e m ode n = 3 correspond au
einquicm e harm onique 2/i — I = 5. Le deuxièm e harm o n iq u e ne peut pas être produit
d an s un tuyau se m b la b le, m ais on p eu t p ro d u ire tous les m odes N o tez eg a le m en t
q u e, d a n s un tel tu y a u , le te rm e « tro is iè m e h a rm o n iq u e » d ésig n e le ran g 2n - 1
de riia rm o n iq u e, et non le troisièm e harm onique possible.
La longtieui d ’un instrum ent de m usique reflète la g.ammc de fréquences poui Itiquclle
il a été conçu ; ainsi, une petite longueur im plique d e hautes fréquences. La figure 3.16

» • >ЯЧ<J/Imir b.tmon
ЛЩшмттвктт угшщртщли
|\<^1и‘1И'lilNt

figure 3.16 Les laniilles des saxophones


CI des vktliins. gui illii.s(rciii la lelaiion
entre la longueur d'un insfrument
et sa gamme de frcquetKes. La gamme
de fréquenees (le chaque instnjiiient
est indiquée par une bande hori/onfale
verte que l'on peut comparer à la gamme
de fiéquences d'un clavier de piano;
la frcquciK;e augntemc eu allant vers
ta droite.
3.7 Les battements 67

Figure 3.17 Des undes sonores produites par a) une flûte, b) un hautbois et e) un saxophone
lorsqu’ils jouent tous la meme note, c’est-à-dire lorsqu'ils émettent la même fréquence
fondamentale.
a)

cti donne un exem ple en présentant la fam ille des sax«)phone,s et celle dc.s violons, avec
leur gam m e respective de fréquences par rapport un clavier de piano. N otez tiue les
plus basses e t les plus haute.s fréquences de chaque instrum ent ch evauchent celles de
rin stru m c n l voisin.
D ans tout systèm e o.sdllant qui d o nne naissance à un son mu.sical. q u ’il s ’agisse
d ’une cordc de violon ou d ’un tuyau d 'o rg u e, le m ode fondam ental est habituellem ent
produit en m êm e tem ps que un ou plusieurs harm oniques sujiérieiirs. V<hi.s les en ieiv le/
sim ultaném ent, c ’e.st-à-diie q u 'ils se .sinierposent et forment une oiulc résultante. I^iisquc
la m êm e n ote est jo u é e par d iffé re n ts in stru m en ts, ces d e rn ie rs p ro d u isen t la m êm e
fréquence fon d am en tale, m ais avec d es in ten sités d iffé re n te s p o u r les h:iim oniqiies
supérieurs. Par exemple, le quatrième harmonique du ilo central petit cim joué rclativcincnl
fort par un instru m en t, et faiblem ent ou pas du tout par un au tre instrum ent. P uisque
différents instrum ents produi.sent d es ondes résultantes différentes, vous les percevez
dificrcm m cnt m êm e s ’ils joueni la m ême note, ce qui produit le limhrc dc.s insli um ents
Temps
de m usique. C e .serait le cas des trois ondes résultantc.s illustrées à la figure .1.17, qui ont
été produites par différents instrum ents jo u an t la m êm e note.

'VÉRIFIEZ VOS CONNAISSANCES 4; Un tube A de longueur L et un tul>c ü de longueui 2L

Exemple 3.6
r ont leurs deux extrémités ouvertes. Quel harmonique du tube H a\a même fréqiieiK’e
que le mode fondamental du tube A ?

Le faible bruit de fond d'une chambre Ihit résonner au mode fondamen­ b) Si vous collez votre oreille contre une extrémité du iiihc. quelle
tal un tube de carton de longueui L ~ 67.0 cm, ous'en aux deux extrémi­ triquencc fondamentale enlcndrcz-vous dans le tube ?
tés. Le module de la vitesse du son dans l’air du tube est de 343 m/s
SOLUTION: On applique maintenant le tontcpl dé suivant : si votre oreille
a) Quelle fréquence entendez vous dans le tube '? olHiire une extrémité du tube, il y a encore un ventre à l’cxtreniilc
SOLUTION: Le concept dé qui s’applique est celui-ci : lorsque les deux ouverte, mais on trouve maintenant un nteiid à l’autre extremité
extrémités du tube sont ouvertes, l’onde stationnaire esi symétrique, (celle qui est obturée par votre oreille), l.a représentation de l’onde
elle a un ventre à chaque extrémité du tube. La représentation de stationnaire est celle du haut de la figure 3.15 b). La fréquence
cette onde sonore stationnaire sous la forme d’une onde transversale fondamentale est donnée par l’cquation 3.41. avec « “ 1 ;
stationnaire dans une corde est illustrée à la figure 3.14 b).
La fréquence fondamentale est donnée par l’équation 3.39, avec « = 1 ; ^ _ (2n - l)v (l)(343 m/s)
/ 2,, 1 — 128 H/., (réponse)
(l)(343 m/.s) (4K0.670 m)
= 256 Hz. (réponse)
2L ~ (2)(0.670 m)
Si le bruit de fond produit des harmoniques supérieurs, comme le Si les bruits de fond protluisent d ’autres harnKiniques. ils sciont des
deuxième harmonitiue. vous devriez également entendre les fréquences multiples impairs de 128 Hz. Cela signifie que la fréquence de 256 Hz
qui sont des multiples entiers de 256 Hz. (un multiple pair) ne pourra être prtxluite.

3.7 Les battements


Si des gens écoulent, à quelquc,s m inutes d 'in te rv a lle , deux sons d o n t les fréquences
sont, pttr exemple, de 552 Hz et de 564 Hz, la plupart d'antre eux ne (xiurroni les distinguer
l’un de l’autre. T outefois, si les sons atteig n en l leurs o reilles en m êm e tem ps, le son
q u 'ils enten d ro n t aura une fréquence d e 558 H z. soit la moyenne des fréquence.^ des
deux sons qui sc superposent. Ils retnarqueront égalem ent une variation frappante dans
l'in te n sité de ce .son. qui augm entera et dim inuera suivant de lents h a tte n ie n ts q u i se
répéteront à une fréquence de 12 Hz. soit la différence entre les fréquences d es deux
ondes sonores .superposce.s. L a figure 3.18 illustre ce phénom ène de batteiTient.s.
S oit d eu x o n d es so n o res se p ro p ag e an t d a n s la m êm e d irec tio n et a tteig n an t
un détecteur, qui p o u rra it être une o reille, situ é à Jt = 0. O n su p p o se q u e les o n d es
p o ssèdent la m êm e am p litu d e el des fre q u e n c e s /, e t /2 v o isin es, c ’e si-à -d ire q u e la
différence des fréquences est inférieure à quelques dizrtines de hertz. Pour chatjue onde
.sonore, on peut donc réécrire l ’équation 3.13 représentant le d éplacem ent en fonction
du tem ps produit par une onde sonore de la fa^tin suivante;

.1, == .Sn, cos((o,i) et cosloj,/). (3.42)


68 Chapitre 3 Les ondes sonores

Figure 318 a), h) Ix s déplaceinenis longitudinaux d'un pelii volume d'air puKluii par deux ondes
sonores émises séparément Les fréquences des ondes sont prest]ue les mêmes, c) Le déplucemenl
*)il resultant de rélcmcnt d'air lorsque les deux ondes sont perçues simultanément, qui correspond
à la superptisilion des graphiques a) et b).

Scion le principe d e superposition, le dcplaceiTKni résultant est


h) = s, + .^2 = «M (COS(<0,r) + COSt«>2/)J.
bn utilisant l’identité trigonom éiriquc (voir l'anne.xe D).
cos a + co s fi = 2 c o s | j í t r - /i) | c o s [ 5( a + /J)j ;
on peut donc écrire le déplacem ent résultant sous la form e
s = 25,„COs [ î ( o>| - <Ui)r| COs[i(fa>, + ÎOi)/]. (.3.43)
Si on pose que
Tfiiip»
ÛJ' = Í (w , — tUi ) Cl Cü = + <«>2 ), (3.44)

on peut alors écrire l'éq u atio n 3.43 com m e suit :

i(r> = | 2.t,„ cos(&)'n] cos(cor). (3 .4 Í)

P u isq u e / , e t /2 sont d es fréq u en ces v o isin es, les fréq u en c es an g u laires itj, et îd ,
des ondes qui se superposent sont presque les m em es, ce qui signifie que w ^ te' (selon
l’équation 3.44). La fréquence angulaire de l’onde sonore perçue est œ. c a r eu' co rres­
pond. en général, à une fréquence trop fiiihle pour être perçue com m e un son par la iilupart
des détecteurs. Selon l’oquation 3.44, cl en rem plaçant tu par I ttJ, la fréquence perçue f
p ar le d é te c te u r e.si d o n n ée p a r / = ( / j + iz)l2 . ce qui c o rre sp o n d à la m o y en n e des
fréquences des o n d es sonores qui sc superposent. O n peut donc v oir l'é q u a tio n 3.45
conuiie une fonction cosinus dwit la fréquence angulaire est tu et dont ram pliiudc (qui n’est
pas constante m ais varie iivec la fréquence angulaire tu ') est la quantité entre crtichets.
La v ariation de l ’am p litu d e de l’o n d e .sonore résu ltan te en fo nciiim de cos(<u'r)
produit une variation d 'in ten sité sonore qu e l'o n nom m e battem ents. L 'intensité varie de
façon périodique dans le tem ps : elle est m axim ale à tous les tem ps / dont co s(tu '/l = ± 1
et nulle pour tout tem ps t .satisfaisant la condition cos(iu'r) = 0 U ne in ten sité sonore
m axim ale sc produit chaque fois que cos(tu'/) a la valeur + 1 ou — 1 dans réqualKiii 3.45,
ce qui se produit deux fois dans une (vériode de la fonction co.siiius. Pui.sque cos(cu'r)
a une fréquence angulaire <u'. la fréquence an g u laire 01^,, à laquelle les b attem en ts se
produisent est = 2o)'. En utilisant l'éq u atio n 3.44, on peut ensuite écrire

= 2io' = (2 )(V)(<y, - ii>2) = tU] - üjj.


Puisque ù) — 2:if. on obtient la fréquence de battem ents

A m ~ l/i “ / ’l (la fréquence di battement*). (3 46)

Les musiciens uiiliscnt le phénom ène de battem cnis pour accorder leurs instruments.
Si on accorde un insirumeni à l’aidc de l’ém ission d 'u n e fréquence siandard (le la majeur
du hautbois. parc.xem plc) cl q u 'o n ajuste la fréquence de l insirum cni /^ ju sq u 'à ce que les
battem ents disparaissent (Aai (>)• scion l'équalion 3.46. l'in stru m en t sera alors accordé
avec cc standard. À V ienne, capitale m usicale, le la de concert (fréquence de 4 4 0 H t)
est disponible par téléphone, pour le bénéfice des nom breux m usiciens professionnels
et am ateurs de cette ville.

Exemple 37
Vous voulez accorder la note la de vpiie piann à sa fréquence exacte deuxième harnitmiquc priKluira des hnlicmcnis avec la fréquence
de 220 Hz. Vtnj.s avez à voire disposition un diapason dont la fré­ de 440 Hz du diupason. Pour ajuster cette corde, il vous suffira iréeoii-
quence CM 440 H / ( 'nmiueni allez-vous procéder"' ter CCS battements et de tendre ini de détendre la corde de manière
à diminuer la fréquence de b.ittcment.s. jusqu'à ce qu’elle di.sparaissc.
(OltnUW: l Jn devTO emploYci ces deux (Oiw^ tfo ; 1) Les deux fréquences
sont trop éloignées pour produite des hattciiK'nLs. 2 l toutefois, la coidc ✓ vérifiez VOS CONNAISSANCES 5; )ans rexem plc précétlciu.
de piano oscillera non seuIcnKni à son mode fombmcntal (à 220 M/, si vous tendez la corde et la fréquence de battements augmente de
elle est Iricn aeamkie). mais mlvsi à son deuxieme harmonique (à 44ÎI Hz). 6 Hz Devriez-vous continuer à tendre la corde ou devriez.-vous
Donc. SI la corde est queique peu désaccordée, la fréquence de vm la détendre pour qu'elle soit bien accordée ?
3.8 L'effet Doppler 69

3.8 L'effet Doppler


U ne voiture de police est stationnée en bordure de l’autoroute, e t sa sirène de 1 0 0 0 Нл
fonctionne. Si vous êtes égalem ent stationné sur le bord de l’autoroute, vous entendrez
cette m êm e fréquence. P ar contre, s’il y a un m ouvem ent relatif entre vous et la voiture
de police, que voas vous en rapprochiez ou que vous vous en éloigniez, vous entendrez
une fréq u en ce d ifféren te. P ar ex em p le, si vous co n d u isez en vous rapprochunt de la
voilure de police à 120 km /h, vous entendrez une fréquence plus élevée (\ 097 H z. soit
une augm entation de 97 Hz). Si vous vous éloignez de la voiture d e police ég alem ent
à 120 k m /h , vous en ten d rez un e fré q u en c e plus basse (903 H z, so it un e d im in u tio n
de 97 Hz).
C es v aria tio n s de fréq u en ce reliées au m o u v e m en t re la tif so n t des ex em p les
de l’e ffet D oppler. Cet effet a été dém ontré (sans être com plètem ent vérifié) en 1842 par
le p h y sicien autrichien Johann C h ristian D oppler. Il a é té vérifié cx p criu icn talcm en t
en 1845 p ar B uys B allo t en H o lla n d e, « à l ’aid e d ’une lo c o m o tiv e tiran t un u a g o n
découvert sur lequel prenaient place plusieurs trom pettistes».
L ’effet D oppler s ’applique non seulem ent aux ondes sonores, m ais aussi aux ondes
électrom agnétiques, qui incluent les m icro-ondes, les ondes radio et la lum ière visible.
D ans ce ch ap itre, on ne traitera to u tefo is qu e des o n d es so n o res, en p ren an t com m e
ré fé re n tie l (sy stèm e de co tird o n n ées) le su p p o rt m atériel d an s leq u el ces o n d es sc
propagent, c’est-à-diie l’air. C ela signifie q u ’on m esurera la vites.se d ’une source S. émettant
des ondes sonores, et celle d 'u n détecteur D, recevant ces m êm es ondes, par rapport au
support m atériel des ondes sonores. (S au f ind icatio n contraire, l’air est im m obile par
rapport au sol (pas de vents), et les vitesses peuvent égalem ent être m esurées p ar rapport
au st)l.) O n supposera que 5 et D se déplacent soit en se rapprochant directem ent, soit en
s ’éloignant l’un de l ’autre, à des vitesses inférieurc.s à la vitesse du son.
.Si le détecteu r, la so u rce ou les d eu x se d ép lac en t, la fré q u en c e / émise e t la
f r é q u e n c e /' détectée sont reliées par

f = f: ( г effet Doppler, cas général), (3.47)

où V e st le m odule de la vitesse du son d an s l ’air, Vp est le m o d u le d e la vitesse du


d étec teu r m e su ré e p ar ra p p o rt à l’air, e t v's est le m o d u le la v itesse d e la so u rce par
rapport à l’air. L a règle suivante déterm ine les choix des signes.

lorstjiie le motivcnxîiit du détecteur s’effectue en s'approchant de la source que l’on suppose


immobile (ou que le mouvement de la source s’effectue en s’approchant du détecteur que
l’on suppose immobile), le signe devant le module de sa vitesse doit avrrir pour effet de faire
augmenter la fréquence détectée. Ixirsque le mouvement du détecteur s’effectue en s’éloignant
de la source que l’on .suppose immobile (ou que le mouvement de la source s’effectue
en s’éloignant du détecteur que l'on suppose iinmohile'). le signe devant le mtxliik
de sa vitesse doit avoii pour effet de faire diminuer la fréquence délectée.

Pour résum er ce point, on peut dire q u ’un rapprochement im plique une augmentation
de la fréquence, et q u ’un éloignement im plitjuc une diminution de la fréquence.
Voici quelques exem ples de l’application de celte règle. Si le détecteur se déplace
en s ’a p p ro c h an t de la .source q ue l'o n su p p o se im m o b ile, m êm e .si e lle est. d an s la
réalité, en m ouvem ent par rapport à l’ait, on utilise le signe positif dan.s le num érateur
de l’équation 3.47 de m anière à faire augm enter la fréquence. S ’il s’en éloigne, on utilise
te signe négatif dans le num érateur de manière à la faire tlimimier. peu im porte 1< m ouve­
m ent qu'effcx tu c la мчлгсс. S ’il c-st im m obile par rapport à l’air, on sub.stitue 0 à Vp.
On dém ontrera à présent les équations de l’e ffet D oppler dans les deux situations
qui suivent, ci l 'équation 3.47 dans une .situation q u ’on peut qualifier d e générale

1. Ixtrsquc le détecteur se déplace par rapport è Pair alo is q ue la source est im m obile,
le m ouvem ent change la fréquence à laquelle le détecteur intercepte les fronts d ’onde,
donc la fréquence de Ponde sonore.
2. L orsque la source se déplace par rapport à P air cl que le déreetcur est im m obile, le
m ouvem ent change la longueur d ’onde de Pontle sonore, donc la Ifétjuence détectée
(rappelez-vou.s q ue la fréquence e.st reliée à la longue.ui rPondc).
/0 Chapitre 3 Les ondes sonores

hgure3.19 l ne source sonore immobile


S émet des fi onts d’onde sphériques,
üislmils uiirc eux tl’uiic longueur
d'onde, qui se propagent à la v itesse
du «on n. Lin détecteur de son D,
représente par une oreille, se déplace
avec une viiessc v’ncn s’approchiml
de la source. Le déletueur capte
une plus haute Irécjtience ?» cause
de son mouvement relatif.

Détecteur en mouvement et source immobile


D ans la figure 3.19, un détecteur D (rcpré.scnlé par une oreille) se déplace à la vitesse i',,
I vers une source im m obile qui ém et des fronts d ’onde sphériques, de longueur d ’onde A
i et de fré q u e n c e / . se p ro p a g e a n t d an s l’air à la v itesse du son v. L es fio n ts d ’o n d e
•iJ a) représentés sont séparés par une distance d ’une longueur d 'o n d e. I.a fréquence captée
p ar le détecteur D est le taux auquel D intercepte les fronts d ’onde (ou les longueurs
T'’ d 'o n d e indiv iduelles). Si D était im m obile, ce taux se ra it/ m ais puisque D sc déplace
en allan t à la ren co n tre des fro n ts d ’onde, le taux d ’in tercep tio n est p lu s grand, et la
vl
fréquence d é te c té e /' est donc plus grande q u e /.
C onsidérez m aintenant la situation où D e.sl im m obile (figure 3.20). En un tem ps t,
les fronts d ’onde .sc déplacent vers la droite d ’une distance vr. Le nom bre de longueurs
b) «a
»t. (l’onde com pris dan.s cette distance ci est le nom bre de longueurs d ’o nde interceptées
par D en un tem ps t, cl ce nom bre est w /A . Le taux auquel D interv epte les lo n g u ew s
d ’onde, qui e.si la fréquence d é te c té e /p a r A esl
ci/A V
/ = , = r- (-3.48)
figure 3.20 Ix s fronts d'onde
de la figure 3.19, qu'on suppose Dans cette situalion où A est imm obile, il n 'y a pas d ’c lïc t D oppler; la fréquence détectée
plans, a) atteignent et b) dépas.sent par A est la fréquence ém ise par S.
un détecteur immobile /) : Exam inez m aintenant la situation où A .se déplace dans la direction oppo.scc à celle
ils sc déplacent d’une distance vt d es fronts d ’o n d e (fig u re 3.21). E n un tem p s t, les fro n ts d ’o nde se d ép lac en t d 'u n e
vers la droite en un temps t. distance vl vers la droite, com m e précédem m ent, m ais A sc déplace m aintenant d ’une
distance I'd/ vers la gauche. D onc, durant ce tem ps /, la distance parcourue p ar les fronts
d 'o n d e p ar rappo rt à D est vt + vrî. Le nom bre de lo n g u eu rs d 'o n d e co m p rises dans
cette distance relativ e i t f est le nom bre d e longueurs d ’o nde interceptées p ar D
en un tem ps t, et est (vt + v,^t)/k. Le taux auquel A intercepte les longueurs d ’onde dans
cette situation e.st la f ré q u e n c e /', donnée par

a) / '= ^ (3.49)
î r>
D ’après l ’équation 3.48, on a A = v/f. L ’équation 3.49 devient alors
vl -U v„i
*' f V'D V + Vj5
,r = (3.M))
^’t f
Note/, que. dans l’équation 3 . 5 0 ,/ ' doit être plus grande q u e / sau f si Vp 0 (détecteur
1.0 <H^-.
im m obile).
D e la meme façon, (vn peut trouver la fréquence détectée pai A lorsque D s ’éloigne
de la source. D ans cette situation, les fronts d 'o n d e parco u ren t une d istance vt — l'pf
- ;i - par rapport à A et e n un tem ps t. et / ' est donnée p ar
V - Vl,
fignn» .3 71 Les fronts d'onde a) atteignent / '= / (.3.51)
et b) dépassent le détecteur f>, qui
se déplace dan.s la direction opposée Dans l'éq u atio n 3 .5 1 ./ ' doit être plus petite q u e / sap f si Vf, O.
à lit leur. I !n un temps t. les fronts ü n peut résum er les équations 3.50 et 3.51 ainsi ;
d’onde se déplacent d'une distance rf
V ers la droite alors que D sc déplace V ± V|)
/ '= / (un détecteur en mouvenKnt. une source immobile 1. (3.52)
d'une distance Vf,/ vers la gauche.
3.8 L'effet Doppler 71

Source en mouvement et détecteur immobile


Suppo.sc/ m alm enant que le détecteur D est im m obile p ar rapport à l’air, et que la source
S .se déplace vers D à la vitesse 7’^ (figure 3.22). Le m ouvem ent de S m odifie la longueur
d ’onde des ondes sonores q u ’elle ém et cl, par le fait m êm e, la fréquence détectée par A
Pour observ er ce changem ent, on sait que T {— 1//) est le tem ps qui s’écoule entre
les ém issions de deux fronts d ’o nde successifs, 1 0 \ et l'O^- D urant 7, le front d ’onde
FOi parcourt une distance vT et la source parcourt une distance v ^ r A(irês un tem ps T,
le front d ’onde FO^ est ém is. Dans la direction du déplacem ent de S, la distance entre
FO^ et FO^. qui est la longueur d ’onde X' des ondes se propageant dans cctic tlircction.
est \'T - v^T. Si D détecte ces ondes, il détecte la f ré q u e n c e /' donnée par

vT - vsT v/f-vs/f
»7
= / V— (3.53)

N otez que / ' doit être plus grande q u e / sau f si vj = 0.


D ans la direction oppo.sée à celle prise par S, la longueur d ’onde À' des ondes est
v7 + Vij7. Si D détecte ces ondes, il détecte la f ré (|u e n c e /' donnée par

,r= / — — (3 54)
V + t's

I c i , / ' doit être plus petite q u e / . sau f si = 0.


O n peut résiunci les équations 3.53 et 3.54 ainsi :

/ '= / (une .source en mouvement, un détecteur immobile). (3.55)


V ± Vs

L’équation générale de l'effet Doppler


O n peut m a in ten an t d é riv e r l ’éq u a tio n g é n é ra le de l'e ffe t D o p p ler en r e m p la ç a n t/
d ans l ’éq u a tio n 3 .55 (la fré q u en c e a sso cié e au d é p lac em e n t d e la so u rce) p ar f
de l’équation 3.52 (la fréquence associée au déplacem ent du détecteur). Le résultat est
l’équatio n 3.47, ex p rim an t l'e ffe t D o p p ler lorsque la source S et le d étec teu r D sont
en m ouvem ent.
C ette équation générale s ’applique non seulem ent lorsque le détecteur et la source
sont tous les deux en mouvement par rapport à l’air, mais également dans les dcu.v situations
spécifiques que l’on vient d ’étudicr. D ans la situation où le détecteur est en m ouvem ent
alors que la source est im m obile, la substitution de Vç = 0 dans l’équati<Mi 3.47 donne
l’équation 3.52. q u ’on vient de trouver. D ans la situation où la source est en niouvem eni
alors que le détecteur est imm obile, la substitution de Vp * 0 dans l'équation 3.47 donne
l’équatitm 3.55, q u ’on vient égalem ent de trouver. D onc, l’équation 3.47 est l’équation
à retenir.

Rgure 3.22 I in détecteur D est immobile


et une source 5 sc déplace vers lui
à la vitesse Cj. Le front d'onde T-'O,
a été émis lorscjue la source était à 5,.
cl le front d’onde FO-,, lorsqu'elle était
à S-,. Au moment de la détection,
la source est à S. Le détecteur capte
une frétjuence plus élevée parce
que la source en mouvement,
en s’approchant de scs propres fronts
d’onde, émet une longueur d’onde
réduite X’ diuis la direction
de son déplacement
72 Chapitie 3 Les ondes sonores

La navigation des chauves-souris


Les chauves-souris règlent leur navigation et leur chasse en ém ettant des ultrasons et c r
détectant ensuite leurs réflexions. O n appelle ultrasons des ondes sonores qui ont de.»
fréquences qui excèdent la ffcquencc maximale, d 'environ 20 kHz, audible chez l'hum ain.
A in si, une ch au v e-so u ris ém et d es u ltraso n s à 83 kH z, so it bien au -d e là d e la lim ite
supérieure de 20 kH z de l'au d ib ilité hum aine.
A p iès avoir été ém is par les narines de la chauve-souris, le so n p eu t être réfléchi
(st)us form e d ’écho) p;u- un papillon de nuit et retourner aux oreilles d e la chauve souris.
L es m ouvem ents de la ch au v e-so u ris et du p ap illo n p a r rap p o rt à l ’a ir font v arie r de
quelques kilohertz la fiéquence entendue p ar la chauve-souris, par rapport à la fréquence
q u 'e lle a ém ise L a c h a u v e -so u ris trad u it au to m atiq u em en t c e tte d ilfé re n c c en une
vitesse relative entre elle et le papillon, e t peut ainsi .se d irig er droit su r lui.
C ertains papillons évitent d ’être capturés en s ’éloignant de l’endroit d 'o ù viennent
les ondes ultrasoniques. C e choix de trajectoire de vol réduit la différence de fréquence
entre les ém issions de la chauve-souris et celles q u 'e lle entend, et elle peut alors ne pas
rem arquer l’ccho. Le papillon peut aussi éviter la capture en produisant une ciépitalion
sonore afin de produire ses propre.s ultrasoas, ce qui « b ro u ille» le systèm e de détection
et peut confondre la chauve-souris. (C hose su rp ren an te, les papillons et les chauves-
souris font tout cela sans jam ais avoir étudié la physique )

.Souue Déiecteur
✓ vérifiez VOS CONNAISSANCES 6 : On a illustré ci-contre six
u) ---------• liiimotiile
sitiiaiions indiquant l’orientation de la vitesse d'une source sonore
Cl celle d’un détecteur, par rapport à l’air immobile. Dans chacune b) -«--------- • Iiiiiiiobilc
des situations, la fréquence détectée est elle plus élevée ou moins «1
élevée que la fréquence émise, ou est-il impossible de le déterminer d)
sans avoir plus d'information sur les vites.ses réelles ?
e)
0

Exemple 3.8
Une fusée est projetée à une vilc.sse de 242 m/s direc:iemem vers un 2. La fréquence du son émis par la source (le poteau) est égale
poteau immobile (par rapport à l’air immobile) et émet des ondes à/ ’. la fréquence du son intercepté et réfléchi par le poteau.
sonores n une fréquence/ = I 250 Hz.
On peut récrire l’équation 3.47, .sous forme de fré q u e n c e /' de la
a) Quelle fréquence/' est mesurée par un détecteur attaché au poteau '.’ source Cl de fréquence/ " détectée, ainsi ;
SOIUÎION: ü n peut trouver/' à l’aide de l’équation .3.47 relative à l'effet , V± rp
Doppler généra). Le concept tie est eelui-ci : puisque la source sonore r = .f V ± l’s
(3..S6)
(la fusée) se déplace daas l'air en s'approchant du détecteur immobile
ultai:hé au |Xiteau, on doit utiliser le signe devant qui aura pour effet On fait maintenant intervenir un troisième coatepUlé. Puistjue le
il'augmenter fréquence dn son. Donc, dans réc]uation .3.47, on utilise détecteur (fixé à la fusée) se déplace dans l’air en s ’approchant de la
le signe négatif dans le dénominateur. On .substitue ensuite 0 au source immobile, on doit utiliser le signe devani l’p qui a pour effet
module de lu vitcs.se du détecteur vj,. 242 m/s au module de la vitesse d'augmenter la fréquence du son. On utilise donc le signe positif dans
de la stHircc v,;, 34.3 m/s ,iu module de la vitesse du .son v (d’après le le nunicratciir de l’équation 3..36. Ensuite, on insère l'n = 242 m/s.
tableau I ) et 1 250 Hz à la frequence émise/ On trouve alors Vj = 0, V = 343 m/s e t / ' = 4 245 H/. On trouve ainsi

.343 m / s ± 0 .343 m /s -1- 242 m /s


/ ’ = / ^ - = (1250 Hz.) / " = (4 245 Hz)
V --fc l's 343 m /s 242 m /s 343 m /s - (I
:r- 4 245 Hz 4.25 kllz. (réponse) - 7 240 H / 7.24 kHz, (réponse)

ce iiiii est eftectivement une fréquence plus élevée que la fràitiem e ce qui est effectivement une fréquence plus élevée que la fréquciKC
émise. du son rélléchi pat le poteau.

h) {.'ortnincs ondes sonores qui altcignciil le poteau «»ni réfléchies


vers la fiisé.c sous fni iiied'échos Quelle fré q u e n c e u n détecteur fixé ✓ VÉRIFIEZ VOS CONNAISSANCES 7 ! Dans l’exemple précédent,
sur Infusée détecterait il dan.s l’écho'? si un vent met l'air en mouvement vers le poteau à une viics.se
ayant un module de 20 m/s, a) quelle valeur devra-t-on utiliser
501UTION On utilise d'abord deux contoplsdes.
p<iiir décrire le module de la vitesse de la source vj dans la solution
1. Le poteau est maintenant la source du son (étant la source de de la partie a)'.’ b) Quelle valeur devra-t-on utiii.seï pour décrire
l’écho), et la fu.sée est maimenani le détecteur (puisqu’elle détecte le module de la vitesse du détecteur dans la solution
l’écho). de la parue b)
3.9 l<es vitesses supersoniques et les ondes de choc 73

3.9 Les vitesses supersoniques


et les ondes de choc
Si une source est en m ouvem ent vers un cléiecieur im m obile à une vitesse ég ale à la
vitesse du son. donc si = v; les équations 3.47 et 3.55 perm ettent de prédire qu e la
fréquence d étec tée/ ' sera infinim ent gran d e C ela signifie que la source se déplace si
Vite c|u'elle a la m ême vitesse que scs propres fronts d 'o n d e sphénques. com m e l'illustre
la figure 3.23 a). Q u ’arn v e-t-il lorsque la vitesse de la source excède la vitesse du son

a)

figwe 3.23 a) Une source sonoie S


se déplace à la vitesse T's. égale
à la vitesse du son, donc aussi rapide Dans le cas de telles vitesses su p erso n iq u es, les équations 3.47 et 3.55 ne s’appliquent
que les fronts d ’onde qu'elle génère. p lu s; autrem ent, elles donnent une valeur d e / ' négative, ce q ui physiquem ent n ’a pas
b) Une source S se déplace à la vitesse de sens. La figure 3.23 b) illustre les fronts d ’onde sphériques issu.s de la source alors
Vj. supérieure à la vitesse du son, q u 'elle se trouvait à différentes positions. D ans cctic illustration, le rayon de tout front
donc plus rapidemeni que ses propres d ’on d e est vt. où v c.st le m odule de la vitesse du so n et / est le tem p s éetu ilé d epuis
fronts d’onde. À la position S ,, l’ém ission de ce front d ’onde par la source. N otez que tous les (ront.s d'o n d e sont d em crc
la source génère le front d'onde A , la source S (vj > v) et sc regroupent en form ant une espèce d ’enveloppe en form e de V,
et à la position elle génère le from com m e le m ontre le dessin en deux dim ensions de la figure 3 23 b). En réalité, les fronts
d’onde FOh- Tous le.s fronts d’onde d ’o n d e se dép lo ien t en tro is d im en sio n s, et leu r reg ro u p em e n t fo rm e un cô n e q u 'o n
sphériques se propagent à la vitesse
nom m e cône de Much On dit i|u 'il y a une o n d e d e choc à la surface de ce cône parce
du son dont le module est y
que le regrou p em en t des fronts d 'o n d e cau se une augm entation et une ch u te rapides
et SC regroupent à la surface
de la pression de l’air au m om ent ou sa surface traverse un jxiim quelconi|ue D ans la
d'un cène nommé cône de Mach.
figure 3.23 b), on voit que le demi angle b du cône, nom m é demi angle du cône de Much,
fonnam une onde de choc.
est donné par
La surface du cône a un demi angle f)
cl est tangente à tous les fronUs d’onde. l’i !•
sin ft = = — (le demi-angle du cône de Machi (3.57)
vst l'S
Le rapport i-j/r se nom m e le nombre de Much. Si on vous dit q u ’un avion a volé à
la lite sse M ic h 2,3. cela signifie que sa vitesse correspondait à 2.3 fois la vitesse du son
dans l’air où il volait. L’onde de choc générée par un avion (figure 3.24) ou des projectiles
su|X 'rsoniques produit un son nom m é hang supersonique. causé par une augm entation
subite de la prc.ssion de l’air, suivie d ’une dim inution de pression aussi subite et d ’un
retour à la norm ale. U ne partie du son entendu lo rsq u ’un projectile est tiré est le bang
supersonique produit par le projectile Un bang sui'tersoniquc peut aussi être entendu en
faisant clatjucr rapidem ent un long fouet : quand le fouet atteint la fin de son mouvement,
son ex tré m ité se d ép lace plus \ ite qu e le son e t p ro d u it un p etit h ang su p erso n iq u e,
le claquement du fouet.

Figure 3 24 Ondes de c Ihk; produites par les ailes d'un avion miliiaire à réaction LA 18.
F.llcs sont rendues visibles par la diminuùi'n subite de 1« pression de l’air, qui cause
la coiulensation de,s molécules d'eau. priMluisant ainsi un brouillard
74 Chapitre 3 Les ondes sonores

e•
«Ê V JS IO N E T « i S ü ' £5'
сг{Г(Г -

L c!>vn d ts sunorta Les oncles sonores sont des ondes mécaniques L’intensité à une di.stance r d’une source sonore poïKiuellc ayant un-
longitiKliiiales qui peuvent se propager clans les solides, les lic]uides et pui.ssance Kj est donnée par
les g a/ l.e mcKlulc de la vitesse v d’une onde sonc;rc dans un milieu
ayant un module de compressibilité D et une masse volumique p est fi (3.28
/ =
4тгг-
V — Л/ — ^tc mcKlulc de ia 4'hcsse du .son). ( 3 .3 )
V P Le niveau sonore en décibels l e niveau sonore fi en dédhcL
Dans l'ail h 20 'T . le nuKliile de la vitesse du son est de 343 m/s. (dB) SCcléfinit ainsi :
Une oncle sonore cause un déplacement lon^tudinal ,vd'un élément
de masse dans le milieu; te dcplaccment est donné par l'équation fi = (lO d B )lo g —. (3.29;
7(1
s Sm cosiAcr —tuf). (-L13)
où /„ (= 10"'- W/nv, ce qui est le seuil d ’audibilité) est le niveau
où v,„ CM l'iim piitude de dépincem rnt (l.a grandeur maximale du d'intensité de référence auquel toutes les intcnsité.s sont comparées.
drplaienieni) p.ar r.ipport à l’équilibre, k — 2/t / à ci to = 2nf. K Pour chaque/ia ieiir de lOd'augnx'niuùoiiclansrintensité (A = f y 10).
ei / Otant la longueur d'onde et la fréquence de l'oncle sonore. L'oncle on ajoute 10 dB au niveau .sonore (fi> = )S, + 10).
soiioïc cause é.ealcmeni une variation de pression Sp du milieu, par
rapport il la pression d'équilibre ; fie.v ondes stationnaires d a m un tuyau Ucs ondes sonores
stationnaires peuvent être prixluiics dans un tuyau. Un tuyau ouvert
Л/» * Д/>„ xinUv —eui), (3.14) à scs deux exircniifés résonnera aux fréquences
ou I am plitude de pression c.st
oil n = 1,2, 3.... (З.ЗЧ)
(3.15) K 2L '
■^Pm = (»'/’*>)*пг
où Vest le mcxlulc de la vitesse du son dans l'air du tuyau. Dans un
Ij'ilU e rftren c e L inierference de deux ondes sonores ayant des tuyau dont une exircmité est ouverte et l’autre fermée, les fréquetice.s
longueurs d'onde identiques qui iiaverscni le meme point dépend de de ré.sonaiiee sont
la différence de plusse é entre elle.s à ce (Winl .Si les ondes sonores
ont été émise.s en phase et se propagent approximativcmcni dans la , V (2 л — I )v
/2 „ -, = ^ -• w « /i 1 .2 .З .... (3 4 1 )
même direction, é est donné par Лгп I 4L

AI
2n, (3.21) Les battements l'ies battements se prodtii.sent lorsque deux ondes
ayant des fréquences légèrement cliffcrcnte.s,/i et f i , sont détectées
cxi Ai, est la différence entre la longueur de leurs parcours ( la dit férence siniiillanémcnt. La frcc]uenee des battements est
dans les distances parcourues par les ondes pour atteindre le point
/ы = yu - / i l t3.46)
commun). L'iie interférence comtruaive se prcxluil lorsque d>esi un
multiple entier de 2tt.
L ’e ffe t D oppler L'effet D oppler est un changement observé
<}) = ni(2rt). où ni = 0. r L ± 2 .... (3.22) dans la fréquence d ’une onde lorsque la source et/ou le détecteur
se déplacent par rapport au milieu dans lequel l'onde sc propage
et. de layon éc)ui valente. lorsque A/, est reliée à la longueur d'onde à par
(l'air, par exemple). Pour le .son. la fréquence observée/' rclativcnxtnl
A/- = mA.. où m = 0. ± L i 2 . ... (3.23) à la fréquence de ia source/est donnée par

Une interférence destruetive se produit lorsque é’ est un multiple V ± V|1


/ '= / (l'eflcl Doppler, cas géréral). (3.47)
impair de rr. I- ± l’s

à) — t2m + I ),T, iHi ni = 0. I l . ± 2. ... (3.24) où v„ est le module de la vitesse du détecteur pai rap|x>rl au milieu,
l's est le nюdulc de la vitesse de la source par rapport au milieu et v
et, (le façon éc|iiivalcnte, lorsque AL est reliée à À par
est le module de la vitesse du son dans le milieu, I.x signe devant Гр
AI, = (m t-î)À . oil w = 0, ± 1, l 2 . ... (3.25) (l's) est choisi de manière à faire augnienter f lorsque le momeraeni
du détecteur (de la source) s’effectue en s’approchant de la source
(du détecteur) que l’on suppose immobile, et à faire diminuer f '
L'intensité snnnre I 'intensité / d'nne onde sonore sur une sur­
lorsque le mouvement du détecteur (de la source) s’effectue en
face est le taux moyen par unité de surface aucjiiel l'énergie de l'oncle
s'éloignant de la source (du détecteur) que l’on suppvisc immobile
passe à travers ia surface.
P L ’onde de choc Si la vitesse d'une source par rapport au milieu
/ (3.26)
À' excède la vitesse du son dans ce milieu, l'équation 3.47 de l’effet
où A est l'iiirr de la surface qui intercepte le son et / ’est le taux d’énergie Doppler ne .s’applique plus. Dans un tel cas, des ondes de choc se
(la puissance) de l’onde sonore passant à travers A. L'intensité / est produisent. Ix demi-angle Wdp cône de Mach est donné par
reliée à l'amplitude de déplacement .v„ de l'onde sonore par

(3.27) sin V = (le demi-anplc ducène de Mach). (3..^7)


v.s
Ouestlons 75

P U C S T IO N S 4 s iJ ;u * ..û

1. La figure 3.25 présente les par- » — :------------------------ On .suppose que l’inccrscction de cette droite [lerpendiculairc et de
eours de deux impulsions sonores Paixoun. 1-' __________ ^ la ligne allant de 5, à S-, est le point P,, a) Les ondes qui atteignent P3
émises en même temps et devant — 7-}---- ---------- j— sont-elles en phase, .sont-elles en opixisition de phase ou ont-elles
franchir la même dislaiKe dans l'air Parrmirs une différence de phase intermédiaire b) Quelle est la réponse si on
La seule différence entre les deux augmente de 1.7/. lu disuncc entre les deux sources ?
Rgt)re3.25 Question I
parcours est qu'il y a une région 7. Dans un tuyau, une onde stationnaire a cinq iKcuds et cinq ventres
d 'a ir chaud (faible masse volumique) dans le parcours 2. Quelle a) Combien d’extrémités caiveiies le tuyau a-t-il ? b) Quel est k- nombic
impulsion franchira le plus rapidement cette distance ? harmonique de l'uiidc résultante?
2. Une onde sonore qui a une longueur d'onde л et une amplitude 8. On produit le sixième harmonique dans un tuyau, a) t'om bien
de déplacement s'mentre dans un passage (un tuyau, l'ouvenure d'une d'extrémités ouvertes le niyau a-i-il (il en a au moins une)? b) Y a-t-il
oreille, etc.). Lorsqu'un petit dispositif installé dans le passage détecte un nieud. un ventre ou un état intermediaire au milieu du tuyau ?
celte onde, il cinet une autre onde sonore (que l'on nomme aniisoii) 9. a) Pendant la répétition d’un orclicstie, la respiration des musiciens
capable d ’annuler la première onde, de .sorte que rien n'est entendu fait augmenter la température de l'air (et diminuer la masse volumique
à l’extrémité éloignée du passage. Pour qu'une telle annulation de l'a'u) ilans les inslmnicnu à vent. Les frequenees de tésonanec de
soit possible, quelles doivent être a) la direction de propagation, ces instruments augmentent-elles ou tliminiient-elles’’ ht l .oiviue la
b) la longueur d'onde etc) l'amplitude de déplacement de la deuxième coulisse d'un trombone est pous.sée vers l’avant, les fréquences de
onde ? d) Quelle doit cire la différence de pliasc entre les deux ondes ? rcsoniuice de rinslnim ent augmentent-elles ou diminuent-elles
(On utilise des dispositifs semblables pour éliminer les sons indésirables
10. Voici quatre des six fréijiienees harmoni(|iies inférieures à I 000 Н/.
dans un milieu bruyant.)
que l’on trouve dans un tuyau; 3(Ю Ilz. 600 Ha. 750 Hz et 900 Hz.
3. La figure 3.26 présente deux Quelles sont les deux frequences manquantes
sources ponctuelles. S, cl ,V,, qui
11. Le hiyau A a une longueur L et une extrémité ouverte Le tuyau «
sont en phase et qui émettent
a une longueur 2L et deux extrémités oiivcitcs. Quels hannnniques
des ondes sonores identiques dont
du tuyau B présentent une fréquence corirsiiontlaiU à une fréquence
la longueur d'onde est de 2,0 m. Figure3.26 Question 3
harmonique du tuyau A ?
Exprimée en fonction de la longueur d'onde, quelle est la différerKc
12. La figure 3.29 nioiitrc une corde tendue de longueur L. de même
de parcours entre les ondes qui atteignent le point P a) si Ц = 38 m
que les tuyaux n, h, c et d, doni les longueurs respectives sont L, 2L,
et Lj = 34 m, et b) si L, = .30 m et = 36 m '? c) Si on suppo.se que
L/2 et LI2. La tension dans la corde est ajustée jusqu’il ce que le
la distance entre les sources est beaucoup plus petite que L, et L?,
module de la vitesse des ondes qui la pareoiireiil soit égal au niixlule
quel type d ’interférence se produit à P dans les situations décrites
de la vitesse des ondes sonores duns l'air. On produit eicsuiic le пюйе
en a) et en b)?
fondamental d ’oscillation dans la corde Dans qucl(s) tuyau(xl le son
4. Dans la figure 3.27. des ondes
produit par la corde produira-t-il une résonance, ei quel miKle
sonores de longueur d'onde À sont Panneau Pan ours 2 d’oseillalion y ohscrvera-l-on ?
émises par une source ponctuelle S.
En suivant le parcours 1, elles sc

hH
^ ^ P am m rs 1
dirigent direcicineni vers un détec­
teur /) ; en suivant le parcours 2, Figure 3.27 Question 4
elles sont réfléchies par un panneau avant d ’atteindre le détecteur
Le panneau est d’abord très rapproché du parcours 1. et les ondes qui
atteignent D en suivant les deux parcours sont presque en phase. 'Lu..
Puis, on éloigne le panneau du parcours I, eomme illustré, jusqu'à
ce que les ondes soient complètement déphasées en atteignant D. Figure 3.29 Q u e s t io n L

Quelle est alors la différence de parcours A/ = ~ 7 1 entre les


13 I In de vos amis prend place, successivement, dans trois manèges
ondes du parcours 1 ci celles du parcours 2 '?
rapides où il effectue un mouvement circulaire uniforme, tout en
5. La figure 3.28 présente deux tenant une source .sonore qui émet de façon isotrope à une certaine
sources pontUiclIes, .5, et .S',. Les frequence. Vous restez éloigné de chacun de< manège.s. l a fréquence
sources sont en phase, émettent des que vous entendez, provenant de chacun de.s manèges où prend place
ondes sonores identiques ayant une votre ami. varie en fonctiiMi de la niiaiion des iiiaiicgc.s. Lc.s variations
longueur d’onde à . et le point P sc .V,
de fréquence des trois manèges sont représentées par les trois courbes
trouve à égale distance des sources. de la figure 3.30. Clas.sez les courbes en ordre décroissant scion
Rgure 3.28 Question 5
Par la suite, .V, est éloignée de P
a) le module de la vitesse i de la source sonore, b) les vite.vscs
d'une distance égale à >74. Au point P, les ondes sont-elles alors en angulaires o>des manèges et ci les rayons rd es niiutègcs
phase, sont-elles en opposition de phase ou ont-elles une différence
de phase intermédiaire a) si A, est déplacée vers le point P d'une
;
distance égale a A/4 et b) si .S'i est éloignée du point P d’une distance
égale à 3X/4 ?
4 Dans la figure 3.9 a) de l'exemple 3.3. les ondes qui atteignent
le point P] sur la droite passant perpentlieulairement au centre du
segment de droite reliant les deux sources sont en phase ; ce qui
revient à dire que les ondes piovenant de .V, et de S2 tendent toujours
à déplacer un élément d’air dans la même direction au point P,. Figure 3.30 (Question 13
]k Chapitre 3 Les ondes sonores

EXEf?CIC€S E T PR O etÆ M ES

éP Dans un certain métal, le module de la vitesse du son est V.


www I.a süluiimi se trouve sur le site Web. à Padresse ci-dessous ;
On donne un coup à une extrémité d'un tuyau fait de ce métal et ayant
www.dlcnttgmvi'hill.cii/physique
une longueur /,. Une personne à l’autre extrémité entend deux sons,
un de Ponde qui se propage dans le luyau ci l’autre de Ponde qui sc
^aiii indication contraire, utilisez ces valeurs dans les exercicca et propage dans Pair, a) Si le niCKlule de ki vitesse du son dans l’aii est i.
problèmes ■ quel intervalle de temps / s ’écoule entre l’arrivée des deux sons?
b) Supposez que i = 1.(X) s et que ce métal est de P acier 1 rouvez la
inrKlulc de lu vitesse du son dans l'air = 343 il)/s,
longueur L.
J9. On lai.ssc tomber une pierre dans un puits. On entend le son de
et masse volumique de l’air -= 1,21 kg/m \
la pierre qui frappe Peau 3.00 s plus tard. Quelle esi la profondeur
du puits?
SECTION 3.2 La viiesse du son
SECTION 3 3 Les ondes sonores progressives
It Concevez une rèj;le vous permettani de calculer la distance en
kilom ètres d’un éclair en comptoni les scctMKles écoulées entre l’instant 8E. La gamme de fréquences audibles chez l'humain qui n’éprouve pas
où vous voyez; l’éclair cl l’inMani où vous entendez le tonnerre. de problème d’audition va d ’environ 20 Hz à 20 kHz. Quelle est la
Sup|)nM.v que le sim voyage en ligne droite. gamine de longueurs d’onde associée à ces ondes stmores audibles?
2E Vous assistez ù un collect i cn plein air cl vous vous trouvez à .VX) m 9E. Des ultrasons ayant une fréquence de 4,50 MHz .sont utilisés pour
Je.s haut-patlcurs. I x concert est également diffusé en direct par examiner les tumeurs dans les ti.ssus humains, u) Quelle est la UHigiieur
saiellite (ù la vitesse de la lumière, soit à 3,00 x K)'' ni/s). Un audi­ d'onde de ces ondes sonores dans Pair? b) Si le module de la viles.se
teur capte rém ission du coiiccri à 5 000 km de distance. Oui est le du son est de 1 S(X) m/s dans les tissus humains, quelle cn est la
prciiucr à entendre lu mu.siquc, vous Ou l'auditeur éloigné ? Quel est longueur d’onde?
le ilclai entre les deux écoutes? lOP a) Une source d'oscillations est attachée à un lies long ressort
3L A un iititlch de ftxilball au stwlc Percival-Molson de l’L'nivcrsité Cl lui transmet une onde longitudinale sinusoïdale progressive.
McOill, deux spectateurs voient, et entendent un in.stant plus tard, La soua^e a une fréquence de 25 Hz. et il y a toujours une distance de
le h.allon botté sur le terrain. Le délai entre le.s deux perceptions 24 cm entre les points successifs d’expansion maximale dans le
(visuelle cl auditive i est de 0.23 s pour un des spectateurs, et de 0,12 s ressort. Trouvez le module de la vitesse de l’onde, b) Écrivez l’équation
pour l’autre. Les lignes de vue des deux spectateurs se croisent à un de Ponde si la grandeur maximale du déplaceineni longitudinal d'un
angle de a) À quelle distance chacun des spectateurs se irouvc-l-il élément de ressort est de 0,30 cm cl que Ponde sc propage daas la
du joueur’.' b> Quelle dislaiiee y a-t-il cnlic les deux .spectateurs ? direction négative de Paxe des x. Mettez .r = 0 à la source, et suppose/
4E. L'ne colonne de soldats qui marchent à un rythme de 120 pas que le déplacement est nul à ect endroit à t 0.
à la minute garde la cadence cn suivant le battement d ’un tambour IIP I.a variation de pression produite par une onde sonore qui .sc
à r.'ivant de la colonne. On observe que les soldats à l’arrière de la propage dans un certain milieu est donnée par l’équation
colonne posent le pied gauche lorsque le batteur pose le pied droit.
Quelle est 1« longueur approximative de la colonne ? Sp — (I..50 Pa) sin{(0.9(X) m"'law — (315 s " ') 7t/J.
SP. Des tremblements de terre génèrent des ondes sonores à l’intcneuT Trouvez al Pamplitude de pression, bl la fréquence, c) la longueur
de la Terre. Contrairement à un gaz, la Terre peut être parcourue à d’onde et d) le motlule de la vitesse de Ponde.
la lois par des ontles sonores transversales (S) et longitudinales (P)
Généralement, le module de la vitesse des ondes S est d’environ
SECTION 3.4 L'inlerférenie des ondes sonores
4 , kni/s, et celui tics oiKles Post d’environ K,0 km/s. Un sismographe
enregistre les ondes P et S causées par un tremblement de terre. I2P. Deux sources ponctuelles séparées par une distance D = 2 .0 /
Les premières ondes P arrivent ,3.0 mm avant les premières ondes S émettcnl des ondes sonores d ’amplitude et de longueur d'onde /
(figure .3.31 ). Si on suppose que le.s ondes sc propagent en ligne identiques. l.es sources sont en phase al Combien y a-t-il de points
droite, à quelle distance du sismographe le tremblement de terre où le son est maximal (présentuni une interférence constructive)
SC produit-il? www le long d’un grand cercle entourant les sources et qui est centré par
rapport au point mitoyen entre les .stiurec.s ? b) Combien y a-t-il de
points oit le son est minimal (présentant une intei fércnce destructive)
j= le long de ce grand cercle entourant les sources ?
£- O n d es P O ndes S
13P Dans la figure 3.32. deux haut-
Îc S^ parleurs distants de 2,00 ni sont cn
phase. Supposez que le son prove­
- S<) / w m / nant des haui-pai leurs a approxi­ 2.00 m Haut-j>iirl«*«rc
s -a
mativement la même amplitude à
Pcndroii où se trouve un auditeur, AiidiU'iii
^=1
B qui est en face d ’un des haui- K
E — 3,7.^ n i -----
parlcurs, ù 3,75 m. a) À quelles
fréquences audibles (entre 20 Hz Figure 3 32 Problème 13
2 H I et 20 kHz) Paiidiicur eiiicnd-il un
l'eiiip» (mill)
son minimal ’ h) A quelles frequen­
figure 3.31 Problème ^ ces te son est-il maximal ? www
exercices et problèmes 77

I4P. Deux ondes sonores provenant de deux sources differentes d ’une source de 50 000 W. en supposant que les fronts d'onde sont
et ayant la même fréquence de 540 H i se propagent dans la meme sphériques.
direction b 330 m/s. Les sources sont en phase. Quelle est la 26P. Trouvez les rappons (du plus grand sur le plus petit) a) entre les
différence de phase entre les ondes à ün point situé à 4.40 ni d’une intensités, b) entre les amplitudes de pression et c) entre les amplitudes
source et 4 4,00 m de l'autre ? de déplacement, générés par deux sons dont les niveaux sonores
I5P. Deux haut-parieurs sont distancés de 3.35 m sur une scène exté­ présentent une différence de 37 dB.
rieure. L'n auditeur se trouve à 18,3 m de l'un et à iy,5 m de l'autre. 27P. Une onde sonore se propage uniformément dans toutes les
Pendant les tests de son, un générateur de signal alimente les deux directions b partir d’une source ponctuelle, a) Démontrez l’expression
haut-parleurs en phase avec la même amplitude ei la même frequence. suivante du déplacement x du milieu dans lequel l'onde se propage
La fréquence du signal se situe dans la plage audible (entre 20 Hz et en fonction de la distance r de la source :
20 kHz), a) Quelles sont les trois plus basses fréquences auxquelles
h
l'auditeur entendra un son minimal ? b) Quelles sont les trois plus basses Í = - sin (kr — V/),
fréquences auxquelles l’auditeur entendra un son maximal ? www r
16P. Dans la figure 3.33, un son d'une Uingueur d’onde de 40.0 cm se où h est une constante. Analysez la situation en considéram, plus
propage vers la droite à partir d'une source, dans un tube composé particulièrement, la direction de propagation cl l’intensité de l’ondc.
d’une section droite et d’un demi-cercle. L'ne partie de l’onde sonore b) Quelle est la dimension de la constante h'* www
se propage dans le demi-cercle et rejoint ensuite le reste de l’onde,
28P. Une source ponctuelle isotrope émet un son de 30.0 W. Un petit
qui se propage dans la section droite. Cette superposition des ondes
microphone intercepte le son sur une surface perpendiculaire à la
sonores effectuant différents parcours produit de rinicrférencc Quel
direction tic propagation de l’ondc de 0.750 cm-, à 2(X) m de la
est le plus petit rayon r qui produira un son minimal au détecteur?
source. Calculez a) l’intensité du son à cet endroit et b) la puissance
sonore interceptée par 1c microphone.
29P' La figure 3 34 montre un interféromètre aciHisliquc rempli d’air,
Source üctccte.ur
qui sert b démontrer l’interférence des ondes sonores. S est une .source
sonore constituée d’un diaphragme oscilldni ; D est un détecteur de son
Rgure 3.33 Problème 16 (une oreille ou un microphone, par exemple). Le irarcours .V/î/7 a une
longueur variable, alors que le parcours SAP est constant. À D,
SFCTION 3.5 L'intensité sonore et le niveau sonore l’onde sonore qui suit le parcours SBD interfere avec celle qui suit le
1IL Une source isotrope émet des ondes .sonores. À 2.50 m de la source, parcours SAP Lors d’une démonstration, l'intensité sonore à /7 a une
l'intensité des onde.s est de 1.91 x 10*'' VV/m^. En supposant que valeur minimale de 100 unités à une position donnée de la section
l’énergie des ondes est conservée, déterminez la puis.sance de la source. coulissante, et celte valeur augmente continuellement jusqu ,4
une valeur maximale de 900 unités lorsque la section est déplacée
18E. Une source ponciuellc isotrope de 1.0 W émet des ondes sonores.
de 1,65 cm. Trouvez a) la fréquence du son émis par la source et
En supptrsant que l’énergie des ondes est conservée, trouvez l'intensité
b) le rapport entre ram phtude de déplacement de l’oiulc du parcours
a) à 1.0 m de la source et b) ,3 2.5 m de la source.
5AD à P et celle de l’onde du parcours S’Ftn. c) Comiiicnt cc.s mules
19F Une onde sonore a une fréquence de 300 Hz et une intensité de peuvent-elles avoir des amplitudes de déplacement différenies si elle.s
I,(K) /iW/m^. Quelle est l'amplinKle des oscillations de Pair causées sont émises par la même source ?
par celle onde ?
20F. Deux sons présentent une différence de niveau sonore de l,(X)dH.
(Jucl est le rapport de la plus grande .sur la plus petite intensité ?
21F. On augmente de 30 dB le niveau sonore d ’une source. Par
quel facteur a) son intensité sonore et b) son amplitude de pression
sont-elles augmentées ?
22L La source d’une onde sonore a une puis.sance de 1.00 /xW. • I)
S’il s’agit d’une source ponctuelle, a) quelle est l’intensité de l'onde
Figure 3.34 Problème 29
à 3,(X) m de distance ? b) Quel est le niveau sonore en décibels à celte
distance ?
SECIION 3 6 Les ondes slolionnoites dons les (donnes d'oir
23F. a) Si deux ondes sonores, l’une sc propageant dans l'air et l'autre
dans l’eau, ont la même intensité, quel est le rapport de raniplilude de 30F. l'n e cordc de violon, d'une longueur de 15,0 cm et fixée à scs
pression de l’onde .se déplaçant dans l’eau sur celle se propageant dans deux extrémités, oscille à son mrxle fondamental I es ondes sur
Pair ? Les niasses volumiques de Peau et de Pair, à une température la corde ont une vitesse ayant un module de 250 m/s. et le son a
de 20 ®C, .sont de 0,998 v 10' kg/ni* et 1,21 kg/m’ respectivement une vitesse dans l'air dont le module est de 348 tii/s. Quellc.s .s<ait
b) Si les amplitudes de pression étaient égales, quel serait le rapport a) la fréquence et b) la longueur d'onde de l'onde sonore émise?
(cau/air) entre les intensités des ondes ? 31F. Le tuyau la (A) d'un orgue, dont les deux cxtréniitcs sont ouvertes,
24P Ln train de marchandises bruyant circulant sur une voie ferrée a une fréquenr e fondamentale de 3i)0 Hz. Le troisième harmonique
droite émet une onde sonore dont les fronts d'onde sont cylindriques. du tuyau si (S), alors qu'une seule de ses extrémités est mivcrte. a hi
On suppose que l'absorption île l’énergie sonore par Pair est négligeable. même fré<]ucnce que le deuxième harmonique du tuyau A. Quelle est
Comment l’amplitude de Ponde varie t-cllc en fonction de la dis­ la longueur a) du tuyau A ' b) du tuytm B"'
tance r par rapport au train mesurée perpendiculairement à cclui-ci ? 32£. Un tube de verre vertical a une longueur de 1,(K) m et .son ntvemi
25P a) Démontrez que l’intensité I d ’une onde est le priKluit de d'eau peut être ajusté à n’importe quelle hauteur. Un diapason
l'énergie de l'oiKle par unité de volume, u, avec le rrwxlule de sa vitesse, v. vibrant à 686 Ilx est irmi diicciemcnt au dessus de l'ouverture du
h) [.es ondes radio ,sc propagent à une vitesse ayant un module tube afin de produire une onde sonore stationnaire dans la partie du
de 3 IK) X 10* m/s. Déterminez « pour une omlc radio située à 480 km tube <iiii est remplie d’air. (C'eut portion du iuIk , remplie d’air, agit
78 Chopitie 3 Les ondes sonores

tomnic im lutic donl une extrémiié scr:iii ouverte cl l'autre fermée.) il y a un n au d ou un ventre de déplacement '.' b) Pur analogie avec ui
A qiiellue hauteurs du niveau d'eau y a-t il une rés<.)iiance ? tuyau dont une des exiiémiics est ouverte, démontre/, que la péritidi
33L a) Trouvez le module de 1я viiesxe des ondes dans une cortlc de pulsation T est donnée par
de violon dont la ina.sse ем de XOO mg et la longueur de 22.0 cm,
SI la fréquence fondamentale est de 020 Hz. b) Quelle c.st la tension
i
dans b corde’' Au mode fondamental, quelle e.st la longueur d'onde
C) des ondes dans la cordc et ü) des ondes sonores ém ises par où R est le rayon de l'étoile à Téquilibrc et v est le module de I:
b corde ? www vites.se moyenne du .son dans la matière de l’étoile c) I .es étoiles de fypt
3^P Line conte tie vitilon a une longueur de Я0 cm entre ses extrémité« naine hlant'lie sont composée.s d'une matière dont le module de coin
Itxes et une masse do 2.0 g. I.a corde « libre •>(sans doigt appliqué) émet preksibilité est 1.33 x Itf - pa et la mas.se vxilumiquc de 1.0 x 10“' kg/ni'
la note //] (440 Hz), n) Pour jouer la note do (52.^ Ilzi. où doit-on Elles ont un rayon correspondant à 9.0 x 10 ^ fois k rayon du Soleil
applujiier un doigt .sur la tô t de ? b) Quel est le rapport entre la longueur Quelle est la période de pulsation approximative d’iirc naine blanche ';
d'onde requise dans les ondes de la corde pour jouer un la sur celle 4CP. Le tuyau A. ouvert aux deux extrémités et d'une longueur tk
rcqiii.se ptiur jouer un du ^ c) Que! est le rapport eiilte la longueur 1.2 m. oscille ù sa iroi.sième plus ba.ssc fréquence harmonique U est
d'onde tie l'onde sonore du la sur celle du Jol rempli d’air, où le module de la vitesse du son est de 343 m/s
35Г. Ouri.s la rigutc 3-3.1. la stiume Vest un petit haut-parleur alimenté Le tuyau B. fermé à une seule extrémité, oscille à sa deuxième
par un trscillateur et un amplificateur audio, et dont la fréquente ne plus basse iiéqucncc harmonique. Les tuyaux A et B oscillent dan.s
peut être ajustée qu'cmrc I IX)0 Hz et 2 000 Hz. Le tube D. fabriqué ces modes à la même fréquence, a) Si on pose un a.xc des v à l’in­
à partir d'une feuille luélalliquc. agit coiiniic un détecteur. Il a une térieur du tuyau A, avec x "= 0 à une extrémité, où les nreuds de
longueur de 45.7 cm et est ouvert uiix deux e.vtrémirés a) Si le module déplacement sont-ils situés sur l'axe ’’ bl Quelle est la longueur du
de lit vitesse du sntt rlaiK fairest tie .Ч4 m/s à la température ambiante, tuyau fl ■’c I Quelle est la phis basse ftéqucnce harmonique du tuyau A ?
à quelles frequences y aura-t-il une résonance dems к tube lorsqu'on fera 41P. Une corde de violon d'une longueur de .30.0 cm cl d'une masse
varier de I ÜOO Hz à 2000 Hz la fréquente émise par le hiiui-parleiir'.’ linéique de 0.650 g/m est platée près d'un haut-parleur alimenté par
b) r« itcs uii entquis (ilaiis le .style de la figure .3 14 b) de Tonde un oseilluicur audio de fréquence variable. Un constate que la corde
MalioiiiMlre correspondanl a chaque fréquence de réstHwiiec. vrwv* oscille seulement aux fiéqucncesdc 880 H/ cl de I 320 Hz. en laisam
varier la ftéqucnce de l'oscillateur de 5(X) Hz à I 500 Hz. Quelle est
la tension dans la cordc ?

SEQION 3.7 Les battements

42L La corde du la d'un violon est un peu trop serrée. On entend


quatre battements par seconde lorsqu’on lait résonner la corde
en même temps qu’im dHip.-ison qui oscille préciscmenl a la note lu
figure 3 35 P ro b lè m e .35 (440 H/). Quelle est la période d’oscillation de la corde de violon?
43E. Un diapason dont la fréquence est inconmic prrxluit trots batte­
36P. Une fordc de violoneellc a une longueur L, pour Inqiiclic la fré­
ments par seconde quand il oscille en meme temps qu'un autre
quence fondamentale est j'. a) De quelle longueur / la torde doit-elle
diapa.son demi la fréquence est de 384 Hz. lot frct|iience de hallements
être écourtée, en la touchant du doigt, pour que la béquenie fondamen­
diminue lorsqu’on met un petit morceau de cire sur la pointe du
tale devienne r / ? b) Quelle est la v.ileur de / si /. — 0,80 m et
premier diapastHi, diminuant ainsi sa fréquence fondamentale Quelle
r 1,2? c) Dans le cas où / 1,2, quel est le «ipport entre lu
longueur d'onde de lu nouvelle onde sonore émise par la corde sur est la fréquence de ce diapason?
celle de Tonde sonore émise avant qu'on ne touche à la cordc? 44P Vous avez cinq diapasons qui oscillent à ries fréquences rappro­
chées quoique toutes differentes. Quels soni a) le nombre ninxitiium
37P. 11 y a de Teau dans le tond d'un puits dont les parois sont verti­
et h) le nombre minimum de fréquences de battement.s différentes
cales et qui résonne à 7.00 Hz. mais non à des fréquences plus busses.
que vous pemvez produire en faisant osciller deux diapasons à la l’ois,
(I.a portion du puits où il y n de Tair agit coniinc un tuynu ayant une
dépendamment de la différence entre les fréquences des diapasons
extrémité l'erincc et l'autre ouverte.) 1,'air dans le puits a une ma.sse
voliiniique de 1.10 kg/m-' et un module de compressibilité de utilisés ?
1.33 X 10' Pa À quelle profondeur du puits se trouve la surface 4SP Deux cordes de piano idemii|ucs ont uik’ fréc|ucnce roiidiimciitale
de l'e a u ’' de 6CX) Hz lorsqu'elles ont In même tension. Quelle augmenlalion
relative de la tension dans une des cordes lA r/r) aura pour effet
38P I !n iiihe d'une longueur de 1.20 m est fermé à une extrémité. Un fd
de produire 6 baticments/s en faisant oscilkr les deux cordes simul
tendu c.st placé près de Tc.slréniité ouverte, la- fil mc.surc 0,330 m
lanémcnl ’?
de longueur et a une masse de 0.60 g. Il est fi.xé à scs deux extrémités
et oscille dans .son mode londamenial Par résonance, il fait osciller
lu colonne d 'air dans le tube à la fréquence fondamentale de cette S£(TI0N 3 8 L'offe» Doppler
colonne. Trouvez a) cette fiéquence et 1>) la Iciision dans le fil. 46£ L’agent de police fl est à la poursuite du conducteur A sur un
.19P. I .a {lérioite d'une étoile qui émet des pulsations variables peut segment droit d'autoroute l.esdcux circulent à une vitesse ayiiiil un
être estimée si on suppose que les couthes de l'étoile, en efléciuart module de IWI kiii/li. L’agent de police H. ne pouvant rattraper
des oscillations lonptudiiuilcs raduile\. pnxluiseni une onde siationnairc le contrevenant, .veiionnc sa sirène. Po.sez que le module de la vitesse
dans le mode fondamental ; en d'autres mots, le rayon de l'étoile du .son dans Tuir est de 343 m/s et que la fréquence de la source est
varie pcritHliqucment avec le temps, et celle étoile présente un ventre de 500 Hz. Quelle est la varmuon de frcquerrec causée par l’effet
de déplaconieni à sa surface, a) Diriez-vous qu'au centre de Ténàlc Doppler dans la fréquence entendue par le condtiaeur A ?
Exercices et problèmes 79

47E. Л quelle fréquence sera entendu le vronibissenicnt de 16,0 kHz 53P. Un système d’alarme sonore se compose d'une source qui
des turbines d ’un avinn à réaction qui vole à une vitesse ayant un émet des onde.s à 28.0 kHz. Quelle est la fréquence de battements
module de 200 m/s par le pilote d’un autre avion qui essaie de entre les onde.s émises et les ondes réfléchies par un cambrioleur
dépasser le premier avec une vitesse dont le module est de 250 m/s? qui marche à une vitesse moyenne de 0.95U m/s en s’éloignant
48E. Une ambulance dont la sirène ciiici un son à 1 6(K) Н/. dépasse de l’alarme? vmw
un cycliste qui circule à une vitesse ayant un module de 2.44 m/s. Après S4P. Une chauve-souris virevolte dans un sous-sol et règle sa navigation
avoir été dépas,sé. le cycliste entend une fréquence de 1 590 Hz. Quel en émettant des ultrasons. Supposez que la fréquence des émissions
est le mixlule de la vitesse de rambulanee? sonores de la chauve-souris est de 39.0 kHz. Lors d’un mouvement
49P. Un sifflement d’une fréquence de 540 Hz se déplace à une vitesse rapide vers la surface plane d’un mur. la chauve-.souris se déplace
angulaire de 15.0 rad/s dans un mouvement circulaire uniforme dont à 0,025 fois la vitesse du son dans l'air. Quelle fréquence perçoit-elle
le rayon est de 60.0 cm. Quelles sont al la plus basse et b) la plus dans les ultrasons réfléchis par le mur ?
haute iréquenee entendues par une personne éloignée et immobile 55P l'n e fillette est assise près de la fenêtre ouverte d'un train qui
par rapport au centre du cet d e ? «»чоу se dirige vers l’est à une vitesse dont le module est de I0,(K) m/s.
SOP Un détecteur de mouvement immobile émet des ondes sonores à Son onde sc tient près de la voie ferrée et regarde le train s'éloigner.
une fréquente de 0.150 MHz en direction d’un camion qui approche Le sifflet de la Ux'omotive émet un son й une fréquence de 500.0 Hz.
à une vitesse ayant uti module de 45.0 m/s. Quelle est la fréquence Cl il n ’y a aucun vent, a) Quelle fréquence l'oncle enierd-il?
des ondes réllcchies par le camion captées par le détecteur? b) Quelle fréquence la filictte entcnd-cllc? Un vent provenant de
51P. Un sous-marin français et un sous-marin américain sc dirigent l'est se lève et souffle à 10.00 m/s. c) Quelle fréquence l'oncle
l’un vers l’autre lors de manteuvres dans le nord de rix;éan Atlantique, entend-il maintenant? d) Quelle fréquence la fillette entend-elle
où l’eau est immobile (ligure 3.36). Le module de la vitesse du sous- maintenant? vivov
raarin français est de 50.0 km/h. et celui du sous-marin américain est 56P. Une sirène de 2 (КЮ Hz et un agent de la paix sont immobiles
de 70,0 km/h. Le sous-marin français envoie un signal sonar (onde par rapport au sol. Quelle fréquence l’agent cntcnd-il si le vent
sonore sous-marine) à 1 000 Hz. Les ondes sc propagent dttns l’eau souille à 12,0 m/s a) de la source sonore vers l'agent cl b) dans la
à une vitesse ayant un module de 5 470 km/h. a) Quelle est la direction opposée ?
fréquence du signal détecte par le sous-marin américain? b) Quelle S7P. Deux trains se dirigent l’un vers l ’autre il .10,5 m/s par nippon au
fréquence est détectée par le sous-marin français dans le signal sol. Le train A émet un sifflement й .“iOO Hz. a) S’il ii'y a aucun vem,
réfléchi par le sous-marin ainéncain? quelle fréquence est entendue dans le train B b) Quelle fréquence
est entendue dans le train K si le vent souffle à 30.5 m/s du train B
vers le train A ? c) (Quelle fréquence est eiiteiHiue si hi direction du
vent est inversée ?

50,0 kra/li 70.0 kin,ai


Sedion3.9 Les vdesses supenoniques et k ; ondes de (hoc
EiSUfe3.34 Problème 51
58L Un pmjectile est tiré à une vitesse ayant un module de 68.5 m/s.
52P Une source sonore A et une surface de réflexion B se déplacent TriHjvez l'angle formé par l'onde de choc et la trajectoire du projectile.
directement l’une vers l'auti«. Par rapport à l'air, le module de la 59P L'n avion à réaction pa.ssc au-dessus de vous à une altitude de
vitesse de la source A e.st de 29,9 m/s, celui de la surface B est 5 (ХЮm Cl à une vitcs.se de Mach de 1.5. al Trouvez l'angle du cône
de 65.8 ni/s. et considérez que le module de la vitesse du son est de Much, b) Combien de temps après que Tavion soit passé au-dessus
de 329 m/s. La source émet des ondes à une fréquence de 1,20 kHz. de votre tête l’onde de choc vous atteint elle ? Le module de la
tel qu'elle est mesurée dans le référentiel de la .source. Dans le vitesse du son est de 331 m/s
référentiel du rétlectciir, quelles sont a) la fréquence et h) la longueur 60P. Un avion vole à U 5 fois la vitesse du son. Son bang supersonique
d ’onde des ondes sonores détectées ? Dans le référentiel de la source, atteint un homme au sol une minute après qu'il soit passé üukIcssus
quelles sont c) la fréquence et d) la longueur d ’onde des ondes de lui. Quelle est l'altitude de l'avion? Le module de la vitesse du
sonores réfléchies vers la source ? son est de 330 m/s.
4 La réflexion
et la réfraction
de la lumière

Quand une camète suit une trajectoire autour du Soleil, lo gloce s'évapore à sa surface et laisse derrière elle un nuage
de poussière et de particules chargées. Le «ventsolaire», qui est lui-même chargé électriquement, force les particules
chargées à prendre la forme
d'une «queue» rectiligne,
orientée de façon radiale
dans la direction opposée
ou Soleil. Toutefois, la poussière
n'est pas offectée par le vent
solaire et devrait normalement
continuer sa course en suivant
l'orbite de la comète.

Pourquoi, alors, la queue


inferieure formée
par la maieure partie
de la poussière est elle courbée
dans cette photographie^
U 'éponse X Irixiv« doQ le chapitre.
4.1 Le spectre électromagnétique 81

4.1 Le spectre électromagnétique


L a principale réussite de Jam es CIciL M axw ell fut de d ém ontrer q u ’un rayon de lum ière
csl une onde progressive com posée d ’un cham p électrique ci d ’u n cham p m agnétique -
une o n d e é le c tro m a g n é tiq u e - et qu e l ’opliquc, l’étude d e la lum ière visible, constitue
une branche d e l’électrom agnétism e. D ans ce chapitre, on abordera ces deux sujets; on
conclura d ’abord l’étude, d ébutée au volum e 2 . des phénom ènes strictem ent électriques
et m agnétiques, et on posera ensuite les fondem ents de l’optique géom étrique.
Л l’époque de M axwell (le m ilieu du xix*^ siècle), les seules ondes électrom agnétiques
connues étaient la lumière visible, le rayonnem ent inltarouge et le rayonnem ent ultr aviolet.
Toutefois, H einrich H c ru , stim ulé p ar les recherches de M axw ell, d éco u v rit ce q u ’on
appelle m aintenant les ondes radio, et Ht la preuve exfiérim em ale q u ’elles se déplaçaient
à la m êm e vitesse que la lum ière visible.
C om m e l’illustre la figure 4.1. on connaît m aintenant un large .spectre (ou gam m e)
d ’o n d es é le c tro m a g n é tiq u e s, q u ’u n é c riv a in im a g in a tif a nom m é « Г arc -en -ciel d e
M axw ell ». E ssayez d e vous rep résen ter ju sq u ’à quel point vous êtes entouré d ’ondes
électrom agnétiques appartenant à ce s|3eclrc. L e Soleil, d ont le rayonncnx;nl a déterm iné
l’environnem ent auquel l’êlrc hum ain s’est adapté et où il a évolué en tant q u ’espèce.
en c o n stitu e la p rin cip ale .source. Vous ôtes ég alem en t su b m erg é p ar les sig n au x dc
té lév isio n e t d e rad io . L es m ic ro -o n d e s des sy stèm e s de rad a r et de tran sm issio n s
téléphoniques peuvent aussi vous atteindre. Les am poules élcctriqrrcs. les nxrteurs des
au to m o b ile s d ég a g e a n t d e la chaleur, les é q u ip em e n ts u tilisa n t d e s ray o n s X, les
en seig n e s lu m in eu ses et les m atériaux rad io a ctifs en fo u is prodiriscnt ég a le m en t des
ondes électrom agnétiques. S ’ajoute à cela, venant du ciel, le rayonnem ent ém is par les
étoiles et d ’autres corps célestes de notre G alaxie, sinon d ’autres galaxies. Les ondes
électrom agnétiques voyagent aussi dans la direction f>pp<»séc. D epuis IQ.'rO, environ, des
signaux de télévision sont transm is de la Terre e t envoient des nouvelles des hum ains
(incluant les épi.sodes d e Star Trek, q u oique très faiblem ent) aux év en tu els habitants
d ’au tre s p la n ète s assez é v o lu ée s en o rb ite a u to u r des q u elq u e 4 0 0 étoilc.s les p lu s
rapprochées.
D ans l'éc h elle de longueurs d 'o n d e de la figure 4.1 (et dans l’échelle de fréquences
correspondante), chaque div ision représente un changem ent de longueui d ’onde selon
un fa c te u r d e 10. C e tte é c h e lle n e se te rm in e pas au x e x tré m ité s rep rése n tée s.
R em arquez le facteu r én o rm e , plus de lO^", en tre les plus longues lo n g u eu rs d ’onde
représentées ( 10* m, ce qui est de T ordre de la distance Terre Lune) et les plus courtes
( 10 m, ce qui est dix fois plus p etit que le rayon d ’un proton).

l ongiieiirs d ’onctc (um )


700 (i(H) .¡tOO 400

S jx rtrc visible

Lonjçiieiii d'oiKlc (m )
10" 10^ 10* Ю* ИТ И)’ to 1 to ’ to * 10--^ КГ’’ 1Г-’ КГ" 10" КГ® КГ^ К Г'"Ш ” 1 0 10-*®10''^КГ‘^111
-I ■ . 1___ I,
.••Ori-flir*«;ÎOngUeîa • tiudtS r a io lefrQoijgcs НдаглтХ ' , ■! 'Г,1^у1>их^ап111м'v

10 10* 10* l(T' 10® 10* lo" 10® 10® 10’" 1 0 " Kl’® K)'® lO'^ lO’® 10*" lO'® I0 ‘® K)'® 10*® 10®' 10®® lO®" io®<
Fréquent e (Hz)

Radio F \î
( 'anaux
Radiodiirii.sion \ (!■ lélêvlsioii .'Vpplit allons m aritim es |
Applit aüoii.s .sur oiide.s eo\n les. \ j et aérotiautiques, télépitotiie i
m atitim es Raditi applirations m aiiliines \ -J" fpllulaire, comnmnirations |
Ci aéroriauliques ,\M et aéionatiiiqties et \ r- tivec le» .satellites i
i ~ 1— ~ l—
10^ KP 10" 10® 10® 10® 10'" lOK
trfquenee (II/)
figure 4.1 [ e spectre électromagnétique
82 Chapitre « La rêflexton et la réfraction de la lumière

C ertaines régions du spectre électrom agnétique de la figure 4.1, com m e celles des
raynm X cl des omles ruüiu, oni de.s apfiellalioiis qui vous sont familières. Ces appellations
d écriv en t som m airein em des plag es de lo n g u e u rs d ’o nde fréq u em m en t u tilisées par
certaines sources et certains dctccieurs d ’ondes électrom agnétiques. N otez que ces régions
n ’ont pa.s, en général, d e lim ites bien définies. D 'au tres régions de la figure 4 . 1, com m e
celles où l ’on indique les signaux de télév ision ei de radio A M , représentent des plages
spécifiques de longueurs d ’oiidc réservées à certaines utilisations com m erciales ou autres,
el d é fin ie s p ar la loi. 11 n ’y a pas d ’esp ac e lib re d an s le sp e ctre é lec tro m ag n é tiq u e,
et lotîtes les ondes électrom agnétiques, peu im porte où elles se situent dans le spectre,
‘НЛ1 4Г)(» MM) :V)0 (iOn Г,Г,0 7()() se propagent dans к vide à une vitesse ayant le m êm e m odule c.
L o ilü iin ii (1 o n ilr (m il) La région visible du spectre préscnlc évidem m ent un intérêt parUculicr dans le cadre
de ce volum e. La figure 4.2 illustre la sensibilité relative de l’ceil hum ain aux différentes
Figure4.2 l,a RonKibilitc relative
longueurs d 'o n d e de la lum ière. I æ m axim um de sensibilité se siuie à environ 555 nm ;
de l'ceil humain au.\ unJc.->
il produit la sensation visuelle que l’on appelle jaune-vert.
élcuronuijini'liijiics en fonrtinn
I es lim ites du spectre visible ne sont pas clairem ent définies, car la courbe de .sensi­
de la lt)Tigiieur d'nnde. Г ptte région
bilité de l’ceil tend vers zéro de m anière a.syniptoti(iuc en présence de longues longueurs
du spectre électrumagnetique
d ’onde com m e en présence de courtes longueurs d ’onde. S i on fixe de façon arbilraire
■yc iiuiiiim; Ut lumièrr vlsihlr.
les lim ites d es lo n g u eu rs d ’onde qui p eu v en t être perçu es p ar un mil selon le critère
voulant que la .scn.sibilité est réduite à I % de sa valeur m axim ale, ces lim ites sont environ
à 4 3 0 nm et à 6 9 0 nm ; to u te fo is, l’œ il p eu t d é te c te r des o n d es éle c tro m a g n é tiq u e s
au-delà de ces lim ites si elles sont as.sez intenses.

A.2 L’aspect qualitatif des ondes


électromagnétiques progressives
C ertaines ondes électrom agnétiques, dont les rayons X. les rayons gam m a et la lum ière
visible, rayonnent (sont émi.scs) à partir de sources dont la taille est d 'o rd re atom ique ou
nucléaire, et où les lois de la physique quantique s ’appliquent. O n traitera ici de la façon
do n t les autres o n d es é lec tro m ag n é tiq u es sont gén érées. P our sim p lifier les choses,
on se lim ite ra à la rég io n du sp e ctre (lo n g u eu r d ’o n d e A. « 1 m ) où la so u rc e d e
rayonnement est à la fois m acroscopique et d e dim ension appréciable.
La figure 4.3 illustre de façon sim plifiée la génération de telles ondes. A u creui du
systèm e se trouve un oscillateur LC. qui a une fréquenœ angulaire tu ( = 1f-/L C ). À cette
fré q u en c e, les c h a rg es et le s co u ra n ts v arie n t de façon sin u so ïd ale d a n s ce circ u it,
com m e c ’est illustré dans la figure 12. l du vnlume 2. U ne source externe, une génératrice
de courant alternatif, par exem ple, doit être incluse pour fournir l’énci-gic qui com pense
à la fois les pertes th erm iq u es dans le circu it et l ’én erg ie ém ise d an s le rayonnem ent
de fo n d e électrom agnclique.
L’oscillateur ZXT de la figure 4.3 est relié par un transform ateur e t une ligne de trans-
mi.ssion à une antenne, qui est form ée, essentiellem ent, de deux m inces tiges conductrices
et rigides. D ans cette connexion, le coui-ani variable sinusoïdal de l’o scillateur produit
une oscillatio n sinu.soïdale le long des tiges d e l ’antenne, à la fréquence an g ulaire co
de l’oscillateur LC.
courant dans les tiges, associé à ce m ouvem ent de charge, varie aussi de façon
sinusoïdale à une fréq u en ce an gulaire w. L’an tenne agit com m e un dipôle électriq u e
dont le m om ent dipolaire varie de m anièro sinusoïdale, le long de l’antenne.

, \ \
Oncle progrrs.<iivc

.Sèiin e
^V-nergii-
Ligne
de- rr;insmi4.sintf
l— v W -
Апг^ппс
(dipôU* )

Figure 4.3 Un disposiiif permettant de générer une ontle éleclminagnétique dans la plage dev <indes
courtes radio du spectre . un oscillateur Li ' produit un courant sinusoïdal dans fanlennc.
qui génère l’ondc. F est Un point éloigné où un détecteur peut enregistrer l’onde qui le traverse.
4.2 L’aspect qualitatif des oncles électromagnétiques progressives 83

C om m e le m om ent dipolaire varie, le cham p électrique produit p a r le dipôle varie

r ‘^ A r
г
Cham ps
égalem ent. De plu.s, com m e le courant varie, le cham p m agnétique produit par ce courant
varie égalem en t. T o u tefo is, les variatio n s dans les ch am p s élec triq u e e t m ag n étiq u e
ne SC p ro d u isen t pas p arto u t de façon in sta n ta n é e ; ces v aria tio n s s ’é lo ig n e n t plutfii
rie m odule maxirnal de Tantenne à la vitesse de la lum ière. C es cham ps variables form ent une onde électro ­

u; h)
a) _
Й<уЛр
гТ
b)
m agnétique qui se p ro p ag e en s ’élo ig n an t de T an ten n e à un e vitesse d ont le m odule
est c. L a fréquence angulaire de celte onde est со. la m êm e que celle de l'o scillateu r LC.
La figure 4.4 m ontre de quelle façon le cham p électrique £ et le cham p m agnétique И

tF * '"
' C lium j» nuis '
i varient dans le tem ps lo rsq u 'u n e longueur d ’onde com plète (un cycle com plet) franchit
le point éloigné P de la ligure 4.3 ; dans chaque partie de la figure 4.4. Tonde se dirige
directem ent en so rtan t de la page. (On choisil un point suffisam m ent distant pour que la
courbure des ondes illustrées à la figure 4.3 soit a.s.sez petite pour être négligée R elati­
K) c) vem ent à de tels points, on parle d ’une onde plane, ce qui sim plifie grandem ent Tétiide

t l de Tonde.) N otez les nom breuses caractérisliques illustrées dans la figure 4 .4 . on trouve
toujours CCS caractéristiques clés, peu im porte la façon dont Tonde a été produite :

îf
Й
rd)
,
1. Les cham ps électrique E et m agnétique B sont toujours perpendiculaires à la direction
de p ro p ag a tio n de l ’ondc. Tl s ’agil d o n c d 'u n e onde transversale, le ty p e d ’o n d e
q u 'o n a étudié dans le chapitre 2.
2. Le cham p électrique est toujours perpendiculaire au cham p magnétique.

Х
CliHinf»
Г J 3. L e produit vectoriel E x B donne toujours la direction de propagation de Tonde.
4. I æs cham ps v arien t toujours de façon sinusoïdale, co m m e les ondes transversales
étu d iée s d an s le ch a p itre 2. De plus, les cham p s varient avec la темпе fréquence
(If iim iluli' m axim al
et sont en pha.se T un avec l’autre.
c) Si Ton tient com pte de ces caractéristiques, on peut supposer que Tonde élec tro ­
Figure 4.4 a) à h) l.a variation m a gnétique SC d ép lace v ers le p o in t P dans la d irectio n p o sitiv e de T ax e des д, que
du champ électrique E et du champ le cham p électrique de la figure 4.4 oscille parallèlem ent à T axe des y, et que le cham p
magnétique fi à un point éloigné P m agnétique o scille donc parallèlem en t à T axe d es z (en utilisant bien sû r un systèm e
de la figure 4.3, alors qu’une longueur de coordonnées droit). On peut alors décrire les cham ps m agnétique et électrique com m e
d’onde de l’onde électromagnétique des fonctions sin u so ïd ales de la p o sitio n x (le long de la d irection de p ro p ag atio n de
traverse ce point. Dans ce schéma. Tonde) et d u tem ps t :
Tonde se dirige directement en sortant
de la page. Les deux champs varient E = sin(£x - cot). (4.1)
de façon sinusoïdale. Notez qu’ils sont
toujours perpendiculaires à la direction B = B,„ sinikx — cot). 14.2)
de propagation de Tonde et Tim
par rapport à Tautre. où £',„ cl B-„ .sont les am plitudes des champ.s et, com m e on Ta vu dans le chapitre 2, co et
k sont respectivem ent la fréquence angulaire et le nom bre d ’onde de Tonde. Cc.s équa­
tions m onlient que non seulem eni les deux cham ps fom ienl Tonde électrom agnétique,
m ais q u e chacu n form e « sa p ropre onde ». L ’équation 4.1 donne la composante élec­
trique de T o n d e éle c tro m a g n é tiq u e , et T éq u a tio n 4 .2 en d o n n e la com posante
magnétique. C om m e on le v erra bien tô t, ces deux composantes de l'onde ne peuvent
exister de façon indépendante.
G râ c e à T éq u a tio n 2 .1 2 , on sa it (|ue le m odule d e la v itesse d e l’o nde esl coik.
T ou tefo is, p u isq u ’il s ’agil d ’une o nde élec tro m ag n é tiq u e, le m o d u le de sa v itesse
(dans le vide) est donne par le sym bole r plutôt ijiic par c. O n verra dans la prochaine
section que c a la valeur

<■ = ----- (le module de la vitesse d’une onde électromagnétique Jaics le vide). (4.3)
s/B oP(i

ce qui équivaut à environ 3,00 x lO** m/s. b n d ’autres m ois:

Toutes les ondes électromagnétiques, incluant la luniic;rc visible, ont une vitesse
dans le vide ayant le niênxi nuidiile r.

Vous verrez égalem ent que le m odule de la vitesse d ’une o n d e électrom agnétique c et
les am plitudes des cham ps électrique et m agnétique sont reliés par

- c (le rapport des ampliiudesï. (4.4)


84 Chapitre 4 Lo réftexion et la rèfroction de la lumière

Si cm divise l’équaiion 4.1 par réquiUion 4.2 et q u 'o n insère le résultat dans l'équation 4.4,
on découvre que les com posantes des cham ps, à tout instant et en tout point, sont reliées
par

—= c (le rapport des composantes). (4.5)


B

On peut représenter l’onde électrom agnétique par un rayon (une ligne indiquant la
dircciion de propagation de F onde) ou p ar des fronts d 'o n d e (surfaces im aginaires où le
cham p électrique de F onde est partout le m em e), ou les deux à la foi.s, com m e on le voit
dans la figure 4.5 a). Entre les deux front-s d 'o n d e illustrés dan.s la figure 4.5 a), il y a une
distan ce d 'u n e lo n g u eu r d ’onde À ( = 2тт/к). (L es o n d es qui se propagent ap p ro x im a­
tivem ent dans la m êm e direction form ent un faisceau, sem blable à un rayon loser, qui
peut aussi cire représenté par un rayon.)
O n p eut aussi rep rése n ter un « in stan tan é » d e F o n d e, co m m e à la fig u re 4 .5 b ),
o ù les vecteurs dc.s cham ps électrique et m agnétique .sont illustrés à un instant précis
Les courbes reliant les extrém ités des vecteurs représentent les oscillations sinusoïdales
données par les équations 4.1 cl 4.2 ; les com posantes t et Й de F onde sont en phase,
perpendiculaires l'u n e par rapport à l'au tre, et perpendiculaires à la direction tie propa
gation de Fonde
L 'interprétation de la figure 4.5 h) est plus difficile. Des dessins similaires tl'im c onde
transversale dans une corde tendue, dont on a discuté dans le chapitre 2. représentaient
les déplacem ents de sections de la cordc. au m om ent où F onde les traversait (quelque
cho.\e se lU^phnait vraiment). L a figure 4.5 b) est plus abstraite. À Finstani illustré, les
cham ps élcetn q u e et m agnétique o nt chacun un certain m odule e t une certaine direction
(m ais to u jo u rs p e rp e n d ic u la ire à F a x e des x) à ch a q u e p o in t situ é su r l'a x e des x.
Puisqu'on choisit d e représenter ces quantités vectorielles par une paire de flèches à chaque
point V, on doit d onc tracer des flèches de différentes longueurs aux d ifféren ts ptnnts.
q u i s’éloignent toutes de F axe des .x, com m e les épines su r une tige de rose. Toutefois,
les flèches représentent seulem ent les valeurs des chanips aux points situés sur F axe d esx .
N i les flèches ni les courbes sinusoïdales ne représcnicnl un m ouvem ent transversal de
quoi que ce soit, et les flèches ne relient pas les jxiinls sur F axe d es .r à d ’autres points
situés hors de Faxe.
Les dtnisins sem blables au dessin utilisé à la figure 4.5 vous aident à visualiser une
situ a tio n ré e lle m e n t trè s co m p lex e. O n c o n sid è re d ’ab o rd le ch a m p m a g n é tiq u e ;
puisqu’il varie de façon sinusoïdale, il induit (d ’après ta loi de l'in d u ctio n de Faraday)
un ch a m p élec triq u e p e rp e n d ic u la ire qui v arie ég a le m en t d e faço n sin u so ïd ale .
Ttmtefois. (Hiisque ce chiuiip électrique a une variation sinusoïdale, il induit (d 'ap rès la loi
de l ’induction de M axw ell) un cham p iiw gnéüque perpendiculaire qui varie égalem ent
de faço n sin u so ïd ale. Et ain si d e suite. L es d eu x ch a n ip s se créen t m u tu ellem en t et
continuellem ent par induction, et les variations sinusoïdales résultantes des cham ps .sc
prop ag en t com m e une onde - F o n d e électro m ag n étiq u e. S an s ce résu ltat stu p éfian t,
on ne pourrait rien v o ir; en fait, p u isq u ’on a bc.soin des ondes électrom agnétiques du
Figure 4.5 a) Une onde élccimnragiiétique
Soleil pour nMiintenir la tem pérature de la Г е те, on ne pourrait sim plem ent pas exister
leprésenice par un rayon et deux fronts
sans ce résultat.
d’onde ; les fronts d’onde sont distants
d ’uiK longueur d'onde À. ht La meme
onde représentée par un « instantané»
de ses chanips électrique F. ^ r a i i b c l'o n d f iR a v o ii д
et magnétique H à différents points a)
sur l'axe des .r, le long duquel Fonde
SC propage à une vitesse r Loitqiie
F n iu le traverse le point F. les champs
varient comme c'est illustré duns
lu ligure 4 4 \j\ cimiposjuite éleeirique
ik l outle est itniquemont constituée
par les eham|>i électriques ;
la cnmposaiiie magnétiqtic c.si
uniquement eon.stiluéc' par les champs
magnétiques. Le rectangle pointillé
au point P est utilisé dans la figure 4.6.
4.3 L’aspect quantitatif des ondes électromagnétiques progressives 85

Une onde très étrange


Les ondes dont on a discuté dans les chapitres 2 e t 3 ont besoin d ’un m lieu (d’un support
m atériel) dans lequel, ou le long duquel, elles peuvent se propager. On a étudié des ondes
se propageant dans une corde, dans la terre cl d an s l ’air. Toutefois, une o nde électro ­
m agnétique (on em p lo iera aussi les term es umie lumineuse ou lumUire) présen te un e
d iffé ren ce étra n g e p a r rap p o rt à ces o n d es, d an s ce sen s q u ’elle n ’a besoin d ’aucun
m ilieu pour .se propager. E lle peut effectivem ent se p ropager dans un m ilieu com m e l'a ir
ou le verre, m ais elle peut égalem ent le faire dans le vide de l’espace qui sépare la Terre
d 'u n e étoile.
À la suite de la reconnaissance de la théorie de la relativité restreinte, bien après la
publication des travaux d ’E instein, en 1905, la conm iunauté scientifique a dû adm ettre
que la vitesse des ondes lum ineuses avait une propriété très spéciale. En effet, la lum ière
p o ssèd e la m êm e vitesse, peu im p o rte le référen tiel à p artir duquel elle est m esurée.
Si vous en v o y e z un faisceau d e lu m ière le lo n g d ’un ax e et d e m a n d e / à p lu sieu rs
observ ateu rs de m esurer sa vitesse alors q u ’ils se d ép lacen t eux-m êm es à d ifféren tes
v itesses le long de cet axe. soit dans la d irectio n d e la lu m ière, so it d an s la d irection
opposée, ils m esureront tous la meme vitesse. C e résultat est suiprenant cl très différent
de ce q u ’on aurait obtenu si ces observateurs avaient m esuré la vitesse de tout autre t y p “
d ’onde ; dans le cas des autres ondes, la vitesse relative des observateurs ’d urait affecté
leurs m esures.
L e m ètre est m aintenant défini en fonction du m odule de la vitesse de la lu m ièic
(ou de toute onde électrom agnétique) dans le vide qui possède la valeur e.xacte de

c = 299 792 458 m/s.


q u ’on peut u tiliser com m e un étalon stan d ard . E n fait, si vous m esu rez le tem p s que
prend une im pulsion lumineu.se pour se rendre d ’un point à un autre, vous ne m esurez
pas réellem cm le m odule d e la \ iies.sc de la lum ière, m ais p lu tô t la d istance en tre ces
deux points.

4.3 L’aspect quantitatif des ondes


électromagnétiques progressives
On d é m o n tre ra m a in te n an t les éq u a tio n s 4..3 et 4 .4 et. ce q ui e st p lu s im p o rta n t,
on ex p lo re ra l ’in d u c tio n m u tu e lle d es ch am p s é le c triq u e e t m a g n étiq u e p erm e tta n t
de produire la lum ière.

L’équation 4.4 et le champ électrique induit


I.e rectangle en pointillé dans le plan xy de la figure 4.6 possède les dim ensions dx et h.
et il est fixé au point P sur T axe des x (on le voit à la droite de la figure 4.5 b). Lorsque
T onde électro m ag n étiq u e trav erse ce rectangle vers la d ro ite, le flux m agnelique
p assan t à trav e rs le rec tan g le ch a n g e et, selon la loi d e T induL iion d e F arad ay ,
des cham ps cletlriq u es induits apparais.scnt dans toute la région du rectangle. O n pose
que £ et £ + dE sont les cham ps induits le long des d eux grands côtés du rectangle.
(7es cham ps électriques induits sont en fait la com posante électrique de Tonde électro­
m agnétique.
Considérez ces cham ps à l’instant où la com posante m agnétique de Tonde traversimt
le rectangle est représentée par la petite section m arquée en rouge dans la figure 4.5 b).
À cet in.stant p récis, le ch am p m ag n étiq u e traversant le rec tan g le est ot iciilé dans la
direction positive de Taxe des et son m odule dim inue (le m odule était plus grand jiisie
Figure 4.6 Quand l’onde électromagné-
avant T arriv ee de la sectio n ro uge). P uisq u e le ch am p m ag n étiq u e d im in u e, le flux
liciue se propage vers la droite m agnétique traversant le rectangle dim inue egalem ent. l.V après la loi de Faraday, des
en dépas.sant le point P de la figure 4.5, cham ps électriques induits s ’opposent à cette variation de flux, ce qui produit un cham p
la variation sinusoïdale par rapport
m agnétique B dtms la direction positive de Taxe des
au temps du champ magnétique fi,
Selon la loi de Ixinz, cela signifie que .si on sc repre sem e les bot dures du rectangle
dans un rectangle centre pat rapport
com m e une bo u cle co n d u e tn e e , un co u ra n t induit d an s le sens a n tih o ia ire d ev ra it y
au point / ’ Induit des champs électriques
apparaître. Il n’y a év idem m ent pas de boucle conductrice ; m ais celle analyse dém onlie
le lr>ng du rectangle. A l’instant
représenté, le nuxlule de 6 diminue, que si les vecteurs du ch am p électriq u e induit £ et £ + dli sont orien tés d e la façiai
et le module du champ électrique induit illustrée dans la Figure 4.6, le m txlule de £ f </£ doit être plus g rand que celui do £ .
est dont plus grand sur le côté droit S ’il n 'e n était pas ainsi, le cham p électrique induit ne [Muirrait pas produire un courant
que sur le cc'tté gauche du re<-tangle. en sens antihorairc autour du rectangle.
86 Chapitre 4 La réflexion et la réfraction de la lumière

O n peut m aintenant appliquer la loi de l’induction de Faraday,

/ E-iCs = —
di '
(4.6)

dans le .sens anlihoraire su r le périm ètre du rectangle de la ligure 4.6. Il n ’y a aucune


contribulion à l'in té g ra le pour les d eux sections du rectan g le parallèles à l’axe de.s x,
parce que E et ds sont toujours peipendiculaires. L’intégrale a alors la valeur

/ E - ds = (f: 1- d t )h - Eh = h dE. (4.7)

Le flux <I>R travcnsanl ce rectangle est

- (R)Ui dx). (4.8)

nil B est le m odule du cham p inoyen B dans le rectangle, et h dx cst I'a ire du rectangle.
Ln dcrivant I’^qiiation 4.8 par rapport a I, on obtieni

d<t>fi . .
— = hdx — , (4.9)
dt dt
Si on insère les équations 4.7 et 4.9 dans l’équation 4.6, on déterm ine que

dB
h dE = - h dx —
dt

dE dB
ou (4.10)
dx dt
En réa lité , B et E sont to u s les d eux de.s fo n ctio n s de deux v aria b les, x el t, com m e
l’im pliquent les équations 4.1 et 4.2. Toutefois, en évaluant dEldx, on doit supposer que t
est constant, pu isq u e la fig u re 4 .6 rep résen te un « in sta n ta n é » . De plus, en év alu an t
dB/dt, on doit supposer que x est constant, puisqu’on évalue la variation de B par rapport
au tem ps à un endroit particulier, soit le point P de la figure 4.5 b). D ans ces cirainstances,
les dérivées sont des dérivées partielles, et l’équation 4.10 doit prendre la form e

lÆ dB
(4.11)
Ih

Le signe n ég a tif dans cette éq u atio n est n éc essaire p u isq u e E au g m en te av ec x dans


le rectangle de la figure 4.6, m ais B dans ce m em e rectangle dim inue avec t.
D ’après l’équation 4.1, on a

dE
— = kE cos(Lr — tôt)
dx

et, d ’après l’équation 4.2, on a

dB
— = ~tüB„ cos(A.ï — fi)t).
dt “
L équation 4.11 devient alors

kE„, cos(Arv — tôt) - <oB^ cos(Lr - rut). (4 12)

l.e rapport iofk d ’une onde progressive correspond au mcnliile de sa viiesse, que l’on a
désigné pai c. L’équation 4.12 devient donc

—c (le rapport des amplitude,s). (4.13)


B„

ce qui correspond à l’équation 4 4.


4.3 L'aspect quantitatif des ondes électromognétiques progressives 87

L'équation 4.3 et le champ magnétique induit


L a figure 4.7 illustre un autre rectangle en pointillé ce n u é au point P d e la figure 4.5 ;
c e p en d a n t, ce lu i-ci se tro u v e dans le p la n L o rsq u e l’o n d e c le c tro m a g n é iiq u e se
J ____/ . ' déplace vers la droite en traversant ce nouveau rectangle, le flux électrique <l>// traversant
le rcclangle change et. selon la loi de l’induction de M axw ell, des cham ps inagnctiqucs
î iix' induits apparaissent dans toute la région du rectangle. Ces cham ps m agnétiques induits
figure 4.7 La variation sinusoïdale sont, en fait, la com posante m agnétique de l’onde électrom agnétique.
par rapport au temps du champ On peut voir, grâce à la figure 4.5. q u ’à l’instant choi.si pour le cham p m agnétique
élecD'ique L, dans un rectangle centré de la figure 4.6, le cham p électrique traversant le rectangle de la figure 4.7 est orienté de
par rapport au point f" de la figure 4.5. la faqon qui est illustrée. Rappelez-vous q u ’à l’instant choisi, le cham p m agnétique de la
induit des champs magnétiques le long figure 4 .6 dim in u e. P u isq u e les d eux ch am p s so n t en p h ase, le ch am p élec triq u e de
du rectangle. À P instant représenté, la figure 4.7 doit égalem ent dim inuer, to u t com m e le flux électrique qui travci-sc le
le même que celui de la figure 4.6, rec tan g le . E n ap p liq u an t le raiso n n e m e n t q u ’on a tenu d an s le cas de la fig u re 4.6,
le module de F. diminue, et le nusdulc on voit q u e le fiux v ariab le in d u ira un ch am p m a g n étiq u e, et g u e si les v ecteurs
du champ magnétique induit est donc fi et fi (IB sont orientés tel q u 'o n le voit dans la figure 4.7. fi + t/fi doit avoir un plus
plus grand sur le cêté droit que sur grand m odule que fi.
le côté gauche du rectangle. On applique ici la loi de l'in d u ctio n de M axw ell,
^ с7Ф/,
fi (4.14)

en procédant d an s le sens an tilio iaire sur le périm ètre du rectan g le en p o intillé d e la


figure 4.7. Seuls les grands côtés du rectangle contribuent à l'in tég rale, dont la valeur
est

B- ( f s = - { f i г dB)h 1 Bh = - h dB. (4.15)

Le flux Ф, traversant le rectangle est

(E){h dx). ( 4 .1 6 )

où F. est le m odule du cham p m oyen E dans le rectangle. En dérivant l ’équation 4 16


par rapport à t, on obtient
(EW dE
-— = h dx .
dt dt
Si on insère ce résultat et l’équation 4.15 dans l’équation 4.14. on déterm ine que

- h dB = dx .

On peut rem placer les dérivées par des dérivées partielles, com m e on l’a fait auparavant
pour l’équation 4.10. en utilisant des argum ents équivalents,
gfi BE
(4.17)
lit
Ici encore, le signe négatif dans cette équation est nécessaire car. m em e si fi augm ente
avec r dans le rectangle centré au point P de la figure 4.7. E dans ce rectangle dim inue
avec t.
En évaluant l’équation 4 . 17 à l'a id e des équations 4 1 et 4.2, on obtient

-4:fi,n cos(fcr — o>t) = — cosfToi: - (ot).

que l'o n peut écrire sous la form e

_ _ 1 _ ^ I
B^ tinf-nc
lin com binant ce ré.sultat avec l’équation 4 П , on obtient finalem ent

(• = _ (le m odule de la vitesse d 'u n e onde électrom agnétique dans le vid e ), ( 4 .1 8 )


V fin m
ce i|ui corres-p«md exactem ent à l'éq u atio n 4.3.
Chapitre 4 La réflexion et la réfraction de la lumière

✓ vérifiez VOS CONNAISSANCES): -c champ magnéiique B traversant le rectangle


de la figure 4.6 est représenté ici à un instant ditterem, dans la partie I de l’illustration ;
B est orienté dans le plan xz. parallèle à l’axe des z. et son module augmente, a) Complétez
la panie I en dessinant les champs électriques induits, illustrez les directions et les modules
des champs relatifs au.x deux côtés du rectangle parallèles à l’axe des y (comme dans la figure 4.6).
b) Pour le même instant, complétez la partie 2 de l'illustration en traçant le vecteur champ
électrique de l'onde électromagnétique. Illustrez également les champs magnétiques induits,
en indiquant les directions et les modules relatifs aux deux côtés du rectangle parallèles à l’axe
des ; (comme dans la figure 4.7).

A '

2)

4 Л Le transport de l'énergie
et le vecteur de Poynting
Toutes los personnes qui prennent des bains de soleil savent q u ’une onde électroniagnctique
peut transporter de l ’énergie cl la transférer à un objet sur lequel elle se pose. l,e taux
de transfert d ’énergie par unité de surface dans une telle onde est décrit p ar un vecteur S,
que l’on nom m e v e c te u r d e P o y n tin g , en l ’honneur du physicien John H enry Poynting
( 18.'î2-1914), qui fut le prem ier à signaler ses propriétés. O n définit S ainsi ;

5 = J_ £ X B (le vecteur de Poynting). (4.19)


Mo

Son mtxUile S est relié au taux auquel l’énergie, transportée par une onde à un instant (inst)
donné, passe à travers une surface im iiaiie perpendiculaire à la direc tion de propagation
de l’onde :

_ /énergie/tem psN /puissanceN


(4.20)
V aire /i V aire Д
On peut conclure que l’unité SI de S est le w att par m ètre carré iW /m ’).

À n’importe quel point, la direction du vecteur de Poynting S d’une onde clccuomagnétique


donne la direction de propagation de l’onde, cl la direction du transport d’ciietgie à ce point.

Pui.scjue, dans une onde électioniagnétique, È et B sont perpendiculaires, le m odule


de L X B est EB. Le m odule de S est donc

S = — EB, (4.21)
IM)

où S. E Cl B sont des valeurs instantanées. l,es valeurs E et B sont tellem ent lices l'u n e
à l’autre que l’on peut n 'e n considérer q u 'u n e ; on choisit E surtout parce que la plupart
des in stru m e n ts serv an i й d élec ter les o n d es élec tro m ag n é tiq u es .sont sen sib le s à la
com posante électrique tic l’onde plutôt c|u’à sa com posante m agnétique. Ln utilisant le fait
que B — FJc selon l’équalion 4.5, on peut récrire l’équation 4.21 .sous la form e su iv a n te.

5 - — £• (le taux de transfert d’énergie insiantané). (4 22)


ЦМ)

Ln in.séranl E ~ sin(Lx — (ut) d an s l'é q u a tio n 4 .2 2 . on p o u rra it o b te n ir une


éq u a tio n du tau x de tran sp o rt d ’é n e rg ie en (o n ctio n du tem p s. D an s la p ratiq u e, en
général, on m esure plutôt le taux m oyen de transport d ’énergie dans le tem ps ; on dí^it
4.4 Le transport de l’énergie et le vecteur de Poynting 89

donc tro u v e r la v a le u r m o y en n e de ,S d a n s un in te rv a lle de tem p s su ffisam m en t


long, q u ’on écrit et q u ’o n nom m e in te n sité / d e l’onde. D ’aprè.s l’équation 4.20,
l ’intensité / est donc
_ /éneigie/tem p.sX _ /p u is .s a n c e \
-'moy (4 .2 3 )
aire /in o y \ a ire /m ü y

À l'a id e de l’équation 4.22, on déterm ine que

f •'moy
él’mov. — t^ ü l s in " (fc ï (4.24)
t’Mo

D ans un cycle com plet, la valeur m oyenne de sin^ 0. relativem ent à to u t argum ent ft est
de 5 (voir la figure 12.14 du volum e 2). D e plus, on définit com m e suit une nouvelle
quantité F ^ t , la valeur efficace du duunp clectrUtue :

¿cir " (4.23)


s/2
O n peut alors récrire l’équation 4.24 sous la form e

/= — £ ,> 14.26)

Puisque £ — r f t et que c est un nom bre très grand, on poun ait conclure que l'énergie
associée au cham p électriq u e e st b eau co u p p lu s gran d e que celle asso ciée au ch am p
m agnétique. Cette conclusion est incorrecte : les deux énergies son! pat faitem ent égales.
P our le dém ontrer, on procédera d ’abord avec l'éq u atio n 5.23 du volum e 2. qui donne la
densité d'én erg ie % ( — dans un cham p électrique, en substituant c 8 à E t on peut
alors écrire

Si on substitue m aintenant l’équation 4.3 à c, un oblicnl

Bt —
2/io

L’équation lü.57 du volume 2 indique que B% jUoesl la densité d ’énergie «« d ’un cham p
m agnétique B, et on voit alors q u ’en tout point d ’une onde élecirom agnétitjue, - u„

La variation de l'intensité en fonction de la distance


L’in te n sité d ’un ray o n n e m en t é le c tro m a g n é tiq u e v arie so u v en t d e faço n c o m p lex e
lo rsq u ’on s ’élo ig n e de la source de ray onnem ent, spécralem eni lorsque la source (un
projecteur dans une soirée de gala, pai exem ple) ém et le rayonnem ent dans une direction
particulière. Dan.s certaines situations, cependant, on peut supposer que la source est une
source ponctuelle qui ém et la lum ière de façon isotrope, c'est-à-d ire avec une intensité
\ - '
/ égale dans toutes les directions. Une section iransversale des fronts d 'o n d e sphériques
qui se propagent à un insiant donné à p<mir d 'u n e telle source ponctuelle et isotrope S
est illustrée à la figure 4.8.
, / y S u p p o se / q ue l'é n e rg ie d es o n d es est co n serv ée en .se p ro p ag e an t à p a rtir d e la
source. Im aginez aussi une sphère de rayon r, centrée p ar rapptirt à la source, com m e
U - dans la figure 4.8. Toute l'én e rg ie ém ise par la source doit traverser la sphère. D onc, le
taux d ’énergie de rayonnem cnl passant à travers la surface d e la sphère doit être égal au
taux auquel l’énergie est émi.se par la snurt;e, c ’est à-dire à la puissance Pj de la source,
l.’inicnsité / à la surface de la sphère d<iii alors être

figure 4.8 tTue source ¡xinctuelle S (4 .2 7 )


/ = A .
émet des ondes électromagnétiques
unifonriénicnt dans toutes les directions.
Les fronts d’onde sphériques traversent où Aitr^ est f a ir e de la sphère. L’équation 4.27 indique que l'in ten sité du rayonnem ent
une sphère imaginaire de rayon r. électrom agnétique d 'u n e source ponctuelle isotrope dim inue en fonction du carré de la
centrée par rapport à .y. di.slance r de la source.
90 Chapitre 4 La reflexion et la réfraction de (a lumière

✓ v é r if ie z VOS CONNAISSANCES 2 : jt schém a c i-c o n tre m o n tre


1c champ électrique d'une onde électromapnéiiquc à un certain point
et à un certain instant. L’onde transpt»rte de l’énergie dans la direction
négative de l’axe des Quelle est la direction du champ magnétique
de l’onde à ce point cl à cet instant ?

Exemple 4.1
Un observateur se trouve à une distance de 1.8 m d’une source Le troisième temept dé est celui-ci ; les modules des chumps élec­
luimiicu.NC ponctuelle et ivitmpe. iloni la puissance l \ e.st de 2.‘>0 W. trique et nrugnétique d'une onde électromagnétique, à tout instant et
Calculez la valeur efficace des champs électrique et magnétique en tout point de l’onde, sont reliés par le module de la vitesse de la
prixluiis par la source, à la position de l'observateur. lumière c. selon l’équation 4.5 {E/B = f). Donc. Ic.s valeurs efficaces
SOIUHON: Les deux premier.s tontepls dés sont les stiivaiiis ; de ces champs sont aussi reliées par l'équation 4.5. et on peut écrire;

I- La valour efficace du champ électrique £,.,f de la lumière est reliée


Ecü _ 4 8 .1 V /m
h riiitcasité / de la liiinit're par l’équation 4.26 (J - fieB =
~ ~ djx) X 10» m/s
2. Piiisi|iir la source est une source ponctuelle qui émet la lumièie
avec une intensité égale dans loulc.s les d ira lions, rintensité / = 1.6 X 10 ’ T. (réponse)
à tonte distance r de la source est reliée a la puissance de la
source par l’équ.'ition 4.27 (/ - Notez que ( “ 48 V/in) peut être mesurée à l’aide d'apparciLs
Cn comhiiiaiii ces deux cniicepis. on nhticiil standard utilisés en laboratoire, tandis que fi.» (-=■ 1.6 x 10 ’ T)
représente une très petite valeur. Cette différence permet d'expliquer
I = = S ’f , pourquoi lu plupart des instruments utili.sés dans la détection et la
c/tü ’ mesure des ondes éicctroinugnétiques sont conçus en fonction île la
ce qui donne
composante électrique de l'onde On ne peut dire, toutefois, que la
cumpo.sante électrique d’une onde électromagnétique est « plus
grande » que sa composante magnétique. On ne peut comj'arer des
quantités qui possèdent des dimensions différentes. Comme on l’a
/(250 W)(3.00 X 10« m/sK4-T x lO^'^ H/m) vu, les composantes électrique et magnétique sont égales du point
~ V (4n^)(1.8m)- ~ de vue de leur propagation tians le vide, et leurs énergies moyennes,
= 4«, 1 Vyui RS 48 Vym. (réponse) qui peuvent être comparées, sont exactement égales.

4.5 La pression de radiation


Les ondes électrom agnétiques ont utie quantité de m ouvem ent et de l'cneigic. Cela signifie
que vous jxvuvez exercer une pression, une pre.<ision d e ra d ia tio n , sur un objet en dirigeant
de la iiitnicrc vers lui. Toutefois, cette pression est très petite puisque, par exem ple, vous
ne ressentez rien lo rsq u 'u n e perstvnne utilise un flash pout vous photogiaphicr.
Pour trouver rexpresston de la pression, on peut diriger le faisceau d ’un rayonnem ent
électrom agnétiq u e, de la kiinicre, p ar ex em p le, .sur un objet pendant un in tervalle de
tem p s At. O n su p p o se ici q u e l'o b je t est lib re d e b o u g e r ci q u e le ray o n n c n ien l est
entièremejit absorbé (capté) p ar l’objet. C ela signifie que. durant rin ie rv a llc At, l'o b je t
acquiert une quan tité d ’én erg ie A V provenant du rayonnem ent M axw ell a dém ontré
que l'o b je t acq u iert ég alem en t une q u an tité d e m o u v em en t. La v ariation du imKlule
de la quantité de m ouvem ent de l’objet Ap est reliée à la variation de l’énergie AU par

AU
Ap = d'absorption totale). (4.28)

où c est le nutdule de la vitcs.se de la lum ière. L a quantité de m ouvem ent de l’objet a la


m êm e directitKi que celle du rayon inc ident absorbe par l'o b jet.
Au lieu d ’etre absorbe, le rayonnem ent |x:ut cire réfléchi par l'tibjct ; le rayiiniK-ment
est alors renvoyé dans une autre direction en rebondissant su r l'o b jel. .Si le rayonnem ent
CM entièrement réfléchi le long de .sa trajecto ire initiale, la variation d u m odule d e la
quantité de niouvcnieni de l'o b je t est le double de celle donnée précédem m ent, ou

< 2 A i/
Ap = ------ (la rénexiiin totale le lottg de la trajei loire initiale). (4.29)
c
4.5 La pression d e radiation 91

D e la m êm e façon, u n o b jet acquerra deux fois plus de quantité de m ouvem ent si une
balle de tennis p arfaitem en t élastique rebondit .sur lui p lu tô t q u ’une balle .sans aucune
élasticité (une boule de pâte hum ide, p ar exem ple), les deux ayant, par ailleurs, la m êm e
m asse et la m êm e v itesse. Si le ra y o n n e m en t in cid en t est p a rtie lle m e n t ab so rb é et
partiellem ent réfléchi, la variation du mtxlule de la quantité de m ouvem ent de l’o b jet se
situera entre A l//c et 2 lU /c .
À l’aide de la deuxièm e loi de N ew ton, on sait q u ’un changem ent dans le m odule
de la quantité d e m ouvem ent est relié au m odule de la force m oyenne par

f = i4..Wj

Pour trouver une expression du m odule de la fome m oyenne exercée par le rajvnncm enl
en fonction d e l’in ten sité / du ray o n n em en t, on p eu t su p p o ser q u 'u n e su rfa ce plane
d ’aire A, perpendiculaire à la d irection de propagation du rayonnem ent, in te n ep ic le
ray o n n e m en t. D an s un in te rv a lle d e tem p s Ai, l’é n e rg ie in te rcep tée p a r la su rfa ce
d 'a ir e /4 est

A (/ = lA Ai. (4.31)

Si l’énergie est entièrem ent absorbée, l ’équation 4.28 indique aloix que A/i — lA \U c et,
d ’après l’équation 4 30, le m odule de la force m oyenne exercée sur la surface d ’airc A est

M
r = — (l’absorption totale). (4 .32)
V

D e la m em e façon, si le rayonnem ent est entièrem ent réfléchi le long de .sa traiectoirc
initiale, l’équation 4.29 indique que Ap = 2M A i/r. et. d ’après l’équation 4.30,

11 A
F = (la réflexion totale le long de la trajectoire initiale). (4.33)
c

Si le rayonnem ent est partiellem ent absorbe et partiellem ent réfléchi, le m odule de la
force m oyenne su r la surface d ’aire A se situera entre les valeurs de /A/t et U Air.
Le m odule d e la force m oyenne du rayonnem ent par unité ckt su rface su r un objet
co n stitu e la p ressio n de rad ia tio n p ,. O n p eu t la tro u v e r dans les situ atio n s décrites
par les équations 4.32 et 4.33 en divisant les deux m em bres de charnue équation par A.
On obtient ainsi

P, = — (rabsorption totale) (4..34)


r

11
et (la réflexion totale le long de la trajectoire initiale) (4.35)

V eillez à ne pas confondre le sym bole p ,, qui désigne la prcs.sion de radiation, asec le
sym bole p, qui désigne le m odule de la q uantité de m ouvem ent, L ’iinilé SI d e la [ires-
sion de radiation est le new ton par m ètre carré (N/m^), q u ’on nom m e pascal (Pa).
Le développem ent de la technologie des lasers a perm is aux chercheur,s d ’aUeindre
des pressions de radiation beaucoup plus élevées qu e celle du flash d un appareil photo,
par exem ple. C ela s ’explique par le fait qu e le faisceau lum ineux d ’un laser, contraire­
ment au faisceau lum ineux ém is par le filam ent d ’une lam pe, peut être concentré en un
m ince faisceau de quelques longueurs d ’o nde de diam ètre seulem ent. C.'ela perm et de
transm ettre de grandes quantités d 'én e rg ie aux petits objets soum is à cc rayonnem ent.

i / VÉRIFIEZ VOS CONNAISSANCES 3 ; Inc lum ière d ’ intensité u n ifo rm e b rille perpendi» iila iic in c n t
à une surface parfaitement absorbante, l’éclairant pleinetnent -Si on diminue Taire de la surface,
est-ce que a) la pression de radiation et b) le rrK'diiIc de la force mnveniic du ravtmm.'mnil
sur la surface augmentent, diminuent ou demeurent les mêmes ?
92 Chapitre 4 La réflexion et la réfraction de la lumière

Exemple 4.2
La poussière produite par une comète ne suit pas la irajcctoirc En iasérant ensuite les équations 4.37 et 4.39 dans l'équation 4.36,
orbitale de la comète, parce que la pression de radiation de la luiuicic et en isolant R, on obtient
solaire 1« pousse de favon radiale dans la direction op|x>séc au Soleil.
Suppt)sce qu'une ['»artiailc de pt>us.sière sjitiériquc txttsêde un rayon R 3Ps
R =
et une masse volumique p - 3.5 X 10’ kg/ni’. cl absortx: eniièrcniail \ 67rcpGMs
la lumière du soleil qu'elle intercepte. Sachant que la puissance En utilisant la valeur donnée de p. celle de G (annexe B) et celle
émise par le Soleil est de 3.9_X |( P W. pour quelle valeur de K la iorcc de Ais (annexe C), on peut évaluer le dénominateur:
graviiaiiiinndle du Soleil exercée sur la particule équilibre-t-elle
la torce de radiation F, exercée sur elle p.ar U lumière solaire ? (l6;r)(3,(K) X 10" m/s)(.3,5 x lo 'k g /n r ')
SOlUIlOII: On peut supposer ici que le Soleil est assez éloigné de la X (6,67 X 10“ " .\ m V kg’ )( 1.9^1 X 1 0 ^ 'kg)
particule pour agir comme une source lumineuse ponctuelle et isoii’0|Tc.
= 7.0 X 10-^^ N/s.
Comme on soit que tu prc.s.sioii de ratliulioii |)ou.s.sc la pariiculc de façon
ladlale dans la direction oppo.sce an .Soleil, la force de radiation F, En utilisant la valeur de fs donnée, on a alors
exercée sur la particule doit être orientée de façon radiale dans la
direriinii opposée au centre du Soleil En meme temps, la force (3)(3.9 X IÜ-* 3\')
R = = 1,7 X lO“ "' ni (réponse)
grnvii.'itionncllc /g exercée sur la particule est orientée de façon 7,0 X 10” N /s
ludirtle ifi.i le cciiire du Soleil. Pui.sqiie F, doit équilibrer F^.
Notez que ce résultat ne dépend pas de la distance r entre la particule
les mcHliile.s de ces forces doivent être égaux, d’où :
Cl le Soleil.
F,. (4..36) Les particules de poussière dont le rayon est /? = 1.7 x 10“^ m
suivent une Irajccloire approximativement rectiligne, cnmine la
On peut ici considérer ces forces séparément.
particule qui a été libérée lorsque la comète était à la position 1 dans
Force de raJiiiiion . Pour évaluer le membre de gauche de la figure 4.9. et qui se déplace, par la suite, suivant la tmjecioine h
l'équation 4.36, on utilise tes trois (oncephdér suivants.
tangente à l'orbite de la comète. Quand les valeurs de R sont plus
1. Puisque la particule est parfaitement «bsorhantc. le mixlule de la grandes, une comparaison entre les équatioii.s 4.37 et 4.39 démontre
fori c ik' radiation F, peut être trouvé en fonction de l'intensité / que, pui.squc F, varie en fonction de Af’ et que F, varie selon R~, le
de In lumière du soleil A l'endroit où se situe la particule, et de module de la force gravitationnelle surpa.s.sc le incxJulc de la force
l'a iic A de la .section transversale de la partictile. il l'aide de de radiation Donc, de telles particules suivent une trajectoire
l iquation 4 32 (P = ¡A/c). incurvée vers le Soleil, comme U trajectoire c de la figure 4.9. De la
2. Coininc on suppose que le Soleil est une .source lumineuse pone même façon, dans le cas des valeurs de R jilus petites, c’est la force
tuclle et isotrope, on peut utili.ser l'équation 4.27 (/ = P^/Attc-) de radiation qui domine, et la poussière suit une trajectoire incurvée
pr>ur relier la puissance du Soleil â l'intensité / de la lumière dans la direction opposée au Soleil, comnK la trajectoire a. L’ensemble
.solaire, relativement à une distance r du Soleil. de ces particules de poussière forme la queue de poussière de la comète.
3. Puisque la particule est sphérique, sa section transversale est un
disque d’aire A = itR- (et non b moitié de sa surface splrénquc).
En combinant ces trots concepts, on obtient

_ M _ PsyrH- _ PsR^
(4.37)
’ c 4 tt- c 4 r-r

Force ntm-iriitioimelli' : la; concept tié ici est la loi de la gravita­


tion de Newton (équation I4.I du volume 1). qui donne le module de
la force gravitationnelle sur la particule:

CA^m
F. ■- (4 38)

ni) Mÿ est la mas.se du Soleil et tu est la masse de la pariieule. (3n relie


ensuite la masse de la particule à sa masse volumique /> et à son
volume P ( = ^rrfî’. pour une .sphère) par

m W figure 4 9 Exemple 4 .2 1 Ine comète est à la positism 6. 1,a jxiussière


= T7 = qu'elle a produite à scs cinq positions précédentes a été poussée
de façon radiale par la pression de radiation de la liimicrc du soleil.
Hn isolant m et en in.sérant le résultat dans l'équation 4 38. on obtient Cette poussée produite par la radi«tion dévie les particules
de poussière de la tnijccioire d^ la comète .suivant les différentes
F, = (4.39) trajcctoirc.s illiLstrécs en pointillé, formant ainsi la queue de poicssicrc
incurvée de la comète.
A.6 La réflexion et la réfraction 93

4.6 La réflexion et la réfraction


Les om bres bien définies des objets produites par la lum ière solaire par tem ps clair e t le
faisceau lum ineux d ’un p ro jecteu r d e film s sont des situ atio n s, parm i de nom breuses
autres, o ù les ondes lum ineuses possèdent des trajectoires approxim ativem ent rectilignes.
D ans ces situations, on repré.senie lc.s ondes lum ineuses sous form e de rayons lum ineux.
U n ra y o n lu m in e u x est une droite fléchée tracée dans la direction de propagation de
l'onde lum ineuse ; elle équivaut à un faiscetiu lumineux dont le diamètre est extrêmem ent
p etit. L es ray o n s lum ineux d ’une o n d e so n t d o n c des d ro ite s p e rp e n d ic u la ire s aux
fronts d ’ondc. L ’étu d e des pro p riétés des ondes lumineuse.s q ui tieni co m p te de celte
approxim ation se nom m e l’o p tiq u e g é o m é triq u e . D ans le reste de ce chapitre et dans
le chapitre 5. on traitera de l’optique géom étrique de la lum ière visible.
La photographie de la figure 4 .10 a) m ontre un autre exem ple d 'o n d es lum ineuses
se prop ag ean t en suivant des trajecto ires ap p ro x im ativ em en t rectilig n es. L e rayon à
a)
gauche dans la photo est un m ince faisceau de lum ière, le fai.sceau incident, provenant
Konnale d ’en haut. 11 se propage dans l’air vers le ba.s, suivant un certain angle, e t rencontre une
I
surface plane de verre. U ne p artie de la lum ière est réfléch ie par la surface, fœ m an t le
rayon en haut à droite ; la lum ière se propage vers le haut com m e si le rayon initial avait
rebondi sur cette surface, com m e une balle de tennis. Le reste de la lum ière traverse la
surface, se propage dans le verre vers le bas, et form e le rayon en bas à droite. Puisque
la lum ière peut voyager dans un verre de ce type, on dit que ce v erre est transparent,
ce qui sig n ifie q u ’o n p eu t v o ir à trav ers. (D an s ce c h a p itre , on lim ite ra l’étu d e aux
m atériaux transparents.)
L a transm ission de la lutnière à travers une surface (ou une interface) qui sépare
deux m ilieux se nom m e ré fra c tio n , et on dit alors que la lum ière est réfractée. A m oins
q u ’un rayon de lum ière incident ne soit perpendiculaire à une surface, la réfraction par
une su rfa c e ch an g e g én é rale m en t la d ire c tio n de p ro p ag a tio n d e la lu m ière. C ’est
pi^urquoi o n d it que le rayon est « dévié » par la réfraction. Notez, dans la figure 4 .10 a),
figure 4.10 a) Une photographie que la dév iation se produit seulem ent à la surface ; à l’intérieur du vci rc. la lum ière se
montrant la réflexion et la réfraction p ro p ag e e n lig n e d ro ite. L es ray o n s lu m in eu x se p ro p ag e n t en ligne d ro ite d an s un
d’un rayon lumineux incident, m ilieu hom ogène.
sur une surface de verre plane. D ans la fig u re 4 .1 0 h), les ray o n s lum ineux de la p h o to g rap h ie sont rep résen tés
(Une portion du rayon réfracté (de m êm e que les fronts d ’onde) par un rayon incident, un rayon réfléchi et un rayon
dans le verre n'apparaît pas claiicnient réfracté. C h a q u e ray o n p o ssè d e une o rie n ta tio n p ar rap p o rt à u n e d ro ite, n o m m ée
dans la photographie.) Lé rayon est la n o r m a le , q ui est p erp en d icu laire à la su rface et qui p asse p ar le po in t où le rayon
perpendiculaire îl la surface courbée incident rencontre la surface. D ans la figure 4.1 0 b), l’a n g le (T in cid en ce est l'a n g le
du bas de la photographie ; la réfraction d e ré fle x io n est 6 [ cl l'a n g le d e r é f ra c tio n est $2 , tous ces angles étant ^nesurés p a r
ne dévie donc pas le rayon à cet endroit, rapport à la normale, com m e c ’est illustré. I.e plan co iu en an i le rayon incident e t la
hî Line représentation de a) en utilisant norm ale est le plan d'incideiu e ; il correspond au plan de la page dans la figure 4 .1 0 b).
lies rayons lumineux. Les angles L’expérim entation m ontre que la réflexion et la réfraction respectent deux lois.
d'incidence (ff,), de réflexion (fti) L o i d e la r é fle x io n : le ray o n réfléch i e st réfléc h i d a n s le p lan d 'in c jd e n c e et
et de réfraction (02) t=oiit indiques. l’angle de réflexion est égal à l’angle d ’incidence. D ans la figure 4 10 b ), cela signifie
que

(la loi de la réflexion). (4.40)

(On laissera m aintenant tom ber le sym bole prim e dans la notation de l’angle de léflrxion.)
L o i d e la r é f r a c tio n : le rayon réfracté e st ré fra c té d an s le plan d ’in c id en ce et
l’angle de réfraction O2 est relié à l’angle d ’incidence 0 , par

M; sin 62 ■* ” 1 sin 01 (la loi lie la refraction). (4.41)

C hacun des sy m b o les et n? e st une co n stan te san s d im en sio n , n om m ée in d ice de


réfraction, qui est as.sociée au m ilieu im pliqué dans la réfraction. O n dém ontrera cette
équation, nom m ée loi de Snell-D escaries, dans le chapitre 6 . C om m e on le verra p ar la
suite, l’in d ic e d e ré fra c tio n d ’un m ilieu est égal à r/v, où v est le m odule de la vitesse
de la lum ière dans ce m ilieu et r. celui de sa vitesse dans le vide.
Ix: tableau 4.1 d onne les indices de réfractio n du vide et de q u elques su bstances
com munes. D aas le vide, n est exactem ent égal à 1 («„j^ — c/v — d e ) : dans l’air, la vites.se
V4 Chapitre 4 La réflexion et la refraction de la lumière

de lu lum ière est iégcrcm cnl inferieure à celle d an s le vide ; r a donc une valeur près
de 1.00 lune apiKoxinmiion que l'o n jtilise ra № uvcnl). A ucune substance n 'a un indice
de réfiac tio n in férie u r à 1, car, selo n la d é fin itio n d e n. c e la im p liq u erait qu e v > t ,
ce qui est physiquem ent im possible. La vitesse de la lum ière dans un m ilieu ne |KUl c*tre
plus grande que celle q u 'e lle possède dans le vide.
O n peut réarranger ainsi l'éq u atio n 4 .4 1 .

sin = — sin в, (4.42)

afin de com parer l'an g le de réfraction avec l'an g le d ’incidence H,. O n peut alors voir
a) b)
que la valeur de ff, dépend des valeurs relatives de n, et « 2. En fait, un peut tirer de celle
\n n n a lr équation les trois résultats suivants.

1. Si «2 à ” i' alors égal à H,. D ans ce ca s. la réfraction ne d év ie pas le


rayon lum ineux, qui continue dans la /nè/ne direction qu e le rayon incident, com m e
on le voit dans la figure 4 . 11 a). D e plus, il n ’y a pas de réflexion.
2. Si »2 est plus grand que ti,, W. est alo rs plus p etit qu e 6 , D ans ce cas. la réfraction
dévie le rayttn lu m in eu x en le rapprochant de la n o rm ale, com m e o n le voit d an s
la figure 4.11 b).
3. St «2 CM plus petit que n ,. ^2 ‘-'SI alors plus grand qu e D ans cc cas. la rériactio n
«9<n, d é v ie le ray o n lu m in eu x en V éloignant d e la n o rm ale, co m m e on le v o it dans
c) la figure 4 11 c).
Figure 4.11 l in rayon lumineux La réfraction ne peut faire dév icr un rayon au point où le rayon réiiacté serait du m êm e
9« prupatcdiil initialenieiil côté d e la norm ale que le rayon incident.
ilanv un niilieii clom rindico de rétmciion
e.M «I rcnumtrc imc inicrfiii c plane
La dispersion chromatique
l'no panio tie ce rayon est réfractée
JaiiN un milieu (liait I'intlicc de réfm iinn L 'in d ic e d e réfra ctio n n d an s to u t m ilieu , au tre qu e le v id e. déi>end d e la lo n g u e u r
CM H;, a) Ix: rayon n « i pas. dévié d ’onde de la lum ière. C elle d ép en d an ce en irc w et la lo n g u eu r d 'o n d e im p liq u e que.
liasque rif = n¡: la lumière réfractée lorsqu'un faisceau lum ineux se com pttsc de rayons de différentes longueurs d ’onde, les
$e propeiic niors dans lu nu'nie rayons .seront réfractés à différents angles par une surface (conséquem m ent à la loi de la
direction que le rayon iiicidciu réfractio n ) ; la lu m ière sera ain si dispersée p ar la réfractio n . C e tte d isp e rsio n de la
(ligne pointilice). Le rayon est dévie lum ière se nom m e d is p e rs io n r h r n m a liq u e , où « c h ro m a tiq u e » réfère aux co uleurs
en b), vers la normale puisque > n,.
ass(x:iées aux longueurs d 'o n d e individuelles. Les réfractions des figures 4. Il) et 4.11 ne
et en c). en s'éloignant de la normale
représentent pas une d isp ersio n ch ro m atiq u e, p arce qu e les faisceaux lum ineux sont
puisque Mj < n¡.
nionoehroniatiqiies (d ’une seule longueur d 'o n d e ou couleur).

IACIEAU 4 1 Quelques inices de réfraction*


Milieu Indice Milieu Indice
Vide Exaclcnieni I Verre crow n typique I ,«
Air(TPN)** I.OOO 29 Chlorure de sodium (sel) l..‘i4
Eau (20 ' O 1..33 Polystyrène l.^5
Acétone I.36 Disulfure de carhonc l.f>3
Alcixrl éthylique (éthanol) L36 Verre flinl lourd 1.65
Solution sucrée (3 0 9 ) I.38 Saphir I.77
Quartz fondu (SI0 2 ) I.46 Verre flint très lourd I.K9
Solution sucrée (80%) I.49 Diamant 2. I 2
* Duiiiicn |K)ur une longueur cW nin trait! jaunr d'uiK* lainpc au sodiumu
•• TPN ^icnifie •• tevnpératun; cl prvKsmn normales » (0 '^'Cci I atini.

figure 4.12 L'indice de réfnK.iiim du quart/ Imidu en l'nnclion'de la longueur d'onde, I e graphique
indique qu'un faisceau de lumière de petite longueur d'ondc. pour lequel l'indice de réfraelion
est plus élevé, est da\ aniage dévié en entrant (Xi en sortant du quartz qu'un rayon de lumière
) ongiiciii il'nndi- (uni) ayant une graixie longueur d'ondc. l'atnre milieu étant l'air
4.6 La réflexion et la réfraction 95

Normalt* En général, pour la lum ière visible, l'in d iec de refraelion d ’un m ilieu donne c.st plus
grand pour les petites longueurs d ’o nde (correspondant à un e lum ière bleue) que pour
les grandes longueurs d ’onde (lum ière rouge). La figure 4.12 m ontre com m ent l'in d ice
de réfiaction du quart/, fondu (verre d e silice) dépend de la longueur d 'o n d e de la lumière.
Une telle dépendance signifie que, lorsqu’un faisceau com portant des ondes lum ineuses
bleue et rouge est réfracté par une su rface, en passant de l’air au q u a rt/ ou vice versa,
la composante bleue (le rayon correspondant à T onde lum ineuse bleue) est davantage
déviée (réfractée) que la com posante rouge.
Un faisceau de lumière blanche pos.sède toutes (ou presque toutes) les com posantes
(les couleurs) du spectre visible, et à des intensités appioximati vement égales. l,orsquc vous
apercevez un tel faisceau, vous percevez le blanc plutôt que les cotileiirs individuelles.
D ans la figure 4.1,3 a), un faisceau de lum ière blanche dans l'a ir rencontre une surface
de v erre. (P u isq u e les p ag es de ce liv re so n t b la n c h e s, on rep rése n te un faisceau
de lu m ière blan ch e p ar un ray o n g ris, et un rayon de lu m ière m o n o ch ro m atiq u e est
généralem ent représenté par un rayon rouge.) Dans la lumière réfractée de la figure 4.13 a),
^blltlîcrr NoniiaU' 1.iimièrc
blanrhe seules les com posantes rouge e t bleue sont illustrées. Puisque la com posante bleue est
} blanche
davantage déviée que la rouge, l’angle de réfraction de la com posante bleue csip lu s
petit que Tangle d e réfraction 02< ctMiiposanie rouge. (K appelez-vous iiiie les angles
sont m esurés par rapport à la norm ale.) D ans la figure 4.13 b), un faisceau de lum ière
blan ch e se pro p ag ean t d an s le v erre ren c o n tre un e in te rfa ce verre-air. D e n o uveau,
la com posante bleue est davatiiuge déviée que la com posante rouge, m ais, m aintenant,
^2h est plus grand que .
Pour accentuer la séparation des couleurs, on peut utiliser un prism e de %erre ayant
une section transversale triangulaire, com m e dans la figure 4 14 a). La dispersion p ar la
prem ière surface (à gauche dans la figure) est alors accentttée par celle ilc la seconde
surface.
Figure 9.13 Dispersion chromatique L ’arc-en-ciel co n stitu e T exem ple le plus charm ant de la dispei vion clironiatiquc
de la liiinicre blanche. La composante Q uand la lum ière blanche du soleil est interceptcc par une goutte de pluie, une partie de
bleue est davantage déviée que la lum ière est réfractée à la prem ière surface de la goutte que rencontre la lum ière, puis
la composante rouge, a) En passant elle est réfléchie par la seconde surface de la goutte, et finalcm cnl, elle est de nouveau
de l’air au verre, la comp<isantc bleue réfractée à la prem ière surface vers l'ex térieu r de la goutte (voir la figure 4 15) C om m e
a le plus petit angle de réfraetion. dans un prism e, la prem ière réfraction sépare la lum ière du soleil en ses etutqvisantes,
b) En passant du verre à l’air, et la seconde réfraction accentue la séparation.
la composante bleue a le plus grand L ’arc-cn-ciel que vous voyez est produit par de la Itim ièrc réfractée par un grand
angle de réfraction. nom bre de g o u tte s; la lum ière rouge vient des gouttes fom iaiii un angle par lapport au
sol plus gnm d que celui des autres gtHiltes dans lo ciel, la co u leu r bleue vieiu des gouttes
form ant un angle plus ¡jctit p ar rapport au sol. et les couleurs interm édiaires viennent
des gttuttes form ant des angles interm édiaires. Toutes les gouttes qui vous envoient les
différentes couleurs form ent un angle d ’environ 42° par rapport à une droite passant par
vous et le Soleil. Si la pluie se prolonge et est très éclairée par le Soleil, vous verrez un
arc circulaire contenant plusieurs couleurs, rouge au-dessus et bleu dans le bas. L ’arc-
en-ciel que vous voyez vous est exclusif, car tout autre observateur intercepte la lum ière
provenant d ’autres gouttes.

Liiini^fP
b la riijir.

a) b)
Figure 4.14 a) Un prisme triangulaire sépare la lumière blanche en se« eompnsante« (couleurs)
h) La dispersion chromatique «e produit à la première surface et est acreilliice à la seaaule surface.
9é Chapitre 4 La réflexion et la réfraction de la lumière

figure 415 aj Un arc-cn-ciel est tinijours


un aie circulaire centré sur la droite
passant par vous et le Soleil,
b) La séparation des couleurs
qui se produit lorsque la lumière
du soleil est rcfraciéc en entrant Urmièic du soleil rKiuttes d ’eau
c l en soitam des gouttes de pluie
produit un arc-cn-ciel. La figure
repré.scnie une .situation où le Soleil
est à rtiorizon (les rayons du soleil
sont alors honzontaux). Les parcours
des rayons rouges et des rayons bleus
réfractés par deux gouttes sont illustrés.
De nombreuses autrc.s gt)Ulle.s
contribuent aussi à la formation
de rayons rouges c l de rayons bleus,
et de rayons des autres couleurs
Intermédiaires du spectre visible. l’obserr’atciir
ai b)

^ V É R I F I E Z VOS CONNAISSANCES 4 : x q u e l de ces tro is dessins (le cas échéant) représente


une ré fra c iio n [ih y s iq iie m e n t p o ssib le ?

» = 1,5 k

b)

Exemple 4.3
a) Dans la figure 4.16 a), un rayon lumineux monochromatique est
réfléchi et réfracté au ptiint A, à l'interface entre le matériau 1. dont
l'indice de réfraction est «| = 1,33, et le matériau 2, dont l'indice de
réfraction est = 1,77 Le rayon incident forme un angle de 50^
avec l’interface. Quel est l’angle de réflexion au point A Quel est
l’angle de réfraction au même endroit ?

SOlUÎION: Le contept dé est le suivant, dans toute réflexion, l’angle de


réflexion c.st égal à l’angle d’incidence De plus, les deux angles s<int
mesurés entre le rayon lumineux et la normale à l’interface passant
par le point de réflexion. Dans la figure 4.16 a), la nonnttle au point
A est la ligne poinlillée traversant le point. Notez que l’angle
d'inciJcnce ^1 n’esi pns l’angle donné de .50". mais vaut pluiOi
90" ~ 50" = 40’' L’angle de létlexinn est donc
a)
(X fl, = 40' (r6pon.se) figure 4.16 F.xempic 4.3 a) l,a luniicrc est réfléchie et réfractée
La lumière qui pas.se du mûtériau I au matenau 2 subit une réfraction au point >1. à l’interface entre les matériaux I et 2. h) I a iimiière
au piùnl A. à l’intcrfarc enire les deux matériaux l.e tomepidé est qui traverse le matériau 2 est réfléchie et réfraclcc au point fi.
que. dans toute retmction, on peut relier l’angle d’incidence, l’angle à l’interface entre les maténaux 2 et 3 (air)
de réfiaetion cl les indices rie réfraciion des deux matériaux à l'aide
de l'équation 4 dl r
En résolvant réqiiation 4.43 p<Hir trouver fl-, on obtient
W-. sin fl-> «I sut fl. (4.43)

Rappelons qu’on mesure les angles entre les rayons lumineux et la fli — sin"' s i n - s i n 4 0 ’^
nta'male pavsaiit par le point de réfraction. Doik , dans la figure 4.16 a),
l’angle de réfraction est l’angle fl,. = 28,88" - 29' (réponse)
La réflexion totale Interne 97

Ce résultat signifie que le rayon est dévié en s’approchant de la Ensuite, la lumière qui passe du matériau 2 à l'air subit une
normale (il formait un angle de 4œ et en forme mainienant un de 29’^). réfraction au point B, avec un angle de réfraction dj. Donc, le tortcpl dé
Cela s’explique par le fait que la lumière qui passe à travers l’interface consiste encore ici à appliquer la loi de la réfraction, mais ceUe fois
entre dans un matériau dont l’indice de réfraction est plus élevé. en écrivant l’équation 4.43 sous la forme
h) l a lumière qui entre dans le matériau 2 au point A atteint ensuite
tij sin f t = fjj sin ft.
le point B, à l’interface entre le matériau 2 et le matériau 3, qui est
de l’air, comme l’illustre la figure 4.16 b). L’interface au pomt B est Eu résolvant cette équation pour irouvcr f t , on obtient
parallèle à celui au pointé. À B. une partie de la lumière est réfléchie
et le reste entre dans l’air. Quel est l’angle de réflexion ? Quel est
l’angle (le réfraction dans l'air? 0., = s i n - sin «2) = s i n - '‘•n 2K.8B")
SOlirnON: On doit d'abord relier un des angles au poini B à un angle
connu au point A Puisque l'interface au point B est parallèle à - 58,75" a 59". (icponsc)
celui au point A, l'angle d'incidence à B doit être égal è l'angle de
Ce résultat .signifie que le rayon est dévié en s’éloignant de la
réfraction ^2. comme l’illustre la figure 4.16 h). Quant à la réflexion,
normale (il formait un angle de 29° et en forme maintenant un
on utilise le même conerpt (lé que dans la partie a) ; la loi de la réflexion.
de 59"). Cela s'explique par le fait que la lumière qui passe à travers
EJonc, l’angle de réflexion à B est
l’interface entre dans un matériau dont l’indice de réfraction est
e; = ft = 28.88" = 29". (rcpon.se) moins élevé.

4.7 La réflexion totale Interne


l.a figure 4.17 représente des rayons lum ineux inoiux lm m iatiques, ém is par une source
ponctuelle S dans le verre, qui rencontrent une interface verre-air. D ans le cas du rayon a,
qui est perpendiculaire à l'in lerface, une partie de la lum ière est réfléchie à l’interface,
et le reste se propage dans l’air sans changer de direction.
D ans le cas des rayon.s h k e , dont les angles d ’incidencc à l’interface sont progrès
.sivement plus grands, il y a égalem ent réflexion et réfraction à l’interface. Puisc|ue les
rayons lum ineux entrent dans un m ilieu dont l’indice de réfraction est plus petit, l'an g le
de réfraction est plus grand que l’angle d ’incidence. Lorsque l’angle d'incidence augmente,
l ’angle de réfraction augm ente égalem ent ; pour ce qui est du rayon e, il est de 90°, ce qui
signifie que le rayon réfracté pointe directem ent le long de l’interface. L’angle d’incidencc
qui pnvluit ccttc situation se nom m e l'a n g le c ritiq u e 0^. Qt^and les angle.s d ’in cid en œ
.sont p lu s g ran d s q u e co m m e d an s le cas d es r a y o n s / e t g, il n ’y a p as d e ray o n
réfracté et toute la lum ière est réfléchie ; cet effet sc nom m e la réflex io n to ta le in te rn e .
Pour trouver on utilise l’équation 4.41 ; on associe arbitrairem cnl l'in d ice I au
milieu cai les rayons incidents se propagent et l'indice 2 à l'autre milieu, celui où les rayons
réfractés, s 'il y en a, se propagent O n substitue ensuite h 0i et 90° à (ij, et on trouve
tii sin = «2 sin 90°,
ce qui donne

I «2
B, = s in - ' — (l'angle critique). (4 .4 i)
«1

Puisque le sinus d ’un angle ne peu! pas être plus grand que 1. «2 (leut pas excéder «i
dans cette équation. C ette restriction vous indique que la réflexion totale interne à une
interface ne j« u t pas se produire lorsque la lum ière incidente cM dans le m ilieu ayant
le p lu s p e tit in d ice d e réfra ctio n . Si la so u rce S ét.ait d a n s l’air d an s la fig u re 4 17.

Figure 4.17 Une réflexion totale interne


de la lumière provenant d’une source
ponctuelle S dans le verre se produit
à tous les angles d’incidence plus
grands que l'angle critique 6,.
À l'angle critique, le rayon réfracté
pointe le long de l'intcrlace air-verre.
98 Chapitre k Lo réflexion et lo réfraction de la lumière

Figuri 4.18 La lumière enirant à une extrémité d une tibre optique comme « Iles illuMnêc: ici
est transmise àVcsirémité opposée avec peu de perte de lumière le long de la fibre.

tous sc.s rayons incideni.s à I’interface air-verre (in c lu a n t/e t î ;) seraient à la fois réfléchis
et réfractés à l'in terface.
L a réflex io n to tale in te rn e a de n o m b reu se s a p p lica tio n s d a n s le d o m ain e de la
tcchnohtgie m édicale. Par exem ple, un m édecin peut reehcrclicr un ulcère dans I'csioraac
d ’un p atien t en in sé m n l d eu x m in c es faisc ea u x de fib re s optiques d an s la g o rg e
du patient (voir la figure 4.18). Lu lum ière introduite à rex lrén iU é extérieure d ’un des
faisceaux sub it d es ré tie x io n s totale.s in te rn es rép étées d an s les fib res de so rte q u e.
m êm e si les fibres suivent un parcours sinueux. In m ajeure partie de la lum ière jxirvicnt
à l ’autre extrém ité et éclaire l’iiuéricur de l'cslo n m c. Une partie de la lum ière réfléchie
de r in lé r ie u i rev ie n i e n s u ite de lu m êm e façon d an s le seco n d faisc ea u , e t c lic est
d étec tée et co n v e rtie en im ag e su r l’écran d 'u n m o n iteu r qu e le m éd ecin p eu t v o ir
l.e s fib res o p tiq u e s so n t g é n é ra le m e n t faites d e v erre ou d e p la stiq u e cl ont des
diam ètres variant de quelques m icrom ètres à 50 /itii.

Fxcmple 4 4 . Kèltrxion
,• J lV lolale in ip m e
La figure 4.19 présente un prisitte triangulaire de verre dans l'air.
Un rayon Im idem i entre d aiK le verre perpendiculairement à un côté
et est totalement réfléchi ù l’interface verre-air adjacente à celle
que traverse le rayon incident, comme le montre l'illustration. Si fí¡
vaut 4.S", que pouvez vous dire sur l’indice île réfraction n du verre 7
Figine4.19 Exemple 4.4 Le rayon incident i subit une réflexion totale
lOlUllON l e premier (ontspt <lt est le suivant, pui.squc le rayon
luraincu,\ e.si complètement léiléchi à l’interface, l'.inglc critique interne à riniei'face verre-air et devient le rayon rélléchi r.
à ccitc interface don être plus petit que l’angle d’incidence de 45".
I
Le deuxième tantepl de est qu’on peut relier l’indice de réfraction n ou n> = 1.4. (гéponsc^
du verre ù t). à l'aide de la loi de la réfraction, qui mène à l'équa­ sin 45
tion 4.44. En insérant itj •= I (pour l'ait ) et ~ n (pour le verre) L’indice de réfraction du verre doit êlrc plus grand que 1.4.
dans l'êquadon. un obtient sinon il ii’y aurait pas de réflexion totale interne pour le rayon
incident illustré ici Remarque/ que. dans cette situation, le prisme
n . = sin l' " - ’ = sin
0 • - I' -•.
a fait dévier le rayon lumineux incident de 90". On utili.se des
«1 n
pri.smes .semblables dans certains instruments d ’opiiquc comme
Puisque N. doit être plies petit que l'angle d'incidence de 45". on a les jumelles.

s i n - '- < 4 5 " . / VÉRIFIEZ VOS CONNAISSANCES 5; Suppose/ que. dans


l’exemple 4 4. le prisme a un indice de réfraction n = 1,4.
ce qui donne Aura-t-on encore une réflexion totale interne de ki lumière
si le rayon incident reste bori/onlal mais que le prisme est tourné
- < sin 45' s) de 10" en sens horaire ou b) de 10' en sens antihorairc
n dans la figure 4.19?

V1^

Lc.v ondes électrom agnétiques I /ne onde élcciromagnéiique de toute onde âcvtromagnétique dans le vide est c, que l’on peut écrire
se cnniposc de champs électrique et magnétique oscillants. Les difte sous la forme
rentes trêquciK'es possibles des ondes cleciioinagnétiqirs forment un
spcctrt. dont la liiiiiière visible couvre veiileinent une petite partie. - - * (4 5 .4 .1 )
B s/w^O
l ’ne onde olceiromagnétiquc se propageant dans la directimi ixisitive
de l'axe des i .1 un champ électrique h et un champ magnétique B où £ et B sont les valeurs des composantes des ciKtmps prises au
iliini les compocantps sont des fonctions de .i et Je /. même instant et au même endroit.

F ~ f.,„ sint*v — fot) £ e transport de l'énergie \je «ux par unité de surface auquel
l'énergie est transmise par une Onde électromagnétique est donné par
B simf r - wt). (1 1 .4 .2 ) le vecteur de Poynting ;
où t'ai et B„ sont les amplitudes de £ et fl Le champ électrique -V = — £ X fl. (4.191
induit le champ magmUique. cl vice versa. Le nimlule de la vilcs.se flu
Questions 99

La direction de S (qui est aussi la direction de propagation de l’onde) La pression de radiation est le module de la force moyenne pat
est perpendiculaire aux directions de £ et de Й. Le nxxlule du taux unité de surface :
moyen par unité de surface auquel l’éiicigic est transmise est 1
que l’on nomme Vinrensile l de l’onde : /J ,—- (l'alxsorplion totale) (4.34)
c
21
1 = (4.26) et P, '■
Ф0‘
(la réflexion totale le Uaig tie la trajectoire initiale). (4.35)
où £(,fr = E J s /l. Une .ïonrceponcWie//«'d’ondes électromagnétiques
émet les ondes de façon hatroiH', c’est-à-dire avec la même intensité L ’optique géométrique L’optique géoméinque est l’étude de la
dans toutes les directions. L’intensité des ondes à une di.stancc r lumière où les ondes lumineuses peuvent être représentées par des
d’une source ponctuelle de puissance est rayons lumineux.
L a réflexion ei la réfraction Gcncralenicnl. Kir.squ’un rayon
I = (4.27) lumineux rencontre une inierfaco entre deux milieux iraiisparciiis. un
47ГГ-
rayon réfléchi et un rayon réfracté apparaissoni. I es deux rayons
sont dans le plan d’incidence défini par le rayon incident et la nor­
Lu p ressio n de radiation Lorsqu’une surface intercepte un
male. L'angle de réflexion est égal à l'angle d'incidence, et l'angle
rayonncmeni électromagnétique, une force et une pression s’exercent
de réfraction est relié à l’angle d'incidence par
sur elle. Si le rayonnement est totalement absorbé par la surface,
le module de la force moyenne est nj sin = «, sin в, (la loi de la rctracnon). (4.41)

où ri| et «2 sont les indices de refraction des milieux dan.s lesquels les
F — ~ (l’absoqition loliile). (4.32)
f rayons incidents et réfractés se propagent.

où / est l'intensité du rayonnement et A est l’aire de la surface La réflexion totale interne Une onde lumineuse qui rencontre
perpendiculaire à la direction de propagation du rayonnement. Si le une interface où l'indice de réfraction du milieu dans lequel sc
rayonnement est totalement réfléchi le long de la trajectoire initiale, propagent les rayons réfractés, s’il y en a. est le plus faible subira
le module de la force moyenne est une réflexion totale in te rn e si l'angle d'incidence est plus grand
qu'un angle critique oii
2JA
F ^
I f^z (4.44)
= sin ‘ — d'angle critiqiic).
(la réllexion totale le long de la liajecioirc initiale). (4.33) Ht

O U € $ tlÔ f^ S

1. Si le champ magnétique d’une onde lumineuse oscUlc parallèlement 4. La figure 4.22 illustre des rayons
à un axe des y et est donné par B,. = sin(Â;<' <üt), a) ilans quelle lumineux monochromatiques tra­
direction fonde sc propage-i-clie et b) parallèlement à quel axe le versant trois substances, n. h et r
champ électrique associé oscille-l-il ? Classez ces substances en ordre
2. La figure 4.20 montre le champ électrique et le champ magné­ déeroi.ssant selon leur indice de
tique il'une onde électromagnétique à un instant donné. L'onde se réfraction.
propage t-clle en entrant ou en sortant de la page ? 5. Dans la figure 4.23, la lumière
se propage, à partir du matériau a,

П,
à travers trois couches d ’autres Figure 4.22 Q u e s t io n 1
matériaux dont les surfaces sont
parallèles, puis dans une autre couche du matériau a. Les réfractions
Rgwe 4.20 (Q uestion 2 (mais non les réflexions a.ssociées) aux surfaces stmi illus(récs.
Classez les matériaux en ordre décroissant selon leur indice de
3. La figure 4.21 présente les réflexions multiples d'un rayon réfraction.
lumineux dans des corridors de verre dont les murs sont soit parai
lèles. soit perpendiculaires les uns par rapport aux autres. .Si l’angle
d ’incidcnce au point « est de 30‘ . quels sont les angles de réflexion
du rayon lumineux aux points h. c, d. e e t / '.'

Figure 4.23 Question 5


100 Chapitre 4 La réflexion et la réfraction dé la lumiéré

b. Chaque situation de la figure 4.24 représente un rayon lumineux 8. La figure 4.15 représente certains rayons lumineux du soleil
qui est réfracté à l’interface entre deux matériaux. Le rayon incident produisant un arc-en-ciel primaire (qui implique une réflexion à
(en gris dans la figure) sc compose de lumière rouge et de lumière l'intérieur de chaque goutte d’eau). Un arc-en-ciel secondaire, plus
bleue. L’indice de réfraction approximatif pour la lumière visible [mlc et moins fréqueni (qui implique deux réflexions à rimérieur de
relativement à chaque matériau est indiqué. Laquelle des Irois chaque goutte d ’eau), formé par les rayons qui soTleiit des gouttes
situations représente une réfraction physiquement possible ? d’eau, comme le montre la figure 4.26 (sans indication de couleurs),
peut apparaître au-de,ssus d’un arc-en-ciel primaire. Lequel des
rayons a et b correspond à la lumière rouge ?

figure 4.24 Question 6

7. a) La figure 4.25 a) représente un rayon lumineux du soleil qui


pas.se 1(И11juste au-dessus d’un bâton vertical dans une piscine remplie
d'eau Г c rayon se termine-Ml dans la région du point a ou <lans celle
du point h ? b) Quelle composante de la luniicrc, entre la bleue et la
rouge, finit le plus près du bâton ? c) La figure 4.25 b) représente un 9. l-a figure 4.27 représente quatre longues couches horizontales de
objet plat (une lame de rasoir, par exemple) qui flotte dans l'eau différents matériaux, avec de l’air au-dessus et en dessous. L,’indice de
stagnante et est éclairé verticalement La force gravitationnelle léfract'ain de chaque matériau est diinné. Des rayons lumineux aiteigiienl
exercée sur l’objet et la cohésion de l’caii produisent la courbure les extrémités gauches de chaque couche, comme il est illu.stré. Dans
illustrée à la surface de l’eau. Dans quelle région {ii, b tai <;) se trouve quelle couche (indiquez l’indice de réfraction) est-il possible que la
le bord de l’ombre de l'objet? (À la droite de l’ombre, de nombreux lumière soit totalement retenue de sorte que, après de nombreuses
rayons liitnineiix du soleil sont concentrés et prwluisent une région réflexions, toute lu lumière atteigne rexirémité droite de la couche ?
spécialement brillante, que Гоп dit camtique.)

L3
M I П M
t.4
(T.
-•---c— • ----- 1,3
nhc
a) b) ,\ir
figure 4.25 Question 7 Figure 4.27 (Question 9

Ш З Ш Е Ш Ш .

«WW La Solution se trouve .sur le s ite Web. à l’adresse ci-dessous : surface d'environ 10 (XX) km-. Llle avait pour fonction de transmettre
des signaux aux sous-marins en profonde immersion. Si la Itxigueur
w w w .d lc n K g ra w h ill.c a /p h y s iq u e
d'onde dans l’air était de 1,0 x 10* rayons de la Terre, quelle était
a) la fréquence et h) la période du rayonnement émis ? HabiUiellentent,
SEOION 4.1 le spectre éledromognétique les rayonnements électromagnétiques ne pénètrent pas facilement dos
lï. a) Combien de temps taut-il à un signal radio pour franchir une condiKteurs tels que l'eau de mer.
distance de 150 km entre un émetteur et une antenne de réception 3E. a) A quelles longueurs d’onde l’œil d’un observaieiu moyen a-t-il
b) La pleine lune est visible grâce à la refiexion de la lumière du la moitié de sa sensibilité maximale b) Quelles sont la longueur
soleil. Combien de temps s’écnule-l-il entre l'instant où la lumière d’onde, la fréquence et la période de la lumière â laquelle l’œil est le
quitte le Soleil et celui où vos yeux l’interceptent ? La distance entre plus sensible ?
la terre et la l.une est de l.X x IO-* km. et celle entre la Terre e,i le 4E. lin la,scr à hélium néon émet une lumière rouge dans une très mince
■Soleil est de 1,S V Ky* km. c) Quelle est la durée du trajet aller-rctoiir plage de longiK'urs d'onde centrée à 632,8 nm et couvrant une « largeur
de la lumière entre lu Teire et un engin spatial en orbite auloui de bande ». exprimée en longueurs d’onde, de 0,010 0 nm. Quelle est
dr .Saturne, qui est à 1. 1 v 10’ km de la Terre '' d) La nébuleuse la « largeur de bande» exprimée en fréquences corre.spondani à celle
du ( 'rahe qui est environ à f> 500 années lumière (al) de ta lèrrc, émission ?
semble c ire le résultat tic l’explosion d’une supernova observée 5P. Une méthode utilisée pour mesurer le module de la vitesse de la
pui des ustronnnic.s chinois tu l’an 1054. F,n tiiicllc année (approxi- lumière, basée sur le-s observations de Ole Roiik r en 1(S7ft, consistait
nmiivcmcni) l'explosion s'est-elle reollemem produite ' à observer tes durées apparentes des révolutions d’une des lunes de
2i. Le projet Sealurcr consistait en im audacieux pmgramnK visant Jupiter. La période exacte de révolution est de 42.5 h. a) PuisqiK la
à construire une énorme antenne souterraine sur un site ayant une vitesse de la lumière est finie, dites qualitativement de quelle façon
Exercices et problèmes 101

la durée apparente d ’une révolution devrait changer lorsque la Terre présente un angle de dispersion de 0,880 /rrad, quelle est l’aire de
SC déplace sur son orbite du point jc au point y, dans la figure 4.28. la surface de la Lune éclairée par le rayon ?
b) Quelles observations doivent être faites afin de calculer le module
de la vites.se de la lumière ? Négligez le mouvement orbital de Jupiter.
La figure 4.28 n'est pas à réchcllc.

Figure 4.29 Exercice 13


Orbite V
d<!Jupiter \
14E. Quelle est l’intensité d’une onde électromagnétique plane si /),„
vaut I.Ü X lO"’' T ?
15E. Dans une onde radio plane, l’amplitude de l.a composante
X 1 T erre A , L / - l.une
I / d e Jupiter
du champ électrique c.si de 5,00 V/m. Calculez a) Fampliludc de lu
\ i composante du champ magnétique et b) Fintensité de Fonde.
\ /
\ / 16P. La lumière du soleil a une intensité de 1,40 kW/ni^, juste à
\ ✓
l’extérieiu de Faimosplière terrestre. Calculez E,„ et 5„, relativement
à la lumière du soleil à cet endroit, en supposant qu’il s agit tl'iine
onde plaiK.
figure4.28 Problème!) 17P. À une distance de 10 ni d ’une source lumineuse ponctuelle
isotrope, le module du champ électrique maximal est de 2,0 V/m.
Quels sont a) le module du champ magnétique maximal et b) Fintcn.sitc
SECTION 4.2 L'aspect quaiitolif des ondes électromagnétiques progressives
moyenne de la lumière à cet endroit’’ c) Quelle est la puissance de la
6t. Quelle est la longueur d’onde de l’onde électromagnétique émise source'? www
par le système antenne oscillateur de la figure 4.3, si /, — 0,253 /xH I8P Frank D. Diake, inspecteur dans le programme SFTI (Search for
e t r = 25,0 pF? Extra-Terrestrial Intelligence, ou programme de recherche d'intelli­
7L Quelle inductance doit être reliée à un condensateur de 17 pF gences extraierre.stres), a un jour «léclaré que le grand radiotélescope
dans un oscillateur pouvant générer des ondes électromagnétiques d ’Arecibo (voir la figure 4.30). à Porto Rico, «pouvait détecter
de 550 nm (donc visibles) ? Commentez votre réponse. un signal qui a. sur toute la surface de la "lerre, une puissance de
seulement un picowatt». a) Quelle est la puissance que l ’antenne
d’Arecibo recevrait d’un tel signal?Le diamètre de l'antenne e.si de
SECTION 4.3 L'ospect quantitolif d«s ondes életlromognétiques progressives
300 m. b) Quelle serait la puissance d'une .source qui pounaii émettre
8f. Une onde électromagnétique plane a un champ électrique maximal un tel signal du centre de notre Galaxie ? Le centre de la Galaxie se
ayant un module de 3,20 x 10’ V/m, Trouvez le module de .son trouveà2,2 x 10* al de la Ferre. Supposez que la source est isotiope
champ magnétique maximal.
9L Le champ électrique d ’une onde électromagnétique plane est
donné par = 0 ; t ’v = 0 ; £ , = 2,0 cos[jr x 10‘'(r — .r/c)|.
où r = 3,00 X 10“ m/s et où toutes les qiiantité.s sont en unités SI.
L’onde .SC propage dans la direction positive de l'axe des x. Écrivez
les expressions des composantes x, y et z du champ magnétique
de Fonde, wviw

SECTION 4.4 le Itansporl de l'énergie et le vecteur de Poynting

lOE. Démontrez, en trouvant la direction du vecteur de Poynting.


qu’en tout point des figures 4.4 à 4.7 les directions du champ électrique
et du champ magnétique sont cohérentes en tout temps par rapport
aux directions de propagation de Fonde données.
IIL Certains lasers au vcrrc-néodyme peuvent fournir 100 TW de
puissance en 1,0 ns, produiwint ainsi une très courte impulsion ayant
une longueur d’onde de 0.26 fum. Quelle quantité d’énergie y a-i il Figure 4.30 Problème 18 I .e radiotélescope d’Areeibo
dans cette impulsion ? www
19P. L'n avion qui vole à une distance de 10 km d'un émetteur radio
12t. L'étoile la plus rapprochée de la Terre, Proxima du Centaure,
reçoit un signal d’une intensité de 10/yW/m^ Cali-ulez al l’ampliriide
est distante de 4,3 al. Il semblerait que des signaux de télévision
du champ électrique produit par ce signal à l ’endroit où se trouve
de notre planète aient pu atteindre cette étoile, et être captés par
I avion, h) l’amplitude du champ magnétique au même endroit et
d’éventuels habitants d'une planète hypothétique en orbite autour
c) la puis.sance totale de l’émetteur, en supposant qu’il émet de façon
de l’étoile. Si l’on suppose qu’une station de télévision sur Ifcrrc
isotrope, wwy/
a une puissance de 1.0 MW. quelle serait l’intensité de ce signal sur
Proxima du Centaure
SECTION 4.5 La pression de rodioticn
13E. l e rayonnement émis par un laser se disperse en prenant la
forme d ’un cône étroit dont la section transversale est circulaire. 20t Un morceau de carton noir parfaitement ahsoibani et ayant une
L’angle 6 du cône (voir la figure 4.29) est l’angle de dispersion surface d ’aire A = 2,0 cm ' intercepte une lumière d ’une intensité
du faisceau. Un laser à 1 argon, rayonnant à 5 14.5 nm, est dirigé vers de 10 W/m- émise par un éclairage de caméra. Quelle pression de
la Lune daus le cadre d'une expérience scientifique. Si le rayon radiation esl produite pai la lumière sur le carton?
102 Chapitre 4 La reflexion el la refraction de la lumière

21t. Des lasers ilc grande puissance sont miliscs pour comprimer un 28P. Prouve/ que Lapression moyenne d'un jet d ’une séné de pmjcciiles
plasma (un gaz ilc particules chargCesl par prcvsion tic radiation frappant perpcndiculairemem une suiiace plane v-aut le double de la
Un laser géncraiil des iinpulsiuas d'une puissance de 1.5 x 10* MW densité d'énergie cinétique dans le jet avant qu'il frappe la surface.
est fosalisc sur 1,0 mm^ de plasma à haute densiie électronique. Suppose/, que les projectiles sont totalement absorbés par la surface.
Trouvée la prcssitiii cxcicéc sur le phi.siiia si ce dernier réfléchit toute Corapaie/ ce problème avec le problème 27.
la liiiuicrc dircctciiicm le long de sn trajectoire initiale 29P. Un petit engin spatial d'une masse de 1..5 x 10* kg (avec un
22L Le layonneinent du soleil qui aiieinl la Terre (juste à l'extcneur astronaute à btird) dérive dans l’espace. Les forces gras liaiionnelles
de rainiosphcrc) a une inicnsité de 1.4 kW/m- a) En supposant que qui s'exercent .sur lui .sont négligeables. Si ru.stroiiautc allume
la Terre (cl son aimosphère.) se comporte comme un disque plat un rayon la,ser de 10 kW. quel sera le module de la vitesse que
perpendiculaire aux rayons du .soleil et que toute l'énergie incidente l’engin atteindra en 1.0 jour à cause de la quantité de iiiouvcnicm
est absoitiéc. calculez le module de la force exercée sur la Terre par iran.sportéc par le rayon ia.scr?
la pressioii de radiation, h) Comparez cette force à celle exercée par 30P On a proposé qu'un engin spatial soit propulsé dans le système
rattraclion gravtlationnclle du Soleil solaire par la pression de radiation à l'aide d ’une grande voile faite de
23L Quelle est la pression de radiation à 1.5 m de distance d'une feuilles métalliques. Sncbani que la puissance de la railiarion émise
ampuulc clti trique de *'00 W ’ Siipfiose/ t|ue la siirtacc sur laquelle par le Soleil est de 3.9 x lO^*' W. quelle dimension la voile doit-elle
s'exerce la pression taisant face à l'am poule est partaiiemciu avoir pour que le module de la force de radiation soit égal à celui
ahsorhante et que l'am poule émet »on rayonnement de façon de l’attraction gravitationnelle du Soleil’.' Supposez que la ma.sse de
isotrope, wvfw l’engin et de la voile est de I 500 kg. que ht voile est parhaiicment
24f Un laser à hélium-ncnn. du tj'pc des lasers qu’on trouve .souvent rcfléchissanie et orientée perpendiculairement aux rayons du soleil.
dans lofi lühoratotrcs de physique, u une (niissuncc de rayonneinenl de (Voir l’annexe C pour les autres données requises.) (Avec une plus
5,00 niW et une longucui d'onde de 6.13 nm Le rayim est focalisé par gronde voile, l’engin s’éloignerait continudlcincnt du Soleil.)
une lentille afin de prixliiire un point lumineux dont le diamètre est 31P. Une particule dans le système solaiic subit l’intlucncc coinbincc
approxiniaiicernent égal à 2 longueurs d'onde. Calculez u) l'intensité de l'aitraction graviiaiionncllc du Soleil et de la force de radiation
du rayon laser localisé, b) lu pression de liKlifltion exercée .sur une sphèm exercée par les rayons solaires. Supposez que la particule est une
puifaitcmciit ah.sorhaïuc diml le diamèire est égnl à celui du point sphère dont la masse v olumique est de 1.0 x 10* Lg/m*. et que toute
lumineux, c) le module tic ht force exercée par le rayonnemem sur celle la lumière incidente est absorbée, a) Démunirc/ que, si son rayon
sphère cl il) le module de l'accélération produite par le rayonnemem est infeneur à un certain rayon critique r. la particule sera éjectée
Supposez que la .sphère a une ma.ssc volumique de 5.(X) x 10*kg/in’. du système .solaire, b) Calculez le rayon criiiquc. La puissance du
25P. Une onde élecirom.agnétique plane, dont la longueur d'onde rayonnement émis par le Soleil est de 3.9 x lO-'’ W.
est (le 1,0 m, se propage dans le vide dans la direction de l’axe
des X posiiils. et son champ ciccthque Ë d'une ampliiudc de 300 \/m SEGION 4.6 La réflexion el la reftoclion
est tirienié le long de l’axe des y. a) Quelle est la fréquence /
32L La ligure 4.32 pré.senic un rayon lumineux qui est réfléchi
de l'onde b) Quelle» sont la direction ci l'am plitude du champ
sur deux sui faces. A et fi. perpendiculaires Trouvez l’anglc entre le
magnétique associé à l'onde'.* c) Quelles sont les valeurs tic A:et de <o
rayon incident i et le rayon sortant /'.
ai E — siniLt — Ml) d) Quelle est l’intensité de celte onde
e) St l'ixidc rcncimtrc une feuille parfaitement absorbante ayam une
surface de 2.0 m-, à quel taux la quantité de mouvement esi-clle
traasmisc à la feuille ci quelle est la pression de radiation exercée »ur
la feuille'.’ yrww
26P. Dans In figure 4 .1 1, un rayon laser d'une puissance de 4.60 W et
d’un diamètre de 2.60 mm est dirigé vers le haut sur la base circulaire
d'un cyhiidrc parfaiiemcnl réfléchissant (d'un diamètre d < 2.60 mm),
qui lévite grâce à la pression de radiation du rayon laser. Ln masse
volumique du cylindre est de 1.20 g/cni* Quelle est la h.aiiieiir II 33E Un faisceau lumineux dans le vide est incident à la surlacc d’une
du cylindre '} |Woi de verre. Dans le vide, le faisceau forme un angle de 32.0'' avec
la normale à la surface, el dans le verre, il forme un angle ilc 21,0'
2.60 mm -----
avec la normale. Quel est l'indice de réfraction du verre’’
34t Vers l’an 1.50 .après J. C.. Claude Ptoléntée mesura les valeurs
-- suivantes de l'angle d’incidence B, et de l’an g icd e réfraction f),
d'iin faisceau lumineux passant ik' l’air à l'eau;
H
e. A fl.

m
figure 4.31 Problème 26
l( r
20°
30"
8“

22"V)'
50°
60’
70°
3.5"
40° .«V
45'30'
40" 29" 80° 50"
22P Prouvez que, dans le vas d'une onde électruraagnéiique plane
micignaiii pci|>cndit nlaireineni une sut lace plane, la pression de a) Ces données soiii-clles cohérentes avec la loi de la réfraction?
r.adialion sur la surface est égale à la densité d'énergie tic radiation h) Si oui. quel est l'indice de réfraction de l'e a u ? Ces données
près de la surface. (Cette relation entre la pression cl lu densité sont intéressantes car elles ennstitueni peiit-èire les plus anciennes
d'énergie s’npplu|uc peu importe la Iraclion d'énergie incidente qui mesures de physique à avoir été consignées. (Notez, que 30' signitic
est réfltx;hic ) .10 minutes d'arc, soit 0,5'’ )
Exercices et problèmes 103

3Sf. Le réservoir rk métal rectangulaire de la figure 4.33 est rempli sur sa direction initiale, mais en étant décalé latéralement, comme le
au maximum d’un liquide inconnu, et un observateur dont les yeux montre la figure 4.36. Démontrez que, d.'ins le cas de petits angles
sont à la hauteur du dessus du réservoir ne peut voir que le coin h \ d'incidence 0 . ce décalage latéral est donné par
on aperçoit dans la figure un rayon réfracte à la surface du liquide se
dingeaot vers l’observateur. 1 nmvez l'indice de réfraction de ce liquide. X = tft-

N orm ale à ta surface


où n est l'indice de réfratiion du verre et ffest mesuré en radians, www

1" — •
36P. Dans la figure 4.34, une lumière incidente torme un angle figure 4 36 Problème .IV
«I *• 4(1. I* à la frontière entre deux matériaux transparents. Une
partie de la lumière se propage alors vers le bas dans les trois autres 40P. Dans la figure 4 37, deux miroirs perpendiculaires et à 45*
couches de niaicriaux inmsparenis. et une partie de cette lumière est de rh o n zo m ale tormeni les cOtés d’un récipient rempli d’eau
réfléchie vers le haut cl s’échappe dans l'air. Quelles sont les valeurs a) L'n myon lumineux provenant du liaiii est incident scion lu noriiuilc
de a) et b) à la surface de l’eau. Démontrez que le imyon émergent est parallèle
au ravon iiicidcnL Supixtsc/. qu'il se pioduii deux réflexions à la surface
des miroirs, b) Reprenez votre analyse dans le cas d ’une incidence
obbque, où le rayon inckleni est dnn.s le plan de la tigiivc.

figure 4.37 Problème 40

41P. Dans la figure 4 3R. un rayon c.si incident sur une face latérale
d un pnsme de verre iriangiilairc qui .se trouve dans l'air i.'angle
d'ineidence fl est choisi de manière que le rayon émergent forme
V f Dans la figure 4.35, un poieuii vertical d’une loiigueiii de 2,OC) m le mf'me angle fl avec la normale de l'autre côté. Démontrez que
est fixé au fond d'une piscine et culmine à 50,0 cm au-dessus l’inditc de réfraction « du prisme de verre est donné pur
de l'eau. La lumière incidente du soleil forme un angle de SS.O'’
s in ( ^ ( f f <^)J
«U dc.ssu.s de l'horiKon. Quelle c.st la longueur de l’ombre du poteau n =
au fond de la piscine ? sin
où 4>CM l'angle ou sommet du pnsme, et iff est langle tie dé\iatum
c'est-à-dire l’angle total par lequel le rayon est dévié en traversant le
prisme. (Dans ces conditions, l'angle de déviation V' « la ph'« petite
valeur possible, que l'on immnK tmitle de dcviaùnn minimnle.) www

Air

38P ITn rayon de lumière blanche forme un angle d'incidence de 35°


avec un côté d'uii prisme de verre de silice (quart/ fondu) ; la section
transversale du prisme forme un triangle équilatéral. Faite.s un crtxjuis
de la lumière qui traverse le pri.sme, en indiquant le.s parcotirs suivis
par les rayons représentant u) la lumière bleue, b) la lumière jaune-
verte et c) la lumière rouge. SECTION 4.7 lo réflexion totale inlotne
39P Prouvez qu'un rayon lumineux incident à la surface d'une pbque 42f L indice de réfraction du benzène est I .g. t^iol est l'angle cniiqu
de verre d'épaisscui t émerge de l'autre côté de la plaque parallèloncnt d un rayon lumineux traversant le benzène en direction d'une cuuch
104 Chapitre 4 La réflexion et la réfraction ae la lumière

d’air située au-dessus ? L'iiucrfaec entre les deux milieux est uniforme 49P. Dans la figure 4.41. un rayon liiiniiicux cuire dans un prisme au
et plane. point P avec un angle d ’incidence 0, l’angle au sommet du prisme e.si
43f. Dans la figitre 4..3‘J, un rayon lumineux entre dans un bkx; de verre de 00“ et une partie de la lumière est réfractée au point Q avec un
au point A et subit ensuite une réflexion totale interne au pt>int B. angle de réfraction de 90“. a) Quel est l’indice de réfraction du
D'après ces données, quelle doit être la valeur minimale de l’indice prisme exprimé en fonction de «? b) Quelle est. numériquement, la
de réfraction du verre ? valeur maximale que l’indice de réfraction peut avoir'' Expliquez ce
qui arrive à la lumière à Q, si l’angle d ’incidence à P c) augmente
légèrement et d) diminue légèrement

..

Figure 4.41 Problème 49

Problèmes supplémenloires

50. Dans la figure 4.42, un albatros plane horizontalement au-dessus


44t. Une source pontluclle de luniicrc sc trouve au fond d’uiK piscine du sol à une vitcs.se constante ayant un tnodule de 15 m/s. Il se déplace
à 180 cm sous la surface de l’eau Trouvez le diamètre du disque dans un plan vertical dans lequel se trouve également le Soleil.
d’où la lumière émerge à la surface de l’eau. Il plane vers un mur d’une hauteur h = 2,0 m, qu’il parviendra tout
4SI. Dans la figure 4.40, un rayon lumineux c.si perpendiculaire au juste à survoler. A ce moment de la journée, le Soleil forme tin angle
cOté ub d'un prisme de verre (« — 1.52). TVouvez la valeur maximale 0 de 30“ avec le sol Quel est le module de la vitesse de I ombre de
de l’angle é permettant au rayon d’Ctre lutalenicnt réfléchi sur le l’albatros qui se déplace a) sur le sol et b) le long du mur"’ Supposez
côté ur si le prisme est a) dans l'air et h) dans feau. www que, par la suite, un faucon vient planer en suivant le même parcours,
et à la même vitesse. Vous remarquez que, lorsque son ombre atteint
le mur, le module de la vitesse de l’ombre augmente sensiblement,
c) Le Soleil est-il maintenant plus haut ou plus bas dans le etc! par
rappoit au moment où l’albatros planait? d) Si le module de la vitesse
de l’ombre du faucon sur le mur e.st de 45 m/s, quel est, à cei instant
précis, l’angle 6 du Soleil''
Figure 4.40 Exercice 45
Ravon du soleil
4éP. Un rayt'n de lumière blanche se propage dans un verre de silice ------------
(quartz fondu) entouré d’air. Si toutes les compt>santes de la lumière
subissent une réllexion totale interne à la surface. la lumière réfléchie
forme alors un raytni réfléchi de lumière blanche. Toutefois, si la
composante île couleur à une extrémité de la bande visible (bleue ou i
rouge) est partiellement réfractée et continue dan.s l’air après avoir
Figure 4.42 Problème 50
traversé la surfac'e, une fraction irnporiantc de celle-ci disparaît dans
la lumière réfléchie. La lumière réfléchie n’est plus blanche dans ce
51. À la redurche de tombeaux. Dans un site de fouilles archéolo­
cas. mais prend la teinte de l'extrémité opposée de la biUidc visible.
giques, on peut hrcaliser et répertorier des tombeaux non identifiés
(Si Je bleu disparaît partiellement lors de la réfraction, le rayon
à l'aide d ’un l’éonidar sans perturber le site, l e géoradar émet
réfléchi est alors rougeâtre, et vice versa.) Est-il pos.sible que le rayon
une impulsion d ’onde éiectromagnetique directement dans le sol ;
réfléchi soit a) bleuâtre, ou b) rougeâtre c) Si oui, quel doit être
l’impulsion est ensuite partiellement réfléchie vers le haut par toute
l'angle d'incidence de la lumière blanche initiale à la surface du
interface souterraine Cela signifie qu’une impulsion est réfléchie
quartz?(Voir la figure 4.12.)
vers le haut par toute frontière horizontale qui modifie la vitesse de
47P Un cube de verre, dont les côtés mesurent 10 mm et dont l'indice l'inipulsion, L'in.siruiiKnt détecte la réflexion et enregistre l’intervalle
de réfraction e.st 1,5. possède une petite bulle d ’air à son centre, (le temps entre l’émission et cette détection. En répétant l’expérience
a) Quelles parties de chai|uc côté du cube doivent être couvertes afin en plusieurs endroits, un archéologue peut déterminer la forme des
qu’on ne puisse voir la bulle d'air, peu importe la direction d’où on structures souterraines.
a'gartfc le cuIk: ? (Ne tenez pas compte de la lumière qui est réfléchie Un instrument .semblable a été utilisé à huit endroits le long
â rintérieiirdii otibe puis réfractée dans l’atr) b) Quelle fraction d’une ligne droite au niveau du sol. et ces endroits ont été numérotes
de Ifl .surface du cube doit être aiiiii couverte ? d’ouest en est, comme le montre la figure 4 43.11 y a une distance de
ittîl* Supposez que le pnsme de la figure 4.3R a un angle au sommet 2,0 m entre chaque endroit. Un tombeau vide, composé de parois de
tf>= 80.0“ et un indice de réfraction ti = ! .60. •}) Quel est le plus pierre horizontales et verticales de même épaisseur, se trouve .sous
petit angle d’incidence d pcniicttani à un rayon d ’entrer par le côté CCS endroits ; les parois horizontales forment la base et le dessus du
gauche du prisme et d ’en ressortir par le côté droit ? b) Quel angle tombeau, et les parois verticales en forment les murs. Le tableau
d'incidencc 0 doit on avoir pour que le rayon sorte du prisme suivant indique les intervalles de temps Ar (en naiK'sccondes) enre
avec un :ingle de réfraction identique 0, comme, on le voit dans la gLstrés pont les impulsions émises aux huit endroits À l’endroit 4,
figure 4 38 ? (Voir le problème 41 ) par exemple, l’impulsion initiale émise daas le sol a pnxiuit ((iiatre
Exercices et problèmes 105

impulsions rcflcchics, la première étant détectée 63,00 ns après


Endroit I 5 8
l’émission et la dernière, 86.54 ns après l'émission.
Supposez que les impulsions ont une vitesse ayant un module de Ai Aucune 63,00 bi,0ü 63,00 63.00 63.00 63,00 Auaine
10,0 crn/n.s = c/3 dans le sol au-dessus, en dessous et à cfilé du
115.8 66,77 66,77 66.77 66.77 93,19
tombeau, de 10.6 ern/ns à travers les parois de pierre, et de 30 cm/ns
dans l’air du tombeau. Trouvez a) la profondeur de la surface 82,77 82,77 74.77 74.77
supérieure du dessus du tombeau, b) la longueur horizontale du
tombeau le long de la ligne est-ouest comprenant les huit endroits et 86.54 86,54 101.2 78,54
c) les dimensions verticales de l’intérieur du tombeau.
52. À quelle distance, environ, devez-vous tenir vos deux mains
Est l’une de l’autre pour qu’elles .soient séparées de 1,0 nanoseconde-
6 T 8 lumière ?

figure 4.43 P ro b lè m e 51
5 Les miroirs
et les lentilles

Depuis sa création, en 1881, le tableau BarauxFolies-Bergèred'Édouard Manet n'a cessé d'éblouir les amoteurs d'œuvres
d'art. Cette foscinotion est créée en partie par le contraste qu'on y observe entre un public d'abord soucieux de se divertir
et une serveuse dont les yeux trahissent la fatigue. Le charme du tableau repose également sur une distorsion subtile
de la réalité que Manet sovait
intégrer à ses peintures-
une distorsion qui donne
à la scène une atmosphère
troublante, avant même
qu'un observateur puisse
distinguer ce qu'il y voit
d'« anormal».

Quelles sont ces subtiles


distorsions de la réalité?
lo répnm« « trouve dons ce (hopibe.
5.1 Définitions 107

5.1 Définitions
P o u r q u e vous p u issie z v o ir un o b jet - un p in g o u in , p a r ex e m p le - vo tre œ il d o it
intercepter certains rayoœs lum ineux provenant du pingouin et les diriger vers la rétine,
à l’intérieur de rtu il. Votre système visuel nerveux, qui com m ence son travail dans la rétine
cl le term ine dans le cortex visuel, qui est situé dans la partie postérieure de votre cerveau,
traite de fitçon incoascienle cl automatique l’inform ation donnée par la lumière. Ce système
reconnaît les limites des objets, de m êm e que leurs ta ienlaüons. leurs textures, leurs formes
e t leurs couleurs, cl im pose à votre conscience l’im ag e du pingouin (une reproduction
dérivée de la lu m ière): vous reconnaissez le pingouin et le percevez dans la direction
d ’où proviennent les rayons lum ineux, et à la bonne distance.
Votre systèm e visuel traite ces inform ations m êm e si les rayons lumineux ne viennent
pas directem ent du pingouin, com m e lorsqu’ils sont rétléch is vers vous par un miroii
ou réfractés par les le n lilk s d ’une paire d e jum elles. Vous voyez alors le pingouin dans
Figure 5.1 Deux types, de faisceaux : la d irection d ’où v iennent les ray o n s luinineiix après av o ir été réfléchis ou réfractés,
a) divergent ; M ainvergciil m ais la distance que vous percevez à ce m om ent peut être très différente de la distance
réelle du pingouin.
P our a n a ly se r les p h én o m èn e s o p tiq u e s en je u . il fau t d ’abord é ta b lir ce rtain e s
définitions. L ’ensem ble des rayons lum ineux issus d ’un m êm e point ou allant vers un
Sysurme m em e p oint s’appelle faisc ea u lu m in eu x . f.a figure .“i.! a) illustre un fai.sccau divergent
OptKjUC
et la figure 5.1 b) illustre un faisceau convergent.
U n .systèm e o p tiq u e e s t un d is p o s itif qui d év ie le faisceau lu m in eu x in cid en t.
I x faisc ea u p eu t être d év ié à la su ite d ’une réflex io n (m iro ir) ou d ’une ré fra c tio n
lo rsq u ’il traverse un m ilieu d ’indice différent (lentille, d io p tie, œ il, etc.). Un sy.slème
o p tiq u e peut cré er u ne im ag e à p artir d ’un o b je t d o n t les c a ractéristiq u es (p o sitio n ,
grandeur, orientation et nature) peuvent être différentes de celles de l’objet.
a) O bjet réel ; /> > 0 O n distin g u e deux types d ’objets. O n reconnaît un o b je t ré e l loraquc le faisceau
in c id en t est d iv e rg e n t (fig u re 5.2 a). U ne ch a n d elle qu e l’on p la ce devant un m iro ir
Système constitue un exem ple d ’o b jet réel. L e faisceau issu de chaque point de la chandelle est
optique d iv e rg e n t lo rsq u ’il est réfléc h i par le m iroir. La d istan c e en tre l'o b je t et le sy stèm e
o p tiq u e s ’ap p e lle d is ta n c e o b je t, d ésig n ée p a r la le ttre p. P o u r un o b je t réel, cette
O
d istan c e est p o sitiv e. Par co n tre , si le faisceau a rriv a n t su r le sy stèm e o p tiq u e est
convergent (figure 5.2 b), on a un o b je t v irtu e l. Fl faut prolonger les rayons incidenus de
l’autre côté du systèm e o p tiq u e p o u r d éterm in e r le p o in t d ’in tersectio n qui d onne la
------ P- position de l’objet virtuel. La distance objet p pour celle situation est négative.
h) O b jet virtuel ; /> < ü Il existe aussi deux types d ’images. O n o btient une im a g e réelle si le faisceau qui
ém erge du systèm e optique est convergent (figure 5..1 a). La d ista n c e im a g e , désignée
Figure 5.2 I.e.s deux types d ’objets :
par q. est d éfin ie co m m e la d istan c e en tre le sy stèm e o p tiq u e e t l’im ag e fo rm ée.
a) réel ; h) virtuel
C elle disuince e st positive lorsque l’im age form ée est réelle. L ne im a g e v irtu e lle est
form ée lorsque le faisceau qui ém erge du systèm e optique est d ivergent (figure 5.^ b),
l e point d ’in te rsec tio n du p ro lo n g em en t des ray o n s ém erg en ts d o n n e la p o sitio n
Svstènie de l’im age. La distance im age q est négative p o u r une im age virtuelle.
opti()u(-
I .a ( onventinn de signes utilisée pour les distances objet et im age est donc la suivante :

► Une distance objet p ou image ç est positive .si l’i'bjet (ou l’image) eorrespondani est réel.
Une distance objet p ou image q est négative ,si l’objet (ou l’image) coiTcspomiant est virtuel.

a) Im age réelle; ç > O


Un mirage fréquent
Svstcinc
optique Voici un exem p le as.se/, fréqiienl d 'im a g e v im ie llc ; une flaque d ’eau apparaît dev an t
vous, à une ce rta in e d istan c e sut la ro u te, lors d ’une jo u rn é e e n so le illé e et v o u s ne
pouvez ja m ais l’atteindre L a flaque e st un « tirag e (un type d ’illu sio n ) form é p ar les
rayons lum ineux provenant d ’une région peu élevée du ciel devant vous (figure 5.4 a).
F.n ap p ro ch an t d e la ro u te, d o n t la tem p ératu re est h ab iliieliem en l assez é le v é e , les
K '/ rayons reneontrent de l’air rie plus en plus chaud. L orsque la tem pérature de l ’air au g ­
m ente. la v itesse de la lu m ière d an s l’air au g m en te au ssi lé g èrem en t et l’in d ice de
b) Im age vhiuelle; </ < O
réfiaction de l’air, quant à lui, dim inue légèrem ent. I x s rayons qui dcscendciil en ren­
Figure 5.3 Les deux types d’images : co n tran t des indices de réfraction de m oins en m oins élev és sont alors dév iés d an s la
a) réelle, b) virtuelle direction h o ri/im tale (figure 5.4 b).
10В Chapitre 5 Les miroirs et les lentliles

Figure 5.4 a) Un rayon pmvenant Kavoii lum ineux


d’une région peu élevée du eiel
est réfracté en traverstuit l'air réchauffe ' Mirage la llaque
I le Route
par la route (sans atteindre lu route).
a)
Un observateur <jui intercepte la lumière
la perçoit comme s’il .s'agissait
d’une llaque d’eau sur la roirtc. Rapide

//tív
C haud CJraïul
b) UeviatitTit (exagérée) d'un rayon
lumineux se dirigeani vers le ba.s
et iruvcr.siuU une fronlière imaginaire
enue l'oir chaud cl l'air plu.s chaud. Flus rapide
c) Dcplacenicnt des fronts d'omle
et déviaiion associée d'un rayon, b) «) d)
causée par le déplacement plus rapide
des cxiréiiiitéa basses des fronts d'unüc
Les rayons, qui se dirigent maintenant horizontalement un peu au-dessus de la surface
dans l'air plus chaud, d) Déviation
de la route, .sont dévies de nouveau, c a r la partie inférieure d e chaque from d ’o nde qui
d'un rayon qui remonte en traversant
leur est associé sc trouve dans l ’air légèrem ent plus chaud et se déplace légèrem ent plus
une froniicir imaginaire entre l'air
vite q u e sa p artie supérieure (fig u re 5.4 c |) . C e m ouvem ent non u n ifo rm e d es fronts
plus chaud et l’air chaud.
d 'o n d e fait d év ier les ray o n s vers le haut. L es rey o n s c o n tin u e n t à ê tre d év iés vers
le haut en rem ontant, ca r ils traversent des indices d e réfraction progressivem ent plus
élevés (figure 5.4 d |).
Si vous in te rc e p te z une p artie de cette lu m ière, v o tre sy.stème v isuel en d éd u it
auttinialiquenient q u ’elle constitue un prolongem ent des rayons que vous avez interceptés
et suppose, pour donner un sens à cette lum ière, q u ’elle vient de la surface de la route.
Si la lum ière est teintée de bleu à cau se du ciel, le m irage sem ble b leu , com m e l’eau.
Fhiisqiie la chaleur cause souvent une turbulence de fair, le mirage scintille, com m e .s'il y
avail d es vagues à la surface d e l ’eau. L a teinte bleutée et le scin tillem en t renforcent
l’illusion d 'u n e flaque d 'e a u , m ais ce qu e v o u s voyez e s t en fait une im age virtu elle
d ’une région inférieure du ciel.

5.2 Les miroirs plans


Lin m iro ir est une surface qui peut réfléchir un rayon lumineux dans une direction au lieu
de le disperser dans plusieurs directions ou de l’absorber. Une surface m étallique brillante
agit com m e un m iroir ; un mur d e béton ne le fait pas. D ans cette section, on exam inera
les im ages q u 'u n m ir o ir p la n (une surface téfléchissam c plane) peut produire.
l a figure 5..5 présente une source ponctuelle de lum ière O, q u ’on appellera Vobjet,
située en face et à une distance perpendiculaire p d 'u n m iroir plan. La lum ière incidenlc
à la surface du m iroir est représentée par des rayons qui sc propagent en provenance de O
l-a réflexion de cette lumière est représente^: par des rayons qui ,se propagent en provenance
du miroir. Si l’on prolonge les rayons réfléchis vers l’a n iè re du miroir, on observe que
les prolongem ents form ent une intersection à une d istan ce p erp en d icu laire q d errière
ce miroir.
Si vous regardez dans le m iroir de la figure 5.5, vos yeux interceptent une partie de
la lum ière réfléchie. Pour d onner un sens à ce que vous voyez, vous percevez une source
p o n ctu elle de lu m ière situ é e au p o in t d ’in te rsectio n du p ro lo n g em en t d es ray o n s
rcllcchis. C ette source ponctuelle co n stitu e l ’im age / de l’o b jet O. O n l ’appelle point
image parce q u ’il s’agit d 'u n point, et il s ’agit d ’une im age virtuelle puisque les rayons
Figure 5.5 Une «tunee ponctuelle réfléchis (qui ém ergent du systèm e optique) sont divergents.
de lumière O. qu'on appelle Voltjei. La figure 5.6 présente deux rayon.s choisis parmi les nom breux raytins de la figure 5.5.
située en face cl à une distance L’un d 'e u x atteint le m iroir perpendiculairem ent au point b. l ’autre l’atteint à un point
perpendiculaire p tl'un miroir plan. q u elco n q u e a, en form ant nn angle d 'in c id e n c e в. Le p ro lo n g em en t des deux rayons
Les rayons lumineux issus de O réfléchis est égalem ent illustré, I.es triangles rectangles Oba et Jba ont un côté com m un
.«lin rrficchiv lorsiju'iis aiieignent
et trois angles égaux, et sont, de ce fait, congruents (identiques), et leurs côtés horizon­
le miroir. Si voiie leil intcacpie
taux ont donc la m êm e longueur. f)onc.
tcriains de l es rayons réfléchis,
vous percevez une :>ourcc iNiiicliiclIr Ib = Ob, (5 I)
de lumière / dcM ieic le miroir,
cl une di.siaive perpendiculaire q. o ù ¡h et O h .sont les d istan ces resp ectiv es entre le m iro ir et l'im a g e et entre le m iro ir
J Ji STMirce p erçue / est une im age et l’objet. L ’équation 5.1 indique que l’im age csl à la m em e distance derrière le m iroir
virtuelle de l'objet O. que l ’objet devant lui. Selon la convention de signes établie pour les distances objet cl
5.2 Les miroirs piaras 109

image, la distance objet p est positive (l’objet est réel) ci la distance imafte q e.st négative
( l’im age est vinueJle). D onc, l’équation 5.1 peut s’écrire sous la form e 1^1 = p, ou

q = —p (l’équation de.s miroirs plans). (5.2)

L e sig n e n é g a tif d e c e lte éq u a tio n in d iq u e qu e l ’o b je t cl l’im ag e .sont to u jo u rs


de n atu res op p o sées. P ar ex em p le, si l’objet est virtu el, l’im age e s t réelle. F aites un
diagram m e d e rayons p o u r vous en convaincre. S euls les ray o n s assez rap p ro ch és les
uns des autres peuvent attein d re l’ccil après av o ir été réfléch is p a r un miroir. D ans la
position de l’œ il illustrée dans la figure 5.7, seule une petite pttrtion du m iroir située
figure 5.6 Deux rayons de la figure 5.5. près du point a (une portion plus petite qu e la pupille de l ’œ il) sert à fo rm er l ’im age.
Le rayon Oa forme un angle d'incidence P our trouver cette portion, ferm ez un œ il et regardez l’im age d 'u n petit objet dans un
quelconque 0 avec la normale à m iroir (le bout d ’un crayon, par exem ple). D éplacez ensuite le bout de votre doigt à la
la .surface du miroir. Le rayon Oh surface du miroir, ju sq u ’à ce que vous ne puissiez plus voir l’im age. Seule cette petite
est perpendiculaire au miroir. portion du miroir, cachée par votre doigt, a produit cette im age.

Les objets étendus


D ans la figure 5.8, un objet étendu O, représenté par une flèche orientée vers le haut, est
en face et à une distance [perpendiculaire p d ’un m iroir plan. Chaque petite portion de l’objet
qui fait face au m iroir agit com m e la source ponctuelle O des ligures 5.5 et 5.6. Si vous
interceptez la lumière réfléchie par le miroir, vous [percevrez une image virtuelle I qui est
com posée des points im age virtuels de toutes les portions de l’objet et qui sem ble être
Miroii
située à une distance q d en ière le miioir. Les distances q t X p sont reliées p ar l’équation 5.2.
On peut aussi kK'aliser l’im age d ’un objet étendu de la m anière dont on l’a fait dans
la cas d ’un point im age dans la figure 5.5 : on trace certains des rayons qui atteignent
le m iro ir et p ro v ie n n e n t d u d essu s d e l ’o b je t, on trace en su ite les ray o n s réfléc h is
correspondants e t on prolonge ces rayons réfléchis derrière le m iroir ju sq u ’à leur point
d ’in te rsec tio n , où ils lo rm e n t une im age du d essu s d e l ’o b je t O n p ro cè d e en su ite
de la m êm e façon avec les rayons qui proviennent du bas de l ’objet. C om m e o n le voit
dans la figure 5.8, on constate que l’im age virtuelle l a la m ême orientation cl la m ême
hauteur (m esurée parallèlem ent au m iroir) que l’objet O.

Les « Folies-Bergère » de Manet


D ans lia r aux Folies-Berbère, vo u s v o y ez la réflex io n du b ar d an s un g ran d m iro ir
reco u v ran t le m u r d errière une p réposée au bar. m ais cette réflex io n est su btilem ent
faussée de trois façons. O bservez d 'ab o rd les bouteilles à la gauche. M anet a peint leurs
réflexions dans le miroir, m ais en les plaçant plus loin vers l’extérieur du bar q u ’elles ne
figure 5.7 Un mince faisceau lumineux devraient l ’être.
provenant de O atteint l'œil après NtPlez ensuite la réflexion de la serveuse. P uisque vous la regardez directem ent de
avoir été réfléchi par le miroir. Seule face, sa réflex io n d c \ rail être d errière elle, et vous ne d ev rie z en ap e rcev o ir q u ’une
une petite portion du miroir située pente partie, tout au plus. Pourtant, M anet a peint sa réflcxupn tout à fait à la droite.
près du point a entre en jeu dans Enfin, o b serv ez la réflexion d e l ’hom m e qui lui fait face ; cet hom m e devrait être
cette réflexion. I.a lumière semble vous, puisque la reflexion indique q u ’il .se trouve directem ent en ftKC d e la serveuse, et
venir du point 7, situé derrière il d evrait donc être celui qui o b se rv e le tableau. Vous reg ard ez l’œ u v re de M anet et
le miroir. voyez votre réflex io n d écalée à la droite. L ’effet est tro u b lan t, car il d iffè re de celui
auquel on s ’attend norm alem ent d ’une peinture ou d ’un miroir.

♦^VÉRIFIEZ VOS CONNAISSANCES! Dans le schéiHU ci-de.vs(4is. v(hi.s regardez dans un système
formé de deux miroirs verticaux ri parallèles. A et B. séparés par une distance d. Une gargouille
est perchée au point O. à une distance de 0.2d du miroir A. Chaque iniroii pnxluii une première
image de la giugouille, celle qui e.st située le plus prés denière le.s miroirs. Puis, chaque miroir
prtKluit une seconde image, rcprcseniant l'objet créé par la première image dans le miroir
opposé. Par la suite, chaque miroir produit une troisième image montrant l’objet créé
par le seconde image dans le miroir opposé, et ainsi de suite - vous pouce/, alors voir
des eenlaines d'images de gargouille.s. À quelle distance derrière le miroii Л sont la première,
la deu.xièmc cl la uoisième image produite par le miroir A ?

TI l (I
ip w 5.8 Un objet étendu O et son image
«Stuelle / dans un miroii plan.
no Chapitres Les miroirs et les lentilles

5.3 Les miroirs sphériques


On peut maintenant passer de.s images pruduilcs par des miroirs plaas aux im ages prtxluites
p ar d e s m iro irs d o n t les su rfa ce s so n t co u rb es, ü n a n a ly se ra p lu s p articu liè rem e n t
les m iro irs sp h ériq u es, qui sont sim p lem en t d es m iro irs ayant la fo rm e d ’une p etite
section de la surface d 'u n e sphère. Un m iroir plan est en fait un m iroir sphérique dont
le rayon de courbure est infinim ent grand.

Créer un miroir sphérique


O b se rv e / d ’abord le m iro ir plan d e la figure 5.9 a), dont la su rface réfléch issan te est
orientée vers la gau ch e en d irectio n de l'o b je t réel O illustré e t d 'u n ob.scrvateur qui
a) n ’est pas représenté. O n obtient un m ir o ir co n cav e en courbant la surface du m iroir afin
de la rendre concave (du mot latin cavus, qui signifie « c re u x » ), com m e on le voit dans
la figure 5.9 h). Rn courbant ainsi la surface du miroir, on m odifie plusieurs caiactcris-
tique.s du m iroir et de l’im age q u ’il produit.

1. L e centre de courbure C (le centre de la sphère dont fait partie la surface du m iroir)
était infinim ent loin dans le m im ir plan ; il est m aintenant plus rapjiroclié. m ais toujours
en face du m iroir concave.
2. Le champ de vision - l'éten d u e de la scène, constituée d ’objets réels, réllcchie vers
l'o b serv ateu r - était la rg e ; il est m aintenant plus petit.
3. L ’im age d ’un objet réel était aussi loin derrière le m iroir plan que l’objet était loin en
face du m iroir; l’image peut m aintenant être plus loin ou plus près du m iroir concave.
4. La h a u te u r de l'im a g e é ta it ég a le à ce lle d e l ’o b jel ; la h a u te u r de l'im a g e peut
m aintenant être plus grande ou plus petite. C ette caractéristique explique pourquoi de
nom breux m iroirs utilisés pour se m aquiller ou se raser sont concaves - ils produisent
une image virtuelle agrandie du visage lorsque celui-ci est suffisam m ent près du miroir.

O n peut c ré er un m ir o ir co n v ex e en courbant la su rface d ’un m iroir plan afin de


la rendre convexe, com m e on le voit à la figure 5.9 c). C ela a pour effet 1) de déplacer
le centre de courbure C derrière le miroir, 2) d'augm enter l ’étendue du cham p de vision,
3) de produire à p artir d ’un objet réel une im age virtuelle plus près du m iroir e t plus
petite que l ’objet. L es m iroirs d e su rv eillan ce dans les m agasins sont h ab itu ellem en t
convexes, afin de produire un cham p de vision agrandi, ce qui perm et de surveiller une
grande partie du m agasin à l’aide d ’un seul miroir.

Les foyers des miroirs sphériques


Pour un m iroir plan, la valeur absolue de la distance im age q est toujours égale à la \ aleur
absolue de la distance o b je t p. A vant de pouvoir déterm iner com m ent ces deux distances
Figure S.9 a) Un objet réel O forme sont re lié e s d ans un m iro ir sp h é riq u e . On d o it c o n s id é re r la réflex io n de la lu m ière
une image viniicllc / dans un miroir plan, p ro v e n a n t d ’un p o in t o b je t d , situ é à l ’in fin i d ev a n t le m iro ir sp h é riq u e , su r l'a x e
h) Si on courbe la surface du miroir o p tiq u e du miroir. C et axe traverse le cen tre de co urbure C et le som m et S du miroir.
de manière à la rendre com ave
A cau.se de la g ran d e d istan ce en tre l'o b je t et le m iroir, les fro n ts d 'o n d e proven an t
et que Tobjei réel est suffisamment
de l'o b je t sont de.s o n d es p lan es lo rsq u 'ils atteig n en t le m iro ir p erp en d icu lairem en t
près du miroir, l’image s’élrrigne
à l'a x e optique. C e la .signifie qu e les ray o n s rep rése n tan t les o n d es lu m in eu ses sont
et devient plus grande, c) Si on la courbe
tous parallèles à l’axe optique lorsqu’ils atteignent le m iroir (voir la figure 4.5).
de manière à la rendre convexe, l'image
L o rsq u e CCS ray o n s p a ra llè le s a tteig n en t un m iro ir co n c av e co m m e c e lu i d e la
SC rapproche et devient plus petite.
figure 5. K) a), les rayons pm ehes de l’axe optique sont réfléchis à un point com m un b situé
sur l'ax e fiptique ; deux de ces rayons réflécliis .sont illustrés dans la figure. Si on plaçait un
(petit) écran au point F, un point image du point objet O infinim ent distant y apparaîtrait.
(Cela se prrxluirail avec tout objet infinim ent distant.) Le point F se nom m e le fo y er du
miroir, et la distance le séparant du som m et du m iroir est la d is ta n c e f o c a le / du miroir.
Si on rem place m aintenant le miroii concave par un m iroir convexe, on observe que
les rayons parallèle.s près de l’axe optitjuc ne sont plus réfléchis vers un [Xiini com m un,
m ais q u 'ils sont plutôt diveigent.s, com m e le m ontre la figure 5.10 b). Toutefois, si votre
reil intercepte une partie de la lum ière réfléchie, vous percevrez la lum ière com m e si
elle provenait d ’un point source situé derrière le miroir. C ette source perçue est située
à l’endroit où les rayons réfléchis traversent un point com m un ( f dans la figure 5.10 b).
5.4 Les images produites par les miroirs sphériques III

C e p o in t est le fo y er F du m iro ir co n v e x e, ci la d istan c e e n tre le fo y er e t la su rfa ce


du m iroir est la d istan ce f o c a le /d u m iroir. Si on p laçait un écran à ce foyer, aucune
im age de l’objet O n ’apparaîtrait sur l’écran ; ce foyer est donc différent de celui d ’un
m iroir concave.
A fin de d istin g u er le lo y e r réel d ’un m iro ir co n cav e d u foyer perçu d ’un m iroir
convexe, on nom m e le p rem ier fo yer réel et le se c o n d ,/o v e r virtuel. D ’autre part, on
considère la distance f o c a le /d ’un m iroir concave com m e une quantité positive, et celle
d ’un m iro ir co n v ex e co m m e une q u an tité n égative. D an s les d eux ty p es d e m iro irs,
la distance f o c a le /e s t reliée au rayon de courbure r du m iroir par

i-------/
k - f~ 2 distance focale d’un miroir sphérique). (5.3)
a)
tiù, pour conserver l’association entre les signes et la distance focale, r est une quantité
positive dans le cas d ’un m iroir concave, et une quantité négative dans le cas d ’un m iroir
convexe.

5.4 Les images produites


par les miroirs sphériques
M aintenant q u ’on a défini le fo y er d ’un m iro ir sphérique, on peut tro u v er la relation
entre la distan ce im age q e t la d istan ce objet p d an s les m iroirs sp h ériq u es co n cav es
et convexes. O n p lace d ’abord un o b jet réel O d e m an ière q u ’il so it plus rapproché
du miix)ir concave que le foyer c ’est-à-dire entre le m iroir et son foyer/- (figure 5.11 a)).
L4i observateur peut alors voir une im age virtuelle l à e O dans le m iroir; l'im age sem ble
être derrière le m iroir et a la m êm e orientation que l’objet.
Si o n é lo ig n e m a in te n an t l ’o b je t du m iro ir ju s q u 'à ce q u ’il se tro u v e au foyer,
Hgure 5.10 P<xir un fai.sceaii incident
l'im a g e s ’élo ig n e d av an tag e du m iroir, ju s q u ’à une d istan ce in fin ie (fig u re 3 .1 1 b]).
constitué de rayons lumineux paiallèlcs
L orsque l’objet réel ü est au foyer F, les rayons réfléchis par le m iroir et le prolonge­
à Taxe optique, les rayons lumineux
m ent des ray o n s d errière le m iro ir ne .se ren co n tren t pas ; l’im age est donc à l’infini.
émergents a) d’un miroir concave
C ette im age peut q uand m êm e être p erçu e par un o b se rv a teu r ; les éto ile s sont à une
convergent vers un foyer réel en h,
distance infim e par rapport à l'o u v ertu re de l’œ il (les rayons lum ineux issus des étoiles
du même côté du miroir que les rayon.s
et entrant dans l ’œ il stint parallèle.s), et cela ne nous em pêche pas de bien les distinguer.
lumineux incidents, et b) d’un miroir
Si on déplace maintenant l’objet rét;l pour qu’il soit « l ’extérieur du / l y e r - c ’est-à-dire
convexe semblent diverger à partir
plus éloigné du m iroir que le foyer - les rayons réfléchis par le m iroir convergent pour
du foyer virtuel F. du côté du miroir
form er une im age renversée de l ’objet O devant le tniroir (figure 5.11 c |). Celle im age
qui est opposé aux rayons lumineux
se rapproche d e F lo rsq u ’on éloigne d avantage l'o b je t au-delà de F. Si vous tenez un
incidents.
écran à la position d e l’image, l’im age apparaîtra su r l'é c ra n - on dit alors que l'im a g e
est focalisée sur l’écran par le miroir. Puisque eetle im age est form ée à partir de rayons
ém ergents convergents, il s’agit d 'u n e image réelle - ici, les rayons se croisent vraim ent

RgufeS.ll a) L^n objet réel O plus


rapprcK-hé que le foyer d'un miroir
concave, et son image virtuelle /.
b) Un ob|ei réel c.st au foyer F.
c) Un objet réel plus cMoigné
que le foyer, et son image réelle /.
d) l in objet viniiel O et son image
réelle / «)
112 Chopitre 5 Les miroirs et tes lentilles

pour créer l’im age, q u ’un o b serv ateu r soil p résen t ou non. L a disiance im age q d ’une
im age réelle est u ne quantité positive, contrairem ent à celle d 'u n e i n i ^ c vinuelle.
Si l ’o b jet O est v irtu el (peu im p o rte sa p o sitio n ), l ’im a g e fo rm é e est rée lle
(figure 5.11 d). Puisqu’un objet virtuel est lo n n é à partir d ’un faisceau incident convergent,
les rayons qui .sont réfléchis p a r le m iro ir concave convergent davantage, donnant une
im age réelle.
C om m e on le dém ontrera dans la section 5.9, lorstiue des raytm s lum ineux [irovenant
d ’u n o bjet ne fo rm en t que d e p etits angles av ec l'a x e o p tiq u e d ’un m iro ir sphérique,
une équation sim ple relie la distance objet p, la distance im age q et la distance fo c a le /:

1 \ _ ^
(l'équation des miroirs sphériques). (5.4)
P Q ~ f

O n su p p o se q u e les an g les p ar ra p p o rt à l ’axe o p tiq u e d e la fig u re 5.11 sont p etits,


m ais les ray o n s y sont d essin és avec des angles ex ag érés, p ar souci de clai té. A insi,
l’éq u a tio n 5.4 s ’ap p liq u e à to u t m iro ir c o n c av e , co n v e x e ou plan. D an s le ca s d 'u n
m iro ir co nvexe ou p lan , à p a rtir d ’un o b jet réel, se u le u n e im ag e v irtu e lle p eu t être
créée, ix;u im porte la p o sitio n d e l ’o b jet réel su r l’axe o p tiq u e. C o m m e c ’est illustré
dans l’exem ple d ’un m iroir convexe dans la fig u re 5.9 c), l ’im age virtuelle d ’un objet
réel est toujours plus petite que l’objet et a la m em e orientation que celui-ci. P uisqu’un
o bjet reel est form é à partir d ’un faisceau incident divergenU les rayons qui sont réfléchi.s
par le m iroir convexe divergent davantage, donnant une im age virtuelle.
l a taille d ’un objet ou d ’une im age, m esurée perpendiculairement à T axe optique
du miroir, se nom m e la hauteur de l’objet ou de l’image. Soit h la hauteur de l’objet, et h'
celle de l’image, la; rapport h'/h se nom m e le g ra n d iss e m e n t tra n s v e rs a l m produit par
le miroir. P ar convention, le grandissem ent transversal p rend un signe p o sitif lorsque
ro rien latio n de l’im age est la m ême que celle de l’objcL et un signe n ég aliflo rsq u c cette
orientation est opposée à celle de l’objet. C 'e s t pourquoi on écrit ainsi rétju alio n de m :

h'
1ml = — (le grandissement tran.sversai). (5..5)

O n dém ontrera bientôt que le grandissem ent transversal peut aussi s’écrire sous la form e

m = — fie grandissement transversal). (5.6)


P

P our un m iro ir plan, où q — —p, on a m — f I . Le grandi.sscm cm de 1 sig n ifie


que l’im age a la m êm e taille que l'o b jet. Le signe p ositif signifie que l’im age et l’objet
ont la m êm e orientation. D ans le cas du m iroir concave de la figure 5.11 c), m ~ —\ ,5.
Les équations 5.3 à 5.6 s’appliquent à tous les m iroirs plans et .sphériques, q u 'ils soient
concaves ou convexes.

Localiser des images en faisant des tracés


des rayons principaux
l es figures 5 .12 a) ei h) représentent un objet réel O dev ant un m iroir concave. O n peut
lo c aliser l'im a g e de to u t p o in t d e l’o b je t situ é en -d e h o rs d e l ’ax e en tra ç a n t un
diagramme de ravonx com prenant au m oins deux des quatre rayons principaux passant
par ce point.

1. Un rayon incid en t p ara llèle à l ’ax e o p tiq u e est, p o u r un m iro ir co n cav e, réfléch i
en p assan t p ar le fo y e r / (ray o n 1 d an s la fig u re 5 .12 a), et, p o u r un m iro ir
convexe, réfléchi de sorte que s<^ prolongem ent passe par le foyer F (rayon 1 dans
la figure .5.12 c).
2. Un rayon incideni p assan t par le foyer /-'(ray o n 2 dans la figure 5.12 a) r>u d ont le
p ro lo n g em en t p asse par F (ray o n 2 d an s la fig u re 5 .1 2 c) ém erg e p ara llèlem e n t
à l ’axe optique.
3. l n rayon incident passant par le centre de courbure C (rayon 3 dans la figure > 12 b)
ou d o n t le p ro lo n g em en t p asse p ar C (ra y o n 3 d an s la fig u re 5 .1 2 d ) e s t réfléch i
le long de son parcours initial.
4. Un rayon incident se dirigeant vers le scmimet S du m iroir est rciléchi sym étriquem ent
par rapport à l'a x e optique (rayon 4 dans les figures 5.12 b et d).
5.A Les Images produites par les miroirs sphériques 113

a) b)

Figure 5.12 a), b) Quatre iracés


de rayons permettant de trouver l'image I.’itnagc d 'u n p o in t o b je t .se situ e à l'in te rse c U o n d es ray o n s lu m in eu x p rin cip au x .
d’un objet ricl dans un miroir concave. G énéralem ent, prrur localiser l’image, tieux raynas principaux suffisent, mais il est toujours
Par rapport à la position de l’trbjei préférable de tracer un troisièm e rayon afin d e v alider voire d iagiam m e. N o te / q u e la
illustrdc. l’image est réelle, rensersée
co u rb u re d es m iro irs d a n s la fig u re .‘ï.1 2 est fo rtem en t ex a g érée. A fin d ’au g m en te r
Cl plus petite que l’objet c). di Quatre
la précision de v o s diag ram m es de rayons, d essin ez ties m iroirs ay an t de très faibles
rayons similaires dans le cas d'un miroir
courbures.
convexe. Dans le cas d'un miroir
En traçant des diagram m es de rayons lum ineux à partir d ’objets reals pour les deux
convexe et d'un objet réel, l'image est
d iffé ren ts ty p es d e m iro irs, rem p lisse z le tab leau 5.1. S o u s la m en tio n Position de
toujours virtuelle, et elle est plus petite
la rubrique hna^e, notez si l'im ag e est du m êm e côté du m iroir que l'o b jet, ou du côté
et a la meme orientation que l'objet.
(Dans la partie c) le rayon 2 incident est trpposé. Sous la mention Tvpe. indiquez si l'im age est réelle ou virtuelle. Sous la mention
dirigé vers le foyer F. Dans la partie d) O rientation, in d iq u ez si l'im a g e a la m êm e o rien tatio n q u e l ’o b jet, ou si e lle est
le rayon 3 incident est dirigé vers renversée. Sous la rubrique Silène, indiquez le signe de la quantité, ou indiquez ± si le
le centre de crxirbure C ) signe est am bigu. P ar la su ite, to u jo u rs à l ’aide de d iag ram m es rie ray o n s lu m ineux,
rem plissez le tableau 5.2 pour des objets virtuels.

ÎABIFAU 5.1 Tableau des torortéistiques des images des miroirs pour des objets réels
Image Signe
Type Prrsitton
de miroir de l’objet réel Posititrn Type Orientation /o u r ‘1 yn
Plan N’importe oii
Entre F et le miroir
CtHicavc
Entre Fcx l'infini
C onvexe N’importe où

ÎABIEAU 5.2 Tableau des carottérsliques des images des miroin pour des obiets virtuek
Image Signe
Type Pnritinn
de miroir de r objet virtuel Position Type Orientation fo u r H m
Plan N’impone où
Concave N’importe où
Entre F et le miroir
C onvexe
Entre F et l'infini
114 Chopitre 5 Les miroirs et les lentilles

Démonstration de l'équation 5.6


On peut m aintenant cléinonircr Tequation 5.6 (m = —qtp), qui exprim e le grandissem ent
transversal produit p ar un m iro ir sphérique. O b serv ez le rayon A d e la fig u re 5.12 b).
Il est réfléchi au po in t S, de sorte que, selon la loi de la réflexion, les rayons incident
el réfléchi form ent des angles égaux avec l’axe du m iroir (l’axe optique cl la norm ale se
txtnfondenl à ce point).
Les deux triangles rectangles ahS et deS de la figure sont sem blables (ils com porteni
les m êm es angles) ; on peut donc écrire
de
Sa
La q u an tité du m em b re d e gau ch e (on ne se p réo ccu p e pas du sig n e ici) est le g ran ­
dissem ent transversal m produit par le miroir. C om m e une im age renversée doit avoir un
giandi.s.semenl négatif, on utilise la notation —ni. Ibulefois. Sd — q e t Sa = p, donc

m = —— (le grandi.ssemcnt transversal), (5.7)


P
qui est la relation que l’on voulait dém ontrer.

Exemple 5.
Une tarentule de hautcui h se tient prudcninient devant un miroir Puisque l'objet et T image ont la même orientation, on sait que m doit
.sphérique dont la distance focale a une valeur absolue l/l 40 cm. être positif : m = ‘ 0.20. Si on insère ce.s données dans l’équation 5.6
L’imucc de la taiciiiulc prixiiiire iwr le miroir a la meme orientation et qu’on la résout pour trouver q. on obtiem
que la laronuilc, et sa hauteur c.st h' ~ ü.20/i.
q — 0.20/1.
a) 1 image est-elle réelle ou virtuelle?
mais cela ne semble pas être d'une grande utilité pour trouver
SOLUIION: Le concept clé utilisé ici est le suivant: pui.squc l’image a la Toutefois, cc résultat peut être utile si on l’insère dans l’équation 5.4.
même orientation que la tarentule (i Objet), le grandissement tn Cette équation donne alors
est positif. .Selon l’équation 5.6, m et p étant tous les deux positifs
1 I I 1 1 1
iro b jcl est réel), q (— —mp) doit être négatif ce qui indique que -7 = ---- h - = O -угГ ----p ~ —(“
—0.20/7 p
5 + 1),
rim agé est virtuelle (Vous pouvez facilement verifier ce résullal / 4 P
SI vous avez rempli le tableau 5.1.) ce qui permet de trouver
b) Le miroir est-il concave ou convexe, et quelle est sa distance
focale/ (indiquez le signe) ? /=

SOLUTION: On ne peut déterminei le type de miroir d'après le type On y est maintenant : puisque p est positif,/doit être négatif, cc qui
d'image, car les deux types de mimir peuvent prcxliiire des images .signifie que le miroir est convexe et que
virtuelles. De la même façon, on ne peut déterminer le type de miroir
en detenrunant le signe de la distance female/ à l'aide des équations 5.3 / = 40 cm. (réponse)
ou 5.4, car il inanquc trop de données pour qu’oii puisse utili.ser ces
équations, foutefois - et voilà le concepf clé à utiliser on peut tirer
profil des données concernant le grandissement. On sait que le ✓ vérifiez VOS CONNAISSANCES 2 Une chauve-souris vampire
rapport de la hauteur de l’image h' à celle de l'objet h est (1,20. de l’Amérique centrale, somnolant sur l’axe optique d’un miroir
Dont, d'après réquaiion 5.5, on a sphérique, est agrandie d'un facteur ni = - 4 . Son image est-elle
a) réelle ou virtuelle, b) renversée ou présente t-cllc lu même
orientation que l’objci ei c) ilii même côté du miroir que l'objei,
Ihil = - r = 0.20.
h ou du côté opposé ?

5.5 Les dioptres sphériques


On étudiera m aintenant les im ages form ées p ar la réfraction des rayons lum ineux passant
à trav ers les su rfa ce s de m atériaux tran sp a ren ts, co m m e le verre O n n e s'a tta c h e ra
qu aux surfaces sphériques, que l'o n appelle égalem ent d io p tre s sp h é riq u e s, prc.scntant
un rayon de courbure r et un centre de courbure C. La lum ière sttra ém ise par un point
objet O, dans un m ilieu où l’indice île réfraction est ri| ; elle sera réfractée, en pas.sant
à travel's une surface sphérique, dans un m ilieu où l’indice de réfraction est
O n veut sav o ir si les ray o n s lum ineux, après leur réfractio n , fo rm en t une im age
réelle ou une im age virtuelle L a réponse dépend d es valeurs relativ es de H| e t d e « j,
et de la configuration géom étrique de la situation.
5.5 Les dioptres SDhériques HS

Figure S.13 Six diCférenlcs situatioiiü d’une image pnxluite


a) b) par la réfraction de nivoni^ lumineux travei\am iin diopirc xphcriiinc
de rayon r e t de centre de courbure C. Le dir>ptie »phéiitjuc sépoic
un milieu où l'indice de rétraction est n¡ d'un milieu où l'indice
de réfraction est fh. la: iioinl objet réel O est toujours duns le milieu
V ir tu e lle
où l’indice de réfraction est n,, à la gauche du rlioptre I e matériau
ayant le plus petit indice de réfraction est non coloré ('.supixiscr
qu’il s’agit de l'air, et que l’autre matériau est du verre). Des images
réelles se forment en a) et b) ; des images virtuelles se fomient
dans lc.s quatre autres situations

Six situations possibles sont illustrées dans la ligure 5.13. D ans chaque partie d e la
figure, le m ilieu ayant le plus grand indice de réfraction est cokiré, et l’objet léel O est
to u jo u rs situ é d an s le m ilieu où l ’in d ice d e réfractio n est »?,, à la g au c h e du d io p tre
sp h é riq u e . D an s ch a q u e p a rtie , on voit un ray o n q u i a é té réfrat té p a r le dwiptrc
sphérique. (D ans chaque cas, il suffit de ce rayon et d ’un rayon le long de Taxe optique
pour déterm iner la position de l'im ag e.)
A u point de réfraction de chaque rayon, la not male à la surface de réfraction est une
ligne radiale passant p ar le centre de courbure C. À cause de la Tcfraciiou, le rayon est
dévié en s ’approchant de la noim ale s ’il entre dans un n u h eu où l'in d ice de réfraction
est p lu s élev é, e t en s ’élo ig n a n t de la n o rm ale s 'il en tre d an s un m ilie u où l ’in d ice
de réfraction est m oins élevé. Si le rayon réfracté se dirige ensuite vers l’axe optique,
il fo rm e ra, avec les au tres ray o n s non illu strés, u n e im age réelle su r cet axe. S ’il se
d irige en s'é lo ig n a n t d e l ’axe o p tiq u e, il ne peut fo rm e r une im age réelle ; toutefois,
le prolongem ent de ces rayons rcfi aciés vers l’axe optique form e une im age virtuelle.
D es im ages réelles l sont form ées (à une distance im age <?) dans les parties a) et b)
de la fig u re 5 .1 3 , où la réfra ctio n d irig e le ray o n vers l ’axe o p tiq u e. D es im ages
virtuelles sont form ées dans les parties c) et d), où la réfraction dirige le rayon dans la
direction opposée à l'ax e optique. D ans ces quatre parties, notez que les im ages réelles
se fo rm e n t lo rsq u e l’objet est relativ em en t élo ig n é du d io p tre sp h ériq u e, et qu e des
’. i-i : . im ages virtuelles se form ent lorsque l ’objet est plus près du dioptre. D ans le.s dernières
:1
situations présentées (figures 5.13 e et 0 . la réfraction dévie loujoiu's le rayon dans une
d irec tio n qui l’é lo ig n e d e l'a x e o p tiq u e, et le.s im ag es qui sc fo rm e n t sont to u jo u rs

virtuelles, peu im porte l:i distance de l'objet.
Dans la section 5.9, on dém ontrera que, dans le cas des rayons lum ineux ne form ant
que de [jetits angles avec T axe optique.

r>2 «2 ni
(l’équaiion des dioptres spliciiques). (5.8)
" 4 ï . P 9

C om m e dan.s le cas des m iroirs, la disiance o b j e t e s t positive pour un objet réel, néga­
f: a/ ! tive pour un objet Vim icl. e t la distance image, q est positive lo rsq u 'il s'ag it d 'u n e im age
réelle, et négative lo rsq u 'il s ’agir d ’une image virtuelle. Toulelbi.s, afin d 'em p lo y er les
signes adéquats dans l'cq u alio n 5.8, on doit utiliser la règle suivante en ce qui concerne
le signe du rayon de courbure r

► Le rayon de couiburc r est positif lorsque le centre de coiubiire C est du côté d’où émerge
le faisceau I orsqiic le centre de eranbe C est du tfrté du faisceau iiu ident. r est négatif

Cet insecte acte fossilisé dans l’ambre


pendant environ 25 millions d’années ^V ÉR IFIFZ VOS CONNAISSANCES !: (Inc abeille plane «levant un dioi^lic sphérique concave
ComnK on le voit à travers une stirface d’une sculpture de verre, a) I aqiielle des situations de la figure 5 .11 corresptmd à celle-ci ''
de réfraction courbe, l’image que l’«vn b) l .'ittiagc protliiitc par le dioptre cst-cllc réelle ou virtuelle, et csl-elle du même côté
perçi'ii ne coïncide pas avec fin.scctc. que l’abeille ou du c6té opposé
116 Chapitre 5 Les miroirs et les lentilles

Exemple 5.2
Un moustique de l’crc jurassique a été découvert, fossilisé. dan.s
un morceau d ’ambre dont l’indice de réfraction est 1,6. La surface
de l’amhre est sphérique et convexe, et son rayon de courbure est
/ O C
de ,L0 mm Uigure 5.14). lête du moustique est située sur l'axe
optique de cette surface et. lorsqu on la regarde le long de l'a.xe. elle
semble être située à 5.0 mm à l’intérieur de l’ambre. Quelle est sa
position réelle ?

SOIUIION. Le coïKcplde utilisé ici est le suivant; la tête semble être


à 5,0 mm à l’intérieur de l'ainbrc parce que les rayons lumineux figure 5.14 Exemple 5.2 Un morceau d'ambre emprisonnant
interceptés pai un observateur sont dé\ iés, pui réfraction, à la surface un insecte de l'ère juiussique, dont la tête est située au point O
convexe de l'ambre. La distance iniage q dil lère de la distaiKc objet Le dioptre sphérique, dont le centre de eouiburc est C, produit
réelle p, selon l'équalion 5.8. Pour bien utili.scr cette équation une image / pour un observateur qui inlereepie les rayons
Cl trouver la di.siaiKC objet réelle, on noie d'abord ce qui suit. en provenance de l'objet O.
1. Puisque l'objet (la tête) et son image siHit du même côté de la surface
de réfraction, les rayons émergents sont divergents, alors l'image ,Si on insère ces données dans l’équation 5.8,
iliiit CiTv virtuelle, de sotte que q = -.5.0 mm.
ni ny ff — «1
— + — _ -= -------- - ,
2. Puisqu'on suppO'SC que l'olijei est dans un milieu où l'indice de P q r
rétraction est u,. on doit avoir n, = 1,6 et «i - 1,0.
1.6 1,0 1,0 - 1.6
3. Piiis(|ue le centre de courbure est du côté d'où proviennent les on obtient
P —5,0 mm —3.0 mm
rayons im idents, le rayon de courbure r e s t négalit, de sorte
que r — - 3,00 mm. et P = 4.0 mm. (réponse)

5.6 Les lentilles minces


U ne le n tille est un dou b le d io p tre dont les axes o p tiq u es co'incident. L ’axe o p tiq u e
com m un c.st Taxe optique de la lentille. L orsqu’une lentille est entourée d ’air, la lumière
est réfractée en passant d e l’air à la lentille, traverse la lentille et est ensuite réfractée,
une seconde fois, en pa.s.sunt de la lentille à l’air. C haque réfraction peut changer la direction
de propagation des rayons lum ineux. O n se contentera d ’analyser le cas particu lier des
len tilles m in c es - des lentilles dont la partie la plus épaisse esl négligeable com parée
à la distance objet p. à lu d istance im age q cl aux rayons de courbure r^ et des deux
surfaces de la lentille.
U ne le n tille q u i fait c o n v e rg e r d es ray o n s lu m in eu x in cid en ts p arallèles à l ’axe
optique se nom m e (généralem ent) une len tille co n v e rg en te . Si elle fait plutôt diverger
les ray o n s in c id en ts p ara llèles à l'a x e o p tiq u e , il s 'a g it d 'u n e le n tille d iv e rg e n te .
O n peut reconnaître au touché st une lentille est convergente ou divergente. Si l’indice
lie réfraction de la lentille esl plus grand que l'indice de réfraction du milieu qui l’entoure,
lu partie centrale d ’une lentille co nvergente sera plus épai.s.se que son co n to u r (voir la
lig u re 5.15 a). P our une len tille d iv erg en te, le co n to u r .sera plus ép ais que la p artie
centrale (voir la figure 5.15 c). C ette caractéristique s'in v e rse si l’indice de réfraction
du m ilieu am biant est plus grand que celui d e la lentille. L orsqu'on fait un croquis d 'u n e
situation quelconque m ettant en jeu des lentilles, on représente une lentille convergente
par le sym bole t et une lentille divergente par ï . L o rsq u ’un objet réel est placé devant
une de ces lentilles, la réfraction, par les su rfaces d e la lentille, des rayons lum ineux
provenant de l'objet peut produire une im age de l’objet.
On traitera seulem ent du cas où les rayons lum ineux form ent d e petits angles avec
Taxe optique (ce.s angles sont exag érés dans les figure.s présentées dans cette section).
D ans la section 5.9. on d ém o n trera q u e, p o u r c e s ray o n s, un e le n tille m ince a une
distance focale f. D e plus, q c i p sont reliés entre eux par

1 _ 1 1
(l’équation des leptilles minces). (5 9 )
/ r ^ 9
5.6 Les lentilles minces 117

Figure 5.15 a) Des rayons incidents


parallèles à l’axe oplitjue d’une lentille
convergente sonl déviés par la lentille
et convergent vers un foyer réel F' . *
La lentille est en réalité beaucoup plus
mince que celle illustrée dans la figure.
Son épais.seur est. en fait, semblable
à celle de la ligne verticale qui
la traverse, où toutes les déviations
b)
de rayons se produisent, b) Agrandi.s-
senient de la partie supérieure
de la lentille illustrée en a) ; les normales
aux sut faces sont tes traits pointillés.
Notez que les deux réfractions
font dévier le rayon lumineux vers
le bas, soit vers l’axe optique
c) Les mêmes rayons incidents parallèles
divergent en traversant une lentille
divergente. l.es prolongements
des rayons divergents passent par
un foyer virtuel F'. d) Agrandissement
de la partie supérieure de la lentille
illustrée en c) ; rsHcz que les deux
réfractions devient le rayon lumineux qui est la m êm e éq u a tio n qu e ce lle d es m iro irs. O n d é m o n trera ég a lc m i ni qu e,
vers le haut, dans la direction opposée lorsqu’une lentille m ince ayant un indice d e réfraction n est em otiiée d 'air, sa distance
à l'axe optique. focale est donnée par

l
1) (la distance focale d’une lentille mince dans l’air). (.5, lOl
T “ '" V'-i n)
q u ’on appelle souvent la formule des opticiens. Ici, r, est le rayon de courbure de la surface
de la le n tille q u i in te rc e p te les ray o n s in c id en ts, e t Vj e st ce lu i d e l'a u tre su rface.
L es sig n e s île ces ray o n s sonl d o n n és p a r les règ les d e la se ctio n .5 ,S co n c ern an i
les ray o n s des d io p tre s sp h ériq u es. Si la le n tille est d an s un au tre m ilieu que l'a ir
(dans l’huile de m aïs, par exem ple), ayant un indice d e réfraction on rem place «
par dans l’cquation 5.10.

I Inc lemille ne produit une image d’un objet que parce qu’elle peut dévier les rayons luniiiieux,
mais elle ne peut dévier ce.s rayons que si son indice de réfraction est différent do celui
du mJieu environnant.

L a figure 5.15 a) m ontre une lentille m ince et ses surfaces d e réfraction convexes,
ou faces. L o rsq u e d es ray o n s in c id en ts p ara llèles à l’ax e o p tiq u e d e la le n tille se
prop ag en t à trav e rs la le n tille, ils so n t réfra cté s d eu x fo is, co m m e on le vent dans
l’agrandissem ent de la figure 5.15 b). C ette double réfraction fait co nverger les rayons
in c id en ts e t les fait p asser p ar un p o in t co m m u n F', à une d ista n c e / d u cen tre de la
lentille. Donc, cette lenlillc est une lentille co n v erg en te; de plas, elle possixlo un foyer
réel F' (puisque l’im age qui est produite est réelle, le faisceau ém ergent de la lentille est
convergent), et la distance f o c a le /q u i lui est asscKiée est positive.
l,a figure 5.15 c) représente une lentille m ince ayant des faces concaves. L orsque
des ray o n s in c id en ts p ara llèles à l'a x e o p tiq u e d e la le n tille la tra v e rse n t, ils stm i
ré fra c té s deux fo is, co m m e on le voit d an s l ’a g ra n d isse m en t d e la fig u re 5.15 d ) ;
ces rayons divergent et ne passent jam ais p ar un point com m un ; cette lentille est donc
une le n tille d iv e rg e n te. T o u tefo is, les p ro lo n g em en ts d es ray o n s ém erg en ts p assen t
p a r un point com m un F', situé à une d is ta n c e / du centre de la lentille. D onc, la lentille
possède un foyer virtuel à 7 ' et la distance f o c a le /e s t négative. (Si vos yeux interceptent
certains rayons divergents, vous percevrez un point brillant à F', com m e s ’il s ’agissait
de la source lum ineuse.)
1 ine lenlillc m ince pas.sède deux foyers qui sont situés à égale distance d e chaque
cô té du centre d e la lentille. L e foyer im age F' d ’u n e lentille co rresp o n d au p o in t de
c o n v e rg e n c e sur l’ax e o p tiq u e d es rayon.s ém erg en ts lo rsq u e les ray o n s incideni.s
и8 Chapitre 5 Les miroirs et (es lentilles

sont parallclc.s à T ax e o p tiq u e, tandi.s q ue le fo y er o b jet F co rresp o n d à l ’en d ro it sur


l’axe optique où l’on doit placer un objet pour que les rayons qui ém ergent de la lentille
soient parallèles à l’axe optique. La figure 5.16 a) illustre l afplication de la définition pour
déterm iner la position du foyer im age F' pour une lentille conveigente, et la figure 5.16 b)
illustre l’applicatio n de la d éfin itio n du fo y er objet h p o u r une le n tille co n v erg en te.
On constate que ces deux foyers sont réels parce que l’image produite en F' et l’objet situé
à F .sonl réels. La figure 5.16 c) illastre, jxtur une lentille divergente, la déviation de rayons
in c id en ts p ara llèles à l ’ax e o p tiq u e p o u r d é te rm in e r la p o sitio n du fo y er im ag e F '.
O n o b se rv e q u e les ray o n s qui é m e rg e n t n o n t pas de p o in t d 'in te rs e c tio n ; il fau t
effectuer leurs prolongem ents vers L arrière pour déterm iner un point com m un. Puisque
l'im a g e form ée e st virtuelle, le foyer im age d 'u n e lentille divergente est \ irtuel et est
situé du côté d ’où proviennent les rayons incidents. La figure 5.16 d) illustre, pour une
le n iille d iv e rg e n te , le p arc o u rs d es ray o n s lo rsq u e ce u x -ci ém e rg e n t p arailèiem eiil
à l’axe optique. P u isq u e ce tte len tille est d iv erg en te, il fau t qu e le s ray o n s incidents
convergent vers la icntiilc afin q u ’ils ém ergent parallèlem ent à l ’axe optique. On doit
alors piolonger vers l’avant la trajectoire des rayons incidents pour trouver la position
On peut allumer un teu en tocalisaiu du p o in t o b je t q ui va c o rre sp o n d re à la p o sitio n du fo y e r o b jet F d e c e tte len tille.
la lumière du »olcil sur un point précis Г e foyer objet est virtuel, puisque l’objet est virtuel, cl il est situé du côté d ’où les rayons
à l'aille d*uiw leniille convergente faiie ém ergent. Il est très im portant de noter que, p o u r une lentille divergente, les pasitions
fie glace transparente. La lentille dcA foyers objet el im age sont inversées par rapport à celles d 'u n e lentille convergente
il été fabriijiicc en fai.snnt fondre (cette rem arque prendra tout son sens lorsque vous tracerez les trois rayons principaux
le« (ieu.'i côtés d’un hloc de glace pour fléierm iner la position de l’im age form ée par une lentille).
dans un récipient | ku profond dont
P u isque les p o in ts focaux d ’une len tille co n v e rg en te so n t réels, on attrib u e aux
le tond est recourbé ; les côtés prennent
distances focales a s s o c ié e s /u n e valeur positive, com m e on l a fait dans le cas du foyer
ainsi une forme convexe.
réel d ’un m iro ir co n cav e. T o u tefo is, la co n v e n tio n de sig n es d o it être co h é ren te en
optique ; il est donc préférable de la vérifier à l ’aide de l ’équation 5.10. Le côté gauche
d e ce tte équatio n est p o sitif s i / e s t p o sitif; m ais q ii’cn cst-il du cô té d r o it? O n j^ieut
ex a m in e r ici l’éq u a tio n un term e à la fois. Pui.squc l'in d ic e de rélractio n n du verre,
ou de toute autre substance, est supérieur à 1, le term e (n — 1) doit être positif, l’uisquc
la source lumineu.se (qui est l’o b jet) e s t à la gauche, et si on considère que la lentille
est biconvexe, le cen tre d e co urbure de la p rem ière face con v ex e (la face de gauche)
se situ e du c ô té d ’o ù le faisceau ém erg e ; le ray o n de co u rb u re r , est d o n c p o sitif,
scion la co n v e n tio n de sig n e s rela tiv e aux d io p tre s sp h ériq u es vue à la .section 5.5.
De la m ême façon, le centre de courbure de la deuxièm e face (la face- d e droite) se situe
du côté du faisceau in c id en t; le rayon de co u rb u re Гз est alors négatif, toujours selon
cette convention de signes. D o n c, le term e ( 1/Г| - l / r , ) est p o sitif, tout le côté droit
de l ’équation 5.10 e.st positif, e t tous les signes sont ainsi cohérents.

a) b)

r - f 1
d)
Figure S.16 a) boyer image F' d'une lentille convergente, h) Foyei- objet /• d’iine lentille
convergente, c) Foyer image F' d'une lentille divergente, d) 1ôyer objet F d'une lentille
divergente
5.6 L es lentilles m inces 119

Les images produites par une lentille mince


O n peut m aintenant considérer les iy|ies d 'im ag es form ées par les Icnlilies convergentes
et divergentes. La figure 5.17 a) m ontre un objet réel O, plus éloigné qu e le foyer objet
f d ’u ne le n tille co n v e rg en te . O n v o it, p a r les d eu x ray o n s d essin és dans la fig u re,
q u e la lentille fo rm e un e im age / réelle et ren v ersée de T o b jet, du cô té de la lentille
oppo.sé à l'objet.
L orsque l'o b je t réel e.st situé en tre la lentille et le foyer objet F. com m e l'illu.strc
la figure 5.17 b), la lentille form e une im age virtuelle / qui .se situe du m êm e côté de
la le n tille que l ’o b je t, et qui a la m êm e o rien tatio n . L es ray o n s in cid en ts p ro v en an t
de l’o b je t .sont tro p d iv e rg e n ts p o u r q ue la le n tille p u isse les re n d re co n v erg en ts.
L o rsq u 'ils ém ergent de l'au tre côté, ils sont divergents, m ais ces rayons divergent m oins
q u e les rayon s incidents. D onc, une len tille co n v erg en te peut fo rm e r so it une im age
réelle, .soit une im age virtuelle, selon que l’objet réel est situé au-delà du foyer objet F
ou entre la lentille et le foyer objet.
La figure 5.17 c) illustre l'im a g e réelle / form ée par un objet virtuel O. Un objet
virtuel étant défini par des ray ons incidents convergents, les rayons ém ergents convergent
d av a n ta g e, d o n n an t une im age rée lle de m êm e o rien tatio n . Peu im p o rte lu p o sitio n
de l'objet virtuel O, r im a g e sera toujours réelle, plus petite et d e m em e orientation.
La figure 5.18 a) pré.sente un objet réel O devant une lentille divergente. Les rayons
incidents tom b an t su r la p rem ière face de la len tille sont d iv erg en ts (d éfin itio n d 'u n
objet réel). Donc, les rayons ém ergents de cette lentille divergente vont diverger davan
ta g e ; ain si, il n 'y au ra ja m a is d e p o in t d e renetm ire réel en tre les raytm s ém ergents.
Les prolongem ents veis l’arrière des rayons ém ergents donnent la position de l'im ag e
virtuelle form ée. D onc, quelle que soit la di.stancc de l'o b je t réel, cette lentille produit
une image virtuelle qui est située du mêiiK’ côté de la lentille que r<ibjel et qui a la m ême
orien tatio n . P ar la su ite, les fig u res 5 . 18 b) et 5.18 c) pré.sentcni les im ages ft>rmées
d ’un objet virtuel O par une lentille divergente. Si l'o b je l virtuel O sc trouve entre la
lentille cl le foyer objet F (fig u re 5 . 18 b), l’im age l est réelle et d e m êm e orientation.
Pf)iir form er cet objet \ irtuel entre la lentille et le foyer, les rayons incidents doivent être
très convergents. Q uand ils traversent la lentille d iv erg en te, ils div erg en t par rapport
à leu r directio n in itiale, m ais pas su ffisam m en t p o u r être d iv erg en ts (ils sont encore
convergents, m ais un peu m oins que les rayons incidents). Si l'o b je t virtuel O se situe
au -d elà du fo y e r o b jet f ’ d e la le n tille d iv e rg e n te, l’im age est v iitu c lle et ren v ersée
(fig u re 5.1 8 c). Ici. les ray o n s in c id en ts .sont un peu m o in s c o n v e rg en ts si on les
com pare à la situation p résentée à la figure 5.18 b). 1-a lentille est en m esure de faire
d iv e rg e r c e s ray o n s in c id en ts peu co n v e rg en ts. la*s p n ilo n g c m e n ts vers l ’arrière
p c rin e iie n t de d e te rm in e r un p o in t d ’in te rsec tio n q ui v a co rre sp o n d re à la p o sitio n
de l'im ag e virtuelle.
C om m e on l ’a vu avec les m iroirs, la distance im age q est positive lorsque l’im age
est réelle, et négative lorsque l'im ag e est virtuelle.

Les images réelles se fwinent du côté des rayons éineigenis til faut que ces rayim.s
convergent pour avoir un |X)int d’intenKx'tion réel), cl 1rs images virtuelles sc forment du côté
des rayons incideni.s (les rayons émergents divergent, donc il taul les prolonger vers l’arriéré
pour déterminer un point commun, ce qui nous amène du côté d'où provenaient les rayons).

I-
l>)
figure S.17 a) fine image /. réelle et renversée, est formée par une lentille a'nvcrgenic lorsque
l'objet réel O est situé au-delà du foyer obict F. b) L'image / est virtuelle cl a la même orientation
que O lorsqiK' O est situé entre la lentille et le foyer objet, c) L'image / est réelle et a la même
orientation que O lorsque l’objet O est virtuel, peu importe sa position.
120 Chapitre 5 Les miroirs et les lentilles

<)

Figure 5.18 a) Une lentille divergente forme une image virtuelle /, dont l’orientation est la même
que celle de l’objet réel O, que O soit plus éloigné ou plus rapproché que le foyer image h"
de la lentille, b) Pour un objet virtuel O. l’image I est réelle et de même orientation si robjei
se trouve entre la lentille et le foyer objet F\ c) si l’objet virtuel f> se situe au-delà du foyer
objet F. l'image / est virtuelle et renversée.

Le grandissem ent transversal m produit par les lentilles convergentes et divergentes est
donné p ar les équations 5.5 et 5.6, co m m e dans le cas des m iroirs. Un grandissem ent
tran sv e rsal m n é g a tif in d iq u e q u e l’im ag e / e s t ren v e rsé e p a r rap p o rt à l ’o b je t U ;
si le grandissem ent m est positif, l'im a g e / a la m êm e orientation que l’objet O (im age
droite).

■( ‘. OL.U OM 3£ PPOtíLt-'MC'J m
l” flrutégie Confusion des signes uvec les miroirs et les lentilles
Faites aiirniiiin; un miroir dont la surface c.st convexe a une distance focale/po.sitive, contrairement à une lentille dont les surfaces sont
focale / négative, contrairement à une lentille dont les surfaces concaves. La confusion entre les propriétés des lentilles et celles
sont convexes Un mintir dont la surface est concave a une distance ties min>irs est une erreur fréquente.

Localiser les images d'objets étendus en traçant


les rayons principaux
L a figure 5. IQ a) m ontre un objet réel Ü plus éloigné que le foyer objet F d ’un e lentille
convergente. O n peut localiser graphiquem ent l’im age d e tout point hors de l’axe d ’un
tel objet (com m e la pointe de la flèche dans la figure 5.19 a) en traçant un diagram m e
de rayons com prenant au m oins deux des trois rayons principaux passant p ar ce point.
On xous conseille fortem ent de tracer le troisièm e rayon, car celui-ci pci met de valider
le schém a obtenu. C es rayons, ch o isis parm i tous ceu x qui trav ersen t la lentille pour
form er l’im age, sont les suiv ants.

1. Un rayon incident parallèle à l’axe optique est dévié, pour un e lentille convergente,
vers le foyer im age F ' (rayon 1 dans les figures 5.19 a, b el d), et. p o u r une lentille
divergente, de sorte qu e son prolongcm eni passe par F ' (rayon 1 de la figure 5.19 c).
2. U n rayon incident passant par le foyer objet f (rayon 2 dans les figures 5.19 a et d).
ou dont le prolongem ent passe par F (rayon 2 dans les figures 5.19 b et e). ém ergera
de la lentille parallèlem ent à T axe optique.
3. U n rayon in cid en t d irig é vers le cen tre de la le n tille ém erg era d e la le n tille sans
changer de direction ( r a y o n d a n s toutes les figures 5.19). parce que ce ray«)n tom be
sur les deux faces de la lentille à l’endroit tàt elles sont presque parallèles.

L ’image du point est ItKalisée au point d ’intersection des rayons ém ergents. .Si les rayons
ém ergents sont convergents, cette image csl réelle et la distance image q est alors positiv e.
Si les ray o n s ém erg en ts sont d iv e rg e n ts, la p o sitio n du p o in t im ag e est d éterm in é e
par les prolongem ents des rayons ém ergents vers f a riiè re . C e point com m un donne la
position de l’im age qui est virtuelle el dont la distance im age q est négative. On trouve
l’im age de l’objet en localisant les im ages de deux ou de plusieurs de ses points
L a figure 5.19 b) illustre de quelle façon les prolongem ents dc.s trois rayons pciiv ent
servir à locali.scr l’im age virtuelle d ’un objet réel plus rappr<x;hé que le foyer F d ’une
lentille convergente. N otez qu e c ’est le prolongem ent du rayon incident 2 vers l’arrière
qui passe par F.
5 6 Les lentilles minces 121

b)

par une lentille mince, d’un objet réel a) plus éloigné que le loyer ob|ct F de la lentille
convergente, b) plus rapproche que le foyer <45jet de la lentille convergente, ou c) qti'il soit
n'importe oil en fac-e d'une lentille divergente, d) L'image formée d'un olijet virtuel
par une lentille convergente.

L a figure 5.19 c) illustre un exem ple du parcnur.s de.s iroi.s rnyon.s principaux pour
une lentille diveigenie à partir d ’un objet réel O n rem arque que le layon I qui ém erge
de la lentille ne passe pas par le foyer image F ' mais esl dévié scion sa direr (ion (c ’esl son
prolongem ent vers Гагпеге qui passe p a rc e foyer). L e rayon incident 2 ne passe jras par
le foyer objet F, il est .seulement orienté vers ce point (c 'e st son prolongem ent qui passe
p ar ce point). P our cet ex em p le, la p o sitio n de l’im age .se tro u v e à l ’in te rsec tio n du
rayon 3 et des prolongem ents vers l'arrière des rayons 1 et 2.
La figure 5.19 d) illustre l'im age formée d 'u n objet virtuel pariiiie lentille convergente
Four déterm iner la position de l’objet, il faut prolonger les rayons incidents convergents
de l’autre côté p o u r d éterm in e r un pt.nnt coin n u tn ; ce point co rresp o n d à la p osition
de l’objet virtuel.
En traçant des diagram m es de rayons lum ineux à partir d ’objets fécl.s se rapporianl
aux lentilles minces convergentes et divergentes, remplissez le labicati 5.3. Sous la mention
Position de la ru b riq u e Inuifte, n o tez si l'im a g e e s t du m êm e cô té de la le n tille que
l ’o b jet, ou du cô té o p p o sé. S o u s la m e n tio n Type, in d iq u e z si l'im a g e esl rée lle ou
virtuelle. Sous la m ention Orientation, indiquez si l’tm agc a la m ente orientation que
l’objet, ou si elle est renversée. Sous la rubrique Signe, in d iq u e/ le signe de la qiiaritilé,
ou indique/. ± .si le .signe est am b ig u . P a r la su ite, to u jo u rs à l ’aid e d e d iag ram m es
de rayons lum ineux pour des lentilles m inces, lempli.sse/. le tableau 5.4 pour des objets
virtuels.

TABl fAU SJ Tobtefflj d« (OfoctéristKiUKdes iiroges des lentilles mintes pour des objets féeh
Image Signe
Type Position
de lentille (Je l’objet réel Position Type (Jrientation
Entre F et la lentille
C o n v e rg e n te-------— ----
Entre /■ et l'infini
Divergente N’importe où

BBLtAU 5.4 TiAleou des cormiéristiques des images des lentilles ininres pour des objets virtuek
Image Signe
T)pe Position
de lentille de l'objet virtuel Po.siiion Type Orientation
Convergente N'importe où
Entre F et la lentille
üivergenfe
Entre F et l'infmi
122 Chapitre 5 Les miroirs et les lentilles

Les systèmes de deux lentilles


P our déterm iner, à p artir d 'u n objet O, la po<iilion de l ’im age linale 1 fo rm ée lorsque
les ray o n s in c id en ts tra v e rse n l un sy stèm e d e d eu x le n lillcs d o n t les ax es o p tiq u e s
coïncident, on procède par étapes. Soit L^, la prem ière lentille rencontrée par les rayons
et ¿ 2, la deuxièm e lentille que traversent les rayons lum ineux.

1" étape Soit p , la distance objet entre l'o b je t O , cl la lentille L, O n trouve la distance
d e l'im a g e p ro d u ite p a r la le n tille so it en u tilisa n t l'é q u a tio n 5.9 , .soit en
traçant à rc c h c llc le parcours des trois rayons principaux traversant cette lentille.
2*étape Ensuite, on ignore la lentille L, et l'in rag c trouvée à la prem ière étape devient
l'objet de la lentille L^. En effet, les rayons émergent.s (im age) d e />, deviennent les
rayons incidents (objet) de L^. Si les rayons incidents sur la deuxièm e lentille sont
convergents, la d istan ce objcl p 2 de la lentille L-, est considérée co m m e négative.
Si les rayon s du nou v el o b je t so n t d iv e rg e n ts lo rs q u 'ils a rriv e n t au n iv eau de
la d eu x ièm e le n tille , alo rs cet o b je t est réel et la d istan c e o b je t p j esl p o sitiv e.
O n trouve e n s u ite la d ista n c e q i de l'im a g e (fin a le ) p ro d u ite p ar la le n tille Lj
en u tilisa n t l'é q u a tio n 5.9 ou en traçan t à l'é c h e lle le p arc o u rs d es trois rayons
principaux qui la traversent.

C ette m éthode par étapes peut être utilisée quel que soit le nom bre de lentilles, ou si une
ou d es lentilles sont rem placées par un ou des m iroirs.
Le grandissem ent transversal total M produit par un systèm e de deux lentilles est
le produit des grandissem ents transversaux w , e t nu générés par les deux lenlillcs ;

M = шриг- (5.11)

Exemple 5.3
f u c iiiaiiic: irtigieusc sc tieni le long de l'axe optique à 20 cm d'une cette face sc situe du côté d'oi'i émergent les rayons ; en employant
lentille mince ayant des rayons de courbure identiques. Le grandisse­ la convention établie à la section des dioptres sphériques, le rayon
ment transversal de la ntnnte produit par la lentille est m = -0 ,2 5 , r, est positif et vaut + r. De même, la deuxième face rencontrée
et l’indice de réfraction de In lentille est 1,65. pai la lumière est concave. Le centre de courbure se situe du côté
des rayons incidents ; alors fi = —r.
a) Determine/ le type d'image (réelle ou virtuelle) produit par la lentille
3. On peut relier ces rayons de courbure i la distance focale /
et le type de la lentille ; détermine? aussi si l'objet est situé entre la
par la formule des opticien.s, l’équation 5.10 (la seule équation
lentille et le foyer ou entre le foyer et l'infini, le côté de la lentille oti
mettant en jeu les rayons de courbure des faces d'une lentille).
l'image apparaîl et si l'image est renversée.
4. On peut relier/ à la distance objet /; et à la distance image q
SOlUflON : Le contepl tié ici est que la \ aleur donnée de m permet d'en par l'équation 5.9.
savoir heauccHip .sur la lentille et l'image. On voit, d'après l'équation 5.6
On connaît />, mais non q. On détermine </ en utilisant l'équa­
(m = qlp), que
tion 5.6 du gnuidis-sement im et p sont connus) et on obtient
q = ~mp — 0.25p.
q = (0.25) (20 cm) = 5.0 cm.
On peut donc répondre aux questions avant meme de terminer Ic.s
ealciils Pnisqtie l'objet est réel, p esl positif; q doit aussi être posiiif L'équaiion 5.9 donne maintcnanl
dans ce ea.s. Cela signifie qu'on a une image réelle et, de ce fait, le seul I _ 1 I _ 1 I
type de lentille pouviuit ptoduire une image réelle à partir d'un objet réel
f p q 20 cm 5.0 cm
esl une lentille eonvergcmc. En effet, selon l'équaiion 5.9 des lenlillcs
minces, lorsque les valeurs de p et de ^ sont positives, celle d e/d o it qui permet de trouver / — 4.0 cm.
être ixwitive. ec qui est une cametéréstique tfiine lentille eonvergente. L’équation 5 .10 donne alors
L'objet réel doit se situer enire le foyer objet et l'infini pour que les
rayons émergents puissent converger afin de produire une image
réelle (vous pouvez également démontrer, en remplaçant q par 0.25;;
dans l’équation 5.V, que p = 5f. donc que l’objet c.st cinq fois plus loin
ou. en in,scrant les valeurs connues,
de la lentille que son foyer objet). Piii.sqiic la di.stanec image q esl
positive, les rayons émergents convergent pour donner une image 1
=(1.65-1)-.
rfrlle situer du eiVé rie la lentille oppisé à l'objet. L.e grantlissemem m 4.0 cm r
est négatif, ce qui indique que l’image est renversée par rapixin à l’objet.
ce qui donne
b) Uucis sont lus dcu.x laym s de courbure ilc la lentille'’
r = (0.65)(2)(4,0 cm) 5,2 cm. (reponse)
SOtUnON Les toiKephti« som les suivants
1. Puisque la lemillc esl syméirique, (le rayon de euiirhiire de la ^ V É R I F I E Z v o s CONNAISSANCES 4 ; )ne le n tille m in c e ayant
surface qui iiiierceptc les rayons incidents) et r , ont la même des rayons de courbure identiques produit Limage d ’une empreinte
grandeur r. digitale avec un grandissement de + 0,2 lorsque l’empreinte est siiuéc
2. Piii.sque la lentille esl convergente, la première face rencontrée 1,0 cm plus loin que le foyer par rapport à la lentille. Quels .sont le
par les rayons lumineux est conv exe. I-c centre de courbure de type et rorientation de l’image, et quel e.si le type de celte lentille ?
5.7 L'œH 123

Exemple 5.4
La figure 5.20 a) représente une graine de piment O, placée L,
devant deux lentilles coaxiales et i.j. dont les di.stanccs focales
so n t/i = +24 cm et = + 9,0 cm. et qui .sont séparées par une
distance 0 = 1 0 cm. La graine sc trouve à 6.0 cm de la lentille
Où le système de deux lentilles produit-il une image de la graine ?
O,
Í
SOUlîlON: On pourrai! localiser l'im age produite par le système de
lentilles en traçant des rayons incidents provenant de la graine et
traversant les deux lentilles. Toutefois, le totitepldé ici est qu'on peut
localiser cette image en analysant le système par étapes, une lentille P\ 1
à la lois. On commence par la lentille la plus rapprochée de la graine. a) i
1
On veut localiser l'iniagc finale, c’csl-à-dire riniuge /j produite 1
par la lentille f.i l-i 1
Lcntillt L\. On Ignore la lentille cl on localise l’image J
pnxluilc pur /.| en appliquant l'équation 5.9 à la lentille L, sculeineni ;
1
1
I I I к• Г
---- 1----- = - - è, 1
P\ 9i fl 1
1
1
L’objet i/, de la lentille L\ e.st la graine, située à 6.0 cm de la lentille, 1
1
et est réel ; on insère donc p, = + 6.0 cm. On insère également lA . 1
la valeur donnée de/,, et on a alors />“»•
/ , - t
_ 1___ _ 1 b) 1
!
-I-6 .Ü cm q, +24 cm ' 1
1
1
ce qui donne = “ 8.0 cm.
l J valeur négative obtenue pour la distance image indique que
les rayons émergents divergent (image virtuelle). On doit prolonger aü
les rayons émergents vers I’arricrc pour trouver la position de
l'image, ce qui signifie que l’image se retrouve du même efué que n
l’objet, à 8.0 cm de 1« lentille. Le grandissement est positif (p est B J
positif et q est négatif), ce qui indique que l’image a la même oricn- T
talion que la graine, comme l'illustre la figure 5.20 b).
IjmUHe Lj. A" celte deuxième étape, le contepl dé est qu’on peut
— Vï -
considérer l'im age l\ comme l'objet O , de la deuxième lentille.
O ■fti'
Cl ignorer la lentille L,. On note d'abord que cet objet O, est plus
éloigné que le foyer de la lentille L.. D’apres le tableau 5.3. l’image Figure 5.20 LxcmpIc 5 4 al One graine O, est située à une distance/i,
produite par la lentille L, doit donc être réelle, renversée et située de la lentille f-i I ticiix lentilles /,, et ¿2 sont séparées par
du côté opposé de la lentille par rapport à Oi. On doit vérifier ces une distance D. La flèche indique ro i ieiUalion de la graine,
affirmalums à l’aide de calculs appnipriés. bl L'image /, produite par la lentille /.,. c) L'image /, agit coitiitr.’
Iji distance objet P2 entre eel objet Oj cl la lentille L, c"*'- l’obiet O2 relativement à la lentille Z.2. ce qui prtxluii rimage
d'après la figure 5.20 c). finale I

P; = f) + lq,l = 10 cm + 8.0 cm = 18 cm.


Le signe positif de la distance image confirme les suppositions faites
L'équation 5.9 applirpiée à la lentille donne alors l'im age /, produite par 1« lentille /..j e.st réelle. Le quotient q/p
exprimant le graïulisscnicnt donne une valeur négative. Ce qui
I I I implique que l'im age est renversée. Le signe positif de q indique
+ 18 cm q2 + 9 .0 cm que les rayons émergents tk- la deuxième lace de la lentille eonvergem,
donc que l'image est située du côté opposé de la Icnlillc L; pitr rappiirt
Donc. ç: = +18 cm. (réponse) à i>2, comme on le voit dans la figure 5.20 c).

5.7 L’œ il
L ’œil hiirnain (voir la figure 5 .2 1) est un globe d 'u n diam ètre d 'en v iro n 25 mm. l,a partie
exierne de l'œ il est fo rm ée d 'u n e m em brane transparente, c|ue Гоп nom m e la cornée.
ayant une forte courbure et un indice de réfraction d ’ens'iron 1..33 Les rayons lum ineux
entrant dans l’œ il sont d ’abord fortem ent réfractés par la cornée, puis traversent, sans
être dévié.s. un liquide d 'tn d ic c 1.3.3 appelé I h um cur aqueuse Par la suite, les rayons
124 Chapitre 5 Les miroirs et les lentilles

Distjuc du lum ineux subis.senl une seconde déviation en traversant le cristallin, q u i est une lentille
0|>lK|UC biconvexe de distance focale variable et d ’indice d e réfraction d ’environ 1,40. L a lum ière,
Ncil
optique finalement, se propage dans un autre liquide d ’in d iœ 1,33, rh iin ic u r vitrée, pour term iner
sa course sur la rétine qui tapisse le fond de l’ceil. La rétine est une m em brane form ée
de cellules nerveuses (les cônes et les bâtonnets) sensibles à la lum ière et dont les influx
nerveux sont achem inés au cerveau par le n erf optique. Les deux déviations que subissent
les rayons lum ineux (environ le quart de la dév iation totale est causée p ar le cristallin)
pciTnetient la form ation d ’im ages réelles .sur la rétine.
Le systèm e optique de l ’œil étant très com plexe, il est souvent suffisant d ’utiliser
un nuxlèle simplifié com m e celui de la figure 5.22. D ans ce mtxlèle. la cornée et le cristallin
sont c o n sid é ré s co m m e un e le n tille m ince co n v e rg en te « é q u iv ale n te » d e di.stance
focale variable, /v„, baignant dans l’air. L a rétine esl située à une distance fixe f d e la
lentille (envinm 15 mm), et e st équivalente à un écran sur lequel on recueille les im ages
Ciliomîdr réelles produites par la lentille.
S rlir o tiiim - l 1 rtiM Hlliii Que l'objet soit éloigné ou rapprtxîhé de l’œil, l’image réelle produite sur la rétine doit
être nette, afin que la vision soit nette. Puisque ( est constante (la rétine ne peut s ’ajuster
Kihres
ctiwires à la position de l'im a g e selon la position de l'o b je t), l’œil s'acco m m o d e aux différentes
distances objets en faisant varier la distance focale du cristallin ; on ap[>elle ce prixx;ssus
(ÎQurfi 5.21 Les difféi'ciilc^ utinpoNunle.s l ’ac c o m m o d a tio n . D e petits muscles ciliaires, en .se contractant, dim inuent les rayons
(le l'œ il humain l ^ lumière pénétrant
de courbure d es faces du cristallin . L orsque les m uscles ciliaires sont relâchés, l'œ il
dan.'» l'ccil C.SI réfrtiticc par la cornée
clam au repos, l’o bservation s'effe ctu e sa n s a c c o m m o d a tio n ; les rayons de courbure
ei le cristallin qui agis.sent comme
du cristallin sont alors à leur v aleu r m axim ale et, con séq u em m en t, la d istance locale
une Icniille eonvergenie. permettant
du cristallin (de m êm e que celle de la lentille équivalente) est m axim ale. Si les m uscles
tun6Î la foiiiiation d’inmpes rccllc.s
ciliaires sont co n tra cté s au m axim um , l ’e ffo rt étant m ax im al. l ’œ il utilise alo rs son
Kur la rétine I, action des muscles
a c c o m m o d a tio n m a x im a le , les v aleurs des ray o n s d e co u rb u re du cristallin et de la
ciliaires sut le crisiallin permet
de faire varier sa distance focale. distance focale de la lentille équivalente sont donc m inim ales.
O n définit la p u issa n c e d ’u n e le n tille P par ;
1
P = -. (5.12)
/
L ’u n ité S I de P est la d io p tr ie , sy m b o lisée p ar D, q u i, .selon l'é q u a tio n 5 .1 2 , est
l’équivalent de l'inverse du m ètre (m “ '). La position de l ’objet dont on obtient une image
nette sur la rétine sans accommodation est appelée le p u n c tu m re m o tu m (PR). Lorsque
l'œ il perçoit nettem ent un objet à cette position, sa puissance est minim ale, P„„„, puisque
sa distan ce focale est m a x im a le,/„a * . De la m êm e façon, la position de l ’ohjel dont
on o btient une im age nette su r la rétine avec l'acconunodation maximale est appelée
le p u n c tu m p ro x im u m (PP). Pour observer un objet avec une visitxi nette au punctian
proximiim. la p u issan ce de l’œ il d o it être m axim ale, p uisque sa d istan ce lo c a le
est m in im ale, . L a fig u re 5 .23 rep rése n te un œ il av e c, d an s ce cas particu lier,
les positions de son PP et d e son PR. L '(cil peut produire une im age nette su r la rétine
de tous les objets situ és en tre son PP et .son PR. C ette rég io n de l’esp ace située entre
Figure 5.22 Modèle simplifié de l'ccil le PP et le PR où la vision est nette icprésente le d o m a in e d e v isio n d istin c te à l’œil nu.

Figure 5.23 Representatinn des positions du PP et du PR. pour un œil donné,


ainsi que de son domaine de vision distincte à l’œil mi
5 7 L'œil 12S

a) b)
Figuie 5.24 Pour un <p Mnormal,
a) le PR est situe à rinfini, l’a il Un œ il e m m é tro p e possède un P R situé à l’infinu ce q ui constitue l’une des carac­
est emmétrope, et b) le PP est siiué téristiques d ’un œ il norm al ; la figure 5.24 a) illustre cette situation. A partir d ’un objet
environ à une dizaine de centimètres éloigné, l ’œil produit sans accom m oder une image réelle .sur la rétine B œst donc ¡lossibk
de l’œil pour une personne d’une p o u r l’œ il d e n ettem en t d istin g u e r san s effo rt les o b jets lo in tain s. U ne v isio n nette
vingtaine d'années. lo in ta in e n ’est pas une v isio n qui p e rm e t de p e rc e v o ir d ’in fim e s d étails d 'u n o b je t
élo ig n é : clic est, en fait, le co n traire d ’un e vi.sion floue. O n peut, p ar ex em p le, bien
distinguer la dém arcation entre le contour de laL u n e et le fond du ciel noir l e PP d*uii œ il
norm al est situé à une dizaine de centimèlre.s de Pieil pour une pei'sonnc d 'u n e vingtaine
d ’années. C e tte d ern iè re d istan ce au g m en te av ec l'â g e au fur et à m e su re q ue l’œ il
com m ence à souffrir de presbytie. T a figure 5.24 b) illustre le parcixirs des rayons lumineux
issus d ’un objet situé au PF d ’un œil norm al alors que son accom m odation est m aximale.
O n peut évaluer la puissance de l ’œil ou d ’une lentille en com binant l'éq u atio n des
lentilles m inces (équation 5.9) et l’équation 5.12, ce qui perm et d 'cc rirc ;

r. ' < 1
P = 7 = -F - . (5.13)
f P Ч
Par exem ple, si l’objet est situé au PI’, l’ccil d o it être à sa puissance m axim ale F™, afin
que la vi.sion soit nette ( / e s t m inim ale). L a d istance o b je t/> corresp o n d à la di.stancc
entre l’œ il et son PP, que l'o n imte i/pp (voir la figure 5.23). I^a distance im age q vaut i.
O n peut donc écrire l ’équation 5.13 sous la form e :
1 1 1
Рпшх -- (5 14)
/n iil dpp
Si l’o b jet est situ é au P R , à une d istan ce de l'œ il, la p u issan ce d e l'œ il d o it être
minimale pour pnxiuire une image nette sur la rétine. L ’équation 5.1 ( p erm et d ’ccrirc

f^rnin — (5.15)
r/pK (
La p u iss a n c e d 'a c c o m m fx la tiu n (ou am plitude d ’accoinmcxlation) est. par iléfmi-
tio n . la d iffé re n c e e n tre la p u issa n c e m a x im a le e t la p u issan ce m in im ale de l ’ccil
- ^no.y ) En soustrayant l’équation 5.15 de l ’équation 5.14, on obtient

_1 I
Ptx-c ( .S I fi)
r/pp dpR

La myopie
D ans la plupart des cas, la m y o p ie est causée par un globe o culaire qui est trop long par
rapport au rayon de courbure de la cornée et à la puissance du cristallin. La rétine étant
trop éloignée d e la lentille équivalente, l’icil du m yope est donc trop convergent, cc qui
entraîne une mauvaise vision des objets élnifinés. F n effet, com m e l'illustre la figure 5.25 a),
l'im a g e correspondant à un objet h l'in fin i se fait en av an t de la rétine, que l ’œ il soit
accom m odé ou non ; il en résu lte sur celle-ci une tache floue qui rend im possible une
vision nette d es o b je ts élo ig n és. D ans un tel c a s, le punctum remotum n 'e s t pas
à l'infini (dpR Ф 14. ) m ais à une distance devant l ’œ il qui peut varier de quelques mètres
126 Chapitre 5 bes miroirs et les lentilles

a)
Figure 5.25 Pour un œil myope, a) la vision des ohjeLs éloignés
n’est pas nette puisque l’image sc forme devani la rétirx' ;
h) le PR n’est donc pas à l’infini, mais situé devani l’œil
à une distance qui dépend de la gravité de la myopie,
c) On corrige la myopie à l’aide d’une lentille eonectrice
divergente qui d ’un objet 0 | à l'infini produit une iitiage
vinuelle /iPsituéc au PR de l’œil.

à t|uelques centim ètres selon la gravité de la m yopie (voir la figure 5.25 b). Un leil est
donc considéré co m m e m yope si 0 < d i^ < peu im porte où se situe son punctum
proximum.
P o u r p erm ettre au m yope d ’o b se rv e r des objets lointains sans accoinmodci; tout
com m e une p erso n n e em m étro p e, on p la ce ra d ev a n t son œ il u ne len tille co rrec trice
d iv e rg e n te Ly, co m m e c e lle de la fig u re 5 .25 c), q ui d 'u n o b je t Oy situ e à T in fin i
produira une im age virtuelle /,V 'à .son PR. C ette im age virtuelle de la lentille correctrice
deviendra un objet réel OiR p o u r l'œ il, qui ptrurra en faire l'o b serv a tio n sans accom -
tnodalioti.

Exemple 5.5
Une personne myope ne peut nettement distinguer les objets au-delà L’objet O, des lentilles correctrices L, étant situé à l’infim, on a
de 2,00 m. (Supposez que les yeux de la personne sont identiques.) P (V).Puisque l’image /|V 'de /.., est virtuelle {q < 0) et située
au PR. alors q ~ “ c/pK —2,(X) in. En insérant ces valeurs de />
a) Üéterminci' la prescription (la puissance) de ses lentilles correctrices.
cl de q dans l'équation des lentilles minces, on détermine la valeur
SOlUilON; Les concepts clés sont les suivants.

1. Puisque la personne a une vi.sion floue de tixis les objets éloignes


de plu.s de 2.(K) m, son PR est situé à 2,00 m de ses yeux, donc = —0 ,5 0 0 0 . (réponse)
2,00 m
= 2,00 m.
2. Pour corriger sa myopie, on doit placer devant scs yeux des
lentilles correctrices divergentes ¿i (voir la ligure 5.25 c) qui b) Si son PP est situé à 20.0 cm de ses yeux, quelle est sa distance
d ’un objet Oi siiiié à l’infini produiron! une image viriuelle /,V' minimale de vision distmclc avec ses verres coiTccteur.s ?
à .son PR. Cette image virtuelle deviendra un objet réel (>•>/?
pinii l’œil situé à son PR. il pourra donc en faire l’observation SOLUTION. Sans ses verres correcteurs, celle personne possède un
.sans aecomnuxlation. domaine de vision distincte de 20.0 cm à 2,(K) m de ses yeux
3. On applique l’équation des lentilles mim es P _ — — fétiua- La présciKe des verres correcteurs de —0,,5()0 O déplace la distance
P q nuLxinuitf de vision distincte, qui est de 2,(X) m à l’oeil nu, à l'infini.
tlo n s I a u x l e n t i l l e s e o r r c c t n e c s a f in d e d é t e r m i n e r le u r On doit donc s’attendit; à cc que sa distance miniwnh' de vision
piiU x an ce p. distincte, de 20.0 cm à l’œil nu. soit également déplacée. En effet,
4. I o rs d e s c a lc u ls , o n p e u t n é g lig e r la d ista n c e e n tre le s y e u x e t le s la personne perçoit non pas les objets places devant les lentilles
v e r re s c o i r r e t c u i s i ai ir lle - e .i e s t, g é .n e ra lc m e m . heauccH ip p lu s correctrices, mais les images produitc.s par cdics-ci. qui deviennent
fa ib le q u e les a u tre s d ista n c e s (o b je t, im a g o et fo c a le ) n é c e s sa ire s ries objets piiur l’œril. Ces images doivent absolument être situées
à la r é s o lu tio n d u p ro b lè m e . entre .son PP et son PR afin que leur perception soit nette.
57 L œ ll 127

Figwe 5.26 Exemple 5.5 La distance


ininiinale de vision distincte avec
une lentille correctrice e.st déterminée
en calculant la distance objet p
de la lentille ccarectrice lorsque Ui
son image est au PP de l'teil.

Le concept clé utilisé ici est donc le suivant : on doit déterminer où L'objet est donc à une tliswnec
se situe l’objet O, des lentilles correctrices qui produit une image
virtuelle /, Vau PP La figure 5.26 illustre la situation. P “ 22.2 cm. (réponse)
On applique de nouveau l'équation des lentilles mince* ce qui correspond й sa distance minimale de vision disiincic avec sc»
P. = - + - ! - aux lentilles eorreclriccs ayant une puissance verres correcicnrs. Remarque/ que la personne myope possède avec
P V scs verres une distance minimale de vision distincte supérieure par
de —0,500 D Puisque l’image virtuelle /,V est située au PP de l’œil, rapport à celle à l’œil nu, ce que démontre, tl’aillcuis. la figure 5.26
on a q = —r/pp = 0,200 m et l'équation des lentilles devient ; (l’objet OiR est plus éloigné de l'œ il que le PPl, Cette légère perte
de vision nette près de l’œil (env iron 2,2 cm dans cei exemple) esi
1 très largement compensée par le très grand gain en vision éltiigncc
-0 ,5 0 0 n = +
P —0.200 m (de 2.00 m à l’infini).

L’hypermétropie
L’h y p erm é tru p it* est gén éralem en t cau sée p ar un globe oculaire qui C.5I trop co u rt par
rapport à la puissance de la lentille équivalente de l’œ il. La rétine étani im p rapprcK hcc
d e la le n tille é q u iv a le n te , l ’œ il d e l'h y p e rm c titip c n 'e s t d o n c pas assez co n v e rg en t.
S i la p erso n n e h y p erm é tro p e n 'a c c o m m o d e p as lu is q u 'e lle u h s e iv e un o b jet situ e
à l’infini, l'im a g e se fo rm e d errière la rétine (voir la figure 5.27 a) Afin de di.stingucr
nettem ent les objets éloignés, rh yperinctropc est dont obligé d ’accom m oder (d’auam eniei
la p u issan ce de son c rista llin p a r l'a c tio n de ses muscle.s c ilia ire s), ce q u i provrxpic
à la lo n g u e un e fatig u e q u i p eu t se tra d u ire p ar des m aux de tête. P u isq u e l ’œ il d un
h y p en n étro p c débute déjà son accom m odation pour des objets éloignés, son acctminuT-

a)

Figure 5.27 Pour un œil hypermétrope, a) la vision


des objets éloignés sans aeconimodalion n’est
pas nette puisque Vimage sc ibrme derrière la rétine
d ’œil n’e.si pas assez convergent), et h) le PR est
im objet virtuel situé derrière l'œil, dpR < 0.
c) ()n corrige l’hypermétropie à l’aide d’une lentille
correctrice convergente L, qui d ’un c>tijet 0^ à l'infini
produit une image réelle /jR située au PR de Pœil.
I2B Chapitre 5 Les miroirs et les lentilles

dation m axim ale esl donc atteinte pour un objet situé plus loin de l’œ il que pour un œil
norm al, ce qui peut entraîner une m auvaise vi,sion des objets rapprochés.
L a fig u re 5.27 b) illu stre la p o sitio n du PR p o u r un œ il h y p erm é tro p e. P u isq u e
l’œ il, sans accom m o d atio n , n ’est pas su ffisam m en t co n v erg en t, les ray o n s in cid en ts
de l’œii doivent déjà être convergents afin que l ’œil les focali.se sur la rétine. L e punctum
remotum de l’œ il hyperm étrope est donc un o b jet virtuel (faisceau incident convergent)
situé derrière l’œ il, ce qui im plique q ue dpn < 0 .
P o u r q u e l ’œ il p u isse o b se rv e r des o b jets lo in tain s sans accommoder, on place
devant lui une le n tille c o rre c tric e co n v erg en te ¿ | . co m m e celle de la figure 5 .27 c),
qui d ’un o bjet O , situe à l’infini produira une im age réelle /,/? à son FR. C ette image
réelle àc la lentille correctrice deviendra un objet virtuel POUf situé à son PR ;
il p ourra donc en faire l’observation sans accom m oder.

Exemple 5 6
Une personne hypcnncinipc possède des verres Lxxmiairs de -r-2.00 D SOLUTION. La personne perçoit avec scs verres non pas les objets
et un KP situe à 20.0 cm de ses yeux. placés devant les lentilles correctnecs mais les images produites
par celle.s-ci. qui deviennent des objets pour l’œil Oimc. l'iniage
a) Où c.sl situe son PR
la plus rapprochée de l'œil que la lentille peut produire i|in qu'elle
SOUIIIOH' La posrtion du PR est déterminée à l'aide des deux tonreph dis soit nettement perçue par ta personne doit être située à son PP.
suivants Le toixept dé utili.sé ici est donc le suivant : on doit déterminer où
«e situe l’objet des lentilles correctrices qui produit une image
1. Selon la figure 5.27 cj. les verres correcteurs de +2,00 D
virtuelle /jP a u PP. La figure 5.28 illustre cette situation.
produisent, à paitir d’un obtcl 0 i à l'infini, une image réelle l,R
On applique de nouveau l'équaiion des lentilles minces
siiiiée an PK de la personne : cette image devient un objet virtuel
I 1
0 )V pour l’itil situé B.Min PR que la personne peut donc nettement Pj. = ---- 1- — aux lentilles correctrices ayant une puissance fi.
distinguer sans accommoder. P <i
I 1 de +2,00 D. Pui.sque l’image virtuelle /,V est située au PP de l’œil,
2. On applique l'équation des lentilles minces Pc — I- -
P <7 on a V = ~dfT = - 0,200 m et l’équation des lentilles devient ;
(équation 5.13) aux lentilles correctrices afin de déterminer la
dislance image q.
+ 2.00D = - -+ .
L'objet O, des lentilles correctrices étant situé à l’infini, on a P —0,200 ni
P - SC. Puisque l’image réelle (q > 0) /,R des lentilles correctrices f ,

devient un objet virtuel OjV' pour l'œ il situé à son PR (i/pR < 0), L'objet O, esl donc à une distance
on peut écrire q — —dpp. En insérant les valeurs de Pc et de p
dans l'équaiion des lentilles minces et en remplaçant q par —dpp, P = 14,3 cm. (réponse)
on détermine la valeur de ■
ce qui correspond à sa distance minimale de vision distincte avec
+ 2,(X)D = — , ses verres correcteurs. Remarquez que la personne hypermétrope
oc - i/p R
possède avec ses verres une distance minimale de vision distincte
</pR = -0.500 m. (réponse) intérieure par rapport à celle de 20,0 cm à l+cil nu, ce que démontre
également la figure 5.28 (l’objet (>iR est plus près de l’œil que le PP).
Le PR de la personne est donc situé derrière elle, à 50,0 cm de
Les lentilles correctrices d'une personne hypermétrope ont donc deux
scs yeux.
actions bénéfiques sur .sa vision : elles lui permettent d'observer des
b) Quelle est sa di.staiKe nûniinaie de vision distincte avec ses verres objets éloignés sans accommoder (sans effort) et clics lui procurent
correcteurs ? une meilleure vision rapprochée.

Figure 5.28 Exemple 5 f> La distance


minimale de vision distincte avec
une lentille correctrice esl déterminée
en calculant la distance objet p
de la lentille correctrice lorsque LH
son image est mi PP de l'ieil.
5.7 L œ il 129

La presbytie
La p m b y t i e est une conséquence du vieillissem ent des m uscles ciliaires et du cristallin,
ce dernier perdant de son élasticité avec les années. Le vieillissem ent des organes de l’œil
sollicités dans le prtx;cssas d 'accom m odation entraîne donc une dim inution progressive
de sa p u issan ce d ’ac co m m o d atio n . L a p resb y tie u ’a lfe c ie pas la v isio n d es o b je ts
éloignés, m ais la vi.sion des ob jets rap p ro ch és d evient pro g ressiv em en t p lu s d ifficile
à m esure que l'acco m m o d atio n est de plus en plus restreinte. P our un leil em méirojx;,
le PR dem eure à l’infini, mais le PP, avec l’âge, s’éloigne de l’icil. La personne presbyte
doit donc lire de plus en plus loin de ses yeux et la lecture prolongée peut devenir difficile
et fatigante, su rto u t à p artir de l’âge de 45 ans. Le PP m oyen d’un œ il em m étrope est
situé à 25 cm de Vieil. U n œil emmétrope est donc presbyte si son PP est situé au-delà
de 25 cm de l’œ il, ce qui correspond, selon l’équation 5.16. à une pui.ssance d ’accora
m odation de 4,0 ü . G énéralem ent, on coasidère q u '« « œJl xouffre de presbytie ¡nrsifue
sa puissance d'accommodation devient inférieure et 4.1KJ D. L a presbytie s’ajouie aux
autres défauts de l’œ il : un œil myo|x* ou hyperm étrope deviendra presbyte avec VAgr.
et il sera alors nécessaire de lui pre.scrire une lentille correctrice à foyers progressifs.
La figure 5.29 illustre un exem ple pour la correction d e la presbytie. Pour ticnnct-
Ire à un œ il p resb y te d ’o b se rv e r d es o b je ts rap p ro ch é s, on lui pre.scnt un e len tille
c o rrec trice L, qui à p artir d ’un o b je t O, situ é à 2 5 ,0 cm d e l ’œ il (le PP m oyen d ’un
œ il n o rm al) p ro d u ira une im age à .son P P q u 'il p o u rra d o n c n ettem en t p erc ev o ir en
accom m odant au m axim um . A vec la lentille correctrice, la distance m inim ale d e vision
d istin cte sera d o n c d e 2 5 .0 c m ; la pcr.sonnc p resb y te p o u rra alo rs relâ ch e r q u elq u e
peu son accom m odation, ce qui lui perm ettra de lire con to rtab lem eiu un texte place à
environ 40,0 cm devant elle.

Rgijfe5.29 Un exemple
de la correction de la presbytie.
La lentille correctrice L, produit,
à partir d’un objet situé
à 25,0 cm, une image au PP de l’œil.
Cette image devient un objet ¡iK (\Ii
pour l’œil qui peut être nettement
distingué lorsque celui-ci
accomiiKxlc au ma,\imum.

Exemple 5.7
Soit un œil emmétrope ayant un PP situé à 40.0 cm de celui-ci. 2. On doit lui prescrire une lentille correctrice (voir la figure 5.29)
qui, â partir d’un objet 0 | situé il 2.5,0 cm de rœ il. jiroduira
a) Quelle est la puissance d’accommodation de l’œil ’’
une image à son PP qu'il pourra donc nettement distinguer
SOLUTION: Le concept dé e.sl le suivant: on utilise l’équation 5.16 en accommixlant au ma.xinnim.

^ avec dpp = 0,400 ni et, sachnni que l’œil est 3. On applique l'équation des lentilles minces = — -i- —
dpt> C/HR
emmétrope, dpJ^ = -xj, donc : (équation 5.13) à la lentille correctrice afin de déterminer va
puissance .
I L'objet O, de la Icnlillc correctrice Lj étant situé à 25,0 cm. on >
R... - = + 2.50D . (réponse)
0.400 m 00 P = 0,250 m. Puisque l'image /. V’de L, est virtuelle ( t j < 01 ci siruée
an PP. alors q = -dpj, = -0 .4 0 0 m. F.n insérant ces valeurs de J
b) Déterminez, sa prescription et de <7 dans l'équaiion (tes lentilles mina-.s. on détermine la vdlctn
de P„ :
mUTION : Les conteoh clés sont les suivants.
I. L'œil souffre de presbytie puisque sa puissance d’acc(Hnmod.ition 1 I
Pp = = -r^L.HOD (I
est inférieure à 4.00 D 0.2.50 m -0.4(H) m
130 Chapitre 5 Les miroirs et les lentilles

l^t puissance de la IciUille correctrice étant positive, ccllc-ei est est située au PR de l’œil qui est à l'intini, on a ç = oc et l’équation
convei'grntc. des lentilles devient :
c) Quel est son domaine de vision distincte avec la lentille correctrice ?
SOLUTION, Sans son verre eorrceicur, cet œil possède un domaine + I..50D = i + i-.
P 00
de vision dislinelc de 40,0 u n jusqu'à I'infini. L’œil perçoit non pas
les objets placés dewnt la kniillc correctrice, mais les images prixliiites L’objet O, est donc à une disiaiKe
par ccllc-ci. qui deviennent des objets pour lui. Selon la question a),
la présence du verre correcteur rapproche la distance m in im a le de P = M\7 cm.
vision disiincic à 25.0 cm de l'ieil. En effet, lorsque t’ohjci est à celle ce qui correspond à la distance maximale de vision distincte de l’œil
distance, l'inringe est au PP. avec son verre correcteur. Le domaine de vision distincte avec la
L’image la plus éloignée de l’ocil que la leiuiile peut produire lentille correctrice est donc
afin qu’elle soit ncticmeiit perçue par l’œil doit être située à son PR.
I e nntept (it utilisé ici pour déterminer la distance nuiximale de vision de 25,0 cm à 66,7 cm de l’œil. (réponse)
distincte est donc le suivani. on doit determiner où sc situe l’objet Oi
de lit Itiuillc correctrice qui produii une image /, an PK de l’œil. Lorsque la lentille correctriec est devant l’œil presbyte, tous les
I a tiguie 5.1(1 illustre la situation objets à plus de 66,7 cm de celui -ci produisent une vision floue.
On applique de nouve.'iu l’équation des lentilles minces Mais si on enlève le verre correcteur, la v ision devient iicUc à partir
de 40,0 cm jusqu’à l’infini. Cela explique pourquoi une personne
f,. = — + - à la lentille correctrice de +l,.5() l). Puisque l'image I, presbyte retire scs verres eorrccieurs pour regarder au loin.
P U

Rguie 5 30 txem ple 5.7


l u disinnee maximale ilc vi.siim io/î
distincte avec une Icnliilc
cnrreetnee est déterminée
cil calculant la distaïu c nbjci p
de la lentille correctrice lorsque
son image est au PR de l'œil.

5.8 Les Instruments d ’optique


L ’œ il hum ain est im organe rem arquablem ent efficace, mais sa portée peut être accrue
de plusieurs façons à l ’aide d ’instrum ents d 'o p tiq u e tels que les loupes, les projecteurs
de cinéma, les caméras (incluant les caméras de télévision), les microscoiTcs et les télescojies.
De nom breux dispo.sitifs sem b lab les éten d en t la p o rtée de la vision h um aine au-delà
du spectre visible ; les ca m éras à in fraro u g e d an s les sa te llite s et les m ic ro sco p e s à
rayons X en constituent deux exem ples.
L es éq u a tio n s rela tiv e s aux m iro irs el aux le n tilles m in c es ne s'a p p liq u e n t
qu'approxim ativem en t à la plupart des instrum ents d ’optique sophistiques. L es lentilles
des m icroscopes de la b o rato ires n ’o nt rien de « m ince >>. L a p lu p art des in slru in cn ts
d ’optique sont constitués de lentilles compo.sécs ; ccllcs-d com prennent plusiciir.s com po­
santes, et il esl rare q u e leurs interfaces soient parfaitenx;nt sphériques. O n analysera
ici trois instrum ents d 'o p tiq u e , en supposant, p o u r sim plifier les choses, que l’cquation
des lentilles m inces s ’applique.

La loupe
L 'teil hum ain n o r n u il iveul observ er un objet ci en rendre une im age nette sut la rétine
(à rin iériciii de l'œ il) si l ’objet est situé entre l’infini et un certain point qii’on nom m e
le p u n c T u m p r o x t m u m (o u point p ro ch e) PP. Si vous placez l’o b jet plus près d e l'ie il
que ce point, l’im age p n id u ile su r la rétine d evient floue et la vision n ’esi plus nelle.
L a position du P P varie n o rm alem en t av ec l'â g e . O n a tous connu de.s |scrsonncs qui
prétendaient ne pa.s avoir besoin de lunettes m ais qui lisaient leur jo u rn al en le tenant
à bout de bras ; la position de leur PP s ’esi éloignée de leurs yeux à cause du processus
du v ie illissem en t P o u r tro u v er la p o sitio n d e v o tre p ro p re PP, e n lev e z vos lu n e tte s
5 8 Les instruments d opttque 131

PI*

h)
a)
figure 5.31 a) liii objci i) de hauteur h, >iuié au PP
(jnmcuun /miximum) moyen d'un œil humain à 25 cm de I’leil.
sous-tend un angle ÿ dans le champ de vision de l'œil, b) L’objet
est rapproché davantage afin d'augmenter l'angle que sous-tend
cet objet par rappon à l'œil, mais l'observateur ne peut plus
neticmcnl percevoir l’objet: sa vision à cette distance est floue,
c) Une lentille convergente, qui sert de loupe, est placée entre
l'objet et l'œil, l'objel étant .situé entre le foyer objet F et la lentille.
L'image virtuelle produite par la lentille, qui devient l’objet de l’œil,
est alors suffisamment cloigttée pour que l'œil puisse la distinguer
nettement ; cette image sous-tend un angle W* plus grand que l'angle 6
de l’objet O placé à 25 cm en a). r)

ou vos v erres d e co n tac t si vous en |>t)rlez. fe rm e / un œ il, et rap p ro ch e z ce tte page


tic votre œ il jusqu’à ce q u 'e lle devienne floue. D ans celle section, on x itw la position
du PP moyen à 25 cm de l'frU, .soit un peu plus que la valeur norm ale pont une ¡lerMitme
de 20 ans.
L a fig u re 5..31 a) p rése n te un o b je t C) situ e au P P d e l'œ il. L a ta ille de l'im a g e
de l'o b je t produite sur la rétine dépend de l'an g le ^ q u e l’objet .sous-tend dans le cham p
de vision de l’œ il. E n rapproehatii l ’objet de l’ieil, com m e l’illustre la figure .5.31 b),
VOU.S pouvez augnKïnicr l'an g le et. par le fait m êm e, la capacité d e leconnaîire les détails
de l ’o bjet. T outefois, p u isq u e l'o b je t est alors p lu s près de l ’œ il que son PP. l ’im age
n ’est plus nette et la vision devient Houe.
Vous pouvez obtenir une vision nette en regardant O à travers une lentille conver­
gente. qui .sert de loupe, placée de m anière qu e O soit entre le foyer objet F et la lentille
(figure 5.16 c). C e que vous voyez est l'im a g e virtuelle de O produite par la lentille.
C ette im age, qui d ev ien t l’o b je t de l'cc il, p eu t être n ettem en t {>erçue si elle est plus
éloignée de l’œ il que son PP.
P ar ailleurs, l’angle d 'stm s-tc n d u p a r l'im a g e virtuelle est plus g rand que le plus
grand angle d que l'o b je t sans loupe, ou sans autres instrum ents d 'o p tiq u e , jviif snus-
tendre tout en étant nettem ent perçu. 1.c gro ssis.srm en t a n g u la ire <7 (à ne pas confondre
avec le grandissem ent transversal m) d e ce qui est perçu est défini par

ft’
G =— (le grossissement angulaire) (5 17)

Littéralem ent, le grossissem ent angulaire d 'u n instrum ent d 'o p tiq u e , com m e la loupe,
e s t une co m p araiso n en tre l’an g le so u s-iem lu p ar l ’im age finale qu e l’in stru m e n t
d ’optique produit et l’angle sous-icndu par l’objet p la ré au PP de l'œ il
D ’ap rè s la fig u re 5.31 et en se fo n d an t su r l ’ap p ro x im atio n q u e lén ft ^ 0 et
tan (F = ft' dans le cas de petits angles, on a
h h
=— et if = .
25 cm P
O n trouve alors que

2 5 cm
(7 = (le gTossissenKail de la l(Hipc). (5 .1 8 )
132 Chapitre 5 Les miroirs et les lentilles

Le g ro ssisse m e n t c o m m e rc ia l G,.,^ est le grossissem ent angulaire obtenu p ar un


instru m en t d ’optique lorsque l'im age finale q u ’il p ro d u it est à l'infini. P our la loupe,
l’image produite par celle-ci est à l ’infini si l’objet est situé au foyer objet de la loupe,
c ’e s t-à -d ire si p = / . Le g ro ssisse m e n t c o m m ercial de la lo u p e est d o n c d o n n é par
l’équation 5.18, en rem p laçan t p p a r/ ;

25 cm
(le gros.sisscmcni commercial de la loupe). (5.19)
/

Le microscope composé
La fig u re 5.32 rep rése n te un m ic ro sco p e co m p o sé c o n stitu é de lentille.s m inces.
L 'in stru m e n t se co m p o se d ’un o b je c tif (la len tille situ ée p rès de l’o b jet à o b se rv e r)
de distance f o c a l e e t d ’un o c u la ire (la lentille près de l’œ il) de d istance lo c a le /^ .
O n l ’utilise pour v o ir de petits objets placés très près de l’objectif.
L ’o b jet réel 0„h à o b serv er, p a r ex em p le un insecte, est lég èrem en t p lu s élo ig n é
de l ’o b je ctif q u e son foyer objet La d istan ce en tre les lentilles est ensuite ajustée
de m anière que l'im ag e produite p ar l’objectif, et qui est réelle, agrandie et renversée,
soit légèrem ent plus rapprochée de l’oeulairc qu e son foyer objet l a distance entre
les point.s focaux et est ap p e lé e la longueur optique s (v o ir la fig u re 5.32)
E lle est g énéralem en t assez g ran d e p ar rapport z. f ^ et /o^. P u isq u e l'im a g e rée lle (ob
d ev ien t un objei réel p o u r l ’o cu laire e t que celu i-ci est p lu s rap p ro ch é d e l’tK u laire
que son foyer objet F ^ , l'o cu laire agit donc com m e une sim ple loupe, et l’observateur
voit une im age finale (virtuelle e t tenverséc) à travers lui.
Le grossissem ent de l’instrum ent e st donné par l’équation 5.17 (C = (i’ifi), où 0'
est l'an g le sous-tendu par l'im a g e finale du m icroscope, représenté à la figure 5.32,
et 0 est l ’angle que sous-tend l ’objet à 25 cm de l'œ il sans m icroscope (v o ir la figure
5 .3 1 a). À l'aid e d e la figure 5.32 et de l’approxim ation des petits angles, <mi déterm ine
q u e tí' = tan tí' = h'Ip^^. Si on rem p lace d an s l'é q u a tio n 5 .17 l ’a n g le tí' p ar h'/p^.
et l'a n g le tí par /t/25 cm (v o ir la section su r la loupe), le grossissem ent du m icroscope
peut donc s ’écrire sous la form e

Й' 25 cm
G= (5.20)
h Рос

Le p rem ier facteu r h'/h de ce tte éq u atio n est, selo n l ’éq u atio n 5.5, le g ran d issem en t
transversal mot, ro b jc c tif . alors q u e le second fac teu r 25 cm/p„,. rep résen te, .selon
l'équation 5.18, le gro.s.si.s.scmcni angulaire G„,. de l 'oculaire. Le grossissement angulaire G
du m icroscope est donc

C - (w ,*)(G „).

Rgure 5.32 Reprc.sentation d'un microscope composé constitué de lentilles minces


<l'illnsiraiion n'e>.'t pas à l’echelle). L'objcctil produit une iniage réelle de l'objet réel
cl cette image est IcgèrciiKiir plus гаррпчЬее de l’oeuiaire que son foyer objet f L'image U
agit alors comme un objet réel prtur l'oculairc. qui produit une image virtuelle finale celle
qui est perçue par l’observateur. L’objectif a une distance Г(ка1е/,*: l'oculaire a une di.staiKe
focale /^ ; la distance entre les points focaux F^^, et F^c longueur optique ,v.
5.8 Les instruments d’optique Ш

L’o bjectif d u m icroscope sert donc à produire, à partir de l’objet à observer, une image
réelle agrandie qui est, par la suite, observée par l’œ il au m oyen de l ’oculaire, qui agit
com m e une loupe. S elon l ’éq u atio n 5.6 , on p eu t é c rire le g ran d issem en t transversal
produit p ar l’o bjectif sous la form e

Çob
(.5.21 )
PiA> h'
où est la distance entre l’o b jectif et son objet et e.si la distance entre l ’objectif
et .son im age /(*. Si on utilise l ’équation 5.21 pour substituer h ’/h dans l ’équation 5.20,
l'expression du grossissem ent du m icroscope devient

^ 9 ob 25 cm
Cl — —--------- — (le gro.vsissement du microscopcj. (5.22)
A * Poe

Le signe n ég a tif indique qu e l'im a g e finale de I'lx u la ire est renversée par rapport
à l’objet O,*, de l’objectif. Le m icroscope produit donc des iim g e s renversées des objets
que l’on veut .scruter.
P(Hir obtenir le grossissem ent com m ercial du m icroscope, l’im age finale produite
par l’cKulaire doit être à l’infini. Far conséquent, l'objet réel de rtxiulaire, qui correspond
à l ’im ag e /f* de l ’o b ic c iif, d o it sc situ e r au fo y er de l ’o c u la ire , d onc .
P uisque la d istance im age q ^ de l'o b je c tif est alors donnée p ar + x (voir la figure
5..32), on sub stitu e cette v aleu r de q,^ d an s l ’éq u atio n des len tilles mincc.s appliquée
à l’o b je ctif, Уроь + l/(7ob = qtJ<^ 1 récrit d ’abord, en la m u ltip lian t par
so u s la fo rm e q j p ^ b + 1 = W / ô s P our o b te n ir q,Jpç^> = i U + 'V/ob “ > “ '//ob-
Le g ro ssisse m e n t co m m ercial est o b ten u en rem p laça n t d an s l ’éq u a tio n 5.22
4JPob par t//ob et Poe par/oc. ce qui donne

S 25 cm
(le gruKsissemeni ciimnicicial du microscope). (5 .2 3 1
/<* /u

O n rem arque qu e le grossissem ent angulaire d ’un m icroscope est d ’autant plus grand
que les distances focales sont pctiic.s (généralem ent entre quelques m illim etres et une
vingtaine de m illim ètres) et que la longueur optique s est graïule (environ 16 cm )

La lunette astronomique
II existe plusieurs types de lunettes astronomiquc.s Celui q u ’on décrit ici est le m odèle
le p lu s sim p le, com po.sé d ’un o b je c tif et d ’un o cu laire ; ces deux co m p o san tes .sont
représentées à la fig u re 5.33 avec d es len tilles m inces sim ples m êm e si. en p ratique,
la plupart des lunettes astronom iques com portent des lentilles dont char une est. en fait,
une lentille com posée.
L’arrangem ent des lentilles est sensiblement le meme dans le-s luneiics a.xtroïKaniqucs
et d ans les m icroscopes, m ais les lunettes astronom iques sont conçues pour o b serv er
de gros objets à de grandes distances, com m e d es galaxies, d es étoiles ou des planètes,
alo rs que les m icro sco p es so n t co n ç u s p o u r o b se rv e r de p etits o b je ts à de p etite s
distances. Cette différence fait en sorte que, dans la lunette iLsironomique de la figure 5.33.
le foyer im age de l’objectif csi très près du foyer objet de rrx:ulairc (à quelques
m illim ètres), alors que, dans le m icro sco p e de la fig u re 5.32, ces p o in ts .sont séparés
par la longueur optique s d ’environ 16 cm .
D ans la fig u re 5.33 a), de.s ra y o n s parallèle.s p ro v en a n t d 'u n o b je t très élo ig n é
(à l’infini) tom bent sur l’o b jectif en form ant un angle Havec l'a x e d e la lunette astrono­
m ique et form en t une im age réelle et ren v ersée au foyer im age /'„f, de l o b iectif.
D ans une lunette astronom ique, on ajuste la distance entre les lentilles afin que l ’image
/,* de l’o b jectif soit légèrem ent p lu s rapprochée de l'o cu laire qu e son foyer objet
P uisque l’im age réelle devient un objet réel p o u r l’oculaire et que cclui-ci c.st plus
rapproché de l’o cu laiic qu e son foyer objet / l’oculairc agit donc com m e une sim ple
loupe, et l’o b serv ateu r voit une im age finale (virtuelle et renversée) à travers lui.
134 Chapitres Les miroirs et les lentilles

Oculaire

b)

Figure 5.33 aj Repré.semulion Le grossi.s,sentent angulaire (7 de la lunette astronom ique est, .selon l’équation 5.17,
d ’une lunette astrononrique formée ff/H. L’angle O' reprdsente à la figure 5.33 a) est. com inc ptiui tous les autres instrum ents
de lentilles mincc.s. L'ohjcctif produit d ’optique, l’angle sous-tendu p ar l'im a g e finale de la lunette a.stronomiquc. On doit,
une image réelle /,*, provenant par contre, m odifier la définition de l ’angle 0 ( = 25 cm//?) ulili.sée p o u r la loupe et le
d'une souicc lumineuse tellement m icroscope, car on ne peut approcher un objet éloigne, com m e la Lune ou une planète,
éluigm'e (l'objet) que les rayons à 25 cm de son œil. L ’angle O sera donc l’angle que soas-tend l’objet à l'œ il nu lorsque
luiiiineuA incidents près de !■ub.jec^ll l’objet dem eure éloigné ; cet angle est egalem ent représente à la figure 5.33 a). D'aprè.s
snni approximativemenr parallèles la ligure 5 33 b), on peut écrire, en considérant que les rayons form ent de petits angles
(On suppose qu’une extrémité de l’objet avec l’axe optique, que 0 = tan 0 = et O' = tan 0' = h'/pg^. ce qui donne
cal iSitucc sut l'axe optique.) I/image
reelle /,^ se forme au loyer image ob
lie I'lib ja tif. l a distance entre G = - ( le grossissenieni de la lunette asirrmomique). (5.24)
P<x
les lentilles est clioisic afin
que l'image /,g,. qui agit comme où le signe « m oins » indique que l’im age finale de l'oculaire e.st renva-sée par rapport
nil objet réel (.viiir l’oculaire, soit plus it l'objet l'objectif.
lapprodiéc de l’oculaire que son foyer I,c gros-sissemcni com m ercial de la lunette astronom ique est obtenu lorsque l’image
ob|ct F,^. L'oculaire produit alors finale loc p ro d u ite p ar l ’o cu laire est à l'in fin i. D an s c e lte situ atio n , p u isq u e l’o b jet
une image finale virtiielle réel de l'oculaire, qui correspond à l'im a g e /<* de ro b je c iif, doit se situer au foyer objet
à une grande distance de l’observateur foc de l’o cu laire, les p o in ts fo ca u x et d o iv e n t co'm cidcr, donc
L’objectif a une distance focale Aj,. Par conséquent, le grossissem ent com m ercial G^.,^ est, selon l'éq u atio n 5.24, donné par
l’oculaire a une distance localeX«..
b) L’image /„(, a une liautcur h'
Gcoin = (le grossissement commercial delà lunette asurstomique). (5.25)
cl soiis-tend lin angle 6' mesuré
JVC
par rapport à l objectif, et un angle 0
mesuré par rapport à l’oculaire. Le grossissement angulaire d 'u n e luncile astronom ique est donc d'autant plus grand que/g,
est grand par rapport à /oc ; cela explique pourquoi une lunette astronom ique doit être
assez longue : elle doit être plus longue qu e /^,, qui peut, pour une bonne lunette, dépasser
le mètre.
Le grossissement n’est q u 'u n des facteurs importants dans la conception d ’une lunette
astronom ique, cl il est assez facile à obtenir. Un bonne lunette astronom ique doit pouvoir
« recu eillir» clficâcem cnt la lum ière, laquelle déterm ine la brillance de l’image. C’ela est
particulièrem ent im portant lorsqu’on veut ob serv er des objets pâles aussi éloignés que
des galaxies ; on obtient ce résultat en fabriquant un objectif ayant le plus grand diam ètre
possible. Une lunette astronom ique doit aussi avoir un bon pouvoir de résolution, qui c?it
la capacité de bien distinguer deux objets éloignés (deux étoiles, par exem ple) dont l’écart
angulaire est faible, l e champ de vision constitue égalem ent un param ètre im poilant dans
la conception. Une lunette astronom ique conçue pour observer les galaxies (qui w iiv ren t
un cham p de vision étroit) est très différente d 'u n e autre conçue pour ob serv er la laine
nu les com ètes qui passent près de la Terre (qui couvrent un cham p de vision plus lai-gc).
I,es co n cep teu rs d e lu n ettes astro n o m iq u es d o iv en t ég alem en t ten ir com pte des
différences im portâm es entre les lentilles réelles cl les lentilles m inces idéales dont on
a discuté ju s q u 'à m aintenant. U ne lentille réelle est dotée de su rfaces sphériques qui
ne prodiii.seni pas d 'im a g e s très nettes lorsque les rayons lum ineux ne form ent pas de
p etits a n g les avec son ax e o p tiq u e ; on n o m m e ce d éfau t aherration de sphéricité.
De plus, puisque la réfraction p ar les d eux su rfaces d ’une v raie lentille d épend de la
longueur d ’oiulc, q u ’tm a apptdée le phénom ène de dispersion, une lentille ne ( oneenlre
pas les rayons liiinincux de différentes longueurs d 'o n d e en un m êm e point ; on nom m e
cet autre défaut aberration chromatique.
C e lte brèv e in tro d u ctio n ne décrit pas de m an iéré ex h au stiv e les p ara m ètre s de
c o n c ep tio n des té le sc o p e s ou des lu n ettes astro n o m iq u es - il y a p lu sieu rs aiiiie.s
param ètres à considérer O n pourrait d 'ailleu rs en énum érer autant pour tCMit instrum ent
d'opiicjiie donnant des im ages d e grande qualité.
5.9 Trois démonstrations 135

5.9 Trois démonstrations


L'équation des miroirs sphériques (équation 5 4)
L a fig u re 5.34 p rése n te un p o in t o b jet ré e l O situ é su r T ax e o p tiq u e d 'u n m iro ir
sphérique concave. L ’objet est p lu s élo ig n é du m iro ir qu e son cen tre de co u rb u re C.
Un rayon lum ineux issu de O cl fo rm an t un an g le u avec l’ax e o p tiq u e cro ise l’axe
au point / après avoir été réfléchi par le m im ir au point a. U n rayon qui quitte O paral­
lèlem ent à l’axe optique est réfléchi au point S, revient sur son propre parcours et passe
égalem ent par /. D onc. I est l’im age de O : c 'e s t une im age rfclle p uisque les rayons
Figure 5.34 Un miroir spliérique lum ineux ém ergeant du m iroir après réflexion sont convergents.
cw cave forme un poim image réel I Le théorèm e suivant sera très utile ici : un angle extérieur d ’un triangle est égal à la
en rélléchissant les rayoïis issus som m e des deux angles internes opposés. Si on applique ce tliéorèm c aux triangles OaC
d’un point objet réel O. et O al dans la figure 5.34, on obtient :

P =a + a et 7 = O' + 2 d.

Si on élim ine d e n tre ces deux équations, on déterm ine que


a + y = 2p. (5.26)

O n peut écrire les angles a . /6 et y en radians, sous la form e

O? <ïÎ aS
a
Id ^ ~p' ^ ^ SC ^

aS
et (5.27)
li (i
Seule l’équation de p est exacte, puisque le centre de courbuie de l'a ic aS .sc trouve à L
Toutefois, les équations de a et de y sont approxim ativem ent correctes si ces angles sont
assez petits. D es rayons lum ineux form ant de petits angles avec l'ax e optique soin dits
paraxiaux. En insérant les équations 5.27 dans l’équation 5.26, on obtient

aS aS Уп.Ч
— -b
P <1 r

puis, en utilisant l’équation 5.3 pour rem placer r par 2 / cl en s im p lifia n t ДО, on a rrive
exactem ent à l’équation 5.4. Mp -b l/</ = Mf, qui est la relation que Гоп voulait dcmontrci.

L'équation des dioptres sphériques (équation 5.8)


D ans la figure 5.35, le rayon incident issu du |x)int objet réel ü qui «Utcint le p o in t a
d 'u n e su rfa c e de ré fra c tio n sp h é riq u e y est réfra cté suivant la loi de la réfractio n
(équation 4.41 ).

«I sin d| — «2 i’in d . .

Si a est |x:til, d, et (6 le .seront aussi, et selon ra|ip ro x im aiio n îles |tetits angles on peut
rem placer les sinus de cc.s angle.s par les angles eux-m êm es. D onc, ré q u a tio n t i-dcssu.s
devient

II) di = «2 (5.28)
On utilise encore le fait q u 'u n angle cxiérieui d ’un triangle est égal à la som m e des deux
angles intérieurs opposés. .Si l’on tient com pte de ce fait dans le cas de.s li iaiiglcs COa
et Cia. on obtient

d, = c r f r i et )4 = d2 + y . (5.2У)
.Si on utilise les équations 5.29 p o u r élim iner d, et d-, dans ré q u a tio n 5.28, ou déterm ine
que
P — — <1- — - ■
«(O' t i i 2y = ( n j n ,)/t. (5.30)
figure 5.35 Le point image réel I
d’un point objet réel O est formé M esurés en radians, les angles a , P et y valent
par la réfraction des rayons lumineux
par une surface sphérique convexe aS „ aS r i?
a ^ — : P — — : y% — . (.5.31)
séparant deux milieux. P r q
136 Chapitre 5 Les miroirs et tes lentlttes

S eu le la deuxièm e de ces éq u atio n s e st exacte. L es d eux aiio e s so n t ap p ro x im ativ es


puisque / c l O ne sont pas le centre du cercle d o n t üS fa it p artie. T outefois, si aS est
assez petit (dans le cas des rayons paraxiaux), les équations 5.31 sont approxim ativem ent
c o rre c te s. En in sé ra n t les é q u a tio n s 5.31 d an s l'é q u a tio n 5 .3 0 e t en sim p lifian t,
on arrive directem ent à l’équation 5.8, « ,/p + n J ç - («2 — «i)/»; dont on voulait faine
la dém onstration

L’équation des lentilles minces (équations 5.9 et 5.10)


L a technique con.sistcia ici à considérer chacune des surfaces d ’une lentille com m e une
surface de réfraction individuelle, et d ’utiliser l’im age produite par la prem ière surface
u>mme l'objet de la seconde surface.
O n co m m en ce avec la « le n tille » do v erre é p a isse illu strée à la fig u re 5 .36 a),
d ’épaisseur L et dont les surfaces de réfraction gauche et droite ont des rayons de courbure
r' et r", respectivem ent. Un objet réel ponctuel ( / est placé près de la surface de gauche,
com m e on le voit dans l'illustration. Un rayon partant de O ’ parallèlem ent à l’axe optique
n’est pas dévié en entrant dans la lentille, ni en en so rta n t
Un deuxièm e rayon partant de ü ' en foim ant un angle a avec l'a x e optique atteint
la su rfa c e de gau ch e au p o in t a', est réfra cté , e t cro ise e n su ite la .seconde su rfa ce
(ce lle de d ro ite) au p o in t a". L e ray o n est de nt>uveau ré fra c té e t c ro ise l ’axe à /",
qui est le point d'intersection des deux rayons issus de O ' et qui constitue l’im age finale
du point O', form ée après la réfraction aux deux surfaces.
I.a fig u re 5.36 b) m o n tre que la su rfa ce de g au ch e fo rm e ég a le m en t une im ag e
virtuelle de O', à / '. Pour localiser f', on utilise l'éq u atio n 5.8 :

^ ^ _ »2 - rii
P q / ■
En posant n, = 1 pour l’air et /I2 = n pour le verre de la lentille, on obtient
1 - ” ~ ^
(5.32)
q' r'
L a fig u re 5.36 c) rep résen te d e n o u v eau la d eu x ièm e su rface. À m oins q u 'u n
observateur se trouvant au point o ”connais.se l’existence de la prem ière surface, il pourrait
croire que la lum ière qui arrive à ce point est issue du point / ' de la figure 5.36 b), et que
la rég io n situ é e à la g au ch e d e la su rfa c e est en v erre, co m m e c ’e st illu stré. D onc,
l ’im age virtuelle / ' form ée par la prcm ièie surface sert d ’objet réel ( T p o u r la deuxièm e
surface. L a distance entre cet objet et la deuxièm e surface est

f = L - q. (5.33)

«2 = 1.0

Figure 5.36 a) lieux rayons lumineux, issus du point


ohjei léel O', sont rétractés en traveri.'im chacune
flHv (Wii» siirfîici«: sphenques d une < ■lentille >'
prniliilxani lin point iniapp léH /" Par stnici
de vlurtc, on a fortement exagéré les angles par rapport'
Й l’axe optique qu’etfecnic le rayon lumineux pass.snl
par les points о ' et я" ; il faut supposer que ce rayon est
paraxial. La representation de la surface de réfraction
b> gauche et c) dr<'ite de la « lentille » illustrée en a).
Révision et résumé 137

D ans ce tte é q u a tio n , on a in sé ré un signe « m o in s » d ev a n t q' car, l’im ag e / ' étan t


virtuelle, q ' est une v aleu r négative. En effet, on s ’in téresse ici à la di.stancc positive
en tre O" et la d eu x ièm e su rfa c e de réfra ctio n pui.sque l'o b je t O" est réel (/;" > 0).
P our ap p liq u er l’éq u atio n 5.8 à la d eu x iem e su rface, on doit po.scr n, — « et /h = 1,
puisque l’objet est m aintenant effectivem ent im briqué dans le verre. Si on substitue ces
valeurs et q u 'o n rem place p" par L — q' selon l’équation 5.33, l’équation 5.8 devient ak»rs
I 1 -
(5..34)
L-q'
O n suppose m aintenant que l'épaisseur L de la « lentille » de la figure 5.3b a) est si petite
q u ’on peut la négliger, par rappon aux autres quantités linéaires (com m e p ', q'. p", q",
r' et r"). D ans to u t ce qui su it, on u tilise ra cette approximation des lentilles m inces
En insérant L = Q dans l’équation 5.34 et en réarrangc;uit le côté droit d e cette équation,
on a
n I H — 1
------ -f- — = --------------. (5.35)
q' q" r"
En additionnant les é(|uations 5..32 et 5..35, on obtient

• ' . ^ *A

Enfin, en désignant sim plem ent p;ir p la distance objet initiale et p ar q la distance image
finale, on a

-p + -q = ( « - D I \ r-' - -r " } . (5 3 6 )

ce qui, avec un petit changem ent dans la notation, correspond aux équations 5.У et 5.10.
soit les relations qu e l’on voulait dém ontrer.

REVISION E T R É S U M t

Les objets réels et virtuels Un objet est le point de rencontre réelles SC forment du côté du miroir où les rayons sont réfléchis,
des rayons lumineux incidents. Lorsque les rayons incidents sur un et des images x'irtuelles se forment du côté opposé
système optique sont divergents, il s'agit d'un objet réel -, si les
2. Diopire sphérique :
rayons incidents sont convergents, il s'agit d’un objet virtuel.
Les iMaffes réelles et virtuelles Une ima/te est le point de «1 ^ П2 П2 — U|
(5.S)
rencontre des rayons lumineux émergeant d'un système optique.
I ne imufte réelle est formée lorsque les rayons qui émergent du
où n, est l’indiec de réfraction de la .substance dans laquelle
sysièinc optique convergent. Si les rayons émergents divergent,
on retrouve les rayons incidents, ii, est l'Indice tie léfrai lion de hi
il s'agit d ’une inutfje virtuelle. Il faut alors prolonger les rayons
substance d'où les rayon.s émergent et r e s t le rayon de courbure
émergents vers l’arrière pour déterminer la position de l'image.
de la surface. Quand le centre de courbure de la surlaec es( du coté
L a fo rm a tio n d ’im ages I^cs miroirs sphériques, les dioptres où les rayon.s émergent (cette surface est convexe par rapport au côté
sphériques et les lentilles minces peuvent former des images provenant où les rayons incidents se propagent), le rayem rc.st p<isitif. Quand
d’une source lumineuse - l’objet —en réorientant les rayons qui le centre de courbure de la surface est du côté des ntyons incidents
émergent de la source. L’image se forme lorsque les rayons réorientés, (celte surface est concave par rapport au côté contenant les rayons
provenant d'un point tibjet, possctient un point de rencontre commun. incidents), r e.si négatif. Des images réelles sc forment du côté d'où
S i les rayons torment de petits angles avec l’ove optique (rayons émergent les rayons, et des images virtuelles se forment du côté
paraxiaux) du miroir spliériquc. du dioptre spiterique ou de la lemille d’où piovieniKiit les rayons incident.s.
Itnince. nn a les relations suivantes qui unissent la distance objet p
3. Lentille m ince:
(qui est positive pour un objet réel cl négative p<Hir un objet virtuel)
« la distance imape q (qui est positive dans le cas d'images réelles,
I I I / I I \
et négBtive dans le cas d’images \ irtuelles) : = _ = ( , , - ! ) -------- . (.<5.9.5.10)
P q f \ '■i n )
I. M iroir sphérique :
o ù /e s t la distance fiK ale de la lentille (positive pour une lentille
1 1 1 - - “
(5.4, .5.3) convergente et négative ptair une lentille divergeniei, n est I indice
p q ~ f ~ r'
de réfraction du matériau composant la lentille, et r, et sont les
OÙ/'est la distance focale du miroir et r est son rayon de courbure. rayons de eotii biire ties deux faces de la lentille. t|ui sont tlc,s siirlace.s
Ces deux distances .sont positives pour un miroir concave et elles sphériques. L n rayon de courbure est positif lorsque le centre de
ion négatives pour un miroir convexe. Un miroir plan constitue courbure est du côté d’où les rayons émergent ; il est négatif lorsque
n cas particulier, où r —> de sorte que q = - p . Des images le centre de ctnirburc de la .surface est du côté d'où proviennent
138 Chapitre 5 Les miroirs et les lentilles

les rayons incidenn.. Des images léelles .se forment lorsque les rayons OÙ 0' est l’angle sous-iendu par l'im age finale que l’instrument
qui émergeni de la lentille convergcm. de sorte que l’image formée d’optique produit et 6 esl l’angle que sous-lcnd l’objet à l’œil nu.
sera du côté des rayons émergents. Des images virtuelles se forment Le grossissement commercial est le grossissement angulaire
lorsque les rayons cmergenLs divergent, et l’image formée se retrouve obtain pai un instrument d ’optique loi"sque l'image finale qu’il produit
du côte des rayons incidents. esl à l’infini.
Le grandissem ent transversal Le grandissement transversal Voici trois instruments d’optique qui permettent d'augmenter
m pnjdiiif jiar un miroir sphenque ou une lentille mince est la portée de la vision humaine.

m = -i. (.'5.6) 1. La lou¡>e, ou lentille simple, produit un grossis-sement angulaire G


t> donné par
La giundeur de m est donnée par
25 cm
(5.5) G = (5 18)
h P
où h et II' sont rcs|)cciiveinent la hauteur (perpendiculairement OÙ P est la disianee entre la loupe et l’objet.
ù Taxe opiiqiie) de l'objet et celle de l'image. Le grossis-seroent commercial de la loupe esl obtenu par
IJeeil Le processus à'accanmu>datioH de l'rcil permet d’obtenir 25 cm
une vision iicuc d’objet» situé» à différentes distances de l'œil G... (5.10)
TT~’
l e puncnmi remotum (PR) est la position de l'objet ilont l’œil peut
produire une image nette sur la rétine .sans qu'il accommode. o ù / est la distance focale de la loupe.
Le piinrium proximiim (PP) est la position de l’objet dont on obtient
une image nette sui la réiiiie lorsque l'acrominodation de l'œil 2. Le microscope composé produit un grossissement G exprimé par
est maximale. La région de l’espace située entre le PP e( le PR où ,, * * 2-5 cm
la vision est nette est appelée le domaine de vision distincte à l’œil nu. (.522)
Pilis Pne
La puissance d'accommodation d un œil est donnée par
1 où wip(, est le grandissement transversal produit par l ’objectif
P = (5.16)
rfpi’ 'fpR est le grossissement angulaire produit par l’oculaire,
OÙ i/pp Cl i/pp .sont les distances respectives entre l’œil et son PP, distance entre l ’objectif et .son image, />„(, est la distance entre
cl entre l'œil et son PR l’objectif et son objet et p ^ est la distance entre l'oculaire et l'image
l'n œil normal possède un PP à moins de 25 cm de l’œil et, de l’objectif.
de plu», il est ernméiroite. c’c.st-à- dire que -son PR est situé à l’infini Le grossis-sement commercial G„,„ du microscope est obtenu à
{df^ — » ) . L’œil peut souffrir d ’un ou de plusieurs défauts qui l’aille de
peuvent être corrigés par le port de verres eorrectciirs.
l'n œil est e.onsitléré comme myope si 0 < d¡.n < v . Pour 23 cm
frenm f f (5.23)
permettre au myope d'ob.scrver des objets lointains .sans acconmioder, ./ob Jeu
on lui prescrit uik: lentille eorrcciiiee divergente qui d’un objet situé
où /,t, ti f^ sont respectivement la distance focale de l'ob jectif et celle
à r infini produit une image virtuelle à son PR.
de l’oculaire, et .s est la longueur optique.
Un œil est hypermétrope si son PR est un objet virtuel situé
derrière l’œil, î/,,,, < ü. Pour que l'œil hypermétrope puisse observer 3. La lunette astronomiepte produit un gros.sis.semenl angulaire G
des objets lointains sans accommoder, on lui prescrit une lentille donné par
correctrice convergente qui d ’un objet situé à l’infini produit .une
image réelle à son PR. G= - (5.24)
I .a preshxtie est une consétjuence du vieillissement qui entraîne foc
une diminuí ion progressive de la puissance d ’accommixlation de l’œil. où /„s est la distante focale de l'objectif et est la distance entre
On considère qu’un œil souffre de presbytie lorsque sa puissance l'oculaire et l'im age de l'o b jectif Le signe «m oins» indique que
d'aeenminodation est inférieure à 4,00 D Pour |iermettrp à un œil l'im age finale de finstrum ent est renversée par rapport à l’objet
presbyte d'observer des objets rappnK’hés, on lui prescrit une lentille observé.
correctrice qui, à partir d’un objet situé à 25,0 cm de l’œil, produit Le grossissement commercial lunette astronomique
une image à son PP qu'il peut alors Deilemcnl percevoir en accom- est calculé à l’aide de
niodani an maximum.
./ob
I.f.s instrum ents d'optique I .e gwssissement angulaire G d'un G,. (5.25)
instrument d ’optique est défini par
Joc
où sont respectivement la distance fcx'alc de l’objeelif et t elle
G =Î. (5.17,
de l’oculaire.
Questions 139

Q U E S TIO N S

1. Les nuinsires lacustres, les tntonycl les sirènes ont souvent été
« apervus » par des observateurs qui étaient situés sur un nvage ou
sur le pont intérieur d'un bateau. D’une telle perspective, où le point
de vue est peu élevé, un observateur peut intercepter des rayons
lumineux qui proviennent d’un objet flottant (un marsouin ou un
tttorceau de bois, par exemple) et qui sont légèrement déviés vers le
bas. en direction de l'observateur (un rayon réfracte .semblable est
représenté avec exagération dans la figure 5 37 a). L'observateur
pcryciit alors l’objet comme s’il s’étendait vers le haut à partir de la
surface de l’eau (et il le verrait probablement osciller, à cause de la
turbulence de l’aù ), dans un mirage qui pourrait ressembler i) l’une de
CCS créatures mythiques. Dans la figure 5.37 b), on indique plusieurs
courbes de la luuiteur. par rapport à la surface de l’eau, en foiKtion de
la température de l’air. Laquelle de ces courbes illustre le mieux les
conditions de temperature de l'air qui favorisent 1« déviation des
rayons lumineux et la production de ce mirage ?

a) I»
Figure 5.37 Ouestion 1

Z La ligure 5.38 nrontre un poisson et un pécheur dans l’eau,


a) Le pécheur voit-il le poisson d*ns hi région du ptnnt a ou dans
celle du point h? b) Le poisson voit-il les yeux du pêcheur dans la
légion du point c ou dans celle du point d ?

a)

Figure 5.38 QiK-stion 2

3. Dans le labvTintlie de miroirs de la figure 5.30 a), de nombreux


«couloirs virtuels» semblent sc prolonger de votre point de vue
parce que vous voyez plusieurs réflexions des miroirs qui couvrent
les murs du labyrinthe. Ces miroirs sont placés sur certaines faces
Figure 5.39 Q ia s iitm 3
des triangles équilatéraux el adjacents eotivrant le planciier. Le plan
du plancher d’un labyrinthe similaire, quoique différent, est illustré
d a n s la figure 5.30 b) ; dans ce labyrinthe, chaque section de mur
est couverte d'un miroir. Si vous vous tenez à l’entrée .t, a) lequel 4, fin pingouin se dandine parallèlement à l'axe optique d'un miroir
des monstres ti. b el c cachés dans le labyrinthe pouvez vous voir concave, à partir du foyer en allant vers l’infini, a) Comment son
dans les couloirs virtuels qui sc prolongent à partir de re n tré e ? image .sc déplace t-cllc ’ b) La hauteur tie son image angmeme-i-ellc
b) Combien de fois chaque monstre visible apparaît-il dans un continiiellciTiem. diminiie-t-cllc continuellement ou changc-t-ellc
couloir .’ c) Que voit-on à l’extréniité d’un couloir? {ImJire: Les deux de fayon plus complexe
rayons illustrés ftroviennem de couloirs virtuels; suivcv.-les dans le 5. laiisqii'uu tyrannosaure poursuit la jeep dans le film La - p u re
labyrinthe en appliquant la loi de la réflexion à chaque miroir situé ju ra s s iq u e , on voit une image réfléchie du monstre dans un miroir
sur le parcours de thacun des rayons. Passent-ils par un triangle latéral sur lequel est inscrite la mention suivante iqui relève de
où il y a un m onstre? Si oui. combien de fois? Pour approfondir rhumour noir dans le présent cas); «Aucniion. les objets sont plus
cette analyse, voir J. Walker, «The Amateur Scientist». Scientific près qu’ils apparaissent tians le miroir. » S'agit-il d'un miroir plan,
Amerktm, vol. 254, juin 1986, p. 120-126.) convexe ou concave'’
140 Chapitre 5 Les miroirs et (es lentilles

6. La fitrure 5.40 pn!,<;ente quatre ieniillcs niinces. fanes du mcnie ou virtuelle, et si elle a la même orientation que l’objet ou si elle est
matériau, dont les faecs sont planes ini ont un rayi>n de courbure de renversée.
10 cm. .Sans faire de calculs explicites, classe/, les lentilles en ordre 1/•nulle I/'utille 2
décroissant selon la grandeur de leur distance frx:alc.

'’1 J i

Figure 5.41 Quesiion 9

Situaliw Lentille 1 Lentille 2

1 Convergente Convergente Pi < / t


7 Soit une lentille mince convergente dont les grandeurs des deux T C.'onvcigcme Convergente P i> /i
rayons de courbure sont identiques. On suppose que l’objet de la
3 Divergente Convergente P\ < f\
lentille est vitliicl. ü i distance image augmente-t-elle, diminue-t-elle
ou rcstc-t-ellc la même si on augmente a) l’indice de rétraction n 4 Divergente Convergente Pi > f\
de la lentille, b) les rayons de courbure des deux faces de la lentille 5 Divergente Divergente Pi < fi
et c) l’indice de rétraction »i„„| du milieu environnant, en gardant 6 Divergente Divergente Pi > / i
la valeur de n^, inferieure à celle de ri’’
8. Un miroir concave ci une lentille convergente (verre: « = 1.5) 10. C'est à la cornée que se produit la plus forte déviation des rayons
ont tous les deux une distance focale de 3 cm dans l’air. Si on les met lumineux qui rend la vision humaine possible (à l’interface entre l’air
dans l'omi (où « ■= I 33). leur distaiKe focale est-elle plus grande, et la nwmbrane transparente externe de l'ocill. La comcc a un indice
plus petite ou égale à 3 cm ? de refraction légèrement plus élevé que celui de l’eau, a) Quand votre
9. Le tableau suivant indique six situations |x»,ssibles dans la disposition leil est immergé dans une piscine, la déviation des rayons lumineux
des detix lentilles minces presentees à la figure 5.41. (Les points /^i à la cornée est-elle plus grande, plus petite ou la même que dans
et f ^ «om les foyers des lentilles I et 2.) Un objet reel est à une l’air? b) Un poisson d ’,\mérique centrale, ÏAïuihleps anablcps. peut
distam e /i, !t la gauche de la lentille I, comme c'est illustré dans voir sous l'eau et au-dessus de l'eau simultanément parce qu'il nage
la figure 5.20. a) Quelles situations vous permettent de dire, sans en gardant une partie de scs yeux au-dessus de la surface de l'eau.
calculs, .si l’image Finnic (prtKiuite par la lentille 2) est réelle Pour assurer une visitm nette dans les deux tniLeux. le rayon de cour
ou virtuelle, cl SI clic a la même orientation que l’objet? b) Pour bure de la panic submergée de la cornée doit-il être plus grand, plus
chacune ilc ces situations possibles, dites si l’image finale est réelle petit ou le même que celui de la panic qui e.si dans l’air’’

EHSBBSSBS
de O verrez-vous? b) Indiquez, dans chaque ca-s, la localisation
wivw La solution sc trouve sur le site Web. à radre,s.sc ci-dessous ;
et l’orientation de l'iiTiage (comme dans la figuie 5.42 b)
\vxv-w.dlcmcgrawhiil.ca/physique
4P. Reprenez l'exercice 3 en fonction d ’un angle t) valant a) 45®,
b) 60 ' et c) 120’’. d) E.xpliquez pourquoi il y a plusieurs réponses
StniO N 5.2 Les miroirs pions possibles cn C).
Il Un papillon au niveau de vos yeux est à lü cm d'un miroir plan :
vous êtes dcrricrc le papillon, à .30 cm du miroir, (^lelle est la distance
entre vos yeux et la irosiiion apparente de l’image du papillon d;ms le
niimii ? vrww
2E. Vous regarde/., à travers l’objectif d'iinc caméra, l'im.'ige d'un
colibri dans un miroir plan. La caméra est à 4..30 m du miroir et lui
fait lace L'oiseau est à la hauteur de la carnera. à 5,ÜÜ ni à votre
droite et à 3.30 m du miroir. Quelle est la distance entre la caméra
cl la posilioii apiiarenie de l'image de l'oiseau dans le miroir?
3f l a figure 5.42 a) montre une vue en plongée de deux miroirs a)
plans vcnicmix et d ’un objet O placé entre eux. Si vous regardez dans
les miroirs, vou.s vove/ des im.iges multiples de O. Vou.s pouvez figure 5.42 Exercice 3 et problème 4
les locali.scr en traçant les dificrcnics réflexions pniduiics par
les miroirs, comme on l'a fail avec la première réflexion sur le miroir 5P. Prouvez que. si on tourne un miroir plan d ’un angle a , le rayon
de gauche a la figure 5.42 b). Vous trace/ ensuite la réflexion réfléchi subit une rotation d'un angle 2o? Pémontrez que ce rcsulUil
(le la reflexion. Voue continuez ainsi tant avec le miroir de gauche est vérifié dans le cas où » = 45’.
qii'iivci le miroir de droiie. jiiMjii'a ce que les rétle.vions se chevau­ 6P La figure 5.43 montre une vue en plongée d'un corridor à
chent ou serotrouvent à r arrière des deux miroirs. Vous pouvez ensuite rexirém ilé duquel est placé un miroir M. Une cumbriolcu.se C
compter le noiiibrc d’images de O. a) Si d ~ VO ". combien d’images SC faufile dans le corridor, directement vers le centre du miroir.
Exercices et problèmes 141

Si <7 = 3.0 m. à quelle distance du m iroir la cambrioleuse C sc TABLEAU S.5 Problème 10 : Miroirs
trouvera-t-ellc lorsque le gardien de sécurité G pourra la voir dans Image Image
le miroir? Type T m réelle '.’ renversée ?
/ H P
a) Concave 20 + 10
b) + 10 + 1.0 Non
c) + 2ü +30
d) +60 -0 ,5 0
c) -4 ü -1 0
n 20 + 0,10
g) Convexe 40 4.0
h) +24 0..50 Oui
Rjure 5.43 l*robléme 6

TP. Vous placez une source lumineuse ponctuelle S à une disiatx:e d IIP. Un ix'tit objet rectiligne de kmgucur L c.st parallèle à l’axe optique
devant un écran E. Quelle est la variation de l'intensité lumineuse d 'u n miroir sphérique. Si l'une de ses exiréniiiés e.st viiiiée à une
au cemie de l’écran si vous placez un miroir totalement réfléchis.sant distance p du miroir, a) dénionirez que son image duns le nuioir a une
M à une distance d derrière la source, comme c ’est illustré à longueur L' donnée par
la figure 5.44 ? (indice : Utilisez l'équation 4.27.)

M A

(Conseil; lanraliscz les deux extrémités de l'objet ) bj Démontre/


que le grandi.vscmcut longitudinal m (= L'IL) est égal il /»-, où m est
le grandissement tnmsvcrsal. wvvw
12P. a) Un point lumineux sc déplace à la vitesse de module Vq vers
un miroir sphérique ayant un rayon de courbure r. parallèlement
à l'ax e optique du miroir. Démontrez que l'im age de ce point se
Rgure 5.44 Problème 7 déplace à la vitesse de imxlulc
BP L« figure 5.45 nwintre une petite ampoule électrique suspendue
au-dessus de la surface de l’eau d’une piscine Le fond de la piscine
Vo.
est recouvert d'un grand miroir. À quelle distaiKC est située l'image
de l’ampoule sous la surface du miroir ? (Conseil : Faites un diagranune
de deu.x rayons lumineux comme celui de la figure 5 6. mais tenez où P est la distance entre le point lumineux et le miroir, ù tout instant
compte de la déviation par réfraction des rayons lumineux. Supposez donné. (Conseil: Utilise* d'abord l'équation 5.4.) Supposez main
que les rayons forment de petits angles avec un axe vertical passant tenant que le miroir est «incavc. q u e r = 15 cm et que !■„ = 5,Ocin/<;.
par l'am poule, cl utili.sez l'approxim ation pour les pieiits angles TriHivez le module de la vilc.ssc de l’iinagc lorsque b) p “ 30 cm
sin « = tan B.) (beaucoup plus éloigné que le foyer), c) p — 8,0 cm (légèrcnicni pliss
éloigné que le foyer) et rt) p — 10 mm (très près du miroir).

SECTION 5.5 les dioptres sphériques

13P. Des rayorus lumineux parallèles, (xovniant d'un Luver. .sont incidents
ù une sphère rigide et transparente dont l’indice de réfraction est n
(figure 5 46). a) S'i un point image est produit à l'arrière de la splière,
quel e.st l’indice de réfraction <k la sphère ? b) Quel indice de rétf.scbon,
le ca.s échéant, produirait un point image an centre de la sphère

Figure 5.45 Problème 8

SECTION S 4 Les imoges produites par les miroirs sphériques

9E Un miroir à maquillage concave a un rayon de courbure de 35.0 cm.


Il est placé de telle sorte que l'image (verticale) d'un visage a 2.50 fois
la taille du visage réel. À quelle distance du visage le miroir est-il placé ? Figuie5.4A P ro hlè inc 13
lOP. Complétez le tableau 5.5. où sont indiquées sur chaque ligne
les données d'une situation référant à un objet réel placé devant 14P. Complétez le tableau 5.6. où sont indiquées sur chaque ligne les
un miroir plan, un miroir splicriquc convexe ou un miroir sphérique données d’une .situation référant à un point objet réel placé dev am un
concave. Les distances sont exprimées en centimètres. Si un dioptre sphérique séparant deux milieux dont les mdiccs de réfraction
chiffre n’a pas de signe, trouvez le signe approprié. Faites un tracé sot« différents. Les distances snni expi iniées- en centimetres r''aiic.s
des rayons lum inciu |>riiiLipaux pvjur chaque situation afin de un tracé des rayons lumineux pnncip«nx pour chaque situation afin
loc.aliser l’image de l’objet réel. de localiser l'image de l'objet réel.
142 C h apitre 5 L es miroirs e t le s lentilles

TABLEAU5.6 Problème 14 : Dioptres sphériques SECTION 5.6 les lentilles minces


Image 20C. Un objet est placé à 20 cm à la gauche d ’une leniillc mince
"i n? P <7 r renversée ? divergente dont la distance ftx'ale est de .10 cm. Quelle e.st la distance
a) 1.0 1.5 + 10 + 30 image i/ ? Trouvez la positimi de l'image à l’aide d’un tracé des rayoas
lumineux principaux, www
b) I.O 1.5 MO -LT
21E. Vous produisez une image du Soleil sur un écran h l’aide d’une
c) 1.0 1.5 -t-600 -t-30
lentille mince dont la distance ftx;ale est 20.0 cm. Quel est le diamètre
d) I.O f 20 -2 0 -2 0 de l’image .'(Voir l'annexe C pour les données concernant le Soleil.)
e) 1-5 1.0 + 10 - 6,0 221. L ne lentille dont les deux faces sont convexes est fabriquée avec
f) 1,5 1,0 -7 .5 -3 0 un verre dont l’indice de réfraction est 1,5. Le rayon de courbure
d'une des faces vaut le double de celui de l'autre, et la distance focale
g) 1.5 1,0 +70 + 30
est de 60 mm. Quelles .sont les valeurs des rayons'.'
h) 1,5 + 100 +600 -3 0
23E. Une lentille est faite d ’un verre dont l'indice de réfraction est
1,5. Une face de la lentille est plane, l’autre est etmvexe et a un raytm
ISP. En rcgurJuiii ver.*« le buh, vou:s upcixeve^. une pièce de nrannaic de courbure de 20 cm. a) Trouvez, la distance focale de lu lentille,
qui reiKisc au fond d'une piscine où l’eau a une hauteur h et un indice b) .Si on place un objet à 40 cm devant la lentille, où l'image sera-t-elle
de rclraction n (figure i.47). Puisque vous regardez avec vos deux située ■'
yeux, qui interceptent des rayons lumineux differents en provenance
de la pièce de monnaie, vous voyez la pièce au point d’intersection
24f. I .’equation I i -1
l> ‘i J
des proIongcnicni.s des rayons, à une hauteur /i, plutôt que h. se nomme la forme gowv.ïicn/ic de l'équation des lentilles minces.
En supixisiiiii que les rayun.s imcrccptds par vos yeux de la figure 5.47 On obtient une variante de cette équation, la forme newtonnirrme.
effèctueni de petits angles avec f axe vertical passant par la pièce de en coiisidcrunt la di.stance x entre l ’objet et le premier foyer, et la
luonnair. déniontn.*/ tiiie h, = h/n. (Consi'il.• Ulilisez l'approximation distance .r' entre le second foyer et l'image. Démontrez que
pour les fictits angle.s .sin ( / - hin ft)
.V.Ï- = r-
csi la forme ncwionnienne de l'cquation des lentilles minces.
2SE. Un appareil photo ayant une lentille (simple) dont la distance
ftKalc est de 75 mm ciipic l’image d’une personne mesurant liiO cm
située à une distance de 27 m. Quelle est la hauteur de l'image de cette
personne sur la pellicule du film ’’
26P. Une diapo.sitivc est placée à 44.0 cm d’un écran. À qucllc(s)
disiancc(s) de la diapositive une lentille ayant une distance focale
de lü.O cm doit-elle être placée pour produire une image nette de la
diapositive sur l’écran'.' www
27P Dans la mesure du possible, complétez le tableau 5.7. où sont
indiquées sur chaque ligne les données d'une situation rclcranl à un
objet réel placé devant une lentille mince. Les distances stait cxpnniées
en centimètres. .Sous la rubrique Type, indiquez C s’il s’agit d ’une
lentille convergente cl D s’il s’agit d'une lentille divergente. Si un
chiffre n'a pas de signe, trouvez le signe approprié (saut dans le cas
16P. Dans un réservoir, une couche d’eau (où n = d'une de l'indice de réfiaciion). Faites un trace des rayons lumineux princi­
épaisseur de 20 mm (lotte sur une couche de tétrachlorure de caibone paux ptîur chaque situation afin de localiser l'image de l'objet.
(où ri = 1.46) d ’une épaisseur de 40 mm. Une pièce de monnaie 28P Une lentille convergente ayant une distance focale de 24,0 cm
repose au h'iid dti réservoir. À quelle prolondcur sous la surface de produit une image trois fois plus grande à partir d'un objet réel.
l’eau voyez.-vous la pièce de monnaie? ( C o n x r i / : Utilisez le résultat Déterminez, la distance objet p si l'image e.st a) virtuelle et b) reelic
et les hypothèses du problème 15. et faites uii tracé de rayons 29P Soit un objet réel situe à 20,0 cm d'une lentille. Si la lentille pro­
lumineux s’appliquant à cette situation, i duit une image réduite dont la hauteur est de 6 0 'SI- celle de l'objet,
17P Vou.s tenez au dessus d’une pièce de monnaie reposant sur le sol déterminez, le type de lentille et la distance focale de la lentille
un bloc en acrylique ( n = 1.42) ayant une épaisseur de 40.0 cm. si l'image est a) réelle et b) virtuelle.
I.H face inférieure du hliK- est à 25.0 cm de la pièce. Déterminez 30P. Démontrez que la distance entre un objet réel et .son image réelle
la position de l’image, lorsque vous observez la pièce à Havers le formée par une lentille mince convergente est toujours nu moins
bUx' d’acrylique quatre fois plus grande que la distance liKale de la lentille, www
18P, Une fourmi se trouve devant une sphère en verre (n = 1.50) 31P. Une lentille divergente ayant une distance focale de 15 cm
de rayon 6.00 cm. Déterminez In position de l’image forim''-e pai cl une lentille convergente ayant une distance fiK-ale de 12 cm ont
la sphère quand la fourmi est à une di.stanec de a) lü.O em e( de le meme axe optique. Il y a une distance de 12 cm entre les lentilles.
b) .*0,0 cm du contre de la sphère. I In objet d'une hauteur de 1.0 cm est placé à 10 cm en face de la lentille
IW -Voit une tige de verre {n “ 1,50) ayant scs deux extrémités divergente, sur l'axe optique commun, a) Où la combiiiai.son des
ticuscs et iloni les giaiidctirs des rayons deeoiirhnie sont de K.(X) em. lentilles produit-elle l’image finale de l'objet ite llc produite par
Un petit objet est placé le long de Taxe de la tige à une distance la seconde lentille, qui est convergente) ? b) Quelle est la h-auteur
de 50.0 em de la partie creuse. Déterminez la position de l’image de cette image ? c) L’image est-elle réelle ou virtuelle ? d) L'image
si la longueur de la tige est de 15,0 cm. a t elle la même orientation que l'objet ou est-elle renversée?
Exercices et problèmes 143

TABLEAU $7 Problème 27 : Lentilles minces


Image Image
Type ./ ''l 9 P n m réelle ? renversée?
a) C 10 ^20
b) ■HO +5.0
c) 10 - 5 .0 > 1.0
d) 10 +5.0 < 1.0
e) + 30 -3 0 + 10 1.5
0 -.30 -.30 + 10 1.5
g) -.30 -6 0 *10 1.5
h) + 10 0.50 Non
i) + 10 -0..50

32P. Une lentille convergente dont,la dislunce focale est ck + 2Ü cm 3/P. Un objet est placé à 20.0 cm à gauche il'ime lemitle coiivcrgcnic
est située à Kl cm à la gauche d'une lentille divergente ayant une d(MU la distance finale cm de -i 12.0 cm. À 14.0 cm à droite de cette
distance focale de —15 cm. Un objet est place à 40 cm à la gauche lentil le se iroust: un miroir spikrique avant un rayon de 18,0 cm. Trouvez
de la lentille convergente. Locali.sez riinagc finale produite par la la position de l’image finale formée par ce système si le mitoir est
lentille divergente et f'aitc.s-en une description complète a) concave ; b) convexe. iliiJU e: L’image finale est formée lorsque
33P. Lin objet est placé à 20 cm à la gauche d’une lentille ayant une les rayons réfléchis par le miroir traversent ù nouveau la lentille )
distance focale de + 10 cm. Une seconde lentille, ayant une distance
focale de ; 12.5 cm. est à 30 cm à la droite de la première lentille,
a) Locali.se/ l'image finede et indique/ sa taille relative, b) Vérifie/ SECTIONS.? L'œ il
vos conclusions en dcs.sinant il l'cchclle le systcnic des lentilles cl
38E. Une personne dont le punctum proximum est situé à 120 cm
en faisant un tracé des rayons lumineux principaux, c) L'image finale
de ses yeux veut lire un contrat (écrit en petits caracièncs !) situe à
est-elle réelle ou virtuelle 2 d) bsl-elle renversée'?
25 cm d’elle. Quelle tkiit être la puissoïKC de ses verres correcteurs '.’
34P Deux lentilles minces de distances focales/ , et / j sont juxta­
39E. Une personne inyo|>e est incapable de nettement voir les objets
posées. Démontre/ qu'elles équivalent à une lentille mince simple
situés à plus de 150 cm de ses yeux. Détermine? la puissanie
dont la distance focale est
des lentilles correctrices cpii lui pc'rmcttra de nettement distinguer
les objets éloignés.
/ = —/*-^1—
^ h + h ■ 40E. l-’ccil gauche d ’une personne possède un PP situé à 25,0 cm
de l’teil et une puissance d’accommodation de b,(X) D. a) Où est situé
35P. Dans la figure 5.48. l'im age réelle et renversée / d’un objet
le PR de cet oeil ? b) De quel défaut souffre-i il c) Quelle lentille
réel O est formée par une lentille (non illustrée); la distance entre
coméenne doit-on lui pnesenre’’
l'image et l'objel c.st <1 = 40,0 cm, en la mesurant le long de l’axe
optique de la lentille. La taille de l’image vaut la moitié de celle 41P. Le domaine de vision distincte d ’une personne à l’œil nu. suit
la région où elle peut ueiiement percevoir les objets, sc situe entre
de l'objet a) Quel type de lentille utilise-t-on pour produire cette
im age? b) À quelle di.stance de l’objet la lentille est-elle placée? 15.0 cm et 80.0 cm. a) Quelle est sa puissance d ’aecomniodation'^
c) (Quelle est la distance focale de la lentille ? b) De quel défaut soul trc -l elle ? C) Quelles lunettes doit-on lui
prescrire afin de corriger ce defaut ? d) Quelle est sa di.stance minimale
O de vision distincte avec *-s lunettes'’

t Axe
optique
T
42P. l.'ne personne emmétrope (son PR ent à l’infini) possède d e s
verres correcteurs de -*-2,25 D a) De quel défaul souffrc-t-cllc'?
b) Où .se situe son PP'’ c) Quelle est sa distance maximale de viskxi
- — ------------ H distincte avec ses verres ?
43P. lin hypermétrope ayant une puis.sance d’accommodation de
Figure 5.48 Problème .35
3.(K) D utilise des lentilles correctrices de 1-1,25 D lorsqu'il regarde
au loin, a) Où se situe son PP? b) Quelles lentilles correctrices
36P. Un objet lumineux cl un écran sont à une distance fixe P l’un de
l’autre, a) Démontre/ qu'une lentille convergente de distance fwalc doit on lui prescrire afin q u 'il puisse lire son journal à 25.0 cm ?
f. placée entre l’objet et l’écran, fornx;ra une image réelle nette sur c) Cet individu étant hypermétrope et presbyte a donc besoin de lentilles
l’écran aux deux positions de la lentille séparées par une distance à foyers progressifs, la partie supérieure corrigeant son hypermé­
tropie et la partie inférieure sa presbylie. Quel est son domaine de
d = y /ÏH h 4/). vision distincte avec ses lentilles correct!ices lorsqu’il regarde avec
la partie supérieure de ses lentilles?
b) Ix-montrez que 44P Une personne peut nettenKnt percevoir les objets situés entre

(IH)' 35.0 cm et STO.Ücm de scs yeux, a) De quel(s) défaut(s) souffre-t-ellc?


b) Déterminez la (les) puissance{s) de scs lentilles correctrices.
c) .Spécifiez son domaine de vision distincie avec chaque type
donne le rapport entre les tailles de l'image pour ces deux positions
de la lentille, www de lentilles correctrices.
144 Chapitre 5 Les miroirs et les lentilles

SECTION 5.8 Les insiruments d'optique la rétine situé à rinléricur de l’reil, là où commence le traitcnient des
informations visuelles. Toutefois, lorsqu’un objet est rapproché
45E On utilise une loupe d'une distance fucale de 12.0 cm pour de l’œil, les mu.scles doivent modifier la forme du cristallin afin que
observer une pièce de monnaie de collection. Si la pièce est à les rayons forment une image nette, réelle et renversée sur la rétine
9,00 cm de la loupe, calculez a) le grossissemem angulaire et (figure 5.49 cl), a) Suppose/ que, dans le cas des rayons parallèles
b) le grandi.s.scincnt tiwnsversal. des figures 5.49 a) et b), la distance f o c a le /d e la lentille mince
46E l'n insecte de 3,00 mm de longueur e.st placé devant une loupe équivalente de l’reil est de 2,50 cm. Pour un objet situé à une distance
ayant une distance focale de 15,0 cm. Si l'image produite par la loupe P = 40.0 cru, quelle distance focale f ’ la lentille doit-elle avoir pour
est située à 30,0 cm de celle-ci, a) quel est le grossissement angulaire que l’objet soit nettement perçu ? b) Les muscles de l’œil doivent iis
et b) quelle esi la longueur de l'image de l'insecte formée par la loupe ? augmenter ou diminuer les rayons de courbure de la lentille de l’œil
47E. l'n objet est placé à 10,0 mm de ro b jec iif d ’un micro.scope pour obtenir la distance focale/ ’?
compo.sé. l^ s lentilles sont distantes de 300 mm et l'image intermé­
diaire est à ,50.0 mm de l'oculaire. Quel est le grossissement angulaire Ciiscilliii
produit par l’instrument?
48E. Dans un microscope du type illustre à la figure 5.32, la distance .\tv isc lf\
f.o m c e
focale de l’objectif est de 4,00 cm et celle de l'oculaire est de 8,00 cm.
La dnstaiice entre les Icniillcs est de 25.0 cm. a) Quelle est la longueur
optique i? b) Si l’image /„b de la figure 5.32 est très légèrement I.iiniière
plus rapproeWe de l'oculaire que son foyer objet , à quelle distance piovciiaiit
de l’objectif l’objet doit il être ? Quels .sont alors c) le grandissement d ’u n oEijci
transversal m,,^ de l'objectif, d) le grossissement angulaire éloigné O a)
de l'oculaire et e) le grossissement global G du microscope? Lcntilli'
éqiiivalenlc Réiiiii*
49L Une luiieiic astronomique possède un objectif ayant une distance
fixtale de 2,(K) m et un oculaire dont la distance focale est de 1,50 cm.
Déterminez le gios.sissemcul tuigulaire si l'image finale de l'instrument
est a) à rinfini et b) à 30,0 cm de l’oculaire.
50P. Vous achetez une loupe ayant un grossissement commercial
de 2,50 pour observer un timbre, a) Si l’image de la loupe se forme b)
au PP normal de I’reil à 25,0 cm, quel est le grossissement angulaire

î
O
de la loupe? b) Si le grandissement de la loupe est +5,00, quel
est alors le grossissement tie la loupe?
~ ~ T j;
SIP. Lne loupe de distance fiK ale/ est placée près de l'reil d’une
personne dont le PP est situé à 25 cm de l’œil, l ’n objet est placé - / '— J
h P
de manière que son image dans la loupe .soit située au PP de l'reil.
a) Déterminez l’expression du grossissement angulaire de la loupe c)
en fonction de/. b) Quelle est I’expres.sion du grossissement angulaire Figure 5.49 F5'oblème 54
si l’objet est déplacé de façon que la loupe produise une image
à l'in fin i? c) Évaluez les grossissements angulaires de a) et de S5P. La figure 5.50 a) présente la structure de base d’un apj.'areil
b) quand / = 1 0 cm. (Observer une image située au PP requiert photo. La lentille peut être déplacée vers l’avant ou vers l’arrière
un effort mu.sculaire. de l’teil, alors que. pour bien des gens. oEiserver pour prtxliiire une image sur la pellicule située dans la partie arrière
une image située à l'infini ne requiert aucun effort.) de l'appareil. Dans un appareil photo dans lequel la distance q entre
la lentille et la pellicule est réglée à / = 5.0 cm. des rayons lumineux
52P. La distance focale de l’objeclif d'un microscope est de 8.00 mm
parallèles provcimnt d'un objet O très éloigné convergent vers un
et celle de son txtulaire de 2,50 cm. Si la distance entre les lentilles
point image sur la pellicule, comme le montre l’illustration. L’objet
est de 17,0 cm et que l'image finale de l’instrument se trouve au PP
est ensuite rappriK-hc à une distance p = 1(X) cm. et la distance entre
d’un (EÜ normal, à 25,0 cm de l'oculaire, a) où se situe l'objet par
la lentille et la pellicule est ajustée de manière qu’une image nette,
rapport à l’objectif et b) quel est le grossissement angulaire’’
réelle et renversée sc forme sur la pellicule (figure 5.50 b|V a) Quelle
53P. L’objectif et l’oculaire d’une lunette astronomique ont respec­
est la nouvelle distance q entre la lentille et la pellicule ? b) De quelle
tivement une distance focale de 1„50 ni et de 2,50 cm On utilise
distance a-l-on déplacé la lentille ?
la lunette pour observer la Lune qui .sous-iend un angle de 0,52°
lorsqu’on l’observe à l’reil nu. Si l’image finale de la lunette est
â 60,0 cm de l'oculaire, déterminez a) le grossissement angulaire,
b) la longueur de la lunette et c) l'angle sous-tendu par la Lune
lorsque l’œil, accole à l’oculaire, l’observe à travers la lunette.
54P. La figure 5.49 a) prc.scntc la stnielure de twisc de l’œil humain,
lai lumière est réfractée dans l’œil à travers la cornée et e.st ensuite
réorientée par une iemille, le cristallin, dont la forme (donc la capacité
à fi>caliscr la lumière) est contrôlée par des muscles. On peut considérer
la cornée et le cristallin comme une simple lentille mince équivalente
(figure 5.49 h]). Un ceil «norm al» peut concentrer des rayons b)
lumineux parallèles provenant d ’un objet éloigné O en un point sur Figure 5.50 F°roblème 55
6 L’interférence

À première vue, le dessus de l'aile du papillon morpho présente une surface teintée de bleu et de vert. La couleur
a toutefois quelque chose d'étrange car elle est presque luisante, contrairement aux couleurs de la pluport des objets -
et si vous vous déplocez ou si l'aile bouge, cette couleur change. L'aile est irisée, et la teinte de bleu vert que l'on voit coche
lo « vraie »couleur brune
qui apparaît sur la surface
intérieure de l'aile.
146 Chapitre 6 L'interférence

6.1 L'interférence
C om m e on peut le voir dans un ate cn-ciel, la lum ière du soleil se com pose de toutes les
couleurs du spectre visible. L es couleurs individuelles apparaissent dans l ’arc-en-ciel
p arce q u e les lo n g u eu rs d 'o n d e in cid en tes .sont d év iées su iv an t d iffé ren ts an g les en
traversant les gouttes de pluie qui créent l arc-en -d el. Les bulles d ’eau .savonneuse et les
nappes d ’h u ile p eu v e n t au ssi rév é ler des co u leu rs saisissan te s, p ro d u ites non p a r la
réfraction, m ais p ar une in te rfé re n c e consli-uctive et destructive de la lumière. Les ondes
SC com binent soit pour supprimer, soit pour accentuer certaines couleurs dans le spectre de
la lum ière solaire incidente. I .’interférence des ondes lum ineuses est donc un phénom ène
de supeiposition. sim ilaire aux phénom ènes dont on a discuté dans le chapitre 2 .
On a trouvé de nom breuses applications à l’accentuation et il la suppression sélective
des longueurs d ’oncle. L orsque la lu m ière frappe une su rfa ce de v erre o rd in aire, par
exemple, environ 4 % de l’énergie mcidentc est réfléchie, ce qui affaiblit d ’autant le faist^Citu
transm is. C ette perte de lum ière peut constituer un sérieux pi'oblème dans les systèm es
o p tiq u e s co m p re n an t p lu sieu rs co m p o san tes. U ne p ellic u le m ince et tran sp a ren te
déposée à la surface du verre peut réduire la quantité de lum ière réfléchie (et augm enter
la q uantité de lum ière transm ise) g râce à l'in te rfé rc n e c d estru ctiv e. L a teinte bleutée
des lentilles d'appareil photo révèle la présence d ’une telle pellicule. Les pellicules peu\ ent
égalem ent servir â acen n ta' - plutôt q u ’à dim inuer - la réflectivité d ’une surface.
Four com prendre l’interférence, on doit aller au-delà des restrictions habituelles de
l’optique géom étrique, et exploiter pleinem ent l’optique physique. L n fait, com m e vous
le verrez, le phénom ène d 'in terféren ce est peut-être la preuve la plus convaincante de la
nature ondulatoire de la lum ière car l'in terféren ce ne peut s ’expliquer q u ’à partir du
concept d ’onde.

6.2 La nature ondulatoire de la lumière


I £ physicien danois C hristiaan H uygens fui le premier, en 1678. à avancer de m anière
convaincante une théorie ondulatoire de la lum ière. M êm e si elle était beaucoup m oins
com plète que la th éo rie su r l ’éleclro m ag n étism e q u e M axw ell p ro p o sa par la suite,
la théorie de H uygens était plus sim ple d ’un point de vme m athém atique, et dem eure très
utile aujourd'hui. Scs grands m érites sont de perm ettre d ’expliquer les lois de la réflexion
e t de la réfraction à partir du concept d ’onde, et de définir l'in d ice de réfraction du point
de vue de la physique.
L a th é o rie o n d u la to ire de la lu m ière d e H u y g en s rep o se su r une c o n stru c tio n
géom étrique qui perm et de prédire où se trouvera, en tout tem ps, un front d ’onde donné
si on connaît sa position actuelle. Cette construction est basée sur le p rin cip e d e H uygens,
décrit ci-dessous.

► Tous le.s points d’un front d’onde servent de points sources à de petites ondes secondaires
sphériques. Après un temps I, la nouvelle position du front d’onde sera celle de la .surface
tangente à ces ondes secondaires.

Voici un exem ple sim ple : dans la partie gauche de la figure 6 .1, la position actuelle d ’un
front d 'o n d e d 'u n e onde plane se propageant vers la droilc dans le vide est représentée
par le plan ab. p erp e n d icu la ire an plan d e la page. O ù le froni d ’o nde scra-t-i! après
un tem ps A t'i De nom breux points (les points noirs) contenus dans le plan tih servent
de sources à des ondes sphériques secondaires qui sont ém ises à / — 0. À l’instant At.
le rayon d e toute,s ces ondes sphériques atteindra cAt. où c est le m odule de la vitesse de
la luimère dans le vide. O n trace le plan de tangent à ces ondes secondaires à l’instant At.
Nouvillr C e plan rep rése n te le front d 'o n d e d e l’o n d e p lan e à l’in stan t A t ; il est p arallèle an
plan ah. et se trouve à une distance pt'ipendiculaire cAi de ce deniicr,

A l'iii.vlant
/ - A/ La loi de la refraction ^
Figure 6.1 i.» propagaiinn d ’une onrle O n u tilise ra m ain ten an t le p rin cip e d e H uygens p o u r d é riv e r la lot de la réfrat lion,
plane dans le vide, représentée l’équation 4.41 (loi de S n ell-D cscaites). La figure 6.2 m ontre trois étapes lors rie la
on fonction du principe de Huygens. réfraction de plusieurs fronts d ’onde à une interface plane entre l’air (milieu 1) et le v e n e
6.2 La nature ondulatoire de la lumière 147

(m ilieu 2). O n pose arb ilrairem en l qu e la d istan ce sép aran t les fronts d ’onde dans le
faisceau de lum ière incid en te est À ,. la longueur d ’onde dans le m ilieu 1. Le m odule
de la vitesse de la lum ière est v, dans l’air, et V7 dans le verre. O n suppose que vs < v„
ce qui est effectivem cnl le cas.
D ans la figure 6.2 a), est l’angle form é entre le front d ’onde ci l'in te rfa c e ; il a la
m em e valeur que l'an g le form é entre la normale au front d ’t)nde (c’est-à-dire le rayon
incident) et la normale à l’interface. W, est donc l’angle d'in cid cn cc.
L o rsq u ’une o nde .se p ro p ag e dans le v erre, un e o n d e se co n d a ire de H u y g en s au
point e s ’étend et passe par le point c, à une distance A, du point e. L’intervalle de tem ps
requis p ar cette ex p an sio n est cette d istan ce d iv isée par le m odule de la vitesse de la
petite onde, ou Aj/v,. N otez que, durant ce m ême intervalle de tem ps, une petite <*nde au
point h s’étendra et passera par le point g, à la vitesse de m odule m oins grand i i cl avec
une longueur d ’onde A,. C et intervalle do tem ps doit donc aussi être égal à A^/v;. Si on
m et en équation ces tem ps de parcours, on obtient la relation

il l'i
(6 . 1)
A-i

qui dém ontre que les longueurs d ’onde de la lum ière dans les deux m ilieux « in i p ro p ir-
tionnclles aux m odules de la vitesse de la lum ière dans ce.s m ilieux.
D ’après le princi|Te d e H uygens, le front d ’onde réfracte doit être tangent à un arc
de rayon A2 dont le centre est h (au point g, par exem ple). Le front d ’onde réfracté doit
égalem ent être tangent à un arc d e rayon A, dont le centre est e (au iw int r. par exem ple).
L e rayon réfracté doit alors être o rien té tel q u ’il est représenté N otez qu e (h, l'an g le
entre le front d 'o n d e réfracté et l’inicrfiice, est réellem ent l’angle de réfraction.
En sc référant aux triangles rectangles hcc et heg de la figure 6.2 b), on |reui écrite

■sin f)i = — (relativement uu triangle hce)


ne
A2
et sm 0-, = — (relativement au triangle ht g),
ne

E n divisant la prem ière de ces équations pio" la seconde, e t en utilisant l ’équation 6.1.
on déterm ine que

sin W| Al ÏL ( 6. 2 )
sin r-i

i>) L 'in d ic e d e réfracticin n de cliaquo m ilieu peut être défini com m e le rapport entre
le m odule de la vitesse c de la lu m ière dans le vide et le iniKlulc de la vitesse i- d e la
lum ière dans le m ilieu. D onc,

M= - (l’indice de réfiactiiHi). (6 .,^)


!•

D ans le cas des deux milieux d e la figure 6.2. on a donc

c e
n, = et 117 ^ (6.4)
Vl ‘ V)

Si on com bine les équations 6.2 cl 6.4, on déterm ine que


figure 6.2 La réfraction d'une onde plane
à une interface air-vcrre. représentée sin fl| c /H | «2
en fonction du principe de Huygens. (6.5)
sin fil C/H2 W|
La longueur d’owlc dans le verre
est plus petite que celle dans Pair
Pour simplifier, on n’a pas représenté
ou U| .sin fif = ih sin fil (lu loi de la réfnielion), ( 6 .6 )
fonde rcficchie Les parties a)
à c) représentent les trois crapes
successives de la réfraction q u ’o n a présentée dans le chapitre 4
148 Chapitre 6 L’Interférence

✓ VÉRIFIEZ VOS CONNAISSANCES 1 : La f.gnre ci-contre


représente un rayon de lumière monochromatique
SC propageant en traversant des interfaces parallèles, à partir
d’une substance initiale a, puis en traversant des couches
de substances h et c, pour se trouver de nouve.'iu dans
la substance u. Classe/ les substances en ordre décroissant,
selon le module de la vitesse de la lumière datis chacune d ’elles.

Longueur d ’onde et indice de réfraction


O n a vu q u e la lo n g u e u r d ’o n d e d e la lu m ière v ariait en fo n clio n du m o d u le d e .sa
vitesse, com m e cela se produit quand la lum ière traverse une interface en passant d ’un
m ilieu à un autre. O n sait aussi, grâce à l’équation 6.3. que. dans tout milieu, le m odule
de la vitesse de la lum ière dépend de l’indice de réfraction du milieu. Il s'en su it que la
lo n g u eu r d ’onde d e la lu m ière dans un m ilieu d ép en d de l’indice de réfractio n de ce
milieu. .Soit une lum ière m onochrom atique ayant une longueur d 'o n d e À et une vitesse
de m odule c dans le vide, el une longueur d ’o nde À„ et une vitesse de m odule v dans un
m ilieu dont l’indice de réfraction est n. O n peut m aintenant récrire ainsi l’équation 6 . 1 :

(6.7)
c

Si on utilise l’éqiialion 6 3 ¡x>ur substituer Mn à vtc, on a alors

k
- ■ ( 6 . 8)
n

C ette équation relie la lo n g u eu r d ’o n d e de la lu m ière dans tout m ilien â sa lo n g u eu r


d ’onde dans le vide. Elle indique que plus l’indice de réfraction d 'u n m ilieu est élevé,
plus la longueur d ’onde de la lum ière sera petite dans ce milieu.
M ais q u ’en est-il de la fréquence de la lu m ière? S o ii/„ la fréquence de la lum ière
d an s un m ilieu d o n t l'in d ic e d e ré fra c tio n e st n. D 'a p rè s l ’éq u a tio n 2.12 (v = k f ) ,
on peut écrire

f = — •

Si on insère le.s équations 6.3 et 6.8 dans l'cq u atio n précédente, on obtient;

c/n c
f , = 7a /Tu = a■ = /

o ii/ c s t la fréquence d e la lum ière dans le vide. Il s’ensuit donc que. m êm e si le m odule
de la vitesse et la longueur d ’onde de la lum ière sont différents dans un m ilieu com para­
tivem ent à ce q u ’ils sont dans le vide, ki fréquence de lu lumière dans ce mUieu est lu
même que dans le vide.
L e fait que la lo n g u eu r d 'o n d e d e la lu m ière dépende de l ’indice de réfraction,
com m e le m ontre l’cquation 6 .8. est im portant dans certaines situations m ettant en jeu
l’intcrfcTcncc des ondes lumineuses. Piir exem ple, daas la ligure 6.3, les rnvnm lumineux
(où les ondes re p rése n tée s so n t des ray o n s) o nt d es lo n g u e u rs d 'o n d e id cn liq iies k.
el sont initialem ent en phase dans l’air <n I ) U ne des ondes voyage dans le m ilieu 1
de longueur L e t dont l'in d ice de réfraction est Hi. L ’autre voyage dans le milieu 2, aussi
de lo n g u eu r L, d o n t l ’indice de réfractio n e.st l-or.sque les ondes q u itten t les deux
m ilieu x , e lle s ont la m êm e lo n g u e u r d ’o n d e leu r lo n g u e u r d ’o n d e À d an s l ’air.
Toutefois, com m e lt;urs longueurs d ’onde ont été m oditiées dans les deux m ilieux, clics
ne sont peut-être plus en phitse. ^
Figur« é.3 Deux rayons de lumière
SC proitatcnl dans deux milieux
l/a différence de phase entre deux ondes lumineuses peut varier si les ondes viiyagc^nt dans
qui ont des indices de réfrac tion
des matériaux différents ayant des indices de réfiaction différents.
diflércnts.
6.2 La nature ondutotoire de ta lumière ]49

C om m e on le v erra bientôt, celle vaiialion d e phase peut d éterm in er la façon dont les
ondes interféreront si clics atteignent un point com m un.
P our tro u v er la d ifférence d e phase, ex p n m éc en fonction de la longueur d ’onde,
on co m p te d 'a b o rd le nom bre A'i de lo n g u eu rs d ’o n d e co m p rises d an s la lo n g u eu r L
du m ilieu 1. D ’après l’équation 6 .8 , la longueur d 'o n d e dans le m ilieu 1 est = À/«, ;
il .s’ensuit donc que
L Lnf
(6.9)

D e la m êm e faço n , on co m p te le n o m bre d e lo n g u eu rs d ’o n d e co m p rises d an s la


longueur L du m ilieu 2, où la longueur d ’onde est = A/h i :

Ny ( 6 . 10)
A
P o u r tro u v er la d ifféren ce de phu.se en tre l’o nde 2 cl l ’o n d e 1, on so u strait A'i de N^.
O n obtient
/n i
Ni = — -----------^ ^ («2 «,). (6 . 11)
A

Supposons par exem ple qu e l’équation 6 .1 1 indique que les ondes o n t m aintenant
une différen ce de phase de 4 5 .6 longueurs d 'o n d e. C ela éq u iv au t à p rendre les ondes
initialem ent en phase et à d ép h ascr l une d ’elles de 45 ,6 longueurs d 'o n d e. Toutefois,
un déphasage d ’un nom bre entier de longueurs d ’onde (com m e 45) rcm ctirait le.s ondes
en p h ase, e t se u le la fractio n d éc im a le (0 .6 d a n s c c cas) c.si d onc im p o rta n te. U ne
différence de (Aase de 45,6 longueurs d ’onde équivaut à une difference effective de phase
de 0,6 longueur d'onde.
U ne d ifféren ce de pha.se de ' longueur d ’o nde a pour effet de placer les ot»des en
o p p o sitio n d e p h ase. Si les o n d es o n t d es a m p litu d e s é g a le s et a tte ig n e n t un p o in t
com m un, elles subissent une interférence destructive, ce qui prinluii de l’obscurité à t e
point. A vec une différence de phase de 0 .0 ou de 1.0 longueur d ’onde, elles subissent
plutôt une interférence coasiruciive, cc qui prtxluit une illum ination m axim ale au point
com m un. 1^ d iffé re n c e de p h ase d e 0 .6 lo n g u e u r d ’o n d e rep rése n te une situ atio n
interm édiaire, quoique plus pioche de l’interférence destructive, et le.s ondes illum inent
partiellem ent le point com m un.
O n jx u t aussi exprim er la différence de jihasc en radian.s ou en degrés, com m e on
l'a déjà fait. U ne différence de phase d 'u n e longueur d ’onde équivaut à des d ifféren tes
de phase de 2;r rad ou de 360®.

Exemple 6.1
Dans la ligure 6.3, le.s deux ondes lumineuses représentées par leurs (pour l'airl. Hj = l.6(M). I = 2.6IM) /tm. et A = 550.0 nm. L'équa­
rayons ont une longueur d’onde de 550.0 nm avant d ’entrer dans tion 6.11 donne alors
les milieux 1 et 2. Elles ont des amplitudes égales et sont en phase,
l e milieu I est l'air, et le milieu 2 est une couche de plastique Nz — Nj = {tf} — « |)
transparent ayant un indice de réfraction de l,6(X) et une cpais.seur A
de 2.600 /im. _ 2.600 X 10“ ®ni
(1.600 I.nOO)
~ 5,.5(X) X 10-'' m
a) Quelle est la différence de pliase des ondes qui émergent, exprinréc
= 2.84 (réponse)
en longueurs d’oixle. en radians et en degR^ (Quelle est leur différence
de phase effective (en longueurs d ’onde)? La différence de pha.se entre les deux ondes émergentes est donc
de 2.84 longueurs d'onde. Puisque 1.0 longueur d'onde équivaul à
SOlUTION: On utilise le tontept clé selon lequel la différence de phase
2jt rad ou à 360®, on peut dcinontrer que cette différence de phase
enirc deux ondes lumineuses peut varier si les ondes se propagent dans
équivaut à
des milieux differenu présentant des indices de réfraction différents.
Cela s’explique par le fait que leurs longueurs d'onde sont différentes différence de phase = 17.8 rad “■1 020®. (réponse)
si elles sont dans des milieux différents. On peut calculer la variation
Scion un autm tontepl de, la dittéfeiKC «.le ptia.se effective est la portion
de la différence de phase en comptant le nombre de longueurs d’onde
déciiiiale de la différence de phase exprimée en longueur!: d'onde.
compri.ses dans chaque milieu, et en soustrayant ensuite ces nombres
On a donc
l.nrscjue 1rs longueurs pareoiirties par les ondes sont identiques,
rétpiation 6.11 permet de résoudre le problème. On a ici = l.CXX) différence de phase cfl'eclive = 0.84 longueur d'onde (réponse)
ISO Chapitre 6 L’interférence

On peut démontrer que cela équivcuit à 5,3 rad ou à environ 3(Xr. d’ondc. Les ondes produiraient donc un type d’interférence intermé­
Attentton : on ne nvuve pas la différence de phase effective en prenant diaire qui sc rapprocherait d'une interférence constructive - et elles
la portion décimale de la différence de phase exprimée en radians ou produiraient un point relativement brillant.
en degrés. Par exemple, on ne prend pas O.X rad d'une différence de
phase complète de 17,8 rad.

b) Si les rayons des ondes étaient légèrement dévies afin que les ondes
K VÉRIFIEZ VOS CONNAISSANCES 2: Les ondes lumineuses
des rayoti-s de la ligure 6.3 ont la même longueur d’orxle et la même
atteignent le même point sur un écran éloigné, quel type d’interférence
amplitude, et sont initialement en phase, a) Si 7.60 longueurs
les onde.s prtKluiraient-cIlcs à cc point
d ’ondc sont comprises dans la longueur du matériau du dcssu.s,
SOLUTION; Le tontepi dé consiste ici à comparer la différence de phase et 5.50 longueurs d ’or.de sont comprises dans la longueur
effective des ondes aux différences de phase qui produi.sent les situa­ du matériau du des.soas, lequel de ces matériaux a l'indice
tions extrêmes d’interlérence. Dans ce cas, la différence de phase de réfraction le plus élevé '>b) Si les rayon» .sont légèrement
effective de 0.84 longueur d'onde se situe entre 0,5 longueur d'onde déviés de sorte qu’ils atteignent un point commun sur un écran
(valeur d line interférence destructive, ou le résultat le plus sombre) éloigné, l’interférence produira-t-cllc l’illumination la plus
«l 1.0 longueur d’onde (valcui d'une inicrféreiKe constructive, ou le brillante po.ssihle. une illumination moyennement brillante,
résultat le plus brillant), mais el le est plus rapprochée de 1,0 longueur un point moyennement sombre ou un point sombre'^

6.3 La diffraction
D ans la prochaine .section, on parlera tic l’expérience qui dém ontra pour la prem ière fois
que la liim icie r.si title onde. M ais on doit d 'ab o rd approfondit le concept de d iffra c tio n
des ondes, tin phénom ène q u 'o n étudiera plu.s en détail dans le chapitre 7. E^ssentiellemenl,
le concept est le suivant : si une «rnde rencontre un obstacle qui prc.sente une ouverture
de dimension similaire à sa Irxigueiir d ’ondc, la partie de l ’onde qui passera par l’ouverture
s ’étendra - fera l'o b je t d ’une d iffractio n - dans la région au-delà de l’obstacle. C ette
expansion est cohérente quant à l'ex p aasio n tics petites onde.s diui.s la représentation de
H uygens de la figure 6 .1. L a d iffractio n sc p m d u it avec tous les types d ’onde, et pa.s
seulem ent avec les ondes lum ineuses ; la figure 6.4 m ontre la diffraction d’ondes qui sc
propagent à la surface de l’eau dans un réservoir peu profond.
La ligure 6„5 a) représente schém atiquem ent la situation d ’une onde plane incidente
de longueur d ’onde X qui traverse une fente de largeur о - 6,0 к et qui .s'éteixl perpendicu­
lairement au plan de la page. 1.'onde se di.spct.se de l’autre côté de la fente. Les figures 6„5 b)
(où a = 3,0k) cl 6.5 e) (où il = 1,5л) iUustrcni la principale caractcrisliquc de la diffraction :
plus la fente est étroite, plus grande est la dispersion de fo n d e transm ise par la fente.
diffraction pose des lim ites à l’optique géom étrique, où l’on représente une onde
Figuie 6.4 1.a diffraction des ondes
électrom agnétique par un rayon. Si on cssiue réellem ent de fo rm er un rayon en irans-
dans un bac à ondes. Les ondes
m ettanl de la lum ière à travers une fente étroite, ou à travers une série de fentes étroites,
sont produites par un oscillateur situé
à la gauche. En se déplaçant de gauche la diffraction po.sera toujours un problèm e parce q u 'e lle produit toujours une di.spcrsion
à tiroitc. elle.s se dispttrsciu en pa.ssant du fai.sccau de lum ière. En effet, plus on fera des fentes étroites (dans l ’espoir d ’obtenii
à travers rouveriure faite dans un rayon plus étroit), plus la dispersion sera grande. L’optique géom étrique s'applique
un obstacle placé à la surface de l'eau. donc seulem ent lorstiue la dim ension des fentes ou des autres o u v en u rcs pouvant êlrc
situées dans le parcours de la lum ière n 'e st pas com parable ni plus petite à la longueur
il’ondc d e la lum ière.

6 A L'expérience de Young sur l'interférence


E.I1 1801. T hom as Y oung d ém o n tra ex p érim en talem en t qu e la lum ière est une (inde,
infiirnanl ainsi cc que la m ajonté des autre.s .scientifiques croyaient alors. H y parvint en
dém o n tran t que la lu m ière p roduit une interféreiKe,. com m e Ic.s ttndt s se p rt'pageani
dans l'eau , les ondes .sonores el les ondes de tous les autres types. De plus, il fut capable
de m esu rer la lon g u eu r d ’o nde m oyenne de la lum ière du .soleil ; la v aleu r à laquelle
il arriva. 570 nm. est sensiblem ent rapprtx:hée de la valeur m oderne, établie à 555 nm.
ü n exam in era ici re x p é n e n c e de Y oung en tant q u ’illiistration de l’in terféren ce des
ondes lununcu.ses.
La ligure 6.6 représente rarra n g em c n i de hase utilisé dans l’expérience de Young
l>a lum ière issue d’une souri e m onochrom atique éloignée illum ine la fente .Vo de l’écran
A. La lum ière qui ém erge .s'étend ensuite, sous l’effet de la diffraction, et éclaire deux
fentes. 5[ et S 2, de l ’écran /?. La diffraction de la lum ière à ces deux len tes transm et des
ondes circulaires se chevauchant dans la région située de l'au tre côté de l’écran B, où les
ondes en provenance d ’une fente interfèrent avec les onde.s provenant de l’autre fente.
6 4 L’expérience de Young sur l’interférence 151

Oiidf Onde
■iK'idt'iiU' clifftartêe
H

T
n
J.
(6,(IA)
ülrt-
(S.OA) C
F.rraii t>) <:)
figure 6.5 Representation schématique de la diffraction l.ians le cas d’une longueur d’onde
donnée À, la dilïraction est plus prononcée lorstiue la largeur a de la fente est plus petite.
Les figures représentent les situations relativement à a) une fente de largeur a — 6.0/.
h) une lente de largeur a .l.OÀ. et c) une fente de largeur a — 1,5a . Dans les trois ca.s,
l’écran et la longueur de la fente s’étendent pcrpendiculaiieTncnl au plan de la page, cl lu lumière
transmise à droite des écrans ne passe que par les fentes

L’« in stan ta n é» d e la figure 6.6 illustre l’interférence dc.s oncles qui se superposent
T outefois, on ne peut v o ir la preuve d e l ' in terféren ce, sa u f à l'e n d ro il où un écran C
intercepte la lumière. lair.sqiie cela se p ro d u it des points oi'i l’interférence est constructive
form ent des rangées brillantes nom m ées batuks hrilUmws, franges brillantes ou rnmima
qui se répartissent sur l'éc ra n (perpendiculairem ent au plan de la page dans la figure
6 .6 ). L es régions som bres - nom m ées bandes sombres, franges sombres ou minima -
résultent d ’une in terferen ce d estru ctiv e et sont visib les en tre deux fran g es b rillantes
a d jac en te s. (L es maxim a e t les minima se ra p p o rte n t p lu s prcci.scm cnl au cen tre
d ’une b an d e .) l e m ofif de fran g es brillantes e t som bres su r l'é c ra n se nom m e fig u re
d ’in te rfé re n ce ou p a tro n d ’in te rfé re n ce . La figure 6.7 est une photographie d 'u n e punie
de la figure d ’interférence, lorsque l’obseï vateui se situe à g auche dans la figure 6 ,6 .

Max

Viax

Max
Onde
inddcnfe Max
MxK
Max

May
M ax
Max

Max

Max

Figure 6.6 Dans l'expérience de Young sur l’interférence, nm: liimièu: monochromatique
incidente est diffractée par la fente ■Vthqui agit ensuite comme une source ponctuelle de lumière
émettant de.s front.s d’onde semi circulaires. Quand cette lumière atteint l’écran fi, Hic est
diffractée pai les fentes ,5| et Sy. qui agissent à leur tour connue des .sources ponctuelles
de lumière 1r s ondes liiriiinruses se ixopagrant à partir des fentes .V, et iS-. ,se chevauchent
et produi.scnt de l'interférence, formant une ligure d'intei férence composée de maxima et
de minima sur l'ceran C. Cette figure représente une coupe transversale: les écrans, les fentes
et la figure d’inin'lérciux; .s’éiendenl perpendiculairement au plan de la page F.ntro les écrans H
et C. les fronts d’onde semi circulaires ayant (xtui centre illustrent les ondes qu on observerait
.si seule la fente i'a était tiuverto. f>e la même fayon, les (ic)nt.s d ’onde ayant .5, pour centre
reprcscnleni les ondes qu’on ob.servcrait si seule la fente était ouverte.
152 Chapitre 6 L’interférence

Localiser les franges


Les ondes lumineuses produisent des franges d am Vexpériente des deux fentes île Young.
M ais q u ’csl-cc qui d éterm in e ex actem en t l’em p lacem en t des franges ? Pour répondre
à cette question, on utilisera la représentation de la figure 6.8 a). On y voit une onde plane
de lu m ière m o n w h ro m a liq u e q u i est in c id e n te au x fen tes A’, et 6% su r l’écran H-,
la lum ière est diffractéc par les fentes et produit une figure d ’interférence sur l’écran C.
On dessine un axe central reliant le point central entre les fentes ei l’écran C. O n choisit
ensuite un poini arbitraire / ’ sur l’écran, à un angle Wpar rap|X)i1 à l’axe central. C e point
est le point d ’intersection entre l’onde du rayon r, issu de la fente du bas et l’onde du
rayon r , issu de la fente du haut.
I o rs q u ’elles passent à travers les deux fentes, ces ondes sont en phase, car elles ne
sont que des parties de la m êm e onde incidente. Toutefois, après avoir traverse les fentes,
les deux ondes d o iv en t p arc o u rir des d istan ces d iffé ren te s p o u r attein d re le |X)int P.
On a vu une situation sem blable avec les ondes sonores dans la section 3.4. el on a alors
conclu ce qui suit.

► La diftérencc de pha.se entre deux ondes peut varier si les ondes suivent des parcours
de longueurs différentes.

Figure 6./ Une de la П{шге L a variation de la d ifféren ce de p h ase est d ue à la difference de marche A L dans les
d’irtcrtcrcncc produite pai' l’arrange p a a o u rs suivis par les ondes. On peut co n sid érer ici deux ondes qui sont initialem ent
rneni lUuslré dans la figure 6.6. en phase el qui suivent des parcours présentant une dificren ce de m arche A L puis qui
(I.a pFioi(igra[Aiie est une vue de face se com binent en un point com m un. L orsque AL e st nulle ou vaut un nom bre en tier de
d’une pmtie de l'écnui C.) Les rnaxirna longueurs d ’onde, les ondes qui arrivent au point com m un sont parfaitem ent en phase,
et minima а1|спк-л 'te nonimcni et p ro d u isen t à c e p o in t une in te rfé re n c e c o n stru c tiv e . Si ce la est vrai d an s le cas
les.frtOT^f.ï d'interférerwe (car ils des ondes des rayons lum ineux r^ et ri de la figure 6 .8. le point P se trouve alors dans
ressemblent aux franges décoratives
une fran g e b rilla n te. Si, au co n tra ire. Al. est un m u ltip le im p a ir de la m o itié d ’une
utilisée.s dans les vêlements et les lapis).
long u eu r d ’o n d e, les o n d es q ui a rriv e n l au p o in t co m m u n so n t d ép h a sé es d e n rad
et produisent à ce point une interférence destructive. Si cela est vrai dans le cas des rayons
lum ineux fj c l r j, le p o in t P se tro u v e alo rs d an s un e fran g e so m b re. (Ht. bien sûr.
il existe aussi des situations interm édiaires d ’interférence, donc des situations de brillance
interm édiaire au point P. )
D onc, on peut affirm er ce qui suit.

1^ Dans l'cxpcricncc des deux fentes de 'Young, cc qui apparaît à chaque point de l'6 :ran
est détemâné pai la différence de marche Al. des rayons qui atteignent ce point.

Figure 6.8 a) ondes issues des fentes .S'i cl Sj (qui s’étendent perpendiculairement au plan
de la page) se combinent à 7) un point arbitraire sur l’écran C, situé à une distance y de l’axe
central. L’angle ê permet de localiser P. b) Si /J d, on peut donner tomme approximation
que les rayons r, et sont parallèles, à un angle Dde Taxe central.
6.4 L’expérience de Young sur l’interférence 153

On peut situer sur l'écran chaque frange brillante ou sombre en donnant l’angle é»entre
l ’axe central et cette frange. P our trouver ft on doit le relier à AL. D ans la figure 6.8 a),
on U'Ouve le long du raytm ri le point h séparé de P par une distance égale à la distance
entre ^2 et P. L a différence de m arche entre les deu.x rayons e.st alors la distance entre
Sj et h.
L a relatio n e n tre c e tte d istan c e (d ista n ce en tre 6'i et h) e t 6 e.st co m p lex e , m ais
on peut la sim plifier si on lait en sorte q ue la di.stance D entre les fentes et l’écran soit
beaucf)up plus gran d e que la distarK e d qui sépare les deux fentes. O n peut alors faire
l’approxim ation que les raytm s r, et r2 sont parallèles et form ent un angle Wa\ ec l'ax e
central (figure 6.8 b). O n peut aussi faire l’appntxim ation que le triangle form é par i',,
et b est un triangle rectangle, et qu e l ’angle interne de ce triangle à S 2 est ft D ans ce
triangle, sin B — AL/d, il s ’ensuit que

AL = d S.in$ (la différence de marche). ( 6 . 12)

Pour q u ’une frange soit brillante, on a vu que A l d o it être nulle ou égale à un nom bre
entier de longueurs d ’onde. Si on utilise l’équation 6.12, on peut écrire cette condition
sous la form e

AL = d sin 0 — (un nom bre entier) (À ), (6.13)

ou .sous la form e

if sin 6 = trik, lorsque ni = 0 , ' l, - 2 .... (les maxima - franges hrillames). (6.14)

O n rem arq u e qu e p o u r ch aq u e frange au -d essu s d e l ’axe cen tral (;ii > ü), il y a une
frange située de façon symétrique sous l'a x e central. Ces franges sont obtenues en insérant
des valeurs négatives de m dans l’équation 6.14.
P o u r q u ’une fran g e so it so m b re. A l d o it être un m u ltip le Im p air d ’u n e d em i-
longueur d ’onde. .Si on utilise de nouveau l'éq u atio n 6.12, on {teut écrire cette condilirm
s(»us la form e

AL — J sin 6 = (un nom bm im pair) (¿À). (6.15)

ou sous la form e

rf sin 6 = {m + 1) A, lorsque m = 0, ± 1, ± 2 . ... (les minima Iranges sombres). (6.16)

L es valeurs négatives de m décrivent les franges som bres sous l’axe central
À l’aide d es éq u a tio n s 6 14 et 6 .1 6 , on p eu t tro u v e r l ’an g le B ûc c h a cu n e des
frangc,s, donc leu r p osition ; d e plus, on p eu t u tiliser les valeurs d e in jxuit id en tifier
les franges. 1n isq u c la valeur de m — 0 . l’équation 6.14 indique qu une fnm gc brillante
c.sl à 0 — 0, c 'e st-à dire sur l'a x e central. C e mo.xininm ctnitml est le point où les ondes
issu es des deux fen tes o n t un e d ifféren ce de m areh e A L - 0 , et ne p résen ien l donc
aucun déphasage.
L orsque ni — Jz2. par exem ple, l’équation 6.14 indique que de.s franges hrillotUr.x
sont à l’angle

au-dessus et en dessous de Taxe central. Les ondes is.siies des deux fentes arri vent à ces deux
franges avec une d ifférence de m arche égale à lA /J — 2X. et so n t déphasées de deux
longueurs d ’onde. On nom m e ces franges Aca. frun^tts de deuxième ordre fcc qui .stpmfk
que m ± 2), ou les deuxièmes nui.ximu (les deuxièm es m axim a adiaccnts au maximum
cen tral), on |x;ut aussi les d éfin iro x n m e étant les deuxièm es franges à partir du maximum
central.
L orsque m = 0. l'équatiim 6 16 indique que la prem ière frange som bre au-de.ssus
de l'ax e central .se trouve à l’angle
154 Chapitre 6 L'interférence

L a prem ière frange som bre sous Taxe central se trouve à l ’angle opposé

« -s in '( ^

que l’on trouve en inscranl la valeur m - —1 dans l’équation 6.16. Les ondes issues des
deux fentes arrivent à ces deux franges avec une différence de m arche \ÙJj = j A e t .sont
donc déphasées d ’une d em ie lo n g u eu r d ’onde. O n nom m e ces fran g es les premières
franges sombres ou les premiers minima, puisqu'elles sont les prem ières franges sombres
à partir de l’axe central. (Les prem ières franges som bres, ou deuxicme.s m inim a, sont
.situées au point où w = 1 et /n = —2 dans l’équaiion 6.16.)

O n a dériv é les é q u a tio n s 6 .1 4 e t 6 .1 6 re la tiv e m e n t à la situ atio n où D 3> </.


T outefois, elles s ’ap p liquent ég alem en t si on p lace une len tille con v erg en te en tre les
fentes et l'écran d'observation et q u ’on rapproche ensuite l'écran d ’t)bscrvation des fente,s,
au fo y er de la len tille. (O n dit alo rs qu e l'é c ra n est dans le plan fo c a l de la len tille,
ou dans le plan perpendiculaire à T axe central au foyer.) U ne lentille convergente a la
propriété de fo ca lise r tous les ray o n s p arallèles au m em e point d an s son plan foeal.
D onc, les rayons qui arrivent m aintenant à n 'im p o rte quel point de l’écran (dans le plan
focal) étaient parfaitem ent (et ntin approxim ativem ent) parallèles lo rsq u 'ils o n t quitte
les lentes. Ils sont com m e les rayons initialem ent parallèles de la figure 5.12 a) qui sont
dirigés en un point (le foyer) p ar une lentille.

^V ÉR IFIEZ VOS CONNAISSANCES 3; )ans la figure 6.8 a), quelles sont A L (en multiple.s
de longueurs d onde) et la diliérencc de phase (en longueurs d'onde) des di:ux rayons
si le point P (situé au-dessus de l’axe central) est a) le inu.sièmc matdmum et h) le troisième
minimum

Exemple 6.2
Sur l'écran C'dc la figure 6.8 a), quelle est la di.stancc entre les maxima Si on établit l’égalité entre les deux expressions de Oet qu’on résout
adjucents situés près du centre de la figure d'interférence? La longueur celle équation pour trouver y„, on détermine que
d'oiiile A tte la lumière est .546 nm, la distance entre les tentes est de nùj)
0 , 12 mm, et celle entre les fentes et l’écran est de 55 cm. Supposez >■« = —p - (6.17)
d
que, dans la figure 6.8, West assez petit pour penneltre l’approximation
sin 0~ tan 0 = 0, où 0 est en radians. Dans le cas du pnx;hain maximum en s’éloignant de l’axe central, on a
(m + l)Â /l
SOtUIlON Tout d ’abord, on prend un maximum oil la valeur de n i Утт1 (6.18)
d
est petite, ce qui le situe donc près du centre de la figure. Le premier
On trouve la distance entre ces deux maxmia adjacents en
concept clé. qui repose sur la di.sposilion géométrique de la figure 6.8 a),
soustiayant l’équation 6 17 de l’équation 6.18:
est que la distance verticale y„ entre le maximum et le centre de la
figure e.st reliée à son angle Oavee l’axe central par AD
3m+l 3»l
d
tan 0 = 0 = — . t_546 X 10" m)(55 x 10 - m)
D
^ 0 ,!2 x 10 m
Le deuxième contepl dé, qui vient de l’équation 6.14. est que cet = 2..50X 10 ’ m s=2.5nun. (répon.se)
angle 0 pour le ènième maximum est donné par
Tant que d et 0 sont petits dans la figure 6.8 a), la distance entre les
nA (ranges d’interférence est indépendante de m ; autrement dit. les franges
sin 0 = 0 = -
sont toutes également espacées.

6.5 La cohérence
Pour q u 'u n e figure d ’in terféren ce piiis.se apparaître su r l'é c ra n d ’o b serv atio n dans la
figure 6 .6 , les onde.s lumineu.ses atteignant un point P quelconque su r l’écran doivent
présenter une différence de phase constante. C ’est le cas dans la figure 6 .6 , puistiuc les
ondes qui traversent les fentes .Ç, et '*<'ht des parties de la m êm e onde lum ineuse qui
é c la ire les fentes. P u isq u e la d iffé ren ce d e p h ase d em eu re co n stan te, on d it qu e la
lum ière issue des lentes .S, et ,S'2 est parfaitem ent co h é ren te .
6.6 L’intensité de la figure d'interférence produite par deux fentes 155

La lum ière directe du soleil est partiellem en t co h éren te ; autrem ent dit, les ondes
lum ineuses provenant de la lum ière du soleil interceptées à d eux p oints ne présentent
une d iffé re n c e d e p h ase c o n stan te q u e si les p o in ts so n t très rap p ro ch e s. Si vous
regardez de près un de vos o ngles à la lum ière du soleil, vous pouvez, voir une faible
figure d ’interférence qui d o nne l'im p ressio n que l’ongle est c o u \e rt de p etites taches.
Vous observez cet effet parce que les ondes lum ineuses issues de points très rapproches
sur l’ongle sont assez cohérentes pour produire d es interférences entre elles lorsqu’elles
atteignent vos yeux. D ans l ’cxpéiience avec deux fentes, toutefois, les fentes n e sont pas
assez rap p ro ch ées et, à la lu m ière du so leil, la lu m ière aux fen te s sera incohérente.
Pour obtenir une lum ière cohérente, il faudrait transm ettre la lum ière du soleil à lravci><
une fente sim p le, co m m e d an s la figure 6 .6 ; co m m e cette uniq u e fente est p etite, la
lum ière qui la traverse est cohérente. D e plus, la petitesse de la fente disperse la lum ière
p ar diffraction et éclaire les deux fentes dans rcx p é ric n ce des deux fentes.
Si on rem place les deux fentes par deux sources de lum ière sim ilaires m ais indépen­
dantes et m onochrom atiques, com m e deux m inces lils incandcscent.s. la différence de
phase entre les ondes ém ises par les sources varie rapidem ent et de m anière aléatoire.
(Cela se produit parce que la lum ière est émi.se p ar un giand nom bre d ’atom es dans les
fils, qui agissent de fa(,on aléatoire e t iinlépcndante, et en des lap.s de tem ps extrêmem ent
courts, de l ’ord re d es nanosecondes.) U en résulte q u 'e n to u t point d onné sur ré c ra n
d ’ob.servation l’interférence entre les deux ondes issues des deux sources vaine rapidem ent
et de laçon aléatoire, allant d ’une interférence constructis c à une interférence destructive
L ’œ il (com m e la plupart d es dctccteur.s o p tiq u es co m m u n s) ne peut su iv re de telles
variations, et aucune figure d ’interférence n ’est alors visible. I e s franges dispnrai.ssenl
et l’écran sem ble uniform ém ent éclaire.
Le laser diffère des sources lum ineuses com m unes en ceci que ses atom es ém ettent
la lum ière d ’une m anière crm rdonnée, rendant ainsi la lum ière co herente. D e plus, la
lum ière ém ise est presque m ouochrom atii]ue. et elle form e un m ince faisceau de faible
d ivergence et peut cire fo calisée à une larg eu r q ui co rresjxind p resq u e à la lo n g u eu r
d ’onde de la lum ière.

6.6 L’Intensité de la figure d ’interférence


produite par deux fentes
L es équations 6.14 et 6.16 indiquent la position des m axim a et des m inim a de la figure
d ’in te rfé re n ce p ro d u ite p ar un sy stèm e de deux fen tes su r l ’écran C de la fig u re 6 .8,
en fonction d e l’an g le 6 q u ’on y trouve. O n veut m aim en an t d ériv er une ex p ressio n
de l ’intensité 1 des franges, en fonction de 6
L a lum ière qui quitte les fentes est en phase. Toutefois, un p eu t supposer qu e les
ondes lum ineuses issues des deux fentes ne sont pas en phase lorsqu'elles allcigncnt le
point P Les com posantes du cham p électn q u c de ces ondes au point P ne sont donc pas
en phase, et leur variation avec le lem ps fx*ul s'éc rire sous la form e

Et ~ E,,, sin cot (6 19)

et ¿2 ~ Sin(W/ + <i>). (6 20)

où (O est la fréquence angulaire des ondes et </> est la constante de pha.se de fo n d e t , -


N otez q u e les d eux o n d es o n t la m em e am p litu d e, et p ré se n te n t un e d iffé ren ce
de phase de é- Puisque la différence de phase ne varie pas. les ondes sont cohérentes.
O n d ém o n trera que ces d eu x o n d es sc co m b in en t à P p o u r p ro d u ire une in te n sité /
donnée par

/ = 4/„cos^ (<i!>/2). ( 6 .21 )

et que

, 2nd
— sm H. ( 6 . 22)
X
156 Chapitre 6 L'interférence

D ans l’équation 6.21, est l ’intensité d e la lu m ière qui atteint l’écran en provenance
d ’une fente alors que l’autre fente est icm ptirairem ent ferm ée. O n suppose que les fentes
sont si é tro ite s co m p arativ em e n t à la lo n g u e u r d ’o nde q u e c e tte in te n sité p ro d u ite
par une seule fente est essentiellem ent uniform e dans la région d e l'é c ra n où on veut
exam iner les franges.
Les équations 6.21 et 6.22, qui indiquent com m ent l'in ten sité / de la figure d ’inter­
férence varie selon l’angle fl de la figure 6 .8, contiennent nécessairem ent de fin lo rm alio n
concernant la position des m axim a et des m inim a. P eut-on extraire celte inform ation et,
ainsi, trouver les équations concernant ces p o sitio n s?
En étudiant l’équation 6.21, on voit que l'in ten sité m axim ale se produit lorsque

é t l = m n, lorsque m = 0 , ± l , c .2, ... (6.23)

Si on insère ce rcsultal dans l’équation 6.22, on déterm ine que

ImTT sin ft lorsque «i = 0 , "^ l, ...


X
ou d sin d = mk, lorsque m = 0 . ± 1. ± 2 .... (les maxima). (6.24)

ce qui revient exactem ent à l'équation 6.14, l’expression q u ’on a précédem m ent det ivec
relativem ent aux positions des maxima.
Les inim ina dans la figure d ’interférence se produisent lorsque

<f)l2 = (m + j ) 7r, lorsque iri = 0, c. 1. ± 2 ....

S i on com bine cette relation avec l’équation 6.22, on arrive directem ent à

d sin f) = (ni t- j)A. lorsque /w = 0 , ± 1, ± 2,... (les numma). (6.2M

qui c o rresp o n d ex a ctem e n t à l ’é q u a tio n 6 .1 6 . l ’ex p re ssio n qu on a p réc éd em m en t


dérivée relativem ent aux positions de.s m inim a.
La figure 6.9. qui est une représentation graphique de l'équation 6.21, illustre l’intensité
de la figure d 'in te rfé re n c e d 'u n sy stèm e à deux fen tes en fo n ctio n d e la d iffé re n c e
de pha.se <j>entre les ondes sur l'écran . L a ligne continue h o ri/o n ta le est l^¡, l'in ic n sitc
(unifom ie) sur l'écran lo rsqu'une des fentes est obstruée. Notez que, dans l’équation 6.21
et dans le graphique, l'intensité / varie de zéro, à la position où l ’inicrférence est destructive
(frange som bre), à 4/(,, à la position où l'in terféren ce est constructive (frange brillante).
.Si les o n d es issu es des d eu x sources (les fentes) sont incohérentes, faisant ainsi
q u 'a u c u n e relatio n de p h ase co n stan te n 'e x is te en tre elles, il n ’y aura pa.s d e fig u re
d 'in te rfé re n c e , ci l ’in te n sité au ra la v aleu r u n ifo rm e 2/(, à n ’im p o rte quel p o in t sur
l’écran ; la ligne pointilléc horizontale de la figure 6.9 représente cette valeur uniform e.
L ’interférence n e peut créer ou détruire l’énergie : elle la répartit différem m ent sur
l ’en sem b le de l ’é cran . L ’in ten sité m oyenne su r l'é c ra n d o it d onc av o ir c e tte m êm e
valeur 2/,). que les sources soient cohérentes ou non. C ela se déduit de l ’équation 6.21 ;
si on insère la valeur m oyenne de la fonction cosinus carré, ccitc équation se réduit
^ Auo) ~ 2/n.

Iiitcn.sité au iiivfau
(te l’c cian 4 (fj (d o u x source:. <ohérciiti-s')

-Z ,5 -Z \S< 1 0.5 0 0.5 1 1,5 2 2.5 AI J k


Figure 6.9 Représentulioii grapluque de l'équation 6.21, illusirant l'intensité d’une figure
d'imcifcrcnce à deux fentes en fonction de la différence de phase entre les ondes qui aUeignent
l'écran en provenance des deux (entes. l„ est l’intensité (uniforme) C(ui apparaîtrait .sur l'écran
si une des fentes était obstruée, L intensité moyenne de la figure d'inte? fércnce est 2/n,
et l’inieiisité maximale (dans le ca.s d'une lumière cohérente) est 4/^.
6.6 L’intensité de la pgure d’interférence produite рог deux fentes 157

La démonstration des équations 6.21 et 6.22


O n com binera les com posantes de cham p électrique £ i et £ 2- dttnnécs par les équations
6.19 et 6.20. en em ployant la m éthode des vecteurs de Fresnel q u ’on a étudiée dans la
.section 2.10. D an s la fig u re 6 .1 0 a), les o n d es ay a n t les co m p o san tes £ , e t £2 so n t
représentées par des vecteurs de Fresnel d 'am p litu d e £,„ qui tournent autour d e F originc
à une vitesse angulaire a». Les valeurs de £ , et £2 sont en tout tem ps les projections de.s
vecteurs de Frcsncl co rresp o n d an ts su r l ’axe vertical. La figure 6,1 0 a) représente les
vecteurs de Frcsncl et leurs projections à un inslanl arbitraire f P ar souci de cohéiencc
au reg ard des é q u a tio n s 6 .1 9 et 6 .2 0 , le v ec te u r de Fre.snel de £ , a un an gle de o>t.
et le vecteur d e Fresnel de £2 a un angle de o>i + 4>.
P o u r co m b in er les co m p o san tes de ch am p £ j e t £2 à n ’im p o rte quel point d e la
figure 6 .8. on lait la .somme vectorielle de leurs vecteurs de Fresnel, tel que l’illustre la
figure 6 . 10 b), l ’am plitude de la somm e vectorielle est l’am plitude £ de Fonde résultante
au point P. et celte onde a une constante de phase fi. Ptxir trouver l’am plilude £ dans la
figure 6.1 0 b), on note d 'a b o rd que les deux angles fi sont égaux parce que le triangle
fo rm é e st un tria n g le iso cèle. Si on se fie au th éo rèm e ( s ’ap p liq u aiil aux tiianglc.s)
voulant q u ’un angle ex térieu r (</> dans ce cas. com m e le m ontre la figure 6 .10 h]) est
égal à la som m e des deux angles intérieurs opposés (celte stimiiie csl ici fi + fi), on voit
que fi = 4)12. On a ainsi

E = 2(E ^ cos fi)


figure 6.10 a) Des vecteurs de Fresnel
rer'ésentant. à l’instant t. les armpo- — 2£,„ cos 4>t2.. (6.26)
santes de champ électiique données
par les équations 6.19 et 6.20. Les deux Si on élève chaque cô té de cette relation au carré, on oblieul
vecteurs de Fresnel ont utx; amplitude
et tournent à une vitesse angulaire <u.
£^ = cos^ <^/2 . (6.27)
Leur différence de phase est <^.
b) La somme vectorielle des deux
vecteurs de Fresnel donne le vecteur O n sait égalem ent, grâce à l’équation 4.24, que l’intensité d ’une onde électrom agné­
de Fresnel représentant Fonde tique est proportionnelle au carré de son amplitude. Donc, le.s ondes que l’on com bine dans
résultante, tlont Fainplilude est E la figure 6.10 b), dont les am plitudes sont E„, ont chacune une intensité proportionnelle
et la constante de phase est fi. à £ ^ , Cl l'o n d e résultante, d ’am plitude £, a une intensité I propo n io n n elle à E ‘^. D onc,

l E-
h E^ '

E n insérant l ’équation 6.27 dans cette équation et en réarrangeant le résultat, on obtient

/ 4/g cos’ <6/ 2.

ce qui correspond à l'éq u atio n 6 .2 1, q ue l’on voulait dém ontrer.


Il reste à faire la dém onstration de l’équatitm 6 .22, qui relie la différence de phnsi </»
entre les ondes qui atteignent n’importe quel point P sur l’écran de la figure 6.8 à l’angle 6
qui sert à localiser ce point.
l a différence de phase <f>dans l’équalion 6 2 0 esl associée à la différence de m arche
S\h de la figure 6.8 b). Si Syb vaut alors ф vaut it ; si S,/? vaut X, il .s'ensuit que 4>vaiK
2тг, et ainsi de suite. C ela m ontre que

2.T
(différence de phase) = — (la différence de marche). (6.28)
A

La d ifférence de m arche .S’,6 dans la fig u re 6,8 b) est sin S , l’éq u atio n 6.28 devient
donc
2m I
4) — ----- sin 6,
X.

ce qui correspond à l ’équation 6.22, qu e l'o n vouhait dém ontrer.


158 Chapitre 6 L’interference

La combinaison de plus de deux ondes


D ’une façon plus générale, on peut avoir besoin de trouver la résultante de plus de deux
ondes variant de façon sinusoïdale à un |w in t particulier. Voici la m éthode à suivre.

1. Tracez un ensemble de vecteurs de Frasnel représentant les ondes de\ ant être combinées.
Tracez-les les uns à la suite des autres, en respectant les relations d e phase appropriées
entre les vecteurs de Fresnel qui .sont adjacents.
2. Ira c c z la somm e vectorielle de cet ensemble de vecteurs de Fresnel. La longueur de cette
som m e vectorielle donne l’am plitude du vecteur de Fresnel résultant. L ’angle form é
entre la som m e vectorielle des vecteurs de Fre.snci et le prem ier %ecteu r de Fresnel
corres|)ond à la phase de la résultante p ar rapport à ce prem ier v ecteu r de Fresnel.
L a p ro jec tio n de la som m e v ec to rielle des v ec te u rs d e F resn el su r l’ax e v ertical
donne la variation tem porelle de l’onde résultante.

Exemple 6.3
Trois ondes lumineuses se combinent i» un certain point, où leurs
composoutes de champ clcctrique sont

- £■„ sin lût,


F, = /:„ sin (fût + Jr/3),
Fy = sin {wt — 7tI6). Figure 6.11 Exemple 6.3 Trois veetcurs de Fresnel, représentant
des onde.s d'amplitudes égales et ayant des phases de 0, de zr/3
Trouve/. Ifiir uimixi.same nisulianle FXn à ce point. et de —jr/6. montrés à l'instant f = 0. Les veetcurs de Fresnel
SCcombinent pour donner un vecteur de Frcsncl résultant
SOlUîtON l.’nitdc résultante est d'amplitiid c El) et d’angle fi par rapport à l’horizontale.

£(/) •• + t,[t) ^ £,U). et a un angle de phase p par rapport au vecteur de Fresnel représen­
tant kl de
(ortept cl® ü utiliser comporte deux volets : on peut employer la / 0£ 6t r ^ \
nicthodc des vecteurs de Frc.snel pttur trouver cette somme, et on P = tan ‘ V 2 . . 3 7 £ , j - 0 115
i rad.
peut évaluer les vecteurs de Fresnel à tout instant t Ptuir simplifier
l’opération, on choisit t = 0 . en fonction duquel les vecteurs de On peut maintenant écrire fonde résultante k(t) sous la forme
Fresnel représentant les trois ondes sont illustrés dans la ligure 6. I l .
£ = £ r sintoi/ P)
On peut additionner ces trois vecteurs directement à Laide d’une cal-
culamcc comportant les toiKiions vectorielles, ou procéder en prenant = 2,4£^ sin(car r- 0,15). (réponse)
une composante à la fois. Si on choisit celle dcmicrc approche, on écrit
Prenez soin de bien interpréter l’angle p dans lu figure 6 .11 : c ’est
d’abord la .somme des composantes horizontales des vecteurs sous la
l'angle consUint entre £ r et le vecteur de Fre.snel représentant £ ,.
forme
alors que les quatre vecteurs de Fresnel tournent ensemble autour
de l'origine. L'angle entre £ r cl l'axe horizontal de la figure 6 . 11
S A’h = P-m w s 0 + cos jr/,3 ^ cos(-7r/6) = 2..37fc„.
ne reste pas égal à p.
I .a somiTKi de leurs composantes verticales, qui correspond Ala valeur
de fî à / = n, esl ✓ VÉRIFIEZ VOS CONNAISSANCES 4 : Quatre p .irc s d ’ ondes
I lumineuses arrivent à un certain p>ini sur un écran. Ixs ondes ont
^ A'v = £„, sin 0 : sin 7t/3 t Fn, sin{ -a-/6) = 0,366£„. I la même Irmgucur d’onde. Au point où elle.s arrivent, leurs aniplitudes
et leurs différences de phase sont a) 2£„, 6£ „ et tt rad. b) 3/;„„
L’tmdc rcsulloiiic E(r) a donc une amplitude /Tr de 5£„, et TT rad. c) ')kn. Tt-m c' tad, d) 2£„, 2£„ et 0 rad. Clas.se/
les quatre paires en ordre décroissant selon l'intensité de la liimicre
Ph = n/ ( 2 , . 3 7 £ J 2 + ( 0 .3 6 6 £ „ ) 2 - 2Ak^, à ce point. [Indice : Trace/ des vecteurs de I resncl.)

6.7 L’Interférence produite


par les pellicules minces
I a' s couleurs que Fnii voit Inr.sqne la lumière du soleil éclaire une huile d ’eau snvoniK'use
ou une tache d ’huilc sont pnxluitcs par l’interférence des ondes lum ineuses réfléchies par
les surfaces avant et arrière d 'u n e m ince pellicule transparente. L’épaisseui de la pellicule
de .savon ou d'huile e.st norm alem ent de l’ordrc tie la longueur d ’onde de la lumière (visible)
6 n cause (D e plus grandes épaisseurs nuisent à la cohérence de la lum ière, requise poiii
priHluire les couleurs.)
6.7 L’interference produite por les pellicules minces 159

figure 6.12 Des ondes lunüncuscs, représentées par le rayon i. sont incidentes à une pellicule mince
d’épai.s.seur L et dont l’indice de réfraction est n,. Les rayons r, et r2 représentent respectivement
les ondes lumineuses qui ont été réfléchies par la surface avant et la surface arrière de la pellicule.
(Les trois rayons sont presque perpendiculaires à la |.)cliicule.) L’interférence que les ondes de r,
et rj prodiii.sent entre clics dépend de leur différence de phase. L’indice de réfracium n, du milieu
à la gauche peut différer de l’indice de réfraction du milieu à la droite, mais on suppo.se, pour
l’instant, que ces deux milieux sont l’air, el que n, = n, = 1,0, qui est plus petit que ni-

L a figure 6 .12 reprc.senlc une pellicule m ince transparente d ’épaisseur uniform e L


et dont l’indice de réfraction e.st « 2>éclairée par une lumière intense de longueur d ’onde A
provenant d ’une source |X)nctucllo éloignée. Pour l'in stan t, on suppose que la peiliculc
est entourée d ’air cl que ti, = tlans la figure 6.12. P our simplifier, on .suppose égalem ent
que les rayons lum ineux .sont pre.squc perpendiculaires à la pellicule 0). O n veut
savoir si la pellicule est brillante ou som bre pour un observateur qui la regarde de façon
presque perpendiculaire. (Pui.sqiie la pellicule est fortement éclairée, com m ent pourrait-elle
être som bre ? C ’est ce q u 'o n verra.)
L a lu m ière in cid en te, rep rése n tée par le ray o n i. Irap p e la .surtace g au c h e tie la
pellicule au point a, et subii à la fois une réllexion ci une réfraction à ce point. Le rayon
réfléch i r, esl in te rcep té p ar l ’reil d e l’o b serv ateu r. L a lu m ière ré fra c té e trav e rse la
pellicule au point h de la surface de droite, ofi elle subit à son tour une réflexion e t une
réfraction. 1.a lum ière réfléchie à h traverse de nouveau la pellicule en se dirigeant vers
le point t , où elle subit de nouveau une réflexion et une réfractittn. La lum ière réfractée
à c, représentée p a r le rayon c.st interceptée par l'œ il de ro b serv ateu r.
Si les ondes lum ineuses des rayons / 1 et /-2 so n t parfaitem ent en phase en atteignant
l’œ il, elles produisent aloirt une interférence constructive, et la région ac de la pellicule
apparaît brillante à l’observateur. Si elles sont déphasées de n rad, elles produisent une
interférence destructive, et la région a c apparaît alors som bre à l'observateur, m êvw si
elle est éclairée. D ans le cas d ’une différence de phase interm ediaire, il se produit une
interférence interm édiaire, e t on observe une brillance interm ediaire.
La clé, dans ce qui apparaît à l’observaieut, est donc la d ifférence de phase entre
les ondes d es rayons T| et rj. L es deux rayons sont d ériv es du m êm e rayon i, m ais le
parcours suivi par le rayon fait que la lum ière traverse la pellicule d eux foi.s <de a à b
e t, en su ite , de à c), alo rs q u e, dans 1»î p arco u rs du rayon r ,, la lu m ière 11c trav erse
pas la pellicule. P uisque la v aleu r de Wtend vers zéro, on fait l’approxim ation qu e la
différence de m arche cnlre les ondes de r¡ el r¿ e.st égale à 2L. Toutefois, pour ü ouvei la
différence de phase en tre les ondes, il ne su ffit pas d e trouver le nom bre de longucuns
Iiiu-it'acc d ’onde >- équivalant à une différence d e m arche de 2L. C ette approche est incom plète
pour deux raisons ; 1) la différence de m arche sc produit dans un milieu autre que l’an,
et 2 ) des réflexions se prndui.sent, ce qui peut faire v an er la phase.
a)
La différence de phase entre deux ondes peut varier si une des ondes ou les deux ondes
.sont létlcchies.
Avant

Avant de poursuivre cette étude .sur l ’interférence pro d u ite p ar des pellicules m ilices,
on doit parler des variations de phase causées par les réflexion.s

Figure 6.13 I «s clianpem ent.\ de pha.«* Les déphasages ô la réflexion


q u i 'îu r v ic n n c n l lo r s q u ’u ne im p u ls io n
l a réfiaction à une interface ne cause jam ais de déphasage - mais la réllexion peut en
est ré flé c h ie à l ’inte rface entre d e u x
causer, en fonction des indices d e réfraction des deux côtés de l’interface La figure ô. 1.3
co rd e s tendues dont les m a sse s lin é iq u e s
représente ce q ui se produit lorsque la réflexion cause un déphasage, en donnant com m e
sont dilfé re n te s. L a v ite sse de l’tinde
est p lu s g ra n d e d a n s la co rde ayant
exem ples des impuLsions dans une corde ayant une forte m asse linéique (dans laquelle
une m a sse liné iq u e p lu s faible, le parcours de l ’impul.sion est rela tiv e m en t len t) et d an s une co rd e ay an t un e m asse
a i L ’im p u ls io n in cid e n te est d a n s linéique m oindre (dans laquelle le parcours de l'im pulsion est relativem ent rapide).
ia c o rd e ayant u n e p lu s g ra n d e m asse I .orsqti’une im pulsion se propageant dans la corde ayant une grande m asse linéique
jincique. h ) L ’im p u ls io n incidente de la figure à. 1.3 a) atteint l'in terface entre cette corde et la corde ayant une faible m asse
est d a n s la c o r d c aya n t u n e p lu s faib le linéique, l’im pulsion est particlleinent transm ise et partiellem ent rétléchie, sans changer
in a sse linéique. C ’est s o u lc m e in d'orientation. D ans le cas de la lum ière, cette situation co n esp o n d à l’onde incidente se
là q u 'i l y a u n ch a n g e m e n t d e phase, propageant dans le m ilieu d ’indice d e infraction « plu.s élevé (so u v en ez-v o u s q u ’une
Cl seulement rians l’onde réfléchie. plus grande valeur de n suppose une n u 'in s grande viie.sse). Ltans ce cas, l’onde qui est
160 Chapitre 6 L'interférence

réfléchie à l’interface n e subit pas d e déphasage ; en d ’autres mots, son déphasage à la


réflexion est nul.
Lorsqu’une impulsion se propageant dans la corde ayant une faible mas.se linéique de
la figure 6.13 b) atteint l’interface entre cette corde et la corde ayant une grim dc m asse
linéique, l’impul-sion est de nouveau piuliellem ciit triuism ise et partiellem ent rélléchic.
L’im pulsion transm ise de nouveau a la m êm e orientation que l’im pulsion incidente, mais
l’impulsion réfléchie est m aintenant inversée. Dans le cas d ’une onde sinuso'tdale, une telle
inversion suppose un déphasage de n rad. ou d ’une dem i-longueur d ’onde. D ans le cas
de la lumière, ccItc situation correspond à l’onde incidente se propageant dans le milieu où
l ’indice de réfraction est m oins élevé (donc à une plus grande vitesse). D ans ce cas, l’onde
qui est réfléchie à l’inlct face subit un déphasage de tt rad, ou d’une dem i-longueur d ’onde.
En ce qui concerne la lum ière, on peut résum er ces résultats et les exprim er à l’aide
de riru lice de réfraction du m ilieu où la lum ière est réfléchie.

Réflexion Déphasage
Indice moins élevé 0
Indice plus élevé 2 longueur d’onde

Cet aide-m ém oire p eu t être utile : « P lu s élevé égale dem i-longueur d ’onde. »

Les équations de l’interférence dans les pellicules minces


Ju sq u ’à m a in te n an t, on a vu d an s ce ch a p itre que la d iffé ren ce d e pha.se en tre deux
ondes pouvait changer de trois façons :

1. par réflexion ;
2 . lorsque les ondes suivent des parcours de différentes lo n g u e u rs.
3. lorsque les ondes se propagent d aas des m ilieux d ont les indicc.s de réfraction sont
différents.

Lorsque la lum ière est réfléchie sur une pellicule m ince, produisant les ondes de rayons
/•| et rn illustrés dans la figure 6 .12. ces trois façons sont en cause. O n les analysera ici
une à la fois.
O n réexam ine d ’abord les deux réflexion.s illustrées dans la figure 6.12. Au point u
de l’interface avant, fo n d e incidente (dans l’air) e st réflcehic par le m ilieu ayant le plus
élevé des deux indices d e réfraction, et f o n d e du rayon réfléchi »i est donc déphasée
de i longueur d ’onde. Au point b de l’interface arrière, fo n d e incidente est réfléchie pai'
le m ilieu (l’air) ayant le m oins élevé des deux indices de réfraction, et fo n d e réficchie
à ce point n’e.sl pas déphasée par la réflexion, pas plus que la portion de fo n d e qui sort
de la pellicule et qui est représentée par le rayon r,. On peut structurer cette inform ation
à l’aide de la prem ière ligne du tableau 6 .1. On y indique que le résultat rlu déphasage
p ar la réflex io n e s t qu e les o n d es de r , e t r , so n t d ép h a sé es de | lo n g u e u r d ’on d e,
cl q u ’elles sont donc en opposition de phase.
O n d oit m aintenant co n sid érer la d ifférence d e m arche 2L qui intervient puisque
fo n d e du rayon /s traverse la pellicule deux fois. (C ette différence de IL est indiquée à
la deuxièm e ligne du tableau 6.1.) Si les ondes de et rj doivent être parfaitem ent en
phase afin de prorluire u ne interférence constructive, la différence de m arche 2/ devrait
causer un déphasage supplém entaire de O,.*). 1.5,2„S,... longueurs d'onde. C ’est seulem ent
dans ce cas (|ue le iléphasage net (incluant le déphasage dû à la réflexion) sera un nombre
en tier de longueurs d ’ondc. Il s ’en su it donc que, dans le cas d ’une p ellicu le b rillan te
(interférence constructive), on doit avoir
nom bre im pair
21. 2 — X longueur d ’onde (les ondes en phase). (6.29)

La longueur d ’ondc q u ’on doit avoir ici est la longueur d ’onde À„ de la lum ière, dans le
m ilieu com prenant la longueur de parcours 2L - c ’est-à-dire dans le m ilieu dont l’indice
di' réfraction est n^- D onc, on peut récrire l’équation 6.29 ainsi :
nom bre im pair
2L — — ----------- y A„. ties ondes en phase). (6.30)
6.7 L'interférence produite par les pellicules minces 161

TABLiAl) 6.1 Interférence dans l'air’ Si les ondes sont plutôt en opposition de pha.sc e t qu’il y a une interférence desu-ucüvc,
cousée par une pelikuie m’tnce la d ifférence de m arche 2L ne d ev rait cau ser aucun déphasage supplém entaire, ou un
déphasage de l. 2 ,3 .... longueurs d ’ondc. C ’est seulem ent dans ce cas que le déphasage
ri ri. net sera un no m bre im p air de d em i-lo n g u eu rs d ’onde. R elativ em en t à urte p ellicu le
Déphasages | longueur 0 som bre (irtterfcrence destructive), on doit donc avoir
par réllexion d’onde
2L = nom bre entier x longueur d 'o n d e. (6.31)
Différence
de marche IL
où, e n c o re un e fois, la lo n g u eu r d ’o n d e est la lo n g u eu r d ’o n d e „, d an s le m ilieu
Indice com prenant 2/.. On a donc, cette fois,
de réfraction
du milieu 2L = nom bre entier x k„ (les ondes déphasées). (6.32)
ivii la différence W?
de marche O n peut m aintenant utiliser l ’équation 6.8 (X„ — k/n) pour d onner à la longueur d ’onde
survient de l’onde d u rayon »s dans la pellicule la form e
nombre impair Â
En phase* : 21 (6.33)
2 «2
Hz
Déphasé* : IL - гюшЬге eniier x où À est la longueur d ’onde de la lum ière incidente dans le v ide (et. approxiinativeincnt.
n?
égalem ent dans l'air). L n insérant l’équation 6.33 dan.s l’équation 6.30 et en rem plaçant
* Valkte lorsquc > n, et »Tj > и,. « nom bre im pair/2 » par (m + 1 ), on obtient

2L = (m + i ) — , lorsque w = 0, 1 ,2 ,...
«2
(les maxima - la pellicule brillante dans l'air). (6 3d)

D e la m em e façon, si on rem place « n o m b re en tier» p ar/n , l’équation 6.32 donne

2/, — m — , lorsque ni = 0 . 1, 2, ...


»2
(les minima - la pellicule sombre dans l'air). (б..!.*»)

R e la tiv em en t à une ép a isse u r d o n n ée d e p ellic u le Z, les éqiialinn.s 6 .3 4 e t 6.35


indiquent les longueurs d ’ondc de la lum ière en fonction desquelles la pellicule apparaîi
respectivem ent brillante et som bre. Il y a une longueur d ’ondc qui correspond à chaque
valeur de m. Les longueurs d ’onde interm édiaiies donnent des hrillancc.s interm édiaires.
R elativem ent à une longueur d ’ondc donnée X, les équations 6.34 el 6.35 indiquent les
épaisseu rs des p ellicu les qui ap p araissen t resp ectiv em en t b rillan tes e t som bre? dans
cette lum ière, et il y a une ép aisseu r d onnée pour chaque \ alcu r de m. i.cs épaisseurs
interm édiaires donnent des brillances interm édiaires.
L’ne situation particu lière se p ro d u it lo rsq u 'u n e p ellicu le est très m inee et que L
est très inferieure à X (L < O.IX, par exem ple). 1 ^ différence de m arche 2L peut alors
être n ég lig ée , cl le d ép h asag e e n tre r, et r i n ’e st dû q u ’au d ép h asag e cau se p a r la
réflex io n . Si la p ellic u le d e la lig u re 6 .1 2 . où les réflex io n s ca u sen t un d ép h a sa g e
de ■ lo n g u e u r d 'o n d e , a une ép a isse u r l < O.IX, / [ el /y sont alors en o p p o sitio n d e
phase, et la p ellic u le e st d o n c so m b re, peu im p o rte la lo n g u eu r d ’o n d e cl m êm e
l'in te n sité de la lum ière qui l'éc laire . C ette situ atio n particulière corresp o n d à rn — 0
dans l ’équatio n 6.3.5. (3n co n sid érera (oiite ép a isse u r L < O.IX com m e étan t la plus
petite épaisseu r spécifiée par l’équation 6.35 en fonction de laquelle la pellicule de la
figure 6.1 2 ap p a raît so m b re. (T oute é p a isse u r se m b la b le co iresp o n clra à ni — 0.)
L 'é p a isse u r su iv a n te en fo n ctio n d e la q u elle la p e llic u le p ara îtra so m b re est celle
correspondant à w = 1.
L a figure 6 .14 m ontre une p ellic u le «l’eau sav o n n eu se leniic il la v citic ale dont
l’épaisseur augm ente en allant vers le bas. à cause de l ’affai.ssement de la pellicule causé
par la gravité. U ne lum ière blanche ei brillante échiire la pellicule. Toutefois, la partie
supérieure est si m ince q u 'elle est som bre. D ans la partie centrale (un peu plius epais.se)
de la pellicule, on aperçoit des franges, ou des bandes, dont la couleur dépend surtout de
la longueur d 'o n d e à laquelle la lum ière réfléchie subit une interfcrence constructive en
fo n ctio n d ’u n e é p a isse u r p articu liè re. P,n allan t vers le b as, on la p ellic u le esi plus
épaisse, les franges deviennent progressivem ent plus étroites, et les couleurs eom m en
cent à se ch cv au th ei cl à pâlit.
162 Chapitre 6 L'Interférence

figure 6.14 La réflexion de la lumière blanche sur une mince pellicule d'eau savonneuse
suspendue dans les airs à la verticale La portion supérieure de la pellicule est si mince que
la lumière réfléchie produit de rinteifércncc destructive (portion sombre de la pellicule).
Avec l'aiigmcntation de l'épaisseur de la pellicule due à la gravité, une série de franges colorées
apparaît (interference constructive).

L'irisation d ’une aile de papillon morpho


On dit q u ’une surface dont les couleurs sont pnxluites par une interférence causée par
une p ellicu le m in ce est irisée, p arce qu e les co u leu rs o b se rv é e s ch an g en t lo rs q u ’on
m odifie .son point de vue p a r rapport à la su rface. L 'irisatio n o b serv ée su r la surface
supérieure d 'u n e aile de papillon m orpho est du e à l’interférence de la lum ière qui est
réfléchie par de m inces rangées d ’une substance transparente q u ’on trouve sur l’aile, et
qui rc ssc m b le m à des cu ticu les. C es rtuigécs so n t disposée.s à la m an ière d e larges
branches d ’une structure arborescente, qui s'éten d en t perpendiculairem ent à l'aile
S uppose/ que vous regardez ces rangées du haut alors q u 'u n e lumière blanche éclaire
l’aile dircx-iement. La lum ière réfléchie vers vous en provenance de ces rangées produit
une interférence co n siru c iiv e dans la région bleu-vert du sp ectre visible. La lum ière
com prise dans les régions jau n e ou rouge, à l'au tre extrém ité du spectre, est plus faible
parce q u 'elle produit une interférence interm édiaire. La surface supérieure de l’aile vous
apparaît tlonc d ’une couleur bleu-vert.
I jrrsinic vous interceptez la lumière qui est réfléchie par l’aile dans une autre direction,
cette lum ière a suivi un p ara m rs oblique en traversanl les rangées. La longueur d 'o n d e à
laquelle il y a une interférence constructive est donc différente de celle qui prévalait lorsque
la lumière a été réfléchie directem enl vers le haut. Donc, si l’aile bouge dans votre cham p
de vision de m anière à changer l'an g le auquel vous la voyez, la couleur au m om ent où
l'a ilc est la plus brillanlc change quelque |ieu, ce qui produit l’irisation de l’aile.

CM^SOt.UTION DE i"ip06u€MES

1" ilrotégie: Les équations des /wllk ule.s minces


Certains élèves pensent que l’cquatioii 6..'14 donne les maxima et que
^ VÉRIFIEZ VOS CONNAISSANCES 5: Le diagramme ci-dessous
représente qu.atrc siluations où la lumière est réfléchie perpendiculai­
l'équation b. t.S cloiino les minima de toutes les situations où il y a une rement sur une pellicule mince d’épaisseur /. (comme dans
(idlicule mince. Cela c.si inexact. Ces relations n’ont etc dérivées que la figure 6.12) : les indices de réfraction sont indiques, a) Dans quelles
pour la siltiation où «. «| et dans la figure 6 .12. situations la réflexion par les interfaces de la pellicule causera-l-elle
Vous pouvez dériver le.s équations appropriées luix autres un déphasage nul entre les deux rayons réfléchis’.' b) Dans quelles
valeurs relatives des indices de réfraction en suivant le raisonnement situations la pellicule sera-t-elle sombre, si In différence de marche IL
expliqué clans celte section et en construisant de nouvelles version.s cause un déphasage de A longueur d ’onde ?
du tableau b. 1. Dans chaque cas, vous arriverez aux équations
6.34 cl 6.35. mais l'équation 6.34 donnera parfois les minima et I
réquatioii 6.35 donnera parfois les maxima - le contraire de ce qu’on 1.'4 1.3
a vu ici. La situation décrite par une équation donnée est tribuinire du l.-l
fait que les réflexions aux deux iiUcriaces pixxluironi ou non le même iS
déplia.sage.
3) 4)

Exemple 6.4
Une lumière blnnche, ayant une intensité uiiiforine ptxir l’ensemble Pour déterminer laquelle des équations est la bonne, on pourrait
du spectre visible allant de 4(K> nm à 690 nm. arrive à incidence remplir un tableau conune le tableau 6.1. fiiulelois, puisqu'il y a de
normale sut une iicilicule d'eau suspendue dans les airs cl dont l’air des deux côté.s de la pellicule d’eau, la situation dans ce cas est
l'indice de réfraciion est n, = 1.33 et l'cp.'tisseiir est l. = .320 nm exactemeni semblable à celle de la figure 6 . 12. et le tableau serait
À quelle lonpiionrd'ondc / , relativement à un observateur, la lumière exactement identique au tableau 6.1. D’après ce tableau, on voit que
esi cllc réfléchie avec la pins giamle brillance sur la pellicule ? les rayons réfléchis sont en pha.\c (et que la pellicule est donc plus
brillante) lorsque
SOiUTION. Le tomiiil clé. dan.s ce cas. cw que la lumière réfléi hie sur la
liellicule est la plus brillante niix longueurs d ’onde X auxquelles les . nombre impair X
IL ~ ---------------— X
rayons rcflcchis sont en phase. L’équation reliant ces longueurs » 2 ni
d’onde À à l’épaisseur donnée L et h l’indice de léfraclion wj peut être ce qui mène à l'équation 6..34 ;
l’éqiiatinn 6.34 ou 6..35. scion le déphasage par réflexion observé
dans celle pellicule. ?J. - (m + ^)
t>2
6.7 L’interférence produite por les pellicules minces 163

Hn résolv.int cetie équaiion pour trouver л et en insérant L et n,. k — 567 nm, ce qui correspond à la lumière jaune-vcrtc située dans
on déteimine que la partie centrale du spectre visible. Quand /n = 2, à = 340 nm.
2ri2l- (2)(l,33)(320nm ) 851 nm ce qui correspond à la région ultraviolette. Il s’ensuit que la longueur
к =
m + ni + m + d'onde à laquelle la lumière vue par un observateur sera la plus
brillante est
Lorsque m = l). cela donne À = 1 700 nrn, ce qui correspond à
la région infrarouge du spectre. Dans le cas où m = I. on trou\ e к = 567 nm. (réponse)

Exemple 6.5
Dans la figure 6.15, une des faces d’une lentille de verre est enduite
d ’une mince pellicule de fluorure de magnésium (MgFj) afin de
réduire la réflexion à la surface de la lentille. L'indice de réficvctiou
du MgFj est 1,38; celui du verre est 1.50. Quelle est la plus petite
épaisseur d'enduit qui élimine (par interférence) les réflexions au
centre du spectre visible (A = 550 nm )? Supposez que la lumière
est approximativement perpendiculaire à la surface de la lentille.
SOlimON Le '.oncept clé utilisé ici est le suivant : la réflexion est éliminée
si l'épaisseur I de la pellicule est telle que les ondes lumineuses
réfléchies par les deux interfaces sont en opposition de phase.
L’équation reliant i. à la longueur d ’onde donnée A et à I'indice de
refraction rti de la pellicule mince est soit l’équation 6..34, .soit l'équa­
tion 6.35, selon les déphasages causés par la réflexion aux interfaces. Figure 6.15 Exemple 6.5 i)n peut supprimer les réflexions du verre
Four déterminer quelle est l'éqtation appropriée, on templit un (à une longueur d'onde sélectionnée) en recouvrant le verre
tableau semblable au tableau 6.1. À la première interface, la lumière d’une mince pellicule transparente de lltioriire de magnésium
incidente est dans l'air, qui a un indice de réfraction moins élevé d'une épaisseur appropriée.
que le lluorurc de magnésium (la pellicule mince). Donc, on inscrit
î longueur d’onde sous r, dans le tableau (ce qui signitic que les ondes Cela mène h l’éqoaiion 6.34. En lésolvani celle équation pixir tnxiver L.
du rayon r, sout déphasées de sA à la première interface). À la on obtient al(H\ l'épaisseur île la pellicule qui éliminera In icllexion
deuxième interface, la lumière incidente est dans le fluorure de de la lentille et de l'enduit ;
magnésium, qui a un indice de réfraction moins élevé que le verre
de l’autre côté de rinterl’acc. Donc, on inscrit ÿ longucui d ’onde sous rj L = fm -h i ) r — , lorsque m “ 0, 1,2,... (6.36)
' 2«2
dans le tableau.
Puisque les deux reflexions produisent le meme déphasage, On veut trouver la plus petite épaisseur de l’euduit - c ’esi-J-dire la
clics tendent à mettre en phase les ondes de r^ et de ri- Comme on plus petite valeur de L. On chtiisil donc m ~ 0, la plus petite valeur
veut que cc.s ondes soient déphasévx. leur différence de marche IL possible de m Fii inscraiii celte valeur cl les données eonnue,s dans
doit être un nombre impair de demi-longueurs d’tmdc ; réquaiion 6.36. on obtient
nombre impair _ A 550 nm
2L = » W .6 nm. (réponse)
~ 4 ^ ^ (4)(l,38)

Exemple 66 Lumicre imidt-ntc

La ligure 6.16 al représente un bloc de plastique transparent comportant


une lircclie remplie d’ait. à droite. (L’épaisseur de la brèche est exagé­
i 1i I l
rée dans la figure.) lin large faisceau de lumière rouge, de longueur
d’onde A = 632,8 nm. est dirigé vers le bas à travers le dessas du bloc
(à un angle d’incidence de 0“). Une partie de la lumière est réfléchie
w rs le haut aux limites supéiieiire et inferieure de la brèche, qui agit
comme une mince pellicule (d’air) dont l’épaisseur varie uiiitonnémcni
et graduellement de Le, à l’extrémité gauche à à l’extrémité droite.
(Lc.s couches de plastique au-dessus et en dessous de la brèche
remplie d ’air sont trop épaisses pour agir comme une pellicule mince.)
L’n observ ateur regardant vers le ha.s en direction du bloc voit une
figure d’interférence compo.sée de six franges sombres et de cinq
tranges brillantes rouges le long de la brèche. Quelle est la variation
figure 6.16 Exemple 6.6 a) Une lomière rouge incideme tombe
de l’épaisseur AI, (■ = /.„ - ¿c,) le long de la brèclie?
sur une mince brèche remplie d’air dans le côté d'un bloc
SOÜJTION■Dans ce cas. un premier totitepl dé est que la brillance, en tout de plastique iransparcm. L’épai.sseur de la brèche c.st Lr. 6 rexirémiié
point de la longueur, de gauche à droite de la brèche, est causée gauche et Lj, 5 l’extreiiiilé droite, b) Une vue de haut du bloc ;
par l'im erfércncc des ondes réfléchies aux interfaces supérieure une figure d’interférence de six frangc-s .sombres et de cinq franges
et inférieure de la brèche. Un .secomi concept de c.si que la variation brillantes de lumière rouge se forme aii-ilessiis de la région
de brillance dans la figure d ’interférence produisant les franges de la brèche, c) Une représentation du rayon incident i. des rayons
Ihnfianles cl sombres est causée par la variation de l épaisscur de réfléchis r, cl rj. Cl de répai,s.scur L de la brèche.
IM Chapitre 6 L'interference

la brèche. Dans cenaiiies régions, l’épaisseur iix;t les ondes réfléchies ce qui mène à l’équation 6.35 :
en pha.se et produit donc une rcHcxion brillante (une frange rouge
et brillante). Dans d'autres régions, l'épaisseur a pour effet de mettre
11. = m — , lorsque »n = 0, 1, 2,... (6.37)
les ondt's réfléchies en opposition de phase et de ne produire aucune «2
réllexion (frange sombre).
Puisque robservaieiir voit davantage de franges sombres que On a ici un autre concept clé : l’équation 6.37 est vérifiée non seule­
de franges brillantes, on peut supposer qu'une frange sombre est ment par rappon à l’extrémité gauche de la brèche, mais egalement
pnxluitc à cliaque extrémité de la brèche, où les réflexions produisent en tout point le long de la brèche où, l'on observe une frange sombre,
une frange sombre. incluant l'extrémité droite - avec une valeur entière de m differente
Comme l’observateur voit plus de franges sombres que de franges pour chaque frange. La plus petite valeur de m est asscxriéc à la plus
brillantes, on doit conclure que la figure d ’interférenee débute par petite épaisseur de la brèche où l'on observe une frange sombre.
une frange sombre à gauche et se termine par une frange sombre à Des valeurs progressivement plus grandes de m sont associées à
l ’extrémité droite de la brèche. La figure 6.16 b) illustre la figure des épaisseurs progressivement plus grandes de la brèche où Гоп
d’intcrlcrcnce obtenue, laquelle va nous permettre de déterminer la obseï ve une frange .sombre. Soit la valeur à I extrémité gauche.
variation île l’épaisseur le long de cette brèche. La valeur à rextrémitc droite ckiit alors être ntc, -r 5 puisque, d'après
On sait que, pour obtenir une frange sombre, les ondes des la figure 6 16 h), l’extrémité droite est située à la cinquième frange
rayons /1 et r, de la figure 6.16 c) doivent être en opposition de phase. sombic à partir de l’extrémité gauche.
On sait egalement que l'équation reliant l'cpaisseur L de la pellicule On veut trouver la variation de l’épaisseur AL entre rexttémité
à la longueur d ’onde /. de la lumière et à i‘indice de réfraction t/j gauche et rextrémitc droite de la brèche. On doit d’abord résoudre
du l,T pellicule e,sl soit l'équalion 6..14, soit l'équation 6 35, selon deux fois l’équation 6.37 - une fois pour l’épaisscui Lq à l’extrémité
les déphasages allribtiablcs à la léflexion. Pîmii déteniiiner laquelle dc.s gauche, et une autre fois pour l’épaisseur i,) à l’extrémité droite :
équations donne une frange sombre à rextremite gauche de la brèche,
on remplit un tableau semblable au tableau 6. 1. A A
Lç. = (tîio) 2w ¿0 = (»V, + 5)—■ . (ЬМ)
A r Intel face .supérieure de la brèche, la lumière incidente est 1П2 1П2
dans Je plastique, qui a un plus grand indice de réfraction que l'air en
dessous de celte interlace. Donc, on inscrit 0 soirs t \ dans le tableau. Pour m niver la variation de l’épaisseur A/., on peut maintenant
À l’intcrfacc inférieure de la broche, la lumière incidente est dans l’air, .soustraire L,, de L,, et insérer les valeurs connues, incluant n, = 1,00
qui a un indice de réfraction moins élevé que le plastique au-dessus relativement à l’air dans la brèche ;
de cette interlace. Donc, on inscrit i longueur d'onde sous a dans le
tableau. Ainsi, les réflexions tendent il déphaser les ondes de r,
(«1(5 -I- 5)A ПЦ- / 5 A
et de A- Af. — Lu f<i —
Puisque, en fait, les ondes sont déphasées à l’exlrémité gauche 2«2 2«2 2 «2
de la brèche remplie d’air, la différence de marche 2L à l'extrémité 5 6.32.« X l()-^n i
de la brèche doit cire donnée par “ 2 Ш)
A — 1.58 X 10“ ^ m. (réponse)
2L = nombre entier x — ,
t¡2

6.8 L’interféromètre de MIchelson


I In intcrf'éromètre est un instrum ent utilisé pour m esurer les longueurs ou les variaiioiis
de lo n g u e u r avec u n e g ran d e p réc isio n à l’aid e d es fra n g es d 'in te rfé re n c e . V oici la
dcscriplion du m odèle original m is au point et fabriqué par A lbert A. M ichclson en 1881.
F xam inez la lum ière qui quitte le point P de la source étendue S de la figure 6.17
cl qui rencontre le diviseur de faisceau M. t 'n d iv iseu r de faisceau est un m iro ir qui
transm et la m oitié de la lum ière incidente et réfléchit l'au tre moitié. D ans la ligure, on a
supposé, pout simplifier le problème, que ce m iroir a une épais.seui négligeable. La lumière
doit donc .se divi.ser en deux fai.sceaux à M. L 'u n d'eu x csl transm is vers le m iroir /W, ;
l'a u tre est réfléch i vers le m iro ir M 2 . L es o n d es .sont en tièrem en t rc tté c h ie s p a r ces
miroirs, et sont renvoyées vers leurs directions d'incidence, et chaque onde finit sa course
au télescope T. C e q u e l'o b se rv a te u r voit est une fig u re d 'in te rfé re n c e com p o sée de
franges courbes ou approxim ativem ent droites : dans ce cas. les frangeas ressem blent aux
rayures d 'u n zèbre.
1 orsquo les d eux o n d es se réu n issen t au télescope, leur d ifférence de m arche est
— 2f /|. et toute vai iation d e cette fliffércnce de m arche causera un changem ent île la
différence de phase entre les deux ondes, là où se trouve I'ceil. Par exemple, si le m iroir M 2
est déplacé d 'u n e distance p ,, la différence de m arche viuie de A et la figure irin teiféren ce
c sl décalée d 'u n e frange (com m e si chaque rayure soiVibre d 'u n zèbre prenait la place de
la rayure som bre voisine). D e la m êm e façon, si le m iroir M-^ est déplacé de ]A, la liguie
est décalée d ime d em i-frange (chaque rayure som bre d ’un zèbre prend la place d e la
rayure blanche voisine).
Révision et resume 165

M iroir U n décalage dans la figure d ’interférence peut aussi être causé p ar l’iasertio n d ’un
m ince m atériau transparent dans le parcours o p tiq u e d ’un des m iroirs, p ar exem p le le
m iroir A/|. Si ce m atériau a une ép aisseu r L et un indice d e réfraction n, le nom bre de
lotigucurs d ’onde dans le parcours allcr-rctour de la lum ière dans le m atériau est. d ’après
l'équation 6.9,
2L 2Ln
(6..W)

Le nom bic de longueurs d ’onde dans la m êm e épaisseur 2L d ’air avant l'in se n io n de ce


m atériau est
2L
(6.40)
•* X

Lorsque le m atériau est inséré, la lum ière renvoyée par le m iroir ;W| subit un déphasage
(exprim é en longueurs d ’onde) de
2Ln 2L 2L
N = = — (« 1). (6.41)
X X A

À chaqu e dépha.sage d ’une longueur d ’on d e, la figure d ’in terféren ce est décalée
figure 617 L'interférornètre de Michelson. d ’nnc frange. Donc, en com ptant le nombre de franges décalées dans la figure par l’insertion
qui montre le parcours de la lumière du m a téria u , et en su b stitu a n t ce n o m b re à N,„ - d an s l'é q u a tio n 6 4 1. on peut
en provenance du point P d’une source déterm iner l'ép aisseu r L du m atériau, exprim ée en fonction de X.
étendue i’. Le miroir M divise la lumière Au moyen de lechniquc.s semblables, les longueurs des objets peuvent être exprim ées
en deux faisceaux, qui sont réfléchis en fonction de la longueur d ’onde de la lum ière. A l’époque de M ichel v m , l'étalo n tic
aux miroirs et M, vers M et vers longueur - le m ètre - était déterm iné, en vertu d ’une convention intarnationalc, par la
le télescope T. Dans le télescope, d istan ce entre d eux m arques fines su r une lig e de m étal co n serv ée à S èvres, près de
un observateur voit une tipurc P aris. M ichelso n put d ém o n trer, à l ’aide de .son in te rfé ro m e tre. q u e le m ètre étalo n
d'interférence compaséc de franges. équivalait à 1 553 163.5 longueurs d ’onde d 'u n e lum ière rouge m onochrom atique ém ise
par une source lum ineuse contenant du cadm ium . P our cette m esure précise. Michcl.son
reçut le prix N obel de physique en 1907. Son travail m ena à l'ab an d o n (en 1961 ) de la
barre é ta lo n d u m ètre c o m m e référen c e de lo n g u eu r et à une n o u v elle d éfin itio n
du m ètre, ex p rim é en fo nction de la longueur d 'o n d e rie la lum ière. Ln 1983. m êm e
cette référence basée sur la longueur d 'o n d e ne fut pas assez p re d se p o u r satisfaire aux
exigences cro issan tes de la science et de la technologie, et elle fut rem p lacée pat une
nouvelle référence basée sur une valeur définie de la vitesse de la Im ntèrc dans le vide

V»-*“
REVISION E T R ESUM E

//€ principe de H uygens On iicut prédire la transmission iridi- L ’expérience de Young Dans 1 experience de \o u n g su r
mensionnelle des ondes, incluant la lumière, grâce au principe de l'in te rfé re n re , un faisceau lumineux passant à travers une fente
Huygens, qui stipule que tous les points d'un front d'onde servent simple rencontre ensuite deux fentes pratiquées dans un ceiiiii.
de sources ponctuelles à de petites onde.s sphériques secondaires. La lumière qui quitte ces fentes se disperse tpar diffraciirm), ei im
.X|nès nn temps t, la nouvelle position du front d'midc sera celle de la phénomène d ’mtcrfcrcnce se produit do l'autre côté de récruii.
surface tangente à ces petites ondes secondaires. Line figure coiiiposér de franges créée par rinlerlcrcncc sc tormc sur
On peut dériver la loi de la réfraction à partir du principe de un écran d'observation.
Huygens en supposant que findicc de réfraction dans tout milieu est À n’iinixirte quel fKiini sur l’écran d ’observation, l’iniensilé de
n = dv. où Vest le module de la vitesse de la luiiiicre dans le milieu la lumière dépend en partie de la différence de marche entre les
et c est le rmxlule de la vitesse de la lumière dans le vide. rayon-s issus des deux fentes, fit celte différence équivaut à un
nombre entier de longueurs d ’ondc. les ondes produisent une inter
La lo n g u eu r d ’onde et l ’indice de réfraction La longueur
fcrence constructive et il en résulte une intensité maximale Si la
d'onde X„ de la lumière dans un milieu dépend de rintlice de réfrae-
différence éijuivaul à un nombre impair de demi-longueurs d'onde,
lion n du milieu :
il y a ¡nicrfércncc dcsinieiivc et l'iiitcnsitc est minimale Les conditions
A correspondant aux intensités maximale ei iiiiniinale som
X „ = -, (6 X)
4 sin Í) = wX.
où X est la longueur d ’onde de la lumière dans le vide. À cause lorsque W = (), ± I. ± 2 ,... (les maxima fiangcM brillantes). (6.14)
de cette relation, l.i différence de pluise entre deux ondes peut varier
ki les ondes traversent différents matériaux ayant des indices de d vin tí (m + i)X.
réfraction différents. lorsque m = Í). ± 1. ^ 2,... (les m inim a - franges som bres), (6 16)
166 Chopltre 6 L’Interférence

où 0 est l'angle forme par le parcours île la lumière avec un axe cen­ IL = (m + ') — ,
tral, ei d est la distance entre les fentes. - «2
loisqiic »1 = 0, 1. 2, ...
L a cohérence Si ilcux ondes lurnineuscs qui sc rencontrent en un
(les maxima - la pellicule brillante dans l'air). (6.34)
point interfèrent de innnière perceptible, le déphasage entre elles doit
demeurer constant dans le temps ; en d ’autiex mots, les ondes doivent
II. = m — ,
être cohérentes. Lorsque deux ondes cohérentes se rencontrent, «2
rintcnsiic résultante peut être trouvée avec la méthode des vecteurs lorsque m = 0, 1. 2....
de Kresnel. (les iiiiiiiina - la pellicule sombre dans l’air). (6,.)5)

IJ in ten sité de la fig u r e d ’in terféren ce p ro d u ite p a r d eu x


où n-, e.st rindicc de réfraction de la pellicule. /. est son épaisseur
fe n te s Dans l’expérience de Young, deux ondes ayant chacune
et X est la longueur d’oiule de la lumière dans l’air.
une inteasitc /„ produisent une onde résultante d'intensité I sur un
Si la liimicre incidente à une mtcrface entre deux milieux [XsaédaiK
écran d'observation, où un a
des indices de réfraction différents sc trinive dans le milieu ayant
2ml le plus petit indice de réfraction, la rétlexion cause un déphasage
I — 4/|, cos^ [ф/2). où Ф sin в. (6.21. 6.22)
de n nul. ou d’une demi-longueur d'onde, dans l'onde réfléchie
Les équations 6.14 et 6.16. qui donnent les positions des franges Sinon, il n’y a aucun déphasage causé par la réflexion. La réfraction
.sombres et brillantes (les minima ci les maxima), sont incluses dans à une interface ne cause pas de déphasage.
celte relation
L ’in terférom ètre de M ichelson Dans Vmterféromi'tre de
L ’interférence produite par une pellicule m ince Lorsqu’une Mtchehon, une onde lumineuse est divisée en deux faiscenux qui.
lumière est incidente .sur une mince pellicule transparente, les ondes après avoir suivi des parcours tie longueurs differentes, se réunissent
lumineuses réfléchies par les faces avant et arrière intcrlcrcm. Dans et produisent une interference, en formant une figure d’intertircncc
le cas truite incidence proche de la normale, les conditions relatives Eu variant la longueur du parcours d ’un des l'aLsccaux. on (leiii exiiriiiier
à riiuensité maximale et minimale de la lumière réfléchie sur une les distances en longueurs d’onde de la lumière, en comptant le nombre
pellicule \u\pen(lue dans l'air sont de franges tWealées dans la figure à cause de la variation.

• •« *S.<v < , V

Q U E S TIO N S

1 Dans la figure 6 18, trois impulsions de lumière - a, h et c - Négligez le petit angle dans le parcours de la lumière du deuxième
de même longueur d ’oiide sont transmises à travers des couches arrangement, a) Quelle est la différence de marche entre les rayons?
de plastique dont les iiulices ilc réfraction soni indiqués. Classez b) A quelle valeur, exprimée en longueurs d'onde, équivaudrait
les impulsions en ordre ilécrtiissant selon la durée de leur parcours cette différence de marche si les rayons étaient tlcphasés de n rad
dans le plastique. en émergeant, et c) quelle est la plus petite valeur de d permettant
ce déphasage final

-•-d *■"—d-

Figure 6.18 Q uostion 1

? I Inc lumière %c propage le long d’ime nanosiniciiire d’une longueur «


de I 5(K) nm Lorsqu’une crête de l’ontle est à une extrémité de la
nanostructure, y a-t-il une crête ou un creux à l'autre extrémité,
si lu longueur d’onde est a) de 500 nin cl b) de 1 000 nm '* figure 6.20 Question 4
.3. 1^1 figure 6. 1У montre deux rayons lumineux, d'une longueur d’onde
5. Y a t il une interférence constniciive, une interférence destructive,
de 600 nm. qui sont réfléchis sur des surfaces de verre séparées par une
une interférence intermédiaire plus proche du maximum ou une
distance de 150 nm. Ix;s rayons sont initialement en phase a) Duelle
interférence intermediaire plus pioche du miniimim au point P de la
est la différence de marche entre les rayons h) Après avoir quitté
figure 6.8 SI la différence de marche entre les deux rayons est de
lu région (le la réflexion, les rayons sont-ils en phase, en opposition
a) 2.2X. b) 3.5Â. e) 1.8X et d) I.OX? Donnez la valeur de m associée
de pha-sc n u dans un état intermediaire'’
au maximum ou au minimum dmis chacune des situations.
6. Dans une figure d'interférence à deux fentes, lorsque vous passez
d'une frange brillante à la frange brillante suivante, a) la différence
de marche AZ augmente t elle ou diminue-t-clle. et b) quelle est sa
variation, exprimée en longueurs d’onde X?
7. Dans une figure d'interférence obtenue avec ilcux fcnfeA, l’espa­
cement entre deux franges augmente-i-il. diminiie-l-il ou reste-t-il
Figure 6.19 ü u e s t lo n .) le mciiK a) lorsqu'on augmenté la distance entre les fentes, h) lorsque
la couleur de la lumière passe du rouge au bleu, et c) lorsque tout le
4 La ligmc 6 20 montre deux rayons lumineux qui sont imiinlemeni système est immergé dans le sherry ? d) Si les fentes sont éclairées par
en pha.se et qui sont réfléchis sur plusieurs surfaces de verre. une lumière blanche, cst-cc la composante rouge on la composante
Exercices et problèmes 167

bleue de chacun des pics secondaires qui est la plus rapprochée du 10. La figure 6.2.Í présente deux rayons lumineux qui rencontrent
maximum central ? des interfaces, où ils sont réfléchis et réfractés. Lesquelles des ondes
8. Dans chaque partie de la figure 6.21, des vcclcul^ de Fresnel résultantes subissent un déphasage à F interface?
rcprésemeni les deux ondes lumineuses*dims une expérience d’interfé­
rence avec deux fentes. Chaque partie représente egalement un point
diiféreni sur l'écran d'observation, à un instant different. F.n supposant
que les huit vecteurs de Fresnel ont la même longueur, clas.sez les
|X)ints en ordre décroi.ssant scion rintensilé de la lumière qui y est
observée.

A
\ r
11. La figure 6.24 a) montre la
.section transversale d’une |x:lliculc
mince verticale dont la laigeui
U--------

.0 b) <) d) augmente progressivement vers le


Figuf* 6.21 Quesiion 8 bas à cause de l’affaisscmcni causé
par la gravité. La figure 6.24 b)
9. Ljt figure 6.22 montre deux sources, 5| et 5>, qui émettent de.s ondes présente une vue de face de la
radio de longueur d'onde X dans toutes les directions. Les sources pellicule, illustrant quatre franges
sont en phase et séparées par une distance égale 6 l„SÀ. La ligne d ’interférence brillantes iwixluitcs Rgui«6 î4 O uestionll
verticale pointilléc est la bissectrice perpendiculaire à la distance lorsque la pellicule est éclairée par un rayon perpendiculaire de
entre leii .sources, a) Si on procède à partir do point de départ indiqué lumière rouge. Les points de la section transversale qui Loiie\iiomlt-iii
et qu’on suit le parcours 1, l'interférence produit-elle un maximum aux franges brillantes sont indiqués. Oucllc est la Jilïcrcncc. exprimée
tout le long du parcours, un minimum tout le long du parcours, ou en longueurs d'onde de la lumière clans la pellicule, dans /’cpais.scur
des minima e,t <les maxima en alternance? Reprenez l'exercice avec de la pellicule entre a) les points a et h ei h) les piim.s h cl c7?
bl le parcours 2 et c) le parcours X 12. La figure 6.23 montre la trans­
mission de la lumière à travers une
pellicule mince entourée d ’air par
un faisceau perpendiculaire tincliné
dans la figure par souci de clarté),

---- _ V,
________Cl
t’ a) l e rayon r, siihit-il un déphasage
causé par la réflexion? b) En lon­ Figure 6.25 Question 12
gueurs d ’onde, quel est le déphasage par réflexion du rayon r,?
t’oini rie riâparl Point de départ —^
c) Si L est l'épaisseur de la pellicule. <|uelle est la ilifférencc
figure 6.22 Question y de marche entre les rayoas ry et i <?

EXERCICES ET PROBLEM ^ë ,

www La solution se trouve sur le site Web, à l’adresse ci-dessous: 5P. Les vagues de l’oeéan appriH-heiii du rivage à une viiesM: ayant
un module de 4,0 m/s en fonuaiu un angle de ÎO" avec la iHirmale.
w« w.dlcmcgjawhill.ca/pliysique
comme rilliisire la figure 6 26. .Sup|iosez que la profondeur de l'caii
change subilemeni à une certaine disiance du rivage, et qu’h cct
endroit le module de la vitesse des vague* ilimlmic îi 1.0 ni/s Près du
SfCîlON 6 2 La nofure onduloioire de te lumière rivage, c|iiel est l'angle flenirc la direction de la vague cl In inirmalc ?
(Supposez (|iie la loi de la réfraction s'applique coiiiine avec la
IL La longueur d'onde de la lumière jaune (d’une lampe au sodium) lumière.) Expliquez pourquoi la pliipaii îles vagur.s atieigncni le
dars l’air est de .389 nm. a) Quelle est sa fréquciKC ? b) Quelle est sa rivage suivant la normale, alors qu’elles s'cii approcheni suivant îles
)ongiicur d’onde dans un verre dont l’indice <k réfraction est 1,52 ? angles variés lirrsqu’clles sont à une plus grande distiinco
c) D’après les résultats en a) et en b), trouvez le imxlule de la vitesse
de la lumière dans le verre.
IL La lumière voyage plus vite dans le saphir que dans le diamant
{voir le tableau 4 1). Quelle est la différence de vitesse en mètres
par seconde ?
3L Le module de la vitesse de la lumière jaune (d'une lampe un
sodium) dans un certain liquide est de 1.92 x lO** m/.s. Quel est
rjndice de réfraction de ce liquide
41 Quel est le module de la vitc.sse d'une lumière dont la longueur
4'onde est de 530 nm dans le verre de silice?
168 Chapitre 6 L'interférence

6P. Dans la figure 6.27, deux iinpulsir>n.s lumineuses sont transmises figure 6.8) de la frange brillante de troisième ordre (lorsque m — 3)
à travers des couches de plastique dont les indices de rélraclion .sont a) en radians et b) en degrés.
indiqués, et dont l’épais.seur est L ou 2L comme illustré, a) Quelle 12E, Quel e.si le déphast^e entre clcux ondes provcniml de deux fentes
impulsion met ic moin.s de temps à traverser le plastique ? b) Quelle
lorsqu’elles atteignent la énième frange s<.>mbre dans l’expérience de
est la différence, exprimée en Uc. entre les temps requis par les deux
Young sur rinterférencc produtte par deux tentes?
impulsions pour traverser les diverses couches '?
13L Supposez qu’on réalise l’expérience de Young en uiilisani une
lumière bleu-vert d’une longueui d’onde de 500 nm. Il y a une di.siancc
de 1,20 mm entre les fentes, et l’écran d’ob.scrvation est situé à 5.40 m
Im pulsion 1 I l .j ô 1.70 1,60 1.45 1 des fentes. Quelle est la distance entre les franges brillantes ? www
1 ^ 1
14E. Dans un système à deux fentes. la distance entre les fettles est égale
------- ,
Im pulsion 2 L as I..50 à I (X) fois la longueur d’onde de la lumière transmise ptu' les fentes,
a) Quelle est la séparation angulaire en radians entre le maximum
central et le maximum adjacent? b) Quelle est la distance entre ces
Rgure 6.27 Problème 6 maxima sur un écran d’cibservation situé à 50.0 cm des fentes ?
15E. Un système à deux fentes éclairées par une lumière au .sodium
7P. Observe/ la figure 6..1 et supposez que deux ondes lumineuses
ilans l’air, d'une longueui d ’onde de 4(X) nm, sont initialement en
(X = 589 nm) produit des franges d’interférence qui ont une .séparation
phase Une onde sc propage dans une ctniche de verre dont l’indice angulaire de 3..50 x lO” ’ rad. À quelle longueur d'onde la séparation
de réfraction est n — 1,60 et dr>nl l’épaisseur est L. L’autre onde sc serait-cllc 10 % plus grande ?
propage dans une etniche de plastique de même épaisseur et dont 16E. Un système à deux fentes éclairées par une lumière au sodium
l'indice de réfraction est ^ 1.50. a) Quelle est la valeur minimale (X = 589 nm) piXKluil des franges d ’interférence qui ont un angle de
que L dott avoir si les ondes sont déphasées de 5,65 rad en sc retrou­ 0,20“ entre clics. Quel sera l'angle entre les franges si le système
vant de nouveau dans l'a ir’’ b) .Si les ondes émergentes atteignent entier est immergé dans l’eau (/i — 1,33) ?
ensuite un point comiiuin. quel type d’interférence produiiont-elles ? 17E. Deux sources ponctuelles d ’ondes radio situées à 2,0 m l’une
8P Supposez que les deux ondes de la figure 6.3 ont une longueur de l’autre ém ettent en phase à une longueur d ’onde À — 0,50 m.
d'onde (le 500 nm dans l'air. Quel est leur déphasage, exprimé Un détecteur suit un parcours circulaire autour des deux sources dans
en longueurs d'onde, après avoir traversé les milieux 1 et 2, si un plan incluant les sources. Trouvez le mxnbre de maxima qu'il détecte.
a) w, — 1,50, n, = 1,60 et L — 8,50 /rm, b) «i — 1,62, ih ~ 1,72
18E. Les sources A c t B émettent des ondes radio de longue portée
et L = 8,50 /rm . c) tq ~ 1.59. ~ 1.79 et L = 3.25 ^rm?
et d’une longueur d’onde de 400 ni. et l’émission de A est en avance
d> Suppo.sez que, dans ces trois situations, les ondes émergentes
de Ç rad par rapport à celle de B. La distance entre A et un
atteignent un point commun. Classez les situations en ordre décrois.sant,
selon la brillance que les ondes produisent au point commun. détecteur est plus grande de 1(K) m par rappon à la distance corrcs-
pondante r«. Quel est le déphasage au détecteur?
9P Deux ondes lumincu.scs dans l’air ont une longueur d ’onde de
600,0 nm et sont initialement en phase. Elles se propagent ensuite 19P Dans une expérience avec deux fentes, la distance entre les fentes
dans des couches de plastique, comme le représente la figure 6.28, est de 5,0 mm et les fentes .sont situées à 1,0 ni de l’écnui. Deux figures
oit L, = 4,00 /rm, ¿2 ~ /'' h’. «1 = 1,40 et «2 = 1,60. a) Quel est d’interférence peuvent être observées sur l’écran : l’uiie d’elles est
leur dépha.vigc. exprime en longueurs d’onde, après leur émergence produite par une lumière d'une longueur d'onde de 480 nm. et l'autre
des couches? b) Si les ondes atteignetit ensuite un point commun, est produite pat une lumière d'une longueur d ’onde de 600 nm.
quel type d’interférence produiront-elles ■’ www Quelle est la distance sur l’écran entre les franges brillantes de
troisième ordre (m 3) des deux figurtrs d’iincrfércncc ?
- /-Z ^
20P. Dans la figure 6.29, S, d sont des générateurs d’caides en phase
et de même longueur d'ondc. La distance entre les générateurs e.st
d —■3,(X)X. Trouvez la plus grande distance à partir de S,, le long
de l’axe des jr. à laquelle une interférence destructive sc produira.
Exprimez cette distance en longueurs d'onde.

t'i s
Figure 6.28 FYohlème 9

lOP. Observez la figure 6.3 cr supposez que les deux orales lumineuses
ont une longueur d ’ondc de 620 nin dans l’air et sont initialement
déphasées de n rad. Les indices de réfrac tion des milieux .sont figure 6.29 Problèmes 20. 27 et .59
W| ” 1,45 et M- = 1,65. a) Quelle est la plus petite valeur de
l’épaissenr / t|Ui permettra aux ondes d'être en pha.se après a\ i>it 21P Une mince pellicule de mica (n 1.58) e-si utili.sée pour taïuvrir
traversé les deux m ilieux? b) Quelle est la valeur suivante de L une lente dans un système d'interférence à deux fentes. .Sur l’écran
qui iiemieiira la luêna" situation ? d’observation, le ixiint central est maintenant occupé par ce qui était
la septième frange brillante (m = 7) avant que la fente .soit couverte.
Si A — 5.50 nm, (|iielle est l’cpai.sscur du mica'? {Indice: Considérez
SECTION 6.4 l'Bxpériencs tie Young sur l'siferférente
la longueur d’orale de la lumière dans le mica.) www
IlL Une luniicrc verte monocbromaliqiie d'une longueur d’onde 22P. Dans une salle de lecture, la lumière d'un laser ayant une
de 550 nin éclaire deux fentes étroites et patallèles séparées par longueur d'ondc de 632,8 nni est transmise par un système à deux
une distance de 7,70 tum. Calculez la déviation angulaire (fi dans la lentes, est réfléchie sur un miroir situé à 20,0 m à farricre de la salle.
exercices et problèmes 169

el prixluii ensuite une figure d’inierfércnce sur un écran à l'avani ondes réfléchies seront en opposition de phase ? b) De combien doit on
de la salle. Lu dislance enire les franges brillantes adjacentes est de déplacer la plaque pour que les ondes soient de nouveau en phase?
10,0 cm. a) Qiielle est la distance entre les fentes ? b) Qu'arrivc-t il à 311 Une lumière brillante d'une longueur d’onde de 585 nni arrive à
la figure lorsque le lecteur piece une mince feuille de cellophane sur incidence normale sur une pellicule d ’eau savonneuse (ii = 1.33)
une fente, augmentant ainsi de 2,-50 le nombre de longueurs d'onde d'cpais.scur 1,21 /zm, suspendue dans l ’air. L* lumière réfléchie par
dans le parcours incluant la cellophane'f les deux surfaces de la pellicule produit-elle une iiilci férence plutôt
destructive ou plutôt constructive ?
SECTION 6.6 l'inle m ité de b figure d'interférence produite par deux fentes 32E. Supposez que les ondes lumineuses de l’excrcicc 30 sont initia­
lement déphasées de ,t rad. Trouvez une expression pour décrire les
23L Deux ondes de même fréquence ont des amplitudes de 1,0<>et 2,(X). valeurs de L (cxpiiméc en longueurs d’onde À) en fonction dc.squclles
Elles intcrfcicnt à un point où leur déphasage est de rr/.i rad. Quelle les ondes réfléchies seront parfaitement en pliasc.
est l'amplitude ré.sultaïue?
331 Une lumière d ’une longueur d'onde de 624 nm arrive pci pen
24L Trouvez la somme y des quantités suivantes : diculaircmcnt sur une pellicule d'eau savonneuse (« = I ,.33) suspendue
dans l'air. Quelles sont les deux plus petites épaisseurs de b pellicule
y, = lOsiiKt)/ et ys = R.Osintarr 4 n'/6).
en fonction dc.sijuelles les réflexion.s de la pellicule produiront une
25t Additionnez le.s quantités suivantes interférence constructive ? www
34£. Une lentille d'appareil photo dont l’indice de réfraction est .supé­
y, = 10 situar rieur à I..30 est enduite d'une mince pellicule transparente, dont
>s = 15 sinicar + 7t/6) l'iiulice de réfraction est 1,25, pour supprimer par interférence la
V3 = 5,0 sinirar —.tM) réflexion de la lumière de longucui d'oiide k qui ariivc à incidence
normale sur la lentille. Quelle doit être l’épaisseur minimale de
en utilisant la méthode des vecteurs de Fresnel, www l'cnduii. exprimée en à?
26L Une lumière d'une longueur d’onde de 6(K) run est incidente suivant 35E. Les faux diamants utilisés dans les costumes sont du verte dont
la nornwle à deux fentes minces parallèles séparées par une distance l’indice de réfraction est 1.50. Pour augmenter la reflexion, on les
de 0,60 inin. Faites un croquis représentant l'intensité de la figure obser­ enduit souvent d'une couche de monoxyde de siltconc dont l iiidice
vée sur un écran éloigné, en fonction de l'angle Hà partir du centre de réfraction est 2,00. Quelle épaisseur minimale l’enduit doit il
de la figure et dans les limites comprises entre 0 :£ fl £ 0.004 0 rad. avoir pour que la lumière à incidence normale d’une longueur d’onde
de 560 nm soit réfléchie par les deux surfaces et produise une interfé
27P. Dans la figure 6.29, S| et S', sont des sources ponctuelles d'ondes
reiK'e constructive ?
électromagnétiques d'une longueur d ’onde de 1,00 m. Les sources
suni en phase et .séparées par une distance d = 4,00 m, et elles 36E Dans la figure 6.31. une lumière incidente d'une longueur d ’onde
de 600 nm est perpendiculaire à cinq sectittns d’une structure transpa­
émettent à la même puissance, a» Si on déplace un détecteur vers la
rente suspendue (bus l’air I .a stniclure a un indice de refraction de 1 50,
droite à partir de la source S, suivant l’axe des .r. à quelles distaiKCs
L’épais.scur de chtujue section est exprimée en foiuiion de 7. où
de S| les trois premiers maxima d ’interfcrcncc seront-ils détectés'.'
L — 4,(Ml /iin. Dans quelles sections la lumière qui est réfléchie
b) L'intensité du iiiiniinum le plus rapproché est-cllc exactement
par les surfaces du haut el du bas produira-t-elle une iiilcrlércncc
nulle? (/«rfirc.' L’intensité d’une onile issue d'une source ponctuelle constructive ■?
deniciire-t-elle coiislanic lorsqu’on s'éloigne de la source"')
28P. Dans la figure 6.9. la flèche à double sens indique les points sut
la courbe d'intensité oii l’intensité de la fiangc centrale vaut la moitié
de l'intensité maximale. Démontrez que la séparation angulaire àO
entre les points correspondants sur l'écran d'observation e.sl
Ы.
A flu ­
ir/
si. dans la ligure 6.H, 9esl a.s.sc/. petit pour effectuer l’approximation
vin 0 *= (1
A
L
2/.
I
Л/2 /./Ю
29P' Dans une expérience sur rimcrfénence avec dctix fentes, supposez
qu’une des fentes e,st pins large que l'autre, et que riniplifude tic 1« -0 1.) I .) d) <■)
lumière i/ui atteint la partie centrale de l’écran, en provenance d’une figure 6.31 Exercice 36
seule fente, vaut le double de celle de l'autre fente. Dérivez, une eX|X"C.s-
sion pour décrire l'intensité Uiinincuse / à l'écran en fonction de fí: 37E. On veut enduire un verre plat tji — 1,50) d ’un inHicnau trans­
celte expression doit correspondre aux équations 6.21 et 6.22. www parent (n = 1.25) afin de supprimer par imet férence la réflexion
de la lumière d ’une longueur d onde de 6(K) nm. Quelle doit être
SECTION 6.7 L'interférence produite par les pellicules n'inces l’épaisseur minimale de l'enduit ? www
38P. Dans lu figure 6.32. une lumicre incidente est perpendiculaire ft
ЗОЕ Dans la figure 6.30, l’onde I. quatre minces couches d'épaisseur L. Les indices de réfraction des
himineiLse O, est réfléchie une fois ;
quatre couches el des milieux au-dessus el en dessous de ces couchexs
sur une surface réfléchissante, alors sont indiqués. Soit à la longueur d'onde de la lumière dans l'air,
que l’onde f>i est réfléchie deux O, et n,. l’indice de réfraction de chacune des couches. Considérez
fois sur cette meme surface et une seulement la iransini.ssion de la lumière qui ne subit aucune réflexion
fois sur une plaque réfléchissante ou qui subit deux réflexions, ctanme c’est illustre darus la figure 6.32 a).
située à une distance L du miroir. figure 6.30 F.xerciccs 3Í) et 32 Relativement à quelles situations l’expn'ssion
Les ondes sont initialement en phase
et ont une longueur d'tHide de 620 nm. Négligez le faible écartement 2l.r¡i
к = lorsque m = 0. L 2. 3,...
des rayons, al Quelle est la plus petite valeur de L pMir laquelle les
170 Chapitre 6 L'interférence

clonne-l-clle les longueur's d’onde de la lumière transmise qui produit franges sombres à cause de cette pellicule, a) La frange observée
une interférence constructive ? à l’extremilé gauche est-elle brillante ou som bre? b) À partir de
l’extrémité droite, une interférence destructive se produit en différents
endroits, en fonction de differentes longueurs d'onde de la lumière. Se
f ‘ 1,6'
produit-elle d ’abord à rextrem ité rouge ou à l’extrémité bleue du
L • 1,1 1 3.« 1,8
-■ L . — spectre visible ?
V\ i.-i . : 1.4 ' ' 1,6 45F Un large faisceau de lumière d’une longueur d'onde de 630 nm
и forme un angle de 00“ avec une mince pellicule en forme de coin,
’U L 8 1,8 M dont l'indice de réfrat;lion est de 1,50. Un observateur qui imereepte
la lumière tran.smise par la pellicule voit 10 franges brillantes
cl 9 franges sombres le long de la pellicule. Quelle est la variation de
Л) b) i) d)
l'épaisseur de la pellicule sur cette longueur? vww
figure 6.32 Problèmes 3K et 3^ 46P. Une mince pellicule d’acétone (n - 1.25) recouvre une épais,se
plaque de verre (n - 1.50). Une lumière blanche est incidente suivant
3VP. L'n pétrolier endommagé laisse échapper du kérosène (k 1.20)
la normale à la pellicule. Dans les réflexions, une interférence
dans le trolfe Persique. Le kérosène foniK une gr ande nappe à la surface destructive se produil à 600 nm. et une interférence constructive se
de l'caii (/; I..Î0). a) Si vous observez direelemcnt vers le bas. à partir
produit à 71Ю nm. Calculez l’épai.sscur de ta pellicule d'acétone.
d'un avion lors d’iiiK journée ensoleillée, une région de la nap(te dtriit
47P Deux plaques de verre sont jtiintcs à une extrémité ptair former
fépaisseiir est de 4fr() nm, en fonction de quelles longueurs d’onde
une brèche d’air qui agit сенпте une pellicule mince. Un large faisceau
de la lumière visible la réflexion scra-t-cUc la plus bnllante à cause de
de lumière d’une longueur d'onde de 480 nm est orienté à traver.s le.s
rimerférrnee consti uctivc b) Si vous faites île la plongée «rus-niarine
plaquc-s, pcqxrixliculairement й la première plaque, l ri observateur qui
«IMI.S cotte même région de la nappe, en fonction de quelles longueurs
d'onilc de la lumière visible la réflexion .sera-l-ellc transmise avec iniovepie la lumière réfléchie par les plaques voit une figure d’interfé­
rence causée par la btèchc d’air. Quelle est l'épaisseur supplémentaire
le plus d 'jn tc n s itc {Indice: Uttltsez la hgure 6.32 a) en prenant les
indices de réfraction appropriés.) de la brèche à la seizième frange brillante, cixnparativcment à la sixicnx*
frange brillante, à ¡rartir du joint entre les plaques ?
40P. Une onde plane de lumière monochromatique est incidente
48P. Un large faisceau de lumière monochromatique est orienté
suivant la nonnalc a une mince pellicule d ’huile qui recouv re une
plaque de verie. On jretir faire varier la Umgueur d’onde de la source. perpendiculairement à travers deux plaques de verre jointes à une
extrémité pour former une brèche d’air entre elles. Un observateur
On observe une interférence destructive de la lumière rélléchie dans
qui intercepte la lumière réfléchie par la brèche d ’air, qui agil ccriimc
le cas des longueurs d'oiulc de .WO nm et de 700 nm, mais aucune
une pellicule mince, voit 4 (K)l franges sombres le long de la bièchc.
dans le cas il’iinc Inngueur d’onde intermédiaire. Si l’indice de
Lorsqu’on évacue l’air entre les plaques, seulement 4 000 franges
réfraction de l'huile est de 1.30 et celui du verre est de 1.50. trouvez
sombres sont visibles. Calculez l’indice de réfraction de l’airen fonc­
l'épaisseur minimale de la pellicule d’huilc.
tion de CCS données.
41P Dans Pair, une onde plane lumineuse monochromatique anive à
49P La figure 6.34 a) montre une lentille ayant un rayon tle coiirhiire
incidence nonnalc sur une mince pellicule d ’huile qui recouvre une
R dépciséc sur une plaque de verre plane et éclairée du haut par une
plaque de vcnc. On peut faire varier la longueur d ’onde de la source.
On obsen-e une interférence destructive de la lumière réfléchie dans
le ciLs des longueurs d’oncle de 5(KJ nm ci de 700 nm. mais aucune
ilans le cas d’une longueur d’onde intermédiaire. 1.' indice de réfrac tien
du verre est de 1.50. Démontrez cjue l’indice de réfraction de l'huile doit
éire intérieur à 1,50
inri(!rm c
\
\R
42P. La réflexion d'un faisceau de lumière blanche perpendiculaire
è une [icllicule d'eau savonneuse dans l’air (iroduii une interference
ton.striiciive îi 600 uni et une interférence destructive è 450 nm.
et II n y a pas de minimum entre ces valeurs. Si n = 1.3.3 dans la
pellicule, qiiolle est l'épaisseur minimale de la pellicule, que l'on Au A-;*--
su{:^ose uniforme'? , ■■
' Verre ,
43P. Dans la ligure 6.33, un large I.umièi'o in rid e n if a)
faisceau lumineux d ime longiieiii
d onde de 6X3 iim est orienté
directement vers le bas. a travers
la plaque du de.ssu.s d une paire de
iii i 1
fil
plaques de verre Les plaques
ont une longueur de 120 mm,
sont jointes è l’cxiréiniié gauche - 1?li rmn
et séparées par un til de 0,04X mm
dr diamètre il l’e xtrémité droite RgiirB6.33 Pn'blèmes 4.3 et 41
L’an entre les plaques agit comme une priliciilt; mince. Combien
(Ip trange« brillantes seront visibles pour un observateur qui regarde
du liiiiil la platine till (Icssiis'’
44P. Dans la figure 6.33, une lumière blanche est transmise vers le bas
à travers la plaque supérieure d'une paire de plaques de verre.
Les plaques .sont jointes à rcxtiéniité gauche, et sépaiée.s pai un fil
de 0,048 mm de diamètre à l ’exiréniité droite ; l'air entre les plaques l>)

agit comme une pelliciiic mince. Un observateur qui regarde vers


le bas à travers la plaque du dessus voit des frange.s brillantes et des Figure 6.34 Problèmes 49 à 52
Exercices et problèmes 171

lumière de longueur d'onde X. La figure 6.34 b) (une photographie de 57P. Dans la figure 6.36. une chambre hermétique comporUuit des
la lentille vue du haut) fait voir des franges d'interférence circulaires fcncircs de verre et mesurant 5 cm est placée dans un bras d’un imer-
(nommées anneaux de Newton), ass(x:iécs à l’épaisseur variable d féromètre de Michelson On utilise une lumière d’une longueur
de la pellicule d ’air entre la lentille et la plaque. Trouvez les rayons r d'onde X = 500 nm. En évacuant l'air de la chambre, on produit un
de.s máxima de rinierfércncc, en supposant que riR 1. wvyw défilement de 60 franges. À l’aide de ces données, trouvez l'indice de
50P Dans une experienee portant sur les anneaux de Newton {voir le réfraction de Tair à la pres,sion atmosphérique.
problème -79), le rayon de courbure R de la lentille est de 5,0 m,
et le diamètre de cette lentille est de 20 mm. a) Combien d'anneaux Miroii
brillants peut-on ob.server? Suppose/ que X = 589 nm. b) Combien
d'anneaux brillants pixirrait-on ob.server si le système était immergé
dan.\ Teau (/i - 1.33)?
SIP. C)n utilise un dispositif d'anneaux de Newton pour déterminer
le rayon lie courbuic d'une lentille (voir la figure 6.34 et le pro­
blème 49). Les rayons du et du (ni -f 20)“ "“ anneau brillant
mesurent respectivement 0.162 cm et 0.368 cm. La lumière incidente Miroir
a une longueur d'onde de 546 nm. Calculez le rayon de courbure de
la surface inferieure de la lentille. Veis
52P a) Utilisez le résultat du problème 49 pour démontrer que, dans la рош |н'
une expérience sur les anneaux de Newton, la différence de rayon â rid e
entre deux anneaux brillants adjacents est donnée par

Ar = r„+i - ^ yJxR/m.
Figure 6.36 Problème 57
en sup()osant que in'S¡> I. b) Démontre/ ensuite que Taire entre deux
anneaux brillants ,'UlJacenLs est donnée par 58P Ecrivez une expression pour décrire l’intensité observée dans un
interféromètre de M ichclsai (ligure 6.17) en fonction ilc lîi position
A — TtkR.
d’un miroir mobile. M esure/ lu position du miroir X pariii du point
en suppoMim que m 2> 1• Notez que cette aire est indépendante de w. o ù d j - d,.
53P Dans la figure 6.35, un iransmeiieiir de micro ondes situé à une
hauteur a au-dessus du niveau d'eau d'un grand Inc transmet des Problèmes stipplemetilaires
micro-ondes de longueur d'onde X en direction d'un récepteur situé 59. La figure 6.29 montre deux sources ponctuelles, Sj et S,,
sur la rive opposée, à une distance r au-dessus du niveau d'eau. qui ém ettait une lumière d'une longueur d'onde X = 5ÜU nm.
Les micro ondes réfléchies par la surface de Teau interfèrent avec Les émissions sont isotropes et en phase, cl la distance entre les
celles qui arrivent directement du transmetteur. En supposant que lu sources e.st d = 2.00 pm . A n ’importe quel point P sur Taxe de.\ x.
largeur U de ce lac est beaucoup plus grande que a et x, et que X 2: <?. Tonde issue de S, et celle issue de i'i causent une interférence.
en fonction de quelles valeurs de .r le signal sera-t-il maximal nu Lorsque P est très éloigné (.r ^ rc), quels sont a) le dé|)liasagc entre
récepteur? (indice: La réflexion causc-t-clle un déphasage’’) les ondes issues de .5) cl et b) le type d ’interférence que ccs ondes
produisent (plutôt constructive ihi plutôt destructive) ? c) .Si on déplace
ensuite P le long de Taxe des .r en direction de S,, le déphasage entre
les iHides issues de 5, et .Çj aiigmente-i-il ou dimimie-t il ? cl) l aitc,s
un tableau qui indique les prcsitions .v auxi|uclles les dépliasages wmii
rie fi, fi.5i)X. I.OOX__ 2.50X. et indique/,, o chaque position, ie type
corre.spond.aiit d’interférence - soit constiuctivc (c), .soit dcslniclive (d).
60 Vers la fin des années 18(K). la plupart dc.s .scientifiques croyaient
figure 6.35 P ro b lè m e 53 que la lumière (comme toute onde élccirnmagnclique) avait hc.soin
d’un milieu pour se propager, et qu'elle ne pouvait sc propager dans
SECTION 6.8 L'inlerfétom èlre de Michelson le vide. Cette opinion reposait sur le fait que tous les aunes lyiK sd'ondi'
connus des scientifiques ne pouvaient se propagei huis d’un milieu
54E. Une mince pellicule dont l'indice de réfraction est n = 1,40 est Les ondes sonores, par exemple, peuvent voyager daas Tair. dans
placée dans Tun des bras d'un interféromètre de Micliclson, perpen­ l’eau et dans le sol, mais non dans le vide Les scicnlifuiufs pi-nsaicnl
diculairement an parcours optique. Si cela produit un dépltasagc de donc que 1.1 lumière qui se dirigeait vers la l ’erre en prm enaïu r du
7,0 flanges dans une figure produite par une lumière d'une longueur Soleil ou d’une autre étoile ne pouvant sc propager dans le vide, et
d’onde de 5S9 nm. quelle est l'épaisseur de la pellicule? qu’il devait dont exister un milieu qui remplissait Tespacc ou la
55E. Si le miroir Ai>cst déplacé de 0,233 mm dims T interféromètre de Terre sc trouvait et dans lequel la lumière jXHJvait sc propager. On
Michelson (figure 6.17), il sc produit un déplacement de 792 franges. présumait que la lumière avait une certaine vitesse c dans ce milieu,
Quelle est la longueur d'onde de la lumière produisant la figure qu’on nommait Tct/it7 .
d’interférence composée de franges? En 1887, .Michelson cl Edward Morely iililisèrcni une version
56P. La lumière jaune du sodium est formée de deux longueurs de Tinlcrféroinètrc de Vlichcfson afin de vérifier les effets de Tcllicr
d'omie. X, = 589,10 nm et X, = 58Q..59 nm. On utilise la lumière du sur le mouvement de la lumière dans cet instrument F3iisque la Terre
sixliiirn dans un interféromètre de Michelson (ligure 6.17). De quelle tourne autout du .Soleil, le tnouvemcni de cet insinimeni dans Téther
distance doit-on déplacer le miroir pour déplacer la figure aurait dû aflecicr la figure d’interférence produite par le dispositif.
de franges créée p«r une longueur d'onde d’exactement une frange l/:s scientifiques supposaient alors que le .Soleil était approxiuialivr-
de plus que la figure de franges créée par Tautre longueur d'onde ? menl au repos dans ccl éther ei que la vitesse de Tinlrrféromèlrc
172 Chapitre 6 L'interférence

dans l'éther devait ctre la vitesse r de la Terre en orbite autour


du Soleil.
La figure 6 37 a) illustre la dispo.sition du système de miroirs
dans l’exjiérience de 1SS7. Les miroirs étaient montés sur une louide
dalle dcpo.sce sur un ba.ssin de mercure, ce qui pernieiiait à la dalle de
tourner doucement autour d'un axe vertical. Michelson et Morely
voulaient surveiller la figure d'interférence en tournant la dalle, ce
qui changeait l'orientation des bras de l’interféromclrc par rapport
au mouvement dan.s l'éthcr. Lin décalage des franges dans la figure
d’inlcrl'crcnce durant la rotation aurait signalé la présence de l'éther
Lu figure 6.37 b), qui offre une vue en plongée de ce système,
représente le parcours de la Jnrnicrc. Afin d'augmenter la probabilité
d'un décalage, la lumière cUiit réfléchie plusieurs foi.s dans les bras
de l’interféronictrc, et non une seule fois dans chaque bnus, comme
c'ctiiit le cas avec l'interféromètrc de la figure 6.17. Ces réflexions
répétées ont eu pour effet d ’augmenter la longueur elïcctivc de
clmqiie bras à rnviirin 10 m Malgré celte complexité accnie, l'inter-
feromètre de.s figuic.s 6..37 a) et b) fonctionne exactement comme
celui de la tigurc 6.17: on peut donc utiliser l’équation 6.17 dans ce
problème en |tosant simplement que les longueurs tf, et d, du bras
.sttnl toutes les deux de 10 m.
On supposera ici qu’il y a présence d ’un ctlicr dans lequel la
lumière a une vitesse c. La ligure 6..17 c) présente une vue latérale
du bras de longueur d| dans le référentiel de l'éther, alors que l’inter
fémmètre se déplace dans ce milieu avec une vitesse île module v.
(Pour simplificT le problème. le divi.seut de faisceau M de la figure
6.1 / est représente parallèlement au miroir A/j à rextrctniic éloignée
du bras.) La figure 637 d) présente le bnes au moment où une portion
de la lumière (reptésentcc par un point) entreprend son parcours dans
le bras. On suiv'ra cette lumière pour déterminer la longueur du
pa.icours dans le bras, à partir du diviseur tie faisceau jusq u ’au
miroir M, cl de rcniur au diviseur de faisceau.
Comme la lumière se déplace vers la dniite à une vitesse c dans — rf, "T
l'étlieret en direction du niimir M,, ce miroir .se déplace vers la droite
à la vitesse v. I,a figure 6.37 e) présente les positions de M et de M,
au moment où la lumière atteint A/j et y e.st réfléchie. La lumière se
« I

e)
\
déplace mainicnani vers la gaucltc dims l'éiher à une vires.se c, alors
que M se déplace vers la droite. La figure 6.37 f) represente les ,v/1«— ^---------------------- 1 W,
po.siiions de M et de M\ au moment où la lumière est revenue à M.
a) Démontre/ que la durée totale du parcours de cette lumière, de d)
Af à Ai, et de retour à Ai, est

2<r/| ,,J_--------------
t\ = '•2 —
e)
et que la longiicui du paicoius /., suivi par la lumière dans ce bras
est de M M,
[
L\ = cil
.2 -
M,
La figure h.37 g) illustre le hra.s de longueur <U\ ce bras se
déplace égaJeinent vers la droite avec une vitesse de module v dan.s
I éther l’our simplifier le problème, le diviseur de faisceau Ai de la
figure 6.17 CM maintenant représenté parallèlement au miroir Ai,, à
rextrem ité éloignée de ce bras, La figure 6 37 h) pré.scntc le bras au
moment où une portion de Inniièrc (le point) y entreprend son par­
cours. Puisque le bras se déplace vers la dioitc jicndant que la
Inmière se propnge le parwurs de la lumière est dévié vers la droite
vers la position qu'aura ;V/j au moinenr où la lumière atteindra ce K)
2 II) i) .il
miroir (fiaurc 6.37 i). La réflcxUin de la lumière à ,Aii transmet Figure 6.37' Problème 60
Exercices et problèmes 173

la lumière vers la droite vers la position de M au mttinent où la où À est la longueur d’onde de la lumière. Cette différence de phase
lumière revient à M (figure 6.37 j). 6) Démontrez que la durée totale détermine la figure d'interférence produite par la lumière qui atteint
du parcours de la lumière, entre M et A/i et de retour à M. est de le télescope de rinterteromètre.
Tournez maintenant l’interféromcta' de 90“. afin que le bras de
2d'.
h = longueur J , suive la direction du déplacement dans l’cther et que
•Jc^ — le bras de longueur r/j soit perpendiculaire à ce déplacement,
et que la longueur du parcours L, suivi par la lumière dans ce bras est e) Démontrez que le déphasage dans la figure d'intcrlérence, cau.sé
par cette rotation, est de
2cî/2
¿2 = 112 = 2d\^
déphasage —
At.2 •
Substituez d à et dj dans les expressions de et L^. Développez
ensuite les deux expressions à l’aide du développement du hiiWimc f) [évaluez le déphasage, en mettant c = 3,0 x 10* m/s, d - 10 m
{voir iannexe D) \ retenez les deux premiers termes de chaque et A = 300 nm. et en consultant l’annexe C pour les données concer­
développement, c) Démontrez que la longueur du parcours L| est nant la Terre.
plus graixle que celle du parcours L^, et que cette différence A/, cs't de Ce déphasage dans la figure d’interférence aurait dû être facile
à observer. Toutefois. Michelson et Merely n’observèrent aucun
déphasage, ce qui remit fortement en cause l’existence de l’éther.
(.Í Kn fait, l’idée même de l’éther ne tarda pas à disparaître. De plus,
les résultats non concluants de Michelson et Merely menèrent,
d) DénKinirez que la différence de phase (exprimée en fonction de la
du moins indirectement, à la théorie de la relativité restreinte
longueur d’onde) entre les parcours f-, et Z., ^st donnée par
d'Einstein.
M d\^
A À ?’
■%t - 1 La diffraction
et la polarisation

Georges Seuroi a peinf U


ndimancheaprès-midiàl'ikdelaGrandeJattesans utiliser ia technique habituelle des coups
de pinceaux, mais en apposant plutôt sur la toile des myriades de petits points de couleur, dans un style maintenant connu
sous le nom de pointillisme. Vous pouvez voir les points si vous vous approchez suffisamment du tableau, mais ils se fondent
et deviennent indiscernables
à mesure que vous vous en
éingnez. De plus, les couleurs
que vous percevez dans
le tableau changent lorsque
vous prenez du recul -
et c'est la raison pour laquelle
Seurat a peint en utilisant
cette technique.

lo r é ^ s o » trouve dons ce clnpitte.


7.1 La diffraction et la théorie ondulatoire de la lumière 175

7.1 La diffraction et la théorie


ondulatoire de la lumière
D ans le chapitre 6, on a d éfin i som m airem ent la d iffractio n com m e étant l’étalem ent
de la lum ière qui ém erge d’une lente étroite. En réalité, il se produit plus q u ’un étalement,
car la lumière produit un phénom ène d ’inicrlcrcnce que l’on nom m e fig u re d e d iffra ctio n
ou p a t r o n d e d if f r a c tio n . Par exem ple, lo rsq u ’une lum ière n io n fvhrom atiqiic issue
d ’une source éloignée (ou d ’un laser) traverse une fente étro ite e t est ensuite interceptée
p ar un écran d ’o b se rv a tio n , elle p roduit su r l’écran une fig u re de d ilfrac tio n com m e
celle de la figure 7.1. C ette fig u re sc com pose d ’un m axim um central large et intense
(très brillant) et d ’un nom bre d e m axim a m oins intenses et plu.s ctm ils (nonunés m axim a
secondaires) de chaque côté. L,es m inim a sont situés approximaiivemenl entre les m axim a
En optique géoinéüiqiic, une telle figure serait totalem ent inattendue : si la lum ière
se propage en ligne d ro ite com m e le font les rayons, la fente devrait alors perm ettre à
certains des rayons de passer, et ils devraienl form er une réplique brillante et nette de la
fente sur l'éc ra n d ’o bservation. C om m e on l’a fait au chapitre 6 , on d o it conclure que
l’optique géom étrique ne fournil q u 'u n e approxim ation du com portem ent de la lum ière.
La diffraction de la lum ière ne se lim ite pas aux situations où la lum ière passe dans
une (Hivcrture étroite (com m e une fente ou un trou d 'épingle). Elle sc produit égalem ent
Figure 7.1 Cette figiirc de diffraction lorsque la lum ière ren co n tre une arête, com m e les bords d e la lam e de rasoir dont la
apparaît sur un écran d’observation figure d e diffraction est illustrée dans la figure 7.2. O bservez les lignes de m axim a et de
lorsqu’une lumière traverse une fente m inim a plus ou m oins parallèles aux bords intérieurs et extérieurs de la lame. lu
étroite verticale. La diffraction proviKjiie lum icre franchit le bord vertical du côté gauche, clic s ’étale et interfère, ce qui produit
un éialemcnt de la lumière jrcqjendicu- la figure ob serv ée. L a plus gran d e p artie de ce tte figure se form e à l’en d ro it où l’on
laireinenl aux longs côtés de la fente. devrait voir l’om bre d e la lam e, .si les principes de l’optique géom étrique s’appliquaient
L’interférence produit une Figure intégralem ent.
de diffraction se composant d’un large l In exem ple com m un de diffraction est perceptible quand vous regardez, le ciel et que
maximum central et de maxima vous voyez des petites taches et des structures sem blables à des cheveux qui sem blent
secondaires nroias intenses et plus
flotter dans votre cham p de vision. Cea flotteurs, com m e o n les appelle, apparaissent
étroits, qui sont séparés par des minima.
lorsque la lumière rencontre les bords de minces dépôts dan.s ГЬшпеиг aqueuse, la substance
transparente q ui cou v re u n e gran d e p artie de Гге11. Q u an d un flo tte u r se trouve dans
votre ch am p d e v isio n , ce que vous vo y ez est la fig u re de d iffractio n p ro d u ite su r la
rétine par l’un de ces dépôts. Si vous regardez à travers un trou d ’épingle dan.s une feuille
opaque de m anière qu e la lum ière en tre dans votre oril co m m e une o nde p lane, vous
pouvez distinguer des m axim a et des m inim a tlans les figures de diffraction.

La diffraction de Fresnel
E a th é o rie o n d u la to ire de la lu m ière p erm e t d ’ex p liq u e r fac ilem en t la d iflia e tio n
Cette théorie, proptisée initialement à la fin des années 16(H) par Huygens, et dont Young
-se .servit 123 ans plus tard pour ex p liq u er l’interféren ce produite p a r deux fentes, mil
toutefois beaucoup d e tem ps à s ’imposer, surtout p arer q u ’elle allail à l’encontre dc la
théorie dc N ew ion, selon latjucllc la lum ière est un faisceau de particules
Cette idée de N ewton était le point de vue prédominant dans les cercles scientifiques
français au début du xix^ sièt le, à l'ép o q u e où A ugustin Fresnel était un jeune ingénieur
m ilitaire E re sn d , qui croyait à la théorie o ndulatoire de la lum ière, soum it un a i u d e
à Г A cadém ie fran çaise d es scien ces dan.s lequel il décri vail ses e x p é rien c es avec la
liim icre et ses explications dc la théorie ondulatoire
En 1819, l’Acadéinie. dom inée par des partisans dc Newion, lança un défi aux lenani.s
de la théorie on d u lato ire : elle o rg an isa un co n co u rs ayant pour thèm e un es.sai su r la
d iffra c tio n . F re sn el r e m p o r ta I es p a rtisa n s d e N ew to n , to u te fo is, nc fu re n t pas
convaincus pour autant L 'u n d ’eux, D enis Poisson, releva « le résultat étra n g e» de la
théorie de F resnel, qui im pliquait qu e les ondes lum ineuses d ev raien t s 'é la le r d an s la
région de l’om b re d ’une sphère en franchissant les bords de cette sphère, et p roduire

Rgure7.2 La figure de diffraction produite par um' lame de rasoii éclairée jtai une lunicrc
monochronialique. ( Ibservez l’altcrnancc des ligne.s d'intensités maximales et minimales
176 Chapitre 7 La diffraction et (a proiarlsation

un p o in t b rilla n t au c e n tre de la rég io n o b scu re. Le co m ité organi.satcur p rép ara


une expérience pttur vérifier la prédiction du célèbre m athém aticien et découvrit (voir la
figure 7.3) que la loche de Poisson, com m e on l’appelle au jo u rd ’hui, se trouvait bien
à l’endroit prévu. Rien ne donne autant confiance en une théorie qu e d e voir une de ses
prédictions les plus inattendues et surprenantes être confirm ée p ar l’expérience.

7.2 La diffraction produite par une seule fente :


localiser les minima
On peut m aintenant exam iner la figure de diffraction produite par les ondes planes d ’une
lum ière de longu eu r d ’o nde 7. qui sont d iffractées en passant dans une fenlc longue
et étroite de largeur « praiit|uéc dans un écran opaque B, tel que cela est illustré dans la
figure 7.4 a). (D ans cette figure, la fente est p erpendiculaire au plan d e la page, et les
fionts d ’onde incidents .sont parallèles à l'écran B.} Lorsque la lum ière diffractcc atteint
l’écran d ’o bserv atio n C. les o n d es is.sues des d ifféren ts p oints d e la fente interfèrent
Cl p ro d u ise n t su r l’écran un e fig u re de d iffra ctio n co m p o sée d e fran g es b rilla n tes
et de franges som bres (les m axim a cl les mim ma de diffraction). Pour l(x;aljser les franges,
figure 7 3 Uni- ¡rliniiigriiphic de la figure o n u tilise ra une m éth o d e a ssez sim ila ire à ce lle u tilisé e p o u r lo c a lise r les fran g es
de diffiuetion produite pur un dingue. d ’une fig u re d ’in te rfé re n c e p ro d u ite p ar d eux fen tes. T o u tefo is, la d iffra ctio n pose
<)hKervez les antteaux conccntnques un plus grand défi su r le plan m alhcm atique. el les équations q u ’on p ourra trouver ne
el la lai’lie de Poisson nu eenire de la s'appliqueront q u ’aux franges .sombres.
figure Cette expérience est A vant de procéder à cet exam en, on peut quand m êm e expliquer la frange brillante
essentielIcmcnl identique It celle ce n tra le o b se rv é e d an s la fig u re 7.1 en n o ta n t que les p etites o n d es se co n d a ire s de
menée par le comité de rAcadémie H uygens issues de tous les points de la fente parcourent environ la m êm e distance pour
pour vérifier les thconcs de Fresnel, atteindre le centre de la figure et, de ce fait, .sont donc en phase lo rsq u ’elles atteignent
puisque la sphère utilisée lors de cetfe cet endroit. C om m e dans le cas des au tres franges b rillantes, on peut seu lem en t dire
expérience avait u ik section transversale q u ’elles sont approxim ativem ent à m i-chem in entre les franges .sombres adjacentes.
circulaire, comme le disque utilisé ici. Pour localiser les franges sombres, on utilisera une stratégie plus sim ple, qui consiste
à grouper en paires tous les rayons qui traversent la fente et à trouver ensuite les conditions
qui font que les ondes secondaires des rayons de chaque paire s ’an n u len t les unes les
autres. O n applique celte stratégie dans la figure 7.4 a) ¡lour locali.scr la prem ière frange
som bre, au point P,. O n su p p o se d 'a b o rd q u e la fen te e st d iv isée en d eu x zones de
largeurs égales al2. On trace ensuite, ju.squ’au point P,, un rayon lum ineux r, à p artir du
point le plus élevé de la zone d u haut, et un rayon lum ineux r2 à partir du point le plus
élevé de la zone du bas. O n trace un axe central reliant le centre de la fente à l ’écran C,
où P, est situé à un angle 0 p ar rappori à cet axe.
L es pclttcs o n d es de la paire d e rayons r^ et T2 sont en phase en q u ittant la fente,
puisqu’elles proviennent du m êm e front d ’onde qui traverse cette fente dans sa largeur.
P our prrnluirc la prem ière frange som bre, elles doivent touteft)is être déphasées de 7/2
lorsqu’elles atteignent P,; ce déphasage est dû à la différence de m arche entre les tieux
rayon.s, les petites ondes du rayon parcourant une plus grande distance que celles du
rayon r, pour atteindre P|. Pour trouver cette différence de m arche, on situe un point h
sur le rayttn de façon que la longueur du parcours entre h et P, soit égale à la longueur
d u parco u rs du rayon r,. L a d iffé ren ce d e m arche en tre les d eux ray o n s co rresp o n d
à la distance entre le centre de la fente et le point h.
L orsque l’écran d ’observation C est assez iapprrK.hé de l’écran B. com m e tians la
figure 7.4 a), il est d ifficile d e d écrire m athém atiquem ent la figure d e d iffractio n sut
l’étTan C. C et asjrect m atliéinatique peut être siinplifié considérablem ent si la tlisiance
a) D entre la fente et l’écran est beaucoup plus grande que la largeur r/ de la fente. O n peut
alors faire l'approxim ation que les rayons /-| et rj sont parallèles et form ent un angle 6 avec
l’axe central (figure 7.4 b com m e dans les autres livres). On peut aussi faire l’approxi
m ation que le triangle form é par le p o in t b. le point le plus haut de la fente et le point
central de la fente est un triangle rectangle, et q u ’un des angles intérieurs de ce triangle
est 0. L a d iffé ren ce de m arche en tre les rayons r, et (qui co rresp o n d K 'ujoiirs à la
distance entre le centre d e la fente et le point h) est alors égale à (a/2) sin 0.

Figure 7.4 a) Le.s ondes is.sues des points les plus élevés des deux zones de largeur al? subissent
une interférence destructive au point P, sur l’écran d'observation C. b) Quand O '» a. on peut
b) faire l'approximaüon que les rayons r, et rj sont panllèles et torment un angle 6 avec l'a.xc central.
7.2 La diffraction produite par une seule fente : localiser les mimma 177

O n peut rep ren d re celle an aly se av ec toute autre p aire d e ray o n s issus de points
correspondtints dans les deux zones (par exem ple, les points m itoyens des zones) et qui
se prolongent au point P^. C hacune de ces paires de rayons présente la m êm e différence
de m arche (n/2) sin 0. En p osant q ue cette d ifféren ce de m arch e co m m u n e est égale
à À/2 (la condition relative à la prem ière frange som bre), on a
a À
2 - 0 = -,

ce qui donne
a sin 0 = À (le premier minimum). (7.1)
D ans le cas d 'u n e fente de largeur a et d 'u n e longueur d 'o n d e X, l'équation 7.1 indique
l'an g le 0 de la prem ière frange som bre au-dessus et (par syinéiric) en dessous de l’axe
central.
N otez que si on com m ence avec r; > A et q u 'o n rétrécit ensuite la fente en gardant
la longueur d ’onde constante, on augm ente l'angle auquel les prem ières franges sombres
apparaissent ; l'éten d u e de la diffraction (ou l'étalem ent des rayons lumineux et la largeur
de la figure) est plus grande lorsque la fente est plus étroite. En réduisant la largeur de
a) la fente à la valeur de la longueur d ’o nde (de sorte qu e a = X). l'a n g le d e la prem ière
frange .sombre est de 90'^. P uisque les prem ières franges som bres d élim itent les deux
bords de la frange brillante centrale, celle frange brillante doit alors co u v rir la totalité
de l’écran d'observation.
O n trouve les deuxièm es franges .sombres au-dcssiis et en dessous de l’axe central
com m e on a trouve les prem ières franges som bres, sauf q u 'o n divise m ainienam la fcnic
en (jiiatre zo n es de la rg e u rs ég a le s « /4 , co m m e on l'in d iq u e d an s la lig u re 7 .5 a).
O n prolonge ensuite les rayons r,. r,, Tj et r 4 issus de.s points les plus élevés dos zones
ju s q u 'a u point P^, so it la p o sitio n de la d eu x iem e fran g e som bre au d essus de l'a x e
central. Pour produire cette frange, la différence de m arche entre rj et r^. celle entre t 2
et r,. et celle entre /y et /y doivent toutes être égales à X/2.
Lorsque D » n. on peut faire l'aiiproxim ation que ces quatre rayons sont parallèles
et fo m e n t un angle 6 avec l ’axe central Pour trouver kair différence de marche, on trace
une ligne perpendiculaire entre chaque paire de rayons adjaeenis. coiTinie le m onlrc la
figure 7.5 b), et on form e ainsi une série de triangles reclangles. la différence de m arche
fo rm an t un cô té d e ch acu n d e ces tria n g les O n voit d a n s le trian g le d u liaut q u e l.r
différence de m aiehe entre r^ et <y corrc.spond à (n/4) sin 6. De façon sim ilaire, on voit
dans le triangle du bas que la différence de m arche entre ry et r^ correspond égalenictu
à (n/4) sin 0. En fait, la différence de m arche entre chaque pwre de rayons issus de jniiiiu
c o rresp o n d a n ts d an s deux zo n es ad jacen tes est d e (« /4 ) sin H. P uisque la d iffé ren ce
I II) de m arche est égaie à Xf2 dans cliaque cas, on a
Figure 7.S a) Les ondes issues des points a . ^ X
les plus élevés des quatre zones
de largeur a!A subissent une inieriétenee ce qui donne
destructive au point b) Quand
O sin 0 = 2X (le diHixiènic miniilimn). (7.2)
D S> a. i>n peut faire l'approximation
que les rayons r,. rj, et ri -sont On pourrait continuer ainsi à localiser les franges sombres dans la figure de diffraction
parallèles et forment un angle en divisant la fente en un pías grand nombre de zones de Uugenrs égales ( )n )m iul loiijouis
avec l’axe central. un nombre pair de zones jxrur ainsi p n iv o ir regrouper les zones (et leurs ondes) en paire.«,
com me on l'a fait préeédemnxml. I n tel procédé m onta' que les fi:anges sombres nii-dessiis
et en dessous de Taxe central peuvent être loealisées à l’aide de cette équation générale

a sin 0 — tnX. lorsque m = i 1, J:2 . .i 3 . ... (les minima - frangc.s sombres). ( 7..3)

Les vaknins positives de m d écrivent les m inim a au-de.ssus de l'a x e central cl le.s
v aleurs n eg a tiv e s de m d é c riv e n t les m in im a .sous l'a x e ce n tral V ous p o u v ez vou.s
rap(icler l'éq u aiio n donnant les franges som bres de la fa(,-nn siiivanle Tracez un triangle
com m e celui de la figure 7.4 h ), m ais en fn neiion de la fente en lièrc de larg eu r a, Ct
notez que la d ifférence de marr lK* en tre le rayon du dcs.sus ct celui du bas. tous deux
Lssus de la fente, est égale à a sin P. L’équation 7.3 vous indique donc ce qut .suit.

► Dans une expérience .sur la diffraction pn'xiuitc par une seule tente, des l'ranges .sombres
sont produites <Hix endroits où la différence de marche (n Sin fl) entre le r ayon du haut et td u i
(In bas est égale à '"À ..
178 Chapitre 7 La diffraction et la polarisation

C ela peut sem bler inexact, puisque les ondes de ces deux rayons particuliers seront en
p hase parfaite lorsque leur d ifféren ce d e m arche sera un n o m b re en tier de longueurs
d ’onde. Toutefois, ces ondes font toujours partie d ’une paire d ’ondes en opjtosition de
phase l’une par rapport à l’autre ; il s ’ensuit donc que chuciine de ces ondes sera annulée
par une autre onde, et il en résultera de l'obscurité.
Les équations 7.1, 7.2 et 7.3 ont été dérivées en fonction des situations où P ^ a.
E lles s ’appliquent égalem ent si on place une lentille convergente entre la fente et l’écran
d ’t»b.sci vaiion, et q u 'o n déplace ensuite l’écran pour le faire coïncider avec le plan focal
de la lentille. La lentille fait en sorte que les rayons qui atteignent un point quelconque
de l’écran sont m aintenant parfaitement parallèles (et rurn plus de manière approxim ative)
au niveau de la feule. Ils sont com m e les rayons initialem ent parallèles de la figure .“1.12 a),
qui convergent au foyer d une lentille convergente.

^ VÉRIFIEZ VOS CONNAISSANCES 1 On produit une figure de diffraelion sur un écran


d’obscrvaiioii au moyen d’une longue feme mince éclairée par une lumière bleue. La figure
s’'étend-clle en s'éloign.int du centre brillant (les maxima el les minima se déplaçant à p.irtir
du centre) ou s’en rapproche-i-elle a) si on remplace la lumière bleue par une lumière jaune
et b) si on diminue la largeur de la fente?

Exemple 7.1
Une fciilc de largeurs est éclairée par une lumiéiie blanche (composée SOIUTION Dans celle situation, le contepi dé est que, pour une longueur
de toutes les longtiruis d’onde de la lumière visible). d ’onde donnée, le premier maximum secondaire c.st environ à
mi-chcmin entre le premier et le deuxième minimum corrc.spondani
a) À quelle valeur de a k piemier niiniinum sc rapporiain à une luiniètc
à cette longueur d ’onde. L'équation 7.3 permet de localiser cc.s deux
rouge de longueur d’onde k — 650 nm apparaîtra-l-il àfl = 15“?
minima si on établit que, respectivement, m = I et ni = 2. Le prcmiei'
SOIUTIOK: Le tomepi rla, dans ce cas, est que la diffraction .se produit maximum secondaire peut ensuite être localisé approximativement
séparément |x»ur chaque longueur d’onde de la gamme <les longueurs si on pose que m = 1,5. L’équation 7.3 devient alors
rl’tmde qui traversent la fente, et qu’on peut lœaliser les minima de
chaque longueur d’onde à l’aidc de l’équation 7.3 (« sin fl = mk). û sin fl = 1.5 k'.
Lorsqu'on pose m = I (pour le premier minimum), et qu'on substitue
les valeurs données à fl et à à . l’équation 7..t donne En réiolvani l’équation pour trouver k' et en insérant les valeurs con­
nues, on obtient
mk (l)(650nin)
= 2511 nm î» 2,5 /im . (réponse)
sin fl sin 15“ a sin fl (2 511 n m )(s in l5 )
—^ — =430nm . (réponse)
Pour que la lumière incidente s ’étale autant (à 2:15“ au premier
rninimuni). la fente doit cire très mince elle doit équivaloir à environ
La lumière de celte longueur d'onde est violette. Pour une lumière de
quatre fois la longueur d'onde. F.n comparaison, un cheveu humain a
cette longueur d'onde (4.30 nm), le premier maximum secondaire
un diámetro d ’environ 100/tm.
coineidora toujours avec le premier niiniinum de la lumière d'une
h) Quelle CM lu longueur il’onde à' d'une lumière dont le premier longueur d'onde de 650 um. peu importe la largeur ilc la fente. .Si elle
maximum .secondaire est à 15“. ce qui le fait coïncider avec le premier est relativement éd-oitc. l’angle fl auquel ce chevauchement siirvicni
minimum de la lumière rouge? sera relativement grand, et vice versa.

7.3 L’Intensité lumineuse dans une figure


de diffraction produite par une seule fente :
approche qualitative
D ans la section 7.2, on a appris à lo caliser les rninim a et les m axim a clans une figure
(le diffraction p ro d u ite p a r une seu le fente. O n ab o rd e m ain ten an t un pro b lèm e plus
général: trouver une expression pour l'in ten sité / de la figure de diffraction en fonction
de 0. la position angulaire d 'u n point sur l’écran d'o b serv atio n .
Pour ce faire, on divise la fente de la figure 7.4 a) en N ¿ones de largeurs égales iLt,
assez petites p o u r qu e c h a q u e zone ag isse co m m e une so u rce p o n ctu elle d 'o n d e s
secondaires de Muygens. On veut superposer les p etites ondes qui arrivent à un point
arbitraire P de l'é c ra n d 'iib serv atin n . à un angle fl avec l'a x e central, afin de pouvoir
déterm iner fa m p litu d e £ „ de la com posante du ch am p électrique de fo n d e résultante
ît P L’intensité de la Im nière D P est alors proportionnelle au carré de' ccü e am plitude.
7 3 L'intensité lumineuse dans une figure de diffraction produite par une seule fente ; approche qualitative 179

figure 7.6 D ia g r a m m e s d e s ve cte u rs


de Fresnel prair W — 18 vecieurs
de Fresnel, ce qui correspond P our trouver E,„ on d o it connaître la relation de pha.se entre les ondes secondaires
à la division d ’une tente simple qui atteignent un point donne. La différence de phase entre les petites ondes issues de
en 18 zones. Les amplitudes résultantes zxincs adjacentes est donnée par
En sont indiquées en tbnetion
a> du maximum central à F = 0, /d iffcren c e\ _ / 2t r \ /diffi
différcnce\
b) d4m point sur l’écran .situé V de pha.se / “ i T M d c nm arche/
à un petit angle 0 par rapport à l’axe
central, c) du premier minimum D ans le cas d ’un p o in t P à l ’an g le 0, la d iffé re n c e de m arche en tre d es o n d es
et d) du premier maximum secondaire. secondaires is.sucs de zones adjacentes e.st ¿ ir sin F; la différence de phase At/i entre les
ondes .secondaires issues de zones adjacentes est donc

(7.4)
(A \ sin в).
- - ( t )
On suppose que les ondes secondaires qui arrivent à P ont toutes la m êm e amplitudi' A t.
P our trouver l ’am plitude tf, de l’onde résultante à P, on additionne les am plitudes A E
en u tilisa n t la m éthixle des vecteu rs d e F resnel. P o u r ce faire, on fait un d iag ram m e
de N vecteurs de Fresnel, ch acu n co rresp o n d an t à F o n d e seco n d aire issu e de chaque
zone de la fente.
R elativem ent au point Ff, à 6 = 0 sur l’axe cenlral de la figure 7.4 a), l’équation 7.4
indique que la difléren ce de phase A</> entre les ondes secondaires est n u lle : les ondes
secondaires qui arrivent sont donc toutes en phase \j î figure 7.6 a) est le diagram m e des
veeteiirs de F resnel corresp o n d an t ; les vecteu rs de Fresnel adjacents rep réseu ien t les
ondes secondaires issues des zones adjacentes et semt tracé» les uns à la suite des autres.
C om m e il n 'y a aucune différence de phase entre les ondes secondaires, F angle est nul
entre chaque paire de vecteurs d e Fresnel adjacents. L’am plitude E„de F onde nette à
est la som m e v ec to rielle d e ce s v ecteu rs de F resn el. C ci a rra n g e m e n t de vecteu rs
de Fresnel est celui qui donne la plus grande valeur à Fam plitudc Eÿ. O n appelle cette
valeur E ^ ; £ „ est donc la valeur de £« lorsque 0 = 0 .
O n peut m aintenant considérer un point P formanl un petit angle 0 avec F axe central.
L ’équation 7.4 indique ici que la diflérence de phase entre les ondes secondaires issues
de zo n es a d jac en te s n ’est plus n u lle La fig u re 7 .6 h) rep rése n te le d ia g ra m m e de
vecteurs de Fresnel correspondant ; com m e aupaiavani, les vpt ieurs rie Fiesnt I soni Irm és
les uns à la suite des autres, mais il y a m aintenant un angle A</> entre les vecteurs de Fresnel
ad jac en ts. L 'a m p litu d e à ce n o u v eau point e s t en c o re la som m e v e c to rie lle des
vecteurs de Fresnel, m ais elle est plus petite que celle de la figure 7.6 a), ce qui signifie
que l ’intensité de la lum ière est m oindre à ce nouveau jHiint P q u ’elle ne fc ta il à P q.
Si on continue à augm enter la valeur de fi, Fangic A<]>entre les vecteurs de Fresnel
adjacents augm ente égalem ent, et la chaîne de vecteurs de Fresnel foi m e finalem ent un
cercle où l’extrémité du dernier vecteur de Fresnel rejoint l’origine du premita (figure 6 r)
[.'am p litu d e Ef, est m aintenant nulle, cc qui sig n ifie ()uc F in ten sité de la lum ière est
égalcm enl nulle. O n a aiieinl le prem ier minimiini. ou frange som bre, dans la figure de
diffraction. Il y a m aintenani une d ifférence d e phase ilc 27t rad entre le prem ier e t le
dem ie! vecteur de Fresnel, ce qui signifie que la différence de m arche entre les myon.s du
haut de la fente et ceux du bas de la fente correspond à une Inngiiciir d'onde. Rappu-lez-vous
q u e c ’est la co n d itio n q u ’on avail fixée pour le p rem ier m inim um d an s ta fig u re de
diffraction.
Si on continue à augm enter la valeur de fi. F angle A</> entre les vecteurs de I rcsncl
ad jac en ts c o n tin u e aussi à au g m en ter, la ch a în e d e v ec te u rs de F resn el co m m en ce
Chapitre 7 La diffraction et la polarisation

à SC referm er sur eUc-même, et l’enroulement résultant com m ence à rétrécir. L’amplitude £<,
augm ente alors ju.squ’à la valeur maximale dans l’arrangement illustré dans la figure 7.6 d).
Cet airaiigem ent correspond au prem ier maximum secondaire dans la ligure de diffraction.
Si on augm ente un peu p lu s la valeur de ft le rétrécissem ent observe dans l’enrou­
lem ent a pour effet de dim in u er c e qui signifie qu e r in tc n s ilc dim inue égalem ent.
L orsque l’augm enlalion d e B e st suffisante, l’ex trém ité du d ern ier v ecteu r de Fresnel
rejoint de nouveau l'o rig in e du prem ier. On a alors attein t le d eu xièm e m inim um . On
pourrait continuer à exploiter cette m éthode qualitative de déterm ination des m axim a et
des m in im a d an s la fig u re de d iffra c tio n , m ais on u tilise ra m a in te n an t p lu tô t une
m éthode quantitative.

✓ VÉRIFIEZ VOS CONNAISSANCES 2 : Les figures ci-co.«re


rcprcscntenl. d:ui.s une forme plus arrondie (avec davantage
de vecteurs de Fresnel) que celle de la figure 7.6, les diagrammes
de vecteurs de Fresnel correspondant à deux points d ’une figure
de diffraction, situés de ptirl et d’autre d’un maximum donné,
ai I»)
a) De quel maximum s'agit-il ? b) Quelle est la valeur approximative
de m (dans l’équation 13) qui correspond à ce maximum?

7.4 L'Intensité lumineuse dans une figure


de diffraction produite par une fente
simple : approche quantitative
L ’équation 7.3 indique com m ent localiser les m inim a de la figure de ciiffraction jiroduite
p ar une fente sim p le su r l'é c ra n C de la fig u re 7.4 a), e t ce, en fo nction de l’angle B
q u ’on y trouve. O n veut m aintenant dériver une expression p o u r décrire l’intensité 1(B)
de la figure de diffraction, en fonction d e ft On dém ontrera ci-dessous que l’intensité
est donnée par

/s i n » V
m — , (7.5)
a) \ a J

4> Tta
elacK'c ou cr = _ = sin fi. (7.6)
A.

L e sy m b o le cr est sim p lem en t un lien p ratiq u e en tre l'a n g le B qui situ e un point
sur l’é c ran d ’o b se rv a tio n e t l ’in ten sité d e la lu m ière 1(B) à ce p o in t. P ar ailleu rs. 7„
est la plus grande \ alcur d e l’intensité 1(B) cl survient au m axim um central (ofi B — 0).
e t 4>est la différence d e phase (en radians) entre le rayon du haut et celui du bas issus
de la fente de largeur a.
-2 0 -ir> - 1 0 - 5 0 r, L 'exam en de l’équation 1.5 m ontre q u e les m inim a d ’intensité surviendront où
6 (d e g r é s )
•>) a = »CT, lorsque m — ± 1 , ± 2 , ± 3 . .. . (7.7)

f ImensitélrclaLive Si on insère ce résultat dan.s l’équation 7.6, on peut déterm iner que
r \.(.h ' t---
1 i 7K7
1 ' r>/ \ f m n = — sin B, lorsque »t = ± I , ± 2 . ± 3 , .. .
r -! A
- i i (l(tf
ou a .sin B = CTÁ, lorsque w = ± I , ± 2 , ± 3 . .. .
- l f.-— --
t ¡ T (les ininimti - franges sombres). (7.8)
; _ ! 1' -f
¡h ;2 -
ce qui correspond exactement à l'équation 7.3, l’expression q u 'o n a dérivée précédem ment
20 -15 -10 - 5 t) 5 10 1 ivnir localLser les m inim a.
e ( d e g i és)
La figure 7.7 représente graphiquem ent rintcnsilé d ’une figure de dififaction produite
O p a r u n e fen te sim p le, c a lc u lé e à l’aid e d es é q u a tio n s 7.5 e t 7 .6 en fo n ctio n d e trois
Figure 7.7 L'intensité relative dans largeurs de fente : n = À, ci = 5X et n = IOà . N o te / que, lorsque la largeur de la fente
la diffiaction causée par une tente augm ente (par rapport à la longueur d ’onde), la largeur du maximum c e n m d tk diffrcctinn
simple, en foiKtion de tmis valeurs (la partie ccnliale du graphique) dim inue, ce qui signifie qu e la tente produit un m oins
du lappori iifX. Plus la feme est large, grand étalem ent d e la lum ière. La largeur tfes m axim a secondaires dim inue égalem ent
plus le maximum central est étroit. (et les maxima deviennent plus faibles) D ans la lim ite d ’une largeur rie fente n bcaiK-oiip
TA L'Intensité lumineuse dans une figure de diffraction produite par une fente simple : approche quantitative 181

plus g rande q u e la lo n g u eu r d ’o n d e Л, les m ax im a seco n d aires p ro d u its p ar la fente


disparaissent ; on n ’a donc plus de d iffractio n cau sée pttr une fente sim ple (m ais o n a
encore une diffractitHi causée p ar les bords d ’une fente large, com m e celle produite par
les bftrds de la lam e d e rasoir de la figure 7.2).

Démonstration des équations 7.5 et 7.6


L ’arc de v ecteu rs d e F resn el d e la fig u re 7 .8 rep rése n te les o n d es se co n d a ire s qui
alleignenl un pttinl arbiiraire И su r l’écran d ’o b serv atio n d e la figure 7.4. point qui se
situe à un petit angle в. L’am plitude ¿'«de l’onde résultante à f est la som m e vectorielle
de ces vecteurs de Fresnel. Si on divise la fente de la figure 7.4 en zones infinitésim ales
de largeur Д.х, Гаге de vecteurs de Frc.snel de la figure 7.8 se rapproche d 'u n arc de cercle ;
son rayon est R, com m e on l’indique dans cette figure. La longueur de l'a rc doit être
l’am plitude au centre de la figure de diffraction, puisque, si on redressait l ’arc potir le
rendre d ro it, on o b tien d rait Г arran g e m en t de v ecteu rs de F resnel de la figure 7 .6 a)
(illustré en couleur plus pâle dans la figure 7.8).
Figure 7.8 La construction utilisée
L 'an g le Ф dans la partie inférieure de la figure 7.8 est la différence de phase entre
pour calculer l'intensité dans une figure
les vecteurs infinitésim aux des extrém ités gauche et droite de l'a rc Г„,. D ans une pers­
de difftuction prtxluiie par une fente
pective géom étrique, </)csl égalem ent l’angle entre les deux rayons R de la figure 7.8.
simple. La situatiiHi illustrée correspond
L a ligne pointillée, bissectrice de ф. form e alors deux triangles rectangles congruents.
à celle de la figure 7.6 b).
O n peut exprinx-'r l’un ou l’autre de ces triangles sous la form e

sin ■h a 9)
2R'

M esuré en radians, vaut (en considérant com m e un arc circulaire)

, Em

E n isolant R et en substituant le résultat dans l’cquntion 7.9. on obtient

(7.10)
2

C om m e o n l’a vu dans la section 4.4. l’inicnsiié d’une ontlc électrom agnétique est
proportionnelle au ca rré de l’am plitude de son cham p électrique. D ans le cfis présent,
cela signifie que l’in laisité maxim ale /„ (qui sui \ ienl au centre de la figure ilc diffraction)
est p ro p o rtio n n elle à £ ^ , et qu e l ’in ten sité ¡(6) à l’an g le R est p ro p o rtio n n elle â E l
On |rcul donc écrire

m ^ E i
(7 .И )
El

En substituant l'éq u atio n 7.10 à £(, et en insérant ensuite a = on obtient l'ex p ressio n
suis ante pour décrire l’intensité en fonction de :

/s in « Y
К в) /,

C ola co rrespo n d exactem ent à l’équation 7.5, une des d eux équations dont on voulait
faire la dém onstration.
L 'au tre éx|ualion q u ’on v a il dém ontrer relie a h 0. Ijï d illcren ce de phase 4> entre
les rayons i.ssus du haui et du bas d e la fente peut être reliée à la difference de m arche
en utilisant l’équation 7.4, qui indique que

(o sin ft).

où « est la somm e des largeurs A.t des zones infinitcsimalcs. On note toutefois que <i> ~ 2a,
et cette équation se réduit à l’équation 7.6.
182 Chapitre 7 La aiffroction et ta |x>larisation

✓ VERIFIEZ VOS CONNAISSANCES 3:


Deux longueurs d’onde de 650 nm (rouge)
et 430 nni (violet) sont utilisées scparénient
dans une expérience sur la diffraction
produite par une fente simple. La figure
ci-conlre est une représentation graphique
de l'intensité / en foiK-iion de l’angle b.
pour les deux ligures de diffraction.
Si on utilise ensuite les deux longueurs o
d'onde sinuiltanéiiient. quelle couleur
observera-t-on dans la figure de diffraction
combinée a) à l'angle A et bi à l’angle B ?

Exemple 7.2
I rouvey lus intensité.s des trois premiers maxima secondaires dans la / /s in a V A in (« i-f 5 )7rV
figure de dirrntetiun de la figure 7.1 mesurées par rapport à l’intensité — = ( ------) = | ----- r=— ) , loi-squc w = ' ’"l, ± 2 , ± 3 , ...
/m V “ / \ (r>i + \)TT )
du m-aximum central.

SOLUTION; Le premier torcopl dé utilise ici est le suivant : les maxima Le premier maximum secondaire survient à m 1. et son iiilcnsilc
secondaires se trouvent approximativement à mi chemin entre les relative est
iniiiiiiia. dont les piisitioiis angulaires sont données par l'équation 7.7 Zl = ^ t n l.S/rV
(a — mn). Les ixi.siiions des maxima secondaires .vnii alors données
Tm \ (I + j)rr
j)7T ) \ l.5rr j
(approMiiiativcmcnt) ixir
= 4.5UX 10-2 = 4.5%. (réponse)
ni — (m ■ lorsque m = I, ±2, ± 3 .......
Quand m = 2 et m -= 3. on ilétermine que
où U est iiicsuic en r.idiniis.
Scion un autre (ontepldé, on peut a lic r l'imen-silé / de tout point /
de la figure île diffraclipti h rimeiisiié du maximum ccntial à l’aide = 1.6 % et — = 0.K3 %. (repoase)
/n
de l’étiualioii 7 5. Il s’ensuit donc qu’on peut substituer les valeurs
npimixiinaiives de ni aux maxima secondaires dans l’équation 7 5. L’intensité des maxima secondaires suivants diminue rapidement.
et obtenir les intensités relatives à ces maxima On a La figure 7.1 a été surexpo.séc délibérément afin de mieux le.s illastrei.

7.5 La diffraction produite par une ouverture


circulaire
O n e x a m in e ra maintenBiU la d iffra c tio n c a u sé e par une o u v ertu re c irc u la ire — une
o u verture com m e celle q u ’on trouve d an s une len tille circ ulaire à trav ers laquelle la
lum ière peut pa.sser. La figure 7.9 représente l’im age d 'u n e source lum inease ponctuelle
éloignée (une étoile, par exem ple) form ée sur une pellicule photographique placée dans
le plan focal d 'u n e lentille convergente. C ette im age n ’est pas un pt^int. conclusion à
laquelle l’opliq u c g éo m étriq u e p o u rra it m ener, m ais un d isq u e en to u ré d e p lu sieu rs
anneaux secondaires d o n t l'in te n sité d im in u e p ro g ressiv em en t. .Si on co m p are celte
im age avec l.'i figure 7.1, on a tout lieu d e cro ire q u ’il s ’agit bien d ’un p hénom ène de
d iffra ctio n . D an s le c a s p rése n t, to u te fo is. l ’o uverU ire est un c e rcle d e d ia m è tre d,
et non une fente rectangulaire.
L ’analyse d ’une figure de diffraction de ce type s’avère com plexe ; elle dém ontre
cependant q u e le p rem ier m inim um d an s un e fig u re de d iffra ctio n p ro d u ite p ar une
figure 7 9 La figure de diffraction ouverture circulaire de diam ètre d peut être localisé à l’aide de
produite poi une ouverture circulaire.
Observez le maximum central sin = 1,22 - (le premier minimum, ouverture circulaire). (7.12)
et lc.s maxiiiiA sccondaims circulaires d
L’image a été surexposée afin de mettre L 'angle 0 e st ici l’angle form é entre l’axe central et tout point sur ce m inim um (circulaire).
en évideiK-e ces maxima .secondairc.s, O n peut com parer cela avec l’équation 7 . 1,
qui sont beaucoup moins intenses
que le maximiiin cemral. sin 0 = — (le premier mininnun, fente simple), (7 .1 3 )
a
qui p erm et de localiser le prem ier m inim um dans le cas d ’une longue fente étro ite de
largeur a 1^ différence principale est le facteur 1,22, qui intervient à cause de la form e
circulaire de l’ouverture.
7 5 La diffraction produite por une ouverture circoVal»e U3

Figure 7.10 La partie supérieure


tie nilustration représente les images
de deux sources ponctuelles
(des étoiles) formées par
une lentille convergente.
Daas le bas de la figure, on trouve
les représentations des inteasités
lumineuses de ces images.
En a), la séparation angulaire
des .sources est trop petite pour
qu'on puis«: les distinguer. En b),
on peut à peine les distinguer et, en c),
on peut claireiiwni les distinguer.
Les exigeiKes relatives au critère
de Kaylcigh sont satisfaites en b),
où le maximum central d’une figure
de diffraction coïncide avec
le premier minimum de l'autre figure.

La résolution
Le fait que les im ages produites par des lentilles constituent des figures de diffraelion a
son im portance lo rsq u 'o n souhaite séparer (distinguer) deux objets ponctuels éloignés
p résentant une faible sép aratio n angulaire. L a figure 7 .10 m ontre trois cas d ifferen ts
d'apparence visuelle, et les ligures rcpréscniam la distribution de l’intensité lum inctise
de deux objets éloignés (des étoiles) présentant une faible séparation angulaire. D ans la
figure 7.1 0 a), les o b je ts ne sont pas sépares, à cau se de la d iffra c tio n ; leurs fig u res
de d iffra ctio n (su rto u t leu r m ax im u m cen tral) se ch ev au c h en t, et les d eux o b je ts ne
p eu v en t être d istin g u és. D ans la fig u re 7 ,1 0 b), les o b je ts sont to u t ju s te sép arés,
et ils sont com plètem ent séparés dans la figure 7 .10 c).
D ans la figure 7.10 b), la séparation angulaire des deux sources ponctuelles lait en
sorte que le m axim um central d 'u n e stnircc est centré sur le prem ier m iniinuni de l’aulrc
source ; cette condition se nom m e le c ritè re d e R ayleigh. D ’après I équation 7.12. deux
objets qui sont tout ju ste sépares suivunt ce critère doivent avoir une séparation an g u ­
laire 0^ de

(>n — sin ‘
(T)
Pui.sque les a n g les sont p e tits, o n p eu t rem p lace r *in fip p a r où Or est ex p rim é
en radians.

X
Or - 1,22 (le cnière de Rayleigh). (7.14)

Le critère de Rayleigh n’est q u ’apjiroxiinatif. puisque la résolution déjieiKl de iinmliieiix


facteurs tels q ue la b n ilan cc relative des sources et de leur environnem ent, lu lurliuleiice
de l ’a ir entre les so u rces et l’o bservateur, cl le fonetitm iiem cnt du systèm e vi.suel d e
l’o bservateur. D es résu ltats ex p érim en tau x d ém o n tren t q u e la p lu s p etite séparafion
angulaire p erm ettan t la résolution est généralem ent un peu plus gran d e que la valeur
donnée par l’équation 7 14 roiilciois, à des fins de calculs, ou con.sUlércra ici l’ctjuahon 7.14
com m e un critère précis. .Si la séparation angulaire 0 entre les sources est plus grande
que Or , on peut distinguer les sources ; si elle est plus petite, on ne fieui les distinguer.
l.c cn ière de R ayleigh peut expliquer la perception des couleurs ém anaiii du tableau
rie S eurat. Un dinumehe après-midi à l'île de la Grande Jatte (ou de tout autre tableau
peint ,’k l'aidc de la tet:hniquc <lu pointillisme) Lorsque vous êtes assez près de la |.viimiic.
les séparations angulaires 0 des points adjacents sont plus grandes que Or . et les txiinLs
peuvent être vus individuellement l eurs couleurs sont les couleurs de la peinture utilisée
par Seurat. Toutefois, si vous êtes a.ssez éloigné du tableau, les séparations angulaires 0
sont plus p etite s qu e Or . e t les p o in ts ne p eu v en t alo rs être vu.s in d iv id u ellem e n t.
Le m élange de couleurs résultant qui parvient à vos yeux en provenance de tout gm ujic
de points peut alors inciter votre cerveau à « fabriquer« une couleur corresjKUidani is ce
184 Chapitre? La diffraction et la polarisation

groupe - une cou leu r qui n ’ex iste p eu t-être pas réellem ent d an s ce groupe d e points.
A insi, S eurat se sert de votre systèm e visuel pour créer les couleurs de ses œ uvres.
Loi squ’on utilise une lentille au lieu d e ses yeux ptaii distinguer des objets présentant
une faible .séparation angulaire, on souhaite ob ten ir la p lu s petite figure de diffraetion
possible. D ’après l’éq u atio n 7.14. on p eu t y arriv er en au g m en tan l le d iam ètre de la
lentille ou en utilisant une lum ière d 'u n e plus petite longueur d 'o n d e.
C 'e s t p o u r celle raiso n q u 'o n u tilise so u v en t la lu m ière u ltra v io le tte lo rsq u ’on
travaille avec un m icroscope ; à cause de sa {xlitc longueur d ’onde, ecltc lum ière perm et
l,î d ’exam iner des détails q u ’on ne pourrait voir avec le m êm e m icraseopc si on utilisait la
lum ière visible. Dans le chapitre 9. on dém ontre que, dans certaines circonstances, les
fai.sccaux d 'électro as se com portent œ m m e des ondes. D ans un microscope électronique,
ces fai.sccaux peuvent avoir une longueur d ’onde effective valant 10 foi.s lu longueur
d 'o n d e d e la lum ière visible. Ils perm ettent un exam en détaillé de .structures m inuscules,
co m m e c e lle de la fig u re 7 .1 1 . q ui ap p a ra îtra it flo u e si o n utili.sait un m icro sco p e
optique.

/VÉR IFIEZ VOS CONNAISSANCES 4: .Supposez que vous pouvez à peine distinguer deux points
Hgur»7.ll Une micropraphic d’une veine rouges à cause de la diffraction pnxluiic par la pupille de vo.s yeux. Si on augmente l’intcnsitc
сошепит (les globules rouges, obtenue de l’cclairagc ambiant autour de vous et que le diamètre de vos pupilles diminue, votre pouvoir
Й I'nidi- гГип inicroscoiJC élcclrnniquc de résolution augmente-t-il ou diminue-t-il Tenez seulement compte de la diffraction.
(Vous pouvez vérifier expériraentalemeni votre réponse.)

Exemple 7.3
Une lentille convergente circulaire, de diamètre d = .12 niiii et de F.rran sinié
distance fo ca le/ = 24 cm. forme des images de sources ponctuelles
éloignées dans son plan focal. On utilise une lumière de longueur
d’onde K = i5 0 nm.

a) En tenant compte de la diffraction causée par la lentille, quelle


.séparation angulaire les deux sources ponctuelles doivent-elles avoir
pour satisfaire aux exigences du critère de Rayleigh ?

SOLUTION; La figure 7.12 montre deux sources ponctuelles éloignées


Pi et A , la lentille, et un écran d’observation situé dans le plan focal Figure 7.12 Exemple 7.3 Une lumière issue de deux sources
de la lentille. On voit également, dans la partie droite de la figure, une ponctuelles éloignées Pj et traverse une lentille convergente
représenlalnm graphique de I’intcnsiic I en fonction de la position et forme des images sur un écran d’observation, dans le plan focal
sur l’écran du maximum central des images formées par la lentille. de la lentille. Un seul rayon représentant chaque source a été
Notez que la séparation angulaire des objets e.si égale à la séparation illustré. Les images ne sont pas des points, mais bien des figures
angulaire des itnages. On utilise ici le concept dé suivant : si les images de diffraction, dont les intensités approximatives sont représentées
doivent satisfaire aux exigences du critère de Rayleigh concernant la dans la partie droite du graphique. La séparation angulaire
résolution, il s’ensuit que les sép-irations angulaires des deux c6tc5s de des objets est 6„. et celle des images est 6, ; les maxima centraux
la lentille doivent être données par l’équation 7.14 (en supposant que des images sont séparés par une distance Ax.
les angles sont petits). En insérant les données connues, on obtient,
à l’aide de l’équation 7.14,
SOLUTION' r.e concept tié, dans ce cas, consi.sce à relier la distance A.v
» „= e, = 1.2 2 - à l'angle в,, que l'on ccvinaît maintenant Dans la figure 7.12, on peut
voir dans l’un cxi l’auire des triangles formés entre la lentille et l’écmn
(1.22)('550x 10-'* ni)
— = 2.1 X 10 lad. (réponse) d’observation que Й./2 ~ XxH2f). En réarrangeant ce rc.siiltut et en
32 X 10г 3 ,
faisant l'appniximation que tan U « on peut déterminer que
À cause de celte séparation angulaire, le maxinuim central de chacune
A x -/9 „ (7.15)
des deux couibes d’intensité de la ligure 7.12 est centré sur le premier
miniimiin de ratilre courbe où Sj est mesuré en radians. En insérant les données connues, on
obtient easuite
b) Vucllc c.st la distance A\ entre les ccnUcs des iinages dans le plan
focal (ou la distance entre les pics centreuvi des deux courbes) A ï =• (0.24 mK2,l X 10 ’ rad) = 5 ,0 //m. (réponse)
7.6 La diffraction produite par deux fentes 185

7.6 La diffraction produite par deux fentes


D an s les e x p é rien c es su r l’in te rfé re n c e p ro d u ite p a r d eux fen tes du c h a p itre 6 , on
su p p o sa it im p licitem en t q u e les fen tes étaien t très étro ites p a r rap p o rt à la lo n g u eu r
d ’tm de de la lunùère qui les éclairait ; ce qui revenait à d ire qu e n <К Я. A vec des fentes
aussi étro ites, le m axim um ce n tral d e la fig u re d e d iffractio n d e l’une ou l’au tre des
fentes couvrait la totalité de l ’écran d ’observation. D e plus, rin te rfé re n c c de la lum ière
issue des deux fentes prtxluisait des franges brillantes qui avaient approxim ativem ent la
m êm e intensité (figure 6.9).
l.o rs q u 'o n utilise la lum ière visible, toutefois, on ne ren co n tre pas toujours celle
condition (a <K /.). D ans le cas de fentes relativcm cnl larges, Г interference de la liimièrc
issue des deux fentes produit îles franges brillantes qui n 'o n t pas toutes la m êm e intensité,
fcn effet, dans une figure d ’interférence produite par deux fentes, I'inlcnsitc des franges
est m odifiée par la diffraction de la lum ière qui traverse chaque lente.
Par exemple, le graphique de la distribution de l’intensité lumineuse de la ligure 7 .13 a)
représente la figure d 'in te rféren c e créée par deux fentes qui sc produirait si les fentes
étaient infinim ent étroites (o <S: A); toutes les franges d ’interférence brillantes auraient
la m êm e intensité. Le graphique de la figure 7.13 b) correspond à la diffraction causée
par une fente .simple ; la figure de diffraction com porte un m axim um central large ci des
m axim a secondaia-s plus faibles à ± 17". Le graphique de la figure 7.13 c) représente la
figure d ’interférence produite par deux fentes réelles. C e graphique a été fait en utilisant
la figure 7.13 b) com m e une enveloppe du graphique de l'in tcn silé de la figure 7.13 a).
Les positions des franges n’ont pas changé ; seule l’intensité a été affectée.
L a fig u re 7 .1 4 a) rep rése n te une lig u re rée lle d an s la q u elle on voit clairem en t
l’interférence produite par deux fentes et la diffraction. Si une fente est co n v en e, il en
résulte une figure de diffraction produite par une seule fente, illustrée dans la figure 7.14 b).
N otez les correspondances entre les Figures 7.14 a) et 7 . 13 c), et entre les figures 7.14 b)
et 7.13 b). En c o m p aran t ces fig u re s, g ard e z à l ’e sp rit q u e les p h o to g rap h ies d e la
figure 7.14 ont été volontairem ent surexposées afin de m ettre en évidence les m axim a
secondaires, qui étaien t pâles, et q u e deux m axim a secondaires sont illusircs, au heu
d ’un seul.

b)
Rqure7.13 a» Tiraphique de la disiiihution de l’imeasité lumineuse prévue
par rinierférenee produite par deux fentes infiniment étroites.
b) Oraphiqiie de la distribution de l'intcnsitc lumineuse dans une figure
de diffraction créée par une fente de largeur a (non infiniment étroite).
c) Oraphiguc de la distribution de riniensité lumineuse prévue en foiatiioii
de deux fentes de largeur a. La courbe de lu ligure b) agit comme une
enveloppe qui délimite l’intensité des franges de la Figure à deux fentes
de la figure a). Ntilez que les premiers minima de la figure de difft action
de la figure b) éliminent les franges de la figure produite par deux fentes
qui pourraient apparaître à envirtm ± 12 dans la figure c).

<■)
186 Chapitre 7 La diffraction et la polarisation

Figure 7.14 a) Franges cTinterférence

II
dans un sy.stènie réel à deux fentes ;
comparez cette figure avec a)
la figure 7.13 c). b) Figure
de difii action cTééc par une fente

Il I
simple ; comparez eciie figure
avec la figure 7.13 b).
b)

Si on lient co m p te d es effets d e la d iffractio n , rin lc n s ité lum ineuse d e la figure


d ’interférence créée par un systèm e à deux fentes est donnée par

sin a V
К в ) = /,„(cos2 (le système à cfcux fentes). (7.16)
. « )

7T(i
ou sin в (7.17)


cl sin в. (7.18)
^ “ T
Dans celle équation, d représente la di.stance entre les centres des fentes, et a esl la largeur
de chaque lente. P ren ez soin d e no ter que le m em bre de d ro ite de l ’éq u aiio n 7.16 est
le produit de / „ cl d e deux facteurs : 1) le facteur d ’interférence cos} 0, qui correspond
à r in te rf c r c n c c en tre les d eux fen tes sé p arées p a r une d istan c e d (q u ’on d éterm in e
à l’aidc des équations 6.21 cl 6 .22) ; 2 ) \e facteur de diffraction |(sin a)/a]~, qui corres­
pond à la diffractio n d ’un e fen le sim p le d e la rg e u r a (q u ’on d é te rm in e à l ’aide des
équations 7.5 et 7.6).
O n vérifie ici ces deux facteurs. Si on met a - ^ 0 dans l ’équation 7.18, par exem ple,
il s ’ensuit que a 0 et (sin a )/a —> 1. L’équaiion 7.16 se léduit alors, com m e il sc doit,
à une équation décrivant la figure d ’in lcrféren ce produite p a r deux fentes in fin im en t
étro ites sé p arée s par une d istan c e d. D e faço n sim ila ire , si o n p o se q u e d = 0
dans l’équation 7.17, cela équivaut à jo in d re physiquem ent les deux fentes en une seule
fente de larg eu r« . L’équation 7.17 donne alors ^ = 0 et cos- \ . D ans ce ca.s, l’équa­
tion 7.16 SC réduit, com m e il .se doit, à une éq u atio n d écrivant la figure d e diffraction
produite par une fente sim ple d e largeur a.
La figure créée p ar un systèm e à deux fentes décrite par l ’équation 7.16 et illustrée
à la figure 7.14 a) com hine les phénom ènes de diffraction et d ’in terférence. L es deux
sont des effets de .superposition, car ils résultent d e la com binaison d ’ondes présentant
des dil fércnces de phase à un point donné. Si les ondes com binées sont issues d 'u n petit
nom bre tie .sources élém en taires coh éren tes - co m m e dans une expérience avec deux
fentes où « <^ À - , ce processus sc nom m e alors interférence. Si les ondes com binées
sont issues d ’un .seul front d 'o n d e - com m e dans une expérience avec une fente sim ple - ,
ce processus .sc nom m e diffraction. C ette dislinctitin entre l’interférence e t la diffraction
(qui est quelque peu arb itraire et ne fait pas to u jo u rs l’u n an im ité) esl une d istin ctio n
pratique, m ais on ne doit pas ou b lier que ces deux phénom ènes constituent des effets de
superposition et q u ’on peut habituellem ent les ob serv er en m êm e tem ps (com m e on le
voit dans la figure 7.14 a).

Exemple 7.4
Diuis une cxj)ciicm.c N11 lailiffiairiiHu rééi’ par deux fentes la longueur a) Г ombien de franges d'interférence brillantes sont L-oniiirises danx
(fontle X. (le la miuroc lumineuse est de 40.5 nm, la distance d entre les le pic central de l’enveloppe de'diftraclicm?
fwitci c.si lie 10.44 /1 in et la largeur a île chaque fente est de 4.050 /cm.
Itme7 compte de l’intcrfércnvc de lu lumière is-suc des deux fentes SOlllTIOtl On doit d'abord analyser les deux mccani.snics rcs(Kmsablcs
cl ik- la diffraeiion de la liimifre dan.s chac une des fentes. de la figutc optique prihliiitc dans cette expérience
7.7 Les réseaux de diffraction 187

Diffraction produite par une fen te simple : Dans ce cas. le


concept clé CSC que les limites du pic central coïncident avec les
premiers minima de la figure de diffraction créée par chaque fente,
pri.se individuellement {voir lu figure 7.13). Les positions angulaires
de ces minima sont données par l’équation 7.3 (a sin 6 ~ mX).
On écrit cette équation sous la forme a sin 6 = m^X. l'indice I
référant à la diffraction causée par une fente. Pour trouver les
premiers niiniina dans la figure de diffitiction. on remplace m, par 1
et on obtient

Í7 sin W= X. (7.19)

Interférence produite par deux fentes : Daas ce cas. le concept clé


est que, dans une interférence produite par deux fentes, les positions
angulaires des franges brillantes sont données par l’équation 6.14.
qu’on peut écrire sous la forme figure 7.15 Exemple 7.4 Un côté du graphique de la di.stribution
de l’intcnsiié lumineuse dans une expérieiK-e d’interférence produite
sin W= in2X, lorsque ntj = 0. ± I. ± 2 .. (7.2(1)
par deux fentes ; l’enveloppe de diffraction est indiquée
où l’indice 2 réfère à rinterférencc produite par deux fentes. par la courbe pointillcc. Le petit encart montre le graphique
On peut localiser le premier minimum de diffraction dans la de la distribution de l’intensité lumineuse (étendu verticalement)
figure de diffraction produite par deux fentes en divisant l’équition 7.20 dans le premier et le deuxième pic secnndaiic de l’cnvcloppc
par l’équation 7 19, et en isolant mj. En tililLsant ensuite les données de diffraction.
connues, on obtient
Si on divise l’équation 7.20 par l'équation 7.21, on peut déterminer que
_ d _ 1 9 .4 4 ^ _
a 4.Ü50 /cm 2d U )( 19.44 /tm)
m. = 9,6
4.050 /1111
Ce résnll.at indique que la frange brillante correspondant à m, = 4
est comprise dans le pic central de la figure de diffraction à fente Ce résultat indique que le deuxième minimum de ditlraction survient
simple, mais que la frange correspondant à m-> ~ 5 n’est pas comprise juste avant la frange brillante d’interférence associée à K)
dans ce pic. Dans le pic central de diffraction, on a la frange brillante dans l'équation 7.20.1>ans l'uii ou l’autre des premiers pies secotKlaires
centrale (mj = 0) et quatre franges brillances (ju.squ'à 4) de diffraction, on a les franges produites quand luj = 5 b nr. ” 9,
de chat|uc côté, fl y a donc un total de neuf franges taillantes dans la ce qui donne un total de cinq franges hrillames dans la figure
figure d’interférence produite par un système à deux fentes à l’intérieur d'interférence produite par deux fentes (illustrée dans l'Cncart de la
du pic central délimité par l’enveloppe de diffraction. Les franges figure 7.15). Toutefois, si on considère que la frange brillante corres­
brillantes d’un côté de la frange brillante centrale sont illustrées dans pondant à fil; - 5. qui est prc.sque complctcincnt éliminée pur le premier
lu ligure 7.15. minimum de diffraction, est trop fiâle pour être comptée, nn a alors
seulement quatre franges brillantes dans le premier pic secondaire de
b) Combien de franges brillantes sont comprises dans l'un ou l’autre diffraction.
des premiers pies secondaires de l’enveloppe de diffraction ?

SOLUTIOH: l e tontept de est que les limites supérieures des premiers pics /V É R IFIE Z VOS CONNAISSANCES 5: .si on augmente à 550 nm
secondaires de diffraction coïncident avec les deuxièmes minima de la longueur d'onde de la source lumineuse iiientiiiniicc
diffraction, chacun d’eux étant ù l’angle ft donne par a sin 0 — m,À, dans l’exemple 7.4. a) est-cc que la largeur du pic central
quand m, = 2 : de diffraction dimimiera. aiigmciucra ou restera la même ?
b) Est-cc que le nombre de franges brillantes d’inirrfcrciKC
a sin ft — 2X. (7.21) comprises dans ce pic augineniera, diiiiiiiiiera <«i rcsicia le même

7.7 Les réseaux de diffraction


Dan.s l’étude de la lum ière e t des objets qui cm citcnl et absorbent de la lumière, le réseuu
d e d if fr a c tio n co n stitu e l’un d es o utils les plus pratiques C e ilisp o sitil est sim ilaire
à l'arrangem ent de deux fentes de la figure 6 .8, m ats com prend un plus grant! nombre N
de fentes ; il peut com prendre, par exem ple, plusieurs m illiers de fentes pal in illim èu e
La fig u re 7 .16 re p ré se n te un réseau idéal co m p o sé d e se u lem en t c in q fc n ic s .
L orsqu’une lum ière m onochrom atique esi (ransm ise par les fentes, elle form e de im necs
fran g es (]’in te rfé re n c e ; l’analy.sc d e ces fra n g es p erm e t d e d e te rm in e r la longueur
d’onde de la lum ière. (L es réseaux d e diffraction peuvent égalem ent cire des siirfaec»

figure /16 l In rcsenii de diffraction idéal, composé de seulement cinq fentes, qui prrxluii une ligure
d’mtcrférciicc sur un écran d'observation éloigné C.
188 Chapitre 7 La diffraction et (a potarisatlon

In ic n siiô opaques co m p o rtan t de inince.s sillons p arallèles (des traits), com m e les fentes de la
I fig u re 7.16. L a lu m ière e s t alo rs d iffu sé e d ep u is ces sillo n s cl fo rm e d es fran g es
•m = I)
-3 -ü I I 1 2 3 d ’interférence au lieu d ’être transm ise par des fentes ouvertes.)

UULÜU 0
Si o n a une lu m ière m o n o c h ro m atiq u e in c id en te su r un rése au de d iffra ctio n
et q u 'o n augm ente grad u ellem en t la quaiiiiié de fentes de d eux à un nom bre élev é N.
le graphique d e la distribution de rim c n siié lum ineuse change, et du graphique habituel
correspondan! à un systèm e à deux fentes, com m e celui de la figure 7.13 c), on passe
a)
à un graphique plus com plexe qui finit par devenir un sim ple graphique com m e celui de
la ligure 7.17 a). Si on utilisait une lum ière m onochrom atique rouge provenant d 'u n laser
à hélium -néon, p ar exem ple, vous verriez sur un écran d ’observation une figure d 'in te r­
u t l :
-3 -2 ■I m = 0 férence .semblable à celle illustrée dans la figure 7.17 b). Les m axim a y sont très étroits
h) (cl sont nom m és /igne.r ou r a i e . f ) , et séparés par des régions .sombres relativem ent larges.
O n utili.se u n e m é th o d e fam ilière p o u r locali.scr les p ics b rilla n ts sur l'é c ra n
Figure 7 17 a) Le graphique d'observaiion O n suppo.se d ’abord que l’écran est assez éloigné du réseau pour que les
de la diMribuiioii de i’imciLsitc rayons qui atteignent un point donné P sur l'écran soient approxim ativem ent parallèles
lumineuse produit par un réseau lo rsqu'ils quittent le m seau (figure 7.18). O n applique ensuite à chaque paire de fentes
de iliffiaclion comportan! un grand adjacentes le raiso n n em en t q u 'o n a appliqué dans le cas d ’une in terféren ce p ar deux
nombre de fentes se uiminisc de pies fentes. La disianee «7entre les fentes (ou les traits) .sc nom m e lepa.\ du réicatt. (Si /Vientes
eiroits, qu’on a identifiés par l’ordre
occupent une largeur to tale il s ’en su it que d = tIN .) La d ifférence de m arche entre
eoi ir.s(X)iidanl à ehairun. h) Les franges
les rayons adjacents est toujours d sin fí (figure 7 .i 8), où (/ est l ’angle form é entre l ’axe
brillunles euircspondiUiies ohsci véc.s
central du réseau (et de la figure de diffraction) et le p o in t P. Un pic sera locali.sé à P
sur l'écran SC nomment raies ;
lorsque la différen ce de m arche entre des rayons ad jacen ts sera u n nom bre en tier de
elles sont egalement identifiées
longueurs d ’onde, c ’est-à-dtre si
par leur ordre m. La ligne centrale
{m 0) de même que les pics
de premier ordre, de deuxieme ordre d .sin 0 = nj'A. lorsque w = 0, ± 1, ± 2 . .. . (les maxima - raies), (7.22)
et de troisième ordre sont illustrés.

où A est la longueur d 'o n d e d e la lum ière. C haque nom bre en tier m représente un pie
different ; ces nom bres entiers peuvent donc .servir à identifier les pies, coiiune on l’a feit
dans la figure 7 . 17. Les nom bres entiers sont alors appelés l ’ordre, et on nom m e les pics
selon leur ordre: le pic d ’ordre z.éro (le pic central, où m = 0), le pic de prem ier ordre,
le pic de deuxièm e ordre, et ainsi de suite.
Si on ré c rit l ’éq u a tio n 7 .22 sous la fo rm e fí = s in ~ ' (niA/i/), o n v o it q u e, p o u r
un réseau de diffraction donne, l’angle entre l’axe central cl un pic quelconque (le pic
de troisièm e ordre, par exem ple) dépend de la longueur d ’onde de la lum ière utilisée.
D onc, lorsqu’une lu m ière de longueur d ’onde inco n n u e est tran.smisc dan.s un réseau
de diffraction, les m esures de l’angle aux pics les plus grands dans l ’ordre peuvent être
utilisées dan.s l ’équation 7.22 pour d éterm in er la longueur d ’onde. M êm e une lum ière
V ers lo p o in t P
co m p ren an t plu.sieurs lo n g u eu rs d ’o n d e in co n n u es p eu t ê tre id e n tifiée et rec o n n u e
.sur Г(к'гаи
d 'o b s m 'a lio n de celte façon. O n ne p eu t le faire, cependant, avec l ’arrangeincni à deux fen tes de la
section 6.4. m algré le lait que la m êm e équation e t le m êm e rapport à la longueur d ’onde
s’af^liquent. Pour l’interférence produite par deux fentes, les franges brillantes créées par
les différentes longueurs d ’onde ,se chev auchent trop pour q u 'o n puisse les distinguer.

La largeur angulaire des pics


La capacité d ’un réseau tic séparer les pics de différentes longueurs d ’onde dépentl de la
•itfércnrR largeur angulaire de ces pics. O n doit trouver ici une expression pour d écrire la demi
m atclie «‘titre l/trgeur (mgulaire du pic central (celui où m ü ), e t on conviendra d ’une expression
fieux r.ivons arljaeeitts
1 t A y1. _ _
I_ pour décrire les dem i-larg eu rs angulaires des pics des autres ordres. I .a d em i-larg eu r
angulaire du pic central a com m e m esure l’angle AOji qui va du centre du pic à // = 0 .
ju sq u ’il re iu lro il où la zone som bre co m m en ce (fig u re 7. IQ). A un tel m inim um , les
N rayons issus des N fentes du réseau s ’annulent m utuellem ent. (La largeur angulaire
R? яй71Я I .es rayons issus des fcnte.s
d’un réseau fie dilinacrinn qui se dingcnl réelle du pic central est bien sûr 2(Afídi)> m ais on com pare habituellem ent les largeurs
vers un point distant F sont approxima- angulaires des pics à l’aide des dem i-largeurs angulaires.)
tivciiirui parallMrs La différence O n a vu, dans la section 7.2. q u ’un grand nom bre de rayons .s’annulaient atis.si en
de marche e n tic les rayons udiaecnt.s raison de la diffraction produite par une fente simple On a alors développé l’équation 7.3
rsi d sin fi. ni) West inesuié comme qui. à cau.se d e la ressem blance entre les deux situations, isermel de localiser le prem ier
on rindique. (Les fentes s’étendent m inim um dans le cas présent. C ette équation indique que le prem ier m inim um survient
(>ei |X;ndiculaiienieni au plan là où la différence de m arche entre le rayon du haut et celui du bas est égale à A. P our la
de la page.) diffraction produite p a r une fente .simple, cette différence est a sin tí. Dan.s le cas d ’un
7.7 Læ s réseaux de diffraction 189

ll lU 'I l s il C

Figure 7.19 La tlemi-largeur angulaire Rgure 7.20 Les fentes du haut et du bas
du pic central se mesure entre le centre d'un réseau de diffraction comportant
de cc pic et le minimum adjacent daas N fentes sont séparées par une distance Nd.
un graphique de / en fonction de fi. Le rayon du haut et celui du bas. qui
comme celui de la figure 7.17 a) traversent ces fentes, ont une différence
de marche de Nd sin Aft^i, où e.st l’angle
au piemici minimum. (On a exagéré l'angle
dans la ligure par souci de clarté.)

réseau de N fen tes, où ch aq u e fente est sép arée de la su iv a n te p a r la d istan c e d. la


d ista n c e entre l ’élém en t du h au t cl celu i du b a s est Nd (fig u re 7.201. ce q u i fa it que
la différence de m arche entre le rayon du haut et celui du bas est Nd sin A0(,|. Le prem ier
m inim um survient donc à l ’endroit où

N d sin AAi, = À. (7.23)

C om m e est p etit, sin = A^<ji (en rad ia n s). E n in sé ra n t ces d o n n ées dans
l’équatitin 7.23, on obtient la dem i-largeur angulaire du pic central, sous la form e

Aftii — (la demi-largeur angulauc du pic central). (7.24)


Nd
O n adm et sans dém o n stratio n que la d em i-larg eu r angulaire de to u t autre pie dépend
de sa position par rapport à l’axe central, et est

(la demi-largeur angulaire d’un pic à 0). (7 .2 5 )


N d co s ft

N o te / que. dans le cas d 'u n e lum ière d ’une longueur d ’onde A, e t avec une distance d
entre les fentes, les largeurs angulaires des pic.s dim inuent lorsque le nom bre N de fentes
augm ente. Il s ’en su it d onc q u 'e n tre deux ré se au x de d iffra ctio n , c e lu i ay a n t la plu.s
grande valeur de N perm et m ieux de distinguer les longueurs d ’onde, puisque ses pies
de diffraction sont plus étroits et produisent m oins d e chevaurhcm ent.s.

Une application des réseaux de diffraction


Los réseaux de diffraction .sont largem ent utilisés |xiur déterm iner les longueurs d ’onde
émise.s par des sources lum ineuses, q u ’il s'ag isse d ’etoiles ou de sim ples lam padaires
La figure 7.21 m ontre un spertroscope à réseau qui utilise un ré.seaii de diffraction à
cette fin. La lum ière provenant d e la source A est focalisée par la lentille / , sur une fente
verticale F , placée dans le plan focal de la lentille Li. L a lum ière ém ergeant du tulie C
(nom m é cnlUntoteur) est une onde plane qui arrive à incidence norm ale sur un réseau A,
où e lle est d iffra c ié e et fo rm e une fig u re d e d iffra c tio n av e c le pic d ’o rd re m ~ 0
diffracté à l’angle ft O par rapport à l’axe central du réseau.
O n peut observer la figure de diffraction q u i apparaîtrait sur im écran d ’nbservalion
à to u t angle ft en o rien tan t sim p lem en t le té lesco p e T de la fig u re 7.21 h cet angle.
Figurs 7.21 L n type de spectroscope I a lentille du télescope fixialise alors la lum ière diffractée a l’anglC (7 (e t à des angles
S réseau utilisé pour anidysci unpt'ti plus petits <iu lin peu plus grands) dans un plan focal F F ' à l in téiieu rd u téleu-(ij.iH.
les longueurs d’«Hide de la lumière L or.squ'on reg ard e d an s r n c n la ir e l). on voit une v ue ag ran d ie d e ceim im ag e mi.se
émise par la source A. au foyer
190 Chapitre 7 La diffraction et la polarisation

m= l m-2
m=0

10“ 30'’ '10" 50" 60" 70" 80"

figure 7.22 L’ordre zéro, le premier ordre, le deuxième ordre et le tjualrième ordre de.s raie.s
d'émis.sioii visibles produites par une lampe à l'hydrogène. Notez que les raie.s sont plus espacées
à de plus grands angle.s. (Elles sont aussi plus pülcs et plus larges, ee qui n’est pas illusUé ici.)

figure 7.23 Les raies d ’émission visibles du cadmium, telles qu'on les voit avec un spectroscope
à réseau.

L n ch a n g ea n t l ’a n g le f) du lé lcsco p e, o n p eu t e x a m in e r en e n tie r la fig u re de


d iffractio n . P our t«iul o rd re autre qu e m = 0, la lu m ière in itiale e st étalée su iv a n t la
longiictir d ’onde (ou la coulctir), ce qui perm et de déterm iner, à l’aide de l'cquation 7 22.
les lo n g u e u rs d ’o n d e ém ise s p a r la so u rce. Si la so u rce ém et d es lo n g u e u rs d ’onde
discrètes, on voit, en tournant le télescope horizom alem eni et aux angles corre.spondant
à un ordre m, une ligne verticale de couleur p o u r chaque longueur d 'o n d e, la ligne de la
plus petite longueur d ’onde sc trouvant à un plus petit an g le B que les lignes des plus
grandes longueurs d’onde.
Pai exem ple, la lu m ière ém ise p ar une lam pe à l’h y d ro g èn e, qui i^onlieni du gaz
hydrogène, a quatre longueurs d ’onde discrètes dans la partie visible du spectre. Q uand
vos yeux interceptent celte lum ière dirccteinenL elle vou.s sem ble rose. Si vous la voyiez
plutôt avec un spectroscope à réseau, vous po u rriez distinguer, à plusieurs ordres, les
pics des q u atre co u leu rs c o rre sp o n d a n t à ces lo n g u eu rs d ’o n d e v isib les (ces p ics
se n o m m en t raiex d ’ém ission). Q u atre o rd res so n t rep rése n tés d an s la fig u re 7.22.
D ans l’ordre central (m = 0), les pics correspondant aux quatre longueurs d ’onde .sont
superposés, ce qui donne une seule ligne blanche à — 0 . L es couleurs so n t séparées
dans les ordres plus élevés.
L e troi.sièm e o rd re n ’e st pas illu stré d an s la fig u re 7 .2 2 , p a r so u ci d e c la r té ;
en réalité, il chevauche le deuxièm e et le quatrièm e ordre. La raie rouge du quatrièm e
ord re n ’ap p a raît pa.s c a r e lle n ’est pas form ée à l ’aide du réseau u tilisé d an s ce cas.
C ’est-à dire que lo rsq u ’on essaie de résoudre l'éq u atio n 7 .22 en fonction de l’angle 0
afin de trouver la longueur d ’onde rouge correspondant à m = 4, on découvre que sin 6
est plus g ran d q u e 1. ce q ui est im p o ssib le. O n d it alo rs q u e le q u atrièm e o rdre
est incomplet pour ce réseau ; il ne le serait peut-être pas avec un réseau qui aurait un
plus grand espacem ent d, où les raies seraient m oins espacées qu e dans la figure 7 22.
L a fig u re 7.2.3 e s t une p h o to g rap h ie d e s raie s d ’é m issio n v isib les p ro d u ite s p ar
le cadm ium . D ’autres spectres d ’ém ission apparaissent à la figure 10 16.

VÉRIFIEZ VOS CONNAISSANCES 6 : .a fig u re ci-dcss<His représente des raies d'urdrc.s


different-s produites par un réseau de diffraction alors qu’on utilise une lumière
monochromatique rouge, a) Le cenhc de la figure est il à gauche ou il droite'’ h) .Si on utilise
plni('>» une lumière monochromatique verte, le.s demi-largeurs angulaires des pics produits
dan.s les infmcs ordres seront-elles plus grandes, plus petites ou lc.s mêmes que les demi-
largeurs des pics ilIiLslrés ?
7.8 Les réseaux ; te pouvoir dispersif et le pouvoir de résolution 191

7.8 Les réseaux : le pouvoir dispersif


et le pouvoir de résolution
Le pouvoir dispersif
P o u r perm ettre à un o b serv aicu r de di.slinguer des longueurs d ’onde voisines (r^omme
d ans un spectro sco p e à réseau ), un réseau doit p o u v o ir étale r les raies de d iffractio n
a.ssociées aux d iffé re n te s lo n g u eu rs d ’on d e. C e t étale m e n t, q u ’on no m m e p o u v o ir
d isp e rsif, se définit ainsi :

A6
Sur un disque compact, les minces D = -r— (la définition du pouvoir dispersif). (7.26)
sillons d’une largeur de 0.5 ùX
agissent comme un réseau de diffraction.
Lorsqu’une petite source de lumière D ans c e tte fo rm u le, A 0 e s t la sép aratio n an g u laire de d eux raie s d o n t les lo n g u eu rs
blanche éclaiie le disque, la lumière d ’onde ont une différence d e A>.. Plus D est grand, plus grande est la distance entre deux
diffractée forme des « bandes » colorées raies d 'ém issio n dont les longueurs d ’onde diffèrent de AL. O n dém ontrera bientôt que
tjui sont les composantes des figures le pouvoir d isp ersif d ’un réseau à l’angle Ocsi dom ié par
de diffraction produites par les sillons.
m
D (le pouvoir dispersif d’un réseau). (7.27)
(J eos fi

D onc, pour obtenir un grand pouvoir dispersif, on doit utiliser un réseau ayant un petit
espacem ent d et utiliser un ordre m élevé. Note?, que le pouvoir dispersif ne dépend pas
du nombre N de fentes dans le réseau. L ’unité SI de O est le degré par mètre, ou le radian
par mètre.

Le pouvoir de résolution
Pour séparer des raies dont les longueurs d ’onde so n t très lappriK hées (afin de pouvoir
les distinguer), le réseau doit produire des raies aussi étroites que possible. En d 'au tres
m ots, le réseau d o it avoir un grand pouvoir de résolution R. q u ’o n définit ainsi :

X
R ~ (la définition du pouvoir de résolution). (7.28)

Ici, est la longueur d ’onde m oyenne d e deux raies d’ériii.s.sion qui peuvent tout ju ste
être séparées, et A>. est la différence de longueur d ’onde entre elles. l ’IuR R est grand,
plus deux raies d ’ém ission peuvent être rapprochées tout en étan t dislincies On «lénion-
trera bientôt que le pouvoir de résoUiiirm d ’un réseau est doratc par l’expression

R — Nm (le pouvoir de résoluiion il" un réM-au). (7 29)

P o u r o b te n ir un grand p o u v o ir d e résolution, on d o it utiliser de nom b reu ses fciut-s


(une grande valeur de N dans l’équation 7.29).

Démonstration de l'équation 7.27


O n com m ence d ’abord avec l'éq u atio n 7.22, qui ex p n m e les positions des raie..s dans la
figure de diffraction d ’un réseau :

d sin 0 — niX.

O n tronsidère ensuite que 0 e t À sont des variables et on calcule la diffétentiellt d r extte


équation. On déterm ine alors que

d cos Ûdfi = m dX.


192 Chapitre 7 La diffraction et la polarisation

Q uand les angles so n t suffisanuncnl petits, on p eu t écrire les différentielles co m m e s’il


s ’agissait de p etites différences, et on obtient

(I cos 6 A 0 = m à X (7 .3 0 )
ou
m
Aà ilœ s 0
Le rapport du m em bre de gauche est sim plem ent D (voir l’équation 7.26), et on a donc
effectivem ent dém ontré I équation 7.27.

Démonstration de l’équation 7.29


O n co m m en ce av ec l’éq u atio n 7 .3 0 , q ui a été d ériv é e d e l ’éq u atio n 7 .2 2 . laq u elle
exprim e les positions des raies dans la figure de diffraction d 'u n réseau. Ici. A â e.st la
petite différence de longueur d 'o n d e entre deux ondes qui sont ditfractccs par le réseau,
cl A tte st la séparation an g u laire en tre elles dans la figure d e d iffractio n . ,Si A fte st le
plus petit angle perm ettan t aux deux raies d ’être distinguées, il doit être égal (d ’après
le critère de R ayleigh) a la dem i-larg eu r angulaire de ch aq u e raie, qui e st d o n n ée par
l’équation 7.25 ;

A«., -
N d cos 0

Si on substitue Afij, tel q u ’il est donné ici à A O dans l’équation 7.30, on déterm ine que
X
= m Aà ,
A'
résultat duquel on déduit facilem ent que

R = f- =

C ela corre.spond exactem ent à l ’équation 7.29, que l’on voulait dém ontrer.

La distinction entre le pouvoir dispersif et le pouvoir de résolution


O n ne doit pas confondre le pouvoir de lésolution d ’un réseau avec son jxiuvoir dispersif.
L e tableau 7.1 présente les caractéristiques de trois réseaux, tous éclairés pai' une lumière
de longueur d ’onde À = 589 nm, et dont la lumière diffractée est ob.scrvcc dans le prem ier
Î R ordre (ni = 1 d ans l ’éq u atio n 7.22). Vous d ev riez v érifier si les v aleu rs d e D et de R
134
0(degré-s) d o n n ées d ans le ta b lea u p eu v e n t être c a lc u lé e s à l’aid e d es éq u a tio n s 7.2 7 e t 7 .2 9 ,
resp e ctiv em e n t. (D an s les ca lc u ls d e D, vo u s d ev re z co n v ertir les rad ian s p ar m ètre
en degrés par m icrom ètre.)
f i kést■aa A
R elativem ent aux conditions indiquées dans le tableau 7.1, les réseaux A et R ont
le m êm e pouvoir dispersif, et 4 et C o nt le m êm e pouvoir de résolution.
l A La figure 7.24 m ontre les figures d ’intensité (q u ’on nom m e égalem ent /or/ne.v des
1.3.1
«(<-l<;si4.<i) raies) qui auraient été produites par ces réseaux en fonction de deux longueurs d 'o n d e
À| et À2, dont la valeur m oyenne avoisine À = 589 nm. Le réseau B. qui a le plus grand
pouvoir de ré.solution, form e des lignes plus étroites et p erm et donc de* d istin g u er des
R éseau / ”\

A
lignes plus rapprochées, au plan des longueurs d ’onde. Le réseau C, qui a le plus grand
pouvoir dispersif, produit la plus grande séparation angulaire entre les lignes.

TABLEAU7.1 Trois réseaux*


figure 7.24 CXairbes de ta distribution
Réseau N d (nm) (f l) r/fim ) R
d’iniensiié relative pr<Kluite par une
lumière composée <lc deux longueurs A 10000 2 540 13,4" 23,2 K) (XX)
d'onde transmi.se dans les réseaux « 2 0 (KK> 2 540 13,4° 23,2 20000
du tableau 7.1 Le réseau Zi a un plus grand
C 10000 t 170 25,5" 46,3 10 000
pouvoir de résolution, et le reseau C
a un plu.s grand pouvoir dispersif ■* I es données (.-om^pondent i X .SS9 nm cl à m = 1
7.9 La diffraction des rayons X 193

Exemple 7.5
Un réseau de diffraction a 1,26 x Kf* fentes uniformciiicnt espacées à l’angle в = 16.99°, comme le résultat trouvé dans la partie a),
sur une largeur t - 25.4 mm. Il e.sl éclairé à incidence normale par relativement à l’une de ces raies. L’équation 7.27 donne alors le
une lumière jaune produite par une lampe à la vapeur de sodium. pouvoir dispersif:
Cette lumière contient deux raies d'ém ission peu espacées (qu’on
I
nomme le doublet du .sodium) qui ont des longueurs d'onde de D =
58y,(X) nm et de 589,59 nm. dcoüB (2016 nm)(cos 16.99=)
= 5,187 X 10 rad/nm.
a) À quel angle le maximum de premier ordre se produit-il (d’un côté
ou de l’autre du centre de la figure de diffraction) quand la longueur D ’après l'cqualion 7.26, on a alors
d'onde c.st de 589,(K) nm ?
A» = /J AX = (5.187 X 10 <rad/nmK589,.59 nm - 589.00 mn)
SOlUIlOH; Le concept tié, ici, est que les maxima produits par le réseau = 3.06 X 10^^ rad = 0.017 5°. (réponse)
de diffraction peuvent être localisés à l'aide de l’équation 7.22
(d sin 6 = mk). Le pas du réseau d est ici de ■Vous pouvez démontrer que ce résultat dépend du pas du ré.seau d,
et non du nombre de fentes qu' il y a dans le réseau.
25. i X lO"-’ m
c) CJucl est le nombre minimum de tèiucs qu'un réseau doit contenir
N 1 ,2 6 x 1 0 ^
s'il permet de distinguer le dmibict de sinlium dans le premier ordre ?
= 2.016 X m = 2016 nm.
SOUJTION: Un premier contept d« est que le pouvoir de résolution d ’un
Le maximum de premier ordre correspond à m = I . En substituant réseau, dans tout ordre m, est établi physiquement par le nombre N
ces valeurs à c/et à m dans l’équation 7.22, on obtient de fentes éclairées qu'il comprend, suivant l'équation 7.29 (R - Nm).
Un autre (oncepi tié est que la plus petite différence de longueur
^ /(l)(.589.0ünm )\ d’onde AX permettant la séparation dépend de la longueur d'onde
в = su r'
d \ 2016 nm ) moyenne dans le réseau, et du pouvoir de résolution R du réseau,
= I6,9<)" «« 17.0 . (réponse) suivant l'équation 7.28 {R = X^,/AX). Pour que les raies d'éuii.ssicm
du doublet de sodium puissent être tout juste séparées. AX doit cire
b) En utilisam le pouvoir di.spersif du réseau, calcule/, la .séparation égale à leur différence de longueur d’onde de 0.59 nm. et X„y„. doit
angulaire des deux raies du premier ordre. être leur longueur d'onde moyenne de 589..V) nm.
En regroupant ces cnneepis, on découvre que le nombre niinimuni
SOUJTION: Le premier «ixep? de qu’on utilise ici est le suivant ; la séparation de fentes éclairées qu’un réseau doit conienii | xhii poiivoii sepan r le
angulaire des deux raies du premier ordre dépend de la différenee doublet de MKliiim est
de longueur d’onde AX entre ces lignes et du pouvoir dispersif D
du réseau, suivant l’équation 7.26 (O = AO/AX). Le second tonteptelé est R '■mn
N = - =
le suiviuit : le pouvoir dispersif I) dépend de l’angle B auquel on doit ni m AX
l’évaluer. On peut supposer que, dans le premier ordre, les deux raies 589,30 nni
=. 999 fentes. (réponse 1
de la lumière au .sodium sont as.se/ rapprochées pour qu'on évalue D (11(0,59 nm)

7.9 La diffraction des rayons X


L es rayons X sont des rayonnem ents clecirom apnétiques dont les longiieurs d 'o m le sont
de l’ordre de 1 Â ( * 10“ '" m)*. L a figure 7.25 m ontre la (»oduction de rayons X Inrsque
des électrons qui s'éch ap p en t d ’un filam ent chauffe F sont accélérés p ar une dilférence
de potentiel V'et frappent une cible m étallique M.
Un néscaii de diffraction optique conventionnel ne perm et pas de faire la distinction
e n tre des lo n g u e u rs d 'o n d e d e l'o rd r e de ce lle s d es ray o n s X. D an s lé cas ou
X = 1 À ( = 0 , 1 nin) ei </ = 3 (KK) n m . p ar ex e m p le , l’éq u a tio n 7 .2 2 in d iq u e qu e le
m axim um de prem ier ordre .se produit à
wX ,(l) ( ü . In m )
в = sin “ ' Sin = 0.(KM 9°.
.3 000 nm
C et écart avec le m axim um cen tral est tro p petit pnui être d ise en i/ih lr Un réseau où
</= X serait idéal, mais com m e les longueurs d ’onrle dc.s laynns X sont approxim ativement
égales aux diam ètres atom iques, il est im possible de co n stru ire un tel iése,au de faqtm
m écanique.
Figure 7.25 Des rayons X sont générés
En 1912. le p h y sicien allem an d M ax von Latte co n stata q u 'u n so lid e cristallin ,
lorsque des électrons qui s’éch.'ippcnt
eom piw é d ’un réseau rég u lier d 'a to m e s, p o u v ait fo rm er un « ré se a u de d iffra c tio n »
du filameni chauffé F sont accéléré.s
trid im ensionn el p o u r les ray o n s X. C e |ih én o n ièn e est dû au fait que. dans un eiislnl
par une différence de potentiel V
co m m e le c h lo ru re d e so d iu m (N aC l). un m o tif d 'a io in e s <le b a se (n n m m é nuiillr
Cl frappent une cible métallique M
L’ouverture () dans la chambre élém fntaire) sc répète dans l'en sem b le du réseau Dans le NaCI quatre ions de sislium
à vide C laisse passci' les rayons X. * l'nagsIriMn (Â) est une unité de longiieiir uiilisee en physique .uomiqiic.
]94 Chapitre 7 La diffraction et la polarisation

e t quatre ions de ch lo re so n t asso cies à ch aq u e m aille élém en taire. L a fig u re 7 26 a)


représente une coupe transversale d ’un cri.sial de N aC l qui p erm et de d istin g u er celte
unité de base. L a m aille élém entaire form e un cube m esurant Cq de chaque côte.
L orsqu’un faisceau de rayons X en tre dans un cristal co m m e le NaCK les rayons
sont dispersés - c ’est-à-dire q u ’ils sont dirigés dan.s toutes les directions par la structure
du cristal. D ans certaines dircciions, les ondes « red irig é es» subissent une interférence
destructive, qui produit les m inim a d ’in ten sité; dans d ’autres d irectioas, l’interférence
est c o n stru c tiv e , ce qui p ro d u it les m ax im a d ’in ten sité. C e p ro cessu s d e d isp ersio n
e t d ’interférence coastitue une form e de diffraction, m êm e si cette dernière diflère de la
d iffra c tio n de la lu m ière so rta n t d ’une fen te ou ren c o n tra n t un o b sta c le , d ont on a
discute précédem m ent.
M êm e SI le processus d e diffraction des rayons X dans un cristal est com plexe, les
m axim a se trouv en t dans les d irectio n s où les rayons X agissent comme s ’ils étaient
réfléchis p a r un en sem b le de plans réfléchissants parallèles (ou plans cristallins) qui
seraien t cré és p a r les ato m es d an s les c ristau x e t qui co m p o rteraie n t d es réseau x
réguliers d ’atom es. (Les rayons X ne .sont pas réellem ent réfléchis; on utilise ces plans
fictifs pour sim plifier l'an aly se du véritable proces.sus de diffraction.)
La figure 7.26 h) m ontre trois ensem bles d e plans, ayant un espacement interpla­
naire d. sur le.squcls les rayons incidents illustrés sont censés être réfléchis. l.es rayons
1 ,2 et i sont respectivem ent réfléchis su r le prem ier, le deuxièm e et le troisiètnc plan.
Relativement à chaque téflexion. l ’angle d ’incidence et l’angle de réflexion sont représentés
par 0. Contrairem ent à cc qui est habitudicm tm i adm is en optique, ces angles .sont définis
par rapport à la surface du plan réfléchissant, et non par rapport à la norm ale de cette
surface. D ans la figure 7.26 b), l’espacem ent inlerplanaire est égal à la m aille élém entaire
de dim ension «o-
L a figure 7.26 c) m ontre une vu e de côté de la réflexion su r deux plans adjacents.
Les ondes des rayons 1 et 2 sont en phase lorsqu’elles atteignent le cristal. A près leur
réflex io n , elles d o iv e n t ê tre en c o re en p h ase, pui.squc les réflex io n s et Ic.s p lan s
rélléchissanls ont été d éfinis u n iquem ent p o u r ex p liq u er les m axim a d ’in ten sité dans
la diffraction des rayons X par un cri.stal. Contrairement aux rayons lumineux, les rayons X

3 2 I

.C l
XV
V------- (P
1
««

L O - . . : 1-------- i-------è
a) b)

figure 7 26 a) La siriiciiirf cubique du cristal NaCI, qui montre les ions de sodium et de chloiv,
et une maille élémcntaiic (colorée), bj Les rayons X incirlents sont diffraciés par la structure
de a). Les rayons X sont diffractcs comme s’ils étaient réfléchis par un easembie de pLins
paiallclcs. suivant un angle de réflexion égal à l'angle d’incidencc. les deux angles étant mesurés
par rapport aux plans (et non par rapport à une normale, comme c ’est le ca.s en optique"),
c) La (liffércntv de marche entre les ondes effectivement réfléchies par deux plans adjacents
est Id sin a. tl) Dne orientation différente des rayons X incidents par rapport à la structure,
L'n autre ensemble de plans parallèles réfiéchit mainienant effectiveinrni les rayons X.
710 La polarisation 19S

ne sont pas réfractés en entrant dans le cristal ; de plus, on n ’a pas spécifié d’indice de
réfraction pour cette situation. Il s'en su it donc que la phase relative entre les ondes des
rayons 1 et 2 au m om ent où iis sortent du cristal est uniquem ent déterm inée par la dif­
férence de m arche entre ces rayons. P our qu e ces ray cms soient en phase, la différence
de m arche doit être égale à un m ultiple e n tier de la longueur d ’onde À des rayons X. Le
tracé des perpendiculaires (les lignes pointillées) de la figure 7 .26 c) indique que la d if
fércnec de m arche est 2d sin H. E n fait, cela s’applique à toute paire de plans adjacents
dan.s l’ensem b le d e plans représenté d an s la fig u re 7 .26 b). O n arrive d onc au critère
suivant relativem ent aux m axim a d ’intensité dans la diffraction des rayons X ;

figure 7.27 Un cnseinble de plans


dans la slruclure de la figure 7 26 a), 2d sin 6 = mX, lorsque m — 1 ,2 . ?>,... (la condition de Biagg), (7.31 )
et une technique permetlaiil de relier
la longueur du côté a,, d’une maille où m est l'o id re d’un m axim um d ’intensité. 1 .’équation 7.31 se nom m e la co n d itio n d e
élémentaire à respaeement
B ra g g en l’honneur du physicien britannique W illiam I,aw rcnce Bragg, qui fut le prem ier
interplanaire d.
à la dériver. (Lui cl son père W illiam H enry Bragg ont partagé le prix Nobel en 1915 pour
leur asage des rayons X dans l’étude des slmcuires cri.stallines.) Dans l’cqualion 7.31. l’angle
d ’incidence et d e réflexion .se nom m e angle de Bragg. Peu im p o n e l'an g le auquel des
rayons X entrent dans un cristal, il y a toujours un ensem ble de plans sur lesquels ils sont
réfléchis, et on peut donc appliquer la condilion de Bragg. N o te/ que. dans la figuie 7.26 d),
la structure de cristal a la m em e orientation que dans la ligure 7.26 a), mais l ’angle auquel
le faisceau entre dans cctle stru ctu re diffère de celui illustré dan.s la figure 7.26 b). Ce
nouvel angle doit im pliquer la création d ’un nouvel ensem ble de iilan.s réfléi bi.ssanis,
com portant un espacem ent interplanaire d différent e t un angle de B ragg différent, si on
veut expliquer la diffraction des rayons X p ar la condition de Bragg.
La figure 7.27 illustre de quelle façon l’espacem ent inieqilanaire J jieui être relié
à la d im e n sio n d e la m aille é lém e n ta ire a,,. P o u r l’en se m b le de p la n s illu.stré ici,
le théorèm e de Pythagorc donne

5d = \l^ a o .

»0
ou d ^ (7 32)
x /r
Plan
d ’o-stillalinn La figure 7.27 indique d e quelle façon on peut tiouver les diniension.s de la m aille
élém entaire, une fois que l’cspacem enl intetpianairc a clé m esuré par la diffraction dc.s
rayons X.
La d iffra c tio n d es ray o n s X est un o u til im p o rta n t d an s l ’é tu d e d es sp ectres de
X
• C rayons X el de l ’agencem eiit des atom es dans les cn slau x . Pour étudier les spectres, on
cho isit un ensem b le p articu lier de plans dans le cristal, séparés p ar un esp acem en t d
connu. C es plans rélléchissent effectivem ent différentes longueurs d 'o n d e à différents
angles, lîn détecteur pt>uvant cffceuier des m esures à d iltéren ts angles peut alors servir
à d éterm in er la longueur d ’o n d e des rayons qui l’atteignent Le cristal lui-m êm e ix'ut
être étudié à l’aide d 'u n fai.sceau m onoehrom aiique de rayons X , lequel jwrinci de déter­
m iner non seulem eni re sp a e e m e n t en tre les d ifféren ts plan.s du ci istal. m ais aussi la
structure de la m aille élém entaire.

7.10 La polarisation
Ln Angleterre, les antennes de télévision VHS (des m ots anglais verv high frequenry) <rni
une orientation verticale, alors q u 'en A m érique du N ord Icui orientation est h o ri/o iu a lc.
C ette différence est due à la direction des oscillations des ondes élcctioinagnétiqucs qui
1» transportent le signal. Ln A ngleterre, les équipem ents de transm ission soni conçus pour
Rguri;7.28 a) Le plan tl'(>.scillalion produire des ondes q ui sont p o la risé e s verticalem ent ; c 'e s t donc dire qu e leur cham p
d'une onde électromagnétique polarisée, électrique ostàllc verticalcinent. Alors, pour tiue le cham ji électrique des oniles incidentes
b) Pour représenter la polari.sation, de télévision alim entent un courant le long d ’une tmtenne (et lournissent un signal à un
hn visualise «de face» le plan récepteur dt» télévision), l'an tcn n c doit être verticale. En A m érique du N ord, les ondes
d'c.scillation et on indique les directions sont pnlarisée.s horizontalem ent.
du champ électrique oscillant La figure 7.2S a) m o n tre un e o n d e élec tro m ag n é tiq u e dont le t h ainp électriq u e
par une FlèclM: bidiretrllnnnelle. o sc ille p ara llèlem e n t à l’axe d es y L e plan d an s lequel évoliu nt les v ecteu rs L
196 Chapitre 7 La diffraction et la polarisation

se nom m e le p la n d ’oscillation de l’onde (on dit alors que l’onde est polarisée linéairement
dans la directio n d es v). O n p eu t rep résen ter la polarisation de l’onde (le fait q u ’elle
est polarisée) en indiquant les directions des o.scillations du cham p électrique dans une
« vue de face » du plan d ’oscillation, com m e on le voit dans la figure 7.28 b). l.a llcchc
bidirectio n n elle v erticale in d iq u e q u ’au m om ent o ù l ’o nde vous d ép asse, .son cham p
électrique oscille verticalem ent - il change continuellem ent sa direction de haut en bas
le long de l'a x e des v.

La lumière polarisée
l.es oiide.s électrom agnétiques ém ises p ar une station de télévision o nt toutes la m ême
polarisation, m ais les ondes électrom agnétiques ém ises par une source de lum ière quel­
conque (com m e le Soleil ou une am poule électrique, p ar exem ple) o nt une p o la risa tio n
a lé a to ir e ou .sont n o n p o la ris é e s , ce q ui sig n ifie q u ’e n to u t p o in t d o n n é, le ch am p
électrique est toujours perpendiculaire à la direction du parcours dc.s ondes, m ais change
de direction de façon aléatoire. D onc, si on essaie de rcpicsciUcr une vue de face de ces
osdllaiions clans uii intervalle de temps, on n 'obtient pas un sim ple m otif avec une llcchc
bidirectionnelle com m e celle de la figure 7.28 b ) ; on a plutôt un ensem ble dé.sordonné
de flcdrcs bidirectionnelles, com m e celui de la figure 7.29 a).
Hn principe, on peut sim plifier cet ensem ble déso rd o n n é en décom posant chacun
des champs électriques de la figure 7.29 a) en ses com posantes y e t L o r s q u e l’onde vcxis
drqja.sse. la coiiipo.sanie résultante y oscille parallèlem ent à l’axe des y. et la com posante
h) résultante r oscille parallèlem ent à l’axe des c. O n peut ensuite rep résen ter la lum ière
figure 779 «» I j luniièic non polariséf non polarisée par une paire de flèches bidircciionnelle.s, com m e dans la figure 7 29 b)
SC coin|Kisc d'ondes doni les champs L a llè ch e b id ire c tio n n e lle le long de T ax e d es y rep rése n te les o sc illa tio n s d e la
clc('lrii|iH's iim line direcliini aléaUiirtr. com posante résultante v d u cham p électrit|uc. La flèche bidirectionnelle le long de l’axe
Ixs ondes se piopagcni louu s suivani d es rep résen te les o sc illa tio n s de la co m p o san te résu ltan te z du ch a m p éleclriq u c.
le même axe. soiiani dirct lenieni De cette façon, on change effectivem ent la lum ière non ptrltuiséc en une superposition de
(In plan de la page, ei om loulcs la même
deux ondes polarisées d o n t les plans d ’o scillatio n sont p erpendiculaires - un plan est
amplitude F. b) Une autre façon
parallèle à l'axe des y et l’autre est parallèle à Taxe des ;. C e changem ent se justifie par le
de représenter la liimiên’ non polarisée-
la lumière est la superposition de deux lait que le dessin de la figure 7.29 h) est beaucoup plus facile à interpréter que celui de
ondes polarisées dont les plans la figure 7.29 a).
d'oseillatioii sont perpendiculaires O n peut dessiner des schém as sem blables pour représenter la lum ière p artielle m e n t
entre eux. p o la risée tics oscillations de son cham p électrique ne sont pas com plciem cnl aléatoires,
com m e diuis la figure 7.29 a), ni parallèles à un axe, coim ne dans la figure 7.28 b). Pour
représenter cette situation, on peut dessiner une flèche bidirectionnelle plus longue que
l’iiuire, dans l’agencem ent perpendiculaire des flèches.
Il est possible de transform er la lum ière visible non polarisée en lum ière polari.sée
si la lumière e.st transm ise par un polariseiir, conune le m ontre la figure 7.S0. C es pohi-
riseurs, commerc ialisés sous le nom de polaroid on de filtres polaroïd. ont été inventées
en 1932 par Ldwin L and alors q u ’il pout suivait .ses éludes. Un polari.seur est une feuille
m ince qui se com p o se d e certain es m o lécu les longues im briquées d an s du plastique.
L ors de sa fabrication, la feuille est étirée afin d ’aligner les m olécules en rangs paral­
lèles. com m e des sillons dans un cham p labouré. L orsque la lum ière est envoyée à tra­
R a v o n lii in i n r ii x
vers la feuille, les com posantes dn cham p électrique orientées dans une direction passent
IIU llll'lll
à travers la feuille, alo rs que les co m p o san tes perpeiidiculairc.s à celle direction sont
absorbées par les m olécules et disparaissent.
I iim ièrc
non polarisée On ne s ’attardera pas sur ces types de m olécules, m ais on attribuera au polariseiir
Polanseiir un axe de transmisión, dans lequel les com posantes du cham p électrique sont transm ises.

I I iiiiiè re p o lm i « V L.a composante du champ électrique parallèle à l'axe dr tran.sinission est transmise
v e iiit .d e n ie n l
par le polariseur. la composante peipendiculaire à cet axe csl absorbée.
Figure / 30 Iji lumière mm polarisée
devient lailarisée lorsqu'elle passe
il travers un polansein-. Sa direction Le cham p électrique de la lum ière qui ém erge alors de la fouille se com pose strictem ent
de polansalion est alors paifillèlc à celle des com p o san tes parallèles à l'a x e d e tran sm issio n de la fe u ille ; la lu m ière csl donc
du polariscur. qui est représentée polarisée dans cette direction. D ans la figure 7.30, les com posantes verticales du cham p
ici par les lignes verticales tracées élccin q u c sont transm ises pat la feuille ; lc.s com posantes horizonlalcs sont absorbées.
sur eclui-ci. Les ondes transm ises sont alors polari.sécs verticalem ent.
7.10 La polarisation 197

L’intensité de la lumière polarisée transmise


Considérons m aintenant l'intensité de ht lum ière transm ise par un polari.seur. On procède
d ’abord avec la lu m ière nttn p o la risée , d o n t les o sc illa tio n s d u c h a m p é lec triq u e
exprim ées selon les com posantes v et c sont représentées dans la Ligure 7.29 b). O n peut
aussi laire en sorte que l'a x e des v soit parallèle à l’axe de transm ission du polariscur.
A lors, se u les les co m p o san tes y du ch a m p é le c triq u e de la lu m ière so n t tran sm ise s
p a r le p o la risc u r; les co m p o san tes c sont ab so rb é es. Tel q u e ce la est su g g é ré p ar
la fig u re 7.29 b ), si les o n d es in itiales o n t une o rie n ta tio n aléa to ire, la so m m e des
co m p o san tes y et ce lle d es co m p o san tes c so n t ég a le s. L o rsq u e les co m p o san tes z
sont absorbées, la m oitié de l'in te n sité I, de la lum ière initiale est perdue. L’intensité /
de la lum ière polarisée ém ergente est alors
Figure 7.31 Lumière polarisée
s’approchant d ’un polariscur.
7= (7.33)
On peut décomposer le champ
élcctri(|iie E de la lumière en
scs composantes Ey (parallèle O n nonim c ecltc règle la rèj^le de la moitié-, on ne peut Tuiiliser que lorsque la lum ière
à l'axe de transmission du polariscur) qui atteint le polariseur est non polarisée.
et E. (perpendiculaire à cci axe). SupiKwons m aintenant qu e la lum ière qui atteint un polari.seur est déjà polarisée.
La composante sera iransmisc L a fig u re 7.31 rep rése n te un polari.seur o n e n té d an s le plan d e la page, et le ch am p
par le polariscur ; la composante E.
éltx triq u e E d ’une onde lum ineuse polarisée se dirigeant vers le ptilariseur (donc, avant
sera absorbée.
que se p rodu ise to u te ab so rp tio n ). O n peut d éc o m p o se r E en ses d eux co m p o san tes
p a r rap p o rt à l’ax e d e tran.sm ission du p o la rise u r; la co m p o san te p ara llèle £ , est
transm ise par le polari.seur. et la com posante perpendiculaire E. est absorbée. Puisque
l'a n g le entre E et T axe d e iransm is.sion du p o lariseu r e st 6, la co m p o san te p arallèle
transmi.se est

£'„ = F eo s ft (7.34)

R a p p elez-v o u s q u e r in te n s iié d 'u n e o n d e élec tro m ag n é tiq u e (c o m m e fo n d e


lumincu.se) est proportionnelle au carré de l’amplitude du cham p élecu ique (équaiiiai 4.26).
D ans le présent ex em p le, r in te n s iié / de l ’o nde ém erg en te est p ro p o rlio n n d lc à E],
c l rin te n siié £ de l'o n d e initiale est proportionnelle à E^. D onc, en fo n c tio n de l'é q u a ­
tion 7.34. on peut écrire ///, = c o s ' ft ou

/ = /, cos- ft (7.35)

C ette équation est la loi de M ains ; on ne peut l ' utiliser q ue lor.sr|ne la lum icrc (|ni atteint
le polariseur est déjà polarisée. L ’intensité transm ise / atteint alors un m axim um Cl est
égale à l’intensité initiale /,. lorsque l ’onde initiale est polarisée parallèlem ent à i'ax c
de transmis.sion du polaiiscur (lorsque l’angle Öde l'éq u atio n 7.35 est cleO" ou de ISO”).
/ est nulle lorstgie fo n d e initiale est polariséti perpendiculairem ent à f a x e de transm is­
sion du polariseur (lorsque ft vaut 90")
La figure 7.32 présente un arrangem ent tlans letiiid une lum ière inilialcm cnt non
polarisée est d irig ée vers deux p o lan seu rs, £, et A . (O n appelle souvent la prem ière
feuille le polariseur, et la deuxièm e feuille, Vanalyseur). Puisque l’axe de lransmis.sion
de ft, est vertical, la lum ière transm ise par ft, à Pi est polarisée verticalem ent .Si l’axe
Figure 7.32 La lumière transmise de tr.tnsmis.sion de ft. c.st égalem ent vertical, toute la lum ière liansmi.se p ar ft, est au.ssi
par le polariseur ft, est polarisée transm ise par ft2. Si I’axc de transm ission de ft, est horizontal, aneune liiniièie irattsniisc
verticalement, telle qu’elle est par ft, ne sera lransmi.se par fti. On arrive aux mêmes conclusions en con.sidénuil sculemeni
représentée par la flèche verticale les orientations relatives des deux jrolariscurs : si leurs axc.s de tran.smission sont parallèles,
bulircetionnelle t.'inlensité
toute la lum ière tran sm ise p ar le p rem ier p olariseur l’est égalem ent par le d ru x icm c.
lumineuse qui sera ensuite transmise
Si CCS axes .sont perpendiculaires (on dit alors que les polari.seurs .sont croisés), aucune
par le polariseur fti dépend de fanglc
lum ière n ’est transm ise pat le deuxièm e polariscur. Ces deux extrêm es sont repm sem ês
entre la direction de polarisation
de cette lumière et l'a.xe de transmission par des verres polarisés dans la figure 7.33.
de P2 (indiqué par les lignes tracées F inalem en t, si les d eux axes de transm i.ssioii d e la figure 7..32 fo rm en t un angle
dans le polariscur, et par la ligne com pris entre 0 ‘ et 90'^. une partie de la lum ière transm ise par ft, sera iiaiiMiiisc pai ft;
pointilléc). [.’intensité de cette lum ière est déterm inée par l’équation 7.3,5
198 Chapitre 7 La diffraction et la polarisation

Rgure 7.33 Les lunettes de soleil polarisées comportent des lentilles dont l’axe de iransmi.ssion
est vertical quand on les porte, al Des verres superpo.sés transmettent bien la lumière lorsque
leurs axes de transmission sont les memes, mais b) ils bltxjucnt la plus grande partie de la lumière
lorsqu’ils sont eroisés.

I .a lum ière peut être polarisée aulrcm ent q u 'à l’aide d ’un polariseur ; elle peut l’être
p ar la réfiexion, p ar ex em p le (q u ’on ciu d iera dan.s la sectio n 7 .1 1 ), ou p ar diffu sio n
des ato m es ou d es m o lécu les. Ix trs de la diffusion, la lu m ière q u i a é té in te rcep tée
par un objet, une m olécule, p ar exem ple, est renvoyée dans p lusieurs d irections, sans
doute aléatoires. O n en a un ex em ple dans la d iffu sio n d e la lum ière du soleil par les
moIécule.s de l’atm osphère, qui donne sa teinte particulière au ciel.
M êm e si la lum ière provenant d irectem ent du S oleil est non polarisée, la lum ière
qui vient d ’une bonne partie du ciel est partiellem ent polarisée par cette diffusion. Les
abeilles utilisent la p olarisation de la lum ière du ciel p o u r s ’oricniei dans leur
va-et-vient autou r des ruches. Les V ildngs l ’u tilisaienl égalem ent pour naviguer dans
la mer du Nord, lorsf|tic le wdeil du jour était sous l’horizon (à cause de la latitude élevée
de la n ie r du N ord). C e s premier.s e x p lo ra te u rs av aien t d éc o u v ert ce rtain s cristau x
(qu’on appelle m aintenant eordiérites) qui changeaient d e couleur lorsqu’on les tournait
dan,s la lum ière polarisée. F n regardant le ciel à travers ce cristal, en le faisant pivoter
dans leur ligne de vue, ils p o u v aien t lo c aliser le soleil co u ch é et d éterm in e r ainsi la
direction sud.

Exemple 7.6
La figure 7.34 a) rcprésenle un système de trois polariseurs placés
dans le parcours d'une lumière inilialemeni non polarisée. L'axe de
transmission du premier polariseur est parallèle à Taxe des y, celui du
deuxième forme un angle de 60” par rapport à l’axe des y mesuré dans
le sens aniihoraire et celui du troisième esi parallèle à l’axe des s.
LHielle fraction de rintensité initiale /(, émerge du système, et comment 4 -
V b)
cette lumière est-elle polarisée ?
SOlUTIOfL Les concepts dés sont les suivants.
c)
1. On analyse le système en s’intéressant à un polariseur à la fois,
du premier qui intercepte la lumière au dernier.
2. Pour trouver l'intensité transmi.se par un polariseur, on applique
soit la règle de la moitié, soit la loi de Malus, scion que la lumière
qui atteint le polariseur est non polarisée ou déjà polarisée.
3. La lumière transmise par un polariseur est tinijours polaiisée figure 7.34 Exemple 7.6 a) Une lumière initialement non polarisée
parallèlement à l’axe de transmission du ptdariscur. d'intensité /„ est envoyée dans un système de trois polariseurs.
Premier polonseur : L'onde lumineuse initiale est représentée Les intensités 7|, 7j et / , de la lumière transmise par les polariseurs
dans la figure 7..34 b) à Laide de la représentation par flèches bidirec­ sont indiquées. On indique aussi la direction de polarisation,
tionnelles de la figure 7 29 b). Ju sq u e la lumière était initialement vue de face, b) de la lumière initiale et de la lumière transmise
non polarisée. Lintensiié /, de la lumière transmise par le premier c> par le premier polariseur, d) par le deuxième polariseur et
polariseur e.st donnée par la règle de la moitié (équation 7.33) : c) par le tRiisième polariseur.

I'I T
im

Puisque Taxe de transmission du premier polariseur est parallèle à La polarisation de cette lumière transmise est parallèle à Laxe de
Laxe des V. la polari.satioii de la lumière transmise par le polariseur transmission du polariseur qui la transmet - c'est-à-dire 60°
l’est aussi, comme le montre la vue de (ai e de la figure 7..34 c). par rapport à l’axe des y. en sens aniihoraire, comme le montre la vue
Deuxième polnriseur : Piiisijiie la luinicre qui atteint le deuxième de face, ck- la figure 7..34 d).
polariseur est polarisée, l'intensité /. de la lumière transmise par ce Troisième polariseur : Puisque la lumière qui atteint le troisième
polariseur esi donnée par la loi do Malu.s (équahon 7.35). Dans cette polariseur est polarisée, l'intensité /3 de la lumière transmise par ce
regie l'angle Hest l'angle entre la direction de polarisation de la polariseur e,st donnée par la loi de Malus. 13angle fl est maintenant
Inniièn; im iilrnir (parallèle à l’axe des y) et l'axe de transmission du l’angle formé par la diiection de polarisation de la lumière ineidente
deuxième polariseur (f>0" par rapp<'>rl à l’axe des ven .sens antihoraire), (figure 7.34 d) et par l’axe de transmission du troisième polanseur
ce qui fait que fl vaut liO" Alors. (parallèle à l’axe des r), de sorte que fl ~ 3fl”. Donc,

- /, COS-’ 60“. / , = ¡2 cos* 30°.


7 11 La polarisation par réflexion 199

Celle dernière lumière transmise est polarisée parallèlement à l’axe / VÉRIFIEZ VOS CONNAISSANCES 7 : La figure représente quatre
des A (figure 7.34 e). On trouve son intensité en insérant d’abord paires de polariscurs, vues de face. Chaque paire est placée
et, ensuite, /| dans l’étiuaiion suivante • dans le parcours d'une lumière initialement non polarisée
(comme l'étaient le.s trois polariscurs de la figure 7.34 a).
L’axe de transmission de chaque polariseur (indique par la ligne
/3 = /, cos- 30" = (7, eos^ 60") cos- 30"
liointilléc) est mesuré par rapport à un axe horixonial a ou à un axe
= i{iu) cos- 60" cos^ 30" = 0,094/,,, vcriical y. Classez ces paires en ordre décroissant selon la fraction
de riiiten.sité initiale qu'elles transmettent.

Donc. ,30;' •SCI


f-
— ^ ( .ir
- = 0.094. (rcptinse) 30"
^0
^Ü0" •\60"
Cela revient à dire que 9 ,4 '^ de l'intensitc initiale émerge du
sy.sièmc formé des trois polariseurs, (Si on enlevait maintenant le 60"
deuxième polariseur. quelle fraction de l'intensité initiale émergerait
du système ?) a) h) r) (11

7.11 La polarisation par réflexion


Vous p o uvez faire v arie r l’éc la t de la lum ière du soleil qui est rélléch ie p ar l 'ea u en
regardant à travers un polari.seur (com m e des lunettc.s .solaires polarisées) cl en tournant
l’axe de polarisation du polariseur. C ette variation de i’intcnsilé se produit parce q ue la
lumière rctléchie est totalem ent ou partiellement polarisée par la a^flcxioti sur une surface.
La figure 7.35 illu stre un rayon de lu m ière non p o larisée in cid en t à une su rface
de verre. O n décom pose le v ecteu r ch am p électriq u e tie ce rayon de lum ière en deux
com posantes. L a composante perpendiculaire est |ieipeiidicu)aire au plan d'inc idence,
donc égalem ent au plan de la page dans la figure 7.35 ; cette com posante est représentée
par des points (com m e si on vtiyait les bout-s des vecteurs). L a composante parallèle est
parallèle au plan d ’incidence et à celui de la p a g e ; elle c.sl représentée par des flèches
bidirectionnelles. C om m e la lum ière est non polari.scc, les deux com posâm es o u t une
am plitude égale.
E n général, la lum ière réfléchie a égalem ent deux com posantes, mai.s d 'am p litu d es
différentes. C ela signifie que la lum ière réfléchie e.st partiellem ent polarisée - le cham p
électriq u e o scillan t dans une d irection donnée a une plus grande am p litu d e q ue ceux
oscillant dans d ’autres directions. Toutefois, lorsque la lumière est à un tuigle d ’incklcncc
particulier, nommé l'anf^lede Urewsterioa angle de palarisation). du. la lumière rcflcchie.
n ’a q u ’une co m p o san te p erpendiculaire, com m e le m ontre la figure 7..35. l.a lum ière
rcflcchie est alors to talem en t p o larisée, perp eiu lieu lairem en i au p lan d ’iin idcncc
L i com posante parallèle d e la lum ière incidente ne disparaît p as, m ais (tout ( tniim e la
com posante pcipcndiculaire) elle c.st réfractée dans le verre.
Le v erre, l’eau et les a u tre s su b stan ce s d ié le c triq u e s d o n t on a d isc u té d an s la
Rawn
im ulent non R.ij'On section 5.7 du volum e 2 peu v en t p olariser to talem en t ou p ariicllem cnl la lumit'‘re par
rj polari.sé réfléchi réflexion, lo r s q u e vous interceptez la lum ière du soleil réfléchie par une surface d e ce
Ï-V-) type, vous voyez un point brillant (le reflet) sur la surface, là où la réflexion sc produit.
'b, I
L Si la su rfa c e est h o rizo n tale, co m m e ce lle de la fig u re 7.35. la lu m ière réfléch ie est
,-X s I '> •' p artiellem en t ou to talem en t p o larisée hori/.onialeuient P our élim in e r ces reflets des
.\ii
.» = -1.5 Vem: surfaces horizontales, les lentilles des luneties solaires (solarisécs .sont montée s de fa^on
que leur axe de transm ission soit vertical.
. ' S», • ■■
Figure 7.35 Un rayon de lumière mm polarisée dans l’air est incident à une surface do vi'rro
! S ' Rayon à un angle de Brewster 6^. Les champs élce-triqucs de ce rayon ont été décomposés selon
Hiv-ft-aerr une composante perpcndû'nlaire au plan de la page (le plan d’im idcncc. de réflexion
et de réfraction) et une composante parallèle au plan de la page La lumière réfléchie
(loniposanie (H rjiendinilaire ne pos-sède qu’une composante jierpendu ulaire au plan de la pape, cl est doni [Xilarisér
au plan de la jiage dans cette direction. La lumière réfractée possède la composante initiale parallèle au clan
(toiiiposame parallèle de la page, et une composante plus faible (lerpcndtculairc à ce plan ; cette lumién' est
au plan de la page partiellement jxilan.sée.
200 Chapitre 7 La diffraction et ta polarisation

La loi de Brewster
Pour une lum ière incidente à l’angle de B rew ster 0^, on peut déterm iner, expérim enta­
lem ent, que les rayons réfléchis et les rayons réfractés sont perpendiculaires les uns par
rapport aux autres. C om m e le rayon est réfléchi à l'an g le 0^ dans la figure 7.35 et que
le rayon est réfracté à un angle 0,. on a

^ = 90“. (7.36)

Ces tleux angles peuvent aussi être reliés par l'équation 4.41. Si im assigne arbiuairem ent.
d ans cette éq u atio n , l ’in d ice 1 à 1« su b stan ce d a n s laquelle les ray o n s in cid en ts sont
réfléchis, cette équation donne

/ii sin = «2

En com binant ces équations, on obtient

«1 .sin ~ ” 2 sin(90° (^ ) = «2 w>s 0R,

ce qui donne

Wy — t a ï C '^ — ^ (l'angle de Rrcwsicr). CI.?n)

(N o tez bien q u e les in d ices d an s l'é q u a tio n 7 .37 ne sont pas a rb itra ire s: ils doivent
respecter la .signification q u ’on leur a d o n n é e ; /i| est rin d ic c du m ilieu où se propagent
les rayons incidents et réfléchis et ro est l’indice du milieu où les rayons sont réfinctés.)
.Si les rayons incidents et réfléchis voyagent dans l'air, on peut faire l’approxim ation que
M, vaut un. et représenter rh par n pour écrire l’équation 7.37 sous cette form e :

(^ = tan * /1 (la loi de Brew.sicrj. (7.38)

C ette version sim plifiée de l’équaüon 7.37 est nom m ée la loi d e B rew ster. C om m e f^ .
elle porte le nom de S ir D avid Brewster, qui la dém ontra expérim entalem ent en 1812.

La d iffra c tio n Lorsque des ondes rencontrent une arête, un maxima concentriques, et la position angulaire fl du premier minimum
obstacle ou une ouverture dont la taille est comparable à leur est donnée par
longueur d'onde, elles .se dispersent en sc propageant et produisent
de rinterférence, i "est ce qu'on nomme la diffraction. sin fl = 1 .2 2 — (le prcmiei minimum, ouvemire circulaire). (7.12)
II
I m diffraction produite par une fe n te unique Les omles qui
pas.scnl par une fente longue cl mince de largeur a génèrent une L e critère de Rayleigh Selixi le critère de Rayleigh, deux objei.s
figure de diffraction produite par rinterférence entre les différentes sont tout ju.sie .séparés lorsque le maximum central de l'un coïncide
ondes qui parcourent des distances diflërentcs avant d'atteindre l’écran avec le premier minimum de l’autre. Leur sépaniiion angulaire doit
d ’obsci vaiioii. l a figure comporte un maximum central et d'autres alors être d ’au moins
maxima sépares par des minima, dont la position angulaire 6 par
nippori à l iixc central correspond à
Ofi = 1.22 - (le critère de Rayleigh), (7 .1 4 )
a
i7 sln 4 = »iX, lorsque«I = r 1. ±2. ... desminitna» (7.2)
où d est le diamètre de l'ouverture par laquelle la lumière pa.ssc.
Pour un angle 0 donné, l'intensité lumineuse de la ligure de dif
frariion esl l a diffraction produite pa r deux fentes Les ondes qui passent
à travers deux fente.s de même largeiii a dont les centres sont séparés
m
-'-Ctl ■¡ta
■— sin fl
À
(7 .-i. 7 6) par une distance d produisent des figures de dilTraciion dont l'inten­
sité /, à l'angle fi. est

cl où /„ t.st l'imen.sité au centre de la figure.


/(fl) = /„(COS' (le système à deux fentes), (7.16)
I m diffraction prttduiU'p o r une ouverture circulaire l.a dif
traction priuluite par une ouverture circulaire ou par une lentille OÙ /1 = (a-r/ZÀ) sin fl. et of a la même valeur que dans une diffraction
de diamètre d pn>duit un maximum central entouré de mínima et produite par une fente unique.
Questions 20!

La diffraction produite p a r des fen tes multiples L<i diftrac- 2d sin 0 = mk. lorsque m — 1, 2, 3,...
lion produite par N fentes (fentes multiples) produit des maxima (la condition de Bi agg). (7.31)
(pics) aux angles 6 de manière que
«
dsin 0 mk, lorsquern = 0, ± I. J .2,... (lesmaximal. (7.22) Im polarisation Les ondes électromagnétiques sont polarisées si
les vecteurs de leur champ électrique sont tous dans le même plan,
ci les demi-largeurs angulaires des pics .sont données par noimnc plan d'oscillation. Les ondes lumineuses de sources communes
ne sont pas polarisées ; on dit qu elles sont non polar'isées, ou qu’elles
Aftu —- (la dcmi-lmgcur angulauc d’un pic à W>. (7.2.‘H) ont une polarisation aléatoire.
Nd cosO
La polariseurs Lorsqu’un poluri.seui est placé dans le parcours
Les réseaux de diffra ctio n Un réseau de diffraction est une de la lumière, seules les œmposanies du champ électrique de la lumière
série de « fentes » ou de « traits » qui permet de séparer une onde parallèles à l'axe de Iran.smissioii du ixilariscur sont transmises ; les
ineidenie en ses différentes longueurs d'onde, en projetant les niaxima composantc's ijerpendiculaires à l’axe de iransmissiixi sixit absorliccs.
de dil fractioii de ces longueurs d'onde sur un écran. Un réseau est La lumière qui émerge d’un jxilariseur est (tolariséc parallèlement à
caractérisé jiar son pouvoir dispei-sil' D et son pouvoir de résolution R : l’axe de transmission du poiariscur
Si la lumière est initialement polarisée, l’intensité / tramniisc
A» m vaut la moitié de l'intensité imlialc /j;
D = (7.26. 7.27)
Aà d cos H
(7.33)
(7.28, 7.29) Si la lumière e.st initialement polarisée, l'intensité / üansmi.se dépend

de l'angle 6 entre la direction de la polarisation de la lumière initiale
Im d iffra ctio n des rayons X Le réseau régulier des atomes et l'axe de tian.smùssion du poiariscur:
d'un cristal constitue un réseau de diffraction tridimensionnel pour l ^ /, cos2 R. (7.33)
des ondes de petite longueur d’c'nde comme les rayons X. À des fins
d’analyse, les atomes jieuvent être représentés suivant différents I m polarisation pa r réflexion l ’nc onde rélléehie sera totalement
plans où d représente l’espacement entre ces plans. Les maxima de polarisée, et ses vecteurs £ seront jierpendic ulaires au plan d’inci­
diffraction (générés par une interférence constructive) se produisent dence, si elle rencontre une interface à l’angle de B n vvstrr On •
si la direction incidente de l'onde, mesurée à partir de la surface
de ces plans, et la longueur d’onde à du rayonnement satisfont aux -1 ” 2
On " lait (l’angle de Brewster), (7.37)
exigences de la condition de Bragg : «1

1. Une lumière de fréquence/qui éclaire une longue fente mince 5. Dans une cxpér'ience sur U diffraction produite par une fente
produit une figure de diffraction, a) Si la fréquence de la lumière est unique, le rayon du haut et celui du bas de la fente arrivent à un certain
plutôt 1,3/ la figure se dilate-t-elle ou sc rappr(x;he-t-elle du centre ’’ point sur l’écran d'observation avec une différence de marche de
h) La figure sc dilaie-i-elle ou sc contracte-t-elle si on immerge le 4.0À. Dans un diagramme de vecteurs de Fresnel comme celui tic la
système dans du sirop de maïs clair ? figure 7.6. combien de cercles superposés la chaîne de vecteurs
2. Vous mener une expérience sur la diffraction produite pat unc- de Fresnt'l forme-l-cllc ?
fente unique avec une lumière de longueur d’onde À Sur un écran 6. La nuit, de nomhieuses perstmnes voient dc.s anneaux (nommés
d'observation éloigné, qu’cst-ce qui apparaît au point où le rayon halos eniopiitjues) auloui de la lumière des liimp«dairc.s cxtci icurs
issu du haut et celui issu du bas de la fente ont une différence de situés dans des zones olsscures T es anneaux soin les premiers
maiche égale a) à 5k et h) à 4.5À ?
maxima secondaires des ligures de diffiactiun produites par des
3 Si von.s parlez en gardant le même niveau d ’intensité avec et sans structures de la cornee (ou du crisialliiu dans l'o-il de I’cihsiTvairur.
un mégaphone devant votre bouche, daas quelle situation serez-vous (lafs maxima centraux de t es figures chcvauclicril Ic.s hmiièrcs dc.s
perçu avec la plus grande intensité par une persomic qui est directe lampadaires.) a) l'n anneau donné deviendraii-il plus petit on plus
ment en face de vous
grand .si on utilisait une lumière rrmpe pluttSt qu’une lumière bleue
4. La figure 7..3b montre quatre ouvertures rectangulaires possibles dans le lampadaire? b) Si un lampadaire émet une lumière hlancbe.
pour une source d’ondes sonores ou d'ondes lumineuses, l.es côtés la luniiere e.st-ellc bleue ou rouge au bord extérieur de l’anneau ’
ont des longueurs / ou 2/. L valant 3,0 fois la iongucui d ’onde des
7 La figure 7 37 représente les franges hriilaiitc.s comprises dons
ondes. Classez les ouvertures en ordie décroissant aj scion l’étendue
l’enveloppe centrale d ’iine figure de diffnu lion Im t il’cxpénence.s
de l étalemcnl horizontal des ondes causé par la diffraction et
b) scion l’étendue de rétalement vertical des ondes sur la tliffraction prtvluite par deux femes , on a iilili.sé la môme
longueur d’onde dans les deux expéiiences. a) La laigeui u dt la fente,

□ b) la distance d entre les fentes et c) le rap|x>rt dla de l’expérience P


sont lis plus grands, moins grands ou les mêmes que ceux de
l'expérience 4 ’
I) 2) 3) -1)

Figure 7.36 ( ) u e s t io n 4
202 Chapitre 7 Lia diffraction et la polarisation

V ..
11. a) Dans un réseau de diffraction donné, la plus petite différence
1 - -. '■ • Ч • • iX ciitie deux longueurs d’onde pouvant être séparées augmcntc-t-cllc,
diminue-i-elle ou resie-i-elle la même lorsque la longueur d'onde
•s
T
JJ
augmente? b) Dans une plage de longueur d ’onde donnée (environ
550 nm, par exemple), est-elle plus grande dans le premier ordre
ou dan.s le troisième ordre ?
Figure 7.37 Oucstion 7
12. La ligure 7.40 représente une lumière qui atteint un jsolariseur
dont l’axe de lransmis.sion est parallèle à un axe des y. On tourne le
8 La figiin.' 7.38 rcpniscntc une raie rouge et une raie verte du même
jxtlariseur de dO” dans le sens horaire par rapport au parcours
ordre dans la figure prrxiuilc par un lésettu de ditîraction. Si on augmente
de la lumière indiqué. Durant celle rotation, la fraction de l’inten­
le nombre de fenlc.s éclairées dans le réseau, en enlevanl, par exemple,
sité lumineuse initiale transmise par le polariscur augmente-t-elle,
un ruban qui recouvre la moitié des fentes, est-ce que a) les demi-
diminue l-elle ou reste-i-clle la même si la lumière e.st a) initialement
largeur.s angulairc.s des raies et b) la di.stance entre les raies augmen non polari.sée, b) inilialement pttlariséc parallèlement à l’axe desx
leroni, diminueront ou resteront les mêmes ? c) Les raies .seront elles cl e) iiiitialcnient polarisée parallèlement à l’axe des y?
dccalcc.s vers la droite ou vers la gauche, ou resteront-elles au même
endroit ?

L
Figure 7.38 Questions 8 et 9

9. Ketourne/ à la situation déente dan,s la question 8 et à la figure 7.38.


Si on augmente plinC)i le pas du réseau, a) les denii-largcurs angu­
laires des raies et bi la distance entre les raies augmcntcroni-ellcs.
diminucntnt-elles ou resteront elles les mêmes ? c) Les raies seront- Figure 7.40 Question 12
elles décalées vers la droite ou vers la gauche, ou resteront-elles au
même endroit 13. Reportez-vous à la figure 7 34 a). Procédez d ’abord avec la
lumière qui est initialement polarisée paialiclemeni à l'axe des x, et
10. aj La figure 7.39 al représente les raies produites par des réseaux
écrivez le rapport entre .son intensité finale et son intensité initiale
de diffraction A e \B utilisant de la lumière qui a la même longueur l(t en utilisant la forme l jl^ = A eos" 6. Trouvez A. ti cl 6 lorsqu’on
d ’onde , Ic.s raies sont du mcinc ordre et apparai.sseni aux mêmes tourne l’axe de tran.smi.ssion du premier polariscut a) de bO' dans
angles 6. Quel réseau a le plus grand nombre de fentes éclairées? le sens aniihoraire et h) de 9(ri dans le sens horaire, par rapport à
b) La figure 7.39 b) représente les raies de deux ordres produites par l'illustration.
un seul réseau de diffraction utilisant une lumière compttsée de deux 14 .Supposez iTwintenanl qu’on tourne le deuxième polariseur de la
longueurs d’onde, situées toutes les deux dans la région rouge du figure 7.34 a), en alignant d’abord son axe de transmission avec l’a.\c
spectre. Quelles raies correspondent à l’ordre oii m est le plus grand ; des y (6 0) et en l’alignant ensuite avec l'axe des a: (P = ÇO”).
celles de la paire de gauche ou celles de la paire de droite? Laquelle des trois courbes de la figure 7 41 ülustFe le mieux l’intciisitc
c) Le centre de la figum de diffiaciiun est il à gauche ou à droite dans de la lumière qui traverse le système des trois polariscurs pendant
la figure 7.39 a) ? d) Et dans la figure 7.39 b) ? cette rotation de 90” ?

b L T - T -
EU
a) b)

Figure 7.39 Q u e st io n 10 Figure 7.41 (Question 14

www l.a solution SC trouve sur le site Web. à l'adresse ei-dessoiis ■ 3E. Une fente simple est éclairée par une lumièm ayant des longueurs
d ’onde de et de choisies afin que le premier minimum de
ww.w.dlcmcgrawhill.ca/physiqiie
diffractiirn de la composante X„ coïncide avec le deuxième minimum
de la composante X*. a) LJudlc est la relation entre les deux longueurs
SFCnON 7.2 La diffrodicn produite par une seule fente : localiser les minima d'o n d e? b) Y a t il d’autres minima qui coïncident dans les deux
figures de diffraction ?
IF line lumière incidente d’une longueur d’tHide de 633 nm éclaire 4f. La distance entre le premier minimum et le cinquième minimum
une lente mince. L’angle entre les prcnner.s minima de chaque côté. d ’une figure de diffraction produite par une fente simple est de
du maximum central eet de 1.20'’. Quelle est la largeur de la tente ? 0,35 mm lorsque l’écran est è 40 cm de la lente, et que la lumière
2F. Fine Uiiiiièrc monochromatique itu idente d'iinc longneui d’onde a une longueur d'onde de 550 nm a) D éterm ine/ la largeur de la
de t И nm éclaire une tente mince. Sur iin écran éloigné de 2,00 m, fente, b) Calculez la position angulaire A du premier minimuin
la distance entre le deuxième minimum de diffraction et le maximum de diffraction
central est de 1,50cm. a) Calculez l'angle Odu deuxieme minimum, 5L Une onde plane incidente d’une longueur d'onde de .590 nm éclaire
h) Determine/ la largeur de la fente. une lente de largeur a = 0.40 mm. Une mince lentille convergente
Exercices et problèmes 203

d'une distance focale de + 7 0 cm est placée entre la fente et un écran opaque comportant un trou et B est le «négatif» de A. À l’aide
d'observation, et focalise la lumière sur l’écran, a) A quelle distance des concepts relatifs à la superposition, démontrez que l’intensité à P
de la lentille se tntuve l’écran? b) Quelle est la distance, sur l'écran, est identique, qu'on cwploie l’objet A ou fttbjet B.
entre le centre de la figure de diffraction et le premier minimum ?
6P. Des ondes sonores d’une fréquence de il 000 Hz et ayant une
vitesse de 343 in/s sont diffractccs en passant dans l'ouverture
rectangulaire d’un haut-parleur et entrent dans un grand auditorium
L'ouverture, dont la largeur horizontale est de 30.0 cm. fait face à un
mur éloigne de 100 m (figure 7.42). Par rapport ii ce mur, à quel
endroit un auditeur sc trouve-t-il s'il est au premier minimum de
diffraction et qu'il a, de ec fait, de la difficulté à entendre le son '?
(Négligez les réflexions.)
f i n i I» H l'l lirWÉl>IZgW

/,-H aiil-pailcui' Figure 7.43 i’ioblème 12


Axe
-
contrai I3P. a) liémontrez que le.s valeurs de a, où les maxima d'im easilé
d'une figure de difiraciion créée par une fente unique apparaissent,
peuvent être trouvées précisément en dérivant l’équation 7.5 par
---- — 100 m - •) rapport à of et en posant le résultat égal à zéro pixtr obtenir la situation
où tan uf a. b) Trouvez les valeurs de a qui correspondent й eette
Figure 7.42 Problème 6
équation en faisant le graphique de la courbe v = tan a et celui de
7P. Une fente large de 1.00 mm est éclairée par une lumière d’une la ligne droite y a, et en trouvant leur point d'intersection, nu en
longueur d'onde de .“iSO nm. On observe une figure de diffraction sur utilisiint une calculatrice pour trouver une valeur appropriée de a
un écran situé à 3,00 m de distance. Quelle est la distance entre les par la méthode itérative, c) Trouvez les valeurs (non emière,s) tic m
correspondant aux maxima successifs dans la figure de diffraction
deux premiers minima de diffraction du même cOté du maximum
prtxluite par une fente simple. Notez que les maxima secondaires ne
central*’ www
se trouvent pas exactement à mi-chemin enlre les minima.
SEOION 7.4 l'in te iisiié lumineuse dans une figure de diffrotlion produite
por une (ente sintpic SEOION 7.5 lo diffrotlion produite por une ouverture tirtiikê rc

BL Une fente large de 0,10 mm est éclairée par une lumière d'une 14E, Supposez que le lampadaire de la question 6 émet une lumière
longueur d’onde de 589 nm. Soit le point P d’un écran sur lequel on d’une longueur d'onde de 5.50 nm. Si le diamètre angulaiie d’im anneau
observe la figure de diffraction créée par la tente ; ce point est à 30” est 2,5°, quel est le dianièire (linéaire) approximatif de la pupille
de Taxe central de la fente. Quelle est la différence de phase entre les qui cause la perception de l’anneau ?
ondes .secondaires de Huygens qui arnvent au p<nnt P en provenance 151. I^ s deux phares avant d'une voiture qui s’appn«■Ik sont c.spurs
du haut et du milieu de la fente ? (Indice : 'Voyez réc|uation 7.4.) de 1.4 ni a) À quelle separation angulaire et b) à quelle distance
9E. St vous doublez la largeur d’une fente simple, l'im ensité du rnaxiniale l'œil (хшпа-t-il les distinguer'.' Suppixsez que le diamètre
maxiimim central de la figure de diffraction est multipliée par 4, bien de la pupille mesure 5.0 nim et que la longiieui d’onde de la lumière
que l’énergie qui pas.se par la fente ne soit que dcHiblée. Expliquez ce est de 5.50 nm. Supposez egalement que seuls les effets de la diffi action
(ihénoinène quantitativement. limitent le 1юи\'о1г de résolution, et que le critère de Rayleigh peut
être appliqué
loi. Une lumière monochromatique d’une longueur d’onde de 538 nin
est incidenle sur une fente large de 0,025 mm. Il y a une distance 161. Une astronaute à bord d ’une navette spatiale affirme qu'elle peut
de 3,5 ni entre la fente et un écran. Soit un jxnnt sur l'écran, situé tiHjt Juste distinguer deux souice.s ponctuelles à la surface de la'Ierre,
à 1.1 cm du maximum central, a) Calculez l’angle Ode ce point, à une distance de 160 km Calculcz a) la séparation angulaire et
b) Calculez a . c) Calculez le rapport entre l’intensité h ce point h) la distance linéaire entre ces sources, en supixxsnnt que les conditions
et l'intensité au maximum central. sont idéales. Posez que X 540 nm et que le diamètre de la pupille
1IP La largeur à mi-hauteur d'un maximum central de diff raction est de l'astronaute c.st de 5.0 mm.
l'angle entre les deux points- dans la figure où l'intensité vaut la moitié 17F. Trouvc'Z la ilislance entre deux points й la surface de la Lune qui
de celle au centre de la figure (voir la figure 7.7 h), a) Démontrez que peuvent tout juste être distingués par le télesio|v du mont Palimiar.
l’intensité diminue à la moitié de sa valeur maximale lorsque qui a un diamètre de 5.1 ni, en supposant (|iie eeiie distance est
sin^ « = o"’/2. bi Vérifiez si (ï 1.39 rad (environ 80*') est une déterminée pai les effets de la diffraction uniquement La disiance
solution de l'équation transcendante de a), c) Démontrez que la entre la Terre cl la I .une est de 3.82 x 10'' km. et la lumière a une
pleine largeur à la mi hauteur du maximum est AO 2 sin ' ' (0.443X/«), longucui d ’onde de 550 nm. www
où « est la largeur de la fente, d) Calculez la largeur à mi hauteur du I8E. Le mur d’une giamie chambre est n-ciaivcrt de hiik-.s aemi iliques
maximum central pour des fentes larges de 1,00. de .S.(X) et de 10.0 parsemées de petits iriHis di»ni les i. entres son! espncé,s de 5.(1 nrun.
longueurs d ’onde www À quelle distance maximale de ces miles une personne peut-elle sc
12P l.e ¡>riru ii)e de Bcihinet. l n faisceau monocliromatiqiic de lumière trouver et eiM'ore distinguer les trous, si on siippixic que lc,s conditions
parallèle est incident à un trou «collim ateur» de diamètre r à. «ont idéales, que la pupille de la pcrstmiic a un diamètre de 4.0 mm
L/C point P SC trouve dan.s une zone sombre sur un écran élni/>né et que la lumière a une longueur d onde de 550 nm ?
(hgure 7.43 a). Deux ob|cts diffractams, illustrés dans la figure 7.43 h), 19E Estimez la distance entre deux olijci.s sur la planète Mars Si
sont placés tour à tour sur le trou du collimateur A est un cercle ces objets pt uveni tout juste cire distingués de la Terre, dans de«
204 Chapitre 7 La diffraction et la polarisation

conditions idéales, a) à l'œil nu et b) à l’aide du télescope de 5,1 tn 24P. Un obstacle de forme circulaire d'un diamètre donné produit la
de diamètre du mont Palomar. L'tilise?. lc,s données suivantes; même figure de diffraction qu’un trou circulaire de même diamètre
distance Terre Mars = 8.0 x 10’ km, diamètre de la pupille = 5,0 mm, (sauf dans le ca.s des valairs très proches ck 6 - 0). Les gouttes de
longueur d’onde de la lumière = 550 nm. pluie en suspension dans l’air .sont des exemples de tels obstacles.
20L Le système de radar d ’un croiseur de la marine transmet des Quand vous voyez la Lune à traver.s des gouttes d'eau en ,su.sj.ierision,
données à une Jongueiir d ’onde de 1.6 cm et à l’aide d ’une antenne comme dans un brouillard, vous interceptez une figure de diffraction
circulaire dont le diamètre est de 2,3 m. Dans un rayon de 6,2 km, provenant de nombreuses giruites, L’easenible des maxima c'entraiix
quelle est la plus petite distance qu'il peut y avoir entre deux vedettes de diffraction de ces gouttes forme une nappe blanche autour de la
pour que le système de radar puisse encore les distinguer comme Lune et peut l’obscurcir. La figure 7.45 est une photographie où l’on
deux objets séparés voit la Lune obscurcie 11 y a deux anneaux pâles et colorés autour
2IP. Les ailes des cicindcles (figure 7.44) doivent leurs couleurs à de la Lune (le plus grand anneau est peut-être trop pâle pour être
une interférence causée par une substance qui forme de fines couches discernable sur cette photographie). Le plas petit anneau es( sur le bord
semblables à dc.s cuticules. Ces couches sont disposées en plaques cxlcricur des maxima centraux produits par les gouttes ; le plus grand
larges de 60 m et produisent tlifférentes couleurs. La couleur que anneau est sur le Ixird extérieur des plus petits maxima secondaires
vous percevez est un effet de pointillisme et d’interférence de produits par les gouttes (voir la fiiiure 7.V). La couleur est visible
ITclIiculw inincc.s. qui varie selon voue iierspeclive. À quelle distance parce que les anneaux sont adjacents aux minima de diffraction
approximative pouvez-vous observer une aile tout en rc.stant dans (anneaux sombres) dans les figures. (Les couleurs des autres régions
Ic.s limite!) vou.s itei niciiani de distinguer les différentes plaques de de la figure se chevauchent trop pour être visibles.)
tiouleur, d ’après le critère de Rayleigh’? l,a longueur d'onde de la al Quelle est la couleur des anneaux siu’ les bords extérieurs des
lumière est de 550 nm et le diamèire de votre pupille est de 3,tXJ mm. maxima de diffraclion? h) Dans la figure 7.45, l’anneau coloré
2?P. En juin l ‘)85. un faisceau de rayons laser fut émis par la .station autour des maxima centraux a un diamètre angulaire valant 1,35 fois
celui tie la Lune, qui est de 0,50". Suppo.sez que toutes les gouttes ont
d’optique de l’armée de l'air américaine h Maui. en Hawaii, et
réfléchi par la navette Discovery, qui se trouvait à 354 km d’altitude le même diamètre. Quel est le diamètre approximatif des gouttes
aii-drssiis rie la station. l e diamètre du maximum central du rayon d'eau?
liLsci cl la (Hssitinn de la navelle était de ni, el la longueur d'onde 25P. a) Quelle est la séparation angulaire de deux étoiles si Icur.s
du faisceau était de 5(X) iiiii. Quel est le diamèire rie rtnivcmire du laser images sont tout jaste séparées par le télescope de Thaw de l 'obser­
de la station de Miiui ? (Indice • Lin faisceau taseï s’cfciid seulement vatoire Allegheny de Pittsburgh 7 La lentille a un diamètre de 76 cm
sous l’ettct de la diffraclion ; l’ouverture ilu laser est circulaire.) et sa distance focale est de 14 m. Posez 2. = 550 nm b) Trouvez
la distance entre ces étoiles tout juste séparée.s si chacune d ’elles est
23P Les syMènie.s radars à r>ndes millimétriques génèrent un faisceau
à 10 années lumière de la Terre, c) Par rapport à l’image d'une de ces
plus étroit que les radars à micro-ondes conventionnels, ce qui les
étoiles vue avec le télescope, ii'ouvcz le diamètre du premier anneau
rend moins vulnérables aux attaques ries missiles antiradars,
sombre dans la figure de diffraction, tel qu'il serait mesuré sur une
a) Calculez la largeur angulaire du maximum central, produit par
plaque phciograpliiquc placée au plan focal de la lentille du télescope.
un faisceau radar de 220 OHz émis par une antenne circulaire de
Supix>sC7 que la structure de l’image est entièrement associée à la
55.0 cm de diamètre. (La fréquence est choisie pour co'incider avec
diffraction à l’ouverture de la lentille, non aux «cireurs» de la lentille.
une «fenêtre» de faible absorption atmosphérique.) b) Calcule?
la même quantité en fonction du radar du croiseur décrit dans 26P. Dans une expérience franco-soviétique visant à analyser la sur­
l’cxcrcice 20. www face de la Lune au moyen d’un faisceau lumineux, le rayonnement

Figuri' 7 44 Pixiblcme 21 Les cicindclcs doivent leurs couleurs Figure 7 45 Problème 24 La couronne autour de la Lune est
à une interférence causée pat la répartition de fines couches la superposition des figures de diffraction provenant de nombreuses
ponctuelles de ciHilenrs, semblables à des cuiicult's. gouttes d’eau en suspension dans l’air
exercices et proiDlèmes Î05

éiTiiç par un laser à rubis (À = Ü.69 f/iii) a été dirigé vers la Lune par 34f. Un réseau comporte 3 15 fentes/mm. A tjiidlc.s longucui s d’oiiJc
un télescope dont le miroir avait un rayon île 1,3 m. Un réflecteur sur du spectre visible peut-on observer une figure do diftraeUon an
la I cille a agi comme un miniir plan cLrciilaiie dont le rayon est de 10 m, cinquième ordie, lors d’uiic cx|>érieiicc .sur hi diffruetion ?
en réfléchissant la lumière directement vers le télescope sur la Terre. 35E. Un réseau comporte 400 fciiic.s/iiini. Dans le .sficctrc visible (de
La lumière réfléchie a ensuite été détectée après avoir été mise au foyer 400 nm à 7(K) nm). combien d’ordres positifs pcuf-il produire lors
par ce télescope. Quelle IVaction de l’énergie lumineuse initiale a d ’une expérience sur la diftraction, en plus de l’ordre m - 0 ? www
été captée parce dctccteiU'? Supposez que. dans chaque direction du 36E. Pour échapper à leurs prédateurs, certains coléoptères (les gynns)
parcours, toute l’cucrgie est comprise dans le pic central de diffraction. .sont colorés par une interférence optique causée par dos ceaillos dont
Talignemcnt forme un réseau de diftraction (qui disperse la lumière
SECTION /.6 La diffraction produite par deux fentes au lieu de la transmenre). Lorsque les rayons lumineux inddniis sont
27£. Supposez que rcnvclopjx; centrale d’iinc figure de diffraction perpendiculaires au réseau, l’angle entre le maximum du premici
produite par deux fentes contient 11 franges brillantes, et que les ordre et le maximum central (de pari el d ’autre du maximum ceniral)
piemiers minima de diffraction éliminent (en coïncidant avec elles) est d’environ 26’'. pour une lumière d ’une longueur d'onde de 550 lim.
les franges brillantes. Combien y a-t-il de franges brillantes entre les (2uel est le pas du réseau du gyrin ?
premier et deuxième minima de l’enveloppe de diffraction? 37P. Une lumière d'une longueur d’onde de 6(K) nm arrive h incidence
28[. Dans une expérience sur la diffraction créée par deux fentes, normale sur un réseau de diffraction. Deux maxima adjacents se
la distance d entre le centre des deux fentes vaut 2 fois la largeur a produi.senl à de.s angles donnés par sin Й = 0.2 et sin в = 0,3. Il n’y a
des fentes. Combien de tranges brillantes sont comprises dans pas de maxima au quatrième ordre, a) Quelle est la distance entre deux
l’enveloppe centrale de diffraction ? fentes adjacentes? b) Quelle est la plus peiiic largeur de fente qu’on
peut observci dans ce réseau ? c) Quels ordres produits par le réseau
29P. a) Dans une expérience sur la diffraction créée par deux fentes,
sont vi.sibles. si l’on considère les valeurs dérivées île a) et de b) ?
quel rappin dta a jxiur effet d’éliminer par diffraction la quatrième
frange brillante due au phénomène d’interférence entre les deux tentes ? 38P. Un asseau se cmripose de fentes d’une largeur de 300 nm espacées
b) Quelles autres franges brillantes sont également éliminées ? de 91K) nm. Le ré.seau est éclairé par des ondes planes monoebroma-
liques de longueur d’onde X = 600 nm qui sont incidentes selon la
30P. Deux fentes de largeur a et séparées par une distance d sont
normale, a) Combien de maxima y a-t-il dans la figure de diffraction
éclairées par un faisceau de luniièm cohérente de longueur d'onde X.
entière? b) Quelle est la largeur d ’une raie spectrale ob.servée au
Quelle est la distance entre les franges d ’interféicnce brillantes
premier ordre, s’il y a I (ХЮ fentes dans le réseau ?
observées sur un écran situé à une distance D?
39P. Supposez qu'on fixe arbitraiiiement les limiies du spectre visible
31P. a) Combien de franges brillantes apparais.sent entre les premiers
à 430 nm et à 680 nm. Calculez le nombre de fentes par millimètre
minima de l’enveloppe de diffraction d’un côté ou de l'autre du
dans un réseau qui di.sperscra le s|rectre du premier ordre .suivant un
maximum central dans une figure créée par deux fentes si X = .350 mn,
angle de 20". www
d = 0.150 mm et « = 30.0 ¡im ? b) Quel est le rappoit entre l’intensité
40P. Lorsqu’une lumière incidente provenant d’un tube à décharge
de la troisiènx; frange brillante et l'intensité de la frange centrale?
tomlrc selon la normale .sur un réseau dans lequel la distance entre les
32P. Une lumière d'une liHigucui d ’onde de 440 nm passe dans un
fentes es( de 1,73 /rm, de fins maxima de cxHileur verte sont pnxluits
système à deux fentes, créant une figure de diffraction dont le graphique
aux angles (i = j. 17,6'. 37,3'. -.3 7 ,1 ', 65.2' et - 6 5 .0 ’. Calculez la
de l’intensité lumineuse en fonction de la position angulaire (lest
longueur d ’onde de la lumière verte qui cortx»pond le mieux à ces
illustré à la figure 7.46. Calculez a) la largeur de chaque fente
données.
et b) la distance entre les fentes, c) Vérifiez les intensités rcpré.sentées
quand m = 1 et m = 2 pour les franges d’interférence. 41P. Une lumière éclaire un réseau à un angle d’incidence tfr, comme
l’illustre la figure 7.47. Démontrez que des franges brillantes
apparais.seni aux angles dqui correspondent à l’équation

i/(sin -ф + sin в) - wiX. lorsque ж = 0, _ 1, ± 2 . ...


(Compare/, celte équation avec l’équation 7.22.) Dans ce chapitre,
on n 'a traité que du cas particulici où V' = 0.

Figure 7.46 Problème 3?

SEOION 7 7 les réseoux de dîHroriion


42P Un réseau dans lequel d ~ 1.50 /im est éclairé en fonction
33t Un réseau de diffraction large de 20,0 mm comporte 6 (K)0 fentes, de differents angles d’incidence par une lumière ayant une longueui
a) Calcule/ la distance ifcnti-c deux fentes adjaix“nts. h) À quels angles B d’onde de 6(K) nm. Représentez giaphiijuement la déviation angulaire
les maxima d’intensité se produisent-ils sur un écran d’observation, si du maximum de premier ordre par rapport à la direction d’incidence,
les rayons incidents sut le réseau ont une longueur d’onde de 589 nm ’’ en fonction de l’angle d’incidcnce (de 0 à ttO") (Voir te prohlèmc 4/.)
206 Chapitre 7 La diffraction et la polarisation

43P. Dérive? l'équation 7.25, l’expression des demi-largeurs angulaires réfléchissants est de 0.94 nni. Quelles sont les deux longueurs
des raies obtenues pour la figure de diffraction d’un réseau. d'onde?
44P. Un réseau comporte 350 fentes par millimètre et est éclairé à
incidence normale par une lumière blanche. Un spectre se forme
sur un écran situe à 30 cm du réseau. Si on perce un nou de 10 mm de
côté dans l’écran, son cf>lc intérieur se trouvant à 50 mm du maximum
central et parallèle à ce dernier, quelle sera la gamme de longueurs
d’onde de la lumière qui passera par le trou '?
45P*. Dérivez celle expression donnant l’iiiiensilc Iumineu.se de la
figure de diffraction obtenue à part ir d ’un « réseau » comportant trois
fentes :

/ = I t- 4 aw t¡> I 4 cos- 4>),


où (A = (2îrd sin 6)/K. Posez « A. '. basez-vous sur la dérivation de
la formule correspondante ptiur deux fentes fl'éc|ualion 6.21 ). 55E. Un faisceau de rayons X d’une longueur d'onde donnée lonihc
sur un cristal de NaCI, à un angle d’incidence de 30,0’' par rapport
SECÍI0N 7.8 Les résemix. le pouvoir dispersif et le pouvoir de résolution à un ensemble donné de plans atomiques réfléchissants séparés jtar
un espacement de 39,8 pm. Si la réflexion à ces plans se produit
46t La ligne D dans le spectre du sodium csi un doublet dont les
au premier ordre, quelle e.st la longueur d ’onde des rayons X ?
longueurs d’onde sont de 589,0 nm et de 589.6 uni. C aknlez le
46L Un faisceau de rayons X de longueur d’onde A est réfléchi au
iioiiibre uiinimum de fentes requis dans un réseau qui permet de
premier ordre dans un cristal lorsque son angle d ’incidence à une
séparer ce doublet au deuxieme ordre. Voyez l'exemple 7.5.
face du cristal est de 23°, et un faisceau de rayons X de longueur
47E. Une siiurce lumineuse conienam un mélange d’atomes d’iiydrogène
d ’onde tie 97 pm est réfléchi au Iroisicinc ordre lorsque son angle
et de deutérium émet une lumière rouge à deux longueurs d’onde qui
d'incidence est de 60° pai rapport à cette face. .Supposez que les deux
ont une différence de 0 .180 nm et dont la moyenne est de 656.3 nm.
faisceaux sont réfléchis par le même en.semble tie plans attimiques
Trouvez le nombre minimum de fentes qu’un réseau de diffraction
réfléchi.s.sanis. Trouvez a) Tcspacement entre les plans et b) la Itmgneur
doit avoir pour séparer ces raies au premier ordre, wvw
d’onde/1.
48E Un réseau comporte 600 fentes/mm et a une largeur de 5,0 mm.
57P Démontrez q u ’il est impossible de déterminer simultanément
a) Quel est le plus petit intervalle de longueur d ’onde qu’il peut
la longueur d ’onde d’un rayonnement incident et l’espacement
séparer dans le troi.sièmc ordre à A = 500 nm ? b) Combien d’ordres
entre les plans réfléchissants dans un cristal en mesurant Tangle
plus élevés ¡xuvcitt être observés ?
des maxima en fonction de plusieurs ordres.
49E. Démontrez que le jxiuvoir dispersif d ’un réseau est égal
58P. Dans la figure 7.49, une réflexitm au premier ordre se produit aux
à D (lan 0)/Â.
plans réfléchissants illustrés lorsqu'un faisceau de rayons X d ’une
50E Dans un certain réseau, le doublet du sodium {voir l'exemple 7.5) longueur d'onde de 0.260 nm forme un angle de 63,8° avec la face
est observé au üoisième ordre, à KP de la normale, et est tout juste supérieure du cristal Quelle est la taille de la maille élémentaire Op?
séparé. Trouvez a) le pas du réseau et b) la largeur minimale du réseau.
51P, Un réseau de diffraction a un pouvoir de résolution
R Amoy/àA = Nm. a) Démontrez que la gamme de fréquences
correspondante A/; où les fréquences peuvent tout jiustc être .séparées,
est donnée piar A /= c/{NmÀ). b) À l’aide île la figure 7.18, démontrez
que le temps que prend la lumière pour se propager le long d’un
rayon qui pan de la fente du bas diffère d’une quantité Ai {NtUc)
sin a par rapiiort au temps qu’elle prend pour se propager le long d’un
rayon qui part de la fente du haut, c) Démontrez que {Af)(At) ~ 1,
cette relation étant inilépendante des divers paramètres des réseaux.
Posez 7 / » I. Figure 7.49 P io h lè m e 58

S2P a) Rffectuez le produit de la demi-largeur angulaire d’une raie


59P. Examinez une structure tie cristal carrée et bitlimensionnelle, par
obtenue par un réseau et du (Xiuvoir de résolution du réseau,
exemple un côté de la struciurc illustrée dans la figure 7,26 a).
et exprimez-le en fonction de l’angle 0 situant cene raie, b) hvaluez
L’espacement entre les plans réfléchissants corresptmtl A la taille de
ce produit pour le réseau du problème 38, au premier ordre.
la maille élémentaire «n. a) C alculez les cinq plus petits espacements
suivants et faites-en le schéma, b) Démontrez que vos résultats en al
SECTION 7.9 lo diffrottion des royons X corresptindem A la lormulc générale
53t Des rayons X d’une longueur d’onde de 0,12 nm sont réfléchis
au deuxième ordre à un angle de 28°. sur un cristal de fluorure de d = ->
litliium. Quel est l’espacement entre les plans atomiques réfléchissants
iians le eriKtnl 7 iHi ft et X sont des nombres entiers premiers (ils n’ont aucun faeteui
.S4f. I.a figure 7.48 c.st un graphique de la distribution de l’intensité commun autre que l’unité), w w
lumineuse, en fonction de la position angulaire ft, pixir la diffraction 60P Dans la figure 7.50. un faisceau de rayoms X. tluni les longueurs
<Tun faisceau de rayons X par un cristal. Le faisceau comporte d ’onde vont de 95,0 pm à 140 pm, arrive A un angle d ’incidence
lieux longueurs d’onde, et l ’espacement entre les plans atomiques de 45° par rapport A un ensemble de plans réfléchissants séparés
Exercices et proDiemes 207

par un espacement cl = 275 pm. À quelles longueurs d’onde ces Si 10% de l’intcnsilc incidente est transmise par les deux polanseurs,
plans produiront-ils des maxima d’intensité lors de leurs réflexions 7 quelle est la valeur de
6/P. Un faisceau horizontal de lumière polarisée verticalement
et d ’une intensité de 45 W/in^ traverse deux polavi.xcuis l ’axe de
transmission du premier forme un angle de 70° avec lu verticale,
et celui du deuxième est horizontal. Quelle c.st l’mtcn.sitc de la
lumière transmise par le .système des deux polanseurs vrww
68P. Reportez-vous au problème 67 et supposez que la lumière initiale
est non polarisée. Quelle est alors l'intensité de la lumière transmise ?
69P. Un faisceau de lumière ixirticllemciii polarisée jicul être eonsitlcré
comme un mélange de lumière polarisée et non polarisée. Supposez
qu’un tel faisceau traverse un polari.setir, et qu’on lourne ensiilie
61P. Dans la figure 7.50. un taisceau de rayons X d’une longueur d’tmde le polariseur de 360° en le maintenant pcipcndiculuirc au faisceau.
de 0.125 nm tombe sur un cristal de KaCl, à un angle d'incidence de Si l’intensité de la lumière transmise varie par un facteur de 5 ()
45° avec la surface supérieure du cristal et un ensemble de plans pendant celle rotation, quelle fraction de l 'intensité du faisceau initial
réfléchissants séparés par un espacement d = 0.252 nm. A quels est associée à la lumière polarisée du faisceau ? wwv
angles doit-on faire pivoter le cristal sur un axe perpendiculaire au
70P Sur une plage, la lumière est généralement polari.séc partielle
plan de la page si on veut que ces plans réfléchissants produisent des
ment, à cause des réflexions sur le sable et à la surface de l’eau.
maxima d’intensité lors de leurs réflexions?
À un endroit particulier et un peu avant le coucher du soleil, la
composante horizontale du vecteur champ électrique vaut 2,3 fois
SECTION 7.10 Lq pokirisolicn la composante verticale, lin baigneur met ses lunettes solaires
62E. le s équations du champ magnétique d’une onde électromagné­ polari.sées ; les leolilles climineni la composante horizontale du champ,
tique dans le vide sont H, — H sin(Ap -*■ cjt) et B,. = B. = 0. a) Quelle a) Quelle fraction de l’intensité lumineuse reçue avant que le baigncui
est la direction de la propagation ? bl Écrivez les équations du champ mette ses lunettes atteint maintenant ses yeux? b) l e baigneur,
électrique, c) L'onde est-elle polarisée ? Si oui. dans quelle direction ? qui porte toujours scs lunettes, s’étend sur le côté. Quelle fraction de
63E. Un rayon de lumière non polarisée d'une intensité de 10 niW/m- l’intensité lumineuse reçue avant qu’il mette .ses lunettes atteint
est transmis à travers un polariseur. comme l’illustre la figure 7..50. maintenant ses yeux ?
a) TVouvez la valeur maximale du champ électrique du rayon transmis. 71P. On veut changer de 90° la direction de polansation d’un faisceau
b) Quelle est la pression de radiation exercée sur le polariseur ? de lumière polarisée en lui faisant traverser un ou plusieurs polariseurs.
ME Dans la figure 7.51, une lumière initialement non ixilarisée est a) Quel est le nombre minimum de polariseurs requis? h) Quel est le
transmise à travers trois jxiiari.seurs dont les axes de transmission nombre minimum de polariseurs requis [x)ur que l'intensité trattsmise
fonnent des angles de Bj = B; = Bj = 50‘ par rap|wrt à l’axe des y. soit supérieure à 60% de l’intensité initiale ?
Quel pourcentage de l’intensité initiale de lumière est transmis par le
système des trois polariseurs ? (Indice : Prêle/ attention à la façon
dont les angles sont définis tlans la figure 7.51.) SECTION 7.11 Lo pdorisalion par réflexion
65E. Dans la figure 7.51, une lumière initialement non polarisée 72E. a) À quel angle d’incidence la lumière réfléchie sur l’eau sera-t-elle
traverse trois polariseurs dont les axes de transmission forment des entièrement polarisée ? b) Cet angle dépend-il de la longueur d’onde
angles B, - 40°, B2 = 20“ et B, = 40“ par rapport à l’axe des y. de la lumière ?
Quel pourcentage de l’intensité initiale de lumière est transmis par le 73E De la lumière qui sc propage dans l’eau, dciii l'indice de réfraction
système? (Indice : Prêtez attention à la façon dont les angles sont C.SI133. est incidente à une plaque de verre dont l’indice de réfraction
définis dans la figure 7.51.) est 1,55. À quel angle d ’incidence la lumière réfléchie sera-t-ellc
entièrement polarisée
74E. Calculez les limites supérieure et inférieure des angles de Brewster
d ’une lumière blanche incirlente à un verre de silice Supposez que la
longueur d’onde de la lumière sc situe entre 400 nm et tOO nm.

Problèmes suppiémentoires

75. Dans lin système de télévision conventionnel, les signaux sont


transmis des tours de transmission aux postes récepleiir.s. Même si
un récepteur n’est pas dans la ligne directe d ’une t<uir à catisc d’un
obstacle naturel ou d’un édifice, il peut tniil de même intercepter
un signal si cclui-i:i est suffisamment difiracté autour de l’obstacle,
dans la «zone sombre» de l’obstacle. Les signaux de télévision
courants ont une longueur d’onde d'environ 50 cm, mais les
Figure 7.51 E x e rc ic e s (vt et 6 5 nouveaux signaux numériques de télévision qui seront transmis des
tours auront une longueur d’onde d'environ 10 mm. a) Ci- change
toP. Un rayon de lumière polarisée traverse un système de deux ment dans la longueur d ’onde augmentera t-il mi diminuera-t-il
polanseurs. Par rapport à la direction de polarisation de celle lumière la diffraction des signaux dan.s les zones soinbre.s des obstacles?
incidente, l’axe de transmission des polariseurs forme un angle B .Supposez qu’un sijmal pas.se dans une ouverture de 5,0 m de largeur
pour le premier polariseur et un angle de 90° pour le deuxieme. entre deux édifices adjacents Quel est l'étalem ent angulaire du
208 Chapitre 7 La diffraction et la polarisation

maximum central de diffraciion (jusqu’aux premiers minima) par en lace de votre (til droit et tenez la figure du cercle en face de votre
rapp»»n à des longueurs d'onde de b) 50 cm et c) 10 mm œil gauche. Regardez le ciel à travers le trou d’aiguille avec voire œil
76. On po.se ici que le critère de Rayleigh donne la limite du pouvoir droit, en même temps que vous regardez le eerelc avec votre œil
de résolution des yeux d’un a.stronaute qui regarde la surface de la gauche. Avec un peu de pratique, vous pourrez faire coïncider les
Terre depuis une navette spatiale se trouvant à une altitude de 400 km. deux images, et le cercle apparaîtra alors parmi les floueurs.
a) Dans ces conditions, c.stime7 la plus petite séparation linéaire que Ajustez la distance entre le cercle et votre œil gauche jusqu’à ce
rastrnnantc peut distinguer à la surface de la Terre. Le diamètre de la que le cercle ail approximativement la même taille qu'un de.s floticurs
circulaires. Demandez à quelqu'un de mesurer la distance L entre le
pupille de l’astronaute est de 5,0 inm et la longueur d'onde de la
cercle et voire œil gauche (une estimation .suffira), 1л figure 7.53 c)
hnnière visible esi de .5.50 nm. b) L'a.stronaute peut-il distinguer la
représente une vue .schématisée du cercle que vous percevrez : des
Grande Muraille de Chine (Hguie 7.52), qui mesure plus de 3 (HX) km
rayons pas.scnl à (ravers la lenlille (le cristallin) de l’œil et formcnl
de lotig. a entre 5 m et 10 m il’épai.sseur à .sa hase, 4 m à son sommet
une image du cercle sur la rétine, à une distance L' = 2,0 cm derrière
et 8 in de haut? c) L’astronaute peut il distinguer des signes de vie la lenlille. En vous fondant sur celte perception du cercle et sur la
inielligenie à la surfac:e de la Terre'? valeur de L. trouvez le diamètre D' de l’image du c'erclc (et de celle
d’une des figures) sur la rétine
On peut faire l’approximation que le dépôt a une forme sphéritiue
Sa figure de diffraction e.sl alors identique (sauf en .ч т centre) à celle
d'u n e ouverture circulaire du même diam ètre; votre perception
visuelle du dépôt est donc identique (sauf au centre) à ]<i figure illustrée
dans la figure 7.9. De plus, la position du premier minimum
de la figure de diffraction du dépôt est donnée par rétjuaiion 7.12
(sin в = 1.22à/z/). Supposez que la longueur d’onde de la lumière est
de 550 nm. Servez-vous de la figure 7.53 d) pour relier l'angle в
au rayon de l'image du cercle sur la rétine et à la distance x entre
le dépôt et la rétine. Sujjposez que la valeur de x se situe entre 1 mm
et 1.5 mm. Quel est alors le diamètre approximatif des dépôts à la
surface de votre œil 7

Figure 7 52 Problème 76

77. Flotteurs. Comme on Ta mentionné dans la section 7.1. les petites


taches et les structures semblables à des cheveux que vous voyez
parfois flouer dans votre champ de vision .sont, en réalité, des figures a)
de diffraction produites sur votre rétine. Elles sont toujtHirs présentes,
mais elles ne sont disecmables que lorsque vous regardez vers un Rétine Réliiif*
arrière-plan uniforme et clair, comme le ciel ou un mur brillamment
éclairé. Les figures sont produite.s lorsque la lumière frapjje des
dépôts ilans le gel (l’humeur aqueuse) qui remplit une grande partie fy
de vos yeux. I a lumière e.si diffractée autour de ces dépôts et dans
leur « zone sombre», comme la lumière dams Texpérienee de rresnel
décrite diuis la section 7.1. Vous ne percevez pas les dépfiix ед1х-тётех,
4)
mais bien leurs figures de diffraction sur votre rétine. On apixîlle ces
figures des «flotteurs» car, lorsque vous bougez vos yeux, le gel Figure 7.53 Problème 77
se trouble (un peu comme un dessert à la gélatine qu’on secouerait).
Ce qui fait déplacci les figures de diffraciion sur votre rétine Avec Tâge, 78. Dans la figure 7.54, un rayon de lumière dans l’air est incident
à une couche plane (substance 2 ) dont l’indice de réfraction est
le gel peut se troubler davantage, ciu il adhère moins bien aux parois
= 1,5. Sous la substance 2 se trouve la substance 3. dont Tindice de
iniéticuies de Treil ; vous remarquerez donc davantage ces flotteurs
réfraiiitm est w-,. l-e rayt'n incident à Tiiiterface entre l’air et la subs­
en vieillissant, ce qui vous rappellera le phénomène de diffraction.
tance 2 forme un angle de Brewster à cette interface. l,e rayon de
5i vous voulez étudier ees figures, vous pouvez, les rendre plus
lumière réfracté dans la sub.stance 3 est incident à T interface etiire la
facilement discernables en regardant h travers un trou d'aiguille, siib.staiicc 7 et la substance 5 et forme un angle de Brewster à cette
qui agit comme une source ftonetucllc de lumière (comme on le voit interface. Quelle est la valeur de ii, ?
dans lu figure (>.5 c) Vous [Kiurrrz, alors voit des flotteurs de forme
eireiil lire tlotés d ’un centre brillant entouré d’un ou de plusieurs
anneaux sombres (figure 7.53 a) ; ils peuvent aussi avoir la forme
d’un сЬелси comportant nn centre brillant et quelques bandes scanbres
sur les côtés (figtiie 7.55 h).
Рейн estimer la taille des dépôts й la surface de vos yeux, procédez
de In façon .suivante : percez un tiou dans une Icuilic de carton opaque,
à Ш1Г distance du bord de la feuille semblable à la distance entre
votre nez et le centre de vos yeux. Tracez un cercle de diamètre
D 2 mm sur une autre l'cuille de carton. Placez le irhi directemeni Figure 7 54 Problème 78
8 La relativité
restreinte

Aujourd'hui, dans le domaine de lo novigotion sur de longues distances, la position et la vitesse précises d'un avion
sont continuellement suivies et mises à jour, lin système de sotellites de navigation appelé NAVSTAR permet de déterminer
les positions et les vitesses partout sur Terre, avec des incertitudes respectives de 16 m et de 2 cm/s. Cependant, si on ne tenait
pos compte de la relativité,
il serait impossible d'obtenir
une incertitude sur la vitesse
inférieure o 20 cm/s, ce qui est
inocceptoble en navigation
moderne.

lu lépons« K trouYt dam (t chopilre.


210 Chapitre 8 La relativité restreinte

8.1 O u ’est-ce que la relativité ?


I7n d es p rin cip au x o b je ts d e la r e la tiv it é r e s tr e in te est co n stitu é d es m e su res des
événetnents (des p hénom ènes qui arrivent), c ’est-à-dire de l ’instant e t de l ’en d ro it où
ils SC produisent, ainsi que des intervalles qui séparent deux événem ents dans l’esiracc
et d ans le tem ps. L a re la tiv ité a au ssi trait à la tran sfo rm atio n d e m esu res en tre des
référentiels .se déplaçant les uns par rapjxirl aux autres <d'où le nom reUiiiviU'). On en a
déjà parlé dans les sections 4 .9 et 4 .10 du \ olum c 1.
Ln 190.5. les physiciens connaissaient bien les traasform alions et les référentiels en
m ouvem ent. C ’est à cette éprxjue q u 'A lb cri E instein publia sa théorie de la relativité
restreinte. L’ad jectif restreinte signifie que celte théorie ne concerne que des référentiels
d'inertie, ou référentiels inertiels. qui s^)nt ceux où la première loi de N ew ton s’applique.
A utrem ent dit. clic n a irait q u ’aux cas où les référen tiels n ’accélèren t pas, où ils se
déplacent à des vitesses constantes les uns par rapport aux autres. (I a théorie de la relativité
générale. d ’E in stein traite de l ’éq u iv ale n ce locale entre un référen tiel ac cé lé ré et la
gravitation [voir la section 14.9 du volum e Ij. D ans ce chapitre, le term e relativité ne
lélère q u ’à la relativité restreinte.)
À partir de deux postulat.s incroyablem ent simplc.s, Einstein a étonné la com m unauté
scientifique en dém ontrant que certaines vieilles idées .sur la relativité étaient fausses,
m êm e si elles étaient fam ilières à tout le m onde e t que personne ne rem ettait le gros bon
sens en question. Ce gras bon sens était toutefois basé sur des expériences ne mettant en jeu
que des objets sc déplaçant plutôt lentem ent. La relativité d ’Linstein, qui se révéla juste
pour toutes les vitesses possibles, prédisait de nom breux effets qui sem blaient bi7arrcs
à prem ière saie, parce que personne n ’en avait fait l’expérience.
E in stein d ém o n tra en p articu lier qu e l’csp acc et le tem p s fo rm en t un tout q u ’on
appelle l’esp ace-lem p s. le tem ps qui sépare deux événem ents dépend de la distance qui
sépare ces événem ents, et vice versa. De plus, les m esures d ’intervalles de tem ps et de
distances sont differentes pour des obsers ateurs qui se déplacent les uns par rapport aux
autres. L ’enseignem ent qui en résulte est que le tem ps ne s ’écoule pas de façon fixe,
co m m e s ’il était rég lé pai la rég u la rité m é ca n iq u e d ’u n e h o rlo g e qui co n trô lerait
rU n iv e rs . A u co n traire, l ’éc o u le m en t du tem ps peut plutôt s ’a ju ste r; le m ouvcnicni
re la tif p eut m odifier l’éco u lem en t du tem ps. A vant 1905, p ersonne*, sa u f un rêveur,
n 'a u ra it pu l’im aginer. A u jo u rd ’hui, les ingénieurs e t les scientifiques tiennent ce fait
pour acquis, car leur expérience avec la relativité restreinte a rem odelé leur gros bon sens.

Figure B1 OiiiMein. au débiii


fli-s aimées 1900. à son pupitre
du Bureau des l)revels de Berne
f Siiivsex où il travaillaii quand
il n publié sa diéorie de la relatis ité
restreinte

* ! os lésullats d binstein om été déctmveris iJc façon indépendante à la même ép»>quc par le malhematicirn
fl iuiçais Hcnn Poincaré.
8? Les postulats 211

L a relativité restreinte a la rép u tatio n d ’élre co m p lex e E lle n ’cxi рал difficH c du
point de vue m atliém atique, du m oins, pas ici. C ependant, la difficulté provient du fait
que, pour les m esures liées à un cvcnenienl, il faut ten ir com pte de сца m esure quoi et
de comment ces m esures sont p rises (ce q ui peut être difficile, parce que cela |ieut aller
à l ’encontre de notre expérience courante).

8.2 Les postulats


O n exam inera m aintenant les deux postulats sur lesquels sc fonde la théorie U L in s loin.

^ 1. Le postulat de la rd ativ lté ; les lois de la physique sont les memes p<lur ions
les oLservatcurs se trouvant dans des référentiels inertiels Tous les rélifHeniWs ineriielx
sont équivalents.

G alilée croyait que les lois de la mécunUfue étaient les m êm es dan.s tous les rélcrcnticlf
in ertiels. (L a p rem ière loi de N ew ton en est une co n séq u en ce im p o rtan te.) Lin.stcin
a élargi cette idée p o u r y in clu re tom es le.s lois de la ph y siq u e, n o lam m en t celles de
l ’clectro m ag n étism e et de l’o ptique. C e postulat ne dit pas qu e les vnleurs d r to u tes
les grandeurs physiques m esurées sont les m êm es pour tous les obsers atriirs im-rik I s .
certaines le sont (on dit q u ’elles sont invariantes), m ais la plupart ne le sont pas (on dit
q u ’elles sont relatives). C e sont plutôt les lois de la physique, qui fo n t le lien entre ces
m esures, qui sont les mêm es.

P- 2. Le postulat de la vitesse de la luniière : la vitrs.se de la lumtrre dans le vide a la memu


valeur c. dan.s toutes les directions et dans tous les référentiels iiieitiels.

On peut égitlem ent fo rm u ler ce po.stulat de façon à d ire q u ’il y a. d an s la n atu re, une
vitesse c limite, identique dans toutes les directions et dans tous les référcnticl.s inertiels.
C ’est un invarian t relativiste. La lum ière voyage dont- à cette vitesse lim ite, com m e
toute particule sans masse (Ie,s neutrinas en sont peut être un exem ple). Cependant, aucune
en tité qui tran sp o rte de l'é n e rg ie ou de l ’in fo rm atio n ne p eu t dépasseï celle vitc.ssc.
D ’aille u rs, au cu n e p articu le ay an t u ne n iasse ne peut a tte in d re cc tlc v itesse c, peu
im p o rte le m odule et la du rée de son accélératio n . C e tte v itesse lim ite n 'e s t pas une
p ro p rié té de l ’o n d e éle e iro m a g n é tiq u e . m ats bien un e p ro p rié té d e la stru c tu re de
r espace-tem ps.
O n a lesté ces deux [xisiulats de manière exhaustive ; on n’a dcccic aucune exception.

La vitesse limite
Î-’existence d ’une vitesse lim ite pi'iir les électrons accélérés a été dém ontrée lors d ’une
ex p érien ce m enée p a r W. B c rlo /z i en 1964. Il a act'Cléré des électro n s à différen tes
vitesses m esurées (v o ir la figure 8.2 ) r t a égalem ent m esuré leurs énergies cinêtiquc.s
(à l 'aid e d ’un e m éthode in d épendante). 11 a U"ouvé que, à in e siu e qu e la force siii un
é lec tro n trè s rap id e au g m en te , l’én e rg ie c in étiq u e me.snrée d e r é le c tr o n au g m en te
ju s q u 'à de très g ran d es v aleu rs alo rs que sa vitc.ssc n ’au g m en te p as de m a n ière
appréciable. Les électrons o n t atteint une vitesse d ’au moins 0.999 9 ‘)9 999 95 fois celle
d e la lu m ière (d an s c e rta in s a c cé lé ra teu rs d e p articu le s), m ais, b ien q u 'e lle s ’en
Figure 8.2 Les points représentent
approche, cette vitesse est encore inférieure à la viies.sc lim ite c.
les valeurs mesurées de l’énergie
Le m ètre est m aintenant défini d e m anière à ce que la valeur exacte de cette vitesse
cinétique d’un éleciron en fonction
lim ite soit :
de sa vite.vse nie.surce. Peu importe
la quantité d’éncrpic fournie
г = 299 792 458 m/s. ( 8 .1)
à un électron (ou à toute autre parlinile
ayant une masse), sa vitesse ne peut
jamais égaler ni dépasser la vitesse ■lusqu'à maintenant, dans ce livre, on l'a arrondie (de inanici'e appropriée) à 3.0 x 10**m/s ;
limite c. (La ccmrhe qui relie les points clans ce chapitre, on l'arro n d ira plutôt à 2,998 x JO** m/s. Vous ptuive? entrer la valeur
illustre- les prédictions de la théorie exacte dans la m ém oire d e votre caleulatricc (si elle n ’y est pas d éjà) p o u r la rappeler
de la relativité restreinte d’Einstein.) au besoin.
212 Chapitre 8 La relativité restreinte

Tester le postulat de la vitesse de la lumière


Si la vitesse de la lum ière cM la m êm e dans tous les référentiels incrticis. la vitesse d e la
lum ière ém ise p ar une source en m ouvem ent doit être la m ôm e qu e celle de la lum ière
piovcnant d ’une source au repas en laboratoire. Cette affinnatiun a été testée directem ent
lors d ’une expérience d e haute précision. L a « so u rce lu m in eu se» était un pion neutre
(sym bole une particule subatom ique instable à courte vie q u ’on peut produire par
des collisions dans un accélérateur de particules. C ette particule se désintègre en deux
rayons gam m a de la m anière .suivante ;

JT" y + y. ( 8 .2)

P u isque les ray o n s g am m a fo n t p artie du sp e c tre é le c tro m a g n é tiq u e (à très hautes


fréquences), ils obéissent aux postulats de la vitesse de la lum ière, conm ie c ’csl le cas
de la lum ière visible.
lin 1964, au co u rs d ’un e ex p é rien c e, le s p h y sic ie n s d u C L R N , le lab o rato ire
européen de p h y siq u e des p articu les situé p rès de G enève, u n i généré un faisceau de
pions voyageant à 0.999 75c p ar rap p o n au laboratoire. Les expérim entateurs ont alors
m esuré la vitesse des rayons gam m a ém is par ces sources se déplaçant à haute vitesse.
Ils ont constaté que la lum ière ém ise par ces pions était identique à celle q u ’ils auraient
ém ise s ’ils avaient été im m obiles dans le laboratoire.

8.3 Mesurer un événement


L'n év én em en t est quelque chose c]ui se produit et qui m arque une position de l’espace-
tem ps. L'n o b se rv a te u r peut d éc rire ce t év én em en t à l ’aid e d e tro is co o rd o n n é e s de
po sitio n e t une co o rd o n n é e de tem ps. Parm i d e n o m b reu x é v é n em en ts p o ssib les,
on trouve la m ise sous tension ou hors tension d 'u n e m inuscule am poule clectiique. la
collision de deux particules, le passage d 'u n e im pulsion lum ineuse à un point déterm iné,
une explosion et la coïncidence de l'aig u ille d ’une horloge avec une m arque située à la
c irc o n fé re n c e du ca d ran . P a r ex e m p le , un ce rtain o b se rv a teu r, im m o b ile d an s un
référentiel inertiel donné, pourrait attribuer à un ét'énem ent A les coordonnées fournies
d ans le tab leau 8 . 1. É ta n t d o n n é q u e la th éo rie d e la rela tiv ité lie étro item e n t tem ps
et espace, on peut qualifier collectivem ent ces ctKirdonnées de coordonnées à'espace-
tempi de l ’événem ent. Il faut noter que le systèm e de coordonnées lui m êm e fait partie
du référentiel de l'observateur.
U n é v é n e m e n t d o n n é p eu t être o b se rv é par un g ran d n o m b re d ’o b se rv a teu rs,
chacun SC irouvant dans son propre référentiel inertiel. t n général, différents o b serv a­
teurs attribueront différentes coordonnées d ’espace-tem ps au m em e événem ent. N otez
q u ’un événem ent n ’« appartient » pas à un référentiel inertiel particulier. Un événem ent
n ’est q u ’un phénom ène qui se produit, et toute personne se tro u \ ant dans un référentiel
peut l’observer et y attribuer des coordonnées d ’espace-tem ps.
U ne telle attribution peut être com pliquée par un problèm e pratique. P ar exem ple,
su p p o se z q u ’un b allo n éc la te à 1 km à vo tre d ro ite à l’in stan t m êm e où une fusée
pyrotechnique explose à 2 km à votre gauche, les deux événem ents se produisant à 9 h 00.
Vous ne voyez aucun de ces événem ents à 9 h (K) précises, parce que la lum ière émi.sc
par ces événem ents ne vous a pas encore atteint. Liant d onné que la lum ière provenant
de la fusée pyrotechnique doit parcourir une distance plus grande, elle atteint vos yeux
après celle qui provient de réc late m c n l du ballon ; l’explosion de la fu.sée vou.s sem blera
donc .SC produire plus tard que l’éclatem ent du ballon. P our déterm iner l’in.stant précis
où CCS év én em en ts o n t eu lieu, so u 9 h 0 0 , vo u s d ev ez calc uler la du rée du trajet de
chaque rayon lum ineux, puis soustraire ce tem ps de l’heure d ’arrivée de la lum ière.
MEAU8.I Inregistrement de l'événement A C ette m éthode peut s 'a v é re r très co m p liq u ée dans des situ atio n s d ifficiles ; il en
Cwrdoruiée Valeur faudrait une plus facile, qui élim inerait autom atiquem ent toute relation au tem ps de par­
cours entre un événem ent et l’o bservateur. P our élab o rer une telle m éthode, on peut
JC .3..S8 m construire un réseau im aginaire de tiges graduées et d ’horloges dans le référentiel iner­
y 1.29 m tiel de l ’observateur (ce réseau se déplace de m anière rigide avec ro b serv ateu r). Cette
Z 0m construction peut sem bler artilicielle, mais elle vou.s évitera beaucoup d e confusion et
r de calculs, cl vous perm ettra de déterm iner les coordonnées d e po.sition, la coordonnée
34,5 s
de tem ps et les coordonnées d ’espace tem ps de la m anière suivante.
8 (t La retaflvite do (a simuttaneite 213

1. Les coordonnées de position. Im aginez que le sy.stème d e cnordonm'L x tk rniisci-


vaieur est quadrillé par un ré.sciiu h trois dim ensiunb de tige» graduées rapprochées,
chacun des trois axes de coordonnées ayant u n ensem ble de tiges qui lut est ptiniiiela.
Supposez m aintenant qu e l’événem ent est la m ise suu.s ten.siciii d une jTeiiie am poule
électrique. Pour lot'aliscr ré v é n e m e n t, ru b se rv a te u r n 'a besoin que de lire le t triiis
coordonnées de position de l'am jxiule.
2. L a c o o rd o n n é e d e le tu p s. Pout d éteim in et la oK udoiiiicc de icinps. iniiigincz que
chaque intersection des tiges graduées renferm e une m in u u ule hoilnge, titic l'o b se r­
vateur peut lire grâce à la lum ière générée par l'é vé ncm cn i. L a ligure 8.3 suggère un
figuTfl 8.3 Section d’un réseau à trois plan du réseau form e par ces horloges c l ces tiges.
dimensions d’horloges et de tiges Les horloges doivent clic adéquatement .syix;hionisécs. 11 ne s u ffit pas de rassembler
gnuliKV.s cjiii ticrmet à un observateur des horloges identiques, de les régler à la meme heure et de 1r s pincer à leurs positions
d’uUribucr des coordonnées d’cspace-
finales. On ne sail pas. p ar exem ple, si le fait de déplacer les horloges m odilicra leurs
temps à lin événement, comme
tem ps indiqués (il le fera). Il faut donc placer les h o ilo g rs avant de les synchronivu
il un eclair lumineux situé au point A.
Si on pouvait transm ettre les .signaux à une vitesse infinie, cette synchroni.sation
lycs coordonnées de position de
serait sim ple. C ependant, aucun signal connu n ’a celte propriété C 'e s t pcKirquoi on
révenement sont approximativement
choisit la lum ière (dans laquelle on inclut grossièrem ent tout le spectre électro m a
X = 3,7 longueurs de tige, y = 1,2
longueur de tige et z = 0. La coordonnée gnétique) pour transm ettre des signaux synchronisés parce que, dans le vide, la lumière
de temps correspond au temps indiqué voyage à la plus grande vitesse possible, à savoir la vitesse lim ite c.
par l’horloge qui est la plus prés de A V oici l’un e d es nom breuse.s m éth o d es q u ’un o b se rv a te u r p o u rra it em p lo y er
à rinstatit où l’éclair se produit. pour synchroniser un réseau criiorlogc.s à l ’aide de signaux lum ineux ; l’observateur
recourt à un g rand nom bre d 'a ssista n ts tem poraires, soit un pour ch aq u e horloge.
L’observateur se tient au point choisi com m e origine et émet une im pulsion de lumière
quand l’h o rlo g e d ’o rig in e affich e r ^ 0. Q u an d l'im p u lsio n lumincu.se a tte in t
un assistan t, celu i-ci règ le son horlo g e à / — t h , o ù r est la d istan ce qui le sépare
de l ’origine. I.es horloges sont alors synchronisées.
3. L es co o rd o n n é e s d ’esp ace-tem p s. L’observateur peut m ainienam atuibuei les coor­
données d ’cspace-teinps à un événem ent en notant tfiut sim plem ent le Icraps indiqué
sur l’horloge qui est la plus piès de l’événem ent et la position indiquée pur les tiges
graduées les plus près. S ’il y a deux événem ents, l'observateiii déterm ine l’intervalle
de tem ps entre eux en calculant la différence entre les tem ps indiqués sur les horloges
les plu.s près, et leu r di.stance, en calcu lan t la d ifféren ce en tre les co o rd o n n ées des
tiges près de chacun d ’eux. On évite alors le problème pratique qui consiste à calculer
les tem ps de pam ours des signaux entre les événem ents ci fo b serv aieu r.

8.4 La relativité de la simultanéité


Supposez q u ’un observateur, Sam uel, note que deux événem ents indépendants (événe­
m ent Rouge et événem ent Bleu) sc produisent au môme instant. S upposez aii.ssi q u 'u n e
observ atrice, S ophie, qui se dép lace à vitesse co n stan te ï p a r rapport à S am uel, note
égalem ent ces deux m êm es evénem em s. S ophie co n sid érera t elle égalem eni que ces
deux événem ents se produisent au inem e instant'.'
La réponse est géncralcm cni non.

Si deux ob-servaicurs sc déplacent l’un par rapport à l’autre, ils ne seront généralement
pas d’accord sur la simultanéité de deux événements Si un observateur les considère
comme étant simultanés, cc ne sera généralement pa.s le cas de l'autre

O n ne peut pas dire que la m esure de l'u n des observateurs est bonne et que la m esure
de l'au tre observateur est m auvaise. Leurs observations sont toutes les deux valables ; il
n’y a aucune raison de favoriser l'u n e plutôt que l’autre.
Le fait que deux énoncés contradictoires su r un m êm e événem ent physique soient
juste.s est une conséquence de la théorie d 'E in stein qui peut sem bler b i/arre. C ependant,
dans la .section .3.8. on a étudié une autre situation où le m ouvem ent peut influencer les
m esures, sans s'é to n n e r des rcsuliaLs co n tradictoires ; la fréquence d ’une o nde sonore
m esurée p a r un o b se rv a teu r d épend du m o u v em en t de l ’o b so rv alc u r et d e la soiircz'.
D onc, deux o b se rv a teu rs se d ép laç an t l ’un par rap p o rt à l ’au tre peu vent d élerm in ei
liifféren ies fréq u en ces p o u r la m êm e o n d e - les deux m esu res é ta n t co rrec tes, c ’est
l'e ffe t Doppler.
214 Chapitre 8 La relativité restreinte

Voici donc la conclusion q u ’on peut tirer de ce qui précède.

► La siumllanéité n'est pas un concept absolu, mais relatif ; elle dépend du mouvement
de l’obscrvateui.

Si la vitesse relative des o b serv ateu rs est très inférieure à la vitesse de la lum ière, les
d isp arités de m esu res sont si p etites q u ’elles n e sont p as ap p réciab les. Tel e st le cas
de toutes vos expériences quotidiennes ; c ’est pourquoi la relativité de la .simultanéité
vous sem ble étrange.

La relativité de la simultanéité en détail


Un peut clarifier l'id ée de la relativité de la sim ultanéité à l ’aide d ’un exem ple fondé sur
les p o stu lats de la relativ ité et d an s lequel au cu n e h o rlo g e ni lige g rad u ée n’en tren t
directem ent e n je u . figure 8.4 m ontre deux longs vaisseaux spatiaux (le VS Sophie
et le VS Sawuel) qui peuvent .servir de référentiels incrticls pour les observateurs Sophie
e t S am u el. C es d eu x o b se rv a te u rs se tro u v en t au c e n tre de le u r v aisseau re.spectif.
L es vais.seaux se d ép lacen t le long d 'u n axe des x com m un, la vitesse d e Sophie par
rapport à Samuel étant v. L a figure 8.4 a) m ontre ces vaisseaux à l’instant où les deux
observateurs sont m om entaném ent alignés.
D eu x g ro s m é téo rite s h eu rte n t le s v aisseau x , l'iin ém ettan t un é c la ir lum ineux
rouge (événem ent Rouge), l'au tre, un éclair bleu (événem ent Bleu), pas nécessairem ent
sim ultaném ent. C haque cvcncm cnt laisse une m arque perm anente su r chaque vaisseau,
aux positions R. K' et B. B '.
Suppose? que les fronts d ’ondes ém anant des deux événem ents atteignent Samuel
au m êm e instant, com m e le m ontre la figure 8.4 c). S u p p o se / aussi que, mc.surant les
m arques B et K laissées su r son vaisseau. Sam uel constate q u ’il se trouvait exactem ent
à m i-c h em in entre les m arq u es R et B q u an d les d eux év én em en ts se sont p ro d u its.
Il dira ;
« L 'éclair de l'événem ent Rouge et l’éclair de l’événem ent Bleu ont atteint m a position
au m êm e instant. Selon les m arques laissées su r m on vaisseau spatial, je constate que
j e m e trouvai.s à m i-ch em in e n tre les d eu x so u rces q u an d le s éc la irs o nt attein t m a
position. Par conséquent, l’événem ent R ouge et rév én em cn i Bleu étaient sim ultanés. »
C om m e on peut le co n sta te r en étu d ian t la figure 8.4, S o p h ie et le front d ’onde
ém anant de l’évén em en t R ouge se d irig en t l’un vers l'au tre , alors q u ’clle-m êm e et le
fro n t d ’o nde ém anant de l'cv é n em e n t B leu se déplacent dans la même direction. P ar
co n séq u e n t, le fro n t d ’o n d e d e l'é v é n e m e n t R ouge a tte in d ra S o p h ie avant ce lu i de
l’événem ent Bleu. Sophie dira alors :

d ^ S C

a IS iS a in uxl fi ^ P fl ---- X
Fw nfirw 'nr Bleu livéneiuent Rouge
Sam uel otiserve le« rleiix evéïum onts.
aï <■)
,S<»|)|ije ohseiTC Sopllie oli.wi Vf
IVvenom enl Ronge. l’évéïieim-nt Bleu.
— P
...... ^ ) — L_

t c x ............ ____ â . 3

h) d)

figura 8.4 Les vaissMiix sputimix de Sophie et de .Samuel et les deux évciicmenls du point de vue
de Samuel. Le vaisseau de SopIiie .sc déplace vers la droite à une vitesse v. a) L’événement
Rouge se produit iUix positions R. R', l’événement Bien, ati'x positions R, R' ; chaque cvénemetil
émet une onde lumineuse, b) .Sophie observe la lumière de l’événement Rouge, c) Samuel
observe simultanément le.s ondes des événements Rouge et Bleu d) Sophie observe l’onde
de l’événement Bleu.
8.5 LiO relçrtivite dü temps 215

« L ’éc la ir d e l ’év é n em en t R ouge est a rriv é avant c e lu i de l'év é n em e rii B leu


D ’après les m arques laissées su r m on vais.seaii .spatial, je euii.sialc q u t je me trnuvni.s
égalem ent à m i-chem in en u c les deux .sourcc.s. Pur conficyucnU ccr> cvcncm cnts « ’ctaiciit
pas sim ultanés ; il j' a d ’abord eu l’événem ent Rouge, puis l ’és'cncm cnt Rlou. ■»>
C es d eu x ra p p o rts n e c o n c o rd en t pas. L es m e su res d e s (U<ux n h se rv u ie u rs sm u
néanmoin.s Justes.
Il faut bien noter q u ’il n ’y a q u ’un front d ’onde ém anant du site île eliaque événe­
ment, et que ce fro n t d'onde .ve déplace à la même vitesse c dans les deux référentiels,
exactem ent com m e l'in d iq u e le postulat de la vitesse de h» lumière.
Il aurait été possible que les m étéorites hcui tent les vai.sscaiix de m anière à ce que
les deux c o llisio n s parais.seni sim u ltan ées à S o p h ie D an s un tel i a.s. S a n n id nurnit
déclaré que ces collisions n’étaient pa.s simultanée.s

8.5 La relativité du temps


Si des observateurs qui se déplacent l ’un pm n ip p o n à l’autre m esurent l’intcrvallC dC
tem ps e n tre deux év é n em en ts, ils o b ticn d io n l g én é rale m en t des résu ltats d ifferen ts.
Pourquoi ? P arce q u e la d istan ce entre d es év én em en ts peut in flu en cer les mlcrvallcfl
de tem ps m esurés par les observateurs.

L'intervalle de temps entre deux événements dépend du mixjvemeni de rirftscrvaicur.

D ans cette section, on aborde la relativité du tem ps à l ’aide il'n n e x e m p le , ccpcuclanl.


cct exem ple est un cas particulier : pour l ’un des deux observateurs, les deux événements
se produisent au m êm e endroit. L es ex e m p le s g én é rau x se ro n t étu d iés à p a rtir de la
.section 8.7.
La figure 8.5 a) illustre une expérience réalisée par Sopliie pendant q u ’elle voyageait
avec son matériel de laboratoire dans un train se déplaçant à vitesse constante v par rapport
à une gare, l'n e im pulsion lum ineuse q uitte une source B (événem ent 1 ), m onte verdca-
Icm cnt, est réfléc h ie v erticalem e n t vers le bas p a r un m iroir, p u is c.sf d ctcc tce à son
retour à la source (événem ent 2). Sophie m esure un certain intervalle de tem ps Ai,, entre

Mlroii' M iroir

a) à l’aide d'une seule, horloge H située dans le tram, f cite horloge est illastréc deux b is : d'abord
poui l’événement 1. puis pour l’événement 2. b'i .Samuel, observant l'événeiiicnt depuis la gare,
a besoin de deux hiM-lopes synchronisées. Mtii W| à l’événement 1 et W2 à l’événcmcnl 2,
pout mesurer l’intervalle de temps entre les deux év'éncmcnts ; sa mesure de l'intervalle
de temps est .\f.
216 Chapitre 8 La relativité restreinte

ces deux cvcnemenLs. C el intervalle csl relié à la distance D qui sépare la source cl le
m iroir par l’équation suivante ;

^ 2Z)
A/„ = — (Sophie). (8.3)
c

Le.s deux événem ents se produisant au m êm e endroit dans le référentiel de Sophie, celle-ci
n 'a besoin que d 'u n e horloge H à ccl endroit p o u r m esurer cet intervalle. L’horloge H
est illustrée deux fois à la figure R..5, soit au déb u t et à la fin de l'in terv alle.
On peut m ain ten an t ex am in er co m m en t S am uel m esu re ces deux m êm es événe-
monis à partir du quai de la gare où il se trouve quand le train passe. Étant donné que le
m atériel de laboratoire se déplace avec le train d urant le tem ps où la lum ière voyage,
Sam uel perçoit la trajectoire de la lum ière de la m anière illustrée dans la figure R.5 h)
Pour lui, les deux événem ents .se produisent à des endroits différents dans son référentiel ;
pai' conséquent, {xiur m esurer l’intervalle de tem ps entre les deux événem ents. vSainuel
doit utiliser deux horloges synchronisées, H^ e t A/,, la prem ière étan t à la p osition de
l’événem ent I, la seconde, à la position de l’événem ent 2. Selon le postulat de la vitesse
de la lum ière, ccllc-ci voyage à la m êm e vitesse c tant pour Sam uel que pour Sophie.
C e p en d an t, la lu m ière p arco u rt la d istan c e I L e n tre les é v é n em en ts 1 et 2. S am uel
m esure alors fin tc rv a llc de tem ps suivant :

2L
(Samuel), (8.4)

ou L = + n \ (8.Î5)

Scion l'équation 8..3, on peut l’écriie ain si:

L = ^ ( | r A f ) ’ + ( i í 'A f ü ) ^ ( 8 .6)

Si on élim ine L entre les équations 8.4 et 8.6 et si on isole Ai. on obtient :

Afp
At = (8.7)
y/1 - {v/cŸ-

L 'é q u a tio n 8.7 in d iq u e co m m en t l ’in te rv a lle At m e su ré p a r S am u el e n tre les


évén em en ts se co m p are à celui m esuré p ar S o p h ie, A/,,. É ta n t d o n n é qu e v d o it être
inférieure à c. le d én o m in ateu r d e l ’équation 8,7 doit être in ferieu r à 1. D onc. Ai doit
a v o u une valeur supérieure à At„: l’inlcrvallc m esuré p ar Sam uel est supérieur à celui
de Sophie. Samuel et S ophie ont m esuré l’intervalle, de tem ps séparant les deux mêmes
événem ents, mais leur m ouvem ent relatif a rendu leurs mesures différentes. O n en conclut
que le moiivemeni relatif peut faire varier l’ceoulem ent du tem ps entre deux év én em en ts,
la clé est que la vitesse de la lum ière est la m êm e p o u r les deux observateurs.
On distin g u e les m esu res d e S am uel et de S ophie en em p lo y an t la term in o lo g ie
suivante.

► Quand deux événemrnts se produisent au même endroit daas un réféieniiel ineitiel, l’intervalle
de temps, Ain- qui les sépare. iiKsuré dans ce référentiel, est appelé intervalle de temps propre.
Une mesure du même intervalle de temps effectuée dans un autre référentiel inertiel sera
loiijiuirs supérieure

D onc, S o p h ie m esu re un in te rv a lle d e tem p s p ro p re. (L e term e propre sig n ifie qu e


l'in te rv a lle de tem ps a été m esuré p ar une horloge im m obile.) La d ifféren ce en tre un
intervalle de tem ps et un intervalle de tem ps propre coiTcspondani s ’appelle d ila ta tio n
d u temp,s.
8.5 La ralotivitâ du tempe 217

10 O n rep rése n te so u v en t le rap p o rt san s d im e n sio n v/c do l'é q u a tio n X.7 p a r le


sym bole p. appelé p a r a m è tr e d e vitesse, et r in v e r s r de la racine caiiéo sans illmcnsinn
de l'équation 8.7 par y. appelé fa c te u r rela tiv iste ou f a c te u r d e L o rc u tz .

- i- - J y =
v /l-^ 2 v /l - ( . / г Я

On peut alors écrire l’équation 8.7 ain.si :


0,2 0.1 0,6 0,8 1.0
P
Д/ — y A/ç (dilalatiuii du tcnijis). (8.9)
figure 8.6 G raphique représentant
le fa d e u r de L orentz y en fonetion l,e piu am èirc de vitesse fi est toujours in fén e u r à I et y est toujours supérieur OU Cpai
du param ètre de vitesse f) ( - v/c) à I Cependant, la différence entre y et 1 est négligeable, à moin.s que v > ü .lc . Donc,
en général, la « relativité galiléenne » s ’applique quand v < (J. le. mai.s c ’est la relativité
restreinte qui entre en Jeu quand v a une grande valeur. Com m e le nioiitic In ligure 8.(i.
la valeu r de y augm ente rapidem ent q u an d p а р р п к Ьс de 1 (quand i- a p p n x hc île t ).
P ar conséquent, plus la vitesse relative entre Sophie et Samuel rsi grande, plus riiiirrv a llr
de tem ps m esure p ar Sam uel sera grand, ju sq u ’il ce que la vitesse soit assez giandc pour
que rin ic rv a llc soit « in fin i ».
Nous devez vous dem ander ce qu e Sophie dirait il propos de riiiterv alle plies grand
m esuré par Sam uel, l.ti m esure de ce d ern ier ne la .surprend pas parce que, p o u r elle,
il n ’a pas bien synchnm isé scs horloges /7, et /7; m eme s ’il in.sistc pour lui dire q u 'il l a fa it
Rappelez-vous que des observateurs en m ouvement relatif ne s ’iKcondenl pas siii In simnl
tanéité. Ici, Sam uel insiste sur le fait que ses deux horloges indiquaicni sim ultaiiém cnt
le m êm e tem ps quand fé v é n c m c n l 1 s 'e s t produit. C ependant, pour Sophie. Sam uel a
m alencontreusem ent réglé en avance son horloge 2. IXmc, aux yeux de Sophie. .Samuel
y a lu un temi>s avancé à l'événem ent 2. ce qui explique pourquoi il a m esuré un inicrv aile
de tem ps supérieui' à celui q u 'e lle m êm e a m esuré.

Le temps propre est un invariant relativiste


.Sophie et Sam uel ne. s ’entendent pas su r rin tc rv a llc de tem ps entre l'éTTli.ssion de f im
pulsion lum ineuse (événem ent 1) et sa détection (événem ent 2) Par co u iic, il y a une
façon de calcu ler le tem ps propre entre les deux événem ents pour que Sam uel Cl Sophie
obtiennent le m êm e résultat. Si un observateur m esure un inicrvallr de lenipx Ar ci une
distance A.» entre deux événem ents, alors le carré du tem ps propre entre les dcu.x événe
m ents est :

(A/,,)- = ^^tp - (Aiy/i (8.10)

O n v érifie facilem en t q u e S o p h ie et Sam uel vont c a k ulci la tnem e ch o se Pour


S am uel, l’interv alle de tem ps entre le,s deux cvénem ent.s e st = yA/„ m ais les
deux événem ents sont distants de A»s^,,„., = = vyA/ц. Si on insère ces valeurs
dans l'équatio n 8.10. on obtient :

.. .) .. ч .. . . “ï ^ "*y*(A i)*
(A/s„.,„«i) - ( A .f w .i ) ‘ /r - - y iA r ) -----------5------

1 - vV e-

P our Sophie, les deux événem ents .se p roduisent au m êm e point (Aig^phi« ~ et elle
m esure un intervalle de tem ps propre (Ar^,phjç = A/y). En rem plaçant ces valeurs dans
fe q u a tio n 8.10, on obtient

~ (^''viphtr) ~ ~ 0^ = (AtoE-

C om m e Sophie et Sam uel calculent le m êm e résultat, on peut alors dire que l’intervalle
de tem ps propre entre deux événem ents est un in v u rian t relativ iste tous les observateurs
vont calculer le m êm e résultat.
218 Chapitres La relativité restreinte

Deux tests de la dilatation du temps


1. H o rlt^ e s m icro sco p iq u es. Les particules éicmcnutires appelées riiuom sont instables :
q u an d un inuon est pro d u it, il n ’cx iste q ue d u ran t une co u rte p ério d e avant d e se
iUsinté^rer (il SC traiLsforme en particules d’autres types). L a durée de vie d ’un muon
est l’intervalle de tem ps qui sépare sa production (événem ent 1) et sa désintégration
(év én em en t 2). Q u an d les m u o n s so n t im m o b iles et q u e le u rs d u rée s de vie sont
m esurées à l ’aide d ’horloges im m obiles (en laboratoire, pai-exem ple), on obtient une
durée de vie m oyenne de 2,200 /rs. C ’est un in terv alle de tem ps p ropre parce que,
p o u r ch a q u e m uon, les év én em en ts 1 et 2 se p ro d u isen t au m êm e en d ro it d an s le
référentiel du muon, c ’est-à-dire au muon lui-même. O n peul représenter cet intervalle
de tem ps propre par le sym bole de plus, on peut appeler référentiel p ropre du
m uon le référentiel druis lequel cette m esure est prise.
P ar contre, si les m uons se déplaçaient dans le lalxjraU)irc. les m esures de leurs
diirée,s de vie pri.ses à l'aid e des horloges du laboratoire devraient indiquer une durée
plus longue (une durée d e vie m oyenne dilatée). Pour vérifier cette conclusion, on a
m e su ré la d u ré e d e vie m o y en n e d e m u o n s v o y ag ean t à une v itesse d e 0 ,9 9 9 4<-
p ar rapport aux hotioges d 'u n laboratoire. D ’après l’équation 8.8, si /1 = 0 ,999 4.
le facteur de l.orcntz de cette vitesse est
1
7=^ = 28,87.
y/1 - ( 0 .9 9 9 4 )’

L 'équation 8.9 donne alors une durée de vie dilatée nn>yenne de

A/ = y âtc - (28,87X 2,200 //s) = 63,52 /rs.

La valeur expérim entale m esurée correspond à ce résultat.

2. H o rlo g es m a c ro sc o p iq u e s. En octobre 1977, Joseph H afele et Richard K eating ont


réalisé ce qui a tlû être une expérience difficile. Ils ont fait voyager quatre horloges
atom iques portatives deux fois autour du m onde dans des avions de ligne co m m er­
ciaux. soit une fois dans chaque direction. Ils voulaient ainsi « tester la théorie de la
relativ ité d ’E instein avec des h o rlo g es m acro sco p iq u es» . O n v ien t de voir q ue la
p rédiction de la d ilatatio n du tem ps a été co n firm ée à un e éch elle m icroscopique,
m ais sa confirm ation avec une vraie horioge est plus convaincante. D e telles m esures
m acroscopiques sont aujourd’hui possibles grâce à la très haute précision des horloges
atom iques m odernes. H afele et K eating ont vérifié les prédictions de la théorie avec
une incertitude de m oins de 10% . (La théorie de la relativité générale d 'E in stein , qui
prédit que l'éco u lem en t du tem ps est influencé par la position d ’une horloge dans un
cham p gravitationnel, jo u e égalem ent un rôle dans cette ex()érience.)
Q u elq u e s an n ées p lu s tard , d e s p h y sic ie n s d e f u n iv e r s ité du M a ry la n d ont
réalisé une expérience sem blable, m ais avec une précision accrue. Ils o nt fait voler
une horloge atom ique autour de la baie C hesapeake su r d es vols durant 15 h et ont
réussi à vérifier la prédiction de dilatation du tem ps avec une incertitude inférieure
à 1 %. A ujourd 'h u i, quand on transporte des horloges atom iques d ’un endroit à un
autre, pour un étalonnage par exem ple, on tien t toujours com pte de la dilatation du
tem ps causée par leur m ouvem ent.

VÉRIFIEZ VOS C O N N A IS SA N C E S!: le b o u t à côté d 'u n c h e m in de fer. vous êtes w iiid a in e m e n t


surpris par un wagon relativiste qui vous dtiublc ; comme le montre la figure, une personne
bien équipée envoie une impulsion laser de l’avant du wagon vers l’arrière, a) La vitesse
de runpulsion que vous nx;surc7 est-elle supérieure, inferieure ou égale à celle mesurce
par la personne qui est dans le train ? h) La durée du trajet de l’impulsion que la personne
mesure dans le train con.stitue t-eIle un intervalle de temps propre c) Sa mesure et la vôtre
sont-elles reliées par l éqtiation 8.97
8 5 LO relativité du temps 219

Exemples 1
Votre vaisseau spatial passe près de la Terre à une vitesse relative de 4. A partir du rcf£rcmicl dc la Terre, lu incaure dc ITiitervalie
U.999 üt. Après avoir voyagé 10,0 a (votre temps), vous vous arrêtez de temps A/ pour le trajet a l a llo f don «ire siipiSn«iire Aiir„. s e l u i
au poste d'observation LP 13, rirez, puis revenez vers la Terre à la r^uaticiii 8.V (Ar = 'VA/yi lelaiivc A la diluiaiion ilu tenipx.
inêiue vitesse relative. Ix- voyage de retour dure encore 10.0 a (votre ,Si on ulilisc I'cquuiion 8.8 pour rcniplacci yduns roquotion U.y,
temps). Coinbicn de temps le voyage aller retour dure-t-il. selon le on consiatc que
leiiifA mesuré sur la 'l'crrc ? (Néglige/, lotit effet relatif îl raccélération
enjeu lors de rimniobilisation. du virage et du rctubli.s-sement de la
A/ =
vitesse de croisière.)
SOLUilON On commence en n'analysant que le trajet à l’aller, et en = (2 2 ..3 7 )(1 0 .0 u )-2 2 3 .7 n .
tenant compte des concepts dés suivanCs. y /\ - (O.yyyOf/e)-
1. Ce problème met en jeu des mesures prises à partir de deux Lois du voyage de retour, nn a lu nicme situation et les inemes données.
référcmiels (incriicls), l’un relié à lu Terre, l’autre (le vôtre) relié
Donc, le vuyage aller-retour prend 20.0 a .selon votre temps, mais
îl votre vaisseau.
2. À r aller, le voyage comporte deux événements : le départ près de = UK223.7 a) = M l a (ié|xins«)
la l’e rre (3t l’arrivée à LPI3. selon le temps nicsiiré .sur lu Terre. Autrcmcnl dit. vous avez vieilli
3. Votre mesure de 10.0 a pour le trajet A l'aller est l’intervalle de de 20.0 a pendant que la Terre vieillissait de 447 ans. üicn qu'on
temps propre A/q entre ces deux événements, car ceux-ci sc ne pui.s.sc voyager dan.s le passé (scion nos connaissance«), on pcin
produisent au même endroit dans votre référentiel. A savoir dans voyager dans l'avenir dc la l'crrc, par exemple en utilisant le iiuiuvenieiit
votre vais.scaii. relatif à haute vitesse pour régler récuuleiiieiit du lemps.

Exemple 8.2
La particule élémentaire appelée kuan positif {Y. ") a une durée de vie oit A/ est rintervaJle de temps qui sépare les deux rvénemems dans
moyeiiiK dc 0.123 7 jzs quand clic est immobile, c’est-à dire quand n'importe quel référentiel. Si on remplace ensuite v par 0,990c et Ar
la durée de vie est mesurée dans le référentiel propre du kaon. .Si un par 0.123 7 fis dans Téqualion 8.12, ou oMient:
kaon positif a une viies.se de (),990î - par rapport au rcfcreniiel du
f/p„ = (0 990< )Ar
laboratoire quand il est produit, quelle distance pem-il parcourir dans
le lalvoraioire durant sa durée dc vie .selon la physiqui' newtonienne ^ (0,9901(2.998 X 10** ra/s)(ü.l23 7 x 10
(approximation valable pour des vitesses très inférieures à c) et selon = 36.7 m (réponse)
la relativité restreinte (qui est bonne Atoutes les vitesses physiquement
possibles)“’ C ’est la distance que le kaon parcoumiii si la physique uewioiiiemie
s'appliqunit à des vitesses proches de r.
SfllUTION On eomniciK'c avec les concepts dés suivanls-
Relativité restntinte ■Lorsqu’on (iboidr le problème a l’aidC
1. Ce problème met en jeu deux mesures effectuées dans deux de la relativité restreinte, on utilise le tontepf rlé suivant il faut bien
référentiels (inertiels), l'un relié au kaon. l’autre relié au laboratoire s’assurer que la distance et l'intervalle dc temps de ré(|u:itinn 8 11
2. Ce problème implique également deux événements : le départ sont tous les deux mesurés dans le même référentiel, panieuiièieinem
du kaon (quand il est produit) et son arrivée (quand il se désintègre). quand la vitesse sc rapproche dc r. comme c’est le cas ici. Donc, pour
déterminer la distance récllcinciil parcourue. </„. par le kaon, tcllc
3. L'équation .suivante relie la disuuicc parcourue par le kaon entre ces
qu'elle est mesurée tians le référentiel du lalnmmnic ci selon la
deux cvcnenients avec sa vitesse v ei la durée de son mouvement
relativité restreinte, on rcformule l’équaiion 8.11 ainsi ;

distance parcourue d„ ^ f Af. (8 13)


(K .ll)
intervalle de temps où Af est rin tcrv allc de temps qui sépare ces dcu.X événements'
mesurés clans le référentiel du lahnrnioire.
Avec CCSconcepts en tête, on détermine d’abord la distance parcourue Avant dc pouvoir évaluer d„ dans l'équation 8,13. il faut
d'après la physique newtonienne, pui.s en fonction de la relativité déterm iner Af en faisant appel au loncepi dé suivant : l’intervalle
restreinte. dc 0,123 7 fis e.st un intervalle rie temps propre parce que les deux
Mécanique newtonienne : F.ii mécanique newtonienne, on se événements se produisent au même endroit dans le référentiel du
fonde sur ce concept tié: on devrait trouver la même distance et le même kaon. A savoir au kaon lui-mônie Par ciMiséquent. on rejircscnte l'in-
intervalle de temps (dans l'éqiraiion 8.1 11. qu’on les mesure dans le tcrvallc dc temps propre par A;,,. On peut alors utiliser réqiiaiinn 8 q
référentiel du kaon ou dans le rcférentiel du laboraioirc. On n’a donc- (A/ = y A/n) relative Ala dilatation du temps pour déterminer l’intervalle
pas A tenir compte du référentiel où les mesure.s sont prises. Poui de temps A/ tel qu’il est mesuré dans le refcrenticl du hiborntoire. Dans
déterminer la distance parcourue par le kaon, f/p,. selon la physique l'éqii.'ition 8.9, si on remplace y par l’expression de l'équation 8.8.
newtonienne, il faut d’abrird rcformiiler l’équation R. 11 ainsi ; on obtient :
Af/i 0.123 7 X l()-^ s
= VAl ( 8 . 12) Af = ^ 8.769 y 10 ' s .
v /i (v/c)^ y /\ (0.990,/c)-’
220 Chapitre в La relativité restreinte

cc qui représente environ sept lois la duree de vie propre du kaun. Cela représente environ sept fois r/p„. Des expériences comme celle
C'est donc dire que le kaon existe environ sept fois plus longtemps qu'on a décrite ici, qui permet de vérifier la relativité restreinte, font
dans le référentiel du laboratoire que dans son référentiel propre - sa partie de la routine dans les laboratoires de physique depuis des
durée de \ie est dilatée. On peut maintenant évaluer l’équation 8.13 décennies. Quand des ingénieurs conçoivent des ctiunternents scien-
poui déterminer la distance parcourue, d„, dans le référentiel du lifiques ou médicaux qui emploient des particules à haute vitesse,
laboratoire • ils dtîivcnt tenir compte de la relaliviié.
r/„ ^ V Л; = (O.WOr ) Дг
- (0.990X2.998 X lO** ui/s)(8.769 x K)-’ si
“ 200 m. (réponse)

8.6 La relativité de la longueur


P our m esurer la longticur d ’une tige qui csl im niobile p ar rapport à vous, vous pouvez
(à votre guise) noter les positions de ses extrém ités su r une règle et soustraire l une des
lectures de l'au tre. C ependant, si la tige sc déplace, vous devez no\sr sim ultaném m i les
positions de scs extrém ités (dans votre référentiel), sinon votre m esure ne peut s'ap|?elei
longueur. La figure 8.7 illustre la m éthode p o u r m esurer la longueur d ’un m anchot qui
m arche : il faut situ er son dev an t et son d e rriè re en m êm e tem ps e t non à d ifféren ts
instants. É tant donné que la sim ultanéité est relative et q u 'elle fait partie des m esures de
longueur, la longueur devrait égalem ent être une grandeur relative. Et elle l’est.
.Soit la longueur d ’une tige que vous m esurez q u and elle est im m obile (ce qui
sig n ifie que vou s et elle êtes d an s le m êm e référen tiel, son référen tiel p ro p re). P ar
contre, s ’il y a un m ouvem ent relatif de vitesse v entre vous et la tige dans le sens de la
longueur de celle-ci, la m esure sim ultanée vous donnera une longueiu L déterm inée par

Lo
L = Ц у / \ - fi'’' = (la contrai-tion des longueurs). (8.14)

Étant donné que le facteur de Lorentz y est tou jours plus grand que 1 lorsqu’il y a un
m ouvem ent relatif. L est inférieure à Le m ouvem ent relatif provoque une contraction
de la longueur, ci L est appelée longueur contractée. Étant donne que y augm ente avec
la vitesse v, la contraction des longueurs augm ente aussi avec i\

La longiieui £„ d'un objet mesuré dans son référentiel propre est sa longueur propre,
fine mesure de la longueur dans tout autre référentiel en mouvement parallèle à eeUc longueur
est toujours inferieure à la longueur propre.
A'
Attention ; la contraction de la longueur se punluit tou jours dans la direction du mouvement
relatif. L es lo ngu eu rs p eip cn d ic u la ire s au m o u v em en t re la tif ne sont p as m odifiées.
D e plus, la longueur m esurée n ’est pa.s trbligatoirem cnl celle d ’un objet com m e une lige
ou un cercle ; il ¡xmi! aussi s'a g ir de la longueur (ou dislance) entre deux objets se trouvant
dans le m ême référentiel propre - par exem ple, le Soleil et une étoile voisine (qui sont,
du m oins approxim ativem ent, au repos l’un par rapport à l'autre).
L’n objet en mouvement rétrécit-il réellement 'I La réalité est basée sur les observations
e t les m esures ; si les résu ltats so n t to u jo u rs co n sé q u e n ts et si on n e p eu t y d éc eler
aucune erreur, ce qui est o b serv é et m esuré est alo rs réel. D e ce point de vue. l’objet
rétrécit réellem cnl C ependant, il est plus ju ste de dire que la m esure réelle indique un
ob)ei plus court - le m ouvem ent influe sur cette m esure, donc sur la réalilé.
Q uanti vous m e su re z la lo n g u eu r co n tra cté e d 'u n e tig e, p a r ex e m p le , q u ’e.st-re
qu’un ob,servaicur sc déplaçant avec la tige dira de votie mesur e Pour lui. vous n 'avez |>as
situe les deux exirém ités de la lige sim ultanément. (Rappelez-vous que deux observateurs

Figure 8.7 Pour mcsnrrr la loiigucm


il’avanl en airièred’iio mam hnl
qui nimchc. vmis devr? noter l'osiùoii
les posiliinis lie son devant et de son
fletrit>re KitYUiltanémcnl (dans votre
léfércmicl), eoiTinie en a», plutôt
qu’à des instanis ilillérents, comme
en b).
6.6 La relativité de la lençuewr 221

e n m o u v e m e n t l’u n p a r rap ix trt à l’au tre n e so n t pus d ’u eeo rd su r la sim u lta n c ité .) P o u r
l’o b se rv a te u r, v o u s a v e z d ’a b o rd sttu é l extrém iu? av an t d e la liy e e t, lé g b re n irn i p lu s
ta rd , so n e x tré m ité a rriè re ; c ’e st p o u rq u o i la lo n g u e u r q u e v o u s a v e z m e s u ré e e s t
in férieu re à la lo n g u e u r p ro p re.
Démonstration de l’équation 8.14
L a c o n tra c tio n d e la lo n g u e u r e st u n e c o n sé q u e n c e d trc c tc d c la d ila ta tio n d u tem p s.
O n re tro u v e ici n o s o b se rv a te u rs S o p h ie e t S am u el. C ette fo is. S o p h ie e.st assise dans un
train q u i trav erse u n e g are, et S am u el, d eb o u t su r le q u ai d c la g are. Ils v eulent to u s d e u x
m e su re r la lo n g u e u r d u q u a i. S a m u e l, q ui iitili.se un ru b a n à m e su re r, déterm ine q u e
la lo n g u e u r est lo n g u e u r p ro p re p a rc e q u e le q u a i e s t im m o b ile p a r ra p p o rt U lut.
Il c o n sta te é g a le m e n t q u e S o p h ie , chin.s le train , tra v e rse c e tte lo n g u e u r dans u n iemp.s
A i -■ L^/v, o ù V est la v itesse d u train ; c ’e st-à d ire q u e
Lij = V A f (Sam ufq. (H 1S)

C e t in te rv a lle A r n ’est p as u n in te rv a lle d e tem p s p ro p re p arce q u e Ic.s d eu x é v é n e m e n ts


q u i le d é fin is s e n t (S o p h ie p a sse l’e x tre m ite a riic rc d u q u a i cl c lic p a s s e I e x tré m ité
a v a n t) se p ro tlu ise n i à d eu x e n d ro its d iffé re n ts, d e .sorte q u e S am u el d o it uttli.ser d e u x
h o rlo g es syncluronisécs p o u r 1c m e.surcr
C e p e n d a n t, p o u r S o p h ie, le q u ai .sc d ép lace. L llc c o n sta te q u e les dcu.x év étio m en ts
m e su ré s p a r S am u el se p ro d u ise n t tiu même endroit d an s so n ré fé re n tie l. Elit* p eu t lo«
c h ro n o m é tre r a v e c u n e seu le h o rlo g e im m o b ile , d o n c I in lc rv a lle At( q u elle m e su re c.si
un in te rv a lle de tem p s p ro p re D e son p o in t d c vue, la lo n g u e u r L d u q u ai e st d éterm in ée
par
/. = v A /o (S ( ^ ic ) . (8 .1 6 )
Si o n d iv is e l’é q u a tio n 8 .1 6 p a r l'é q u a tio n 8 .1 5 e t q u ’o n y a p p liq u e l ’é q u a tio n 8 .9 .
l’éq u atio n d e d ila ta tio n d u tem p s, o n a
L V A/o I
V А/ y (8 .1 7 )

ou l. =

c e q ui est l ’é q u a tio n 8 .1 4 , soit l’éq u atio n d e la c o n tra c tio n d e s lo n g u eu rs

Exemple 8.3 Soplilc

Dans la figure 8.8. Sophie (située au point A) et le vaisseau spatial


dc Samuel (d’une longueur propre ¿o = 2.10 ra) passent l’un près
dc l'autre à une vites.se relative con.stantc v. Sophie mesure l,.‘'7/us SHfimrl
poui riinervalle de temps nécessaire au vaisseau pour la dépasser
(du passage du poini B au passage du point O . Quelle est la vitesse Гщиге 8 8 Exemple 8,3 Sophie, située au point A, mesure le temps
relative r entre Sophie et le vaisseau, exprimée en fonction de la qu'il faut au vaisseau pour la clépas.scr.
vitesse de la lumière e?
Éiani donné t|u’<in suppose que v sc rapproche dc la viiesse de l:<
SOlUnON: ( )n suppose ici que la \ itcssc v est proche de la vitesse dc la lumière, il faut s’assurer que la disiancc et l’iDlcrvalIc dc temps
lumière. On pcui alors commencer avec les tomepls clés suivants. utilisés dans l’éqiiation 8.18 sont tncsiinis dans le réfémiiicl.

1. Ce problème met enjeu des mesures prises dans deux référentiels On peut choisir le référciiliel que l’on veut pour prendre scs mesures.
(incrtiels), l’un lié à Sophie, l’autre lié à Samuel et son vaisseau Puisqu’on sait que l’intervalle de temps Ar mesuré dans le référentiel
spatial. de Sophie entre le.s deux événements est .3.57 ^s, on utilisera aussi
2. Ce pnihlèine met egalement en Jeu deux événements : le premier la distance L entre ces deux événentents, mesurée dans son rétërentiel.
est le passage du point R vis-à-vis Sophie ; le second, le passage L’équation 8.18 devient alors
du point C.
(8 19)
3. Pour chaque référentiel, l’autre référentiel passe à une vitesse v et Af
parcourt une certaine distance dans l’intervalle de temps entre les
On ne connaît pas la valeur dc L, mais le concept d é suivant donne
deux événements :
la relation entre cdle-ci et la valeur de Lo fournie • la distance mesurée
distance parcourue entre les deux événements dans le réfîrentiel de Samuel est la longuem
(8 18)
intervalle dc temps propre ilii vaisseau L„. ITonc. la distaiK'e L mesurée daas le référentiel
222 Chapitre 8 La relativité restreinte

de Sophie doit être inferieure à conune l’indique l’équation 8.14 Par conséquent, la vitesse relative entre Sophie et le vaisseau est
(L = Lu/y) de la contraction des lonfrucurs. Si on remplace L p a il^ /y égale à 21 % de la vite.sse de la lumière. N ote/ que seul le mouve­
dans l’équation 8.19, puis .si on remplace y par l ’équation 8.8, ment relatif de Sopliie et de Samuel imptirtc ici ; le fait que l ’un soit
on obtient : immobile par rapport à un vaisseau spatial, par exemple, n’est pas
pertinent. Dan.s la figure 8.8, on considère que Sophie est immobile,
^ t V l - (vA )^
mais on peut tout aus.si bien considérer que c'est le vaivseau qui est
ûf ~ dr immobile et que c’est Sophie qui passe à côté. Le résultat sera le
Si on isole U on obtient; meme.

Loc
A /)- -I- L ( i ^ V É R I F I E Z VOS CONNAISSANCES 2 : Dans l'exemple ci-dessus.
(230 m)c .Sophie mesure le temps qu’il faut au vais.scau pour la dépa.sser.
Si Samuel mesure aussi cet intervalle de temps, a) qui de Sophie
X 10» m/s)-(3..57 X IO-<^ s)2 + (2;W m)^
ou de Samuel mesure un temps propre et b) quelle mesure donne
= 0,210t. (réponse) un intervalle de temps plus court ?

.S"
8.7 La transformation de Lorentz
La figure 8.9 m ontre le référentiel inerticl S ' qui se déplace à une vitesse v p ar rapport
au référentiel S, dan.s la direciitm positive com m une de leurs axes horizontaux (nom m és
t.véïiement r et .v'). Un observateur se im iivant dans S me.sure les coordonnées d ’espace-tem ps x, y, z.
f d ’un événem ent ; un observateur ,se tiouvani dans S' m esure x', y', z', i' ]x)ur le même
-------- H événetnent. Q uelle est la relation entre ces deux ensemble,s de m esures ?
11 est m ontré à la section 8.13 qu e Ic.s co o rd o n n ées y et t, é ta n t pcriiendiculaircs
au m ouvem ent, ne sont pas influencées par ce d e rn ie r; autrem ent dit, y = y ' et r ~ z '■
On peut alors .se concentrer sur la relation entre x e t x', pui.s sur celle entre f et
Figure 8.9 IDeux référentiels inertiels;
le référentiel S' se déplace à une vitesse Les équations de la transformation de Golilée
î' par rapport au référentiel S.
A v an t q u ’E in ste in ne p u b lie sa th éo rie de la re la tiv ité restre in te, on c o n sid érait
que le.s quatre coordonnées qui nous intéressent ici étaient reliées par les équations de la
transformation de Galilée :
t' = t
(équations de la transformation de Galilée;
approximativement valides à basse,s vitesses). ( 8 .20 )

(C es équations rmt été formulée.s en fonction de la supposition que / = / ' = 0 quand les
origines de S et d e S ' coïncident.) Vous pouvez vérifier la prem ière équation à l’aide de
la figure 8.9. P ar la se co n d e é q u a tio n , on al firm e q u e l’éc o u le m en t du tem p s e st le
m êm e |xm r les observateurs dans les deux référentiels. A vant Einstein, cette affirm ation
était si évidente aux yeux d ’un scien tifiq u e q u ’il n ’en au rait m êm e p as fait m ention.
Q uand la vite,sse v est jietite par rapport à r, les équations 8.20 s ’appliquent g énérale­
m ent bien.

Les équations de la transformation de Lorentz


O n d ém o n trera à la sectio n 8 .13 qu e les b o n n es éq u a tio n s de tran sfo rm atio n , qui
dem eurent valides pour toutes les vitesses ju sq u ’à la vite.s.se de la lum ière, découlent des
postu lats de la relativ ité. L es résu ltats, ap p elés é q u a tio n s d e la tr a n s f o r m a tio n d e
L o re n tz ’* ou, parfois, (plus fam ilièrem ent) la transform ation de I.oreniz. sont

- y (,r - Vf), y ' - y, z' - z, t' — y ( t V X / i^ )


(ët|uations de la iransformation de I oremz.
valides à toutes les vitesses physiquement pos.sililes). (8.21)

(C es équations ont été form ulées avec la supposition que t - / ' = 0 quand les origine.s
de 5 et de S ' coïn cid en t.) N otez qu e la co o rd o n n ée de position .r et la ccxirdonncc de

* Vous deve/. vous demander pourquoi on ne le.s appelle pas fquaU tm \ île la rransjarmaliim il tin a e in
(ei pounjiioi on n’appelle pa.s y fa n e u r H'EiasleiiO. C'esi que Hendrick Amoon l,orenl/ a dérivé
ces équations avant Hin.su-in. mais, eomnte il l'a liii-ntcme concédé, il n’a pas interprété ces équations
eoniine une description de la vraie nature de l ’espace et du temps. C'est cette inierpréuition. pour
la première lois effectuée par Kinstein . qui est au coeur de la théorie de la rclativilé
8.7 La tranGformallon de Lorentz 223

lABlIAU 8.2 les équations de lo transformation de Lorentz pour des paires d’événements
1. A r = -yfAv' -t- V A /') I '. A x ' = 'y(A<f - V A/)
2. Ar = yiAr' -t vA ïV c’) 2'. A/' — y(A/ v Ajï/ c^)

1
y=
v /l - (v/f)^ v /l
I x rctcrciilicl 6' se déplace à la vitesse ? dans la direction de fax e ties .x ¡sositifs par iaii|xiii
au référentiel S.

tem ps t sont liées d an s la p rem ière e t la d e rn iè re éi|iiatin n . C e lie n eiiire l 'e s p a i r


et le tem ps é ta it un d es p rin cip au x m e ssag e s de la tlién tie d 'F ln s u in , rn essag e
longtem ps rejeté par plusieurs de ses eomemjxHraiiis.
Une condition obligatoire que doivent respecter les éijuations reliitivi.strs csl de .se
réduire aux équations non relativistes fam ilières si on eoiisiilèie qu e t approche l’intini.
A u trem en t dit, si la vitesse de la lu m ière était in lln im e n t gran d e, toutexi Icô v itesses
finies seraient « faibles » et les équations non rclativi.stcs seraicnl toujours vûlidcp. Si on
prend la lim ite c —» oo d an s les éq u a tio n s 8.21. y > I et c e s éq u a tio n s SC réduisent
(com m e on s 'y attendait) aux équ atio n s galilccnncs (équations 8.20). Vous devriez le
vérifier.
La form e des équations 8.21 est utile si on connaît les valeurs de .x e t dc f et q u 'o n
veut déterm iner celles de x et de t '. On peut toutefois voulou' déterm iner Ic.s valeurs de x
et de t à pitrtir de jr' et de D ans ce cas, on isole sim plem ent r c t i dans les étjimtions 8.21
pour obtenir
X = y(x' + i r ') et / = y (/' + vx'/c‘). 18.22)
O n trouve, en les com parant, que les éq u atio n s 8 22 so n t o btenues à p artir d es équa
lions 8.21 en interchangeant les grandeurs prim es et non prim es et en inversani le signe
d ev a n t le m o d u le d e la v ite sse relativ e v (p o u r roh.vcrv ateu r dans le rclé rc n lic l .S",
le référentiel S sc déplace à vitesse v vers la gauche).
Les équations 8.21 et 8.22 relient les coordonnées il’un événem ent unique p a ç u par
deux observateurs. Mais, parfois, on ne veut pas cotinaîlre les coordomiées d 'u n événement
unique, m ais plutôt les d iffé ren ce s en tre les coordonnée.s d un e paire d ’év énem ents.
A lors, dans ce cas, si on nom m e les év én em en is o b serv és 1 et 2, on peut irffcctucr la
relation suivante :

Av = V2 — X) et A i ~ h — f),
com m e le m esure un observateur d ans le référentiel S, ci

Av^ vo a. J et Ai tj f |,
com m e le m esure un obst rvatcui dans le référentiel !>'.
L e tableau 8.2 p rése n te les équaiion.s d e L o re n tz sous la fo rm e d e diffé re n ces,
applicables à l'a n a ly se d 'u n e paire d ’événem ents, l.c s équations de ce tableau ont clé
dérivées sim plem ent en rem plaçant les quatre variables d es équations 8.21 et 8.2? par
des différences (com m e Av e t Aa ' ).
A tte n tio n : en rem p laçan t les v aleu rs p ar les d iffe re n c e s, as.surcz-vous d 'ê tr e
cohérent ; ne m élan g ez pas les v aleurs du p rem ier év én em en t avec CCllCfi du second.
D e même, si, par exem ple. A i a une valeur négative, il faut inscrire le signe négatif lors
de la substitution.

| / V É R I F IE Z VOS CONNAISSANCES 3 : i fig u re illa s tre tro is situ a tio n s où un ré lé rc m ic l bleu


et un rétércnliel vert soni en mouvement relatif «lans la (lin'ctioii commiiix' rie leur,s axe.s
des.v et des a', comme l’indique le vecteur viiesse attaché à l'un d'eux Dans chaiiue cas.
si on dit que c’est le référentiel bleu qui csl iminohilc. les signes devant v dans les équations
du tableau 8.2 doivent-ils être ehancés?

e)
224 Chapitre 8 La relativité restreinte

8.8 Quelques conséquences de la transformation


de Lorentz
Ici, on fait appel aux équations de la transform ation présentées dans le tableau 8.2 pour
obtenir ceitaines des conclusions que nous avons déjà trouvées en utilisant directem ent
les postulats d ’Einstein.

La simultanéité
On prend l'équatio n 2 du tableau 8.2.

(8.23)

Si deux événem ents sc p roduisent à des endroits différents dans le référentiel S ' de la
figure 8.9. A x' a une valeur non nulle dans cette équation. D onc, m êm e si ces événem ents
.sont sim ultanés d an s S ' (A r' = 0 ). ils ne le seront pas dans le référen tiel S (ce qui
concorde avec la conclusion ém ise dans la section 8.4). D ans S. l’intervalle de tem ps
entre les événem ents sera

- A t'
At = y (évciiemeiiLs simultanés dans .V').

La dilatation du temps
On suppose m aintenant qu e deux év énem ents se produisent au m êm e endroit dans S'
(Av' = 0), m ais à des instants différents (A /' ^ 0). L 'équation 8.23 se réduit alors à

A i — y At' (événements se produisant au même endroit dans S'), (8.24)

ce qui confirm e la dilatation du tem ps. P uisque les deux év énem ents se produisent au
m êm e endroit dans S', on peut m esurer l’intervalle de tem ps A t' qui les sépare à l'aid e
d ’une seule horloge, située à cet endroit. D ans ces conditions, on m esure un intervalle
de tem ps propre, q u ’on peut nom m er A/,,. D onc, l’équation 8.24 devient

A t= y A lQ (dilatation du temps),

qui est exactem ent l ’équation 8.9, l'éq u atio n de la dilatation du temps.

La contraction des longueurs


On prend l’équation 1' du tableau 8.2,

A v '= ylAv vA t). (8.2.5)

St une tig e est parallèle aux axes des v et des .v' de la figure 8.9 et est im m obile dans
le référentiel S', un observateur se trouvant dans S ' peut m esurer sa longueur à sa guise.
Il peut alors soustraire les coordonnées d e scs extrém ités. L a valeur de A» ' obtenue sera
la longueur propre L,, de la tige.
S upposez m aintenant que la tige se déplace dans le référentiel 5. C ela signifie q u ’on
peut décrire A ^ co m m e étant la lo n g u eu r L de la tige dans le référen tiel S seu lem en t
si on m esure simultanément les coord o n n ées des ex trém ités de la tige, c ’est-à-d ire si
Al = 0. Si on insère Av' = L t, Av = L et Ar = 0 dans l’équation 8.25, o n obtient

Lo
L —— fcontraciion des longueurs). (8.26)
y

ce qui c o rre sp o n d ex a ctem e n t à l’éq u atio n 8 .1 4 . l'é q u a tio n d e la co n tra ctio n des
longueurs.
8.8 Quelques consequences de la transfôrrTratiori de boreriti 275

Exemple 8.4
Un vaisseau spatial terrien a etc envoyé pour connaître l’état d’un
poste avancé sur la planète P14Î17, dcfrii la lune abrite un groupe de S'
V'ais.'nau
combattants reptuliens, souvent hostiles. .Sur une trajectoire rectiligne
qui (Ta-ssc d’abord près de la planète, puis prè.s de la lune, le vaLsscau
Unie
détcclc une émission de micro-ondes à haute énergie à la base lunaire (ciiussion) I'Luh' u -
des Reptuliens puis, 1.10 s plus tard, une explosion au poste avancé (OV|lll».iMU)
terrien, qui se trouve à 4.1X) x U)^ ni de la base reptulienne. distance
mesurée dans le létércniicl du vaisseiur. Les Reptuliens ont apparcin- Figure 8.10 Lxemplc 8.4 Une planète et sa lune dans te léféienilH
nieni attaqué le poste avtincé terrien, si bien qu'à l’intérieur du vais­ se dcplaecnt vers la droite à une vitesse u pat rap|xiii à un vais.vrau
seau. tm SC prépai e ptiur un atïrontemenl. SC tioiivant dans le référentiel i'.

a) La vitesse du vaisseau par rapport à la planète et à sa lune est de


0. 980.. Quels sont la di.stance et l’intcn'alle de temps entre remission L équaiiiHi 8.27 devient alors
et l’c.xplosion. mesurés dans le référentiel inertiel planctc-lune (donc,
scion la perspective des occupants du poste avancé) ? A.v' - (5,02.5 2)
SOllïïlON On ctimmence avec les conteplstln suivants. X 14.00 X 10“ m - ( +ü.980)(2.998 x 1(У m/s)(1.10 s)|
= 3,86 X 10“ m, (réponse)
1. Ce problciuc met en jeu des mesures prises dans deux référentiels,
le référentiel planète-lune et le référentiel du vaisseau. et l’équation 8.28 devient
2. Ce problème met en jeu deux événements : l’émission et l’expkision.
3. Il tàut tran.sformei en données appropriées au référentiel planète- Af' = (5,025 2)
lunc les données relatives aux temps des deii.x événements et à la (-^0,980)(2.998 x iC т/ч)(4 (У) x 10“ m)
X ( l . l Os ) -
distance les séparant mesurées dans le référentiel du vaisseau. (2,998 X 10* iii/s)-
Avant d'effectuer la liansformation. il faut soigneusement choisir la = -1 .0 4 ,s. (reponse)
notation. On commence en trayant un croquis de la situation, comme
dans la figure 8.10. On y a établi que le référentiel du vaisseau 5 est b) Que signifie le signe négatif dan.« la valeur tic At'
immobile cl que le référentiel planète-lune S’ est en mouvement avec
une vitesse vers la droite. (Ce choix est arbitraire ; on aurait pu consi­ SOLUTION: L’important ici est d’etre cohérent avec la noration que I on
a choisie en a). Rigipelai-vous eonmienl on a d'abord défini l’uitcrvalli
dérer tonuiie immobile le référentiel planète-lune. Il aurait alors fallu
redessiner v dans la figure R. 10 en l’attachant au référentiel 5 et en de temps entre I’emis.sinii cl l’explosion : A/ = q 1« ” 1.10 8.
indiquant un mouvement vers la gauche. les signes devant c auraient Pour être cohérent avec cette notation, on doit définli- A;' i.oiinne
alors etc changés. Les résultats auraient toutefois été les mêmes.) étant r' — . on constate donc que
On décrit l’explosion avec l'indice x et l’émissitHi avec l’indice m.
At' - I’, - >1 = - u n , .
Les données fournies, toutes dans le référentiel S (du vaisseau),
sont alors Le signe négatif indique que iJ,, r, ; c'est donc dire que. dans
Ax = Vx - Xn, = + 4,00 X 10“ ni le référentiel planctc -lune. l ’émission s’est produite 1,04 s iiprUx
l’explosion, et non pas 1,10 s tivani l’explosion, comme on Ta perçu
et = 0 /„.= - 4 . 10 s. à partir du référentiel du vaisseau.
Ici. Av est une valeur positive parce que. dans la figure 8.10. la C) L'émission a-t-elle provcxjué l'explosion ou esi-cc le i ontrairc
coordonnée v, de l’explosion est supérieure à la coordonnée .x„,
de l’émi.ssion ; Д/ est aussi une valeur pcisttive parce que le temps q SOLUTIOH■L'ordre des événements {xiryii dans le référentiel plmiètc lune
lie l’cxplo.sion est supérieur (après) au temps de l'émission. est l’inver.se de l'ordre perçu dans le référentiel du vaisseau Ici,
On ehen he Ад' et Ai ’, que l'on devra obtenir en transformant le concept dé est que. dans l’une ou l autre de ces situations, s'il y a une
les données du référentiel S en données du référentiel planète-lune S' relation de cause à effet entre les deux événements, l’information doit
Liant donné qn'on étudie une paire d’événements, on utilise les voyager d’un endroit à l'autre pour provoijiirr le second évenetnenf.
équations de transformation du tableau 8.2. à savoir les ériuations On vérifie donc la s ilcsse à laquelle cette information doit voyager
l ' c t 2 ’; Dans le réfcienticl du vaisseau, cette vitesse est

A.»' = y(A.r vA/) (8.27)


Av 4.00 X 10* m
Vinfo - ^ = -3.M X 10* m /s.
et 1.10 s
(8.28)

Ici, f -t 0,980t. et le facteur de Loreiiiz e.si vitcs.se impossible parce qu elle dépasse r fJans le référentiel
phinèie-lune. la vites.se ohiennc est 3,'7() x 10“ m/s. également impos­
sible Par conséquent, aucun des evénements ne peut avoir provoque
= .^.0252. l’antre ; autrement ilit. il .s'agit de deux cvcncmcnts
y = Donc, les occupants du vaisseau ne iloivent pas s 'en prentlrc mix
V^l - (v/cŸ \ / l -Н О .Ш к /с У
Reptuliens.
226 Chapitre 8 La relativité restreinte

> > 8.9 La transformation relativiste des vitesses


P a n ic u lc D an s ce tte se c tio n , on u tilise ra les éq u a littn s de la tran sfo rm atio n d e L o re n tz p o u r
n ' ine.suré<* dans S* obtenir une relation entre la vitesse d ’une particule m esurée par un observateur se trouvant
w in ts u rc f <lans S dans un référentiel S e t la vitesse de la m em e particule m esurée par un o b serv ateu r se
trouvant dans un référentiel S '. O n suppose d ’abord q ue S' sc dép lace à un e vitesse v
orientée vci-s l’axe des x positifs par rapptni à S.
Rgure 8.11 Le référentiel .V' se déplace O n su p p o se e n su ite q ue la p a rtic u le , v o y ag e an t à v ite sse co n sta n te o rien tée
ü line vitesse v par rapport au référentiel >V. parallèlem ent aux axes des x et des jc'dans la figure 8 . 11. ém et deux signaux. C haque
Une particule possède une vitesse m' observateur m esure l’intei-valle de tem ps et la distance qui .séparent ces deux événem ents.
par n^port au référentiel S' et une Ces quatre m esures sont reliées par les équations 1 et 2 du tableau 8.2,
vitesse Mpar rapport au référentiel S.
= yfA-Y' + V A /')

et
(8.23)

Si on divise la prem ière de ces équations p ar la seconde, on obtient

Av à x ' + V A /'
Ai A i' v A v '/f^

Si on divise le num érateur et le dénom inateur du m em bre de droite par A/ ', on obtient

Av _ A.vVA/' + V
Ai 1 + V {Ax'/At')lc^

C ependant, lo rsq u 'o n prend la lim ite Ai -> U. Av/Ai devicnl «, la v itesse de la particule
m e su ré e d ans S, et A x '/A t' d ev ien t ti', la v ite sse d e la p articu le m esu rée d an s S '.
On a alors, finalem ent,

U= (transf<-vrrnation relativiste de la vites.se) (8.29)


1 + n 'v/c^

com m e équation de transform ation relativiste de la vitesse. C ette équation se réduit à


une équation de transform ation de la vitesse galilcciinc.

U — m' + V (transformation de la vitesse galiléenne) (8.3Ü)

quand on applique la lim ite non relativiste qui consiste à considérer c A utrem ent
dit, l’équation 8.29 s ’applique à toutes les viles.ses physiquem ent possibles, alors que
l ’équatio n 8.30 e st ap p ro x im ativ em en t v alab le pour d e s viles.ses très in férieu res à c.
R e m arq u e/ que. si on rem place la particule par une im pulsion lum ineuse qui .sc déplace
à la vitesse de la lum ière dans le réléren tirl S ' (donc u' = c), la vites.se dans S esl aussi
égale à c, quelle que soit la vitesse relative v entre les deux référentiels.
L ’équation 8.29 est la transform ation relativiste d e la vite.sse lorsque la vite.s,se de la
p articu le est p a ra llè le à la vitesse relativ e en tre les d eux référen tiels En g én é ral, si
dans le référentiel S ' (qui se déplace dans la direction d e l’axe des x positifs par rapport
au référen tiel S) le v e c te u r v itesse d e la p a rtic u le a d e s co m p o san tes mJ, et
les com posantes du vecteur \ itessc de la particule m esurées dans le référentiel S .seront
données par:

" 1 +

11,. =
y ( \ + ii,'v/r^), (8.31)

U.
y (ï + «.'v/r-)
8.10 L ’effet Doppler retatfviete 22/

8.10 L'effet Doppler relativiste


D ans la section 3.8, on a abordé l’cffel D oppler (dccalaEc de la fréquence m esurée) pour
les o n d es so n o res v o y ag e an t d an s l ’air. D ans le cas d e te lle s o n d es, l'e f f e t o o p p le r
dépend de deux vitesses, à savoir les vitesses tie la sotiice et du détecteu r par rapport
à l’air. (L 'a ir est le milieu qui traasnict Ic.s onde.s.)
C e n ’est pas le cas avec les ondes lum ineuses, car celles -ci (et les autres ondes olee-
tromagiiétiqucs) n’ont besoin d ’aucun milieu de transmission ; clics peuvent racine voyajtor
dans le vide. L ’effet D oppler pour les ondes lum ineuses n e depend que U une viusse, la
vitesse relative v entre la .source et le détecteur, m esurée dans l’un des deux rOfÉreniiCls.
S oit la f ré q u e n c e p r o p r e d e la so u rce, c ’c.st à d ire la fré q u en c e mesurée p a r un
o bservateur dans le référentiel propre d e la source. Soit f la fréquence m esurcc par un
observateur se déplaçant à une vitcs.se v par rapport au rcfcrenticl propre de la source.
A insi, quand l'o b serv ateu r s’éloigne directem ent de la source.

,f = ./n (source ei d é ie e ie iir s 'é lo ig n a iii l ’ un de l'a iu ie ). (8 3 7 )

où P = vtc. Q uand l’o b se rv ateu r se dirige d iicctcm en l vers la source, il faut changer
les signes devant les deux sym boles /8 dans réq u atiim 8 .3 2 .1 dém onstration de l'é q u a ­
tion 8.32 est présentée à la section 8.13.

L’effet Doppler pour de faibles vitesses


D ans le cas de faibles vitesse« (j8 « 1), l'éq u a tio n 8.32 peut être dévelopj>ér en .série,
ce qui donne l’approxim ation snivanle :

f=fç){l ~ P + ( s tu ir c e e id é U ic ie u r s 'é lu ig n a n l,f î <i. 1). (8 33)

L ’équatio n de l ’e ffe t D o p p ler des ondes sonores pour de faibles vitesses p ossède les
deux m êm es jtrem iers term es, mais st>n Iroisièm c term e affiche un co efficient différent.
D onc, l ’effet relativiste pour des sources lum ineuses et des détecteurs voyageant à faible
vitesse n’apparaît q u ’avec le term e p^.
P o u r m esurer la vitesse l’ d 'u n e auto, un cin ém o m ètre d e p olice (com m uném ent
appelé radar) fait appel à l ’effet D oppler su r les m icro-ondes. ITne source placée dans
l ’appareil ém et un faisceau d e m icro -o n d es à un e ce rtain e fréq u en ce (p ro p re) /¡, sur
la route. U ne v o itu re qui sc d irig e vers l’appareil in tercep te ce faisceau , m ais à une
fréquence d écalée vers le haut par l ’effet D oppler, en raison d e son m o uvem ent vers
l ’appareil radar, l/a u to réfléchit le faisceau vers l ’appareil. Étain d onné que l’aum sc
d irige vers l ’ap p areil, le cap teu r de c e d e rn ie r in tercep te un faisceau réfléchi dmii la
fré q u en c e e s t en c o re p lu s d éc alée v ers le h au t. L ’ appareil com pare cette n o u v elle
fréquence av cc/n et calcule la vitesse v de l’auto.

L'effet Doppler astronomique


Lorsque vous o b se rv e / des étoiles, d es gaiaxie.s ou toute autre so n n e lutm ncusc. vous
pouvez déterm iner la vitesse de la source, (pii s'élo ig n e on qui se laiijirodie de voas. en
m esurant le déplacement Doppler de la lum ière qui vou.s piu vieni Si une l'cn ain c étoile
était im m obile par rapport à vous, vous détecteriez sa lum ière à une certaine fréquence
propre / q. C ependant, si cette étoile s'élo ig n ait ou sc rapprochait directem ent d e VOUS,
vous déteeicriez une lum ière possédant une fréquence /’décalée de Jn par relTel Di ipplrr
C ’est le m ouvem ent radial de l'éto ile (s’approchant ou s'élo ig n an t ditvciem eni de vous)
qui provoque ce décalage, de sorte que sa m esnm vous perm et sciilrrnenl de déterm iner
la vitesse radiale v de l'é to ile - soit uniqiirm eiil la compo,sante radiale de la vitesse de
l’étoile p ar rapport à vous.
Supposez que la vitesse radiale v tl'u n e certaine source lum ineuse est assez faible
(P est assez proche de zéro) p o u r que vous puissiez négliger le term e p - d r réq iin iio n
8.33. Placez égalem ent un ± devant le term e P (le signe n égatif cone-spond au inmive-
ment radial s'élo ig n an t de vous ; le signe positif, au nxuivem ent radial .se rap p o s liant)
l,'éq u atio n 8.33 devient alors

^ P ). ( 834 )
228 Chapitre 8 La relativité restreinte

H ab itu ellem e n l, en astro n o m ie, on m esu re la lo n g u e u r d ’o n d e p lu tô t q u e la


fréquence de la lum ière ; on rem place donc / par d k e t/^ p ar r/Ào, où k est la longueur
d ’onde m esurée e t Àf, est la lo n g u e u r d ’o n d e p r o p r e . Si on rem p lace ég a le m en t fi
par v/c dans l’équation 8.34, on obtient

k y c)

k — ki\
ce qui m ène à V = ± — ------ c.

Par convention, on écrit cette équation ain.si :

^k
( vitesse radiale de la source lumineuse, v «: e). (8.35)

où AA = lA. — 2t„l est le déplacement Doppler de la longueur d'onde de la .source Inminease.


Si la source s ’éloigne de vou.s. À est supérieure à Àp ; on parle alors de déplacement vers
le rouge. (C ette expression ne signifie toutefois pas que la lum ière détectée soit m uge
ou m êm e visib le; clic signifie plutôt que la longueur d ’onde augm ente.) D e m êm e, si la
siHirce .se rapproche de vous, k est inférieure à Ay. et on parle de déplacemeni vers le hleu.

^ VÉRIFIEZ VOS CONNAISSANCES 4 ; u figure -Dctc< u-ui


illustre une .source qui émet une lumière
de fréquence pmpreyj, en sc dcplaçtuit
directement vers la droite <1une vitesse r/4
mesurée dans le référentiel S. La figure montre
également un détecteur qui mesure cette lumière
à une fréquence/ > / , . a) Le détecteur
SC déplace l il wns la gauche ou vers la droite h) La vitesse du détecteur mesurée
dans le référeniiel S est-elle supérieure à c/4. inférieure à e/4 ou égale à c/4 ?

L'effet Doppler transversal


Jusqu’à m aintenant, ici et dans le chapitre 3, on n 'a abordé l'effet D oppler que dans des
situations où la source et le dcleeleur se rapprochent ou s ’éloignent directem ent l'u n de
l’autre. L a figure 8.12 illustre une situation différente, dans laquelle une source S passe
près d ’un détecteur D. (Juand S atteint le point P. sa vitesse est perpendiculaire à la ligne
la joignant à D et, à cet instant, elle ne se rapprtx’he m ne s ’éloigne de D. Si cette source
ém et des ondes sonores de fréq u en ce/o . D détecte cette fréquence (sans effet D oppler)
en captant les o n d es ém ises au point P. C e p en d a n t, si la so u rce é m e t d es o n d es
lu m in eu ses, il y à to u jo u rs un e ffe t D oppler, d it tr a n s v e r s a l. D ans ce cas. q u an d la
source SC trouve au point P, le détecteur capte une lum ière de fréquence

f ~ f ( ) V^l — (efld Doppler transversal). (8..36)

P o u r de faibles vitesses (/5 <g; I), l ’éq u atio n 8.3ô peut être dév elo p p ée en sé rie ;
on obtient alors l ’approxim ation suivante :

(faibles vitesses). (8.37)

Ici. le prem ier ternie correspond à ce à quoi on pourrait .s’attendre en présence d ’ondes
■sonores. D e plus, l’effet D oppler relativiste pour les sources et les détecteurs à faibles
vitesses fait intervenir un term e proportionnel à
Figure 8.12 L ne .source lumineuse S
se déplace à une vitesse v en passant fin piincqie, un ciném om ètre de police peut déterm iner la vitesse d ’une auto même
prt Ad lin déiri tnn situé au point F* quand la trajectoire du faisceau radar est perpendiculaire (transversale) au m ouvem ent
La théorie de lu relativité restreinte de l'auto. C ependam .selon l’équation 8.37, étant donné que /5 a une petite valeur même
prodit un eftet Doppler tran.svcr.sal à J10I1I une voilure rapide, le term e relativiste fiV l dans l’effet D oppler transversal a une
rmotant ou la source passe au point R valeur extrêm em ent faible. IT o n c ,/- ./n, de sorte que le ciném om ètre calcule une vitesse
oil 1.1 direciion de «on déplacement mille. C ’est pourquoi les policiers essaien t to u jo u rs d ’alig n er le faisceau radat su r le
c.st praptaidlciilaire In ligne passant trajet de l’auto p o u r obtenir un déplacem ent D oppler donnant la vitesse réelle de l’auto.
par r>. Lii théorie non relativiste T oute d év iatio n p a r rapport à c e t alig n em en t favori.se le co n d u c te u r de l'a u to parce
ne prédit aucun effet semblable. q u ’elle réduit la m esure de la vitesse.
8.Ш Leffet Dof^ier relotivieto 22V

L’effet D oppler transversal constitue égalem ent un autre m oyen permoiuuit do tostor
)a dilatation du tem ps. Si on rcfo rm u lc l'éq u a tio n S.36 en em p lo y an t lu irénotle 7 de
l'oscillation de l’onde lumineuse ém ise au lieu de la fféquonee, <»n tihiieni (fiuisque T UJ )

T - ,____ ,
To
= yT'ü- (8.38)

où 7() ( = 1//Ô) est la période propre de la source. C om m e le dem ontm une comparaiKon
avec l’équation 8.9, l'éq u atio n 8.38 est .simplement la form ule de la dilatation du tem ps,
puisqu’une période esl un intervalle de tem ps.

Le système de navigation NAVSTT^R


C h aq u e sa te llite N A V ST A R ém et co n tin u ellem en t des sig n au x rad io d o n n an t sa
p o sitio n , à une fréq u en ce rég lée et eonlrfiléc p a r dc.s h o rlo g es .itnm iqucs d e gran d e
précision. Q uand un de ces signaux est capté pai le délcctcur d 'u n avion com m ercial,
par exem ple, sa fréquence a été décalée par l’effcl D oppler Un cap tan t sim ultnncm cnl
les signaux de plusieurs satellites NAVSTAR, le ilciccicur peut détci minci la diicclion
de n ’im porte lequel d 'en tre eux et la direction de son vecteur vitc.s.se Hii sc fondant sur
le déplaeem eni D o p p ler du signal, le d étecteu r p eu t d éterm in er In vitesse ilc l'av io n .
(Il faut aussi considérer un effet D oppler causé par la relativité générale. )
O n peut faire un calcul rapide p o n ra m sta ie r l'cfficacité de ce phénomène. La vitesse
d 'u n satellite NAVSTAR p ar rapport au centre de la T erre est appnixiTnalivenicnt de
1,0 X 10^ m/s. La valeur de fi correspondante est d ’environ 3,0 x 10 D onc, le leriiK-
^ -/2 des équations 8.33 et 8.37 (qui est le term e de la relativité) est approxiinaiivrm ern
4,.*; X lO" '°. A utrem ent dit. la relativité m odifie le déplacem ent D opplei du signal iieiyu
d ’environ 4,5 parties .sur 10'*'. ce qui n ’c.st pas très con.sidérablc.
Cependant, ce décalage est quand m êm e im |xrrtanl. Le.s horloges atom iques qui .se
tro u v en t d ans les sa te llite s so n t si p ré c ise s q u e la v aria tio n d e fréq u en ce du sig n ai
du sa te llite n ’est tiue de 2 p arties su r 1 0 ’^. l’ar l’éq u a tio n 8 .3 7 , o n con.statc qu e fi
(et par conséquent v) dépend de la racine carrée àeflfit. D onc, la variation de 2 x 1()-'^
de la fréquence de l ’horloge provoque une variation de

v/T7"îô-'- = 1,4 X 10-*'


de la valeur m esurée de la vitesse relative v entre le satellite et Tavion.
Puisque la valeur de la vitesse relative entre Tavion et le satellite v découle principa
lemeni de la gnuide vitesse du satellite, 1,0 x 10* m/s. cela signifie que v (et par conséquent,
la vitesse de Tavion) peut être délc*rminéc avec une précision approxim ative de

(1.4 X 10~*)(1,0 X 10“* m /s) = l,4 c m /s .

Supposez que T avion vole durant 1 h (3 600 s). Si vous connaissez la vitesse approxi­
m ative à 1,4 cm /s, vous pouvez prédire la position de 1 avion à la fin de cette heure a \ 8C
une précision de

(0.014 m /s)(3 600 s) = 50 m.

ce qui est acceptable en navigation aérienne m oderne.


Si on ne tenait pas com pte des effets de la relativité, il serait impossible de d éter­
m iner la vites.se de Tavion avec une pi-écision inférieure a 21 cm/S. CC qui nC pCrmCUTail
pas de prédire une position avec une incertitude inférieure à /6 0 m

Exemple 8.5
Lit figure 8.13 a) représente graphiquement Tintcnsilc. en fonction SOlUnON: Les (oncepts dés sont les suivants :
tk* la longueur d’onde. J ’iinc lumière provenant de gaz intcrMclIaires
se trouvant a des eûtes opposés de la galaxie tVl87 (figure 8 .13 hj) 1. Si le gaz ne tournait pas autour du cœur de la galaxie, la lumièie
Une courbe afteini un pic à 4W,8 nm, Taulre, à 501.0 nm. l e ga/ qu’il émet serait détectée à une certaine longueur d’onde.
orliitc aiiloui du cn iir de la galaxie à un rayon r = 100 années- 2. Fn raison de l'effct Doppler, le n louvemcnt du gaz nioditie la longueur
lumière ; il se dirige apparemmeni vers la Terre d'un côté du cœur, d’onde dércUée, Taugmrntani dans le cas du gaz qui s'éloigne de
et s'en éloigne du côté opposé. la Terre et la diminuiini dans le ras du gaz ipii s’en lapprochc

a) O n rllr courbe correspond au gaz se dirigeant ver.s la Iérre ! Donc, le pk de la roiirhe J) ,501,6 uni i o rrrs|X )n d au niouvcmciil
¡b) üucllc esl la vitesse de ce gaz par rapport à la Terre (et par rapport qui s’éloigne <lc la Terre ; le pic à 499.8 nm. au mouvement qui s’en
au cœur de la galaxie) ’’ rapproche
230 Chapitre 8 La relativité restreinte

On peut supposer ici que l’augmentation et la diminution de la


longueur d’onde provoquées par le mouvement du gaz sont d’égale
grandeur. La longueur d'onde non décalée, que l’on considérera
comme une longueur d ’onde propre Лц, doit alors être la moyenne
des deux longucui.s d’ondes décalées ;

501,6 nm + 499,8 nm
Ao=------------- ----------—- = 5(X).7 nm.

Le déplaccniem Doppler А/. de la lumière provenant du gaz s’éloignant


de la Terre est alors

АЛ = 1Л - Л,|1 - 501.6 nm — 5(Ю,7 nm


= 0.90 nm. L ongueur d 'o n d e (nm )
.Si on insère cette valeur et A = 501.6 nm dans l'équation 8..55, on a)
ctatstale que la vitesse du gaz est figiiie 8.13 Fxempic 8.5 a) Courbes de l’intensité, en fonction
de la longueur d'ondc. de la lumière émise par des gaz situés
ДА _ 0,9 nm O à des côtés opposés de la galaxie M87 el délectée sur Terre
2,998 X lO** m /s
A 501.6 nm b) l a région centrale de M87. Les cercles indiquent les positions
5 X lO'"’ m/s. (réponse) des gaz dtsni les intensités sont données en a), l e eteui de M87
se trouve à mi chemin entre ces cercles.
b) Le gaz tourne autour du cœur de la galaxie parce qu 'il subit
une force gravitationnelle générée par la ma.sse M de ce cœur.
Si on rassemble ces trois concepts, on oNient:
Quelle e.st cette masse exprimée en multiples de la masse du Soleil,
rWs( 1,99 X 10“' kg)? CMm
— — = m

SOlUnON. Ici, deux tonceph clés entrent en jeu.


r- r
Si on isole M et si on insère les données fournies, on obtient
1. D ’après l’équation 14.1 du volume I, le module F de la force
gravitationnelle exercée sur un élément gazeux de masse m orbi
tant à un ra> on r est v~r
M ----------
G
GM>tt
F = (5..38 X 10^ m/s)^ (l0 0 al)(9,46 x lo'"' m / ^
6.67 X 10 N • m’ / i ^
2. Si ] 'élément gazeux décrit un cercle autour du cœur de la galaxie, = 4 X 10^^ kg (2 X (réponse)
il doit avoir une accélération centripète de module = i-^/r.
orientée vers le cteur. Ce résultat indique qu’une masse éqiiivalanl à deux milliards de
3. Selon la deuxième loi de Newton, fomiulée pour un axe radial .soleils est comprimée dans le cœur de la galaxie, ce qui suggère
sortant du cteur cl sc dirigeant vers l’élémenl gazeux, = nui^. fortement qu'un trou noir extrêmement massif occupe ce ctcur.

8.11 La quantité de mouvement en relativité


Supposez q u 'un c n la in nombre d ’observateurs, chacun dans un référentiel inertici distinct,
observent une collision isolée entre deux particules. En m écanique new tonionne, on a
vu q u e, m êm e si les ob.servateiirs m e su re n t d iffé re n te s vites.ses p o u r les p articu le s,
ils déterm ineront tous que le principe de la conservation de la quantité de m ouvcm cnl
est resp ecté, so it q u e la q u an lilé d e m o u v em en t to ta le du sy stèm e d e p a rtic u le s est
la m êm e avant et après la collision.
Com m ent la rd aliv ité influence t elle œ lte situation ? On constate que, si t>n continue
de définir la quantité de m ouvem ent p d ’une particule par l ’expression rnv, .soit le produit
de sa m asse et de sa vitesse, la quantité de inouvem ent totale « ’est pas conservée pour
des observateurs se trouvant dans des référentiels inertiels différents. O n a alors deux
c h o ix . abandonner le principe de conservation de la quantité de m ouvem ent ou vtnr si
on peut redéfinir la quantité de m ouvem ent d 'u n e particule de façon à ce tjue ce principe
continue de s'appliquer, l.e second choix e st le bon.
Im aginez une p a rtic u le se d ép laçan t à une vitesse co n stan te v d an s la d irectio n
po.sitivr de l'a x e des x. S elo n la d éfin itio n d e N ew to n , le m o d u le de la q u a n tité de
m oiiveincnl de la particule est *

mv = m (quantité de m ouvem ent new tonienne). (8.39)


^r
8.12 L'énergie en relativité 231

où A.C csi la distance parcourue tlan.s l’intci valk de letup.'» àt. Ptnir formuler une cTtprcesion
re la tis is te de la q u a n tité d e m o u v em en t, on re m p la c e T in te rv a lle d e tem p s p a r un
in \a ria n t relativiste ; rin le rv a lle de tem ps projire

Ici, com m e a v a n t A v est la di.stancc p arco u ru e p ar une p articu le en m ouveiuein telle


q u ’elle est vue p ar un ob.scn atcur. C ependant. A/,, est le tem p s requis p o u r parco u rir
cette distance, tem ps me.surc non par l’ob.scrvateur surveillant la particule on m ouve
m ent, m ais par un observateur .sc déplaçant avec ccUc particule. La particule est imiPO
bile par rapport à ce second observateur, ce qui a pour nésultnt que cc dernier m caurc un
tem ps propre.
Si on fait appel à la form ule de dilatation du temps (ét|uation 8 9), on |x lU r c iirr que

Av A/ A.V
P = m ~ T~ 1— ~ T~ "y'
Af() Af Ai, Ai

C ependant, puisque Av/Ai est la vitesse v de la particule.

P = ymv (quaniité de niouveiiKiil). (8.40)

N otez que le facteur de L orentz constitue la seule difféicnee eniic cette exprc.ssion r i la
définition newtonienne qu’est l'cquation 8.39. (.’ependant. cette difidrcncc*" esiim portatilc ;
la quantité de m ouvem ent relativiste s’approche d 'u n e valeui infinie quand » appnx lis c,
ce qui n 'e st pas le cas quand il est question de la quantité d r m ouvem ent new tonienne.
On peut généraliser l’équation 8.40 sous form e vectorielle:

P — yim ’ (quantité de mouvement). (8.41)

C ette équation constitue la bonne définition de la quantité de m ouvem ent pour toutes les
vitesses physiquem ent p o ssibles. D ans le cas d 'u n e vitesse très Inférieure à î , elle s r
réduit à la définition new ionienne de la quantité de m ouvem ent (p — mv).

8.12 L'énergie en relativité


L’énergie au repos
La chim ie s’c'st d ’abra'd développée avec l'idec que, lors des réaciiiHiv rliim iqnrs, l ’r n n g ir
et la m asse étaient conservées de m anière indcpicndantc. Hn 1905. E instein a dém ontré
que, en eon.séquence de la théorie de la relativ ité restreinte, la masse iv u t être considérée
com m e une autre form e d ’énergie (la dém onstration d 'E in sle in est présentée n lu sec
lion 8 . 13). D onc, le p rin cip e de c o n se rv a tio n d e l ’é n e rg ie eM en fait le priiu ipe de
conservation de l’ensem ble m asse-énergie.
D ans une réaction chimique (processus où des atom es cl des m oléfulcs interagis
sent), la quantité de m asse tran sfo rm ée en d 'a u tre s fo rm es d ’én erg ie (ou vice versa)
conslilue une fraction si infim e de la m asse totale en jeu q u ’il e.«t im|->os.siblc de m esurer
la variation de cette mas.se. m ême avec les m eilleures balances. 11 semble que la mafnSC
et l’é n e rg ie so ien t réellem ent co n serv ées séparém ent. C rp en d a n t, d an s une rcartion
nucléaire (où des noyaux et des particules subatom iques interagissait), l’énergie dégag/x-
est .souvent un m illion de fois supérieure à celle d ’une réaction chim ique, de sorte q u ’on
peut facilem eni m esuicr la variation de m asse l e traiisferl nia«sr-énergir qui se produit
lors d ’une réaction nucléaire est connu depuis longtem ps.

“Certains auteurs interprètent fexpns'üiin ym ('(sninip une rnavur rrl.itivist« Nous ni- raisoii.s p,i.s cetU;
imerpréiaiinn newtonienne dans re volume II est prctcrablc de cUnnpci la définition de la quantité
de inoiivemern plutôt que de ehanper la déftnilion de la masse qui apparaît dans la quaniité dr mouvemr.nl.
232 Chapitre 8 La relativité restreinte

TABLEAU B.3 Les énergies ou repos de quelques objels


Objet Masse (kg) Énergie au repos
Electron 4.11 X 10""' 8,19 X 10 (= 511 keV)
ftoton 1,67 X 10-2^ l „ 5 0 x 10 '“ J (= 938 Me V)
Atome d’uranium 3.95 X 1 0 '“ 3,55 X lO"" J (= 2 2 5 Ce V)
Particule de pou.ssière 1 X IO -'5 1 X Kf J ( = 2 kcal )
Pièce de I cent 2,25 X 10-^ 2,02 X 1 0 '''J (= 56.2 GW • h)

L a relation entre la m asse m d ’un objet et son énergie au repos £(> est

Eo = rm '. (8.42)

qui est, sans l ’in d iee 0 , l’é q u atio n scien tifiq u e la p lu s co n n u e d e to u s les tem ps. La
dém onstration de cette relation apparaît à la section 8.13. L’énergie associée à la m asse
d ’un o bjet e st appelée équivalence masse énergie, ou énergie au repos. La seconde
ex p re ssio n suggère qu e Ep e s t u n e én erg ie q u e m êm e l ’o b je t au rep o s p o ssèd e, tout
sim plem ent parce q u 'il a une m asse. (Si vous poursuivez vos études de physique après
ce cours, vous lirez des textes traitant plus à fond de la relation entre la m asse et l’énergie.
Vous pourrez m êm e en lire qui contesteront la nature et la signification de cette relation.)
Le tableau 8.3 donne l’équivalence ma.s,se-énergie, ou énergie au repos, de quelques
objets. Par ex em p le, celle d ’u n e p ièce de I cen t est én o rm e, soit l’éq u iv alen t d ’une
quantité d ’énergie qui coûterait près d e quatre m illions de dollars. Pat ailleurs, toute la
production annuelle d ’énergie électrique de la centrale Roberl-Boura.ssa (la plus grosse
centrale hydro-électrique du Q uébec) ew resp o n d à un équivalent de seulem ent 2 kg de
m atière (pierres, pom m es de terre ou n ’im porte quoi d 'au tre).
En p ratiq u e, on n’u tilise q u e rarem e n t les u n ités SI d an s l’éq u a tio n 8 .42 p arce
q u ’elles .sont tro p g ran d e s p o u r ê tre co m m o d es. O n y ex p rim e p lu tô t les m asses en
unités de m asse atom ique, où

1 u = 1,66 X 10 kg, (8.43)

cl l’énergie, en électronsolts ou en ses m ultiples, où

1 e V = 1,60 X 1 0 - '''J. (8.44)

E xprim ée en unités des équations 8.43 et 8.44, la co astan te de m ultiplication a les


valeurs .suivantes :

= 9,315 X 10" eV/u = 9,315 x H)“' keV/u


= 931.5 MeV/u. (8.45)

On écnit au.s.si parfois la m asse des particules subatom iques en MeV/r^. (Vfur l’annexe B.)
Par exem ple, la m asse d 'u n proton est 938,3 McV/c^. C e la indique que son énergie au
repos est égale à 938.3 McV.

L'energie totale
L'é<iuation 8.42 donne la mas.se au rep«is (ou l'éq u iv alen t m asse-énergie) £„ as.soeiée
à la m asse ;/i d 'u n o bjet, peu im porte que ce d e rn ie r so it au repos ou en m ouvem ent.
S ’il se déplace, il (X)ssèdc une énergie additionnelle appelée énergie cinétique, K. Si on
suppo.se que .srm énergie potentielle e.st nulle, son énergie totale Ee.st la som m e de son
énergie au rep<is et de son énergie cinétique :

£ = bu + K — m e 4 K. (8.46)

O n ne le dém ontrera pas ici. m ais l'én erg ie totale f. peut égalem ent être exprim ée ainsi :

£ — ymr^, (8.47)

o(i y est le facteur de lo r e n tz relatif au m ouvem ent de l'o b jet.


8 .12 Lènergio en relativité 733

D epuis le chapitre 7 du volum e 1, on a éuulié de noiiihreux exem ples oii inici vien­
n en t les vaiiaiion.s d ’én e rg ie to ta le d ’une p m tie u lc ou d ’un sy etem e d e p articu les.
C ep en d an t, on n ’y a pas inclus celles de l ’éncrgio au rep o s p arce que .ses vîiriMiions
étaient soit nulles, soit assez petites pour être négligeables Or, le [)i tiii iire de consi i vaiion
de l’énergie totale s ’applique toujours niéinc si les vanaiions de l’cncrgic au repas sont
im portantes. Donc, peu inifmrte ce q ui arrive à l’énergie au repos, l’énoncé suivant, tire
de la section 8.7 du volum e 1, dem eure vrai :
r<JS-
!► L’énergie totale t d’un système isolé ne peut pas varier.

Par exem ple, si la sontm c des énergies au repos de deux particules en interaction dans
un systèm e isolé dim inue, un autre type d ’éne'igic du système doit augm enter parce que
l’énergie totale n e peut varier.
D ans un systèm e qui subit une réaction ch im iq u e t)u m id é a iie , une variation Je
l’énergie au repos du .système causée par la réaction s ’exprim e souvent pat la viileur ç ,
l a valeur Q d ’une réaction s'o b tien t par la relation

énergie au repos \ énergie au rcpo.s \

<HI
( initiale du sy stèm e/

% = /^oi-1 a
(finale du .sv sièm e / ' ^

À l’aide de l’équation 8.42 (E q = /tir’), on peut récrire cette relation en em p lo y an t la


m asse totale initiale M, et la m asse totale finale Mf ainsi ■
(8.48)

+ Q

ou Q = M.r^ - A/,c2 = -A M c “ , (8.49)

où la variation de m asse causée par la réaction est \ M “ M,- A4;.


Si une réa ctio n se solde p a r un e tra n sfo rm a tio n d ’én e rg ie au rep o s en én e rg ie
cinétique des produits de la réaction, pat exem ple, l’éiicrgic au ne|xis totale fin sy.stcmc
(et sa inas.se totale M) dim inue et Q est positive. Si une réaction requiert p lu tô t que celte
énerg ie soit tran sfo rm ée en énergie au repos, l ’éu erg ie au repos to tale tlu systèm e
(et sa m asse totale M) augm ente cl Q est négative.
P ar exem ple, on peut supposer que deux noy aux d ’hydm gcnc subissent une réaction
de fusion au tx/urs de laquelle ils s ’unissent pour form er un noyau unique et lllTèrein ile.nx
particules. L ’énergie au repos (et la m asse totale) du systèm e comprenant Ic noyau qui
en résulte et les deux particules libérées est inférieure à l’énergie an repos totale (cl à la
ma.sse totale) des noyaux d'hydrogene initiaux. D onc, la valeur Q de la fusion esi positive,
et l’énergie est d ite dégagée (transform ée à partir de l’cncrgic au repos I p ar la réaction.
C e dégagem ent d ’énergie est im portant poin nous tous parce que la fusion des noyaux
d 'h y d ro g è n e q u i se p ro d u it d an s le S oleil fait p artie du proccssirs qui d o n n e le
rayonnem ent du Soleil su r Terre et qui rend la vie possible.

L’énergie cinétique
Dans le chapitre 7 du volum e I. on a défini de la m anière suivante l'én erg ie cinétique K
d 'u n objet de m asse ni se déplaçant à une vitesse v très inferieure à r .

F - ^/rii-2 (approxunation non relativiste) (8..‘50)

C e p en d an t, cette éq u atu m non rela tiv iste n’est q u 'u n e ap p io x im aiio n su ffisam m en t
bonne quand la vitesse en jeu est très inferieure à celle de la lum ière.
[| faut maintenant trouver une expre.s.sif>n poui l’énergie cinétique bonne pour Witter
les vitesses physiquenxînt possibles, y compris celles qui se rapprix lienl de c. .Si on isole K
dans l’éq u a tio n 8 .4 ô et ipTon rem p lace F par son e x p re ssio n d e l'é q u a tio n 8.47,
on obtient

K ^F rnc- ~ yme^ — m n
=ir u fy 1) (énctpic cinétique). (8.51)
234 Chapitre 8 La relativité restreinte

où y ( = 1/-^/! — (y/cŸ ) est le facteur de L orenlz relatif au m ouvem ent d e l’objet.


L a figure 8.14 représente graphiquem ent l ’énergie cinétique d ’un électron calculée
à l ’aide de la bonne définition (équation 8.51) e t selon son approxim ation non relativiste
(équation 8.50), to u tes d eux en fo n ctio n d e v /r. D ans la p artie g au ch e du graphique,
n o te / que les deux courbes coïncident ; c ’est la partie du graphique (relative aux basses
vitesses) qui correspond aux calculs des énergies cinétique.s q u ’o n a effectués ju sq u ’ici.
C e tte p artie in d iq u e q u ’on a eu ra iso n d ’e m p lo y e r l’e x p re ssio n n o n re la tiv is te de
l'éq u a tio n 8.5 0 p o u r calcu ler l ’én erg ie cinétique. C ependant, dans la p artie droite du
graphique (relative aux vitesses se rapprochant d e c). les deux courbes sont com plète­
m ent différentes. Q uand v /r s ’approche de 1,0, la courbe de la définition non relativiste
n ’au g m en te que m o d é ré m en t, alo rs q u e ce lle d e la d é fin itio n rela tiv is te (la b onne)
au g m en te c o n sid érab lem e n t. D onc, q u an d la v itesse v d 'u n o b je t s ’ap p ro ch e de r,
il fiuif u tilisa l’équation 8.51 pour calcu ler l’énergie cinétique.
l.a figure 8.14 révèle égalem ent un fait im portant su r le travail q u ’il faut effectuer
sur un objet pour augm enter sa vitesse, de 1 %, par exem ple. Le travail W néccs.sairc c.st
figure 8.14 Les ctjiiatiotis relativiste
égal à la variatio n de l’é n e rg ie c in étiq u e AX d e l'o b je l. P o u r q ue c e lle v aria tio n se
(équation 8.51 ) et non relativiste
réalise dans la partie relative aux faibles vitesses (gauche) du graphique 8.14, le travail
(cfiuaticin 8.5<)) de l’énetïîie cinétique
d’un électron icpré.sentées graphiquement nécessaire peut être m odeste. C ependant, si la variation doit se produire dans la partie
en fonction de v/r, où re st la vitesse relative aux hautes vitesses (droite), il faut fournir un travail énorm e, parce que l’énergie
de l’électron et l’est la vites.se cinétique K augm en te très rapidem eni dans celle p artie q u and la v itesse v augm ente.
de la lumière. Note/ que les deux P our augm enter la vitesse d ’un objet à c. il faudrait, en principe, une q u an tité infinie
courbes coincident à de faibles d ’énergie ; par conséquent, il est im possible de le faire.
vitesses et qu’elles sont très différentes On exprim e souvent les énergies cinétiques des électroas. protons et autres particules
à des vitc-sses élevées. Les données en éiectronvolts ou en un de scs m ultiples utilise com m e adjectif. Par exem ple, un électron
expérimentalc.s (marquées par des X) p o ssé d a n t une é n e rg ie c in étiq u e d e 20 M eV p eu t ê tre d é c rit co m m e un élec tro n
monlrenl que, à des vitesses élevées, de 20 M e V.
la courhe relativiste décrit très bien
les données cx|jérunentales alors La quantité de mouvement et l'énergie cinétique
que la courbe non relativiste ne décrit
pas du fout ces jxiints. t!n mécanique non relativiste (ou new tonienne), le motlule de la quantité de m ouvem ent p
d ’une particule est w v el son énergie cinétique K est ^mv^. Si on élim ine v de ces deux
expressions, on obtient une relation directe entre la quantité de m ouvem ent cl l’énergie
cinétique ;

/)‘ = 2Km (non relativiste). (8.52)

O n peut trouver une relation .semblable en relativité ; p o u r ce faire, on élim ine v de la


définition relativ iste de la c|uantité d e m ouvem ent (équation 8.40) e t de la d éfinition
relativi.ste de l'é n e rg ie cin étiq u e (é q u a tio n 8.51). On o b tie n t alo rs, ap rès un peu
d 'algèbre.

(pcŸ — K - + IKinc^. (8.55)

Avec l'aid e de 1 équation 8.46, on peut transform er l’équation 8.53 en une relation entre
la quantité de m ouvem ent p et l’énergie totale L d 'u n e p articule;

- (prŸ + (nu-)-. (8.54)

Le triangle rectangle de la figure 8.15 peut vous aid er à vous souvenii de ces relations
utiles. Vous poiivc’/; égalem ent dém ontrer les expressions suivantes dans ce triangle

sin O = et cos 6 = \ /y. (8.55)

D ans l’équation 8.54, on peut co n stater que ic produit pt doit avoir les m êm es unités
riguri>R.I5 Voici un rme nnèmonique que l’én erg ie £ ; la q u an tité dc m o uvem ent p d o it donc .s’ex p rim er com m e une unité
utile p nir retenir les rolatioas relativistes d ’rn c ig ie divisée p ar c. En fait, la q uantité de m ouvem ent en physique des particules
m ire l’énergie lourle B. l’énergie .s’exprim e souvent en M eV /c ou en CleV/<. *
au repos (ou équivalente masse- On peut aussi réécrire l’équation 8.54 sous la form e :
énergie) me'-, l'cncrgic cinétique K
et la quantité dc inouvemcnt /> £,? - ( m r y - F? (pcŸ. (8.56)
в 12 L'énergie en relativité 235

Celte équation a la m êm e form e qu e l'éq u atio n 8 10. qui indique que le tem pa propre est
un in v a rian t rela tiv iste. L ’éq u a tio n 8 .56 in d iq u e q u e 1 é n e rg ie au rep u e fei dniic la
m asse) d ’une particule e.sl un invariant relaiivi.sic L 'équatiun 8.54 eai viiltiblc quel que
soit le référentiel utili.sé.
P our une particule sans m asse (com m e le photon, la partieulo de lum ière qui vei a
introduite au prochain chapitre), l ’équation 8.54 donne une lelaiion e n iir l'éne rgie ci la
quantité d e m ouvem ent :

E = ¡K' tpariiciile sans niawsc) 18.57)

'V É R IF IE Z VOS CONNAISSANCES 5 ; a) L'énergie c in é tiq u e ei b ) l’ém ig ir to ta le d 'u n électron

r ck 1 GeV sont-elles supérieures, inférieures ou égales à celles d'un p ro u in tic 1 (îcV

Exemple 8.6
a) Quelle est l’énergie totale é ’d’un électron de 2,53 MeV? que l'énergie au ie|>os d’un électron est égale à 0.511 McV.
L'équation 8.58 donne alors
50ll!Ti0M■I /t cenrept dé utilisé ici est le suivant : selon l’équation 8 46,
l’énergie loialc F est la somme de l’énergie au repris (ou équivalence, E = 0.511 McV + 2.53 McV = 3.04 MeV. (réponse)
ma.s.se-énergie) de l'électron mc^ et de son énergie cinétitjue ;
h) Quel est le module de la quantité de mouvciiiriil p de l’élcctron.
E = me’ -♦ K. (8.58) exprimé en MeV/c ?

Dans l’énoncé du problème, l’adjectif «2,53 MeV » signifie que SOlUTIOH■Ici, le coraepf dé est le suivant ; on iicut dclermmer p a pat1.ir
l’élcrtron possède une énergie cinétique de 2,53 MeV. Pour évaluer de l'énergie totale E et de l'énergie au ie|xis m r à I’uidc de l 'cqua
l'énergie au repos m r de l’électron, on remplace sa rnas.se m par sa tion 8.54,
valeur donnée dan.s l’annexe B, ce qui donne
E- • f I- (nu 'Ÿ.
mc~ (9.KW X 10-” kg)(2,998 x 10* m/s)^ Si on isole pc. on obtient ;
= 8,187 X I 0 - ‘"J. pc = v/£2 - (mH)2
Si on divise ensuite ce résultat par 1,602 x 10"'^ .l/MeV. on obtient ” V(3,04 MeV)^ - (0.511 MeV)2 - 3.00 MeV.
une énergie au repos de 0.511 MeV pour réleclron. (On peut Finalement, si on divise les deux membres par c. on obiieni :
confirmer cette valeur dans le tableau 8.3. On aurait pu aussi utiliser
la mas.se de l’électron donnée à l’annexe B en McV/c^.) Cela indique P - 3,00 MeV/e. (réponse)

Exemple 8.7
Le proton ayant la plus grande quantité d'énergie jamais détectée (3,0 X 1fi2“ eV)
dans les rayons cosmiques atteignant la Terre possédai! une étonnante y = I+
7938 X 10^ evT
énergie cinétique de 3.0 x 10^’ cV (suffisamment d'énergie pour
- 3 .1 9 8 X lu '* y |ri" (réjvH ise)
faire grimper de quelques degrés Celsius la température d'une cuillère
à thé d’eau).
Cette valeur de y est si grande qu'on ne peut utiliser la déi'iniiioii
a) QiK'ls étaient le facteur de I .orent? y du proton et sa viiejv.se r (tous de y (équation 8.8) pour déterminer v. fcssayez te j votre caluulaci i»«
deux par rapport au détecteur sur rerre) ? vous indiquera que /1 a une valeur égale à 1 Ct que v est égale à r
En fait, Vest très pn clie de r mats on vent une valeur plus précise,
SOl UnCN: On emploie ici un premier tortepl dé ; le facteur de 1.orentz y
que l'on peut obtenir en résolvant d’aboid IVqii/itinn 8.8 cri tonclion
du proton relie l’énergie totale E avec l’énergie au repos me' par
de I /f. Pour commencer, on écrit
l'équation 8.47 ( f ym< ”). Ix deuxième concept dé est que l’énergie
totale du proton est la somme de son énergie au repas me' et de son 1 I
énergie cinétique K (connue). Si on combine ces deux concepts, ^ v^2(1 - / ! ) '
on obtient :
où on a utilise le lait que /< est si près de l’uuiié que I fl passèdi
E n u ' -F K , K
y = — ,= --- 2" — 1 + ----5 ■ une valeur très proche de 2 vitesse que i'on chen hc est contenue
me- mc^ me-
dans le terme 1 fl. Si on isole 1 - fl. on obtient :
On peut olitenir l’énergie au repos me' du proton à partir de sa masse
l 1
(exprimée en MeV/c^ dans l’annexe B), comme on l'a fait [vnir 1-/6
l’électron de l’exemple 8.6 a). Ainsi, nu ' est 938 MeV (comme 2p (2)(3 198 X 1 0'‘)’
l'indique le (jihlcau 8.3). Si on insère cette valeur et l’énergie cinétique - t.O X и г ’ '' 5 V 10'-.^4
connue dans l équaiion précédente, on obtient :
23é Chapitre 8 La relativité restreinte

Donc, = 1 - 5 X I0'2i. 1. Ce problème met en jeu deux me.sures effectuées dans deux
référentiels (inertiels), le référentiel Terre Voie lactée et le référentiel
bl. puisque v — fie, du proton.
V= 0,999 999 999 999 999 999 999 995c. (réponse) 2. Ce problème met aussi en jeu deux événements ; le premier se
pitxluii quand le proton entre dans la Galaxie, le .second, quand il
b) Supposez que le proton traverse la Voie lacice sur son diamètre en sort.
(9,8 X 10^ al). Combien de temp.s lui l'aut-il. appRiximativeineni. 3. Dans le référentiel du proton, on mesure un intervalle de temps
pour iw courir cette distance si on le mesure du rclérentiel aimmun propre A/fl parce que les évéïiement.s s’y produisent au même
de la Terre et de Ja Galaxie ? endroit, à savoir au proton lui-même.
SOtüilON : On vient ju.ste de voir que ce proton uUnirdativisie voyage 4. On peut déterminer l'intervalle de temps propre A/y à l’aide de
à une vitesse trè.s peu ini'crieure à r . Drmc. le concept de applicable ici l'intervalle de temps niesuré dans le référentiel Terre-Voie lactée
est que. selon la définition de rannée-kimicre. la lumière prend 1 a en utilisant l’équation 8.9 (Af = yA/y) de la dilatation du temps.
pour parcourir 1 al ; elle devrait donc prendre 9,8 x K)* a pour par
Si on isole Ar„ dans l’équation 8.9 et si on remplace y par sa valeur
couru 9.8 X Kl* al, et ce proton devrait prendre è peu prè.s le même
obtenue en a) et Ai par sa valeur obtenue en b), on obtient :
temps Par coaséquent, du rélërenticl Terre-Vciic lactée, le voyage du
proton prend
Ai ^
Д/ = 9,8 X l( f a. (réjxmse) “ y ” .T Î 9 8 X 1 0 “
= .î.0 6 x 10 ’ a = 9 ,7 s. (réponse)
c) Combien de temps ce voyage dure-t-il dans le référentiel du
proton ? Dan.s le référentiel Terre-Voie lactée, le voyage dure 98 000 a. Dans
le référentiel du proton, il dure.. 9,7 s ! Comme on l’a énoncé au
SOLUTION: Ici, on a besoin de quatre concepts clés. début du chapitre, le mouvement relatif peut modifier l’écoulentcnt
du temps ; on en a ici un exemple extrême.

8.13 Trois démonstrations


La transformation de Lorentz
La dém onstration des équations 8.21 de la transform ation de Lorentz s ’effectue en trois
étapes. O n considère nos deux observateurs. Sam uel et Sophie. On suppo.se que Sophie
est dans un train qui se déplace à une vitesse v par rapport i) Sam uel. On appellera S le
référentiel de Sam uel et S' le référentiel de Sophie.
D ’abord, il faut d ém o n trer que les longueurs perp en d icu laires au m ouvem ent ne
sont pas changées. N os observ ateurs font rex p é ric n ce suivante : Sophie est dans un train
qui se déplace à une v itesse v et Sam uel est sur le quai de la gare. Sui ce quai, il y a un
mur. Sam uel a tracé une ligne rouge tout le long du mur, en utilisant un pinceau attaché
au bout d ’une ügc d e 1 m q u 'il m aintient perpendiculairem ent au sol. U ne ligne rouge
a donc été tracée s'ur le m ur p ar Sam uel à une hauteur de I m. Sophie trace de la m êm e
façon une ligne bleue sur le m u r en iitili.sant un pinceau au bout d ’un m ètre.
Supposons que la longueur petpendiculaire au m ouvem ent d ’une tige est contractée.
D aas ce cas, pour Sam uel, la tige de S ophie est en m ouvem ent et sera contractée. Cela

Mur

Figure 8.16 a) Pour Samuel, la tige


de .Sophie est en mouvement ; clic est
<lonc eondacrée. l.a lige .se trouvant
dans le tram trace une ligne bleue plus Mur
ba,s.se que Ift ligne rouge, b) Four Sopliic.
e’eat le mur qui est en mouve nieni
et lu hauteur de la ligne rouge .sera
eomraetée. ligne ronge sent
donc plus basse. b)
8.13 Trois demonstfotionfi Eî

produira une ligne bleue à une hauteur plus basse que la ligne rouge, com m e l’ilfusue la
figure 8.16 a). La figure 8.16 b) illustre le.s observations de Sophie : pour elle, c 'e s t le
m ur qui est en m ouvem ent et sa hau teu r (incluant la ligne ntu g e qu e Sanuiel a tiat ée^
sera contractée. D onc, pour S ophie, la ligne bleue sera suiTcricure à la ligne rouge. Il > a
alors une contradiction en tre Ic.s ob.scrvations de Sophie et cclleti de Sam uel. E t cette
fois, on ne peut pas faire intervciiir la relativité de la sim ultanéité pour expliquer la dit
férence dans les observations. O u ne peut pas avoir en m em e tcmp.s une ligne rou^C au
dessus de la ligne b leu e pour un o b se rv ateu r et une ligne b leu e ati-dcs.sii.s d e la ligne
rouge pour l’autre observateur. La seule façon d'élim inet la com radieiinii est île d ite que
Ic.s longueurs perpendiculaires au m ouvem ent d ’un rélérentiol par rappoi t à un aim e ne
changent pas I x s deux lignes colorées seront alors supcrpo.sccs.
Sam uel et Sophie font une deuxièm e expérience pour trouver cette fo is la iransfor
Rquro 8.17 Pour Sophie, l’ampoule in ation des co o rd o n n é e s p o u r un o b jet situ é à r o r ig in e du rél'ércnticl de S am uel.
SC déplace vers la gauche. Elle émet D ’abord, à / = 0 et f' = ü, le sy stèm e de conrdonnée.v d e S ophie (д 'y ', ') ci celui de
une impulsion au temps t' à la position Sam uel (дуг) coïncident. C om m e les dim ensions perpendiculaires au déplacenieni, qui
x' ~ vt'. s ’cflecu ie le long des axes des л et des a ', ne .sont pas changées :

y
г' (8 5d)

S u p p o so n s que d an s le référen tiel S (le rctc re n tie l d e S am u el), il V a une p etite


am poule située à l’origine (a — 0) qui ém et une im pulsion liimineu.ic au tem ps / Don.s
le référentiel S' (celui de Sophie), à quel endroit e t à quel tem ps cette im pulsion ем -elle
ém ise ? P our S ophie, l’am poule .sc déplace vers l’axe des x' négatifs. C om m e l'am p o u le
était à la position a ' = 0 à r' — 0, elle doit se trouver à x' = v f apres un tem p s t',
où t' est le tem ps m esuré par Sophie (voir la figure 8.17). Le terrqjs r — i„ est un temps
propre pour Sam uel, car l ’am poule est im m obile dans son rélércnliel. On obtient alors,
d 'ap rès l ’équation 8.Ч de la dilatation du tem ps ;

t' = yt. (8.(Ю )


C om m e a ' = - vr', alors:
X = (8 6 1 )

P our c o m p léter la d ém o n stratio n de la tran sfo rm atio n d e L o ren tz, on con.sidère une
troisièm e expérience, décrite à la figure 8.18. U ne p an icu le subaiom ique (par exem ple
un inuon) e s t produite. Sophie et Sam uel choisissent la position de ha particule lors de sa
pnxluclion comme étant l'origine de leurs référentiels. Les coordoiiuées d'espare-tem ps de
f= 0 la particule lors fie sa prtxluclion sont alors (f = U ; a — 0) pour Samuel et ( f “ 0 ; a ' - 0)
• --- X p o u r Sophie. L a particule se déplace vers l'a x e des a positifs (et l’axe des a ' ¡losiliis)
Production Desintégratmn
av an t de sc d é sin té g rc i. S am u el m e su re que la p a n ic u le SC d ésin tè g re au tem ps r
a) et à la position a (fig u re 8.18 aj). Pour Sophie, la p articu le sc d csin tcg rc au tem ps f
et à la position a ' (figure 8.18 b]). I.a relation de transform ation entre ces coordonnées
doit être une relation linéaire :

a' = /tv Pt
t' = Dx ■ rt.

où A. B. D e i F ne d ép e n d en t ni d e la p o sitio n n i du tem p s de ré v é n e m e m . D an s la
partie 2 de la dém onstration, nous avons obtenu la traiisfoiTnation pour un événem ent se
situant à ï ü. En p osant a = 0 dans les équ atio n s précédentes, e t en co m p aran t les
résultats aux équations 8.60 et 8.61. on obtient F = y et R ~ - y v . L es transform ations
(■-()
deviennent :
PllKiuclion rVsintéf;ratinn
a' Лл - yv/
h)
t' — Dx 4 yi. (8 6?)
figure 8.18 Une particule est produite
à l’origine et se déplace vers la droite P our ob ten ir les coefficients A et D. on considère les m esures de Sophie (/' et x')
avant de se désintégrer, a) Samuel et celles de Sam uel (r et r). L a durée de vie de la particule est un tem ps propre t„ pour
observe cette désintégration au temps t celle-ci. O tem ps propre p eu t être calculé par n 'im p ttrie quel o b serv ateu r en iitili.sam
et à la position x. ht Sophie observe l’équation 8.10. C e tem ps est un invariant relativiste ;
la désintégration au temps r' et à la
position x'. te = t 'У с' - F - x 'k . ( 8 .6 .1)
238 Chapitre 8 La relativité restreinte

En insérant l’équation de iransform ulion 8.62 dans l’équation 8.63. <in obtient :

— x'Vc~ = + 2yD xt + P^x- — (y4-jr — lA yvxt + y'^v^t-)

~ 11 ~ vVr^] + 2 y fD + /4i'/r-l xt - - / A -] .r^/r*.

C om m e les valeurs d e x et de / sont arbitraires, il faut que le ten u e proportionnel à xt soit


nul et que le term e proportionnel à x^/c^ soit égal à 1 :

D + Av/c^ = 0

La solution de ce systèm e d 'éq u atio n s donne \ A = y ci D — —yv/r^. On obtient alors


les équations 8.21 de la transform ation d e Lorenl/. :

-t’ = y{x - Vf)

t' = y (t — vxJc^). (8.64)

L’effet Doppler relativiste


P our d ém ontrer l’éq u atio n 8.32. on suppose que S ophie est dans .son train en m o u v e­
m ent à une v itesse v en s ’élo ig n an t d e S am u el. S am u el en v o ie une o nde lu m in eu se
(com m e à la lig u re 8 .1 7 ) qui a une fréquence p r o p r e / q . Sam uel m esure une pérkxJe
propre T^. P our S o phie, la p ério d e q u ’elle m esu re sera un in te rv alle d e tem ps dilaté
A /' = y ^ u . De plus, pour elle, la source s’est déphK ée vers la gauche d 'u n e distance
V A i'. L 'o n d e a donc bc.soin de parcourir une distance supplém entaire vAi'. ce q u ’elle
fe ra d ans un tem p s v A /'/r. L’in te rv a lle total en tre d eux fro n ts d ’o n d e o b se rv é s p ar
Sophie .sera donc :

T — à t' + l ’A /'/e = y T tid -t- fi).

C om m e y = l / \ / l — /3* = l / y / d — /5)(1 H- 0 ) , on o btient;

T -T

La fréquence étant T inverse d e la période, on obtient l’équation 8.32

f= f (8.6.S)

L’énergie au repos
L’équivalent inas.se-énergic (ou énergie au repos) a été d écouvert par A lbert Einstein en
1905. L ’aniu’c suivante, il a donné une jo lie et sim ple dém onstration en considérant le
centre de m asse d ’une boîte c|ui contient une source lum ineuse.
Considérons une boîte de longueur L qui est au repos sur une surface sans frottement.
C om m e le présente la figure 8.19, la btutc contient une source lum ineuse. E lle ém et une
Pijsiiion finale Position iniluil>- im pulsion lum ineuse. C o m m e la lum ière tran sp o rte de la q u an tité d e m ouvem ent, la
/
boîte d o it a v o ir un e v itesse d e recul v o rie n té e vers la g au c h e p o u r q u e la q iian tilé
1 5 1 de m ouvem ent soit conservée. D 'a p rè s l’équation 8.57. la q u antité de inouvem cnl de
1 CM
» • i l’im pulsion lum ineuse est /? = E/c. En supposant que la m asse de la boîte, M. soit a-sse?
! grande pour que sa vitesse de recul .soit non relativiste, on obtient :

- M e + E/c - 0. ( 8 . 66)
Figure 8.19 Knr, sonrre rm rt une im|Xilsinr
vers la (lioilc. Pimr l'oiLserver laquamiié Lorsque rm ip u lsio n luniinciise arrive à re x tre m ite tic la boîte, elle est absorbée, ce qui
de tntmvemem. la boîte Uoii reculer iiTiiiiobiIi.se la boîte.
L’iiiipulsion inuispnne une ma*se A nalysons le m tiuvem ent du cen tre d e m asse L a som m e des fo rces ex térieu res
éiiiiivalcntc m F/r vers la droite, qui s’exercent sur la boîte est nulle. C ela im plique que le centre de m asse ne peut pas
de telle Roitc que le centre de maîsc changer de position (voir le chapitre 9 du volum e 1). C ependant, com m e la boîte recule,
ne change pas <k- position elle transporte de la m asse vers la gauche. Pour co m p en ser ee m ouvem ent, il faut que
Rôvivion et résumé 239

l'im pulsion lum ineuse transporte une certaine m asse iti vers la droite. C ela est |x»ssiblt
s 'il y a une équivalence entre l’ctiergie et la masse.
L 'im pulsion lum inease prend un tem ps U l |X)ur se dcpliieci d ’un tô le à l ’autre de
la boîte. Pendant ce tem ps, la boite recule d 'u n e dtstance = tit = m f J r = I n (M i -)
(com m e le côté droit de la boîte se déplace vcis l ’im pulsion. le tem ps sera légétciiienl
in férieu r à Ut', m ais ecUe d iffé re n te est iicgligcnblc car v -4C c l. Le dcpJaCCtYtcnt dU
centre de mas.se (qui est nul) est donné p ar;

-M d mL = 0
IF
niL = M

L
=*m (i6T)

On trouve alors que l’im pulsion lum ineuse, ayant une énergie F, est éqnivalenle à une
m asse m = F/c'.

S S HI
IL
Le.vpostulats La théorie de la relativité restreinte d'Eiiisiein La contraction des longueurs yuand un observateur mesuie
est füiiiléc sur deux postulats. la longueur d’un objet immobile dans son référentiel inerticl, on parle
de longueur ¡iroprp les observateurs se tivuvuni dans îles léléimlieh
1. Les loi.s de la physique sont les mêmes p».)ur tous les ubservateiirs
inertiels (fill se dépltu enr dans un mnuvemeni relaiifà re réfrientirl et
se trouvant dans de.s référentiels incrtiels. Tous les référentiels
parallèle à la longueur de l'objet mesureront une longueur plus
inerticls sont équivalents.
courte. Pour un observateur se déplaçant a une viie.sse relative r
2, La vitesse de la lumière dans le vide pjissède la meme valeur e la longueur contractée est
dan.s mutes les dimetinns et dans tous les référentiels inertiels.

Im vitesse de la lumière < dans le vide est une vitesse limite qu’aucune L- - /32 (««m acikxi des Inngiiriirsi (8 14)
entité transportant de l’énergie ou de rinfoniiation ne iieul dépas.scr.

Les ctwrdonnées d 'u n événement Trois coordonnées de position La transform ation de Lorentz l-es équaliotis de la tioasfomui-
et une coordonnée de temps situent un événement dans Tespacc- l i o n de loniitT irlir.nt les crxtrdonnécs d'cspacc-temps d'un é ' cncineiii

temps. A l’aide de la ivlalivité restreinte, on peut relier les coordoiutées unique tel que l ’ont perçu des observateurs se Iroiivaiil dans deux
d ’un événement attribuées ptu deux observateurs en mouvement référenitcls inerticls. S et S', où S' se déplace par rapport à i à une
uniforme l’nn par rap|xirt à l’autre. vitesse ë dans la direction de Taxe des.» |Xisitibi l es quatre ixxjrdonnécs
.sont reliées de la manière suivante :
Les événem ents sim ultanés .Si deux ohserv'atenrs sont en
mouvement relatif, ils ne seront généralement pas d’.accord quant à la r' = y ( r - w ) , >''= y, z ' - Z, i ' - y(i - t a / r ‘)
sinuillancitc de deux événements. ,Si Tun d’entre eux observe deux {dquatiorb» de In ininvr^wiiiaiioii de Lorcni/ :
événements .simultanés se produisant à deux endroits différents. viilahles à tome', viievtCN |)t))'SHjiirnirrt pnviîhlc.\ï. (K.21 )
Taiiire ne les observera pas comme étant simultanés, et vice versa
La simultanéité n'est / m i s une notion absolue, mats relative, scion le Les transform ations des vitesses Quand une particule se
mouvcnneni tic Tobsei valeur. La relativité de la simultanéité est une déplace il une vites.se « ' dans la direction de l’axe des .v' positifs
conséquence directe de la vitesse limite finie r. dans un référentiel irK-rticl S' qui sc dcpibcc lui-mcmc à une vitesse v
parallèle à la directktn de l’axe des .r d’un deuxième rclcrc-ntirl inerticl
La dilatation du tem ps Si deux événcnicnis successifs sc S, le module de la vitesse » de la pariknilr iiK-MiriT dans ,S’ est
produisent au même endroit dans un référentiel inerticl, Tintervalle
de temps Afi qui les sépare et qui est mesuré à Tatdc d’une horloge
iviicw rclaiivtsic). (8.29)
unique située à l’endroit où ils sc produisent est l'intervalle de temps 1 -f ii'v/c^
propre entre ces événements. U s obscmatriirs se Irniivtinl dans îles
référentirls en mouvement par rapport au référentiel propre mesureront l ' e f f e t D oppler relativisteSt une source émettant des ondes
kn intervalle de temps plus grand. Pour un obscrvaleui se déplaçant lumineuses de fréq u en te/„ s'éloigne direilcm ent d ’un détecteur
à une vitesse relative v. l’intervalle de temps mesuré est à une vitesse radiale relative v. la fréquence f mesurée par le
détecteur (en fonction du paramètre île viteva- fi — r t c ) est
Ato Ato
M =
V 'i - (v/r)2 x /l I -fi
.f=fo (8.32)
1 s-P'
= yAl(, (diluiaiion du tcmp.s). (8.7 à 8.9)
Si la source se dirige direcinuent vers le dclcctcur. les .signes devant fi
Ici, fi = l’/ i est le p ara m ètre de vitesse et y = l/\ /1 —fi^ est le sont inversés dans Tàpralion 8.32.
facteur de L orenU . La dilatation du temps a comme importante Dans les observations astronomiques, on mesure reffcl Dopplei
conséquenre que le.s horloges en mouvement sont plus lentes quand en longueurs d’ondes. Pc'ur des vitesses très inférieures à c, Téqua-
ellei sont observées par quelqu’un qui est au repos. lion 8 .32 mène à
240 Chapitre 8 U j relativité restreinte

AA P = y n tv (quantité de mouvement). (8.41)


■c. (8.35)
E = mc^ f A' = ymc- (énergie totale), (8.46, 8.47)
où AA est le déplacement Doppler de la longueur d'onde (valeur ahsrriue K = mcHy — \) (énergie cinétique). (8A1)
de la variation de la longueur d'onde) cau.sé par' le mouvement.
Ici. -y est le facteur de Lorentz relatif au mouvement de ht particule,
L ’e ffe t Ihtppler transversal Si le mouvement relatif de la source et mc' est l'équivalence masse-énergie, ou énergie au repos, associée
lumineuse est perpendiculaire à une ligne joignant ectic source et le à la masse de la particule. Ces équations mènent aii,\ relations .suivantes ;
détecteur, la tréqucnce captée par le détecteur est
(pi Ÿ = L- - iKnur. (8.53)

(8.36) »i* E:^ = (pf)' + (8.54)


Quand un système de particules subit une réaction chimique ou
C'el effet Doppler transversal est causé par la dilatation du temps. nucléaire, la valeur Q de la ré.action est l'inverse de la variation
d'énergie au repos totale du système :
I jU quantité de mouvement et l ’énergie Les dcfinilions .suivantes
=• A f ^ - A - f ^ = -A A /f*. (8.49)
de la qu.inlité de mouvement /;. de l'cncrgie cinétique K et de
l'énergie totale h d'une particule de masse m s’appliquent à toute où M, est la masse totale du système avant la réaction et M, est .sa
vitesse physiquement possible : masse totale après la réaction.

Q U E S TIO N S

1. Dans la figure 8.20, le vaisseau A envoie une impulsion laser vers b) En chemin, .Samuel envoie une impiii.sioii lumineuse vers Mars.
un vaisseau B en approche pendant que le vaisseau éclaircui C s’éloigne. Chacun mesure la duree du voyage de cette impulsion Qui me.surc
Les vitesses indiquées sont toutes mesurées dans le même référentiel. une durée propre ?
Classez les vaisseaux selon le module de la vitesse de l'impulsion mesu­ 5. I .a figure 8.22 illustre un vai.sseau (dont le référentiel est S')
rée par chacun d ’eux, en commençant par la valeur la plus élevée. passant près de la Terre (dont le référentiel e.st .V). On tire un proton à
une vites.se proche de celle de la lumière sur la longueur du vaisseau,
de l'avant vers I’am crc. a) Le déplacement Ajt' entre le point
de lancement du proton et son impact est-il positif ou négatif?
b) L'intervalle de temps A/' entre ces événements est il positif ou
0,5r
négatif?

Figure 8.20 Questions I et 7

Z La figure 8.21 a) montre deux horloges dans un référentiel immo­


bile S (elles sont synchronisées dans ce référentiel) et une horloge
qui se iléplace dans le référentiel S’. Les horloges 7/| ci ll[ indiquent
zéro à i'in.siant où elles «c croisent. Quand les horloges W,' et Wj Figure 8.22 Question 5
se croisent, a) laquelle indique la valeur la plus basse et b) laquelle
mc.siire un intervalle de temps propre? 6. a) Dans la figure 8.9. on suppose qu’un observateur sc trouvant daas
le référentiel S' mesure deux événements et cxmstale qu’ils .se prrxluiseni
au même endroit (5 x', par extîinpie), niais non au même instant. Un
observateur se trouvant dans le référentiel 5 pcut-il les considérer
t'omnie se produisant au même endroit ? b) Si deux événements
SC produisent simultanément au même endroit pour un observateur,
SC produisenl-ils simultanément pour tous les autres observateurs?

O : c) Se produisent-ils au même endroit pour tous les autres observateurs ?


7. Les vaisseaux /1 et B de la figure 8.20 sc déplacent direi:iemcni
a) b) Lun vers l'autre; les vitesses indiquées sont unîtes mesurées dans le
Figurt 8 21 Questions 2 cl 3 même référentiel. La vitesse du vaisseau A par rapport au vaissoau B
cst-cllc inférieure à 0.7c, supérieure à 0,7c ou égale à 0,7c ?
3 I A figure 8,21 b) montre deux liorloges dans un référentiel immo­ 8. La figure 8.23 montre l'un des quatre croiseurs spatiaux qui font
bile. S' (ellfs sont synchronisées dans cc icfeicntiel) et une horloge une course. ()uand chaque croiseur passe la ligne de départ, une
qui SC déplace dans le référentiel S. I es horloges H, et //,' indiquent navette le quitte pour voyager vers la ligne d ’arrivée En tant que
■/éiii il riiusiani t)ù elles se croisent. Quand le.s horloges H et / / i se juge de la course, \o u s êtes immobile par rapport aux lignc.s «le
eroisem. tt) laquelle indique la s aleur la plus basse cl b) laquelle départ et d'arrivée. I x;s vitc-sses des croiseurs par rapport à vores et
rac.surc un intervalle de tcni|is propre les vitesses \ „ «les nax ettes par japport à leur croiseur respectif sont,
4. Samuel tniiiie Vénus dans un vaisseau spatial en direction de dans cct ordre. 1) 0.70c et 0.40t ; 2) ü.40c et 0.70r ; 3) 0.2(V et 0.90( .
Mars il croise .Sopliie, qui se iriHis'e sur Terre, à une vitesse relative 4) 0.50« et 0.60c. a) Sans calcul écrit, classez les navettes scion leurs
de 0.5c. a) Chacun mesure la durée du voyage Vénus-Mars. vitesses par rapport à tous, en commençant par celle ayant la vitesse
Qui mesure un temps propre; Samuel, Sophie ou aucun «les deux'’ la plus élevée b) Toujours sans calcul écrit. clas.sez les navettes selon
Exercices et problèmes 241

les üiscariees qiic leurs pilotes mesurent entre la ligne de départ et la 9 À bca-d d’un vaisseau spatial, vous interceptez, des signaux provenant
ligne d arrivée, en commençant par la distance la plas longue, c) Chaque des navettes qui s’éloignent ou qui sc rainmxilicnt directement de vouti.
croiseur envoie un signal à sa navette à une certaine fréquence/(, Les signaux ont la même fréquence propre / q 1.a vitesse (par r a j ^ n
mesurée à bord du croiseur. Toujours» sans calcul éciit, clas.sez les à VÜU.S) lies difféienrcs iiaicttcs sont rcspcctivcnirni iij dr 0.3i- *i r.s
navettes selon les fréquences qu'elles observent, en commençant par vous, b) de 0,(>i' vers vous, c) de 0.3c en s’éloignant de vous et
celle ayant la fréquence la plus élevée. d) de ü.6c en s'éloignant de vous. Classez les naveiies selon les
frequences que vous recevez, en cominonçant pai la imveue ayaui la
plus élevée.
10. L’énergie au repos ei l'éneigie lotaleile inils pai iinili-srxpriiircs
en fonction d line consiaïuv A sont resiiei iiveiiieni 1) A. IA , 7.) A. 3A ;
3) 3A, 4A. Sans calcul écrit, classez ces paiiicules selon a) leiii
ma,ssc, b) leur énergie cinétique, c) leur lacteui' de I nreiir/ ei d) leui
vitesse en commençant, dans chaque cas, par la valeur la plus élevée.
i bigne [.igné
I de départ d'arrivée

Figure 8.23 Que.siion 8

EX ER C IC ES E T P R O B LEM ES

6P. Vous voulez faire iiii voyage aller retour dans un vaisseau spanal
www La solution se taïuve sur le site Web, à l’adresse cÎKlessoii.s :
à partir de la Terre. Vous voulez voyager ii vite.vse constante et en
www.dlcmcgrawhiH.ca/physique
ligne droite cliiram 6 moi.v. puis icveim- vers l.i Terre à lu mênic
vitesse constante. De plus, îi votre retour, vous void»-/ ii Iroiivci la
SECTiOH 8.2 Les postuicts Terre telle qu’elle sera dans 1 000 ans. a) À quelle vitesse devez vous
voyagtT? b) l e fait tie voyager en ligne droite est il important? bi, par
IL Outie le fait qu'il subit l'action de la rotation et de la révolution exemple, vous décriviez un cercle durant une année, retrouveriez-vous
de la l'erre, le référentiel d'un laboratoire n'est pas stnetement inerticl la Terre 1 000 ans plus tai d, scion scs horloges ?
parce qu'une particule qui y serait placée au repos ne resterait
généralement pas au rejxv; : elle tomberait. Cependant, les événements SECTION 8.6 loreiotivitédelaiongueui
se pmduiserl souvent si rapidement qu’on peut ignorer l’accélération
7E. Une tige est parallèle il l'axe des ^ »lu rctcrcnticl .S ; elle se déplacé
gniv ii-ationiielle et considéra- le référentiel comme inertiel. Imaginez,
sur cet axe à une vitesse de 0,630c. Sa longueui au repos est de 1,70 in
par exemple, un électron projeté à une vitesse v 0,992t horizon­
talement dans une salle et parcourant une distance de 20,0 cm. Quelle sera sa longueur mesurée dans le référentiel 5 ?
a) Quelle sera la durée de ce mouvement et b) de quelle hauteur 8f Un électron de p - 0.‘W 987 ,se déplace sur Taxe d'un tul»e à vide
rélcctrou tombera-t il durant cet intervalle? Que fieul-on conclure d’une longueur rie 3.00 ni mc.surcc jMir un observateur en laboratoire 6
sur la fiabilité du laboratoire comme référentiel inertiel dans ce cas ? au refHi.s par rapport au tube. 1lu ob.sa vateur .S' au repos ¡rar rapport
à l'électron verrait toutefois ce tube se déplaçant à une viic.ssc v ( —
2L Quelle fraction de la vitesse de la lumière chacune des vitesses
pr). De quelle longueur serait le tulse pour robservareiir .V'?
suivantes représente-t elle ; en d'autres mots, quel est le ¡varamètre de
vitesse P as,socié à ces différentes situations a) La vitesse moyenne 9f t.:n mètre en bois se Uinivant tlans un rélcrrnticl .9' roriiic un angle
de la denve des continents (3 cm/a). b) La vitesse limite .sur une de 30“ av-ec l'axe des .v' .Si ce référentiel .sc dcplacr parallMcnicnl ii
.luloroiite à КЮ km/h. c) Un avion sujjersonique volant à 1 2СЮkm/h. l'axe de.s t du référeiuiel .S îi une \ he.ssc de 0.9(10» pat rapjxirt ii ce
dl La vitc.s.se de libération d’un projectile de la surface de la Terre dernier, »|uulle est la longueur ajiparenir du mf lie dnn.s .i
(1 1.2 km/s). e) La vitesse de récession d ’un quasar éloigné. 101. Pour un observateur, la longueur mesurée d'un vaisseau spaiial
(3,0 X K f km/.s). est égal»; h 0.5(X) fois sa longueur au rcjros. a) En fonction de c, quelle
esi la vites.se de ce vaisseau spatial par rapport au rcfcrCntiCl dC
robservateur ’ b) De quel facteur les horloges du vaisseau spatial
SECTION 8.5 lo relotivilé du lemps sont elles plus lentes qiu- »elles »In réfcrmiirl de l’obstirvale.iir*’
3 t La durée de vie moyenne mest irée de muons immobiles est de 2,2 / î s. lu . Un vaLsseau spatial d'une longueur propre de I 30 m croise une
La durée de vie moyenne mesurée »le muons se déplaçant à haute •Station de chronométrage à une vitesse de 0.74O( a) Quelle est la
vitesse produits j-iar des rayons » osmiques. est de 16 a»s . Deternunez longueur de ce vaisseau telle qu elle est mesurée par la staïUin ?
la vitesse »les muons créés jiar ces rayons cosmiques par rapport à b) Quel intervalle »le temps I hiirltigc de la station mesurera t elle
la Terre. entre les passages de l'avant e.t de riirrii'n»' tlu vaisseau'’
4f. Quel doit être le paramètre de vitesse /? si le t acteur de Lorentz. y 12P a) Fn principe, une pcrs»»nnc peut-elle sc rendre de la ferre au
est a) 1,01 b> 10.0, c) 100 et »!> 1 000 eeiilrc de la Galaxie (une distance d'environ 23 000 al) dans une tlurée
SP. Une particule instable à haute énergie laisse une traie de I .O*? mm de vie normale ’’ Fxpliquez. votre réponse en aKirdiinf la dilatation du
de longueur sut un détecteur avant de se désintégrer. Sa vitesse par temps ou la coniraeiion de,s longueurs, b) Quelle viles«- constante est
rajipoit au détecteur était de 0,Ч92г. Quelle «tst sa dur<V »le vie propre“’ nécessaire ftoui fniie ce voyage en ^0 a (temps propre) ’
•Л тгетеш dit, combien de temps la particule aurait-elle duré 13P. L'n astronaute quitte la Terre à une vitesse de 0 'Wr vers 1 »9nile
avant de se désintégrer si elle avait été immobile par rapport au Véga, qui se trouve a 26.(' al. Quel intervalle de leinjTs aura passé sur
détecteur ? www les horU'ges de la Terre a) quand le viiyageur atteindra Véga et
242 Chapitre 8 La relativité restreinte

b) qimiKl les observiueurs sc trouvant sur Terre recevront stm message Dans chaque cas, comparez votre réponse avec la prédiction de
disant qu’il est arrivé c) De combien d’années l’astronaute aura-t-il l’équation de transformation galiléenne des vite.sses.
vieilli (en fonction de son référentiel) quand il atteindra Véga '? www 23E La galaxie A s’éloigne de la Tcitc à une vitesse nx:suréc de 0,35c.
La galaxie B, située exactement dans la direction opposée, s ’éloigne
SECTION 8.8 QucTquot conséquentes de lo tronsformotion de Lorentz également de la Terre à la même vitesse. Quels modules de vites.se un
observateur sc trouvant dans la galaxie A mesurerait-il a) relativement
14E. L'observateur S rapporte qu’un événement s’est produit sur Taxe
à notre Galaxie et b) relativement à la galaxie B '?
des-f de srtn référentiel à .r = 3.ü() x 10* ni au temps i = 2.50 s.
a) Une observatrice S' et son référentiel se déplacent dans la direction 24E On déduit du déplacement vers le rouge de la lumière émise par le
de l’axe des x positifs à une vitesse de 0,4(X)c. De plus. ,v = .v' = 0 quasar (T, que celui-ci s'éloigne de la Terre à une vitesse de ().800c.
k l = t' = 0 , Quelles sont les coordonn&'sde l’cvénement selon l’ob­ Le quasar Q2 . qui se trouve plus près de la Terre d;ins la même direction,
servatrice S' '? b) Quelles .seraient ces coordonnées .si l’ob-scrvatrice S' s’éloigne à une vitesse de ü.400c. Pour un observateur se trouvant
se dépl<ivait à la même viie.sse dans la direction né^otirr de l’axe des .x ? sur U ,. à quelle vitesse Q, s’cloignera-il 7
15E. L’observateur S aili ibiie les coordonnées d ’espacc-tcanps 25P. Un vaisseau spatial dont la longueur au rcfios est de 3.50 m ne
déplace à une vitesse de 0,82c par rapport à un certain référentiel.
x = l(X)km et / =*= ’0 0 /ts Line micrométéorite, voyageant également à une vitesse de 0,82c
dans ce référentiel, mais dans l'autre sens, envise le vaisseau. Combien
à un événement. Quelles sont les coordonnées de cct evénemeni dans
de temps, mesuré à partir du vaisseau, faut il à cct objet pour traverser
le référcnlicl S', qui se déplace dans la direction de l';uc des x positifs
la longueur du vaisseau '? www
à une vitesse de O.VSOr par rapport à .V’’ Supposez que x = x' = V
26P. Une flotte de vais.seaux spatiaux longue de 1,00 al (dans son
à t = /' “ 0.
référentiel propre) se déphec à une vitesse de O.SOOr- par lupport à
léf. Le référentiel inertiel S' se déplace à une vitesse de 0.60c par
la station terrestre S. Un messager voyage de la queue de la flotte
rapport au rctéreniiet .V(tlgurc 8.9). De plus, x = x' = 0 à / = /’ = 0.
vers l’avant il une vitesse de 0,950< par lapport à S. Combien de
On rapporte deux événements. Dans le référentiel S. l’événement I
temps lui faut-il pour atteindre l’avant tel qu’il est mesuré dans a) son
se produit à l’origine à r = 0 et révénenieni 2 se produit sur Taxe
référentiel, b) dans le référentiel de la Hotte et c) dans le référentiel
des r à .r = 3.0 km et à f = 4,0 ^s. A quels temps l'observateur S'
d’un observateur se trouvant dans 5”?
rapporte-t-il ces mêmes événements ? Expliquez la différence dans
l’ordre des temps.
SECTION 8.T 0 L’eTfel Doppler relativiste
17L Un expérimentateur déclenche deux clignotants simultanément
et produit un gros éclair i l’origine de son référentiel, et un petit 27E. Un vais.seau spatial s'éloignant de la Terre à une vitesse de 0.900c
éclair à ,r = 30,0 km. Une observatrice sc déplaçant à une vitesse transmet un rapport à une fréquence (mesurée dans le réferentiel du
de 0.250c dans la direction de l’axe des .v positifs observe les éclairs. vaisseau) de 100 MHz. A quelle fréquence doit-on régler le récepteur
a) Quel est l’inlervallc de temps entre eux. scion elle ’? b) Quel éclair sur Terre pour capter le rapport
se produit le premier, scltm clic ’? www 28E. La figure 8.24 est un grapliiquc représentant l’intensité, en tonciiwi
18P. Un observateur S voit un gros éclair liiniinciix à 1 2(X) m de lui de la longueur d’onde, de la lumière atteignant la Terre en provenance
et un petit éclair lumineux à 720 m plus |->rés de lui, directement dans de la gala.xie NGC 7319, qui sc trouve à envirtm 3 x 1 0 * années-
l’axe du grtxs éclair. Il détermine que l’intervalle de temps entre les lumière La lumière la plus intense est émise par l’oxygène se trouvant
éclairs e.st de 5,(X1 fisel que le gtt>s éclair arrive le premier, a) Quelle dans NGC 7319, En laboratoire, cette émission possède une longueur
est la vitesse relative v (donne/, le module et la direction) d’un second d'onde de À — 513 nm mais, dans la lumière nous parvenant de la
observateur S' pour qui ces éclairs se prwluisenl au même endroit galaxie, clic est décalée à 525 nm par effet Doppler (toutes les
dans le lél'ércntiel .S’"’ b) Du point de vue de S', quel éclair se produit émissions provenant de NCiC 7319 sont décalées), a) Quelle est la
le premier’?c) Quel intervalle de temps ,5' rncsure-i-il entre les deux? vitesse radiale de NGC 7319 par rapport à la Terre ’’ b) La galaxie
19P Une horloge se déplace sur l’axe des t à une vitesse de 0,6(X>c s’ëloignc-t-cllc ou s’approchc-t-elle de notre planète ?
Cl indique zéro en passant à rorigiiie. a) Calculez son facteur de
Lorenifc. b) Quel temps indique-t clic en passant à » — 180 m ?
«<ю
20P L’observateur S du problème 18 voit les deux éclairs aux mêmes 1- ДЗ.-^ + 12 nm \ ’(iC. 78И1
endroits qu’avant, mais ils .sont maintenant plus rapprochés dans
le temps. Quel intervalle de temps les sépare dans le léfcrcniiel 5 /
5 40n l.oiigiu-iir
[xiur qu'on puisse encore trouver un référentiel S' dans lequel ils se M d ’oililf I
produisent au même endroit ? e n lalH iiatoirc'
c
2ÎHi
SECTION 89 La tionsTormotlon refctiviste des vitesses

?lf l'o r p:micule se déplace sur Taxe des .v' d’un rétérenticl .V' à une 0
vitesse de 0.4Mi. Le léféieiitiel S' se déplace à une vites.se de 0.60c 400 450 5fi0 m i) or»o 700 750
par rapptirl .111 rcférciitiel ,V. Quelle est la vitesse de la particule dans l.r>ngiictii'«l'ondr (n u l)

le réferentiel S ! Fi{|uri8.24 Exercice 28


22£. Le rétcrcnttcl .V'sc déplace n 0.62c dans la direction de l’axe
des X lai.sitifs imr rdp|aiit mi rcféirniirl 5. Dans le référrnticl S', une 29E On observe que certaines longueurs d'ondes contenues dans la
nurliciilc possède une vitesse inesiirée de 0.47c dan.s la direction lumière émise par une galaxie'de la constellation de la Vierge sont
laxsilivc de .v' al O iidlr t si la viiesse de la panicule par rapport à S? 0,4 9f- plus longues que celles de ki lumière correspondante proveiituit
b) Quelle .semit la vitesse de hi paiticulc pai rajpurt à S si elle voyageait de sources terrestres. Quelle est la vitesse radiale de cette galaxie pat
(à 0,47r) dans 1« direction néfiarive de x' dans le référentiel S”? rapport à la ferre ? S’appiTK-he-t-elIe ou s’éloigne-l-elle de la T’e rre’.’
Exercices et problèmes 243

30t. En supposant que l'équatiun 8.35 s’applique, déterminez la 43P Un cachet d’aspirine de 5 grains a une masse de 320 mg. Sur
viics.se à laqiidlc vous tlevne/: vous déplacer vers une lumière rouge txxnhieri de kilomètres réncigie équivalant à cctic masse ¡icuixllc ftiiir
pour qu'elle vous paraisse verte. Prenez 620 nm comme longueur rouler une automobile? ¡supposez une conRommation de 12,75 üm/L
d ’ontlc ik- la lumière rouge et 540 nm'comme longueur d'onde de la et une chaleur de combustion de 3.65 x 10’ J/L |юш Гсл.1спсс uiiliacc
lumière verte. dans Г automobile.
31P. Un vaisseau spatial s’éloigne de la Terre à une vitesse de 0,20r. 44P I.a durée de vie moyenne des muons uu repos est de 2.20 /xs-
Une source lumineuse située à l’arrière du vais.scau .semble bleue Une mesure effectuée en labiiratoirc sur des muons se déplaçant dims:
(Á, — 450 nm) aux passagers du vaisseau. De quelle couleur cette un faisceau émergeant d 'un accélérateur de pam euies donne iinu
souiec serait clic pour un observateur la surveillant de la Terre www durée de vie moyenne de 6.90 us. Quelles sont a) lu vitesse île eec
muon.s dans le laboratoire, b) leur énergie cinctiquc et e I leur quumiié
SECTION 8.12 l'énergie en relolivite de mouvement? La masse d’un muon représente 207 IoIr celle d un
électron.
32C. Q ir-1travail faudrait-il effectuer pour que la vitesse d’un électron
au rc|X)s passe à a) 0.5(k’, b) 0,990c et c) 0,99*> Or ? 45P l^or.s d’une collision й baiile cnrigîr entre un rayon i:osmiqiic cl
une particule située dans la cmiclie .supciicurc de l'atmospiicn. tci iiciirr.
331. Déterminez le paramètre de vitesse ^ et le facteur de Ixircniz y
à I2Ü km au-dessus du niveau de la nier ¡1 y a procliirtioii d’un piiir
pour un électron ayant une énergie cinétique de a) 1.00 keV,
charge. Le pion possède une énergie loialc F lic 1.35 л 10’ Vfi.V ci
bl I.ÜÜMeVetc) 1,00GcV.
se déplace s'erticaleiiieiii veis la .чшГасс de U Tcric. D<ius le léféirniicl
•34f Déterminez le paramètre de vitesse p et le facteur de Lorentz y propre du pion, celui-ci se désiuiègre 35,0 us ajirès sa ciéation A quelle
p«»ur line particule ayant une énergie cinétique de 10,0 MeV si cette altitude au-dessus du niveau île la nier iiicsurée dans к référentiel tic
pailicule est a) un éleciron, b) un proton et c) une particule alpha la Terre, cette désintégration se prodiiii-cllc 1,'éncrgic au repos d ’im
ayant une mas.se de 4.002 6 u. pion est 139,6 MeV. www
351. En fonction de c, c|ucllc est la vite.s.se d’un clcctnin dont l’énergie 46P. Dans la section 8.5 du volume 2. on a dcmonlrc qu’une particule
cinétique est I0Í) M eV’’ de charge q et de masse m se déplaçant à une vilcs.se c ixMiKiidiculaite
36f Dans la réaction suivante ; à un champ magnétique uniforme Й dcciii un icri le de laynit г
déterminé par l'éiiuation 8 16:
P 4 '^F -> a H ''D .

les valeurs des masses sont


qH
nKp) = 1,007 825 U, mia) - 4,(ï)2 603 u.
mff-) = 18,998 405 u, m (0) ^ 15.994 915 u. Il a égalenKni été démontré que la période T du inouvement circulaire
est indépendante rie la vitc.sse de la particule, C’es résultaus sont
Calculez la valeur Q de la réaction en fonction de ces données. valides seulement si v <K c. Dans le ras de particidt-s voyageant à
3/P On croit que les quasars sont des noyaux de galaxies actives dans des vitesses .supérieures, on doit déterminer le rayon de la uaiecioire
les premiers stades de leur formation. Un quasar typique rayonne circulaire à l’aide de l’équation suivante :
son énergie à une puissance de 10"“ W. A quel taux la masse de
ynw
cc quasar dinnnuc-t-cllc pour fournir cette énergie? Exprimez votre
réponse en masses solaires par année, une unité, de masse solaire qR qB
qRyJ\
(I um.s = 2,0 X 10’’” kg) étant la masse de notre Soleil. Celte équation est valable pour touies les vitesses jiliysiqiienienr fiossiblcs.
38P. Quel travail faut-il effectuer pour augmenter la vitesse d’un Calculez le rayon de la trajectoire d'un électron de 10,0 MeV se dépla­
électron a) de 0 ,18c à 0,19c et b) de 0,98c à 0.99c? Notez que la çant pcrpcndiculaireincnl à un champ magnétique unitorttic dc 2,20 1.
vitesse augmente de 0,01c dans les deux cas. à l’aide des formules a) non relativiste et b) relativiste. C) L alculez la
39.1’ (ific ccriaine parciculc de tnas.se m possède une quantité de période T - 2 jrr/v du nxiuvcmeiit ciiculairc à l’uitlc dc la foi mulc rclati
inoiivenient me. Quels sont a) sa vitesse, b) son facteur de Loreniz visic pour t . I.C résultat csi-il indépendant dc la vile.s.sc dc l’elcctron V
et Cl son énergie cinétique ? 47P. Les mesures de iionisation révèlent qu iine cmaint» iiariifulc
40P. Quelle est la vitesse d ’une panicnie a) dont l’énergie cinétique nucléaire dc faible masse po.sscdc une ebnrge de -I-2 p ei qn'ellc su
represente deux fois son énergie au repos et b) dont l’énergie totale déplace a une vitesse de 0,7tOr. Le rayon de eourhure de sa iiajei:-
représente deux fois .son énergie au repos toirc daas un chatnp magnétique dc 1,00 f est de 6,28 m U 'J ira jeeioiie
41P. Quelle doit être la quantité de mouvement d ’une particule de est un cercle donl le plan est perpendiculaire au champ magnétique )
masse m pour que .son énergie totale représente trois fois .son énergie D éterm ine/ la nias.se de la particule et identifiez la. {Indice: les
au repos '' www particules nucléaires de faible masse sont composées dc ncutron.s
(qui n'ont aucune charge] et dc protons [charge = -t-e|, en nombres
42P. a) Si l’énergie cinétique K et la quantité de mouvement /)
d'u n e particule peuvent être mesurées, il devrait être pos.sible de grossièrement égaux. Considéiez, que la masse de chacune dc cc.s
particules e«i 1 00 ii. Voyez aussi le problème 46.)
déterminer la masse m de la particule, donc d’identifier celte
dernière. Démontrez que 48P. Un proton de 10 (icV (un rayon cosmique) sc déplacé dans le
champ magnétique de la Terre R. dans une région où 1« modnlc
- A’5 du champ moyen est 55 //T Sa vitesse v est perpriuliailaiie à R Quel
m --------- :— ■
2Kc- est le rayon de la trajm nire im urvée du promu dans cette région'’
(Voir le problème 46.)
b) Démontrez que celle expression se réduit à un résultat attendu
lorsque u!c 0. où u est la vitesse de la particule, c) Déterminez la 49P I )n électron de 2.50 MeV ,se déplace |icr|Tcndi( ulaircmcnl n un
masse d'une particule ayant une énergie cinétique de 55.0 MeV et champ magnétique en suivant une trajectoire dimi k rayon de coiiduiir
une quantité de mouvement de 121 MeV/c. Expinmez votre rcpon.se est de 3.0 cm. Quel est le rmxtule du champ magiiriiqiir R (Voir le
en fonction de la masse de l'électron, problème 46.)
244 Chapitre 8 La relativité restreinte

50P. 1^ .syiichroti'on à firoions du Fcrmilab (près de CTiicago) accélère Ici, il faut tenir compte de deux événements, événement I : quand
les protons jusqu'à une éiieigie cinétique de 500 GeV. A une telle le pare-chocs arrière passe la porte avant, celle-ci se ferme. Attribuons
énergie, les effets relmivistes .sont importants: notamment, quand la valeur zéro au temps de cet événement pour monsieur Voiture et
la vitesse du prtiton augmente, le temps qu'il lui faut pour faire monsieur Gtuage : = Li = 0. L’évciicmcni sc produit à .r, = .Kj. = 0.
le tour de sa trajectoire circulaire daas le sync+itoüon atigmentc aussi. La figure 8.25 b) illustre révéncnieni I par rapport au référentiel x^.
Dans un cyclotron, où le module du champ magnétique et la Événement 2: quand le parc-choe.s avant atteint la porte arrière, cette
porte s'ouvre. La figure 8.25 c) illustre révéncm eni 2 par rapport
fréquence de l'oscillateur sont fixes, cet effet de dilatation du temps
au référentiel x^.
dcsyiiclironiscra la fréquence du proton avec celle de roscillaieur.
.Selon immsicur Gaiage, a) quelle est la longueur de In limousine
C:e phéntmiêne élimine l'accélération répétée; de plus, le proton
et b) quelles sont les coordonnées U’cspace-icmp.s et ipj de
n'aiieiiKlra pas une énergie aussi grande que 500 GeV. Cependant, dans
révénem ent 2 '.’ e) Pendant combien de temps la limousine est-elle
un synchrotron. le module du champ magnétique et la fréquence « comcmie » dans le garage quand ie.s deux portes sont fermées ?
d'o.seillation varient toutes deux pour permettre la variation causée par Observe/ maintenant la situation du référentiel .x. Dans celui-ci, le
la dilaluliun du temps. garage passe autour de la limousine à une vitesse de —0.998 Oc.
A une énergie de 500 GcV. calcule? a) Je lacteur de Loreni/., Selon monsieur Voiture, d) quelle est la longut'ur du garage qui pa.ssc
h) le paramètre de vitcs.se et c) le module du champ magnétique si cl e) quelles sont les coordonnées d ’espace-temps .t,.2 et f,2 de
l'orhite du proton possédé un rayon de courbure de 750 m. (Voir le l'événeineni 2 f) La limou.sine s’est-ellc déjà trouvée dans le garage
priililèmc 4éi. iiiiliscx 9.5S.3 MeV comme énergie au repos du proton.) pendant que ses deux portes éiaienl fermées ? g) CJuel événement
5IP* Une particule alpha possédant une énergie cinétique de se produit le prcniiei h) De.ssinez les cvéneincnts 1 et 2 tels qu'ils
7.70 W 'V heurte im noyau de '■*N .au repos ; tous deux se transfor­ sont observés par monsieur Voiture. (Les événements ont-ils un hen
ment alors en un noyau île ' 'O et en un proton. Le proton est projeté de cause à effet? Auirenicnl dit. est-ce que l'un provoque l'autre ?)
à 90“ de la direction tic la particule alpha incidente et il possède i) Finalement, qui remporte le pari '?
une énergie cinétique de 4.44 MeV. Voici les masses des différentes 53. Jets superiwniiieux. l-a ligure 8.26 a) montre la trajectoire suivie
particules particule alpha : 4.002 60 U ; '■'N 14.003 07 u ; proton : par une perturbation dans un jet de gaz ionisé expulsé d'une galaxie.
1.007 825 U ; * '0 : 16.999 14 u. Exprimées en MeV, quelles sont La perturbation .se déplace à vitesse constante ? et à un angle 0 par
a) l'énergie cinétique du noyau d'oxygène et b) la valeur Q de la rapport à la direction de la Terre. A l'occasion, la perluihaiion émet
réaction ? {hulire : les vitesses des particules sont très inférieures à c.) un éclair hiinineux capté sur Terre. La figure 8.26 illustre deux éclairs,
séparés par un temps t mesuré dans un référentiel immobile situé près
Problèmes supplémentaires des éclairs. La figure montre les éclairs comme s’ils étaient photo­
graphiés .sur la même pellicule, d’abord quand la lumière de l'édair 1
S2. Le problème de la voiture dans le narage. Monsieur Voiture vient a atteint la Terre et, plus tard, quand la lumière de l’cclair 2 l'a atteinte à
d'acheter la plus grande limousine du momie, dont la longueur propre son tour. La flistancc apparente parcourue par la penurbation entre
est L , - 30,5 ni. Dans la ligure 8.25 a), la limousine est stationnée les deux éclairs est la distance du point de vue d’un observateur
devant un garage dont la longueur propre est = 6,00 nv Le garage terrestre de la trajectoire de la perturbation. Le temps apparent
a une porte avant (ouverte dan.s rilhisirationi et une porte arrière entre les deux éclairs est la différence entre les temps d'arrivée de
(fermée dans rilliistralioii). La liiiKmsine est évideniincni plus longue leurs lumières. La vitesse apparente de la perturbation est donc
que le garage. Q u'à cela ne tienne, monsieur Gmage, à qui appartient = D ^ I T ^ . En fonction de v, t et 0. quelles sont les valeurs de
le garage et qui connaît la contraction de longueur relativiste, parie a) Djpp et b) T.pp? c) Évalue/ quand v = Ü.980î et 8 = 30,0“.
avec monsieur Voiture que la lirntiusine peut prendre plate dan.s le La première fois qu'on a observé des jets supcrlumineux (plus rapides
garage même si .ses deux ptnlcs sont fennées. Monsieur Voiture, qui a que la lumière), ceux-ci semblaient défier la rolativiic restreinte -
abandonné ses cours de physique avant d'éiudicr la relativité restreinte, du moins jii,sc|u'ù ce qu'on comprenne la géométrie de l'observation,
affirme tiu'unc telle chose, même en principe, est impossible. comme l'illustre la figure 8.26.
Pour analyser la situation de monsieur Garage, on fixe un axe
des V.. à la linxiusine, r,. = 0 sc trouvant au pare-chocs arrière, et un
axe des jr^ au garage, = 0 sc trouvant à la p<Hle avant (actuellement
ouverte). Mim.sieur Voiture doit alors conduire la limousine directement
vers la porte avant du garage à une viies.se de 0.998 Or (ce qui e st bien
sûr. techniquement et Rnancièrement im|X)ssible). Monsieur Voiture
est inunobilc {¡ans le référentiel : monsieur Garage est immobile
dans le référentiel Xj.

------- - 1« TK

a)
•>pp

0
h
Kl lair I
Y
K riair 2
6) r) b)
Rgure 8.25 Problème 52 figure 8.7é Problème 53
9 Les photons
et les ondes
de matière

Cette image révèle des traces de minuscules bulles de vapeur dans une chambre à bulles, elle révèle les traiectoires suivies
por des électrons (traces colorées en vert) et des positrons (rouge). Un rayon gammo (qui ne laisse aucune trace en pénétrant
dans la partie supérieure) éjecte un électron d'un des otomes d'hydrogène qui remplissent la chambre, et se transforme alors
en une poire électron positron.
Un autre rayon gamma
se transforme en une seconde
paire, plus bas. Ces traces
(incurvées à cause d'un champ
magnétique) montrent
clairement que les électrons
et les positrons sont des particules
qui suivent des trajectoires
étroites. Néanmoins, on peut
aussi considérer ces particules
comme des ondes.
246 Chapitre 9 Les photons et les ondes de matière

9.1 Une nouvelle direction


L ’élu d e de la th é o rie d e la rela tiv ité d ’E in slcin vous a fa it d éc o u v rir un m o nd e qui
dép asse l’o rd in aire, so it le m onde d es o h jc ls qui se d ép lacen t à d es v itesses p roches
de celle de la lum ière. Entre autrc.s suipri.se.s, la théorie d ’E instein m ène à la prédiction
que l ’éc o u le m en t du tenip.s m esu ré p a r une h o rlo g e d ép e n d de la vile.sse à laq u elle
elle-m ê m e se d ép lac e par rap p o rt à la p e rso n n e q u i l ’o b se rv e : p lu s e lle .se d ép lace
rapidem ent, plus elle to u rn e len tem en t. Ju sq u ’à m ain ten an t, cette p réd ictio n et bien
d 'au tres ont été eo n lirm ées dans tous les lests expérim entaux ; de plus, la théorie de la
relativité perm et de com prendre davantage la nature de l’espace e t du tem ps.
Vous êtes m a in te n an t su r le p o in t de d é c o u v rir un n o u v eau m o n d e h o rs de
votre e x p é rien c e q u o tid ie n n e - le m o n d e su b ato m i(|u c. V«ius ferez d e su rp re n an tes
decouvertes qui, bien q u ’elles puissent parfois sem bler bizarres, ont perm is aux physiciens
d ’approfondir, étape par étape, leur connaissance de la réalité.
La p h y siq u e q u a n tiq iie , votre nouveau sujet d ’étu d e, am ène des réponses à des
questions com m e les suivantes : pouiquoi les étoiles brillent-elles ? Pourquoi les éléments
sont-ils ordonnés ainsi dans le tableau ¡périodique? C om m ent les lasers fonclionnciU-ils?
Q uels sont hrs eonsiituants de la m atière et com m ent inieragissenl-ils ? E tant donné que
la physique quanlique fournit une ex p liau io n à toute la chim ie, y com pris à la bi(x;himie,
on dt)it la com piendre p o u r com prendre la vie elle-m êm e.
C ertain es d es p réd ictio n s q ue p erm et la p h y siq u e q u an tiq u e p araissen t étranges
m ême aux yeux des physiciens et des ¡philosophes qui en étudient les bases. D e multiples
ex|péricnces ont dém ontré le bien-fondé de cette théorie, et beaucoup en ont révélé des
aspects très étranges. Le m onde quantique est un parc d ’attractions rem pli de m anèges
m erveilleux qui éJpranleront le .sens com m un que vous avez acquis depuis votre enfance.
On com m encera rcx p lo ratio n de ce parc quanlique par l’étude du photon.

9.2 Le photon. quantum de lumière


Ixi physique q u a n tiq u e (aus.si appelée mécanique quantiqite et théorie quanlique) est, dans
une large m esure, l’étu d e du m onde m icro sco p iq u e. D an s ce m onde, de nom breuses
grandeurs n’existent qu’en certaines quantités m inim ales (élémentaires), ou en m ultiples
entiers de ces quantités élém entaires ; on dit alors q u ’elles sont quantifiées. L a quantité
élém e n ta ire asscK-iée à une telle g ran d eu r e st ap p elée q u a n tu m (p lu riel ; quanta) de
cette grandeur.
En un certain sens, on peut dire qu e la devise mipnétaire canadienne est quantifiée
p arce q u ’il y a une p ièce ay a n t la v aleu r m in im ale (la p ièce de 1 c e n t par ex em p le)
e t que les valeurs de tous les autres billets et pièces sont des m ultiples de cette valeur
minimale. Autrement dit, le quantum de la dcvi.se canadienne est 0,01 S et toutes les valeurs
sujiérieures ont la form e «(0,01 $), où n est un nom bre entier positif. P ar exem ple, vous
ne pouvez donner à qui que ce soit 0 ,7 5 5 $ = 75.5(0,01 $).
En l ‘)()5, E instein suggéra que le rayonnem ent électrom agnétique (ou, sim plem ent,
la lutniéte) est quantifié et existe en quantités minimales (quanta) q u ’on ai^pelle aujourd'hui
phutuns. 11 est possible que cette suggestion vous semble étrange parce que. dans les der­
niers chapitres, vous avez étudié l’idée classique que la lum ière e.si une onde sinusoïdale,
possédant une longueur d ’onde À, une fré q u e n c e /e t une vitesse (dans le vide) t dont la
relation est

/ - (9.11
A

D e p lu s, dans le ch a p itre 4, on a étu d ié l'o n d e lu m in eu se c lassiq u e co m m e é ta n t la


aim b in aiso n d 'u n cham p électrique et d ’un cham p m agnétique, chacun o.scillanl à une
fré q u en c e/, ro in in e n t celle o nde de cham ps o.scillants peut-elle être constituée d ’une
q u an tité élém e n ta ire de q u elq u e c h o se - le q u an tu m de lu m iè re ? Q u ’cst-c e q u ’un
photon exactem ent ?
l e concept de quantum de luiniète. ou de phonfn, s'av è re beaucoup plus subtil et
mystérieux que ne l’a imagim' Ein.stein. Ln effet, il est encore très peu compris. ( e manuel
n’abordera que ce rtain es d es notions de base relativ es au p h o to n , p rin cip alem en t en
fonction de ce q u 'a suggéré Einstein.
9.2 Le photon, quantom d e lumiOro 247

S cion la suggestion d 'E in ste in , le quantum d 'u n e o nde lum ineuse d e IrtHjueiice j
pos.sèdc une énergie

£ ~ hf (l’cncreic d’un photonl. ip i)

Ici. h est la c o n s ta n te d e P la n c k , doni la valeur est

A = 6 .6 3 x 1 0 s = 4 , I 4 x Jü ‘^ e V -s . fQ ’ )

L 'é n e rg ie m in im ale q u 'u n e o n d e lu m in eu se d e fréq u en ce / p eu t posséder CSl hf,


soit l’énergie d 'u n plHtum unique Si l'o n d e fn>s\èile plus d 'én erg ie, son énergie totale
doil être un m ultiple entier d e hf, tout com m e la devise de l'ex em p le prci edeni ilml êlrc
un m ultiple entier de 0.01 $. La lum ière ne peut avoir une énergie d e nu de 7S.S///'
E instein a aussi avancé que. lorsque la lum ière est absorbée ou ém ise par un objei
(par la m atière), l'absorption ou l'ém issio n sc produit au niveau des atomes de CCI objet.
Q uand un atom e absorbe une lum ière tic fréquence J. l'cn crg ic hf d un photon est trans
féréc de la lum ière à l'ato m e, l e plurton disparaît alors, et on parle d'absorption. Par
contre, quand un atom e ém et une lum ière de fré t|iie n c c / il y a transfert d ’cncigic lij de
l'ato m e à la lum ière. Q uand il y a émission, un photon apparaît soiidainem eni, ém is pai
l ’atom e. D onc, il p eu t y av o ir absorption de pboton.s et émission de phoions p ar les
atom es d ’un objet.
D ans le cas d ’un objet constitué de iiom bieiix alonK.'s, d peut y avoir de nom breuses
absorptions de photons (com m e dans un verre fum é) ou émî.ssious de phtaoii.s (com m e
dans une am poule électrique) Cc|K-iulanl. chaque absorption ou ém ission met toujours
e n je u le transfert d 'u n e énergie égale à celle d ’un photou unique d e lumière
D ans les c h a p itre s p réc éd en ts, les ex e m p le s qui tra ita ie n t de 1 ab so rp tio n ou
de l'é m issio n de lum ière co n cern aien t tellem ent de lum ière q u 'o n n ’avait pas besoin
de faire appel à la physique quaiiliquc. de sorte q u ’on a pu travailler avec la phy.siqiic
c lassiq u e. C e p en d a n t, à la fin du x x ' siècle, la te ch n o lo g ie a p e rm is de réa lise r
des expériences avec des photons uniques et d’en faire une uiilisaiion pi ati(|uc Depuis
la p h y siq u e q u an tiq u e fait p artie de l ’in g én ierie, n o tam m en t en g én ie o p tiq u e Cl en
pholoniquc.

✓ v é r if ie z VOS CONNAISSANCES! Classez les rayonnements siiit^aniv selon les énergies


des photons qui y sont assiKiés, en commençant par le rayonnement ayant la valeui ilVnergie
la plus élevée ; a) uiic lumière jaune émise par nue lampe à vapeur de sodium, bl un ravon
gamma émis par un noyau radioactif, c) une onde radio émise par l'antcnnc d'une station
de radio commerciale, d) un faisceau de nücro-nndes émis par un radar dr t onli'ôlc de
la circulation aérienne

Exemple 91
On installe une lampe à vapeur de sodium au centre d'une grosse On a alors, en fonction do I équatirin 9.2 (£ - hf).
sphère qui absorbe toute la lumière qui l’atteint. Le taux auquel
l.a lamix: émet l’énergie est HMt \V ; supposez que l’émission se fait
uniquement à une longueur d’onde de .SqO nm. À quel taux la sphère hf
absorbe t elle les photons’’
Si on remplace/ par l’exprcs-sion ilc l’cquation 9.1 {J - d k ) et qu'on
SOLUIION. On supim.sc que toute la lumière émise par la lampe atteint insère les doiuiées connues, on obtient
la sphère (donc qu’elle y est absorbée), la; toncepl clé est que la lumière
est émise et absorbée sous forme de photons. Le taux ii auquel les
photons sont absorbés par la sphère est égal au taux ii^ „ auquel les ” ~ lu-
plKMons sont émis par la lampe. Ce taux est (lOOWg-tOn X lO-'^iii)
taux d'émission d’énergie “ (6.fi3 X 10 ’•* J SX2.998 X 10* m/s)
énergie par photon émis li = 2,97 X K)'20 .plwions/.s. (réponse)
248 Chapitre 9 Les photons et tes ondes de matière

9.3 Le rayonnement du corps noir


Le concept des quanta a etc introduit en physique lors de l'élu d e du rayonnem ent ém is
p ar les o b jets chauds. C'ctlc étu d e a co m m en cé au m ilieu du x ix ‘ siècle et a m ené en
1905 à l'introductio n du concept d e quantum d e luinicrc (q u 'o n appelle aujo u rd ’hui le
photon) par A lbert E instein. D ans cette section, nous revoyons ce développem ent, qui
s ’est déroulé en m êm e tem p s que la th erm odynam ique s ’e st établie. En fait, ces deux
sujets étaient liés de très près.
I.d iijju rn r <r<m<!r (/uni) N ous voyons la plupart des objets par la lum ière q u ’ils réfléchissent. C ependant,
Figure 9.1 Un résultat de la mesure les objets émettent aussi du rayonnem ent électrom agnétique et, si leur tem pérature est
de la radiance spcclialc pour un assez élevée, nous p o u v o ir voir, ou bien délecter, ce rayonnem ent ém is. Par exem ple,
intervalle de longueurs d'onde à deux la sen satio n de c h a le u r q u ’on ressen t lo rsq u ’on ap p ro ch e le s m ain s d 'u n e am poule
températures diflcrenres. Le pic élec triq u e ou d ’un feu e st p ro d u ite p ar le ra y o n n e m en t in fra ro u g e q u ’é m c ltc n t ces
sc déplace vers les longueurs d’onde objets. L e rayonnem ent ém is par un corps à cause d e sa leni|)ératurc est appelé rayon*
plus courtes lorsque la lenipératiire n e m e n t th e rm iq u e . C e rayonnem ent a une distribution spectrale continue.
augmente L es chercheurs du x ix ' siècle découvrirent deux propriétés im portantes du rayonne­
ment therm ique ;

À mesure que la lempéiature augmente, l'inieasité du rayonnement émis par un corps


augmente i-apidcinenl.

Plus la température du corps est élevée, plus la longueur d'onde associée au maximiini
de la di.stribulioii du rayonnement est courte.

Les chercheurs curent beaucxmp de difficulté à cxprim ei ces résultats de façon quantitative
ca r le rayonnem ent therm ique d 'u n corps à une certaine tem pérature dépend de la com ­
p o sitio n de l'o b je t et d e l ’état de sa su rface. A u cu n e loi fo n d am e n tale ne se m b lait
ém erger des données, si différentes d 'u n cotps à l’autre.
La solution à cette difficulté a été proposée par Gustav K irchhoff en 1859. Il a montre
que le rayonnem ent ém is par les ob jets qui absorbent tout le rayonnem ent incident ne
dépend pas de la nature de l'objet, ü n objet qui absorbe tout ic rayonnem ent incident est
appelé c o rp s n o ir ca r à des tem pératures faibles, il est noir (il absorbe la lum ière inci­
dente et son rayonnem ent ém is n ’est pas visible ). Le noir de carbone est un exem ple de
corps noir. K irchhoff a indique com m ent construire un corps noir. Il s ’agit de form er une
cavité dans un corps solide dont les parois sont gardées à une tem pérature constante et
uniform e T e l de percer un petit trou dans l'u n e de scs parois. L ’ouverture est un coqjs
noir car la lum ière qui entrera dims la cavité sera absoibée par les parois de celle-ci. Le
ray o n n e m en t qui so rtira par r o u v e r tu r e sera u n iq u e m en t le ray o n n e m en t ém is p ar
le corps noir. (Pour les m êm es raisons, la pupille d 'u n œ il paraît noire, m êm e si c 'e st en
lait une ouverture). L a figure 9.1 illustre la distribution spectrale du rayonnem ent ém is
par un corps noir, pour deux tem pératures différentes.
O n p eu t alo rs éc rire q u a n tita tiv e m e n t le p rem ier én o n c é. L’in ten sité to ta le du
rayonnem ent / (la puissance ém ise à toutes les longueurs d 'o n d e par unité de surface)
omi s par le corps noir est

/ = oT\ (9 4 )

où 7 'CM la tem pérature du eiMps noir, exprim ée en kelvins, et a esi une constante appelée
constante de Stefan-B oltzm ann. La valeur recom m andée est a — 5,670 x 10 * W /m '
(v o ir l'an n ex e B). I .'équation 9.4 est appelée la Ini de Stefan-Boltzm ann. l’A iitricliien
loscf Stefan a découvert cette loi en 1879 à partir d 'u n e série d ’expériences alors que
rA u iric h ic n Ludw ig B ollzm ann l'a dém ontrée de façon théorique qia'lqiies années plus
lai'd. L 'intensité / représente la puissance totale rayonnée, som m ée ou intégrée sur tnincs
les lo n g u eu rs d 'o n d e . D ans la fig u re 9 . 1. l'in te n sité rep résen te l ’aire sous la cou rb e
pour une température donnée. On rem arque que celte aire augm ente rapidement 1огм.ц1г la
tem pérature augm ente.
I e deuxièm e énoncé (xnu être rem placé par la relation

2,898 X 10 'm K, (9.5)


9 3 Le rayonnement du corps noir 249

où est la longueur d ’onde pour latjiielle l'intensilé fin laytmiieiiiem ém is par le corp.s
noir à une icm péraiure T est maximale. L 'cyuation У.5 est appelée la loi du déplacement
d e W icn ; elle a été d éco u v erte p ar Г A llom anrt W ilhelm W ien en I АЧЗ. Г е м е loi est
bien illustrée dans la figure У. 1. En effet, plus la icmpci atiirc est élevée, plus la Itrngiicur
d ’onde asscK ice au m axim um de la courbe est courte. Cette loi exprim e aussi le chanfiC-
m ent de co u leu r d 'u n o b jet à m esure q u ’il est ch au ffe. L o rs q u ’ on chauffe une pioce
m étallique dans u n feu, elle devient d ’abord rouge. Puis, à m esure QUC sa temperature
augm ente, sa couleur change : elle devient m ange, puis jau n e. La loi du déplacem ent dC
W icn est utilisée pour o b ten ir la tem pérature d e ми face des étoiles, lars étoilc.s bleues
Figura 9.2 formule de Planck ont une tem pérature de surface plus élevée que les étoiles rouges.
décnl parfaiiemem les données La loi de .Sicfan-Boit7m ann cl la loi du déplacem ent de W ie n ne présem eiii pas la
expérimentales. situation com p lète de la d isin h u iiu n d ’un corp.s noir. Il m anque i'in tc n s itC du rayon
nem ent en fonction de la longueur d ’onde, c'est-à-dire l’équetion de la courbe reliant les
points de la figure 9.1. O n définit la radiance sjitx trale, /?т-(л). de telle soi le que lu puis
sancc ém ise par unité de surface dans l’intervalle de  à Л t- </> ¡пни un im p s uoii à une
tem pérature T est Rjiy.) d k. L ’intensité / peut être calculée en iniégnuit A,{> ) siii louies
les longueurs d’onde, e ’est-à-dirc

(À) J X .
= Г /?г(>
■h)

À la fin du XIX' siècle, plusieurs suggestions ont été faites jxiur la fonction R ^ik\
mais aucune ne décrivait k» données expérim entales pour toutes les longueurs d ’oiule Pu
1900, l’Allemand Ma.x Planek propos« eette expression q u 'o n peut écrire de façon m oderne

Incalí
R jik ) = (9 0 )
k^ 1

qui e.st appelée la loi d u r a jo n n e m e n t d e P lan e k . D aas cette équation, c c.si lu \ iiesse
de la lum ière, k est la constante de B oltzm ann {k = 1..^8 x lO'^* J/K R iN ^ , où R esi
la œ n stan te des gaz parfail.s et est le nombre d'A vogatlm ) I » constante h est alors une
nouvelle constante introduite p ar Planek Pour cette raison, on 1 appelle la constante de
P la n e k , dont la valeur est donnée à l ’équation 9.3 L ’équation 9 f) dcerii p arfaitem n it
les données expérim entales disponibles pour loiiies les tem pératures et p o u r toutes les
longueurs d ’onde, com m e l ’illustre la lig u re 9.2. Pur exem ple, en 1993, le rayoïm rm cni
ft)s.sile, un reste probable du big-bang, a été m esuré par le satellite Cosmic. Hockgwuful
Explorer (CO BE). Les données suivent la loi de Planek avec une très grandi- précision ;
la lem pérautrc m esurée du rayonnem ent fossile est / “ (2.725 2. 0,002) K, O n discuter.!
de ce rayonnem ent fossile au chapitre 14
En intégrant la loi de Planek (l’équatioii 9.6) su r loutes les longueurs iro m k ' ou
retrouve la loi de Stelan-B oltzm ann (l'éq u atio n 9.4). où la consianio a est donnée par
l : t ‘'k'

De plus, si on trouve le maximum de la radiance spectrale en dérivant la lot du rayoïuiomem


de Planek pai rapport à À et en [xtsani eette d ériv ée égale à zéro, on retrouve la loi du
déplacement de W ien :
lu
= 2.89S X 10 ' ni K.
4.96.5A
Pour obtenir réq u atio n 9.6, Planek s’est d 'abord basé sur les résultats expérinieniaux
et sur la loi de S iefan -B o lt/m an n . Il a ensuite d ëm o n ü é son résultat en eûnsidcranl que
les parois de la cavité se com portent com m e de ¡letiu oseillateurx h.-irmoniques. Il a ensuite
siippo.sé q u e ces o.scillaieurs ne po u v aien t pas o sciller avec une én erg ie q u elco n q u e,
m a is seu lem en t avec un e én e rg ie rep réscn ln n t un m u ltip le t'iitier de hf, o ù / e s t la
fréquence du rayonnem ent que ces o scillateu rs absorbent et éiueiieiit lo rsq u 'ils im ei-
agi.sscnt avec le rayonnem ent p riso n n ie r d e la cavité. O t t e h y p o th èse su rp ren an te a
intrfNiiiil la quaniification et la constante de Planek dans le m onde de la physique et cela
a été le com m eneem eni de la physique quanliqne. Les physiciens de répiwiiie. incluant
Planek lui-m êm e, avaient beaucoup de difficulté à aa-ep ler q u 'u n o.seillaieitr ne puisse
pas absorbeir ou ém ettre de rayonncm cni avec une énergie quelconque, t n 1905, grSce
à sa très grande perspicacité, Einstein redém oniia réq iialio n 9.6 à partir d ’iinc hypothèse
250 Chapitre 9 te s photons et les ondes d e matière

différente ; le rayonnem ent contenu dans la cavité interagit avec les parois com m e s ’il
é tait co n siitu c de q u an ta d 'é n e rg ie ay an t c h a cu n u n e é n e rg ie d éterm in é e p ar
l’é q u atio n 9.2 ( f = hf). F ar co n séq u en t, E in stein a in tro d u it le co n cep t de q u an tu m
de lum ière, q u ’on appelle aujo u rd 'h u i le photon.
Dan.s les prochaines .sections, nous allons revoir les expériences qui m ontrent q ue la
lum ière est constituée de photons.

Exemple 9.2
I ài mu face d'une éini le nVsi pas très bien définie, contrairement à ta b) Quelle est l'intensité du rayoniicinent émis par ces aois étoiles ?
surface d ’une planète comme lu Terre. La plupart du rayonnement
émis est en équilibre thermique avec les gaz formant les couches SOlliTlON: Four calculer rintensité, on utilise les deux concept des suivants ;
extérieures de l'étoile Nous pouvons alors considérer les étoiles les étoiles peuvent être considérées comme des corps noirs et l’intcnsitc
comme des corps noirs. Le table.nu suivant indique la longueur d'un coips noir est donnée par la loi de Stefan-Boltzmann. On aisitlique
d'onde du pic de rayoïincnicnt poni' trois étoiles. l'équation 9.4 aux résultats de lu partie a) :

ttoile Couleur / = oT\


Sinus 0.33 Bleue
AlITUi
.Soleil U.5U Jautn:
= 3,4 X lO'* W/m-. (réponse)
Bétclgeasc 0,83 Rouge
fsolul
al Ouellc est la trmpéralnrc à la surface de ces étoiles ? = 6,4 X 10^ W/in^. (réponse)
SOlUItON: Le concept clé apphcnblc ici est qu’on peut considérer les
étoiles uonmic des cor|j\ noirs. La longueur d'onde et la lempéranire /ikiciïru« = (5-670 X lir * W/m’ K^)(3.5 x I0-’ K)^
associées au maximum île la courbe de la radiance spectrale sont alors = 8.4 X IO*’ W/m^. (réponse)
reliées par la loi de Wien (équation 9.,‘i). ( )n isole alors la température :
c) Le rayon du Soleil est r = 6.96 x 11)" m. Calculez sa luminosité L
2.898 X 10 ’ m K
7 --- (9.7) (la puissance totale rayoïméc).

SOLUnON: Pour calculer la luminosité, on utilise le tonteplde suivant:


Il s'agit easnitc de R-mplacer les valeurs de indiquées au tableau
rintcMsité représente la puissance par unité de surface. On calcule
2.898 X 10"' m K alors la puissance rayonnée en multipliant l'intensité par la surface
Tvilim 8.8 X 10^ K (repoase) du Soleil {A = 4 ;rr’)
0.33 X K) '’111

2.898 X 10 -*m K L = 147rr


rsowa - 5,8 X 10-' K. (réponse)
0.50 V lO'*' m
= (6.4 X 10’ W/m’) 4;r(6.96 x 10" m)’
2.898 X 10 ■'m K
r,.Dv'Uiliivuae ^ 3,5 X to ' K. (réponse) = .3.9 X )()’'’ W'. (réponse)
0.83 X 10-'' m

9.4 L’effet photoélectrique


.Si vous dirigez un fai.sceau lum ineux d 'u n e longueur d 'o n d e suffi.saimneni co u rte .sur
tenitre
en quai12 une surface m ctallitpie propre, la lum ière exp u lsera dc.s électro n s de cette su rface (la
V ide
I > surface énivitru des électrons). C et effet p h o to é le c triq u e est utilisé dans de nom breux
appareils (cam éras de télévision, camé.scopcs, vi.seurs de nuit, entre antres). Einstein a
H î soutenu le bien-fondé de son concept de phoion en l'iitilisanl p o u r expliquer ect effet,
P ^ I I iiiiiieré* q u 'o n ne peut com prendre .sans notions de phy.siquc quanliqiic.
_______ X I im idenir O n analysera m aintenant deux expériences photoélectriques de base, chacune iiti
li.sani l'ap p a re il illu stré dans la figure 9..3, où une lum ière de fré q u e n c e / éclaire une
1' cible P et en éjecte des électrons. U ne différence de potentiel V est m aintenue entre la

\
TC
. i Jiafbiic lisurc9.3 l'n apjwnùl utilisé pour étudier l'effet photoélectrique. I.a lumière incidente atieini

I—-4UL_-^ur__
une plaque P qu’im appellera la cible, éjectant des clcctronSt qui sont captés par le cylimlrc
capteur nans- le circuit, les électrons sc déplacent dans la direction opposée au,\ flcchc.s,
qui leprésenieni le sens du courant électrique Les piles et la résistance variable servent
à produire et à régler la différence ite potentiel.
9.4 L 'e f f e t p h o t o é l e c t r i q u e 251

cible P et le cylindre capteur C pour faire circuler ces électrons, appelés p h o to é lcc tro n s.
C ette capture produit un c o u r a n t p h o to é le c triq u e r m esuré p ar ram pèrernèin:: A

Première expérience photoélectrique


O n règle la différence de potentiel l 'c n déplaçant le contact mobile de la figure V.3 pour
que le capteur C .soit légèrem ent négatil p ar rapj.X)rt à la cible 1*. (. ette tension rülcntll ICiJ
électnins ém is. O n fait ensuite varier V juscju’à un e certaine valeur encore plus negative,
appelée p o te n tie l d ’a r r ê t l'arrêt» à laquelle raiii}ièiem ètre A indique une valeur nulle.
Q uand V — V„rc^, les électrons ém is les plus énergiques rebrousseiu cliem in ¡liste avam
d ’atteindre le capteur C. L’énergie cinétique de ces électrons les plus énergiques, it',,.,,,
est alors

A',,,:,,. = aiifp (‘LH.)

où e est la charge élém entaire.


Les m esures révèlent que, pour une lum ière île fiéquciice donnée. tu- depend
pas de l'intensite de la source lumineuse. Q ue la s o n n e soit ébloiii.ssaiite ou si Inihir
q u 'o n a peine à la détecter (ou d 'u n e intensité intermédiaire), l'énergie cinétique maximale
des électrons ém is a toujours la m êm e v aleur.
C e résultat ex p érim en tal co n stitu e un casse -tctc p o u r la phy.sique classiq u e. Du
point de vue classique, la lum ière ineidenie est une oiule élei:troniagnétiquc oscillant de
m anière .sinusoïdale. E lle d evrait donc faire osciller de façon sim isoïdale un électron de
la cible en raison de la force clcctrique oscillan te q u ’exèrcc so n ch am p électriq u e Si
l'am plitude de son oscillation est as.ser élevée, l'clccfron devrait se libcrer de la fiultacc
de la cib le - c ’est-à -d ire être ém is de la cib le. D onc, si on augm ente ram p litiiclc de
fo n d e et son ch am p électrique o scillan t, ré lc c tro n ilevrail .subir une « p o u ssée» plus
éncigique au moment de son em ission f 'ependnnt ce n 'estp a s ce {jUi seproduU. A une
fréquence donnée, la lumière intense et la lumière faible donnent toutes deux exactcmcni
la m êm e poussée m axim ale aux clcctrotis émi.s
L'explication de cette observation apparaît naturellem ent si on pense en foiK'lion des
photons. O n constate alors que l'én erg ie pouvant être transférée de la lumicR' incidente a
un électron de la cible est celle d ’un photon unique .Si on augm ente l'intensité lumincu.se.
on augim'iitc le nombre de photons en jeu, mais l'énergie de chaque plxiton (déterm inée pat
l'équation 9.2) dem eure inchangée parce que la fréquence ne varie pas. P ar consequent,
l'én erg ie transform ée en énergie cinétique d ’un électron dem eure égalem ent inc hangcc.

Seconde expérience photoélectrique


On fait m aintenatii varier la frcquetice/ de la lum ière incidente et on m esure lé potentiel
d ’arrêt 1^ figure 9.4 représente graphiquenicni en foiiclion d e ^ Note/, cju’il
n ’y a aucun effet p h o to électriq u e si la fréq u en ce est sous une r m a i i i r fréqueme de
seuil fo on, ee qui est éq u iv alen t, si la lo n g u eu r d ’o nde est su p érien re à la liiiigiietir
d ’nnde de .seuil à„ = c/Jq . C ’est ainsi, /vh importe i 'intensité de la lumière hu ideme.
Voici un autre casse-tctc p o u r la physique classique. Si vous considérey la Umiièie
com m e une onde élcctroinagnclique, vous devez vous attendre, m êm e si sa ftéquence est

«il L'Iiiaviolcl

f
U'
•Z
U
U
ri

figure 9 4 1 e potentiel d ’a rrci


en fcin ction de la freq u e n ce f
d<- la lu m iè re écLaiian l u ne cib le
de s o d iu m P d a n s raft|rjreil illu stré
à la fig u re 9..t (D o n iK -e s rapfxirtées
par R . A . M illik . m en 1 91 6)
2S2 Chapitre 9 Les photons et les oncles d e matière

b asse, à ce q u e la lu m ière p u isse to u jo u rs é je c te r d es é le c tro n s si vous fo u rn issez


suifisanim cnt d'énergie - c'est-à-dirc si vous utilise/, une source lumineuse assez brillarte.
Ce n 'est pas ce qui se produit. Q uand la lum ière possède une fréquence inférieure à la
fréquence de se u il/(,. il n 'y a aucun effet photoélectrique, peu im porte rin te n site de la
source lum ineuse.
T outefois, si l'é n e rg ie est transférée par les photons, on doit s ’attendre à ce q u ’il
existe une fréquence de seuil. D ans la cible, les électrons sr)nt m aintenus par des forces
électriq u es. ( S 'ils ne l'é ta ie n t pas, la force d e g rav ité les ferait to m b er au .sol.) Pour
s'éc h ap p e r de la cible, un électron doit acq u én r une énergie m inim ale où est une
caractéristique du m atériau de la cible appelée tra v a il d ’e x tra c tio n . Si l'én erg ie trans­
férée à un électron p ar un phoion excède le travail d'ex tractio n du m atériau (si h f > <I>),
l’électron peut s ’échapper de la cible. P ar contre, si l’énergie transférée n’excède pas le
travail d 'extractio n (c ’est-à-dire si hf < <1>). l’électron ne peut s ’échapper. C ’est ce que
dém ontic la ligure 9.4.

L’équation photoélectrique
Einstein a lait la som m e des résultats de tellc.s expériences photoéIeclriquc.s dans l’équation

<I> (l’cqiiation pfKrtoélectriquc). i9 .9 )


Cctlc équatioti constitue un énoncé du principe de conservation de l'énergie dans le cas de
l’absorption d 'u n photon unique par un m atériau possédant un travail d ’extraction <!'.
L ’énergie égale à celle du photon h f est transférée à un seul électron de la cible. P our
que l'é le c tro n s'é c h a p p e de celle-ci, il d o it acq u érir une énergie au m oins égale à »!>.
T oute én e rg ie a d d itio n n e lle ( h f — <h) que l’élec tro n a c q u ie rt du p h o to n d ev ien t de
l ’én erg ie cin étiq u e K. D ans les m eilleu res circo n stan ces, l’clcctro n p eu t trav e rser la
su rface sans p erd re son é n e rg ie c in é tiq u e ; il q u itte alo rs la c ib le avec le m axim um
d ’énergie cinétique possible.
O n |Tcut récrire l’équation 9.9 en rem plaçant par l’expression de l'équation 9.8.
A près un peu de mani|Tulation. on obtient

(9.10)
Les rap|xirts h/e et ^ / e étant constants, on devrait s'atten d re à ce q u 'u n graphique des
potentiels d ’arrêt m esurés en fonction de la fréquence de la lum ière soit une droite,
com m e dans la figure 9.4. D e plus, la pente de celte droite devrait-ctre hie. Pour vérifier
ce point, on m esure ub cl hc dans la figure 9 .4 et on écrit
ah 2.-45 V - 0 .7 2 V
e bc ( l l , 2 x 1 0 '-* -7 .2 x I0'^)H/
= 4.1 X lO '* ' V s.
Si on m ultiplie cc résultat p ar la charge élém entaire e. on obtient
/i = (4.1 X 10 '^ V s M l.ô x 10 '’ C ) = 6 .6 x 10 s.

cc qui est cohérent par rapport aux valeurs de h mesurées à l’aide de nombreuses méthodes.
A parté: l'é n o n c é d ’une explication de l’effet photo électriq u e nécessite ce rtain e­
m ent des notions de physique quantitjuc. l’endani de nom breuses années, l’explication
d ’E instein a égalem ent constitué un argum ent irrésistible en faveur d e l'ex isten ce des
photons. C ependant, en 1969. on a form ulé une autre explication qui faisait appel à la
physique quantique, mais qui ne nécessitait plus la notion de photon. La lum ière est bel
et bien quantifiée en photon.s. m ais l'ex p lica tio n d ’Einstein de I cffcl pho to élcctiiq u e
n ’est pas le m eilleur argum ent perm ettant de l’expliquer.

VÉRIFIEZ VOS CONNAISSANCES 2 : ,a fig u re représente g ra p liiq u e m e n t des données co m m e


celles de la figure 9.4 pour des tihles de césium, de potassium, de sodium et de lithium, l.es droites
sont parallèles, a) Cla.sse/ les cibles selon leur imvail d ’extraclion. en comuxüiçani pai le Iravail
ayant la valeur la plus élevée,
h) Classez les droites selon
les valeurs de h qu’elles
donnent, en commençant
par la plu.s élevée. f>.2
V-3 L'effet Compton 253

Exemple 9 3
Une feuille de ptHassiuin se trouve à une distance r = 3.5 m d’une 4. Avec l'aidc de l'é<]uailnn 4.23. on peut mciirr en rclmion le luux
source de lumière isotrope émettant de l’énergie au taux Я = 1,5 W. Le de rénergie incidente F„ avec rinieiksitc / de la lumier« tnoidente
travatl d’extraction Ф du potassium est 2,2 eV. Supposez que sur la feuille et 1’шге de la feuille ;
l'énergie transponée par la lumière incidente a été transférée à la
Рш - 1Л.
feuille cible continuellement et doucement (comme si la physique
classique prévalait sur la physique quantique). Combien de temps
Donc, Л/ - —
faudrait-tl à la feuille pour absorber assez d'énergie pour émettre un lA
électron'.' Supposez ici que la feuille absorbe toute l’énergie qui
5. Étant donné que la source de lumière est iiotiiipe. l'inieii.slic
l’atteint et que l'clcciron à éjecter capte l’énergie d ’une zone circu­
lumineuse / à une distance r d c la source dépend du taux
laire de feuille ayant un rayon de 5,0 ж 10"'* m. soit environ celui
auquel l’cncrgic est émise par la source, selon l’équauou 4 27.
d ’un atome type.

SOUrnOH Voici les tontephtlés qu’on utilise ici. 1=


4,T»'
1. L’intervalle de temps Д/ qu'il faut à la zone ptxir absorber l’énergie
On a donc, fiiiulemcni.
Д/Г rléix;nd du taux auquel renergie est absorbée ;
4я; 'Ф
Ai A i-
P Л
L’aire de détection A est n^(5.0 X 10 “ mÉ = 7 ^ 5 x 10 -'' m’, et 1c
2. Four qu'un électron quitte la feuille, l’énergie minimale qu'il travail d'extraction <1>est 2.2 eV — 3.5 x 10 J. St on insère ces
don acquérir de la lumière est égale au travail d’extraetion Ф du résultats et les autres dcniiiées. on obtient
potassium. Donc,
4rr(3.5 m)^(3.3 x 10 I)
Ф ~ (1.5 W)(7.S5 V 10-^' m ')
* 45H 0s ^ 1.3 h. (lé(x-)iisr)
3. Puisque la zone de la feuille absoibe totalement l’énergie, le taux Scion la physique classique, il faudrait drmr atreiulrr plus d’une
d ’absoqttion est égal au taux auquel l'énergie atteint la heure après avoir allumé la .source lumineuse pour qu'un pholoéleeiroii
feuille; c ’est-à-dire soit émis. Cependant, le temps d’attente léel est infcrieui k 1(1"'* s. Il
semble donc qu’un électron n'absorbe pas graduellement l’éneipie de la
Ф lumière incidente. Plutôt, il n'absorbe aïK'inic énergie, on il absorbe un
A r= — •
quantum d’énergie instantanément en absorbant un pliotnu «le lumirrr

Exemple 9.4
Déterminez le travail d'extraction <!' du scxlium de la figure 0.4. hfn - 0 -f '1* - <1>,

SOlllîlOM: Ic-i. le tontept ilê est qu’on peut déterminer le iras ail d’extrac­ Dans la figure 9.4. la fréquenee de seuil est celle à laquelle In
tion à l’aide de la fréi|uciicc de seuil fu (qu’on peut nxisurer sur la courbe croise l'axe horizontal des fréquences, .soit eiiviii III 5.5 x 10''' 11/
courbe). Voici le raisonnement; à la fréquence de seuil, l ’énergie On a «kirs
Cinétique est nulle dans l’cquaiitin 9.9. Donc, toute l'énergie hf
‘t ’ = h }n ~ (b.b3 X 10 ' ' J s)t5 5 X 10” H /)
transférée d’uii phoion à un électron sen à l’émission de ce dernier,
qui néc-essile une énergie de <t>. L'équation 9.5 tloniie alors, s i / = /o- “ .3.6 X I()■''' J — 2,3 cV. (réponsej

9.5 L'effet Compton


En 1916, E iastein a élargi son concept de quanta de lum ière (photons) en su g eéran iq u u
les photons p o ssèd en t une q u an tité de m ouvem ent. Le p h o to n est un e p a iiic u lr sans
m asse fil SC d ép lac e à la v ite sse de (a lu m ière) D ans le oos d un p h o to n ay an t une
énergie hf. l ’équation 8.57 indique que le m odule de cette quantité de m ouvem ent est

Л/ Л .. «.
P — - = — (la quantité de mouvement d un pbnion), (9.11)
е л

où on a re m p la c é / pat son ex p re ssio n d e l’éq u a tio n 9.1 ( f = c/X) D tinc, q u an d un


photon interagit avec la m atière, il y a iraiisfert d ’énergie et de quantité de m ouvem eiu
com m e s 'il y avait une co llisio n classiq u e en tre le photon et la m atière (la no tio n de
collision a été vue dans le chapitre 10 du volum e 1)
2S4 Chapitre 9 Les photons et les ondes de matière

E n 1923, A rth u r C o m p to n ré a lisa un e e x p é rien c e à l’u n iv e rsité W ashington


Doter
de S ain t L ou is, au x EtaLs-L'nis. C e lle e x p é rie n c e p erm it d e c o n firm e r l’h y p o th èse
du irdnsfcrt d ’énergie cl de quantité de m ouvem ent par le photon. Il projeta un fai.sceau
de rayons X de longueur d ’onde X sur une cible de carbone, com m e dans la figure 9.5.
Un rayon X est une fonric de rayonnem ent éicctrom agnctiquc à haute fréquence, donc
de courte longueur d ’onde. C om pton m esura les longueurs d 'o n d e cl les intensités des
rayons X qui étaient diffusés dans les d iflércn lcs directions p ar cette cible de carbone.
La figure 9.6 m ontre les résultiils d e cette expérience. B ien que le rayon X incident
Kcmc4 ne com prenne q u ’une seule longueur d ’onde (X = 71.1 pm ). on ctMislate que les rayons X
r t u « tllI ilI M U 'I ll
diffusés en conlicnnent tout un éventail avec deux pics d ’intensité. C ’est l’cfTet C o m p to n .
Figure 9.5 Appareil de Compton. Un pic est centré près de la longueur d 'o n d e X incidente ; l ’autre, près d ’une longueur
Un Taisecau de. rayons X d ’une d ’on d e X' su p é rie u re à A de A /., d iflé rc n c c ap p elée d é p la c e m e n t d e C o m p to n .
longueur d’onde A. = 71.1 pni est valeur du déplacem ent d e C om plon varie en fonction de l’angle auquel les rayons X
dirigé sur une cible C de carbone. diffusés sont délectés.
lx.s rayons X diffusés par la cible L a figure 9.6 constitue égalem ent un cassc-lêtc pour la physique classique. Du point
sont observés à différents angles tf> de vue classique, le faisceau de rayons X est une onde électrom agnétique sinusoïdale. Il
par rapport à la direction du faisceau devrait donc faire osciller un électron de la cible de m anière sinusoïdale en rai.son de la
incident. Le détecteur mesure
force oscillante q u 'ex e rc e son cham p électrique. D e plus, l’électron devrait o.sciller à la
l intensité de,s rayons X diffusés
m êm e fré q u en c e q u e l'o n d e et d ev ra it ém ettre d es o n d es à cette m êm e fréquence.
et leur longueur d’onde.
com m e s’il était une m inuscule antenne de transm ission. D onc, les rayoas X diffusés par
l'é le c tro n d ev ra ien t a v o ir la m êm e fré q u en c e, c l la m êm e lo n g u e u r d ’o n d e, q u e les
rayons X incidents - m ais ce n’est pas le cas.
Com pton interpréta la diffusion des rayons du carbone en tenant com pte des transferts
d ’énergie et de quantité de m ouvement effectués par les photons entre les rayons X incidents
et les électrons faiblem ent liés de la cible de carbone. On verra m aintenant, d ’abord île
façon conceptuelle, puis quantitativem ent, com m ent cette interprétation de la physique
quanlique peut vous am ener à com prendre les résultats d e C om pton.
Sup|X)sez q u ’un photon unique (d ’énergie F. — h f) est associé à l’interaction entre
les ray o n s X et un élec tro n au rep o s. En g én é ral, le rayon X c h a n g e ra de d irectio n
(le rayon X est dilfu.sé) cl l’électro n aura un recu l, m o n lran i ainsi q u ’il a acquis une
certaine énergie cinétique. L’énergie est cortservée dans celle interaction i.solée. Done,
l'énergie du photon diffusé ( £ ' = h f ) doit être inférieure à celle du photon incident. I a;s
rayons X diffusés doivent alors avoir une fréquence in fé rie u re /' et une longueur d ’onde
supérieu re À' à celle d es ray o n s X incidents, tout com m e le d ém o n tren t les résu ltats
de l'expérience de Com pton dans la figure 9.6.
Pour ce qui est de la partie quantitative, on applique d ’abord le principe de const'r
vation de rdnen^gic. 1.a figure 9.7 suggère une «collision » entre un rayon X et un électron
libre initialem ent au repos dans la cible. Le résultat de ccUe collision est que le rayon X

(Ü-4V' ,

L o n p i f iii dViiiiU' (piii) ri’oiulc (pin)

I 'ï
i 2

Figiiie 9 A 1 .rs rè siilla is de C o m p t o n


rrla tlv e in e n t li q u a ire a n g le s
de d iiïu s iim é N o ic z qu e le d è p la ie m e n i
de C o m p t o n A X a u g m e n te q u a n d
l ’a n g le d e d if fu s io n augm ente. Longueur d'onde (p in )
9 5 L e f f e t Compton 255

de longueur d ’onde À' est diffusé à un angle </>et l’électron e st éiiiis à un angle fl, conuiic
l’illustre la figure. I x principe d e con.scrvalion de l ’éncrgic donne alors

h f = h f ! K,
où fl/ est l’énergie du photon du raytm X incident, h f est l’énergie du pliolon du rayon X
diffusé et K est l’énergie cinétique de l'élcclro n s'cloigntm l. Htant donné que ré lc c tfo n
[■»eul se déplacer à une vitesse cotnparahle à celle de la hm ncrc. il laul uüliscr l'cxprCSSion
relativiste de l ’équation 8.51,

K — nu H y ~ 1),
pour décrire l’énergie cinétique de rélectixtn. Ici. m est la m asse de l ’clcciron Cl y est lu
fa d e u r de Lorentz
I

Si on rem place K par son expression dans rét|tiaiio n de la cuiiscrvatioti de I’cncrgie, oti
obtient
h f ~ h f + »«■■(•y — 1).

Ravon \ Si on r e m p la c e /p a r r /A e t / 'p a r t /A', on obtient la nouvelle équation de la eonservaiion


ÉlpctKin
de l’énergie
X V= 0
fl fl
- = f m c ( y - 1). (9 12)
A A'
Avant
O n applique ensuite le principe de conservation de la quantité de m ouvem ent à la
collision électron-rayon X illustrée dans la ligure 9.7. Selon l’équalion 9.11, le photon
incident p o ssèd e une q u an tité d e m o uvem ent d o n t le mcnlule est fl/A ; dans le cas du
Rayon X
photon diffusé, le m odule de la quantité d e m ouvem ent c.sl /i/A'. Selon l’é.quation 8.40.
l'é le c tro n ém is p o ssè d e un e q u a n tité de m o u v em en t d o n t le m o d u le est y m \ . É tant
V fl donne q u ’on est en présence d ’une collision à deux diinensions, on écrit des cquatuins
distinctes relativem ent à la conservation de la quantité d e m ouvem ent su r les axes des a
et des V pour obtenir
i\prOS

Figure 9.7 Un rayon X de longueur A - = — cos ifi -b ymveos fl d’axe des x) (9 13)
interagit avec un électron au repos.
Le rayon X est diffusé à un angle é,
avec une longueur d'onde plus grande et 0 = sin -t y/HVsin fl (l’axe dc.s vl. (9.141
A'
A'. I,’électron s’éloigtic à une vitesse v
et à un angle d. On veut déterm iner AA ( = A' - A), soit le déplacem ent de C om pum des rayons X
diffusés. Piu iTii les cinq variables de la colli.sion (A, A', v, 4> Ct fl) apparaissant dtuis les
équations 9 .1 2 ,9 .1 3 et 9.14. on choi.sii d ’élim iner v e t fl, qui ne concernent que l ’éleciron.
A vec un peu d ’algèbre (quelque {leu ctim pliquéc), on arrive à une équation perm ettant
de déterm iner le déplacem ent de C om pton en fonction de l’angle tic diffu.sion 4>:

Aa = — ( 1 — cos 4>) (le déplacement rie Ciimptoii). (9 LS)


me

L 'équation 9.15 esl cohérente quant aux rcsulfat.s expérim entaux dC C^ompion.
La quantité hhm de l’équatitMi 9.15 est une constante appelée In iig u ciir d ’o n d e d e
C o m p to n Sa valeur dépend de la mas.se m de la particule qui diffu.se les rayons X. Ici,
celle particule étani un électrcm faiblem ent lié, on rem place m par sa m asse ptnir évaluer
la longueur d ’oruie de Compton dons- le cas de lu diffusion de Compton causée pa r un
électron

Un détail à régler
l e pic présent à la longueur d ’onde incidente A ( = 71,1 pm ) de la fig u ir 9 .6 reste eiKorc
à expliquer. C e pic découle iK'n pas des interactions entre les rayons X et les électrons
liés très faiblem ent dans la cible, m ais des interactions entre lc,s rayons X ct les électrons
qui f>onxfortem ent lié.s aux atom es de carbone constituant la cible. L n effet, chacune de
ces c o lh sio n s-c i se produit entre un rayon X incident e t un ato m e d e carb o n e entier.
2S6 Chapitre 9 Les photons et les ondes de matière

D an s l'é q u a lio n 9 .1 5 . si on rem p lace m p a r la m asse d ’un ato m e d e ca rb o n e


(qui rep résen te en viron 22 ÜÜO fois eclle d ’un électro n ), cm co n state que Д л d ev ien t
22 000 fois plus petit que le d éplacem ent de C om pton d ’un électron trop p etit pour
être d étec té. D o n c, les ray o n s X d iffu sés dans ces c o llisio n s p o ssè d en t la m êm e
longueur d ’onde que les rayons X incidents.

Exemple 9.5
Des layuas X d’une longueur d'oiulc de X = 22 pin (énergie du Selon l'équalion 9.2 (E = hf). on peut remplacer l’énergie iniiiale E
photon = 56 keV ) sont diffusés par une cible de carbone ; on mesure cl l’énergie mesurée E' de» rayons X par l’expression contenant
un angle de diffusion de S i" p.ar rappon au faisceau incideiu. des fréquences. Ensuite, selon l’équalion 9.1 ( / = c/À), on |ieiil
remplacer ces expre.s.siiHis par celles contenant des iongucui.s d'onde.
a) Quel est It*déplacement de Compton des rayons diffusés ?
On trouve alors
5011ITION: la; contept cl4 mi lise ici est le suivant: le déplacement
h f — h f c /'/. — с/Л ' À' — À
de (aiiM|ilon est la variation de la longueur ü'ondc des rayon» X frep =
hf с/х
ocensionnée par leur diffusion, diffusion cllc-mèmc causée par des
élecim ns faihlemeni liés dans une cible De plus, selon l'équa- ДЛ
lion 9.1.5. cc dcplarenifni dé.|Tend de l'angle de. diffusion des rayons X. (9.16)
X -(■ A/.
Si on iiitrixiiiil la valciii' île l’angle égal à 8.^" et la valeur de la irutsse
de l’élcctron c c a k à 9,11 x 10"’' kg (parce que la diffusion est Si on insère les données, on obtient
causée par des cicctron.sl dan.» réquaiion 9 .15. on oliiiem
fre p = --------------------------- = 0.091 ou 9,1 %. (réponse)
' 2 2 p n i4 2.2 lp m
Aa = — ( I — cos tjt)
Bien que te déplacement de Compton AX soit indépendant de
(6,63 X U r ''’* J -s)(l - cos 85 ) la longueur d'imdc X des rayons X iiicidenu (voir réqualiuii 9 15).
(‘Л И "x lU -^' kg)(3.0() X ÏÔ» m /s) la variation relative d'énergie du photon des rayons X dépend, elle,
2.21 X II)” ’’ m ^ 2.2 pin. (réponse) de X : elle augmente quand la longueur d’onde du rayon incident
diminue, comme I'iiidiquc l'équation 9.16.
b) Quel pourcentage de ,son énergie initiale le photon du rayon X
transfèi e-i il à un électron dans une telle diffusion ? VÉRIFIEZ VOS CONNAISSANCES 3 Comparez la diffusion
SOfUHON. Ici, le cofitept dé consiste à déterminer Infraction d'énergie de Compton des rayon.s X (X = 20 pm) et celle de la lumière
perdue (qu’on appellera frep) par les photons diffusés par les visible (X = 500 pin), à un angle de diffusion particulier. Laquelle
électrons : povsède la valeur la plus élevée dans chacun des cas suivants ;
a) le déplacement de Compton, b) le décalage relatif de la longueur
énergie perdue E E' d'ondc. c) la variation relative d'énergie du photon et d) l’énergie
frep =
énergie initiale transmise à l'élcciron ?

9.6 La lumière en tant q u ’onde de probabilité


La question suivante coasliiue un mystère fondamental de la physique ; com m ent la lumière
iwui elle être une onde (qui se propage dans une région) en physique classique, alors qu’elle
e st émi.sc et ab so rb é e sous form e d e p h o to n s (qui n aissen t et q u i disparais.scnl
ponctuellem ent) en physique quanüquc ? L 'expérience des fentes doubles de la section 6.4
se situe au coeur de ce m ystère. Voici trois versions de ectie expérience
Fi.iiigi*4
< r ii i U i f é r rr i rr La version standard
L a fig u re 9.8 est un c ro q u is de l'e x p é rie n c e o rig in a le ré a lisé e par 'l’h otnas Y oung
en 1801 (5oiraus.si la figure 6 .6 ) l a lum ière atteini l’écran B. qui contient deux m inces
fentes parallèles. Les longueurs des ondes ém ergeant des deux fentes se propagent par
diflraction et .se superposent sur l’écran C où. par interférence, elles form ent un patron
où altem en i m axim um s et m inim um s d 'in te n sité . D ans la section 6.4. on a considéré

figure 9.8 line liiniièrc éclaire un écran B dans lequel deux fentes milices parallèles sont
pratiquées. La luinicrc émergeant de ces fentes sc propage par diffraction. Les deux ondes
diffracices sc superposent lorsqu'elles frappent l'écran C ei forment des franges d’interfèrcncc.
Un pciif détecteur de photon D situé daas le plan de l'écran C produit un clic nei chaque fois
qu'il ahsm+ie un photon.
9 6 La lumière en tant qu onde de probabilité W

l'e x isle n c e de ces franges d ’in te rfé re n ce co m m e une p reu v e in d én iab le de la n atu re
ondulatoire de la lum ière.
O n place un minu.scule détecteur de photon D dans le plan de l'éc ra n Г 4 u p |io se /
q u ’il s’agit d ’un appareil photoélcclriquc qui éincl uu clic quand il alt.sodx' un photon.
O n constate que le d étecteu r pa>duit une suite de clics, alcatoircm cnt espacés dans le
tem ps, chaque clic signalant un tran sfert d ’énergie de l’o nde lum ineuse û l ’écran par
l’absorption d ’un photon.
Si on déplace très lentem ent le dcicctcur vers le haut ou le bus. com m e l’imliquc la
tlèchc noire dans la figure 9.8, on constate que le nom bre de clics auginrnic txi dim inue,
passant par des m axim um s et par des m inim um s qui correspondent exaetem eut ii vmix
des lignes d ’interférence.
L’intérêt de cette expérience de l ’esprit est le suivant. O n ne peut prédire l'in stan t
où un photon sera détecté à un point particulier de l’écran f ’ , les pholoas .siml tk'lci-.(é.s h
des points individiœls à des tem ps aléatoires. Cependam , on [vui |w édireqiie la pio/x/Ai/W
relative q u ’un photon soit détecté à un point particulier dans un intervalle de temps spécifié
est proportionnelle à Г intensité de la lum ière incidente a ce ixiint.
D ans la section 4.4, on a vu que rin te n silé / d ’une onde lum ineuse h un ¡Kiinl donné
est proportionnelle au carré de l’aini'Iiaide du vecteur cham p rlcx:iriqiir o v ilbml de
l’onde à ce point. D onc, on p eu t affirm er ce qui suit.

1л probabililé (par unite de te?np.s) qu'un р1югоп soil dét« té dans un priii voluiiir m u ré
sur un point donne dans une onde lumineuse est proponiotmelle au carré du rainpliiude
du vecteur champ électrique de Гогкк: à ce point.

On a m aintenant une description patbabiliste d ’une onde lumineuse, ce qui constitue


une autre façon de considérer la lumière, ( ’d le -c i n ’est pa.s seulem ent une onde clccfro-
m agnetique, m ais aussi une o n d e d e p ro b a b ilité . Autrem ent dit. on pcui a ilrib iic r une
probabilité num érique (par unité de tem ps) au fait de détecter un photon dans un petit
volum e centré à chaque point contenu dans une onde lum ineuse

La version à un photon à la fois


En 1909, G. I. Taylor réalisa une version de rcx p é ric n cc des deux fentes dans laquelle
il n ’y a q u ’un photon à la fois qui se déplace de la source à l’écran C, version rép étée tie
nom breuses fois depuis. À la différence de l’cxpcricncc preeedente, la .source him tnensc
est si faible q u ’elle n 'é m e t q u 'u n photon à la fois, à inlet valles aléatoires. Il est étonnant
de constater que des lignes d'interférence se form ent encore sur l'écran C si l'expériencé
dure assez longtem ps (plusieurs m ois dans le cas de la prem iere cxjiéricnec de Taylorl.
Com m ent peut-on expliquer le résultat de cette expérience à deux fentes e t à un photon
à la fo is? A vant m êm e d ’etudier ce résultat, il lüiil se poser certaines q u estio n s; si les
photons traversent l’appareil un à la fois, dans laquelle tics deux leiilcs de ré e ra n B un
photon d onné p asse t il? C om m ent un p hoton d o n n é |.-ieiii.n « sa v o it » q u ’il y a une
autre fente qui crée la possibilité d 'u n e in terféren ce? Un photon unii|ue |ieut-il pas.sei
par le.s deux fentes et interférer avec lui-m êm e
Il ne faut pas oublier que 1rs iTlxuons sont observables sculnnenl qinintl ils interagissent
avec la m atière il n ’y a aucun m oyen d e les d élec ter en l'abseiiL e tl’un insti iiiiicnt
com m e un d é te c te u r ou un écran . D o n c, dans 1 ex p é rien c e d e la fiu u re 9.8 , on peut
seulem ent savoir que les photons sont générés à la source lumincu.se d di.sparaisscnt ù
l’écran. Entre la source et l’écran, on ne (leui connaîlic la nature ni l’aciiviié du plmum
Cependant, éUuit donné q u ’un patron d ' interférence finit par se li h nxa siu l'ceraii. on peut
suppo.ser que chaque photon se déplace de la source h \’ccтan,1m^sform{'¿Гumon4<’ qui
rem plit l’espiK'e entre ces deux o b jets et qui disparaît lors d e l’ab so ip iio n du photon,
avec un transfert d ’énergie et de quantité de m ouvem ent, à un certain point .sur l’écran.
O n ne p eu t pa.s p réd ire le p t'in i où ce tran sfert se p ro tln iia (où un plioton sera
détecté) d;ins- ce cas. C ependant, on pent prédire la probabilité q u 'il se produira (et q u 'il
y aura tendance à l’absorption d ’un pboion) dans les régitms brillantes des lignes d ’iiuerfé
rcnce qui se form ent sur l’écnui. (h t [seut aussi prédire q u ’il auta tendance li ne pas se
produire (et qu’il n’y aura/w.v tendance à l’absorption d 'un photon) dans les régioas sombres
d e CCS lignes d ’interférence. O n peut donc dire que l’onde se projiageant d ’u n r stnirce
vers l'é c ia n e.sl une mir/e c!c prohahiUlé, et q u ’elle produit des « franges de p ro b ah ililc;>
sur l’écran.
258 Chapitre 9 Les photons et les ondes de matière

rrM Ccroiie
Molécule unique
^ Trajoeroii-e
La version à un photon à la fois et à grand angle
Dans le pa.ssé, le.s physiciens ont tente d 'expliquer Je résultat de l’expérience à deux fentes
M„ et à un photon à la fois par la présence de petits paquets d 'o n d es lum ineuses classiques
> 1 in d iv id u ellem en t p ro jetés v ers les fentes. Ils d éfin issaien t ces p etits p aq u ets com m e
des photons. C ependant, les expériences m odernes invalident cette explication et cette
H d c tin itio n . La fig u re 9.9 illu stre le m o n tag e de l une d e c e s e x p é rien c es, rap p o rtée
en 1992 par M ing Lai et Ican-C laude Diels, de l’U niversité de .New M exico. La source S
contient des m olécules qui cm cltent des photons à des instants bien .sépares. Les miroirs
Figure 9 9 1 ^ '.oiirce S émet des photons
Ml cl M l sont placés de m anière à réfléchir la lum ière vers deux trajectoires di.stinctes.
n des instants bien séparés dont
1 e t 2. sép arées p ar un an g le в qui fait près d e I ВО”. C e m o n tag e d iffè re de celui de
la lumière siiii deux trajectoires
l ’ex p érien ce sta n d ard à d eu x fen te s, d an s lequel l'a n g le e n tre les tra jecto ires d e la
scpmécs cl inlcrlî’ii; avec elle-même
lum ière atteignant les deux tentes est très petit.
au détecteur D après avoir été
reeninhinée par le séparaieur A près avoir été réfléchies par les m iroirs M , et M j. les ondes lumineu.scs suivant
les trajecto ires i et 2 se ren c o n tre n t au .séparateur d e faisc ea u B. (U n .séparateur de
de faisccuu B. (TVaprès Miiiÿ; I^i
et Jean-Claude üiels, Jounuü of the faisceau est un appareil optique qui transm et la moitié de la lumière incidente et réfléchit
ûiirhal Sodeiy o f America B. n' 9, l'au tre m oitié.) D ans la figure 9.9, du côté droit du .séparateur. l’ondc lum ineuse suit la
déccmbiu l ‘J92, p, 2 290-2 294.) trajectoire 2 et, réfléch ie p ar B. se com bine avec F o n d e lum ineuse d e la trajectoirc 1
lransm i.se p ar B. C e s d eu x o n d es im e rte re n t alo rs l ’une av ec l’autre au d é te c te u r D
(un phoUmndtipUcateur pouvant détecter les photons individuels).
Le détecteur ém et alors une suite d'im pulsions électroniques aléatoirem ent e.spacccs.
soit une iniiiulsion pour chaque photon détecté. Dans l’expérience, le séparateur de faisceau
c,sl déplacé lentem ent à l’horizontale (dans l’expérience rapptm éc, une distance maximale
approxim ative d e seulem ent .50 /tm ), et le résultat du détecieur e.si noté dans un lableau.
Ce déplacem ent fait varier les longueurs des trajectoires 1 et 2, jircxliiisant un décalage
de phase entre les ondes lum ineuses qui atteignent le détecteur D. Les m axim um s et les
m inim um s d ’interférence apparaissent dans le signal du détecteur.
Cette expérience est difficile à com prendre d ’un point de vue traditionnel. Par exemple,
quand une m olécule d 'u n e source ém et un plioton, ce photon suit-il la trajectoire 1 ou la
trajectoirc 2 dans la figure 9.9 (ou une autre tra je c to ire )’? C om m ent peut-il suivre les
deux directio n s à la f o is ’? Pour répondre à ces q u estio n s, on suppose que, q uand une
m olécule ém et un photon, une onde de probabilité se propage dans toutes les directions.
D ans F expéricncc. on a choisi deux de ces directions, presque m utuellem ent opposées.
O n co n state ici q u ’il e st p o ssib le d 'in te rp ré te r les trois v ersio n s d e l'e x p é rie n c e
à deux fentes si on suppose q ue 1 ) la source génère la lum ière sous form e d e photons,
2) que la lumière est absorbée p ar le détecteur sous form e de photons et 3) que la lumière
se propage de la source au détecteur sous form e d 'o n d e de pntbabilité.

9.7 Les électrons et les ondes de matière


Un 1924. le physicien fran çais L ouis de B ro g lie fo rm u la l’affirm atio n su iv a n te su r
la sym étrie: un faisceau d e lum ière est une onde, m ais il transfère de l'é n e rg ie e t une
t]uantité de m ouvem ent à la m atière seulem ent en des endroits l«x:ali.sés. par les pht'tons.
P ou rq u o i un faisc ea u d e p a rtic u le s ne p o u rra it-il p as a v o ir les m êm es p ro p rié té s ?
A utrem ent dit. pourquoi ne peut-on penser à un électron en m ou\ ememt (ou à toute autre
paiticulc) com m e à une o n d e d e m a tiè re qui tran.sfcre de l’énergie et une quantité de
m ouvem ent à la m atière en des endroits lot-alisés ?
Plus particulièrem ent. D e B roglie suggéra qu e l'éq u a tio n 9.11 {p — hlKi pourrait
.s’a p p liq u er non seu lem en t au x p h o to n s, m ais au ssi aux électro n s. O n a utili.sé ce lte
éi|uaiion dans la section 9.5 pour attrib u er une q uantile d e m ouvem ent p îi un photon
de lum ière ayant une longueur d ’onde À. On l'u tilise m aintenant, sous la form e

h
(la longueur (l’onde de De Broglie) (9.17)

p(Mii atti ibuei une longueur d'(Mide 2. à une particule ayant une quantité de m ouvement p.
La longueur d ’nmie calculée à l’aide de l’équation 9.17 est appelée lo n g u e u r d 'o n d e de
D e R io p lie de la particule en m ouvem ent. La prédiction de de Broglie .sur l'ex isten ce
des ondes de m atièiv a été vérifiée ex p érim en talem en t pour la prem ière fois en 1927,
par C. J- Davi.sson et I, 11. ftcrm er, du Bell Téléphoné L ab o n u o ries. cl pai G eo rg e P.
T hom son, de l’U niversiié d ’A berdeen, en bcosse.
9.7 Les électrons et les ondes de matière 259

Figure 9.10 Phoioi^raplticN montrant la création dc lignes


d’inierférence par un faiscrau d'électrons dans une cspcnence
à deux fentes comme celle illustrée dans la figure 4.R
Les ondes de matière, comme Ic.s ondes lumineuses, sont
des ondes de pmhnhUité. lie haiil en bas. les nombres
approximatifs dxicetrons en jcii sont 7. 100, 3 000, 20 000
et 70 (ХЮ.

La figure 9 .10 m ontre les épreuves phoingiapliiques d'tiiidcs d c m ntiere tirées lors
d 'u n e expérience plus récente. Au cours dc cette experience, des franges d 'im ei féieiicc
se .sont formées quand les électrons ont été projetés, un o la fois, à travers un appareil a deux
fentes. Cet appareil res.semblait aux appareils utili.sés pour expliquer rim erférenee optique,
sauf que l’écran d 'o b serv alio n ressem blait à un écran dc télévision norm al. Uuand un
électron heuilail l'éc ra n , il générait un éclair lum ineux dont la position ciaii notée
L es prem iers électro n s (les deux photos du haut) ne rcs'élèfcnl nen d 'in ic re ssan i
et scm blatenl avoir louché l’écran au hasard repicndani. après le passage dc plusieurs
milliers d’élccirons, un m otif est apparu sur l’écran, lévélam des fiarg as oit dc nombreux
électrons avaient touché l’écran et d ’autres où peu d'éleeirotiK l'av aien t fait i 'e patron
correspond exactem ent aux attentes norm ales quant à rin te rîé re n ee des ondes Pai mn.sé-
qucnl, chaque élection a traversé l’appareil sous form e d ’ondc de matlCrc - la partie i|iii
traverse une fente interférant avec la partie qui ira v e rv l’autre tente, l. cttC interférence
déterm ine alors la probabilité que ré le c iro n se m atérialise à un point donné d r l’ceran,
en It- tCMichant De nom breux électrons se m atérialisent dans les zones c<mrespondani aux
franges bnllan les en interférence optique, et jx u d ’enirc eux le font dans les zones co r
nespondant aux franges foncées.
Des interférences sem blables ont été dém onm'e.s avec des protons, de.s n cutronf et
divers aïom es. En 1994. elles o nt etc dém ontrées avec des m olécules dMixle 1,. qui sonl
non seulem ent .“KX) ()(X) lois plus m assives que les électrons, m ais aussi beaucoup plus
comple.xc.s. h n 1999. elles fu ren t d ém o n trées av ec des m o lécu les en co re p lu s corn
plex es, so it d es m olccule.s de fullerc'ne (ou hurhninsterfiilleri’ne) et C ( l e s
fnllerèncs sont des m olécules d 'ato m es de rnrhonc. H) atom es dans la C;«, Cl 70 dans la
C 7,,. ayant la form e d ’un ballon dc soccer.) Il sem ble que les ju tits objets, to m m e les
électrons, les protons, les atonies et les m olécules, sc prop ag en t co m m e dc.s ondes de
m atière. Cependant, il existe un point où la taille cl la com plexité des objt ts dc\ ienm-nt si
im ponantfs q u ’il n est plus ju.stifié d ’attribuer une nature om lulalm re à l'objet 11 n y a
plus de cohérence entre les ondes. À ce point, on retombe' dans le m onde non qiianiique
260 Chapitre 9 Les photons et les oncles de matière

qui vous est famiJici; où s ’applique la phy sique des chapitres précédents. Bref, un électron
est une onde de m atière pouvant interférer avec lui-même, niais un chat n ’est pas une onde
de matière et ne peut interférer avec lui-même (c'e qui doit êüx; un soulagement pour les chats).
A u jo u rd ’h u i, la n atu re o n d u la to ire des p a rtic u le s et d es ato m es est te n u e pour
acquise d ans de n om breux d o m ain es .scientifiques et en in g én ierie. P ar ex e m p le , on
utilise la diffraction des électrons et des neutrons pour étudier les structures atom iques
des solides et des liquides, e t on utilise la diffraction des électrons p o u r étudier les struc­
tures atom iques des surfaces.
L a fig u re 9 .1 1 a) illu stre un m o n tag e p o u v an t se rv ir à d é m o n tre r la d iffra ctio n
des rayons X ou des électro n s par des cristaux. O n dirige un faisceau d e rayons X ou
d ’électrons sur une cible constituée d 'u n e poudre de m inuscule cristaux d 'alu m in iu m .
L es ray o n s X ont un e c e rta in e lo n g u e u r d ’o n d e X. O n fo u rn it assez d ’én e rg ie aux
électrons (lourquc leur longueur d 'o n d e de de Broglie soit la m êm e qu e A. La diffraction
des ray ons X ou des électrons par les cristaux produit des patrons d'interférence circulaires
sur une pellicule photographique. L a figure 9.11 b) m ontre la figure de diffraction des
rayons X ; la figure 9.11 c), celle des électroas. Ces figures sont les m êm es, les rayons X
et les électrons étant des ondes.

Les ondes et les particules


l.es ligures 9.10 et 9.11 sont des preuves convaincantes de la nature onthdatoirc de la
matière, mais il existe au m oins autant d ’expériences qui lais.saitcroire à l a nauire dixt rète
de la m atière. On peut revenir, par exem ple, aux trajectoires générées par les élcciiuns
et illustrées dans la p hoto d 'o u v ertu re de ce chapitre. C es traces (qui sont des chaînes de
b u lles lais.sécs dans l ’h y d ro g èn e liquide rem p lissan t la ch am b re à b u lles) suggèrent
fortcm eni le passage d 'u n e particule. Où est f o n d e ?

,'Vnncau
de diffraction
Fiüsr.pa\i incident ciiciilaire
(rayoïi.s X ou électrons)

Cible
(poudre d'almiiiuium)
Figure 9.11 a) Expérience utilisée pour démonircr. par diffraction,
le caractère ondulatoire du faisceau incident. Photographie
de.s figures de diffraction produites quand le fai.sceau incident csi Pellicule
constitué ht de rayons X (onde lumineusei et c) d’électrons (onde photographique
de matière). Notez la rc,s.scmhlance géométrique de hase des figures. a)

h) C)
9.8 L equation a e scnrodinget 261

figure 9.12 Quelques uik-s des nombreuses trajectoires suivies par une paniciile entre les iroiiiLs
de détection J et f . Seules les ondes de matière qui suivent les trajectoires proches de la ligne
I
droite reliant ces points iniertêrem d'une manière construclivc. Dans les «nlrcs cas. les niidts
^ r ~ '^
• I
t qui suivent des trajectoires voisines interterent de manière destrucuve. Donc, une onde de mattere
lais.se une trace linéaire.

P our sim p lifier la .situation, on p eu t en lev e r le ch am p m agnétique, p o u r lairc n i


sorte que le.s chaînes de bulles soient droites. On peut cunsidérei chaque bulle cuinm e
un point de détection de l’éleclron. Les ondes de m atière se propageant entre dCfl points
de détection / e t F e o m m e dans la figure 9.1? exploreront loiilc.s les trajectoires po ssi­
bles ; la figure en illustre quelques unes. En général, k chaque trajectoire reliant / ei F
(sauf pour celle en ligne droitel correspond une traiectoire voisine ; ainsi, les ondes de
m atière qui suivent ces deux chem ins s'anniilcronl m uturllem cnl par in tcrtcrencc. t e
n ’est toutefois pas le cas de la trajectoire en ligne droite reliaiii / et F ; ici, les ondes tic
m atière em p ru n tan t toutc.s les trajecto ires voisines ren fo rcen t f o n d e qui .suit la ligne
d roite. V ous pou v ez im ag in er les bu lles qui fo rm en t celte trace co m m e une suite de-
points de détection où fo n d e d e m atière subit une interférence t tmM niciîve

Exemple 9.6
Quelle est la longueur d'onde de de Broglie d’un électron po.ssédani D ’apres l’équation 9.17,
une énergie cinétique de 120 eV ?

SOIUIIOK: Un premier concept clé est utilise ici ; on peut déterminer la lon­
gueur d’onde de de Broglie À pour l’électron à l’aide de l’équation 9.17
P
( f = h/p) si on determine d’abord le module de sa quantité de
b.63 X 10 s
mouvement p. Le deuxième concept dé est qu’on détermine p à l’aide
.5.91 X 10“ ^'^ kg m /s
de l’énergie cinétique K donnée pour l’électroa Cette énergie cinétique
est très inférieure à l'énergie au repos d ’un électron (0,511 McV. = 1,12 X 10 m = 112 pra. (léponse)
d'après le tableau 8.3). On peut donc utiliser les approximations non
relativiste.s pour le module de la quantité de mouvement p {— mv) et
pour l’énergie cinétique F ( = ^mv^). C ’est environ la taille d'un atome type. Si on angincntc l ’énergie
cinétique, la longueur d ’onde diminue encore plus.
Si on élimine le module de la vitesse v dans ces deux équations, on
ohlient
^ V É R IH E Z VOS CONNAISSANCES 4: c I élection et un prnrnn
p = \Î2mK peuvent avoir a) la meme énergie cinétique, b) la même quantitC
de mouvement et c) la même vitessi- Dans cliaquc cas. quelle
- >/(2)(9.11 X lO -’i kg)(120eV )(L60 x 10 J/eV) particule possède la plus counc longueur d'ondc. de De Hroglic'.'
= .S,91 X 10' kg • m/.s.

9.8 L’équation de Schrôdinger


U ne sim ple onde se propageant, q u ’il s ’agisse d ’une oiule dans une corde, d ’u n r onde
sonore ou d 'u n e o nde lum ineuse, est décrite com m e une g ran d eu r qui varie .selon un
m ode ondulatoire. D ans le cas des ondes lumincu.ses. p ar e.xemplc, ccltc g ran d eu r est
E(x. y, r. I). soit le cham p électrique de fo n d e . Sa valeur observée en un point depend
de la position de ce point et de 1 instant où l’observation est faite.
Q uelle g ran d e u r v ariab le d c v ra it-o n u tilise r pour d écrire une o nde clc m a tiè re ?
O n doit s ’attendre à ce que cette grandeur, app elée fo n c tio n d 'o n d e Î'^(.v, y, c, t), soit
plus com p lex e q u e la g ran d eu r corics|X )ndante d ’une o nde luinm euse. parce q u ’une
onde de m atière, en plus de l'é n e rg ie e t de la q u an tité de inouvem eiit, tran sp n rie une
mas.se et (souvent) une charge électrique. 11 s'avère que ' f , la lettre grecque psi m/qusciilc,
rcpié.scnie tou jo u rs une fonction co m p lex e au sens m athém atique, c 'e s t à d ire q u ’on
peut to u jo u rs expi im t'r ses v aleu rs sou.s la form e a -b ib, où o et b sont d es nom bres
réels cl r = —1.
Dans tous les cas que vous rencontrerez ici, les variables de position peuvent être
grnopées séparém ent de la variable de tem ps et ’4 ' peut s ’rc riie stnis la forme

y, /. t) = ^¡/{x. V, Z) e (9,1«)
262 Chapitre 9 Lies photons et les ondes de matière

où <ij ( = I jr f) est la fréquence an g u laire de l’o n d e de m aiière. N o tez qu e \f/, la lettre


grecque psi m in u scu le, rep résen te seu lem en t la p artie spatiale de la fo n ctio n d ’onde
dépendante du tem ps 'ï''. D ans la section suivante, on utilisera presque exclusivem ent i}f.
Deux questions se posent ; qu’entend-on par fonction d’onde et comment la détermine-t-on ?
Que signifie lafotK lion d ’onde? O n entend p ar fonction d ’onde le fait q u ’une onde
de m atière, com m e une o nde lum ineuse, est une onde d e probabilité. S upposez q u ’une
on d e de m atière attein t un m in u scu le d étec teu r de p a rtic u le s; la p ro b ab ilité q u ’une
particule soit délectée dans un intervalle de tem ps donné est alors proportionnelle à
où \ijA est le m o d u le d e la fo nction d ’o n d e co m p lex e à re m p la c e m e n t du détecteur.
Bien que t/r soit habituellem ent une grandeur com plexe, est toujours à la fois réelle
et positive. Il s ’agit alors d e q u ’on appelle d e a s ité d e p ro b a b ilité , et non de V-',
qui a une signification physiqui-. En gros, cette signification est la suivante.

► La probabilité (par unité de temps) de détecter une particule dans un jKtit volume centré
à un point donné dans une onde de inalièic est proportionnelle à la valeur (fc Itfrl’ à ce point.

É tant donné que \jr est habituellem ent une grandeur com plexe, on déterm ine le carré de
son m odule en m ultipliant ^¡f par le com plexe conjugué de Pour déterm iner xjf*,
on rem place le nom bre im aginaire i dans \jß par - i .
Comment détem dne-t-on la fonction d ’onde? L es ondes so n o res et les o n d es se
propageant dans une corde sont décrites par les équations de la m écanique new tonienne.
Les ondes lum ineuses sont décrites par l ’équation d e S c h rô d in g e r, form ulée en 1926
par le physicien autrichien Erw in Schrödinger.
Plusieurs des cas qu’on étudiera m ettront en jeu une particule se déplaçant dans la
direction de l'a x e des x dans une région où des forces conservatives agissant su r elle lui
donnent une énergie potentielle U{x). D ans ce cas particulier, l’équation de Schrodinger
se réduit à

dhfr ü irfii

(l'équarion de Schrôdinger, pour un mouvement à une dimension). (9.19)

où E e st l'én e rg ie m écanique totale (énergie po ten tielle plus én erg ie cin étiq u e) de la
p a rtic u le en m o u v em en t, (O n ne lien t pas co m p te d e l ’é n e rg ie au rep o s d an s cette
éq u a tio n non re la tiv iste .) O n ne peut d é riv e r l’é q u a tio n d e S ch rô d in g er à p a rtir de
principes fondam entaux ; c ’est le principe de base de la physique quantique.
Si f/(x) a une valeur nulle dans l ’équation 9.19. celte équation décrit alors une p a r-
tk iile libre, e’est-ù-dirc une particule sur laquelle aucune fw ee n ’agit. Dans ce cas. l’énergie
to ta le d e la p a rtic u le e st e n tiè re m e n t c in étiq u e e t, d e ce fait, d an s l’éq u atio n 9 .1 9 .
E c s i jm v C ette équation devient alors

d^ 87r-«i ( m \ ' \
7x^- '

q u 'o n peut reform uler de la m anière su i\ ante;

d~ik
dx-

Pont nhienir celle équation, <in a rem placé m v par la quantité d e m ouvem ent p et on a
regroupé les term es.
S elon l ’équati(»n 9 .1 7 , pth d e l ’éq u atio n ei-d e ssu s peut d ev e n ir 1/X. où À e st la
longueur d ’oinlr de d e Broglie de la particule en m o u v em en t On reconnaît de plus que
2rr/À est le nombre d'onde k. q u ’on a défini dans l'éq u atio n 2.5 Pat cette substitution,
réq iialio n ci-<le.ssus devient *

d~il/
- , + k^rlß = 0 d ’équation de Schrôdinger, iKHir une paniculc libre). (9.20)
9.9 Le principe d'incertitude de Heisenberg 2i3

La solution générale de l’équation 9.20 est

y/(jc) = + Be (9 r u
où 4 cl fl sont des constantes arbitraires. Vous pouvez dém ontrer que cen e êtjuaiion est
en effet une solution de Tcqualion 9.20 en rem plaçant et sa dérivée seconde clans
l’équation et en notant q u 'il en résulte une identité.
Si on com bine les équations 9 . 18 et 9.21. dans le cas de la fonction d onde dépen
duale du leinps 'P d ’une p aiticu le se déplaçant dans la direction x. on trouve

0 - ilf(x)e-"^ = M e " ' H flc “ " ‘ Ic'


(9.22)

Déterminer la densité de probabilité It/tP


D ans la section 2.5, on a vu q u ’unc/o/ic:rtwi F ayant la forme F{kx ± o j i ) loprésente une
onde progressive. C ela s ’applique tant aux fonctions exponcnticllc.S comme celles dc
l’équation 9.22 q u ’aux fonctions sinusoïdales q n ’on a iitili.sécs poin rlrcrirc. Ic.s nndc.s se
pnipageant le long des cordes. En fait, ces deux représcniMiions de fonctions sont rcücc.s jwr

e '" - cos B + I sin 6 et e '* “ cos 6 — I sin fl,

où fl est un angle.
I ,e prem ier term e du m em bre dc droite de l ’cquation 9.22 représente donc une onde
■se propageant dans la direction des v positifs ; le second term e, une onde SCpropageant
dans la direction dc Taxe des -v négatifs. C ependant, ou suppose que la particule libre
q u ’on étudie .se déplace seulem eiii dans la diiection d c l'a x e des x positifs. P our rndiiirc
la solution générale (équation 9 22) au cas present, on pose la con.stantc arbitraire B dans
le.s équalion.s 9.21 et 9.22 ég a le à zéro. En m êm e tem ps, on renom m e la eo n slau te 4
com m e étant ^f/„. 1,’équation 9.21 devient alors

^ (x ) ^ % e“^'. (9.23)

Pour calculer la densité dc probabilité, on calcule le carré du m odule de i>(x). O n obtient


alors

Dc.asiU' d f prot>al>ililc
M aintenant, pui.sque

on obtient

1)^ = Vrd (une constante).


figure 9.13 Une représcmuuion graphique
L^i figure 9.13 représente graphiquem ent la densité de probabilité l<//P en fom tioii tic v
de la densité de probabilité lt/f|’ pcHir
une particule libre se déplaçant dans pour une particule libre - une droite parallèle ù 1 axe des .x allant de —œ à -e a On roustare
la direction positive de l’axe des x. q u e la dcn.sité d c p ro b ab ilité li^P est la m êm e p o u r to u tes les valeur.« d e v , ainsi, la
Piii.sqiic IVd- a la même valeur, qui est pro b ab ilité que la p articu le sc trouve n'im porte où le long dc l’axc dCS X est d égale
constanip, pour tontes les valeurs valeur partout. 11 n ’y a aucune m éthode qui puisse ¡rerm cttie de picdirc q u ’u n r pasition
de X, la probabilité de détecter est plu.s probable q u ’une autre ; c ’est donc dire que toutes tes positions sont possible« de
la particule est la même à tous m anière égale.
les points de .sa trajectoire. On verra cc que cela signifie dans la prochaine section.

9.9 Le principe d'incertitude de Heisenberg


La particule dont la densité de probabilité est lepié.seniée graphiquement dans la figure 9 .13
est lib re, c ’e st-à -d ire q u ’au cu n e fo rc e n ’agit sur e lle, d e so rte q u e sa q u a n tité de
m ouvem ent p doit être co n stan te. O n a su p p o sé q u ’on co n n a issa it ex actem en t cette
quantité de m ouvem ent, sans aucune incertitude T e fai.xaiii nii obtient une particule qui
a une densité de probabilité constante : on ne sait pas du tout où clic se trouve. Si on veut
264 Chapitre 9 Les photons et les oncles de matière

oculaû'c conn aître la position de la particule, on peut faire une m esu re en l’éclairan t avec des
photons. Un photon dévié pourra nous indiquer la position de la particule m ais à cause
de l ’e ffe t C o m p to n , la q u a n tité de in o u v en ien t d e l ’c lc c tro n sera ch an g ée. D an s le
monde quaniique. une m esure change l’état d ’un .sysicme. Ce phénom ène a été découvert
par l'A llem and W erner H eisenberg, (jui a form ulé son p rin c ip e d 'in c e rtitu d e en 1927 :

Ou ne peut pas nxtsurct exactement, de façon simultanée, la position et la quantité


o b jc r lif

V
de inouvemeiii d'une particule.

clei n o n •
Plus précisém ent, le principe d ’incertitude donne les lim ites suivante.s sur les incer-
plioloii
inridi'iii tittides entourant les m esures sim ultanées de la pasition et de la quantité de m ouvem ent;

.SOUKf
- l ï Aa ^ fl.
a)
Av Ap, ^ fl. (9.24)
A." A/> s fl.

D ans ces éqiuitions, nous avons introduit la c o a s ta n te d e P la n c k ré d u ite ft = hl2n =


1.05 X K) J • s (prononcer h-barre ; voir l'an n ex e B pour la valeur recom m andée).
D onc, plus on veut connaître précisém ent la quantité de m ouvem ent d ’une particule
b) et plus r in c e n iu id c su r sa po.sition sera grande. L ’ex em p le de la p articu le libre de la
section précédente est un cas extrêm e : on connaît exactem ent la quantité de m ouvem ent
mais on ne sait pas du tout où est la particule, car la densité de probabilité est constante.
D e m êm e, plus on connaît précisém ent la position d ’une p articu le et p lu s sa quantité
de m ouvem ent est incertaine. D onc, la position d 'u n e particule qui suit une trajectoire
précise perd sa signification dans le m onde quantique.
Regardons cela avec une expérience im aginaire. On utilise un niicro,scope imaginaire
pour m esurer la position x cl la com posante de la quantité de m ouvem ent a d ’un électron.
Ce m icroscope utilise une source de iuinicrc dont la longueur d ’onde est Â, et un systèm e
de lentilles, com m e i'illustre la ligure 9.14 a). Supposons que la pt)sition d ’un électron est
Figure 9.14 a) On mesure la pasition obtenue lorsqu’un pholt)n frappe l'électron et qu'il est dévié vers l’objectif du microscope.
et la quantité de mouvement avec A cause de la diffraction de la lum ière qui pas.se à travers le .sysième optique, le phoion
un microscope imaginaire. Pour détecteur pourra être dévié et on obtiendra la figure de diffraclit'n de la figure 9.14 c).
mesurer la position de l'électron, La position de l'électro n cause une incertitude Aa donnée p ar la form ule de diffraction
un photon doit le frapper pour ensuite
entrer dans l’ohjectif du microscope, A
b) La quantité de mouvement Sin O)
de r électron sera nuxlitice de
|Mi r effet Compton. La composante où d>est illustré dans la figure. On peut réduire l’incertitude sur la position en dim inuant
(W- la quantité de nunivemeiit du photon la longueur d ’onde du photon.
est comprise entre —(/i/Â)sin </> Le plioton qui est dévié par l’électron transporte une quantité de m ouvem ent dont
et (/t/À)sin <f>. I.’ohscnMtcur voit le m odule est p - hfX. Plus on réduira la longueur d ’o nde de la lum ière utilisée (pour
la figure de diffraction de la partie c). obtenir une m esure précise de la position) et plus la quantité de m ouvem ent du photi^n
sera g ran d e. L o rsq u e le p h o to n v a fra p p e r l’é le c tro n , il va lui d o n n e r une ce rtain e
quantité de m ouvem ent par effet C om pton. C ela va changer la quantité de m ouvem ent
de l'électro n d ’une certaine v aleu r égale à l'in c ertitu d e su r la q u an tité de m ouvem ent
du photon dévié Pour être vu. le photon doit pas.ser dans le systèm e optique. 1 l ’après la
figure 9.14 h), pour un photon qui pa.s.se par l’o b jeciil, la com posante x d e sa quantité
de m ouvem ent d o it être co m p rise en tre —(/i/A.)sin <f>et h(/i/A),sin <}>(on a n ég lig é la
variation du module de la quantité de m ouvem ent du plmton). Donc, il y a une incertitude
sui la com posante x d e la quantité de m ouvem ent du photon

A/> — 2(/î/A)sin <f>.

Cela corre.spond à l’incenitude sur la quantité de m ouvem ent de rélectro n . Cette fois-ci.
pour dim inuer rincerliiu tle sui la quantité de m ouvem ent de l’électron, il faut augm enter
la longueur d 'o n d e de la lum ière utilisée.
S u p p o so n s q ii’on veut m e su re r de façon sim u lta n ée la p o sitio n cl la q u an tité
de m ouvem ent de l'électron. Ix’ prinluit de l’in cenitude sur la po.sition et de l ’incertitude
sur la quantité de m ouvem ent est

. . / A \/2/»sin<*\
Ax Aa - -, - . M = 2/i.
\.s in ^ / V A y
9,10 L'ôffôt tu n n e l ÏM

Ce ré su lta t e st in d é p en d a n t du sy stèm e o p tiq u e . Il est c o n sé q u e n t avec le p rin cip e


d'in ccrlilu d c de H eisenberg.
L a lim ite sur la précision d es m esures sim ultanées de la jHisnioii et de la quam lié
de m ouvem ent im plique au.ssi u ne lim ite sur la m esure d e l ’cncrgic d ’une particule et
du tem ps nécessaire pour la irc cette m esure. U n obtient le p n n eip c d ’ineortitudû pour
l’énergie et le tem ps :

S E S r >: h . (9 2-))

Le principe d ’incertitude im plique que l'én e rg ie d 'u n .système n 'e st pas I ia c ; elle
peut fluctuer autour d ’une valent m oyenne [Tendant un inieivallc Ai.

Exemple 9.7
Suppose/, qu'un électron se déplace le kmg de l'axe de.s \ et que vous P, = wv, - (9.11 V lO 'Jt kg)(2,0V y. ni/s(.
mesurez le module de sa vitesse à 2,(to x. 10®ni/s. avec une précision de
= 1,87 X lO-’-'kg-in/s
0,50%. Quelle est l incenitude minimale (comme le iiermet le principe
d'incertitude dans la thwirie quantique) avec laquelle vous pouvez L incertitude sur la vitesse donnée est de 0,.50% de la vilevsc
nTesurei simnlianémeiit la ptisilion de rélectron le long de l’axedesA?
mesurée. Puisque /J, dépend direeiemem de la viievvc, riacetriiudc
SOLUTION. Le (ontcpldé utilisé ici est le suivant : l'incertitude minimale Ap, sur la qiianliié de niouvemeiu doit être de 0,50% d r la qiiiinllic
permise par la théorie ijuantique est déterminée par le principe d'incer­ de nwiuvement :
titude de lleinsenberg de l’équation 9.24. On n'a bestiin de tenir
compte que des composantes de l’axe des x parce que le mouvement Ap, - (0.(K)5 0)A
ne se fait que le long de cet axe et qu’on veut y connaître l’incertitude (Ü.IX).5 0)( I.R7 X 10 " k g ra/sl
Ax quant à la position. Étant ckmné qu’on veut l'incertitude minimale - 9.25 A 10 " k g ni/.s.
permise, on choisit l'égalité an lieu de rinégaliié dans la partie
relative à l'axe des .t de l’cqualiim 9.24 ; on écrit donc Le principe d’ineerlitiide donne alois
Ar Ap ~ fi.
fi (ft.63 X U) ^ J s)/2n-
Pour évaluer l’incertitude A/i, sur la quantité de mouvement, il faut Av =
Ap, 9,35 X 10 27 kg • m /s
d abord évaluer la composante de la quantité de mouvement p , . Étant
^ 1.13 X m ® m I nm. (réponse)
donné que la vitesse de l’élecnon v, est très inférieure à la vitesse de la
lumière C, on peut évaluer/?,. à l'aide de rexprc.ssion non relativiste
ce qiii rcpri5.sente environ 100 diametres atoniique&. 6 tanl donne
de la quantité de nxiuvcmcnt au lien d’utiliser l’expression R-laiivi.\ic. votre mesiire de la v ite w de I’clettion. cssaycr dc mesurer la position
On trouve alors de relecmin avec une plus gtande precision ii’a aucuii sens.

9.10 L’effet tunnel


Si vous lanciez à plu.sieurs reprises une lève en gelée sur une table où l'épose un lis re qui
eou[.Te sa trajectoire, vous seriez très .suqrris dc voir la lève apparaître de l ’ autre CÔtC du
livre au lieu de rebondit vers l’ai rière. Ne vous attendez pas à un tel rc.suJial avec une
fève en gelée. C ependant, un événem ent sem blable. ap|>H(' rlTcl tu n n e l, xt pniduit avec
des électrons et d ’autres particules de [letites masse«
l a figure 9.15 a) m ontre un électron possédant une én erg ie totale F se ilé|ila(jani
parallèlem ent à l’axe des .v. D es forces agi.ssent sur l ’électron, de sorte que son én eicie
potentielle est nulle sauf quand il sc trouve dans la zone 0 < r < / , ou celle èniTgic po.s.sèdc
la valeur constante L'n. On définil cette région conune une b a rr iè r e «l’én e rg ie [Hitentielle
(st)iivent appelée b a r r iè r e d c p o te n tie l) d 'u n e hauteur Va et d ’une épaisseur L.
Du point de vue classique, étant donné que /, <■ i/o, nn é l « Inm .s’approchant d e la
barrière en venant dc la g auche serait réfléchi par la b arrière et lelu o u sse iaii cliemin.
Cependant, en physitjuc quantique, l'clee tron est une onde dc m atière cl i l y a une certaine
possibilité q u 'il « tra v e rse « cette barrière C ela «igninc q u ’il y a n n r p io b a liilit é f in it
que l’électron sorte dc l’autre côté de la barrière, en se déplagani vers la diotte.
On peut déterm iner la fonction d ’onde ^fv(x) décrivant l’éleclron ; pont ce faire, on
résout l’équatio n de S ch ro d in g er (équation 9.19) .séparém ent pour ch acu n e des trois
régions dc la figure 9.15 a) : 1) à gauche de la barrière, 7) dan.v la barriere cl 3) >) «hoilr
de la b arrière l es co n stan tes arb itraires qui ap p araissen t d an s les so lu tio n s peu v en t
2bb Chapitre 9 Les photons et les ondes de matière

liiicrg ic être choisies de m anière à ce que les valeurs de t/f(.v) e t de sa d érivée par rap p o rt à x
soient continues (sans saut ni entortillem ent) à = ü et à jr = L. Si on calcule le carré
du m odule de xl/{x), on obtient la densité de probabilité.
L a fig u re 9.15 b) rep rése n te g ra p h iq u e m e n t le résu ltat. La c o u rb e o sc illa n te à
-f.-m -----
Elwti’on gauche de la barrière (où x < 0 ) est une com binaison de fo n d e de m atière incidente et
de fo n d e de m atière réfléchie (doni l’am plitude est inférieure à celle de fo n d e incidente).
Les oscillations apparaissent parce que ces deux ondes, .se propageant dans des directions
opposées, interfèrent l’une avec f autre, créant ainsi une onde stationnaire.
À l'in té rie u r de la b arrière (où 0 < < L), la d en sité de p ro b ab ilité d im in u e de
rntuiièrc c.xponcnlidlc avec x. L a v aleu r donnée de /. étan t petite, la densité d e proba­
Dnisiu* <!<' {)t'ol)<i)»iliio bilité n*e.st pas tout à fait nulle à .r = L.
À d ro ite d e la b arrière d e la fig u re 9 .15 (où x > L). le g rap h iq u e d e d e n sité de
probabilité décrit une onde transm ise (à travers la b a n iè re ) avec une am plitude faible,
m ais constante. Donc, l’électron peut être détecte dan.s cette région, m ais avec une pro­
babilité relativem ent faible. (C o m p are/ cette partie de la figure avec la figure 9.13, qui
0 I m ontre une particule libre.)
!•) O n peut attribuer un coefficient de transmission 7 à fo n d e de m atière incidente et à
figure 9.15 a) Un diagramme d'énergie la barrière de la figure 9.15 a). C e coefficient donne la probabilité q u ’un élccrion incident
montrant une barrière d’énergie so it « transm is » à trav ers la b arrière - c ’e st-à -d ire la p ro b ab ilité q u 'il y ait un eftè i
potentielle d’une hauteur i/„ et d'une tunnel. Par exem ple, si 7 = 0,020, on peut dire que, à tous les 1 000 électrons projetés
épaisseur L. l ’n électron possédant vers la barrière. 20 (en m oyenne) la traverseront p a r effet tunnel et 980 seront réfléchis.
une énergie totale E s'approche L e c o e ffic ie n t d e tra n s m is s io n / e st a p p ro x im a tiv e m e n t
de la barrière en venant de la gauche,
b) La dcasitc de probabilité T ^ e Ikl. (9.26)
de l’onde de matière représentant
l'électron, elle montre l’effet tunnel qui i
- E)
permet à rélcctron de traverser la bar­ ou on trouve (9.27)
/*2
rière. I^a courbe à gauche de la barrière
représente une onde de matière station­
É tant donné la form e exponentielle de l’équation 9.26. la valeur de T est très sensible
naire résultant de la superposition de
aux trois variables dont clic dépend : la masse de la particule m, l'épaisseur de la barrière L
l’onde de matière incidente et de l’onde
Cl la différence d ’énergie Un — E.
de matière réficcbic.
L 'e ffe t tunnel tro u v e de n o m b reu ses ap p licatio n s te ch n o lo g iq u es, d o n t la diode
tunnel, dans laquelle on peut rapidem ent faire p asser ou stopper un flux d 'électio n s (par
effet tunnel à travers un appareil) en contrôlant la hauteur de la barrière. P u isq u 'o n peut
le faire agir très rapidem ent (en 5 ps). c e t appareil est app ro p rié aux ap p lieatio as qui
dem andent une réponse riq^ide. En 1973. le prix N obel de p hysique a é té p arta g é par
taiis « experts en effet tunnel », Lco Esaki (pour l'effet tunnel dans les semi-conducteurs).
Ivar G iaever (pour l'e ffe t tunnel dans les supraconducteurs) et B n an Josephson (pour
la Jonction de Josephson. in terru p teu r rapide basé sur l'e ffe t tunnel). Le prix N obel de
1986 a été rem is à G erd B innig et à H einrich R ohrer en reconnaissance de leur travail
sur un autre appareil fort utile basé sur l'c rfc t tunnel, le m ieitiscope à effet tunnel

'V É R IF IE Z VOS CONNAISSANCES 5 : Dans la figure 9 15 b), la longueur de l’onde transmise j

r
est-elle supérieure, intérieure ou égale à celle de i’onde incidente '?

Le microscope à effet tunnel


|

A ppareil basé sur l’cffci tunnel, le m icroscope à effet tunnel perinei de tracer une ca n e
détaillée des surfaces, révélant des im ages à r é c h c llc atom ique ay an t une résolution
beaucoup plus grande qu e celle des im ages obtenues à l'a id e d 'u n m icroscope optique
ou électronique La figure 9 16 en donne un exem ple ; chacun des atom es de la surface
est visible.
L a lig u re 9.17 ex p liq u e le cœ u r du m icro.scope à e ffe t tunnel. U ne fin e sonde
m étallique, m ontée à I’intcrscction de trois tiges de q u a rt/ perpendiculaires, est placée
près de la surface à exam iner. U ne petite différence de potentiel, peut-être de seulem ent
K) mV, est m aintenue entre la .sonde et la surface.
Le q u artz cristallisé p ossède une propriété in téfessan tc apivelée piâzo^lei tru ité:
figure 9.16 Un rés»-aii d'atomes quand on applique une différence de potentiel dans un échantillon de q u a rt/ crisüillin. les
de silicium révélé par un microscope dimensions de cet échantillon varient légèrement. C'ette profMnétc fait doucem ent varier la
à effet tunnel longueur de chacune des trois tiges de la figure 9 . 17. ce ijui perm et à la somle de balayer
P é v is io n e t r e s u m e 267

i Figure 917 l.e pnncipe d'un microscope à efici iiirmel, l.es liges de quartz dcplaceni une sonde
I à pointe très fine au dessus de la surlace à étudier en maintenant une séparation consiante
i . .'S-'; entre les deux. La sonde doii se déplacer de tiaiii en bas |X)ur suivre le contour de la aui lace ;
ce mouvement est alors enregistré pour constituer une carte comme celle de la lijjiire V Ifi
Tiges
“ j5 i« tjuari/
*^.|*L*S la surface (dans les dircclions x et v) en luuuuint et en dcsccndiini selon le contour de la
P'iíTí.'*
SchkIc ' surface (dans la direction z).
L ’e sp a c e e n tre la su rfa c e et la so n d e fo rm e un e b arrière d ’én eiE ie p n ie in ie llr,
Surface
co m m e d an s la figure 9.15 a). Si la sonde est su ffisam m en t proche de la su rface, les
élec tro n s prov en an t de l'éc h an itllo n peuvent trav e rser celte b ariièrc p ar e fte t tunnel
pour atteindre la sonde, form ant ainsi un courant par effet tunnel.
Pendant que l'ap p areil balaye la surface, un dispositif de réponse électronique lèglc
la ptfsiiion verticale de la sonde pour g arder constant le courant par effet tunnel. C ela
sig n ifie q u e la sép aratio n en tre la so n d e et la su rfa ce d em eu re ég a le m en t constante
durant le balayage. L’im age produite par l'appareil (par e.xenijilc la fig u ir 9. Ifi) est une
carte vidéo d o n n an t les d iv erses positions verticales de la sonde, et par conséinieiii le
profil de la surface, en fonction de la position de la sonde dans le plan xy.
Les micro.scopcs à effet tunnel sont disponibles dans le com m erce et sont utilisés
dans Ic.s laboratoires partout dans le monde.

Exemple 9.8
Supposez que l’clcciron de la figure 9.15 a), possédant une énergie et, scTon l’cqiialion 9.26. le coefticient de transmission est
totale E de 5,1 cV. s'approche d’une barrière d'une hauteur de
U„ = 6.8 eV et d’une épaisseur de /, = 750 pm. T= - p - ‘o,o - 45 X u r ”. (réponse)

a) Quelle est. ap|iroximdlivemcnt, la probabilité que l’électron traverse Donc, à ch;H]ue million d’clcclroas qui heurtent la barncrc. environ 45
la barrière pour apparaître (et cire détectable) de l’autre côte ? la traverseront par effet tunnel.
SOLUTION Ici. le tontepl dé est le suivant ; la probabilité qu'on cherche h) Quelle est. approximativcnKnt. la probahilitc qu'un proton possédtuu
est le coefficieni de transmission T tel qu'il est déterminé pat la même énergie totale de 5,1 eV iraveice l.t harrirrr pm r apparaître
rét|ualkm 9 26 ('/ « e ‘ où k est (et être détectable) de l'autre côte’’
STT^miUo — £ )
ä2 • SOlUTIOH Ici. le concept clé est que le cocfTicient de iransiiiission T
* •= / (donc, la probabilité de tiansinissioii) dépend de la inas.se de la
Le numérateur de la fraction située sous la racine caivée est particule. En effet, puisque la masse ni est un des facteurs contenus
dans l’exposant de e dans l'équalion de T. la valeur de bi |irnbaliilité
(8,’r-)(9.l l X 10 kg)(6.8 cV - 5.1 cV)
de transmission y est très sensible. C ette l'ois-ci, la masse est relie
x ( l , 6( lx 10-‘’ J/eV )= 1.956 X lO'^’ J kg. d'un proton (1,67 X 1()~2^ kg), qui est largeniem supdrieiuH ii celle
de l’électron dont il est question en a); si on emploie la même
/1,9.56 X 10-^^ J kg méihodc qu’en a), on détermine que ï ~ 10' I >one, la pmlmbiliié
Donc, k =
V (61.63 X 10-'*^ J s)2
= 6,67 X 10'' m
que le proton soit transniis n'est pas nulle, male très peu supéiieiiie A
zéro. Dans le cas de particules encore plus massives et possédant la
La quantité (sans dimension) 2kf. est alors
même énergie totale de 5.1 eV. la probabilité de irarismiicion esi
2kL = (2H6.67 X lO" m~'X750 x )() •■ mj = 10,0 exponciitieileincni pins faible.

RÉVISION E T R ÉSUM É

Le.s quanta de lum ière - photo n s Une onde électromagné­ est T émet un rayonnement continu décrit par la loi du rayonnement
tique (luiiiicrc) est quantifiée, et ses quanta s’iippcllcnt photons. Dans de Planck
le eus d'une onde lumineuse de fréquence / et de longueur d’onde à . 2irr’7i 1
l’énergie F. et la quantité de mouvement p d'un photon sont RtM = fhcOJcT _ ] ’ (9.6)

h — hf (l'énergie du phoion) (9.2)


oùRjtX) est la nidiaiKe spectrale, définie de telle sorte c|ik; la iniissancc
hf _ h émi.se par unité de surface comprise dans un intervalle de A a À 3 dis
c ~ A est Rj-iX) fi) I ’inlnisilé totale niyniinéc est dcmiiéc por la loi dC
Slefan-Rolt^niaiin
1 ^ rayonnem ent du corps noir Un corps noir est un obiet qui
absorbe toute la liiniièrc incidente. L'n corps noir dont la température I - o rC (94)
2i8 ChapKre 9 Les photons et les ondes de matière

I n corps noir ckMii la température est T émet ui\e intensité maximale région où l’énergie [xitentieUe est U(x), on peut déterminer la valeur
pcxjr une longueur d’oncle donnée par la loi du déplacement de Wien de en résolvant l’équation de Schrodinger simplifiée :

= 2.898 X 10-’ m K. (9.5)


I£ - L\.x)]ir = 0 (9.19t
"P ”
L ’e ffe t ph o to électriq u e Quand utie lumière possédant une Comme une onde luinincu.se. une onde de matière est une onde de
Ircqucncc s’uffisammenl élevée atteint une .surface métallique propre, probabilité. ainsi, si un détecteur de particules est inséré dans l’onde,
les interactions entre photons et électrons à l ' intérieur du métal la probabilité qu’il ilétectc une particule dans un intervalle de temps
]irov(Xiuetn une émission d'électrons. La relation qui gouverne ces donné est proporliorMiellc à l\icP, grandeur appelée densité de proba­
éinission.s e.si bilité.
19.9) Dans le cas d'une particule libre (c’c.st-à-dire d'une particule
h f - Л-'шах +
pour laquelle U{x) ~ 0) se déplaçant dans la direction x, l\H’ possède
où ht est l’énergie du plnnon. est l’énergie cinétique des électrons une valeur constante pour toutes les positions le long de l'axe des x.
émis les plus énergiques et ‘l>est le travail d ’extraction du matériau
de la cible - c’est-à-dire l’énergie miiiiinulc nécessaire pour extraire l.e principe d ’incertitude de H eisenber^ La nature proba­
un électi'on de sa surface. Si A/ possède une valeur inférieure à <t>, biliste de la physique quaniique donne une limite importante à la
il 11 y a aucun effet photoélectrique détection de la position et de la quantité de nimivemcnl de la par-
ticuic. Autrement dit, il n’esi pas possible de mesurer simultanément
L 'effet Compton Quand les rayons X semt diffusés par des élec­ la position r et la quantité de mouvement p d’une particule avec une
trons faiblement liés dans une cible, certains des rayons X diffii.sé.s précision sans limite. Les incertitudes sur les composantes de ces
ont une longueur d'onde supérieure à celle des rayons X incidents. grandeurs sont déterminées par
Ce déplacement de Compton est déicnniné par
Д .1 Ip, г Л
A v l i \ > ti (9.24)
Дл — —- (1 — cos </>). (9.1.5) Ar A/x > ti.
me
où <^>est l’angle de diffusion des rayons X. De plus, l’énergie et le temps ne peuvent pas non plus être mesurés
avec une précision infinie. Les incertitudes sur l’énergie et au temps
Les ondes lum ineuses et les phoUms Quand la lumière inter sont déterminées par ;
agit avec la maticie, l’énergie et la quantité de mouvement sont trans­
férées par les photons. Cependant, quand la lumière se propage, on \ F A fS fi. (9.25)
considère l'onde lumineuse comme une onde de probabilité, dans
laquelle la probabilité (par unité tie temps) qu’un photon puisse être
détecté est propmtionnelle à t ’^. où est ramplitiirie du champ L 'effet tunnel Selon la phy skiiie classique, une particule incidente
électrique oscillant de l’onde lumineuse au détecteur. sera réfléchie par une barrière d’energie potentielle dont la hauteur
est suficricure à l’énergie cinétique de la particule. Par contre, selon
Les ondes de maticre (ine particule en mouvement ctmime un la physique quantique, il existe une probabilité finie que la particule
élection ou un proton peut être décrite comme une onde de m atière t traverse une telle barrière grâce à l’effet tunnel. La probabilité
sa longueur d’onde (appelée longueur d ’onde de De Broglie) est qu’une particule donnée de masse m et d ’énergie b traverse une
détemiinée par a. = htp, oi'i p est la quantité de nKxivcmem de la particule. barrière de hauteur i 'o et d ’une épaisseur L est déterminée par le
coefficient de transmission T:
Les fonctions d 'o n d e l ine onde de matière est décrite par sa
fonction d'onde y; l). qu'on peut diviser en une |.iaitie dépendante T ^e (9.26)
de la position (Kjt. y, a) et en une partie dé|iendatite du temps
Dans le cas d’une particule non relativiste de inas.se m se déplaçant £)
où k, - (9.27)
dans la direction .v avec une énergie totale F. constante à travers une

Q U E S TIO N S . ♦ c • Tè f Ш

1. Hntrc les ondes élcctromagncti(|ues générées dans un four à Pourtant, elle émet du rayonnement à une plus grande puissance que
micro ondes et ccllc.s gcncrccs par l’appareil à rayons X de votre le Soleil. Pourquoi ?
dentiste, lesquelles possèdent a) la plus graiulc longueur d'onde, S Dites si chai iinc des affirmations suivantes sur l'effet photoélec
hl la lirqiiciKT. la plus élevée et c) les plioiKnv ayant le plus d’énergie? trique est vraie ou fausse, a) Plus la fréquence de la lumière incidente
? .Si Ions les objets i(ui nous entourent émctieni du rayonnement, est élevée, plus le potentiel d ’arrêt est élevé, bl Plus f intensité de­
pourquoi ne peut on pas les veur la nuit là lumière incidente est élevée, plus la fréquence de seuil e.st élevée,
3, Lit ladiancc siveiialc d'un corps noir est maximale pout une c) Plus le tr.ivail d’extraction du matériau de la cible est élevé, plus le
InngtK'ui d’oiulc J )0 change la tenifiéialure de l’olijel pour potentiel d’arrôl est élevé, d) Plus le travail d’extraction du matériau
que l.i tadiaiicr .spectrale soit maximale à une longueur d'onde Xf/2 de la cible est élevé, plus la fr^ iicn re de seuil est élevée, e) Pins l-a
IX, quel Iacteur l’intensité totale cst-cllc cliangce ? Iréquetice de la lumière incidente est élevée, plus l’énergie cinétique
4 l ‘cloili rtiiielgnise. la géante rouge de la constellation d’Orion, a maximale des électrons émis c.st élevée, fl Plus réiicrgie, des photons
une température de siii face bien plus petite i|iie celle du Soleil. est élevée, plus le potentiel d'arrêt est bas.
E x e r c ic e s e t p r o b lè m e s 269

6. Selon la figure de la rubrique « Vérifiex vos connatssanees » 2, de De Broglie, en eommcnvaiil pai la plus grande : clectrtm, particule
pour une lumière incidente de fréquence donnée, l’énergie cinétique alplia. neutron.
ittaxitnalc des électrons émis est-elle la plus élevée dans le cas d’une 18 T.a figure 9 .18 illustre quatre situations où un électron sc dcplucc
cible en potassium ? * dans un champ électrique. Il se déplace a) dans l.a direetinn opposes
7. Dans l’effet photoélectrique (|x>ur une cible d<mnée et une lumière d ’un champ éleclnque, b) dans la même direction qu’iiii cliami»élec­
incidente tle fréquence donnée), lesquelles de ces grandeurs, le cas trique. c) dans la inêinc direction qu'un chump inagnelique, dl perpen
échéant, dépendent de l’intensité du faisceau lumineux incident : diculairemcni à un champ magnétique. Dans chaque cas, la longueur
a) rénergic cinétique maximale des électrons, b) le courant photo­ d’onde de De Broglie de l’électron augjncnte t elle, dimitnis-i-slls
électrique maximal, c i le potentiel d’arrêt, dj ta fréi)uenec de seuil ? ou rc,ste-t-cllc la même ?
8. St vous dirige? une lumière ultraviolette sur une plaque métallique
isolée, celle-ci émet des électrons pendant un moment. Pourquoi
arrête- t-clle de le faire ?
9. On éclaire une platjuc mélailiquc avec une lumière d’une certaine
<j---------------
/■’
<--------------- o_—
D
——
Í
fréqiiPtK'e l esquels des fadeurs suivants déterminent s’il y a émi.ssion a) b) c) d)
d'éleetnHis : a) l’intensité de la lumière, b) la durée de l'expitsition
de la plaque à la lumière, c) la conductivité thermique de la plaque, Figure 9.18 Question 18
d) Paire de la plaque, e) le matériau de la plaque?
19. lin proton et un deutéron, cliaeun ayant une énergie cinétique do
10. Dans une exiTéricncc sur le déplticemenl de Ctanpioii. un photon ,3 McV. s’approchent d’une barricrc d'énergie potentielle dont la hau­
de rayon K est diffusé vers Pavant, à 0 à la figure 9.5. Quelle teur Un est 10 MeV. Quelle particule a le plus de chances de traverser
quantité d’énergie l’électron acquiert-il durant celte interaction? la barrière par effet tunnel (La masse d’un deuteron représente Cleu.s
11. Selon l’équation 9.15, le déplacement de Compton est le même fois celle d’im pmion.)
pour les rayons X et pour la lumière visible. Pourquoi peut-on alors 20. I jiquelle des actions suivantes influence Ic plus le coeftieieni do
le mesuivr facilement dans le cas clcs rayons X. mais non dans celui tran.smission T dans le cas d'un électron qui traverse par cttct tunnel
de la Ininière visible? une barrière d’énergie prjicniiclle ; a) élever la hauteur de ia barrière
12. I jc photon A possède deux fois l’énergie du photon fi. a) quantité Un de 1 % ou b) abaisser l’énergie ciiiéii(|Hr, F. de l’élrrinm im ident
de mouveinent de A est-elle inférieure, égale ou supérieure it celle de 1 %?
de 5 ? b) l,a longueur d'onde de A est-elle inférieure, égale ou 21. Pourquoi, dans la partie gauche de la figure 9.15 b), les mini­
supérieure à celle de fi mums des valeurs du P sont-ils supérieurs à zéro ’’
13. Le photon A provient d ’une lampe de bronzage à l’ultraviolet, 22. Supposez que la hauteur de la barrière il’énergie potcntirllc
et le phtiton B, d’un transmetteur télévisuel. Lequel pos.sède illustrée dans la figure 9.1.5 a) est infinie, a) À quelle valeur peui-on
a) la longueur d’onde, b) l'énergie, c) la fréquence et d) la quantité s’attendre pour le roefficLcnt de transmr.s.sion des électrons
de mouvement le.s plus élevées ? s'approchant de cette barricrc ? b) L’équation 9.26 pemiei-elle de
H, Les données fournies dans la figure 9.6 ont été pri.ses quand prédire les résultats que vous attendez ?
on a dirige des raytms X sur une cible de carbone. De quelle manière 23. I.C tableau donne les valeurs |xmr trois situations (le rc.xpérienee
ces données seraient-elles différentes, le cas échéant, .si la cible était sur l’effet tunnel illustrée dans ia figure 9.15. ( ’lasse? ces situations
en soufre plutôt qu’en carbone ? selon la probabilité que rc le tiro n traverse la barrière, en coin
I. 9. Un électron et un proton ont la même énergie cinétique. I jcqucl meiu’ani par la probabilité la plus élevée
possède la plus grande longueur d’onde de De Bn'glie ?
16. a) Si s'oiis doublez l’énergie cinétique d’une particule non rela­ Énergie Hauteur Épaisseur
tiviste, comment sa longueur d’onde de De Broglic varie-t-elle? de l’électron de la barrière do la barrière
b) Qu'arrive-t-il si vous doublez le nxxlulede la vitesse de la particule?
a) F 5F. 7.
17 Les particules non relativistes suivantes possèdent tomes la F. 1?E IJ2
b)
môme énergie cinétique. Classcz-les selon leurs longueurs d ’onde F ’>r 71
c)

EX ER C IC ES E T P R O B LÈM ES

la pellicule. Quelle e,st la longueur d’onde la plus élevée pouvant êtie


www La ,so!uiion se trouve sur le .site Web. à l'adres,se ct-dessous :
photographiée’’ Dans quelle région du spectre é)c< (romiignétiqiir
wvv'w.dlcmcjpawhill.ca/physique cette longueur d’onde est elle située ’
3E. Démontrez que. dans le cas d ’une lumière d’une longueur
SfOlON 9,2 Le photon, quantum de lumière d'onde X en nanometres, l'énergie du phtiton Itf en électrofivolts
1E. Hxpriine? la constante de Planck h en employant les unités élcc- est 1 24U/À.
treinvolts (emlo.sccondes. 4f. l a lumière jaune émise par un éclaiiage «lu .sodium sur l'autoroulc
?f. Une lumière monochromatique (luinicre contenant une seule est la plus brillante è une longueur d’onde di. 589 niii. Quelle c.sl
longueui d’onde) doit être absorbée p.ir une pellicule phtitographique, l’énergie des photons de la lumière a cette longueur d’t'nde’’
donc être photographiée. L’absorption du photon se produira ,si SE, ( ’ombteii de pilotons par seconde le Stileil émet-il ? Poui simplifier
son énergie est égale ou supérieure ù l’énergie minimale de 0.6 eV l’exertMee suppose/, que l'ém ission totale du Soleil au taux de
necessaire pour diss<icicr une molécule de AgBi contenue dans 3,9 X lO’’’ W est à la loncueuj d’onde unique de 5.50 mn
270 Chapitre 9 Les photons et tes ondes de matière

6E. Un laser hélium-néon projette une lumière rouge à une longueur 17L Les physiciens des basses températures ne trouvent pas qu’une
d’onde A, = 633 nm dans un faisceau ayant un diamètre de 3,5 mm et tempénuure de 2,0 mK ( -273,148 0 '^U) est une très basse tempéra­
à une puissance de 5,0 rnW. Un détecteur .situé dans la trajectoire ture. Quelle est la longueur d’onde du pic de rayonnement d’un corps
du faisceau absorbe totalement ec dct tiiet. Cotnbien de photons par noir à celle lempcratuic Dans quelle région du spectre électrom.agné-
Siceonde par unité d atre le détecteur absorbe-t-il ? liquc cette longueur d’onde est-elle située ’?
7E. Une raie spectrale est un rayonnement électromagnétique étnLs I8£. Calculez la longueur d'onde du pie de rayonnement et indiquez
dans une plage de longueurs d’onde suffisamment mince pour être dans quelle région du spectre électromagnétique cette longueur
considérée comme une seule longueur d ’onde. Une de ces raies d ’onde est située pour les situations suivantes: a) le rayonnement
iniitoiuimes en a.siionomic possède une lotigueur d'onde de 21 cm. fossile (un reste proliable du big-bting), dont la température est égale
Qtielle est l'énergie du plituon ,sc tmiivani dans eetle longueur d'onde ? à 2,7 K ; b) votre peau, dont la température est de 34 "C ; c) le filament
8k. A quel module de vite.sse un électron doit il se déplacer pour de tungstène d’une ampoule électrique à 1,8 x 10^ K ; d) le Soleil,
avoir une énergie cinétique égale à celle d ’un photon émis par une dont la température de surface est 5,8 x 10’ K; e) l’expUxsion d ’une
lampe au stnlium et possédant une longueur d’onde de 590 nm '? bombe nucléaire dont la température de la Ixtule de feu est 1 x 10’ K
91 On a tléjà défini le mètre comme étant I 650 763,73 longueurs 19P, Une cavité dont les parois sont gardées à une lempcraturc de
d’onde de la lumière orange émise par une source contenant des atomes 1 900 K est munie d’un petit oiifiee de 1,00 min de iliamètrc. Quelle
do krypton 86. Quelle est l'énergie des photons de cette lumière ? est la {Tius.sance émergeant de l’orifice?
lOP. Dans des conditions idéales, l'ctrc htiniain peut voir une lumière 20P. L’ne cavité dont la température est émet du rayonnement avec
.«i la rétine de son leil peut ah-sorher une longueur d’onde de 5,50 nm une puissance de 12,0 inW. Si la température augmente à 2 I \ , quelle
à un taux aussi faible que 100 photons par seconde. À quel taux la est la puis.sance rayonnée?
rétine absorbc-t-cllc alors l'énergie ? 2IP. Le Soleil peut être considéré comme un corps noir dont la
IIP Une ampoule électrique d’un type particulier émet une lumière température de surface est T = 5.8 x 1(>' K. À l'aide de la loi du
morKK.-hromatique d’une longueur d’onde de 630 nm. fin lui fournit une rayonnement de Planck, calculez la puissance par unité de surlace émise
puissance éleclrique de 60 W ; l'ampoule en convertit 93 % en énergie par le Soleil dans l’intervalle étroit de longueurs d'onde compris
lumineuse. C'omhien de photons émet-elle durant sa vie de 730 h ? entre 550 nm et 552 nni. (Intliee: Considérez que l’intervalle de
12P. Un faisceau émergeiint d’un laser à argon de 1,5 W(Â = 31.5 nm) longueurs d'onde est a.sscz petit pour t]iie la radiance spectrale soit
environ constante.)
possède un diamètre r/de 3.0 mm. Ce faisceau est concentré par un
système de lentilles ayant une di.siance focale équivalente f, de
2.5 mm. Le faisceau conccntié touche un écran qui l’absorbe com­ SEGION 9.4 l'effet photoélectrique
plètement et sur lequel d forme une figure de diffraction circulaire
dont le distjue central possède un niyon N deterininé par 1,22/^ À/</. 22k. a) L’énergie ininiinale nécessaiie pour éjecter un électron du
Il peut être démontré que 84% de l’cncrgic incidente aboutit dans sodium est 2.28 cV. Le sodium pré.sente-t il un effet photoélectrique
ce di.sqiie central. À quel taux l’écran ahsotbe-t-ii les photons dans le dans le ctis d ’une lumière rouge d'une longueur d'onde A = 680 nm .’
di.S(.|ue central de la figure de diffraction ? b) Quelle est la longueur d’onde de seuil pour l’émission plioioélec-
trique du sodium ? À quelle couleur correspond-elle ?
13P Une lampe à ultraviolet cinet une lumière d’une longueur d’onde
de KKl nni, avec une puissance de 400 W. Une lampe à infrarouge 23E Vous cherdicz un clément chimique à installer dans une cellule
émet une lumicrc d ’une longueur d'onde de 700 nm, également avec plKHoélcclnquc qui fonctionnera avec de la lumière visible Lesquels
une puissance de 4(K) W. a) Quelle lampe émet des photoas au taux des éléments suivants sont appropriés (le travail d’extraction est entre
le plus élevé et b) quel est ce taux ? parenthèses): tantale (4,2 eV), tung.stène (4,5 eV), aluminium
14P. I In satellite en orbite autour de la Terre maintient un panneau de (4,2 eV), baryum (2,5 cV ), lithium (2,3 eV) ?
cdiule.s .solaires ayant une superficie de 2,60 m- perpendiculaire à la 24k. Les travaux d'exiraclion pour le potassium et le césium sont
direction des rayons du Soleil. L’inlensité de la lumière au panneau respccUveineni de 2,25 eV et de 2,14 cV. a) Y aura-l-i! un effet
est de 1,39 kW/m’. a) Quelle est la puis.sance solaire incidente sui le photoélectrique dans l'un de ces éléments en pré.sence d'une lumière
panneau'.’ b) A quel taux le panneau absorbc-t-il les photons d'une longueur d’onde de 565 nm ? b) d ’une lumière d’une longueur
solaires'’ SiipjKiser.que le rayonnement solaire est monochromatique d ’onde de 5 18 nm ?
et i|ii’il Hune longueur d’onde de 550 nm, et que tout le rayonnement 25E Une lumière éclaire une surface de sodium, provoejuant une
solaire atteignant le panneau est absorbe, c) Uombicn de temps émission photoélectrique. Le potentiel d’arrêt des électrons émis
faudrnil-il au panneau pour absorbei une « mole de phoions » ? est .5.1) V : le travail d’extraction du sodium est 2,2 eV (Quelle est la
15P. Une lamix: au sodium (À 589 nin) de 100 W émet de l'cncrgic longueur d'onde de la lumière incidente?
(iiiirorniémem dans toutes les direeiit'ns. a) A quel taux emei-clle ses 26E. Déterminez l’énergie cinétique maximale des électrons émis pat
photons ’’ bl A quelle distance de la lampe un écran (parfaitement un certain matériau si le travail d ’extraction de cc matériau est
□Ksorbniif) alwoilx;ia-l-il les photons dont le flux est de 1.00 photon/ 2,3 eV et si la fréquence du rayon incident est 3,0 x 10*' llz
(cni^ •s) ? c) Quel est le flux de pilotons (photons par unité d’aire par unité 27E Le travail d ’extraction du tungstène est 4,,50eV. C’ali ulez le
de temps) sur un petit écran ,se trouvant <i 2.00 lu de la lampe’’ www module de la vitesse des électrons les plus rapides émi.s par une .sur
face de tungstène quand celle-ci est éclairée pat une lumière dont les
SECflON 9.3 Lê rayonnement du corps noii pilotons ont une énergie de 5,80 eV.
16L t n 1983. le saleiiitc Infmicd Astronomical Satellite (IRAS) a 28P. a) Si le inivail d ’extraclinn d’un certain métal est 1,80 eV, (|ttel
ob.scrvé un nuage de particulc.s cntouiant l'étoile Véga. La longueur est le potentiel d’arrêt ptnir des électrons éjccté.s de ce métal en
d’onde du pie de rayonnemeni était de M l'm Quelle est la fcnipératuro pré.sence d’une lumière d'une longueur d’cmde de 4(X) nm ? b) Quel
du nuage de particules ? Supposer que le nuage se comporte comme e.st le module de la vites.se maximale des éleclroiis émis?
un corps noir.
E x e r c ic e s e t p r o b lè m e s 271

29P Une lumière d’une longueur d’onde de 200 nm éclaire une surface 39E. Un certain faisceau de rayons X possède une longueur d’onde
d’aluminium. II faut 4,20 eV pour éjecter un électron de l’uluminium. de 35,0 pm. a) (Quelle e.st la fréquence corrc.spondantc ? Calculez
Quelle est l'énergie cinétique des électrons émis a) les plus rapides b) l’éneigie et cf la quantité de laouveinent des photons.
et b) les plus lents? c) Quel est le potentiel d ’arrêt dans cc cas? 40P Des rayons Xdoni l o photons omuiK longueur d'onde de Ü.OIOimi
d) Quelle est la longueur d'onde de seuil pour l’aluminium? fcom diffusés à un angle de 180' par une cihle contenant dej; élct ii iins
30P longueur d’omlc de seuil de l’argent est 325 nni. Déterminez faiblement li(îs. Quels sont a) le dépluccmcnl de Compton, bl k varia
l’ciicrgic cinétique maximale de.s électrons éjecté d’une surface bon correspondante dans l’énergie du photon, c) l'éneiYie cinétique
d'argent par une lumière ullravioleite d’une longueur d 'onde de 2,54 nm. de l’éleetmn émis et cl) la direitiion du iiiouvemeut de féleiiuin
31P Un sütclUte en orbite peut se charger par effet photoélectrique 4IP Démontrez, en analysant une collision entre un photon et
quand la lumière du Soleil éjecte des électrons de sa surface. Les un électron libre (à l'aide de la mécanique relativisiej, qu'un piioion
ingénieurs doivent donc concevoir les satellites de façon que cet effet ne peut pas transterer toute son énergie à un élecimii liliie (jiiiiv
soit réduit au minimum. Suppose/ qu'un satellite est couvert de philine, disparaître)
métal ayant un travail d'extraction très élevé 1<I> - 5.32 eV) TnHive/. 42P Dc.s rayons gamma dont les photons ont une energie de 0,x 11 m p V
la plus grande longueur d’onde de lumière solaire incidente pouvant sont dirigés sur une cible en alumimnm et diffiises' dunv diverses
éjixter un électron du platine. directions par les électrons faiblement liés qui s’y iiouvem a) Quelle
32P. Au cours d’une expérience sur l’effet photoélectrique où vous est la longueur d ’onde des rayons gamma ineideiits? b) Quelle rsi
utilisez une surface de sodium, vous déterminez un potentiel d’arrêt la longueur d'onde des rayons diffusés à ÙO.O' du rayon iiiridrni ?
de 1.85 V pour une longueur d'onde de .3tK) nm et un potentiel d’arrêt c) Quelle est l’énergie des photons diffusés dans crtie direction
de 0.8211 V pour une longueur d’onde de 400 nm. En fonction de 43P. Calculez la longueur d’onde de Compton a) d un électron et
CCS données, déterminez a) une valeur pour la constante de Planck, b) d'un proton. CXiellc est l’énergie des photons d'une onde electro
b) le travail d’exlraelion <t> pour le .sodium et c) la longueur d ’onde magnétique ayant une giandciir égale à la longueur d ’onde de
de seuil A» pour le sodium. Compton c) de l’éleciroii et dl du proton ?
33P Une lumière d ’une longueur d’onde de 491 nm éclaire une 44P. Quel est le déplacement de Compton maximal dans le cas d’une
surface; le potentiel d’arrêt pour les électrons émis par ccitc surface collision de Compton entre un photon et un proton libre?
est 0,710 V. Quand un donne une nouvelle valeur à la longueur 45P. Quelle augmentation relative de longueui d'onde est produiit
(fonde incidente, le potentiel d'arrêt devient 1,43 V. a) Quelle est par une perte de 75% de 1 énergie du photon dans une cnIHvion
celte nouvelle longueur d’onde '? b) Quel est le travail d'extraction photon électron libre ? www
de la .surface? www
46P. Calculez la variation relative d’énergie du photon p<'ndniit une
Vers 1916. Robert A. Millikan trouva les données du potentiel collision (ximmc celle de la figure 9.7 lorsque ij>— 90" et dans le ( as
d’arrel p<Mir le lithium au etxjrs de .ses expériences .sur l’effet photo­ d’un rayonnement dans a) le tour à micro ondes, avec A = 3,0 cm;
électrique. h) la lumière \ isihle, avec X ^ .500 nm •. c) la zone dc.s rayons X,
avec A, = 25 pm et d) la zone des rayons gamma, avec un phorrni
Longueur gamma ayant une énergie de 1.0 McV. c) Que concluez voas quant à la
d’ordc (nm) 433,9 404,7 365,0 312,5 253,5 possibilité de détecter le déplacement de Compton dans ces diverses
Potentiel régiorcs du spectre électromagnétique, en ne tenant compte que de la
d’arrêt (V) 0.55 0,73 i.tw 1,67 2.57 perte d’énergie au cours d'une rencontre photon-éiceiron uiii«|ue?
47P Un électron de masse m et ayant une s îies,\c de iiHulule v (( heurte n
Tracez un graphique, comme celui de la figure 9.4 (qui est pour le un photon de rayon gamiiia ayant une ¡'nergir iiihiale Ii/ q. mesnréc
stxlium), à l’aide de ces données, puis utilisez-le pour déterminer dans le référentiel du lahoraioLre. I e photon est diffusé dans la d ira -
a) la constante de Planck et h) le travail d’extraction du lithium. tion du mouvement de l’électron. Montrez que l'énergie du photon
35P. Supposez, que le rendement d ’une surface de césium (avec un diffusé, telle qu’elle est mesurée dans le référentiel du lahoraroiir. est
travail d’extraction de 1.8(' eV) est de 1,0 x tO"'*', donc qu’en
moyenne, un électron est éjecté à tous les lO“'' photons qui frappent -I
2hh [ T t 3
la surface. Quel .serait le courant d’éleclmns émis d’une telle surtacc £ = hh 1 4
me- \ I
SI clic était éclairée par une lumière de 600 nm produite par un laser
de 2.00 mW cl que tous les électrons émis participaient au courant?
36P On dirige des rayons X d'iiiie longueur d’onde de 71 pm sur une 48P Démontrez que A /’/fi. la perte relative d’cnergtc d’un phOton
leuille d'or, de sorte que les éleetm ns fortement lies sont éjectés durant une collision avec une particule (k- mas,sc m, est dctci mince par
de,s atonies d or. [,es élca:trons émis décrivent alors des Iraicctoires
circulaires (le rayon r dans un champ magnétique homogène B. où
AF.
Ri ~ l.fifix 10 ■‘T m. ITétermineza) l'énergie cinétique maximale —TT — r (1 —( lis (.4j1.
de ces électrons et b) le travail effectue pour les extraire des atomes d'or.

où t est l'énergie du plu'lon ineidenr f est la fréquence du photon


SECTION 9.5 l'effet Complon diffuse et </>est dêFini comme dans la figure 9.7.
37f ( In (linge une lumière d'une longueur d’onde de 2.4 pm sur une 49P. Imaginez une collision entre un photon de la gamine des rayons V
cible contenant des électrons libres, a) D éterm ine/ la longueur ayant une énergie initiale de 50,0 keV et un électron au repos. Au
d'onde de la lumière diffusée à 30'’ de sa riirix-iion initiale b) Faites cours de cette collision, le photon est repoussé vers I’arricrc et Fclec
de ntême dans le etts d'un angle de diffusion de 120'’. iron est projeté vers l’avanl. a) Quelle est l’énergie du photon dltfusé
38E a) Quelle est la quantité de mouvement d'un photon dont vers l'arncrc ' b) Quelle c.sl l’énergie cinétique de Féieeiron ’
l'cnergic est égale à l’énergie au repos d'un électron'’ Quelles sont SOP Quels seraient a) le Jcplacemcnt de Ccmipton. bi le déplacenieni
b) la longueur d’onde et c i la fré<|iience du rayonnement correspondant de Compton relatif et t ) la variation de l’énergie du photon d'une
272 Chapitre 9 Les photons et les ondes de matière

lumière d’une longueur d’onde de 590 nm diffusée par un électron 65R Des ions de sodium ioni.sés une fois sont accélctés à paitir du repos
libre initialement au repos si cette diffusion était à 9<r du faisceau par une différence de ptitentiel de 300 V. a) Quelle est la quantité de
incident '? d) Calculez les mêmes grandeurs pour des rayons X dont niouvcrnciit acquise par un de ces ions? b) Quelle est sa longueur
l’énergie des photons est de 50,0 keV. d’onde de De Broglie ?
51P. Quelle est l'énergie cmétigue maximale des élections éjectes 66P. a) Un photon el un électron ont tous deux une longueur d’onde
d’une mince feuille de cuivre par diffusion de Compitm dans le cas de 1,00 nm. Quelle est I'cnergic du photon et quelle est l’énergie
d’un faisceau de rayons X incident de I / J keV cinétique de l ’électron ? b) Refaites l excrciec pour une longueur
S2P. Dérivez l’équation 9.15. équation du déplacement de Compton, d’onde de 1.00 fm.
en fonction de,s équations 9.12,9.13 et 9 .14 et en éliminant v et fi. 67P. Le gros accélérateur d’électron.s de l’Université Stanford fournit
33P. À quel angle un photon de 200 keV doit-il être diffusé par un un faisceau d ’électrons ayant une énergie cinétique de ,50 GeV.
électron libre pour qu’il perde 10% de .son énergie? Les électrons de cette énergie possèdent de petites longueurs d’onde,
S4P. Démontrez que, lorsqu'un photon possédant une énergie E est appropriées |.xiur sonder les fins détails de la structure nucléaire
diffusé t>ar un électron libre au rcixis, l’énergie cinétique maximale par diffusion. Quelle longueur d'onde de De Broglie un électron
de l'électron qui s'éloigne est déterminée par de 50 GeV a t il ? CJuelle comparai.son peut-on établir cntie cette lon­
gueur d'onde et le rayon d'un noyau moyen, qu'on considère comme
étant de 5,0 fm, appmximali vement ? (Avec cette quantité d'éneigie. vihjs
fr +I nic^/J
? /-1 * pouvez utiliser la limite ultrarclativiste entre la quantité de mimvcmcnt
E
et l’énergie, à s a v o i r —E/c. Cette relation, utilisée pour la lumière,
s’applique quand l’énergie cinétique d’une particule est beaucoup
plus grande que son énergie au repos, eomnx; dans le cas present.)
SFCnON 9 7 les életlrons et les ondes de moliere
68P. Ernest Rutherford a découvert l'existence du noyau atomique
55E. En faisant appel aux équations non relativistes de la quantité en 1911 ; il a adéquatement intciprété quelques expériences au cours
lie mouvement et de l’énergie cinétique, dcmoiitrcz que la longueur desquelle.s un faisceau de particules alplia était diffusé par une feuille
d’onde de De Broghe en nanomètres peut s’exprimer .sous la forme d’atomes de métal, métal qui jxHJvaitêire de l’or, par exemple, a) Si les
A, — l.226f'\/lK , où K est l'énergie cinétique de l’électron en paiticules alpha possédaient une énergie cinétique de 7.5 McV.
éicctronvolts. quelle étau leur longueur d'onde de De Broglie'' b) Aurait on dû tenir
56t. Une balle de fusil de 40 g se déplace à 1 000 m/s. Bien qu'elle compte de la nature ondulatoire des particules alpha ini idemes dans
soit trop grosse pour être considérée comme un onde de matière, l’interprétation de ces expériences ’’ Une particule alplia possède une
déterminez ce que l'équation 9.17 permet de prédire quant à sa ma.sse de 4,00 u (unités de masse atomique), et la di.slance minimale
longueur d ’onde de de Broglie. à laquelle elle sc rappr(x.’hait du centre du noyau était de .30 fm, environ.
(La nature ondulatoire de la matière a été étudiée plus d’une décennie
57F. Dans un téléviseur ordinaire, une différence de potentiel de 25,0 kV
accélère des électrons. Quelle est la longueur d’onde de De Broglie après la réalisation de ces expériences cniciales.)
de tels électrons? fLa relativité n’esi pas necessaire.) 69P Une panieiilc non relativiste se déplace à une vites.se ayant un
module icpnSsentant trois fois celle d'un électron. Le rapport de la
sut. Calculez les longueurs d’onde de De Broglie a) d'un électron de
longueur d’onde de de Broglie de la particule à celle de l’électron est
I ,<X) keV, h) d’un photon de 1,00 keV et c) d ’un neutron de 1.00 kcV.
de 1.813 X lO '’’. En calculant sa masse, identifiez la particule.
59P. l.a longueur d ’onde de la raie sjrectrale jaune du sodium est
70P. Le pouvoir de résolution qu'un microscope peut atteindre n’est
de 590 nm. Quelle est l'énergie cinétique d'un électron qui a une
limité que par la longueur d'onde utilisée, c'est-à-dire que le plus
longueur d’onde de de Broglie égale à celle longueur d’onde ?
[x:tit objet qu'il peut permettre de distinguer possède des dimensions
60P. Si un proton possède une longueur d’onde de De Broglie de
approximativement égales à sa longueur d'onde. Supposez qu’on
100 frn. a) quel est le module de sa vitesse et b') pour quelle différence
veuille «voir» l’inténeurd'un atome. Si on suppciscquc l’atome a un
de potentiel devrait-il être accéléré pour atteindre ce module de vitesse
diamètre de КЮ pm. cela signifie qu'il faut pouvoir bien voir une
à parlir du repos ?
largeur de 10 pm, environ, a) Si on utilise un microscope électronique,
61P. Les neutrons en équilibre Üiermique avec la matière <ml une énergie quelle énergie minimale l’électron doit-il avoir'.’ h) .Si on utilise un
émétique moyenne de (3/2)AT où k est la constante de Boltzmann microscope optique, quelle énergie minimale le photon doit-il avoir?
(voir l’annexe B) et T. qu'on peut considérer comme étant 300 K. c) Quel microscope semble le plus adéquat ? Pourquoi ?
est la teiiipéiatiire des neutrons, a) Q ucIIl' est l'énergie cinétique
71P Quelle différence de |x>tcnticl faudrait- il appliquer aux électrons
moyenne de tels neutrons b) Quelle est la longueur d'onde de
d ’un microscope électronique pour que celui-ci ait le pinivoir de
De Broglie correspondante
résolution atteignable avec des rayons gamma de 1(10 keV (Voir le
62P Un électron et un photon possèdent tous deux une longueur problème 70,')
d'onde de 0,20 nm. Calculez a) leurs quantités de inoiivemeni el
b) leurs énergies cinétiques.
SECTION 9,8 l'équation de Schrôdinger
63P. a) L n photon pos.sède une énergie de 1.00 eV. et un électron
pjsscdc une énergie cinétique ayant la même valeur. Quelles sont 72t. a) Soit n U b ih. un nombre complexe, où a et h sont des
leurs longueurs d'o n d e? b) Refaites l'e-xercice pour une énergie nombres réels (positifs ou négatifs). Démontrez que le produit im*
de 1,00 <IcV. vr*rM est toujours un nombre réel positif b) Soit m = о f iti. on autre
WP inuigini“'/ un ballon rempli d'hélium à la température et à la iKimbrc œmplexe. Démontrez que 1иш1 — 1и1 hwl.
pression amlnaniov t alculez a) la longueur d'onde de de Broglie 73P Démontrez que l’équation 9.21 est effectivement une solution
moyenne des atonies d'héliiiin et h) la ilisiance moyenne séparant les de l'éqiiation 9.20 si on remplace ^¡'(x) et sa dénvee .seconde d.ans
atomes dans ces conditions. L'énergie cinétique moyenne d'un atome l’équaiion 9.20 el si on noie qu’il en résulte une identité.
est égale ù (3/2)kT, où k est la constante de Boltzmann, c) Peut-on 74P a) Écrivez la lonction d’onde tfri.r) présentée dans rét]uation 9.23
considérer les atomes conunc des parlit ules dans ces comlitions sous la forme ф(.\) a + ih, où a et b sont des grandeurs réelles
E x e r c ic e s e t p r o b lè m e s 273

(Supposez que est réelle.) b) Écrivez la fonction tl’imde dépendante particule est parfaitement inconnue. Scion le même principe,
du temps Ÿ(.r, i) qui correspond à ijKx). l’opposé est également vrai, c ’est-à-dire que, .si la position d ’une
75P Dcnioutrcz que le nombre d’onde k dans le cas d’une particule particule est exactement connue, Av = 0, rinccrtitudc sur sa quantité
libre non relativiste d’une masse m peut s’écrire ainsi de mouvement est intlnie.
Imaginez un cas imermédiairc. dans lequel la pusiiiuii d'uiir,
2гГу/2тК particule est mesurée, non avec une précision ultime, mais avec une
к =
incertitude de aJ2^, où à est lu longueur il’uiule île 13e hroglie île la
où K csl l’énergie cinétique de la particule. particule. Démontre/ que l’iiu eiiiiuile sur la quamiié tic minivi-mcm
76P. Uénюntгez que [xj/p = № 1-, en mettant en relation \1/ et Ч' comme (mesurée simultaiiéniem) esi égale k la quantité de mouvcini'.ni
dtins l’équation 9.18. Autrement dit. démontrez que la densité de elle-même. c'esi-îKliie que i/3, —p . Dans ces circoiiMui ices. .'crie/ voiiiv
probabilité ne déixtrui pas de la variable de temps. surpris qu'une quantité de mouvement mesurée soit nulle’.’ qu’elle
/7Р. La fonction tK.v) présentée dans l'équation 9.23 décrit une soit de 0,5/3 ? lie 2/> ’’ tic 12/»
particule libre, pour laquelle on présume que (/(л ) = 0 dans l’équa­ 84P Dans le chapitre 10, vous déiouvilrez que le.v élcuroris ne
tion de Schrodinger (équation 9.19). Suppose/, maintenant que peuvent se déplacer sut des orbiies dériiiic.s h riiitériair de l'atome
L'(jr) = ff,| = une constante dans cette équation. Démontrez comme le foin les planètes du sysièmc solaire Poiii savoir pourquoi,
que l’équation 9.2.3 est toujours une solution de l’équation de on peut essayer « d’ohservcr » un tel clct tron r,n orhile il l'aide d'un
Schriklinger, où microscope opiique jroiir mesiiicr la po.siiloii oriiitalc pré.suniéc- de
l’électron avec une précision, pai'exemple, de H) pin (un atome iiiovcii
к - J ï m Œ - (/ni. a un rayon approximatif de 100 pm). La longueur d’onde de In lumière
b
utilisée dans le microscope déviait eue d’env iron 10 pm. a) Quelle
donne maintenant le nombre d’onde к de la particule, www serait rénergid des photous de cctic lum ière? b) Oucllc quantité
78P. Supposez qu’on a posé A = 0 dans l 'cquation 9.21 cl qu’on a d'énergie un tel photon doiinerait-il à un l'iei irim dans une collision
renomme П comme étant (^ue décrirait alors la fonction d'onde frontale? c) Que vous indiqueiu ic.s icsullals sui la possibilité
résultante'.’ Comment la figure 9.13 serait-elle modifiée, le cas « d ’ob.servei » un électron atomique il deux ou à plusieurs points .sur
échéant ? son orbite présumée? (Indice ■les élta lrons jiéiipheriques sont lies à
79P. Dans l’équation 9.22, gardez les deux termes en établissant que l’atome par des énergies de »euleinent qnelqurs élri imnvolis.)
A — f? = i/ tq. L'équation décru alor.s lu superposition de deux ondes
fie matière d’égale amplitude sc propageant dans des directions
opposées. (Rappelez-vous que ce sont des conditions néces.saires à SECTION 9.101'efMtunnel
l’cxislcncc d’une onde stationnaire.) a) Démontrez que la valeur de 85P. Ln pnrton et un deuteron (ce dernier possède la même cluirge qu'un
f)l^ est altirs déterinincc par proton, mais a deux fois sa mas.se) atteignent une barrière d'cncrgic
potentielle d'une épaisseur de 10 fin et d’une hauteur de 10 MeV,
№(.v, riP = 2t/rô(l -t- cos (2Cv)|. Chaque particule possède une énergie ciiiéiique initiale de 3,0 MrV
b) Représentez graphiquement cette fonction et démontrez qu’elle a) Quel est le coefficient de transmission de chacune >b) Quelles si mt
décrit le carré de l’amplitude d’une onde de matière stationnaire. leurs énergies cinétiquc.s respectives une fois la barnero traversée
c) Démontrez que les meuds de cette onde stationnaire sont situés à (en supposant qu’elles In traversent)? c) Quelles sont leuvo énergies
cinétiques respectives si elles sont réfléchies par la barrière ?
jr = (2n + I )(|Л). où и = 0. 1,2 . 3... 86P. Imaginez une barrière d'énergie iHUMiitiella samhlahlc à celle
de la figure 9.15 a), mais doni la hauieui ffpesi (3,0 eV ei l'épi.sseur I
et que Àest la longueur d’ontlc de De Broglie de la particule, d) Écrivez
est 0,70 nm. (.Juelle est l’énergie d ’un élei non incidciir doni 1r
une expres.sion pour exprimer les positions les plus probables de la
coefficient de tninsmission esi 0,001 il?
particule.
87P Prenez la situation d ’effet tunnel illustrée dans I exemple 9 8.
SKTIOH 9.9 Le principe d'interltlutle de Heisenberg Quelle variation relative dans le coefficient de muismis.sion T tésiilie
d’une variation de l,09è dans a) la hauteur de la banièie. tq répaisseiii
BOE T.a figure 9.13 démontre que. en raison du principe d’incertitude de la barrière et ci l'énergie cinétique de réleciron incideiii ?
de Heisenberg. il est impossible d ’attribuer une coordonnée r à la 88P a) Supposez qu’un faisceau de prolon.s de 5,0 cV atteint une
position de l’élcclron. a) Pouvez vous y attribuer une crxirdonnée y barrière d'énergie potentielle d'im e hauteur de 6.0 cV et d une
ou - ■’ (.Indice: la quantité de mouvement de l’électron n'a pas de épai.ssciir de 0.70 nm à un taux équivalant à un courant dc 1 OOU A
comiiosanle y ni г.) b) Décrivez l'onde de matière en titiLs dimension.s. Coinhicn de U'inps liiudrail il attendre (en moyenne) pour qu un
BIL Imaginez que vous |ouez au baseball dans un univers (qui n’est pas proton traverse la barrière ? b) Combien dc temps faudrait il attendre
le nôtre !) où la constante de Иапск est 0.60.1 ■s. CTuellc serait Г incerti­ si le faisceau était constitué d’électrons au lii ii tli- proioiis'’
tude quant à la position d’une halle de O.fiO kg se déplaçant à 20 m/s le 89P 1 inc voiture dc 1.500 kg roulani à 20 m/s s approche d’une pente
long d'un axe si l’incntitiKle quani au пн diilc de la vites.se est de 1.0 tn/s ’ de 24 ni lie haiiteiii cl de 30 m de longueur. Bien que la voiture Ct
82E L imertinifle quant à la position d ’un électron donné est de la pi'iitc .soient trop grossc.s pour être constdcrces comme des ondns
50 pm, ce qui représente environ le rayon d ’un atome d’hydrogène. de m atiia', déterminez ce que l'équation 9.2(> permet dc prédire
Quelle est l’incertituile minimale quant à tiMite mesure simnllanée de quant au eocflicicnt dc transmission dc la voiture, comme si elle
la quantité de mouvement de cet éleetmn '? dcvaii iiHinter la pente par un effet tunnel, à la manière d une onde
83® La figure 9.13 illustre un cas où la quantité de mouvement i\ dc matière, ronsidérez la pente comme une barrière où l éncrcie
d’une pariiculc est fixe, de sorte que Др, 0 , alors, selon le princiixr potentielle r,M gravitationnelle.
d’incertiiudc de Hein.senberg (équation 9 24), la posilinn лг de la
10 LiO mécanique
ondulatoire

Cette specteculoire image informatique fut réalisée en 1993 au Centre de recherche de IBM à Almaden, en Californie.
Chacun des 48 pics qui forment le cercle marque la position d'un atome de fer sur une surface de cuivre spécialement
préparée. Ce cercle, d'un diamètre approximatif de 14 nm, est appelé corailquantique.

Comment ces atomes


ont-ils pu former un cercle,
et que sont les ondulotions
piégées dons le corail?
lo 'qjonw se trouve dons ce éopitie.

^ i
ÎS. â.-i
10.2 Les ondes dans les cordes et les ondes de matière 275

10.1 La structure de l’atome


A u dcbul du XX' siècle, personne ne sav ait com m cnl Ic.s electrons étaient répartis dans
un atom e, com m en t ü s sc d ép laçaien t, com m ent les atom e« ém ettaien t île la lum ière
ou en absorbaient, ni m êm e pouixjuni les atom es étaient stables /sans e rs contiaîssiinccs.
il était im possible de ctiiTiprendre com m ent les atom es sc lient pour form er tics molécules
ou s’em pilent pour form er des solides. I.e résultat tic cette ignorance était que les ba.scs
de la chim ie (y com pris celles tie la bioclum ic. sciciiec stiiis-jaccnle à lu cm nisréhcnsion
de la nature m êm e de la vie) constiiliaient ]ilus nu mtiin.s un mystère.
En 1926, le développem ent de la physitjue tjiiaiuit|ne |ierinii d e com |)tcn d rc tous
CCS phénomènc-s, et bien d ’autres. Ses prém isses de base sont que les élearnn,s, les pnunns
e t tous les types de particules en m ouvem ent doivent cire considérés couune des ondes
de m atière, dont les m ouvem ents sont régis par l ’équation de bchrôdinccr. Bien que la
th é o rie q u an iiq u e s ’ap p liq u e ég a lc m cn l à des o b je ts m a cro sco p iq u es, il est in u tile
d ’étudier les balles, les automobile.s, les planèlc.s cl d ’autres objets de cc type du point
de vue quantique. En effet, |x u ir ce qui e.sl de cc.s objets m assifs cl Iciils. la ph y siq u e
new tonienne et la phy.siquc quaniique abouiisseni aux m êm es lésiiliais
A v an t d ’a p p liq u e r la p h y siq u e q u a n tiq u e à la siru c iu ie ato m iq u e , vou s d evez
acq u érir certain es notions en ap p liq u an t la théorie q uantique à q u elques cas sim ples
C es « problèm es qiianttques » pourront vous .sembler artificiels, m ais vous cousuuerez
q u ’ils fourniront une ba.se solide à la com préhension d ’un problèm e très réel qui .sera le
sujet d 'an aly se de la section 10.8, soit la structure de ra tn m c d ’hydiogcnc

10.2 Les ondes dans les cordes


et les ondes de matière
D ans le chapitre 2, on a vu q u ’on peut générer deux types d ’ondes dans une corde tendue
Si la corde a une longueur q u ’on peut considérer com m e infinie, on peut y générer une
onde progressive de pratiquem ent n ’im p o rlc quelle fréquence. C ependanl, si la corde
tendue a une longueur finie, peut-être parce q u ’elle c.st solidem ent m aintenue à sc.s deux
exlrémiics, on ne peut y générer que des ondes statiomain s ; de plu-s, ces ondes siaiiotuialies
ne peuvent avoir que des fréquences discrètes. AiitR-ment dit, si on confine l’onde dans
un espace fini, on o b se rv e une quantification du m o u v em en t, e t l ’ap p aritio ii d 'é ta ts
discrets pour f o n d e , q u ’on appelle états stationnaires, chaque état ayant une fréquence
bien définie.
C elle o bserv atio n s’ap p liq u e aux ondes de toutes sortes, y com pris aux ondes de
m atière. C e p en d a n t, p o u r le s o n d es d e m a tière, il est p lu s sim p le de irav u illei avec
l ’énergie C de la particule correspondante q u ’avec la fréquence / de fo n d e . D ans tous
les cas qui suivront, on s ’intcicssera à fo n d e de m atière as.six iéc à un électron, maia les
résultats s'appliqueront à toute onde de m atièrr confinée
Im aginez une onde d e m atière assctciée à un éici Iron sc déplaçant dan.s la direction
de f axe des x jxrsitifs et ne subissant aucune forer Résultante - une pitrtu uU' dite Ithre.
L ’énergie d ’un tel électron peut avoir n ’im porte quelle valeiti, tout com m e mu- niule .'le
p ro p ag e an t d an s un e co rd e ten d u e d e lo n g u e u r in fin ie peut av o ii M 'im |io iir t|u r lle
fréquence.
Im aginez ensuite fo n d e de m atière associée à un électron, peut être un électron de
valence (le m oins fortem ent lié), d 'u n atom e de sodium I In k l électron (m ainlem i dans
l’atom e p ar la force de C oulom b qui le retien t au to u r du n o yau chargé positivem eni)
n 'est pas une particule libre. Il peut seulem ent oxistcr dans un ensem ble d états discrets,
chacun possédant une énergie discrète C e sont des étais stationnaires C elte «iiiiation
ressem ble beaucoup aux états stationnaire.« et aux iféquences quantifiées pns.sibles dans
une corde tendue d e longueur finie. D onc, dans le ca.s d une onde de m atière, com m e
dans le cas des ondes de toutes les sortes, on peut énoncer un principe d e eonfinrnicnt.

► Le L-onlinctnent d'une onde mène à la qtianiificiuion -< 'est-à-din; à l’rxist( iicc d’Ants
stationnaucs possédant des énergies discrètes, l 'oncle ne |ie iu posséiW-r tp ir ces éncrgic.s
276 Chapitre 10 U j mécanique ondulatoire

V ^ -oo V —0 V -> 10.3 Les énergies d'un électron piégé


Les pièges à une dimension
L , ^
On peut exam iner ici l’onde de m atière associée à un électron confiné d aa s une région
X= 0 L
lim itée de l’espace. O n le fait par analogie avec les ondes stationnaires dans une corde
Figure 10.1 Les éléments d’un « piège » tendue de longueur finie, parallèle à l’axe des x ci confinée entre des .supports rigides.
idéalisé conçu pour confiner un électron Piii.squc les supports sont rigides, les deux extrém ités de la corde constituent des nœ uds,
dans le cylindre central. On considère
ou des points où la cordc est toujours au repos. La corde peut contenir d 'au tres nœ uds,
que les cylindres de longueur semi-
mais ces deux nœ uds doivent loujtiurs y être présents, com m e le m ontre la figure 2 .2 1 .
infinie des extrémités ont un potentiel
Les étals, ou m odes d ’oscillation des ondes stationnaires de la cordc, sont ceux qui
négatif infiniment grand et que
le cylindre central a un potentiel nul. cxi.stent quand la longueur I. de la corde est égale à un nom bre entier de deini-lnngucurs
d ’onde. C 'e s t dire que la corde peut seulem ent présenter des états pour lestiucls

HA
— , p o u r« = l, 2, 3 ,.... ( 10. 1)

C h aq u e v aleu r de n décrit un étal de la corde o sc illa n te ; en p h y siq u e q u an tiq u e, on


appelle n o m b re q u a n tiq iie le nom bre en tier >i.
k chaque état de la corde perm is p ar l ’équation 10. 1, le déplacem ent transversal de
tout point X sur la corde est d éten n in é par

/tlTT \
»’„(.x) = A sin y Pi’nr « = 1, 2, 3...... ( 10.2)

où le nom bre quantique h représente le m ode d ’oscillation, et A dépend de l’insliint où


vous ob.scrvez la corde. (L’équation 10.2 est une version sim plifiée de l'éq u atio n 2.47.)
O n constate que. pour toute valeur de n et à tout instant, il y a un point de déplacem ent
nul (nœ ud) à.v - 0 cl à x = L, com m e il se doit. La figure 2.20 m ontre des expositions
photographiques prolongées d ’une telle cordc tendue quand n = 2. 3 et 4.
O n peut m aintenani aborder les ondes de m atière I.e prem ier problèm e consiste à
confiner un électron qui sc déplace sur l’axe des x pour q u ’il dem eure à l’intcrieur d ’un
segm ent fini. 1 ^ figure 10.1 illustre un p /èg e à électron unidim ensionnel concevable. Ce
piège est co n stitu é d e deux cy lin d res qui se p ro lo n g en t à l ’in fin i d an s une directio n ,
chacun ayant un potentiel électrique s’approchant de —’x. ; entre eux se trouve un cylindre
creux de longueur L, dont le potentiel électriq u e est nul. On plarÆ un électron unique
dans ce cylindre central pour l’empri.sonncr.
Le p ièg e de la fig u re 10.1 e st fac ile à analy.ser, m ais n ’esi p as très réaliste.
C ependant, il est possible d ’em prisonner des électro n s uniques en laboratoire à l’aide
de pièges plus com plexes, m ais qui font appel à des notions sem blables. P ar exem ple,
à l ’I'n iv c rsilé de W ashington, des scien tifiq u es o nt piégé un électron unique dans un
piège duranl p lu sieu rs m ois d ’affilée, ce qui leur a p erm is d e m esu rer ses propriétés
avec une très grande précision.

La détermination des énergies quantifiées


La figure 10.2 m ontre l'én e rg ie potentielle de l’électron en fonction de sa position sur
l’axe d es x du p ièg e id é alisé de la fig u re 10. 1. (ju a n d l'é le c tro n sc tro u v e d an s le
cylindre central, son énergie potentielle —cV) est nulle parce que le potentiel V y
est nul. Si l’électron pouvait sortir de cette zone, son énergie potentielle serait positive
et de g ran d e u r in fin ie , p arce que, à c e m om ent là, V' —xi. L a co n fig u ratio n
Figure 10.2 L'énergie potentielle
d 'én e rg ie potentielle de la figure 10.2 s ’appelle p u its d 'é n e rg ie p o te n tie lle in fin im e n t
électrique //(») d'un élerlmn Confiné
p ro fo n d ou, plus brièv em en t, puits de potentiel infini. C ’esI un « p u its » parce q u ’un
dans le cyUndie leiilial du piège idéalisé
de la figiiiv 10 ! Ou constate que électron placé dan.s le cy lin d re central d e la figure 1 0 .1 ne peut s’en échapper. Q uand
(7 0 dan.s le cas où 0 \ ^ L, il s ’approche d ’une des extrém ités du cylindre, une force d ’iin m<vlule praliquem ent infini
et que t/ —> oc dans le cas où x < 0 renverse son m ouvem ent, cc qui l’em prisonne. Liant donné q u ’il ne ix-iit .sc déplacer que
a X ■>1 .sur un axe, on peut appeler cc piège un puits de potentiel infini unidimensionnel.
10.3 Les énergies d'un électron piégé 27/

À l’in.star d e l ’o n d e sta tio n n a ire d an s u n se g m en t d e co rd e ten d u e, l ’o n d e


de m atière décrivant l ’électron confiné doil avoir des noeuds à л = 0 c l àA = 7^. D e plus,
l ’éq u a tio n l ü .l s ’ap p liq u e à un e telle o nde de m a tiè re si on y co n sid ère X com m e
la longueur d 'o n d e de de B roglie associée à l'électro n en mouveiiieiii.
L 'équation 9.17 définit la longueur de de B roglie à coim nc étant > = Ыр. où p i si
le m odule de la q u a n tité d e m oiivem eiu d e ré le c U o n . C e tte g r a n d e u r e s t a.ssoeicc
à l'é n e rg ie cin étiq u e de l’électio n K p ar p — -JlmK, où w esi la m asse de l'é le t iron
D ans le cas d ’un électron sc déplaçant à l ’ intérieur du cylin dre central de la figure lü . l,
où C = 0 . l’énergie (m écanique) totale est égale à l’cncrgie cinétique. A insi, on peut
écrire la longueur d 'o n d e tic de B roglie d e cct électron de la m anicrc Miivaiiic :

. _ Л _ h
( 1 0 .1)
P s/2m E

Si on insère l’expression de l'équation 10.3 dans l ’cquation 10. J et q u ’on isole 1 énergie
II, on constate qu e f, dépend de n de la rnanicie suivante :

jioiir n — 1, 2, .3,... П 0 .4 )

Ici. le nom bre en tier n est le nom bre quantique de l ’étal quaniit|tie d e l'é le rtro n piégé
dans le puits de potentiel.
L’équation 10.4 révèle une chose im portante : étant donné que l’clcctrcm c.st confine
dans le piège, il ne peut posséder que les énergies déterm inées par cette ét)uailon Tl ne
peutpa.'t posséder une énergie sc silnant. par exem ple, entre le,s valcuns n ~ I et n ~ 2.
Pourquoi cette restriction existe t-c lle ? Parce q u 'u n électron est une onde de m atière.
S ’il était plutôt une particule, com m e on le percevrait en pliysique classique, il p n irra ii
p o ssé d er une én erg ie de n ’im porte q u elle v aleu r lo rsq u 'il est co n fin é tians un piège.
La figure 10.3 est un g rap h iq u e in d iq u an t les 5 valeurs cl’c n rrg te s p erm ises les p lu s
basses p o u r un électro n se tro u v an t d an s un p u its in fin i d ont L ~ 100 pni (en v iro n
la taille d ’un atom e m oyen). C es valeurs sont appelées niveaux d ’én etfile: elles sont
illu strées dans la fig u re 10.3 so u s la fo rm e d ’un diagramme des niveaux d ’énergie.
lesquels ressem blent aux barreaux d 'u n e échelle, l ’énergie est représentée à la verticale ;
aucune valeiii n ’est inscrite à rh o riz o n ta lc
L 'étal quantique correspondant au niveau d 'én e rg ie le plus bas possible perm is pat
l ’équation 10.4, £ | , et posséchuii le nom bie quantique n — 1, s ’tippcilc éUit jondanu ntai
de l’électron. L’électron tend à se trouver dans cet état d ’énergie. Tous les élans quantiques
,se trouvant aux niveaux d 'én e rg ie plus élevés (correspondant aux nom bres quantiques
M — 2 e t p lu s) so n t des é ta u excités de l'é le c tro n L 'é ta t du niv eau tT éu crg ic
co rrespondan t au nom bre qu an tiq u e n — 2, est appelé prem ier cuit excité parce qu' il
s ’agit du prem ier des états excités rencontrés dans le thagrom m e des niveaux d énergie.
D e m êm e, l ’élal du niveau d ’énergie £3 s ’appelle deuxième état excité.
UHHi
4'' étal fxiite Les variations d'énergie
l ri électron confiné Iciul à p o sséd er la plus basse én erg ie perm ise, donc à se trouver
800 dans son état fondam ental 11 (leut pas.vet à un étal excité (où il possède plus d ’énergte)
seulem ent si une source externe lui fournit l’énergie additionnelle nécessaire à la variation
S oit £(,ai l’é n e rg ie in itiale d e l ’é le c tro n et / l'é n e rg ie de r é lc c tr o n d an s im état
■Î' clat excité
000 suptirieur dans son diagram m e de niveaux d ’énergie. Iri quantité d 'én e rg ie nécessaire
à l’électron jxxir changer d ’état est alors
S'

400
2'" éi.ii rx^'lté ^iMUt ^-1 (Ш.Й)

200 l " èlal <-x< ilé ^


Rqurc 10.3 Quelques-unes des énergies permises par l’équation 10.4 pour un éleetmn eonfuié
Î'tar tondrtinrntal dans le puits de potentiel infini de la figure 10.2. K i. la largeur Z — 100 pm l n tel graphique
est appelé diagramme des niveaux d'énergie
278 Chapitre 10 La mécanique ondulatoire

E
Figure 10.4 a) Excitation d’un électron
[
qui est passé du niveau d’énergie
de son état fondiunental au niveau 1
1
d’énergie de son Imisièmc état excité
----- fs
b), c) et d) Trois des quatre façons
dont un électron peut ,se désexciter
et revenir au niveau d'énergie
^ J—
if- •
1
" T
i
___ 1___
de son étal fondamental (Quelle K,
façon n’est pas illustrée?) a) h) () d)

Q uand un électron reçoit une telle énergie, on dit q u ’il fait une transition qum tique
de son é tat d 'é n e rg ie le p lu s bits à un état d ’én e rg ie supérieur. 11 est alo rs ex cité,
l a figure 10.4 a) représente une transition de l’ciai fondam ental (le niveau d ’énergie £ ,)
!Ui inrisième état excité (le niveau d ’énergie E^). C om m e l’illustre la figure, cette transition
doit se faire d ’un nive:ui d ’énergie à un autre, mais elle peut éviter un ou plusieurs niveaux
d ’énergie interm édiaires.
Pour gagner de l’énergie afin de faire une transition quantique à un niveau .su|iéricur,
un électron peut absratier un photon. Cependant, cette absorption et la transition ne peuvent
se produire que si la condition suivante e st rem plie.

► Pour qu’un électron confiné absorbe un ptinton, l’éneigie hfdu photon doit être égale
à la différence d’éncigic entre le niveau d’éneigic initial de l’électron et un niveau
d'énergie supérieur.

Par conséquent, l’excitation par absorption de lum ière exige que

h f= AL £haiil £ lhas* (10.6)

Q uand un électron atteint un état excité, il n’y dem eure pas ; il se désexcite rapidem ent
en rédui.sant son énergie. Les figures 10.4 b) à 10.4 d) m ontrent certaines des transitions
possibles dans sa descente à partir du troisièm e état excité. A insi, l ’électron peut revenir
au niveau d ’énergie de son état fondamental en faisant une lran.sition directe (figure 10.4 b))
ou en effectuant de petites transitions en passant par les niveaux d ’énergie intermédiairc,s
(figures 1 0 .4 c le t Jü .4 d )).
L’ém ission d ’un photon constitue, pour un électron, un m oyen de réduire son énergie,
m ais seulem ent si la condition suivante est remplie.

Si un électron confiné émet un photon, l’énergie 7i/dc ce photon doit être égale à la différence
d’énergie AK entre le niveau d’énergie initial de l'électron et un niveau d’énergie inférieur.

D onc, l'équalifin 10.6 s ’applique à la lois à l’ah.siirption et à rém i.ssion de lumière


par uu électron confiné. La lum ière ab.sorhce ou ém ise ne peut alors avoir que certaines
valeurs de /t/e l, d e cc fait, seulem ent certain es valeurs d e f ré q u c n c e /e l de Itm gueur
d ’onde À.
B ien que l’éq u atio n 10.6 et les notions q u ’on vient d ’ahorder su r l ’ab so rp tio n et
l’ém ission d ’un photon puissent s ’appliquer aux pièges à électron physiques (réels), on
ne peut les appliquer aux pièges à électron unidim easinnnels (irréels). L a raison en est
que lors de l'absorption ou de l’ém ission, le m om ent cinétique est conservé, ( ependant
dans cc m anuel, on néglige cette nécessité et on utilise l’équation 10.6 m eme Itirsqu’il
s’agit de pièges à une dim ension.
#
✓ VÉRIFIEZ VOS CONNAISSANCES! i C la s s e / le s p a ire s su iv a n te s d 'é t a ts qiianfifiiie s
I d'un électron confiné dans un puits infini selon les différences d’énergie entre les états.
I en n ’mmençant par les plus élevées : a) « = .1 à n = 1. b l n = 5 à n = 4. t:l n 4àn —^
10 3 L «s énergies d'un électron piégé 279

Exemple lO.I
— M- 3 ; W= H «-
Un électron est contlné dans un puits de potentiel infini unidimen­
sionnel d’une largeur L = I(X) pm.

a) Oucllc est l’cnci-gic minimale que l’élcclron don posséder?


(
■ ■■"| ——~ n ~ '2 »1 - 2
SOLliTIOfi : Ici, le contepl clé est que le confincmciu de l’électron (une onde
de matière) dans le puits mène la quantification de son énergie.
Puisque le puits est infiniment profond, les énergies permises sont a - 1 n= 1
déterminées par l'équation 10.4 (£„ = le nombre a) to cl
quaniique n étant un nombre entier positif. Dans le pré.sent cas,
l'ensemble des constantes se trouvant devant ii^ dans l'équation 10.4 figure 10.5 Exemple 10.1 Désexcitation - réleciron passe du
est évalué ainsi : deuxième état excité à l’état fondamental, soit directement (a]),
soit en passant par le premier état excité (b|, c)).
_ (6,63 X 10
- (8)(9,11 X 10-^1 kg)( 100 X lü -î^ m l^
Dans le cas de la différence d ’énergie A/;,, obtenue en b), cette
= 6.031 X 10“ "^ J. (10.7)
équatiou doniic
L'énergie minimale de l’électron correspond au plus petit nombre hc
A=
quantique, qui est n = I pour l’état fondamental de l'électron. Donc, A Eu
les équations 10.4 et 10.7 donnent (6,63 X 10 '^ J s)(3.0 X 10**m/s)
4.83 X 10 'M
= 4 .12nm. (réponse)

is 6.03 X 10” '^ J = .37,7 eV . (réponse) d) Une fois que l ’clcctron est à son deuxieme état excite, quelles
longueurs d'onde peut-il éna tlrc en sc désexcitant’)
b) Quelle énergie faut-il u-ansferer à l’clectrnn pour qu’il effectue une
SOLUTION; Voici trois comepts cl« qu’on doit utiliser
transition quantique de son état fondamental à son deuxième état
excité ? 1. L’électron tend à sc désexciter au lieu de demeurer dans un état
excite, jusqu’à ce qu'il revienne à son étal roiulainenial (n — 1).
SOlUTIOli On se rappellera, d'abord, qu’on a vu dans la figure 10.3
que le deuxième état d’excitation correspond au troisième niveau 2. Pour sc désexciter, l 'électron doit perdre juste asscï d ’énergie
d’énergie, niveau qui a le nombre quantique n = 3. Ainsi, et il s'agit pour passer à un niveau d’énergie infcncur.
du concept dé à utiliser ici, pour que l’électron passe du niveau « 1 3. S’il doit perdre de l’énergie en éinetiam de la lumière, la perte
au niveau n 3, la variation de son énergie doit cire, .selon d’énergie doit se faire par émission d'un photon.
l’équation 10.5,
Panant du deuxième état excité (à « = 3), l’élection peut atteindre
A £„ - fi, fi,. ( 10.8) l’état tondamenial (n = J) soit en passant directement au niveau
d'cncrgic fondamental (figure 10.5 a|). soit en eflèeiuam deux transi­
Un deuxième contepl tié est que les énergies E , et fi, dépendent du tions distinctes en passant par le niveau h = 2 (figure.s lo 5 li| et c')).
nombre quantique n. selon l’équation 10.4. Par conséquent, si on
remplace fi., et fi, dans l'équation 10.8 par l’expression de l'équation La transition directe met en jeu la même différence, d'cnc.rgir A/.',,
10.4 et qu'on utilise l'équation 10.7, on obtient que celle tju'on a obtenue en b). 1 a longueur d'onde est duiu égale h
celle obtenue en c). sauf qu'il s'agit d'nnc luriiicrc éitii.se cl non pnù
/ absoiliéc. Par conséquent, l’élcctron ftcul passer directement à .son
AErt - \8
I w fi2
état fondamental en émettant une luirucrc d une longueur d’onde

(6.031 X 10 IR J)(8) fi = 4.12 1 (irjxm .se)


= 4.83 X 10 J = 302 éV. (réponse)
t n suivant la méthode employée en b), vous jxxive/ tlemuiiliei t|Ue
les ditïérencc,s d’cncreie dans le cas des transitions de« figun«. lo 5 h)
c) Prxirque l’électron acquière assez d'énergie fwiir passer du niveau
cl e) .sont
d'énergie fi, au niveau E, en absorbant de la lumière, quelle longueur
d’onde cette lumière doii-clle avoir? Afi:,, = 3.016 X 10 .1 et Afiji - ' .«09 < 10 L’
SOLUTION: Scion un premier tomept tfé, si la lumière doit (ransmeitre de
À l'aide de l'équiiUon 10.9. on détermine que la longueni d’onde de la
l’énergie à réleciron, ce transfert doit s’effectuer pat absorption il’iin
lumière émise durant la première de ces iraasitinns (de n ~ t a w = .7)
photon. Un deuxième tonceptdé est que l’énergie du photon doit être
est
égale à la différence d'énergie Afi)entre le, niveau d'énergie initial
de réicctroii et un niveau d’énergie supt'rieiir. selon l’équation 10.6 fi = 6,60 nm. (réponse)
(hf AE). Sinon, il ne peut y avoir absorption d'un photon. ,Si on
rcniplacc f |iai t / fi. on peut récrire l’équation 10.6 ainsi ; et que la longueur d ’onde de la lumière émise durant la deuxieme
transition (de n 2 à « = I ) est
(10 9)
fi-:
A/' 11 O nni (réponse)
280 Chapitre 10 La mécanique ondulatoire

10.4 Les fonctions d ’onde d ’un électron piégé


Si o n réso u t l ’éq u a tio n de S ch ro d in g er ap p liq u ée à un élec tro n p iég é d an s un p u its
de potentiel infini unidim ensionnel de largeur L, on constate que les fonctions d ’onde
de cet électron sont déterm inées par

W O = A s in ^ ^ jc j. pour n 1, 2 , 3. ( 10. 10)

dans le cas où 0 ^ a" s Z, (la fonction d ’onde est nulle hors d e cette plage). O n évaluera
bientôt la constante (l’am p litu d e)/! de cette fonction d ’onde.
N otez que les iônetions d ’onde prennent la m êm e form e qu e les fonctirms de
dép lacem en t v„(.r) d 'u n e o nde statio n n aire d an s une co rd e te n d u e en tre d es su pp o rts
rigides (voir l’équation 10.2). A insi, on peut im aginer un électron em prisonné dans un
puits de potentiel infini unidim ensionnel com m e une onde de m atière stationnaire.

La probabilité de détection
On ne peut en aucune façon détecter ou m esurer directem ent la fonction d ’onde ;
en e lfe t, il est to u t sim plem ent im possible de regarder dans le puits p o u r voir f o n d e ,
com m e on peut le faire dans sa baignoire pour un e onde su r l’eau. L a seule chose que
l’on puisse faire est d 'y insérer une sonde quelconque pour essayer de détecter l’électron.
À l’instant de la détection, l’électron devrait sc m atérialiser au point de détection, à un
certain point de l’axe des x dans le puits.
Si on répétait ce procédé de détection à plusieurs positions dans le puits, on consta­
terait que la probabilité de détecter l ’électron c.st liée à la position x de la sonde dans le
puits. En fait, la probabilité et la position sont reliées par la densité de prohahilité xl/^{x).
D ans la section 9.8. on a vu que, en général, la probabilité q u ’une particule puisse être
détectée dans un volum e infinitésim al centré à un point déterm iné est proportionnelle à
Ici, l’éleetnm étant em prisonné dans un puits à une dim ension, on ne tient com pte
que de sa détectio n su r l’axe des x. D onc, la d en sité de p ro b ab ilité xl/^(x) est. dans le
présent cas. une probabilité p ar unité de longueur su r l’axe des x. (Ici. on peut om ettre
le signe du m odule parce que dans l ’équation H). 10, ^„(x) e.st une fonction réelle, non
une fonction com plexe.) La probabilité p(> ) de détecter un électron à la position » dans
le puits est :

probabilité p{x) de d étec tio n \

(
/d en sité de probabilité ^
dans la laig eu r dx centrée
ee I = 1 , (largeur djf),
I \ a la position x
à la position X

ou p(x) - \l/^(x) dx. (10.11)

À l'a id e de l’équation 10.10, on constate que la densité de probabilité \l/^{x) dans


le cas de l’électron piégé est

~ s in ^ ^ - ^ x ^ . pour/z = 1 , 2 , .3,..., (10.12)

dans la zone 0 ^ £ Z. (la densité de probabilité est nulle hors de cette zone). La figure 10.6
illustre ti'ifjr) quand n = 1.2 , 3 et 15, dans le cas d ’un électron piégé dans un puits infini
dont la largeur L est 100 pm.

figwe 10.6 La densité de probabilité pour quatre états J^un électron piégé dans un puits
infini à une dimension ; leurs nombres quaniiqucs sont n = 1, « ~ 2, n = 3 et n = 11.
mo L’électron est plus susceptible d’être détecté là où la valeur de est élevée le plus,
et il esi moins susceptible de l'être là <mi la valeur de est élevée le moins.
10.4 Les fonctions d ’onde d ’un électron piégé 281

P our d éterm in e r la p ro b ab ilité de d étec ter un électro n dans une sectio n fin ie du
puits (p ar exem p le en tre le p o in t x , e t le point X2), il fau t intégrci /i(c) entre ces deux
points. D onc, selon les équations 10.11 e t 10.12,

/p ro b ab ilité de d étcctio n \ = / p{x)


\ entre JT] et x-> /
'= A ^in’( í^ .,)r /v . (10. n i
- j :

Si la p h y siq u e clas.siquc p rév a lait, on s ’atte n d ra it à ce q u e les p ro b a b ilité s dC


détecter l’électron piégé soient égales dans toutes les parties du puits. Cependant, dans
la figure 10.6, on co n state qu e ce n ’est pas le cas. P ar ex em p le, un exam en dc CCHO
ligure ou de l'éq u atio n 10.12 révèle que, prxir l’état correspondant à n “ 2, il est plus
p ro b ab le de d é te c te r l'é le c tro n p rès d c x - 2S pm et d e .v ’= 75 p m q u ’ailleu rs.
C elle probabilité devient presque nulle près dc jr = 0, dc x = 50 pm e t de x — 100 pni.
D ans la fig u re 10.6, le cas de n — 15 su g g ère que, à me.sure q u e n au g m en te,
la probabilité de détection devient dc plus en plus hom ogène dans l’ensem ble du puits.
C ette tendance constitue un exem ple d ’un principe général appelé p rin c ip e d e eorrex-
p o n d an c e.

Quand le nombre quantique est sutrisainmem élevé, les pédinioiiv de la jiliysiqiR' qiianliqiic
et de la phy.sique classique se rapprochent de pins en plus

Ce principe, form ule pour la prem ière fois par le pliy.sicicn danois NieLs Bohr. s’applique
à toutes les predictions quantiques, il devrait vous rappeler un principe sem blable coiieer-
nanl la théorie de la relativité, qui est q u ’à des vitc.sses suftisammcnl faibles, les predictions
de la relativité restreinte correspondent de plus en plus à celles de la physique newtonienne.

4/ VÉRIFIEZ VOS CONNAISSANCES 2; La figure montre trois puits de potentiel infini doni
les largeurs sont L, 2 L ti IL. chacun contenant un électron dans l’état où n = K). Classez
ces puits selon a) le nombre de maxima de la densité de prohahiliic de dela tion de l'clcctron
Cl b) l'énergie de l’élcctron, en commençant par les valeurs les plus élevées

/. 2L ,x/
ai b) c)

La normalisation
Le produit d o n n e la p ro b ab ilité q u ’un électro n p ié g é dans un p uits puisse être
détecté d ans la zo n e situ ée en tre x et .x H t/.r P u isq u ’on sait que l ’c lre tro n doit etre
quel(/uc pxirf dans le puits infini, alors

ij/^( r ) d x — 1 (!’équation de normali.sation). (10.14)

pui.squ'unc p ro b ab ilité ég a le à 1 co rresp o n d à la ce n iim ie B ien que l'in lé g ra lc soil


prise sur tout l 'ax e des x, seule la région de x = Ü à v - f a une valeur non mille t|iiam
à la probabilité. Crntphiqucinent. l'in té g ra le de l’équation 10 14 représenie f a l i e sous
chacune des courbes de la figure 1Ü.6.
D ans l’ex em p le 10.2. on v erra q u e. si on rem p lace V^„-(x) dans I éqim iûm 10.14
par l’expression de l’équation 10.12. il est possible d ’attribuer une v aleur s p t irigiie à l:i
co n sta n te A qui ap p a raît dan.s l’éq u a tio n 10.12, à sa v o ir A - J l 'L , L a m élhorle
qui coiisi.ste à utiliser l’équation 10.14 poui évaluer la constante d ’une fonction d 'o n d e
s ’appelle normalisatiun de la fonction d ’onde
282 Chapitre tO La mécanique ondulatoire

L'énergie de l'état fondamental


Le rem placem ent d e « = 1 dans l’équation 10.4 définit l ’état où l’énergie est au plus bas
pour un élection p iég é dans un puits de p otentiel infini, soit l ’état Tondamcntal. C ’est
l ’état qui caractérisera l’électron confiné à m oins q u ’on lui fournis.se de l’énergie pour
l'é le v e r à un état excité.
La q u estio n .suivante se p o s e : p o u iq u o i ne p eu t-o n in clu re n = 0 p arm i les
p o ssib ilité s de n d an s l'é q u a tio n 10.4 ? Si on in sé ra it n — 0 d an s c e tte éq u a tio n ,
on o b tie n d ra it un é ta t fo n d am en tal d ’é n e rg ie n u lle. C e p e n d a n t, si on in sè re n = 0
dans l'éijuation 10.12, on obtient égalem ent 0 ,7(x) = 0 pour toute valeur de x, q u ’on ne
peut interpréter autrem ent q u ’en disant q u ’il n ’y a pas d 'électro n dans le puits. O n sait
pourtant q u ’il y en a un. donc « - 0 n ’est pas un nom bre qiiantiqnc po.ssible
Le fait que les .systèmes confinés ne peuvent exister dans des états d ’énergie nulle
est une im portante conclusion d e la physique quantique. Ces systèm es doivent toujours
pos.séder une certaine énergie m inim ale appelée én e rg ie d e l'é ta t fo n d am e n tal.
O n peut rendre l'én erg ie de l’état ibndam cnial aussi petit« q u ’on le veut en élargis­
sant le puits infini, c ’est-à d ire en au g m en tan t L dans l’équation 10.4 lorsque n 1.
D ans la lim ite où L —» l’é n e rg ie d e l’ctat fo n d am en tal s ’ap p rtK h e d e zéro.
C e p en d a n t, d an s ce tte lim ite , où la larg eu r du p u its e s t in fin ie, l ’é le c tro n e st une
p a n ic u le libre, qu i n ’est p lu s co n fin ée dans la d irectio n x. D e plus, étan t d o n n é que
l’énergie d ’une p articu le libre n ’est pas q u an tifiée, elle peut av o ir to u tes les valeurs,
y com pris zéro. Seule une particule confinée doit avoir une énergie de l’état fondam ental
non nulle et ne peut jam ais être au repos.

VÉRIFIEZ VOS CONNAISSANCES 3 : 'h a c u n c dc.s p a rtic u le s suivanfe.s est c o n fin é e dan.s un
puits infini, et les quatre puits oni la même largeur ■a> un électron, b) un proton, c) un deutéron
et d) une particule alpha. Classez leurs énergies de l’état fondainental. en commençant
par la plus élevée. Les particules sont classées selon leur masse, en ordre croissant.

Exemple 10.2
Évaluez la constante de normalisation 4 dans l’équation 10.10 quand x 0 et que y = m quand x = L, de sotte que 0 et nn sont les
p<xir un puits de potentiel infini s’étetxiani tie Jt = 0 à x = L. nouvelles limites. Après tous ces remplacements, l’équation 10.15
devient
SOLUTION: Le concept clé utilisé ici est le suivant : les fonctions d’onde de
l’équation 10.10 doivent répondre aux exigences de normalisation de 7 r
l’équation 10 14, qui dit que la probabilité ((uc l’clcctron puisse être (sin‘ y) dy = 1.
/
détecté quelque part sur l’axe des x est 1. Si on pose l'expression de
l’équation 10 10 dans l’équation 10.14 et si on sent la constante 4 hors
f)n peut utiliser rintégralc 11 de rannexe D pour évaluer l’intégrale ;
de rinicgralc. <in obtient
on obtient alors l’équation
ri- ^ /niT \
1 (10.15) siii2v
rtït
Ou a changé les limites de l'intégrale de et de + « en 0 et
en /. parce que la fonction d ’onde est nulle hors de ces nouvelles Si on év alue aux limites, on obtient
limites.
( )n peut simplifier rintcgration Indiquée en faisant un change- 4^L mr
= 1.
inem de variables, de la variable v à la variable sans dimension y. où njt 2
rm
V— K, (10.16) A= (réponse) (10.IT)
/

dv = rlY Ce résultat montre que la dimension de .d", donc de \lr^(x), est


HTT
l’inverse d'une longueur. C ’est approprié parce que la den.sité de
Ouand on effectue ce changement de variables, on doit aussi modifier prohabililé de l’équaiiiin 10 J 2 est une probabilité par unité de
les limites d ’intégration (encore). L’cqiiation It). 10 dit que y = 0 longueur.
10 5 Un electron dans un puits fnl 283

Exemple 10.3
Un élcclron à l'c ta t fonddmcntal cs>l empri.sonnc dans le puit.s de On doit donc évaluer ce qui suit :
potentiel infini unidimensionnel illiiliré dans la figure 10.2. puits
ayant une largeur L — 100 pm.
p r o b a b ilité » ^ I s iu ^ y li/v .
a) Quelle est la probabilité de détecter un électron dans le tiers
gauche du puits (entre .»| = 0 et X2 = L/i) ? Si on utilise l’intcgraie 11 de l'annc.xc D, on obtient
SOlUnOH: Le picmicr corcepl dé utilisé ici est le suivant : si on sonde le
1 1 .^ 2 A ,sin 2 v A * '
tiers gauche du puits, il n’y a aucune garantie qu'on y détectera probabilité ~ ------ — j -
l'élcclron. Cepcndaiil. on peut calculer lu probabilité de le détecter
avec l’intégrale de l'équation 10.13. En s'appuyant sur un deuxième
Donc, on a
concept (le. on peut dire que la probabilité dépend de l'étal dans lequel
SC trouve l’clcctron - c’est-à-dire de lu valeur du nombre quantique n
de l'électron. Étant donné que rélcciron se trouve ici dans son étal
fondamental, on établit que n ~ 1 dans l'équation 10.13.
(probabililé (le détectúMil _ ^
dans le tiers gauchr /
(réponse)

On établit aussi que les limites de l'intégrale sont les positions C’c.st donc du e que. si on sonde de manière répétée le tiers gai» lie du
X| = 0 et = L/3 et. d’après l’excniple 10.2. que la constante de puits, on pourra détecter l'électron dans 20% des cas. en inoyeiinr.
normalisation A est yPJL. On constate alors que
b) Ouelle est la probabilité de détecter l eleciron dans le nets médian
probabilité de détection\ du puits (entre .Vi - /73 et .Vj ~ 2/73 ) 7

(
dans le tiers gauche /
SOUITtON; On sait maintenani que la probabilité de détection dans le
On aurait pu trouver cette probabilité en remplaçant L par tieisgauchc du puits est de 0.20. l.e premier (cnccpt clé. ici. est que. par
100 X K) '2 n, puj.j en utilisant une eulculalricc à eupMcité graphique syniélrie. la juobabilité de détection dans le tiers droit du puits est
ou un logiciel de mathématiques pour évaluer l'intégrale. On suivra également de 0,20. Le dciixièine conccpl (I# est que. étant donné que
plutôt les éuipcs de l’exemple 10.2. .Selon l’cquaiion 10.16. on ohlieiu, l'électron s'y trouve sans aucun doute, la probabilité de détection
pour lu nouvelle vaiiahle d’intégration y. dans tout le puits est de 1.0. Donc, la probabilité de déieciion dans Ir
tiers médian du puits e.si
L
V= et dx = t/v.
TT probabililé de déieciion

(
) = 1 .0 - 0 .2 0 - 0.20.
Grâce à la première de ces équations, on constate que les nouvelles dans le tiers médian
limites d’intégration sont v'i = 0 pour r, = 0 et vj = n /i pmir.r, = /73. - 0.60. (réponse)

10.5 Un électron dans un puits fini


Un puits d 'én e rg ie potentielle de profondeur iniinie est une idéalisation. La llfjury 10.7
m ontre un puits d'énergie* potentielle piwsiblc. .sj sterne dans lequel l'cn crg ic potentielle
d ’un électron hors du puits n 'e st pas innninieiii grande, m ais a plutôt une valeur positive
finie I/o. appelée p ro fo n d e u r d u puit.s. I,‘analogie entre les ondes dans une corde tendue
et les ondes de m atière ne s'ap p liq u e plus dans le etw de [uiiis de profondeiii liiiie parce
qu’on ne peut plus être sûr d 'y trouver dos nœuds dans l’onde de m aiièir à i — 0 et à a ” ■L,
(O n verra q u ’il n ’y en a pas.l
Pout déterm iner les fonction.s d ’onde qui décrivent les élais quam iquos d ’un élei im n
dans le puits fini de la ligure 10 7. on doit recourir à I’cquation de SohrOUinticr. oqutuiuii
de base de la physique quantique D ans la section 9 R, on a vu t|u 'o n utilise I rquafion
de Schrôdinger dans la form e de l’équation 9.19 quand il est quesiion d ’nn m ouvenx ni
à une dim ension ;

i/V RjT^w r , ,
[ é - U{x)]xfr - 0. ( lO lf i)
dx^

Au lieu d 'essa y er de résoudre eette équation dans le cas d ’un puits fini, on donne sifn
Figure 10 7 T'n puits d’énergie |xitcntielle p le m en t les résu ltats re la tifs à d es v aleu rs n u m ériq u es p a rtic u liè re s d e Un e t d c L.
fini I ji pn ifondrur du puits est Üq l a figure 10 R illu stre les résu ltats sous la fo rm e des g rap h iq u es dc x¡/i{.\), qui CSl la
ex sa largeur est L. ('auiiine dans le puits densité de probabilité, pour un puits «u'i 7/ft = 4 'in eV et /- = KKl pm.
de potenüel infini de la figure 10.2, L a d en sité d e p ro b ab ilité V'nC’^) P*'er eh aq u e graplii()iir d e la fig u re lO.R csl
le nioiivtmienf de l'électron piégé est ici cohérente avec l’équation 10,14, l'éq u a tio n d e n o rn ialisalion, de sorte q n ’on sail qiie
aus.si restreint à la direction jc. l'airc sous les trois courbes de densité dc probabilité est num ériqiicm cni égale à 1.
284 Chapitre to La mécanique ondulatoire

Si vous co m p arez la fig u re 10.8 (p u its fin i) à la fig u re 10.6 (p u its in fin i), vous
c o n staterez une d iffé re n c e fra p p a n te : d an s le cas d ’un p u its fini, l’o nde de m atière
de l’électron pénètre dans les m urs dans une région où la m écanique classique d it que
l’é lec tro n ne peut exister. C e tte p én é tratio n n e d ev ra it pas vo u s su rp re n d re , c a r on
a vu diuis la section 9.10 q u ’un électron peut traverser une barrière d ’énergie potentielle
par effet tunnel. La « fu ite» de l’onde de m atière d e l’électron dans les m urs d ’un puits
d 'é n e rg ie p o te n tie lle fin i e s t u n p h én o m èn e sem b lab le. D an s la fig u re 10.8, les
graphiques vous perm ettent de constater q ue cette fuite est plus grande quand la valeur
du nom bre quantique n est plus élevée.
E ta n t d o n n é q u 'u n e o n d e de m atière pénètre d an s les m urs d ’un p u its fini, lu
longueur d 'o n d e À pour tout état quantique donné est plus grande quand l’électron est
em prisonné dans un puits fini que lo rsq u ’il l'e s t dans un puits infini. A lors, l’équalioii
10.3 m ontre que l'én erg ie E d ’un électron dans un état donne e st plus bas.se dans le puits
fini que dans le puits infini.
On peut ainsi créer un diagram m e approxim atif représentant les niveaux d ’énergie
d 'u n électron em prisonné dans un puits fini. Par exem ple, on p eu t Irtteer un diagram m e
50 0 50 100 150 approxim atif du puits fini illustré dans la figure 10.8, d ’une largeur L = KX) pm et d ’une
X (pni) p ro fo n d eu r Ud 4 5 0 eV. C o n sid érez la fig u re 10.3, qui p résen te le d iag ram m e des
Rgiire 10.8 Densités de probabilité n iveaux d ’é n e rg ie d 'u n p uits infini ay a n t cette largeur. O n en lèv e d ’ab o rd la p artie
pour un électron confiné dans un puits de cette figure se tro u v an t au-dessus d e 450 eV. On abaisse en su ite les trois niveaux
de piHcniie! fini de profondeur d 'é n e rg ie résid u e ls, en abaissan t d av an tag e le niveau d e n = 3 parce que la fuite de
i/g = 450 cV et de largeur I. ■= KH) pm. l'o n d e dans les m urs est m axim ale quand n = 3. Le résultat est approxim ativem ent le
Les seuls étals quantiques où l'électron d iagram m e des niveaux d ’én erg ie dans le puits fini. L e vrai d iag ram m e est présen té
peut être dans ce puits sont ceux dans la figure 10.9.
dont les nombres quantiques sont D ’après celle figure, un électron possédant une énergie supérieure à Uq ( = 4 5 0 eV )
« = J,/) = 2 e t n = 3. est trop én erg iq u e p o u r être p iég é dans le puits fini. 11 n ’est d onc pas co n fin é et son
énergie n 'e st pas quantifiée ; son énergie n’est donc pas restreinte à certaines valeurs.
P our atteindre cette partie non quantifiée du diagram m e des niveaux d ’énergie, donc
pour être libre, un électron piégé doit, d ’une manière ou d ’une autre, acquérir suffisam m ent
d ’énergie pour posséder une énergie m écanique de 4 5 0 cV ou plus.

Figue 10.9 Ix diagramme des niveaux


d'énergie eorrespondanl aux densités
rte probabilité de la figure 10.8. Si un
électron e.st piégé dans le puits de
potentiel fini, il ne peut avoir que les
énergies correspondant à n = 1, n = 2
et n 3. S'il po.ssède une énergie de
4.50 eV ou pins, il n'est pas piégé et
son énergie n'est pas quantifiée.

Exemple 10.4
Suppose/ qu'un puits fini Vu - 450 cV et /. - KH) pm confine un initialement h son état fondamental et possède une énergie
éleciron dans son état fondamental. ¿1 = 24 e \. Dont, pour se libérer à peine, il doit acquérir une énergie de

a) Quelle e.st la longueur d'onde de la lumière qui permei ii réleciron


U„ - C| = 450 cV - 24 eV - 426 eV.
de qiiillcj tout juste le puits de potentiel à l’aide de l'ab.sorplion d’un
seul photon ?
S ’il acq u iert cette én erg ie fie la lu m ière, il doit absorlxir un
^OilITION premier tnntppf fié utilisé ici est le s u iv a iu ■pour que
I.e phnton la [jossédant. S elon l’équation 10.6, si on r e m p la c e /
rélciim n .s'échappe du jniirs de jmiemiel il doit aiquérii suffisam- jrar r/A., on peut écrire ‘
nicnt d’énergie [xiur atteindre la région d'énergie non quarilificc de la
figure 10.9 11 doit donc posséder éventuellement une éiKrgie mini- hc
Vu - E,
niale de Vu<= 450 eV). l e deuxième (oncept dé est que l'électron est
10.6 D’autres pièges à électron 28J

dont on obtient 3. Par conséquent, l’élcetrcwi peut absorber un pbolon de cette lumière.
hc
Non seulement le transfert d'énergie libérera l'élcciron mais,
en outre, il lui fournira davantage d’éne*^o cinétique De pitic.
Vo - ¿1 , éuint donné que l’électron ii’cst plus emprisonne, .son énergie
(6,63 X 10 ” J ■s)(3,()0 X 10*^ m /s) n’est plus quantiliée et son énergie cinétique n ’est soumise ü
(426 cV )(l,60 X 10-'^ J/eV) aucune resmetion.
= 2,92 X 10“ ’ m = 2.92 nni. (réponse)
L’énergie transférée à I’clcclron est l’cnergic d’un photon:
Donc, si l'électron absorbe un photon d'une lumière ayant une longueur c (6.6.^ X 10 ^ T- s)(3,00 X 10’' m/s)
h f = // — = —------------------ — „ ------------------
d’onde de 2,92 nm, ¡1 s’échappe tout juste du puits de (XHentiel. À 2.00 X 10“ ’ m
b) L'électron initialement à l'état ronilamenlal peut-il absoiber de la = 9,95 X 10“ *^ J = 6 2 2 e V .
lumière d ’une longueur d'onde de 2,00 nm ? Si oui, quelle est son Scion les données obtenues en a), I’cncrgic necessaire pour tout juste
énergie finale’? libérer l’clcctron du puits de potentiel est Un t | (~ 426 CV),
SOLUTION; Voici les tonteptsclés. L’énergie résiduelle des 622 cV sc transforme en énergie Cinétique.
Donc. l’énergie cinétique de l’clcctron libéré est
1. Eu a), on a déterminé que la lumière de 2.92 nm .suffira juste
a libérer réleclron du puits de potentiel. /< = h f ~ ( U o - i i , }

1. On est ici eu présence de luinicie d’une longueur d’onde inférieure, 622 eV —426 cV — 196 cV. (icpinsc)
.soit 2.00 lun, doue d’une énergie supérieure par jihoton (/;/ hdk).

10.6 D'autres pièges à électron


O n étudiera m aintenant trois types de pièges à électron artificiels.

Les nanocristallites
Il s’agit peut-être de la méthode la plus directe pour fahriquei un (Xiiis d ’énergie potentielle
en laboratoire et p rép arer un éch an tillo n d ’une m atière sem i co n d u ctrice sous fo rm e
d 'u n e poudre de petits granules (à l’échelle du nanom ètre) de taille hom ogène ( 'hacim
de ces granules (chaque n an o cristailite) agit œ m m e un puits de potentiel ptiur les électrons
qui y sont piégés.
L ’équation 10.4 révèle q u ’on peut augm enter l ’énergie de l’état fondam ental d ’un
électron piégé dans un puits infini en rédui-sar* la largeur /. de ce puits. C ’est aus.si vrai poiu
les puits form és par les nanocristallites individuelles. D onc, plus la n anocristailite est
petite, plus l’énergie de son étal londam cntal c.sl élevée ou, autrem ent dit. plus l 'énCTgic
de seuil pour l’absorption des photons est élevée.
Si on dirige la lum ière du Soleil sur une poudre de nanocristallites. les crlsiallitcs
peuveni absorber to u s les photons possédant une énergie supérieure à un certain seuil
^•seuii ( = O onc. clIcs p c u v c n t a b so rb c r la lum ière d ’une longueiii d ’ onde
figure lO.IO Deux échaniillons de sélénide inférieure à un certain seuil où
de cailmium, un semi-conducteur,
ne différant que par la taille de leurs hc IIO .Iλ
granules. Chaque granule sert de piège
à électron. Les granules de l’échantillon
du haut sont plus gros que ceux P uisque la lum ière non ab so rb ée est d iffu sée , la p o u d re d e n an o crisiallites d iffu sera
de l’échantillon du bas ; par amséqiicni, toutes les longueurs il’ondes supérieures à Às,ui|.
ils pié.senlent de plus jviits espaces O n voit réch aiilillo n de poiirlre grâce à la lum ière q u ’il difhi.se vers les yeux. D onc,
eiilie les niveaux d’énergie et une
en contrôlant la taille des nanocristallites form ani un échantillon, on pi ul contrôler les
énergie de seuil pour l’absorption
longueurs d ’onde de la lum ière diffusée et. par conséiiuent. la couleur de rccluinliilon.
de |ih(itons plus basse. L'échantillon
La figure 10.10 m ontre deux échantillons d ’un sem i-conducicui nomi™' séicnidc de
diffuse aloi.s la lumière non absorbée
c.admium, chacun étant constitué d ’u ne poudre de nanocristallites d e taille hom ogèm
à une longueur d’onde supérieure
L’échantillon du haut diffuse la lum ière à l’extrém ité rouge du sjieetre. L’échantillon du
et paraît rouge. I,’échantillon du bas
bas diffère de celui du haut seulement par la taille de scs nanocristallites, qui sont plus
paraît jaune en raison de .ses petits
granules et. par conséquent, des espaces petites. C ’est pourquoi son seuil d ’énergie est siipt-rieur et. scion l'éq u atio n 10.19,
plus grands qu'il contient entre .sa longueur d ’onde de seuil >*„¡1 est plus courie. I,'échantillon prend une couleur ayant
les niveaux d ’énergie et d’une énergie une plus cou rte lo n g u eu r d ’o nde (ici, le ja u n e). Le co n traste frap p an t entre les deux
de seuil plus élevée pour l’absorption échantillons est une preuve indéniable de la i|u a n tifiratio n des énergies des électrons
de lumière. piégés et de la dépendance, de ces énergies à l ’égard de la taille thi piège à électron.
286 Chapitre 10 Ua mécanique ondulatoire

Figure 10.11 Un point quantiquc. ou « atome artificiel » a) Une couche semi-conductrice centrale
Métal Coiidiicictiir forme un puits d’énergie ptrteniielle où les électrons sont piégés, l.a couche isolante du bas est
suffisamment mince pour permettre un ajout ou un retrait des électrons à la couche centrale
Л
par effet tunnel si une tension appropriée est appliquée entre les conducteurs, b) Une photographie
Sciiii- d’un point quantique réel. La bande violette centrale est la zone de confinement des électrons.
conclucu'iir

Les points quantiques


Les techniques très avancées qui servent à fabriquer des circuits intégrés peuvent également
servir à la construction, atom e par atom e, de puits d ’énergie potentielle individuels qui
se comportent, sous plu.sicuns aspects, com m e de.s atomes artificiels. Ces points q u an tiq u es,
a) com m e on les appelle habiiuelleincnt, peuvent av o ir des applications prometteu.ses en
optique électronique et en technologie inform atique.
P o u r fab riq u er un tel a p p a reil, ou fo rm e un « s a n d w ic h » en in séran t u ne fine
couche d ’un m atériau sem i-conducteur, m ontrée en violet dans la figure 10.11 a), entre
deux couches isolantes, dont l’une est beaucoup plus m ince que l’autre. D es couvercles
de m étal m unis de fils conducteurs recouvrent les extrém ités. On choisit les m atériaux
de façon à ce que l’énergie potentielle d ’un électron se trouvant dans la couche centrale soit
inférieuie à ce q u ’elle est dans les deux couclies isolantes, pour que la couche centrale
agisse com m e un puits d ’énergie potentielle. figure 10.11 b) est une photographie
d ’un véritable point q u an tiq u c; la zone violette est le puits où les électrons individuels
peuvent être em prisonnés.
D ans la figure 10.11 a), la couche isolante du bas (non pas celle du haut) est su ffi­
sam m ent m ince p o u r p crm c llrc aux é lec tro n s de la tra v e rse r p ar e ffe t tu n n el si on
ap p liq u e une tension ap p ro p riée e n tre le.s c o n d u c te u rs. A in si, on p eu t co n trô le r le
nom bre d ’électrons confinés dans le puits. C e m ontage se com porte en effet com m e un
atom e artific ie l, et a co m m e c a ra c té ristiq u e q u ’il p erm e t de c o n trô le r le nom bre
d ’élec tro n s q u ’il co n tie n t. O n peut c o n stru ire des p o in ts q u an tiq u es en réseau x
b id im en sio n n els q ui p o u rra ie n t fo rm e r la base de sy stèm e s in fo rm atiq u es à g ran d e
vites.se et à grande capacité de m ém oire.

Les coraux quantiques


Q uand un m icroscope à effet tunnel (décrit dans la section 9.10 et dans la figure 9.17)
est en marclie, sa sonde exerce une petite force sur des atom es isolés p o uvant se situer
sur une surface régulière. En m anipulant soigneu.sement la position de la sonde, on peut
« tra în e r» de tels atom es isolés sur la su rface p o u r les p lacer où l'o n veut. C ette tech­
nique a perm is aux scientifiques du A lm aden R esearch C enter de IBM de déplacer des
atom es de fer sur une surface de cuivre soigneusem ent préparée pour les di.sposcr en un
cercle q u ’ils ont appelé corail quantique. l,a photo d ’ouverture du chapitre en m ontre
le résultat. C haque atom e de fer form ant le cercle est niché dans un vide de la surface
de cuivre, équidistant des tm is atom es de cuivre voisins les plus proches. C e corail a été
fabriqué à basse tem pérature (environ 4 K) pour réduire la tendance des atom es de fer à
se déplacer aléatoirem ent su r la surface sous l’influence de leurs énergies therm iques.
l e s o n d u la tio n s à l'in té rie u r du co rail so n t cau.sées p ar les o n d es d e m atière
associées aux électro n s qui peuvent se d ép lacer à la .surface du cu iv re, m ais qui sont
em prisonnés dans le corail, l e s dim ensions des ondulations concordent fiirt bien avec
les prédictions de la théorie quantique.

10.7 Les pièges à électron bidimensionnels


et tridimensionnels
D ans la p rochaine se ctio n , on é tu d iera l’atom e d ’h y d ro g èn e c o n stitu a n t un p u its de
potentiel fini à trois dim ensions Pour se préparer à cette étude, on peut élargir l'an aly se
des puits d e potentiel infini à deux cl à trois dim ensions.
10.7 Les pièges à électron bidimensionnels et tridimensionnels 287

Le corail rectangulaire
L a fig u re 10.12 m o n tre une .surface rectan g u laire dans laq u elle un électron peut être
confiné ; il s’agit d e la version bidim casionncllc de la figure lü .2 - un puits de potcnUcI
bidim ensionnel dont les largeurs sont L» cl L,.. Un tel puits est appelé corail rectanguiuin?
C e corail peut se trouver à la surface d ’un corps qui contraint, CTune m anière ou d U ne
au tre, le m o u v em en t d 'é le c tro n p ara llèle à l ’axe des z'. il cra p c e h e d o n c r é lo e im n
de qu itter la surface. Vous d e v e / im ag in er d es fonctions d ’cncrgic potentielle infinie
(com m e U{x) à la figure 10.2) sur tous les côtés du corail, ce qui pcrm ci d ’j cnipriMiniicr
figwe 10.12 Un corail rectangulaire l ’électnin.
- version bidimeasiiMinelIe du puits L a solution de l ’équation de Schrôdinger relativem ent au corail rectangulaire de la
de potentiel infini de la figure 10.2 - figure 10.12 révèle que. p o u r que l ’éleeiron .«)ii em prisonné, .son onde de mnticrc doit
dont les largeurs sont et L,
entrer dans chacune d es deux largeurs de m anière ¡ndé^xfndan^e, loui com m e ro n d e de
m atière d ’im électron em pi i.somié doit entrer dans un puits infini à une dim ension. C ela
signifie que l’o nde est quantifiée scparém cnl scion la largeur /j, cl scion la largeur L,,.
Soit n, le nom bre quantique de fo n d e de m atière .selon la largeur l.^ ei n, celui dt fo n d e
de m atière selon la largeur L ,. C^omme dans le ca.s d 'u n puits à une dim ension, ces nom ­
bres quanliqiies ne peuvent être que des entiers positifs.
L 'énergie de l’électron dépend des nom bres (|tmnlique.s. ainsi que de la som m e de
fc n c rg ic q u ’il posséderait s ’il était confiné au seul axe des x el de l’énergie q u 'il pos­
séderait s ’il était eo n lïn é au seul axe des y. A l’aide d e l’équation 10.4, on p eu t écrire
cette som m e ainsi :

=(4)"' - - £(i ( 10.20)

Les exigencc.s quant à l'excitation et à la désexcitation de f électron p ar absorption


et ém ission d ’un pholon .sont les même.s que d an s le cas des piôgc.s à une diineiisiou.
La seule différence im portante dans le cas du corail à deux diiiiciisinns esi que l’énergie
d ’un é tat d o n n é d ép en d d e d eu x nombre.s q n an iiq iics {n^ et « ,) au lieu d ’un seul (ni.
En général, d es états d ifféren ts (affich an t d ifféren tes p aires de valeurs pour n , et n,.)
possèdent des én erg ies d ifféren tes. C ep en d an t, d an s certain s cas, des étais dirfércin.s
peuvent posséder la m ême énergie. O n dit alors que ces états (cl leurs niveaux d ’cncrgic)
sont dégénérés. Il ne peut pas y av o ir d ’ctais dégénérés dans les puits à une dim ension

La boite rectangulaire
U n élec tro n peut ég alem en t être em p riso n n é d an s un p uits de p o ieu tiel infini k irois
d im e n sio n s u n e hotte. Si la b o îte esl rec ta n g u la ire , co m m e d a n s la fig u re 10.13,
l'équation de Schrôdinger révèle q u 'o n peut d éciiie f én cig ic de l’clcctio n ainsi

(10.21)
l] j

Figure 10.13 Une boîte rectangulaire Ici. H. est un tro isièm e nom bre qu an tiq u e co rresp o n d an t k f o n d e d e m atière selon la
- version indimcnsionnelle du puits largeur L . .
de potentiel infini de la figure 10.2 -
iyam les largeurs et /-. VERIFIEZ VOS CONNAISSANCES 4 ; f)a n ', la n o ta tio n de l ’é q u a tio n 10.20, la q u e lle des valeurs

f:
Miivanlcs représente l’énergie de l'état rondainental d*un électron daii.s un i;orail rectangulaire:
¿an, L'i,«- £iu ou ^?

Exemple 10.5
Un électron est emprisonne dans un corail Ciuré consumant un puits SOLUIION: I x tonteptcli utilisé ici est le suivant • éuuit doiuie <iiie l’élee-
de potentiel à deux dimcnsion.s (figure 10.12) où L, = ê , . fron esl emprisonné dans un puits bidimensionnel rectangulaire,
son énergie dépend de deux nombres quamiques, «, et n^. selon
a) Détermine/ les énergies des cinq plus ba.s niveaux d'énergie de
féquation 10.20. Hutsque le puits est carré, on peut dire que les
rêleetron, puis représente/.-les dans un diagramme.
largeurs sont = l^ .~ L.
288 Chapitre 10 La mécanique ondulatoire

TABLEAU 10T №veaux d'énergie b) Sous forme d ’un multiple de h-l&mLr, quelle est la différence
n. riy Énergie”'’ Énergie* d'énergie entre l’état fondiuncntal et le troisième état excité de
«ï
l’électron?
1 3 10 2 4 20
3 1 10 4 2 20 SOIUTION. Dans la figure 10.14, on constate que l’état fondamental est
2 2 S 3 3 18 l’état ( 1,1), qui a une énergie de 2(/iV8/?/L^. On constate aussi que le
1 2 5 i 4 17 troisième état excité (troisième état en haut de l’état fondamental
dans le diagramme des niveaux d ’énergie) constitue les états
2 1 5 4 1 17
dégénérés ( 1,3) et (3, 1). qui ont une énergie de ÎO{l<’/SmLr). Donc,
1 1 2 3 13
la différence AE entre ces deux états est
.1 2 13

^F.n imilliples de h^/SiiiL-


AE
■'“ ( » » y -’ ( s p ) -Uy (réponse)

On üiiiiplifie ensuite rcgualion 10.20, gui devient


13 P 5.2.A2..1
-„2 ( 10.22)

Les états d ’énergie les plus bas correspondent aux petites 10 A’ia
valeurs des nombres guantigues n, et qui sont les nombres entiers
ivisitifs 1,7. .Si on remplace et par ces noinbres entiers dans Ç b
l'égiiaiion 10.22. en commençant par la valeur la plus basse. 1, on QC
()cul obtenir les valeurs d’énergie données dans le tableau 10.1, O r
peut voir dans ce tableau que plusieurs des paires de nombres guan-
tiques (n „ n j ont la même énergie. Par exemple, les états (1, 2) et fo .l.él
(2, I) possèdent tous deux une énergie de 5{fr№mf.'). Chacune de
ces paires est associée à des niveaux d’énergie dégénérés. Notez
également, ce qui peut paraître étonnant, que les étals (4. 1) et (1, 4)
— - E,
pos.sèdent moins d'énergie que l’état (3, 3).
A l’aide du tableau 10.1 (où les niveaux dégénérés sont soigneu­
sement notés), on peut construire le diagramme des niveaux d’énergie
de la figure 10.14. Rgure 10.14 Exemple 10..5 Diagramme des niveaux d’énergie

10.8 L’atome d ’hydrogène


N ous som m es m aintenant prêts à étu d ier un électron lié à un atom e. Nou.s choisissons
l’atome le plus simple, l'hydrogène. Cet atom e est coastittié d ’un seul électron (chai gc —e)
lié à un noyau cen tral, fo rm é d ’un seul proton (ch arg e -i-c), qui so n t liés p ar la force
a ttractiv e co u lo m b ie n n e qui ag it e n tre eux. L’ato m e d ’h y d ro g èn e , co m m e to u s les
ato m es, est un pièg e à é le c tro n ; il co n fin e so n u n iq u e é le c tio n d an s une rég io n de
l’espace. Selon le principe d e confinem ent, on s’attend à cc que l'électro n ne puisse se
trouver que dans un en.semble discret d ’états quantiques po.ssédant chacun une énergie
donnée. Ü n veut ici d éterm iner les énergies e t les fonctions d 'o n d e de ces états.
Dans le chapitre 4 du volume 2, on a fonnulé l'équation 4.43 relative à l’énergie poten­
tielle (électrique) d’un systèm e form é de deux particules po.s.scdant les charges </| et q i :

V = _L
4m^o r
où r e s t la distance qui .sépare les particules. D ans le cas de l’atom e d ’hydrogène, on p<^ut
décrire l’énergie potentielle ainsi;
I fe)( e) 1 e
U = _ ' = -- - - - - - - - - . (1 0 .2 3 )
Amo r Aneu r
La co u rb e de la fig u re 1 0 .l.“! illu stre le p u its de p o te n tie l trid im en sio n n e l où
l'éleciron de l’alome d ’hydrogène est confiné. Ce puits diflêrc du puits fini illustré dans la
figure 10 7 de la m anière suivante dans le cas de l’atom e d 'h y d ro g èn e, V a une- valeur
négative pour tonies les valeurs de r parce q u ’on a (arbitrairem ent) choisi que le / cri
d 'é n e rg ie p olciuiellc corresp o n d à r Dans le cas du p uits fini d e la figure 10.7.
cependant, on a choisi (également de façon arbitraire) d ’attribuer la valeur nulle d'énorgic
potentielle à la région interne du puits.
10.8 L'otome d ’hydrogène 789

l'(eV) P our obten ir les états qiiantiqucs de l’atom e d 'h y d ro g èn e et leurs énergies, on doit
I ré.soudre l’équation de Schrôdinger, en y rem plaçant l'expression de l’énergie jxttentiellc
d o nnée à l’équation 10.23. C ep en d an t, étant d o n n e qu e l’clcctron q ui se tro u v e dans
l’atom e d ’hydrogène est em prisonné dans un puits à trois dim ensions, il fam utiliser une
form e tridim ensionnelle d e l ’équation de S chrôdinger Avant d ’étudier ces états quaii-
tiques, nous pré.senlons une brève revue historii|iie

Aperçu historique
figure 10.15 L'énergie p<4enlielle U
d'un atome d'hydrogène en fonction D ans le prem ier quart du XX* siècle, un des principaux problèmes de la physique était Ic
de la distance r qui ■»èparc l'élcclron suivant : quelle est la structure de l’alom e '? Il y avait beaucoup de rcsullats cxpcnm cnlnux
du proton central. La courbe est illustrée qui n ’avaient pa.s encore d ’explication. En particulici, on savait que les alotnes ém ellaient
deux fois (à gauche cl à droite) cl absorbaient de la lum ière à des longueurs d’onde bien définies 1 .a figuie 10.1ô illustre
IKHir aider la visualisation du puits les sp e ctres d ’ém issio n d e q u elq u es élém en ts. C e s sp ectres p eu v e n t être o b se rv é s it
й trois diinensioas à symétrie .sphérique l'aid e d ’un réseau, qui sépare les différentes rates de la lum icrc cmi.se par une lam pe à
dans lequel Pélcctron est confiné. décharge contenant un gaz. En particulier, le spectre de l’hydrogène contient quiitic raies
dans le visible.
La figure 10.17 a) illustre une portion, q u ’o n appelle la série d e Balm er, du spectre
d ’ém ission de l’hydrogène. L a figure 10.17 b) m ontre sch ém atiq u em en t que lu série
de B alm er n ’est q u 'u n e p artie du sp ectre d ’ém issio n d c I h y d ro g én c, c o n stitu é e de
p lusieurs séries di.serètes, ch acu n e portant le nom d ’un ch erch eu r qui l ’a d éco u v erte
ou qui l’a expliquée. R em arquez qu e chaque série o ccupe une section finie d u sp e rtie
électrom agnétique. P our la série de Balm er, la rate qui a la p lus grande longueur d ’onde
est une raie rouge et sa longueur d ’onde est À nm. La lim ite de la scflC du CÔtC
des petites lon g u eu rs d ’o n d e est m arquée d 'u n e flèch e et co rresp o n d à une lougiictir
d 'o n d e À = 364,6 nm . qui sc trouve dans ru llrav io let.
Les physiciens de l'ép o q u e, qui travaillaient à partir des fréquences plutôt t|iic des
longueurs d 'o n d e d es raies, essaycrcnl dc relier ces fréquences aux fréq u en c es d ’un
électron orbitant auum r du noyau. Malheureu.sement. ce « m odèle planétaire» de Taiom e
ne peut e x p liq u e r le siiectre de r a to i n e d ’h y d ro g èn e san.s in tro d u ire dc n o u v elles
hypothèses. R egardons pourquoi.
L'ne particule chargée accélérée doit ém ettre de l’énergie sous form e d ’ondes cIc c-
trom agnéliqucs. l in exem ple bien connu est l 'cncrgic cmi.se p ar un ém etteur radio muni
d ’une antenne émettrice. L'énergie est pKxluite pai les chaigcs qui oscillent dans rantenne.
U n électron, supposé décrire une orbite circulaire autour du noy au atom ique, subit une
accélération centripète com m e tout corps qui décrit un arc de cercle. De tefs c k c lro n s en
orbite, en ém ettant leur énergie, décriraient des spirales et tom lieiaient très rapidem ent
sur le noyau. Un m odèle atom ique basé sur un électron classique qui orbite Huiour d 'u n
noyau prédit donc un .spectre continu et des orbites instables.

Figure 1018 .Spectres d’émission


a) de l’hydrogène, h) de rhélium,
c.) du néon et d) du mercure
290 Chapitre 10 La mécanique ondulatoire

Figure 10.17 a) Une série de raies spec­


trales, appelée la série de Balmcr,
émises par l’atome d’hydnigcnc
tlans le visible et dans ruliraviolct
b) Un spectre plus étendu de l'atome
d’hydrogène, qui inclut la série de
Lyman et la série de Paschen. Deux
autres séries, la série de Hrackett
et la série de Pfiinrl. existent dans
rinfmi'ougc lointain, à des longueurs
d’otidc trop grandes pour être illustrées
dans cette figure.

2000

Le modèle de Bohr
En 1913, p lu s d 'u n e d éc en n ie av an t que la m é ca n iq u e q u an tiq u e n e so it in tro d u ite
p a r H eisen b erg cl S h ro d in g er. le p h y sic ie n d an o is N icls B o h r a propo.sé un m o d èle
pour l ’alotTie d ’hydrogène qui repose sur une com binaison astucieuse d e la m écanique
classique et d ’idées déjà existantes de la physique quanlique. M ême si le m odèle de Bolir
a ensuite été rem p lacé p ar un m oilcic qiiantique co m p let, il a q uand m êm e beaucoup
stim ulé les découvertes qui suivirent.
M êm e si B o h r savait qu e d ’après la m écan iq u e classiq u e, les o rb ites d 'é le c tro n s
n'étaicnl pas stables, il s'est pose une question. « Q u'est-ce qui se passerait si la mécanique
classique était incorrecte e t que les orbites étaient stables ? » Selon le langage m oderne,
on peut form uler le prem ier postulat de Bohr de la façon suivante ;

Les électrons d’un atome peuvent exister dans des états stables dont l’énergie c.st fixe,
sans rayonner de l’énergie.

D 'u n seul coup, B ohr a introduit le concept d ’étal quantique, un concept d ont tin
s ’est servi depuis le début de ce chapitre dans l ’étude des pièges à électron.
Bohl a alors introduit un second postulat audacieux :

lin atome émet (rxi absorbe) du rayonnement à une fréquence bien déterminée Iw.squ’un
électron de cet atotnc sc déplace d'un état quarwique à un autre étal ijuantiqne.

Ce pristulat est équivalent à ce qui a été form ulé à la .section 10.3 au sujet d ’un électron
piégé. La fréquence du photon ém is (ou absorbé) est donnée par l'éq u atio n 10.6:

h r
(10.24)

où h est la constante de Planck et L„. et t.„^. sont respectivement les énergies des états initial
et final (où E,.^ et sont inversées dans l'éq u atio n 10.24 quand le photon est absorlié).
A vec ces hypothèses. Bohr réduisit le problèm e de l'ato m e d ’hydrogène à la déter
m ination des états quantiques et d e leurs énergies. À p artir des én erg ies des étals, les
fréquences, et donc les lo n g u eu rs d 'o n d e , du spectre peu v en t être ca lc u lée s à l'a id e
de l'équation 10.24.
La quantification de l’énergie pour un cleclm n qui ,sC déplace au tour du noyau en
form ant des orbites circulaires im plique que le moment eiiiélique /. est quantifié. Pour que
le iiKidèle de B ohr donne les bonne.s longueurs d 'o n d e fiour le spectre de l'atom e d 'h y d ro ­
gène, le m om ent cinétique doit être un m ultiple entier de la t:onstaiire de Planck létluire

L„ - /m'r„ - /1*, ■* (10.25)

où II est le nom bre qiiantiqiie (« > 0) qui décrit les états quantiques, m est la m asse de
l’életitron. et #•„ et v„ sont respectivem ent le rayi'n de l'o rb ite et le m odule d e la vitesse
de l’électron dans l’étal quantique ii.
10.8 L atome d’hydrogène 291

En ap p liq u a n t la m é c a n iq u e cla ssiq u e au m o u v e m e n t d e l’e le c tro n , o n p e u t o b ten ir


le ray o n e t l ’é n e rg ie d e s o rb ite s d e l'é le c tro n a u to u r du p ro to n . S o n m o u v e m e n t e st un
m o u v e m e n t c irc u la ire c a u s é p a r la fo rc e c o u lo m b ie n n e e x e rc é e p a r le p ro to n . C e tte
fo rce e st d o n c une fo rce c e n trip ète ( F ^ im V r). À l'a id e d e 1'e q u a tio n 1.4 d u v o lu m e 2.
o n o b tie n t ;
" •t , ^ il. UO M )
4Treo I""
r
Si on utilise l’équation 10.25 p o u r rem placer v et q u ’o n isole / , on oliticnf (avec h = IiI I tt) :

tu / n ’ /i^ \ 1 ej
r„ \ 4 тг5 м^ У 4 я £о г2

=>r„ = { (1П.27)
Y-WIP*

L a c o n s ta n te a = h~F.^t{7one^) — 5 ,2 9 x 1 0 " " m = 5 2 .9 p m e st a p p e lé e le rayon


d e Bohr e t rep ré se n te le ra y o n d e l’o rb ite d e l'é ta t fo n d am en tal clans le mcKlcIe d e rin lu
D o n c, l'é le c tro n p e u t être su r u n e o rb ite d e ray o n a, on 4/j, ou 9a, . , m ais il ne p eu t pa.s,
p a r e x e m p le , ê tre su r u n e (»"bite d e ray o n 2a o u d e ray o n 6,2a.
ü n p e u t e n s u ite c a lc u le r l'é n e rg ie m é c a n iq u e d e l'é le c tro n e n a d d itio n n a n t so n
én e rg ie c in é tiq u e (K — mv^/2) à son é n e rg ie p o te n tie lle , d o n n ée p a r l'é q u a tio n 10.23 :
1
F„ =
2 4яс(| r„
S elo n l’é q u atio n 10.26. m v- = еУ(4пР(^г). S i on in sère la q u a n litlc a tio n d u ra y o n d e Гог
b ite , selo n l’é q u a tio n 10.27, o n o b tie n t l’é n e rg ie d e s é ta ls q u a n tiq u e s ;
1 c4
F« = r
2 47ti.ur 4aror

2 4теог„
1 ( TttnP \
2 4я £’о yh^eun^J

A p rè s sim p lific a tio n , o n o b tie n t:


me 1.3,6 eV
Ffî — 0 (1 0 .2 8 )
8 t^

L es én erg ie s so n t n é g a tiv e s p arc e q u e la c o n fig u ra tio n corrc.spondant à u n e e n c rg is nulle


Xon quanrifiii- a é té c h o is ie c o m m e la c o n fig u ra tio n o ù l’é le c tro n e st à u n e trè s g ra n d e d is ta n c e du
noyau et au repos. O n v érifie facilem en t q u e l’dcjiiation 10.28 rnène à L„ — 0 poui' r —^ ^ .
c ’e st-à -d ire n —» os. B o h r et p lu sie u rs autre.s phy.siciens o n t utili.sé ce m o d èle a to m u ju c
if p o u r o b te n ir d e s in fo rm a tio n s su r l'a to m e d ’h y d ro g è n e O n p e u t a u ssi g c n r r a lis n Ir
l-b m o d è le a u x é lé m e n ts io n is é s q u i ne c o n tie n n e n t q u 'u n é le c in m (H e* I I ’ ...).
-a .o

-4 .0
»
! П ’
1 u
Séi
St'rie
d<* Pas<’l»eii, L e n o y au a alo rs une c h a rg e Ze, o ù Z est le n u m é ro a to m iq u e. L e ra y o n cl l'é n e rg ie d es
é ta ts so n t alor.s :
i <lr RaliiKi’
hhi)
r„ =
* 6,0 " \m n Z (‘..-j " - T
mZV 1 (1 3 ,6 eV )Z -
E„ - - (1(170)
-6.0*
1 __
(fè* 1и Ix irs q u c B o h r a é ta b li so n m cxlèlc d e l'a to m e d ’h y d ro g è n e , l ’é q u a tio n d c
-lO.Oi- S h rix lin g er n 'a v a it p as e n c o re été a v an cée c l le p rin c ip e d ’im e rtilu d e n ’ctait p a s co n n u .
L es seu ls o u tils q u an tiq u es q u 'il avait à sa d isp o sitio n étaien t le co n cep t dc q u an tificatio n
-1Ж 0'
I Figure 10.18 Un diagramme illustrant quelques-uns de« niveaux d’énergie dc l’atome
I I
9»i d'hydrogène, selon l’équation 10.28. l.es transiiiniis soin gioupcej. en série, chai une jxirliint
г* 14,0 Série <1г 1.vm;ni le ivim d’une personne.
292 Chapitre 10 La mécanique ondulatoire

de Planck et le concept de photon d ’Einstein. Il e.st alors norm al que quelques-unes des
conclu.sions du m o d èle de R o h r so ien t fau sses. P a r ex em p le, on n ’im ag in e p lu s que
l’électron d ’un atom e se déplace sur des orbites bien définies. L ’idée m êm e d ’oitiites
n ’est pas com p atib le avec le p rin cip e d ’in c ertitu d e d e H eisen b erg ea r c e la im p liq u e
q u ’une particule ayant une quantité de m ouvem ent fixe occupe à un instant une position
bien définie ; or, ces deux quantités (la position et la quantité de m ouvem ent) ne peuvent
être connues avec une précision infinie de façon sim ultanée. M alheureusem ent, l'im ag e
courante de l ’atom e continue de répandre ce m odèle orbital incorrect.
C ependant, plusieurs des prédictions du m ixlèle d e B ohr sont valides. L’équation de
l’énergie des états quanliques de l’atom e d ’hydrogène ( l’équation 10.28) est la m êm e
que celle q u ’on obtient en résolvant l’équation de Shrddinger. De plus, le concept d 'états
quantiques d ’énergie fixe et l’équation donnant 1a fréquence des raies ( l’équation 10.24)
font partie d es fo ndem ents d e la m écanique quantique. La q u an tificatio n du m om ent
cinétique est aussi un résultat im portant, m êm e si l’équation 10.25 sera m odifiée pai la
théorie de Shrôdinger.
L a figure 10.18 illu stie les niv eau x d ’én erg ie de l'a to m e d ’hy d ro g èn e calcu lés à
l'a id e de l’équation 10.28. Les lignes verticales correspondent aux transitions, groupées
selon les différentes séries. R em arquez que chaque série de raies spectrales correspond
à différentes valeurs de /i; pour le m êm e état final n , . P ar exem ple, les raies spectrales
pour lesquelles l'é ta t final de l ’électron est /«( = 2 fo rm en t la .série de Balmcr. La flèche
rouge représente la raie de B alm er ayant la plus grande longueur d ’onde et correspond
à la transition de «, = 3 à n, = 2. La lim ite inférieure correspond à la transition de
à n, = 2. La série de Lym an est caractérisée par l’é tat final n, = 1 alors que la série de
Paschen correspond à l’état final n, = 3. Il existe deux autres séries non illu strées: la
série de B rackett correspond à «, = 4 et la série de Pfund correspond à «j = 5 . Les raies
d ’érai.s.sions correspondantes sont dans l’infrarouge.

Exemple 10.6
a) Quelle esi la longueur d’onde de la lumière dans le cas du photon h) (Quelle est la lotigueur d’onde de la limite de la série de Lyman ?
le moins énergique émis dans la série de Lynian des raies spectrales
SOLUTION; Ici, le concept dé est que la limite de la série rorrespond à la
de l’atome d ’hydrogène?
transition entre le niveau = sc et le niveau de référence (n, ~ 1 dans
SOLUTION: Ix: premier fontepl clé utilisé ici est le suivant : dans toute série, la série de Lyman). Selon l’équation 10.28, la différence d’énergie
la transition qui produit le photon le moins énergique est la liansition relative à cette transition est
entre le niveau de néférenct' qui définit la série et le niveau sc trouvant
immédiatement au-dessus. L’n deuxième concept dé est que, dan.s la AE - £ |- ( l 3 . 6 e V )
série de Lyman, le niveau de référence e.si n,- = 1 (figure 10.18). (^ 2 , 2)
Donc, la transition qui produit le photon le moins énergique est celle
( l 3 ,6 c V ) ( 0 - 1 ) = 13.6eV.
qui va du niveau /jj = 2 au niveau »if = I. Selon l’équaiion 10.28.
la différence d’énergie est I..a longueur d ’onde correspondante est déterminée comme en a),
et est
= = - ( 1 3 ,6 e V ) |;^ - 7^1 = H'.2eV.
hr
X=
~KË
Puis, selon l’équation 10.24 (A t ~ hf), si c/X remplacef on a
_ (6.63 X K )-’’’ J ■slO.OO X 10** m /s)
hr _ (6.63 X 10-^'’ J s)(3.(X) X 10^ m/s) (l3,6cV )(1.6f) X 10- 1'* ,f/eV)
X=
AE ~ ( 10.2 eV)( 1,60 X lO - i r i / ë v ï = 9,14 X 10'* m = 91,4 nm, (réponse)
= 1,22 X HV ^ m = 122 nm. (réponse)
La lumière ayant cette longueur d’onde se situe également dans
i a lumière ayant cette longueur d'onde se situe dans fultiaviolet. fultraviolet.
10.8 L'atome d'hydrogène 293

lABUAU 10.2 Nombres quantiques de l'olome dliydrogène


Symbtile Nom Valeurs possibles
fi Nombre quantique principal 1. 2, 3, ...
I Nombre quantique orbital 0, 1 ,2...... / 1 - 1
m, Nombre quantique magnétique - / , - ( / - l ) ...... t ( ( 1 1 .+ /

Les nombres quantiques de l’atome d ’hydrogène


N ous discutons m aintenant la solution d e l'éi]uaiion de Stiiodinijci lorst)iic la t'onnioii
énergie p oten tielle est d onnée p ar l’équation 10.23, q u i est p ro d u ite par rin ie ia c iio n
coulom bienne entre l’électron cl le proton. N ous ne donnerons pas ict les détails de la
so lu tio n m a th ém atiq u e, q u i d ép a sse n t larg em en t le niveau de ce te x te. Les valeurs
ir é n c rg ie s sont les m êm es q u e c e lle s p rév u es p a r le m o d è le de Hnhr. d o n n ée s par
l’équation 10.28. P ar la suite, nous donnons les solutions pour les lo m lions d ’onde.
B ien que Ic.s énergies caractérisan t les états de ra io m e d 'h y d ro g è n e puis.scnt être
décrites com plètem ent par le nom bre qu an tiq u e «, les fonctions d ’onde d éciivaiii ces
états nécessitent trois nom bres qtianüques correspondant aux trois dim ensions où l ’élec­
tron peut .se déplacer. C'es tro is n o m b res q u an tiq u es fig u ren t, accompagnés de leurs
nom s, dans le tableau 10.2.
C haque ensem ble de nom bres quantiques (n, I. m,) d â rit la Fonction d ’onde d ’un
état quantique particulier. Le nom bre quantique n. appelé nombre quantique principal,
apparaît d an s l’équation 10.28. qui d onne 1 en erg ie de l ’ctat. Le nombre qimotique
orbital / est lino m esu re d u m odule du m om ent cin étiq u e asso cié à l ’état q u antique.
L e nombre quantique magnétique /n, est associé à l’oi ientaiion ilans l ’cspacc de son
vecteur momeni cinétique. Les resüictions lelativcs aux valeurs des nom bres quantiques
de l’atom e d ’hydrogène, com m e l'in d iq u e le tableau 10.2. ne sont pas a rb itra iie s, elles
viennent plutôt de la .solution de l'éq u a tio n de Schrôdinger. N o te/ que, dans le cas de
l’état fondam ental (n — 1), les restrictions com m andent que l = O etm j — 0 Autrem ent
dit, l ’atom e d ’hydrogène qui est à son état fondam ental possède un m om ent cinétique nul.

^V ÉR IFIEZ VOS CONNAISSANCES 5: Un groupe d’états quantiques de rntoinc d ’hydrogène


correspond an = S. Combien de valeurs de / sont possibles pour les éuus de ce groiqie
b) Un .soas-groupe d’ctais de l’atoine d’hydrogène contenu dtias le groupe n 5 correspond
à / = 3. Combien de valeurs de m, sont possibles pour les états de ce srnis-gmnix:

La fonction d ’onde de l’état fondamental


de l'atome d ’hydrogène
En résolvant l’équation tridim ensionnelle de Schrodinger et en norm alisant le résuh.u,
on obtient la fonction d 'o n d e suivante pour l’étal fondam ental dc l ’atom e d ’h vdrogénc:
1
xlj{r) - (l’état fondamcnlab. (10.30)

Ici. U = 52,9 pm est le ra y o n de B o h r, qui a été introduit à l'cq u atio n 10.27. Ce rayon
est grossièrem ent considéré com m r le rayon a'ici d ’un atom e d ’hydrogcnc et s'av è re une
unité com m ode d e longueur |H>ur les cas m etianl e n je u des ilinieiisions atom iques
C o n u n e dans le cas des autres fonctions d ’onde, ^ {r) de l’équation 10 30 n 'a pas
de sig n ific atio n physiriue, m ais v'/-’(r) en a une. Il s’agil de la d cnsitc de p ro b ab ilité
(probabilité par unité de volum e) que l'éleelroii puisse être détecté Plus s|iéi ifiqiu iiicnt,
t//’(r ) f/V'est la prolsabilitc que l’élcctron puisse être détecté dans un elem ent dC S’Oiume
dV (infinitésim al) situé à un rayon r du centre de l ' atonie

probabilité dc d étec tio n \ édensité de piobabilité)

(
dans le v olum e d \ | i/\r ) (volum e d V ) (1 0 3 1 )
au rayon r ) l au rayon r j

Ici. p u isq u e i/r-ir) d ép e n d seu lem en i d e r. il e«i lo g iq u e de ch o isir, com m e clém en t


de volum e dV, un volum e contenu entre deux sphères creuses coiicentriqurs d e l ayon.s /
et r t dr C 'e s t donc dire q u 'o n determ ine l'éléraenl dc volum e d t ainsi :

dV ~ ( \7Tr-) dr, (10.52)


294 Chapitre 10 La mécanioue ondulatoire

10 où e.st l’aire d e la sp h ère creu se in tern e et dr est la d istan ce rad ia le q ui sépare


i A i les deu x sphères creuses. Si on com bine en su ite les éq u atio n s 10.30, 10.31 et 10.32,
F on obtient
S! Cu
P; probabilité de d clcc tio n \ ^
' -K

(
s 5 dans le volum e d V 1 = i//-(r) dV - ~ e ' ^ '' “r ’ rir. (10.33)

■.i
i1
# ! au rayon r J ^
s 's . .
) A.
0 r>0 100 l.'K) 2(M) 2'>0 11 est plus facile de d é c rire la p ro b a b ilité d e d étec tio n si on u tilise u n e densité
Kiiy<iii (pin) de probabilité radiale P(r) au lieu d ’une densité d e probabilité La grandeur P(r)
est une probabilité par imite de longueur (elle que
Figure 10.19 Un graphique de la densiié
de prohabilité radiale P{r) lorsque densité de p ro b ab ilité) / , , /d e n s ité d e probabilité)

(is)M
(
l'aionie trhydrogcnc est à l ’état
tondamental. La marque triangulaire r ™ '““ " '"
au rayon r } \ au rayon r /
SC trouve à un rayon tic Bohr
de l'origine, qui représente
ou P(r) dr - ifH>) d \ ' ( 10.34)
le centre de l'aionie.
Si on y rem place Tj/\r) dV par son expression dans l’éq
P O ) = - - r ^ e -Tr/iJ
4
(la densité de probabilité radiale, l'état fondamental de l’atome d'hydrogène). ( 10.35)

La fig u re 10.19 e st une repré.sentation g rap h iq u e d e l ’éqiialion 10.35. L ’a ire sous la


courbe est l'u n ité ; c ’est donc dire que

P ( r ) d r = I. (10.36)
f
Jo
C ette équation indique sim plem ent que. dans un atom e d 'h y d ro g èn e, l'électron doit se
trouver quelque part dans l’espace entourant le noyau.
La m arque triangulaire sur l'a x e horizontal de la figure 10.19 se trouve à un rayon
de B ohr de l ’origine. Le g rap h iq u e in d iq u e q u e, d an s l’é tat fo n dam ental d e l ’atom e
d ’h y d ro g èn e, c ’est à ce lte d istan c e du cen tre d e l ’ato m e que l ’é lec tro n est le plu.s
susceptible de se trouver.
L a fig u re 10.19 co n tred it clairem en t l’idée p o p u laire qui veut q ue les élec tro n s
suivent des orbites bien définies dans les atom es, com m e le font les planètes autour du
Soleil. Cette idée, bien que courante, est erronée. L a figure 10.19 indique tout ce q u ’on
ne saura jam ais sur la position de l'électron dans l'état fondamental de l’atom e d ’hydro­
gène. La question appropriée n ’est pas « à quel instant l’élection arrivera-t-il à tel ou tel
p o in t? » , m ais plutôt « q u elles sont les chances que l ’électron soit détecté dans un petit
volum e centré à tel ou tel point ? ». L a figure 10.20, qu’on peut qualifier de diagram m e
de points, suggère la n atu re p ro b ab iliste de la fo n ctio n d ’o n d e et fo u rn it un m odèle
m ental utile de l’atom e d 'h y d ro g èn e dans son état fondam ental. Im aginez l ’atom e dans
cet état com m e une balle de peluche au contour mal défini et sans signe d ’orbite.
Il n ’est pas facile pour un débutant d 'im a g in er les particules siibaiom iques de cette
m anière probabiliste. C elle difficu lté découle d 'u n e tendance naturelle à im aginer un
électron com m e un m inuscule pois se trouvant à certains endroits à certains instants et
suivant une trajectoire bien définie. Les électrons et les autres particules élém entaires ne
SC com portent tout sim plem ent pas ainsi.
L 'é n e rg ie de l'é ta t fo n d am en tal, d é te rm in é e en in séran t n — I dan.s l ’é q u atio n
10.28. est £., = —13.6 eV. Vous obtenez la fonction d ’onde de l’équation 10.30 si vous
résolvez l ’équatio n de S ch rô d in g er avec celle v aleu r d ’énergie En fait, vous pouvez
tro u v e r une so lu tio n à l'é q u a tio n d e S c h rô d in g e r p o u r toute v aleu r d ’é n e rg ie, par
■••V. exem ple h ~ —11.6 eV ou 14.3 eV. C ela peut suggérer que les énergies des états de
l’atom e d ’hydrogène ne sont pas quantifiées, m ais on .sait q u ’elles le sont.

Figure 10 20 Un « diagramme de |>oints » illustrant la densité de probabilité ijdu-) (non pas la dt'nsilé
de probabilité nuliale Pir)) lorseiuc l'atome d’hydrogène est à l’étal fondamental. La densité
des points chute de manière cx|X)neniii-.llc avec l'augmentation de la distance par rapport
au noyau, représenté ici pai un point rouge, l n tel diagramme fournit une image mentale
<lii « nuage électronique » d'un atome.
10.8 L’atome d’hydrogène 295

C e casse-têle a etc réso lu q u and les physiciens ont co n state qu e ces so lu tio n s de
l’équation de S chrôdinger ne sont pas acceptables du point de vue de la p h j sique parce
q u ’elles donnent de grandes valeurs croissantes quand r —» C es « fo n ctio n s rTondc»
m o n tre n t q u ’on a p lu s d e ch a n ce s d e d étec ter l ’élec tro n loin du n o y au yiic prè.s du
noyau, ce qui n ’a au cu n sen s. On se d é b a rra sse d e ces so lu tio n s non d é sirc c s en
im posant des conditions dites aux limites, qui consistent à acccplci seulem ent les solu
lions de l’équation d e vSchrodinger pour lesquelles ^ ( r ) 0 quand ; oc, l ’csi-à-d irr
q u ’on n 'a c c e p te d e tra v a ille r q u ’av ec d es électrons co n fin és. C om pte tenu de ce iie
restrictio n , les so lu tio n s d e l ’éq u atio n de S chrixitnger fo rm e n t un en sem b le discret,
com portant des énergies quaiuifiées détei m inées par l’équation 10.

Exemple 10.7
rVmonlrcA que la densité de probabilité radiale de l’état fondamenial ■trio
de l'atonie d’hydrogciic est maximale à r = «. dr - y a J ô'
SOLUTION: Un premier cotitepl dé est que la densité de priibabilité radiale Ë: 2r/ß _ ^ Zrfu
d 'un atome d’hydrogène à l'ctal fondamental est déterminée par ~ «3^ O* ^
l'équation 10.35,
Z r ia
r)c
^2^-2r/o
n-
Si on attribue la valeiii zéro au membre de droiu-, ou obtient une
Un deuxième contepl dé est que, pour déterminer le maximum (ou le équation vérifiée si r = «. Autrement dit. dPIJrtsi égal à zéro qnniiri
minimum) d ’une tônclicm, il faut la dériver et attribuer la valeur zéro r = (I. (Notez qu'on a aussi dF/dr O J i» = r ie i à r = oo C ipiidani,
au lésuliai. Si ou dérive 1\r) par rapport à x en utilisant la dérivée 7 ces conditions correspondent à un minimum dans F{r), cwiime on peut
de l'annexe D et la règle pour dériver les pixxluits, on obtient le voir dans la figure lO.lh.)

Exemple 10.8
On peut démontrer que la probabilité p(r) que l’électron d’un atome de rayon r. On cherche le rayon de la splière où />(r) = 0.90. Si on
d ’hydrogène à l’état fondamental soit détecte dans une sphère de insère cette valeur dans l’cxprcssion de p(/'), on obtient
rayon r est determince par
0.90 - 1 - e ’'(l 2x ‘ 2x~)
P (r )= I -r? -^ '(l Id).
ou lOr ^'(1 + 2.x ' 2.d) - 1.
où X, grandeur sans dimension, est égal à r/a. Déterminez r lorsque
On doii üouver la valetir de x qui conr.siKind à cette égalité. 11 n’c.sl
p(r) - 0,90.
pas ptissible d’isolcr x. mais un rcsolvcur d'équations de calculatrice
SOlUTlON: Ici. le coiHept dé est qu’on n’est pas sûr de dcteeicr l’élecnon donne \ = 2,66. Cela signifie qiK- le rayon d’uiic .sphère h l’mlérictir
à une distance radiale r particulière du centre de l’atome d ’hydro­ de laquelle la probabilité de dcleciion de rélcctron .serait de *>{)%
gène. Cependant, grâce à la fonction rlonnée, on peut calculer la est 2.éitv7. Marque/celte position sur Taxe hoi izonlal de la figure 10.19;
probuhililc que l'électron soit détecté cpwlquc pnrt dans une sphère s’agit-t-il d’une réixinse vrai.scniblublc ?

lïlBlfAU 10.3 les nombres quonliques Les états de l’atome d’hydrogène quand n =2
des états de l'atome
Selon les dtmnéev du tableau 10 2, il y a quatre états de l'atom e d ’hydrogène quand « — 2 ;
d'hydrogène quand n 2
leurs nom bres quantiques sont présentés dans le tableau 10.3. P ieiie/ J ’ahord l'rhat où
#7 / m.
n = 2 et / = Mt, = 0 ; sa densité de probabilité est représentée p ar le diapramme de points
2 n 0 de la figure 10.21. N otez que ce diagiamiTx*, com m e celui de l’état londam ental illu.stré
? 1 t1 d ans la figure 10.20, a une sy m étrie sphérique. C 'e s t-à d ire que, dans un systèm e de
0 coordonnées sphérique com m e celui défini rian.s la figure 10.22, la densité de probabilité
2 1
est fo n ctio n d e la seu le cix ird o m tée rad iale i et esi in d ép en d an te d es co o rd o n n é es
2 1 -l
angulaires fi e t <f>.
O n co n state q u e tous les états q u an tiq u es, q u an d I ~ 0 . possèd en t des foiu iions
d ’onde qui ont une sym étrie sphérique. C 'est s rai.scmblablc parce que le nom bre quan
tique / est une m esure du m om ent cinétique ttssocié à un étal donné. St / “ 0, le m om ent
cinétique est égalem ent nul, ee qui fait que la densité de probabilité représentant l'état
n ’a pas d ’axe d e sym étrie pr6d(>minant
296 Chapitre 10 La mécanique ondulatoire

L a figure U) 23 rep rése n te les d iag ram m es d e p o in ts d e tp~ (r, в) d es tro is états
quand и = 2 cl / = 1. Les densités de probabilité de ces ciais quand m, = I l et гиу = - 1
sont identiques. B ien que ces diagram m es soient sym étriques par rapport à l’axe des z,
ils /t'o n t pas de sym étrie sphérique, c ’est-à-dire que les densités de probabilité de ces trois
étals .sont fonction de r et de la coordonnée angulaire в.
Voici un c a s s e -tê te : q u ’e st-c e q u i, d an s l ’ato m e d ’h y d ro g èn e , é ta b lit l’axe d e
sym étrie si évidetil de la figure 10.23 ? Réjw nse : uhsolum -nt rien.
I-a solution d e ce casse-tête apparaît quand on constate q ue les trois étals illustrés
dans la figure 10.23 pos.sèdent tous la même énergie. Rappelé? voas que l’énergie d ’un état,
déterm inée par l'éq u atio n 10.28, dépend seulem ent dit nom bre quantique principal n et
est indépendante de / et de nt,. En fait, dans le cas d 'u n atom e d ’hydrogène isolé, il n ’y
figure 10.21 Un diagramme de poinis illus­ a aucun m oyen expérim ental de distinguer les trois états illustrés dans la figure 10.23.
trant la densité de probabilité f \ r ) de Si on additionne les densités d e probabilité des 3 états où n = 2 et i = 1. la densité
l'atome d’hydrogène dans l’éiat quantique de probabilité com binée se révèle avoir la sym étrie .sphérique, sans axe unique. O n peut
où n = 2, / — 0 et W; = 0. Le diagramme
alors penser q ue l’électron passe le tiers d e son tem ps dans chacun des 3 états illustrés
a une symcirie sphérique. |wr rapixni au
d ans la fig u re 10.23. et on peut im ag in er la som m e pond érée des 3 fonctions d ’onde
noyau ccnlia;. L'espace vide dans le mont
indépendantes com m e la définition d ’une sous-couche à sym étrie sphérique, décrite par les
de densité marque une surface sp^iériquc
où = 0. nombres quantiques n ~ 2,1 = 1 .1 es états individuels révéleront leur existence distincte
seulem ent si on place l’atom e d ’hydrogène dans un cham p électrique ou m agnétique.
L es 3 états de la so u s-co u c h e n = 2, l = 1 au ro n t alo rs d iffé re n te s é n e rg ie s, et
l’orientation du cham p établira l’axe de sym étrie nécessaire.
l ’état и = 2. / = 0, dont la densité d e probabilité est illustrée dans la figure 10.21,
possède également la m êm e énergie que cliacun des 3 états illustrés dans la figure 10.23.
On peut im aginer q ue les quatre états d ont les nom bres quantique,s sont nom m és dans
le tab leau 10.3 fo rm e n t une c o u c h e sy m é triq u e sp h é riq u e , d éc rite u n iq u em en t p ar
le nom bre quantique n. L ’im portance des couches e t des sous-couches vous deviendra
évidente dans le chapitre 11. où vous étudierez les atonies possédant plus d ’un électron.
Pour com pléter l’im age q u 'o n se fait de l’atom e d ’hydrogène, on observe, dans la
Figure 10.22 La relation entre les coordon­ fig u re 10.24, un d ia g ra m m e d e p o in ts de la d en sité d e p ro b a b ilité rad ia le d ’un état
nées X. y et Z du sy.stèmc de ctiordonnées de l’ato m e d 'h y d ro g è n e ay an t un n o m bre q u an tiq u e rela tiv e m en t élev é (/? — 4 5 ) et
cartésien et les coordonnées r. S et du le nom bre q u an tiq u e o rb ital le plus élevé p erm is p ar les restrictio n s du tableau 10.2
sy.stème de coordonnée.s sphérique. Ce (/ = « — I = 44). La densité de probabilité radiale form e un anneau sym étrique autour de
dernier est plus approprié dans l’analy.se Taxe des z, anneau qui repose très près du plan des ry. Le rayon moyen du rayon est n^a,
des situations menant en jeu la symétrie, où a est le rayon d e Bohr. C e rayon m oyen représente plus d e 2 000 fois le rayon réel de
sphérique, comme dans le cas de l’atome l’atom e d ’hydrogène à l’état fondam ental.
d’hydrc>gcnc.

n tl 1 1

a) Ы
figure 10.24 Un diagrainine de ptsints de la densité de probabilité
figire Ifl 23 niagranim««: de point.*; de la densité de probabilité ^/^r, 6} radiale P(r) de l’atome d’h y d r^èo e dans iin état quantique ayant
de l'ütomc d'hvilmgcnc* dans 1rs étals où n - 2et I = l a ) Oia- un nombre quantique principal relativement élevé (à savoir « ;= dS)
gramme de m, ^ l) b) Diagramne de 1 et m, = —I. et tm nombre quantique de moment cinétique 1 ~ n ~ 1 44.
Ces deux diagrammes montrent que la densité de probabilité Les points se iix>u\-ent près du plan des .rv ; l’anneau de points
est syméiritiue par rapivirt à l’axe de.s z. suggère une orbite éiccironique classique.
Revision et résumé 297

L a figure IÜ.24 lai.vsc penser que l’électron suit une orbite décrite p ar la physique
classique. O n est donc en p résence d 'u n e autre illustration du principe d e co rresp o n ­
dance d e B o h r - à savoir que, en présence d ’un nom bre quantiqiic élevé, les p réd it lions
de la m écanique quanlique co riesp o n d cn i de plus en plus à celles de lu pliy.suiuc clu.s
sique. Im aginez l'asp ect d ’un diagram m e de point.s com m e celui de lu ligure I0.2'l dans
le cas de valeurs irés élevées de n et de / ; /t — 1 (MH) et 7 “ 9 W , p ar exem ple.

C S iJI--ll.l!l:AJ:a4ll!ll
L e p rin cip e de c o n fin e m e n tLe principe de confinenient L'equation 10.14 mène à lu eonsiaiatioii i|ue la plus basse
s'applique aux ondes de toutes .sortes, y compris les ondes dans une énergie permise dans le cas de l'éleclron n'est pas nulle, mais qu’elle
cordc et les ondes de matière décrites en physique quanlique. Selon est l’énergie qui correspond ù n = 1. Cette énergie la plus faible, pai
ce principe, le confmcmeni mène à une quantification - c’est-à-dire ailleurs, correspond à 1 énergie de l'ériit foniUimenial du système
à rcxi.stcncc d'etats discrets possedam des énergies discrètes. électron-puits.

L 'électro n d a n s un p u its de p o te n tie l in fin i Un puits de L ’électron dans un puits de jw icntiel fin i I in puit.s ilc potcniitl
poteniiel infini est un appareil pernietiaiu de confiner un électron. fini en esi un où l’énergie potentielle d'un électron s’y trouvant est
Fn MT fondant sur le piinci|^ de confinciiienl. on s’attend à ce que inférieure d’une quantité finie Un à celle d'un électron sc trouvant ù
l’onde de matière représentant un électron emprisonné ne puisse rextcrieiir [.a fonction d'onde d’un cicciron piégé dans un tel puit»
exister que dans un ensemble d’étals di.scrcts. Dans le cas d'un puits péncfre dans les murs.
de |xi(cniiel infini, les énergies as.siKiées à ces états quantiques sont
Les pièces à électron bidim ensionnels et tridim ensionnels
Les énergies quantifiées d'un électron piégé dans un puits de potentiel
pour и = 1,2. 3, (10.4) bidimensionnel infini qui forme un corail rectangulaire sont

où L est la largeur du puits et n est un nom bre quantique. Pour que ( 10. 20 )
...... ц ) ‘
l’électron passe d ’un étal à un autre, son énergie doit varier d’une
quantité
où n, est un nombre quanlique décrivant la direction parallèle à la
tIO.5) largeur L, du puits, et /î, est un nombre quantique corivspondam à la
direction parallèle à la laigcur du puits. De même, les énergies d'un
où est l’énergie In plus élevée et est l’énergie la plus basse
électron empnsonnc dans un puits de potentiel iridimensionnel infini
Si la variation se fait par absoiption ou émission d’un photon, l’énergie
qui forme une boîte rectangulaire sont
de ce dernier doit être

hf= (10.6) /|2


(10.21)
Les fonctions d ’onde a.ssociées aux états quantiques sont 8»я

A U ) = A sin^ - ^ . ï j , pour и = 1 .2 ,3 .... (Ш .Ю ) Ici, n. e.M un troisième nombre qiiantiquc décrivant la direction
parallèle à la largeur f i du |xiiis

1XI densité de probabilité Vr rXx) d’un état possible a comme significa­ L ’atom e d ’hydrogène La fonetiiiii O’èiieigie jviieniielle pour
tion. en pliysique. que \ îo(a ) d \ est la probabilité que l'électron soit l’atome d’hvdrogène est
détecté dans la zone située entre .r e t .» + dv. Dans le cas d’un électron
I
se trouvant dans un puits infini, les densités de (irobabilité sont U (in.ljl)
4ТГГП r

^,f(.v) - / 4 ’ sin’ ^ — A^, pour « — 1,2, 3, ... (1012) I .es énergies des états i)uantiqurs de raioinc d ’hvdrogènc .sont deter
minées par la forme iiiilimensionnelle de l’équafioii de Schriidingcr
En présence de nombres quantiques élevés n. l’électron tend à ou par le iiKidèle de B nhi.
adopter un comportement classique en ce sens qu'il tend à occuper 13.6 cV
toutes les parties du puits avec une probabilité uniforme. Ce fait pourn - 1 .2 ,3 ... (10.28)
n-
mène au principe de correspondance ; quand les nombres quantiques
sont suffisamment élevés, les prédictions de la physique quantique tiii II est le nom bre qii:inlH|iie principal. l a descrijaion tom[ilrtr de
correspondent de plus en plus à celles de la pliysique classique. l’atome d'hydrogène néce.ssile troi.s iioinhics quaniiqia-.s ; Iriiis iinins
et leurs v.ilcurs possibles sont présentés dans le tableau 10 2
La norm alisation e t l ’énergie de l ’état fo n d a m e n ta l On
La densité de p ro b ab ilité rad iale f fr ) d 'un état tic l'aionic
peut déterminer la constante de norinalisation 4 de l’équmion 10.12
d'hydrogène est définie de telle soi le (jti'clle ем la jMohabilitc que
à l'aide de l'équation de normalisation. l'éleclron soit détecté entre deux sphères crcüM s ciHKeniiiqucs qui
sont centrées au noyau île Г а п п п е ei diiiii les i ayons si ml r rt r -l th
Фr^s)d\ 1. (10 14) Dans le cas de l étal fondainema) de ГаЮше d’hydrogène
L
qui montre que l'éleclron doit sc trouver quelque part dans le puits, 4
r ir )= .r^c (lü-tS)
car la probabilité I im|>lique la certitude. fiJ
298 Chapitre 10 La mécanique ondulatoire

où a, le rayon de Buhr, est une unité de longueur d ’une valeur rapport à l’axe des z, mai.s, additionnés, ils ont aussi une symétrie
de 52,9 pm. La figure K) 19 est une représentation graphique de PÇr) sphérique.
de l’état fondamental. Quand n = 2, les 4 états ont tous la même énergie et peuvent
I.XS figures 10.21 et 10.23 représentent Ic.s densités de proba- pratiquement être con-siderés comme une couche, nommée couche
bilité (non pas les densitcs de probabilité radiale) des 4 états de n = 2. Les 3 états de la figure 10.23, pris ensemble, peuvent être
l'atome d ’hydrogenc quand n = 2. Le graphique de la ligure 10.21 considérés conune la suus-co u ch e n = 2, l = 1. Il e.st impossible
(« = 2,1 = 0. m, = 0) a une symétrie sphérique. Ixs graphiques tic la de séparer les 4 états n = 2 expérimentalcmetit. il moins que l’atoinc
figure 10.2,3 (n = 2, / = I, w, = 0. I- 1, —U sont symétriques par d'hydrogène ne sc trouve dans un champ électrique ou magnétique,
ptxir établir un axe de symétrie défini.

r m

1. Si vous doublez la largeur d’un puits de potentiel infini unidimen-


27
sioniiel. a) l’éneigic de l'étal fondamental de l’électron emprisonné — •2.5
csf-t-clle multipliée par 4, par 2, p a r i- pat ^ ou par un autre
---- 19 20
nombre ? h) Les énergies des états supérieurs sont-elles multipliées pai'
ce facteur ou par un autre, selon leur nombre quantique ? ---- 16 .... 10

2. Trois électrons sont piégés dans trois dilTérents puits infinis à une 12
dimension doni les largeui-s sont a) 50 pm. b) 20() pm cl c) l(K) pm. - 9 «
Clas.sez CCS électrons scion les énergies de leur état fondamental, 4
2 — .5
en commençant par l’énergie la plus élevée. ____1
3. Si vous vouliez utiliser le piège idéali.sé de la figure 10.1 pour tt C I)
emprisonner un positron, auriez vous besoin de modifier a) la Figure 10.26 Question 6
géométrie du piège, b) le potentiel électrique du cylindre central ou
c) les potentiels électriques des deux cylindres semi-infinis des 7. L’énergie de l’état fondamental d’un proton emprisonné dans un
extrém ités? (Ln positron possède la même niasse qu'un électron, puits de potentiel infini à une dimension est-clIc supérieure,
mais est chargé positivement) inférieure ou égale à celle d’un électron emprisonné dans le même
puits de prHenliel ?
4. Ln électron est emprisonné dans un puits de potentiel infini à une
dimension dans un état où m = 17. Combien de points de a) probabilité 8. 1In proton et un électron sont j>iégés dans des puits de |X)tcntiel
nulle et b) de probabilité maximale son onde de matière pc>ssède-t-elle? infini à une dimension identiques ; chaque particule est dans son état
fondamental. Au centre des puits, la densité de probabilité du proton
5 La figure 10.25 illiisüc trois puits de potentiel infini, chacun sur
est-elle supérieure, inférieure ou égale à celle de l’électron ?
un axe des e. Sans calcul explicite, déterminez la fonction d’onde
d’un électron à l’étal fondamental piégé dans chaque puits. 9. Vous voulez niixiifîer le puits de potentiel fini de la figure 10.7
pour permettre à l'électron qu’il contient d’exister dans plus de trois
rj
états quantiques, a) 1.e ferez-vous en élargissant ou en amincissant le
-, P lîl puits ? b) Le ferez-vous aussi en augmentant la profondeur rlu puits
--f ' t >
(' a ' .1 é s
i', ou en la réduisant ?
■r ■ f ■■■ *<

•».C-.-'T “ , -g. ._ ... , ,


1 4
:~r-. >
■é
^ ir. V ’
10 Un électron est piégé dans iin puits de potentiel fini suffisamment
•2L 0 /., 2 - / ./ 2 +L/2 profond pour lui permettre d’exister dans un état où n ~ 4. Combien
de points a) de probabilité nulle et h) de probabilité maximale son
Figure 10.25 Question 5 onde de matière possède-t-elle à ITntérieur du puits’’
11. En vous fondant .sur un examen visuel de la figure 10.8, classez
6. I a figure I0.2fi donne les nix’eaux d’énergie les plus bas (en élec­ les nombres quantiques dc.s trois états quantiques selon la longueur
tron volt s) dans cinq cas où un électron est piégé dans un puits de d’onde de de Broglie de l’clectTon, en commençani par la longueur
[Xiientiel infini à une dimension. Dans les puits R. C, l) et L, l’électron d’onde ayant la plu.s grande valeur.
est dans l’état fondamental. ( )n excitera l’électron ,se trouvant dans le
12. Un électron, piégé dans un puits de potentiel fini semblable à
puits A pour le faire monter au quatrième étal excité (à 25 eV».
celui de la figure 10.7. esi diuis son état fondamental, a) Sa longueui
L’élcciron poiina alors se désexciter poiii revenir à l’état fondamental
d'onde de de Broglie, b) le module de sa quantité de mouvement
en émettant un ou plusieurs photons, correspondant à une longue
et c) son énergie sont-elles supérieures, égales ou inférieun-s à ce
transition ou à plusieurs petites transitions. Quelles énergies de pho
qu’elles seraient si le puits de potentiel était infini, comme dans la
tons émi.v lors de celle désexcitation correspondent à une énergie
figure 10.2 ?
d’iin photon qui peut elre absorbé (à partir de l étal fondamental)
chez les quatre autres électrons Domic/ k-s nombres quantiques qui
y coircspumlfiK
exercices et problèmes 299

13. Le tableau présente des nombres quanliques de einq états de même masse qu’un proton, mais a une chiuge opposée. Selon vous,
l’atome d’hydrt>gène proposés. Lcsquel-s sont impossibles ? le spectre de l’anti-hydrogène est-il identique à celui de l’hydrogène
normal, ou en est-il different?
1 m, 15. a) À l’aide de la figure lO.lR, qui est le di.agramme des niveaux
d’énergie de ratom c d’hydrogène, voua ixiuvw; démontrer que l'sncr
a) 3 2 0 gie du photon correspondant à la deuxieme raie spectrale de la séiie
h) 2 3 1 de 1 ynian est égale à la somme des énergies des iihoton» tor icspoiidunt
C) 4 3 -4 à deux auties raies, üiiclles sont ces deux raies ? b) L’cncrgic du photon
eorrespondant à la deuxième raie speeu'ale de la série de I .ynaii esi
d) 5 5 n aussi égale à la (liffârenct' entre les énergies îles iiIkxoïiv rorirsj* imlani
c) 5 3 -2 à deux autres raies Quelles sont re.i raies’’
lé. Un atome d’hydrogèiK est dans son troisième état exeiti' À quoi
14. Fn 11)%, des physiciens travaillant dans un laboratoire contenant état (cUmne/. le nombre quantique n) devrait il passer pour aj ('’inetire
un accéléiaieur ont réussi à pnKluire des atomes d ’anti hydrogène, une lumière ayant la plus graiKlè longueur d'onde possible, Iq onieiiie
l n tel atome est constitué d’un positron se déplaçant dans le champ une iumièie ayant la plus courte longueur d'onde possible ei c) atisor-
électrique d ’un antiproton. Un positron po.ssède la même masse ber la lumière ayant la plus grande longueur d'onde |xissihle ■>
qu’un électron, mais a une charge oppt'sée. Un antiproti« possède la

-■s-yvvv
•v< v ■

les niveaux « « 3 cl n 4 ? b') Quelle paire (le cas échéant) ariicliei a


www La solution SC trouve sur le site Web. à l’adresse ci-dc.ssotts :
deux fois celle différence d’énergie ?
www.dlcmcgrawhill.ca/physiqiie
IIP Un électron esi piégé dans un puits infini à une dimension d’mx-
largeur de 2.‘>0 pm, el il se rmuve dans son état foiidanicntal S'il n’y
SECTION 10.3 Les énergies d'un électron piégé a absorption que d'un .seul photon. quelles soni les quaiie pins grandes
longueurs d’onde de la lumière pouvant I exciter (par rapport a son
lE. Quelle est l’énergie de l’état fondamental a) d’un électron et b) d’un
état fondamental) '? www
proton si chacun est piégé dans un puits de jxuentiel infini à une
12P. Supposez qu’un électron piégé dans un puirs infini à une dimen­
dimension d’une largeur de 100 pm ?
sion d’une largeur de 250 pm passe de son premier éiai excité à smi
2E. Vous voulez réduire de moitié l’énergie de l’étal fomiamental
troisième état excite, a) Ln électronvolts. quelle énergie doit-il
d'un électron piégé dans un puits de potentiel infini à une dimension.
absorber ptnir effectuer celte transition quunliquc’/ Si J'cicctron Sê
De quel facteur tleve/.-vous modifier la largeur du pirits de potentiel ?
désexcite ensuite en émeiiaiit de la lumière h) quelles longueurs
3E. Imaginez qu’un noyau atomique est équivalent à un puit.s de d'onde pcut-il émettre et c) do quelle façon (ei dans quel onirc) pcui-11
potentiel infini à une dimension d’une largeur de ~ 1,4 x 10 m, les émettre ? dl À l’aide d’un diagramme des niveaux d’énergie, illiisiiez
qui est le diatnètre nucléaire moyen. Quelle serait l’énergie de l’état le.s différentes façons possibles de désexciter l’éleetron
fondamental d ’un élet tron piégé dans un tel puits de potentiel ? ПР. Ln électron est confiné dans un mince tulx: vide d'une longueur
{Nnif: les noyaux ne contiennent pas d’électroa) de 3,0 ni ; ce tube fonciionne comme un puits de potentiel infini à une
41 Quelle doit être la hmgeur d ’un puits de potentiel infini è une dimension, a) En électronvolts, quelle est la difféicnet d’cncrgic
dimension pour qu’un électron qui y est piégé dans l’état n .1 ait entre l’état fondamental de l'éleciron ei son pieniier éiai rx c iic ”
une énergie de 4.7 cV ? b) À quel nombre quantique n la différence tl’éneigic entre des
SE. Un proton est confiné dans un puits de potentiel infini à une niveaux d’energic adjacenis .serait-elle de 1.0 eV (qui est inesiiralilc
dimension d'une largeur de 100 pm. Quelle est l’énergie de son état contrairement au résultat de a]) À c r nonibir quaiuiquc. t ) (|iicllc
fondamental ’’ serait l’énergie de l’életurim et d) ccliii-i i scraii-il lelativistc
6L L’énergie de l’état fondamental d’un clcciron piégé dans un puits
de |x>tenlici infini à une dimension est de 2,6 eV. Quelle valeur SECTION 10.4 Les fondions d'onde d'un éledron piégé
auiail elle st la largeur du puiLs de potentiel était doublée ?
14E. 1in électron piégé dans un puits de poici ilicl infini à une dimension
7E. 1 In élecni'n piégé dans un puit.s de potentiel infini à une dimension d ’une largeur L passe de son élat fomiamciilnl à son premier étal
d une largeur de i-SO pni sc iroiive dans son étal fondamental. Quelle excité. Ce phénomène augmeme-i-il, diminuc-i-il cmi biis.v;-i-il iiuduiu
quantité trénergie doit-il absorber pour monter à l’état où « - 4 ? gêe la probabilité de détecter l’éleciron dans un petit segment de
8P. Un électron est piégé dans un puits <le potenrit l infini à une dimen­ l’axe des x a) au centre du puits et b) près de Г un des murs du puits ?
sion. Quelle paire de niveaux d ’énergie adjacents fie cas échéant) 15t. Soit A /r^ la ditlérenee d'énergie entre deux niveaux d'éiicigic
afficlie une diftérencc d’énergie égale à l’énergie de l’électron qui est adjacents dans le cas d ’un electron piégé dans un (xiits de pnn nricl
dan.s l’état où a) « ~ 5 et b) où « = 6 ’’ infini à une dimension. Soit L l’énergie de l'un des deux niveaux
9P, Un électron est piégé dans un puits de poicniiel infini à une a) Démontrez que le rapport Aé.„j,/£' s'approche de la valeui 2/n
dimension. Démontrez que la dilTérenee d ’énergie A /' entre Ic.s quand le nombre qiiantique n a une x aleur élevt4>. t,*uand n —» f',
niveaux qiiantiques n et n -* 2 est (H'/2ml ')(n 1 1). W A/-;„jj. cf E ou d) s’approchent ils de zéro >e) signi
lOP Un électron est piégé dans un puits de potentiel infini à une fient ee.s résultats par rapport au principé de correspondance .'
dimension a) Quelle paire de niveaux d ’énergie adjaee.nts (le cas 16P. Une particule est confinée ibns le puits de ptilcniii I Infini à une
échéant) affii hera trois fois la différence d ’énergie qui existe entre dimension de ta figure 10.2. .Si la p'iriicule sc trouve dans son étal
300 Chapitre 10 La mécanique ondulatoire

fonüamcnial, quelle est la prohabilité de la détecter entre a) .r = 0 27P. Dans le problcnie 26. quelle est la fréquence de la lumière que
et .ï = ().25L. b) X - 0.75/. et .r = i. et c) .t = 0,25/. et x -= 0 ,7 5 t ? l'électron doit absorber ou émettre pour effectuer des transitions
I7P L'n électron est piégé dans un puits de potentiel infini à une entre les cinq niveaux d’énergies les plus b as? Exprimez votre
dimension d’une largeur de 100 pin; il est dans son état tondamental. réponse en multiples de hlbmLr. www
Quelle est la probabilité île le détecter dans une largeur de = 5,0 28P. Une boîte cubique dont les largeurs sont 6 , = L, = L. = L
pni centrée h .v = a) 25 pin, b) 50 pm et c) VO pm ? (ImJict : la largeur contient un électron. En multiples de h^lHniLK où m est la ma.sse de
\ x est si mince que vous ptniw / considérer que lu densité de proba­ l'électron, quelles sont a) fenergie de l'étal fondamental de l’clcctron,
bilité y est constante.) b) l'énergie de .son deuxième étal excité cl c) la dift'crence entre les
énergies de ses deuxième et troisième élat.s excilé,s ? Combien d ’états
SiCnON ] 0.5 Un élccifon tkins un ptiils fini dégénérés possèdent l'énergie d) du premier état excité ci e) du
cinquième état excité ?
18L a) Démontrez que les termes de l'équation de Schrôdinger
29P. Dans le cas énoncé dans le jirohlèrac 28. quelle est la fréquence
(équ.ation 10.18) possèdent les inemes dimensions, b) Quelle est
de la lumière que l'électron doit absorber ou émettre pour effectuer
l’unité SI commune à chacun de ces termes
des transitions entre les cinq niveaux d'énergie les plus bas?
15£. l'n électron dans l'ctiit n = 2 sc trouve dans le puits de potentiel Exprimez vihtc réponse en multiples de hJimL*.
fini illustré dans la figure 10.7 ; il ubsuibc une énergie de 400 cV
d’une source externe. Quelle est son énergie cinétique après cette
absorption, en supposant qu’il se déplace vers une position où x > Z. ? SECTION 10.8 L'oiome d'byàogène
20L L t figure 109 préseme les niveaux d’énergie d ’un électron piégé
ЗОЕ. P ro u v e/q u e la constante apparaissant dans l’équation 10.28
dans un puits d’énergie potentielle fini d ’une profondeur de 450 eV. est 13.6 eV.
bi rélcclroii c.st dans l'état ii 5. quelle est son énergie cinétique ?
31E Un atome (qui n’est pas d'hydiogène) absorbe un pbt4on dont la
21F Comme le suggère la ligure 10.8. la den.sité de probabilité île la fréquence est 6.2 x 10*'* Hz. De quelle quantité l’énergie de ratom e
région V> L dans le puits de potentiel fini de la figure 10.7 chine de augnieme-t-eile?
maniorc exponentielle .selon
32E. Un atonie (qui n’est pas d'hydrogène) absorbe un ¡■»hoton dont la
V'-(v) = Ce longueur d'onde est de 375 nm, puis émet immédiatement un plioton
dont la longueur d'onde est de 580 nm. Quelle quantité d ’énergie
où r est une constante, a) Démontrez que la fonction d'onde tir(x) résultante l’atome absorbc-t-tl durant cet événement''
qu'on peut déterminer à l’aide île cette équation est une .solution de
33E. Quel est le rapport entre la plus courte longueur d’onde de la série
l’équation de .Schrodinger à une dimension, b) Quelle doit être la
de Balmer et la plus courte longueur d’onde de la série de Lyman ?
valeur lie k pour que ce soit \-rai ''
34E. a) Quelle est l'énergie E de l’électron de l'atome d'hydrogène
22P. Comme le suggère la figure lO.R. la densité de probabilité pour dont la densité de probabilité est représentée par le diagramme de
un électron SCtrouvant dans la région 0 < x < i à l’intérieur du puits points de la figure 10.21 ? b) Quelle quantité minimale d’énergie est
de potentiel fini illustré dans la figure 10.7 est sinusoïdale, nécessaire [xmr extraire cet électron de l’atome ?
pui.squ'elic est donnée par
35L Quels sont a) fenergie, b) le module de la quantité de mouve
t!r-(.i) B sin’ iLv. ment et c) la longueur d'onde du photon émis quand un atome
d’hydrogène passe d’un état où it = 3 à un autre où n = I ?
où ß est une constante, a) Démontiez que la fonction d'onde ^(x)
3éf. Refaites le.s questions de l'exemple 10.6 en fonction de la série
qu’on peut déterminer à l'aide de cette équation est une solution
de Balmer de l’atome d'hydrogène.
de réqti.'ilion de 5>cbrôdinger à une dimension, b) Quelle doit être la
37£. Un neutron possédant une énergie cinétique de 6,0 eV heurte un
valeur de k pour que ce soit i rai ?
atome d’hydrogène immobile à son état fondamental Dites piHirquoi
23P. DénKmtrcz que. dans la région .t > Z. du puits de potentiel fini de
la collision doit cire élastique ou, autrement dit. pourquoi l’énergie
la figure 10.7, i/r(x) — De*' est une .solution de l'équation de
cinétique don y être coaservée. (Indice : démontrez que cette colli.sion
Schrôdinger à une dimension, où D e.st une constante et it a une
ne peut résulter d'une excitation de l’atome d'hydrogène.)
\ ale.ur positive. Sur quoi se ba,se-l-on pour affirmer que cette solution
38f. Dans le cas de l'atom e d'hydrogène à son état fondamental,
riathcmatiquemeni acc-epiablc est physiquement inacceptable ? w w
calculez a) la densité de probabilité tlcHr) et h) lu den.sité de proba­
bilité radiale P (r) lorsque r = a, o ù a est le rayon de Bohr.
SEGON 10.7 Les pièges à électron bidimensionnels et tridimensionnels
39E Calculez la densité de probabilité radiale P (r) de l’atome
24L Cn clcdroo est confiné dans le corail retituigulaire illustré dans la d'hydrogène Ji son étal fondamental lorsque a) r = 0. b) r = я
figure 10.12. scs largcuisi .sont Z., 800|im elf-, - 1 bOOpin. Quelle et e) r = 2«. où « est le rayon de Bohr.
e.st l'énergie, cn électronvolts. de l'éiai fondamental de l'éleciron? 40E Un atome d’hydrogène passe de son état fondamental à l'étal où
25F tin éleciron est eonrlnc dans la boîte lectangulairc illustrée n = 4. a) Quelle quantité d’cnergic doit il absorber ? b) Calculez
daii.s la figure 10.13 : ses largeurs .sont = 800 pm. / , = 1 600 pm l’énergie lies photons pouvant être émis quand l'électron regagne son
et L. = 400 pm. Qiii-llt- est l’éneigie., en électronvolts, de l’ctat état fondamental, puis rq>résente/-les dans un diagramme de niveaux
londamcmal de l’électron 7 d'énergie
2AP IIn corail leciangiilaire dont les largeurs sont L. L et C 21. 41P Quel travail faut il effectuer pour séparer jusqu'ù l’tnfini l’élcctnin
contient un électron. F.ii multiples tic )r/8m t^. où ni est la mti.ssc de et le proton qui forment l’atome d ’hydrogène si l’élcciron est initia­
l’élcctron. quelle* .sont a) l'éiingie de l 'étal fondamental de l’élcctron. lement a) à son état fondamental et b) à l’état ihi n = 2 ?
b) 1 énergie de son premier état excité, c) l'énergie de ses étais 42P Un atome d'hydrogène, iniiinlement au repos à l’étal quaniique
dégénérés les plus bas et d) Il différence entre les énergies de son n = 4. effectue une transition jusqu'à son état fondamental en émettant
deuxième cl de son troisième état ? un plioion. Quelle est la vitesse de recul de l'atome d'hydrogène''
Exercices et problèmes 301

43P. Un alome d ’hydrogène émci de la lumière d ’une longueur SSP. Quelle est la probabilité qu’un électron dans l’état fondamental
d’onde de 486.1 nm. a) Quelle iransition e.st responsable de ce rayon- de l’atome d’hydrogène se trouve entre deux .sphères creuses dont
nemeni '! b) À quelle série cette transition appartient-elle ? les rayons sont r et r A»; a) si r 0,500// et Ar ~ 0,OHi/i
44P. Quelles sont les largeurs des intervalles de longueurs d’onde sur et b) si r = l.OOii et Ar — 0.0)0«. où u est le rayon de R uhr?
lesquels a) la série de Lyman et h) la série de Baliner s’étendent'.' (Indice: Ar est suflisammcnt petit pour permettre a la densité de
(Chaque largeur toiiiincnce à la plus grande longueur d’onde et finit probabilité radiale d'être constante entre r et /• -^ Ar. j
è la limite de la série.) c) Queili» sont les largeurs des intervalles de 56P Pour quelle valeur du nombre quantiguc principal n le rayon
licquciitcs qui correspondent à ces intervalles de longueurs d’onde? cficctif serait- il de 1,0 mm, coiiuiic le montre un di^ram m e de pomts
Exprime/; les intervalles de Iréqucnccs en téralK-rtz ( I THz = 10'^ Hz). de l’alonie d’hydrogène'? Supposez que / ait sa valeur ma.v;iniaic
45P. Dans l’état fondamental de l’atonie d'hydrogène, l’électron possède àn — I. (Indice; revoyez la figure 10.24.)
une énergie totale de —13.6 eV. Quelles sont a) son énergie cinétique 57P* Dans l’exemple 10.7, on a démomré que la ilciisiié de proba­
et h) son énergie potentielle si l’életlron sc trouve à un rayon de Bohr bilité radiale de l’état fondanicnial de l'aioinc d’hydrogène rsi mnxi
du noyau central ? male quand r —a, où «/ est le rayon ilc Bohi. Démontrez que la vulcui
46P, a) À l’aide du dii4aiunme des niveaux d’énergie de la figure 11). 18. m ovenne de r, décrite connue étant
déterminez les nombres quaniiques correspondant à une transition au
cours de laquelle la longueur d ’onde du rayonnement émi.s est de
121,6 nm. b) À quelle série celte transition appartient-elle ?
-I P(r)f dt.

est égale à 1,5«. Dans celte expre.ssion de chaque valeur de P(r)


47P l ’n atome d’hydrogène dans un état possédant une énergie
de liaiw n (énergie requise pour extraire un électron) de 0,85 eV est pondérée en fonction de la valeur de ro ù elle sc iniuvr Notez que
effectue une transition vers un état qui a une énergie d ’excitation la valeur moyenne de r est supérieure h la valeur de / [lour laquelle
(différence entre l’énergie de te l état et rénergic de l’état fonda­ /'(/■) est un maximum.
mental) de 10,2 cV. a) Cluelleest l’énergie du photon émis en rai.son de 58P. La fonction d’onde de l’état quantique de l’atome d’hydrogène
cette transition ? b) Identifiez cette transition à l’aide du diagiaminc cor respondant au dingiamme de points de la figure 10.21, où « ~ 2
des mveaux d’énergie de la figure 11). 18. et / = 777| — 0, est
48P. Vérifiez les longueurs d'onde présentées dans la figure 10.17
des raies spectrales visibles de la série de Balmer.
49P. Quelle est la probabilité de trouver rélcclron à un rayon plus
OÙ U est le rayon de Bohr et l’indice de ^/(r) donne les valeurs des
grand que le rayon de Bohr dans un atome d ’hydrogène à son état
nombres quantiques n, l, m,. a) Représentez graphiquemeni '//^,(r)
fundanientar'(W ice.-voir l’exemple 10.8.) www
et démontrez que votre graphique est cohérent avec le diagramme de
50P. Un atome d'hydrogène émet de la lumière d’une longueur
points de la figure 1 0 .2 1 h ) Démontrez analyiiquement que t//juu(r) a
d'onde de 102.6 run. Quels sont les nomhi'es quantiqiies initial et final une valeur maximale à r = 4«. c) Déteiminez, la dcnsiir de proba­
de la transition effectuée?
bilité radiale P2ud(r) de cet état, d) Démontrez que
S1P. L’équation de Schiftdingcr appliquée aux états de l’atome
d’hydrogène dont le nombre quantique orbital / est zéro est
Pm M dr 1,
\ d ( - ,d ^ \ 877 f
donc que l’expression donnée ci-dc.ssus de la fonction d’ondc
Démontrez i|uc l ’équation 10.30. qui décrit l’état fondamental de tfr-ÿiofr) a été normalisée.
l’atome d’hydrogène, est une solution de cette équation. 59P [zîs fondions d’onde des trois états des diagrammes de pomts
52P. Calculez la probabilité que l'électron de l’atome d’hydrogène, illustrés dans la figure 10.23. où « — 2. / = i et »1, = 0, I et —l,
dans son état fondamental, sc trouve entre les sphères creuses sont
dont les rayons sont a et 2îi. où a est le rayon de Bohr. {Indice .• voir
V'zioC'; W = (1 /4s/2w )(« "-V/uW '-’” '’ cos 6.
l'exemple 10.8.)
y//2, ,, ( r ft é) - -(l/8 c A r )( fl- ’ ’)(/y«)f-’'^'’(sin
53P. Démontrez que l’équation 10.35, qui donne la densité de pro­
babilité radiale de l’étal fondamental de l’alomc d ’hydrogène, e-st <l>2i-\(i: ft </j) — (1/8cAT)(«“ ’' ’)(r7«)c“"'*"(sin ftlc
normalisée. En d'autres moU vérifiez que l’équation
où les indices de tlr(r, fl. </>) donnent les valcui s des nombres quan
tiques n. / ni,, alors que les anglc.s sont défmi.s duii.s la liguie 10.22.
P{r)dr - I Notez que la première l’o iidioii d ’onde est lérilc, mais que Ic.s
./o
autres, qui mettent en jeu le nombre imaginaire /, sont eomplcxc.s.
est \ raie. a) Déterminez la densité de probabilité de chaque foin tion d'oiulr cl
démontrez que chacune est cohereme par rapport ù son diagrannne
MP a) À une valeur donnée du nombre quaniiquc principal n,
de points de la figure 10,23. b) Additionne'/ les tiois tlensiiés <lr pm-
i:oinhien de valeurs possibles le nombre quantique orbital / peut-il
babilite calculées en a) et démontre/ que leiii somme a une syiiiéirh:
avoir ’ b) À une valeur donnée de l, combien de valeurs pox.\ibles le
sphérique, puisqu’elle ne dépend que de la civiidoiinée radiale r
nombre quantique magnétique m, peut-il avoir? c) À une valeur
donnée de n. combien de valeurs possibles m, peut il avoir?
11 Les atomes

Peu oprès leur invention, dons les années I960, les lasers sont devenus de nouvelles sources de lumière utilisées
dans les laboratoires de recherches. Aujourd'hui, ils font l'objet de nombreuses opplications et on les trouve dans tous
les domaines, comme la transmission de la voix et de données, l'arpentoge, la soudure et la lecture des codes à borres.
La photographie ci-contre
montre une intervention
chirurgicale effectuée
à l'aide d'un faisceau loser
transmK par fibres optiques.
La lumière d'un laser, comme
celle de toute autre source,
est émise par des atomes.
11.2 Ouelques propriétés des atomes 303

11.1 Les atomes et le monde qui nous entoure


A u début du XX* siècle, de nom breux scientifiques im portants doutaient d e l’existcncc
des atom es. D e nos jo u rs, toute personne bien inform ée .sait qu e Ic.s auimex cxistriit cl
q u ’ils con.stitueni les unités d e base du m onde in atcricl. O n peut m êm e, au jo u rd 'h u i,
.saisir un atom e et le déplacer. C ’est ainsi q u ’a été form e le corail quantiquo preçiontè it
la prem ière page du chapitre 10. O n peut facilem enl y com plet les 48 atom es Rn inatii un
cercle. Il est m êm e p o ssib le de p h o to g rap h ier ch a q u e ato m e g râce à la lu m ière q u ’il
ém et. P ar exem ple, le ptiint bleu pâle de la figure 11.1 est le ré s u lu i de la luinièrc ém ise
par un seul ion de baryum confiné dans un ptege conçu à l’université dc W ashington.

11.2 Quelques propriétés des atomes


Vous devez penser que la physique atom ique est un dom aine h m ille lieues de votre vie
de tous les jours. Voyez plutôt com m ent les propriétés suivantes des atom es (lellem em
fondam entales q u ’on y pense rarem ent) influencent votre environnem ent.

La!s atom es sont stables. E ssen tiellem en t, tous les ato m es q ui fo rm en t le m onde
tangible existent depuis des m illiards d ’années et n’ont connu aucune m txlification.
figure 11 1 Lc point bleu est
À quoi le m onde rcs.seniblerait'il si les atom es changeaient dc form e apres quelques
la photographic cl'une lumière émise
sem aines txi quelques an n ées?
par un seul ion de baryum confiné
durant une longue période dans un piège Les atomes se combinent entre eux. Ils s ’unissent pour form er de,s molécules Stables
conçu à rumversité de Washington. et s’em pilent pour form er des solides rigides. Bien q u ’un atom e soit principalem ent
Des techniques spéciales ont amené constitué d ’espace vide, vous pouvez vous icnii sut un plancher - form é d ’ntomos
l’ion à émettre encore et encore sans pour autant passer au travers.
de la lumicrc en effectuant
C es prop riétés fon d am en tales des atom es p eu v en t s’ex p liq u er par la p hysique t|u an -
des transitions dans la même paire
tiqiic, tout com m e les trois propriétés m oins apparentes qui suivent.
de niveaux d ’énergie Le point
représente l’émission cumulée
de nombreux photons. Les atomes sont classés de manière systématique
L a figure 11.2 illustre un exem ple dc propriété répétitive des clém ents en fonction de
leurs positions dans le tableau périodique (annexe F). Elle constitue une représentation
graphique dc l’én e rg ie d ’io n isatio n des élém ents : l'én erg ie retjuise priur ex traire l'clrc*
tron le plus faiblem ent lie d ’un atom e neutre y e st représentée en fonction de la position
d ’un élém ent donne dans le tableau périodique. Les sim ilitudes rem arquables eiitie les
p ro p rié té s c h im iq u e s et p h y siq u e s d es élém e n ts d a n s c h a q u e co lo n n e v erticale du
tableau périodique sont as,scz évidentes p o u r q u ’on puis.se constater que les atom es SOhl
construits selon des règles systém atiques.

Numéro atomique 7
Figure 11.2 Un graphique »le.s énergies d ionisation des éléments en lonciion de leur niiiném
atomique ; il illustre la répétition pcnodiquc des propriétés dans les .six périodes hi'irizontulec
du tableau périodique. Le nombre d’éléments contenus dans chacune dc ces périodes est
indiqué.
304 Chapitre 11 Les atomes

D ans le tableau périodique, les élém ents sont classés en six p é rio d e s horizontales
(il y a au ssi une sep tiè m e p ério d e , m a is qui e s t in c o m p lè te) ; ex c e p tio n fa ite d e la
prem ière, chaque période co m m ence à gauche par un m étal alcalin hautem ent réactif
(lithium , sodium , potassium , etc.) et se term ine à droite p ar un gaz noble chim iquem ent
in e rte (néon, arg o n , k ry p to n , etc.). L a p h y siq u e q u a n ü q u e e x p liq u e les p ro p rié té s
chim iques de ces élém ents. L es nim ibrcs d ’élém ents contenus dans les six p ério tk s sont

2, 8, 8, 18, 18 et 32.

La physique quantique perm et de prédire ces nom bres.

Les atomes émettent et absorbent de la lumière


Rgure]].3 Un niotlck-classique
O n a vu que les atom es ne peu v en t ex ister q u 'à l'in té rie u r d 'é ta ts quantiques discrets,
illustrant une particule de masse nt
chaque état possédant une énergie donnée. Un atome peut effectuer une transition d ’un état
et de charge —e sc déplaçant
à un autre en ém ettant de la lum ière (p o u r pa.sscr à un niveau d 'én erg ie plus bas, ou
à une vitesse v sur un cercle de rayon r.
en ab.sorhanl d e la lum ière (pour passer à un niveau d ’énergie plus haut. £|,am). C om m e
La |Tarticiile qui se déplace possède
vous l’avez vu dans la section 10.3, la lum ière est ém ise ou absorbée sous form e d 'u n
un moment cinétique L ^ ? x p,
où P {~ mv) est sa quantité photon possédant une énergie
de mouveniem. Le mouvement
de la pailiculc est équivalent h f^ ( 11 . 1)
bas*
à une boucie de courant ayant
un moment magnétique associé
et orienté dans la direction opposée à L. D onc, le problèm e qui consiste à d éterm in er la fréquence de la lum ière ém ise ou
absorbée par un atom e se réduit à d éterm in er les én erg ies des états quan tiq u es de cet
atom e. On peut le faire (du m oins en principe) grâce à la physique quantique.

Les atomes possèdent un moment cinétique


et un moment magnétique
La figure 11.3 illustre une particule chargée négativem ent qui décrit une orbite circulaire
autour d ’un centre fixe. O n a vu dans la section 11.4 du volum e 2 que la particule en
oibite possède un m om ent cinétique Z et (puisque .sa tr^ ecto ire équivaut à une m inuscule
boucle de courant) un m om ent dipolaire m agnétique /1. (Par souci d e brièveté, on om et
l'in d ice «oibj> utilisé dans le chapitre 1 1 du volum e 2.) C om m e le m ontre la figure 11.3.
les vecteurs L c y ji st>nt tous deux perpendiculaires au plan de l'o t bite, mais, puisque la
charge est négative, ils .sont orientés dans des directions opposées.
Le m odèle illustré dans la figure 11.3 est strictem ent classique et ne représente pas
fidèlem ent un électron dans un atome. En physique quantique, le iruxlèlc orbital rigide a
é té rem p lacé p ar le m o d èle à d en sité d e p ro b ab ilité, m ieux rep résen té p a r un nuage
de )X)inls. Cependant, en physique quantiqiie, il est encore vrai que, en général, chaque état
quantique d ’un électron dans un atonie met en jeu un m om ent cinétique Z et un m om ent
d ip o la ire m ag n étiq u e ¡1 o rie n té s d an s des d ire c tio n s o p p o sé es (ces d eux grim deurs
vectorielles sont dites couplées).

L’expérience d Einsteln et de De Haas


En 1015, bien avant le d év e lo p p e m en t d e la p h y siq u e q u an tiq n e, A lbert E in stein et
W J, de H aas, un physicien néerlandais, o nt réalisé une expérience ingénieuse destinée
à dénrontrci que le moment cinétique e t le monioni m agnétique d ’un atom e soin couplés.
E instein cl H aas ont suspendu un cylindre de fer à une m ince fibre, com m e le
m ontre la figure 11.4 a). Ils ont placé un solénoïde autour du cylindre en évitant tout
contact entre les deux élém ents. Initialem ent, les m om ents ilipolaires m agnétiques /7 des
ato m es dit cy lin d re étaie n t o rien tés d an s d es d ire c tio n s a lé a to ire s, d e so rte q u e les
actions de leurs cham ps m agnétiques externes s ’aniuilaieni (figure 1 1.4 a |) Cependant,
quand un courant circule dans le solénoïde (figure 11.4 b] ), p n xluisant ainsi un cham p
m agnétique B parallèle à l'a x e du cy lin d re, les nionients d ip o laires m agnétiques des
atom es du cylindre se réorientent, s'alig n an t avec ce cham p .Si le m om ent cinétique L
de chaque atom e est couplé à son m om ent m agnétique //, l’alignem ent d es m om ents
m agnétiques atom iques doit provot)ucr un alignem ent des moments cinétiquc.s atom iques
Opposé au cham p m agnétique.
It 3 Le spin de l'électron 305

M in c e fib r e ■

C)iil)(1ir
de 1er
— Siilinoïile —'

a)
figure 11.4 Le montage de l’cxpéricnce d’Einstein et de De Haas a) Iniiialemcnt, le etianip
magnétique du cylindre de fer est nul et les vecteurs moments dipolaires magnétiques fi
de ses atomes sont orientés de manière aléatoire Les veneurs moments einéiiqties atomiques
(non illustrés) sont orientés dans les directions opposées à celles des vecteurs iiiomniis dipolaires
magnétiques ; ils sont donc, eux aussi, orientés de inaiiiérc aléatoim. li) Ouund on produit
un champ magnétique fl parailèlcmciil à l'axe ilu cylindre, les vecteurs municnts dipoluircii
magnétiques s'alignent parallèlement à C, ce qui signifie que les vecteurs moments cinétiques
s'alignent dans la direction opptiséc à fl. litant donné que le cylindre est initialement isolé
de toute force de torsion externe, .son moment cinétique est conserve et il commence à tourner.

Dans l'expérience d ’fcinstcin et de De 1laas, aucune force externe de torsion n airit .sur
le cylindre ; donc, son m om enl cinétique doit conserver sa valeui nulle initiale. ( x[)ciidaiu,
q uand R e st ac tiv é et qu e les m o m en ts c in étiq u es a to m iq u e s s a lig n e n t de m anière
antiparallèlc à B, ceux-ci tendent à donner un m»iiTH-iii cinétique résultiint au cylindre
(orienté vers le bas dans la figure 11.4 b]). Pour co n serv er son m om enl cinétique nul,
le c y lin d re co m m en c e à to u rn e r au to u r de son axe c e n tra l et p ro d u it un m om ent
cinétique £r,„ orienté dans la direction opposée (vers le haut dans la figure 11 4 hj).
Si ce n ’était de la fibre, le cylindre continuerait de tourner aussi loiigiem ps que le
cham p m agnétique serait présent. Cependant, la torsion de la fibre p ro d u it rapidem ent
un m om ent de force qui stoppe m om entaném ent la rotation du cy lin d re et qui le fait
to u rn er dans l’autre sens ju s q u ’à ce qu e la torsion devienne nulle F n sn ilr. la fibre v
tordra et sc détordra à m esure que le cylindre oscillera autour de son orientation iniiialc
dans un m ouvem ent harm onique sim ple angulaire
L ’observation de la rotation du cylindre perm et de \ érilier que le moment cinéiii jiie
et le m o m en t d ip o la ire m a g n étiq u e d ’un ato m e so n t c o u p lés d an s d es d ireo tio n s
opposées. De plus, elle dém ontre clairem ent que les m om ents cinétiques associés au.x
é ta ts q u an liq u es d es ato m es peu v en t se so ld e r pat un e ro tatio n visible d ’un o bje t
m acroscopique.

11.3 Le spin de l’électron


C om m e on l'a vu dans la section 11 4 du volum e 2, q u ’un c'Iertron .soit rnnfkié dans
un atom e ou q u 'il soit libre, il possède un moment cinétique intrinsèque S, vmiv» nt
sim plem ent ap p elé spin. (K ap p clez-v o u s qu e l' a d je c tif m m n s e q u e Sianific q u e V
est une caractéristique de ba.se d ’un électron, com m e sa m asse et sa charge clectnque.)
O n v e rra d an s la p ro ch ain e sectin n que la valeui de S est q u an tifiée cl d ép e n d d 'u n
nombre quantique de spin s, qui est to u jo u rs -, dans le cas de», é le e n n n s (ainsi qui
dans le cas des protons et des neutrons). De plus, la composante de 3 m esurée sur im axe
est quantifiée et dépend d 'iin nombre qnantique magnétique de spin m ,, qui rte peut
avoir que les valeurs L ^ ou — ' .
304 Chapitre 11 Les atomes

TASIEAU i 11 Les états eledroniques d'un otome


Nombre quamique Symbole Valeurs possibles As.socié
Principal n 1,2, 3 ,... à la dLstance du noyau
Orbital l 0. 1.2....... in - 1) au moment cinétique orbital
Magnétique m, 0, ±1, - 2 . . . . . ± / au moment cinétique orbital
(composante Z)
Spin "i, au spin (composanie z)
Fous les états ayant la même valeur Tous les états ayant les mêmes valeurs
(le H forment une touche. de « et de / forment une sous-couche.
Une couche contient 2n- état.s. Tous le.s état.s contenus dans une souscouche
pos-sèdent la même énergie.
Une .sous-couche contient 2(2/ + 1) états.

L 'e x isten ce du spin de l’électron a été rév élée, sur une base em pirique, p ar deux
étudiants dipU'unés. G eorge U hlcnbcck et Sam uel G oudsm it, qui se fondaient sur leurs
é tu d es du sp ectre ato m iq u e. L a h ase d e la p h y siq u e q u an tiq u e qui traite du sp in
de l ’é lec tro n a été fo u rn ie q u elq u es an n ées p lu s lard p a r un p h y sicien b rita n n iq u e,
Paul A drien M aurice D irac, qui a form ulé (en 1928) une théorie q uantique rclaiivi.stc
de l’électron.
Il est tentant d ’expliquer le spin de l ’éleclrun en im aginant ce d ern ier com m e une
m inuscule sphère tournant autour d ’un axe. C ependant, ce m odèle classique, com m e le
m odèle classique des orbites, ne lient pas la route. E n phy.siquc quantique, au m ieux,
on ex p liq u e le spin com m e une p ro p rié té in trin sè q u e de l ’é lc ttr o n ; on ne p eu t to u t
sim plem ent pas le visualiser en fonction d ’un m odèle classique
I,e tableau 11.1. qui est un prolongem ent du tableau 10.2, donne les quatre nom bres
quanliques n, I, m, et m, qui décrivent com plètem ent les états quantiques de l'électro n
dans un atom e d ’hydrogène. (Le nom bre quam ique s en est absent parce que tous les
électrons possèdent la valeur ç = | . ) C es nom bres quantiques spécifient égalem ent les
étals possibles de chaque électron contenu dans un atom e à plusieurs électrons.

11.4 Le moment cinétique et les moments


dipolaires magnétiques
C h aq u e é ta t q u am iq u e d ’un é lec tro n dans u n ato m e p o ssèd e un m o m en t cin étiq u e
orbital et un m om ent dipolaire m agnétique orbital correspondant. C haque électron, q u 'il
soit confiné dans un atom e ou q u ’il soit libre, possède un m om ent cinétique intrinsèque
et un m om ent d ip o la ire m a g n étiq u e d e sp in co rresp o n d a n t. O n v c iia d ’a b o rd ces
grandeiii s indiv iduellcm ent, puis quand elles sont com binées.

Le moment cinétique orbital et le magnétisme


I e m odule L du m o m e n t c in é tiq u e o rb ita l / d ’un électron dans un atome est quantifié,
c ’est-à-dire q u 'il ne peut avoir que certaines valeurs. Ces valeurs sont

L - x/7i/-l \)h. (11.?)

OÙ / est le nom bre quantiqnc orbital et h — /i/2;rcsi la constante de Planck réduite. Selon
le tableau 1 1 1 . / doit avoir une valeur nulle ou celle d ’un nom bre en tier |X)silif égal ou
inférieui à « — 1. Dans le cas d ’un étal on n ~ 3, p ar exem ple, seules les valeurs l ~ 2,
/ = 1 et / 0 sont possibles.
On a vu dans la .section 11.4 du voUinic 2 ijii’iin dipôle m agnétique est associe au
m om ent cinétique orbital d 'u n électron dans un atom e. C e dipt'Mc m agnétique possède
un m o m e n t d ip o la ire m a g n é tiq u e o r b ita l ([ui est relié au m om ent cinétique par
l’équation 11.8 du volum e 2 ;
11.4 Le moment cinétique et les moments dipolaires magnétiques 307

e ~
(11.3)
2m

Le signe négatif signifie que e.st orienté dans la direction opposée à L . Étant donne
que la valeur de Z est quantifiée (équation J 1.2). celle de doit aussi être quantifiée
et donnée par

(1 1 4 )

Il est im possible de m esurer Cependant, on pcia m esurer les eoniposanies


de ces deu x v ec te u rs le lo n g d ’un axe don n é. Im ag in ez q u e l ’atom e sc tro u v e dans
un c h a m p m a g n étiq u e ff \ suppo.scz q u ’un axe dc.s ; est o ricn ié d an s la d irec tio n
d es lignes d e ch am p . V ous p o u v ez alo rs m e su re r les eo m p o saiitcs z d e /*(„,1, et d e L
le long de cct axe.
l ^ s com ixisantes , du moment m agnétique orbital sont quantifiées cl données
par

(11.5)

Ici. m, e s t le n o m b re q u an tiq u e m a g n étiq u e du tab leau 11.1. cl Po es! le mugnéfun


de Bohr :

ch ch
Pb = = — = 9,274 X 1 0 ‘“'*J/T (le magneton de Bohr), ( ll.f i)
4Tr«i 2ni

où m est la m asse de l ’électron.


I ^ s compasantc.s L. du m om ent cinétique soûl égalem ent quantifiées e t données par

L. — m,h. (1 1 7 )

La figure 11..5 illustre les cinq com posantes quantifiées L. du m om ent em etique orbital
d ’un électron quand 1 — 2, ainsi que les orientations as.srx:iées du iiiDiiienl i:inélic|iir 7 .
C e p en d a n t, /7 ne faut pa,s considérer cette fig u re au prem ier déféré, c a r on ne peut
en aucune m anière d étec ter L . D onc, un e illu stratio n com m e celle d e la fig u re 1 L.*»
co n stitu e p kis ou m o in s une aide visuelle. O n peut e o in p létcr celte aide vi.sucllc en
disant que L form e un certain angle 0 avec l ’axe des z, de telle sorte que

e<ts. fi — (11 R)
T

On peut dire que fi est un angle semi-classique com pris entre le vecteur L et T axe des z .
car il eonslilue une m esure classique d ’une chose impo.ssiblc à mesurer, selon la lheonc
qiianlique.

Le spin et le moment magnétique de spin


Le m odule S du spin S d 'u n électron, libre ou confiné, possède la seule valeur
Figure 11.5 I es valcnr« p<w;sihlcs de
d'un électron dan.s iiii état quantiqne
oit / = 2 Pour tout vecteur moment
x/.s(.v 4 l)/î
cinétique orhiial L présenté dans
la figure, il y a un vecteur oricnié = 1 1)7!« 0 .866 Tl, (119)
dans la direction opposée, représentitnl
le module et la direction du moment
dipolaire luagnétiqnc nrhilal ¡îo±. oil ,v ( — I ) est le nom bre quantique de spin de l'électm ii.
308 Chapitre 11 Les atomes

On a vu dans la section 11.4 du volum e 2 q u 'u n électron possède un dipôle m agnétique


iiiirinsèquc associé à son sp in 5. q u ’il so it co n fin é d an s un atom e ou q u ’il so it libre.
Ce dipôle m agnétique possède un m o m e n t m a g n é tiq u e d e sp in qui est relié au spin
par l’équation 11.2 du volum e 2 ;

/7, = - - S. (1 1 .lü )
ni

Le^siçne nég atif de cette relation signifie que fi, est orienté dans la direction opposée
à 5 . E tant d o n n é q u e le m o d u le de S est q u a n tifié (éq u a tio n 1 1.9), c e lu i d e fi , d o it
égalem ent être (|uantiiîé et donné par

+ l)f'- ( 11. 11)


2m

Il est im possible de m esurer 5 ou Ji,. C ependant, on peut m esurer leurs com posantes le
long d 'u n axe donné - on peut l'a p p e le r l’axe des i. L es com posantes du spin sont
Figure 11.6 Les valeurs possibles de
quantifiées et données par
cl de II,, d'un électron

S, — m ji. ( 11. 12)

où m , est le n o m b re q u an tiq u e m ag n étiq u e d e spin d o n n é d an s le tab leau 11.1.


Ce nombre quanlique ne peut avoir que deux valeurs : m, = I-1 (on dit alors que l’électron
a u n spin vers le haut, oriente dans la direction positive de l’axe des r.) et n\, = - 1 (on
dit alors que l’éleclixm a un spin vers le bas, orienté dans la direction négative d e l’axe
des :).
1 jcs co m posantes . du m om ent m ag nétique d e spin sont ég alem en t quantifiées
et données par

l^s.i = “ (11.1.3)

L a figure 11.6 illu stre les deux co m p o san tes 5 . q u an tifiée s du sp in d ’un é lec tro n et
les o rien tatio n s asso ciées <lu v ecteu r S. E lle m o n tre ég alem en t les co m p o san tes ,
quantifiées du m om ent m agnétique de spin et les orientations asstxiiées de fi ,.

La combinaison du spin et du moment cinétique orbital


D ans le cas d ’un atom e contenant plus d ’un électron, on définit un m om ent cinétique
total J. qui est la som m e vectorielle des m om ents cinétiques des électrons individuels
(de leur .spin et de leur m om ent cinétique orbital). Le nombre d ’électrons (et le nom bre
de protons) dans un atom e neutre est représenté p ar le n u m é ro a to m iq u e Z. D onc, dans
le cas d ’un atom e neutre.

J = (L , + 1,2 3 Lv + ... + ¿ z ) + t- Ô2 + -Si + ... + S j). (11.14)

De m êm e, le m om ent dipc'lairc m agnétique total d ’un atom e à plusieurs électrons


est la som m e v ec to rielle d es m o m en ts d ip o la ires m a g n étiq u es (o rb ital el d e sp in )
de chacun de ses électrons. C ep en d an t, en raison du facteur 2 dans l'éq u a tio n 11.1.3,
le m om ent dipolaire m ag nétique résu ltan t d e l’atom e ii’csl pas o rien té dans la m ême
direction que le vecteur J , il form e plutôt un certain angle avec c'a dernier. Le m o m e n t
d ip o laire m a g n é tiq u e effec tif de l’atom e est la com posante de la som m e vectorielle
lies m om ents dipolaires m agnétiques orientes dans la direction de —7 ( ligure 11.71.
Vous verre? d an s la p ro ch ain e se ctio n q u e, d an s les ato m es ty p e s, les m o m en ts
cinétiques orbiianx el les spins de la plupart des électro n s ont une som m e vectorielle
nulle. A insi. J et /îrfi d e ces ato m es so n t le résu lta! d ' u n n o m b re relativ em en t petit
d'cicctrons. s<Mivrnt d ’un seul élcc trou de valence.

Figure 11.7 Un rrKxlelc c lassique illusiram y


létique (total J
le vecteur moniciit cinétique ^ VÉRIFIEZ VOS CONNAISSANCES! ? Un électron se trouve dans un étal i|iiantique où
ei le vecteur ni<'nicnt ma
magnétique le module de .son moment linétique orbital L est 2y/}h. Combien de projections de son
rt'fcciif il,fi I montent
111 dipol.-iiie magnétique orhital le long d'un axe des : sont possibles'*
11.5 L'expehence de Stern et Geiiach 309

11.5 L’expérience de Stern et Gerlach


E n 1922, O tto S tem et W alther G erlach, de l'u m v e rsite de Hamhourjx, en ЛИепшрпо,
ont dém ontré expérim cntalem eni tjue le m om eut m agnétique (1еч ain m rs de ecxiiim cm
quantifie. Dan.s l'ex p é rien c e de S tern et G erlach, com m e un l’appelle niuinlenant, un
vaporise de l ’argeiu dans un Ib u r; certains des atom es de cette vapeur s ’échappent par
une m ince fente pratiquée dans la paroi du to u r et passent dans un tube ou l'o n entretient
un vide poussé. Q uelques-uns de ces atom es échap p es tra v e n e n t en su ite une seconde
Collilllillrlll
fente pour form er un m ince faisceau (figure 11.8). (I.cs atom es wmt alors dits nilliiitait w
£lprtro:umant - alignés en faisceau - et la seconde fente e.sr appelée i itllinwteiit ) l e laisc eau passe
ensuite entre les pôles d 'u n électro aim an t p o u r frapper une plaque déiecirice eu verre
Plaque (iêtertrice
<41 vene où il form e un dépôt d 'arg en t.
Q uand Г électroaim ant est hors tension, le dépôt d ’argent form e un p etit point. Far
Figue 11.8 A|iparcil utilisé par Slern
contre, quand l'électroaim ant est sous tension, le dépôt d ’argent s’étend verticalem ent.
et Gerlach
C e phénom ène se produit parce que les atom es d ’argent sont des dipoles m agnétiques,
de so rte q u 'ils réa g isse n t aux fo rc es m ag n étiq u es vcrticalc.s q u an d ils tia v c rse n i le
cham p m agnétique de rélectro aim an t ; ces forces les font dévier légèrenieiii vers le haul
ou le bas. D onc, en analysant le dépôt d ’argent su r la plaque, ou peut d éterm in er les
déviations subies par les atom es dans le cham p m agnétique. Q uand Stern et G erlach ont
analysé la trace d 'a rg e n t sur la plaque d étectrice, ils ont Cil un e su rp rise. G cpendaill.
av an t d 'e n p a rle r et d 'a b o rd c i ces im p licatio n s q u an tiq u cs, on d o it v o ir la force de
déviation qui agit sur les atom es d'argent.

La force de déviation magnétique exercée


sur un atome d ’argent
O n n ’a pas encore parié du type de force m agnétique qui fait dévier les atonie.s il’argent
d an s l'e x p é rie n c e de S tern et G erlach . Il ne s ’a g it/w v ici d e la force de ilcviation
m agnétique qui agit sur une particule chargée qui ^ déplace, com m e on peiit ro b leiiii
à l’aide de l’équation 8.2 du volume 2 = r/v x Ai) La raison en est sim ple; un alornc
d ’argent étant électriquem ent neutre (sa charge résultante q est nulle), ce type de force
m agnétique est égalem ent nul.
L a force m ag n étiq u e rech erch ée est g én érée pai un e in ie ra d io n en tre le ch am p
m agnétique B de l’électroaim ant et le dipôlc m agnétique de l’atom e d ’argent. O n peut
dériver une expression de la force prc.sente dans celle interaction en partant de l ’énergie
potentielle U du dipôlc daas le cham p magnétique. L'équation R 38 ilu voliimc 2 itionirc que

u = - il-B. (II.I.S)

où fl est le m om ent dipolaire m agnétique d 'u n atome d ’argent. Dans la figure 1 1 8. l'ax e
des Z positifs et B sont orientés vcrticaiem ent vers le haut. Donc, on peut ccriro l'é q u a
tion 11.J 5 en utilisant la com posante q . du m om ent d ip o laire m agnétique de l'a io n ic
orienté dans la direction de B ;

U = -fl,B. (11.16)

Ensuite, si on utilise l’équation 8.20 du volum e (Av------<H!td\) dans le cas de Г аке


des Z illustré dans la figure 11.8. on obtient

dV dB
/. = Fr (11.171
dz dz'

■Voilà ce q u ’on cherchait, soit une équation poui décrire la force m agnriiqiic qui ili'vif
un atonK- d'argent traversant un cham p m agnétique non uniform e.
I x‘ term e dBJd- de l’équation 11 17 est le gradient du cham p inagnetique parallèle Ù
Taxe des Si la com posante du cham p m agnétique parallèle à cet axe ne varie pas (s’il
est uniform e ou absent), alors dB/dz = 0 et l’atom e crargent n 'rsi pas d r i ié en ¡lassaiil
entre les pôles d e l'aim ant. D ans l’cx péncnce d e S tern ei G erlach, les ixile«. soin cons'i's
d e faço n que le g ra d ie n t dB/d: ait u n e v aleu r m ax im ale afin qu e léf ato m es
d ’argent su b issen t un e d é v iatio n v erticale m ax im ale et q u e ce lle -c i p<iraisse su r la
plaque de verre.
310 Chapitre 11 L>es atomes

Selon la physique classique, les com posantes ju. des atom es d ’argent qui trjiversent
le cham p m agnétique dans la lig u re 11.8 dév iaien t avoir des valeurs situées entre —¡x
(le m om ent dipolaire /1 est orienté directem ent vers le bas de l’axe des c) et d ¡î (ji est
o rien té d irec tem en t vers le h au t d e l ’axe d es ;) . D onc, d ’a p rè s l’éq u a tio n 11.17,
il devrait y avoir une p lag e de fo rces agissant su r les ato m es et, p ar co n séq u en t, une
plage d ’atom es s ’étalant de la déviation m axim ale, vers le bas, à la déviation m axim ale,
vers le haut. Ainsi, on devrait s'atten d re à v o n ces atom es form er une ligne verticale sur
lie faisceau la plaque de verre. Ce n ’est toutefois pas ce qui .se produit.
Figure 11.9 Résultats d ’une versit)n
mrxlcrnc de l’expénence de Stem
et Gerlat'h. Quand réleciroaimant est La surprise expérimentale
hors tension, il n’y a qu’un faisceau; C e que Stern et G erlach ont constaté, c ’est que les atom es ont form é deux points distincts
quand il e.sl s'ous tension, le faisceau
sur la plaque de verre : l’un au -d essu s du po in t où ils auraient abouti sans dév iatio n ,
initial se divise en deux sou.s-faisccaux.
l ’autre à la m êm e d istan c e, p lu s bas. L e g rap h iq u e d e la lig u re 1 1.9 illu.stre bien ce
Les deux sous- faisceaux cone.s(X)ndcnt
p h é n o m è n e ; il s ’ag it du ré su lta t d ’un e v ersio n ré c e n te de l’e x p é rien c e d e S tern et
à un aliRnenKnt parallèle et
à un aliRncincnl iuitiparallèk G erlach. D ans cette version, un faisceau d ’atom es de césium (des dipôles m agnétiques,
des momenrs magnétiques des atomes com m e les atom es tl’arg en t de l’expérience o riginale) traverse un ch am p m agnétique
de ccxsium avec le champ magnétique ay an t un im portant g rad ie n t dü/dz v ertical. C e ch a m p p o u v ait être m is h o rs ou sous
extérieur. tension, et le détecteur pouvait sc déplacer de haut en bits.
Q uand le cham p était hors tension, le faisceau n ’é ta it b ia i .sflr, pas dévié ; le détecteur
a alors enregistré la courbe com prenant le pic central de la figure 11.9. Pai" contre, sous
tension, le cham p a divisé le faisceau verticalem ent en deux petits faisceaux. l'u n plus
haut q ue le faisceau original, l ’autre plus bas. En se déplaçant verticalem ent à travers
ces deux petits faisceaux, le détecteur a enregistré la courbe à deux pics de la figure 11.9.

La signification des résultats


Dans l'expérience de Stem et Gerlach originale, il y a eu formation de deux points d ’argent,
non d ’une ligne v erticale, su r la p laq u e de v erre. C e la sig n ifie q u e la co m p o san te
parallèle à l’axe de B (et à l'a x e des z) ne peut avoir n ’im porte q uelle valeur entre
—P et + p , com m e le laisse su p p o ser la physique classique. E lle e st plutôt restreinte
à deux v aleu rs, une p o u r c h a q u e p o in t su r le v erre. D o n c, l ’e x p é rien c e de S tern et
G erlach originale a dém ontré que p est quantifiée, ce qui im plique (avec raison) que p
l ’est égalem ent. D e plus, étant donné que le m om ent cinétique L d ’un atom e est associé
à P , cc m om ent cinétique et sa com posante sont égalem ent quantifiés.
La th éo rie q u an tiq u e m oderne p erm et d ’e x p liq u er la fo rm atio n des deux points
dans l’expérience de Stem et G erlach. On sait maintcmuil q u ’un atom e d ’argent contient
plusieurs électron s, chacun p o sséd an t un m om ent m ag n étiq u e de spin et un m om ent
m agnétique orbital. O n sait égalem ent q ue tous ces m om ents m agnétiques s'an n u le n t
vectoriellem ent, exception faite d ’un seul électron, et q ue le m om ent dipolaire orbital de
c e t électro n est nul. D onc, le m om ent dipolaire résultant p d’un atom e d ’argent est le
moment m agnétique de spin de cet électron unique. Selon l’équation 11.13, cela signifie
que p ne p eu t avo ir q ue deux co m p o san tes p o ssib les le long de l’ax e des z d an s la
figure 11,8 : une com posante pour le nom bre quantique m, = + ÿ (l'électron unique a un
spin vers le haut), l'au tre com posante pour le nom bre q u an tiq u e wi, = —| (l’élextron
unique a un spin vers le bas). Si on insère ces valeurs dans l’équation 11.13, on ohiieni

p ,, = 2( (-i)pB = - P b ht p,_, - - 2 ( - | ) p „ - + p (11.18)

Si on nm iplace ensuite p . par ces expressions dans l’équation 1 1 .17. on coastate que la
com posante /■ de la force qui dévie les atom es d 'a rg e n t dans le cham p m agnétique ne
peut avoir que deux valeurs

(d B \ (d R
(11.19)
■ "«U ) “ ' ‘ ■ ■ ’■■“ " ( Â ) -

ce qui donne les deux points d 'arg en t sur le verre.


11.6 из résonance magnétique 311

Exemple 11.1
Dans rexpeneuce île Siern et Gerlach illustrée dans la figure 11.8, Étant donné que cette accélération est constante, on peut utiliser
un faisceau ü ’atomes d’argent trarW se un gradient de champ l’équaiicm 3.15 du voluinc 1 (lablcau 3 .11 ixiiii déciiiï la déviation d
magnétique ¿WMz d'une grandeur de 1,4 T/mm sur l’axe dc.s z. Cctic parallèle à l’axe des z :
région a une longueur I de 3 ,i cm dans la direction du faisceau initial.
, Л/n(c//^/</г)^
La vitesse des atomes est de 730 m/s. De quelle distance d les atomes d ~ Vtfc.t ~ U u ’ = Ot Ü1.2U)
=( — — J '- '
sont-ils déviés quand ils quiueni le gradient du champ magnétique?
La masse M d 'u n atome d’argent esi 1,8 x U) kg.
fiiani domic que la I'orcc de ckividtion agit pcrpcndiculmrcincnt
SOIUTIOK; Ici. le premier concept cfé est qtie la déviation d’un atome ä la direction du rnouvenicnt initial dc Га1о т с , cllc nc moditic pas
d’argent dans le faisceau est causée par une interaction entre le dijiôle la composanic du vcctciir vitesse dc Га1о т с dam. In direction
magnétique de l’atome et le cliamp magnétique, en raison du gradient originale du diiplacciuciil Done, I’utonic prcntl un icnip.s / //r,.
i I B M z . La force de déviation est onentee dans la direction du gradient pour tiaver.sei la longueur / dans cctic direction. Si on icmplucc (
du champ (dans la direction de l’axe des et est donnée par par /Л; dans l’6 :jniifion 11.20. on <4)ilciit
l*éc|Uation 11.17. On ne s’attarde ici qu’à la déviation orientée
dans la direction positive de l'axe des z; on devrait donc utiliser , , (iib{d B nlz)\ ( I i i i i { d B / d . ) P
K= de l’écjuation 11 lé.
Le deuxième concept clé est qu’on suppose que le gradient du = (9.27 X 10-^'* .T/THl 4 X 10’ T/m )
champ (IB/dz possède la même valeur dans toute la région traversée
(3.5 X 10-^ m)^
par les atomes d’argent. Donc, la composante / . de la force est
constante dans cette région. Ainsi, selon la deuxième loi de Newton, (21(1,8 к lO- ” kg)(/50m /5)2
la composante de I’acceleration a, d’un atome causée par F. c.st = 7,85 X 10“ ^ m 0.079 mir. (réponse)
egalement constante et est donnée par
lai distance qui sépare les deux sous-fai sceaux représente deux fois
f- iJ-{dB/(lz) cette valeur, soit 0.16 mm. Llle n’est pas grande, mais facilement
M M mesurable.

11.6 La résonance magnétique


O n a brièvem ent vu dans la seetion 11.4 du volum e 2 q u ’un proton possède un .spin .V
et un m oineni m agnétique de spin ¡1 orientés dans la même direction (parce que le proton
a une charge positive). Si le proton se trouve dans un cham p m agnétique uni form e B
orienté parallèlem ent à l'ax e des z, la com posante vectorielle z du m om ent m ajîncliquc de
spüi ¡À ne peut avoir que deux directions quantifiées ; soit paiallèic à B .sthl aiilijvarallclc
à B, com m e le m ontre la figure 1 1.10 a). G râce à l ’équation 8.3R du volum e 7 on .sait
que ces deux directions ont une différence d ’énergie de 2/< /?, qui repréveiiie l’énergie
en jeu dans l ’inversion d ’un dipôle m agnétique dans un cham p m agnétique uniform e
L’état d ’éncrgie inféricui est celui où /x. est parallèle à B: l’état supérieur, celui OÙ M- C.st
Л) aniiparallèle à fi.
Supposons q u ’on place une goutte d ’eau dan.s un chttmp m agnétique uniform e fi ;
L chacun des protons dans les alom<“s d ’iiydrogènc dc.s m olécules d ’eau possède un îx. qui
V
Ff, est soit parallèle, soit aniiparallèlc à fi. Si on sounicl cnsuilc la goutte à un cham p clcc
. k. 2 ц ,в
c trom agnétique altern atif d ’une certaine fréq u en ce/ ro ric n ta tio n d c /1 . des protons ,se
c
trouvant dans l'état d'énergie inférieur peut subir une inversion. C’ette inversion est appelée
retoumement dc spin (parce que l'in v ersio n du m om cnl iiiagnéliquc dc .spin d ’un pm lon
b)
n éc essite une re io u rn em en i du spin de c e p ro to n ) La fré q u e n c e / rcqui.se p o u r le
Figure 11.10 a) Un proton (point rouge), rctournem ent de spin est déterm inée par
dont 1,1 composante vectorielle <lii spin
dans la diiection d'un cliamp magnétique h f= 2ß.B, (1 1 2 1 )
cxtérieiii a un module dc \h . peut
condition appelée résonance magnétw|ne- E n leim es précis, pour t|ti’uii eb am p élci -
ix ciiper l'une des deux orientations
trom agnétique alternatif provoque un retournem ent de spin des protons q u 'il contient,
quantifiées dans un champ magnétique
les photons associés à ce ch am p doivent posséder une énergie h f égale ?t la différence
Si les conditions posées par I équation
2/lî;B entre les deux onentations possibles de / l . (rlonc au spin du proton) dtms ce champ.
11.21 sont remplies, on peut induire
un changement d’orientation chez Une foi.s fjuc le proton a subi un retournem ent dc spin vers l'état d'énergie supeneur.
les protons de l'échantillon, il peut reto m b er à l'é ta t d 'e n e rg ie in fé rie u r en ém ettan t un p h o to n a y a n t la m em e
b) Normalement, on trouve plus én erg ie /t/^detenninée par l’équation 1121 O n tro u v e n o rm alem enl p lu s dc, protons
de protons dans l'étal d’énergie d ans l'é t a t d ’é n e rg ie in fé rie u r q u e d an s l'é ta t .supéiieui. co m m e Ir su g g è re la
inicneurque dans l’état d'énergie figure 11.10 b). C ela signifie q u 'il y aura une i t h s n r p t i n n léM ikanir détretah ic d ’énergie
supcriciir. en provenance du cham p électrom agnétique aliem aiif.
312 Chapitre 11 Les atomes

h'gwe 11.11 Un spcure de résonance magnétique de l'éthanol. Les raies spectrales représentent
l absorption d'énergie a.ssociée aux retournements de spin des protons. Les trois groupes de raies
correspondent, comme on l’indique dates la figure, aux protons du groupe OH, à ceux du groupe
CH, et à ceux du groupe CHi de la molécule d’éthanol. L’axe hwiamial couvre n»ins de 10 T.

L e ch am p con.sjam B d o n t le m o d u le a p p a ra ît d an s l ’éq u a tio n 11.21 « 'e s t pax


le cham p extérieur dans lequel se trouve la goutte d]^caii ; il s’ag it plutôt du cham p
m odifié p ar le p e tit ch am p m a g n étiq u e local in tern e g én é ré p a r les m om en ts
m agnétiques des atonies et des noyaux voisins d ’un proton donné. D onc, on peut récrire
l’équation 11.21 ainsi:

h f= 2 p , ( 11.22)

P our atteindre la réso n an ce m ag n étiq u e, on g ard e h ab itu ellem en l fixe la f r é q u e n c e /


des oscillations électrom agnétiques cl on m odifie ju s q u ’à ce que l’équation 11.22
soit respectée et q u ’un pic d ’absorption soit enregistré.
[.a résonance m agnétique est une pm priété qui constitue la base d ’un outil d ’analyse
très utile, notamment dans l’idenlification d e com posés inconnus. La figure 11.11 illustre
un spectre de résonance magnétique d e l ’éthanol, dont on peut écrire la form ule ainsi ;
CH^-CH^-OH. Les divers pics de résonance représentent les retournem ents de spin des
protons. C es retournem ents se p roduisent toutefois à d ifféren tes valeurs de parce
que les environnem ents locaux des six protons contenus dans la m olécule d ’éthanol sont
différents les uns des autres. Le spectre de la figure 11.11 est la signature particulière
de l'éthanol
L a technolog ie du spin, appelée imagerie par résonance magnétique (IRM), a
une application pratique en m édecine diagnosliqiie. Les protons des divers tissus du coiyis
hum ain sc trouvent dans de nom breux environnem ents m agnétiques locaux différents.
Q uand le corps, ou une d e ses parties, est plongé dans un puissant cham p m agnétique
extérieur, ces différences environnem entales peuvent être détectées par les technique.s
de retournem ent de spin et tiaduites en im ages par un ordinateur, im ages qui ressem blent
à celles des rayons X. L a figure 11.12, par exemple, montre une section transversale d ’une
tête hum aine m ise en im age p ar cette méihixle.

figure 11.12 Une vue transversale d'une tête et d ’un cou humains produite pai la résonance
magnétique ÎIRjVl). Certains des ilétails de eette image n’apparaîtraient pa.s sur une image
produite par rayons X. ni même sur l'image pniduitc à l’aide d’un tomodensitomètre axial
a.s.sisté par ordinateur (scanographe).

Exemple 11.2
l île goutte d’eau est suspendue dans un champ magnétique B dont de spini, l’énergie des photons hf de l’onde électromagnétique doil
le module est l ,?10 T. et on y applique un champ électromagnétique être égale à la différence d ’énergie 2/(,fl. donnée par l’équation
alternatif- dont la fréquence est réglée pour provrqiier des retourne 11.21. A l’aide de cette équation, on trouve que
inents de spin chez les protons contenus dans l’eau. La compf>santc
/r.du moment magnétique de spin d’un proton, me.suréc parallèle­ 2n:B (2)(L41 X l( r ^ ‘>J/T X L 8 0 T )
/ -
ment à l'axe de fl, est 1,41 V 10 J/T. Supposez que les champs h 6,6.^ X 10 J s
magnétiques locaux sont négligeables par rapport à B. Quelles _ 7.66 X lO’ Hz = 76.6 MHz. (réponse)
sont la fréquence/et la longucm d ’onde X du champ alternatif ?
L.a longueur d’onde correspondante est
SOU/flON. Le piemier lomepldéniilisé iciest le suivant: quanti un proton
SC trouve dans un champ magnétique B. il possède une énergie , r 3 .0 0 X 10*^ m /s , _
potentielle parce q u îl constitue un dipôle magnétique. I e deuxième ^ = 7 .6 6 7 W hzT -
concept dé est que cette énergie potentielle est limitée à deux valcuis
ayant une différence de 2// fl. Le troisième tontepltlé est que, pour Celte fréquence et cette longueur d ’onde, sc trouvent daii' le domaine
que le proton pa.ssc de l’une à l’autre de ces énergic.s (retournement des ondes radio du spectre électromagnétique.
11.8 Plusieurs électrons dans des pièges rectangulaires 313

11.7 Le principe d'exclusion de Pauli


D ans le chapitre 10, on a vu divers pièges à électron, qui allaient des puits de potentiel
fictif à une dim ension au puits d e potentiel réel en trois dim ensions que représente un
atom e d ’hydrogène. Dan.s tous ces ex em p les, on était en p résence d un seu l électron
p iég é. C e p en d a n t, d a n s r é tu d e d es p iè g es co n te n a n t d eu x o u p lu sie u rs élec tro n s
(co m m e on le v erra d an s le s deux prixrham cs .scctioasi, il fau t te n ir co m p te d ’un
principe qui gouverne toute pariicule dont le nm nbic quaiitic|uc tic spin .v n ’est ni ré io
ni un nom bre entier. C e principe s ’applique non seulem ent aux éleeiro n s, mai», aussi
aux prolon.s et aux n eu tro n s, qui ont to u s -v = C e p rin cip e est connu SOUS Ic nom
de p r in c ip e d ’ex c lu sio n d e P a u li, d 'a p rè s W tilfgang Pauli, ipn l’a én o n ce en 1925.
Foui les électrons, il dit ce qui .suit.

Deux électrons confinés dans le meme piège ne peuvent po.»sétler tleiu snMiiiililes
de nombres quuntiques identiques.

D an s la sectio n 11.9. on v erra qu e ce p rin c ip e sig n ifie q u e d eu x é le c iio n s dans


un atom e doivent d ifférer p ar au m oins un nom bre quantique. Si ce n 'é la il piw le ca.s,
les atom es s'effondreraient, et vous et le m onde que vous connaissez n ’existériéZ pas.

11.8 Plusieurs électrons dans des pièges


rectangulaires
P our vous préparer à étudier les cas où plusieurs électrons sont contenus tlanv un atom e,
on p eut vo ir le cas où deux élec tio n s sont co n fin és dans les pièg es rectan g u laires du
chapitre 10. On utilisera encore ici les nom bres quantiques déterm inés pour ces pièges
quand seulem ent un électron y était confiné. Ce¡x-ndanl, on incluía les spins des deux
élcctroas. P o u rc c faire, on suppose que les pièges se trouvent dans un cham p m agnétique
uniform e. A insi, selon l’équation 11.12. un électron peut avoir un spin vers le haut avec
w , — I ou un spin vers le bits avec m, = (On suppose que le cham p m agnétique est
très faible, de sorte q u ’on peut négliger les énergies p otentielles q u ’il génère c h c / les
électrons.)
Q uand on co n fin e d eux élec tro n s d an s l’un d es p iè g es, il fa u t a v o ir le p rin cip e
d'ex clu sio n de Pauli en lêie, selon lequel les électrons ne jx.*uvent avoir deux ensem bles
identiques de nom bres quantique.s.

1. Le piège èi une dimension. Faire cnirei fo n d e d 'u n électron dans la largeur Í du piège
à une dim ension de la figure 10.2 nécessite un seul nom bre quaniique. n P a r coii»,é
qucni, tout électron confiné dans cette cage doit avoir une certaine valeui de n, ei son
n o m b re q u a n tiq u e m, p eu t être + ^ ou — ' . L es d eu x élec tro n s p o u rra ie n i av o ir
différentes valeurs de «. ou bien la m em e vah ui de ri si l’un a un spm vers le haut
et l’autre a un spin vers le bas.
2. Le corail rertangidnire Faire eiilier l’oniV d ’un éici fron dan.s l(.s largeurs et L. du
corail rectangulaire de la figure 10.12 nécessite les deux nom bres quam ique» ii„ cl u,..
D onc, tout électro n co n fin é dans le piège d o it av o ir c e rtain es \ aleurs de t es deux
nom bres qiianliques, et son nom bre quantique iii^ peut être soit f ' . soit * (il V a
donc tro is n o m b res q u an tiq u es). D ’après le p rin cip e d ’ex c lu sio n d e P au li, deux
électrons confinés dans le piège doivent afficher différentes valeurs à au moins l'u n
de CCS trois nom bres quantiqiies
3. La boîte rectangulaire. Faire entrer fo n d e rl’iin élerln in dtms les largeurs et L.
de la boîte rectan g u laire de la figure 10.1.3 nécessite 1r s trois nom bres q u an tiq u es
M,, et n.. D onc, to u t élec tro n co n fin é dans le piège d o it av o ir re rta in e s valeurs
(le ces trois nom bres quantiques. et son nom bre q uantique w , peut Ctrc I- 2 “ a
(il y a donc m aintenant q u alie nom bres quantiques). Scion le principe d ’excln.ston
de Pauli, deux électrons confinés dans le puits doivent afficher des valeurs différentes
à au m oins l'u n de ces quatre nom bres quantiques.
314 Chapitre 11 Les atomes

S upposez q u ’on ajoute plus d ’un électron, un par un, dans uii piège rectangulaire
de la liste précédente. Les prem iers électrons ajoutés v ont naturellem ent dans le plus bas
niveau d ’énergie disponible - on dit q u ’ils occupent ce niveau. C ependant, le principe
d ’exclu.sion fin it par ne plus p erm e ttre l'a d m issio n d ’élec tio n s ad d itio n n els d an s ce
niveau d ’én erg ie, d e so rte q u e le p ro ch ain électro n d o it o ccu p er le niveau d ’én erg ie
su iv a n t. Q uand un n iv eau d ’é n e rg ie ne p eu t p lu s a c c e p te r d ’é lec tro n s en ra iso n du
princqic d ’exclusion de Pauli, on dit q u ’il est re m p li. P ar contre, un niveau qui n ’est
occu p é p ar aucu n é le c tro n est d it v id e . D an s les cas in te rm é d ia ire s, le n iv eau est
p a rtie lle m e n t occu p é. L a cpnfif’urution élecm m ique d ’un systèm e d ’électrons piégés
est une liste ou un tracé des niveaux d ’énergie q u ’cx:cupent les électrons, ou l’ensem ble
des nom bies quantiques des électrons.

La détermination de l’énergie totale


Pltis loin, on essaiera de déterm iner l'énergie d 'u n système form é de deux ou de plusieurs
électrons co n fin é s d an s un p ièg e re c ta n g u la ire . C 'e s t d o n c d ire q u ’o n d é te rm in e ra
l ’énergie totale d ’une configuration d 'éle ctro n s piégés
P our sim plifier, on doit supposer q u ’il n ’y a aucune interaction électrique entre les
électrons ; autrem ent dit. on négligera les potentiels électriques des paires d ’électrons.
O i' calculera donc l'énergie totale d ’une configuration électronique en détemiinant l’cncrgie
de chaque électron, com m e on l ’a fait dans le chapitre 10, puis en fai.sant la som m e des
énergies. (Dans l'exem ple 1 1.3, on le fera en fonction de sept électrons confinés dans un
corail rectangulaire.)
P our bien c lasse r les v aleu rs d ’én erg ie d ’un sy.stème d ’électro n s donné, on peut
tracer un d iag ram m e des n iv eau x d ’én erg ie correspondant à ce système, co m m e on
l ’a fait dans Je,s cas où un électron unique était confiné dans Ic.s pièges du chtipitre 10.
Le niveau le plus bas. dont l’énergie e.st£f^„ correspond à l'état fondiimental du système.
Le niveau qui lui est ju ste supérieur, dont l’énergie est £pj, correspond au prem ier état
excité. Le niveau su iv an t, d ont l’én erg ie est co rresp o n d au d eu x ièm e état excité
du systèm e, cl ainsi de suite.

Exemple 11.3
Sept électrons sont confinés dan.s le corail carré de l'cxcinplc 10.5. électron. On représente cc deuxième électron par une flèche vers le
C e corail est un puits de potentiel infini k deux dimensions dont les haut (pour le spin vers le haut) sur le niveau E ,, dans la figure 11.13b).
largeurs sont /., — L = L (figure 10.12). Sup)X«cz que les électrons Ici, un autre contepi dé entre en jeu : le niveau de l’énergie E u est
n’ont aucune interaction électrique entre eux. maintenant rempli, d(>nc le troisième électron ne peut avoir cette
énergie. Par conséquent, cet électron se place dans le niveau suivant, qui
a) Quelle est la configuration électronique du système des sept
contient les énergies c<¡uivalente.s E j, et E,7 íce niveau est dégénéré).
électrons dans l’état tbndamental ’?
Chez ce troisième électron, les nombres quantiques et n, peuvent
SOLUTION: On peut détenniner la configuration clectronitjue du système avoir les valeurs respectives I cl 2 ou 2 et 1 ( et électron peut également
en plaçant les sept électrons un par un dans le corail. Le premier avoir un nombre quantique m , de 5 ou de —j . On lui attribue
coatepl fié est le suivant: puisqu’on pré.sume que les électrons n’ont arbitrairement les nombres quantiques 11, = 2, n„ = 1 et m, -
aucune interaction entre eux, on lient utiliser le diagramme des On le leprésentc aim s par une flèche vers le bas sur le niveau de
niveaux d’énergie pour un électron unique piégé afin de bien suivre i ' 1.2 £>.1 dans la figure 11.13 c).
l'ordre d'entrée des electron.s dans le corail. Ce dia^romme des Vous pouvez démontrer que les trois électrons suivants peuvent
niveaux d'énerKie à un électron est présenté dans la figure 10.14 et aussi se placer dans le niveau des éneigies E , , et E ,7. à condition
partiellement reproduit ici, dans la figure 11.13 a). Rappelez-vous qu’aucun ensemble de trois nombres quaniiques ne soit parfaitement
que les niveaux srint nommés fe„ „ en fonction de leurs énergies, répété. Cc niveau contient alors quatre électrons, dont les mimbres
l’ar exemple, le niveau le plus bas a l’énergie É’ij. où le nombre quanliqiies (n^, n^„ m,) .vint
qiianlitiue n, est 1 et le nombre quantique n^ est 1.
(2, I, | | , ( 2 . 1. + | ) , t l , 2 , - ^ ) . ( L 2 , + |) ,
l,e deuxième concept de est que les électrons piégés doivent obéir
au principe d’exclusion de Pauli, c’est-à-dim que deux électrons ne et le niveau est rempli. Donc, le septième électron se place dans le
peuvent avoir le même ensemble de valeurs pour leurs nombres niveau suivant, qui est Ej.?- Du lui attribue arbitrairement un spin
qunmiqiies, n,. n, et/w,. vers le ba.s, où m, = —j
1e pi emler électron se place dons le niveau d énergie E ,, et peut La figure 11.13 d) illustre les sept électrons dans un diagramme
avoii m, i ou m , ----- -j. On choi.sit arbitrairement le deuxième des niveaux d’énergie des élec|rons. Le corail renferme maintenant
et on trace une flèche vers le bas (pour représenter le spin vers le bas) sept électrons, qtii sont dans la configuration d’énergie la plas basse
sur le niveau E , , »lans la figure 11.1.3 a). Le deuxième électron se en vertu du priiici|5e d'exclusion de Pauli. Donc, la configuration
place également dans le niveau E, ], mais doit avoir m, = + j afin du système dans l’état fondamental est celle représentée dans la
que l'un de ses nombres qimntiques diffère de ceux du premier ligure 11.13 d) et nommée dans le tableau 11.2.
11.8 Plusieurs electrons dans des pièges rectangulaires 315

11) 10 10 10 ^ £ si-£ i4

8 ....... г M 8 8 Í
■# t
i
1
' " ^ llT
L I f'
5 — -j— ^Vi-^i-î F. . 2 f-r.
i f i f
:îi ' ‘ l’--
..il..,. £ 2 It r
T ^ ^ 1 a *•* V»
tt T T ^ '" tl “ 4»

a» b) 0 d) e) n
figure 11.13 a) Diagranitnc des niveaux d’énergie pour un élcetroii dans un corail carré de largeurs L (L'éncrgic L est en multiples de irliiml.'.)
Un élcciron ayant un spin vers le bas occupe le niveau le plus bas. b) Deux cltxrlrons (l’un ayant un spin vers le bas, l’auire, un sjmii veic
le haut) occupent le plus bas niveau du diagramme des niveaux d’énergie pour un electron, c) Un troisième électron occupe le niveau
d’éucrgic suivant, d) La configuration du système à sept électrons dans l’état fondamental e) Irois transitions possibles (XHii que le xyxirnic
à sept electrons pas.se à .son premier état excité 1) Le diagramme de,s niveaux d’energic représentant les imis éiieigies míales tes plus liasses
du système (en multiples tic hVVimL})

TAfilEAU 11.2 Configurotioii et énergies dons l'étot fondamental 3. Le principe d ’exclusion de Pauli doit toujours s’appliquer; ainsi,
m. Énergie* un élcciron ne peut pas.scr à un niveau rempli.
n, >h
1 On peut examiner le.s miis iran.sitions illastrces dans la figure 11.13 C);
2 2 8
mutes trois sont permises par le principe d exclusion de Pauli parce
2 1 .5 que les élcclrons passent tous dans des états vides ou panielleiiiem
2 1 5 occupés. Dans l’une de ces Iran.sitions possibles, un électron passe
du niveau £i_| au niveau partiellement occupé L ij. La variation
1 2 5 d ’énergie est
1 2 1 5
2
2 A£ —£ 3.3 £*1.1 “ ® - 7 ------ -------------- r
1 1 8m/.
1 1 “ 2 2
(On suppo.se que l'orientation du spin de l’électron en iransii peiii
Total .32 changer au besoin.)
* tn miilliples de h '/H iiiL - Dans une autre îles transitions possibles de la figure 11.13 c), un
électron passe du niveau dégéneté £11 et £, , au niveau pariiellcnicm
occupé / 'j j . [.a variation d’cncrgic est
b) Quelle est l’énergie tiualc du système à .sept électrons dans son
état fondamental, exprimée en nmlliples de h - f H m L - ? Л-
A£ = £ 3,3 - £ 3., = 8 - 5 T - 3
SOlUllON Ici. le lomeptdé est que l’énergie totale Ef,, du syjiièmc dans Rm£ Hm£^
son état fondamental est la somme des énergies individuelles
Dans la troisième iransition possible de la figure 11.13 e),
dc.s électrons contenus dans ce système. On peui trouver l’énergie
l’élcciron du niveau £ 3,3 passe au niveau vide et dégénère £ , , 01 £ .,.
de chaque électron dans le tableau 10. 1. partiellement reproduit dans
le tableau 11.2, nu dans la figure 1 1.13 d). Étant donné qu'il y a variation d ’cnergic est
deux électrons dans le premier niveau (le plus bas), quatre dans le
-8
Rm77- i J *
deuxième niveau et un dans le troisième, on a A£ *■ £*1.1 £ 3.3 '
Rwi£2

De ces trois iraiisitioiis pos.siblcs. celle qui nécessite la moins


grande variation d’énergie A£ est la derniere. D autres iransuioni
sont cnvis'ageablcs. mais aucune ne nécessiterait moins d'ènergir
= 32 - (réponse) Donc, pour que le système pa.sse do son état fontlamenial à sou pirniîcr
état excité, l’électron du niveau £ , , doii passer au niveau £ ( j cl £ 3,1
vide et dégénère ; l’énergie requise est
C) (Quelle énergie faul-il transférer au système pour qu’il pa.ssc à son
premier état excité, et quelle esi l'énergie de cet état /?■*
A£ 2 —
^LUIION : Voici les (onieph dés qu'on iiiilise ici. V>mL^

1. Pour que le système soit excité, un de ses sept électrons doit L’cnci gie £pf du premier étal excité du sysiètiie eu alors
cffeetiier une transition qnantique vers le haut dans le diagramme
des niveaux il'énmgie de la ligure 11.13 d). Eps = A £ -3 2 (réponse;
8m£2 " 8m £- ' 8m
?. Pour que cette transition se produise, la variation d’énergie A/s
de l’électron (donc du système) doit être A £ = On peut représenter cuite énergie et l’éiirigic £f„ du système dans
(équation 10.5). où £(,„ est le niveau d’énergie initial de la l’étal fondamental sur un diagraniine- des niveaux d ’énergie du
transition et est son niveau d ’énergie final. système, comme dans la figure II 13 D.
316 Chapitre 11 Les atomes

11.9 La construction du tableau périodique


Les quatre nombres quantiques du tableau 11,1 décrivent les états quantiques des électrons
in d iv id u els d an s un atom e à p lu sie u rs éle c tro n s. L es fo n ctio n s d ’o n d e de ces u o is
états n e sont to u tefo is p as les m êm es qu e ce lles des états co rresp o n d an ts d e l’atom e
d’hydrogène parce que, dans les atomc,s à plusieurs électrons, l’énergie potentielle a,ssocice
à un élecintn donné est déterm inée non seulem ent p ar la charge et la position du noyau,
m ais aussi p ar les ch a rg es et les p o sitio n s de to u s les au tres électro n s p résen ts dans
l'atom e. L es solutions de l’équation de Schrodinger appliquées aux atom es à plusieurs
électrons peuvent être réalisées m athém atiquem ent (en principe du m oins) à l'a id e d 'u n
ordinateur.
On a vu dans la section 10.8 que tous les états où les nom bres quantiques n et / ont
les m em es valeurs respectives form ent une sous-couche. Pour une valeur donnée de /, le
nom bre quantique m agnétique m, peut avoir 21 + I valeurs ptissibles et, pour chaque
valeur de m ,, le nom bre quantique de spin m, peut avoir deux valeurs possibles. D onc,
il y a 2(21 I ) étots dans une sous-couche. 11 s'av ère que tous les états d'une sous-couche
donnée possèdent la mente énergie, dont la v aleu r e st d o n n ée p rin cip ale m en t p ar la
valeur de n et, dans une m oindre proportion, par la valeur de l.
Q uand on n o m m e les so u s-co u c h es, on u tilise d es lettres p o u r re p ré se n te r les
valeurs
1 = 0 1 2 .3

d f
P ar exem ple, la sous co u c h e n = 3, / = 2 pc^urrail être nom m ée sous-couche 3d.
Q uand on attribue des électrons aux états d 'u n atom e, il faut respecter le principe
d ’exclusion de Pauli expliqué dtins la section 11.7 ; autrem ent dit. deux électrons contenus
dans un atom e ne peuvent posséder le m em e ensem ble de nom bres quantiques n. /, m,
et /«,. Si ce n ’était de cet im portant principe, tous les électrons contenus dans un atom e
p o u rra ien t passer au n iv eau d 'é n e rg ie le plus bas, ce qui p o u rra it élim in e r la ch im ie
des atom es et des m olécules, donc la biochim ie, ü n peut étudier les atom es de quelques
élém ents p o u r co n n aître le rôle du principe d 'e x c lu sio n de Pauli dans la construction
du tableau périodique.

Le néon
L ’atom e de néon po ssèd e 10 électro n s. S euls d eu x d ’en tre eux en tre n t dans la sous-
couche du plus bas niveau d ’énergie, la sous-couche l.v. C es doux électrons affichent
tous deu x les v aleu rs n - 1, / = 0 et /«/ = 0 , m ais l ’un affich e ni, — -1- i e t l'a u tre ,
w , = - | . Selon le tableau 1 1.I, la sous-couche l i renferm e 2(2/ + 1) = 2"états. Étant
donné que cette sous-couche contient tous les électrons perm is par le principe de Pauli,
elle est dite ferm ée.
D eux d es h u it élec tro n s résid u e ls rem p lisse n t la so u s-co u c h e su iv a n te , la
sous couche 2,v. Ix-s six derniers électrons rem plissent la sous couche 2p qui, .si / = 1,
com prend 2(2/ 1) = 6 états.
Dans une sous-couche fermée, toutes les com posantes c possibles du vecteur m om ent
cinétique oibital L sont présentes et. com m e vous pouvez le vérifier dans la figure 11
ces co m p o san tes s ’an n u len t d an s la so u s-co u c h e co m p lète ; p o u r to u te co m p o san te
positive, il y a une com posante négative de m êm e grandeur. De même, les com posantes z
des spins s ’annulent égalem ent. D onc, une sous-couche ferm ée n 'a ni m om ent cinétique
ni m om ent m agnétique d 'au cu n e sorte. D e plus, sa densité de probabilité a la sym étrie
sp h ériq u e. D onc, avec scs tro is so u s-co u c h es ferm ée s (l.v, 2s e t 2p). le n éon n ’a
aucun élec tro n .seul p o u v an t e n c o u ra g e r les in te ractio n s ch im iq u e s av e c le.s au tres
atomc.s. C om m e les autres g a z n o b le s qui form ent la colonne située à droite du tableau
pénodique, le néon est chim iquem ent inerte.

Le sodium
L e sodium vient ap rès le n éon d an s le tab leau p ério d iq u e ; il co m p te 11 élec tro n s.
Dix d ’entre eux foi ment un ereur ferm é sem blable à celui du néon, qui a un m om ent
cinétique nul, com me on l’a vu. L’électron résiduel est lar|;emcnt hors île ce cceur inerte, soit
d an s la sou.s-couchc 3.r (la proc h ain e so u s-co u c h e p o sséd an t le m o in s d ’én e rg ie).
É tant d o nné que cet é le c tro n d e v a le n c e du sivilium se tro u v e dans un état o ù / = 0
( c ’est-à-dire dans un état .s), le m o m en t cin étiq u e et le m o m en t d ipolaire m agnétique
de l'ato m e de sodium doivent être générés entièrem ent par le spin de son seul électron
de valence
11.10 Ues rayons X et la numérotation des éléments 317

Le sodium se com bine facilem ent avec d ’autres atom es qui o nt une « vacance » que
son électron d e valence faiblem ent lié peut com bler. C om m e le.s autres mctuux alca lin s
qui form ent la colonne d e gauche du tableau périodique, le sorhum est cliimUjiieniem
actif.

Le chlore
L’atome de chlore, qui possède 17 électrons, com porte un cteu rd e lOéleaum.«; .vrmlilahlc
à celui du néon et 7 électrons résiduels. Deux de ces derniers remplissent la souiwronclie
ce qui en lais.se cinq pour la stHis-couchc 3/>, qui est la sous-couche suivante avec le uxûns
d'énergie. C ette dernière sous-couchc, où / 1. peut co n ten ir 2(2/ + 1} = b élec’tronv,
de sorte q u ’il s 'y trouve une vacance, nu un « tro u » .
Le chlore peut réagir avec un autre atom e qui possède un électron de valence pou
vant com bler ce trou. I.e ch lo ru re de soelium (N aC l), par exem ple, est un composé très
stable. Le chlore, com m e les autres h alo g èn es qui (orm eni la colonne VIIA du tableau
périodique, est chim iquem ent actif

Le fer
L ’arran g e m en t d es 26 électro n s de l’au inie de fer peut être rcpré.scnlé de la m anière
suivante :

2.V' 2/7* 3.v^ 3p*’ 4 j ^.

L es sous-couches .sont nom m ées en o rd re num érique Ct, par co nvention, un expt>s:mt
donne le nom bre d 'électro n s contenus dans chacune. D ans le tableau 11.1, on constate
q u 'u n e sous co u c h e s p eu t c o n te n ir 2 éle c tro n s, une so u s-co u ch c p. b éle c tro n s,
e t une sous-couchc cl. 10 électrons. ITonc, les 18 prem iers électrons du fer form ent les
5 prem ières sous-couches rem plies indiquées p tu le crochet, ce q m laisse 8 électrons
à placer. S ix d e ces huit électrons se trouvent dans la sous- couche i d , les deux antres
vont dans la sous-couche 4s.
L es 2 d e rn ie rs élec tro n s ne s ’m sta llc n t p as d an s la so u s-co n c h e 3r/ (qui p eu t
co n ten ir 10 électro n s) parce q u 'u n ato m e e st d an s un é ta t d ’én crg ie in férie u r q uand
il sc trouve dans la configuration 3cf'4s~ plutôt que clans la configuration U n atom e
de fer dont la sous-couche 3 d contiendrait 8 électrons (plutôt t|iie 6) efféctuciait rapidement
une transition à la configuration 3 rf’4s^, ém ettant un rayonnem ent élecirom apiiétic|ue.
Il faut retenir ici que, sau f chc,r les élém ents les plus sim ples, il se peut que les éUils ne
se rem plissent pas en suivant une suite q u 'o n pourrait considérer com m e logique.

11.10 Les rayons X et la numérotation


des éléments
Q uand une cible solide, com m e du cuivre ou du tungstène, esi liomlTardée par des c'it-ctmn.s
ayiuit une énergie cinétique située dans rim e rv a lle des kiloélec ironvolls. il y a 6mis.sion
de rayonnem ent électrom agnétique dans le dom aine des rayons X O n ,s’intcrc.s,scra ici
à ce que ces rayons (dont les applieaiions m édicales, dentaires cl industrielles sont très
connues et rép an d u es) p erm etten t d 'ap p ren d re sur les atom r.s qui les cm cttcn t OU les
absorbent. La figure 1 1 1 4 m ontre le sjseclrc de longueurs d ’onde des rayons X produits
(|uand un faisceau d 'électro n s de 35 keV frapitc une cible de m olybdène. Il ,s ag it d un
large spectre co n tin u de ray o n n e m en t sui lequel s’élèv e n t d eux p ics de longueurs
d 'o n d e nettem ent définies. L e spectre continu et les pics sont g énérés de diffcrcnte.'i
m anières, q u 'o n abordera séparém ent.

Le spectre continu de rayons X


On étudiera ici le spectre continu de rayons X de la figure 11 14 en ignrx iini, poui l ' instanr
FigurR 11.14 1 ilistrihniion scion les deu x p ic s qui s ’en d étac h en t. Im ag in ez un elec tro n ay an t une én e rg ie cin éiiq iu
la longiRMir il’onde des rayons X in itiale A'o qui h eu rte (in te ra g it avec) un des ato m es d e la cib le coniiiic rilln .s iie la
proflnits quand des électrons de 35 keV figure 11.15. L 'électron peut perdre une énergie AX, qui piem se tiadnin- par rém issio n
frappent une cible de niolybdcnc. d'u n photon de rayon X qui rayonne à partir du site lie la collision, fil y a très piai d ’énergie
Les pics éiroiis ct le spectre continu transférée au recul d e f atom e en raison de la m asse relativem eni élevée de ce dci n ier,
d’oil ils s’clèvent sont générés on peut d o n t la négliger.) C e spectre con tin u d e ray o n s X esi apjielé le rayo n iien irn t
par des mécanismes différents. de freinage (ou « b rcm sstrah lu n g » ).
318 Chapitre ii Les atomes

L’électron diffusé de la figure 11.15, ayant maintenant une énergie inférieure à Xo, peut
heurter un autre atom e de la cible, générant un deuxièm e photon, dont l'énergie sera géné­
ralem ent différente de celle du photon produit dans la prem ière collision. Ce processus
de diftusion peut aw ilinucr ju sq u ’à ce que l’élcclron soit presque imm obilise. C hacun des
photons générés p ar ces collisions form e une ptutie du spectre continu d e rayons X.
D ans le spectre illustré dans la figure 11.14, on voit bien la l o i ^ c u i ' d ’o n d e d e seuil
nettement définie, X,„in, sous laquelle le spectre continu n’exLste pas. C elte longueur d ’onde
m inim ale corrcsptm d à une unique collision frontale avec un atom e cible, collision dans
laquelle un électron incident perd toute son énergie cinétique initiale X(,. Pratiquem ent
toute l’cncrgic de l’électron est transférée à un photon unique, dont la longueur d ’onde
associée (la plus petite longueur d ’onde possible du rayon X ) csl déterm inée par
figure 11.l 5 Un électron ayant une énergie hc
cinétique passe près d’un atome K ^ = h f= ^ .
de la cible et peut générer un photon
de rayon X tout en perdant une partie hc
ou ^min — (la longueur d’onde de seuil) (11.23)
de son énergie dans le pixrcessus. Ko
Le spectre continu de niyon X
est généré de cetre manière I,.a longueur d ’onde de seuil est totalem ent indépendante du m atériau de la cible. Si on
Ce rayonnement est appelé changeait le molybdène de la cible pour du cuivre, p ar exemple, Uxiies les parties du spectre
le rayonnement de freinage. du rayon X de la figure 11.14 varieraient, exception faite de la longueur d ’onde de .seuil

VÉRIFIEZ VOS CONNAISSANCES 2: A longUfUi d ’onde de seuil du .spectre conlimi


de rayon X augmentc-t-cllc. diminue-t elle ou re.ste-i-clle la iiicmc si vous a) augmentez
l’énergie cinétique des électrons qui heurtent la cible de rayon X. b) projetez les électrsMis
sur une cible formée d’une mince feuille au lieu d ’un bloc épais de niatéiiau, c) changez
la cible pour un élément de numéro atomique plus élevé ?

Exemple 11.4
L’n faisceau d’électrons de 35,0 keV frappe une cible de molybdène possédant la fréquence la plus élevée possible et la plus petite
et génère les rayons X dont le spectre est illustre dans la figure 11.14. longueur d’onde possible. Selon l’éqiiation 11.23, on a
Quelle est la longueur d ’onde de seuil ?
SOUfTION; [ci, le tontepl clé est que la longueur d ’onde de seuil hc (6,63 X 1 0 '^ J • s)(3,00 X 10* m /s)
correspond au tran.sferi (approximativement) complet de l'énergie (35,0 X 10’ eV )(l.60 X K )-'« .1/cV)
d ’un électron à un photon de rayon X. produisant ainsi un photon = 3.55 X 10 m = 35.5 pm. (réponse)

Le spectre des rayons X caractéristiques


O n p e u t m ain ten an t s ’a tta rd e r aux d eux pics de la fig u re 11.14, n o m m és IQ e t K^.
C es pics (cl d ’autres apparaissant au-delà de la plage de longueurs d ’onde illustrée dans
la figure 1114) form ent le sp e c tre des ray o n s X c a ra c té ristiq u e s du matériau de la cible.
C es pics sont générés dans un proce.ssus en deux parties. Prem ièrem ent, un électron
énergique heurte un atom e de la cible cL pendant sa diffu.sion, expulse un des électrons
d'u n e couche profonde (basse valeur de n) de l'atom e. Si l'électron de l'ato m e se trouvait
dans la couche définie par n = 1 (appelée, pour des raisons historiques, la couche K f
il laisse une vacance, ou trou, dans cette couche. D euxièm em ent, un électron se trouvant
dans une des couches à énergie plus élevée vient alors com bler le trou dans la couche K.
Pendant cette transition, l'ato m e ém et un photon de rayon X caractéristique. Si l’électron
qui com ble la vacance de la couche R provient de la couche où n — 2 (appelée couche
/,). le rayonnem ent ém is est la raie de la figure 11.14; s ’il provient de la couche où
« = 3 (a p p e lé e c o u c h e Ai), il p ro d u it la raie etc. Le trou lais.sé d an s la co u c h e
Z ou M sent com blé p ar un électron provenant d ’une couche supérieure de l ’atome.
D ans r é tu d e d es ray o n s X , il est p lu s co in n io d e de c o n s id é re r le trou créé
profondém ent dans le nuage électronique de l’alnm e que de co n sid érér les variations
d ’état quantique des électrons en transit pour com bler ce trou. C ’est cxactem cni cc que
l’on fait dans la figure 11.16; il s ’agit d 'u n diagram m e d es niveaux d ’énergie pour le

Figure 11.16 Un diagramme des niveaux d ’énergie alomique simplifié pour le molybdène,
montrant les transitions (des trous plutfit que des élections) qui génèrent certains des rayons X
caractéristiques de cet élément. Cihaqiie ligne horizontale représente l’énergie de raloinc
romprenuni un trou (un électron marit|uant) dans la couche Indiquée.
11.10 Les rayons X et la numérotation des éléments 319

m olybdène, l’élém ent dont il est question dans la figure 11.14. L a ligne de ba.se ( f ’ = 0)
représente l’atom e neutre à son é tat fondam enutl. l.e niveau m arqué K (à h — 20 keV )
rep rése n te l ’ciicrg ie d e l ’atom e d e o n d y h d èn c eom ix)itant un trou dan.s In co u ch e K.
D e m êm e, le niveau m arque L ( h E = 2,7 k eV ) représente l’atom e alfichant u n trou dans
la couche L, et ainsi de suite.
Les transitions m arquées et Kf, dans la figure 11.16 sont celles qui produiseni
les deux pics de rayons X dans la ligure 11.14, La raie spectrale par exem ple, est
générée quand un électron provenant de la couche L com ble un trou dans la couche A'.
D ans la fig u re 11.16, celte tran sitio n co rresp o n d à une tran sitio n \crf> /c bas du trou,
du niveau K au niveau L.

La numérotation des éléments


En 1^13. un p h y sic ie n b rita n n iq u e, H en ry G . J. M oseley, a g én éré des ray o n s X
ca ractéristiq u es en u tilisan t tous les élém en ts q u ’il a pu tro u v er (38), P o u r ce ta ire ,
il b tnnbardait ces clém en ts avec des élcctron.s d an s un tu b e à v id e d e sa co n cep tio n .
À l’aide d ’un chariot m anipulé par des ficelles, il pouvait déplacer chaque cihlc ilans la
trajectoire du faisceau d ’électrons. II a m esuré les longueurs d ’onde des rayons X ém is
à l’aide de la m éthode d e diffraction des cristaux dccrile d ans la section 7.0
M oseley rech erch a (et tro u v a) alors des co n stan ces dans ces spectres en passant
d ’un élém ent à l’autre du tableau p én o d iq u e. Il nota, en particulier, dans le cas d ’une
raie sp e ctrale d o n n ée co m m e Xj,, q ue s ’il rep rése n tait g rap h iq u e m en t, p o u r ch aq u e
élém ent, la racine carré de la fré q u e n c e / en fonction de la p osition de l ’élém ent dans
le tableau pcrio<lique, il en résultait une droite. I j \ figure 11.1/ m ontre une partie dc ses
nom breuses données. L a conclusion de M oseley fut la suivante •

«Nous avons ici une preuve qu’il y a une grandeur fondamentale dans rmome, qui augmente
par étapes régulières à me.sure qu’on passe d’un élément à réléincm suivant. ( efte grandeur
ne peut être que la cbaige du noyau central. »

G râce au travail de M oseley, le spiectrc des rayons X caractéristiques fut adopte com m e
signature universelle d ’un élém ent, perm ettant lu solution d ’un bon nom bre de cas,se-tête
diins le tableau périodique. A vant cette époque (1913). les positions des élém ents dans
le tableau périodique étaient attribuées en fonction du p n id \ aïom ique, bien q u ’il tû t
nécessaire d ’in v erser cet o rd re ch ez p lu sieu rs p aires d ’élém en ts en raiso n d ’in d ices
chim iques irréfu tab les; M oseley dém ontra que c ’était la charge nucléaire (C’CSt à dire
le numéro atomique Z) qui devait être la base lé c llr de la num érotation des élém ents.
E n 1913, le tableau {périodique présentait plusieurs cases vides, et il y eut un nom bre
surprenant de réclam ations pour y inclure de nouveaux élém ents l e s{irc:lrc de raytHis X
constitua alors un test final quant à ces réclam ations. Les lanihanides. .souvent apjxrlcs
élém en ts des te rres nu-es, étaie n t o rd o n n és de m an ière im pm faite en raison d e leu rs
p ro p riétés chim iq u es sim ilaires. U ne fo is le trav ail d e M oseley publié, ces élém en ts
fu re n t ad é q u atem e n t o rd o n n és. P lu s réc em m e n t, les id e n tités de cenaiii.s élém ent.s
se tro u v an t a u -d e là de T u ra n iu m so n t d ev e n u es in c o n tesiab les q u an d des q u a m llc s
suffisantes ont perm is une étude de leurs spectres de rayons X respectifs.

Figure 11.17 Un graphique de Moseley


de la raie Ka des spe< 1res de rayons X
caraetéri«tiques de 21 éléments.
La fréquenee est ealeiilée en fonction
dc la longueur d'onde mesurée.
320 Chapitre 11 Les atames

Tl n ’esi p as d iffic ile d e sa v o ir p o u rq u o i le sp e ctre d e ray o n s X c a ractéristiq u es


affiche une constan ce si im pressionnante d ’un élém e n t à l ’autre, alors que le spectre
optique des dom aines visibles et près du visible ne le fait pas ; la clé de l ’identité d ’un
élém en t est la charg e de son noyau. L ’or, par e.xem ple, est ce q u ’il est p arce q ue ses
atom es possèdent une charge nucléaire de +19e (c’est-à-dire que Z = 79). Un atom e dont
le noyau passède une charge élém en taire de plus est du m ercure ; un atom e ayant une
charge de m oins corresp o n d au platine. Les électro n s K, qui jo u e n t un rôle im portant
dans la production du spectre de rayons X, se trouvent très près du n oyau et sont des
soixlcs .sensibles de .sa charge, l-e spectre optique, par contre, m et en je u des transitions
des électrons les p lu s éloignés du noyau, voilés par les autres électrons de l ’atom e, et
qui, par conséquent, ne constituent pas des sondes .sensibles de la charge nucléaire.

L'explication du graphique de Moseley


Les d o n n ée s ex p é rim en tale s d e M o seley , d o n t le g rap h iq u e de la fig u re 11.17 ne
représente q u ’une p artie, p eu v en t se rv ir d irec tem en t à p lacer les élém en ts aux bons
endroits dans le tableau périodique. C ’est possible m êm e si aucune ba.se théorique des
résultats de M oscicy n 'e st établie. C ependant, une telle base existe.
Selon l’équation 10.28, l’énergie de l’atom e d ’hydrogène est
me"* I 1 3 ,6 eV
L/i = - —f T-i ~2 = -------- i— ■ pour n = 1, 2. 3 ,... (11.24)
SêqAt t r n~
Im aginez m aintenant deux des électrons les plus internes de la couche K dans un atom e
à p lusieurs électrons. En raison de la p résen ce de l'a u tre électro n dans sa co u ch e, le
prem ier électron « voit » une charge nucléaire effective approxim ative de (Z - 1)e, où e
est la ch a rg e é lé m e n ta ire e t Z est le n u m éro ato m iq u e de l'é lé m e n t. L e fa c te u r e*
de l’équation 11.24 est le prcnluit de (carré de la charge nucléaire de l ’hydrogène) et
de ( —r)^ (carré de la ch aig e de l’électron). Dans le cas d ’un atom e à plusieurs électrons,
on p eut é ta b lir un e ap p ro x im atio n de l ’én e rg ie e ffe c tiv e de l ’a to m e en re m p la ç a n t
le facteur par (Z — lŸ e- x ( - r ) ‘, ou eHZ — 1)^ dans l ’équation 11.24. ce qui donne

,„ .2 5 ,

O n c o n sta te q u e le p h o to n d e ray o n X K^, (ay a n t une é n e rg ie hf) est g én é ré q u an d


un électron effectue une transition d e la couche Z. (où n = 2, énergie E i) à la couche A
(où « = 1, énergie /i|). D onc, si on utilise l’équation 11.2.*), on peut exprim er la variation
d ’énergie ainsi;

A £ = A2 - El
- ( 1 3 . 6 e V ) ( Z - 1)2 - ( 1 3 , 6 e V ) ( Z - 1)2
22 V
= ( l Ü . 2 e V ) ( Z - 1)2.

L a fréquence ( de la raie Ai, est alors


( 1 0 . 2 e V ) ( Z - 1)2
/■ = — =
• h (4,14 X lO-'-'* eV-s)
= (2.46x I0' ' ' H z ) ( 7 - 1)2 (11.26)
Si on extrait la racine carrée des deux m em bres, on obtient

^ = C 7 -C . (11.27)

où r c'st une constante ( = 4.96 x lO^ Hz'^’). I .’équation 11.27 est l’équation d ’une droite.
Elle dém ontre que. si on représente graphiquem ent la racine carrée de la fréquence de la
raie spectrale Ku des rayons X en fonction du n um éro atom ique Z, on devrait obtenir
une droite. O n voit dans la ligure 1117 qu e c ’est exactem ent ce q u ’a trouvé Mo.seley.

^V ÉR IFIEZ VOS CONNAISSANCES 3: I.es rayons X K„ générés par une cible de cobalt (Z 27)
ont une longueur d'omle approxiitialive de 1/9 pni I si lonpienr d'onde des rayons X Ka
générés par une cible de nickel (Z 28) esi-i'lle siipcrienre inférieuie à 179 pm V
11.11 Les lasers et lo lumière d ’un laser 321

Exemple 11.5
On bombarde une cible de cobalt avec des électrons el on mesure Si on divise la deuxième ccjiiaiion par ia picmièrc, on élimine C.
les longueurs d’onde du spectre de ray6ns X caractéristiques généré. cc qui donne
Il y a aussi un tleuxicmc spectre caractéri.stique plus laible, qui est Zx - 1
généré par une inipureté dans le cobalt. Les longueurs d ’onde des
raies /(u sont 178,9 pnt (cobalt) et 143,5 pm (impureté), ci le numéro
V 7Co - 1
atomique du cobalt c.st '¿c„ = 27. Identifier l’impureté à l’aide de ces Si on y insère les données connues, on obtient
données.
/178,9 pm ¿x -1
SOLUTION; Le torcepldé utilisé ici est le suivant: les longueurs d ’onde des \ / T43.5 pm 27-
raies X<ï produites par le cobalt (Co) et parrinipurcié tX) ,sc trouvent
dans le graphique de Moseley, qui est donné par l’équation 11.27, Si on i.sole l’inconnue, on obtient
Si on lemplacc/piu' c//, dans cette équation, on obtient Zx - 30.0. (réponse’I

et = C Z x -r. Un coup d’œil au tableau périodique vous jxTmeiira de coiisiaiei que


\lh l’impureté est du zinc.

11,11 Les lasers et la lumière d ’un laser


À la lin des ann ées 1940 et au début des années 1960. la p hysique q u am iq ue perm it
deux développem ents technologiques m ajeurs: le tr a m is to r ,q u i révoliilionna r in fo r
m a tiq u e , c l le la s e r. L a lu m ière d ’un laser, to u t co m m e la lu m ière d 'u n e ani|xiiik-
ordinaire, est ém ise quand des atonies effectuent une transition d ’un état quantiquc Ü un
au tre de plus faib le én e rg ie. C e p en d a n t, d an s un laser (cc qui n ’csi pa.s le ea.s d an s
d ’autres sources lum ineuses), les atom es travaillent de concert p o u r produire une lum ière
ayant des caractéristiques spéciales.

1. La him ière d ’un Utser est fortem ent monochromatique. La lumière d ’une am poule
in c an d e sce n te o rd in a ire c o u v re un e p la g e co n tin u e do lo n g u e u rs d 'o n d e e l u ’e.si
c e rtain e m e n t pas m o n o c h ro m atiq u e L e ray o n n em en t d ’un néon Muore.sccnt est
m onochrom atique, à environ 1 partie p ar 10'’. Cependant, la netteté de définitiuii de
la lum ière d 'u n laser peut être plusieurs fois supérieure, ju sq u 'à 1 partie p ar Kl’-’.
2. L a lum ière d ’un laser est fortem ent cohérente. L es lo n g u es o n d es in d i\ id u elles
(trains d ’onde.s) de la lumière laser peuvent m esurer plusieurs centaines de kilom èires
Q uand deux faisceaux distinct.s ayant parcouru de telles distances sur des traicctoircK
séparées se com binent de nouveau, ils « se souviennent » de leur origine com m une et
peuvent form er des patrons d’inierférencc La Imif^ueio de c aht rcncc correspondante
d ans le cas de train s d ’o n d es ém is p a r u ne aiiifwiule é lec triq u e es( ly p iq iirn iriii
inférieure à un mètre,
3. ! m lum ière d ’un laser est fo rtem en t directionnelle Un faisicaii laser diverge très
peu ; il s ’écarte du parallélism e strict seulem ent en raistin d ’une diffraction à l'o u ­
verture de la sortie du laser. P ar exem ple, une im pulsion laser utilisée pour tiiestirt r
la distance de la Lune génère un point d e seulem ent quelques m ètres d e diam ètre h la
surf ace du satellite naturel de la Terre. On peut concentrer la lum ière d une am poule
ordinaire en un fai.sceau presque parallèle à l'aid e de Icnlillcs. m ats la divergence dC
ce faisceau est beaucoup plus grande qu e re lie de la lum ière lasci ( 'li<u|iic jioint du
filam ent d ’une am poule form e son propre faisceau dtsiincu et la divergence angulaire
du faisceau total com posé est fonction de la taille du filam ent
4. La lum ière d 'u n laser p e u t être nettem ent focalisée 9i d n ix faisceaux lum ineux
transportent la m êm e énergie, le faisceau qui peut être focalisé su r le plus pwii jmliu
affichera la plus grande inlensité A ce poinl. D ans le cas de la lum ière d un laser, le
point concentré peut être si petit (gTiiiie inten.silc de H)' ' V>7cm" peut farilcnicnt Ctrc
atteinte. L a tlam m e d ’un chalum eau oxyarétyléniqiie. par m in p araiso n , n une inten­
sité approxim ative de seulem ent l(d w /cm *
322 Chapitre 11 Les atomes

Les lasers ont de nombreuses applications


I ^ s plus petits lasers, utilises pour la transm ission de la voix et de données dans les fibres
o p tiq u es, ont co m m e m ilieu a c tif u n cristal sem i co n d u c te u r ay an t env iron la taille
d 'u n e tête d ’ép in g le. M êm e à cette ta ille , les la sers p eu v e n t g én é rer un e p u issan ce
appRixim ative de 2Ü0 mW. L es plus gro.s lasers, utilisés dans la recherche sur la fusion
n u cléa ire ainsi q u ’en astro n o m ie e t d an s d es ap p lic a tio n s m ilita ire s, rcm plis.sent
un g rand im m eu b le. U n la se r d 'u n e telle ta ille p eu t g én é rer d e b rèv es im p u lsio n s
de lum ière laser ayant une p u issan ce d ’env iro n 10‘‘* W. C ette p u issance est q u elques
centaines de fois supérieure à la capacité de production électrique totale des État.s-Unis.
Pour év iter une brève panne nationale durant une im pulsion, on stocke l’énergie requise
à taux régulier durant rin tc rv a lle relativem ent long qui sépare deux im pulsions.
P arm i les n o m b reu ses a p p lic a tio n s d es la sers, o n tro u v e la le ctu re d es c o d e s à
barres, la fabrication et la lecture de disq u es co m p acts, la rcali.sation d 'in ie rv c n tio n s
chirurgicales (voir la photo d 'o u v ertu re du chapitre), la surveillance, la taille de tissus
Figure 11.18 La rétine décollée dans l’industrie du vêtem ent (plusieurs centaines de couches à la fois), la .soudure et la
il’iinc pKiiciup evt replaçai' cl «uidée réalisation d ’hologram m es.
à l'aide il'nn laser dirigé tliin.s l’ail.

11.12 Le fonctionnement des lasers


Le m ot « laser » est l’acronym e de l’expre.ssion anglaise U ^hi Amplification bv Stimulated
Em ission o f Radiation (am p lifica tio n d e lu m ière p ar ém issio n stim u lée de ray o n ­
nem ent). O n peut donc constater que l'é m issio n stim u lé e est la clé du fonctionnem ent
du laser. Ein.stein a élab o ré ce co n cep t en 1917. Bien qu e le m on d e dût atten d re les
an nées 1960 p o u r v o ir un la ser fo n ctio n n er, les b ases de son d év e lo p p cm en i furent
établies des décennies plus tôt.
Im aginez un atom e iso lé p o u v an t e x iste r so it d an s so n é tat d ’é n e rg ie m in im ale
(son é tat fo n d am en tal), d o n t l’én e rg ie est £ „ , so it dans un étal d ’én e rg ie su p érieu re
(un état excité), dont l’énergie est Voici Iroi.s p rte e ssu s qui perm ettent à l'ato m e de
passer de l’un de ces états à l’autre.

1. IJahsorption. La figure 11.19a) m ontre l ’atome initialem ent dans son état fcxidantental.
Si cet atom e est placé dans un cham p électrom agnétique oscillant à une fréquence f.
il peut ab.sorber une énergie h f de ce cham p et passer au niveau d ’énergie supéneur.
Selon le principe de conservation de l’énergie, on a

h f= £ , - f o ­ il 1.28)

C e processus est appelé a b s o rp tio n .


2. L 'ém issitm sp o n ta n ét. D ans la figure 11.19 b), l’atom e est dans .son é ta t ex cité et
aucun ray o n n em en t e x té rie u r n ’est p résen t. A p rès un c e rta in tem p s, l ’atom e
effectuera de lui-même une transition vers son état fondam ental, ém ettant un photon
d ’énergie h f dans le processus C e pnxiessus est appelé ém issio n s p o n ta n é e , il est
dit spontané p arce q u e l’év é n em en t n ’est induit p a r au cu n fa c te u r ex térieur. La
lum ière du filam ent d ’une am poide électrique ordinaire est générée de cette m anière.
N orm alem ent, en m oyenne, les atom es dem eurent excités 10*** s environ avant
que l’ém issio n sp o n ta n ée se p ro d u ise. C e p en d an t, d a n s le cas d e c e rta in s états
excités, la durée d ’excitation pi'til être 10"* fois plus longue. Des états ayant de si longues
durées sont dits niéta.stables ; ils jo u e n t un rôle importani dans le fonctionnem ent des
lasers.
3. L'ém ission stimub'-e. Dans la figure 1 1.19 c). l’atom e est encore dans son étal excité,
m ais, ce lle fois, un ray o n n em en t ay an t un e fréq u en ce d é te rm in é e p ar l ’é q u atio n
1 1 28 csi présent Un photon qui a une é n e rg ie /if peut stim uler l’atom e e t l’am ener à
effectu er une transition vers son état fo n d am e n tal, durant cci év én em en t, l’atom e
ém et un p hoto n ad d iiio n n e! ay an t ég a le m en t un e é n e rg ie hf. C et év é n em en t est
appelé ém issio n stim u lé e ; ccllc-ci est dite stimulée parce q u ’elle est induite par un
photon extérieur. Le photon ém is est pat failem cnl identique au photon qui a stim ulé
son ém ission. D onc, les ondes associées aux photons ont la m êm e énergie, la m êm e
phase ei la m êm e polarisation, et suivent la m êm e direction.
11.12 Le fonctionnement des lasers 323

Figure 11.19 L'interaction entre Aval 11 Processus Après


le rayonnement et la matière pendant
les processus a) d’absoiption, i<j
b) d’émis.sion spontanée et -» a)^ Alwoiption .\u iiiii
c) d’émissirm stimulée. Un atome ------- L„
(matière) e.st rcpré-scnié par le point
itHige -, l’atome est dan.s un état
quantiiiuc (bndamentai ayant ■ h/
Knii.ssioti
une éncrpic ou dans un étal b) A iujin
s[>oiilanéc
quantique excité ayant une énergie _1
En a), l'atome absorbe un photon
d’énergie h f d’une onde lumineuse
incidente. En b), il émet une onde
himineusc en émettant un pht>ton
d'énergie hj. En c). une onde lumineuse stimulée
incidente dont les photons ont
une énergie /if provoque l’émission Rayoïmcinem Matière Maiirr<* Ravonnrmcni
d'un photon de même énergie
par l'atome, augmentant ainsi
l’éncrsie de l’ontle lumineuse.

La fig u re 1 J.1 9 c) d écrit r é m is s io n stim u lé e d an s le cas d 'u n alo m c u n iq u e.


Supposez q u ’un échanlillon contient un grand nom bre d 'ato m es en équilibre Iberm iqiie
à une tem pératu re T. A vant q u 'u n rayonnem ent atteig n e l'éc h an tillo n , un nom bre
de ces atom es se intuvcni dans leur état fondam ental et ont une énergie £ „. et un nombre
d ’atom es se trouvent dans un état d 'én erg ie excité £ , . Ludw ig B oltzm ann a dém ontré
que N y s ’exprim e au m oyen de A/(, par

N, = (11.29)

OÙ k - 1,38 X J/K = 8,62 X 10“ ^ eV /K e st la co n stan te de B o ltzm an n . C elle


équation sem ble vraisem blable. La grandeur k T est l'én e rg ie cinétique m oyenne d 'u n
atom e à la tem pérature T. Plus la tem perature est élevée, plus les atom es (en moyenne)
.seront « p oussés» par agitation therm ique (c’c.st-à-dirc par les colli.sions entre les atom es)
au niveau d 'é n e rg ie supérieur £ ,. D e plus, étant d onné qoe £ , > £f,, l’équation 11.29
impo.se que A, < N q : c'e.st donc dire q u ’il y aura toujours m oins d ’atom es dans l ’état
excité que dans l’état fondam ental. C ’est ce qui est prévu si lc.s populations et A, des
niveaux ne sont détem iinécs que par l’action de l’agiiaiion tlicrmiquc. La figure 11.20 a)
illustre cette situation.
Si on in on d e m a in te n an t les ato m es d e la fig u re 11.20 a) de p h o to n s ayant im e
énergie Ey £'o, les pho to n s seront abstirbés p ar les atom es dans l’état fondam ental.
Cl des p h o to n s se ro n t g én é rés p a r ém issio n stim iilcc d es ato m es se tro u v an t dans
l ’étal excité. Einstein a m ontré q ue la probabilité par atom e que ees d eux processus se
déroulent est égale. D onc, étant donne q u ’il y a plus d atom es dans l étal fondaïucnuü
que dans l’état excité, l’action résultante sera l’aKsotpttion de photons.
P o u r o b te n ir un laser, il fau t plus d ’éniission.s qu e d 'a b s o rp tio n s d e p h o to n s,
-tJL. - A' c ’e st-à-d ire q u ’il fau t une situ atio n où rcm is.sion stim ulée dom ine L e m oyen direi i
de le faire est d e c o m m e n c e r avec p lu s d ’ato m es d an s l e ta t ex c ite q u e d an s l'é ta t
.*«**.«*»*__r*, 0
fondam ental, com m e dans la figure 11.2(1 b). Cependant, p u isq u 'u n e telle inversion de
M p o p u la tio n s ne peut c ta ' produite avec l’éq uilibre therm ique, il faut trouver un autre
Figure 11.20 a) La repartition à réqiiilibiie m oyen d ’en établir une et de la maintenir.
des atomes entre l'étal fondamental
£i, et l'état excité Ey. expliquée Le laser hélium-néon
par l'agnation ihermiquc
La figure I L2J illustre un type de laser q u ’on trouve l ouram m ent dans les laboiatoires
b) Une population inversée obtenue
par des méthorles spéciales scolaires. Il a été conçu en 1^61 par Ali .lavan cl ss^s collaboiateurs. I r tube à deuharge
Une telle inversion est essentielle en verre est rem pli d ’un m élange contenant 20 % d ’hélium et 80 % de néon g az eu x ;
au fniu'tionne.menr d’un latter. le néon est le m ilieu oi'i fa c tio n du laser se produit.
324 Chapitre il Les atomes

laiiccaii La figure 11.22 p rése n te des d iag ram m es de n iv eau x d ’én e rg ie sim p lifiés p o u r
J.' Tube à décharge p — laser les deux ato m es. L n co u ra n t élec triq u e trav e rsan t le m élan g e h éliu m n éo n p erm e t
(par les collisions entre le.s atom es d ’hélium et les électrons du courant) de faire passer
Ml
TT 1/
M .>
de nom breux atom es d ’h élium à l ’état qui est inétastable.
(stmi- L’én e rg ie d e l’é tat d e l ’h éliu m E% (20,61 eV ) est très p rè s d e l ’én e rg ie d e l ’état
iranspareiii) du néon £2 (20,66 eV ). D onc, q uand un atom e d ’hélium m étastable (E 3) et un atom e
de néon à son état fondtunental (£ 0) sc heurtent, l’énergie d ’excitation de l’atom e d ’hélium
Figure 11.21 Les composantes
d'un laser hélium-néon. L'ne diflérence est souvent transférée à l’atom e de néon, qui pa.sse alors à l’état £ 2. A insi, le niveau £2
de potentiel appliquée projette du néon illustré dans la figure 1 1.22 peut devenir plus occupé que son niveau £ ,.
des électrons ù travers un tube C e tte in v ersio n d e p o p u latio n s est rela tiv e m en t facile à o b te n ir p arce q u e. p re­
à décharge contenant un mélange m ièrem ent, il n ’y a essentiellem ent pas d ’atom es de néon dans l ’état £ , ; deuxièm em ent,
d’hélium et de néon gazeux. la nature m étastable du niveau £3 d e l'h éliu m assure une alim entation facile en atom es
Le.s électrons heurtent les atomes de néon dans i’étal £ , cL troisièm em ent, les atom es dans le niveau £ , pas,sent rapidem ent
d’hélium, qui heurtent eJisiiite tes atomes (à travers des niveaux interm édiaires non illustrés ici) à l’état fondam ental du néon.
de néon, qui émettent de la lumière S u p p o se/ m aintenant q u ’un photon unique est spontaném ent ém is quand un atom e
dans l’axe du tube. La lumière traverse
de néon passe de l’état £2 à l’état £ ,. Un tel photon peut provoquer une ém ission stim ulée
des fenêtres transparentes F et est
qui. à .son tour, peut provoquer d 'au tres ém issions. G râce à cette réaction en chaîne, un
réfléchie vers l'arrière et l’avant
fai.sceau cohérent de lum ière laser rouge, .se p ropageant parallèlem ent à l’axe du tube,
du fuite par les miroirs M, et M2
prmr jirovoquer encore plus d’émissions est rapidem ent créé. C ette lum ière, d ’une longueur d ’onde de 632,8 nm , traverse le m be
chez les atomes de nérm. L'ne certaine à d éch arg e p lu sieu rs fois en raison d es réflex io n s su ccessiv es d es m iro irs M j et M 2
quantité de lumière est transmise (figure 11.2 1 ), accum ulani des ém issions additionnelles d e photons à chaque pas.sage.
à travers le miroir Mi pour former Le m iroir M | réfléchit toute la lum ière, m ais le m iroir M , est légèrem ent transparent,
le faisceau laser. de sorte c|u’une petite fraction de la lum ière s ’échappe pour form er un faisceau externe
utile.

V'VÉRIFIEZ VOS CONNAISSANCES 4: longueur d'onde de la lumière du laser A (laser


hélium-néon) est 632.8 nm; celle du la.ser R (laser à ga/ carbonique) est 10,6 /<m. La longueur
d’onde du laser C (laser à rarséniure de gallium) est 840 nm. Classe/ ces la.scrs en fonction
de la différence d’énergie entre les deux états quantiques responsables de leur action,
en commençant par la différence la plus élevée.

F.rat niétaswblc
h

l’ü 'E.,
Colhsion.'i
He-N<‘ F'i l umière du laser
'^ ■' (I1.32.8 mn)

Kxciiaüon Pégradiitioii
pai rapide
collisions

10

I lat Ibndameiil.il
F.lats Flats (omnnm
de l’iiélium flu néon

figure 11.22 Qm<ire niveaux d’énergie essentiels des atomes d ’hélium et de néon dans un laser
hélium néon. L’action laser apparaît entre les niveaux et E, du néon quand il y a plus d ’atomes
dans le niveau £2 Ouc dans le niveau £,.
Révision et résumé 325

Exemple ] 1.6
Dans le laser hélium-néon de la figure 11.21, l’aclion laser apparaît kT - (8.62 X 10 ^ eV/K)(3Ü0 K) = 0,025 9 cV.
entre deux éUils excités de l’atome dfe néon. Cependant, dans de
Si on insère les deux derniers résultats dans l’équation 11.30, on
nombreux la.sers, elle apparaît entre l’état fondamental et un état
obtient le rapport entre les populations à température ambiante :
exeité, comme le suggère la figure 11.20.
//V = ^-aK.cV>Hl,«2S0cV)
a) Imaginez un tel laser (jui émet à une longueur d’onde de À = .S.^0 nm.
S’il n'y a pas d’inversion de popidations, quel e.st le rapport entre la = l„3 x 10-’’*. (léponsc)
population d ’atomes dans l’état E , et la population d ’atomes dans
l’ctat fondiuneiital £«, cc.s atomes étant à la température ambiante ’’ C’e.st un nombre extrêmement petit, il n’csi loiiicroi.s pas inviai.sciii-
blable. Un atome dont l’cnergic d ’agitation thermique movenne
SOLUTION: Le premier concept clé qu’on utilise est le suivant; le rapport est seulement de 0.025 9 eV ne pnx urera ]ias souvent une énergie
des populations W^/ÎV(, apparaissant naturellement dans les deux de 2.26 eV à un auln- atome clan» une collision.
états est fonction de l'agitation thermique des atomes gazeux ; selon
l’équation 11.24, on peut écrire b) Dans les conditions décrites en a), à quelle température le rapport
ÎV,/A'„ serait-il ^ ?
NJN„ =_ e„ -ie.-E,wï (11.30)
SOLUTION; Les deux concepts dés de a) s applicjiieni ici, male on veut,
Pour déterminer A',/W„ à l’aide de l’équation 11.30, il faut calculer cette fois, que la température T,soit suffisante pour que l’agitalion
la différence d’énergie Ljf C» entre ces deux états. Un deuxième thermique pousse assez d ’atomes de néon au niveau d'Cnerpie
tOiKcpl clé est employé ici : on peut obtenir — Eq en utilisant la supérieur (XHif obtenir j Si on insère ce tappori dans
longueur d’onde donnée de 5iO nm pour l’action laser entre ces deux réc|uation 11.30. qu’on prend le logaritlime naturel des deux membres
états. On obtient et qu’on isole T, on obtient
. „ hc E ^ - £o _ 2.26 e.V
Afin 2) (8,62 X K)"’' eV/K)(ln 2)
(6,63 X 10 J ■s)(3.00 X 10** m/s) (réponse)
= .3.78 X 10’' K.
“ (550 X lU -’ m )(l,60 X lO* ’’ J/eV)
ce qui est txiaucoup plus chaud que la surface du Soleil. Tl est clair
^ 2.26 cV.
que. si on veut inverser les populations de ces deux niveaux, il faut
Pour résoudre l’équation 11.30, on doit aussi ulili.ser l’énergie moyenne un mécanisme spécifique, c ’est-à-dire qu’il faut pou-sser les atomes.
de l’agitation rlieriniqiie kT pour un atome à température ambiante Aucune température, quelle qii clic soit, ne provoquera naturellement
(supposée à .300 K), qui est une inversion de populations par agitation theiinique

i;i M i.- ii » llii :w ;i aM U

Quelques propriétés des atomes Les énergies des atomes sont Un dipôlc magnétique est associé au momeni cinétique / d'un
quantifiées, c'est à-dire que les atomes ne possèdent que certaines électron dans un atome. Ce clipéle magnétique a un niuiiii'iil
valeurs spécifique'i d’énergie asscxriées à differents états quanliques. lairc' m agnétique urhital oriemé dans la diiw.;iion npjxisée à /..
Ixîs atomes peuvent effectuer des transitions entre differents états
- - * r (il.3)
qiiantiques en émettant ou en absorlrant un pholon. dont la fréquence Mo* 2m
/ est déterminée pai' où le signe négatif indique des tliicctiim.s op|>osécs I.a compcvianic
/Zprt,. du moment dipolaire magnétique orbital le long de Taxe des ;
¥= - F-b. ( 11. 1)
est quaniitîée et mesunihlc et peut avoir les valeurs
où £haii' est le niveau d’énergie supérieur cl £h„, est le niveau
d'énergie intérieur de la paire des états quanliques en jeu dan.s la (11 S)
transition 1 atomes pos.sèdent egalement des moments cinétiques où j/jj est le nmgnétim ilt liohr.
et des moments dipolaires magnétiques quantifiés.
eh pji
Les m om ents cinétiques et moments dipolaires magnétiques Mb - _ q?74 Л 1() -"ТЯ. ( 11 . 6 )
4тгт
Un électron piégé dans un atome possède un moment cinétique
itrhiuil L dont le mcxlule est 1 n électron, ciHifiné ou libie. |x>ssèdc un mmucut <iiit'uqut m liiti-
sèque (ou .simplement un spin) S dont le module est
L = ^ /ш + m . ( 11. 2 )
S — Y^•(v 1)ù. (11.9)
oil l est le nombre quantique orhital (qui peut avoii' les valeurs indiquées
dans le tableau 11.1 ) et où la constante « h-barre » est h hf2n . OU .rest le notnhre quuntiquede spin (Ic I’clcciron qiii cm loujiNini
1Æcomposante L. de L le long d’un axe des z. arliitraire est quantifiée La cwnposiinte 6’- dc 5 Ic long d’un axe des c .'u-bitraim est quantifiie ct
et mesurable et peut avoir les valeurs me.siirable el peut avoir les valcurs

L, ~ Wih. (11.7) .V = m,h ( 11 . 12)

OÙm, est le nombre quontique magnétique (qui peut avoir les valeurs OIÎ «ij csi le nombre quantique île spin de Гélectron ft 1X411 être ) i
indiquées dans le tableau 11.1), on Î- I
326 Chapitre 11 Les otomes

Un électron possède un dipôie magnétique intrinsèque associé celles qui Contiennent un nombre maximal d’électrons, les moments
à son spin S. qu'il soit confiné dans un atome ou libre. Ce dipôie cinétique et magnétique résultants des électrons stmt tous les deux nuis.
magnétique possède un m om ent m agnétique de spin ¡1^ orienté
!>es rayons X et la num érotation des élém ents Un spectre
dans la direction opposée à 5 ;
continu de rayons X (le rayonnement de freinage) est émis quand
K = ( 11. 10) des électrons à haute énergie perdent une certaine partie de leur
m énergie dans utie collision avec, des noyaux atomiques. La longueur
La composante . du moment magnétique de spin ¡1, le long d ’onde de .seuil Á,„¡„ est la longueur d’onde émise quand de tels
d'un axe des :. arbitraire est quantifiée et mesurable et peut avoir les électrons perdent toute leur énergie initiale dans une seule de ces
valeurs
reneontres ; elle est donnée par
(I! 13) hc
. (11.23)
bn
L e spin et la résonance m agnétique Un proton possède un
spin s et un moment magnétique de spin Ji tou|ours orienté dans où Kq est l’énergie cinétique initiale des électrons qui frappent
la cible.
la même direeiion que .V Si un proton se trouve dans un champ
Le spectre de rayons X caractéristiques apparaît quand des
magnétique ti extérieur, la composante vectorielle Ji.. de p parallèle
électrons à haute énergie éjectent des électrons présents dans les
à un axe des i (décrit comme étant orienté dans la direction de B)
couches profondes de l’atome ; quand le « ir<m » qui eti résulte est
ne peut avoir que deux orientations quantifiées: parallèle à B
comblé par un électron provenant d ’une couche supérieure, il y a
ou amiparallelc à B. La dittcrcncc d ’cncrgic entre ces oncnialions
est Ip H. I, énergie requise par un photon pour faire passer le spin émi.ssion d lin photon du spectre de rayon.s X caractéristiques.
du proton d’une orieniuiion à l’aiiire est lin 1^)13. le physicien britannique Henry G. J. Moscley a
mesuré la fréquence des rayons X caractéristiques d'un certain
h f= I- ( 11 . 22 ) nombre d’éléments. E a noté que, lorsqu'on repré.sente graphiquement
la racine carrée de la fréquence des rayons X en fonction de la ptisition
où représente maintenant le champ magnétique extérieur et de rélém eni dans le lableaii périodique, il en résulte une droite,
esi le champ magnétique k a a l généré par les atomes cl les noyau.x comme dans le graphique illustré dans la figure 11.17. .V1o.sclcy en a
eiilouram le proton La détection de tels retournements de spin peut conclu que la propriété qui détermine la position d'un élément dans
conduire à la création tle spectres de résonance magnéticjue qui le tableau périodique n’esi pas sa masse atomique, mais son num ero
permettent ridemificaiion de substances particulières. atom ique Z (c’est à dire le nombre de protons contenus dans son
Le principe d ’exclusion de Pauli Les électrons confinés dans noyau).
un atome ou dans un autre piège obéissent au principe d ’exclusion L es lasers et la lum ière d ’un laser La lumière d ’un laser
de Failli, qui dit que deux particides dont le nombre quunlique de provient d'une émis.sion stimulée, c’est-à-dire qu’un rayonnement
spin n'est f>as entier {connue les électrons, les protons et les ncinrons) ayant une fréquence déterminée par
dans un même atome ou dans un autre type de piège ne peuvent avoir
le même ensemble de valeurs de nombres (/uantiques. hf (1 1.28)

La construction du tableau p ériodique Les éléments sont peut provoquer chez un atome la transition d'un niveau d’énergie
classes dans le tableau périodique suivant l’ordre croissant de leurs supérieur (d'énergie lip à un niveau d'énergie inférieur (d’énergie A(i),
numéros atomiques Z; la charge nucléaire est Ze; Z représente à la ce qui entraîne l'ém ission d ’un photon de fréquence/. Le photon
lois le nombre de protons contenus dans le noyau et le nombre stimulé et le photon émis sont identiques en tous points et .se combi
d'électron.s contenus dans l’atome neutre. ncni pour former la lumière d'un laser
L es états ayant la même valcut de n forment une couche, Pour que rémission prédomine, il doit normalement y avoir une
et ceux ayant les mêmes valeurs respectives de n et de / forment une inversion de pupulatiuiis. c'est-à dire que l’état d’énergie supérieur
sous-ctiuche. Dans les couches et les sous-cout hes/eimée.s, qui sont doit contenir plus d'atomes que l’étal inférieur.

Q U E S TIO N S í ’ l- iii.iíí .... ■

1. Un électron confiné dan.s un alome d'or est dans un état où n — 4. 7. De quels nombres qiianti>|iics l'énergie d’un élei'tron dépend-elle
Lesquelles de ces valeurs de I sont alors possibles : —3,0. 2. 3. 4. 5 ? a) dans un atome d'hydrogène et h) dans un atonie de v-anailiiini?
2. L.in atonie d'argent comporte des sous-couches V/ et 4<( fermées 8. Dites si les énoncés suiv.anis .sont vrais ou faux, a) Une (cl seulement
Ces suus-iouche.s «omptent elles le niênie nomlirc d'électrons, ou y une) de ces sous-couches n e peut exister 2p if, 3d. l/>. h) Le nonihrc
en a-t-il nue t|ui en compte davantage ? de valeurs de w, possibles dépend seulement de / et non de n. c) Il y a
3. Un atome d'uianiuni comixirte des sous-coiiehes fÿ) et 7.i fermées. quatre soiis-i;niichcs où n ~ 4. d) La valeur minimale de n associée
Guelle sous couche compte le plus grand nombre d'électrons ’' à une valeur donnét' de / est / - L e) Dans tous les élaUs où / ■= 0. on
<1, Un électron eonicnu dan.s un atoiiK de mercure se trouve dans la trouve aussi nil = 0 fj II y a n xous-couches pmii chaque valeur de n.
.sous-cout he )() (^luillcs valeurs de m; pt ut-il avoir' 3, 1,0. 1.2'.' 9 l csqiielles des aftinuations suivantes sur l’expérience d'Einstein
5. a) Combien de soiis-couehes la <aiiichc n — 2 compte-i-el l e '? et de Haas ou sur scs ré.sultats sont vraies (le cas échéant) ? a) Les aloiiK.’s
combien d étals électroniques '? b) Répondez aux mêmes questions ont un moment cinétiigic b) l e moment cinétitiuc des atomes est quaiv
pour la couc hc it = 5. tifié. c) Le,s atomes ont des numients magnétiques, d) Les moments
6. De quel atome de chacune des paires suivantes est-il plus facile magnétiques des atomes sont quantifiés, e) Le moment cinctiqiic
d'exir.iire un électron a) Krypton ou brome b) Rubidium ou d'un alome est fortement couplé à son moment magnétique,
cérium et Hélium ou hydrogène ? f) L’expérience repose sur la conservation du moment cinétique.
Exercices et problèmes 327

10. Considérez les cléments krypton et rubidium, a) Lequel est de composantc.s la raie Ka possède-t-elle ? b) I3c même, combien de
le plus approprié dans une expérience de Stern et Geiiach du type composantes la raie Kj] possède-l-elle ''
décrit en fonction de la figure 11.8? b) Lequel, le cas échéant, 13. Lesquelles (s’il y en a) des situations suivantes sont essentielles
ne l'onciionnerait pas du tout ? à l’apparition d'un effet laser entre deux niveaux d'énergie d'un
11. Le spectre de rayon X de la figure 11.14 est généré par des élec- atome ? a) 11 y a plus d’atomes dan.s le niveau supérieur que dans le
iron.s de 33.0 keV heuiiaiit une cible de molybdène (Z = 42). Si vous niveau inférieur b) l.e niveau su|icrieur est mcia.stablc. c) Le niveau
remplacez cette cible par une cible d'argent (Z = 47), les valeurs intérieur est métastahlc. d) Le niveau inteneur est l’etat tonctametit.'il
a) de h) de la longueur d'onde de la raie A'u et e) de la longueur de l'atome, e) La matière aciive e.si un ga/
d’onde de la raie Kf\ augmenteront-elles, diminueront-elles nu 14. La ligure 11.22 présente des dUgiammes paitiels de niveaux
resteront elles inchangées'' d’énergie dc,s atomes d'iiélium et de néon en icu dans un laser
12 I.a raie K^. pour tout élément, est générée par une transition hélium-néon On ditcju un atonie (niélninulans l'état rijiieut lietiriei
entre la couche = 1) et la couche L {n — 2). La figure 11.14 un atome de néon dans son étal fondaïutnUil et l’clever à i'clui Lj,
montre cette raie (dans le cas d'une cible de molybdène) apparais.siint L’énergie de l'état de rhéliiim (20.(il «Vl est prés île I «nergie de
à une seule longueur d'onde. A résolution plus élevée, cependant, l’étal El du néon (20,66 cV), san,s être txaclaiKia égale. CoiniiKiit le
la raie se divise en plusieurs composantes de longueurs d'onde transfert d'énergie peut il se produire si ces énergies ne sont pas
parce que la couche L n’a pas une énergie unique, al Combien exactetiieni égales ?

wwvf I.a .solution SC trouve sur le site Web, à l’adresse ci dessous : 13P. Si vous mesurez le moment cinétique orbital l parallèlement a
l'axe des Z, par exemple, ixnii obtenir la valeur de i , , démontrez que
wvi'W.dlcmcgravifiill.ea/
IL? + L?)''2 = |/(/ H- Q -

SKTIOH 11.1 Le moment cinétique et les moments dipoloires mognétiques


est le plus que vous pouvez dire sur les deux atitros conqKisanies du
1E. Drmoiitrcv que fi = 1.06 X 10 J ■s ~ 6.58 x 10 eV • s. moment cinétique orbital.
2 t Combien d’étals électroniques tmuvc-l-on dans ces soirs couches : 14P. (Un problème traitant du principe de correspondance. ) Lstimez
a) « - 4. / = 3 ; b) n = 3. / = 1 ; c) « = 4, / = 1 ; d) it - 2 ,1 0 ? a) le nombre quantique / associé au mouvement orbital de la Tt'rre
3E a) Combien de valent s de / sont associées à n = 3 b) Combien autour du Soleil et b) le nombre d ’orieniations possibles du plan
de valeurs de m, sont associées à / = I ? de l ’orbite de la Terre, selon les règle.s de la quantificaiion spatiale
c) Déterminez. le demi-angle du plus petit tôiic poux ant être
11. a) Quel est le module du moment cinétique orbital dans un état
balayé par une perpendiculaire à foibilc de la Terre autour du Soleil.
où / = 3 ? b) Que' est le module de sa plus giamlc compo.sanie le long
d’un axe des z imposé 7
5F Combien d ’états électroniques trouve-t-on dans les couches SEOION 11.5 L'expér'ience de Stern et Gerlath
suivantes; a) « = 4, h) n = 1, c) « =• 3, d) n ^ 2? 15E. Calculez, les deux angles possible.? entre le vecteur .spin de
61. lacrivez tous les nombres quantiques associés aux états qui forment l’électron et le champ magnctiqiic de rexcinplc 11 .1. Ayez en tctc
la sous-couche où « = 4 et / = 3 que le inriment cinétique orbital de l’élcetron de valence de l’alomc
7F Un électron présent dans un atome d’hydrogène est dans un état d ’argent est nul.
où 7 = 5. Quel est l’angle minimal possible entre /, et L ? 16E. Supposez que. dans l'cxpérientc de Stern et Gcilach iitili.sunt
Rt Un électron ixinfiné dans un atonie à plusieurs électrons attichc t-4 des atomes d'argent neutres, le champ magnétique R possitle un nvuliilc
comme valeur maximale de m,. Que ^louvez-volls dire sur le reste de 0.50 T. a) Quelle est la difféience «I'cncigic ciilie les iirirmatioiis
de ses nombres quantiques? des moments magnétiques des atomes d'argent dans Ica deux sous
faisceaux'? bl Qiuillc est la fréquence du rayonnement qui induirait
9E. Un électron piéscnt dans un atome à plusieurs électrons possède une transition entre ces deux états'' c) Quelle est sa longneur î I'oikIh
1c nombre qiianiique / = 3 Quels sont alors les nombres quamiques et à quel domaine du spectre éIeciroinagnéii<|iie appaitient-elle''
n, m, et m, possibles ? Le moment inagiiélic|ue d'un atome d ’argent nciilrc rsl égal h
M- Combien d’états électroniques trouve-t-on dans une couche I magneton de Bohr
définie pai le nombre qiiantique n = 5 ’ 17E Quelle est l'accélér.'Uion d'un atome d ’argcni quand il iiavrr.sr
1 IP. Un électron se trouve dans un étal où / 3. Qiicllcs sont les l'aim ant d én c u cu r dans rc x p c n c n tc de Stern et (Jcriach de
grandeurs a) de /, et b) de jj c) Réalise/, un tableau affichant les l'exemple 11 1 ?
valeurs possibles de m,, de /. (en fonction de h ). de /./„rt,.; (t't fonction IBP. Supj'KJsez qu un atome d'hydrogène dans son état fondameiii.il
de //„) et «Je l'angle semi-classique Wcompris entre I. et la direction parcourt 80 cm dans un champ magnétique v c riiia l. son dépldcr-
positive d t l'axe des r. www ment est orienté perpendiculairement an champ, dont lo gr.adiem
12P, Un electron ,se trouve dans un étal où n = 3. Quels sont est dBldi 1.6 X I0-’ T/m. a) Quel est le module de la force exercée
a) le noinbic de valeurs possibles de /. b) le nombre de valeurs p<issibles p;u' le gradient du champ sur r a io n r en raison du niomciii inngnciiqiir
de njf, c) le nombre de valeurs possible.? de m„ d) le nombre d’états de « 'n électron, qu’on considère comme égal à I magnet«'« de Bohr ?
dans la couche ri 3 et e) le nombre de sous-couches dans la couche h) Quel est le déplacement veitical de f atome ilans le parcours de
n ' 3'? 80 cm si la coni|X)sante iiorizonlalc de sa vitesse est 1,2 X 10' m/s?
328 Chapitre II Les atomes

SECTION 11.6 Lo résonance mognétique 26P. Dans le cas décrit dans l’exercice 24, quelles sont, en fsmetion
de ЬЩт1г. les énergies a) du premier étal excite, b) du deuxième état
19L Quelle csl la longueur d ’onde associée à un photon qui induira
excité et c) du troisième état excité du système à sept éleclrons ’?
une transition du spin d’un électron de parallèle à antiparallèle dans
d) Tracez un diagramme des niveaux d’énergie représentant le.s quatre
un champ magnétique dont le module est de 0,200 T ? Supposer que
niveaux d’énergie les plus bas du système.
1=0.
27P. Une boîte cubique dont les largeurs sont L, = i , = L. = L
20L Le proton, comme l'électron, a un nombre quanlique de spin ,r
contient huit électrons. En fonction tic h-Kml.\ quelle est l’énergie
de L'atome d'hydrogène dans son état fondamental (n = I et / = 0)
de l'état fondamental du système à huit électrons '? Supposez, que les
compte deux niveaux d 'énergie. dans le premier niveau, les spins
éleclrons n'inicragissem pa.s entre eux et ne négligez pas le .spin, www
de réicetron et du proton sont parallèles et. dans le deuxième niveau,
28P. Dans le cas décrit dans le problème 27, quelles sont, en fonction
le.s spins sont antiparallèles. Si un atome effectue un tctourncmeni
de h ’/StrUJ. les énergies a) du premier étal excité, b) du deuxième état
de spin, passant d'un étal d’énergie supérieur à un étal d'énergie
excité et c) du troisième état excité du .système à huit électrons?
inférieur, il y a émission d'un photon d'une longueur d’tmde de 21 cm.
d) Tracez un diagramme des niveaux d’énergie représentant les quatre
Les radioastronomes observent ce rayonnciiKnt de 2 1 cm provenant
niveaux d’énergie les plus bas du système.
Je l’espace. Quel est le ciiump magnétique effectif (produit par
le mtnneni diixilairc magnétique du proton) que rélcctron subit lors
SECTION П .9 Lo lonstruction du tobleau périodique
de l’émission de ce rayonnement
211;. Des atomes de sodium excités émettent deux raies spectrales 29E. Démontrez que, .si les 63 éleclrons d’un atome d’europium étaient
rapprochées (le doublet du Kodium ; voir la figure 11.2.1) ayant des distribués dans les couche.s selon une suite «<logique» de nombres
longueurs d’onde de .5il8,9d5 nm cl de 589„‘i92 nin. a) Quelle est la qiiantique,s, cet élément serait chimiquerntmt semblable au sodium
différence d’énergie entre les deux niveaux d’énergie supérieurs ? ЗОЕ. Soit les éléments sélénium (Z = 34), brome (Z = 35) et krypton
b) Cette différence d’énergie apparaît parce que le spin de l'éleeiron (Z = 36). Dans leur partie du tableau périodique, les sous-couches
( — 1 magneton de Bohr) peut avoir une direction parallèle ou anti- des états électroniques sont remplies dans l’ordre suivant
parallèle au champ magnétique interne as,sodé au mouvement orbital b 2s Ip 3s 3p 3d 4i 4p ...
de l’électron. Faites appel aux résultats obtenus en a) pour déterminer
le module de ce champ magnétiqvic interne. Pour chacun de ces élénicnis. 1ю т т с г la sous-couche ixxiupéc la plus
élevée et dites combien d'électrons elle contient.
:.4,2= 1 {' 3IE. Supposez que l’élcetron n 'a pas de spin et que le principe
d’exclusion de Pauli s’applique toujours. Quel.s gaz nobles actuels
demeureraient, le cas échéant, dans cette catégorie ?
32E Quels sont les quatre nombres quantiques des deu.x électrons de
l’hélium quand cct élément est dans son étal fondamental ?
33P. Dans l'atome de lithium (Z = 3). deux électrons ont les nombres
quaniiqucs n = 1, / = 0, m, = 0 et »n, = Quels nombres
quanliques le troisicnie électron doit-il avoir pour que l'atome son
a) dans son état fondamental et b) clans son premier état excité ?
n - ». / = 0 — ----- 34P. Supposez qu’il y a deux électrons dans le même atome. Uius
Figure 11.23 Exercice 21 deux avec n = 2 et / = I . a) Si le principe d’exclusion de Pauli ne
s’appliquait pa.s. combien y aurait-il de combinaisons d’états possibles ?
22E. On applique un champ magnétique oscillant externe d’une fré­ b) Combien d’états le principe d’exclusion de Pauli interdit il ? Quels
quence de .14 MHz à un échantillon contenant des atomes d’hydrogène. .sont CCS élat.s ?
On observe une résonance quand le module du chantp magnétique 35P. Démontrez que le nombre d’étals possibles partageant le même
extérieur constant e,st égal à 0.78 T. Calculez, le module du champ nombre quanlique n est 2 tr.
niagncliquc local au site des protons qui subis.scnl des retournements
de spin, en supposant que les champs externes et locaux y sont SECTION 11.10 Les royons X et la numérololion des éléments
parallèles Pour les protons, д = 1.41 x 10 •"’.l/T.
3ÉE Л l'aide de quelle différence de polenuel minimale un électron
clans un tube Йrayons X doit-il йге ac-céléré pour prcxluiie des rayons X
SECTION 11 8 Plusieurs éleclrons dons des pièges redanguloires d’une longueur d'onde de 0.100 nm ’
23F .Sept électrons sont piégés dans un puits de praentiel infini à une 37E. Sachant que la longueur d'onde minimale d'un rayon X produit
dimension d’une largeur L. En fonction de Ir/HniJ?, quelle est l’énergie par des électrons de 40,0 keV frappant une cible est de 31,1 pm,
de l’état fondamental du système à .sept électrons Supposez, que le.s déterminez la constante de Planck h.
électrons n’interagissent pas entre eux et ne négligez pa.s le Npin. 38E Démontrez que la longueur d’onde de seuil (en picximèires) dans
24E. Un corail rectangulaire de largeurs L et L, 21 contient le spectre continu de rayons X, pciur toute cible, est déterminée
sept électrons. En fonction ilc IrKmL-. quelle est l’énergie de l’état par - 1 2'^0/V,' où V'est la différence de potentiel (en kilovolLs)
foiKlamental du système à sept éli-cimns ? Supposez i[ue les électrons qui acxélèrc les électrons avant qu’ils heurtent la cible.
n'interagissent pa.s entre eux et ne négligez, pas le spin. 39P. Des rayons X sont générés dans un tube à rayoïi.s X par des
29P. Dans le cas décrit dans l’exercice 2L quelles .sont, en fonction électrons accélérés au moyen d’une différence de potentiel de .50.0 kV.
de Л^/8ш1Л les éneigies ai du |)rctnierétai excite, li) du detixième étal 1 In électron subit trois collisions avec la cible avant de s'immobiliser:
excite et c) du troisième état excite du système à sept éleclrons? il perd la moitié de son énergie résiduelle dan.s chacune des deux
d) Tracez un diagiammc des niveaux d’énergie représentant les qiiaire premières collisions. Déterminez les longueurs d ’onde des phoions
niveaux d’énergie les plus bas du système. prcxiuiLs. (Négligez le recul des atomes de la ctble.) www
Exercices et problèmes 329

40R Un électron de 20 keV s’inuiiobilise en subissant deux œllisions Zr Nb Mo Te Ru


successives et semblables à la collision décrite dans la figure 11.15;
il transfomie ainsi son énergie cinétique en énergie de deux photons. Z 40 41 42 43 44
1.a longueur d’onde as.si.x;iée au scconcf phoion est .supérieure de 130 pm Ejr(kcV) 18.00 18.99 20.00 21 04 22.12
à la longueur d ’onde assœ iée au premier photon. a) Détermine/,
l'énergie cinétique de l’électron après sa première collision, b) Quelles
(Indice : une substance absorbera prmctpaleracnt un seul ravonne-
sont les longueurs d'iHide et les énergies ass<x;iées de.s deux photons?
mciii X si rén cig ie des iiIhiioiis du pi cinier i ayon X est suffisante
41P Démontrez qu'un éicetrtm en mouvement ne peut spontanément
pour éjecter un électron d ’une couche K d ’un de scs alomes alors que
SC transformer en photon de rayon X dans un espace ouvert. Pour
les pilotons de l'aulre rayon X ii'oiii pa.s a.sse/ d’ériwyie. i
qu'un tel phénomène se produise, il faut la présence d'un troisième
49P L ue cible de tungstène IZ - 74) est Ixmihartlèc par des électrons
corps (atome ou noyau). Pourquoi cette présence est-elle nécessaire ?
dans un tube à rayons X. a) Quelle est la valeur minimale de la
{Imlice : tenez compte des principes de conservation de l’énergie et
différence de potentiel d'acrélération qui ix'rinenra la iirtHliiriinn tirs
de conservation de la quantité de mouvement.)
raies kp cl Xu caractéristiques du tungstène? b) l’our cette même
42P. Quand des électrons bombardent une cible de molybdène, ils dil'Céreni e tie poieniiel d’accéléialion. quelle est la valeur iiiiiitmale
produisent des rayons X à la fois continus et caractéristiques, comme
de Àmi.i'-' c) Quelles sont les longueurs d ’onde de A,, et de /qi .*
dans la figure 11.14. Dans ectfe figure, l’énergie cinétique des I^s niveaux d ’énergie A. /. et M du ningstéiie tvoir la lintitv 11 Iti)
électrons incidents est de 35,0 keV. Si la différence de potentiel
possèdent des énergies respectives de 69,5 keV, Je 11,3 keV et
d’accélération est aiigmcntcc à 50,0 keV. quelles nouvelles valeurs de 2.30 keV
a) de Â^,. b) de la longueur d'onde de la raie et c) de la longueur
50P l,es énergies de liaison des électrons des cnucties K et /
d’onde de la raie Kfi en résulteront ?
du cuivre .sont re.spcciivemenl de 8,979 keV et tic 0,951 keV, Si un
4.3P Dans la figure 11.14, les rayons X sont générés quand des rayon X K(, provenant du cuivre atteint un cristal de ohlortire de
électrons de 35,0 keV heurtent une cible de molybdène (Z - 42). sodium et produit une réflexion de Bragg du premier ordre à un angle
Si la différence de potentiel d ’accélération est maintenue à cette de 74.1 °, mesuré par rapport aux plans Parallèles des atomes de
valeur, mais en présence d’une cible d’aigent (Z = 47), quelles valeurs sodium, quelle esl la séparation entre ces plans parallèles*’
a) de À„,„„ b) de longueur d ’onde de la raie Ka et c) de longueur
51P a) A l’aide de l’équation 11 26, estimez les rapports des énergies
d’nndr de la raie Kp en résulteront ? Les niveaux atomiques K, L et M
des photons générées par les transitions X,, dans deux atnnirs dont
d e l’argcnl (à comparer avec la figure 11. 16) sont 25,51 keV,3,56keV
les numéros aloniiqucs sont Z et Z', hl Quel est cc rapporl dans le cas
el0J)3keV.
de l’uraniuni et de l’aluminium ? c) de furamum et du lithium 7
44P La longueur d’omle de la raie Ka générée par le fer est 193 pm.
52P Détermine/ connucni les énergies des photons des rayons X AL
Quelle esl la différence d ’énergie entre les deux états de l’atome
théoriques, telles qu’elles sont obtenues à l’aidc de l’cquation 11.'27.
de fer responsables de ce raytinnemeni ?
se rapprochent des énergies mesurées chez les éléments de faibles
45P C alculey. le rapport entre la hingucur d'onde de la raie du ma.sses allant du lithium au magnésium. Pour ce faire, a) trouvez
niobium (Nb) et celle du gallium (Ga). Consultez l’annexe E pour d'abord la constante C dans l'équation 11.27 à cinq chiftres signiRcatils
connaître les données ncce.ssaircs. en la determinant sous la torme des constantes londamentales de
46P. En vous référant à la figure 11.14. calculez approximativement l'équation 11.24, puis en utilisant les dotinées de l 'annexe B h) Bixune,
la différence d ’énergie E, E^, du molybdène. Comparez cette calculez les pourcentages des déviations des énergies Ihéoriqiies par
différence avec la v aleur pouvant Être obtenue si vtHis utilisez plutôt rapport aux énergies mesurées, c) Finaleiiieiil. lepiésenlei gia|)liique-
la figure 11.16 ment CCS déviations cl commentez la icntlancc Í es énergies inr.surécs
47P Voici les longueurs d’onde K^, de quelques éléments : (eV) des pilotons Kn de ces éléiwms sont les suivanu s

Élément À (pm) Élément À (pm) Li .54..3 O 524.9

275 C'o 179 Bc 108,5 F h 78,8


Ti
V 250 Ni 166 B 183.3 Ne 848.6

229 154 C 277 N.T 1n i 1


Cr Cu
Mn 210 Zn 143 N 392,4 Mg 1 254

Fe 193 Ga 134
(Il y a, en lait, plus d'un rayon X K„ en riii.son de lu division du nivcuu
Tracez un graphique de Moseley (comme celui de la figure 11.17) d'énergie /„ mais cct effet est négligeable clans le cas des cléments
Ijoiir représenter ces ilonnées et vérifie/ si sa pente est cohérente par qui sont nommés ici.)
rapporl h la valeur de C obtenue dans la section 1 1.1(1.
48R On bombarde une cible de molybdène (Z ~ 42) avec des électrons SECTION 11.12 Le fonctionnement des ItBerc
de 35.0 keV et 11en résulte le spcctic de rayons X de la figure 11.14 S3E, Les lasers peuvent servir a générer des impulsions lumineuses
1es longueurs d’nndc de kp et de Ka sont rcspecii venicnt de 6.3,0 pm de durée aussi courle que 10 f s a) C iM iibieii de longueurs d ’Otidr
et de 71,()pm. a)Quclle,s s<>nl le.s énergies des phiXons correspondants'’ lumineuses (X “ 500 ninl ainticnl une (elle impulsion b) Détcrnimci
h) r m veut f i l w ces rayonnements à faidc d’un iiiatcriau qui absorbera la grandeur manquante A’ (»m années) ;
la raie Kp beaucoup plu.s fortement que la raie Kn Quelle substance
utilisera t on? Les énergies d'ionisation des électrons K dans le lo is I .s
inolyhdènc et dans quatre éléments voisins sont les suivantes ; T7 T
330 Chapitre 11 Les atomes

54t Dans les conditions décrites dans l’exemple 11.6 a), combien 65P. Dans un certain la.ser. une lumière laser ayant une longueur
de moles de néon faui-il pour amener 10 atomes à leur état excité E, ? d'onde de 694 nm est générée par un milieu actif d ’une longueur de
55£. Dans un atome imaginaire, les niveaux d’énergie sont uniformé 6.00 cm et d'un diamètre de 1,(Юcm. a) Considérez cc milieu comme
ment séparés par une énergie de 1,2 e V, A une température de 2 000 K. une cavité résonante optique semblable à un tuyau d’orgue fermé aux
quel est le rapport entre le nt)mbre d’atomes dans le 13' état excité deux extrémités. Combien de nœuds l’onde stationnaire compte-t-eUe
et le nombre d'tOomes daas le 1 1 'étal excité ? dans l’axe du laser'l h) Quelle variation de la fréquence Д/ du
56E. En mesuraiil la durée de rallcr-rctoiir d’une impulsion laser faisceau va faire augmenter ce nombre de nteuds de I .' c) Démontrez
entre un ob.servaloire situé su rlérrc et un réllccteur situé .sur la Lune, que ù / est simplement l'inver.se de la durée d ’un aller-retour de la
il est pos.sible de mesurer la distance qui sépare ces deux objets, lumière dans l'axe du laser, d) Quelle est la variation de fréquence
a) Oocllc est la valeur prés uc de cette durée b) On peut mesurer la relative correspondante, Д ///? L’indice de réfraction approprié
distance avec une précision de 15 cm. A quelle incertitude quant à du milieu actif (un cri.stal de rubis) est 1,75. www
la diiiéc du voyage cette précision coi ies|X)nd-cllc ? 66P. Des atomes imaginaires pos.sèdenl deux niveaux d ’énergie, dont
57t Un atome imaginaire possède seulement deux niveaux d’énergie, la longueur d'onde de la transition est de 580 nm. Dans un échan­
séparés par une énergie de 3.2 cV. Supjxiscz que. à une certaine tillon paniculier à 300 K, 4,0 x 10*’ de ces alonie.s se trouvent dans
aliiUitlc dans raliiKXsphère d une étoile, il y a 6 .1 x 10*' de ces atomes l’état inférieur, a) Combien d’atomes se trouvent dans l’éiat
pai centimètre cube dans le niveau d'énergie supérieur et qu'il y en a supérieur, dans des eondition.s d'équilibre ihermique ’ b) Supposez
2.5 y lO*' par centimètre cube dans le niveau d ’énergie inlérieiir. maintenant que 3.0 x 10*’ de ces atomes .sont « pompés » dans l'état
Ouelli: c.si la itnipcraiure tie l’atmosphère tie l’étoile à celte altitude '? supérieur par un processus cxlcrne et que 1.0 x 10*’ atomes
demeurent dans l'étal inférieur. Quelle énergie maximale les atomes
58E. On décrit stttivcnl une inversion de populations entre deux
peuvent ils dégager dans une impulsion laser unique si chaque atome
niveaux d'énergie en altiibuaiu au .sy.sième une lcrnpéralurc en kelvins
effectue une transition entre ces deux étals (soit par absorption
négative. Quelle tenipcralure dcehrail un système t)ù la ptipulalion
ou parémis.sion stimulée) ?
du niveau d'énergie supérieur excéderait de U)% celle du niveau
inférieui. et où la différence d’énergie entie les deux niveaux .serait 67P. Peui-on détruire un missile balistique intercontinental à l'aide
de 2.1 eV ? d’un fai.sceau laser intense ? lîn faisceau d'une intensité de 10” W7nr
bifilerai! et détruirait probablement un missile (sans rotation) en 1 s.
59E 1In laser à impulsions émet une lumière d’une longueur d’onde
a) .Si le laser avait une puissance de 5,0 MW, une longueur d'onde de
de 694,4 nm La durée d’une impulsion est de 12,0 ps. et son énergie
3.0 Min et un diamètre de 4.0 ra (un laser très puis.sani). pourrait-il
est de 0.150 J. a) Quelle est la Itmgueui de l'impul.sittn ? b)0)m bien
détruire un inis.sile à une di.staiice de 3 0(X) k m ’’ b) Si on pouvait
de phtilons sonl émis dans chaque impulsion ?
changer lu longueur d'onde, quelle en serait la valeur maximale
60E Un laser hélium néon émet une lumière la,ser à une longueur adéquate? Utilisez l'équation du disque central présentée dans
d ’onde de 632,8 nm et à une puissance de 2,3 mW. À quel taux cet l’exercice 6 1
appareil émet-il des photons '*
6BP. Le faisceau d'un laser à argon (d'une longueur d’onde de
61E. Un fai.sceau laser à haute puissance 1à - 600 nm) ayant un diamètre 5 15 nm) a un diamètre d de 3,00 mm et une puissance continue
de 12 cni est projeté vers la Lune, à une distance de 3,8 x 10-' km. de 5,00 W? Le faisceau est fcKalisé sur une surface diffuse par
Le faisceau ne diverge que pai' diffraction. La position angulaire du bord une lentille dont la distance fo ca le/e st de 3,50 cm. Une figure de
du distjuc de diffraction central (voir la figure 7.12) est donnée par diffraction comme celle de la figure 7.9 est formée, le rayon du
disque centra! étant déterminé par
1.22X
sin —• - .
d I,22/À
R=
i<ù d est le diamètre de l’ouverture du faisceau. Quel est le diamètre
du ilisque de diffraction central à la surface de la Lune'.' (voir l’équation 7.12 et l'exemple 7.3). On petit démontrer que
le disque central contient 84% de la puissance incidente, a) Quel est
62E -Suppose? qu'on ait des lasers dont on peut « régler » avec précision
le rayon du dusqiie central? b) Quelle est l’intensité moyenne
les longueurs d'onde ii'imp<.)rle w'i dans le domaine visible, c’c.st-à-dire
(puissance par unité d'aire) dans le faisceau incident ? c) Quelle est la
dans le drstvunc 450 mit •' à < 650 luii. Si chaque chaîiK- de television
puissance moyenne dans le disque central ?
occupe une largeur de bantlc de 10 .VIH?, combien de chaînes peuvent
entrer dans cette plage de longueurs d'onde ?
Problèmes supplénientaires
63E. \jt volume actif d'un laser coiistriiil avec le semi-eonductcur
OaAlAs n'est que de 2tt0 /rin' (plus petit fiu’iin grain de sable), et le 69. l,e laser à carbonique mariten. Les molécules de dioxyde
laser peiii fournir de faijon cnnlinue une puissance de ‘>.0 niW' à une de carbone se trouvant à une altitude aiiproximativc de 75 km dans
longueur d’oncle de 0,80 /nn. A quel taux généie t il des pholons ? l'atmosphère de Mars subisseni une action laser nattirelle lors(|iic la
64P Dans la figure 1121, les iniroiis du laseï, séparés de 8,0 cm, lumière du Soleil brille. Les niveaux d’énergie en jeu dans l’action
formcnl une , aviié opiitiue dans laquelle on peut établir des ondes sont illustrés dans la figure II 24; une inversion de populations .se
sudionnaires de lumière laser. Chaque onde stationnaire possède un pixKliiit entre les niveaux E, et fc). a) Quelle longueur d'onde de la
nombre, ri de demi longia urs d'onilc dans la cavité de 8.0 cm, où « lumière solaire excite les molécules dans l’action laser? b) A quelle
H une valeur élcvér et où 1rs ondc.s diffèrent légèrement de longueur longueur d’onde l’action la.sci apparaît-elle ? c) Dans quel domaine
d'ondr Près de X — ‘133 nm. quelle est la variation de la longueur du spectre électromagnétique les longueurs d ’onde d’exdrarion ei
d’onilc cmrc (leux ondes stntioiuiaires'? d ’action laser sc situent-elles ?
Exercices et problèmes 331

■ = 0.Ü89 c \ la lumière du Soleil; elles sont manquantes parce qu’elles sont


absorbées par l’atmosphère du Soleil quand la lumière la traverse.
Quand une comète approche du Soleil, l’effet Doppler provcxjué
par la vitesse relative de la comète change la longueur d'onde des
■£ | = O.H>5 «V raies de Fraunhofer, amenant apparemment un chcvauchcmeni de
l’une d’elles ei de la longueur d ’onde retiuise pour provtKiuer l’cxcitaiion
vers le niveau Ej des molécules OH. L’invorsion do populations se
produit alors parmi ces molécules, cl celles ci font des vniissions
stimulées (figure 11.25 hj). Par exemple, quand la uim ètc
Koul'.oulck .s’est approchée du Soleil en décembre 1(172 et en janvier
Rgtre 11.24 P ro b lè m e 6У 1974, elle a (>rtKliiii une émis.sion .stimulée à cnviruii 1 666 M lîr
durant la m oitié de janvier, a) Quelle est la dittérenee d energie
70. ¡/(mission sUnmiée d'une comi>te. Quand une comète s’approche Ej — E, de cette cnii.s.sion ? bl Dan.s quel domuinc du spectre élcwiio-
du Soleil, la chaleur crois-sante provoque l'évaporation de la glace magnétique cette émission se situait elle ?
présente à la surface de Son noyau, créant ainsi autour de lui une
mince atmosphère de vapeur d’eau. La lumière solaire peut ensuite
■L,
dissix;ier la vapeur d’eau en H et en OH. F.lle peut également exeiter
les molécules OH pour les faire passer à de.s niveaux d’énergie
supérieurs, dont deux sont illustrés dans la figure 11.25.
Quand la comète est encore relativement loin du Soleil, la lumière
de ce dernier provoque une excitation égale vers les niveaux Ej et £i
Itigure 11.25 a]). Ainsi, il n ’y a aucune inversion de populations
entre les deux niveaux. Cependant, quand la comète approche
du Soleil, rexciiation au niveau £ , diminue et une inversion l'o ----- En
de populations apparaît. Cela est prrxluit par l’une des nombreuses a) b)
longueurs d’onde (appelées mies de Fraunhofer) manquantes dans figure 11,25 Problème 70
]2 La physique
nucléaire

Des noyaux rodioactifs injectés à un patient s'accumulent dans certaines parties de son corps, subissent une désintégration
radioactive et émettent des rayons gamma. Ces rayons peuvent être captés par un détecteur et traduits en images colorées
sur un écran vidéo. Dans l'image ci contre (à gauche, vue frontale d'un patient; à droite, vue arrière d'un patient),
les régions brunes et les régions
orangées indiquent les sites
d accumulation de noyaux
radioactifs (colonne vertébrale,
bassin et côtes).
12.1 La découverte du noyau 333

12.1 La découverte du noyau


Au tout déb u t du xx* siècle, les scientifiques connaissaient peu la structure de l’atom e,
hon n is le fait q u ’il contenait des électrons. L ’électron a été découvert par J. J. Thom son,
en 1897 ; sa m asse était alors inconnue. Il était donc im possible de spécifier le nom bre
d ’élec tro n s ch a rg és n ég ativ em en t co n ten u s dan.s un au tm e d o n n é. L es iUimic.s ctuiil
électriquem en t n eutres, ils devaient au ssi co n ten ir une certain e charge po sitiv e, m ais
personne ne savait quelle form e cette charge positive com ponsm oire revêiaii,
Ln 1911, Ernest Rutiierford ém it l'hypoihèse (¡tie c'eiie charge jKtsiiive ciaii coiiLeiiln^c
au centre de l’atom e, form ant son n o y au , et que, de plas. u : noyau rcpr6^icnlцit la plu.s
grande partie de la m asse de l’atome. C ette hypothèse de R utherford n ’ctait pas gratuite,
mais largement basée sur le.s résultats d ’une experience q u ’il avait conçue et qui fuiréulisce
p a r ses c o lla b o ra te u rs, H ans G c ig c r {connu p o u r so n co m p ieiir O eig e r) et F in est
M arsden. un étudiant de 20 ans qui n 'av a it pas encore son baccalauréat
À l’époque d e R utherford, on savait qu e ceriain.s éléiiKuits, q u 'o n di.sait rad io actH s,
sc transform aient spontaném ent en d ’auftc.s é lé m c n u et ém ctuiicnt alors des purtieulcii.
Le radon en est un ; il ém et des particules alpha ( a ) possédant une énergie approxim ative
de 5,5 M eV. On sait au jo u rd ’hui que ces particules sont des noyaux d ’ atomes d ’helium .
L’idée de R utherford était de diriger les particules alpha énergiques sur une m ince
feuille de métal e t de m esurer l ’angle de la déviation q u ’elles subissaient en la traversant.
L es particules alpha, environ 7 3(XJ fois plus m assives q u 'u n électron, ont ntic t harge
de + 2e.
La fig u re 12.1 illu stre le m o n ta g e de l ’exixli ien ce de O c ig e r et M ar.sdcn. L i iir
source alpha était un tube de verre à paroi m ince rem pli de radon gazeux. T.’exjiérience
a in sista ii à com pter le nom bre de particules alpha déviées à divers angles <f>.
La fig u re 12.2 illu stre leurs résu ltats. N otez que r é c h e lle v erticale est lo g arith
inique. On constate que la plupart des particules .sont déviées à des angles plutôt |ieiits,
m ais (et ce fut là la grande surpri.se) q u ’une très petite fraction d ’entre clics sont déviées
à de très gran d s an g les, ap p ro ch an t des 180°. S elon R u th erfo rd ; « C ’c.sl .sûrem ent la
chose la plus incroyable q u ’il m ’ait été donné de voir. C ’était presque aussi incroyable
que de voir un obus de 15 pouces rebondir sur un m ouchoir de papier. »

S o n n e alplia

figure 12.1 Un montage (vu en plongée) Figure 12.2 Les points représentent
utilisé dans le laboratoire de Rmherfortl des données sur la diffusion «les jiailiciilrs
de 1911 à 1913 i^nir ctudier la diffusion alpha à travers une feuille d'ut, obicnucs
des particules alpha par de miiKc.s feuilles Iiariiciger d Maisden à l’aide de l'appareil
de métal. Le dclci:tcur peut tourner illustré dans la figure 12. l. La courbe
à divers angles de diffusion <i>. t.a source cominue est la préditiion tltéoriquc,
alpha était du radon gazeux, produit fondée sut l'hypoilièse que l’atome
de la désintégration du radium. < e simple possède un petit noyau niassil cliargé
appareil tenant sur une table a permis posilivemciu, Nnie/.qiir IVchrlIe verticale
la découverte du noyau atomique est logarithmique, couvrant six ordres
de piandcur Lc\s données ont été ajustcei'i
au point expérimental entouré d'un oercio
pour correspindrp it la riHirhc ihénric(iif.
334 Chapitre 12 La physique nucléaire

Pourquoi R u th erfo rd fut-il si s u rp ris? À l’épo q u e où l ’cx p cricn ce a é té réalisée,


la plupart des physiciens croyaient au m odèle dit plum -pudding (pudding aux prunes)
de l’atom e, p ro p o sé p a r J. J. T hom son. O n p en sait alors qu e la c h a rg e p o sitiv e était
dispersée dans tout le volum e d e l ’atom e ; on croyait que les ékx'trons (les « p n in es »)
v ib raien t a u to u r d e p o in ts fixes à l'in té rie u r d e c e tte sp h ère de ch arg e p o sitiv e
(le « pudding »).
D ans celte perspective, la Jdree d e d iffu sio n m axim ale qui aurait dû ag ir su r une
p articu le alpha trav ersan t une si grosse sphère ch a rg ée p o sitiv em en t aurait aloix clé
beaucoup trop petite pour dévier la particule alpha ne seniit-ce que de 1®. (La diffusion
attendue avait été com parée à ce q u ’on aurait observé si on avait projeté une balle de
fusil à travers un sac d e boules de neige.) D e m êm e, les électrons contenus dans l’atom e
f- Paninili-s n innrlpntfs auraient égalem ent très peu agi sur la m assive particule alpha énergique Ils auraient, en
j Feuille fait, été eux-m em es fortem ent diffusés, tout com m e un essaim de mouchcTons aurait été
/ de m étal cible dispersé pai une pierre le traversant.
R utherford pensa que, pour faire dévier la particule alpha vers l’arrière, il fallait une
grande force ; cette force n ’c tail p o ssib le qu e si la charge positive était co n cen trée au
centre de l ’atom e au lieu d 'ê tre dispersée dans tout son volum e. D ans ce cas, la particule
incidente p o u rrait s ’ap p ro ch er très p rès de la charge pt>silive sans la p én é trer; un tel
rappnx;hem ent se traduirait par une grande force de déviation.
L a figure 12..') illu stre d es trajecto ires p o ssib les suivies par d es p articu les alpha
traversant les atom es de la feuille de m étal cible. O n peut co n stater que la p lu p art ne
X ----------- ■ ' f -* sont pas déviées ou ne le sont peut-être que légèrem ent, alors qu e quelques-unes (celles
qui passent, par hasard, très près d ’un noyau) .sont dév iées d ’un grand angle. A partir
de ces données. R utherford conclut que le rayon du noyau devait être plus petit q ue celui
d e l ’ato m e d ’un facteu r d ’e n v iro n 10'’. A u trem en t d it, l ’a to m e est p rin c ip a le m e n t
com posé d 'esp a ce vide.

Hgure 12.3 L’angle d e déviation d’une particule alpha dépend de la distance qui .sépare
sa trajectoire du noyau atomique. Les grandes déviations sont le résultat de passages
' Noyau très rapprochés.

Exemple 12.1
Une particule alpha de 5,30 Me'V fonce, par hasard, directement sur k' — ^ ^aÇAu
le imyau d’un atome d'or, qui contient 7Ù protons. A quelle distance “ 4 tk'() d
la particule alpha s’approchc-l-clle du noyau avant de s’immobiliser
momentanément et de renverser sa trajectoire? Négligez le recul
où Ça (= 2 e ) est la charge de la particule alpha (2 protons)
du noyau.
y.\u 1~ 7*^0) est la charge du noyau d'or (79 protons).
SOIUTION. Le (onteptdé utilisé ici est que, durant tout le processus, Si on remplace les charges et si on isole d. on obtient
l 'éneigie mér.tniquc totale F du système composé de la particule alpha
et (lu itoyau d’('rcst conservée. Ainsi, l’énergie mécanique initiale ^ (2c)(79e)
(lu systènx!, avant l'inteotction pariicule-noyau, esT égale à son énergie 4;K|,£r„
mécanique finale ff, alors que la particule alplia s'inuiiobihse momen­ (2 X 79K I.60 X l ( r ' ‘' 0 2
tanément. l.’énergie initiale F, correspwul à l’énergie cinétique IÇ,
(47t)(8,85 X 10 F/m)(5,.3()McV)(1.60 x 10 1? .1/MeV)
(le la particule alpha incidente, l ’énergie finale E, correspond
= 4.29 X 10 (réponse)
à l'énergie potentielle U du système (Ténergie einctique est alors
nulle) ( )n peut déterminer f / à l'aide de l’équation 4 4.3 du volume 2
( O - q,^./(47rci)r)). C ’est une petite distance par rapport à l’atome, mais non ptu' rapport
•Soit d la distance qui sépare le centre de la particule alpha et le au noyau. Ln fait, elle est eonsidcrablcmenl plus grande que la
centre (lu noyau d’or quand la particule se trouve immobilisée. somme des rayons dn noyau d’or et de la particule alpha. Donc, cette
On peut alors exprimer la conservation de l’énergie £, £', de particule alpha inverse son mouvement sans vraiment «toucher» le
la manière suivante ; noyau d’or.
12.2 Certaines propriétés nucléaires 335

12.2 Certaines propriétés nucléaires


L e tableau 12.1 p rése n te c e rta in e s c a ractéristiq u es de q u elq u es n o y au x ato m iq u es.
Q uand on s ’intéresse particulièrem ent aux caractéristiques associées au noyau lui-m êm e
(p lu tô t q u ’à ses ca ractéristiq u es en tant que co n stitu an t d ’une p artie de l’ato m e), on
nom m e ce noyau un n u cléid e.

La terminologie nucléaire
L es noyaux se co m p o sen t d e proion.s et de neutrons. l.e nom bre d e p n ilo n s contenus
dans un noyau (appelé n u m é ro a to m iq u e du noyau) est représente pai' le sym bole Z ;
le n o m b re d e n e u tro n s est représenté par le sym bole N. Le nom bre tot'il rie neuirmu.
et de protons contenus dans un noyau est appelé n o m b re d e m asse A, donc

Л Z + N. U2.1)

Les neutrons ci les protons sont des nucléoas


O n rep résen te les nu cléid es p ar des symbole.s seiublable.s à ceux d e la p rc n iiè ir
co lo n n e du ta b lea u 12 1 O n p rend ici ’’ ’Au co m m e ex em p le. L ’ex p o san t 197 est
le nom bre de mas.se Л \ £ sym bole chim ique Au indique qu e cet élém ent est l ’or, dont
le num éro atom ique est 79. Selon l'éq u atio n 12.1, le nom bre de neutrons de ce nuclôlde
est 197 - 79, ou 118.
L es nucléides ayant le m êm e num éro atom ique Z , m ats des nom bres différents de
neutro n s N sont ap p elés iso to p e s. L ’o r a 35 iso to p es, de à d o n t un seul
( '''^Au) est stable ; les 34 autres sont radioactifs. De tels ra d io n u c le id e s subissent une
d é s in té g ra tio n en ém ettant une particule et. de ce fait, sc transform ent en des nucléides
différents.

Classer les nucléides


Le,s atom es neutres de tous les isotopes d 'u n élém ent (avant tous le m êm e Z ) pc'ssèdeni
le m êm e nom bre d ’élec tro n s cl les m êm es p ro p rié té s ch im iq u es ; ils e n tre n t dans la
m êm e case du tableau périodique Or, les propriétés nucléaires des is(JtO|K-.s d ’un même
é lé m e n t so n t très d iffé ren te s d ’un iso to p e à l’autre. D onc, le tableau p ério d iq u e e \i
d 'u n e utilité lim itée jxiur les physiciens, les chim istes et les ingénieurs nucléaires.

TABtEAU 12.1 Quelques propriétés de certains nudà'des


Masse** finerpie iIh liMiciiii
Nucléide Z N Л Stabilité* (U) Spin*** ^MeV/iiiicIrtHii

'11 1 0 1 9*^985% 1.1Ю7 825 1

■’Ll 3 4 7 7.016 003 3 5,60


2
Mp 15 16 31 100% 30.973 767 1 8,4«
Л
“ Kr .16 48 84 .57.0% 83,911 507 0 8,77
‘‘"Sn 50 70 120 32,4% 119,902 199 O 8.11
■’’Gd 61 93 157 1.5.7% 156,923 956 3 8,21
J
"” Au 79 118 197 100% l% ,966 543 3 791
7
227Ac 89 1.38 227 21,8a 227,02.7 750 1 7.65
7
2«1>U 94 145 239 24 ИХ) a 239,0.52 158 j. 7..16
* Dans le cas ries iim Icidcs siahlcs, rahmulaiicc it«^ âsotopi'.i cm dunnce. .soit la ftactiou Jet alomct
Je ce type SCtrouvanl dans im ccnanlillon carxlérisliqtie dr cri cléiiiciil Dans K l aa dc.s iuiclciJc.s
radioactifs, on donne la dena-vie
** Selon la pratique standaid. la masse tkimiéc est celle de l'atome neutre, non telle du novau seul.
*** I .r spin est en nnilé.s de h
336 Chapitre 12 La physique nucléaire

Figure 124 Un graphique représentaiil les nudéides conmis. La wjnc verte esi constituée
des luicléides stables ; la zone beige contient les radionucléides. Les nucléides stables
de faibles mas.ses po,s.sèdenl environ des nombres égaux de neutrons ei de protons ; cependant,
plus les nucléides sont massifs, plus leur proponion de neutrons est grande. La figure montre
qu 'il n'y a aucun nticléide .stable pour Z > 83 (bismuth).

On classe les nuclcitles dans un ta b lea u des n ucléides com m e celui de la figure 12.4 ;
ce type de graphique présente les nucléides en fonction du rapport entre le nom bre de
protons et le nom bre d e neutrons q u ’ils contiennent. Les nucléides stables sont en vert ;
les radionucléides sont en jaune. On constate que les radionucléides tendent à se trouver
de chaque côte (et à l’extrém ité supérieure) d ’une bande bien définie de nucléides stables.
N otez égalem ent que les nucléides stables légers tendent à se trouver près de la ligne où
A' = Z, ce qui signifie q u 'ils ont environ les m em es nom bres de neutrons et de protons.
P ar contre, les nucléides les plus lourds tendent à avoir beaucoup plus de neutrons que
de protons. P ar exem ple, on constate qu e le ’’’ A u possède 118 neutrons et seulem ent
79 protons. st)it un excès de 39 neutrons.
ü n p eu t se p ro cu rer un tab leau de n u c lé id e s sous la fo rm e d ’un tab leau m ural
dans lequel chaque petite case contient des données .sur le nucléidc q u ’elle représente.
La ligure 12.,'5 en m ontre une partie, dont le centre est ‘^’^Au. O n y trouve les abonilanecs
relatives (habituellemenL com m e on les trouve sur Terre) dans le cas des nucléides stables
et les dem i-vies (m esure de la durée de vie m oyenne) dans le cas des radionucléides.
L a d ro ite en pente in d iq u e une lig n e d 'i.s o b a re s, d es n u cléid e s p arta g ea n t le m êm e
nom bre de m asse, /i — 198 dans le cas présent.
D epuis le début des années 2000. des nucléides ayant des num éros atom iques aussi
élevés que Z = l \ 4 (A = 289) ont clé observés au cours d 'ex p érien ces de laboratoires
(aucun élém en t ayant un n u m é ro ato m iq u e su p é rie u r à 92 n ’e x iste d an s la nature).
Bien que tes g ro s n u cléid es so ie n t g én é rale m en t g ran d em en t in stab les el ne d u ren t
q u 'u n bref instant, certains nucléides superm assifs sont relativem ent stables e t o nt une
durée de vie plutôt longue. C es nucléides superm assifs form ent un îlot de stabilité à des
valeurs élevtV s d e / et de /V sur un tableau de nucléides. com m e cc4ui de la fig u ir 12.4.

✓ VERIFIEZ VOS CONNAISSANCES I ! .Scion la figure 12.4, qtiels nucléides. paimi les siiivanrs.
I .sont le plus susceptibles de ne pas être détectés ; (Z = 26), ^ ’As (Z = 3.3),
1 ‘'"Nd (Z = 60). ' ’T.U (Z = 71 ), (7 = 82) ?
12.2 Certaines propriétés nucléaires 337

Figure 12 5 Une partie agrandie du


tableau des nucléides de la figure 12.4, - - -. r— , ç ,
dont k centre e.st *®’Au. Les cases 197p i»Spb. s«»pb I ; «>2rb-
82 - l,5ti , 21,511 ,1 933ti. '53000«! 2,16d
venes représentent les nuclcidcs ■> 45 min 2,4
stables, dont les abondances relatives ■p— r -
sont diMinccs. Les cases beiges rI \ I ue-n
IfïS'n j 200-j-i
20 ïui rj * soa n^
repa^Æntent les radionucléides, 2.83 h i 7,4I h j.I 26,1
26,16h ■; 79,(
79,0 6 }. 12,2d
dont les demi-vies sont données. -k
Les lignes d’isobares de nombre
'««llg ‘»’Hg ! •'"Hg j I =^'"Ug
de masse A constant forment 10,9 % I 23,1 ÔL ! 13,2 %
ï 9 3 li o.ir,%
des lignes diagonales, comme
le montre la droite A = 198 107
IWAu 'Au i ‘«'Au ! »»’Au
de l'exemple. .'®‘Au '*AU
39.4 h 166 d 6,!8d i 100% ' 2.64« ! 3,14d j4«.4 nitn

IS:7|,( i ISOp, ISÏp, , I Pt


1
; 32,9% 333 % • 253 % 18,3 6 ; 7 , 2 \ ^ 30.8t»ôr,

i9î,r : 104,,. , '«■’Il '««Il "% r ^ ir :


73.« a 1 62,7% 19.26 ; 2>lh 52 s ; 5.8 min
---- - ----------
j
; ,7
'»‘fis ' i‘« o ! '«■‘Oi, l» ü s 1 '«‘'Us - 4
76
15,4 d 41,0% 30.511 ; 6.0 a 0.5 imn S5 ujiii „1
__ i__ - i J___l _ .. JL
115 116 117 IIH 119 120 121
Nonibro de neutrons V

Les rayons nucléaires


L e fem tom ètre ( = 10 m ) est u n e u n ild co m m o d e p o u r e x p rim e r le.s d ista n c e s à
l’échelle de.s noyaux. Cetle unité est .souvent a p p e lé e /m « i ; ces deux term es pariagcni
la m êm e abréviation. D onc,

1 ferntom èlrc = 1 ferm i = 1 tm = 10 m. 112.2)

Poui connaître davantage la taille et la structure des noyaux, on peut les bom barder
avec un faisceau d ’électrons à haute énergie et observer com m ent ccux-ci .sont dcviéfi par
les noyaux. Les électrons doivent posséder suffisam m ent d ’énergie (au moins 2iKl M eV)
pour avoir des longueurs d ’onde de D e B roglie plus petites que les structures mit léaires
visées.
Tout com m e l’atome, le noyau n ’est pas un objet plein ayant une surface bien définie.
D e plus, bien que la plupart des nucléides soient .sphériques, certains .sont ellipsoïdes.
N éa n m o in s, les e x p é rie n c e s d e d iffu sio n des éle c tro n s (ain.si qu e les experiences
d 'au tres types) perm ellent d ’attribuer à chaque nuclcidc un rayon effectif déterminé par

/-0/4 lO (12.3)
OÙ A est le n o m bre de m asse et /•<) « 1.2 fm. U n co n state que le vo lu m e d ’un noyiiii,
qui est proportionnel à r^, est d irectem en t proportionnel au nom bre de m asse A cl esl
indépendant des valeurs respectives de / et de A'
L ’éq u a tio n 12.3 ne s ’ap p liq u e pas aux t m d t ’ itle y à fu iJa , dc.s n u cléid e s iic h e s
en n eu tro n s p rixluits p o u r la p rem ière fo is en la b o ra io ires d an s les an n ées 1980
C es nucléides sont plus gros que ce qu e perm et de prédire l'éq u atio n 12.3. cai ccilains
de leurs neutrons form ent un halo au to u r d ’un cœ u r sp h ériq u e fo rm e p a r les protons
el les autres neutrons. Les isotopes du lithium en sont un exem ple, (^uand un neutron
s ’a)outc au *Li pour form er le '’l.i, dont aucun n ’esi un nucléidc à halo le layun (T fe u if
augm ente d ’environ 4 %. C ependant, quand deux ueuim ns s ’ajoutent au ^Li pour form er
l'isotope “ Li riche en neutrons (le plus gros des isotopes du lithium ), ils ne ee joipneni
pas au noyau existant, m ais fo rm en t plutôt un h ait' autour de lui, aupnieruaiit le rayon
effectif d 'en v iro n 3 0 % . 11 sem ble qu e cette configuraiion en halo nielie. en jeu moin.s
d 'é n e rg ie qu un t:œ ur co ntenant 11 nucléons. (D ans le prissent r hapitre, on considère,
de façtin générale, que l’équation 12.3 s’applique.)
338 Chapitre 12 Lk3 physique nucléaire

Les masses nucléaires


De nos jours, il csl possible de m esurer les m asses atom iques avec une grande précision,
ü a n s la section 1.6 du v o lu m e I , on a vu q u ’on ex p rim e d e te lle s m asses en un ités
de m asse atom ique u, d efin ies d e m anière à ce que la m asse ato m iq u e (non la m asse
nucléaire) du '"C .soit exactem ent de 12 u. L a relation entre cette unité et le kilogram m e
est approxim ativem ent (une valeur plus précise apparaît à l’annexe B)

1 u = 1,661 X 10 ^’ kg. (12.4)

Le nom bre de m asse A d ’un nueléide est nom m é ainsi parce que ce nom bre représente
la m asse du nueléide, ex p rim ée en unités de m asse ato m iq u e et arro n d ie au nom bre
entier près. Donc, la masse atom ique de '^ A u est 196,966 373 u, q u ’on arrondit à 197 u.
D ans les réactions nucléaires, la relation Q = Am (équation 8.49) est un outil
indispensable. O n a vu dans la section 8.12 que Q est l’énergie dégagée (ou absorbée)
quand la m asse d ’un systèm e ferm é d e particules interagissant entre elles varie de Am.
On a égalem ent vu dans la section 8.12 que la relation d’E instein E = me- indique
que l’énergie au repos d ’une masse de 1 u est de 931.5 MeV. Donc, selon l’équation 8.45,
on peut utiliser

r* = 931,5 M eV/u (12.5)

com m e co n v ersio n co m m o d e e n tre l ’é n e rg ie e x p rim é e en m illio n s d ’é le e iro n v o lts


et la m asse exprim ée en unités de m asse atom ique.

Les énergies de liaison nucléaire


L a m asse M d 'u n noyau est inférieure à la m asse to tale Xw d e ses p ro to n s et d e ses
neutrons C ela signifie que l’énergie au repos M t^ d ’un noyau est inférieure à l’énergie
au repos totale 2 (w r~ ) de ses protons et de scs neutrons. l.ji différence entre ces deux
énergies est appelée é n e rg ie d e liaiso n du noyau ;

AL| = 2 ( m r ‘) Mc- (l’énergie de liaison). ( 12. 6)

A ttention: l’é n e rg ie d e lia iso n n ’est pas un e én e rg ie q ui résid e d an s le noyau.


Il s ’agit plutôt d ’une différenc e entre les m asses au repos d ’un noyau et de ses nucléons.
Si on pouvait diviser un noyau en scs nucléons, il faudrait transférer une énergie totale
équivalente à àE^i à ces particules durant le prrKcssus. B ien q u ’on ne puisse pas diviser
un noyau de celte m anière, l'én erg ie d e liaison nucléaire dem eure une m esure com m ode
de la Ibree qui m aintient les parties d ’un noyau ensem ble.
l 'é n e rg ie d e liaiso n m o y e n n e p a r n u c lé o n . AE^„ constitue une m eilleure m esu re;
il s’agit du rapport entre l’énergie de liaison A/Tj d ’un noyau e t le nom bre A de nucléons
q u ’il contient :

ALj
A/:,„ (l’énergie de liaison moyenne par nucléon) (12.7)
T

O n peut considérer l’énergie de liaison m oyenne par nucléon com m e l’énergie m oyenne
nécessaire pour diviser un noyau en ses nucléons individuels.
La figure 12.6 représente graphiquem ent l’énergie de liaison m oyenne par nucléon
A£|„ en fonction du nom bre de m asse A d 'u n g rand nom bre d e noyaux. C eux qui ,se
trouvent dans la partie supénciu'c du graphique sont très fortem ent liés ; c ’est d o n t dire
q u ’il laiidrait fournir une grande quantité d ’énergie par nucléon pour d iviser f un de ces
noyaux I e s noyaux qui se trouvent plus bas dans la courbe (à droite et à gauche) sont
plus faiblement liés ; il faut donc m oins d 'én e rg ie i:>ar nucléon pour les diviser.
C es sim ples énoncés su r le contenu de la figure 12.6 ont de profondes im plications.
1a*s nucléo n s d 'u n n o yau se tro u v an t d an s la p artie d ro ite de la co u rb e seraient plus
fo n e m e n t liés si ce noyau était div isé en d eux au tres se trouvant p rès du som m et de
la courbe. Un tel pr<x:essus, appelé fissio n , se produit n atu rellem en t che? les noyaux
m assifs (nom bre d e m asse A élev é ), co m m e celu i d e l’u ran iu m , qui p eu t su b ir une
fis.sion sp o n ta n ée (c 'e s t-à -d ire san s cau se ni so u rc e d 'é n e rg ie e x té rie u re s). C ’est
12,2 Certaines propriétés nucléaires 339

Figure 12.6 l ’énergie de liaison moyenne


par nucléon de ceruiins nucléides
représentatifs. Le nucléide de nickel
*'-Ni possède l’énergie de liai.son par •
nucléon la plus élevée (environ
8,794 60 MeV/nuclcon) de tous
les nucléides stables connus. Notez
que la particule alpha C'He) ptisscde
une énergie de liaison ptu' nucléon
plus élevée que cdlcs de scs voisines
du tableau périorlitnie et, par conséquent,
qu’elle est également particulicrcmcnt
stable.

ce p ro ce ssu s qui est à la b ase d es arm es n u cléaires, d an s le sq u elles on p ro v o q u e la


fission sim ultanée de nom breux noyaux d ’uranium ou de phitoniuin iwnii produire une
explosion.
L es nucléons d 'u n e paire de noyaux se trouvant dans la partie gaucho de la courbe
•seraient plus fortem ent lié.s si ces noyaux se com binaient pour form er un noyau unique
se trouvant près du som m et d e la courbe. Un tel processus, appelé fu siu n , se produit
naturellem ent dans les étoiles. Si cc n était pas le cas, le Soleil ne brillerait pas et la vie
sur la Terre ne pourrait exister.

Les niveaux d'énergie nucléaire


L’éneigie des noyaux est quantiliéc, com m e celle des atom es, c ’cst-à-dirc que le.s noyaux
ne peuvent exister que dans des états q uantiques di.screts, chacun possédant une énergie
bien définie. La figure 12.7 illustre certains de ces niveaux d ’énergie du nucléide *®A1.
nucléide type de faible m asse. N otez que l’échelle d ’cncrgic est en m illions d 'clcctro n
v o lts p lu tô t q u ’en éle c tro n v o lts, u n ités u tilisé es av ec le s ato m es. Q u an d un noyau
e ffe c tu e u n e tran sitio n v ers un n iv eau d ’én e rg ie inférieur, le p h o to n ém is se situ e
typiquem ent dans le dom aine des rayons ganuna du s p c a ie élctU om agnétiquc,

Spin et magnétisme nucléaires


D e nom breux nucléides ont un moment einétujue tmiJtkiiir m tiinscquc. on spin, et un
s-1
m om ent m a^nétiqnr nucléaire in trin sèq u e qui lui est asso cié. Bien q ue les m om en ts
c in é tiq u e s n u cléa ire s aient ap p ro x im ativ e m e n t le m em e m o d u le qu e les m o m en ts
cinétiques des électrons atom iques, les m om ents m agnétiques nucléaires sont boaueoiip
plus petits que les m om ents m agnétiques atom iques types.
2-
La force nucléaire
La force responsable des m ouvem enis des éle< irons aïoiuiqiics esr la foice é lertu u n a-
i.Éf- gnétique que l’on connaît O p e n rla m , p o u r lier les parties du noyau en sem ble, il faut
une im portante force attractive d 'u n type totalem ent d ifféirn i, suflïsam m ent lo tie p o u r
surpas.ser la iiirce de répulsion de.s protons (chargés positivem ent) agissant en lrr n ix cl
pou! m ain ten ir les protons et les neutrons d an s le m inuscule volum e q ue eonslitue le
noyau. C ette force nu cléaire d o it égalem eni av o ir une courre jrortée de so rte qu e son
influence ne déborde pas trop de la « surface » nucléaire.
0 S elon la conception actuelle, la force nucléaire qui lie les neutrons et les p rotom
^"Al dans le noyau n’est pas une force fondam entale de la nature, m ais un effet secondaire de
Figure 12.7 Les niveaux d ’énergie l'in te ra c tio n fo rte qui lie les quarks p<nir foraior les neutrons et les piotous D e même,
du nucléide ’*Л1. déierminés à l’aide la force attractive entre certaines m olécules neiiires est un effet serontlaire de la fou e
d’expériences de réactions nucléaires électrique coulom bienne qui agit dans les liaisons in icrn rs di v nm léculcs
340 Chapitre 12 La physique nucléaire

Exemple 12.2
On peut imaginer tous les niicléides cttmmc un mélange de neutrons Le deuxième concept clé est que le rayem r est determine par l’équation
et de protons qu’on peut appeler matière nucléaire. Quelle est la 12.3 (r = roA'^^), où r(,est 1,2 fm (= 1.2 x 10“ ” m). Si on remplace
masse volmiiiquc de la matière nucléaire? r par cette expression, on obtient
Am m
SOIUÏION' lei, le premier concept dé est qu’on peut determiner la masse P =
^ш ^А
volumique (moyenne) p d'un noyau en divisant sa masse totale par
son volume. Soit m la masse d ’un nucléon (soit d ’un proton, .soit d’un Note/ (]ije A s’est simplifié ; donc, cette éejuation de la masse volumique
neutron, puisciue ces particules ont environ la meme masse). Donc, la P s’applique à tout noyau pouvant être considéré comme sphérique
triasse d'un noyau contenant A nucléons est Am. Ensuite, on suppose et ayant un rayon déterminé par l’équation 12,3. Si on utilise
que le noyau est sphérique et possède un rayon r. .Son volume est 1,67 X 10“^ kg comme masse m d’un nucléon, on a
alors I rrr^ ; on |>eut décrire la masse volumique du noyau ainsi
1.67 X 10“ ^’ kg
P = 2 X lO'^ kg/m-’, (réponse)
Am |л<1.2 X 10 m)-’
P =
ce qui représente environ 2 x lO’“' fois la masse volumique de l’eau.

Exemple 12.3
Quelle est l’énoigic de liaison moyenne par nucléon asstxiiée au Ce.s 50 électrons peuvent être combinés aux 50 protons pour former
nucléidc ‘-'■'Sn'.’ 50 atomes d’hydrogène neutres. On a alors
SOlUnCH: On utilise ici deux comeph clés. (atome ‘’"Sn) —r ,50(atomes H séparés) 70(neutrons séparés). (12.9)
1. On peut nhrenir l’énergie de liaison moyenne par nucléon A£|„
si trn raltiile d'aboal l’éiieigie de liaison AE¡, puis qu’on la divise Dans le tableau 12.1. la masse d’un atome '^.Sn est de
par le nombre de nucléons A contenus dans le noyau, .selon l’équa - 119,902 199 u. et la masse d ’un atome d ’hydrogène est de
lion 1? 7 (AF|„ = A/;,/4). 1,(Ю7 825 u ; la masse d'un neutron est 1,008 665 u. Donc,
2. On |>eut obtenir A£, en calculant la différence entre l'énergie l’équation 12.6 donne
au repos M c - du noyau et l’éncrgic au repos totale 1 , ( m c - ) des
A£| = X(nic^) Mc‘
nucléon.s individuels qui fonnent ce noyau, selon l’équation 12.6
(A/q - Ifim ?) - Me>). — 50(/Ин<‘^) + 7(X’/n„r’) —Л/snC^
Dans le tableau 12.1, ou constate que le noyau '^Sn compte .SOprotons = 50(1.Ш7 825 u )r’ + 70(1.008 665 u)t^
(Z .“>0) et 70 neutrons (N — A Z - 120 — .^0 = 70). Donc,
il faut imaginer qu'un noyau de ‘‘°Sn est divi.sé en .50 protons et - (119,902 199 u)c-
en 70 neutrons. = (1.095 601 u)i^ ^ (1,095 601 UK931.5 MeV/u)
(noyau '®Sn) 5(Xptotons séparés) 7(Xnaitrons ,séparé.s), (12.8) I 020,6 McV.

puis calculer la différence qui en résulte dans l’éncirgie au repos. l'équation 12.5 (c“ ~ 931.5 MeV/u) fournissant une conversion d'unités
Pour effectuer ce calcul, il faut les masses d'un noyau '-®.Sn, commode. Notez que futilisation des masses atomiques au lieu
d’un proton et d’un neutron. Cependant, étant donné que la mas.se d'un des masses nucléaires n’influence pas le ré.sultat, parce que la masse
atome neutre (le noyau plw t les électrons) est plus facile à mesurer que des 50 électrons contenus dans l’atome '^ S n est soustraite de celle
la masse d’un noyau sans électrons, les calculs des énergie.s de liaison des- électrons contenus dans les 50 atomes d’hydrogène.
sont traditionnellement effectués en masses atomiques. D ont, on Maintenant, l’éqnation 12.7 donne l’cncrgic de liaison moyenne
modifie l'étjualion 12.8 pour que .son membre de gauche comporte par nucléon ;
un atome '^ S n neutre. Pour ce faire, on insère .50 électrons dans le
membre de gauche (qui correspondront ainsi aux 50 protons contenus A7^, 1020.6 MeV
dans le noyau '^ ’Sn). On doit également additionner 50 électrons AFi„ = A 120
au membre de droite pour obtenu un équilibre dans l’équation = 8,51 MeV/nuclénn. (réponse)

12.3 La désintégration radioactive


On peut voir, dans la figure 12.4, que la plupart des nuclcides identifiés sont radioactifs.
Un nucicide rad io actif ém et sponiancm cnt une p articu le et se tran sfo rm e alo rs m un
niicléide différent, occupant une case différente d an s le tableau des nucléidc.s.
La désintégration radioactive a fourni la prem ière preuve que les lois qui gouvernent
le m onde siibatom ique sont statistiques. On peut prendre, p ar exem ple, un échantillon
de 1 mg d ’uranium . Il contient 2,5 x I 0 ‘®atom es du radionucléide ayant une très
longue vie. Les noyaux d e ces atom es existent sares 4e désintégrer depuis leur création
(bien avant la form ation de notre systèm e solaire). C haque seconde, il n ’y a environ que
12 d es noyaux présents dares l ’échantillon q ui se désintègrent en ém ettant une particule
alpha, se transform ant alons en noyaux de ^^Th. C e p en d an t on peut constater ce qui suit.
12.3 La désintégration radioactive 341

Il n’y a absolument aucun moyen de prédire si un certain noyau contenu dans un échantillon
radioactif fera partie de ceux qui se désintégreront dan.s la prochaine seconde. Tous k s noyaux
de l’échantillon ont la même probattilitc de le faire.

Bien q u ’on ne puisse prédire quels sont les noyaux d ’un échantillon qui se désinté­
g rero n t, on p eu t d ire q u e, si un éc h an tillo n c o n tie n t N n o y au x ra d io a c tifs, le taux
( = —(¡Nldi) auquel ils sc désintégreront est pmjTonionnel à N:
dN
------- = AyV. (12.101
dt
où À, la c o n s ta n te d e d é s in té g ra tio n « p o ssèd e une v aleu r ap c ciliq u c p o u r cluiquc
radionucléide. Son unité SI est l'in v erse de la seconde (s
P our déterm iner N en fonction du tem ps /. on léci il d ’alxird ré t|iia iio n 12 lO ■
dN
— À dt, (1 2 .Ш
N
piii.s, si on effectue l'in tég rale des deux m em bres, on obtient

ou In (V - In Л/о ~ - À ( t - /(il- (12.12)


D ans celle équation. N q est le nom bre de noyaux radioactifs conrcnii.s daii.% réid ian tillo n
à un tem ps initial arb itraire f„. Si ou pose qu e q, = 0 et .si on récrit ré q u a iio ii 12.12,
on obtient

i n ------- —Al. (12 13)


^0
Si on calcule la fonction exponentielle des deux m em bres (la fonction exponentielle est
la fonction inverse du logarithm e naturel), on obtient

N,

ou N(,e -A t (la désintégration radioactive). (12.14)

où N q e.st le nom bre de noyaux radioactifs contenus dans l’échantillon à / 0 cl


le nom bre de n o y au x résid u e ls à to u t in stan t su b sé q u en t t. N o tez q u ’u n e am p o u le
électrique (par exem ple) n ’obéit pas à une telle loi de désintégrntit'n exponentielle Si on
effectue un test pour me.surer la durée de vie de 1 ООП am poules élertrit|u es, on «’attend
à c e q u ’elles se « d ésintègrent » toutes (ù ce q n ’elh v grillent) plus on m oins au mémo
instant. La désintégration des radioniidéitics obéit a tine loi très difféiem e
O n est souvent plus intéressé par le taux de désintégration R (= dNIdr) que par N
lui-m êm e. Si on dérive l ’équation 12.14, on obtient
dN
R - ------- - X N (,e-''
dl

ou R = R q€ -A l (la (lé.sinlégration radioaciivet. (12 И )

une autre form e de la loi de la désintégration radioactive (éq u atio n 12.14). Ici. Ht, est
le taux de d é s in té g ra tio n au tem p s / — 0 , et R est le tau x ù un te m p s su b sé q u e n t t.
On peut m ainten an t récrire ré q iia lio n 12.10 en ftm etion du taux d e d ésin tég ratio n H
de l'échantillon :

R ÀN , (12.10)
342 Chapitre f2 La physique nucléaire

o ù K e t le n o m b re d e n o y a u x ra d io a c tifs iV qui n e se so n t p a s e n c o re d é sin té g ré s d o iv e n t


être é v a lu é s au m êm e instant.
L e ta u x d e d é sin té g ra tio n to ta l R d 'u n é c h a n tillo n c o n te n a n t un o u p lu sie u rs
rad io n u cléid es e st ap p elé l'a c tiv ité ra d io a c tiv e (ou sim p lem en t l’activité) d e l'échantillcat.
L 'u n ité SI d e l’a c tiv ité e.st le b e c q u e re l, e n l’h o n n e u r d u p h y sic ie n H en ri B e c q u e re l,
d é c o u v re u r d e la rad io a c tiv ité :

1 b ecq u erel = 1 B q = 1 d ésin té g ra tio n p a r se co n d e

U n e u n ité p lu s an cien n e, le c u rie , e st e n c o re u tilisé e :

I cu rie ~ I C i = 3,7 X 10'** Bq.

V o ici u n e x e m p le d ’u tilis a tio n d e c e s u n ité s : « L e 15 ja n v ie r 2 0 0 0 , l ’a c tiv ité d e la


p astille d e co m b u stib le n u cléaire u sée r f 5 658 était d e 3,5 x 10'-’ B q ( = 9 ,5 x K d C i). »
D o n c , c e tte jo u rn é e -là . 3.5 x Ю '“* n o y a u x ra d io a c tifs d a n s la p a stille se d é sin té g ra ie n t
c h aq u e seco n d e. L ’id en tité d es ra d io n u cléid es œ n te n u s d a n s la p astille, leurs c o n stan tes
d e d é sin té g ra tio n Л et les ty p es dir rad ia tio n s q u 'ils é m e tte n t n't>nt a u cu n ra p p o rt av ec
cette m esu re d e l’activ ité.
S o u v en t, u n d é te c te u r p la c é p rè s d ’un éch a n tillo n ra d io a c tif n ’e n re g istre p as to u tes
le s d é s in té g ra tio n s q u i s 'y p ro d u ise n t, à c a u se d e la g é o m é trie ou d e so n in e ffic a c ité .
D an s ces circon.stances. il m esu re u n e activ ité p ro p o rtio n n e lle (et in fé rie u re ) à l'a c tiv ité
réelle d e l’éch an tillo n . U ne telle m esu re ne s'e x p rim e p as en becquerels, m ais sim p lem en t
e n cix ip s p a r u n ité d e tem ps.
Il ex iste deux m esu res d e tem p s co u ra n m ie n t u tilisées p o u r e x p rim e r la d u rée d e vie
d ’un ty p e d e ra d io n u c lé id e d o n n é . L ’u n e e st la d e m i-v ie un rad io n u cléid e, qui est
le te m p s o ù /V et R a u ro n t tous d eu x a tte in t la m o itié d e leu rs v aleu rs m itrales. L 'a u tre
m e su re e st la d u r é e d e v ie m o y e n n e r , q u i e st le te m p s o ù N et R a u ro n t to u s d eu x
atteint f' ' d e le u rs v aleu rs in itiales.
P o u r re lie r Ту2 e t la c o n s ta n te d e d é s in té g ra tio n Я, o n in s è re R = dans
l’é q u a tio n 12.15 e t o n re m p la c e t p a r O n o b tie n t
-Л7„,
■yRn — R(\e

Si o n c a lc u le le lo g arith m e n atu rel d e s d e u x m e m b re s e t q u ’o n iso le T j^, o n o b tie n t

À •
D e m êm e, p o u r re lie r r c l À, o n in sère R —e '/?„ d a n s l’é q u a tio n 12.15, o n re m p la c e t
p a r r et o n iso le r ; o n o b tie n t alors
1
r =
A

O n ré su m e ces résu lta t ainsi ;

T
/|P = '"—2 = r ,In .2. (1 2 .1 7 )
A

/ VERIFIEZ VOS CONNAISSANCES 2: i nucléide '" I est radioactif, ayant une demi vie de 8,04
jours. À midi, le premier janviei. l’aelivitc d’un certain échantillon est de 6(K) Bq. En fonction
du concept de demi-vie. et sans calcul écrit, déterminez, si le 24 janvier, à midi, l'activité
de l'échantillon sera un peu inférieure à 2(X) Bq, un peu supérieure à 200 Bq. un peu inférieure
à 75 Bq ou un peu supérieure à 75 Bq.
12.3 La désintégration radioactive 343

Exemple 12.4
Le tableau ci-dessous donne quelques mesures de rac tiv itc d ’un
échantillon de '^1. un radionucleide souvent utilisé en médecine ptiur
mesurer le taux d’absorption de l’kxle par la glande thyroïde.

Temps (min) И (coups/s) Temps (min) R (eoup.s/s)

4 .392,2 132 10,9


36 161,4 164 4.56
68 65,5 196 1.86
mo 26.8 218 1,00

IJélerminez la constante de désintégration X et la demi-vie de ce


radionucléide.
SOIUTIOH: Le premier comeptdé utilisé ici est le suivant' la constante
de désintégration X dctenninc la diminution exponentielle de l'activité R figure 12.8 Exemple 12.4 Un graphique du logariflimc naturel de
en fonction du temps i (comme l’indique l’équation 12.15). Par consé l’activité d’un échantillon de ‘^'*1 en fonction du temps, hast* sur les
quciit. on devrait pouvoir déterminer X en représentant graphiquement données du tableau
les valeurs de R en fonction du temps t.
Cependant, il est difficile d ’obtenir X à l’aide d’un graphique La figure 12.8 montre le graphique do In K en fonction du temps i
de R en fonction de t parce que R diminue de manière exponcnlielle pour les mesures données. La pente de la droite qui mile Ic.s points du
avec t, scion l’équation 12.15. Donc, un deuxième concept clé e.sl ici graphique est
néce.vsairc ; il consiste à transformer l’cquation 12.15 en une fonction 0 “* 6) ^
linéaire de I pour facilement déterminer X. Pour ce faire, on calcule pente = — r - — r — —0,027 5 min" '
225 min —0
les logarithmes naturels des deux membres de l’équation 12.15.
On obtient ainsi Donc, -X —0,027 5 mm '

In /? = ln(/fo^ = In R„ i ln(c-^ ') ou X - 0,027 5 mm 1,7 (réponse)

= in ffn -X/. (12.18) Pour obtenir la demi-vie 7i/->du radionucléide, on fait appel au concept dé
selon lequel le temps qu’il faut à l’activité R pour diminuer do moitié
Étant donné t|ue l’équation 12.18 est sous la forme y = h t- im , où est donné par l'équation 12.17 C/’i/j = In 2/a ). Ln SC basant sur Cétte
b et m sont des constantes, il s’agit d’une équation linéaire donnant équation, on obtient
la grandeur In R en fonction de i. Donc, si on représentait graphique­
ment In K (au lieu de R ) en fonction de t, on devrait obtenir une In 2 In 2
Tin - — ~ 25 min. (réponse)
droite. De plu.s, la pente de la droite devrait être égale à —X. 0,0275 niiii -l

Exemple 12 5
On constate qu’un échantillon de 2.71 g de KCI provenant d ’un iVln2
(12.191
entrepôt de produits chimiques est radioai tif et qu’il se tiésiniègrc R
à un taux constant de 4 490 Bq. On attribue les désintégrations au
potassium et. particulièrement, à l’isotope ‘•"K, qui constitue 1,17% On sait que la valeur de A( dans celle équation esi de l . n ^ d i i imhiiIiic
du pota,ssium naturel. Calcule? la demi-vie de ce nucléide. total d'atomes de potassium contenus dans l'éi haiitillon. On sait
également que N¡¡_ doit être égal au nombre N kci ilc molécules
SOLUTION' Le premier concept clé utilisé ici est le suivant : étant donné contenues dans réchantillon. On peut obtenir N ^ n à partir de
que l’activité K de l’échantillon semble constante, on ne peut la masse molaire A/kci de KCI (masse d’une mole de KCl) e t de la
déterminer la demi vie 1\,2 en représentant graphiquement In R niasse donnée de rcehantillon
en fonction du temps t comme on l’a fait dans l’exemple [2.4
(on obtiendrait simplement une droite hori/nntale). Cependant,
on j-ieul faire, appel aux deux conce|^ tiés suivants.
Nv
л о т Ь т de moles
(
dans réchaiiiillon :)
N,
MécYi ,,
TT
Mkci
( 12, 20)

1. On peut cak iilei la demi-vie Тщ à partir de la constante de où est le nombre d’Avogadro (6,02 x 10^’ mol ' S i o n consulic
désintégration X à l ’aide de l'équation 12.17 {Тщ lu 2/X). l’annexe L, on constate que la masse molaire du potassium est
de .^y,l()2 g/iiiol et que celle du chlore esr rie T'^.455 g/m ol. dom .
2. On peut ensuite relier X à l'activité donnée R de 4 490 Bq à faide la masse in o la it r de K C I est 74..5.55 g/mol. L’équation 12.2U donne
de l’équation 12.16 (/( = XW). où N est le nombre de noyaux alors
de '***K(donc, d’atomes) contenus dans l’échantillon.

Si on combine les éciuations 12.17et 12.16, on obiieni ^K( 1 = 2.188 У К Я


74..555 g/m ol
344 Chapitre t2 La physique nucléaire

t;f)mnie étant le nombre de molécules KCl contenues dans l’échan- (2,560 X K >^)ln2
tilion. Donc, le nombre total d’atomes de potassium est également T,n -
4 490 s - '
2,188 X l ( P . et le nombre d’atomes de "“K dans l’cchantilloti doit être
= 3 .9 5 x l()“^ s = 1 . 2 5 x 10® a. (réponse)
N = 0,01 I 7/Vk = (ü.ül I 7)(2,188 x 10=^).
Cette demi-vie de *K e.si appréciable à récliclle de l’âge de l’Univers.
= 2,560 X 10''®atomes.
Donc, l’activité du “"K contenu dans l'échantillon entreposé diminue
Si on fcinplaccN (tar cette valeur et /f par l’activité donnée de 4 4‘X) Bq tri'S lentement, trop lentement pour qu’on la détecte en quelques
(= 4 4V0 s~') dans l'équation 12.19, on obtient jours d’observation, ou tnême dans l’espace d’urx: vie. Une fraction
du potassium contenu dans notre corps est constituée de cet isotope
radioactif, ce qui signifie que nous sommes tous légèrement radioac­
tifs. Notre corps possède aussi d’autres radionucléide.s comme le ‘*C.

12.4 La désintégration alpha


(Jiiand u n m>yau su b it une d é s in té g ra tio n a lp h a , il se tran sfo rm e en un nu cléid e d ifféren t
en ém ettan t u n e p aiticu le alpha ftin noyau d ’h éliu m , "*Hc). P a r ex em p le, q u an d ru ra n iu m
su b it u n e d e sin te g ra tio n alp h a, il se tran.sform e e n th o riu m ’^ T h ;
2ÎSJJ 23-1J h q. lHe_
( 12.21)

C e tte d é s in té g ra tio n a lp h a d u --’"U p e u t se p ro d u ire s p o n ta n é m e n t (s a n s a p p o rt


d ’én erg ie ex térieitre) parce q u e la m asse to tale d es p roduits d e dé,sintégration *’‘* Th et '^He
e s t in fé rie u re à la m asse d u '’’’'U o rig in a l. D o n c, l ’é n e rg ie a u re p o s to tale d e s p ro d u its
d e d é sin té g ra tio n e s t in fé rie u re à c e lle du n u c lé id e o rig in a l. C o m m e o n l'in d iq u e d an s
l ’c q u a tio n 8 .4 9 , d a n s u n tel p ro c e ssu s, la d iffé re n c e e n tre l ’é n e rg ie a u re p o s in itia le et
l’é n e rg ie au re p o s tïn a lc to ta le e st a p p elée la v a le u r Q d u p ro c e ssa s.
D an s le c a s d ’u n e d é sin té g ra tio n n u c lé a ire , o n d it q u e la d iffé re n c e d a n s l’én e rg ie
au rep o s e s t Vénergie de désintégrauon Q. D an s l’éq u atio n 12.21, la v a le u r d e Q d e cette
d ésin tég ratio n e st 4 ,2 5 M eV o n d it q u e cette én erg ie e,st d ég ag ée lors d e la désintcgraticai
alp h a d u p a r la tran sfo rm atitm d e l’é n e rg ie au rep o s en én e rg ie cin é tiq u e d e s d eu x
4,25 MeV p ro d u its.
l,a d e m i-v ie d u a s s o c ié e à c e p ro c e s s u s d e d é s in té g ra tio n e s t 4 ,5 x 10® a.
P o u rq u o i c.st c lic si lo n g u e '? Si le - ^ U p eu t se d é sin té g re r a in si, p o u rq u o i les n u c lé id e s
d ’ un é c h a n tillo n n e se d c s in lé g rc n i-ils p a s to u s en m ê m e te m p s ? P o u r ré p o n d re
à ces q u e stio n s, il fau t é tu d ie r le p ro cessu s d e d é sin té g ra tio n alpha.
O n p e u t im a g in e r un m o d è le d a n s leq u el u n e p a rtic u le a lp h a (d é jà fo rm é e ) ex iste
d a n s le n o y au a v a n t d e s ’e n éch ap p er. L a Figure 12.9 d o n n e l’é n e rg ie p o te n tie lle U(r)
ap p ro x im a tiv e d u sy stè m e c o n stitu é d e la p artic u le alp h a et du n o y a u - ’"'Th résid u el en
Figure 12.9 Une fonction énergie fo n ctio n d e la .séparation r d e ce u x -c i. C e tte é n e rg ie e st u n e c o m b in a iso n 1 ) d e l’én e rg ie
potentielle lors de l’émis.sion p o ten tielle asso c ié e à l ’in te ra c tio n fo rte (attractiv e) qui ag it à l'in té rie u r d u n o y au e t 2)
d’une particule alpha ptir le nucléidc d u p o ten tiel c o u lo m b ie n a.s.socié à la fo rce é le c triq u e (ré p u lsiv e ) q u i a g it en tre les tleux
^•'’'U. La droite horizontale noire p articu les av an t e t ap rès la d ésin tég ratio n .
nommée Q = 4,25 MeV indique L a d ro ite h o riz o n ta le n o ire n o m m é e Q = 4 ,2 5 M eV in d iq u e l’é n e rg ie de d é sin té ­
l’éncrgic de dé.sinlégraiion. l/épaissc g ra tio n du p ro c e ssu s. Si o n su p p o se q u 'e lle repré.sentc l'é n e rg ie to ta le diî la p a rtic u le
partie gruse de cette droite représente alp h a d u ra n t la d é sin té g ra tio n , la zo n e d e la co u rb e U(r) situ ée au -d e ssu s d e c e tte d ro ite
les séparations r interdites à la particule c o n s titu e u n e b a rriè re d 'é n e rg ie p o te n tie lle se m b la b le à c e lle d e la Figure 9 .1 5 . C e tte
alpha du |)oint de vue de la physique
b a rriè re n e peut ê tre su rm o n té e . Si la p a rtic u le a lp h a p o u v a it se tro u v e r à u n e c e rtain e
classique. La pailicule alpha c.st
d ista n c e r à l’in té rie u r d e la b a rriè re , so n é n e rg ie p o te n tie lle / / e x c é d e ra it .son é n e rg ie
leprésemée par un poinL à l’intcrieur
de la barrièir. fl’éiHTgic potentielle
to ta le E. D u p o in t d e v u e d e la p h y s iq u e c la s s iq u e , c e la sig n iF ie ra it q u e so n é n e rg ie
(à gauche) et hors de cette barrière
cin é tiq u e K (q u i est é g a ie h E - U) sera it n ég a tiv e - une .situation im p o ssib le.
On ]veut m aintenant voir pourquoi la particule alpha n’est pas im m édiatem ent ém ise
(à droite), après qu'elle l’a traversée
piir eftet tunnel. La droite notre du noyau d u : ce noyau e.si en to u ré d 'u n e im p ressio n n an te b arrière de potentiel
horizontale nommée Q ' - 6,81 MeV qui occujie (si vous l'im ag in ez en trois dim ensions) Ic3 volum e se trouvant entre deux
indique l énergic de désintégration couches sphér iques (dont les rayons sont environ de 8 fm et de 60 fm). C et argum ent est
alpha du nucléidc ’^U. Cl.es deux si convaincant q u ’on modifie m aintenant la dernière q i’iestion : étant donné que la barrière
i.sotopcs ont la même fonction énergie sem ble em prisonner en perm anence la particule dan.s le noyau, com m ent le noyau ^**11
potentielle parce qu'ils ont la même pcut-il ém ettre une particule alpha ? Voici la réponse ; vous avez vu dans la section 9 .1 0
charge nucléaire.) q u ’il y a une p ro b a b ilité finie q u 'u n e p a rtic u le p u isse trav erser, p a r e ffe t tu n n el,
12 A La désintégration olpha 34S

u n e b a rriè re il’ é n e rg ie p o te n tie lle q u i s e m b le in s u rm o n ta b le d u p o in t d e v u e d e la


p h y siq u e c lassiq u e. E n fa it, la d ésin té g ra tio n alp h a c.st un ré su lta t d e l'e ffe t tu n n el.
É tan t donné q u e la d em i-v ie d u nu cléid e e.st très longue, la b arrière sem b le ne pas
« fu ir» b eau co u p . L a p articu le alp h a, qui fait d es allers e t reto u rs dans le noyau, .suppose-
t-on. d o it to u c h e r la su rfa c e in te rn e d e la b a rriè re e n v iro n 10-^ lo ts av an t d e p a rv e n ir a
la tra v e rse r p a r e ffe t tu n n e l, so if e n v iro n U é” foi.s p a r seco n d e d u ra n t e n v iro n 4 X 10'^
an n ées (l’â g e d e la T erre) ! O n n e p eu t, b ien sûr, q u ’a tten d re à r e x te n e u r p o u r co m p te r
les seu les p artic u le s alp h a q u i réussissent à s ’éch ap p er.
O n p eu t v érifier le b ien -fo n d é d e c e tte e x p licatio n e n e x a m in a n t d 'a u tre s su o stan ccs
ém ettrices de p articu les alpha. O n p ren d ici u n ex em p le co n trastan t, soit la d ésin icg n u itm
a lp h a d ’un au tre iso to p e d e l'u ra n iu m , le n u c lé id e d o n t l’é n e rg ie d e d ésiiiiég raiio n
Q', q u i est d e 6.81 M eV , e s t e n v iro n 6 0 % su jiérieu re â c e lle d u (I,a valcui d e Q '
e st é g a le m e n t repré.seniée p a r u n e d ro ite h o riz o n ta le n o ire d a n s la fig u re 12 .1 .) O u a vu,
d an s la sectio n 9 .1 0 , q u e le c o e ffic ie n t d e tra n sm issio n d ’u n e b a rriè re c.st très sen sib le
au x p etites vai ialio n s d ’é n e rg ie to tale d e la p a rtic u le q u i e.ssaic d e la p én étrer. D o n c, on
p eu t s ’a tten d re à c e q u e la d é sin té g ra tio n alp h a sc p ro d u ise p lu s fa c ile m e n t d a n s le cas
d e t-e n o y au q u e d an s le c a s d u b t c ’esL en effet, ce qm sc p ro d u it. Lc tableau 12.2
m o n tre q u e s a d e m i v ie n ’e s t q u e d e 9 .1 m in u te s . U n e a u g m e n ta u o n . d ’u n fa c te u r
d e se u le m e n t 1,6, d e la v a le u r d e Q d im in u e la d e m i-v ie (c ’est-ii d ire l'e ffic a c ité d e la
b a rriè re ) d ’u n fa c te u r d e 3 x lO'"*. C ’e s t c e q u ’o n p eu t a p p e le r d e la sen sib ilité

1A61EAU 12.2 Comparaison de deux émetteurs de partkuies olpha


Radionucléides Q Dcnii-v ic
/.rsp' 4.2.S MeV 4.5 X 10’ a
6.81 MeV 9.1 min

Exemple 12.6
On a les niasses atomicjues suivantes : en utilisant les masses atomiques Si on urilise les masses atomiques
données dans l’cnoncc du problème, l'équation 12,22 dr-vieni
238,030 79 U *Hc 4,002 6 0 u
Q = (238,050 79 U)c= - (2.A4,fl4^ pa u + 4.002 60 n)c’
2^<Th 2.34.043 63 U 'H 1.007 83 U
= (0,004 ^6 u)f- = (0,004 36 UKU31,3 MeV/u)
2^’Pa 237.051 21 u = 4,25 MeV (reiviiise)

Pa est le symbole de l’élément protactinium (Z 91 j. Notez que l'utilisation des masses aloiniqiics au lieu des masses
nucléaires n’innnence pas le résuliai parce que la masse, totale
a) Calculez l’énergie dégagée durant la désintégration alpha de
des électrons dans tes produits est sousiraiie de la inasst- îles iiiitléons
I.e processus de désintégration est
-v électrons de l’atome ™l) original.
i- 'He.
b) Démontrez que le nucléide ■
’ '*U ne peut spontanément émettre un
N ote/ la fayon dont la charge nucléaire est conservée dans cette pnrton.
équation, la somme des numéros atomiques du thorium (90) et
SOLUTION; Si c’était le cas, le pnx-es.sus de désinrégiation serait
de l ’hélium (2) donne le numéro atomique de l’uranium (92).
Le nombre de nucléons est également conservé 238 = 234 + 4. »Kj; i17p^ I,.j

SillimON Ici, le toreepiclé esi i|ue l’énergie ilégagée lois de la désintégra­ (Vous devez vérifier si la charge nucléaire et le nombre de nucléons
tion est Q. qii on petit calculer en utilisant la vaiiation de masse \ M sont t<His deux conservés lians ce processus.) Si tm (ail ajipel au nié me
causée par la désintégration du ^'*U. On utilise l’équation 8.49, conrepi (lé ei à la même mcihode qu’en a), on »onsiaïc une la mas.se
des deux produits de désiniégralion ( - 237.0.'il 21 u 4 l.(K)7 83 u)
( 12.22)
e x e rd e ru it telle du -^“fl de \ m ~ f),0OR 25 u. avec une énergie de
où la masse initiale Af, esi celle du ^’’*11 et la masse finale M, est la désintégration de Ç — 7,68 MeV. Le signe négatif indique qu'il
somme des masses du et du 'He. Cotiime dans l'exemple 12.3. l’.Hit/oKrtiir 7.68 MeV à un noyau de ’'"’l? iviur qu'il ém rnr un pioiiin .
on doit eriecliior ee calcul pour des atomes neutres, c ’esi à dire une telle émis,sion ne se prixJuira certainement pas spsintanétncnt
346 Chapitre 12 La physique nucléaire

12.5 La désintégration bêta


U n n o y au qui se d ésin tèg re sp o n tan ém en t en ém ettan t un électro n ou u n p o sitro n (particule
d e la m a sse d ’u n é le c tro n d o n t la c h a rg e e s t + e ) s u b it u n e d é s in té g r a tio n b é ta .
A l’in s ta r d e la d é sin té g ra tio n a lp h a , il s ’a g it d ’u n p ro c e ssu s sp o n ta n é , p o ssé d a n t une
én e rg ie d e d é sin té g ra tio n d é fin ie c l u n e d e m i-v ie . E t c o m m e la d é sin té g ra tio n a lp h a, la
d é sin té g ra tio n b ê ta e st u n p ro cessu s statistiq u e, d é fin i p a r les é q u a tio n s 12 .1 4 et 12.15.
D a n s la d é s in té g ra tio n b êta m o in s {fi ), un n o y a u é m e t u n é le c tro n , c o m m e d a n s la
d ésin té g ra tio n
Vr 12'S + e - + i/ ('/1^, = 14,.3j). (1 2 .2 3 )
D a n s la d é s in té g ra tio n b ê ta plus u n n o y a u é m e t u n p o s itro n , c o m m e d a n s la
d ésin tég ratio n
«C u '’^Ni + e {Ti/2 = 1 2 J h). (1 2 .2 4 )
L e sy m b o le a re p ré s e n te u n neutrino, u n e p a rtic u le n e u tre d e trè s fa ib le m a s s e o u
d e m a sse n u lle é m ise p a r le n o y au e n c o m p a g n ie d e l ’é le c tro n o u du p o sitio n p e n d a n t
la d é s in té g ra tio n . L e s n e u trin o s n ’in te ra g is s e n t q u e trè s fa ib le m e n t a v e c la m a tie rc ;
c ’e s t p o u rq u o i ils .sont si d iffic ile s à d é te c te r c l le u r p ré s e n c e e.sl lo n g te m p s p a ssé e
iiraperçue*.
I.a c h a rg e e t le n o m b re d e n u c lé o n s s o n t to u s d e u x c o n s e rv é s d a n s le s d e u x
pr<x:cssus c i-d e ssu s. D a n s la d é sin té g ra tio n d e l ’é q u a tio n 12.23, p a r e x e m p le , o n peut
e x p rim e r la c o n se rv a tio n d es c h a rg e s ain si ;
{ + L V ) = ( f I6 e ) + ( e) t (0 ),
p a rc e q u e le p o ssè d e 15 p ro to n s, q u e le ’“S p o ssè d e 16 p ro to n s e t q u e le n e u trin o ;/
a u n e c h a rg e n u lle. D e m êm e, o n p e u t n o te r la c o n se rv a tio n d e s n u c lé o n s d e la faço n
su iv an te
(3 2 ) = (3 2 ) t- (0) + (0),
parce q u e le -^P et le p o ssè d e n t c h a c u n 3 2 n u c lé o n s et q u e ni l ’é le c tro n m le n eu trin o
n e .sont d es n u cléo n s.
Il p e u t v o u s s e m b le r s u rp re n a n t q u e le s n o y a u x p u is s e n t é m e ttre d e s é le c tro n s ,
des p o sitro n s e t des neutrinos, p u isq u ’o n v ous a d it q u ’ils n e so n t fo rm és q u e d e n eu tro n s
e t d e p ro to n s. C e p e n d a n t, o n a vu p a ic é d e m m e n t q u e les a to m es é m e tte n t d e s p h o to n s,
m ais il n ’a ja m a is été d it q u ’ils e n « co n ten aien t ». O n d it q u e les p h o to n s sont c réés d an s
le p ro c e ssu s d ’ém issio n .
Il e n va d e m em e d es é lectro n s, d e s p o sitro n s et d es n eu trin o s é m is p a r les n o y au x
p en d an t la d é sin té g ra tio n ; ils so n t c ré é s p e n d a n t l ’ém issio n . D a n s le cas d ’u n e d é sin té ­
g ra tio n b ê ta m o in s, un n e u tro n se tra n s fo rm e e n p ro to n d a n s le n o y a u d e la m a n iè re
su iv an te :
n P+ c + 2^. (1 2 .2 5 )
D an s le c a s d e la d é s in té g ra tio n b ê ta p lu s, un p ro to n .se tran .sfo rm c e n n e u tro n d e la
m an ière su iv an te ;
P -V n ^ e* + O. (1 2 .2 6 )
C e s proc!es.sus d ém o n tre n t p o u rq u o i le n o m b re d e m asse A d ’u n n u cléid c su b issa n t une
d é sin té g ra tio n bêta n e v arie p a s ; un d e scs n u clé o n s c h a n g e sim p le m e n t d e p ro p riétés,
cc qui e st coliéreni a v ec l’é q u atio n 12.25 o u l’é q u a tio n 12 26.

I a desintepratinn Kîia com prend aussi l,i capture d 'é le ctro n un noyau so diSiimèprc en absorhani
un de ses (‘leoiroas atom iques, cinettam un neutrino au cours du processus. ( )ii ne s ’attardera pas ici
à c c processus. l)e plus, la particule ém ise dans la désintégration »léfiiiic par 1 équation 12.2Î est,
en fait, un antinrutrino. distinction q u 'o n ne fera pas dans celte iniriKiia-lion.
12.5 La désintégration bêta 347

D a n s le s d é s in té g ra tio n s a lp h a e t b é ta , la m ê m e é n e rg ie e s t d é g a g é e à c h a q u e
d é s in té g ra tio n in d iv id u e lle d ’u n ra d io n u c lé id e . D a n s la d é s in té g ia tio n a lp h a d ’u n
radionucléide pruticulier, le.s particu les alpha ém ises po ssèd en t toutes la m êm e énergie cin é­
tiq u e n ettem en t d éfin ie. C ep en d an t, ilans la d ésin tég iatio n bêta m o ias d e l’éq u atio n 12.25,
av ec é m issio n d ’un é le c tro n , l ’é n e rg ie d e d é sin té g ra tio n Q e st p a rta g é e (en p ro p o rtio n s
v a rié e s) e n tre l'é le c tro n e t le n e u trin o é m is, l ’a rfo is , l'é le c tro n o b tie n t p re sq u e to u te
l’én e rg ie , p a rfo is c ’e st le n e u trin o q u i o b tie n t la m ajeu re p a rtie d e l'é n e rg ie . D a n s to u s
les cas. cep e n d a n t, la so m m e d e l’én e rg ie de l'é le c tro n d d e l ’é n e rg ie d u n eu trin o d o n n e
figure IZlO La répartition dc.s énergies to u jo u rs la m êm e v a le u r Q. L'n p a rta g e se m b la b le d ’é n e rg ie , d o n t la stim m e e st é g a le
cinciiqiics des jxisitronî. émis pendant à Q, SC p rotluif d a n s la d ésin té g ra tio n b ê la p la s (éq u atio n 12.26).
la désintégration bêta de '^ u . L'énergie
cinétique maximale (7é„,,) est 0,653 MeV.
D o n c, d an s la d ésin tég ratio n bêta, l’é n c i^ ic d e s élcctron.s o u d e s p o sitro n s om is poui
Dans toute.s les désintégrations de '’‘’Tu.
se s itu e r e n tre z é ro e t u n e c e rta in e v a le u r m a x im a le L a fig u re 1 2 .1 0 illu s tre
eette énergie est partagée entre le positron la ré p a rtitio n d e s é n e rg ie s d e s p o s itro n s d a n s u n e d é s in té g ra tio n b ê la d u ^’* ru (v o ir
et le neutrino, dans des proportions l'é q u a tio n 1 2.24). L 'é n e rg ie m a x im a le d u p o sitro n d o it ê tre é g a le h r é iic r g ic de
variées. L énergie la p lu s prohahU' d é sin té g ra tio n Q p a rc e q u e le n e u trin o e n ijx irte a p p ro x iin a tiv rm c n t u n e é iu 'ig ic n u lle
d'un positron émis est environ q u an d le p o sitro n e m p o rte ; c ’est-à-ilire
0,15 McV.
0 = fi„ (1 2 .2 7 )

Le neutrino
W o lfg a n g P a u li fu t le p re m ie r à s o u p ç o n n e r l'e x is te n c e d e s n e u trin o s , e n 1930
N o n se u le m e m so n h y p o th èse ren d a it-e lle p o ssib le u n e c o m p ré h e n sio n d e la rép artitio n
d e l’é n e rg ie p a rm i les é le c tro n s e t le s p o sitro n s d a n s la d é sin té g ra tio n b ê ta , m a is e lle
p e rm e tta it ég a le m e n t d e réso u d re un au tre c a ssc -tc tc d e la d é sin lé g ra lio n b eta, celu i q ui
c o n c e rn a it le m o m e n t c in é tiq u e « m a n q u a n t» .
L e n eu trin o est u n e pat ticu lc v raim en t insaisissab le ; on a calcu lé q u e le libre parcours
m o y e n d ’u n n e u trin o é n e rg iq u e d a n s l'e a u n ’e s t p a s iiifc ric u i à p lu s ie u rs m illie rs
d ’am iées lu m ière. P a r ailleu rs, les n e u trin o s lib érés p a r le B ig B a n g , qui a vi aisem b la-
b le m e n i p ro v o q u é la c réatio n d e l’U n iv ers, so n t les p a rtic u le s p h y siq u e s le,s plu.s ab o n
d an tes. D es m illiard s d ’en tre eu x trav ersen t nos coq.» ch aq u e seco n d e, n ’y laissant au cu n e
trace.
M a lg ré le u r c a ra c tè re in sa isissa b le , d e s n e u trin o s o n t etc détccté.s e n la b o ra to ire
C e fu t e ffe c tu é p o u r la p re m iè re fo is en 1953 p a r F. R e in e s et C l . C o w a n à l ’aid e de
n e u trin o s g é n é ré s d a n s u n ré a c te u r n u c lé a ire à h a u te p u is s a n c e . (L n 1 9 9 5 . Rcincs,
m em b re su rv iv an t d e l’équijie, a reçu le jirix N obel |v m r son travail.) B ien q u e la détection
d es n e u trin o s c a u se d es d iffic u lté s, l'é lu d e d e c e s p artic u le s est a u jo u rd 'h u i u n e b ran t hc
b ie n d é v e lo p p é e d e la p h y .siq u e e x p é rim e n ta le , d o n t fo n t p a rtie d e s c h e rc h e u rs
c h e v ro n n é s d an s d e n o m b re u x lab o ra to ire s p a rto u t d an s le m o n d e. L e C a n a d a p o ssè d e
u n d es p rin cip au x lab o rato ires v o u és à l’étu d e et à la d étectio n dos n eu trin o s L e Siulbm y
N e u trin o O h se rv a to ry (S N O ) e s t s itu é d a n s u n e a n c ie n n e m in e , à u n e p ro fo n d e u r
d e 2 k m . p rès d e .Sudbui y. e n O n tario .
L e Soleil ém et én o rm ém en t de n eu trin o s grâce a la chaudière nucléaire que co n stitu e
so n c œ u r e t, la n u it, c e s m e ssa g e rs d u c e n tre d u S o le il p a rv ie n n e n t ju s q u 'à v o u s pui
en dcs.sous. la T e rre ctan i p o u r eu x pre-squr p a rfa ite m e n t tra n sp a re n te . F n fé v rie r 19S7.
la lu m ière d ’u n e é to ile en e x p lo sio n (u n e s u jx ,T n n v u ) d a n s ic i irancl N u ag e d e M ag ellan
figure 12.11 Une explosion de neutrinos
(u n e g a la x ie v o is in e ) a a tte in t la T e rre a p rè s a v o ir v o y a g é d u ra n l 170 OflO an s. Dc>
par la supernova SN I987A. qui s'est
qu an tités én o rm es d e n eu trin o s o n t été g én érées d an s c e n e ex p lo sio n , en v iro n 10 d 'e n tre
produite à un temps (relatif) 0. se détache eu x o n t été ca p té s p a r un d é te c te u r d e n e u trin o s sen sib le situ e au J a p o n ; la fig u re 12 11
de Vém ission h abiitielle. (Dans le cas illu stre un e n reg istrem en t d e leu r p assag e.
de neutrinos. 10 rcpiésenle
une «cxplo.sion».) Les particules La radioactivité et le tableau des nucléides
ont été détectées par un détecteur
sophistique installé profondément
O n p eu t a u g m e n te r l’in fo rm a tio n d o n n é e d an s le ta b le a u d e nucléidCS d e la fig u re 12.4
dans une mine du .lapon. La supernova e n n ip rc se n ia n t g rap h iq u cm cn i l’cxcL s d e m a s s e d e c h aq u e n u clcid c d an s u n e d ire c tio n
n ’était visible que dans l’hémisphère p e rp e n d ic u la ire au p lan d e s N-Z. L 'e x c è s d e m a sse d u n n u c lè id e e s t (o n d ep ii d e son
sud, de sorte que les neutrinos ont dû n o m ) u n e én erg ie q u i e st u n e a p p ro x im atitm de l’c n c rg ic d e liaiso n totale d e cc n u eléid e
pénétrer la Terre (lianicie insignifiante Il e st d efin i pai- (m - / t ) r o ù m est la m asse a to m iq u e d u n u eléid e e t A e st son n o m b re
pour eux) pour atteindre le détecteur. d e m a sse , to u s d e u x e x p rim é s e n u n ités d e m asse ato m iq u e , e t csi 9 3 1,5 M c’V/u
348 Chapitre I2 La physique nucléaire

Figure 12.12 Une partie de la vallée


de nucléides ne montrant que
les nucléides de faibles masses.
Le deutérium, le tritium et l’hélium
SC trouvent à restrém itc rapprochée
du giaphique. alors que l’hélium
se situe au point élevé. La vallée
s’étiie en .s’éloignant de l’ob-servateur,
le graphique s’arrêUuil veis Z 22
cl N — 3.S. Le.s nucléides affichant
de grandes valeurs de A , qui .se
trouveraient bien au-delà de la vaUée.
peuvent se dé.sintégrcr dans la vallée
par cmi.vsions alpha répétées
et par fission (division d’un nudéidef

L a .su rface a in s i fo rm é e d o n n e u n e re p ré s e n ta tio n g ra p h iq u e d e la s ta b ilité


n u cléaire. C o m m e le m o n tre la fig u re 12.12 (d a n s le cas d e s n u c lé id e s d e faib le m as.se),
c e n e su rfa c e p ren d la fo rm e d ’u n e « v allée d e n u c lé id e s» d o n t le fo n d e st c o n stitu é d e
la b an d e d e sta b ilité d e la fig u re 12 .4 . L es n u c lé id e s .se tro u v a n t d u c ô té ric h e en p ro to n s
d e la v a llé e se d é sin tè g re n t v e rs le b a s d e c e lle -c i en é m e tta n t d es p o .sitro n s; ceuK d u
cô té rich e e n n e u tro n s le fo n t en é m e tta n t d es électro n s.
/V ÉR IFIEZ VOS CONNAISSANCES 3 ! 1^ nudéide ■'’"‘U .se dé,sintègrc en par l’émission
d’une particule alpha. S’ensuit une chaîne de désintégrations radioactives, soit alpha, soit
beta. Finalement, ces désintégrations atteignent un nudéide stable et plus aucune autre
(Jesintegration n’e.st passible. Lequel des nucléides stables suivants est le produit final
de la chaîne de désintégrations radioactives du : ■'’''Pb, -'”Pb, ^”*Pb ou -"’'P b ’.'
{¡ndice.: v o u s p o u v e z le d é te r m in e r e n te n a n t c o m p te d e s v a ria tio n s du n o m b re d e m a s s e A
¿LS,S(.)ciccs a u x d e u x ty p e s d e tlé sin lc g ta iin n s .)

Exemple 12.7
Calculez l’énergie de désintégration Q de la désintégration Am - (a if i- 16/»,;) — (m p t ISm^).
bêta moins du '-P, telle qu’elle est définie par l’équation 12.23.
Les quantités entre parenthèses sont les masses atomiques du
Les masses atomiques iiét-e.ssaircs sont 3I.')73 91 u pour le ’’P
et du ’’P, donc
61.11.972 07 U pour le ''-S.
Am —
SOUJnON: Le concept clé utili.sé ici est que l’énergie de désintégration Q
de la dé.simégration bêta est la variation de l’cnergie au repo.s causée On constate alors que. si on soustrait seulement les masses atomiques,
par la désintégration. La valeui de est donnée par l'équation 8.49 la mas.se de l’électron émis entre aiilomatiqnement en jeu. (Cette
(Q ~ —A M C-). Cependant, étant dc'nncqu’un électron individuel méthode ne lonctionncra pas dans le cas de l’émission d'un positron.!
est émis (et non un électron lié dans les couches de l’atome), il faut L’énergie de la désintégration du ^-P est alors
bien distinguer masses nucléaires iqu’on ne connaît pas) et masses Q ------ A m
atomiques (qu’on connaît). On représente les ma.sses nucléaires
-----(31.972 07 U - 31,973 91 u)(93L5 MeV/u)
du *-P et du ■’^S par les symboles en gras m.. et m s. et leurs mas.ses
atomiques, par les symboles en italique irtf, et Ws. On peut alors noter 1,71 McV. (réponse)
la v a r ia tio n de masse associée à la désintégration exprimée par fc.xpérimcntalemeni, la quantité calculée ici est égale à K,„,„ l'énergie
IVqiiaiion 12 23 ainsi. maximale que peuvent avoir les électrons émis. Bien que 1.71 McV
.soit émis chaque foi.s qu’un noyau de '-P .se dé,sintègre, il apparaît
Am — m«' m ,) m p.
que. dans pratiquement tous les cas. l’électron emporte une énergie
o ù iHp e s t Id mds.sc lie r é U x i i o n Si on a d d iiio n n e e t s o u s tra it I moindre. l.e neutnno obtient tout le reste de l’éneigic. l’emportant
Clans le m e m b re d e d ro ite , o n o b tie n t hors du laboratoire.
12 6 Lo datation par radioactivité 349

12.6 La datation par radioactivité


S i v o u s c ttn n a issc z la d e m i-v ie d 'u n ra d io n u c lé id e d o n n é , v o u s p o u v e z , e n p rin c ip e ,
u tilis e r la d é s in té g ra tio n d e c e t é lé m e n t p o u r m e s u re r d e s in te rv a lle s d e te m p s.
P a r e x e m p le , la d é sin té g ra tio n d e n u c lé id e s à très lo n g u e v ie p eu t s e rv ir à m e su re r l’âg e
d e s ro c h e s , c ’e s t-à -d ire le te m p s é c o u lé d e p u is le u r fo rm a tio n . D e te lle s m e s u re s
d e l’â g e d e ro c h e s p ro v e n a n t d e la T e rre , d e la L u n e e t d e m é té o rite s ré v è le n t u n â g e
m ax im al c o n sta n t d 'e n v iro n 4 ,5 x 10® a.
P a r e x e m p le , le ra d io n u c lé id e '*'^K se d é sin tè g re e n *®Ar, iso to p e siah ie d e l’arg o n ,
u n g a z n o b le. L a d em i-v ie d e c e tte dé.sintégralion e st 1,25 x 10‘^ a. l’o u r d é te rm in e r l’âge
d ’u n e ro c h e , o n p eu t d é te rm in e r le ra p p o rt e n tre le s n u c lé id e s d u K e t d u '*”A r q u ’il
p résen te. O n p eu t u lilistir d ’au tres d ésin té g ra tio n s d e lo n g u e d u ré e , c o m m e c e lle d u '
e n ’*'’’P h (m e tta n t e n je u u n c e rta in n o m b re d ’e ta p e s in te rm é d ia ire s ), p o u r v é rifie r
ce calcu l.
P o u r m e s u re r d e s in te rv a lle s p lu s c o u rts , à l’é c h e lle h is to riq u e , la d a ta tio n p a r
le cartH ine 14 s 'e s t a v é ré e in e stim a b le . L e ra d io n u c lé id e (d o n t 7 ,^ = .“i /Mi a) est
p ro d u it à un tau x c o n sta n t d a n s les c o u c h e s su p é rie u re s d e ra tm o s[7 h ère q u an d l’azo te
atm o .sp h ériq u e est b o m b ard é p a r les ra y o a s co sm iq u e s. C e rad iu c u rb o n e .st m é la n g e au
c a rb o n e n o rm a le m e n t pré.sent d a n s l ’atm o sp h è re (so u s fo rm e d e ( '<),), d e so rte q u 'il y a
e n v iro n u n a to m e d e ' “C p o u r 10'^ a to m e s d e ‘^C sta b le s D e s a c tiv ité s b io lo g iq u e s
co m m e la p h o to sy n th è se e t la resp iratio n fo n t q u e les a to m e s de carb o n e a lm o sp h é n q u c
c h a n g e n t d e p lace aléa to ire m e n t, un à la fo is, av ec les ato m es d e carb o n e p ré se n ts d an s
c h aq u e ê tre v iv a n t, in clu an t le b ro c o li, les c h a m p ig n o n s, le s p iiig o u in s et les hiim aiii.s
C et é c h a n g e finit p a r a tte in d re une siiu a tio n d 'é q u ilib re o ù r c n s c m b le d e s a lo m c s de
carb o n e c o n te n u s d an s c h a q u e ê tre v iv an t c o m p te u n e p etite fractio n fix e d e n u c lé id e s
’"‘C rad io actifs.
C et éq u ilib re p ersiste aussi lo n g tem p s q u e l’o rg an ism e vit. Q u a n d il m eu rt, l’cch an g c
avec l’atm osphèR ’ cesse et, étani tlonné q u e le rad io carb o n c em jx iso m ié d an s l’o rg an isn x '
n ’est p as rem placé, sa quantité d im in u e en fonction d ’une dem i-vie d e 5 730 a. F.n m esurant
la q u an tité d e rad io carb o n c iiar g ram m e d e m atière o rg an iq u e, o n peut d é term in er le iein|)s
q u i s ’e st écm ilé d e p u is la m o rt d e l’o rg an ism e. L e c h a rb o n de b o is dc.s fe u x d e cam p , les
U n fragm ent des m anuscrits de la m er m a n u sc rits d e la m e r M o rte e t de n o m b reu x a rte fa c ts p réh isto riq u es o n t é té d ates ainsi.
M orte et les grottes où ils ont été L 'â g e d e s m a n u sc rits a é té d é te rm in é au m o y en d e la d atatio n , p a r le c a rb o n e 14. d ’un
trouvés é c h a n tillo n d u lin g e utili.sé p o u r b o u c h e r les v ases d a n s le sq u e ls ils é ta ie n t sc ellés

Exemple 12.8
La spectrométrie de masse des atomes de potassium et d'argon Étant donné qu'on ne peut mesurer on réliniine des éqiiaiiiiiis
conicnus dans une nxtie de la I^une révèle que le rappon entre le iKimhre 12.28 et 12.2d. Après avoir fait un jx-u d'algchrc. ou iibiïciii
d'aloine.s de ‘“'Ai (stables) et le nombre d’atomes de “’K (radioactifs)
est. dans cette roche, de 10,3. Supposez que nais les atomes d’argon
aient été produits par la désintégration des a to m e s de |X )tassium . avec A/ ( 12.3U)
une demi vie de 1,25 x U)'' a Quel e s t l'âge de ccitc pierre?
où p e u ! être mesuré. .Si on is<>lt* f r i si on fait «inxl h
SOlUTION. Le concept dé uiilisé ici est le suivant, si N„ atomes de
l’équation 13 17 pour remplacer À par (In un oliticni
potassium étaient présents au moment de la formation de la pierre
par solidification d'un e matière en fusion, te nombre d’atomes
de potassium résidue.l.s au moment de l’analyse est, selon l'éq u a­
^ _ 7'i/2ln(l +
tion 12.14, ln2
(1.25 l()''u)|ln(l ) lü.J)]
a
A/k = (12.28) Iii3
où rest l'âge de la roche Chaque lois qu’un atome de potassium 4,.37 X 10^ a. (réponse)
SC désintègre, il y a formation d’un atome d'argon Donc, te nombre
d’atomes d’argon présents au moment de l’analyse est En ulilLsani d’annes éidiantillons de niches liiriain S ou trrrestrt'S, Oli
peut mesurer des âges inférieurs, mais non supérieurs. Donc, le sy»
A'o - A^K- (I2,2‘>) icmc solaire doit avoir environ quatre milliards et demi d ’années.
350 Chapitre 12 U i physique nucléaire

12.7 Mesurer la dose de rayonnements


ionisants
L ’a c tio n d e s ra y o n n e m e n ts io n isa n ts, c o m m e les ra y o n s g a m m a , le s é le c tro n s e t les
p a rtic u le s a lp h a s u r le tissu v iv an t (n o ta m m e n t le n ô tre ) est un su je t d ’in té rê t p u b lic .
D an s la n a tu re , d e te ls ra y o n n e m e n ts io n isa n ts p ro v ie n n e n t d e l’e sp a c e et d ’é lé m e n ts
ra d io a c tifs p ré s e n ts d a n s l’é c o rc e te rre s tre . O n tro u v e é g a le m e n t d e s ra y o n n e m e n ts
io n is a n ts a s s o c ié s à c e rta in e s a c tiv ité s h u m a in e s , c o m m e l ' u tilis a tio n d e ra y o n s X
et d e s rad io n u c lé id e s en m éd ecin e et d an s l’in d a strie .
O n n ’e x p lo re ra p a s ici les d iv e rs e s so u rc e s d e ra y o n n e m e n ts io n is a n ts ; o n se
c o n te n te ra sim p le m e n t d e d é c rire les u n ités u tilisé e s pi:>ur e x p rim e r le u rs p ro p rié té s e t
leu r actio n . O n c o n n a ît d é jà Vactivit^^ d ’u n e so u rce rad io a c tiv e . O n v e rra ici d eu x au tres
d im e n sio n s du p h én o m èn e.
1, D o se absorbée. M e su re d e la d o se d e ra y o n n e m e n ts io n isa n ts (é n e rg ie p a r u n ité d e
m a sse ) ré e lle m e n t ab so rb ée p a r u n o b jet particu lier, c o m m e u n e m ain o u la p o itrin e
d ’u n p atien t. S o n u n ité S I e st le g r a y (G y). 1 e r a d (d e l’a n g lais n u lia tio n a b so rb ed
d o se ) e st u n e u n ité p lu s a n c ie n n e to u jo u rs c o u ra m m e n t utili.sée. C e s te rm e s s o n t
d é fin is d e la faço n su iv a n te :
1 G y = I J/k g = 100 rad. (1 2 .3 1 )
V oici u n é n o n c é ty p e s u r la d o se ab so rb ée : « U n e a b so rp tio n d e 3 G y ( = 3(K) ra d ) de
ra y o n s g a m m a à c o u rt te rm e c a u s e ra la m o rt d e 50 % d e la p o p u la tio n e x p o sé e . »
H e u re u se m e n t, la d o se a n n u e lle m o y e n n e q u ’on a b so rb e a c tu e lle m e n t, d e so u rc e s
n atu re lle s e t artific ie lle s, n ’e st q u e d e 2 m G y ( — 0 .2 rad).
2. É q u iva len t d e do se. B ie n q u e d iffé re n ts ty p e s d e ra y o n n e m e n ts io n is a n ts (ra y o n s
g a m m a e t n e u tro n s , p a r e x e m p le ) p u is s e n t fo u rn ir la m ê m e q u a n tité d ’é n e rg ie
a u c o rp s, ils n 'o n t p a s la m e m e a c tio n b io lo g iq u e . L ’é q u iv a le n t d e d o s e p e rm e t à
r e x p é r im e n ta te u r d e d é c rire l’a c tio n b io lo g iq u e en m u ltip lia n t la d o s e a b so rb é e
(en g ra y s ou en rad s) p a r un fac te u r d e q u a lité Q (à n e p a s c o n fo n d re av ec l’én erg ie
d e d é s in té g ra tio n ), sa n s d im e n s io n . P a r e x e m p le , d a n s le c a s d e s ra y o n s X e t d e s
é le c tro n s, Q = \ '. d a n s le c a s d e s n e u tro n s le n ts, Q — 5 -, d a n s le c a s d e s p a rtic u le s
a lp h a . Q ~ 10. L e s a p p a re ils d e m e su re in d iv id u e ls , c o m m e le d o s im è tre
p h o to g rap liiq u e |ic rso n n e l, e n re g istre n t l’é q u iv a le n t d e d o se.
L ’u n ité S I d e l’é q u iv a le n t d e do.se e st le s ie v e r t (S v ). I.e r e m , u n e a n c ie n n e
u n ité, e st to u jo u rs c o u ra m m e n t u tilisé . L e u r relatio n est
I S v = 100 rem . (1 2 .3 2 )
V oici un ex em p le d ’une b o n n e u tilisatio n d e c e s term es ; « S elo n u n e reco m m an d atio n
d u N a tio n a l C o u n c il o n R a d ia tio n P ro te c tio n (C o n se il n a tio n a l |X )ur la p ro te c tio n
c o n tre le s ra y o n n e m e n ts io n is a n ts ), u n o rg an L sm e a m é ric a in , a u c u n e p e rs o n n e
e x p o sé e (d a n s u n c o n te x te n o n p ro fe ssio n n e l) a u x ra y o n n e m e n ts ioni.sants ne d o it
re c e v o ir un é q u iv a le n t d e d o se s u p é rie u r à .S m S v ( = ()„'> re m ) e n u n e a n n é e . »
C e la c o m p re n d les ra y o n n e m e n ts io n isan ts d e to u s les ty p es ; b ien sûr, il laul u tiliser
le fa c te u r d e q u a lité a p p ro p rie à c h a q u e type.

Exemple 12.9
( )ii a vu qii’urK- dose ilr 3 fïy de rayons gamma est mortelle pour la On peut supposer que c, la ci^ e itc themiique massique du eoips humain,
iiioiiié des gens qui y .sont exposes. ,Si une énergie ériuivalcntc était est la niéiiic que celle de l’eau, soit 4 180 J/(kg ■K). On obtient alors
iihsoiliée sous lormc de clialcui, de combien la température du coqw
augmenterait elle? M = i ^ ^ _ 3 J / k g ^,2x10 ’ K;
r 4 180J/(kg K)
(réponse)
SOlUTION; Le pn-mier ronrupt clé utilisé ici est le suivant; l'énergie
ali.soibée Ç est reliée à l'augm entation de température A7 qui en Il est évident que les dommages provoqués par le.s rayonnements
léxulir par réqnation g nir A T Uans cette équation, m est la masse ionisants n om rien à voir avec^eux causés par la chaleur. Les effets
lie l’objet abxnibant l’énergie et c e.st la capacité tticrmiqiie massique dangereux proviennent du fait que les rayonnements endommagent
de col objet. Le second concept de est qu’une dose absorbée de ,3 Cîy l’ADN et interfèn-ni avec le fonctionncincnt normal des tissus qui les
correspond à une énergie absorbée par unité de masse de .3 J/kg. absorbent.
12.8 Les modèles nucléaires 351

12.8 Les modèles nucléaires


L e s n o y a u x so n t p lu s c o m p le x e s q u e le s a to m e s. D a n s le c a s d e s d ’a to m e s, la fo rc e
re sp o n sa b le de la d y n a m iq u e e sl la fo rce é le c triq u e d e C o u lo m b . C e lle fo rce est donntSc
pai' u n e e x p re ssio n sim p le e i il e x iste u n c e n tre d e fo rce n atu rel, le n o y a u . D a n s le cas
d e s n o y a u x , p a r c o n tre , la fo rc e n u c lé a ire e s l c o m p le x e e l ne p e u t en fa it, ê tre é c rite
e x p lic ite m e n t d a n s to u s ses d é ta ils. D e p lu s, le n o y a u fc o n stilu c d e p lu sie u rs p ro to n s
e t d e p lu s ie u rs n e u tro n s ) n e p o s s è d e p a s d e c e n tre d e fo rc e n a tu re l p e rm e tta n t de
sim p lifie r le s calcu ls.
L n l ’a b se n c e d ’u n e th éo rie n u c lé a ire c o n ip lè ie , on se lo u rn c v o is la in n c c p ilo n
d e m odèles n u c lé a ire s. U n m tx lè le n u c lé a ire p e rm e l sim p le m e n t d ’o b s e rv e r le n o y a u
et d ’im a g in e r le p lu s g ra n d n o m b re p o ssib le d e se s p ro p rié té s p h y siq u e s. O n m e su re
r u ü lité d ’un m o d è le piU" sa ca p a c ité de. p e rm e ttre d e s p ré d ic tio n s p o u v an t ê tre véi ifiées
e x p érim en talem en t
D eux m odèles du noyau se .sont avérés utik.s. flicn q u ’ils sem blent s’cxclurc l’un rau tro .
c h acu n p e rm e t d e trè s b ie n e x p liq u e r un g ro u p e d e p ro p ric te s n u c lé a ire s sp é c ifiq u e s.
A près les av o ir d écrits sép arém en t, o n v e rra c o m m en t o n p e u t co m b in er ces deux modèles
p o u r fo rm e r u n e se u le im ag e co h é re n te d u n o y au ato m iq u e.
Le modèle collectif
D a n s le m odèle collectif, é la b o ré p a r N iels B o h r, o n im a g in e q u e les n u c lé o n s, q ui se
d é p la c e n t au h a sa rd d a n s le n o y au , in te ra g isse n t fo rte m e n t e n tre e u x . to u t c o n u n c le s
m o lé c u le s c o n te n u e s d an s une g o u tte de liq u id e. L n n ucléon d o n n é h c u ite fré q u e m m e n t
d 'a u tr e s n u c lé o n s à l'in té rie u r d u n o y a u , so n lib re p a rc o u rs m o y e n é ta n t s u b sta n tie l
lem en i in fé rie u r au ray o n n u cléaire.
I .e m o d èle c o lle c tif p e rm e t d ’éta b lir u n e co rrélatio n en tre d e n o m b reu x faits asso ciés
au x m a sse s n u cléaires e t aux én erg ie s d e liaiso n ; il e st u tile (co m m e v o u s le v e rre z plii.s
lo in ) p o u r e x p liq u e r la fissio n n u c lé a ire . Il e sl é g a le m e n t u tile p o u r c o m p re n d re un
g ran d n o m b re d e ré a c tio n s n u cléaires.
S o it u n e ré a c tio n n u c lé a ire q u e lc o n q u e , e x p rim é e so u s la fo rm e

A I a Y h h. 112.33)
O n im a g in e q u e le p ro je c tile a e n tre d an s le n o y au c ib le X, fo rm a n t u n n o y a u c o m p o sé
C e t lui d o n n an t u n e c e rta in e q u an tité d ’é n e rg ie d ’ex cita tio n L e p ro jectile, p eu t-ê tre un
n e u tro n , est im m é d ia te m e n t c a p tu ré p a r le s m o u v e m e n ts a lé a to ire s q u i cai a etéi Lseni
l ’in té rie u r d u n o y a u . Il p e rd ra p id e m e n t so n id e n tité (p o u r a in s i d ire ) ei l'é n e r g ie
d ’e x c ita tio n q u ’il a p p o rte d a n s le n o y a u e s t ra p id e m e n t p a rta g é e e n tre to u s le.s auu-es
n u clé o n s p ré se n ts d a n s C.
I .’étal m étastab le repré.senté p a r C d a n s l’é q u a tio n 12..33 (leut a v o ir u n e d u rée d e vie
m o y en n e d e 10“ ''’ s av an t d e se d é sin té g re r en Y e t en b . À r é c h c llc n u cléaire, c ’c.sl irèii
lo n g , so it e n v iro n u n milli<in de fin.s le tem p s q u 'il lau l à u n nuclcisn a y a n t une cn c rg ie
d e q u e lq u e s m illio n s d ’c lc c tro n v o lts p o u r tra v e rse r un n oyau
L ’é lé m e n t c e n tra l d e ce c o n c e p t e sl q u e la fo rm a tio n d u n o y a u c o m p o s é el sa
d é sin té g ra tio n su b séq iien le so n t d e s é v é n e m e n ts p arfaitein en i iiidéjxzluiants. A l'in stiin l
d e sa d é sin té g ra tio n , le n o y a u c o m p o sé a « o u b lié » la fayon d o n t il a été lo n n é . A insi,
so n m<xle d e d ésin tég ratio n n ’e,sl p;is in flu en cé p a r son m o d e d e to rm a lio n . P ai ex em p le,
la fig u re 1 2 .13 illu stre tro is lyp»\s de fo rm atio n po.ssible.s d u n o y au c o m p o sé ’"N e cl tro is
ty p e s d e d é sin té g ra tio n p o ssib le s. C h a c u n d es tro is modems d e Id rm a lio n p eu t c o n d u ire
'"O + a __ ■'‘F + ' ” à l’un o u l’au tre d e s tro is m o d es d e d i'sin lcg ralio n
'’«N,. Liiii^ '«Nr 1 II
Le modèle en couches
•’() J- ‘Hc D a n s le m o d èle c o lle c tif, o n su p p o se q u e les nuclé<ins se- d ép lacen t aléatiM ienieni ci .v
Tinis 'IVobi h e u rte n t m u tu e lle m e n t, e t c e , fré q u c m m rn l I <■ im nlèle en r o w /le». q u a n t <3 lui. e st ba-^»'-
T||fv|»*\ su r la su p p o sitio n op|x>sée, à sa v o ir q u e chaque n u cléo n d em eu re d an s u n état q u an tiq u c
rie tOmialion <1^ r1^«infrgr;uic»n b ien d é fin i à l’in té rie u r d u n o y au e t n e su b it a u c u n e c o llisio n . L e n o y a u , c o n trn irem o n t
Rgint 12.13 I es nuxlcs de fonixition à l ’a to m e , n e p o s s è d e a u c u n c e n tre d e c h a rg e f ix e ; d a n s eu m o d è le , o n s u p p o se q u e
et les m odes dcsintépraiion du noyau c h a q u e n u c lé o n se d é p la c e d a n s un p u its d e p o te n iie l d é te rn iin é pai tes iiio u v em eiiis
(m o y e n s) d e to u s les au tres n u i léons.
com posé
352 Chapitre 12 La physique nucléaire

U n n u c lé o n se tio u v a n t d a n s u n n o y au , to u t co iim ie u n é le c tro n d a n s un a to m e , est


a sso c ié à u n e n se m b le d e n o m b re s q u a n tiq u e s q ui d é c riv e n t l ’é ta t d e so n m o u v e m e n t.
D e p lu s, les n u c lé o n s o b é isse n t au p rin cip e d ’ex c lu sio n d e P a u li, to u t c o m m e le fo n t les
é le c tro n s : d e u x n u c lé o n s c o n te n u s d a n s un n o y a u n e p e u v e n t o c c u p e r le m ê m e é ta t
q u a n tiq u e a u m ê m e in sta n t. A in si, le s n e u tro n s et les p ro to n s so n t tra ité s sé p a ré m e n t,
c h aq u e ty p e d e p a rtic u le p o ssé d a n t son p ro p re e n se m b le d ’états q u an tiq u es.
L e tiüt que les nucléons o b éissen t au frin c ip c d ’ex clu sio n d e Pauli p erm et d ’e x p liq u er
la stab ilité relativ e d e s états d es n u cléo n s. P o u r q u e d eu x nucléon.s co n ten u s d ans le n o y au
subi.s.sent u n e co llisio n , l’én erg ie d e ch acu n après la co llisio n d o it co rresp o n d re à l’énergie
d ’un é ta t iiuiccupé. Si au cu n é tal d e la so rte n ’est d isp o n ib le , la co llisio n n e jxîut sim p le ­
m e n t p as se pro d u ire. D o n c, to u t n u clé o n a sso c ié à d e s « o c c a sio n s ra té e s d e c o llisio n s»
m aintiendra l’étal d e son m ou veinent suffisatnincnt lo n g tem p s ptxir d o n n er une signification
à l'é n o n c é qui d it q u ’il e x iste d a n s u n é ta t q u an tiq u e e t pos.sède u n e én e rg ie bien d efin ie.
D an s le d o m a in e a to m iq u e , les ré p é titio n s d e s p ro p rié té s p h y siq u e s e t c h im iq u e s
q u ’o n tro u v e d a n s le ta b le a u p é rio d iq u e .sont a sso c ié e s à u n e p ro p rié té d e s é le c tro n s
a to m iq u e s, so it c e lle q u i fait q u e c e s é le c tro n s se ré p a rtisse n t en c o u c h e s affic h a n t une
plus g ran d e stab ilité q u an d elles so n t rem plies. O n p e u t co n sid é re r les n u m éro s ato m iq u es
d e g a z n o b les
2, 10. 18. .36. .54. 8 6 ,...
c o m m e d e s nom bres m agiques d'électrons q ui m a rq u e n t le rem p lissag e (o u la ferm etu re)
d e telles co u ch es.
I x s n o y a u x p ré s e n te n t é g a le m e n t d e te ls e ffe ts d e c o u c h e s fe rm é e s a s s o c ié s à
c e rta in s n o m b re s m a g iq u e s d e n u c lé o a s ;
2, 8 ,2 0 ,2 8 ,5 0 , 8 2 . 1 2 6 ,...
T out n u cléid e d o n t le n o m b re d e p ro to n s 7. o u le n o m b re de n e u tro n s N affic h e l'u n e de
c e s v aleu rs p o ssè d e u n e trè s g ra n d e sta b ilité p o u v a n t .se tra d u ire d e d iv e rse s m an ières.
V oici d es e x e m p le s d e n u c lé id e s « m a g iq u e s » : '**0 (Z = 8 ), '**’C a (Z = 2 0 , A = 20),
'•M o (N = 5 0 ) CI -"'*Pb (Z = 8 2 , W = 126). L e s n u c lé id e s -«>Ca e t ^^'^'Pb p e u v e n t to u s
d e u x ê tre q u a lifié s d e « d o u b le m e n t m a g iq u e s » p a rc e q u ’ils c o n tie n n e n t à la fo is d es
c o u c h e s d e p ro to n s re m p lie s e t d es c o u c h e s d e n eu tro n s rem p lies.
L e n o m b re m a g iq u e 2 a p p a ra ît d an s la stab ilité e x c e p tio n n e lle d e la p a rtic u le alp h a
(^H e), q u i est d o u b le m e n t m ag iq u e, c a r Z = A = 2. P a r ex em p le, su r la co u rb e d ’én erg ie
d e liaiso n d e la fig u re 12.6, l’én e rg ie d e liaiso n p a r n u cléo n d e c e n u c lé id e se situ e bien
a u -d e ssu s d e c e lle d e l ’h y d ro g è n e , d u lith iu m e t d u b é ry lliu m , .scs voi.sins d u ta b le a u
p é rio d iq u e . E n fa it, la p a rtic u le a lp h a e st si fo rte m e n t lié e q u ’il e st im p o s s ib le d ’y
ajfMJtcr u n e autre p articu le ; il n ’y a au cu n n u cléid e stab le d o n t A = 5.
L a n o tio n d e c o u c h e fe rm é e la isse p e n se r q u ’u n e p a rtic u le seu le se tro u v a n t h o rs
d ’u n e c o u c h e fe rm é e s ’e x tra it re la tiv e m e n t fa c ile m e n t, ta n d is q u ’il fa u t dépen.ser une
é n e rg ie c o n sid é ra b le p o u r e x tra ire u n e p a rtic u le c o n te n u e d a a s la c o u c h e e lle -m ê m e .
D an s r a lo m c d e so d iu m , p a r e x e m p le , o n tro u v e u n é le c tro n (d e v a le n c e ) h o rs d ’une
c o u c h e é le e tn m iq u e ferm ée. Il ne fau t q u e 5 eV p o u r e x tra ire cet é le c tro n ; c e p e n d a n t,
p o u r e x tra ire u n deuxièm e é le c tro n (q u i d o it être a rra c h é à u n e c o u c h e fe rm é e ), il fau t
2 2 eV . D a n s le d o m a in e n u c lé a ire , o n p e u t c o n s id é re r l’é lé m e n t ‘^ 'S b (Z = 51 ), q u i
re n fe rm e u n p ro to n u n iq u e h o rs d ’u n e c o u c h e fe rm é e d e 5 0 p ro to n s. P o u r e x tra ire ce
p ro to n s o lita ire , il fa u t 5 .8 M eV ; p o u r e n e x tra ire u n d e u x iè m e , il fa u t u n e é n e rg ie
d e 1 1 M eV . Il exi.ste d e n o m b re u se s a u tre s p re u v e s c x p é rim c n ia lc s q u e le s n u c lé o n s
c o n te n u s d a n s u n n o y au f ilm e n t d es c o u ch es fe rm é e s et q u e c e s c o u c h e s p o ssè d e n t d es
p ro p rié té s d e stab ilité.
O n a vu q u e la th é o rie q u a n tiq u e p eu t c x p lit|u e r le s n o m b re s m a g iq u es d ’é lectro n s,
c ’est-a -d ire les p o p u la tio n s d e s so u s-c o u c h e s d an s lesq u els les é le c tro n s ato m iq u e s sont
reg ro u p és. D e la m ê m e lay o n , d an s c e rta in e s co n d itio n s, la th é o rie q u an tiq u e p eu t tout
a u ssi b ie n e x p liq u e r le s n o m b re s m a g iq u e s d e n u c lé o n s. I jc prix N o b el d e p h y siq u e
d e IQ 63 a é té re m is à M aria M a y e t e t à H a n s Je n s e n « p o u r le u rs d e c o u v e rte s su r la
stru c tu re e n c o u ch es d u n o y a u » .
12.8 Les modèles nucléaires 3S3

Un modèle combiné
O n p eu t c o n sid é re r un n o y au d an s le q u e l un p e tit n o m b re d e n eu tro n s (o u d e p ro to n s)
se tR iuveiit h o rs d ’un c œ u r d e c o u c h e s fcrm cc.s q u i c o n tie n t d e s n o m b re s m a g iq u e s d e
n e u tro n s o u d e p ro to n s. L e s n u clé o n s e x te rn e s o c c u p e n t d es états d ’é n e rg ie q u a n tifié s
d a n s un p u its d e p o te n tie l c ré é p a r le c œ u r c e n tra l, p ré s e rv a n t ain si l'é lé m e n t c e n tra l
d u m o d è le d e s c o u c h e s . C e s n u c lé o n s e x te rn e s in te ra g is s e n t a u s s i a v e c le c œ u r, le
d é fo rm a n t e t y g é n é ra n t d es m o u v e m e n ts « o n d u la to ire s » d e ro ta tio n e t d e v ib ra tio n .
C e s m o u v e m e n ts c o lle c tifs d u c œ u r p ré se rv e n t l ’é lc m c n t cen tra! do c e tnodolo. L^n tel
m o d è le d e stru c tu re n u c lé a ire ré u ssit a in si à c o m b in e r Ic.s p o in ts de v u e ap p a re m n ie m
ir ré c o n c ilia b le s d u m o d è le c o lle c tif e t d u m o d è le en e o u e h e s . il ré u s s it d e fa ç o n
re m a rq u a b le à e x p liq u e r les p ro p riétés n u cléaires o b se rv é e s

Exemple 12.10
Considère/. la réaction suivante, dans laquelle il y a capture d'un neu
iron.

'®^Ag -t- n —» *”’Ag + y , (12.^4)

iluranl laquelle il y a formation d 'u n noyau composé ( '" ’Ag). l.a


figure 12.14 représente graphiquement le taux relatif de production
dii noyau coiniiosé en fonction de l’énergie du neutron incident.
On cherche à déterminer la durée de vie moyenne de ce noyau
composé en faisant appel au principe d'incertitude de Heisenberg
pour l’énergie et le temps (exprimé par l'équation 9.25)

X E At ■= Й. (12.35)

Ici, l i t c.st une mesure de rincertitiide quant à l’énergie d’un état. La RgurelZH Exemple 12.10 Graphique du nombre rolatil de réactions
grandeur Ai c.st une mesure du temps disponible pour mesurer cette du type décrit par l’équation 12.34 en fonction de I énergie du neutron
énergie. En fait, ici, Ai est simplement i^ ,,, la durée de vie moyenne incident I.a demi largeur A£ du pic de résonance est d’environ 0.20 éV.
du noyau composé avant qu’il sc désintègre pour retrouver son état
fondamental.
l’incertitude A£' étant d ’environ 0.20 eV. Donc, l'équation 12.25
SOlUIlON: On constate que le taux relatif de la réaction connaît donne
un pic abrupt quand le neiilron passède une énergie d'environ 5.2 cV.
Cela laisse croire qu’on est en présence d’un unique niveau d’énergie h 1,05 X 1 0 - I s
excité du noyau composé " ”Ag. Quand l’énergie di.sponible (du Xt - U
A£ “ (0 ,2 0 eV )(1 .6 0 x lO -'^T /cV )
neutron incident) correspond à l’énergie de ce niveau supérieur
à l’état fondamental de ""Ag, on est en présence de «résonance», : 3 X 10 ” S (réponse)
cl la réaction de l’équation 12.34 « fonctionne » vraiment.
Cependant, le pic de résonance n’est pas infiniment aigu, mais Ce résultat représente plusieurs centaines tic fois le irintw qu'il faul
a une demi-largeur approximative (A £ dans la figure) d’environ à un ncuirun de 0,20 cV pour parcourir le diamètre d'un noyau de
0,20 eV. Ici, le toficept dé est qu’on peut expliquer cette largeur en disant '"‘^Ag. Par conséquent, durant cet intervalle de 3 x 10 ''' s. le neutron
que le niveau excité ne possède pa.s une énergie parfaitement définie, fa i t en quelque sorte p artie du noyau.

RÉVISION E T R ÉSUM É

îa;s n u c l ê i d e s On connaît environ 2 000 nurléides. Chacun L’éiicrgic nu rcpo.s


L e s é c h a n g e s e n tr e m a s s e e t é n e r g ie
est caractéri.sé par un num éro atom ique Z (nombre de protons), d ’une unité de masse atomique (u) est 931,5 McV. La courbe de
par un nom bre de neutrons N et par un nom bre de m asse A l’énergie de liai.son démontre que les nucléidcs de masse movenne
(nombre total de nucleoas - protons et neutrons). Donc, A = Z + N. «ont les plus stables, et que la fi.ssion d ’un noyau de forte masse ei
Les nurléides ayant le même numéro atomique mais de.s nombres la fusion de deux noyaux de faible ma.sse dégagent de réiiergie
differents de neutrons sont des isotopes d’un élément. Ix s noyaux
Les parties des noyaux sont maintenues
lu t f o r c e n u c lé a ir e
ont un rayon moyen r donné par
ensemble par une force iraiiiaciion qui agit cnüc k s midcons On
r = r o A '' (12..3) croit qu’il s’agit d'un effet secondaire de niitcruction forte agissant
entre les quarks qui torment tes nucléons I .es noyaux pciivcm exister
où r„ œ ] ,2 fm.
dans un certain nombre d’états d’énergie discrets, chacun avant un
moment cinétique et un moment magnétique caractéristiques
334 Chapitre 12 La physique nucléaire

L a d é s in té g r a tu m r a d io a c tiv e La plupart des nucléides Les nucléides radioactifs


L a d a ta tio n p a r r a d io a c tiv ité
connus simi ladioactits ; ils sc désintègrent spontanément. l ’activité présents dans la nature permettent de dater approximativement des
H ( ■ —(IM Jl) rcpicsciite le taux de désintégration, cl est proportion­ éléments historiques et préhistoriques. Par exemple, on peut souvent
nelle au nombre /V d’atomes radioactifs présents, la constante de déterminer l'âge des matières organiques eu inesuraiit leur teneur
proportionnalité étant la constante de désintégration ?.. ce qui mène en ‘"*C; on peut dater des échantillons de roche à l’aide de l’isotope
à la loi de la désintégration ; radioactif
On utili.se trois unités pour
I m m e s u r e d e la d o s e r a d io a c tiv e
N = K = K„e
-A» exprimer l'exposition aux rayonnements ionisants. Le Irccquerel
fia désiiitcgralion radioactive). (12.14, 12.15, 12.16) (1 Bq = l désintégration pat seconde) mesure l'activ ité d ’une
stiurce La quamilé d’énergie vraiment absorbée s'exprime en grays,
1 Gy correspondant à 1 J/kg. L'action biologique estimée de l'énergie
La dcini-vic ^>12 = (In 2)/À d'u n nucléide ladioactif est le temps absorbée est expnméc en sieverts ; une dose équivalant à I Sv provoque
requis pour que l’activité R (ou le nombre/V) d'un échantillon atteigne le même effet biologique, peu importe le ty'pe de rayonnement en jeu.
la moitié de sa valeur initiale.
Selon le modèle collectif de la structure
L e s m o e l l e s n u c lé a ir e s
Certains nucléides sc désintègrent
Im d é s i n t é g r a t i o n a lp h a nucléaire, les nucléons se heurtent continuellement et des noyaux
en émettant une particule alpha (noyau d'hélium , '‘He). Une telle com poses ayant des vies relativement longues sont formés quand
désintégration esl inhibée par une barrière d'énergie potentielle il y a capture d'un projectile. La fonnalion d’un noyau composé cl sa
impénétrable du point de vue de la physique classique, mais qui esl désintégration subséquente sont des événements parfaitement
sujette à l’effet tunnel selon la physique quantique. La perméabilité indépendants.
de la barrière, donc la demi-vie de la désintégration alpha, est très Scion le modèle en couches, chaque nucléon se déplace, es.seniiel-
sensible à l’énergie de la particule alpha émise. Icnicnt sans collision, dans un état quantique à l’intérieur du noyau.
Ce modèle mène aux niveaux de nucléons et aux nombres magiques
L a d é s i n té g r a t io n b ê ta Dans la désintégration héta, un noyau de nucléons (2, 8 , 20, 28, .50, 82 et 126) associés à des couches
(■met un électam ou un positron, qui est accompagné d’un neutrino, tennées ; les nucléides comprenant Pune de ces quantités de neutrons
l es particules émises se partagent l'énergie de désintégration ou de protons sont particulièrement stables.
disponible. Les électrons et les positrons émis dans la désintégration Le modèle combiné, dans lequel les nucléons supplémentaires
bêta possèdent un spectre continu d'énergie s'étalant de presque eém (xicupent des états quantifiés hors d’un caeur central de cout+)cs fermées.
à une énergie limite {= Q - —Am c^). pcniKt de prédire avec succès de nombreuses propriétés nucléaires.

' ^ .a» ♦ V9 ^
Q U E S TIO N S • . .J J V

1. .Supposez que la particule alpha de l’exemple 12.1 est remplacée A/ = Z de la figure 12.4? b) Les deux fragments se .situent-ils
par un proton ayant la même énergie cinétique initiale et fonçant au-dessus ou en dessous de cette ligne? c) Ce.s fragments sont-ils
aussi directement sur le noyau de l’atome d'or. Le proton, compara­ stable.s ou radioactifs ?
tivement à la particule alpha, s’immobiliscra-t-il plus loin, plus près 9 Les radionucléides sc désintègrent de manière exponentielle,
ou à la même distance du centre du noyau ’’ comme dans l’équation 12.15 De fait, les piles, les étoiles et même
2. Votre corps contient-il plus de protons que de neutrons, plus les étudiants se désintègrent; par «désintégrer», on entend ici
de neutrons que de protons, ou approximativement le même nombre «se consumer». Ces cléments se désintègrenl-ils de manière
de protons et de neutrons exponentielle?
3. Le nucléide -‘’’’Pu (Z 94) émet des particulc.s alpha. Lequel des 10. K l - 0, un échantillon du radionucléide A affiche la mciiic
nucléides suivants devient-il après sa désintégration : ’’®Np (7 = 93), activité qu’un échantillon du radionucléide B r = .30 min. Les
(7 = 92), -“'’Cm (Z = 96) ou -““Am (Z = 95)? constantes de désintégration sont X,, et ofi X,, < kg. Les deux
4. Un certain nucléide esl considéré comme étant particulièrement échantillons présenteront ils éventuellement (simultanément) la même
stable. Son énergie de liaison moyenne par nucléon sc situe t elle activité? (In d ic e : représentez, graphiqueiiK-nl leurs activités.)
légèrement au-dessus ou légèrement en rlcs.sous de la courbe d’énergie 11 À f = 0. l’activité d’un échantillon du radionucléide A rcprc.scntc
de liaison de la figure 12.6 ? deux fois celle d ’un échantillon du radinniiciéidc H. l.es cnnst.intc.s
5. L excès de masse d'une particule alpha (utiliser une règle pour de désintégration sont X4 et kg. où X^ > k g . l es deux échantillons
lire la valeur dans le graphtque de la figure 12.12) est-il supérieur ou présenteront-ils éventuellement (simultanément) la même activité
inférieur à l’énergie de liaison totale de la particule (utilisez l'énergie 12 I.a figure 12,15 présente l’activité de trois échantillons radioactifs
de liaison par nucléon donnée dans la figure 12 .6 ) ? en fonction du temps. Classez ces échantillons selon a) leurs demi-vies
6. Le radionucléirle ''"'Ir se désintègre en émettant un électron,
a) Dans quelle ea.se de la figure 12.5 se irouvc-l-il après sa tran.sfor
mation .' b) Subit-il des désintégrations subséquentes?
7. Un nucléide de plomb contient 82 protons, al S’il contenait
également 82 neutrons, ou se situerait-il dans le graphique de
la figure 12.17 h) .Si un tel noyau était forme, émettrait il des
IK)xiimu,v. émettrait-il des électrons ou resterait-il st.able? c) Selon
la ligure I 2 . I, combien de ncutions vous atirndez.-vous à irtiuver.
approxunaiiveiiient, ilans un nuelénie de plomb stable?
8. Le nucléide ^’’’U (Z ^ 92) peut se diviser en deux fragments ayant
des numéros aïomiqiies et des nombres de masses identiques.
n) I c nucléide .sc trouve-i-il au-dessus ou en dessous de la ligne
Exercices et problèmes 355

et b) leurs constantes de dcsintcgiatitHi, en commençant par les valeurs 16. Les radionucléides ’’^At et -'-Po émettent des particules alpha
les plus cicvccs. (In d ic e : en a), utilisez une règle pour trouver la ayant des énergies respectives de 7,83 MeV et de 8,95 MeV. Quel
demi vie à partir du graphique.) nucicidc ptissède la demi-vie la plus longue ?
13. Si la mas.se d ’un échantillon radioactif est doublée, a) l’activité 17. Les nombres magiques associés aux noyaux sont donnes dans la
et h) la constante de désintégration de I’cchanlillon augmentent-elles, section 12.8: 2, 8,20,26,50,82 et 126. Les nucléides sont-ils magiques
iliniinucnt-elles ou restent-elles les memes? (c’e.st-à-dirc particiiliereincnl stables) quand a) seul le nombre
14. À / = 0, on commence à observer deux noyaux radioactifs de masse A est égal à l'un de ces nombres, b) seul le numéro atomi­
idcntiquc.s ayant une demi-vie de 5 min. À f = 1 min, un des noyaux que Z est égal à ruii de ces nombrc,s. c) seul le nombre de neutrons TV
se désintègre. Cet événement augmcntc-t-il ou diminue-t-il la proba­ est égal à l’un de ces nombres ou d) 7 ou Al (ou les deux) sont égaux
bilité que le second noyau se désintègre dans les 4 minutes suivantes, à l’un de ces nom bres? Choisissez toutes les réponses qui sont
ou n'a-t-il aucun vapjmrt avec le second noyau ? correctes.
15. Le radionucléide ^‘’.Sc a une demi-vie de 57,0 min. À / = 0, 18 a) Lesquel.s des nucléifles suivants sont inagiijiies- ' ” Nn ’‘” 5n
il affiche une activité de 6 0(K) eoups/min au dc.ssus de l’activité ‘'*'Cd. ''"'Au. ’™Pb? bl Lesquels, s’il y en u, sont doublement
tie fond générale, qui est de 30 coups/niin. .Sans calculs explicites, magiques ?
ditc.s s'il faudra environ 3 h, 7 h ou 10 h, ou plus de 10 h, pour
que son activité soit approximativement égale à celle de l’activité
de fond.

E X E R C IC tS E T P R O B L È M E ^ ,

9F. Constniisez un tableau de nucléides semblable à celui de la figure


iv w La solution SC trouve sur le site Web, à l’adresse d-dessous ;
12.5 pour les 25 nucléides suivant.s: "*“*“ lc. *'’ '^'Sb,
www.dlcmcgTawhill.ca/
iis-iiyjn, et •'*'Cd. Tracez-y et nommez a) toutes les droites
isobares (Л constante) et b) touie.s les lignes d ’excès de neutrons
constants, définis par /V —Z.
SECTION 12.1 Lo découverte du noyou
lOE. On peut illustrer l’important excès de neiitron.s (defini par N — 7)
lE. Supposez qu’un noyau d ’or possède un rayon de 6.23 fm des noyaux de masse élevée en iioiani que la pluijart des nuclcidcs dc
et qu'une particule alpha a un rayon de 1.80 fm. Quelle énergie une forte masse ne ¡xmi raient jamais se diviser en deux noyaux stables
particule alpha incidente doit-elle posséder pour «toucher» le noyau sans laisser des neutrons dans rnjtératinn On peut prendre comme
d’or scion le type de calcul employé dans l’exemple 12.1 ? exemple la fission spontanée d’un noyau du inirléidr en deux
2E. Calculez la distance minimale séparant une particule alpha de ncyaicxfiliaiion stables ayant îles mm ici as aUmiique.s 39 cl 53. ü) Sclwi
5,30 MeV Cf le noyau d’un atome de cuivre durant une collision l’annexe F, qucl.s .sont les éléments filiation’’ .Scion lu figure 12.4.
frontale. combien y a-t-il b) de neutrons dans les isoUipcs tiliation et c) de neu
3P Quand une particule alpha subit une collision élastique avec un Irons qui sont libérés ?
noyau, ce dernier effectue un mouvement de recul. Supposez qu’une 11E. L’énergie poienliellr élecliiquc d'une sphère homogène de
particule alpha de 5,00 MeV effectue une collision frontale élastique charge g cl de rayon r est
avec un noyau d’or initi.alement au repos. Quelle est l’énergie ciné
tique a'i du noyau qui icculc et b) de la particule alpha qui rebondit? U -
20лг(1/

SEGION 12.2 Cerioines propriétés nudéaites a) DctcnTiincz l’énergie potentielle électrique du nucléidc -'’’'Pu,
qu’on suppose sphérique et ayant un rayon de 6,6 1fm. h) Comparez
4L À l’aide de la diffasion d’électroas. on mesure le rayon d'un noyau l'énergie potentielle élecliique par nucléon, ci aii.ssl l’ciicigîe
S()h6riquc. 3.6 fin. Quel est le nombre de masse probable de ce noyau ? potentielle électrique pur proton de ce nucléidc uvee l'cneigie de
5E. Dans le tableau des nucléides de la figure 12.4, situez les nucléides liaison moyenne par nucléon de 7,so Mp V c) Qu 'en conrluez vous ’
présentes dans le tableau 12.1. Vérifiez .s’ils se trouvent dans la zone 12E. Calculez et comparez al la masse volumique nucléaire p„. et
île .stabilité. h) la densité de charge volumique nucléaire du nuclèide de masse
6L Une étoile à neutrons est un corps céleste dont la masse volumique plutôt Uihie ’■'^Mn et <lii nucleide plutôt massif ’‘“ Bi. 11 1es differriiees
est approximativement celle de la matière nucléaire, telle qu'on l'a sont-elles celles que vous attendiez ? Expliquez. viHrr n')xmsc
c-nlculée dans I exemple 12.2. Suppisezquc le Sàilcil s’effondre sur 13E. Vérifiez, la valeur de l'énergie de liaison moyenne par nucléon
lui-même et devienne une telle étoile sans perdre sa masse actuelle. présentée dans le lableau P 1 pour le •’-'’Pu Г.е.ч mas.ses atomiques
Quel serait .son rayon dont vous aurez besoin sont 2.39.052 16 u (’’'''Pu). I,(X)7 83 u (41)
7E a) Combien de protons et h) combien de neutrons le niudéitle "C et 1.СЮ8 6 / u (neutron)
tonticnt-il ? 14E. a) Démontrez qu’une formule approximative pour determiner la
8E À l'aide d’un tableau de nucléidc.s. écrivez les symboles mas.se M d'un atome est M ~ Ani^,, oii A est le nombre d« masse et
â) de tous les isotopes stables dont Z 60, b) de tous les nucléides est la masse du proton b) Quel poun ciiiage ir r n r iii irouve-i-on
radioactifs dont /V - 60 et c) de tous les nucléides dont A ~ 60. quand on utilise cette formule pour calculer les masses des atomes
356 Chapitre 12 La physique nucléaire

nominé.s dans le tableau 12.1 ? La inas.se du proton isolé est de supposez que la pièce de un cent est entièrement compo.sée d ’atomes
1,007 276 U. c) Celte formule est-elle assez piéeise poui permettre de *’Cu (mas.se de 62,929 60 u). Les masses du proton et du neutron
le calcul de l’énergie de liaison nudeaire ’’ sont respectivement de 1,(X)7 83 ii et de 1,008 67 u.
ISE On peut mesurer les rayons nucléaires par la diffusion d’élections 24P Étant donne que le neutron est électriquement neutre, il faut
i haute énergie par des noyaux, a) Quelle est la longueur d’onde de déterminer sa tnas.se par un autre moyen que la spectroiiictrie de
De Broglie des électrons de 21ЮMeV ? h) Ces electrons consiitucnt-ils masse. Quand un neutron et un proton sc rencontrent (supposez,
les sondes appropriées compte tenu de l'objectif poursuivi? que le.s deux sont presque immobiles), ils se combinent ptnir former
I6E. l e temps nudeaire caractéristique est une mesure utile, mais un deutéron, en cinctiani un rayon gamma ayant une énergie de
loin la définition est imprécise; il s’agit du temps nécessaire à un 2,223 3 MeV. Les masses du proton et du dcuicron (sans aucun
mcléon pt).4.sé(lanl une énergie de quelques millions d'électronvolts électron) ,s<mt respectivement de 1,(X)7 276 467 uctde2,013553213u.
KHir paicouiir une distance égale au diamètre d’un niK-léide de classe A l’aide de ces donnée.s, déterminez la masse du neutron, au nombre
noycnne. Quel en est l’ordre de grandeur? Imaginez des neutrons de chiffres significatifs permis. (Voici une valeur du facteur de
le MeV traversant un diamètre de ’’'’Au; utilisez l’équation 12.3. conversion masse-énergie r- plus précise que celle donnée dans le
7E. Etant donné (|u‘iin nucléon est confiné dans un noyau, on peut manuel ; r ’ = 931,494 McV/ii.)
»nsidéierque Г incertitude quant й sa position correspond approxiinati 25P Démontrez que l'énergie de liaison totale Ey d'un nucléide est
/ement au rayon du noyau r. Que dit le principe d’incertitude sur
t , = ZA„ H ZA„ - â .
’énergie cinétique d'un nucléon contenu dans un noyau dont, par
ixemple,A = 100 ?(/«<:/tcc-considérez Г incertitude quant à la quantité où Ah. a ,, et a simt les excès de mas.se appropriés (voir le problème 22).
le mouvemcnl Др comme étant la quantité de mouvement réelle /7.1 À faide de cette méthode, calculez l’énergie de liai.son moyenne par
8E. Les masses atomiques de 'H. de 'Ч’ et de ’’’'U sont rc.spectivement nucléon du nucléide '‘'^Ati. Comparez votre résultat avec la valeur
le 1,007 825 U, de 12.(ХЮ 000 u (celle-ci est exacte par définition) donnée dans le tableau 12.1. Les excès de masse nécessaires, arrondis
t de 238,050 785 u. a) Cruelles seraient ces masses si on avait défini à trois chiffres significatifs, sont Ah = -1-7,29 MeV, A„ = I-8,07 MeV.
’unité de masse atomique de façon à ce que la masse de 'H corres­ et A |97 = —31,2 MeV. Notez l’économie de calculs quand on utilise
ponde à (exactement) 1,000 000 u? b) En vous inspirant de votre les excès de masse au lieu de la masse réelle, www
ésullal, indiquez pourquoi cette définition n’a pas été choisie.
9P. a) Démontrez que l’énergie associée à la force nucléaire entre SEOION 12.3 La désintégration radioactive
5S nucléons contenus dans un noyau est proportionnelle à A, soit le
ombre de mas,se du noyau en question, b) Démontrez que l’éncrgic 26E. Lin nucléide radioactif a une demi-vie de 30 a Quelle fiaction de
ssociée à la force coulombienne entre les protons contenus dans un réchantillon pur initial de ce nucléide ne sera pas désintégrée au bout
oyau est proportionnelle à Z(Z — I ). c) Démontrez que, à mesure que a) de 60 a et b) de 90 a ?
;s noyaux grossissent (voir la figure 12.4), la force coulombienne 27E. La ilemi-vic d’un isotope radioactif est de I40j. Combien de jours
ugmentc plus rapidement que la force nucléaire. faudra-i-il pour que l'aclix iié d’un échantillon de cct isotope chute
OP. On vous demande de séparer une particule alpha ( ‘Hc) en au quart de sa valeur initiale ?
xtrayant. à tour de rôle, un proion, un neutron et un proton. Calculez 28E La demi vie d’un certain isotope radioactif est de 6,0 h Si un
) le travail requis à chaque étape, h) l’énergie de liaison totale de la échantillon contient initialement 48 x lO*'* atomes de cct isotope,
articule alphti et c) l’énergie de liaison moyenne pai nucléon. Voici combien en contient-il apres 24 h ?
uelques masses atomiques utiles. 29E Prenez un échantillon pur initialement de 3.4 g de *'Ga. isoitqje
ayant une demi-vie de 78 h. a) Quelle est son activité initiale ?
^He 4,(Ю2 60 u 2.014 10 u
h) Quelle est son activité 48 h plus tard'.’
’H 3.01605 u ‘H l.(X)7 83 u 30E. À l’aide des données présentées dans les deux premiers
paragraphes de la section 12.3. déterminez a) la constante de
n 1.008 67 u
désintégration à. et b) la demi-vie du nucléide •’ ’"U.
IP Lin tableau périodique peut donner la masse atomique moyenne 31E. l.in isotope de mercure radioactif '“’’Hg ayant une constante de
J magné.simn, qui esi de 24,312 u. Cette valeur moyenne résulte de désintégration de 0.010 8 h ' .sc désintègre en or, '^^Au. a) fCalculez sa
¡/esée des masses atomiques des isotopes de magnésium selon leur demi-vie. Quelle fraction d ’un échantillon de cct élément resiera-t-il
xinclance naturelle sur la Terre le s trois isotopes et leurs masses au boni b) de trois demi-vies et c) de 10.0 jours ’’
mt ^'Mg (23.08« 04 u). ''M g (24.985 84 u) et -"Mg (25,982 59 u). 32L L’isotope de plutonium ^'®Pu est un sous-produit généré par les
’abondance naturelle de ’'*Mg représente 78.99% de la masse réacteurs nucléaires qui s'accumule dans l’environnement Radioactif,
’esl-à-dire que 78,99% de la masse d’un échantillon de magnésium il a une demi-vie de 2.41 x 10* a. a) Combien de noyaux de Pu consti­
ésent dans la nature est composée de ^'*Mg). Calculez l’abondance tuent une dose chinmiuemcni mortelle de 2.00 ing ? b) Quelle est
;s deux autres i.vitopes. l’activité d'un échantillon de 2,(X) mg?
'P Pout sim plifier les calculs, on présente parfois les mas.ses 33E, Les cellules cancéreuses sont plus vulnérables aux rayons X et
ninuiues non sous la forme de la masse atomique réelle m. mais aux ravtins gamma que les cellules saines. Dans le pas.sé, on utilisait,
iii.s la forme (ni A)r'^. oii 4 est le nombre de masse exprimé en en radiothérapie, l’isotope radioactif *‘’Co conuiie source standard
lités de masse atomique <e h r grandeur habituellement exprimée de rayonnements ionisants ; cet isotope se dé.sintègre en un état excité
I millioas dVIrcironvolls. est apjielée erei’s/fe w /i.w et représentée de '’".Ni avec une demi-vie de 5,27 a. 1,'isotope de nickel émet immé­
II le symbole .A A l'aide des données de l’exemple I2.,i, diatement deux photons de rayons gamma, diacun ayant une énergie
•terminez les cxtè.s de masse a) de 'H. h) du neutron et c) du '•“’Sn. approximative de 1.2 MeV. Combien de noyaux de '’"Co radioactif
P. Lnc pièc-e de un cent a une masse de 3,0 g Calculez l’énergie contient une source de 6 000 Ci du type utilisé dans les hôpitaux ?
icléaire qu’il faudrait pour séparer les uns des autres tous les neii- (Aujourd’hui, la radiothérapie tait appel à des panicukvs énergiques
>ns et tous les protons qu'elle contient Pour siinplifiej le problème. émanant d’accélérateurs linéaires.)
Exercices et problèmes 357

34P. Le radionucléide *^Cu a une deiiii-vie de 12,7 h. Si un échan­ Après un bombardement beaucoup plus long que 2.58 h, I’activitc
tillon pur contient imtialemcnt 5.50 g de ^ u à f = 0, quelle quantité de la cible, en raison de la présence du ^’Mn, est 8,88 x 10'“ Bq.
se désintégrera entre t = 14.0 h et / = 16,0 h ? a) À quel taux R constant les noyaux de ''*Mn sont-ils produius
35P. En 1902, apres un long effon, Marie et Pierre Curie ont réussi à dans le cyclotron pendant le bombardement ? b) À quel taux se
séparer d ’un minerai d ’uranium la première quantité importante de dcsintcgrcnt-ils (durant le bombardement également)? c) Quel est
ntdium. soit un décigiammc de RaCL pur Le radium était l’isotope le nombre de noyaux de ^'’Mn présents à la fin du bombardement ?
radioactif dont la demi-vie est de 1 600 a. a) Combien de d) Quelle est leur masse totale ?
noyaux de radium les Curie ont-ils isolés 7 b) Quelle était raciiviié 44p. (Voir les problèmes 41 cl 43.) Une source de radium cvinticnt
de leur échantillon, exprimée en becquerels ? vww 1,(X) mg de -^Ra. qui .se désintègre avec une demi-vie de 1 WX) a jKuir
3éP Le radionucléide '^P ('/’1,2 = 14,28 j) est souvent utilisé comme produire du ‘^’Rn, un gaz noble. Cet isotope de radon sc dcsintcgrc à
traceur txiur suivre le cours des inactions biochimique.s mettant en son lour pat émission alpha avec une demi-vie de 3.82 j. a) Oiielle esi
jeu le phosphore, a) Si son activité initiale daii.s une expérience l'activitc de dans la source ? b) Combien de temps fant-il an
donnée est de 3 0.50 coups/s, combien de temps lui faudra-t-il pour radon pour atteindre un équilibre séculaire avec son paiciil ladiuiu ’
tomber à 170 coups/s 7 b) Une solution contenant du '-P est donnée c) À quel taux le radon sc désintègre t il alors 7 d) Quelle quantité de
aux racines d’un plant de tomate expérimental piHir mesurer l’activité radon est en équilibre avec son pareni radium 7
du ’’P dans une feuille 3,48 jours plus tard. Par quel facteur doit-on 45P Un des dangers des retombées radioiielives d'une euplosinti
multiplier cette lecture pour corriger la désintégration survenue nucléaire c.st le ““Sr, qui a une drmi-vic de 29 ans. I;Uint tlonnc qu’il
depuis le début de l'expérience ? possède des propriétés chimiques semblables à celles du calcium,
37p. Une source contient des radionucléides de phosphore k strontium, s’il est ingéré par une varhe se conrentre dans le laii l'n e
’^P (/],2 = I4..3 j) et ’’P (7'|,j = 25,3 j). Initialement, 10,0% des certaine quantité du ““Sr aboutit alors dans les os de la personne qui boit
désintégrations proviennent des ” P. Combien de temps faut-il le lait de cette vache. Les électrons cncrgiquos émis par la désintégra
attendre pour que 90,0 %■en prov iennent 7 tion bêta du '“'Sr endommagent la moelle o.s.seuse ei affaiblis.scnt la
38P Un isotope tic plutonium ‘^''Pu ayant une demi-vie de 24 100 a production des globules rougc.s. Une bombe de 1 mégatonne produit
subit une désintégration alpha. Combien de milligrammes d ’hélium environ 400 g de ‘“ Sr. .Si les retombées se répandent tinifurmémeni
sont produits par un échantillon pur contenant initialement 12,0 g sur une superficie tie 2 000 k m ’, quelle sui)ciTicie présenterait une
de ^-’®Pu au bout de 20 CXXl a ? (Tenez compte de l’hélium produit radioactivité égale à la limite «permise» pour une personne, qui c'd
directement par le plutonium et non par un sous-produit du processus de 74 000 Bq?
de désintégration.)
39p. Un échantillon de 1,00 g de samariiini émet des particules alpha SECflON 12.4 La désintégrolion alpha
à un taux de J20 particules/s. L’isotope responsable est le '"” 5111.
qui forme 15% du samarium naturel. Calculez la demi-vie du '^’Sm. 46E. imaginez, qu’un noyau de - **U est constitué d'une particule alpha
40P. Après une brève irradiation de l’argent par des neutrons, on C'He) et d'un noyau résiduel (^“ Th) Représentez graphiquement
trouve deux isotopes; ITsotopc *®*Ag (Ti/2 = 2,42 min), ayant une l’énergie potentielle élcetrique f '(/ ), oh r est la distance entre ces
activité initiale de 3,1 x lO-' Bq, et l’isotope ''"A g (7 ,^ = 24.6 s), particules. Couvrez la plage approximative de 10 fm < /■< 100 fm,
ayant une activité initiale de 4,1 x 10'' Bq. Tracez un graphique cl comparez votre giaphique avec celui pirésrnté dans la figure 12 9.
semblable à celui de la figure 12.8 montrant l’activité combinée 47L Gcncralcmcnt. plus la masse du nucléide est grande, plus il est
des deux isotopes en fonction du temps. d c / = 0 à f = 10 min. instable par suite de la désintégration alpha, Bar exemple, l ’isotope
On utilise la figure 12.8 pour illustrer l’extraction de la demi-vie dans d’uranium le plus stable, le “^’"U. possède une demi vie de 4,5 x 10^ a.
le cas d'une désintégration simple (un i.soloiie) En vous fondant sur L’isotope de plutonium le plus stable est le *"Fu dont la demi-vie est
votre seul graphique de l’activité totale du système à deux isotopes, 8,0 X l()“a;dan.slecas du ciiiium, l'isotope le plus stable est le ’'“*Cni,
suggérez, une analyse permettant de déterminer les demi-vies des dont la demi-vie esi 3.4 X 10' a. Quand la moitié de réchantillon
deux isotopes. original de ^ U s ’est désintégrée, quelles tractions des échantillons
41P (.In produit un certain radionucléide dans un cyclotron, par originaux des isotopes tie, pliitoniimi ei de eurnim meniionné.s reste-i -il 7
exemple, à un taux constant R. Il se désintègre en fonction d'une 48P. Imaginez qu’un noyau de omet a) une particule alpha ou
constante de désintégration A. Supposez que le processus de production b) une suiie ncutmn. proton, neutron, proton. Cakulcz. l'cncrgic
se fait durant un intervalle de temps suffisamment long comparative­ dégagée dans chaque cas, c) Convainquez voue, par des arguments
ment à la demi-vie du radionucléide. Démontrez que le nombre de qualitatifs et par des calculs, que la dilféience eiiire ces tien* nom­
novaux radioactifs pré.sents après un tel intervalle de temps demeure bres e.st égale h l’énergie de liaison tolalc de la parliLiilr alpliii.
constant et qu’il est donné par N — K lk . Maintenant, démontrez que Caleuicz. celte énergie de liaison. Voici quelques mas.sc.s d ’atonic.s et
ce résultat est toujours bon, peu importe le nombre de noyaux de particules qui voas .seront necessaires :
radioactifs présents inilialemeni r>n dit de ce nncléide qu’il est en
équilibre sé a ttm re avec sa stvurcc ; dans cet état, son taux de désinté­
=-'Tl 238.051) 7V U ^’'Tli 234,043 63 U
gration (.son activité) est égal à son taux de priHluctioii. www 217,1 217.048 73 U ^He 4,007 60 II
42P. Calculez la masse d'un échantillon de l’isotope ‘‘"K (initialement 2x.pa 236.048 91 U 'H 1,007 83 U
pur) présentant une activité initiale de 1,70 x l(é Bq. L'isotope a une
demi-vie de 1,28 X 10“ a. “ 'Pa 235.045 44 U 11 1.008 67 U
43P (Voir le problème 41.) 1^ radiotuii léide ’’'’Mn a une demi-vie
49p. Im noyau de ; emet une particule al['h:i de 4 190 MeV r ainile/
de 2.58 h. 11 est produit dans un cyclotron par un bombardement
l’énergie de désintégration 0 tie ce prot esstis en tenant romptr dr
de deutérons sur une cible de manganèse. La cible ne contient que
l'énergie de recul du novau résiduel ' ’’’Tli,
l'isoiopL' de manganèse stable ''^Mn : la réaction inanganèse-deutéron
50P. Ix s gros radionucléides émettent une particule alpha pluifti que
qui produit le '''’Mn est
d'autres combinaisons de nucicides parce que la particule alpha
‘’’’Mn + d ^M n p. possède une structure stable fortement liée. Pour conlirnier i.erie
358 Chapitre 12 La physique nucléaire

aftirmation, calcule2 les étiergiei. de ces déiinlégiatîons hyportictiqucs 56P. Certains radionucléides se désintègrent en capturant un de leurs
et explique/, la signitlcation de vos résultats : propres électrons atomiques, comme un électron de la couche K.
Eti voici un exemple ;
a) 235y 2 ''rh + 'He.
I,) ’i-'u -♦ 2” Th + ’He. «V -é e ■'"Tl 7'i/2 = 3 3 !j.
C) -’3.SU 4. Démontrez que l'énergie de désintégration Q de ce processus est
donnée par
Voici les ma.s.ses atomiques dont vous avez liesoin.
Q = (wv - - Ef.-
2.’I2.0.’I8 I U 'Hc 3.016Ou
où nty et m,, sont les mas.ses atomiques respectives du '"V et du '"Ti,
■!'‘Th 2.31,0.36 3 U 'H c 4.002 611 et Ef, est l'énergie de liaison de l'électron de la couche K du vanadium.
2''Th 230.033 111 ’He 3.012 2 u { In d ic e : soit iiiy el ni,j les masses nucléaires correspondantes;
continuez en employant la méthrxle de l'exemple 12.7.)
'" t l 235.043 Vu 57P Un neutron libre se désintègre selon Péqiiation 12.26. St la
différence de masse entre le neutron et l’atome d'hydrogène est
51 P. D a n s ce rta in e s c irc o n sta n c e s rares, un n o y a u peut se d é sin té g re r
840 /;u. quelle est l’énergie cinétique maximale du spectre
e n é iiicitaiu u ne p a rticu le p lu s m a ssiv e qu*unc particule alplia. V o y e z
d’énergie de l'électron ?
e r s tlcsintc.gralions
58P Déterminez l'énergie de désintégration Q du '"V' lorsqu'il y a
“ 'Ra ^ -"''Pb + "C capture d'un électron de la couche K, comme le décrit le problème .56.
Voici les données qui vous sont nécessaires: niy, = 48,948 52 u,
et >^'‘^Kn ' -Hc. /«TI = 48.947 87 u et 5.47 keV.
a) Calcule/ les valeurs Q de ces désintégrations et démontrez que 59P Le radionucléide "C .se désintègre de la manière suivante :
toutes vieux sont éneqii(|uenient possibles, hj l a hauteur de la barrière
"C -» "B + + v; 'En = 20.3 nun.
coulombienne de rémission de la particule alpha est de .30,0 McV.
Quelle est la hauteur de la barrière de l'émission du "C ? Voici les L'énergie maximale des positrons émis est de 0.% 0 MeV.
mas.ses atomiques dont vous avez besoin.
a) Démontrez que l'énergie de désintégration Q de ce processus est
2^’Ra 223,018.50 U "C 14.00.3 24 U dctcrniinée par

’’"'Pb 208.981 07 U 'He 4.002 60 U Q {m e - «lu - 2m, )i ^

2'’Rn 219.009 48 U oil m e et ni|j sont les masses atomiques respectives du "C et du "B,
et m, est la masse d ’un positron, b) A l’aide des valeurs de masses
Wc = 1LOI 1 4.34 u. («H 11,009 .305 u et m, = 0,000 .548 6 u,
SfCTiON 12.5 La désintégration bêlo calculez Q et comparez sa valeur avec l’cnctgic maximale du position
52f. I.es radionucléides de masse élevée, qui peuvent émettre des émis donnée précédemment, { ¡ » d ir e : soit nie ™i. mas.se.s
particules alpha ou beta, appartiennent à T une des quatre séries nucléaires, continuez en employant la méthode de l'exemple 12.7.
de désintégration, .selon que leur nombre de masse A a la forme Notez que la désintégration bêla plus est une exception <à la règle
An, An I- 1, 4n 1-2 ou 4n -I- 3, <>ù ri est un nombre entier positif,
générale qui dit que. .si des masses atomiques sont utilisées dans les
a) Justifiez celle affirmalion et démontrez que, si un nucléidc appartient calculs des désintégrations nucléaires, la masse de l’électron émis est
à Pline de ces séries, tous scs prixluits de désintégration appartiennent automatiquement considérée.)
à la même série, b) ria sse z ces nucléides scion la série à laquelle 60P Deux nuelcidcs instables par rapport à la désinlégratiim alpha,
ils appjffiienneni : ’” U, ” ^1.1. ^"*11. ^"'Pu, -'^Pu, -■’’Cm. m, ’' ’Cf, le -'’’U et le -'^Th, et un autre, instable par rapport à la vlésintégratiiMi
ct^” Fm. bêla, le '"K, sont suffisamment abondants pour contribuer de manière
53L. Un eciiaiii nucléidc stable, après ahsvirplion d’im neutron. érraU notable au récliauffcmcnt de la Terre par l’énergie de désiniégratiiHi
un électron, le nouveau nudéide formé se divise spontanément qu'ils génèrent. Ix s isotopes instables par rapport à la désintégration
en deux pailicules alpha. Idcniilicz le nucléidc. alpha génèrent des chaînes de desintégrauons qui cessent à la lormaiion
des isotopes stables du plomb. L'isotope " ’K subit une simple
S4£. Un nucléidc do masse moyenne (/1 = 150, par exemple) émet
désintégration bêta. Voici les données du problème
un élcelruii avec une énergie einéliqiie de 1.0 MeV. a) Quelle est la
longueur d'onde de De Broglie de Pcicciron b) Caltailez le rayon du Mode Point
noyau qui l'éinel. c) Un électron peut-il être confiné sous la forme de Demi-vie final Q /
d'une onde stationnaire dans une «boite» ayant de telles dim en­ Parent désintégration (a) stable (MeV) (mg/kc)
sions" (il Pouvez-vous faire ap|iel à ces nombres pour contredire
l'argumeni (abandotmé) qui veut que les élecirons existent vraiment 33«U a 4.47 X 10" ''’"Ph 51.7 4
dans les noyaux " -” 'Tli a 1.41 X II)'" ‘"»Pb 42.7 13
55f. l/i.sotopc de césium est présent dans les retombées “ ’K P 1,28 V 10’ "C a 1.31 4
H'rxplosioMs nu( léaircs aii-<lessus du sol filant di'iiné qu'il se
ilésintèfif r.ii "'R a avn une demi-vie lente (30.2 a) en dégageani Dans le tableau, Q e.st l'cncrgic totale dégagée par la désintégration
constJsiablcinenl d'cncigic. il .simicvc une inquiétude du poinl d’un noyau parent au point fi/fo/ stable, et /e st la concentration de
rte vue environnemental, l.es masses atomiques de Cs et de Ba sont l’isotope en tnilligr.immes par kilogramme de granit a) Démontrez
respectivement de 136,907 1 u et de 136,90.5 8 u; calculez l'cncrgic que cc.s subsianees génèicnt de l’énergie sous forme de chaleur au
totale dégagée par une telle désintégration. taux de LO z: 10 VV par kilograinine de granit, h) Hn supposant
Exercices et problèmes 359

qu’il y a 2,7 x kg de granit dans une couche sphérique de 20 km 70P. L ne pei sonne de 75 kg rex^oit une dose de rayonnements ionisants
d’épaisseur à la surface de la Terre, estim e/ la puissance de cette sur tout le corps de 2.4 x 10 Gy, émise par des particules alpha
désintégration sur toute la Terre. Com paret celte puissance avec la dont le (acteur de qualité est 12. C alcule/ a) l’énergie absorbée
puissance totale du Soled captée par la Terre, soit 1,7 x 10” W. en jtxilcs ei b) l'équivaieni de dose en sieveris et en rem.
61P'. Le radionucléide ” P se désintègre en comme le décrit 71P. S'affairant près d’un surgénérateur, un travailleur de 85 kg ingère
l’équation 12.23. Dans une certaine désintégration, un électron de aixidentcilcmcnt 2.5 mg de poussière de ’"’l’u. Ce radioisotope a une
1.71 MeV est émis, ce qui correspond à la valeur maximale possible. demi vie de 24 100 a, et sc désintègre par désintégration alpha.
Quelle est l’énergie cinétique de recul de Talome de '^S durant cet L’énergie des particules alpliit émises est de 5,2 McV. avec un lactcur
événement ? (In d iv c : dans le cas d’un élexlron. on doit utiliser les de qualité de KL Supposez que le plutonium demeure dans le corps
exprcs.sions rclaiivisteji pour l'énergie cinétique et la quantité de du travailleur pendant 12 heures et que 95 51. dc.s part’u nies alpha
mouvcmeiiL La mécanique newtonnienne peut s'appliquer dans le cas émises y sont capturées. Calculez u) le nombre d ’atomes de plutonium
de l’atome de *^S. dont le mouvement est relativement lent.) www absorbés, bi le nombre d ’cmre eux qui se désiiiiègreiii diiraiii le.v
12 heures, c) réncrpic absorlicc pai le corps, d) lu dose résulluutc
SECTION 12.6 la dolalion par radioatlivité en grays et e) l’équivalent de dose en sieverts.

62f Le 'Y ’ contenu dans un échantillon de charbon de 5,(K) g provenant


SEOION 12.8 Us modèles nucléaifcs
d’un ancien leu de camp a une activité de 03.0 désintcgration.s/min.
Dans un arbre v ivant, l’activité du ” C pour un échantillon de 1,00 g 72E. Ln noyau intermédiaire impliqué dans une réaction nucléaire
est de 13.3 désintcgralions/min. La demi vie du ' ‘*C est 5 730 a. particulière se désintègre 1.0 x 1(1 —s après sa formation a)QiielIeest
Quel est l’âge de l’échanlillon de charbon'? l’incertiludc AL quant à notre connai.vsaiicc (.t i ci clal îiilcrnxrdiain. ?
WE. Le nucléide se désintègre en ’^ P b avec une deini-v ie de b) f k;t étal peut il s'apixler « noyau composé » ? ( Voir l’exemple 12.10.'l
4,47 X 10^ a. Bien que cette désintégration se produi.se en plu.sieurs 73L Nommez, parmi les noyaux suivants, a) ceux dont le,s couchc.s
étapes, la première étape possède de loin la plus longue demi-vie : de nucléons sont remplies, b) ceux qui présentent un nucléon hors
par conséquent, on peut souvent considérer que cette désintégration d’une eixiche remplie ei c) ceux ()ui afOi hem une vacant e dans une
mène directement au plomb. Ainsi. couche autrement remplie . T . 'H ). •'’K . "‘T l. ’”K i.'’'7x ^ 'Mo. ‘^'.Sb.
■” N d . •’".Sm . '“T l et '"'P b
2Mj_l -®*>Pb ‘ divers produits de désintégration.
74P. Voyez les trois processus de formation du noyau composé ‘'’Ne
On constate qu’une roche contient 4.20 mg de et 2,1,35 mg illustrés dans la ligure 12.13. Voici quelques unc.s des inas,scs atomiques.
de ^'“’Pb. .Si cette roche ne contenait pas de plomb à sa fonnation. ■’Ne 19.992 44 U 4,002 60 U
(le .sorte que tout le plomb présent actuellement provient de la
désintégration de ITininium. a) combien d’atomes de -^'*l'et de ■ ’"^Pb ”Ь 18,998 40 U 1.007 83 U
la pierre conticnt-cllc maintenant ? b) Combien d'atom es de ’^’*U
'Ю 15.994 91 U
la rot hc conicnait-cllc à sa formation ? c) Quel est l’âge de la roche ?
64P. ü n croit qu’une certaine roche a 260 millions d’années. Si clic Quelle énergie a) la particule alpha, Ы le proton et c) le photon dp
contient 3.70 mg de combien de ^ P b devrait clic contenir’’ rayons gamma doivent-ils avoir pour fournir une énergie d'excitation
iVoir l’exercice 63.) de 25.0 McV au noyau composé ’? www
65P. l ’iie roche trouvée profondément sous terre contient 0.86 mg
de 0,15 mg de ^’’’'Ph et 1,6 mg de “’Ar. Combien de '"'K Problèmes supplémcnloires
dcvrail-cllc ctMitcnir? Les demi vies à connaître .sont indiquées dans
75. A la fin de la .Secoixle Oiierre nmntliale. le.s atmirilés néerlaïulauce»
It; problènx' 60.
ont arréic l'arti.stc hollaiulais Ilaii.s vau Meegeicii pour iiahi.soii jmicc
qu’il avait vendu un tableau de maître au nazi Hermann Gocrine
SECTION 12.7 Mesurer lo dose de rayonnemenls ionisunis jicndant la guerre. Le tableau, Ln H encontre à i.r im o u s du peintre
Johannes Vermeer (1632-167S), a éié tlrcimvrrt rii И 17 jiai van
66E. Ln compteur Ciciger enregistre 8 7(X) coups en 1 minute Calcule/
Meegeren après une disparition de près de 300 ans. Peu après su
l'activite de la source en becquerels et en curies, en supposant que
découverte, les experts en arts alfiriiiérent t|ii hmmaiis éiaii
le compteur enregistre toutes les désintégrations.
piobabicineiii le plus grand V mneci jam ais vu. La vciili: d 'un tel
67L Ijc nucléide '”*Au. qui a une demi vie de 2,70 j. est utilisé pour
tiésor national à Tennemi lui considérée comiiK' une (rahisoii sons nom.
combattre le cancer. Quelle nias.se dr ce nucléide est nécessaire pour Cependant, peu après son arrestation, van Meegeren annonv'a
produire une activité de 250 Ci ? soudainemenl que c’élaii lui ei uim Vermeei qui av.iii peint tn m n n n
68E. Qn pilote de ligne vole en moyenne 20 h par semaine à une altitude Il expliqua qu’il avait soigneuscnit nt imité le style de Vcrmccr.
de 10 km, oil l’cquivalenl de dose de rayfinnements co.smiques est et utili.sé une toile vieille de 300 ans et les pigments tic Vcrmccr ;
de 7.0 /iSv/Ii Quel est l'équivalent de dose annuelle (52 semaines) il avait ensuite signé le nom de Vermeer sur le liibleau el cuit ce
provenant de cette seule source'? N ote/ que l'équivalent de dose dernier pour lui donner un a.speci vieilli
annuelle maximale permise (provenant de uhjics les sources) pour la Van Мссцсгеп incntait-il pour échapper îi une accusation de
population en général est de ^ mSv; dans le cas des travailleurs tiahison '>Hspcniii il être at;eiisé de fraude, un crime moins grave?
soiimi.s aux rayonnements, clic est de 50 misv. Du point de vue des experis, F m iuails trssrm blaii vriiimcrii îi un
i9P I ,oi s d’une radiographie pulmonaire, le patient revoit une dose Vermeer mais, à répixjuc où s'est tenu le pniccs de van Mccrgeren
type de 250 /wSv. dégages par des rayons X dont le facleui de qualité (1947), il n ’y avait aucun moyen seientifit|iie de résoudre rénigm e
est de 0,85. En supposant que la masse du tissu exposé rcprésciile Cependant en 1968. Bernard Kei,sch. de ruiiiverviir Carnt-gii -
la moitié de la masse du paiieni de 88 kg, calculez l'énergie absorbée Mellon, a pu s’attaquer au inv-stcic grâce ù des techniques nouvelles
tm (oulcs. d’analyse par radioactivité.
3i0 Chapitre 12 La physique nucléaire

il analysa notaninieni un pelil échantillon d ’un pigment blanc a) Quel est le processus de désintégration du '"Mo en '"Te ? b) Si on
contenant du plomb prélevé sur t'm m aüs. Ce pigment est une particule injecte un échantillon de 8.2 x 10' Bq de '"Te à un patient, combien
raffinée provenant d ’un minerai de plomb, dans lequel le plomb de photons de rayons gamma .sont initialement produits dans le patient
esi produit par une longue suite de désintégrations radioactives qui chaque seconde c) Si le taux d ’émission des photons de rayons
conimencc par le instable et sc termine par le ^*’Pb stable. Pour gamma d'mie petite tumeur qui a capturé le '"Te est de 38 par seconde
suivre l’esprit de l’analyse de K cisji, penche/-vous sur la panic à un certain moment, quelle quantité de '"Te à l’état excité .se trouve
abrégée suivante de la suite des désintégrations, dans laquelle les dans la lunicur à cet instant ?
radionucléides intcnnédiaire.s à vie relativement eoune ont été omis : 79. À cause de l’explosion et de l’incendie qui ont eu lieu dans le
réacteur nucléaire de Tchernobyl (nord de l’Ukraine) en 1986,
^’"Th - > ^-'■’Ra 2'"Pb ^ ^“<’Pb.
75,4k» ;16d une partie de l'Ukraine est contaminée par du " ’Ce. qui subit une
désintégration bêta moins ayant une demi-vie de 30.2 a. En 1996,
Ijis plus longues et les plus importantes demi-vies de cette partie de l'activité totale de cette contamination sur une superficie de
la suite de désintégrations y sont indiquées.
2,6 X 10' k n r était estimée à 1 x lO'*’ Bq Supposez que le " ’Ce
a) Démontrez que, dans un échantillon témoin de minerai de plomb, est répandu uniformément sur celte superficie ei que les électrons
le taux auquel le noinbic de noyau.\ de change est déterminé pai de la désintégration bêla se déplacent directement vers le haut ou
riyV.>i( directement vers le bas. Combien d’électrons de la désintégration
~ ^226^226 ' ‘21(A210- bêta auriez-vous intercepté.s si vous vous étiez couche sur le .sol dans
clf
cette région pendant I li a) en 1996 et b) aujouid’hui? (Vous devez
OÙ A'jK, et sont le nombre de noyaux de ^'‘'Pb et le nombre estimer votre superficie fmmale, qui est la superficie qui intercepte
de noyaux de contenus dans l échantillon, et Àjm et X22« sont ces électrons.)
les constantes de désintégration correspondantes. 80. En iKtobre 1992, la police suisse a arrêté deux hommes qui
Hiant donné que la suite de désintégrations est active depuis e.ssayaicnt de pas.ser de l’osmium en contrebande vers l’Europe de
des milliards d années et que la demi-vic de -"^Pb est de beaucoup l’E-st. Cependant, ces hommes avaient pris, pai erreuu du " ’Ce. On a
intérieure à celle de ^^Ra, les nuclcides -^'Ra et sont en équilibre. rapporte que chatiue contrebandier avait transporté un échantillon de
c'c-st-à-dire que leur nombre, ou leur concentration, dans l’échantillon 1,0 g de '-’’Ce dans une poche. En becquerels et en curies, quelle était
ne varie pas. bl Quel est le rapport R^tiJl^ixa clcs activités de ces l’activité ik chaque échantillon“' I x " ’Ce a une demi-vie de 30,2 a.
nucléides daas l’échantillon témoin de minerai de plomb? cl Quel est (Les activités des radioisotoiies couramment utilisés dans les hôpitaux
le rapport N 22fJbt2 w Je leur nombre ? soni tle l’ordre de quelques millicuries.)
Quand le pigment de plomb est raffine à panir du minerai, le 81. La figure 12.16 montre une partie d’un schéma de dé,sintégraiion
e.st pratiquement éliminé. Suppevsez ici qu' il en reste seulemem du ^” Np dans un graphique présentant le nombre de masse A en
1.00%. Juste après la production du pigment, quels sont les rappt>rts fonction du numéro atomique Z ; cinq segments de droites représentant
J) ^22e^^2ic e) 'V-)2fWjio ? la désintégration alpha ou la désintégration bêta moins relient les
Keisch constata que. avec le temps, le rapport R226^^2in J'J pigment points qui représentent les isotopes. Quel isotope sc trouve à la fin
passerait graduellement de la valeur présente dans le pigment fraîchc- des cinq désintégrations (accompagné d’un point d ’interrogation
iiKiit raffiné ù celle dans le minerai où il y a équilibre entre le ^"’Pb dans la figure 12.16)?
et le ^^'^Ra résiduel. Si tr m ia U s avait été peint par Vermeer et que
l'cchantillon qui y a été prélevé avait .100 ans à son examen en 1%8,
le rapport serait près de la réponse obtenue en b). S'il avait été peint
par van Meegeren dans les années I9.H0 et que l’échanlillon n ’avait
que 30 aiLS, le rapport serait près de la réponse obtenue en d). Keisch
trouva un rapport de 0,09. f) E m m aiis est-il un Vermeer ’’ va»w
/ 6. Pendant îles essais nucléaires au-dessus du sol, de la poussière
radioactive a etc projetée dans les couches supérieures de l’atmo­
sphère Les courants aériens ont alois répandu celle poussière sur
l’ensemble de la planète avant qu'elle s'incorpiw au sol et à l’eau.
Lin tel essai a etc réali.sé en octobre 1976. Quelle fraction du ’’'Sr
produit par cette explosion restera-t il en octobre 2fX)6 La demi-vie
du '”^‘Sr est 29 a.
77. Dans un laboratoire, on prépare un échantillon radioactif destiné
à l'irradintion d’un patient dans un hôpital voisin. L’échantillon a une
demi- vie de 83.61 h. Quelle devrait être .son activité initiale pour 82. L’isotope ’"K. qui a une demi-vie de 1.26 x 10’ a. peut se désin
qu’elle soil de 7.4 x 10* Rq au inoiix-ni où on l’utilisera pour irradier tegrer en ‘"'Ca ou en ’''Ai. Le rapport entre le Ca et l'A r qui sont
le patient, ¿4 h plus tard profluits est lie 8,.S'1/1 = R,.'v4 Initialement, un échantiUon ne possédait
que du ’'’K. Il possède niainienant des quantités égales de '”’K et
78. ( )n peut injecter le micléide radioactif '’^Tc dans le sang d ’un
de ■’''A r; c'e.st donc dire que le, rapport entre le K et l’Ar est 1/1 = 1
parient piHir, entre autres, suivre Sa circulation, mesurer le volume
.sanguin nil loi.alisiT une tumeur l.e micléide est priyJuit dans un Quel est l’àgc de cet échantillon (liuliee: traitez ce probicine comme
hôpital pai une « vache» eontenani du '''^Mo, un radionucléide qui se tout aiiire pniblèmc mettani en jeu la datation par radioaetiv ité, excep­
drsiiiiègir en “'^ c avec une demi vie de 67 h Une fois par ituir. on tion faite que cette désintégration a deux prixluits au lieu d’un .seul.)
« Iniii » la « vaclic » (xjur obienir le produit dans un état excité 83. Dans certaines cavernes, l’air contient une quantité importante
par le '".Mo, le '" l’c 5C désexcite vers .son état fondamemal en émettant de radon, un gaz pouvant provoquei un cancer du poumon s’il est
un photon de rayons gamma, qui esr enregistré par des détecteurs respire pendant une longue période Dans les cavernes britanniques,
placés autour du patient. La dcsexcilation a une demi-vie de 6,0 h. l’air de caverne présentant la plus grande quantité de ce gaz a une
Exercices et problèmes 361

activité volumique de 1,55 x 10" Bq/m \ Supposez que vous passiez 85. Quelle e.st l'énergie de liaison moyenne pai' nucléon du “ -Bh ?
deux jours complets à explorer (et à dormir) dans cette caverne. La masse de l'atome est 262,123 1 u.
Quelle quantité approximative d’atomes de ' '^Rn inspireriez-vous et 86. Combien d’années faut-il pour que l’activité du chute
expireriez vous durant votre séjour? Le radionucleide ^^-Rn contenu à 0,020 de .sa valeur originale ? La demi-vie du ‘‘*C est de 5 730 a.
dans le radon gazeux a une demi-vie de 3,82 jcxirs. Vous devez
87. Quelle est l’activité d’un échantillon de 20 ng de ''-’Kr, dont
estimer votre capacité pulmonaire et votre taux moyen de respiration.
la demi-vie est de 1.84 s ?
84. Un noyau de ^.^i aymit une énergie cinétique de 3,00 MeV est
pnijeté vers un noyau de -” Th. Quelle est la distance minimale entre
les centre» des deux noyaux, en supposant que le noyau de -’^Th
(le plus massif) ne bouge pas ?
,L, -f ^ 'f •
kl#-••* <==• ]3 L’énergie
nucléaire

À l'époque de la Seconde Guerre mondiale, le monde a été frappé de stupeur par cette image. Après avoir ossisté
à la première explosion atomique, Robert Oppenheimer, le directeur de l'équipe qui a conçu la bombe atomique,
nota la phrase suivante, tirée
d'un texte sacré hindou :
« Je suis maintenant lo Mort,
destructrice des mondes. »
Í3.2 Lo fission nucléaire ; principe d e base 363

13.1 L'atome et son noyau


Q u a n d o n o b tie n t d e l’é n e rg ie e n b rû la n t d u c h a rb o n d an s u n e c h a u d iè re , on m an ip u le
le s a to m es d e c a ib o n e e t d ’o x y g èn e, ré a rra n g e a n t leu rs électrons e x te rn e s p o u r en faire
d e s c o m b in a is o n s p lu s sta b le s. Q u a n d o n o b tie n t d e l ’é n e rg ie d e T u ra n iu m d a n s u n
ré a c te u r n u c lé a ire , o n b rû le e n c o re un c o m b u stib le , m ais o n en m an ip u le ici les n o y a u x ,
ré a rra n g e an t leurs nucléons p t)u r e n fa ire d e s c o m b in a iso n s p lits stab les.
L es électro as so n t m aintenus d an s les atom es p a r la tb ree électro m ag n étiq u e ; il ne faut
q u e q u e lq u e s c lc c im n v o lis p o u r en e x tra ire un. P a r co n tre , les nui Ico n s sont m ain ten u s
d a n s les n o y au x p a r l’ititeraclio n fo rte, c l il la u t q u e lq u e s milüoius d ’clectro n v o ltft }X)ur
e n ex tra ire un. C e fa c te u r d e q u e lq u e s m illio n s se reflète ihins le rap p o rt e n tre f è n e rg ie
q u ’o n p e u t e x tra ire d 'u n k ilo g ra m m e d 'u ra n iu m e t c e lle q u 'o n p e u t o b te n ir d 'u n
k ilo g ra m m e d e c h arb o n .
D ans les co m b u stio n s lU om ique et nucléaire, le d ég ag em en t d ’énergie est acco m p ag n é
d 'u n e d im in u tio n d e m a sse , sc io n l'é q u a tio n Q = —A w L a p rin c ip a le d iffé re n c e
e n tre la c o m b u stio n d e l’u ra n iu m et c e lle d u charlroii e st que, la p re m iè re coii.sum c une
p lu s g ra n d e fractio n d e la m asse d isp o n ib le (en co re p ar un la c lc u r de quclquci> m illio n s)
q u e la seco n d e.
L es d iv e rs p ro c é d é s u tilisé s d an s la c o m b u stio n atomique o u nucléaire fournissent
d iffé re n ts n iv eau x d e p u issan ce (les tau x au x q u e ls l ’é n e rg ie e.si d é g ag ée). D an s le cils du
n u cléaire, o n p eu t c o n su m e r un k ilo g ra m m e d 'u ra n iu m u n e vites.se cxplo.sive a v ec une
b o m b e o u le fa ire le n te m e n t d a n s un réacteu r. D an s la c o m b u stio n a to m iq u e , o n peut
m e ttre e n p a ra llè le l’e x p lo s io n d ’un b â to n d e d y n a m ite et la d ig e s tio n d ’u n b is c u it
au ch o co lat.
Læ ta b le a u 1 3 .1 m o n tre la q u a n tité d 'é n e rg ie q u 'o n p eu t exti'aire d e 1 k g d e m atière
p a r d iv e rse s m éth o d es. A u lie u d e d o n n e r d ire c te m e n t une v a le u r à l'é n e rg ie , le ta b le a u
in d iq u e r in tc r v a llc d e te m p s d u ra n t le q u e l l ’é n e rg ie e x tra ite p o u rra it a lim e n te r u n e
a m p o u le électriq u e de 1(X) W . S euls les tro is p rem iers p ro céd és o n t réellem en t fait l ’o b jet
d e te s ts ; le s tro is a u tre s re p ré se n te n t d e s lim ite s th é o riq u e s in a c c e ssib le s en p ra tiq u e
L e d e rn ie r, so it l’a n n ih ila tio n m u tu e lle to ta le d e la m a tiè re e t d e r a ïu im a tiè ie , e s t
u n o b je c tif u ltim e d e p ro d u c tio n d ’én erg ie. D an s ce p ro c e ssu s, toute l'é n e rg ie au rcpo,s
est tra n sfo rm é e en d ’au tres fo rm e s d ’én erg ie.
N o te z q u e le s c o m p a ra is o n s d a n s le la b le a u 13.1 .sont c a lc u lé e s (x iu r u n e nias.se
d e I kg. P oui un m êm e k ilo g ram m e, o n o h iie n i p lu sie u rs lu lllio a s d e fois p lu s d 'é n e rg ie
d e r u r a n iu m q u e d u c h a rb o n o u d ’ u n e c h u te d ’e a u . P a r c o n tre , o n tro u v e b e a u c o u p
d e c h a rb o n d an s l’éco rce te rre stre , et l'e a u re v ie n t fa c ile m e n t en lu n o n t d 'u n bai iag e

13.2 La fission nucléaire : principe de base


E n 19 3 2 , le p h y sic ie n Ja m e s C h a d w ic k d é c o u v rit le n eu tro n . Quelques années plus lard,
à R o m e, E n ric o F e rm i c o n stata q u e , si tm b o m b a rd e d iv ers é lé m e n is av ec îles n en iro iis,
il y a p ro d u ctio n d e n o u v eau x clénK 'nts rad io actifs F erm i av a it p ié d itig ic le n cu iio n . ne
p o rtan t a u cu n e c h a rg e , serait un p ro je c tile n u cléaire u tile ; c o n tiiiiie m e n t au p ro to n o u î)
la p a rtic u le a lp h a , il ne su b it a u c u n e fo rc e c o u lo m b ie n n e ré p u lsiv e q u a n d il so tro u v e
p rè s d 'u n n o y au . M ê m e les neutrons therinu/ues, d e s n e u tro n s len ts en é q u ilib re th e i-
m iq iic av ec la m atière e n v iro n n a n te à te m p é ra tu re am b ian te et q u i ont u n e é n e rg ie ein é-
tiq u e m o y e n n e d ’cnx iro n ().(M eV . c o n stitu e n t d e s p ro jectiles u tile s dans les etudes sur
la p h y siq u e n u cléaire.

TABLEAU13.1 par 1 kg de matière


Matière Processus Temps*
r.au Chute de .“iO rn .“i.s
l'harbon Combustion 8h
UO 2 enrichi f ission dans un réacteur 690 a
Fission l'i'inplète ^ )c Ifé a
Deutérium gazeux chaud f usion complète 3 y ltr*a
Matiète et antimatière Annihilation complete 1 A Ifl’ a
C'ette colonne indique le tem ps dur,mt lequel l’cncrgic générée pourrait alinwntei
une am poule électrique de 100 W.
364 Chapitre 13 U’energie nucléaire

À la fin d e s an n é e s 1930, la p h y sic ie n n e L is e M e itn e r et le s c h im iste s O tto H a h n


e t F ritz S lra ssm a n n , tra v a illa n t à B e rlin cl s ’in té re s s a n t au tra v a il d e F e rm i e t d e scs
c o lla b o ra te u rs , o n t b o m b a rd é d e s s o lu tio n s d e se ls d ’u ra n iu m a v e c d e te ls n e u tro n s
th erm iq u es. A p rès le b o m b ard em en t, ils o n t cotLstaié la p résen ce d ’un c e rta in n o m b re d e
n o u v e a u x rad io n u c lé id e s. E n 1939, d e s te sts ré p é té s ont p e rm is d ’id e n tifie r le b a ry u m
c o m m e l’u n d es ra d io n u cléid es p ro d u its d e celte façon. H ahn et S trassm an n se sont alo rs
d e m a n d é : « C o m m e n t c e t é lé m e n t d e m a sse m o y e n n e (Z = 5 6 ) p e u t-il ê tre p ro d u it
p a r le b o m b ard em en t d e l’u ran iu m (Z = 9 2 ) av ec tics n e u tro n s ? »
C e cas.se -tê te a é té ré s o lu e n q iic lq u e s se m a in e s p a r M e itn e r et so n n e v e u O tto
F risc h . Ils a v a n c è re n t l’id é e d ’un m é c a n is m e fa isa n t q u ’u n n o y a u d ’u ra n iu m a y a n t
ab-virlié un n eu tro n th e rm iq u e p o u v a it se scinder, a v ec d ég a g e m e n t d ’é n e rg ie , e n deux
frag m en ts gro.ssièrem ent é g a u x , d o n t l ’u n p o u rra it b ien être le b a ry u m . F risc h n o m m a
c e p ro c e ssu s fissio n .
L e rô le d é te rm in a n t jo u é p a r M e itn e r d a n s la d é c o u v e rte d e la fissio n n ’a p as été
p lein em en t re co n n u Ju sq u ’à ce iiue d e s re c h e rc h e s récen tes en h isto ire le m e tte n t à jo u r.
E lle n 'a p a s p a rta g é le p rix N o b e l d e c h im ie re m is à O tto H a h n en 1944. C e p e n d a n t,
H a h n et M e itn e r o n t to u s d e u x e u l’h o n n e u r d e d o n n e r le u r n o m à d e s é lé m e n ts :
h ah n iu m (sy m b o le H a, Z - 105) e t m e itn e riu m (sy m b o le M t, Z = 109).
La fission vue de près
L a fig u re 13.1 p ré se n te la d istrib u tio n se lo n le n o m b re d e m asse d e s frag m en ts p ro d u its
p a r le b o m b a rd e m e n t d u n u clé id c ’ ’•‘’I J a v ec d e s n e u tro n s th e rm iq u e s, l.e s n o m b res d e
m a sse les p lu s p ro b a b le s, a p p a ra issa n t d a n s e n v iro n 7 % d e s é v é n e m e n ts, so n t c e n tré s
a u to u r d e A » 9 5 e t d e A ~ 140. C u rie u se m e n t, la co u rb e à d eu x p ics d an s le gi ap h iq u e
d e la fig u re 13.1 n ’e st p a s e n c o re c o m p rise co m p lètem en t.
D a n s u n e fissio n ty p e d e un noyau de a b so rb e un n e u tro n th e rm iq u e ,
p ro d u isan t un n o y au d e d an s un é ta t ex cité élev é. C 'e s t ce n oyau qui .subit réellem en t
la Fission, se s c in d a n t e n d e u x fra g m e n ts. C e s fra g m e n ts é m e tte n t ra p id e m e n t d e u x
n e u tro n s , la is s a n t (d a n s u n c a s ty p e ) d u ' ‘*°Xe (Z = 5 4 ) e t d u ''’’S r (Z — 3 8 ) c o n m ie
frag m en ts de fissio n . D onc, l ’é q u a tio n de fissio n g én é ra le p o u r ce t é v é n e m e n t e st
^ 236JJ i « x e + ^ S r + 2n. (1 3 .1 )
N o te z q u e , d u ra n t la fo rm a tio n et la fissio n d u n o y au c o m p o sé , il y a c o n se rv a tio n du
n o m b re d e p ro to n s e t d u n o m b re d e n e u tro n s e n je u d a n s le p ro c e s s u s (d o n c d e le u r
n o m b re to tal e t d e la c h arg e n ette).
D an s l ’é q u a tio n 13.1, les fra g m e n ts d e ‘‘“’X c e t d e ‘’"S r .sont to u s d eu x h a u te m e n t
in stab les, su b issa n t une d é sin té g ra tio n b êta (av ec la c o n v e rsio n d ’un n e u tro n e n p ro to n

Figure 13.1 I a distribution selon le nombre de masse des fragments tiouvés lors de réliide
de nombreuses fissions de ^’'(1. Note/ que l’échcllc verticale est logarithmique.
13.2 La fission nucléaire ; principe de base 365

c l l ’é m issio n d ’u n é le c im n e t d ’u n n e u trin o ) ju s q u ’à ce q u ’ils a iic ig n e n t u n n u c lé id e


final .stable. D an s le cas d u x én o n , la sé rie d e d csin ic g ra iio n e st
I40xc ^ '«»Cs '‘‘“B a ^ '^ ’L a '‘*<*Ce
■fi/2 14s 64 s 13 J 40h S tab le
(1 3 .2 )
Z 54 55 56 57 58
D ans le c a s d u stro n tiu m , la .série est
"‘‘.Sr 'V.I

76 s 19 min S tab le ( l i i)

Z 38 39 40
S i o n se fie a u x n o tio n s p ré .se n lé e s d a n s la .sectio n 1 2 .5 . o n p o u v a it s 'a tte n d r e à c e
q u e les n o m b re s d e m as.se (1 4 0 e t 9 4 ) d e s fra g m e n ts d c in c u re n t m e h a n g e s d u ra n t c e s
d é s in té g ra tio n s b ê ta , e t à c e q u e le s n u m é ro s a to m iq u e s (in itia le m e n t 6 4 et 3 8 )
a u g m e n te n t d ’u n e u n ité à c h a q u e étap e.
U n e x a m e n d e la b an d e d e stab ilité d u tab leau d e s n u cléid es d e la fig u re 12.4 ré v è le
pourquoi les frag m en ts d e la fission .sont instables. L e nu cléid e ^ U , q u i est le n oyau fissile
d a n s la ré a c tio n d e l'é q u a tio n 13.1, p o ssè d e 92 p ro to n s et 236 - 92 = 144 n e iim m s,
ce q u i d o n n e u n ra p p o rt n e u tro n s/p ro to n s a p p ro x ira a iif d e l ,6. L e s p ie m ie rs fr a g n iriirs
form é,s im m é d ia te m e n t a p rè s la fissio n a ffic h e n t e n v iro n c e m êm e la p jio rt neutron.s/
p ro to n s. C e p e n d a n t, c h e z le s n u c lé id e s s ta b le s se tro u v a n t d a n s la z o n e d e s m a sse s
m o y e n n e s, c e ra p p o rt e st p lu s p e tit, s o it e n u e 1,3 e t 1,4. L es p re m ie rs fra g m e n is so n t
d o n c riches en neutrons (ils p o ssè d e n t tro p d e iK u tro n s), d e .sorte q u ’ils e n e x p u lse ro n t
q u elq u es-u n s. deux d an s le ca s d e la réactio n d e l’éq u atio n 13.1. T r-s fiag m en ts qui resten t
s o n t e n c o re tro p ric h e s e n n e u tro n s p o u r ê tre s ta b le s . L a d é s in té g ra tio n b ê ta p e n n e i
au x n u c lé id e s d e se d é b a rra sse r d e s n e u tro n s e n e x c è s, n o tam m en t en les tra n sfo rm a n t
e n p ro to n s à l'in té rie u r d u n o y au .
O n p e u t e s tim e r l’c n c rg ic d é g a g é e p a r la fis s io n d 'u n n u c lé id e m a s s if e n te n a n t
c o m p te d e l'é n e rg ie de liaiso n to tale p a r n u clé o n £i„ av an t e t a p re s la fissio n . L ’id ée e st
q u e la fissio n e st p o ssib le p a rc e q u e l'é n e rg ie au re p o s to ta le d im in u e ra , c ’r s i- à - d ir r
q u e augm entera p o u r q u e les p ro d u its d e la fissio n so ien t plus fo riein eiii liés. D o n c ,
l’é n e rg ie Q d é g a g é e p a r la fissio n est
n - ( ü a is o n \ / é n e rg ie d e lia iso ii\ (H 4)
^ ~ \ to tale fin ale / initiale

P o u r é ta b lir l'e s tim a tio n , o n su p p o se q u e la fissio n tra n sfo rm e un n o y au m a.ssif in itial


en d e u x n o y au x d e m asse m o y e n n e ay an t le m ê m e n o m b re d e n u c lé o n s O n a alo rs
_ ^ AEi„ j ^ n o m b re fm al'^ _
\finale / \ de nucléons /
^ Ai
(initiale K nombre mmal\
de nucléons f (1 3 5)

D iuis la fig u re 12.6, on co n state q u e, d an s le cas d ’un n u cléid e m a ssif (2l ~ 24U). l'é n e rg ie
d e lia is o n p a r n u c lé o n e s t d 'e n v ir o n 7 ,6 M e V /n u c lé o n . D a n s le c a s d e s n u c lé id e s
d e m as.se m o y e n n e (A ~ 120 ), e lle e st d ’e n v iio n 8..5 M e V /n u c lé m i. D o n c , l ’c r c r g ic
d é g a g é e p ar la fissio n d ’un n u cléid e m a ssif en <leux n u c lé id e s d e m asse m o y e n n e est
_M cV \ /. „ nucléons \
(s .5
Q -
n u cléo n /I (2 noyaux) \1120 n o y au /I
MeV \
- 17,6 ' I (240 nucléons) ~ 2,0 X 10^ MeVi (13 6)
nucléon !

^VÉRIFIEZ VOS CONNAISSANCES! Voici une fission générale :

-■ n - > x ‘ Y \- 2ii
Parmi les paires de nucléides suivanies quelle est celle ou quelles sont celles qui ne peuvent
être rcpréscntée.spar X et
V a) '“"Xc et''*Sr ; b) *” C.sct '''R b ; c) '''’N dcl '“G c; d) '^'In et " 'R u ?
366 Chapitre 13 L'énergie nucléaire

Exemple 13.1
Déterminez l’énergie de désiniégraiion Q dans le cas de la fission de neutrons du membre de droite. La différence de mas.se dans lu réaction
l’équation 13 1. en tenant compte de la désintégration des fragments de l’équation 13.7 est
de fisston inscrits dans les équations 13.2 et 133. Voici quelques masses
d ’atomes et celle du neutron dont vous avez besoin pour résoudre A«i = (139.905 4 u -r 93,906 3 u H- 1.008 67 u) - (235,043 9 u)
le problème : = -0,22.3 53 u,

235,043 Ч U '^•Ce 1.39.905 4 U et l’énergie de désintégration corre-spondiuite est


n l.fXI8 67 u 93,906 3 U Q = -A m (P- = - ( - 0 .2 2 3 53 uX931,5 MeV/u)
= 2083 MeV, (répon.se)
SOlUîlOM Voici les contepH dés utilisés ici ; I) l’énergie de désintégra­
tion Q est l'énergie au repos transformée en énergie cinétique des qui est en accord avec rcsiimation présentée à l'équation 1.3.6.
produits de la désintégration et 2) = —A/n où Awi est la variation Si la fission se pnxiuil dans un corps solide, la plus grande partie
de la masse. Puisqu’il faut inclure la désintégration des fragmctits de de cette éttergie de désintégration, qui se traiisfonne d'abord en énergie
fis.sion, on combine les équations 13.1, 13.2 et 13.3 pour écrire ainsi cinétique des pnxluits de la désinlcgratioii, se traduit finalement par
la transformation générale : une augmentation de l’énergie ititerne de ce corps, se révélant par
une augmentation de la température. Cependant, environ 5 % chi 6%
I4(^e • ■>‘'Уг + n. (1-3.7)
de l’énergie de désintégration est associée aux neutrinos émis durant
Un .seul neutron apparaît ici parce que le neutron initiateur présent la dé.sintégralion béta des premiers fragments de fis,sion. Cette énergie,
dans le membre de gauche de l'équation I 3.I annule l ’un des deux emportée hors du système, est perdue.

13.3 Un modèle de fission nucléaire


P eu a p rè s la d é c o u v e rte d e la fissio n , N ie ls B o h r et Jo h n W h e e le r o n t u tilisé le m o d èle
c o lle c tif du n o y au (.section 12.8) e t étab li u n e a n a lo g ie e n tre un n o y au e t u n e g o u tte de
liq u id e c h a rg é e p o u r e n e x p liq u e r le p rin c ip e d e b ase. I.a fig u re 13.2 m o n tre co m m en t
c e m o d è le e x p liq u e la fissio n . Q u a n d u n n o y au m a s s if, p a r e x e m p le le a b so rb e
u n n e u tro n lent (th e rm iq u e ), c o m m e d an s la fig u re 13.2 a), c e n e u tro n to m b e d a n s un
p u its d e p o te n tie l a s s o c ié a u x in te ra c tio n s fo rte s a g is s a n t à l ’in té rie u i d u n o y a u .
L ’é n e rg ie p o te n tie lle d u n e u tro n est a lo rs tra n sfo rm é e en é n e rg ie d ’e x c ita tio n in te rn e
d u n o y au , c o m m e le m o n tre la fig u re 13.2 b). L ’é n e rg ie d ’e x c ita tio n q u 'u n n eu tro n lent
a p p o rte à l ’in té rie u r d ’u n n o y au e st é g a le à l’é n e rg ie d e lia iso n du n e u tro n d a n s ce
n o y a u , q u i e st la v a ria tio n d e l’é n e rg ie au rep o s d u sy stè m e n e u tro n n o y au p ro v tx ju ée
p a r la c a p tu re d u n eu tro n .
L e s fig u re s 1 3 .2 c ) e t 13.2 d ) m o n tre n t q u e le n o y a u , se c o m p o rta n t c o m m e une
goutte de liquide ch arg ée oscillant fortem ent, fo rm era tô t ou tard u n c o u rt « col ». Le noyau
a u ra a lo rs la fo rm e d ’u n h a ltè re . S i la ré p u lsio n é le c triq u e é lo ig n e su ffis a m m e n t les
d eu x p a rtie s d e l’h a ltè re l’itne d e l’aittre p o u r ro m p re le co l, les d e u x frag m en ts, c h acu n
p o ssé d a n t u n e c e rta in e é n e rg ie d ’e x c ita tio n ré sid u e lle , .sc .sép arero n t (fig u re s 13.2 e)
et 13.2 f). L a fissio n sen t alo rs term in ée.
NeiilToii

0
Ix- iiovaii rte ^^ tl Formation fl’iin iiorau l e iiioiivenient
аЬчг)г1х* un nriiumn 'le ^ ‘''t ' |4)«4»41ant peut pisKliure
thermique. un extécleiil rl'éiiejRie; un col.
il oseille violcinuieiu.
a) b) c)
^ / N< iiiro iis , J

I- »•
9 9
I es tors es I J fission I.fS fr^cnirtys
couloinbienne.s яе prtKluit. si|>2<re'nt:
13.2 Les éta)x;s d’une fission étivciii le noyau. d e s iioutr<»ii.s
•ype selon le modèle collectif de Bohr son» éjrclés.
et Wheeler a> 0
13.3 Un modèle d e fission nucléaire 367

C e m odèle constitue une bonne description qualitative d e la fi.ssion. Il reste toutefois


à savoir s ’il peui répondre à une question difficile : pourquoi certains nucléide.s m assifs
et -^'*Pu, par exem ple) sont-ils facilem ent fissiles si on les bom barde de neutrons
therm iques alors que d ’autres, ég alem en t m assifs et “^’A m , p a r ex em p le), ne le
sont pas 7
B o h r e t W h e e le r o n t p u ré p o n d re à c e lte q u e s tio n . L a fig u re 1 3 .3 re p ré s e n te
g rap h iq u e m e n t l’é n e rg ie p o te n tie lle d ’un n o y au à d iv e rse s étajies d e la fissio n , d ’ap rès
l'a ra in è li'e fie flistursioii r (liii) le u r m o d èle. C e tte é n e rg ie e st rep ré se n lé e g ia p h iq u e in c n i e n fo n ctio n d u ¡H inim ènr i k
figure 13.3 L’ôncrgic poietiiielic d is to r s io n r. qui e st u n e m esu re g ro ssiè re d e ré tirc m c n l q u e co n n aît u n n o y au o scillan t
à iliverses rtape.s île la fi.>«ion, p a r ra p p o rt à sa fo rm e sp h ériq u e. I a lig u re 13.2 d> illu stre cc p a ra m è tre ju ste av an i la
comme ijei mel de le prédire le modèle fissio n . Q u an d les fra g m e n ts so n t é lo ig n é s, cc p a ra m è tre ne re p ré se n te q u e la d ista n c e
collectif de liohr et Wheeler. La valeur q u i sép a re le u rs cen tres.
de y de la réaction (environ 200 MeV) La d ifléren ce d ’én erg ie entre l’état initial et l'é ta t final d u n oyau en fission (c'est-à-d ire
et la haiiieur de la baiT ière de fission l’é n e rg ie d e d é s in té g ra tio n Q ) e st d o n n é e d a n s la fig u re 13.3. O r, o n le iu a rq iie , d a n s
siait toutes lieux indiquées. c e tte fig u re , q u e la c o u rb e d ’é n e rg ie p o te n tie lle a tte in t n u in a x im u in à u n e i e ria ln r
v a le u r d e r. D o n c , il y a u n e I x i r r i è r e d e ¡ m i e n i i e l d 'u n e h a u lc u i F(, q u i d o it e tn :
•surm ontée (o u trav e rsé e p a r e ffe t tu n n el ) p o u r q u e la fissio n sc p ro d u ise. C c q u i rappelle
la d é sin té g ra tio n a lp h a (fig u re 12.9). p ro c e s s u s e g a le m e n t in h ib e p a r u n e b a rric rc d e
p o ten tiel.
O n en d é d u it q u e la fissio n sc p ro d u ira se u le m e n t si le n eu tro n ab so rb é fo u rn it u n e
é n e rg ie d 'e x c ita tio n k„ s u ffis a m m e n t g ra n d e p o u r s u rm o n te r c e tte b a rn e re . Q irace à
l'e ffe t tu n n el d e la p h y siq u e q u a n tiq u e . U n 'e s t to u te fo is p as n é cessaire q u e celle é n e rg ie
so it a u s s i g ra n d e q u e la h a u te u r de la b a rriè re
Le tableau 13.2 m ontre le test q u i vise à savoir si un neutron therm ique peut provo­
quer la fission de quatre nuclcidcs m assifs. R elativem ent ü Chaque nucléidc, le tableau
donne la hauteur de la barrière du noyau form e par la capture du neutron e t l’cncrgie
d ’excitation £„ générée par la capture. Les valeurs de E„ sont calculées en fonction de la
vanation de l ’énergie au repos prov(X]uéc par la capture du neutron.
V oici un ex e m p le du ca lc u l e ffec tu é p o u r £ „ de la p rem ière lig n e du tab leau ,
qui représente le proces.sus suivant de la capture d ’un neutron :

-H il ^ ” f’U .

L e s m a sse s en je u so n t 2 3 5 ,0 4 3 9 2 3 u d a n s le c a s d u 1,(X)8 6 6 5 u d a n s le c a s du
n e u tro n e t 2 3 6 ,0 4 5 5 6 2 u d a n s le c a s d u Il e st facile d e d é m o n tre r q u e, e n raiso n
d e la c a p tu re d u n eu tro n , la m a sse d im in u e d e 7 ,0 2 6 x 1 0 “ ^ u. D o u e, il y a tra iisle rt d e
l ’é n e rg ie au rep o s à l’é n e rg ie d ’e x c ita tio n £„. Si o n m u ltip lie la v ariatio n d e m asse p ar
<r < = 9 3 1 ,5 M eV /u ), o n o b tie n t E„ = 6 ,5 M eV , v a le u r q u i e st d o n n é i' d an s la p n em ièif
lig n e d u tab leau .
L es p rem ier et tro isièm e résu ltats d u tab leau 13.2 o n t u n e iin p o rtaiio e lii.storique, c a r
ils c o n stitu e n t les raisim s p o u r Iesi|uel1es le s d eu x bonihe.s aïo in iq u e s lan c é e s à la fin de
la S e c o n d e C îiicrre m o n d iale co n te n a ie n t d u (p rem ière Ix n n h e) e t d u ’ ^ u (sei:nnde
b o m b e). A u tre m e n t d it, p o u r le cl le ^’“Pu. £ „ > £i,- C e q u i sig n ifie q u e lu fi.ssion
p a r a b s o rp tio n d ’un n e u tro n th e rm iq u e e s t p o s.sib lc c h e z c e s n u c lc id c s . D a n s le cas
d e s d e u x au tres n u c lé id e s (-'•'U e t ^''^A m ), o n a E„ c ’esi d o n e d ire q u e le noiirm n
th e rm iq u e n e Ib u rn it p as s u ffis a m m e n t d ’é n e rg ie p o u r q u e le n o y a u e x c ité su rm o n te
la b a rriè re d e p o te n tie l ou la tra v e rse p a r e ffe t tu n n el A u lien d e se scin d er, le n o y au
se d é b a rra sse d e eo tte én e rg ie d ’e x rita fio n en c n x ’tta n t un p h o to n d e ray o n gam m a.

TABLEAU 13.2 Test de fissiMité de quatre nudéides

Nucleidc Niicléide subissant £n(MeV) L,(M eV) Fission par neutrons


cible la fis.sion llieriiilqw.s'’

™ ll 6..S Oui
-•'HJ 4.8 5.7 Non
2Vlpu - “’Pu 6.4 4.8 Oui
.s,.*) 3.8 iXon
348 Chapitre 13 L'energie nucléaire

11 e st to u te fo is p o ssib le q u e le e t le ^ -'A m su b isse n t une iissio n s ’ils ab so rb e n t


un n e u tro n ay an t a sse z d ’é n e rg ie (au lie u d ’un n eu tro n th e rm iq u e ). P a r e x e m p le , d an s
le c a s d u le n e u tro n a b s o rb é d o it p o s s é d e r u n e é n e rg ie m in im a le d e 1,3 M eV
p o u r q u ’u n e r a p id e puis.se se p ro d u ire.
L e d é g a g e m e n t d ’é n e rg ie e x p lo s if e t d é v a s ta te u r d e s d e u x b o m b e s a to m iq u e s
e m p lo y é e s p en d a n t la S e c o n d e G u e rre m o n d ia le d é p e n d a it d e la c a p a c ité d e s n e u tro n s
th e rm iq u e s d e p n tv o q u e r d e s fissio n s p re sq u e sim u lta n é e s c h e z les n o m b re u x n u c lé id e s
m assifs q u e c e s b o m b e s c o n ten aien t. L a p rem ière b o m b e u tilisait du - ’■“'U parce q u ’o n en
a v a it s u ffis a m m e n i tiré d ’ u n m in e ra i d ’u ra n iu m p o u r fa b riq u e r c e tte b o m b e e t u n e
b o m b e d ’e.s.sai. (Ixi m in erai c o n tie n t p rin cip itlem en l du q u i, c o m m e o n le sait, n ’e st
p a s fissile p a r d e s n e u tro n s th e rm iq u e s.) l,a se c o n d e b o m b e u tilisa it d u ’ ^'^Pu; e lle fut
corv;:ue à l’aid e d es seu ls calc u ls th é o riq u e s résu m és d an s le tab leau 13.2, c ar il n ’y av ait
p as su ffisa m m e n t d e ad d itio n n e l d isp o n ib le q u a n d e lle fu t œ m m a n d é e .

À 20 m du module 4 du réacteur 13.4 Le réacteur nucléaire


de Tchernobyl Iprès de Kiev), après
P o u r o b te n ir un d é g a g e m e n t d ’é n e rg ie à g ran d e é c h e lle , u n e fission d o it e n p ro v o q u e r
SOI) explosion en avril 1986. Presque
d ’a u tre s , p o u r fa ire en s o rte q u e le p ro c e s s u s s e ré p a n d e d a n s to u t le c o m b u s tib le
tous les radionucléides volatils présents
dans le réacteur ont été projetés
n u c lé a ire , c o m m e u n e fla m m e d a n s u n e b û c h e . L e fa it q u e la fissio n p n x iu ise p lu s de
dan.s l’atnio-sphèrc.
n e u tro n s q u ’e lle n ’e n c o n su m e au g m e n te la p o ssib ilité d 'u n e telle ré a c tio n e n ch aîn e»
d u ran t laq u elle c h a q u e n eu tro n g é n é ré p eu t p o te n tie lle m e n t p ro v o q u e r u n e an tre fissio n ,
l a ré a c tio n p e u t ê tre ra p id e (c o m m e lo rs d ’u n e e x p lo s io n n u c lé a ire ) o u c o n trô lé e
(c o m m e d a n s un ré a c te u r n u cléaire).
S u p p o s e z q u ’o n v e u ille c o n c e v o ir u n ré a c te u r fo n d é s u r la fis.sion d u par
d e s n e u tro n s th e rm iq u e s . L 'u ra n iu m n a tu re l c o n tie n t 0 ,7 % d e c e t iso to p e , le s a u tre s
W ,3 % é ta n t c o n stitu é s d e n o n fissile p ar d e s n eu tro n s th e rm iq u e s. O n p eu t ten ter
d ’a u g m e n te r l ’e ffic a c ité d u ré a c te u r e n e n r i c h i s s a n t a rtific ie lle m e n i l ’u ra n iu m
c o m b u stib le p o u r q u 'il c o n tie n n e p e u t ê tre 3 % d e Il reste n é a n m o in s e n c o re tro is
d iffic u lté s à su rm o n te r p o u r c o n c e v o ir u n ré a c te u r fo n ctio n n el
1. L a fu i t e d e n e u tr o n s . C e rta in s d e s n e u tro n s p ro d u its p a r la fissio n s ’é c h a p p e ro n t du
ré a c te u r e t n e p articip ero n t p as à la réaction en ch aîn e. L a fuite est un effet d e su rface ;
so n im p o rtan ce e s t p ro p o rtio n n e lle au c a iré d e la lo n g u e u r d 'u n ré a c te u r ty p e (T aire
totale d ’u n cu b e d ’une lo n g u eu r d ’arête a est 6r/^). C ependant, la production de neutrons
s ’e ffe c tu a n t d a n s to u t le v o lu m e d u c o m b u s tih le , e lle e s t p ro p o rtio n n e lle au c u b e
d ’u n e d im e n s io n ty p e (le v o lu m e du m ê m e c u b e e s t a^). O n p e u t ré d u ire
au m in im u m la fra d io n d e n eu tro n s p erd u s par fuite en fab riq u an t un c œ u r d e réacteu r
s u ffis a m m e n t g ra n d ; o n ré d u it a in si le ra p p o rt e n tre la s u rfa c e e t le v o lu m e
6/rt d an s le cas d ’u n cu b e).
2 . L 'é n e r g ie d u n e u tr o n . L e s n eu tro n s g é n é ré s p a r la fissio n sont ra p id e s ; ils p o ssè d e n t
u n e én e rg ie c in é tiq u e d ’e n v iro n 2 M cV . C e p e n d a n t, la fissio n e st p lu s e ffic a c em e n t
in d u ite p a r les n e u tro n s th e n n iq u e s. Il e st p o ssib le de ra le n tir les n e u tro n s ra p id e s en
m élan g ean t l’u ran iu m à u n e su b sta n c e , ap p elée m o d é r a te u r , ay an t d e u x p ro p riétés :
c lic ra le n tit le s n e u tro n s a u m o y e n d e c o llis io n s é la s tiq u e s e t e lle n e re tire p as les
n e u tro n s d u c œ u r p a r a b so rp tio n . E n A m é riq u e du N o rd , la p lu p a rt d e s ré a c te u rs
n u c lé a ire s u tilis e n t T e a u c o m m e m o d é ra te u r; les n o y a u x d ’h y d ro g è n e (p ro to n s )
p résen ts dans l’e a u en .siint les co m p o san tes actives. D a a s le ch ap itre 10 d u v o lu m e 1,
ou a vu q u e, si une p articu le en m o u v em en t su b it u n e co llisio n fro n tale é la stiq u e avec
une p artic u le iin n io h ile, la p a rtic u le e n m o u v e m e n t perd to u te so n é n e rg ie cin é tiq u e
si les d eu x p articu les ont la m êm e mtesse. D o n c, les p rotons co n stiliien i des m odérateurs
e ffic a c e s p a rc e q u ’ils o n t a p p ro x im a tiv e m e n t la m êm e m asse q u e les n e u tro n s d o n t
n n v eu t réd u ire la v itesse.
3 . I a i c a p t u r e d e n e u tr o n s . Q u a n d l’é n e rg ie d e s n e u tro n s ra p id e s (2 M e ’V ) g é n é ré s
p a r la fissio n e st ra m e n é e au n iv eau d es é n e rg ie s th e rm iq u e s (en v iro n 0 ,0 4 e V ) p a r
le m o d érateu r, il y a un in te rv a lle d ’é n e rg ie c ritiq u e (d e 1 à 100 eV ) au c o u rs d u q u el
1rs iic u tiïin s so n t p a rtic u liè re m e n t s u s c e p tib le s .d ’c trc c a p tu ré s sa n s fis s io n p a r
les n o y a u * d e ^’*11. U n e te lle c a p t u r e p a r r é s o n a n c e , q u i se so ld e p a r l ’é m is s io n
d ’un ray o n g a m m a , retire le n eu tro n d e la c h a în e d e fissio n . F o u r réd u ire ces c a p tu re s
san s fission, l’u ran iu m co m N istib le e t le m o d érateu r ne so n t p as étro item en t m élangés ;
ils sont plu tô t « re g ro u p é s » , o c c u p a n t d iffé re n te s z o n es du v o lu m e d u réacteur.
13.А Le reacteur nucléaire 369

D a n s un ré a c te u r ty p e, l’u ra n iu m co m b u stib le p ren d la fo rm e de p a stille s d ’o x y d e


d ’u ra n iu m , in sérées b o u t à b o u t d an s d e lo n g s tu b e s m éta lliq u e s. L e s g ra p p e s fo rm é e s
p a r c e s c ra y o n s d e c o m b u s tib le b aig n en t d an s le m o d é ra te u r liq u id e, fo rm a n t le c œ u r
d u ré a c te u r. C e tte d is p o s itio n g é o m é triq u e a u g m e n te la p ro b a b ilité q u ’u n n e u tro n
ra p id e , p ro d u it d an s u n c ra y o n d e co m b u stib le , so it d an s le m o d é ra te u r q u a n d il trav erse
l ’in te rv a lle d ’é n e rg ie c ritiq u e . U n e fo is q u e le n e u tro n a a tte in t le n iv e a u d ’é n e rg ie
th e rm iq u e , il p e u t e n c o r e ê tre c a p tu ré san s p ro d u c tio n d e fissio n ( c a p tu r e th e r m iq u e ) .
C e p e n d a n t, il re to u rn e ra p ro b a b le m e n t d a n s un cra y o n d e c o m b u stib le e t y p ro v o q u e ra
u n e fission.
l,a fig u re 13.4 illu s tre l’é q u ilib re d e s n e u tro n s d a n s u n ré a c te u r n u c lé a ire ty p e
fo n c tio n n a n t à p u issa n c e c o n sta n te . O n p eu t .suivre le c y c le c o m p le t d ’un é e b a m illo n
d e 1 (KX) n c u tro a s th e rm iq u e s, u n e g e n e r a tio n , d an s le c œ u r d ’u n réacteur. Ils p ro d u isen t
1 3 3 0 n e u tro n s p a r fissio n d a n s le c o m b u s tib le e t 4 0 n e u tro n s p a r rissio n d a n s
le c e q u i d o n n e 3 7 0 n eu tro n s d e p lu s q u e les I 0 0 0 n eu tro n s in itiau x , to u s iapidi:s.
Q u a n d le ré a c te u r fo n c tio n n e à p u is s a n c e c o n .stan tc. le n o m b re d e n e tilro n s p e rd u s
p a r fu ite d u c œ u r e t p a r c a p tu re .sans fissio n est to u jtiu rs e x a c te m e n t le m êm e (3 7 0 ), ce
qui la isse 1 0(Ю n e u tro n s th e rm iq u e s p o u r c ré e r la g é n é ra tio n su iv a n te R ien sûr, d an s
c e c y c le , c h a c u n d e s 3 7 0 n e u tro n s p ro d u its pai fissio n re p rc sc n ic u n déix'it d 'c n rrg itr
d an s le c œ u r du réacteu r, c e q u i le réch au ffe.
L e f a c t e u r d e m u ltip lic a tio n к (im p o rta n t p a ra m è tre d u réac te u r) e s t le ra p p o rt e n tié
le nom bre d e neutrons présen ts au d éb u t d ’u n e g én ératio n e t le n o m b re d e neutron.s présents
au d é b u t d e la g é n é ra tio n s u iv a n te . D a n s la fig u re 13 4 , le fat le u r d e m u ltip lii a iio n
e s t 1 0(Ю/1 (XX), so it e x a c te m e n t l’ u n ité. Q u a n d A = l, le fn iietiim n em en i d u ré a c te u r
e s t d it e x a c te m e n t c r itiq u e , et c ’est c e q u e l’o n v e u t p o u r a v o ir u n e p u issa n c e sta b le
E n fait, o n co n ço it les réacteu rs d e façon q u ’ils soient x iq x r c r itiq u e s d e n tan icre inhérem c
( k > I ) ; o n a ju ste e n su ite le fa c te u r d e m u ltip lic a tio n p o u r o b te n ir le fo n c tio n n e m e m
critiq u e (A = 1,) en e n fo n çan t d e s b a r r e s d e c o n trô le d an s le c œ u r d u réacteur. C es b arres
c o n tie n n e n t u n e su b sta n c e , c o m m e le c a d m iu m , q ui a b so rb e fa c ile m e n t les nt u tr o n s ,
o n p e u t le s e n fo n c e r p ro fo n d é m e n t p o u r ré d u ire la p u issa n c e d ’o p é ra tio n o u les retirer
p o u r a u g m e n te r c e tte p u issa n c e ou p o u r c o m p e n se r la ten d a n c e d es ré a c te u rs a attein d re

Fiiiie de neurron.s
the Iiniques

(ii] m ires
üicniiique.s

t'aplurcs
par résonance

themiqiies

Fuite
de neutrons
rapides \

Fissions
iapitle.s
figure 13.4 (.'oniptabilitc des neutrons dans un réacteur. Lhic génération de I (XX) neutrons
thermiques interagit avec le combustible la matrice ' ”’1 ; ei le mocieraieut. Ils prtxliii.seni
I 370 neutrons par fission ; 370 d enire eux sont perdu.s [lai cai«iirc san.s fi.ssion («i par fiiiU'.
II re,ste donc 1 000 neutrons thermiques pt'ur former la generation suivante La figure illustre
un réacteur fonctionnant à puissance constante
370 Chapitre 13 L’énergie nucléaire

un niveau s o u s - c r itiq u e à m esure que les pnxluits de fission (qui absrrrbent les neutrons)
s'accum ulent dans le cœur.
Si vous retiriez rapidem ent l’une d es barres de contrôle, à quel taux la p u issance
augm enterait-elle ? Le te m p s d e r é p o n s e dc^^end d ’une situ atio n p articu lière e t fasci­
nante, soit celle où une p etite fraction d es neutrons gén érés p ar fission ne s'é c h a p p e
p as im m éd ia te m en t d es frag m en ts d e fissio n n o u v ellem en t fo rm é s, m ais p lu s tard,
au m om ent où les fragm ents ,se désintègrent p ar ém ission bêta. D es 370 « nouveaux »
neutro n s p ro d u its d an s la fig u re 13.4, p a r e x em p le. 16 p eu t-être .sont retard és, étan t
ém is par les fragm ents apres les d ésin tég ratio n s bêta, dont les dem i-vies d urent entre
0,2 s cl ."îS s. C es neutrons retardés sont jieu nom breux, m ais ils sont utiles pour ralentir
le tem ps de rép o n se du réa cte u r d e façon q u ’il co rresp o n d e aux tem p s des réactions
m écaniques.
l.,a figure 13.5 est le .schéma d ’ime centrale électrique à réacteuo à e a u s o u s p r e s s io n .
type de centrale couram m ent utilisé aux É tats-l.'nis. D ans un tel réacteur, l’eau sert de
m odérateur et de m ilieu d e transfert de la chaleur. D ans la p r e m iè r e b o u c le , elle circule
dans l’appareil du réacteur et transfère l’énergie à tem pérature et pression élevées (600 K
et 150 atm p o ssib les) du cœ u r chau d du réa cte u r à la ch au d ière, qui fait p artie de la
d e u x iè m e b o u c le . Au C anaila, les réacteu rs CAKDLT (C a tu id io n d e u te r iu m - u r a n i u m )
utilisent de l’eau lourde ( D 3O ) com m e modérateur et c-ommt? milieu de transfert de chaleur
circulant dans le ereur. D ans la chaudière, l’évaporation fournit une vapeur à pression
élevée qui p< rm el d ’actio n n er la turbine qui en traîn e le g én érateu r d ’électricité. Pour
c o m p léter la d e u x i è m e b o u c le , la v ap eu r à faib le p ressio n de la tu rb in e est refro id ie
et coïKlenséc en eau, puis pous.séc dans la chaudière par une pompe. .Sachez q u ’un réacteur
typique d 'u n e centrale (électrique) de 1 000 M W peut atteindre une hauteur de 12 m et
un p o id s de 4 M N . L ’eau circ u le d a n s la p rem ière bo u cle à un d éb it a p p ro x im a tif
de 1 M I7m in.
L’utilisation de réacteurs nucléaires engendre inévitablem ent une accum ulation de
déchets radioactifs, dont des produits de fission et des nucléides tr a n s u r a n ie n s lourds,
com m e le plutonium et l’am éricium . L e taux auquel ces déchets dégagent de l’énergie
sous form e therm ique etm stitue une m esure de leur radioactivité. La figure 13.6 m ontre
la puissance therm ique générée par les déchets produits en une année d ’opération dans
une grande eentralc nucléaire type. N otez que les deux éch elles sont logarithm iques.
1 ^ plupart des crayons de eombu.stible « épuisés » des réacteurs nucléaires sont entrepo.scs

Vaix-ut (pression élever)

Encrée du liquide
<l«r

Sortit du liquide'
relroiilis.seiiieui

Kcnrl< ni
sous pression F.au
(pression

élevée) . ‘ r .
(p ressio n faible)

Pré’iiiièr« hniirle D eu x ièm e b o u r îe

figure 13.S Schéma simplifié d’un réacteur nucléaire à eau sous pression. Il y manque de nombreux
élérnent.s. dont le clispositil de lefroidisse.mcnt du cneur en cas d ’urge,ncc.
13.4 Le réacteur nucléaire 371

figure 13.6 Graphique de la puissance thermique dégagée par les déchets radioactifs résultani
d’une année d’opération clans une grande centrale nucléaire type en fonction du temps.
La courbe est une superposition des actioas de nombreux radionucléides, qui on( une grande
diversité de demi-vies. Notez que les deux échelles sont logarithmiques.

in sim , iin in e rg é s d a n s l'e a u ; o n d o it d o n c e n c o re c o n stru ire tics site s d ’entrciK isagc


p e rm a n e n t p o u r les d é c h e ts n u cléaires.
U n e g ran d e q u an tité d e d éch ets i-adioactifs p ro v em u ii d ‘a rm es, ac c u m u lé s d iin u il la
S e c o n d e G u e rre m o n d iale el a u etn irs d e s an n é e s su b sé q u e n ie s. so n t e n c o re cm rep o sés
d a n s d e s site s c o n s tru its à c e tte lin . P a r e x e m p le , la fig u re 13.7 m o n tre u n e n se m b le
d e ré se rv o irs so u te rra in s e n c o n stn ic tio n au site d e H an fo rd , d an s l’É tat de W ash in g io n ;
c h a q u e g ra n d ré se rv o ir eo iu icn l l M l. d e d é c h e ts liq u id es liau tcm en i rad io aeiifs. C e site
c o m p te a u jo u rd 'h u i 152 ré se rv o irs D e p lu s. be.aueoup d e d éi h c is so lid e s, faib lem en t
rad io actifs (d es v ê te m e n ts c o n ta m in é s, p a r ex e m p le ) et h a u te m e n t ra d io a c tifs (les piles
n u c lé a ire s d e so u s-m arin s d é m o n té s, p a r e x e m p le !, so n t e n se v e lis d an s les tran ch ées.

Figure 13.7 Un ensemble de té sa voirs


souterrains en construction pendant
la Seconde Guerre mondiale au site
de llanford, dans l’État de Washington.
Observe/ les camions et les ouvriers.
Chaque grand réservoir contient
actucLlenient 1 ML de déchets
hautement radioactifs.

Exemple 13.2
Une grande centrale électrique est alimentée par un réacteur (= 3,4 X lO'’ J/s); et ?) selon l'équation 13.0. l'énergie Q dégagée
nucléaire à eau sous pression, La puissaiKC thermique produite dans par chaque fission est d’envimn 200 MeV. Попе, pour arriver à imr
le eteur du réacteur est de 3 400 M W . la centrale génère 1 100 MW opération stable (F constante), on obtient :
d’élecirieilé l a masse de combustible est 8.b0 x 10* kg d'uramum,
sous forme tl’oxyde d’uranium, distribué dans 5,70 x 10* crayons.
_ F _ / 3.4 X iO’ J/s \ / 1 MeV
L’uranium est enrichi de 3,0% de
^ ~ Ü “ \2(Ю М сУ /Ь х5ЮП) V l.bl) X K )-'M
a) Quel est le rendement de la centrale ?
= 1.00 X 10"'’ lis,sions/s
SOIÜTIOU' 1/>. ccntepl tié est la définition du rendement de cette centrale l , I X 10^” hssions),s. (réponse)
électri<iiie ou de tout autre appareil ■rapport entre la puissance de
production (taux auquel l’énergie utile est obtenue) et la puissance
c) À que) taux (en kilogrammes par jour) le combustible
fournie (taux auquel l’énergie doit être fournie), le i, le rendement
disparaîi-il Tenez compte des conditions présculrs au début des
iiend) est
opérations.
paiissance lie priKliietion 1 КЮ N W (électrique)
rend = SÜIUTIOK' Ici, le contepitié est que le -'■''l. disparaît en raison île deux
puissaiKe fournie 3 fOO MW (thenuique)
procédés: 1) la fission au taux calculé en b), el 2) la capture sans
0,^2, ou 32 %. (rcpon.se)
fission de neutrons à environ le quart de ce taux. Donc, le taux total
Le remlement (comme ilans unîtes les centrales électriques) obéit au de disparition de r est
deuxième principe de la thcrmodyiianiique. Four que cette centrale
fonctionne, il doit y avoir un taux de 3 400 MW — 1 100 MW. soit (I > 0,25K I.06x 1 0 ^ aïome.s/s) 1.33 X 10*'momes/s.
2 3(X) MW. de perte d’énergie ; cette énergie est dispersée sous forme
tlienniquc dans l’environnement. On doit maintenant avoir la masse de ohaijtie atome de On ne
peut utiliser la masse molaire de riiraniiini donnée dans l'armrxr T
b) À quel taux K la fission se produit-elle dans le cieiir du léaeleur?
parce que c’est celle de l'isi'tope '**U. l’isotope dUranium le plus
SOUITION Voici les tontepls tiés qu'on doit iitiliseï pour résoudre le abondant, il laiil plutôt supposer que la masse de chaque atome de
problème : 1 1 les fissions fournissent une puissance F de 3 4(X) MW ^” U en unités de masse atomique est égale au nombre de masse A.
372 Chapitre 13 L’énergie nucléaire

Donc, la ma.s.se de chaque atome de est 235 u (—3,90 x 10 kg). e) À quel taux la masse est-elle convertie en d’autres formes d’énergie
Donc, le taux auquel le combustible disparaît est par 1a fission du dans le cceur du réacteur ?
SÜlUTION; Ici, le concept tié est que la conversion de l’énergie au repos
■(1,33 X 10-’" alomc.s/sK3,90 x 10 kg)
ih en d’autres formes d ’énergie est uniquement liée à la fi.ssion qui
= .5,19x K )'=kg/s»4,5kg/j. (réponse) fournit la puissance (3 400 MW), non à la capture sans fission
des neutrons (bien que ces deux processus inl lucnccnt le taux de
d) À ce taux de combustion, combien de temps la ré.\crve de com­ consonimaiion de ■^’L'). Donc, en s’inspirant de la relation d’Einstein
bustible durera-t-elle? £), = TOC-, on peut écrire
SOlUIlON; On Sait déjà que la niasse totale de correspond à 3,0%
des 8 ,6 X 10^ k g d’oxyde d’uranium Le concept clé ici est donc que dm d L ,)/d i 3,4 X 10‘^ W
le temps T requis pour consumer cette masse totale de au taux ~di c~ (3,00 X 10« m / s Ÿ
constant de 4,5 k g /j est ^ 3.8 X I0~* kg/s
(0.030)(8.6 X 10^ kg) = 3.3 g/j. (réponse)
T = »,570j. (réponse)
4.5 kg/i
F,n pialiciiic. il faut remplacer le,s crayons de combustible (liabituclle- On constate que le taux de conversion de la mas.se corre.spond à
nient en groupes) avant que le ■'"U qu’ils contiennent soit entièrement environ la masse d ’une pièce de monnaie par jour, cc qui est
consumé. considérablement mcin.s que le taux de consommation calculé en c).

13.5 Un réacteur nucléaire naturel


Le 2 d éc e m b re 1942, q u a n d leur p re m ie r ré a c te u r e st d e v e n u o p é ra tio n n e l (fig u re 13.8),
Elit ieo F e rm i e t .scs c o lla b o ra te u rs é ta ie n t en d ro it d e su p p o se r q u 'ils a v a ie n t c o n çu le
p re m ie r ré a e te u r à fissio n d e l’h isto ire d e la T erre. C ep e n d a n t, e n v iro n .30 an s p lu s tard,
u n a d é c o u v e rt q u e , s ’ils l ’o n t v ra im e n t p en sé, ils é ta ie n t d a n s l'e rre u r.
Il y a q u elq u e d e u x m illiard s d 'a n n é e s, d ans un d ép ô t d 'u ra n iu m m ain ten an t ex p lo ité
p rès d e F ra n c e v ille a u G a b o n , e n A friq u e é q u a to ria le , u n ré a c te u r à fissio n n atu rel s 'e s t
apparem m ent m is en m arche et a peut-être fonctionné d u ran t plusieurs centaines d e m illiers
d 'a n n é e s av a n t de s'é te in d re . Il e st p o ssib le de v érifie r si c e t é v é n e in e n t s 'e s t réellem en t
p ro d u it e n ré p o n d an t à d eu x q u estio n s.
1. Y a va ii-il assez de com bustible ? D an s un réacteu r à fissio n d 'u ra n iu m , le co m b u stib le
d o it ê tre IT so to p e fa c ile m e n t fissile , q u i n e c o n stitu e q u e 0 .7 2 % d e l'u ra n iu m
n a tu re l. C c ra p p o rt a c lé m e su ré d a n s d e s é c h a n tillo n s te rre stre s, d a n s d e s ro c h e rs
p ro v e n a n t d e la L u n e et d a n s d e s m é té o rite s ; d a n s to u s les cas, le ta u x e.st le m êm e.
L’in d ice qui a m e n é à celte d é c o u v e rte au G ab o n éta it q u e l’u ran iu in d e c e d ép ô t était
d é fic ie n t en -^^U, c e rta in s é c h a n tillo n s a ffic h a n t d e s ta u x a u ssi fa ih lc s q u e 0 .4 4 % .
L es é tu d e s su g g é rè re n t q u e c e d é fic it e n ' ’'U p o u rra it s ’e x p liq u e r si, à u n e c e rta in e
é p o q u e , le ’-’ ‘’U a v a it é té p a rtie lle m e n t c o n s u m é p a r l’o p é ra tio n d 'u n ré a c te u r à
fi.ssion n atu rel.

Figure 13.8 Tableau tnoiurani le premier


réacteur nuc.léaii’e. a“iscniblc ix-ndaiit
la Seconde Gue rre mondiale sur
lin terrain de squash à rUniversité
de i'hicago par une •'■qiiqx'. «lirigcc
par Fnrii-o Fermi. Ce réacteur, devenu
opiSiflIionm-l le 2 dcccnibie 194?..
a été consrriili avec des boules
d’uraniuin ciKasiiées dans des bl(x:s
de Kranitr 11 servir de prototypir
aux réacteurs ultériciii's, dont le but
était rie fabriquer du plutonium;
on se servait du plutonium pour
construire des armes nucléaires.
13 5 Un réacteur nucléaire naturel 373

U n sérieu x p ro b lèm e d e m e u re : si o n trav aille en fo n c tio n d ’u n tau x d ’iso to p e d e


se u le m e n t 0 ,7 2 % , la c o n s tru c tio n d ’u n ré a c te u r (c o m m e l’o n t a p p ris F e rm i et son
é q u ip e ) re p o s e s u r u n e c o n c e p tio n rig o u re u s e e t o n d o it p rê te r u n e a tte n tio n
sc ru p u le u .se a u x d é ta ils . L e s c h a n c e s q u ’u n ré a c te u r n u c lé a ire d e v ie n n e c ritiq u e
« n a tu re lle m e n t» se m b le n t m illes.
C e p e n d a n t, d a n s u n lo in tain p a ssé , le s c h o se s é ta ie n t d iffé re n te s. L es iso to p e s
et so n t to u s d eu x rad io actifs, et o nt d e s d em i-v ies resp ectiv es d e 7 ,0 4 x 10** a
e t d e 4 4 ,7 x 10* a. D o n c , la d e m i-v ie d e l'i.so io p e -'*1 J. q u i e st facilem eiu fis-sile, est
e n v iro n 6 ,4 fo is p lu s c o u rte q u e c e lle d e l’isotojie P u isq u e le ’“ II se d é sin tè g re
plus ra p id e m e n t q u e le il é ta it p lu s a im n d an t q u e c e d e rn ie r d a n s le p assé II y a
d e u x m illiard .s d ’année.s, e n fa it, s o n ta u x dan.s Г u ra n iu m n ’é la it ран d e 0 .7 2 'Л .
c o m m e m ain ten an t, m ais d e 3 ,8 %. C e qui co rre sp o n d au tau x d an s l’u ra m u m n atu rel
q u i e s t a rtilic ie lle m e n t e n ric h i p o u r s e rv ir d e c o m b u s tib le dans le s réacteurs
n u cléairas m o d ern es.
E n rai.son d e la d is p o n ib ilité d e c e c o m b u s tib le ta c ile m e n t fis s ile (c l d ’a u tre s
c o n d itio n s), la p ré se n c e d ’u n ré a c te u r n a tu re l e s t m o in s su rp ren an te. L e co m b u stib le
était là. Il y a d e u x m illiard s d 'année.s, la fo rm e d e vie la p lu s d év elo p jiée était I nlgiie
bleue.
2. Q uelle est la p reu ve de l ’existence de ce réacteur? L a .seule re d u c tio n d u dan.s
un d é p ô t d e m in erai ne p ro u v e p a s q u ’un ré a c te u r à tis.sion n atu rel ait exi.sté. Il fau t
c h e rc h e r de,s p re u v e s p lu s co nvaincante.s.
.S’il y a d éjà eu un réacteu r, il d e v ra it a u jo u rd ’h u i y a v o ir d e s p ro d u its d e tissio n .
P a rm i la tre n ta in e d ’é lé m e n ts q u i p ro d u ise n t d e s iso to p e s sta b le s d a n s u n ré acteu r,
c e rta in s d o iv e n t e n c o re ê tre p ré se n ts. L a m e su re de le u r a b o n d a n c e p o u rra it fo u rn ir
la p re u v e d o n t o n a besoin.
Pai ini le s n o m b re u x é lé m e n ts é tu d ié s, le c a s d u n é o d y m e e s t p a itin iliè ie m e n t
c o n v a in c a n t. L a fig u re 13.9 a ) d o n n e l’a b o n d a n c e d e s se p t iso to p e s d u n é o d y m e
q u ’o n tro u v e n o rm a le m e n t dan.s in n a tu re . tig u rc 1.1.9 b) d o n n e c e s a b o n d a n c e s
te lle s q u ’e lle s a p p arai.ssen t d a n s le s p ro d u its fin a u x s ta b le s tic la fis.sion d e ’-“ U
L es d iffé re n c e s n e sont p a s su rp re n a n te s, c o n sid é ra n t le s o rig in e s to ta le m e n t d iffé
ren tes d e s d eu x en.sem bles d ’iso to p e s. N o tez p a rtic u liè re m e n t q u e le '^^N d. iso to p e
d o m in an t d a n s l’é lé m e n t n am rel. e st ab sen t d a n s les p ro d u its d e fissio n .
L a p rin c ip a le q u e s tio n e st a lo rs la su iv a n te ; à q u o i re s s e iu b le n i le s iso lo fies
Figure n.9 La distribution selon d e n é o d y m e tro u v é s d an s le m in erai d ’u ra n iu m d u G a b o n ’’ Si un ré a c te u r n atu rel a
le nombre de masse des is*itopes ftinctionné à ce t en d ro it, on .s’attend à tro u v e r des iso to p es p ro v en an t d es deux sources
de néodyme tels qu’ils apparaissent (c ’e st-à -d ire a u ta n t d e s isotopexs n a tu re ls q u e d e s i.sotopes p ro v e n a n t d e là fissio n ),
a) dans les dépôts terrestres naturels L a fig u re 13.9 c) p rése n te les abondaiK Æ s d e c e s iso to p c.s; on a te n u co m p te ici d es
des minerais de eei élément et b) dans deux so u rces e t q u e lq u e s c o rre c tio n s o nt é té a p p o rtées aux d o n n e ra . La com pnrai.son
le combustible épuLsé d’une centrale d e s fig u re s 13.9 b ) et 13.9 c ) in d iq u e q u ’il y a b el et b ie n e u un ré a c te u r à li.ssion
nucléaire, c) l.a distribution (après n atu rel ay a n t fo n c tio n n é à c e t e n d ro it.
plusieurs corrections) du néodyme
trouvé dans la mine d’uranium L e fait q u e les p ro d u its d e fission de ce réacteu r naturel d u G ab o n n ’aient p as voy ag é Uiin
du Gabon, en Afrique équaionale. d u site d e le u r prcxliiction d e p u is d e u x m illia id s d ’an n é e s peut eo iisiiu iei un argiim eni
Notez que b) et c) sont pratiquement en fa v e u r d ’un enirep<>sage à lo n g term e d e d é c h e ts rad io aiaifs ilans un environneiT ient
identiques, et plutôt difterents de a). g é o lo g iq u e ad éq u at.

9.Ч
■]

Й? •20

y
I Ift-
<

И З 11-1 1L> Mb 148 И2 1)3 141 U .") I4f'» 148 150 112 143 144 140 148 1.30
Nombr« <lc; iiiabSi' Л Nomlrtf <!»■iiiasw .4 .Moinliic <1г mass»- Л
a) b)
374 Chapitre 13 L'énergie nucléaire

Exemple 13.3
Le rapport entre le -■’•''U et le dans un tlépôt d’uranium naturel N5(0) _ ,V5(0
est aujourd’hui (),(K)7 2. Quel était ce rapport il y a 2.0 x 10'' a?
Le.s demi-vie.s des deux isotopes sont respectivement de 7.04 x 10" a On calcule les ciaislanles de désintégration en utilisant l’équation 12.17
et de 44,7 x 10" a. cl les valeurs des demi-vies, ce qui donne
SOIUTIOH. Ici. te conrept clé est que le rapport entre le ‘’•'’L et In2 In2
X, = = 9.«.Sx 10 '«a '
le au temps t = 0 n’était pas égal à 0,007 2 (rappon actuel, T l/ 2 .5 7,04 X 10« a
au temps t = 2,0 x 10" a) parce que ces deux isotopes se sont
désintégrés à des taux diliérents. Soit Ai',(0) et A/r(0), les nombres In 2 In 2
et Ak = = l,.S5 X 10l-lll 0-1
d’i.sototxxs dans un échantillon d’uranium à / ~ 0. et .V,(r) cl les T'i/J.s '44.7 X 10« a
nombres d’isotopes à un temps ultérieur t. Donc, on peut utiliser L'exposant de l’équation 13.8 est alors
l’éiiualion 12.14 pour écrire le nombre de chaque isotope au temps i
(À5 - As )f = ((9.8.S I..S5) X 10- a - ‘](2 x 10" a)
par rapport au nombre au temps / = 0 :
= 1.66.
/V,(/) - AT,(0)ir~'»' et WsO) - tVgiOjf -Afif
L’équation 13.8 donne
OÙ A, et sont les constantes de désintégration correspondantes. ,Vs(0) Als(/)
Si on les divise, on obtient _ (0.007
TV8(0j W«(/)
•RU = 0,037 9 = 3,89?'. (réponse)
TVs« A^x(0) I x rapport entre le et le - ’"U était encore plus élevé (environ
Puisqu’on cherche le rapport /V,(0y/Vn(0). on réarrange cette équation 30% ) à l’époque de la formation de la Terre, il y a 4,5 milliards
d’année.s.

13.6 La fusion thermonucléaire :


principe de base
La courbe de l’énergie de liaison parn u eléo n de la figure 12.6 dém ontre qu’il peut y avoir
dégagem ent d ’énergie si deux noyaux légers se com binent pour form er un noyau plus
gros, processus ap p e lé f u s io n n u cléa ire . C e p ro c e ssu s est in h ib é p a r la rép u lsio n
coulom bienne qui em pêche les deux particules chargées positivem ent de se rapprocher
su lilsam m en t po u r se tro u v er dans la portée d e leurs interactions fortes attractives et
« fu sio n n er» . La h au teu r de c e tte b a r r i è r e c o u l o v i h i e n n e d ép e n d d es ch a rg es e t des
rayons des deux noyaux en interaction. D ans l’exem ple 1.3.4, on dém ontre que. dans le
cas de deux protons ( Z ~ 1 ). la hauteur de la barrière est 400 keV. 14ans le cas d e par­
ticules ayant des charges plus élevées, la barrière a, bien sûr, une hauteur supérieure.
Four générer des quantités utiles d ’énergie, la fusion nui léairc doit se protUiire dans
une m atière solide. La m eilleure façon de ptxxluire cette fusion est d ’élever la tcmiiératuTC
du m atériau ju sq u ’à ce que les particules possèdent suffisam m ent d ’énergie (en raison
de leurs seuls m ouvem ents therm iq u es) pour p én étrer dans la b arrière coulom bienne.
Ce processus est apjselé fu sio n th e rin o n u c lé a ire .
D ans les études des pliénom ènes therm onucléaires, les tem pératures sont exprim ées
en énei^ie cinétique K des particules en interaction ; cette expression découle de la relation

K = kT. (13.9)

où K est l’énergie cinétique correspondant à la vitesse Ut p l u s p r o b a b le de.s particules en


jcii et k est la constante de B o lt/m an n (voir l’annexe B) ; la tem pérature T est exprim ée
en kelvins. D onc, au lieu de dire « la tem pérature au centre du Soleil est 1,5 x 10’ K ».
il est plus courant de dire « la tem pérature au centre du Soleil est 1,3 keV ».
L a te m p ératu re am b ian te c o rresp o n d à ~ 0 .03 cV ; une p a rtic u le p o sséd an t
seulem ent eette én erg ie ne pourrait su rm o n ter une b arrière aussi hau te que 4 0 0 kcV.
p ar e.xetnple M êm e au ce n tre du S o leil, où k T = l.T keV . les p o ssib ilités de fusion
lh erm onueléaire ne sem blent pas p rom etteuses à p rem ière vue. On sa it que la fusion
therinonucléaire non seu lem en t se produit dans le creiit du Soleil, m ais q u ’elle e st le
principe dom inant de ce txtrps et de toutes le.s autres étoiles.
13.6 La fusion thermonucléaire príncipe de base 375

Figure 13.10 La courbe nommée n{K) donne le nombre volumique (le nombre par unité de volume)
de protons par unité d’énergie au centre du Soleil. La courbe nommée p (K ) donne la probabilité
de pénétration dans la barrière (et, ainsi, de lu.sion) par rapport aux collisions proton-proton
à la température du cœur du Soleil. La dmite verticale indique la valeur de k T à celte température.
Notez que les deux courbes sont tracées à des échelles verticales arbitraires (différentes).

C e casse-tête est résolu q uand on co n state d eu x fa its: 1) l’cn crg ic calcu lée avec
r é q u a tio n 13.9 est c e lle d e s p a rtic u le s ay an t la v ite sse la p lu s p ro b a b le ; il y a des
pailicules qui possèdent des vitesses beaucoup plus élevées et, [iarconséi|iieni, ties éner­
gies beaucoup plus élevées ; 2) les liaiitcui.s d o hai ricrcs q u 'im n ca k iilcc.'« icpicv.aitcnl
leurs p ic s. La trav ersée d es b arrières par e flé t tunnel p eu t sc p ro d u ire à des énergie»
considérablem ent inferieures à celles de ces pies, com m e dans la désintogniiion alpha,
principe q u 'o n a vu dans la section 12.4.
La figure 13.10 en fait la .somme. La courbe nonunée «(A') y repiéseutr la distribution
de Maxw ell des protons dans le cœ u r du Soleil ; elle est tracée ptiiir c o rn siTondrr à In
te m p ératu re ce n trale du S o leil. P lu s sp é cifiq u em en t, l ’ex p re ssio n ni K ) d K d o n n e lu
probabilité q u ’un proton ait une énergie cinétique se situant entre A et A -t à K . La valeur
de k T dans le cœ ur du Soleil est indiquée par la droite verticale dan.s la ligure ; notez que de
nombreux prtrtons dans le cteur du .Soleil possèdent des énergies siipéiieiiies il cette valeur
La courbe nom m ée p(K ) dans la figure 13.10 représente la probabilité de penetration
dans la barrière de deux protons en collision. Les deux couibcs de cette figure suggèrent
q u ’il ex iste une énergie p articu lière du proton à laquelle les fusions p ro to n -p ro to n se
p ro d u isen t à un taux m ax im al. À d es én e rg ies bien su p é rie u re s à ce tte v aleur, la
barrière est suffisarajTiem transparente, m ais trop peu de protons possèdent ces énergies
pour que la réaction de fusion puisse être m aintenue. Pai contre, de nom breux protons
po ssèd en t des én erg ies très inférieures à cette valeur, m ais la b arrière co ulom bienite
est trop im portante.

^VÉRIFIEZ VOS CONNAISSANCES 2 . Laquelle des réactions de fusion potentielles suivantes


n e se soldera/v/s par un dégagement d’energie nNiilianic - a) '’Li + "Li. h) ^lle + '1 le I- ^llr.
c) d) + ^'Ne, c) ^^Cl + ’•‘’Cl ou f) '''N + ” C1 ? (In d ice ■Observe/ la ( oiirbe
d’cticrgic de liaison de la figure 12.6.)

Exemple 13.4
Supposez qu'un proton est une sphère ayant un rayon A « 1 fm. Deux
protons sont projetés l’un vers l’autre avec la meme énergie cinétique K. lÔTTPoA
a) Quelle doit être la valeur de K si les particules .sont immobilisées (1 Y .0 k 1 0 -''’ ( V
par leur répulsion conlomhiennc mutuelle à l’instant où elles se (1 6 jr)(S ,8 5 X 10 ‘2 b /m )(l x 10 m)
« louchent » ? On peut con.sidércr cerfe valeur de K comme une 5.7.5 X 10 .1 = 360 keV (SI 0,4 MeV (réiainse)
mesure repiéscnrative de la hauteur de la barrière coulombienne.
b) À quelle température un jiroioii contciui dans un gaz de proioii.s
SülUIlOh’ Le tontef» tié utilisé ici est le suivant : l’énergie mécanique h aura l'énergie cinétiiiuc moyenne calculée en a) cl, de ce fait, aura
du système de tieux pndons est amservée quand ceux-ci se dirigent l’un donc une énergie égale à la hmitenr de lu hurrière eoulombienne
vers rautre et s’immobilisent momentanément. Plus particulièrement,
l’énergie mécanique initiale h, est égale à l’énergie mécanique A, SOLUTION: Ici, le tontepi clé consiste à considérer le gaz de protoivs
comme un gaz parfait; donc, l'énergie moyenne des protons est
•à rin.slaiil où les particules s’immobilisent L’énergie initiale A,
n’est consiituéc que de l’énergie cinétique totale 2K des dtaix protons, ^inu) ~ 2 ^ ^ ^ cnnstaiiic de nolt/inaim (réqiiation 13.9
donne l’énergie cinétique associée à la vitesse la plus probable,
fjnand les protons s ’immobilisent, l’énergie Af n ’esi con.siituée
et non l ’énergie cinétique moyenne). St on isole T dans cette ét]iiui ion
que (le l’énergie potentielle élettriqiie U du système, comme on le
et qu’on utilise le résultat obtenu en a), on a
détermine à l’aide de réqualion 4.43 du volume 2 (ri = q^qdA ne,/).
Ici, la distance rqui sépare les protons à l’instant où ils s'immobilisent ? a;moy (?>(5 75 X 1 0 -”' ,1)
est la distance entre leurs centres. 27?, et leurs charges Çi et sont 7 =
,3A (3 )(l.3 8 x 10 23.1/K)
toute.s deux égales à la charge élémentaire e. On peut donc exprimer
3 X 10® K (rcpoiisv)
la conservation d'énergie A| “ Af ainsi :

1 La température du cœur du Soleil n’est apiiniximaiivemem que dr


2A = -----— 1,5 X 10^ K , il c.sl donc cluir que la fusion clans le cœur du Soleil doit
4iTfio 27?
mettre en jeu des protons possédant des énergies de loin siipérlemes
Ce qui donne, d’après les valeurs connues. à l’énergie moyenne
376 Chapitre 13 L'énergie nucléaire

13.7 La fusion thermonucléaire dans le Soleil


et dans les autres étoiles
L e S o le il ra y o n n e d e I’c n e rg ie à iin la u x d e 3 ,9 x 10^^ W d e p u is p lu s ie u rs m illia rd s
d ’an n ées. D ’où vient to u te c ette én erg ie ? L a co m b u stio n c h im iq u e e st e x clu e ; si le Soleil
était cotTiposc de charbon e t d ’oxygène (dans des proportions adéquates p o u r la com bustion),
il n ’au rait d u ré q u ’un m illier d ’années. U n e au tre p o ssib ilité serait que le Soleil rétrécisse
len tem en t .sous l’actio n d e ses p ro p res forces g rav itatio n n elles. E n tran sfo rm an t l’én erg ie
p o te n tie lle g ra v ita tio n n e lle e n é n e rg ie th e rm iq u e , il p o u rra it m a in te n ir sa tem j'iératu ie
e t c o n tin u e r à ray o n n er. L es c a lc u ls d é m o n tre n t to u te fo is q u e c e m é c a n ism e n ’e st p as
le b o n ; il p ro d u ira it u n e d u ré e d e v ie so laire tro p co u rte p a r u n fac te u r d ’au m o in s 5(K).
C e q u i n e la is s e q u e la fu s io n n u c lé a ire . Le S o le il, c o m m e v o u s le v e rre z , n e b rû le
p a s d u c h a rb o n , m a is d e l’h y d ro g è n e , e t d a n s u n e c h a u d iè re q u i n ’e st ni a to m iq u e n i
c h im iq u e, m ais b ien n u cléaire.
La ré a c tio n d e fu sio n d a n s le S o leil e st u n p ro c e ssu s c o m p o rta n t p lu sie u rs étaj'ies
d an s leq u el l’h y d ro g è n e esi tra n sfo rm é e n h éliu m , l’h y d ro g è n e é ta n t le « c o m b u s tib le »
et l’b é liu m éta n t les « c e n d re s » . L a fig u ie ! 3.11 illu stre la c h a în e p r o to n - p r o to n (p -p )
q u e c e p n x e s s u s suit.
L a c h a în e p -p c o m m e n c e p a r la c o llisio n d e d e u x p ro to n s ('H + 'H ) q u i fo rm e n t
u n d e u té ro n (^H ), a v e c c ré a tio n s im u lta n é e d 'u n p o s itro n ( e ’^) et d ’u n n e u irin o (v)
l e p ositron rencontre rapidem ent un électron libre (e ‘ ) d an s le S oleil, e t ces deux particules
s ’a n n ih ile n t (v o ir la sectio n 1.6 du v o lu m e 2), le u r én e rg ie au rep o s a p p a ra issa n t so u s la
fo rm e d e d eu x p h o to n s d e ray o n g a m m a (y ).
U ne p aire d e tels é v é n e m e n ts a p p a ra ît d an s la lig n e d u h a u t d e la fig u re 13.11. C es
é v é n e m e n ts so n t e x trê m e m e n t rares. E n fait, il n ’y a fo rm a tio n d ’u n d e u té ro n q u ’u n e
fo is to u te s les 1Ü-* c o llisio n s p ro to n p ro to n , e n v iro n ; d a n s la g ra n d e m a jo rité d e s cas,
les d eu x p ro to n s re b o n d isse n t sim p lem en t l’un su r l’au tre, d e m an ière é lastiq u e. C ’e st la
le n te u r d e c e p ro c e ssu s en « g o u lo l d ’é tra n g le m e n t » q u i ré g u le le ta u x d e p ro d u c tio n
d ’én erg ie e t em p êch e le S o leil d ’exploser. M alg ré cette lenteur, il y a tellem en t d e p ro to n s
d an s le gigante.sque e t d e n se v o lu m e d u c œ u r d u S o le il q u e le d e u té riu m y e st p ro d u it
ainsi au ta u x d e 10'^ kg/.s.
U ne fo is p ro d u it, le d eu téro n h eu rte ra p id e m e n t u n au tre p ro to n ci fo rm e un n oyau
d e 'H e , co m m e l’illustre la lig n e du cen tre d e la figure 13.11. D e u x n o y au x de 'H c peu v en t
fin a le m e n t (en 1 0 ' a, ils o n t su ffisa m m e n t d e te m p s) se tro u v er, fo rm a n t u n e p articu le
alp h a C 'H e) e t d e u x p ro to n s, c o m m e l’in d iq u e la d e rn iè re lig n e de la fig u re.
D an s la fig u re 1 3 .1 1, on v o it q u e la c h a în e p -p é q u iv a u t à la com binai.son d e q u atre
p ro to n s e t d e d e u x é le c tro n s p o u r fo rm e r u n e p a rtic u le a lp h a , d e u x n e u trin o s e t six
p h o to n s d e ray o n g a m m a , c ’e s t à -d iie
4 'H + 2 e ~ ■‘H c + 2iy + 6y. (1 3 .1 0 )
O n ajo u te d e u x é le c tro n s à c h a q u e m e m b re d e l’é q u a tio n 13.10 p o u r o b te n ir
(4 'H + 4 c - ) (“H e + 2 e ) + 2i^ + 6 y . (13.11 )
L es g ra n d e u rs e n tre les d e u x e n se m b le s d e p a re n th è se s rep ré.sen ten t a lo rs d e s a to m es
(n o n p a s d e s n o y a u x .seuls) d ’h y d ro g è n e e t d ’h é liu m . C e q u i p e rm e t d e c a lc u le r le
d é g a g e m e n t d é n e rg ie d e la ré a c tio n g é n é ra le d e l ’é q u a tio n 1 3 .1 0 ( e t d e l’é q u a tio n
13.11) a in si;
( ) ------Aw
= -|4 .(K )2 6 0 3 u (4)( 1,007 82*5 u )l(9 3 l,5 M eV /u )
= 26,7 M eV .

‘n • * - +•r ^ + » ((2= 0.42MeV) *H ^ ‘H + e+f »• 0.42 MeV)


e r e"-4 y + y Cß L02 MeV) ,,+ 4. y I- y (Q ■= l.OS MeV)

Rqurc 13.11 La chaîne proton-prolon


qui explique la production d'énergie
dans le Soleil. Dan.s ce priK'essus. "H ^ ‘a - » t e H y t Q = 5.40 MeV) + ‘H-> ’H c + y -5,4i) MeV)
les protons fusionnent |wur former
une particule alpha •'■'He), chaiain
des événements dégageant une énergie * .. . . ..t _ ''
nette de 26.7 MeV. ?Hc t ^H r-4 'H c 1^ 'H 1- 'H « 2 = 12.86 MeV)
13.7 La fusion thermonucléaire dans le Soleil et dans les autres étoiles 377

où 4,002 603 U est la mas.se d ’un atom e d ’hélium et 1,007 825 u est celle d ’u n atom e
d 'h y d ro g è n e . L es neiitrinos ay an t tout au p lu s u ne m asse n ég lig ea b le et les p h o to n s
de rayon.s gam m a ay an t une m asse n ulle, ils n ’en tren t pas dans le calcu l de l ’én erg ie
de désintégration.
C ette m em e valeur de Q découle (com m e il se doit) de l’addition des videurs de Q
dans les étapes distinctes de la chaîne protoii-jirotoii de la figure 13.11. D onc.
Q = (2)(0,42 M eV ) + (2)(1,02 M eV ) + (2)(5,49 M eV ) + 12,86 M eV
= 26,7 McV.

D e cette quantité d 'én erg ie, environ 0..5 McV csi liansixtilé b o ts du Soleil pai les deux
neutrinos présents dans les équations 1 3 .1 0 el 13.11 ; le reste ( = 26.2 M eV) se dépose
d ans le cœ ur du S oleil sous form e d ’én e rg ie th erm iq u e. C’ette én erg ie th e rm iq u e est
e n su ite g ra d u e lle m e n t tran sp o rté e vers la su rfa c e du S o leil d ’où e lle la y o n iie en
s'éloignant sous form e d ’ondes électrom agnétiques, dont la lumière visible luit iw lie .
La com bustion de l’hydrogène dans le cœ ur du S oleil est analogue à de l'aleh im ie
à g ran d e é c h e lle q uand on co n sid ère q u ’un élém e n t e.si iian .sfn in ic eu un u iilir.
C ependant, les alchim istes du M oven  ge cherchaient davantace ù changer le plom b en
or que l’hydrogène en hélium . D ans un .sens, ils étaient sur la Ixmiic voie, sauf que leurs
fo u rs n ’étaie n t pas assez ch au d s. A u lieu d etre à une te m p é ra tu re de 6 0 0 K. p ar
exem ple, ils auraient dû atteindre une tem pérature d 'a n mtiins 10^ K
Il y a com bustion de l’hydrogène dans le Soleil depuis environ 5 x 10'^ a ; les calculs
dém ontrent q u ’il reste suffisam m ent d ’hydrogène p o u r que cet astre brille encore autant
de tem ps dans le futur. Ainsi, d an s cinq m illiards d 'an n ées, le ciru r du Soleil, qui sera
alors largem ent compo.se d ’héhum . com m encera à se refroidir et le S oleil com m encera
à s'effondrer sur lui-même en raison de sa propre gravité. Ce phénom ène fera augm entci
la température du cœ ur et provoquera une expansion de Tenvelopjve externe, tranvformant
le Soleil en ce q u 'o n appelle une /g éa n te ro u g e .
Si la tem pérature du cœ u r atieinl envim n UT" K, la fusion peut encore prtxluirc de
l’én erg ie, cetlc fois p a r la co m b u stio n de l’héliu m , qui form e du carb o n e A m esure
q u 'u n e étoile év o lu e d avantage et devient encore plus chaude, les réactions de fusion
peuvent form er d 'a u tre s élém ents. Ce|Tendaiu. les clém ents plus m assifs que ceux situes
près du pic de la courbe de l'én erg ie de liaison illustrée dans la figure 12 6 ne peuvent
être produits par des fusiorus ultérieures.
On croit que le.s élém ents ayant des nombres de ma.sse qui vom au-delà de ce pic sont
form és p ar la capture de neutrons pendant les explosions stellaires cataclysm iques q u 'o n
appelle s u p e r m v a ' (figure 13.12). A u cours d 'u n tel événem ent, la couche ex tern e de
l'étoile est soufflée au loin, où clic se m êle au milieu ténu qui remplit l’c'space interstellaire
Fiÿwe 13.12 a) i .’étoile appelée
C 'e st à p an ir de ce milieu, continuellem ent enrichi p ar des débris d'explosions stellaiies,
Sanclulcak telle qu’elle apparaissait
q u e les n o u v elles éto ile s sc fo rm en t p ar effo n d re m en t so u s l'in flu e n c e de la force
jusqu'en 1987. b) À cette date,
gravitationnelle.
la lumière de sa supernova est parvenue
L 'abondan ce, sur terre, d 'élém en ts plus lourds que l ’h ydrogène ci rh é liu m lal.ssc
jusqu’ici; l’explosion fut 100 millions
penser que le systèm e solaire s'c.st form é à partir de m atière interstellaire contenant des
de fois plus grosse que le Soleil
et pouvait être vue à l’teil nu. Cette restes de id lc s explo.sions. P ar conscqucnl, tous les élém ents qui vcas entourent (y compris
explosion a eu lieu 11 I ‘>5 000 années- ceux qui com|Xisent votre organism e) ont été fabriqués à l'intérieur d ’étoiles i|ui n existent
luniièrc de la Terre ; elle .s’est donc plus, l 'n .scientifique a déjà dit que les êtres hiiniains sont, en vérité, les cnfanl.v des élnilcs
produite il y a 155IHX) ans.
378 Chapitre 13 L énergie nucléaire

Exemple 13.5
À quel taux dm /dr l’hydrogène cst-il consume dans le cœur du Soleil que 26.2 MeV ( = 4,20 x 10^^^ J) d ’énergie thermique sont
par la chaîne p-p illustrée dans la figure 13.11 ? produits lorsque quatre protons sont consumés, c ’esl-à-dire que
SOIÜIION: Le (ontepi dé utilisé ici est le suivant ; le taux d tJ d t auquel la A/:' = 4.20 X 10“ ’’ J pour une consommation ilc masse de
Am = 4(1.67 X 10'^’ kg) St on in.sère ces données dans l’équa­
consommation »l’hydrogène (proton) produit de l’cncrgie est égal
tion 13 .12 et q u ’on utilise la luminosité L du Soleil donnée dans
au taux L (qu’on appelle la luminosité du Soleil) auquel le Soleil
l’annexe C. on constate que
rayonne l’énergie ;
l‘îL
dl dm ÙJI1 4(1.67 X 10--^ kg)
(З.иО X 10^^ W)
~dt ~ ~ ~4.20 X 10 12 J
Pour in,scier le taux de consommation de la musse dni/d t dans cette
équation, on peut la récrire ainsi : = 6.21 X lO' kg/s. (réponse)

dK dE dm ^ \ E dm
L = (13.12) Donc, une gigantesque quantité d ’hydrogène est consumée pur le
dt dm dl Д т (Il
Soleil ù chaque seconde. Cependant, tic craignez pas que le Soleil
où Ait’ est l’énergie produite quand les protons de masse à m éptii.sc son hydrogène, car sa masse de 2 x lO-’i’ kg lui [lennettra de
sont consumés. Dans le texte de cette section, il est démontré briller durant très, très longtemps.

13.8 La fusion thermonucléaire contrôlée


La prem ière réaction therm onucléaire .sur terre s ’est produite dans l’atoll F niw clok le
I "■novem bre 19 5 2 , quand les ElaLs-l nis ont fait exploser une bom be à fusion, générant
un dégagem ent d ’énergie équivalent à 10 m illions de tonnes de TNT. L es tem peratures
élevées et les densités d e particules nécessaires pour d éclen ch er cette réaction ont été
atteintes à l’aide d ’une bom be à fission qui a servi d ’am orce.
Il est considérablem ent plus diiricilc de concevoir une source a in iin u c et contrôlable
de fusion (un réacteu r à fusion faisant partie, p ar exem ple, d ’un e centrale électrique).
Plusieurs pays du m onde poursuivent néanm oins sans relâche cet objectif, car nom breux
sont ceux qui co n sid èren t le ré a c te u r à fu sio n co m m e la so u rce d ’én e rg ie du futur,
du m oins pour la production d ’clcclricitc.
La chaîne (>p présentée dans la figure 13.11 n ’est pas appropriée pour un réacteur
à fu sio n te rre stre parce q u ’e lle rep rcse n lc un p h én o m èn e b ea u co u p tro p lent.
C e prtK essu s ne se p ro d u it q u ’à r in té r ie u r du S tdeil en raiso n de l ’é n o rm e d en sité
des protons s’y trouvant. Sur terre, l’utilisation de deux réactions deutércMi-deutéron (d d)
sem ble plus réalisable,
2H -f 2H ) -y n [Q = + 3 ,2 7 M eV ), (13.13)

4 2Ц + 41 (G = + 4 ,0 3 M eV ), (13.14)
tout com m e la réaction deutéron-trilon (d-t)’к
-H + -4 '‘He -f n (G - 4 iz .-iO M c V ). (1.3.15)
L ’iso to p e d eu tériu m , so u rce d e d élitero n s p o u r ces réa ctio n s, a une ab o n d an ce de
seulem ent 1 partie par 6 700 dans les échantillons d ’hydrogène, m ais il est disponible en
quantité ilLm itée en tant que com posante de l’eau de mer. Les prom oteurs de l ’énergie
nucléaire décrivent le choix énergétique iillimc de l’clrc hum ain (lorsque le com bustible
fossile sera épuisé) com m e « la combiistitiii de rochers » (fission »le l’uranium extrait de
m inerais) ou « la combu.stion de l’eau » (fusion du deutérium extrait de l’eau).
T rois condition,s d o iv e n t être rem p lies p o u r q u ’un ré a c te u r th e rm o n u c lé a ire
fonctionne.
1. U n n o m b r e v o lu m iq u e n é le v é . Le n o m bre v o lu m iq u e d e p articu le s in te rag issan t
(nom bre de deuiérons par unité de volum e, pai exem ple) doit être assez grand pour
p e rm e ttre un tau x d e co llisio n s d d su ffisam m en t im p o rta n t. A ux te m p ératu res
élevée.s ret]iiises, le deulcriiini serait com plètem ent ionisé, form ant un p la sm a neutre
(gaz ionisé) de déliterons cl d'électro n s.
2. U n e te m p é r a tu r e d e p la s m a é le v é e T . Le plasm a doit être chaud. Sinon, les deutérons
en otillision ne posséderaient pas l’énergie suffisante pour pénétrer dans la barrière
roiilom bieiine qui tend à les m ain ten ir élo ig n és lés uns des autres E n laboratoire,
on a réussi à atteindre une tem pérai ure de plasm a ionisé de S.*! keV^ ».-orrespondant
à 4 X 10" K, soit environ .30 fois celle du centre du Soleil.
Ir noyau de l’isotope d’hydrof.cnc *H (Iriiium) c.cl appelé triton. C es! un rachonudeidr
avani une demi vie de I7..3 a
13.8 La fusion thermonucléaire contrôlée 379

3. U n e lo n g u e ¡x rifM Îe d e c o n fin e m e n t t . Un problème majeur consiste à maintenii' le plasma


chaud à une densité et à une température suflisam m cnt clcvécs pour perm ettre la fusion
d’une quantité appréciable de com bustible. Pour c e faire, on doit conserver le plasm a
chaud a.sse/ longtem ps, et il est clair q u ’aucun coiUciianl solide n e p eu t su p p o rter
les te m p ératu res req u ises ; il est d o n c n éc essaire de co n c ev o ir des te ch n iq u es d e
coiifincm ent appropriées; on verra bientôt deux d ’entre elles.

Il p eu t être d ém o n tré q u e, p o u r q u ’un ré a c te u r tlrc rm o n u c lc airc fo n ctio n n e


en em ployant la réaction d-t, il est nécessaire d ’avoir

HT > s /m \ (13.16)

C e tte co n d itio n , ap p e lé e c r i t è r e d e L a w so n , in d iq u e q u ’on n le ch o ix de c o n h n e r


une grande quantité de particules durant une courte période ou d 'e n confiner peu durant
une longue péritxle. O utre ce critère, il est nécessaire que la lem pérature du plasm a soit
figure 13.13 L e réacteur d ’e ssa i de fu s io n
suffisam m ent élevée.
T o k a in a k à r i . ’n iv c rsité de P rin c e to n
Deux m éthodes de arn tm lc de l'énergie nucléaire font actuellem ent l'o b je t d ’études.
Bien q u ’aucune n e soit encore concliianle. on eti poursuit l ’étude p arce q u 'e lle s sont
prom etteii.ses et p arce que la fusion co n trô lée o ffic un ixitcnlicl im portant q u an t à la
résolution des problèm es énergétiques m ondiaux.

Le confinement magnétique
Dans une version de cette m éthode, un cham p m agnétique ayant une ftirm e puriiculiére
confine un plasm a chaud dans une cham bre tom ïdale sous vide a|ipel«- to k aiiïak (ce nom
est une contraction de trois mots russes). I æs forces magnétii)iies agi.ssant sur les particules
chargées qui form ent le plasm a chau d em pêchent le plasm a d e toucher les parni.s de la
cham bre. La figure 13.13 présente un tel appareil ; il se trouve au laboratoiie des plasmas
de l’U niversité de Princeton.
O n chauffe le plasm a en y induisant un coûtant cl en le bom bardant avec un faisceau
de particules accélérées à rextéricur. Le prem ier objc-ctif de cetts médiode est d ’atteindm
l’é q u ilib r e é n e r g é tiq u e , c e qui se p ro d u it q u an d le critère d e I .aw snn est atteint nu
dépassé. L ’o bjectif ultim e est l’ig n itio n , qui correspond à une réaction iherm onucléaire
a u to su ffisa n te , ac co m p ag n ée d ’une p ro d u ctio n n eü e d ’é n e rg ie. Lti 200fl. l'ig n ifio n
n’avait pas encore été réussie, ni dans un tokaniak ni dans un autre ajuxuril de confinement
m agnétique.

Le confinement inertiel
C eue m éthtxle de confiiiemenl et de réchaiiffeinent du com bustible de fusion pour o btenu
une réaction therm onucléaire consiste à «bom bardei » une pastille de unnbu.sliblc solide;
de tous les côtés avec d ’intenses faisceaux la.sci, ce qui fait éva[xnrr une certaine quantité
de matière de la surface. Cette m atière cngcndic une onde de choc qui se dirige vers f i n
térieur et com prim e le cœ ur de la pastille, augm entant à la fois la den.sité do kor pariieules
et sa tem pérature. C e processus est ap|telé co n fin m n n m r in e r tie l parce que 1c coiiibiisiiblc
est rtw ^ w 'd a n s la pa.siillc cl que. en rai.son de leiii in e r tie (leur m asse), les particuie.s ne
.s’échappent pas de la p astille ch au ffée du ran t le très co u rt laps de temps que dure Je
bom bardem ent.
La fu sio n p a r laser, utilisant le confinem ent inertiel, est étudiée tlaas de nom hm ux
laboratoires aux États- Unis et ailleurs. P ar exem ple, au laboratoire L aw rence l iveinore
des pa.siilles de dcutcrium-tritiiitTi plus petites q u 'u n grain de sable (figure 13 14) «ont
b om bardées p ar 10 im p u lsio n s la.ser à h au te p u issan ce sy n c h ro n isée s et d isp o sées
symétri(|iiemeni auuHirdc chacune d elles. Le.s impulsions last>r sont conçues pour fournir
une é n e rg ie to ta le d e 20f) k,I à c h a q u e p astille en m oins d ’une n an o sec o n d e C ela
rep résen te un ap p o rt de p u issan ce d ’en v im n 2 x 10'^ W du ran t l ’im p u lsin n . ce qui
constitue, grossièrem ent, 100 fo is la capacité de production d clcctricitc (m aintenue)
dans le mernde '
Figure 13.14 L e s petites sphèrc.s présentes D ans un réacteur iherm oiuieléaire du type fusion par laser, 1rs p astilles île co m ­
su r une pièce de 2i> cents am é ricain s sont bustible doivent ex p lo ser (com m e ties bom lx s hydrogène itiitrialim s) i) un taux qui
d e s pa.siilles de cornbtisiible. d e iité riu in pourrait être de 10 à 100 par seconde. L.a faisabilité de la fusion par la.scr pour produire
tritium c o n ç u e s p o u r être u tilisé e s de f e n e r g ie th e rm o n u c lé a ire n ’av ait p as en c o re été d ém o n trée en 2000. m ais les
d a n s u n e c h a m b re à fiis io n au laser. rccherclies continuent à une cadence accélérée.
3£0 Chapitre 13 L’énergie nucléaire

Exemple 13.6
Supposez qu'une pastille de ct)mbusliblc dans un appareil de fu-sion où m., et » 1, sont le.s masses respectives d’un atome de deutérium et
par laser contient un nombre égal d ’atomes de deutérium et de iriiium d’un atome de tritium. On peut remplacer ces masses par les masses
(sans aucune autre substance). La masse volumique p = 200 kg/ni‘ molaires données ; ainsi
de la pastille est augmentée par un facteur de 10' par l’aciion des Md M,
impulsions la.ser. i«d = et ni, =
/Va ■" ■“ Na '
a) Combien de particules par unité de volume (deutérons et tritoms) où est le nombre d’Avogadro. Après avoir effectué ces substitu­
la pasLlIc comprimée contient-elle ? La masse molaire A-/j des atomes tions et remplacé la ma.ssc volumique comprimée p ' par 1 OOOp.
de deutérium est de 2.0 x 10 ' kg/mol ; la masse molaire des on isole n dans l’équation 13,18 pour obtenir
atomes de tritium est de 3.0 x 10"' kg/mol.
2 000p /Va
SOLUTION: Ici. le cont^ dé est le suivant. dans le cas d'un système constitué Md -H M,
d 'un seul type de particule, on peut décrire la masse volumique ce qui donne
en employant la masse des particules et leur nombre volumique
(2 Ü(X))(200 kg/m-')(6.02 x 10=^^ m o l"')
(nombre de particules par unité de volume):
(2,0 X 10 ' kg/niol) (3,0 X lO '-' kg/miil)
ruimbre volumique H masse d’une paiticule,\ = 4.8 X 10-^' m"-’ (rdpon.se)
( masse volumique,
kg/m' )=(' m~’ kg )
(13.17) b) Scion le critère de Lawson, combien de temps la pastille doit-elle
m am tem ree nombre volumique de particules pour que l’équilibre
Soit n le nombre total de particules p«r unité de volume dans la énergétique soit atteint '!
pastille comprimée. Le nombre d’atomes de deutérium par unité de
volume est n/2. et le nombre d’atomes de tritium par unité de volume SOLUTION; Le coruept dé utilisé ici e.st le suivant : pour que l'équilibre
e.st aussi n/2. énergétique soit atteint, le nombre volumique comprimé doit être main­
Ensuite, on peut élargir l’équation 13.17 au sy.sième constitué de tenu durant une période r donnée par l'équation I3 .l6 (n r > lO^^s/m’).
deux types de panicules en décrivant la niasse volumique p ' de la pa.siille Donc, on a
comprimée comme la somme des ma.sses volumiques individuelles: 10^’ s/m ’
r > - = lü->2 s. (réponse)
4,8 X lO'i n i- î
n n
P = 2 "'d ^ 2 '" "
(13.18)
(La température du plasma doit egalement être suffisanimeiil élevée.)

RÉVISION ET RÉSUMÉ

Les processus nucléaires sont environ un


L 'é n e r g i e d u n o y a u bienne mutuelle, l.a fusion peut se produire d.ins la matière solide
million de fois plus efficaces, par unité de masse, que les processus seulement si la température est suffisamment élevée (c'est-à-dirc
chimiques pour transformer de la mas.se en énergie. si l’énergie de la particule est suffisamment élevée) pour que se
produise un effet tunnel appréciable.
1.'équation 13.1 illustre une fission du
I j i f i s s i o n n u c lé a i r e
L’cncrgic du Soleil provient principalement de la comhiwtion
induite par le bombardement de " 'U avec des neutrons Ihcrniiqucs.
ihcrnionucléaire de l'hydrogène, qui lorme de l’hélium en suivant
l.cs équations 13.2 et 13.3 clonneni les chaînes de désintégration
la chaîne p ro to n proton décrite dans la figure 13 II. Une fois
bêta des premiers fragments, l.’énergie dégagée par une telle fission
l’hydrogène conihuslihle d ’une étoile épuisé, d’autres proce.ssus
est Ç = 200 MeV
de fusion peuvent prtxluire des éléments ju.squ’à A ~ .Sfi (le pic de la
I.a fission peut être comprise à l’aide du modèle collectif qui
courbe d'energie de liaison).
compare un ixiyau à une goutte de liquide chargée portant une certaine
énergie d’excitation Pour que la fission se produise, une Karrière de L a fu s io n c o n tr ô lé e On n’a pas encore réussi à contrôler la
potentiel doit ctre traversée par cifct tunnel. La fissibilité dépend de fusion therm onucléaire pour générer de l’énergie. Les mécanismes
la relation entre la hauteur de la bamere et l’énergie d’exciialion £„. les plus prometteurs soni les réactions d d et d t. Pour qu’un réacteur
L.es neutrons libérés pendant la fission rendent possible une à fusi(>n fonctionne, il doit satisfaire au critère de I.au.son,
rrariin n PMl'Iiaînt' île fi.vsions. La figure 13.4 illustre l’équilibre des
neutron,s dans le cycle d'un léacteur iyp<‘ l-i figuie 13..5 pré.senie le
nT > 1 0 * ' s/nv’. (13.16)
schéma d'une centrale nucléaire complète. et le plasma doit avoir une température î ’suffiHiiniment élevée.
Le dégagement d'énergie généré par
f a fu s io n n u c lé a ir e Dans un tokainak, le plasma est confiné par un champ magné­
la füKlon do doux noyaux légers est inhibé par leur barrière coulom­ tique Dans la fusion p a r laser, on utilise le confinement inertiel
exercices et prabtèmes 381

. . *Î t i

1. Dans le tableau 13.1. la relation Q '= - A m s’applique-t-elle de contrôle pour réduire la puissance du réacteur à 25 % de sa valeur,
a) à tous les processus, b) à tous les proc &ssus sauf celui de la chute le facteur de multiplication est-il alors a) un peu inférieur à l’unité,
(J’cau, c) à la fission seulement, d) à la fission et à la fusion seulement '? b) substantiellement inférieur à f’uniié tai c) c-ncore égal à l’unité?
2 Selon la figure 13.1, la fission du par bombardement de neutrons 8. Le cœur d'un réacteur nucléaire doit avoir le plus |x;fit ni|)|ioii
thcmtiques en deux fragments de masses égales se produit dans environ .surfaceA'olume possible. Classez les solides suivant.s selon leurs rapporta
un cas sur a) 10 000, b) 1 000, c) lOO, d) 10. surface/volimic. en comnx-nvani par le rapixm le plus élevé a) un nilTt*
3. Les liragments initiaux formés par la fission ont-iLs a) plus de prtxons d ’arête a, b) une sphère de rayon a, c) un cône de hauteur u cl de
que de neuti'on.s. b) plus de neutnsns que de protoas ou c) à peu prés rayon de hase « ci di un cylindre de rayon a et de lumteiir « (1,’aire
le même nombre de neutrons et de protons d<’. la surface co u rb e du cône est i f î m r et son voluinc c.si rr«V3 )
4. Soit la réaction de fission 9. La figure 13.6 illu.stre cotnmeni la chaleur générée par les tlérliets
nucléaires produits par une année d operation dans une gronde ccntmlc
2.''U n -> X + Y I 2n. ma'léaire diminue avec le temps. Par quel facteur approvunatif ce
dégagement d'énergie thcrmiqia'; a-t-il diminué au Iront de 100 ans
Classez les nucicides suivants selon la possibilité qu’ils ont de repré­ a) 20, b) 2U0. c) 2 ÜÜO ou d) plus de 2 (XXI ?
senter X (ou Y), en comtnençant par le plus susceptible de le faire : 10. Lequel des éléments suivants n 'e st p u s produit par la fusion
a) '**’Nd. h) c) ’”'In, d) "^Pd. e) ‘"'Мо. ijn d ice: Voir la figure 13. J.) Ihermoniidéaire il l’iniérieur des étoiles : a) carbone h) stlicium,
5. Choisissez dans chacune des paires suivantes I’i.soiope le plus c) chrome, d) brome.
susceptible d’être un fragment initial formé par une fission; 11. Rnviron 2% de l'énergie générée dans le cieur du Soleil par la
a) ou '•'-'Ru, b) ‘“’'’Gd ou c) '■''^Nd ou ‘^’Lu. (In d ice : Voir la chaîne p-p asi transixnlée hors de l'étoile par Ir.s ncutriniw. L'énergie
figure 12.4 et Г annexe F.) associée à ce flux de neutrinos cst-cllc a) égale, b) supérieure ou
É. Supposez qu'un noyau de -'^*1? absorbe un neutron, pui.s se désin­ c) inférieure à l'énergie qui rayonne de la surface du Soleil sous
tègre, non par fission, mais en émettant un électron et un neutrino. forme de rayonnement électromagnétique ?
Quel nucléide cette désintégration produit-elle : a) ^^Pu. b) ■'■’^Np. 12. Le critère de Lawson ass<x:ié à la réaction d-i (équation 13.16)
c) ^"“Np ou d) -'"Pa? est HT > 10^’ s/m’. Dans le cas île la réai-tion d-d, vous atrendez-
7. Un réacteur nucléaire fonctionne à une certaine puissance, son vou.s à ce que le nombre fin membre île ilroiie .soit a) le même,
facteur de multiplication к étant réglé 5 l’unité. Si on utilise des barres b) inferieur ou c) supérieur?

EXERCICES ET PROBLÈMES

WW La solution se trouve sur le site Web, à l’adresse ci-dessous : 5E. Vérifiez si, comme on l'affirme dans la section 13.2, les neutrons
en équilibre avec la matière à une température ambiante de .300 K
www.dlcmcgrawhill.ca/physique
possèdent une énergie cinétique moyenne approximative de 0,04 cV
(Indice : l.'éneigie cinétique moyenne est Xnx.i ~ ik T /2 .)
SECTION 13.2 ta fission nucléaire : principe de base
6t. Calculez l ’énergie de désintégiaiion Q dans le cas de la fission
lE, a) Combien d'atom es 1.0 kg de pur contient-il ? b) Quelle de ‘'"Mo en deux novaiix identiques. I x s niasses que vous devez
énergie, en joules, est dégagée par la fission complète de 1.00 kg connaître sont V7,‘X)5 41 u pour le ‘'"Mo et 4X.050 02 u pour le ■‘‘'Sc.
lie ? Supposez que Q = 2(K) MeV. c) Combien de temps cette Si la valeur de O .s'avèic positive, tliics (xiungioi ce pmccvsiix ne .'ie
énergie aliincntcrait-elle une ampoule électrique de 100 W ? produit pas spontanément.
2E. Complétez le tableau suivant, qui se rapporte à une réaction 7(. Calculez l’énergie de désintégration Q dans le cas de la fission du
de fi.ssion typique f n X 4 V I- />n. ■
‘'^Ci en deux fragments égaux 1r s masses que viuis cleve/ ciimunlre
sont 5 1,V40 51 u pour le ’’■'Cr et 25.982 50 u pour le '"Mg.
X Y h 8t. L’isotope subit une désintégration alpha avec une demi vie
de 7.0 X 10" a 11 «e désintègre également (nirement) par fission
'4'Xe — 1 spontanée et. en l’ab.scnce de désintégration alplia, su demi vie
— 2 associée à ce seul processies serait de 3,0 '■ 10' ' a. a) À quel taux les
fissioas spontanées se priKluisenl-elles dans LO g de ? b) f omhion
— ‘""Zr 2
de désintégrations alpha de -^''L' y a-t-il à chaiitic fi.vsion spontanée
’•"Us — 9E Call liiez l'énergie dégagée par la réaction de fission

3E À quel taux les noyaux de doivent il .subir une fission par ’"‘U 4 n -4 ""Cs 4 "'Rb 4 2n.
iionibandciiieiit de neutrons pour générer une puissance de 1.0 W ?
Supposez que Q 200 McV. Voici les masses des atomes et dex panimlev dont vous ave-/ besoin
IL Les propriétés de fi.ssion de l’isotope ’^Fii sont très semblables à
(■elles de l’isotope L’énejgie moyenne dégagée par fis.sion est "■'U 235,013 92 u «'Rb 02,921 57 u
de 180 MeV. Quelle énergie, en MeV. e.st dégagée si tous les atome,s
contenus dans I .(K) kg de ■^"'Pu pur subissent une fission? '^'Cs 140.919 63 u II 1.008 67 II
38Z Chapitre t3 L'énergie nucléaire

10£. Vérifie/ si, comme l'indique le tableau 13.1, la fission de isolées. De tels radionucléides sont fabriqués en abondance dans les
contenu dans 1,0 kg de UÜ 2 (enriclii pour que le représente réacteurs nucléaires et peuvent être séparés chimiquement du
3.0% de l'uraniuni total) pourrait alimenter une ampoule électrique combustible épuisé. Le ‘’"Pu ( ’/ ’„2 = 87.7 a) est un radionucléide
de 100 W durant 690 a. approprié ; il subit une désintégration alpha dont Q = 5250 MeV. Quelle
IIP. Au cours d'une lïs.sir)n ijarticulière dans laquelle l'isotope est la puissance iliermique générée pai 1,00 kg de celle matière ?
subit une fission par bombardement de neutrons lents, aucun neutron 20P. (Voir le problème 19.) Parmi les produits de fission pouvant être
n’est émis et l'un des premiers liaginents de fission est l’isotope “-'Ge. extraits chimiquement du combustible épuisé d’un réacteur
a) Quel est l’autre tragment b) Comment l'énergie de désintégration nucléaire, on trouve le ‘" ’.Sr ( r ,;2 = 29 a). Cet isotope est produit dans
Q = n o MeV est-elle divisée entre les deux fragments e) Calculez les grands réacteurs nucléaires types au taux approximatif de 18 kg/a.
la vites.se initiale (juste après la fission) de chaque tragment. www Par .sa radioactivité, U génère de l’énergie thermique au taux de
I2P. Soit la fission du par bombardement de neutrons rapides. 0,93 W/g, a) Calculez I'cnctgie de désintégration efficace associée
Au cours d ’une fission, aucun neutron n 'csl émis et les produits à la désintégration d’un noyau de ““Sr. (Q ^, comprend les contributions
finaux stables, après la désintégration bêta des preniier.s fragments de de la désintégration des produits de filiation de l’isotope ‘““Sr dan.s .sa
fission, sont •‘’"Ce et '’“Ru. a) Combien tie désintégrations bêta tonnent chaîne de désintégration, mais non celles des neutrinos, qui s’échappent
les (le.ux chaînes, pri.ses ensemble? b) Calcule/, l’énergie de désin­ complètement de l’échantillon.) b) On veut construire une source
tégration Q de ce processus de fission. Voici les musses des atome.s et d’énergie générant 150 W (d’électricité) pour alimenter un ajipareil
des particules dont vous avez hc,soin : électronique installé dans une balise acoustique sous-marinc. Si la
source énergétique est basée sur l’énergie thcmiique générée par le ““Sr.
238.050 79 u '^’Ce 139,905 43 u et si le rcndemejit de la ciuiversion thermiquc-électriquc est de 5.0%,
““Ru 98,905 94 11 quelle quantité de '*’Sr faut-il
I .(K18 67 u
21P Nombreux sont ceux qui craignent que, en aidant les nations à
I3P. Imaginez que, immédiatement après la fission de ■*’‘’11 décrite acquérir la technologie des réacteurs nucléaires, on augmente la
par l’équation 13.1. les surfaces des noyaux de ‘‘‘"Xe et de “''Sr résul- possibilité de guerre nucléaire parce que ces réacteurs peuvent .servir
tant.s se touchent, a) En supposant que les noyaux sont sphériques, non seulement à produire de l’électricité, mais aussi à fabriquer
calculez l'énergie poteniiellc électrique (en MeV) associée à la l’isotope ‘’“Pu, pnxluit secondaire de la capture de neeitrons par ‘’"U,
répulsion entre les deux frajmicnts. (Indice : l Itilisez l’équation 12.3 qui est peu cher cl qui peut servir de « combustible » dans les bombes
pour calculer les rayons des fragments.) b) Comparez cette énergie nucléaires. Quelle série simple de réactions mettant en jeu la capture
à l'énergie dégagée dans une fission typique. de neutrons et la ilésintcgration bêta donne cet isotope de plutonium ?
14P. I fn noyau de se sépare en deux fragments de masse moyenne, 22P. Le temps de génération d’un neutron dans un réacteur est le
'’"'Xe et “‘’S i. a) En pourcentage, de combien l’aire totale des produits temps moyen requis ptxir que le neutron rapide émis dans une fission
de fission diffère t elle île celle du noyau initial de ? b) Quel est, ralentisse dans le modérateur et pnmique une autre fission. Supposez
en pourcentage, le changement de volume ? c) Quel c.st, en pourceni-age, que la puissance électrique prixliiitc dans un réacteur au temps i = 0
le cliangementdc l’éneigie potentielle électrique ? f.’éncigic potentielle est Py. Démontre/ que la pui.ssance électrique produite au temps 1
électrique d ’une sphère de rayon /■et de charge q homogène e.sl ultérieur est P (t). avec

P (t) =
5
où k e,si le facteur de mulliplicalinn. Lors d’une production d'électricité
à une puissance constante, k I.
SECTION 13.4 le réocleur nutlétirc 23P. L ne bombe atomique de 66 kilotonnes (voir le problème 18)
15E. Un réacteur à fission de 200 MW a consumé la moitié de son est alimentée avec des isotopes de ‘” 0 purs (figure 13.15). dont 4,0 %
combustible en 3,(X)a. (Quelle quantité de contenait-il initialement ? subissent réellem ent une fission, a) Quelle quantité d'uranium la
Supposez que toute l’énergie générée provient de la fission de
et que ce nucléidc n’esf consumé que par fission.
I6E. Refaites l'exercice 15, mais en tenant compte de la capture de
neutrons sans fission par le
I7E. La fission lie ^’’’Np nécessite 4,2 MeV. Pour extraire un neutron
de ce nuclcide. il faut dépenser une énergie de 5,<1 MeV. 1 e “” Np
est-il fissile par bombardement île. neutrons thermiques ?
18P. Le dégagement d’energie dans l’explosion d'une bombe atomique
est provoqué par la fission inconlrfilée de *'"Pu (011 de ■*’^U). La
puissance nominale de, la bombe correspond à l’énergie dégagée,
exprimée en masse de TNT recjuisc jxhu prixluire le même dégagement
d’cncrgie. Une mégatonne ( K/’tomics) de TNT dégage 2,6 x KV’’ MeV
d’énergie a) Calculez la puissaiiie nominale, en tonnes de TNT.
d’une honihe atomique contenant 95 kg de ”“Ehi. dont 2,5 kg subis­
sent vraiment une lis-sion (voir l’exerciee 4) b) Pourquoi les autres
'/4.5 kg de ‘'“Pu sont-ils iiéw-vaiiies s'ils ne subissent aucune fission ?
19P ( .’énergie thermique généree quand la matière absorbe du rayon­
nement provenant de radionucleides peiii servir de mécanisme
de base à une petite sinirce énergétique dans les satellites, dans les Figure 13.15 Problème 23. Un « bouton » de ‘’’U, prêt à être relondu
stations méféorologiques éloignées et dans d’aiiires installations et usiné pour servit dans iiiv' ogive.
E x e r c ic e s e t p r o b lè m e s 383

bombe contient-elle? b) Combien de premiers fragments de fission 33E. Calculez la hauleui dti la barrière coulombienne de la collision
sont protluits ? c) Combien de neutrons générés dans les fissions sont tiontalc de deux deulérons. Considérez que le rayon réel d’un deiitéron
libérés dans l’environnement ’’ (.En moyenne, chaque fission produit est de 2,1 fm.
2,5 neutrons.) » 34E. Vérifiez si la fusion de 1,0 kg de deutérium par la réaction
24P. Le temps de génération d’un neutron (voir le problème 22)
dans un réacteur particulier est 1,0 ms. Si le réacteiu fonctionne à une ‘H + -H ‘Hc r n(Q - -t-3.27 MeV)
puis,sancc de 500 MW, environ combien de neutrons libres sont |xait alimenter une ampoule électrique de 1(X) W pendant 3 x lO* a.
présents daas le réacteur à tout instant ?
35L D’aulres méthodes que le lédiaulfciiicnt de lu mulièrc pour
25P. Le temps de génération d’un neutron (voir le problème 22) daas sunnonter la barncrc coulombienne ont etc suggérées pour provoquer
un reacteur particulier e.st 1.3 ms. Le réacteur génère l’énergie à une une fusion Par exemple, on peut envisager riiiilisaiiiHi (raccéléraleiiix
puissance de 1 200 MW. Pour effectuer certaines vérifications de particules. .Si voas deviez en uillisci deux pout ucccicrcr deux
d ’usage, on doit temporairement réduire cette puissance à 350 MW. faisceaux de douterons directement Lun vers 1 autre pour qu il y art
On veut que la transition vers celte puissance réduite prenne 2,6 s. collision frontale, a) quelle diflérence de potentiel chaque accélérateur
À quelle valeur (constante) doit-on établir le facteur de multiplication devrait-il générer (xiiir que les dcuiéroits eu collision suiinontciii In
pour cITcclucr la transition dans le temps voulu ? www barrière coulombienne ? b) Scion vous, pourquoi cette méthode
?6P lin réacteur fonctionne à 4(ï) MW avec un temps de génération n’cst-clle pa.s utilisée actuellement?
d'un neutron (voir le problème 22) de 30.0 ms. ,Si cette puissance 36P. ( alculcz la hauteur de la barrière coulombienne de deux noyaux
augmente durant 5,00 min par un facteur de multiplication de 1,0003. de ’Ll pro|etés l’un vers l’autre à l’aide de la même énergie cinétique K.
quelle est la puissance ite prixluction électrique à la fin des 5,(X) imn ? (fn d iri’ ■l liilisez l’éqnation 12.3 pour calculer les rayons îles noyaux.)
27P. a) L n neutron de ma.s.se ayant une énergie cinétique K .subit 37P. Dans la figure 13 lü. l’équation de n(A"). 1^ mmihre volumique
une collision frontale élastique avec un atome au repos de masse m. par unité d’énergie, est
Démontrez que la |3crtc relative d’énergie cinétique est donnée par
K ,k T
^f< 4шпП/ nlK) - 1.13n
(7.7)
{m + rua)^ ■
où n est le nombre volumique total de particules. Au centre du Soleil,
b') Détemiinez, А Ю К piair ehiieune des particules suivantes agissant la température est de 1,50 x 10’ K. cl l'énergie moyenne d'un
comme atome au repos : rhydrogène, le deutérium, le carbone et le proton, , est de 1,94 keV. Déterminez le rap|inn entre le nnnilw
plomb, c) Si, initialement, K = 1,(H) MeV, combien de collisions volumique des protons à 5,00 keV et le nombre volumique des
frontales faudrait-il pour que l’cnergic cinétique du neutron atteigne pinions dont l’énergie est égale à l'énergie moyenne.
une valeur thermique (0,023 cV) .si les atomes au repos qu’il heurte
sont des atomes de ileiitérium, un modérateur couramment utilisé ?
(Dans les vrais modérateurs, la plupart des colli.sions ne sont pas SECTION 13.7 Ln fu«on lherinonutléoife dons le Soleil
frontales.) ef dons les niiires étoiles
38E. Démontrez que l’cncrgic dégagée, quand trois particules alpha
SECTION 13.5 Un réotleur nucléaire naturel fusionnent pour former f isotope. ’’C est de 7.27 MeV. La masse
28F À quelle époque le rapport était il de 0 ,13 dans les atomique de ’’I le est de 4.(H)'2 6 u , celle de ” C est de 12.000 0 u.
dépôts d’uranium naturel ’’ 39E. On a vu que le facteur Q, dans la chaîne île fusion proton-pt tmni
29t. On estime que le réacteur à fis.sion naturel dont on parle dan.s complète, est 26,7 MeV. Coriinieiu |xiuvc/-vous leliei cette valcui
la section 13.5 a généré 15 gigawatts-années d’énergie durant sa vie. avec les valeurs de 0 de.s téaclions qui forment celle chaîne, comme
a) Si le réacteur a eu une vie de 200 000 a. à quelle puissance l’illustre la figure 13.Il ?
moyenne a-t-il fonctionné’’ b) Combien de kilogrammes de a-t-il 40E Au centre du Soleil, la masse volumique csi de 1..5 x 10’’ kg/ra’
consumés liurant .sa vie ? et la composition massique est, pour l’essentiel, la .suivante; 35*4
30P. On a constaté que certains échantillons d’uranium provenant du d'hydrogène et 65% d’hélium, a) Uucl est le nombre voltiinique de
réacteur naturel décrit dans la section 13 5 étaient légèrement enrichis protons au centre du Soleil? b) De combien est-il supérieur à celui
de plutôt qu'appauvns. Expliquez cet état en tenant compte de des particules dans un g a/ parlait à lempéradire et il pression
l’absorirtion de neutrons par l'isotope qui est abondant, et des normales (0 ‘'C et 101 kPa) ? (In d ic e . Pour un gaz parfait, p V —
désintégrations bêta et alpha de scs produits. ou P est la pression, V est le volume, N est le nomhie de particules.
k est la constante de Boltzmann et T c.st la temperature en kelvins.)
3IP Aujourd’hui, l’iiratiium extrait des mines contient 0,72 % de ^’4.'
fissile mélangé au -^U, soit tnippeu pour tbrmerun combustible destiné 4ÎP. Vérifiez les quatre valeurs de (> des réactions présentées dans k
à la fi.viion par bombardement de neutrons thermiques. C'est pourquoi figure 13.11. Voici les masses des atomes et des particules doni vous
il faut l’enrichir en ^’’L'. Les isotopes ’’'l! (f ,« = 7.0 x 10^ a) avez besoin
ei ^'”11 ( T y j = 4,5 X 10’’ a) sont tous deux radioactifs À quelle
éiinquc l'uraniuro naturel constiliiait-il uii comhiLsiible pour réai:teur. 'H 1.007 825 u ''He 4.002 603 u
possédant un rapport de 3.0%? www ’11 2,014 102 u c' 0.(MX)54Kôu
'Hc 3.016 029 u
SECTION 13.6 Lo fusion thermonudèaire : principe de bose

32t. En utilisant l’information donnée dans le texte, trouvez et notez (Indice • Distinguez bien la masse des atomes tle la masse des fioyaux,
les hauteurs approximatives des barrières coulombiennes a) de la et tenez compte des positrons.)
désintégration alpha de et b) de la fission de ' par bombardc- 42P. I.e Soleil a une masse de 2,0 x 10"’ kg et il ravonne de l’énergie
iiK:nt de neutrons thermiques. avec une puissance de ,3,9 x 10’* W. a) À quel taux le Soleil
384 Chapitre 13 L'énergie nucléaire

transfonne-t-il sa masse en d ’autres formes d ’énergie? b) Quelle 49p. l,a valeur de Q efficace pour la chaîne proton-proton de la figure
fraction de sa masse initiale le Soleil a-t-il perdue depuis qu’il a 13.11 est de 26,2 MeV. a) Exprimez-la en énergie par kilogramme
commencé à consumer l’hydrogcnc qu’il contient, il y a environ d’hydrogène consumé, b) La puissance du Soleil est de 3,9 x 10'*’ W,
4,5 X 10^'a? Si son énergie provient de la chaîne proton-proton, à quel taux
43P. Calciilev: et comparez l’énergie dégagée a) pat la fusion de 1,0 kg perd-il son hydrogène? c) À quel taux perd-il sa masse? Expliquez
d ’hydrogène au creur du Soleil et b) par la tïssion de 1,0 kg de la différence dans les résultats de b) et de c). d) La masse du Soleil
dans un réacteur à fission. e.st de 2.0 x 10’*' kg. S’il en perd au taux constant calculé en c).
44P, a) Calculez le taux auquel le Soleil génère des neutrinos. combien de temps lui faui-il pour perdre 0.10% de sa raas.se?
Considérez que la production d ’énergie est enliciement provoquée SOP. La figure 13.16 illustre une des premières idées de bombe à
par la chaîne proton-proton, b) A quel taux le,s neutrinos solaires hydrogène, l.e combustible de fusion est le dexitcrium, 'H. La haute
atteignent- ils la Terre'' température et le nombre volumique élevé de particules nécessaires à
45P. Dans cetlaines étoiles, le cycle CNO est plus probable que la la fusion sont fournis par une bombe atomique agis.sant comme
chaîne proton-proton. Ce cycle est «détonateur», qui met en jeu la fission de ' ” U ou de ” ’Pu. disposé
de- façon à imprimer une onde de choc implosive et compressive au
'2C •+ ‘H ' ’N y. 0. 1.95 McV, deutérium. La réaction de fusion résultante est
' ’N ^ ” C * e ’ + H (-?: = 1.19 MeV. 5 'H ’Hc f 'Hc < 'H 4 2n
'•’C - 'H ^ '“N t- y. 0 ^ = 7,55 MeV,
a) Calculez le facteur Q de la fusion. Pour connaître les masses
•’’N 'H ^ 'i'O 1- y. 04 = 7.30 McV, atomiques ikml vous avez besoin, voyez le problème 41. b) Calculez
i5q J.sfsj ). e - -h 1/. 0s = 1.73 MeV. la puissance nominale (voir le problème IR) de la partie fusion de
■'N -t- 'H - r ''C - “He. la bombe si celle-ci contient 500 kg de deutérium, dont 30,0%
0s = 4.97 MeV.
subit une fusion.
a) Démontrez que ce cycle de réactions a des effets exactement
équivalents à ceux de la chaîne proton-proton de la figure 13.11. w s u ou
b) Vérifiez si les deux cycles possèdent, comme prévu, la même 2S9p„
valeur de Q.
■'H
46P La comhastKMi du chaitxrn est la réaction suivante : C -• O 2 ^ COi.
La chaleur de combustion est de 3,3 x 10' J/kg de carbone ato­
mique consumé, a) hxpriinez-la en énergie par atome de carbone,
b) Exprimez-la en énergie par kilogramme de réactifs initiaux, le figure 13.16 Problème 50
carbftne el l'oxygène, c) Supposez que le Soleil (raas.se = 2,0 x 10’’' kg)
est composé de carbone et d’oxygène en proponions combustible.s
SEOION 13.8 La fusion thcrmonucléoire contrôlée
et qu’il continue de rayonner son énergie avec sa puissance actuelle
de 3,9 X 10'* W. Quelle serait sa durée ? 51E. Vérifiez les valcius de Q utilisées dans les équations 13.13, 13.14
47P. Considérez que le coeur du Soleil représente le huitième de sa et 13.15. Voici les masses dont vous avez besoin :
masse et qu’il est comprimé dans une splière dont le rayon est le quart
de celui du Soleil. De plus, considérez que la composition du cœur 'H 1,007 825 U ’Hc 3,016 029 U
compte 35% d ’hydrogène par masse et qu’cs.sentiellement 'H 2,014 102 H ‘Hc 4.002 603 U
lotiie l'énergie du Soleil y est générée. Si le Soleil continue à brûler 3,016 049 U
’H n 1.ÎK)R665 U
l'hydrogène au taux calculé dans l’exemple 13.5, dans combien de
temps I■hydrogène sera- t-il entièrement consumé ? La masse du ,Solei I 52P. L’eau ordinaire contient tq^rroximativement, par masse. 0.015 0 %
est de 2,0 X lO’’*' kg. d’eau lourde, dans laquelle un des deux atomes d'hydrogène est
48P. Une étoile convertit tout son hydrogène en hélium, et finit remplacé par le deutérium, -H. Quelle puissance de fu.sion moyenne
par être composée à ]0()% d'hélium File convertit ensuite l’hélium peut-on obtenir si on «brûle» tout le 'H contenu dans I litre d ’eau
en carbone pai le proce.ssu,s triple-alpha: en 1 jour d ’une manière quelconque, en provoquant la fusion du
deutérium à l’aide de la réaction 'H + 'H ’He -t n ?
"He ‘He 1- -“He -» ‘'C; r 7,27 McV.
53P Dans la réaction de fusion deutéron-triton de l'équation 13.15,
La masse de l’étoile, est ilc 4,6 x 10’' kg, et elle génère l'énergie à cnmrncnt l’énergie de réaction Q est elle partagée entre la particule
une puissance de 5.3 x 10-*” W. A cette puissance, combien de temps alpha et le neutron Négligez les énergies cinétiques relativement
faudra-t-il pour convertir tout l’hélium en carbone? faibles des deux particules qui seeomhinent.
14 Les quarks,
les leptons
et le Big Bang

Cette image est une photographie de l'Univers âgé de seulement 379 000 ans, ce qui représente une remontée
d'environ 1 3 x 1 0 ^ années dans le passé. C'est ce que vous auriez vu à l'époque en regardant dans toutes les directions
(lo vue a été condensée dans cette image ovale). Des taches lumineuses provenant des atomes traversent le «ciel »,
alors que les galaxies,
les étoiles et les plonètes
ne sont pas encore
formées.
386 Chapitre 1* Les quart<s. tes teptons et le Big Bang

14.1 La vie sur une corde raide


Les p h ysiciens parlen t souvent de « physique m oderne » q u an d ils font référen ce aux
théories de la relativ ité et de la physique quantique, pour les distinguer des théories de
la m écanique new tonienne et de rcicctn n n ag n étisin e m axwelheii, q u ’ils regroupent sous
le term e « physique classique ». A vec les années, le term e « m oderne » .semble de m oins
en m oins approprié pour désigner ces théories, dont les bases ont été élaborées au début
du xx"‘ siècle. N éanm oins, le q ualificatif a toujours un sens.
D ans ce d e rn ie r c h a p itre, on s ’in té resse ra à deux c h a m p s d ’étu d es v raim ent
« m o d e rn e s» , bien q u 'ils aient les plus anciennes racines. Ils tournent autour de deux
quc.stions désespérém ent sim ples ;

D i' q u o i r U n i v e r s e s t- il fa i t ?
C o m m e n t l ’U n iv e r s e s t il d e s e n ii c e q u ’il e s t ?

Ces dernières décennies, de rapides progrès ont perm is d ’y répondre.


D e nom breu x nouveaux concepts .sont fondés su r des ex p érien ces réali.sécs dans
de griuids accélérateurs de particules. C ependant, à m esure que les physiciens projettent
des pM liculcs les unes sur les autres à des énergies toujours plus élevées à l’aide d ’accé
lératciirs de plus en plu.s gros, ils se rendent à l’évidence q u ’aucun accélérateur terrestre
ne p o u rra g é n é re r des p articu le s p o sséd an t su ffisam m en t d 'é n e rg ie p o u r te ste r leur
lh 6 iric ultime. U ne seule source de particules a déjà possédé ces énergies, soit l’U niveis
lui-m êm e, dans les prem ières m illisecondes d e son existence.
D ans ce c h a p itre , vo u s a p p re n d re z une série de n o u v eau x te rm es et u n g ran d
no m b re d e nom s d e p articu le s qu e vous n ’a u re z p as à retenir. Si vo u s v'ous sen tez
tem poraiiem ent perdu, vous partagerez alors le .seniiment des physiciens qui ont connu
ces p ro g rè s et qui, p a rfo is, n e voy aient rien d ’au tre qu e la co m p lex ité cro issan te de
cho.ses q u ’ils avaient peu d 'esp o ir de com prendre. C ependant, si vous persévérez, vous
partagerez I cxcitatio n ressentie p ar les physiciens q ui fo n t d e nouvelles découvertes
grâce aux m agnifiques accélérateu rs m odernes, grâce aux th éo ricien s t|ui m etten t de
l’avant des idées chaque jo u r plus audacieuses et grâce à cclu; lueur qui brille enfin dans
l’obscurité.

14.2 Une « ménagerie » de particules


Dans les années 1930, de nombreux scientifiques croyaient que le problèm e de la structure
ultime de la m atière était en bonne voie d ’etre résolu. On pouvait alors com prendre les
atom es en faisa n t référen c e à trois p articu les ; l'é le c tro n , le p ro to n et le neu tro n . La
phy.sique quantique expliquait bien la structure de l’atom e et la dé'sintégration radioactive
alpha. L ’existence du neutrino avait été postulée et. bien que cette particule n 'e û t pas
encore été observée, elle avait été incoiporée par E nrico Ferm i à une théorie fructueuse
de la désintégration bêta. Il y a\ ait espoir que la théorie quantique, appliquée aux protons
et aux neutrons, expliquerait rapidem ent la structure du noyau. Q u ’y avait il d ’a u tre ?
L 'euphorie n ’a pas duré. L a fin de cette m êm e décennie fut l’aube d ’une période de
découvertes de nouvelles particules qui se poursuit encore au jo u rd ’hui. Les nouvelles
particules ont des nom s et des sym boles com m e n iu n n ( / i ) , p i o n (rr), lo u w (K) et .vigmw
(X). Toutes les nouvelles particules sont instables, c ’est-à-dire q u ’elles .se tran.sforment
sprm taném cnt en d ’autres types de particules .scion les m êm es fonctions de tem p s qui
s’ap p liq u en t aux n o y au x in stab les. D onc, si N q p a rtic u le s d ’un c e rta in ty p e so n t
présentes dans un échantillon au tem ps / = 0. le nom bre N de ces particules présentes à
un tem ps ultérieur t est donné par l’équation 12.14 ;

N - N qC (14.1)

Ij ? taux de désintégration (ou activité) R. à p artir d ’une valeur initiale R o , c.st déterm iné
par l’équation 12. LS:

R = R „e~\ (14.2)
14.2 U ne « m énagerie» de particules 387

c l la d em i-v ie , la co n stan te de d é sin té g ra tio n  et la d u rée d e vie intrycnne t


sont reliées par l’équation 12.17 ;
ln 2
Tm = = г In 2. (14.3)

Les dem i-vies des nouvelles particules se situent entre lü"*’ s et lü s. fcn fait, certaines
des panicules ont une durée de vie si courte q u ’elles ne (xuvent être directement ilétectccs.
leur présence ne peut être eonlirm ée que p tu des indices indirects.
C es nouvelles particules sont habituellem ent produites lors de collisions frontales
entre de.s protons ou des électrons accélérés à des énergies élevées dans des accélérateurs
situés en des lieux com m e le Fcrm ilab (près de C hicago), le C tR X (près de G enevo),
le S L A C (à S tanford) et le D E SY (près d e H am bourg, en A llem ag n e). O n d o it leurs
déeouvcrles à des détecteurs de particules de plus en plus sqiliistiqués i voir la figure 14.1 )
qui pcuvenl c h a c u n rivaliser avec V e n s e m b le d e to us les u c c é lé r a ie u r s d ’il y a quelques
décennies.
A u jo u rd ’h u i, o n co n n a ît p lu sieu rs ce n tain es d e p articu les. Pout les nom m er, on
a é p u isé l'a lp h a b e t g rec, ei o n attrib u e un n o m bre à la p lu p art d V in re elles d an s la
Rgure 14.1 I -e tlélecltur de partieulcs com pilation périrxliquc T h e R e v ie w o f P a n i c l e P h y s ic s (publiée p ar le Р и п и le П а ш
OFAL (de l’anglais om ni-purpose G r o u p , dont le site W eb est http://pdg.lbl gov/). P our s ' y retro u v er dans cci ensem ble
a p p a m tiis) au CHKN, le laboratoire
im pressionnan t de p an icu le s, on recherche des critères p h y siq u es sim[iles. On peut à
eun'péen de physique des particules
tout le m o in s fa ire un e p rem ière d iv isio n g ro ssiè re p arm i les p a rtic u le s tics trois
situé près de Genève, en Suisse.
m anières suivantes
OPAL est conçu pour mesurer
les énergies des particules produites
par les collisions prisibrrn électron Fermion ou boson ?
ayant chacun une énergie de 50 GeV. Toutes les parlietiles ont un m om ent cinétique intrinsèque appelé sp in , com m e on Га vu
Bien que ce détecteur soit de taille lorsqu’o n a étudié les électrons à la section 1 1.3. Si on généralise la notation em ployée
imposante, il est petit en comparaison dans cette section, on peut écrire la comixwante du spin S (b a s toute direction (on suppo.se
de l’accélérateur lui-même, qui est q u ’elle est orientée dans la direction de l’axe des i ) ainsi :
un cercle de 27 km de circonférence.
S = m ,h p o u rra , * л, .v - 1 ,..., s, (14.41

où h est égal à h /2 ji, est le n o m b r e (ju a n liq u e m a g n é tiq u e d e s p in et s est le n o m b r e


q tia n fiq u e d e .spin. C e d ern ier peut avoir des valeurs deini-eniières ijositives (4 , 4^...)
ou des valeurs entières non négatives ( 0 ,1 , 2 ,...). Par exem ple, pour un éleetm ii, r - ^
A insi, la com posante du spin d 'u n électron (m esuré dans n im p o rle q u elle direction)
peut avoir les valeurs

Sj sh (spin vers le haut)

ou = —y/i (spin vers le bas).

Une chose qui peut porter à confusion est que le tci me spin est utilise tic deux façons :
il fa it référen ce, d e m a n ière ap p ro p riée , au m o m en t c in é tiq u e in trin sè q u e S d 'u n e
particule, m ais il d ésig n e so u v en t, p ar ex ten sio n , le n o m bre q u an tiq u e de spin do la
paiiieiile s. D ans ce dernier cas, par exem ple, on dit d ’un électron iju’il e.st une particule
de spin J .
Les particules dont le nombre quantique de spin est demi entier (to m m e lc.s clcttrons)
s ’appellent ferm ions, d ’après Lnrico Fermi, qui décotivnl (on m ëm t' temps que Paul ü irac)
les lois statistiq u es qui g o uvernent leu r com portem ent, to u t com m e les électro n s, les
protons et les neutrons ont .v ~ f et sont des ferm ions
I £ s particules dont le nombre qiianiiqiic de spin exst entier ou nul sont appelées h<Kson.x,
d ’après le physicien indien Satyendra N ath Bose, qui découvrit (avec A lbert Linstcin)
les lois statistique.v qui gouvernent r e s particules. Les phoious (jtii ont .v — 1, soni des
bosons ; vous connaîtrez bientôt d ’antres pariienles faisant p an ic de ce tte classe.
C ette façon d e classer les p articules peut .sembler rudim entaire, m ais elle est très
im portante, pour la raison suivante.

Les fermions «-«bèi.vscnt an pritK.i(x: d’exclasion de Pauli, qui viait qu’un état quaiitiquc donné
ne peut accueitlii qu’une seule panicule à la fois (x-s bosons « o heisscni p a s à ce principe
N"importe quel nombre de bosons peut oceuixsr un étal qiiuiitiquc donné.
388 Chapitre 14 Les quarks, les leptons et le Big Bang

O n a vu l’im p o rtan ce d u p rin cip e d ’ex clu sio n d e P au li q u an d o n a « c o n s tru it» d es ato m es
e n a iirib iia n i d e s é le c tro n s (sp in ^ ) à d e s é ta ts q u a iu iq u c s in d iv id u e ls. C e p rin c ip e a
p e rm is d e d o n n e r u n e e x p lic a tio n co m p lè te d e la stru c tu re et d e s pixvpriétés d e s a to m es
d e dif féren ts ty p es.
É tant d o n n é q u e les b o so n s n 'o M s s e n t p a s au p rin cip e d e Pauli, ces p articu les tendent
à s ’e m p ile r d a n s l ’é ta t q u a n tiq n e a y a n t la p lu s fa ib le é n e rg ie . E n 1 9 9 5 , u n e é q u ip e
de B oulder, au C o lo ra d o , d irig é e p a r E ric C o rn ell c i C ari W iein an , a réu ssi à p ro d u ite un
conden.sal d ’e n v iro n 2 (XX) a to m e s d e ru b id iu m 87 (qui so n t d e s b o so n s) d a n s u n é ta l
q u a n tiq u e u n iq u e d ’én e rg ie a p p ro x im a iiv e m c m n u lle.
P o u r q u e ce p h é n o m è n e se p ro d u ise , le ru b id iu m d o it ê tre à l’é ta l d e v a p e u r; sa
te m p é ra tu re , trè s b a sse, et so n n o m b re v o lu m iq u e , trè s g ran d , fo n t a lo rs q u e la lo n g u e u r
d 'o n d e d e D e B ro g lie d e c h a q u e a to m e e s t p lu s g ra n d e q u e la d is ta n c e m o y e n n e q u i
sép a re le s ato m es. L u e fo is c elte c o n d itio n atte in te , les fo n c tio n s d ’o n d e resp e c tiv e s des
a to m es se c h e v a u c h e n t, e t to u t l'e n s e m b le dev ien t u n sy stè m e q u a n tiq u e u n iq u e, appi’lé
c o n d e n s â t d e B o s e - L in s te in . L a fig u re 14.2 d é m o n tre q u e, lo rsq u e la tem p é ra tu re d e la
v a p e u r d e ru b id iu m e st a b a issé e à e n v iro n 1,70 x 10 ’ K , c e .systèm e « s ’e ffo n d re su r
lu i-m ê m e » d ans un état n cu cm cn i defini co rresp o n d an t à une v itesse ap p ro x im ativ em en t
n u lle d es atom es. D ’autres g ro u p es o n t réu ssi à p ro d u ire d es co n d en sais d e B o se-E in stein
à p a rtir d e v a p e u rs d 'é lé m e n ts d iffe re n ts , c o m m e le lith iu m e t le so d iu m . E n 2 0 0 1 ,
C o rn e ll e t W ie m a n (a in si q u e W o lfg an g K eltc rle ) o n t re ç u le p rix N o b el d e p h y siq u e
p o u r le u r d éco iiv crie.
Hadron ou lepton ?
O n p eu t égalem en t c la sse r les p articu les en fo n ctio n des q u atre in teractio n s fo n d am en tales
q u i ag issen t su r e lles. U in te r a c tio n g r a v ita tio n n e lle ag it su r to u te s les p artic u le s, m ais
son a c tio n au n iv e a u d e s p a rtic u le s é lé m e n ta ire s e st si fa ib le q u 'o n n ’a p a s à e n te n ir
c o m p te (du m o in s p as d an s la rec h e rc h e actu elle). \S i n te r a c ti o n é le c tr o m a g n é tiq u e agit
su r to u tes les p artic u le s c h a r g é e s é le c triq u e m e n t-, so n actio n est b ien c o n n u e e t on p eu t
e n te n ir c o m p te , a u b e so in ; o n ig n o re ra c e tte in te ra c tio n d a n s la m a je u re p a rtie d u
p résen t c h ap itre.
Il re ste V in te r a c tio n fo r t e , q ui lie les n u clé o n s e n tre eu x . et V in te r a c tio n fa ib le , qui
e st e n je u d a n s la d é.sin tég ratio n bêta e t le s p ro c e ssu s se m b la b le s. L 'in te ra c tio n faib le
ag it su r to u tes le s p a rtic u le s ; l'in te ra c tio n fo rte, seu lem en t su r q u e lq u e s-u n e s.
O n p e u t a lo rs g ro ssiè re m e n t c la sse r les p a rtic u le s e n d é te rm in a n t si l’in te ra c iio n
fo rte a g it ou n o n s u r e lle s. L e s p a rtic u le s su r le s q u e lle s l 'i n t e r a c t i o n f o r t e a g it s o n t
ap p e lé e s h a d r o a s . C e lle s s u r lesq u e lle s l'in te ra c tio n fo rte n 'a g it p as. laissan t la p la c e à
rin tc ra c tio n faib le, so n t a p p e lé e s le p to n s . L es p ro to n s, les n eu tro n s e t les p io n s .sont des
h ad ro n s ; les é le c tro n s e t les n eu trin o s so n t d e s lep to n s. V ous c o n n a ître z b ie n tô t d ’an tres

Fijurt 14.2 'l'rois grripliiqucs


di* la ilistribmion de la vitesse
dc.s atomes rir luhidiuin H7
à l'éiat de vapeur I a leinpéraiiire
de la vapeur est successivement réduite
du grdjjhique <i) au graphitiuc t ).
I .P graphique c) monire un pic net
centré autour de la vitesse n u lle ;
c ’est donc dire que tous le.s atomes
so n t duns le meme état quantique,
l n tel cnn(1eiu..'it d e BoKe-hinsiein,
souvent appelé Saini-rnaal
(le la phyvupie atomique, a finalement
été observé en IW5.
14.3 Un interlude 389

m em b res d e c e s cla.sses. O n jic u i faire u n e au tre d istin c tio n dan.s la c la sse d es h ad ro n s


C e rta in s h ad ro n s so n t d es b o so n s (o n les ap p elle les m é so n s) ; le pion e n e st un ex em p le.
L es autres h ad ro n s sont d es ferm io n s (o n les ap p elle les b a rd o n s ) ; le p ro to n et le n eu tro n
en so n t d es ex em p les.
Particule ou antiparticule ?
En 1928, D irac p ré d it q u e l'é le c tro n c" d ev ait a v o ir u n e cn iiiic p a rtic d e m asse c l de spin
identiques, m ais avec une d ia rg e positive. C e lle coiitrepai tic, le ¡lusHnm fut d eco u v erte
d an s le s ray o n s cosm ique,s e n 1932 p a r C ari A n d erso n , l^ar la su ite, les p h y sic ie n s o n t
g ra d u e lle m e n t c o n s ta té q u e ch a q u e p a rtic u le é ta it a s s o c ié e à u n e untlpurtieult!.
L es m e m b re s d e c h a c u n e d e c e s p a ire s a v a ie n t la m êm e n ia sse et le m êiiie sjiin,
m a is d e s c h a rg e s é le c triq u e s d e sig n es o p p o sé s (si elles sont c h a rg é e s) et d es n o m b res
q u an tiq iics (dont on p arlera d an s les sectio n s su iv a n te s) d e sig n e s o p p o ses.
A u d é b u t, le te rm e p a rticu le faisait réfé re n c e au x p a rtic u le s c o m m u n e s, c o m m e les
électro n s, les p ro to n s et les n eu tro n s : le te rm e antiparticule, q u an t lu i, faisait référw iee
à le u rs c o n tre p a rtie s, ra re m e n t d éle c té e s. P lu s tard , o n a attrib u é le s te rm e s particule et
antiparticule aux p a rtic u le s m o in s e o m tn iin cs (x iu rê tre co llèren t av ec c e iia iiis prineijx-s
d e c o n s e rv a tio n q u i s e ro n t v u s p lu s lo in d a n s c e d ia p itr e (o n e m p lo ie p ai foi.s in d if-
fé re m m e n t le te rm e p a rticu le q u a n d on p a rle d e p arlie iilc s et d ’a n tip a rtie u lc s e t q u ’o n
n ’a p as à le.s d istin g u er). S o u v en t (p as to u jo u rs), on re p ré se n te u n e an tip a rtic u le p a r u n e
b a rre a u -d e ssu s d u sy m b o le d e la p a rtic u le c o rre sp o n d a n te . A in si, p e st le sy m b o le d u
p ro to n , e t P (p ro n o n cé « p b arre »1 est le sy m b o le d e l'a n tip ro to n .
Q u a n d u n e p artic u le ren co n tre son an tip articu le, elles p eu v e n t s annihiler m u iu d le
m e n t : la p artic u le e t ra n iip a m c u le d isp a ra isse n t c l leu rs én erg ie s com binées (in c lu a n ;
l’én e rg ie au re p o s) ré ap p araissen t so u s d ’a u tre s fo rm es. D an s le cas d ’u n e a n n ih ilatio n
d ’une paire électron-positron, l'é n e rg ie d es deux particules est tran sfo rm ée en d eu x photous
d e ra y o n s g am m a :
e -t- e~ y + y. (1 4 5 )

S i l ’élcclrtm e t le p o sitro n so n t à p eu p rès au iepo,s au m o m en t d e le u r a n n ih ila tio n , leu r


é n e rg ie to ta le e st é g a le à la so m m e d e le u rs rn erg ic.s au re p o s, é n e rg ie alo rs p at lag ee
égalem en t en tre le,s d eu x photons. P out co n serv er la q u an tité de m oiivcm eat. cl p u isq u ’ils
n e p eu v e n t ê tre au rep o s, ces p h o to n s s ’é lo ig n e n t d a n s d es diretiio ii.s np(Ki.sées
t n 1996, les p h y sicien s du C E R N o n t réassi à créer, du n iiti q u elq u es b rè v e s nano.se-
co n d es. une p o ig n é e d ’ato m es d 'a n tih y d ro g è n e , c h a c u n é ta n t c o n stitu é d 'u n {xisiiion et
d ’un an tip ro to n lies e n se m b le (to u t co m m e l’é le c tro n et le p ro to n le so n t d an s u n ato m e
d 'h y d ro g è n e , v raisem b lab lem en t). U n tel en se m b le de p artic u le s est apjielé uniir)intü>re.
p o u r le d istin g u e r d ’u n e n se m b le d e p ariitm les e o m n u iiies (m atière)
O n p e u t su p p o se r q u ’il e x iste d e s g a la x ie s d ’an lim a ilè re , c o m p ta n t d e s atom c.s. dc.s
m o lécu les e t m êm e d es p h y sicien s. O n p eu t m êm e lin a g in e r le d ésastre q u i se p roduirnif
si, p a r e x em p le, u n asténVide éc h a p p é d ’une telle g alax ie h eu rtait la Ic rrc (en a n n ih ila n t
le p o in t d ’im p a c t), llc u re tu sc m e n t, les c o n n a is s a n c e s a c tu e lle s in d iq u e n t q u e n o tre
G a la x ie e t le re ste d e r f în iv e r s s o n t la rg e m e n t c o n s titu é s d e m a tiè re au lieu d 'a n ti­
m atière. (C ette asy m étrie p o u rrait être ex p liq u ée pai une asyuK5trir de l'In ieiaL iln n faible.)
Figure H 3 a) C'nc photo.ciaphie
d'une chambre à bulles où se produit 14.3 Un interlude
une suite d ’événemenrs provoquée
par un antiproton qui y entre
A v a n t d e se p ré c ip ite r p o u r c la s s e r le s p a rtic u le s , o n p e u t p re n d re un m o m e n t p o u r
par la gauche, b) Les trajccioire.s
s a is ir l’e s p rit d e la re c h e rc h e e n p h y siq u e d e s p a rtic u le s e n a n a ly sa n t u n é v é n e m e n t
sont tracées et annotées, pour les rendre ty p e ce lu i illu stré p a r la p h o to d e la c h a m b re à b u lle s d e la fig u re 14..3 a)
plus claires. Les points 1 et 2 indiquent L es tra c e s d e c e tte figure so n t c o n s titu a s d e b u lles fo rm é e s Ir lo n g îles irajciu o ires
les sites des événements secondaires d e s p at ticu les ch a rg é e s q u i se d é p lacen t d an s une ch am b re le n ip h c d ’h v d ro g c n c liquide.
spécifiques drierits tbns le texte. O n p e u t id e n tifie r la p a rtic u le q u i fo rm e une tra c e p a rtic u liè re e n m e su ra n t fg s p u e e
r.cs trajectoires sont courbées re la tif e n tre les b u lle s (en tre au tres m o y en s). I jt c lia in b rr b a ig n e d an s un c h a m p m ag n é
parce r.iu’un champ magnétique présent tique liom ogène q u i d év ie d an s le .sens co n traire d es aig u illes d ’une m ontre les traiectoires
dans la chambre exerce une Force d e s p a rtic u le s d o n t la c h a rg e é le c triq u e e st p o sitiv e, e t d a n s le sen s d e s aig u illes d 'iin c
de déflexion sur chaque particule m o n tre les tra je c to ire s d es p articu les d o n t la c h a rg e é lc c u iq n c est n ég ativ e F.n m esu ian i
chargée qui se déplace. le ray o n d e c o u rb u re d ’une traje c to ire , o n p eu t c a lc u le r la q u an tité d e m o u v e m e n t d e la
390 Chapitre Les quartes, les leptons et te Big Bang

TA81EAUI1.I Les particules et les antiparticules ossociées à l'événement de la Figure 14.3


Particule Symbole Charge q Masse (MeV/c^) Spin s Classe Vie moyenne (s) Aniipaiiicule
\
Neutrino U 0 0 2 l.epton Stable V
0,511 1 I,epton Stable
Électron c -l 2 e'
105.7 ï l.epton 2,2 10 "
Miion p~ -1 5 X P'
Pion TT'*' 1-1 139.6 0 Méson 2.6 X K)-“ n
9.38.3 1 Baryon Stable
Proton P -H 2 P

p a rtic u le q u i l'a fo rm ée. L e ta b le a u 14.1 pré.sentc c e rta in e s p ro p rié té s d e s p a rtic u le s et


d e s a n tip articu les asso ciées à l’év én e m e n t d é c rit d a n s la fig u re 14.3 a), y co m p ris celles
qui n ’ont pas laissé d e traces. E n vertu d ’u n e p ratiq u e cou ran te, o n ex p rim e les m asses des
p articu les n o m m ées d an s le tab leau 1 4 .1 (et d a n s les au tres ta b le a u x d e c e c h a p itre ) en
M eV /e^. O n e m p lo ie c e tte u n ité p arc e q u 'o n a p lu s so u v e n t b eso in d e l’é n e rg ie au rep o s
d ’u n e p a rtic u le q u e d e sa m asse. A in si, d an s le ta b le a u 1 4 .1 , o n d o n n e 93X .3 M cV /c^
c o m m e m a sse d ’u n p ro to n . P o u r d é te rm in e r l’é n e rg ie au re p o s d u p ro to n , il su ffit d e
m u ltip lie r c e tte m a sse p a r r~ p o u r o b te n ir 9 3 8 ,3 M eV .
P o u r a n a ly se r u n e p h o to g ra p h ie c o m m e c e lle d e la fig u re 14.3 a), o n u tilis e d es
o u tils tels q u e les p rin c ip e s de c o n se rv a tio n d e l’én e rg ie , d e la q u a n tité d e m o u v e m e n u
d u m o m en t cin étiq u e e t d e la c h arg e électriq u e, en plus d ’au tres p rin cip es de conserx ation
q u ’o n n ’a p as e n c o re étu d iés. L a fig u re 14.3 a) e st. e n fait, l’u n e de d e u x p h o to g ra p h ie s
p rise s e n s té ré o ; e n p ratiq u e, c ette a n aly se e st e ffe c tu é e en tro is d im en sio n s.
L ’év én em en t de la fig u re 14.3 a) est p ro v o q u é p a r un an tip ro to n én erg étiq u e (p ) q u i,
a p rè s a v o ir été g é n é ré d an s u n a c c é lé ra te u r d u la b o ra to ire L a w re n c e B e rk e le y , e n tre
d ans la ch am b re du cô té g auche. Il y a tro is so u s-év én em en ts distincts ; d eu x se p ro d u isen t
aux p o in ts 1 e t 2 d e la fig u re 14.3 b), le tro isièm e .se p ro d u it h o rs d u c a d re d e la fig u re.
O n e x a m in e ra ici c h a c u n d 'e u x .
I. L ’a n n ih ila tio n p ro to n -a n tip ro to n . D a n s la fig u re 14.3 b ), au p o in t I, l’a n tip ro to n
in itia te u r (tra c e b le u e ) h e u rte u n p ro to n d e l ’h y d ro g è n e liq u id e c o n te n u d a n s
la c h a m b re , c o llisio n q u i ,se .solde p a r u n e a n n ih ila tio n m u tu e lle . O n p e u t d ire q u e
l ’a n n ih ila tio n s ’e st p ro d u ite q u a n d l ’a n lip ro lo n in c id e n t é ta it e n v o l p a rc e q u e la
p lu p art d e s p a n ic u le s g én érées par la ren co n tre se d ép lacen t d a a s la d irectio n in itiale -
c ’e st-à -d ire v ers la d ro ite d e la fig u re 14.3. S e lo n le p rin c ip e d e co n .scrv atio n d e la
q u an tité d e m o u v em en t, l ’an tip ro to n in cid en t d ev ait av o ir u n e q u an tité d e m o u v em en t
v ers la d ro ite à l’in.stant d e so n a n n ih ilatio n .
L ’é n e rg ie to tale e n je u d an s la co llisio n d e l’an tip ro to n et du p ro to n est la so m m e
d e l ’é n e t^ ie cin étiq u e d e l’an tip ro to n e t d es d eu x é n erg ies au rep o s (id en tiq u es) d e ces
d e u x p a rtic u le s (2 x 9 3 8 ,3 M eV , o u I 8 7 6 ,6 M e V ), ce q u i c o n stitu e su ffisa m m e n t
il’én erg ic p o u r c ré e r un certain n o m b re d e p articu les d e faible m asse et leu r d o n n e r de
l’én erg ie cinétique. D ans le pré.sent cas, l’annihilation p n x lu it q u atre pions positif s (traces
ro u g es d e la fig u re 14,3 h) e t t|u atre p io n s n ég atifs (traces vertes). (P o u r sim plifier, on
su p p o se q u ’a u cu n p h o to n d e ray o n g a m m a n ’e st p ro d u it, p h o to n q u i n ’aurait laissé
aucune trace p art e q u ’il est électriq u em en t n eutre ) I e p ro cessu s d ’an nihilation est alors
■47r*^ -t- A n . (1 4 .6 )
D ’ap rès le ta b le a u 14.1, o n c o n sta te q u e les p io n s p o sitifs {n^ ) so n t d e s p a rticules et
q u e les p io n s n ég a tifs (n ) so n t d es antiparticules. L a ré a c tio n d e l’é q u a tio n 14.6 e st
c a u sé e p ar l’in te ra c tio n fo rte, p arce q u e to u tes les p a rtic u le s en je u so n t d es h ad ro n s.
O ti v érifie m ain ten an t si la c h a rg e élec triq u e e st c o n se rv é e d a n s la réactio n . P o u r
c e faire, on p eu t d é c rire la c h a rg e é le c triq u e d ’u n e p a rtic u le c o n u n c qe. o ù q est un
nombre quantique de charge. D onc, détei m in er si la c h arg e électriq u e est c o n serv ée
d a n s u n proce.ssus é q u iv a u t à d é te rm in e r si le n o m b re q u a n tiq u e d e c h a rg e net in itial
est égal au nom bre quantique d e ch arg e net final. D ails le p rocessus d e l’équ atio n 14.6. la
charge nette initiale est ( 1 1) + ( - 1 ) = 0 et la charge nette finale est 4( * 1 ) H 4( 1) = 0.
D o n c, la c h a rg e est c o n serv ée.
14.3 Un interlude 391

Le p rin cip e de co n se rv a tio n d e l’é n e rg ie im p liq u e q u e l ’é n e rg ie d isp o n ib le


provenant d’une annihilation p-p est au moins la som m e des énergies au repos du proton
et de l’antiproton, soit 1 876,6 MeV. L ’énergie au repos d ’un pion étant 139,6 MeV,
l’énergie au repos totale des huit pions est donc de 8 x 139,6 MeV, ou 1 11 6,8 MeV.
C e qui laisse au m oins 760 M eV à partager entre les huit pions sous form e d ’énergie
cinétique. Donc, le principe de conservation d 'én e rg ie e st facilcm cni respecté.
2. Im d é s 'm ié g r a û o n d u p io n . L es pions sont des particules instulrles ; les pions chargés
se désintégrent après une vie m oyenne de 2.6 x 10 **s. A u point 2 de la figure 14.3 b),
un des pions positifs se désintègre spontaném ent en un antim uon p, ' {trace violette)
Cl en un neutrino i/ :
■P* + IA (14.7)

Le neutrino. sans charge, ne laisse aucune trace. L ’antim uon et le neutrino soni tous
deux des leptons, c ’est-à-dire que ce sonidc.s p aiticu les mu IrMiutdlcs rin u -raclto n
forte n ’agit pas. D onc, la d ésin tég ratio n décrite p ar T cquation 14.7 e s t cau see par
l’interaction faible. L 'én erg ie au repos d 'u n antim uon e.st 1051,7 MeV 11 reste donc
139,6 MeV — 105,7 MeV = 33,9 M eV (en plu.s de l'énergie cinétique qu'avait le pion en
se désintégrant) à partager entre l’antimuon et le ncutnno sous form e d ’é n e rp e cinétique.
On peut vérifier si le m om ent cinétique est conservé dans le jmicessuv de l’équa­
tion 14.7, ce qui équivaut à détei m iner si la com jiosanlc S . du .spin rcsullanl dans lu
direction d ’un axe des z. arbitraire peut être conservée par le processus. L es nom bres
quantiques de spin s des particules de ce p rtx essu s sont 0 pour le pion n*' ei | pour
l ’antim uon /u" e t le n eu trin o i/ D onc, la co m p o san te du pion d o it être ü/i alors
que la com posante S . de l’antim uon et du n cu tn n o doit être + ¿fi ou - i h .
L a com posante 5 , e.st conservée dans le processus d e l’équation 14,7 s ’il existe
M17 m oyen par lequel la valeur initiale d e S . Ofi) peut être égale à la valeur fin a le
d e S . résu ltan te . O n co n state q u e, si l ’un d es p ro d u its, so it p ~ ou v , a S . = f i f i
et l'au tre a S = — \ h . leur valeur resultante finale est üfi. Dixie, puisque 6^ peut être
conservée, la désintégration de l’équation 14.7 p e u t se pividuire.
Selon l’équation 14.7, on constate égalem ent que la charge est conservée ¡lar le
processus parce que, avant celui-ci, le nom bre quantique de la charge est + 1 et, après
ce processus, il est 4 1 + 0 = + 1 .
3. l.o d é s i n t é g r a t i o n d u m u o n . L es m u o n s ( p ~ o u / r ’) so n t eg a le m en t in sta b le s. SC
d ésin té g ra n t ap rès une vie m o y en n e de 2 .2 x 10 '' s. B ien q u e les p ro d u its de
désintégration n 'ap p araissen t pas dans la figure 14.3 b), l’iintim uon produit dans la
réaction décrite pat l’équation 14.7 sc désintègre spontaném ent de la maniere .suivante ;

•c P + l'. (14.8)
L’énergie au repos d e l’antimuon est 105,7 M rV ; i elle du positron n’i'sl qun0..511 MeV,
ce qui lais.se 105,2 M eV (en plus de l’cncrgic cinétique q u ’avait le m uon en se desin
tégrani) à partager sous form e d ’énergie cinétique entre les trots particules produites
dans la désintégration décrite par l'éq u atio n 14.8.
Une question vous vient jx-m-être à l'esprit ; pourquoi d e ia neutrinos dans l’équa­
tion 14.8 ‘7 Pourquoi non pa.s un seul, conunc dans la désintégration du pion décrite dans
l’éq u atio n 14.7 ? U ne rép o n se p o ssib le est qu e les nom bres q u an iiip ies de spin d e
l’aniim uon, du positron et du neutrino sont respectivem ent \ , .s’il ii’y avail qu 'un
seul neutrino, la com posante .V- résultante du spin ne p o u rra irêtre eonservée dates la
désintégration de l'antiinuon décrite par l’équaiion 14 8.1 lue autre raison sera donnée
dan.s la .section 14 4

Exemple 14.1
Un pion positif au repos peut sc désintégrer de la manière .siiivanie. K I I t m ,;’ ■ K^..

fi ’ 1 K Étant donné que le pion était au repos, son énergie cinéiitjne Kj ^ était
nulle Ainsi, d'aprè.s les masses donuccs au tableau 14 I. ou obtient
Oaellc CM l'énergie cinétique de rantiniuon Quelle est l’énergie
cinétique du ncutriiK) ? K^, t K,, — - - w,/--
SOLlillON; I..e concept clé ici est que, dans la désintégration dn pion il doit = 1.39.6 MeV 105.7 MeV 0
y avoir conservation de l’énergie et de la quantité tie mouvement. = <1,0 MeV (IdO l
On exprime d’abord de la manièi'e suivante la ctimservaiion de
l’énergie (énergie au repos m c ‘ plus l'énergie cinétique K ) dans et K . ne peuvent êta- détcriiiinées séiiaréiiicni dans l'ciquaiioii
la désintégration : 14.9. On applique donc le pnnci|’e de conservation de la quantité de
392 Chapitre 1A Les quarKs. les leptons et le Big Bang

mouvement a la désintégration. Puistjuc le pion est au repos quand il Si on insère l'équation 14.11 dans le membre de gauche et dans le
SCdésintègre, ce principe exige que le niuon et le neutrino se déplacent membre de droite de l’équation 14.12, on obtient
dans des directions opposées après la désintégration. Suppose? que leur
K l -r = K l ^ 2 K ,m j- \
mouvement suit un axe. Ainsi, on peut exprimer de la manière suivante
la conservation de la quantité de mouvement dan.s la direction de l’axe Si on pose que la masse du neutrino = 0 (comme on l’indique
pour la désintégration ; dans le tableau 14.1), on insère K^, = 33,9 MeV — K^^, qui provient
de l’équation 14.9. puis on isole K^^ et on obtient
Ps P IJ. Pi/•
qui dtinne, si p„ - 0. ^ (33,9MeV)2
^ “ (2 )(.3 3 T ^eV
Pu = -/V - (14.10)
(33,9 MeV)2
On vein relier les quantités de mouvement et —p^, cl les
(2)(33.9M cV H 105,7 MeV)
énergies cinétiques et pour tléterniiner ces dernières. Étant
donné qu’il n ’y a aucune raison de croire que la physique non = 4,12 MeV. (réptmse)
telativj.ste ptiisse s’appliquer au mouvement du muon et du neutrino,
L’énergie cinétique du neutrino est alors, d’après l’équation 14.9,
nn utilise l'équation 8.53. soit la relation quantité de tnouvement-
énergie cinétique de la relativité restreinte • K^, - 33,9 MeV - = 33,9 MeV - 4,12 MeV
= 29.8 MeV. (réponse)
{pc)^ - 2 K n,c'. (14.11)
On constate que. bien que les modules des quantités de mouvement
Grâce à l’équation 14.10, on sait que
des deux particules qui s’éloignent soient les mômes, le neutrino
( P ij c f {p„cŸ- (14.12) obtient la plus grande part (88%) de l’énergie cinétique.

Exemple 14.2
Des protons au repos dans une chambre à bulles sont bombardés par On obtient, pour la réaction précédente.
des pions négatifs énergiques, ce qui provoque la réaction suivante ;
Q = + uipc’) - (m ^r' 4 mvC^).
-T- I- P -» K - 4 I + . = (139.6 .MeV 4 938,3 MeV)
- (493,7 MeV h- 1 189,4 MeV)
Les énergies au repos de ces particules sont = -6 0 5 ,2 MeV. (réponse)

n- 139,6 MeV K" 493,7 MeV Ix signe négatif signifie que la réachon est endotherm ique c’est-à dire
que le pion iiKidcnt (;r~) doit pos.séder une énergie cinétique supcrieuie
P 938.3 MeV 2 I 189,4 MeV
à une certaine valeur de seuil pour |wovoquer la réaction. L’énergie
de seuil est supérieure à 605 MeV parce que la quantité de iiaxivement
Quelle est la valeur Q de cette réaction ’?
doit être conservée, ce qui signifie, d'une part, que le kaon ( K“) et le
SOlüTlON : Ici, le comepf clé est que la valeur Q d’une réaction est sigma (2 ) doivent être créés et que, d’autre part, ils doivent aussi
obtenir une certaine énergie cinétique. Un calcul relativiste dont les
( énergie au rcpos\ /énergie au repos\ détails dépassent les notions abordées dans le présent volume
totale iniiiaie / \ totale finale / ’ démontre que l’énergie de seuil de cette réaction est 907 MeV.

14.4 Les leptons


Dan.s cette section et dans la prochaine, nous traiterons de certaines des particules do la
m éthode d e c la ssific a tio n p rése n tée à la se ctio n 14.2 ; les lep to n s et le.s h adrons.
N ous co m m en cero n s par les Icplons. ces p a itic u le s su r lesq u elles l'in te ra c tio n fo rte
n 'a p i i p a s . L es leptons vus ju s q u ’ici sont l’électron fam ilier et le neutrino qui l’accom ­
pagne d an s une d ésin té g ra tio n bêta. L e nuion. d o n t la d ésin tc g ra lio n e st d éc rite par
l'équation 14.8, est un autre m em bre de eciie classe. Les physiciens onl graduellem ent
appris que le neutrino qui apparaît dan.s l’équation 14.7, associée à la production d ’un
m uon. n ’e s t p a s id e n t iq u e au neutrino p ro d u it dans la d ésin tég ratio n hêta, asso ciée à
l'apparition d 'u n électron Le prem ier est ap(:)elc neutrino muonique (//,^) et le sec tmd.
neutrino électronique ( ) quand il faut les distinguer.
C es deux types de neutrinos .sont considérés eon u n c des particules différentes parce
que. SI un fai.sceau d e neutrinos nuioniques (p ro d u its par une désintégration de pions
com m e celle décrite dans l'éq u a tio n 14.7) h eu rte une e.ible solide, il y a u n i q u e m e n t
p r o d u c t i a n d e m u o n s , ja m a is d 'é le c tro n s. P ar co n tre, si d es n eu trin o s élec tro n iq u es
(prcxluits par une désintégration bêta des produits de fission dans un réacteur nucléaire)
heurtent une cible solide, il y a u n iq iu tm e iit p m d i,u :tio n d 'é le c tr o n s , Jam ais de m uons.
14 4 Les leptons 393

E n 1975, on a d éco u v en un autre lepton, le ta u o n , au SLA C ; celui q u i a fait cette


decouverte, M an in Perl, a partagé le prix N obel de physique en 1995. L e tauon pos.sèdc
son propre neutrino, different des deux autres. Le tableau 14.2 énum ère tous le.s leptons
(paitieules et antiparticules) ; tous ont un nom bre quantique de spin s- de
Il y a des raisons pour div iser les leptons en trois fam illes, chacune constituée d ’une
particule (électron, m uon ou tauon), du neutrino qui lui est assexie cl des anlipariicules
correspondantes. De plu.s, il y a lieu de croire q u 'i l n ’y a q u ^ les trois fam illes de lepton.s
nom m ées dans tableau 14.2. Les leptons n’ont aucune structure interne ni aucune dim en­
sion m esurable ; on croit q u ’il .s’agit de particules fondam entales vraim ent ponctuelles
quand elles interagissent avec d ’autres particules ou avec des oiide,s éleciromngnétiquc.s.

La conservation du nombre leptonique


Les expériences m enées m ontrent que les interactions de purlieu les qui mettent e n je u
d es lep to n s o b éissen t à un p rin cip e d e c o n se rv a tio n d 'u n n o m b re q u a n iiq u e ap p elé
n o m b re le p to n iq u e L . Dans le lablcau 14 2, on attribue le nombre- leptonique ! — 1
à ch aq u e p a rtic u le (n o rm a le ) et L = l à ch aq u e an tip artic u le . O n attrib u e L — 0
à toutes les autres particules, qui ne sont pas des leptons. D e plus, scion les expériences,

dans toutes les uiteraaions de (»iticules, le nouibie leptonique net ih iu i i lu u ju t fa m ille


est conservé .séparément.

11 y a donc vraim ent crois nom bres leptnnique.s, et , la s<imme de i h iu u n doit


d em eu rer in ch an g ée p en d an t une in ieraciio n de p articu les. C e fa it ex p é rim cn lal est
a p ix lé príncipe de co n se rv a tio n d u n o m b re lep to n iq u e .
On peut illustrer ce principe en réexam inant In dcsinlcgration de l’antim uon décrite
dans l’équation 14.8, qui est plus com plète ainsi :

fX c + (14.13)

C o n s id é re /-la d ’abord en ten an t co m p te d e la fam ille des m uons p arm i les lep to n s
Le ia ’ est une antiparticule (voir le tableau 14 2), donc le nom bre leptonique miioniquc
est = —I . Les deux particules e ' et n ’appartiennem pas à la Camille des m u o n s.
elles ont donc ~ 0. Ce qui lai.sse à droite qui, étant une arilip-ianiciile, jvissède aussi
le nom bre leptonique m uonique — —1 D one, les deux m em bres de l'éqiialiou 1 4 1 3
possèdent le m ôm e nom bre leptonique m uonique, à savoir l — —1 , si t e ii’élait pa.s
le cas, le ne sc désintégrerait pa.s ainsi.
Étant donné q u ’aucun m em bre de la famille des électrons n ’apparaît dans le incmbre'
de gauche de l ’équation 14.13, le nom bre leptonique électronique net doit être L f ~ l).
D ans le m em b re de d ro ite d e l ’éq u a tio n 14.13, le p o sitro n , é ta n t une a n ttp a rttc u le
(voir le tableau 14.2), possède le nom bre leptonique électronique — —1. Le iieini inn
électronique v^, étant une particule, possède le nom bre leptonique élecireuiique L, — t 1
D onc, le nom bre le p to n iq u e é lec tro n iq u e n et pour ces d eux p articu le s du m em bre
de droite de l’éq u atio n 14.13 est égalem ent 0 ; le n o m bre leptonique électronique est
égalem ent conservé dans le processus

TABLEAU 14,2 Les leptons’


Mas.se
Famille Particule Symbole (MeV/r^) Charte q Anliparticuie

Électron e‘ 0.511 f e*
Électron Ncuiritio
élecTmnique-’'-'' 0 0 '4
Muon i‘ 10T7 1
Muon Neutrino
m uonii]U c** II n
‘'u "a
Tauon 1 777 -1 T*
Tauon .Neutrino
lauotiique*’'' •'t 0 n

* Tous le.s leptons ont ur nombre qu.nntiqiie de spin do * cl sont dont des fermiotiv
** Los mas.sc.s do.s neutrinos sont très laibles On les iiéglipe dans ce manuel.
394 Chapitre 14 Les quarts, les lep*ons et (e Big Bang

A ucun m em b re d e la fam ille des ta u o n s n ’a p p a raît d an s les d eu x m e m b res de


l ’équation 14.13 ; on doit donc avoir L, = 0 dans chaque m em bre. D onc, les nom bres
quantiques Icptoniques et dem eurent inehangés durant la désintégration décrite
p a r l ’é q u a tio n 14.13, leurs v aleu rs c o n stan tes étan t resp e ctiv em e n t — 1, 0 et 0.
C et e x e m p le n ’e st q u 'u n e illu stra tio n de la co n se rv a tio n d u n o m b re Ic p to n iq u e ;
il s ’applique à toutes les interactions de particules.

✓ vérifiez VOS CONNAISSANCES!: a) Le méson n * se désintègre ainsi : n * + u.


À quelle (amillc de leptons le neutrino r appartient-il ? b) Ce neutrino est-il une partiaile
I iiu une aniiiwrtieulc? e) Quel est son nombre lepionique ?

14.5 Les hadrons


On peut m aintenant traiter d e s had ro n s (b a ry o n s et m éso n s), ces p articu le s qui sont
soum ises à l’interaction l'orte. O n com m ence en ajoutant un autre principe de con.serva-
tion à la liste tléjà établie ; la conservation du nom bre baryonique.
Pour décrire ce principe de con.servation, on prend la désintégration du proton.

P ^ e* + (14.14)

C e processus n e se produit /rwm/s. O n doit s 'e n réjouir, ca r s ’il se produisait, tous les
protons de l’U nivers se transform eraient graduellem ent en positrons, ce qui aurait des
conséquences désastreuses. Pourtant, cette désintégration ne viole aucun des principes
de conservation dont on a parlé jusqu'ici, y com pris la conservation du nombre Icptonique.
C e tte a p p a ren te .stabilité du p ro to n (ain si q u e l’ab sen ce d e n o m b reu x autres
processus qui p o u rra ien t se p ro d u ire au trem en t) s ’ex p liq u e p a r un n ouveau nom bre
q u an tiq u e, le nombre baryonique B , c l un n o u v eau p rin cip e de c o n se rv a tio n ,
la conservation du nombre baryonique.

On attribue fi = + 1 à chaque baryon ; B = —1 à chaque anlibaryon ; S —0 à tous


les autres types de particules. Un processus mettant enjeu des particules ne peut sc produire
s’il fait varier le nombre baryonique net.

D ans le p ro c e ssu s d e l ’é q u a tio n 14.14, le p ro to n a un n o m b re b a ry o n iq u e de


fi = H- ! ; le p o sitro n et le n eu trin o ont to u s d eux un n o m b re b a ry o n iq u e d e fi - ü.
P uisque ce processus ne con.serve pas le nom bre baryonique. il ne peut se produire.

✓ vérifiez VOS CONNAISSANCES 2 : Le mode de désintégration suivant « ’est p a s observé ;

Oucis principes de con.servation. parmi les suivants, ce processus viole t-il ; a) principe
de conservation de l’cncrgic, b) principe de conservation du moment cinétique, c) principe
de conservation de la quantité de mouvement, d) principe de conservation de la charge,
e) principe de conservation du nombre leptonique, f) principe de conscivation du nombre
barviiniquc ? Ixs masses sont - 9.39,6 MeV/o^, — 9,38,3 MeVVt- et = O,,*«11 MeV/< ’

Exemple 14.3
Déterminez si un proton au repos peut .se désintégrer de la manière nombre quantique de charge initial est +1. cl que le nombre quaii-
suivante: tiquc de charge final est 0 I = s 1. l'Hinc, la charge est conservée
P —» ;r “ + . / r '. durant la désintégration nombre Icptonique est aussi conservé, cai
aucuno des trois particules ii’esi un le|iton. de sorte que leurs nombres
Les propnotes du proton et du pion .t ’’ sont énumérées dans le
leptoniques sont tous 0.
tahleaii 14 1 pion jr* a une charge nulle, un spin nul et une
La quantité de mouvement peut également être conservée ; étant
énergie au repos de 135,0 MeV
donné que le proton est au repos, et qu’il a une quantité de niouvement
SOLUTION. Le concept tié utili.sé ici est le suivant : il faut déterminer si la nulle, les deux pions doivent simplement sc déplacer dans des dircctirtis
désintégration proposée viole l'iin de.s principes de conservai ion dont opposées, mais avec des modules de quantité de mouvement égaux
on a parlé. Dans le cas de la charge électrique, on constate que le (de sorte que leur quantité de inouvemcnt totale est mille), pour
14.6 Un autre principe de conservation 395

ctMiscrvcr la quantité tic mouvement. Le fait que la quantité de mou­ Le moment cinétique est-il conserve par la désintégration’?
vement p u isse eue conservée signifle que le proee-ssiLS n’en viole |ias Cette question équivaut à déterminer si la composante S. résultante
le pnncipe de conservation. du spin sur un axe des z arbitraire peut être conservée dans la désin­
Y a-t-il de l’énergie pour la désintégration? Étant donné que le tégration. Les nombres quantiques de spin s des particules dans le
proton t»t au repos, cette question équivaut à demander si l’énergie processus sont \ pour le proton et 0 piHir les deux pions. Donc,
au repos du proton suffit à produire les cncrgic.s au repos et les éner­ dans le cas du proton, la composante S peut être + *A ou —' A,
gies cinétiques des pions. Pour y répondre, on évalue la valeur Q de cl dans le cas de cliaquc pion, elle est OA. On conMaïc que .5. ne peul
la clcsintcgration : cire conservée d’aueunc lacon. Ainsi, le moment ciiictiquc n’est
pas conservé, et la désintégration proposée du proton no peut so
( énergie au rcpos\ _ /énergie au repos\ produire.
Q totale initiale / \ totale finale /
l a désiiiiégraiion viole égalenietil le print ijrc de i unservatkiii
= m - (n iffs
‘ - m .c^) du nonibic baiyonique. le piototi a un nombre baryonique de
= <Î1K,3 M eV - (1.15.0 MeV I O y,6 M eV) i? = -1- 1 , et les deux pions ont un nombre barvonique de l i — 0.
Cette non-con.servation du nombre baryonique eonstiuic une
* 66-1.7 MeV.
autre raison qui permet d ’affinnei que la désiniégraiion ne pein
I.e fait que f j ail une valeur positive indique que l’énergie au repos se produire
initiale est supérieure à l'énergie au repos finale. Donc, le proton possède
suirisammcnt d’énergie au repos pour créer la paire de pions.

Exemple 14.4
Une panieiile appelée xi-moins, repirésentéc par le symbole H , se b) L a d é sin té g ra tio n c o n s rr v e - t -c llc lc.s tro is noinlm :,s Iciito iiiq iie s
désintègre lie la manière suivante :
SOlUnOH. lei. le coiitei^ dé est que tout proccs.sus doit coascrvei séparéiiKnt
5 - ^ A«*-!- nr-. le nombre leptonique net de ehaqw famille de leptons du tableau 14.2.
La particule A” (appelée lambda-zéro) et la pai lieule n sont toutes Ou examine d'abord le iiombie leptonique électronique L ,, qui esl )
deux instables. La désintégration suivante se produit en c a sc a d e pour Péleetron e , —1 pour l'antineutrino élretm niqiir î/^ ci (1
jusqu'à ce qu’il ne reste que des produits stables : pour les autres particules de la désiniégraliou totale de l’équa­
tion 14.15. On constate que net est 0 avant la Ucsmiégration
+ rj, + l'e- et 21+1 ( —1)1 + 2(0 I 0) - 0 apres la dcsintcgration. Lfonc. le
a) La pai'ticule 3 esl elle un Icpton ou un hadron ? S’il s’agit d'un nombre leptonique électronique esl conservé. Vou.s pouvez ainsi
hadron, csi-ce un baryon on un meson ? dénionirei que le nombre leptonique muonique cl que le nombre
leptonique lallmiique sont également conservés
SOUlîlON; Pour répondre à la première question, on utilise le concept (lé
suivant : il n’existe que trois familles de leptons (tableau 14.2), et c) Que pouvez-vous dire au sujet du spin de la particule 3 "
aucune ne comprend la paiticule H". Donc, cette particule doit être
un hadron. SOLUTION: Le tomepi dé c.st ici que la désintégration totale décrite par
Le tontept tié, pour répondre à la seconde question, est qu’il faut l’équation I f. 15 doit conserver la composante S , du spin résultant.
déterminer le nombre baryonique de la particule S . S’il est 1 wi —I. Doiif, on peut déterminer la composante S . du spin de lapaiticule 3
la particule 5 est un baryon ; s'il est 0, il s’agit d’un méson. Pour le dans le membre de gauche de l'équation 14.15 en tenant compte
savoir, on écrit la désintégration totale, de la particule 3 initiale aux des composantes 5- des neuf particules du membre de droue l-ea
priKliiils finaux stables. île la manière suivante ; neuf particules sont des pam eiiles de spin | ; elles oni clniit une
composante S. de + ^ A nu de — iffi. Peu im|xirie le choix fait m ire
S" >p + 2(e + T > c ) (14,15) ces deux valeurs |xjssiblcs |Xiur S la valeur 5- résuliaiiic de cc.s ncul
Dans le membre de droite, le proton a un nombre baryonique de -• 1 ; particules doit être un multiple d em i-en tier de A. Done, la pariieiile
les électrons et les neutriiio.s ont un nombre baryonique de 0. l')oiic, 2 doit avoii une eimipo.sanli: ,5. d ’uii d rn ii-e n tle i mulllplit |iai A
le nombre baiyonique net du membre de droite c.st i I , ce qui iloil ce qui .signifie que son nombre quaiitiquc de spin s doit être un demi-
alors être le nombre baryonique de la particule uiiiijue du membre de entier. (fcn fait, le nombre qiianiiqiie est ; la pariiaile H se trouve
gauche. On en conclut que la particule S est un baryon. avec d'autres bai yons de spin l dans le tableau 14 1.)

14.6 Un autre principe de conservation


L es p articu les o n t dos p ro p riétés intrin-sè-ques on plus do ocU cs q u i ont é té vuos Justin’ii i
la m asse, la ch a rg e , le sp in . le n o m b re lep to n iq u e c l le n o m b re b a ry o n iq u e . L a p re m iè re
d e CCS pixjpriéiés ad d itio n n e lle s a été d éo o iiv erte q u a n d les o b o n hours ont olistT vé qiio
c e rta in e s n o u v e lle s p a rtic u le s, c o m m e le k ao n (K ) et le sig m a ( 1 ) . ô ta ie n t a p p arcin m cn l
to u jo u rs p ro d u ite s d e u x p a r d e u x . 11 se m b la it im p o ssib le d e n ’en p ro d u ire q u 'u n e seu le
à la fois. D onc, si un faisceau de p io n s é n e rg é tiq u e s in te ra g it av ec les p ro to n s p résen ts
d an s u n e ch a m b re à b u lles, la ré a c tio n
7T^ + p - > K " + i ' ( i LI f t )
396 Chapitre 14 Les quorks. les (eptons et le 6lg Bang

SC produit souvent. P our sa part, la réaction

TC ' + P Л" + 2 (14.17)

qui ne viole a u c u n e d es lo is de c o n s e rv a tio n co n n u e s d an s les p rem iers jo u rs d e la


physique des particules, ne se produit jamais.
O n a p lus tard p ro p o sé (M u rray G cll-M a n n aux É tats U nis cl K. N ish ijim a au
Japon, chacun de leur côté) qu e certaines particules possédaient une nouvelle propriété,
a p p e lé e c t r a n g c tc , ay an t son p ro p re n o m b re q u an tiq u e S et so n p ro p re p rin cip e d e
co n serv atio n . (Il n e fau t p as c o n fo n d re le sy m b o le 5 av ec ce lu i d u sp in .) Le term e
é t r a n g e i é d éc o u le du fait q u e. av an t q u ’on ne d éco u v re l'id e n tité d e ces p articu les,
on les considérait com m e des « particules étranges » ; le nom leur est resté.
L e p ro to n , le n eu tro n et le p io n o n t 5 = 0 . c ’e st-à -d irc q u ’ils ne so n t pas
« é tra n g e s » . Il ap p a raîi, p ar co n tre , q u e la p a rtic u le K ‘ a une é tra n g e té 5 = + 1 et
q u e 2 ' a 5 = — 1. D ans la réa ctio n d éc rite p ar l'é q u a tio n 14.16. l’é tra n g e té initiale
est 0 ; l’étrangeté finale est aussi 0. D onc, l’ctran g eté est co n serv ée dans la réaction.
C e p en d an t, d an s la réa ctio n d écrite p ar l’é q u atio n 14.17, l’é tra n g eté fin ale e st - 1 ;
ré tra n g e ié n ’y étant pas con serv ée, cctie réaction ne peut se produire. Il sem ble donc
q u ’il faille ajouter un autre p rincipe d e co n serv atio n à la liste déjè établie, à sav o ir la
co n serv atio n d e l’é tra n g e té .

L’ctr.'uigeié est conservée dans les proeessu.s mettant en icii l'inleraclion forte,
mais clic n’est pas consers'éc dans les processus mettant en jeu l’interaction faible.

11 peut se m b le r m alad ro it de d o n n er une n o u v elle p ro p riété aux p articu le s p o u r


expliquer un petit ca.s.se-téte com m e celui représenté par les équation.s 14.16 et 14.17.
CependanL l'étrangeté et son nombre quantique sont rapidement apparus daas de nombreux
dom aines de la physique des particules, de sorte que l’étrangeté est aujtiurd’hui pleinement
acceptée com m e un attribut légitim e des particules, de pair avec la charge cl le spin.
5= 0
Mc vous lais.sez pas trom per par le caractère singulier de ce nom. L’étrangeté n ’est pas

Г
/ X®
\ г .4 = -1
plus mystérieuse que la charge. U s ’agit de deux propnétés que les particules peuvent (ou non)
avoir, chacu n e étan t d éc rite pai’ un nom bre q u an tiq u e ap p ro p rié. C hacu n e o b éit à un
principe de conservation. D’autres propriétés des particules ont été découvertes, et elles
portent dc.s nom s encore plus curieux, etm une c h a r m é et h o tlo m . mais toutes sont parfai­
tem ent légitim es. O n peut voir, par exem ple, conunent l’étrangeté « ju stifie sa présence »
\ /
5 -2 en perm ettant de dévoiler d ’im portantes rcg u lan tés dans les propriétés des particules.
7=-l 7= 0 7 = HI
л)
14-7 La voie octuple
A près la d cttiu v é rtc d ’une cen tain e de hadrons, les physiciens sc sont dem andé co m ­
K»--------- K* S= +1 m ent classer ceux-ci de fayon plus précise. En con sid éran t les b ary o n s de plus (aible
masse, on trouve huit baryons (neutron et proton inclus) qui ont un nom bre quantique
de spin de '. l.c tableau 14..3 donne certaines tie leurs autres propriétés. La figure 14.4 a)
^ n \ m ontre le fa.scinant m otif qui apparaît si l’on représente graphiquem ent l’étrangclé de
r" n 5 0
ces baryons en fonction de leur nom bre quaniique d e charge, cl q u 'o n utilise un axe en

\ ’
K‘ .-----------K"
/ S= -1
pente pour désigner les nom bres quanliques de charge. .Six des huit bary o n s form ent un
hexagone au centre duquel .se trouvent les deux b ary o n s résiduels O n dit que les huit
baryons cotLStiiuent un m ultiplet de huit particules : un octet.
7= 1 7*0 7 = -H O n peut m aintenant pa.sser des baryons aux m ésons de faible masse. N eu f m ésons
b) ay an t un spin de 0 so n t n o m m és d an s le tab leau 14.4. .Si on les rep rése n te d an s un
diagram m e en pente étrangeté-charge, com m e dans la figure 14.4 b), le m êm e schém a
Fgure 14 4 a> L e s d ié r n a de l ’octet
m o n lta n l le s liiiii b a r y o n s de sp in
fascinant apparaît. C ette fois-ci. les m ésons form ent un octet (groupe d e S particules)
n ie iilio n n é s d a n s le tableau l'1.3.
et un singulct (une particule seule). D ans ces diagram m es, les particules sont reliées par
I CS particules stml représentées des opérateurs de sym étrie qui form ent ce que les m athém aticiens appelleni le groupe
par des disques sur un graphique SU(3). Ils ont été proposés en 1961 île façon indépem iante p ar M urray G ell M ann, du
étranpeté-charge. où l’on utilise C alifornia In.sututc o i Technology, ci Ynval N e’einan, d e Г Imperial C ollege, à LonJrc.s.
un axe incliné pour désigner le nombre G cll-M ann les a nom m és la m i e o c tu p le * parce (|uc le groupe SI !(3) contient huit opéra-
quantique de charge, b) t'n 4'héma leuI^ de .symetne. I.es deux .schémas de la figure 14.4 sont des exem ples de schém as de
semblable pour les neuf mésons de spin m ultiplets rcpré.sentant des groupt's de bary o n s et de mésons.
Zéro mentionnés dans le tableau 14.4.
• Emprunt au bnuddhisn’u?.
14.7 La voie octuple 397

L a s y m é trie d e la v o ie o c tu p le p ré v o it u n d é c u p le t d e b a ry o n s d e sp in | (n o n
illu stré ic i), c ’e st-à -d ire u n g ro u p e d e lü b a ry o n s d isp o sé s d a n s un sch ém a en tria n g le
re sse m b la n t à la d isp o sitio n d es d ix q u ille s d an s u n je u . C e p e n d a n L q u a n d c e ty p e de
s c h é m a a é té p ro p o s é p o u r la p re m iè re fo is, se u le m e n t n e « /d e c e s p a rtic u le s é ta ie n t
co n n u e s ; la « q u ille c e n tra le » était a b sen te, t n 1962, g u id é p a r la th é o rie e t la sy m é trie
d u sch ém a, G eli-M an n lit u n e pré^diction q u i c o n sista it e n ce q ui su it ;
11 existe un baiyon de spin | ayant une charge de - 1 . une étrangeté d t —.3
et une éneigic au repos d’environ 1 680 McV. Si vous cherchez cette particule
o n u ’g a m oins (comme je propose de l'appeler;, je jTense que votis la Itouvere/..

U n e é q u ip e d e p h y s ic ie n s d irig é e p a r N ic h o la s S a m io s . d u B ro o k h a v e n N a tio n a l
L a b o ra to ry , re le v a le d é fi e t tro u v a la p a rtic u le « m a n q u a n te » , c o n firm a n t toute!; les
p réd ictio n s d e ses p ro p riétés p a r G cll-M an n . R ien n e vaut u n e œ nT irm atiori exjiérim en tale
p o u r stim u le r la co n fia n c e e n u n e thétirie.
L a v o ie tH.’tu p le est à la p h y s iq u e d e s p a rtic u le s ce q u e le ta b le a u p é rio d iq u e c.sl
à la c h im ie . D an s c h a q u e c a s, il y a u n m o tif d ’o rg a n isa tio n d a n s leq u el les vacancc.s
(p a rtic u le s m a n q u a n te s o u é lé m e n ts m an q u an ts) so n t é v id e n te s c o m m e le n e z au milieu
d u v isag e, g u id an t les c h e rc h e u rs. E n c e q u i a tra it au ta b le a u p ério d iq u e , so n e x iste n c e
m ê m e la isse fo rte m e n t c ro ire q u e le s a to m e s d e s é lé m e n ts n e so n t p as d e s p a rtic u le s
fo n d a m e n ta le s, m a is q u ’ils o n t u n e s tru c tu re s o u s-ja c e n te D e m ê m e , les sc h é m a s d e
m u ltip le ts s u g g è re n t fo rte m e n t l’id ée d ’u n e s tru c tu re so u s-ja c e n te au x m é so n s r i a u x
b a ry o n s, à l’aide d e laq u elle o n p o u rra it co m p ren d re leu rs p ro p riétés C e tte stria turc peut
ê tre e x p liq u é e p a r le m odèle d es quarks, q u e n o u s ab o rd e ro n s d an s la .section su iv an te.
TABlfAU 14.3 HuH baryons de spin formant un odet
Nombres quantiques
Particule Symbole Masse iM cV/r') Charge <i Étrangete i

Proton P 9J8.3 +1 0
Neutron n 939,6 0 0
Lambda A« 1 115,6 0 -l
Sigma 1 189,4 +1 -1
Sigma 1 192,5 fi -1
Sigma S 1 197,3 -1 1
Xi l 314,9 0 -O

Xi 1 321.3 -l -1

m m 14.4 Neuf mésons' de spin 0 formant un octet et un singulet


Nombrc.s quantiquus
Particule Symbole Masse (MeV/c-) riiarge q Étrangeté 5

Pion 135.0 fi 0
Pion TT'*' 139,6 -c 1 (1
Pion 7T~ 1.39,6 -1 fi
Kaon K+ 493,7 +1 n
Kaon K 493,7 1 -1
Kaon K» 49/,7 0 1
Kaon K" 497,7 fi -1
ftta V 547-5 0 fi
Eta prime v' 957.8 0 0
* Tous les mesons wmiI des bosons ayani dcs spins de t). 1 . 1 .... Tons reiix qui sont Homines
ici ont un spin de 0
398 Chapitre 14 Les quarks, les leptons et le Big Bang

uu
lUI
i-0 14.8 Le modèle des quarks

y
■d

S
iid ud
\ l .S = - I
En l% 4 , GeH-Marin et G eorge Z w eig affirm creiit, chacun de leur côté, q u ’on pouvait
iacilem eni com prendre les schéma.s de la voie octuple si Гоп com prenait que les m ésons
et les bai yons sont com posés de sous-unités, que G cll-M ann a nom m ées q u a rk s . O n peui

\ rt
,SS/
U \ S - -2
voir d ’abord trois d 'e n tre eu x ; le q u a r k u p (u), le q iu tr k d o v iti (d) et le q u a r k éirunf> e
(s px)ur xtra n fie). et leur attribuer les propriétés m entionnées dans le tableau 14..'5. (Les noms
de ces q u ark s, à l'in s ta r d e ceu x d es tro is a u tre s d o n t on tra ite ra p lu s loin, ne sont
que des étiquettes com m o d es ; ils n ’o nt aucune sig n ificatio n p articulière. O n appelle
q = -\ ç= 0 î= +' collectivem ent ces nom s s a v e u r s ¿les q u a r k s . O n au rait tout aussi bien pu les appeler
a) vanille, c h tx o la t et frai.se au lieu de up. dow n et étrange.)
TI est [xis.stblc que les nombrc.s quanliques d e charge fraelioim aires des quarks vous
agacent un peu. Cependant, ne p o rte / aucun jugem ent ju sq u ’à ce que vous constatiez la
ds u$ S = +1 clarté avec laquelle ces ch a rg es fra ctio n n aires ex p liq u en t les ch arg es ob.scrvées des
m ésons et des baryoas. D ans toutes les situations norm ales, soit sur la Terre ou dans tout
prcKessus astronom ique, les quarks sont toujours regroupés par deux ou par trois pour
^ dit uu des raisons qui ne sont pas encore tout à fait expliquées. C ependant, en 2(KX), certains
dCi uil S^O
accélératcui^ avaient lînalcm ent pu projeter des atom es dans des cibles avec siinisam m enl
û
\ sd
\
^ s=
d cncrgic pour que la collision sépare les quarks Itxs uns des autres pendant un bref instant.
O n sa it com m en t on p eu t lie r d es ato m es en co m b in an t leurs élec tro n s et leurs
noyaux. On peut voir m aintenant com m ent on peut form er des m esons et des bosons en
-1 - 0 com binant tk s quarks. U ne chose apparaît évidente :
b) Il n'exi.stc aucun méson ni harytxi dont on ne peut coniprcndre les propriétés à l'akle
Figure 145 a) Le.s composiiioas d’une combinaison de quarks. Inverseineni, il n ’y a pa.s de combinaison possible de quarks
des quarks des huit baryons de spin j qui ne corresponde pa.s à un méson nu à un baryon.
figiirani dans le tableau 14.3. (Bien que
les deux baryons eentraux partagent E xam inons d ’abord les bary o n s du tableau 14.3.
la même structure de quark, le sigma
est un êlat excité du lambda ; Ouarks et baryons
il SC désintègre en lambda par l’émission C haque baryon est constitué d ’une com binaison d e trois quarks ; certaines de ces com bi­
d’ut) photon de rayon gamma.) n aiso n s sont p ré se n té e s d a n s la fig u re 14.5 a). P ar rap p o rt au n o m b re b a ry o n iq u e ,
b) Les eoniptrsiiinns des quarks on constate que tout en.scmble de trois quarks (chacun avec fi = + ^ ) donne un baryon
des neuf mésons de .spin U figurant (où fi - + 1).
dans le tableau 14.4. La situation est sem blable avec les charges, com m e on peut le voir dans les trois
exem ples. P our le proton, la com position des quarks est uud. et le nom bre quantique de
charge est
r/(uud) = I + 5 5) = + 1
Pour le neutron, la com position des quartes est udd, et le nom bre quantique de charge est
^(udd) = I + ( - i ) + ( - i ) = 0 .
P o u r la p articu le (sig m a m o in s) la co m p o sitio n des q u ark s e.sl d d s, et le nom bre
quantique de charge est
g ( d d s ) = - i + ( - ' ) - t - { - i ) = - 1.
La situation est la m ême avec les nom bres quantiques d ’étrangeté. Vous pouvez le vérifier
en con.sultani le tableau 14.3 dans It- cas du X et le tableau 14.5 dans le cas des quarks,

TAB11AU14.5 Lesquorks’
Nombres quantiques
Mas.se N om he
Panicule Symbole (MeV/c^) Charge q Étrangeté S baryonique fi Antiparticule
Up U 5 0 lï
1
Down d 10 % 0 a
Charmé c 1 500 0 c
Étrange 200 1
s 3 -1 s
Top 1 175 0(Ю 0* T
Bottom b 4.300 _ i 0
4 + 1 b
• Tnus les qiiarks (y com pris les antiqimrks) <'n» un spin de ^ d sont donc des fermions. Le nointifc
qu.iniiqiie q. .Sou B d ’on antiquark est l'opposé du nombre quantique du quark correspondant.
14.8 Le modèle des quarks 399

Quarks et mésons
L es m ésons so n t d e s p a ire s q u a rk -a n tiq u a ik ; ce rtain e s de le u rs c o m p o sitio n s so n t
» p ré se n té e s d an s la fig u re 14.5 b). Le m o d èle q u ark -an tiq u ark est co h éren t q u an t
au fait que les tnéso n s ne sont pas des b ary o n s, d onc q u e les m ésons o n t un n o m bre
baryonique S = 0. Le nom bre b aryonique d ’un quaik est + 1 e t celui d ’un antiquark est
- J ; donc, la com binaison des nom bres baryonique.s dans un m éson e.si O.
On peut s ’intéresser au m éson n , qui est constitué d ’un quark up (u) et «l’iin quark
an tid o w n (d ). Si o n reg a rd e le tab leau 14.5, on co n state q u e le nom b re q u an liq u e
de ch a rg e du q uark u p e st + § e t que c e lu i d u q u ark an lid o w n est f ' (le signe est
opposé à celui du q u ark dow n). 11 s ’ag it là d e n o m b res qui s ’ad d itio n n en t bien p o u r
donner un nom bre quantique de charge de -+• 1 au m éson rr ; c 'e st donc dire <jue

ç(ud) = ^ + i: = + 1 .

Tous les nom bres quantiqiics de charge et d ’étrangeté de la figure 14,5 b) sont t riliéients
par rapport à ceux du tableau 14.4 et de la ligure 14.4 b). C onvainque/.-vous que toutes
les com binai.sons q u ark -an tiq u ark iip. dow n et étran g e so n t u tilisé es e t q u e tttus les
mé.sons de spin zéro connus sont expliqués.

VÉRIFIEZ VOS CONNAISSANCES 3 : La com binaison d ’un qu ark dow n (d) et d ’un quark
<nnttup (ü) s'appelle-l-cllc a) un mdson b) un proton, c) un meson n ~ , d) un méson tt’’
ou e) un neutron?

Un regard neuf sur la désintégration bêta


À q u o i re sse m b le la d é sin té g ra tio n b êta du p o in t d e vu e du m o d èle d es q u ark s ?
L 'équation 12.23 présente un exem ple tyj.ie d e ce processus ;

«S 4 + 1/.

A près la découverte du neutron et le travail de b e rm i sur sa théorie de la U e sinlégraiioii


bêta, les physiciens en sont venus à considérer le proces.sus fondam ental d e la desinte
gration bêta com m e une transform ation d ’un neutron en proton à l ’intérieur du noyau,
transform ation s ’effectuant de la m anière suivante:

n P I e ~ + P»,

où le neutrino est identifié d e m anière com plète. De nos jo u rs, d es o b serv atio n s plii.s
ap profondies p erm etten t de co n stater q u 'u n neutron (luld) jieui se tia n sfo rm er en un
proton (uud) en transform ant un q uark dow n en quark up O n considère m aintenant le
processus fondam ental d e la désintégration bêta com m e étant

d —> U i - c 4-17».

Donc, en développant chaque jour davantage les connaittsances sur la nature Ibndameniate
de la m atière, on peut exam iner les processus fam iliers encore plus à fond. O n constate
égalem ent que le m o d èle d es q u ark s, en plu.s d ’aid er les eh e rch ru rs à co m p ren d re lu
siructiiie des particules, clarifie les inU'raclion,s entre ces particules.

Encore d'autres quarks


Il existe d ’autres particules et d ’autres m ultiplets dont on n 'a pas parlé Pont les expliquer,
on d oit p résenter to n s autres quarks, le q u n r k i h a r m f (c). le q u a r k to p (t) et le q im r k
b o tto m (b), D t'nc, il existe un total t k six quarks, qui .sont énum érés dans le tableau 14,5.
N otez que trois quarks sont exceptionnellem ent mas.sifs. le plus m assif d ’entre eux
(top) l’étant presque 17(1 fins plus q u ’un proton P o u rg é n é re i des partit nies qui co n ­
tiennent de tels quarks, qui pos,sèdcnt des éncrgic.s an repos très élevées, on doit produire
des q uantités d ’én erg ie de plus en p lu s g randes, raison pour Inquelle ces trois quarks
n ’ont pas été découverts plus tôt.
400 Chapitre 14 Les quarks, les leptons et le Big Bang

La prem ière particule contenant un quark charm é à être observée fut le m éson
dont la structure des quarks est cc. Elle fut découverte sim ultaném ent et indépendam m ent
en 1974 par des groujjes diriges p ar Sam uel Ting, au BrtK)khaven N ational L aboratory,
et par B urton Richter, à I'Liniversitc Stanford.
La production en laboratoire du quark top a déjoué tous les efforts ju sq u ’en 1995.
quand son existence a finalem ent été dém ontrée dans le Tevatron, un grand accélérateur
de particules du Ferm ilab. D ans cet accélérateur, on oblige les protons et les antiprotons,
chacun ayant une énergie de 0,9 TeV ( = 9 x 10 " eV ), à entrer en collision aux centres
de deux grands détecteurs d e particules. D ans de rares occasions, la collision entre un
prtrton et un antiproton crée une paire de quaiL s top-antitop ( tt) . qui ,se désintègre tr è s
rapidem ent en particules détectables ; de la présence de ces particules on peut déduire
l’existence de la paire top-antitop.
R evenez brièvem ent au tableau 14.5 (les quark s) et au tableau 14.2 (les leptons)
et notez la sym étrie claire entre ces deux « paquets de six » particules, chacun sc divisant
naturellement en trois familles de deux particules correspondantes. Selon les connaissances
actuelles, les quarks et les leptons sem blent vraim ent être des paiticuies fondam entales
ne possédant aucune structure interne.

Exemple 14.5
La particule S " a un nombre quantique de spin s de j , un nombre quantique de clwige ^ est - I, et le nombre quantique de charge q
quantique de charge <y de —I et un nombre quantique d’étrangeté S de chaque quark étrange est de —3 . Donc, le troi«èmc quark x doit
de —2. On sait également qu'elle ne contient aucun quark bottom. avoir un nombre quantique de charge de - j po“’’ qu'on puisse être
Quelle combinai.son de quarks forme la paiticule H ' ? en présence de

SOIÜTIOH ; Grâce à l'exem ple 14.4, on sait que la partieule H e.si q ( E ) = q(ssx)
un baryon. Le tontepl cli est alors qu’elle doit consister en trois quarks
= - J + ( - } ) + ( - 5 ) = -1 -
(non pas en deux, comme dans le cas d’un méson).
Outre le quark s. les seuls quarks affichant g = ~ \ sont le quark
On prend ensuite l’étrangeté 5 = —2 de 2 “ . Ici. le tontepl tie est
dow n d et le quark bottom b. Étant donné que l’énoncé du pioblcme
que seul le quark étrange s et le quark aniiétrange s ont des valeurs
élimine le quark bottom, il reste le quark dowin. Cette conclusion esi
d’étrangeté non nulles (voir le tableau 14.5). De plus, étant donné
également cohérente quant aux nombres quantiques baryoniques :
que seul le quark étrange s a une valeur d ’étrangeté nt'f’a tive, H~
doit contenir ce quark. En fait, pour que c ait une étiangeté de —2. « ( = - ) - B(ssd)
elle doit contenir deux quarks étranges.
Pour déterminer le troisième quark, qu'on nommera x, on peut
tenir compte des autres propriétés connues de H . Son nombre Donc, la composition des quarks de la particule H est ssd.

1A.9 Les interactions fondamentales


et les particules messagères
Du catalogage des particules, on passe à l’étude des interactions qui les anim ent.

L'interaction électromagnétique
À l’échelle atomique, on dit que deux électroas exercent une interaction électrom agnétique
l’un su r l'a u tre selo n la loi d e C o u lo m b À un n iv eau inférieur, cette in te ractio n est
décrite par une brillante théorie appelée c le c tro d y n a m iq u c q u a n tiq u e . D e ce point de
vue. on dit que ch aq u e électro n sent la présence de l'a u tre en échangeant de.s photons
avec lui
O n ne peut détecter ces photons parce que. une fois q u 'ils sont ém is par un électron,
ils sont absorbés par l'autre très peu de tem ps après. A cause de cette existence indétectable,
on les appelle p h o to n s v irtu e ls. P uisque ces photons établissent des com m unications
entre les deux particules chargées, on les appelle parfois p a r t ic u l e s m e s s a g è r e s .
.Si un électron ém et un photon en dem eurant lui-m êm e inchangé, l'én erg ie n ’est pas
co n serv ée . L e p rin cip e de co n serv atio n d e rén e rg iV est to u tefo is sau v eg ard é par le
principe d ’incertitude, décrit par l'éq u atio n 9.25 ;

A E • A l <= h . (14.18)
14.9 Les interactions fondamentales et les particules messagères 401

Ic i, on in terp rète c e tte re la tio n d e la fa ç o n su iv a n te : v o u s pouvez. « m ettre à d é c o u v e rt »


u n e q u an tité d ’én erg ie Æ , violant ainsi le p rin cip e de la cc«iserv ation de ré n c rg ic ,/;o iu v «
q u e v o u s la « r e to u r n ie z » d a n s u n in te rv a lle A t d é te rm in é p a r fil A F , d e so rte q u e la
v io latio n n e p u isse être d étectée. L es p h o to n s v irtu els le fom . A insi, q u an d un électro n A
ém et un p h o to n v irtu el, le d éco u v ert e n én e rg ie e,st rap id em en t co m b lé q u a n d cc t électro n
re ç o it u n p h o to n v irtu e l d e l ’é le c tio n B, et la v io la tio n d u p rin cijx i d e c o n se rv a tio n d e
l’é n e rg ie p o u r la p a ire d ’é le c tro n s e st d issim u lé e p a r rin c e rtitu d e in h éren te.

L’interaction faible
L a th é o rie d e l'in te ra c tio n faib le, q u i s 'a p p liq u e à to u te s les p a rtic u le s, a r i r fo rm u lée
p a r a n a lo g ie a v e c la th é o rie d e l ’in te ra c tio n é le c tro m a g n é tiq u e . L e s m e s s a g e rs q u i
tra n sm e tte n t l ’in te ra c tio n la ib le e n tre les p a rtic u le s n e so n t to u te fo is p a s d e s p h o to n s
(sa n s m a s s e ), m a is d e s p a rtic u le s m a s s iv e s , re p ré s e n té e s p a r le s s y m b o le s W ' , W~
e t Z °. C e tte th é o rie fu t si fru c tu e u s e q u ’e lle p e rm ii d e c o n s ta ie i q u e l ’im e ra c iio n
é le c tro m a g n é tiq u e e t l ’in te ra c tio n fa ib le é ta ie n t d eu x a sp e c ts d ’une se u le in le r a r tio n
é le c tro fa ib le . C e ré su lta t e st u n e su ite lo g iq u e au trav ail d e M ax w ell qui a déc o u v e rt
q u e les fo rc e s é le c triq u e et m a g n é tiq u e é ta ie n t d iffé re n ts a sp ects d ’u n e seu le in teractio n
é le c tr o n u ifin é tiq u e.
L a th é o rie é le c tro fa ib lc p e rm it de p ré d ire le s p ro p riétés d e s p a rtic u le s m essag ères.
O n a p réd it, p a r e x e m p le , q u e leu rs c h a rg e s et leu rs m asses é taien t c elles im 1i(|iiées d an s
Particule Charge Masse le tab leau c i-c o n tre . R a p p e le z -v o u s q u e la m a sse d u p ro io n n ’est q u e d e O.d'lR O eV fr^ ;
W ±e 80,6 GeV/c^ c e s p artic u le s so n t m a ssiv es ! L e p rix N o b e l de p h y siq u e d e 1Q79 a é té remi.s à S h eld o n
Z 0 9L2C.CV/H
G lash o w , S te v e n W ein b e rg e t A b d u s S a la m jxiur le u r trav ail su r la th é o rie de l’in te ra c ­
tio n électro faib lc.
C e tte th é o rie a d ’a b o rd é té v é rifié e d a n s les an n é e s 1070. D e p lu s, u n e ét]ui[ie du
C E R N , d irig é e p a r C a rlo R u b b ia, l’a co n firm é e e n 1983 d e fa ç o n écla ta n te Ch o b se rv a n t
d ire c te m e n t les tro is p a rtic u le s m e ssa g è re s et e n m e s u ra n t le u rs m assr.s q u i xc so n t
a v é ré e s é g a le s au x p ré d ic tio n s th é o riq u e s . L e p rix N o b el d e p h y s iq u e d e 1984 a été
rem is il R u b b ia e t à S im o n van d ci M e a ' p o u r c e brillan t trav ail ex p érim en tal.
O n p e u t m e ttre en é v id e n c e la c o m p le x ité d e la p h y siq u e d e s p a rtic u le s d ’au in iir-
d ’hui e n la c o m p a ia n t à u n e an cie n n e e x p é rie n c e e n p liy siq u e d es particules q u i ittcna à
la d éco u v erte d u n eu tro n , et qui valut à son au te u r le p rix N o b el C ette dét o u v erte d ’u n e
im p o rta n c e v itale a été fa ite au c o u rs d ’u n e e x p é rie n c e sim p le <ni des particules é m ises
p a r dc.s su b stan ces rad io activ es serv a ie n t alo rs d e p ro jectiles ; e lle fut dsicum entée e n 10 3 2
so u s le titre « Po.ssiblc E x isten ce o f a N eu tro n » (E x isten ce p o ssib le d 'u n n e u u m ) , d o n t le
seul au te u r é ta it Ja m e s C h ad w ick .
Par co n tre, l’ex p érien ce q u i a m en é à la d éco u v erte des particu les m essag ères en 1983
a é té ré a lisé e d a n s u n g ra n d a c c é lé ra te u r d e p a rtic u le s a y a n t e n v iro n 7 k m d e C ircón
féren ce et fo n c tio n n a n t à plu.sieurs c e n ta in e s d e m illia rd s d ’cIcctronvolL s. L e d é te c te u r
d e p artic u le s p rin c ip a l p èse à lui .seul 2 0 M N . C^ette e x p c n c n c c a n é c e ssite le tra v a il de
130 p h y sic ie n s ap p a rtc n a n i à 12 c e n tre s d e rec h e rc h e d e 8 p a y s d iffé re n ts, a p p u y é s p a r
u n e im p o rta n te éq u ip e .
L’interaction forte
O n a é g a le m e n i fo rm u lé u n e th é o rie s u r l’in te ra c lio n fo rte , q u i esi la fo rce q u i lie les
q u ark s e n sem b le. D an s c e c a s. les p articu les m es.sagères sont ap p elées g lu o n s cU co m tiK
le p h o to n , c lic s n ’o n t p a s d e m a sse . D a n s c e tte th é o rie , o n s u p p o s e q u e c h a q u e
« sa v e u r» d e q u a rk e x iste en tro is v a rié té s q u i. ix iu r p lu s de com m cK litc. o n t été a p p e lé e s
ronge, jaune et bleu. 11 y a d o n c tro is q u a rk s u p . so it un d e c h a q u e couleur, e t il e n va de
m em e p o u r les a u tre s quark.s. I ^ s an tiq u a rk s ex isle n i é g a le m e n t en tro is c o u le u rs, q u ’un
ap p elle antirouge, antijaune e t antihleu. Il n e faut to u telo is pa.s crotrc q u e les q u ark s so n t
c o lo rés, c o m m e d e s fèv es e n g elée. C e s n o m s sont atirihinN p a r sim p le cornm cxliu'. m ais
on v e rra q u ’ils o n t u n e certa in e )u stificatio n fo rm elle
lai force qui agit entre les q u aik s est appelée in te ra d in n fo rte , et la tlicorie sous iaccntc.
par analogie à féle c tro d y n a m iq u e quantique, est appelée c h ro m o d y n a m iq u e q iu m tiq iic .
Il sem b le q u e les quark.s n e p eu v en t s as.scm bler q u 'e n co m b in aiso n neutre, san s co u leu r
Il y a (leux m o y e n s d ’a tte in d re la n eu tra lité d e co u leu r. D an s la th e o n e , la c o m b i­
n a iso n ro u g e -f- ja u n e b le u d o n n e b la n c , q ui est n e iilre : d o n c , o n p e u t ré u n ir tro is
q u ark s p o u r fo rm e r u n b a ry o n . à c o n d itio n q u il y en ait un l.iune, u n ro u g e et u n bleu
I x a iin h in a iso n an tiro u g e + an tijau n e f an tib lcu d o n n e ég alem en t b lan c, d e sorte q u o n
402 Chapitre 14 Les quarks, les leptons et le Big Bang

p eu t u n ir tro is an tiq u ark s (d es b o n n es a n tico u lcu rs) p o u r fo rm e r un a n tib ary o n .


F inalcm enl, la com binaison rouge + aiuirouge ou jau n e + antijaune ou bleu + antibleu
donne aussi blanc. D onc, on peut form er un inéson en com binant un quark cl un anti-
quark. La règle de la neutralité de couleur perm et aussi des com binaisons plus com plexes
com m e la com binaison rouge + ja u n e + bleu + rouge + antirouge. L a particule ainsi
form ée est appelée pentaquark ; elle a été observée en 2003 p ar une équipe au Japon et
par une autre équipe aux Ètais-U nis.
L 'interaction forte ne lie pas seulem enl les quarks en baryons et en m ésons, elle lie
aussi cerlaines de ces d ern ières p articules en tre c lic s ; dans ce cas, on l’appelle trad i­
tio n n e lle m e n t fo rce n u cléaire. A in si, non se u lem en l l ’in te ra c tio n fo rte lie -t-e lle
en sem b le les q u ark s p o u r fo rm er d es p ro to n s e t d es n eu tro n s, m ais e lle lie au ssi les
p ro to n s et les n eu tro n s p o u r fo rm e r des noyaux. C e c i est éq u iv a le n t à l’in te ractio n
ék'c trom agnétique qui lie les électrons e t les noyaux pour fo rm er des atom es, m ais aussi
les aïom es ensem ble pour form er des m olécules.

Le rêve d'Einstein
L ’u n ific atio n des in teractio n s fo n d am en tales d e la nature en une in teractio n unique
(qui occupa Einsiein vers la fin de sa vie> est un sujet de recherche actuel. O n a vu q u ’on
a réussi à com biner l’interaction faible avec l'élc'ctrom agnétism e et q u ’ainsi on a pu les
considérer com m e une in te r a c tio n é le c t m f a i h l c unique. Les théories qui perm ettraient
d ’ajo u ter l’interaction forte à cette com b in aiso n , ap p elées f> randea t h é o r ie s u n ifié e s ,
sont à l'é tu d e . L es th é o ries q ui p e rm e ttra ie n t de c o m p lé te r le trav ail en y ajo u tan t
la gravite (parfois appelées th é o r ie s d u to u t) en sont à une étape en courageante, m ais
spéculative.

14.10 Une pause pour réfléchir


On peut m ettre ce q u ’on vient d ’apprendre en perspective. Si tout ce qui voius intéiesse
est la structure du m onde qui vous entoure, vous vous en tirerez bien avec l’électron, le
neutrino, le neutron e t le proton accom pagnés du phm on com m e p articu le m essagère.
C o m m e q u e lq u ’un l ’a d ii, on p eu t p lu tô t bien n a v ig u e r su r le « v a isse a u T e rre » en
connaissant seulem ent ces particules. O n peut observer quelques unes des particules les
plus exotiques en les cherchant dans les rayons cosm iques ; cependant, p o u r observer la
plupart d ’entre elles, il faut construire des accélérateurs im posants et travailler à grands
frais en déployant de grands efforts.
L ’effort à fournit est grand parce que. du point de \u e énergétique, on vit dans un
m o nde de te m p craiu res très b asses. M ôm e au ce n tre du S oleil, la v aleu r de AT n’csl
que de 1 keV environ. Pour produire ces particules exotiques, il fau t pouvoir accélérer
les protons ou les électrons à des énergies de l’ordre des G eV et des TeV, et plus.
Il fui une période où la tem pérature é t a i t p arto u t su ffisan te p o u r fournir une telle
én e rg ie, et m êm e p lu s. C e tte p ério d e d e te m p ératu re s e x trê m em en t é le v é e s d ate
du début de l’U nivers, pendant le Big B an g , au m om ent où l’U nivers est né (ainsi que
l ’espace et le tem ps). D onc, les scientifiques étudient les p articules à hautes énergie,s
pour com prendre, entre autres, com m ent était TU nivers ju ste après sa naissance.
On verra bientôt que to u t l'espace contenu dans l’I h iv ers était iniiialcnxmt minuscule,
et que la tem p ératu re des p articu les co n ten u es d an s c e t esp ac e était in cro y ab lem en t
élevée. A vec le tem ps, cependant. l’U nivers s ’est dilaté et a refroidi à des tem pératures
plus basses, jusqu’à atteindre la taille et la tenifiératurc q u ’on voit aujourd’hui
En fait, l’expression « o n voit aujourd’h u i» est com plexe; quand on observe l’e,space,
on regiude dans le passé, car la lumière provenant des étoiles eJ des galaxies a mis beaucoup
de tem ps à atteindre la Terre. Les objets les plus éloignés q u ’on peut détc'Cter sont les
q u a s a rs (de l’anglaLs q u a s i s t e l 'a r o b je c ts : objets quasi .stellaires), qui sont des a c u r s de
gala.xies extrêm em ent brillants situés aussi loin que 14 x l()‘^ al de la Terre. C hacun de
ces cœ urs contien t un giganie.sque trou n o ir; q u an d la m atière (des g az e t m êm e des
étoiles) est attirée vers un de ces trous noirs, elle s’échauffe et ém et une im pressionnante
quantité de lumière, suffisante pour q u ’on puisse la détecter malgré l’incroyable distance.
On « v o it» le q u asar tel q u 'il était ja d is , à l’in stan t où c e tte lu m ière co m m en ça son
voyage vers la Terre, il y a des m illiards d’années.
14.1t L ’Univers est en expansion 403

1A.11 L’Univers est en expansion


D ans la section 8.10, on a vu q u 'il est possible de m esurer les vitesses relatives auxquelles
les galaxies s ’approchent ou s ’éloignent de la Terre en m esurant le déplacem ent D oppler
de la lo n g u eu r d 'o n d e d e la lu m ière q u 'e lle s cm eiten i. Si on o b se rv e se u lem en t les
galaxies éloignées, au-delà des galaxies voisines de la Гсггс, on constate un luit étonnant ;
elles s'éloignen t to u te s de nous.
En 1929, Edwin P. Hubblc a établi une relation entre la vitesse de récession appai'cntc i
d ’une galaxie et sa distance r d c la T c n c . C ette relation indique que la vitesse de réces.siim
e t la distance sont directem ent proportionnelles ;

V= Hr (la loi de Hul4ile). (14 19)

où H , une constante d e p ro p o rtionnalité, est appelée eunstunt«* de H ubble. ! a valeur


de H est h ab itu ellem en t ex p rim ée en k ilo m ètres p a r se co n d e p ar in ég ap arscc. où le
m é g ap arscc est u ne u n ité d e lo n g u e u r co u ra m m en t u tilisé e en astro p h y siq u e cl en
astronom ie :

1 M pe = 3,084 X lO'^' km = 3 ,2 « ) x 10® a I. (14.201


La co n sta n te de H iibble H n ’a p as c o n s e rv é la m êm e v aleu r d ep u is la n aissan c e de
l’U nivers. Il e.si très difficile de déterm iner sa valeur actuelle parce q u ’il faut faire des
m esures en fonction d e g alax ies trè s élo ig n ées. U ne étu d e récen te a étab li la valcui
actuelle de H à (70 ± 7) km /(s • M pe). alors q u ’une au tte propose fortcm cnl une valcui
de -S8 km /(s M pe). D ans le présent chapitre, on utilisera une valeur de com prom is ;

H = 63,0 km /(s • M pe) — 19,3 inin/(s - a 1) (14.21)

L a récession des galaxies s ’explique par l'ex p a n sio n de l U nivers ; clics agisscnl
com m e les raisins secs qui s'é lo ig n e n t les uns des autres q uand la pâte à pain qui 1rs
contient se dilate en cuisant. Des observateurs se üniivant dans toutes les auiies galaxies
coaslatcraient égalem ent une récession des galaxies éloignées, com m e le prévoit la loi
de Hubble. Pour poursuivre dans l’analogie développée, on jieut dire q u ’aucun raisin SCC
(galaxie) n ’a une vue unique ou idéale.
La loi de H ubble est cohérente avec l'h y p o ih èse qui veut que l'U n iv ers soit né avcc
le Big B ang el q u ’il se dilate depuis. Si l’on suppose q u 'il se dilate à un laiix constant
(c’est-à4lire que la valeur de W a été con.stantc). on peut estlm ei son âge rLtniYm ^ l'aid e
de l’équation 14.19. On siippo.se égalem ent que. depuis le B ig Bang. n*im{ioiie t|tielle
partie de l'U n iv ers (une galaxie, par exem ple) s’éloigne de la Terre à une vlicsv .-1 dc'ier-
ininéc par l'équation 14.19. A insi, le tem ps nécessaire p o u r que cette partie s’éloigne à
une distance r est

r i
(l’âge estime de l’Univers). (14.22)
Wr H

Si on utilise la valeur de com prom is donnée à H dans Péqualion 14 21. se révèle


être 15 X 10’’ a. D es éludes plus poussées sur l'expansion de 1 I inivers attrihiicnl .à
des valeurs entre 12 x 10® a et 15 x 10® a

Exemple 14.6
déplacement Doppler de la longueur d’omlc lumineuse d'un quasar 2, R X 10** m/4
domtc indique que ce quasar a une vitesse «le récession fie 2,K x lié m/s ( 1 000 mm/ni)
'' H 19..3 mrn/(s ■a 1)
(soi! 93 <y<- de la vites.se de la lumière). À quelle distam.e apja'oximativc
= I4.S X in'" al fréimiisel
ce quasar sc Irouvc-t il de la Terre ?
Ce n'csl qu'une approximation, parce que !c quasar ne s'csi pus
SOlUnOH; Le (tmeept dé utilisé ici c.st le .suivant : il faut appliquer la loi KHijoiirs éloigné de la lerre a la meme vitesse i . e'esi-it-dire que H
de Hubblc à la vitesse r doiinéA;. Selon les équations 14.19 et 14.21, n’a pas toujours eu sa valeur actuelle durant toute l’cxpfinsis'n de
on a n Hivers.
4(И Chapitre Les quarKs. les leptons et le eig Bang

Exemple 14.7
Une raie d’émission pariieulicrc, détectée dans la lumière d’une ce qui mené à
galaxie, a une longueur d’onde = 1.1X, où Àest la longueur d’onde cA k
propre de la raie. A quelle distance cette gala.xie se irouve-i-elle de la (14.24)
lik '
Terre ?
Dans cette équation,
SOLUTION Ici. un premier (ontepi tié est qu’il faut supposer que la loi de
HubMc (v = H r) s’applique à la récession de la galaxie. Un deuxième A k = k, - k - I.IÂ - Л =0.1À.
concept dé est qu’il faut .supposer que le déplacement Doppler astrono­ Si on insère ce résultat dans l’équation 14.24. on obtient
mique de l’équatinii 8..^5 (e = c AÀ/À) s’applique au déplacement
de la longueur d’onde provoqué par hi récession. On peut alors établir г(0.1Л) O .lc
r =
que les membres de droite de te s deux équations sont mutuel Icment Hk H
égaux CI ainsi obtenir _ (0.1K3.0 X 10^ m/s)
(1 0 ( Ю т т /т )
cA k
19.3 m m /(s- al)
Hr - (14 2.T) (réponse)
~1T = 1,6 X lO’ al.

14.12 Le rayonnement cosmique fossile


E n 1965. A rn o P e n z ia s c i R o b ert W ilso n , q u i tra v aillaien t d an s ce qui était alo rs les Bell
T elcp h o ite L a b o ra to rie s, te sta ie n t u n ré c e p te u r à m icro o n d e s se n sib le u tilisé d a n s les
re c h e rc h e s en té lé c o m m u n ic a tio n s. Ils d é c o u v rire n t un fa ib le b ru it d e fo n d q u i re sta it
d 'im cn .sité c o n sta n te p eu im p o rte o ù ils p o in taien t l’an te n n e . Il d e v in t b ie n tô t c la ir q u e
P en zias e t W il.son o b se rv a ie n t u n ra y o n n e m e n t c o s m iq u e fo ssile, g é n é ré au d éb u t de
l’ex isten ce d e l’U n iv ers cl rem p lissan t l’esp ace pre.sqiie u n ifo rm ém en t C e гауоппепкШ ,
d o n t l’in te n sité m a x im a le a ttein t u n e lo n g u e u r d 'o n d e d e 1,1 m m , a u n e d istrib u tio n d e
lo n g u e u rs d ’o n d e id e n tiq u e à c e lle d ’un ra y o n n e m e n t d ’u n c o rp s n o ir m ain te n u à une
te m p é ra tu re d e 2 .7 K . D a n s c e cas. le co rp s n o ir e st l’L’n iv ers entier. P en zias cl W ilso n
o n t reçu le prix N o b el d e p h y siq u e d e 1978 prtur leu r d é co u v erte.
C e ray o n n em en t a été p ro d u it en v iro n 379 0 0 0 an s ap iè s le B ig B ang, q u an d l’Lînivers
e.si so udainem ent devenu transparent p o u r les on d es élcctrtim agnétiques (les onde.s n ’etaient
alo rs p lu s im m é d ia te m e n t ab so rb é e s p a r les p articu les). À c e tte é p o q u e , le ra y o n n e m e n t
c o rre sp o n d a it à un ra y o n n e m e n t d 'u n c o rp s n o ir d o n t la te m p é ra tu re a tteig n ait p e u t-ê tre
1 0 ' K. A v ec l'e x p a n sio n d e l'U nivcT s. la tem p ératu re a ch u té à sa v aleu r actu elle d e 2 ,7 K.

14.13 La matière sombre


À l’o b se rv a to ire n atio n al d e K itt P eak , en A rizo n a, V era R u b in e t son c o é q u ip ie r K ent
F o rd o n t m e su ré le s v ite sse s d e ro ta tio n d ’u n c e rta in n o m b re d e g a la x ie s é lo ig n é e s .
P o u r c e fa ire , ils o n t m e su ré les d é p la c e m e n ts D o p p le r d ’a m a s b rilla n ts d 'é to ile s se
tro u v a n t à d iffé re n te s d istan ces d u c e n tre d e n o tre G alax ie. C o m m e on p eu t le v o ir d an s
DiswiKe du ren tre
la fig u re 14.6. les ré su lta ts o b te n u s so n t su rp re n a n ts : la v ite sse o rb ita le d es é to ile s au
galacliqiie ( I0'*a I) b o rd e x te rn e v isib le d e la G a la x ie e st e n v iro n la m em e q u e celle d e s é to ile s sc Iro u v an t
p rès d u c e n tre g alactiq u e.
figure 14 6 La vitesse de rniation
C o m m e l’a tte s te la c o u rb e e n tra it p le in d e la fig u re 14.6, c e n ’e s t p a s c e q u 'o n
des étoiles dans une galaxie type
en fonction de la disiatice qui .sépare
.s'attendrait à tro u v er si toute lu m asse d e la G alax ie était rep résen tée p a r la lu m ière visible.
chacune d’elles du centre galactique.
C e n 'e s t p a s n o n p lu s c e q u 'o n tro u v e d a n s le sy stè m e so laire. P a r e x e m p le , la v ite sse
courbe théorique en trait plein
o rb ita le d e P lu to n (p la n è te la p lu s é lo ig n é e d u S o le il) n 'e s t q u e le d ix iè m e d e c e lle de
démontre que. si une galaxie
M ercu re (p lan ète la p lu s p rè s d u S o leil).
ne contenait que la masse (|iii est La se u le e x p lic a tio n q u a n t au x d é c o u v e rte s d e R u b in e t d e F o rd q u i p e u t ê tre en
vi.sihle, les vitesses observées c o n fo rm ité av ec la m éca n iq u e n ew to n ie n n e e st q u 'u n e g a la x ie tj'p e c o n tie n t b e a u c o u p
diminueraient avec la liLstaiKe p lu s d e m a tiè re q u e c e lle q u ’o n p e u t ré e lle m e n t v oir. E n fa it, la p tm io n v isib le d ’u n e
dans le cas des étoiles éloignées. g a la x ie n e re p ré se n te e n v iro n q u e 5 % à 1 0 % d e sa ma.s.sc to tale. E n p lu s d e ces étu d e s
I.CS points représentent les données su r la ro ta tio n d e s g alax ies, d e n o m b reu ses a u tre s o b se rv a tio n s m è n e n t à la co n c lu sio n
expérimentales, qui démnnli cnf q u e l'U n iv e rs a b o n d e en m atière q u ’o n n e p e u t voir. '
que la vitesse de rotation est Q u e lle e st d o n c c e tte m a tiè r e s o m b r e q u i se tro u v e d a n s u n e g a la x ie ty p e e t q u i
approximativement constante l'e n to u re c o m m e u n h a lo d o n t le d ia m è tre re p résen te p eu t-ê tre 3 0 fo is celu i d e la g alax ie
à de grandes distances visible ? Lc.s « ch o ses >' qui peu v en t être de la m atière .stjmbrc se ran g en t d ans d eu x classes.
14 Le 6ig B ang 405

c u ric-u sem en t a p p e lé e s la c la s s e d e s w im ps (d e l’a n g la is W IM P , w ta k ly in te ra ctin g


m assive p u riid e s: p a rtic u le s m a ssiv e s so u m ise s à l’iiiie ra c tio n fa ib le ) e t la c la sse des
m aclw s (d e l’a n g la is tnassive com pact halo ohjecls: o b je ts m a ssifs c o m p a c ts d u h alo ).
Si les n eu trin o s o n t une m asse, ils so n t des can d id ats wimps po.ssibles. L es m m iuis i)cuvcnt
c o m p re n d re d es o b je ts c o m m e les tro u s n o irs, les naine.s b la n c h e s e t les n ain e s b ru n e s,
q u i so n t d e la taille d e Ju p ite r e t d o n c tro p |x :tilcs p o u r p io d u iic d e la iu sio n u u v lc a u e .
L e s n ain e s b ru n e s ne b rille n t d o n c p a s, c e q u i fait q u ’elles n e \im i pa.s v isib les A vant
2 0 0 0 , il y a e u d e s p re u v e s c o n v a in c a n te s q u e les m achos e x is te n t d a n s n o tre O a la x ic .
Im a g in e z q u ’u n m a ch o (in v is ib le ) sc tro u v a n t d a n s n o tre G a la x ie p a sse , p a r h a sa rd ,
e n tre la T e rre e t u n e é to ile d ’u n e g a la x ie v o is in e . D a n s sa th é o rie d e la re la tiv ité
g é n é ra le , E in ste in a p ré d it q u e d e s ra y o n s lu m in e u x p a ssa n t p rè s d ’un o b je t in a s s if
seront d év iés p ar la m asse d e cc d ern ier (v o ir la section 14.9 du volum e 11. D onc, si ré to ile ,
le m acho c l la T erre so n t a lig n é s, le m acho a g ira c o m m e u n e lentille grax'itationnelle,
c o n c e n tra n t le s ra y o n s lu m in e u x d e l ’é to ile q u i p asse n t p rè s d e lui et p ro rliiisan t u n e
im ag e p lu s b rilla n te d e l’é to ile alo rs q u e le m acho lu i-m ê m e l’cclip.sc.
Il y a e u s u ffis a m m e n t d ’o b s e rv a tio n s d e te ls é v é n e m e n ts p o u r e o n v a in c rc le s
astro n o m e s q u e les m achos p e u v e n t c o m p o se r une fractio n su b sta n tie lle (c e rta in s d isen t
5 0 % ) d e la m atière so m b re p résen te d an s n o tre G alax ie. L es oh.scrvalions .sc ¡xiursuivcnL

14.14 Le BIg Bang


E n 1985, un p h y sic ie n fit la rem tu'que su iv an te lo rs d ’u n e réu n io n s c ie n iin q u e :

Il est aus.si sûr que l’Univer.s est né lors d ’un Big Bang il y a envuen I .“i milliards d’années
qu'il est sûr que la Terre tourne autour du Soleil.

C e tte a ffirm a tio n m o n tre le n iv eau d e c o n fia n c e d e s scien tifiq u es d an s la th éo rie d u B ig


B a n g , fo rm u lé e p o u r la p re m iè re fois p a r le p h y sic ie n b elg e G e o rg e s L e m a ître
Il n e faut p as im a g in e r q u e le B ig B an g a ressem b lé à l’e x p lo sio n d ’u n g ig a n te sq u e
p é ta rd ni q u e , en p rin c ip e d u m o in s, v o u s a u riez p u v o u s te n ir à c ô té p o u r l’o b se rv e r.
Il n ’y a v a it p a s d e « c ô té » p a rc e q u e le B ig B a n g re p ré s e n te le c o m m e n c e m e n t de
l ’e sp a c e -te m p s lu i-m ê m e . D u p o in t d e v u e d e l’U n iv e rs a c tu e l, il n ’y a a u c u n e n d ro it
d a n s l’e sp a c e q u e v o u s p o u v e z p o in te r e t d o n t v o u s p o u v e z d ire : « L e B ig B a n g s ’e st
p ro d u it ici, » Il s 'e s t p ro d u it p arto u t.
D e p lu s, il n ’y a p a s d ’« av an t B ig B a n g » p a rc e q u e le te m p s a com m encé avec cet
é v é n e m e n t c ré a te u r. D a n s cc c o n te x te , le te rm e « a v a n t» p e rd to u te s a s ig n ific a tio n .
O n p eu t to u te fo is é m e ttre d e s c o n je c tu re s s u r c e q u i s ’e,st p a ssé au cours d ’in te rva lle s
d e te m p s su c c e ssifs q u i o n t su iv i le B ig B an g .
t ~ 1 0 s. C ’est la p re m iè re p ério d e d u ra n t laq u e lle on ne p e u t n e n d ire d e sig n i
ficatif su r le d év elo p p em en t d e l’U nivers. C ’est à cei instant q u e les co n cep is tl’espacv
e t d e lem p s a c q u iè re n t leu rs signification.s aciu d le.s. e t q u e les lo is d e la p h y siq u e
telles q u ’o n les c o n n a ît m a in trn a n l tlevie.nncnl ap p lic a b le s. A c c t in stan t. l’U n iv o rs
e n tie r e st p lu s p etit q u ’u n pro to n , e t sa te m p é ra tu re e st d ’e n v iro n 11)’’ K .
t ~ s. J u s q u ’à c e t in s ta n t, l ’LTnivers a su b i u n e d ila ia iio n iiu ro y a h le in e n i
rap id e q u ’o n a p p e lle iiifla tio iL au g m e n ta n t sa ta ille p a r un fa c te u r d ’e n v iro n 10'".
U est d ev en u une so u p e ch au d e de p h o to n s, de quai-k.s e t d e lcpton.s, à une tem p ératu re
d ’e n v iro n 10^’ K.
/ = 1 0 ■’ .s. L e s q u a rk s p e u v e n t n iain ien an i se combiiUT p o u r fo rm e r d e s p ro to n s t i
d e s n e u tro n s, e t le u rs a n iip a rlic iile s. L e s p a rtic u le s de m a tiè re e t d ’a iilim a liè it ,sc
h e u rte n t e t s ’a n n ih ile n t m iitu ellcm em . l 'n lé g e r e x c è s d e m atière, e n l ’ab se n c e d e
p a rte n a ire s d 'a n n ih ila tio n , s u rv it p o u r fo rm e r le m o n d i d e m a lic rc t|u ‘oii c o n n a ît
a u jo u rd 'h u i.
f « t m in . L 'U n iv en s s ’e st su ffisam n K n t refro id i p o u r q u e les p ro to n s et les n eu tro n s
q u i en tre n t e n c o llisio n p u isse n t re ste r u n is p o u r fo rm e r d e s n o y au x d o faib le m a sse
^H. ^H e, *He et ^l,i. L es ab o n d an ces préd ites d e ces n oyaux .vont c elles (iii'o n obsci ve
d an s l’U n iv ers actuel. 11 y a p lein de ray o n n em en ts, m ais la lum ière ne p eu t p aren iirir
d e g ran d es d ista n c e s av a n t d ’in te ra g ir av ec un n o y au . L 'U n iv e rs est d o n c o p aq u e.
406 Chapitre 16 Les quarirs. tes leptons et le Big Bang

t = 379 000 a. L a tem pérature a chuté à environ Ift* K ; les électrons peuvent rester
liés aux noyaux après q u ’ils sont entrés en collision, form anl ainsi des atom es. Étant
donné que la lumière n ’interagit pas de m anière appréciable avec des particules (sans
charge) co m m e le s atom es n eu tres, elle p eu t p arc o u rir d e g ran d es d istan ces. C e
rayonnem ent to rm e le rayonnement cosmique fossile dont nous avons parlé dans la
section 14.12. L es atom es d ’hydrogène et d ’hélium , sous rin flu c n c e de la gravité,
com m encent à s ’unir, am orçant ainsi la form ation des galaxies et des étoiles.

Les prem ières m esures suggéraient que le rayonnem ent cosm ique fossile était hom ogène
dans toutes les directions, cc qui im pliquait que, 379 (KX) a après le Big Bang. toute la
m atière de l'U niv ers était répartie uniform ém ent. C ette découverte était étonnante parce
que la m atière de l’U nivers actuel n ’e st pas uniform ém ent répartie, m ais rassem blée en
galaxies, en am as de galaxies et en superam as de galaxies. 11 y a aussi de vastes vides
contenant relativem ent peu d e m atière, et il y a des régions tellem ent pleines de m atière
q u ’on les appelle murs. Si la théorie du Big B ang est m êm e approxim ativem ent juste,
les germ es de cette répartition non hom ogène de la m atière doivent avoir été présents
avant les 379 000 a d e rU n iv ers. et ils se révèlent m aintenant .sous la form e d 'u n e répar
tition non hom ogène des rayonnem ents cosm iques fossiles.
En 1992, des me.sures prises par le satellite С О В Е de la NA,SA ont révélé qu e le
rayonnem ent cosm ique fossile n ’est pas parfaitem ent hom ogène. Ln 2003, de nouveaux
résultats ont été ob ten u s par le satellite W M A P de la N A SA . L im age de la prem ière
page de cc chapitre a été réalisée à l’aide de ces m esures, après une année d ’observation,
et illustre l’aspect de l'im iv e rs lorsqu’il n 'éta it âgé que de 379 OtX) a. Les zones rouges
m ontrent des régions plus chaudes alors que les zones bleues m ontrent des régions plus
fm ides. Vous pf>uvez alors voir que des rassem blem ents de m atière avaient déjà co m ­
m encé. Donc, la théorie du Big Bang s ’avère, en principe, fondée. D e plus, les résultats
du W M AP indiquent que l’âge de TUnivers est de ( 13.7 ± 0,2) m ilhards d ’années.

14.15 Une récapitulation


Lnaginez, dans ces derniers paragraphes, où cette iiccum ulation rapide de connaissances
sur l’U nivers mène. Elle procure, bien sûr, une satisfaction incontestable aux physiciens
et aux astro n o m es cu rieu x . C e p en d a n t, c e rta in s la v o ien t co m m e une ex p é rien c e
d ’hum ilité : chacun des progiès de la connaissance sem ble révéler plus à fond la relative
insignifiance de l’hum ain dans le grand schèm e de la m atière. En ordre grossièrem ent
chronologique, on en est venu à constater ce qui suit.

La Terre n ’e.sl pas le centre du systèm e solaire.

Le .Soleil n ’est tin’une étoile piu^mi tant d ’autres dans notre Galaxie.

N otre G alax ie en est une parm i b eau co u p d ’autres, et n o tre S oleil est une éto ile
insignifiante qui en fait partie.

l ’âge de la Terre représente peut-être seulem ent le tiers de celui de l'U n iv ers, et on
p eu t a ffirm e r av ec c e rtitu d e qu e n o tre p la n ète di.sparaîtra q u an d le S o leil aura
éjiiiisé son com bustible et deviendra une géante rouge.

Ntnre espèce habite la T erre depuis m oins d ’un m illion d ’années, un clin d ’œil dans
le tem ps cosm ique.

Bien q u e notre position d an s PUm ivers sem b le in sig n ifian te, les lois de la p hysique
q u ’on a découvertes (et celles qui sont encore â découvrir) sem blent s ’appliquer dans
tout l’Univers, et (pour autant q u ’on le sache) elles existaient au début et existeront dans
l ’avenir. Il n ’y a au cu n e p reu v e que d ’au tres lo is s ’ap p liq u en t d an s d ’a u tres p arties
de l’U nivers. Donc, ju.squ’à ce que quelqu’un se plaighc. on est habilité à valider les lois
de la physique «découvertes sur T erre». 11 en reste lieaucoup à découvrir; « L ’IInivers
est rem pli de choses m agiques, qui attendent patiem m ent qu e notre esp rit s’affine. »
O u estio n s 407

ВЗВВШШЕ I« i • ,V», i 4 ■* *,

Les leptons et les quarks Les recherches actuelles confirmem une de 0. Ces quatre signes algébriques sont inverses chez les anii-
l’idée tjuc toute matière est faite de six types de leptons (tableau 14.2), quarks. Les quarks ont aussi une aiuleur (rouge, vert ou bleu).
lie six types de quarks (tableau 14.5) et de douze types d'antiparticules, l e s hadrons : les buryons et les m ésons I .es qiiaiks .se com­
une pour chacun des leptons et des quarks. Chaque quark se divise en binent piiur former des particules qui interagissent tortement et qu’on
trois types (rouge, ven et bleu). Toutes ces particules ont des nombres apjxtlle hadrons. Ix s iKtryoits чип des iiadroiis qui uni des mmibies
quantiques de spin de j et sont, de celait, des fermions (des particules quanliqucs de spin demi entiers ( J ou J). Les m ésons sont des
dont le nombre quantiqiie de spin est dcini-entier). hadrons qui oui des nombres qumiliqucs de spin entiers (0 ou 1) et
L iS interactions Les particules chargées électriqucincni iiitcr- qui sont, par consequent, des bosons, l es haryons sont des fermions
agis.senf .sous I’aciion de l’interaction électromagnétique en échangeant Les mé.sons oui un nombre baiyoïiique égal h zéro; Icsbaryons ont
des photons virtuels. Les leptons peuvent interagi rentre eux et avec un nombre baryonique égal à I I ou a —1. La chromodyiumùquc
les quarks grâce à ce que l ’on nomme l’interaction faible, en q iian tiq u e permet de prédire les combinaisons possibles seules
échangeant des particules massives W^. W et Z'^ . De plus, les les combinaison.s de couleur neutre sont pos.siblcs (par exemple
quarks interagissent entre eux grâce à l'intcm ction forte en rouge t vert • bleu ou rouge + antirouge) (cette prédiction est
échangeant des gluons. 1,’imeraciion électromagnétique et l’interaction cohérente au regard ik s expériences menées).
faible .sont differentes manifestations de la même interaction, appelée L 'exp a n sio n de l ’V nivers Les indices actuels suggèrent
interaction l'Ieelri^aiblr. fortement que rU nivers est en expanxmri. Ic.s loimaiiics galaxies
t^ S leptons l’rois types de lepions d ’électron, le imion et le .s'éloignent de la Terre à une vitesse v donnée par la loi de H u b b le .
tauon) ont une charge électrique égale à —c. Il existe aussi trois V Иг (b loi de Hubble). 114.19)
n eu trinos (qui sont également des leptons) non chargés, un pour
Dans ce volume, on considère que H, la constante de H ubble,
chacun des leptons chaigés. Les neutrinos ont de très petites masses,
possède la valeur
qui sont peut-être même nullcs. Ixs amipaiiicules associées aux leptons
chargés ont une charge électrique piKiiive. t l = 63,0 km/s/Mpe ^ 19,3 mm/(s al). ( 14.2) 1
Les quarks Chaam des six quarks (u, d, s, c, b et t) a un nombre L’cxpausion décrite par la loi de Hubblc et l'onmiprésence d’un rayon-
bai VIinique de cl une charge égale à -r- ou —Je. Le quark s liemeni cosmique fossile suggèrent que l’Univers est né lors du Hig
possède une étrangeté de —1. alors que tous les autres en atfichent Bang, voilà 12 à 15 milliards d ’années.

O U E S TIO N S

1. Il n’y a piis que les particules comme les électrons et les pilotons suivants serait viole s’il le faisait : a) nombre Icptomquc électronique,
qui peuvent être considérées comme des fermions ou des bosons. b) moment cinétique, c) charge, d) nombre leptonique muoniquc,
Des atonies entiers peuvent au.ssi l’élre, selon que leurs nombres c) quantité de mouvement, f) nombre baryonique?
quandques de spin résultant sont, respeclivenient. des demi entiers 7. On a vu que la structure de quarks du méson n est dii. Quels
ou des entiers. Laquelle des affirmations suivantes est juste par rapport principes de conservation des éléments suivants seraient violés si
aux isotopes d’hélium ’Hc et ’’H c? a) Tous deux sont des fermions, un jr~ était plutôt terme d ’un quark d et d ’un qimrk u ■a) énergie,
b) Tous deux sont des bosons, c) "'Hc est un ferimon ; *He est un b) moment cinétique, c) charge, d) nombre lepioiiiqur e) qiiamité
Ixison. d) *He est un feniiion ; ’He est un boson. (1 es deux électrons de mouvement, f) nonibie bai yoiiiqw ?
de l’hélium forment une couche fermée et ne jouent aucun rôle dans 8. Une particule 1 ' a les nombres qiianiiques suivants : étrangeté
celte ilétcrniinaiion.) 5= I, charge q — +1 cl spin .v — I i - Laquelle des lombinuisons
2. Le champ magnétique de la figure 14.3 b) st>n il de la page ou y de quarks suivantes est correcte, a) dds, b) s.s, c) uus, d) ssu ou
entre-t-il’.' e) uus?
3. bequel des huit pions de la figure 14.3 b) possède le moins 9. Ci-des.sous, la coloniK de gauche donne des concepts appartenant à
d’énergie cinétique? la physique atomique ; i elle de (Imite, des concepts de la physique des
4. L’n électron ne peut se désintégrer en deux neutrinos Quels particules. Associez les concept.s dc.s deux colonnes.
principe.s de con.servaiion des éléments suivants seraient violés s'il
I . chimie a) voie tx-iuplc
le pouvait : a) énergie, b) moment cinétique, c) charge, d) nombre
leptoni(|ue. e) quantité de mouvement f) nombre haryoïiiquc? ? élci tmns b) hadrons nianqutmts
5. Un proton ne peut se désintégrer en un neutron et en un neutrino. 3. tableau ivenodique c) i hmmodynamiqiic quanlitiiiL
(,)ucls pniKipes de coiuscrvation des éléments suivants seraient violés 4. éléments manquants d) physique des parrii nies
s'il le pouvait - a) énergie (en supposant que le proton soit au repos), 5. mécanique qiiantuiiie e) quarks
b) moment cinétique, e) charge, d) nombre leptonique. e) quantité
de ntoiivcmcnt. fl nombre baryonique ? 10. Soit le neutrino dont le symbole est r , . a) .S'agit tl d ’un qiiiirk,
6. Un proton possède suffisamment d’énergie au rc|H)s pour se d’un Icpion. d'un méson ou d’un baryon? b) S'agit il d’une particule
désintégrer en ¡et com|-K>sé d ’électrons et de neutrinos, et de leurs ou d’une antiparticule? c) .S’agit il d ’un boson ou d ’un ferm ion''
antiparticules. Quel principe de conservation d ’un des éléments d) Eist-il stable?
Chapitre t4 Les quar4<s. les leptons et le eig Bang

11. Asscx;ie7 les éléments de ces deux colonnes : 12. Classez les particules suivantes selon leur masse, en conmien^ant
par la plus légère; a) proton, b) neutrino, c) méson n , d) quark
1. tauon a) quark
étrange, c) tauon, f) électron et g) 2 .
2. pion b) Ic-pton
13. Quels sont les nombres leptoniqucs de ces particules : a) n~,
3. proton cl iiiésoii b) c ,c )iX ', d) T-, e) ?
4. posilron d) baryon
5. chiU'iné e) anliparticule

«*i^ >f • r '


EX E R C IC E S E T P R O B LÈM ES

IIP. a) Une particule au repos, 1, se désintègre en deux particules,


www La solution se trouve sur le site Web, à l’adres,se ci-dessous :
2 et 3. qui s’éloignent avec des quantités de mouvement de même
www.dlcincgrawbill.ca/physique module, mais dans des directions opposées. Démontrez que l’énergie
cinétique de la particule 2 est donnée par
SECTI0N14.3 lin interîixie
1E. Calculez, la différence de masse, en kilogrammes, entre le muon BjŸ E}},
el le pion de l'exemple 14. l
2t. Une di.staiice r sépare un électron et un positron. Déterminez où fi,. E2 et sont les énergies au repos correspondantes. {Indice :
le rapport entre la force gravitationnelle et la force électrique qui Suivez. le laisonncmenl de l’exemple 14.1, sauf que, dans ce cas,
agissent entre ce,s particules. D’après votre résultat, que pouvez-vous aucune des particules créées n’a une masse nulle.) b) Démontrez que
conclure sur les forces agis.sanl entre les particules détectées dans une le résultat en a) donne l'énergie cinétique du muon telle qu’elle est
chambre à bulles? calculée dans l’exemple 14.1.
3t. Un pion neuti'c SCdésintègre en deux rayons gamma • tr®—» -y 4- y.
Calculez la longueur d’onde des rayons gamma produits par la StOION14.5 Leshdrom
désintégration d’un pion neutre au reptxs.
]2£. Vérifiez si le schéma de désintégration hypothétique du proton
4E. Un pion pessitif subit la désintégration décrite dans l’équation 14.7 ;
de l’équation 14.14 ne viole pas les principes de conservation
TT* —» IJ* + U. Quel doit donc être le scliéma de désintégration du
a) de la charge, b) de l’énergie, c) de la quantité de mouvement el
pion négatif? {Indice: Le rr ‘ est l’antiparticule du Tr"^.)
d) du moment cinétique.
5t. Quelle énergie serait dégagée si la Terre était annihilée pat une
13E. Quel principe de conservation est violé dans chacune des désin­
collision avec une anti-Terre?
tégrations suivantes (supposez que la particule initiale est au repos
5P. Certaines théories d ’unification des interactions prédisent que et que les produits de désintégration ont un moment cinétique orbital
le proton est instable, sa demi-vie clanl d'environ 10’* années. nul) :a.) P -> e b) p. z e + i/^ + c) -> ,t ’ + ?
En supposant que ce soit vrai, calculez le nombre de dé.siiitégrations
14P La particule Ai et ses produits se désintègrent de la manière
de protons que vous vous attendriez à voir en une année dans l’eau suivante :
d’une piscine olympique d’une capacité tle 4.32 x 10’’ L.
7P. l,es observations de ncutrinos émis par la siipemova SN 1987a A 2 ■ ' /)” + TT* , P* s ' + V ^ V,
(figure 1.3.12), située dans le Cnand Nuage de Magellan, établissent p"- - rr* + TT“ , JT~ —> p ~ + V,
une limite supérieure de 20 eV à l’éncrgic au repos du neutrino n* ip * V. p~ V ■{ V.
électronique. Supposez que l’cnergie au repos de ce neutrino, au lieu
d’être nulle, est égale à 20 cV. De combien inférieure à la vitesse de a) Quels sont les produits de désintégration finaux stables ? b) Selon
la lumière serait la vitesse d'un neutrino de 1.5 MeV émis lors d’une les produits, la particule est-elle un fermion ou un lioson ? Est­
désintégration bêta ? elle un méson ou un baryon? Quel est son nombre baryonique?
{Indice : Voir l'exemple 14.4.)
8P. Un pion neutre a une énergie au repos de 135 MeV et une vie
moyenne de 8,3 x 10 ” s. S’il est produit avec une énergie cinétique
initiale de 80 MeV et .se »lésintègie après une durée de vie moyenne, SECTION 14.6 Un uüire printipe de tonsetvalion
quelle est la plus longue trace que cette particule pourrait laisser dans
15t. La réaction zt’ - i p ^ p - i - p + n s e produit par interaction
une chambre à bulles ? Utilisez la dilatation du temps.
forte. En appliquant les principes de conservation, déduisez le
9P. On ne peut mesurer directement les énergies au repos de ntimbre quantique de charge, le nombre baryonique et l'étrangeté de
nombreuses particule.s ayant de courtes vies ; on doit en trouver les l’antineutron.
valeurs à l'aide des quantités de mouvement mesurées et des énergies
16E. 1-n étudiant l'étrangeté, déterminez., parmi les désiiitégration.s
au repris connues des produits de désintégration. Prenez le méson
ou les réactions suivantes, celles qui sc produisent par interaction
qui se désintègre par la réaction —> ?r ' - t - . Calculez l’énergie
forte : a) K" —» tt* + n~, b) f p —> 2 '’' H n, c) /V + p —» f n~,
au repos du méson sachant que la qti.'uiiitc de muuvemcnl de
d) K H p -» 3®
chaque pion créé a un mixiule de 358.3 MeV/t. Voyez le tableau 14.4
pour connaître les énergies au repos des pions. 17E. Quel principe de conservation est violé dans chacune des
réactions ou des désintégrations suivantes (suppasez que les produits
lOP. L’n tauon positif (T ^ énergie au repos — 1 777 MeV) se déplace
ont des moments cinétiques orbitaux nuis); a) A® p -*■ K .
avix; une énergie cinétique de 2 20(1 MeV en suivant une Irajecloiie
b) l i >2 H TT®(.Ç = - 3 .9 + I. et wi = I 672 MeV7<-^ pour fl*),
circulaire ixipcndiculaire à un champ magnétique homogène de 1.20 3.
c) K + P -> A® H TT'’^?
a) Calculez la quantité de mouvemeni du tauon en kilogrammes-
mètres par seconde.. Il faut tenir compte dc.s effets relativistes. 18t. Calculez l’énergie de désintégration des réactions suivantes;
b) Déterminez le rayon de lu traii:ctoirc circulaire. a) TI ■ + P -» 2 '’' H K’' et h) K t- p -> A® f jt®
Fxercices et problèmes 409

19E. Une particule 2 “ sc déplaçant avec une énergie cinétique de 31E. Quelle est la longueur d’onde observée d'une raie d’hydrogène
220 MeV se désintègre de la manière suivante: 2 —> -r n. de 656,3 nm (première raie de Balmcr) émise par une galaxie .située
Calculer l’énergie cinétique totale des produits de désintégration. à une distance de 2,40 x K)** a I ? Supposez que le déplacement
Doppler de l’équation 8.35 et que la loi de Hubble s’appliquent, www
SEGION14 7 La voie octupic 32L En laboratoire, une des raie.' du sodium c.st émise à une longueur
d’onde de 59{),() nm. Cependant, dans la lumière d'une galaxie
20P, Dcmonlre/, l'énoncé suivant : si, au lieu de représenter graphi­
donnée, cette raie est ob.servée à une longueur d'onde tie 6()2,l) nm.
quement réirangelé S en fonction de la cliargc q des baryons de spin j
Calculez b distance de la galaxie, en supposant que la loi de Hubble
de la ligure 14.4 a) et des mésons de spin nul de la figure 14.4 b),
et que le déplacement Doppler de l'cquaiion 8.35 s'appliquent.
on représentait la quantité Y = B ‘ S en fonction de la quantité
T i = q — jtf l S ) , le schéma hexagonal apparaîtrait avec des axes
33P. L’Univers coniinuera-t-il de sc dilater indéfiniment 7 Pour répondre
qui ne seraient pas en pente (perpendiculaires), (La grandeur Y est à cette question, faites la supixisition (vraisemblable 7) que la vitesse
apiielée hyiM-rchurae et est relie à une grandeur appelée isospin.) de récession v d'une galaxie se trouvant à une distance r tie la Terre
est .seulement déterminée pat Ut malièrc sc trouvant dans une sphère
21P. Faites appel aux principes de conservation pour identifier la
de rayon rdont nous sommes le centre .Si la masse totale a rm téi ieiir
particule appelée x dans chacune des réactions suivantes, qui
de cette sphere est M, la vitesse de libération V|,|, de cette sphère est
SC produisent par interaction forte • a) p -t- p -> p t A*’ t x,
l'iii, — ^2GM/r (équation 14.27 du volume I). a) Dcnioutrcz que.
b) P 1 p —* n -ï, c) 71 -r p —» E'* + K“ + X . wvrw
pour prévenir une expansion illimitée, la masse volumitjiie moyenne
??P. Soit la désintégration -» p + n- , où A” est au repos, p à r intérieur de la splièrc doit au moiii.s être égale à
a) t'îilculez l’énergie de désintégration, b) Déterminez l ’énergie
cinétique du proton, c) Quelle est l’énergie cinétique du pion? 3 //^
(Indice : Voyez le problème 11.) (8.-T<;)'

SECTION 14.8 le modèle des quarks b) Évaluez numériquement cette «mas.se volumique critique»;
exprimez votre réponse en atomes d’hydrogène par mètre cube.
231. Les combitiaisons de quarks qui forment le proton et le neutron Les mesures de la masse volumique réelle sont difficiles à pi entire,
sont respectivement uud et udd. Quelles sont les combinaisons notamment à cau.se de la piéscncc de matière sombre, mais les mc.surcs
de quarks a) de l'antiproton et b) de l'antineutnm? récentes du satellite WMAP indiquent que la masse volumique e.si
241. À l’aide des tableaux 14.3 et 14.5. déterminez l’identité des égale à cette masse volumique critique.
baryons formes par les combinaisons suivantes de quarks (vérifiez 34P. Le.s vitesses de réces-sion apparenic.s des galaxies et des quasars
vos réponses avec l’octet de baryon illustré dans la figure. 14.4 a]) ; éloignés sont proches de la vitesse de la lumière ; il faut donc utiliseï
a) ddu, b) uus, c) ssd. la formule relativiste tlu déplacement Doppler (équation 8.32). Le
25t En utilisant soilement les quarks up, down et étrange, construisez, déplacement vers le rouge est rapporte comme étant un déplacement
si possible, un baryon a) dont q ~ +1 et l’étrangeté S = —2. relatif Z = AX/Ào- a) Démontrez que le paramètre de la vitesse de
et b) dont q = -^2 et l'étrangeté 5 = 0. récession (i — vk. exprimé par rapport à z. est déternune par
?6f Quelles combinaisons de quarks faut il pour former a) un A*’ ,2 -I- 2z
et b) un 5®? P -
f 2z i- 2 '
27(. Il y a 10 baryons qui ont un spin de | . Leurs symlxiles et leurs
nombres ciuantiques pour la charge q et l’étrangeté S sont les suivants : b) Un quasar ayant / — 4.43 a été détecté en 1987 Calculez .son
paramètre de vitesse, c) Détcmiinez b distance du quusui, en .supptisiint
9 5 Q 5 que la loi de Hubblc s’applique h cette distance.
A -1 0 S*" 0 -1
SECTION 14.12 Le rayonnemenl cosmique fossile
A» 0 0 X*' -1 -1
A* ^1 0 -1 -2 35p. En raison de l’orrmiprésencc du rayonnement eosmiqtio fosvile, ta
températuie minimale possible il’un gaz dans l'espace inlcrsirlbiiic
A * -f2 0 0 2
ou iiitergalaeliquc n'est pas ü K. mais 2.7 K. Ce qui implique qu’une
a;*- -1 -1 li -1 -3 traction impirtanie des molécules de resjiace ¡xHivani 01x 14101 des
états excités as.sociés ù une énergie d’cxcilation faible peuvent, en
Représentez ces baryons dans un graphique charge étrangeté, fait, être dans ces état excités, i-a désexcitation snliséqtietiie devrait
en utili.sani le système de ctKirdonnées en pente de la figure 14.4. nx-ncr à rémis.sion de rayonnements (Jcteclablc.-). Soit une molécule
Compare/ votre graphique avec cette figure. (hypothétique) ne comportant qu'un état excité, a) Que devrait être
28P 11n’y a aucun méson connu dont le nombre quantique de charge l’énergie d'excitation poui que 25% des molécules soient dans l'étal
q = + I et l’étrangeté S 1, ou dont q ~ - I et .V = f l . Dites excité7 {Indice : Voyez l'equation 11.20.) b) (^icllc semi- In longueur
pour(|uoi, à l'aide du modèle îles quarks. d’onde du phoion émis dans une traiisiiinn vers l'état fondami mal ?
?9P I e baryon 5.*® de spin ^ (voir l'exercice 27) a une énergie au
repus de I 385 MeV’ (dont l’incertitude intrinsèfjue e.st ignorée ici); SECTION 14.13 Lo matière sombre
le baryon 2® de spin 2 a une énergie au repos de 1 192.5 MeV.
36t. Quelle desTait être la masse du .Snlei I pour que Elutnn (la planete
Si chacune de ces particules a une énergie cinétique de 1 (MK) MeV,
la plus éloignée du Soleil, la plupart du temps) ait la vitesse orbitale
laquelle, le c.a.s échéant sc déplace le plus rapidement, et quelle e.st la
que Mercure (la planète la plu.s prè.s du .Soleil 1 a aciucllt niuil ®
dittérence de vitesse enue les deux 7
(itilisez les données de l’annexe ( ' et exprim e/ votre répon.se
en nnisse.s actuelles du Soleil (.Supposez, que les orbites sont
SECTION 1411 L'Univers est en expansion circiilaims.)
30t. Si on applique la loi de Hubble à des distam e.s liés grandes, 37p. Supposez que le rayon du Soleil a migmenté à 5 v lOi« m
à quelle distance la vitesse de récession apparente devient elle égale (rayon moyen de l'orbite de Pluton, planète la plus éloignée
à la \ ite.ssc de la lumière 7 du .Soleil), que la niasse volumique de ce .Soleil dilalé est homogène
410 Chapitre 14 Les quarks, les leptons et le Big Bang

et que les planètes orbitcni à l'intérieur de cet objet ténu, a) Calcule/, dans une suite de champs magnétiques et de détecteurs dispo.sés de la
la vite.'tNe orbitale de la Terre dans celte nouvelle configuration et manière illustrée dans la figure 14.7. Le premier champ magnétique
compare/-la avec sa vitesse orbitale actuelle de 29.8 kni/s. Supposez Ml déviait la trajectoire de toute particule chargée qui le traversait;
que le rayon de l'orbite de la Terre demeure inchangé, b) Quelle de plus, ce champ était réglé de façon que seules les particules qui
serait la nouvelle période de révolution de la Terre (La masse étaient chargcc.s négativement (soit un p ou un j r ') et qui avaient une
du Soleil demeure la même.) www quantité de mouvement de 1,19 GeV/c en cniergem pour atteindre le
38P Supposez que la matière (étoiles, gaz, poussières) d'une galaxie deuxième champ magnétique (Q1 ). Q1 était un type particulier de
dtinnée. d'une ma.sse totale M, est distribuée uniformément dans une champ magnétique (un champ qtuulrupoUtire) qui fai.saii converger
splière de rayon R. L ne etoile de masse m tourne autour du centre de les particules qui l'atteignaient de manière à former un faisceau leur
la galaxie sur une orbite de rayon r < A?, a) Démontre/ que la vitesse permettant de traverser un trou pratiqué dans un épais écran et de .se
orbitale Vde l’ctoile est donnée par diriger vers un compteur à xdnrillaiiom S I. Le passage d'une particule
chargée dans un tel compteur provoqiMil un signal (tout comme
V = ry / C M / H ^ , l'écran d'un télévi.seiir normal émet une impulsion lumineuse quand
et que. par conséquent, la période de révolution T csi un électron le heurte). Donc, chaque signal indiquait le passage d'un
TT■ de 1.19 Ge'V/r ou. vraisemblablemeni, d'un p de 1.19 GeV/c.
T = In y/R ^fC M , Après avoir été de nouveau concentrées par le champ magné­
tique Q2. les particules étaient dirigées par le ehainp niagnéliquc M2 à
indéixiidaiilc de r. Ignoicz tome force de résistance h) Quelle est ki
travers un deuxième compteur à sciniillutions S2, puis à iravcrs deux
formule eorrcs|X)ndniitc de la [lériixle orbitale, en supposant que la
(¡électeurs Cerenkov C I et C2. Ce,s derniers peuvent être conçus pour
masse de la galaxie est lorlcincnt concentrée vers son centre, de sorte
produire un signal seulement quand la particule qui les traverse a une
que, essentiellement, toute la masse se trouve à des distances
vitesse se trouvant entre deux limites. Dans rcxpcrience. une par­
inférieures à r du centre '?
ticule qui avait une vitesse supérieure à 0,79c déclenchait C l ; une
particule qui avait une vites.se entre 0,75c et ü,78c déclenchait C2.
S in iO N H .M le Big Gang
Il existait alors deux moyens de faire la distinction entre les
391. I.n longueur des (Huics élecirouiagiiétiqucs les plus intcases émises rares antiprotons prédits et les abondants pions négatifs. Ces deux
¡wr uii coi'ps noir à une température T est donnée par la loi de Wien ; moyens reposaient sur le fait que la v itesse d'un p de 1.19 GeV/c
(voir réquaitun 9.5). a) Démontrez que Tcncrgie E d'un photon devait être différente de celle d ’un de 1,19 GeV/c: 1) selon les
correspondant à cctlc longueur d'onde peut être calculée à l'aide de calculs, un [5 déclencherait l'un des compteurs Cerenkov, et un n
déclencherait l'au tre; 2) de plus, l’intervalle de temps Ai entre
¿' = ('1,28 X 10 '"M eV /K ir. les signaux de SI et S2. qui étaient séparés de 12 m, aurait une valeur
dans le cas du p et une autre valeur dans le cas du tt~. Donc, si le bon
b) A quelle température iniiiiiiiale ce photon peut-il créer une paire compteur Cerenkov était déclenche et si rintervallc de temps Ar avait
électron positron (comme on l'a sni dans la section I C dn volume 2)'? la bonne valeur, rexpérienec prouverait l’existence des antiprotcHi*.
40E Faites appel à la loi <fc Wicn (voir l'cxcrcice 39) pour répondre
aux questions suivantes; a) Le raynimcineni cosmique fossile
présente un pie d'intensité à une longueur d’onde de 1,1 mm. A I Faisceau de protons
quelle temperature cette longueur d'onde conespond-clle ■?b) Hnviron
379 000 a aptf)s le Big Bang. I Lnivcrs devint transparent au rayon­
nement élrciromagnéiique. Sa température était alors de 10^ K.
Quelle était la longueur d'onde à laquelle le rayonnement cosmique '- i iihlc
tos.sile était le. plus intense ?

Problèmes supplémentaires

41. La figure 14.7 montre une partie du montage expérimental qui


permit ta decouverte tk: raiitiproion dans les années 1950. Un faisceau
de protons lie 6.2 OeV émergeait d'un accélérateur de particules et
heurtait des noyaux dans une cible de cuivre. Selon les prédictions
théoriques de l'époque, les collisions entre les protons et les neutrons
coiileuiis dans ces noyau.x devaient produire des antiprotons par les
réactions

p + p-»p?pfpt-p

et p i n —* P + Il ' P + P
Cependant, meme si ec.s rendions se sont produites, elles étaient
rares en i imipaiaison de.s réactions

P t- P —» P I P /r* + /r'

d p t n —* p - ^ n — TT + : 7r

Donc, la plupart des particules proiluitcs par les collisioas entre les
|)i'olon.s de 6,2 OeV et la cible de cuivre étaient des pions.
Pour prouver que les antiprotons existaient et qu'ils étaient pro-
duiu par les eolILsions. on dirigeait les particules quittant la cible Figure 14./ P ro b lè m e 41
Exercices et problèmes 411

Quelle est la v itesse a) d’un antipixiCon ayant une quantité de mou­


vement de 1,19 GcV/c et b) d’un pion négatif ayant la même quantité
de mouvement? La vitesse d’un antiproton traversant les détecteurs
Cerenkov serait légèrement inférieute à celle calculée ici parce qu’un /
antiproton perdrait un peu d’énergie dans les détecteurs. Quel détecteur
Cerenkov a été déclenché c) par un antiproton et d) par un pion /
négatif? Quels intert ailes de temps ài les passages e) d’un antiproton /
,i/
et f) d’un pion négatif indiquaient-ils? (Problème adapté de
O. Chamberlain, E. Segrè. C. Wiegand el T. Ypsilantis, «Observation
of Antiprotons », P/iviiT«//Jcv/cH’, vol. 100, IQ?.*?. p. 947-9.‘)0.) /
42. Un jeu de ixirticttles La figure 14.8 est un croquis représentant
les traces que lai.ssent des particules dans une chambre à bulles fU-iive d
(où un champ magnétique himtogène est perpendiculaire à la page) ; I

le tableau qui suit donne les nombres quaniiques/îcn/f (la fantaisie, /


le sérieux et la coquetterie) associés aux particules qui laissent
CCS traces. La particule A entre dans la chambre, laisse la trace 1
et se désintègre en trois parliculcs. Ensuite, la particule qui laisse N
la trace 6 se désintègre en trois autres particules, et la paniculc
qui laisse la trace 4 se désintègre en deux autres particules, dont
rim e est électriquement neutre; la trajectoire de cette dcriiicre est I
représentée par la droite pointilléc. La particule qui laisse la trace 8
est connue ptnii avoir un rximbrc quantique sérieux nul. Hqure 14.B l’ ioblèm c 42

Particule Charge Fantaisie Sérieux Coquetterie En supposant un principe de conservation ric chacun di s rinmiire.s
quanliques fictifs à chaque point de désintégration, ot en notant les
A 1 1 -2 -2
directions des courbes des ttajeetoircs, as,M)cicz chaLunc des parti­
B 0 4 3 0
cules à une trajectoire. L^nc de,s particules de la liste n ’est pas
C 1 2 -1 formée ; les auties n’apparaissent qu’une fois chacune.
D -1 -1 0 1 43. Déphuemeni vers le rouge cosmujuc. L’expansion de l’Univers
E -1 0 -4 -2 est souvent rcpré.scntéc par un croquis .semblable à celui de la ligure
F 1 0 0 0 14 9 a). Dans ce croquis, la Terre o.st située à l’endroit où se trouve
C -1 1 1 -1 le symbole nommé VI^ ([roui Voie lactée), à I onpinc d un axe dc!^ r
II 3 3 1 0 qui s’éloigne de notre planète de façon ladialc dans toutes les directions.
D’autres galaxies très éloignées y sont au.ssi rcpr6 iciiiéc,s. Leurs sym­
! 0 6 4 6
boles sont accompagnés de leurs vecteurs vitesses calculés
J 1 -6 -4 -6

»
y

VI. O Mi
B

VI.O é
d) • A

S)
c) • • • # s i h
VI A
$ T
1 ' 0 * - #
1 1
\ ' 1 ) 1 1
1 ' .A
va
b) X X q e) •

B ■ d - r ' ' - ' ■ ■• e

■■ •> « * . • ^ f p ■ . .

.r* •■ ■
1 ^ r ’ • iV-.iV ...............t ■ V

0 rs '

a)
Figure 14.9 ProblènK 43
412 Chapitre 14 Les quarks, les leptons et le Big Bang

CQ fonction du déplacement vers le n>uge de la luimère t)ui parvient où V est la vitesse radiale de la galaxie A par rapport à la Terre,
jusqu’à la Terre. Comme l’indique la loi de Huhble. la vitesse de c) Faites appel à la loi de Hubble pour comparer ce résultat de l’effet
chaque galaxie est proportionnelle à la distance qui la .sépare de la Doppler à l’expansion cosmique décrite en d), et déterminez une
Terre. Ue tels croquis peuvent être trompeurs parce qu’ils impliquent valeur pour a . À partir de celte analyse, vou.s pouvez von que les deux
1) que les déplacements vers la gauche sont causés par les mouvemaits résultats, dérives avec des modèles très différents du déplacement
des galaxies par rapport à la Terre, en s’éloignant de celle-ci dans un vers le rouge de la lumière des galaxies lointaines que l’on détecte,
espace statique (stationnaire), et 2) que la Teire est au centre de tout sont compatibles.
ce mouvement. Suppasez que la lumière de la galaxie A que l'o n détecte a un
En fait, l’expansion de rUnivers et la séparation croissante des déplacement vers le rouge de ДлД et que le taux d'expansion de
galaxies sont causées non pas par un éloignement des galaxies dans l’Univers a toujours été constant à la valeur actuelle donnée dans le
un espace préexistant, mais par une expansion de l'espace lui-méme chapitre. 0 À l’aidc des résultats obtenus en b), déterminez la distance
dans l’Univers. l'esfHwe e.\t dynamique, non statique. qui séparait la galaxie de la Terre à rinslanl où cette lumière a été
Les points b), e) et d) de la figure 14.9 montrent une façon dif­ émise. Déterminez depuis combien de temps cette lumière lut émise
ferente (le représenter l’I nivers et son expansion. Chaque partie de par la galaxie g) à l’aide du ré.sultal obtenu en a), cl h) en suppo.sant
la figure représente une sec’tion unidimensionnelle de l’Univers (sur que le déplacement vers le rouge est un effet Doppler. {Indice : En h),
un axe des r), les deux autres dimensions spatiales de l’Univers ne le temps n ’esi que la distance présente à l'insiant de l'ém ission
sont pas illustrées. Chacune des trois parties de la figure illu.stre la divisée par la vitesse de la lumière, car si le déplacement vers le
Voie lactée et six autres galaxies (reprcsciitccs par des points) ; ces rouge n’csi qu’un effet IToppler, la di.staiice ne varie pas durant le
parties sont placées sur un axe de temps vertical, où le temps augmente déplacement de la lumière. Ici, les deux modèles du déplacement
vers le haut. En h), au premier temps des trois parties, la Voie lactée et \'ers le rouge de la lumière présentent des résultats différents.)
les six autres galaxies sont représentées relativement près les unes des i) À l’instant de la détection, quelle distance sépare la galaxie A de la
autres. A mesure que le temps avance vers le haut dans la figure, l’es­ Terre (On suppose ici que la galaxie A existe cnc-orc ; si elle avait
pace se dilate, faisant s'éloigner les galaxies. Notez que les parties de cessé d ’exister, les humains n’apprendraient sa di.sparilinn qu’au
la figure sont dessinées par rapport à la Voie lactée et que, de ce point moment où la dernière lumière qu’elle aurait émise atteindrait
d'observation, toutes les autres galaxies s’éloignent en raison la Terre.)
de l’expansion. Cependant, la Voie lactée n’a rien de particulier - Supposez niaintenaiu que la lumière qu’on détecte de la galaxie
les galaxies s’éloignent également de n’impoile quel site d’observation. B (figure 14,9 gj) a un déplacement vers le rouge de ЛА/Л = 0,080.
Les points e) et f) de la figure 14.9 montrent seuleinent la Voie lactée j) À l’aide du résultat obtenu en b), déterminez la di.siance qui .séparait
et une autre galaxie, la galaxie nommée A, à deux instants particuliers la galaxie B de la Terre quand la lumière fut émise, k) A l’aide du
pendant l’expaasion. En e), la galaxie A se trouve à une distance r résultat obtenu en a), déterminez combien il y a de temps que la
de la Voie lactée et émet une lumière d ’une longueur d ’onde A. lumière a été émise par la galaxie B. I) Quand la lumière de la galaxie
En f), après un intervalle de temps àt, cette onde lumineuse est A qu’on a détectée a été émi.se. quelle distance séparait la galaxie A
détectée sur la Terre. On peut représenter le taux d ’expansion de de la galaxie B ’.*
l’Univers par unité de longueur de l’espace p a r« , qu’on suppose 44. La figure 14.10 est une représentation graphique hypothétique
constant durant fintervalle Al. Alors, durant cet intervalle, chaque des vitesses de récession v des galaxies en fonct ion de leur distance r
unité de longueur de l’e.space, par exemple chaque mètre, se dilate de de la Terre ; la droite présentant le meilleur ajustement est illustrée
a Al, donc d’une distance raAt. L’onde lumineuse de la figure 14.9 parmi CCS points. À l’aide de ce graphique, déterminez Tâge de
c) et f) se déplace à une vitesse c de la galaxie A vers la Terre, l’Univers, en supposant que la loi de Hubble s ’applique et que la
a) Démontrez que constante de Hubble a toujours la même valeur durant l ’expansion
de rUniver.s.
Af =

La longueur d’onde détectée À' de la lumière est supérieure à la


longueur d’onde émise À parce que l’espace s ’esi dilaté durant
I intervalle Ai. Cette augmentation de la longueur d’onde est appelée
déplacement vers le rouge cosmique ; ce n’e.st pas un effet Doppler,
b) Démontrez que la variation de longueur d'onde Aa ( “ A' — A)
est donnée par
Да rn
A c — ra
c) Développez le membre de droite de cette équation, en utilisant le
développement du binôme (pré.senté dans l’annexe D). d) Si vous
retenez seulement le premier terme du développement, quelle esi
l’cqualion résultante de Aa/A
Si Ton suppoM* plutôt que la figure 14.9 a) s’applique et que AA
est c:au.sée pai un effet Doppler, on a alors, selon l’équation 8..15.
AA V
~T ^ c ’ Figure 14.10 Fh‘o b lè m e 4 4
ANNEXE A
Le système international
d ’unités (SI)*

1. Les unités de bose


Grandeur Nom Symbole Définition
longueur mètre in « Le mètre est la longueur du trajet parcouru dans le vide par
la lumière pendani une durée de 1/299 792 4S8 de seconde. »
masse kilogramme kg « Le kilogramme est l’unité de masse ; il est égal a la masse
du prototype international du kilogramme.»
temps seconde s «L a seconde est la durée de 9 192 631 /70 périodes
de la radiation i inrcspondant à la transition entre les deux
niveaux hyperfins de l’état fondaniemal de ratoine
de césium 133. »
courant électrique ampère A « L ’ampère est l'intensité d'un courant électrique constant
qui, maintenu dans deux cimduelcurs parallèles, rectilignes,
de longueur infinie, de section circulaire négligeable
et placés à une distance de 1 mètre 1 un de l’autre dans
le vide, prixiuirait entre ces conducteurs une force égale
à 2 X 1 0 ' newton par mètre de longueur. »
température thermodynamique kelvin K « la: kelvin, unité de température thcmiodynamiquc.
est la fraction 1/273.16 de la température liicrmodynamiquc
du point triple de l’cmi.»
quantité de matière mole me «L a mole est lu quantité de matière d’un système contenant
autant d ’entites élémentaires qu’il y a d'atomes dans
0,012 kilogramme de carbone 12. »
intensité lumineuse candela Cfl «T a canrtela esi l’intensité lumineuse, dans une (.lircclinii
donnée, d ’une source qui éinei iiii layoïmeinem moraichro-
matique de fréquence 5'K) x HV' hertz, et dont Tintensité
énergétique dans cette direction est de 1/6X3 watt
par stévadian. »

* BUREAU INTERNATIONAL DES PtilDS ET MLSURLS. Systèrrif mternatinjuif d'imilH 7' édition. 1998, Pour plui de details.
consultez le site Web du Bureau international des poids et mesun» : www.bi|rni.lf. Dans le présenl manuel, nous n’utilisons pas la candcla

A-1
A-2 A nnexe A Ue sy stè m e International d 'u r tté s (SI)

2. Quelques unités dérivées_ _ _ _ _ _ _ _ _ _ _ _ _ _ _ _ _


Grandeur Nom de Tunité Symbole
angle plan radian rad m/m — 1
iulgle solide sicradian sr =
aire mètre carré n r’
volume mètre cube nv’
fréquence hert7. Hz s '

masse volumique kilogramme par mètre cube kg/nv*


vitesse mètre par seconde ni/s
vitesse angulaire radian par seconde rad/s
acceleration mètre par seconde carrée m/s-
accélération angulaire radian par seconde carrée rad/s^

force newton N kg • m/s-


pression, contrainte pascal Fa NVm-
travail, énergie, quantité de chaleur joule J .N • m
puissance watt W J/s
charge électrique coulomb C A s

différence de potentiel, force électronioirice volt V W/A


cliamp électrique v«»ll par mètre (ou newton par coulomb) V /in N/C
résistance électrique ohm il V/A
capacité électrique farad F C/V
tlux magnétique weber Wb V s

inductance henry H V -s/A


champ magnétique tesk) T Wb/m^
entropie joule par kelvin J/K
capacité tiKrmique massique (chaleur ttiassique) joule par kilogramme kelvin J/(kg K)
conductivité thermique watt par mètre kelvin W/(ni K)
intensité énergétique watt par stéradian W/sr

Facteur Préfixe Symbole Facteur Préfixe Symbole

lO’“ yotia Y 10- ' (léxi d


10’* ¿cita Z 10-* centi C
lO'« exn E 10 ’ milli m
10'^ jx:ta P 10 * micro
10'2 lera T 10 * nano n
lO’ G 10 12 pito P
B'B“
IIF mega M 10 ” femto f
III’ kilo k 10-'» alto a
KF hedo h 10 *' 7.epu» y.
K)' déca da K, M yocto y
* ANNEXE B
Constantes fondamentales
de la physique*
Valeur rcconiiuaiidée
C'oiiManic Symbole Valeur appr<x;hée Valetu " Incertitude''

Vitesse de la lumière dans le vide c COO X 10^ nVs 2,997 924 58 exacte
Charge élémentaire e 1,60 X 10 '"C 1,602 176 462 0,039
Constante gravitationnelle 6 6,67 X )0 ‘ " mV(s* kg) 6.673 1 500
Constante des gaz parfaits R 8,:il J/tmol ■K) 8,314 472 1,7
Nombre d ’Avogadro 6.02 X 10“’ mol ' 6,022 141 99 0.079
Constante de Boltzmann k 1,38 X 10-2’ J/K 1,380 650 3 1,7
C’onstanle de Stefan-Boltzmann iT 5,67 X 10 « W/(m2-K-‘) 5.670 4(X) 7.0
Volume molaire des gaz parfaits à ITN*" 2,24 X 10-*mVmol 2.241 399 6 1.7
Permittivité du vide 8,85 X 10“’2 F/m 8.854 187 817 62... exacte
Perméabilité du vide Pu 12.6 X 10 2 H/m 4/r exacte
Constante de Planek h 6,63 X K) »’'.I s 6.626 068 76 0,078
Con.stanle de Planek réduite h 1.05 X i n - ’” I s 1,0.54.571 .5% 0.078

Masse de l'électron rrtç 0.11 X Ifl ” kg 9,109 381 88 0.079


5,49 X 10 ’ U 5,485 799 1 10 0.002 1
0,511 MeV/c2 0.510998 902 0.040
Masse du proton Wp 1,67 X 10 22 kg 1,672 621 58 0,079
1.007 3 U l.m r/276 466 88 0.000 13
938 MeV/c2 938,271 998 0.040
Rapport de la masse du proton à celle de l’électron m airie 1840 1 836,152 667 3 0,002 1
Rapport de la charge à la masse de l'électron e tm . 1.76 X IO“ C/kg 1,758 820 174 0,010
Ma.ssc du neutron 1.675 X 1 0 -2 'kg 1,674 927 16 0.079
1.009 U 1.008 664 915 78 0.000 54
939,5 MeV/i.-2 939.565 330 0.040
Masse de l’atome d’hydrogène l,(K)7 8 u 1,007 825 0316 0,000 5
Masse de ralom c de deutérium 2,014 l u 2,014 101 777 0 o ,a x ) 5
Ma.sse de l’atome d'hélium 4,002 6 U 4,(M)2 603 2 0,ÎXi7
" ' a ,.
Masse du n u ion 1,88 X 10 2« kg 1.88.3 .3.31 09 0.0H4
Moment magnétique de l’clcctron Mc 0,28 X lü - 2” J/-) 9,284 763 6.7 0 040
Moment magnétique du proton Mp 1 41 X 10 2Sjft 1,410 606 633 0.041
Magneton de Rohr P is 9.2.7 X 10-2” J/T 9.274 008 99 0,040
Magnéton nucléaire Mt; 5.0*1 X 10 27 j / x 5.050783 17 (1.040
Rayon de Uohr Cl 5,29 X 10 " m 5,291 772083 0,003 7
Constante de Rydberg R„. l . l Ox 10' m -' 1,097 373 156 «54 9 7.6 X 10-1
Izingticnr d’onde de ( omplon pour l’électron Ar. 2.43 X 10-12 m 2,426.310 215 OfXT/ 3
l'niié de niasse atomique U 1.66 X 10 27 kg 1.660 538 7.3 0.079
931 McV/f2 931.19401.3 0.<M0
Éleetronvolt cV l.tvO X 10 i” .| 1.602 r /6 462 0,030

" Aux viilturx iun rilex dani cette cdorne. il faut ajouter les mêmes unités et puissances de 10 que les valeurs approchées enrrecpnivlampc
* Ineenilurte relative en parties par million.
s rPM sigrilie « température et pression normales » ! 0 '(_ ci I.O atm (101,.!?^ kPa).
* Source; Pclci J. Mohr and Barry N Taylor, fODATA Recommended Values of the Fondamenial Piivsicul Conatdiit.s. IWS. Jeunud ofPhriU ul
iind Chemical Reference Data, vol. 28, n” 6, et flevieu-.s afMttdern P hym s, vol, 72, n" 2, 2000. (www.physics.nLsl.gov)
im m c
Quelques données
physiques

Propriétés de i'oir et de l'eou


Air (SCC, à 20 “C cl I aim = 101,325 kPa)
Maasc volumique 1,21 kg/m’
Capacilc (i haleui'l massique à pression constante l()IO J/(kg K)
Rapport des capacités (chaleurs) massiques 1,40
Vitesse du son 343 m/s
Rigidité diélectrique 3 X l( f V/m
Masse molaire 0,028 9 kg/mol

Fou
Masse volumique 1 000 kg/m^
Vitesse du son 1 460 m/s
Capacité (chaleur; massique 4 190J/(kg K)
Chaleur latente de fusion 333 kJ/kg
Chaleur latente de vaporisation 2 260 kJ/kg
Indice de réfraction (A = 589 nm) 1„33
Masse molaire 0,018 0 kg/mol

Quelques distances de la Terre


à la Latrie* 3.82 x 10* m au centre de notre galaxie 2,2 x 10^" m
au Soleil* 1,50 X 10“ m à la galaxie d’Andromède 2,1 x lO^-'m
&l’étoile la plus proche (Proxima du Centaure) 4,04 x 10'* m à la frontière de l’Univers observable ~ 10-* m
* Di-Stance moyenne.

le Soleil, la Terre et la lune


Caractéristique Unité Soleil Terre Lune
Masse kg 1,99 X 10^ 5.98 X lO-* 7.36 X 10--
Rayon iw>yen m 6.96 V 10* 6,37 X 10* 1.74 X 10*
Ma.sse volumique moyenne kg/m * 1 410 5 520 3 340
Atreéléialioii en »'hule libre à la surface m/fP 274 9,807 1.67
Vite.sse de libértuion k m /s 618 11.2 2,38
Période de rotation“ 37 j aux pôles* 26 j à l’équateur* 23 h 56 min 27,3 j
Luminosilé'' W 3.90 X 10^* »
“ Mesurée par rapport aux étoiles éloignées
Le Soleil, qui est une boule de gaz. ne tourne pas comna» un corps rigide
‘ Juste à l’extérieur de l'aimosphère, terrestre, le taux d’énergie solaire in< idem (la eonsiante solaire) est de 1 340 W/iir. en supposant une ineidcnce norina)e.
Annexe C Quelques données physiques AS

Quelques propriétés des planètes


Mercure Vénus Terre Mars Jupiter Saturne Uranus Neptune Pluton

Distance tnoyenne dti Solal. x 10* km $7,9 108 150 228 778 l 4.30 2 870 4 500 5 900

Période de révolution, a Oé241 0.615 1,00 1.88 11,9 29.5 84.0 165 248

Période de rolauoiï', j 58,7 -243* 0,997 1.03 0,409 0,426 -0,718* 0.658 -6,3‘1''

Vitcs.se oriitlale. kni/s 47.9 35,0 29,8 24.1 13,1 9.64 6.81 5.43 4.74

biclinai.son de l’équateur 29.6° 57,5°


0* =3° 23.4“ 25,0“ 3,08° 26,7° 97,9“
par rapixrri à l’orhilc

ludinai.son de rtnWte par rapport 1,77“ 17,2°


7.00° 3.39° 1.85“ 1,30” 2,49= 0,77“
à l’oibitc terrestre

Excentricité de l’orbite 0,206 0,006 8 0.016 7 0,093 4 0.{M8 5 0.0550 0 ,0 4 /2 0,008 6 0,250

Diamètre à l'équateur, km 4 880 12 100 12800 6 790 143000 120000 51 800 49 500 2 300

Masse (Terre = 1) 0,055 8 0,815 1.000 0,107 318 95,1 14,5 17,2 0,(K)2

Masse volumique, g/cm^ 5.43 5.20 5,52 3,95 1,31 0,704 1,30 1,67 2,03

Valeur de ^ à la surface*^, ni/s- 3.70 8,60 9,78 3.72 22,9 9.05 7,77 11,0 0,5

Vitesse de libérations kin/s 4,3 10,3 11,2 5.0 59,5 35,6 21.2 23,6 l.l

Satellites naturels connu.s 0 0 1 2 52 30 21 11 1


+ anneau 1- anneaux + anneaux aiineaux

“ Mesurée par rapport aux étoiles éloignées.


* Les rotations tie Vénus, d'I ranus et de Plulon .sont dans le sens opposé à leurs révolutions.
‘ Accélération en chute libre mesurée à réqualeur de la planète.
^ANNEXE D
Formules
mathématiques
Géométrie Signes et symboles mathématiques
— est égal à
Cercle de rayon r : circonfcrcncc •- 2m : aire —n r .
est appimimativcment égal à
S.phère d e rayon r: aire = v o lu m e —
C y lin ilre c irc u la ire d ro it d e ra y o n r et d e h a u te u r h : ~ est de l’ordre de grandeur de
a ire = 2 n r ^ t 2 m h \ v o lu m e = n r 'h . n’est pas égal à
T ria n g le d e h av e a e t d e h a u te u r h ; a ire ~ \u li.
= est identique à, est défini comme

> est supérieur à ( » est de beaucoup supérieur à)


Équation quadratique
< est mféneur à (<K est de beaucoup intérieur à)
—A -t- 4rtc
Si ax- + Ar -e c = 0, alors x = > est supérieur ou égal à (pu n ’est pas intérieur à)
2ü ■
< est inférieur ou égal à (ou n’est pas supérieur à)
± plus ou moins
Fonctions trigonométriques de l’angle 6
axe oc est proportionnel à
y -r
sin 0 = cos 0 = — des V
2) la somme de
V X
tan » = — cotg 0 — - est la valeur moyenne de .r
X y
r r
sec tí = cosec tí =
X V

axe Identités trigonométriques


desx
sin(9{^ tí) = cos tí
cosl'X)*^ — tí) = sin tí
sin tí/cos tí = tan tí
Théorème de Pythagore
sin’ tí + cos^ tí 1
Dans tout triangle rectangle, scc^ tí tan’ tí = 1
r/^+A^ = c’.
cosec- tí cotg^ tí = 1
sin 2 0 — 2 sin tí eos tí
eos 2tí eos* tí — sin* tí 2 eos* tí — 1 — i — 2 sin* tí
Triangles
sintoí 1 /9) = sin a eos /9 1 eos a sin fi
Les angles sont A, K. C. cos(or F /Î) = ctw a eos ^ ^ sin a sin /1
Les côtés opposés sont a, h, c. tan a ± tan /9
tanto ± fi) = ---------------------- --
I,es angles A + B + C — I I T ta n o ta n /9
sin A sin B sin C sin o -t sin /) - 2 sin 5(0 eos |( o "f /9)
a h c
c o s o t- eos ^ = 2 eos *(o *- /9) cos^fo /))
- a' f A- — 2nh cos C
eos o —eos fi = —2 sin j(o fi)%\n 2(0 — p)
Annexe D formules mathématiques A-7

Développement du binôme L'olphobet grec


. > . n(n — , , Alpha A a Nu N F
(1 + jr)" - 1+ - + + . . . (JC^< 1)
Bêla B P Xi 2 Î
Gamma l y Omicron O O
Développement de l'exponentielle
Delta A 6 Pi 11 JT
^ .V' Epsilon E c Rhô P P
^ + + 2^ + 3! + •••
ZeUi Z C Sigma £ (7
Êta H 11 Tau T T
Développement du logarithme
Thêta 0 ft Upsilon V t»

In (l V) = T - ^ + y - . . . (¡ a | < 1 ) Iota I l Phi 4>, If


Kappa K K Khi X
Lambda A À P.si M' t/t
Développement des fonctions
Mu M M Oméga li W
trigonometriques { 0 en rodions)

• Z. n Produits de vecteurs

0'- 0^ Soit i, j et k, les vecteurs unitaires dans les directions x v et c- Alor.s,


cosW= 1 - y
' ■ t = . j ' j = k- k = l, l • j = : j • k = k • i = 0,
tan( ^ = 0 + Y + - î j + . . . i y i = j x j = k x k = 0,

ixj k, jxk=i, i i x i —j
Règle de Cromer
Tout vecteur a. dont les composantes selon les axes des x. ilrs y
Deux ét|iiations simultanées comprcnaiil les inconnues x et >i et des Z sont respecliveinent , n, et a .. peut s’exprimer ainsi -

aix t- i iT = fl et a^x r f t y = Cj a = (tri k Oy\ + fl.k.

ont comme solutions Soit Tl, h et c, des vecteurs arbitraires tic grandeurs (ou de iixxlulcs)
a, b et c. Alors
Cl bi
h2 C\t>2 — Cohy
fl X (i> + ? ) = (a X fc) + (/) X c )
«1 bi a\hi — aohi
Û2 h (.va) y. h = Tl y. (v/t) — ï(a a h) (.v est une valeur scolaire).

et
Soit ft le plus petit des deux angles formés par à et h. Alors
<h fl
07 f2 _ ei\C2 - «2t 1
Tl ■b ~ b - à ~ a,b^ + fl,.ft. a,h, — nh cos fi
"1 b\ 0\b2 ~~ 02b\
02 l>2
. i j k
b •nTt — O.X Il y a-
hx h\. b-
1Oy a? Ur a- 1. «1 üv-
- j hy
1h.» h.V h. + by
= (riyh. — hyd-U f (dybr ft« r) j 1 (ux-bf ft,«vllv

I a X h — fl/) siri fi

à • (h y c ) - b - { r y a ) = C ' ( n y b )

Tl y (h y r) (û • c)b —(a • fi)r


A-8 Annexe D i^ormutes mathématiques

Dérivées et Intégrales
üans les équations suivantes, les lettres u et v représentent des fonctions
1. J dx — X

quelconques de r ; a et m sont des constantes. Il faut ajouter une


constante d’intégration arbitraire à chacune des intégrales indéfinies. 2. j audx — a J udx
Le manuel Handbook of Chemistry' ami Physics (CRC Press Inc.)
propose une liste plus importante.
3. I iu T v)dx = j udx -f- J vc

1. = 1 j^.m+ I
i/i /
x ’"dx = ------- -
il du
2. . (au) — « . m + 1
dx dx
. d ^ du dv f dx
5. I — = In |x|

4. — x"‘ = mx"‘ '


dx
AJIf U dx-- (hK = uv — JIf
6. V— dx
dx
,
<1 1 7. / dx = r*
5. , - In X = -
ih X
I-
d dv du
su — {uvi = H — + r — 8. / sin X dx
dx dx dx
■ h
7. , d' ^
dx
'. cos xd x =
ff. , sin X — cos X
dx 10. f tan xdx = ln|secx |

9. — cc« X — —sin J»
dx • — -^ sin(2x)
• Z x d.x = -X ^

10. T tan X = sec^jr


n. f , sin‘
2 4
dx

,, O , 2 a / . a
II. , COtgjr----- COSCC Jf
llx
dx — ;; (ox 4 De
12. sec X — tan X sec x- 13. 1^ xe a-'-
dx

14
13. — cosec X - - cote .v cosec x
dx - j x^e~'^ dx - f 2<u J 2)c” "*

14. — e = e — 15 n<
dx dx
- L
d du
15. — sin U = cos U—
dx dx
k 2" >0" V “
16. — cos U = -s u if/ .
dx dx 17. / 7 = ^ — î = ln(x t- \ / x 2 f a h
J x/x^
V x ^ + i«7
d

,«. J
f (x2 + «2)3.'2
____ I
ü ^ + «7)1/2

19.
’7 (x7 -I- «2j 3/2 fl2(_j2 .J. rt2)l,/2

20. f x~"«+ U-ÎÎV dx =


. 2«» + '
{a > 0 )
Jo

21. / = X — t/ln(x + d)
f X+ d
Briâtum : Au nuiii^n» 18 de raunesc D ilans les volumes I d 2. Mipprimer la racine carrde.
i i 4' ’■'■
ANNEXE E
Propriétés des éléments
A m o in s d ’in d ic a tio n c o n tra ire , to u tes les p ro p riétés p h y siq u e s c o rre sp o n d e n t à u n e p re ssio n d e I a n n .

Capacité
Élément Symbole Numéro Masse Masse Point Point ilicrmiquc
atomique, molaire, volumique, de fusion, d’ébullition, massique, J/(g • "Cj
L g/mol g/cm' à 20 "C "C Ù25X
Actinium Ac 8*) (227) 10,06 1 323 (3 47.3) 0.092
Aluminium Al 13 26,981 5 2,699 660 2 4.50 0,9011
Américium Am 95 (243) 13,67 1 541 — _
Antimoine Sb 51 121.75 6,691 630.5 1 380 0,205
Argent Ag 47 107.870 10,49 960,8 2210 0,234
Argon Ar 18 39.948 1.662 6 X 10 ’ -189,4 -185,8 0,523
Arsenic As 33 74.921 6 5.78 817 (28 aim) 613 0,331
Asiate At 85 (210) — (302) —

Arotc N 7 14,006 7 I.164 9X 1 0 '’ -210 -195,8 1,03


Baryum Ba 56 1.37,33 3.594 727 1 897 0,205
Berkélium Bk 97 (247) 14,79 — —
Béryllium Be 4 9,012 2 1,848 1 287 2 4'Vl 1,83
Bismuth Bi 83 208,980 9,747 271.37 1 560 0.122
Bohrium Bh 107 262,12 — — - —

Bore B 5 10,811 2,34 2 030 2 550 1.11


Brome Br 35 79,909 3.12 (liquide) -7 .2 58 U.29J
Cadmium Cd 48 112,40 8,65 321.03 765 0,226
Calcium Ca 20 40,08 1,55 8.38 1 440 0,624
Californium Ci 98 (251) — — __
t.iarhnne C 6 12,011 15 2,26 3 727 4 830 0.691
Cérium Ce 58 140,12 6,768 804 3 470 0.1 Kg
CéMiim Cs 55 132.905 1,873 28.40 690 0,243
Chlore CJ 17 35,453 3.214 X 1 0 '’ (0 "O -101 -34.7 0.486
Chiome Cr 24 51.996 7,19 1 857 2 665 (1,44X
Cobalt Co 27 58.933 2 8.85 1 495 2900 0,423
Cuivre Cu 29 63..54 8.96 1 08.3,40 2 595 0.385
Curium r ’m 96 (247) 13,3 — — —
Darmstadtium üs 110 (271) — —

Dubnium Db 105 262,114 —


Dysprosium Dy 66 162.50 8.55 1 409 2 330 0,172
fcinstemium E<; 99 (254) —
Erbium Er 68 167.26 9.15 1 522 2 630 0,16/
Étain Sn 50 118.69 7,298 4 231.868 2 270 0,226
Buropium Ku 63 151.96 5.243 817 I 490 0.163
Fer Fe 26 55,847 7,874 1 536.5 3000 0.447
Â-IO Annexée Propriétés des éléments

Capacité
blément Symbole Numéro Masse Masse Point Point thermique
atomique, molaire, volumique, de fusion, d’ébullition, massique, J/(g •
Z g/mol g/cm' à 20 °C °C “C à25°C

Fcriitiuin Fin KM) (237) — — — _


Fluor F 9 18,998 4 1.696 X l 0 - ’(0 "C) -219.6 -188.2 0.753
Frauciuin Fr 87 (223) — (27) — —

Cadoiilium Cd 64 157,25 7,90 1 312 2 730 0.234


Gallium Ga 31 69,72 5.907 29.75 2 200 0„377
Germanium Ce 32 72,59 5..323 937.25 2 830 0.322
llafiiium Hf 72 178,49 13.31 2 227 5 400 0.144
t^Iassluiii Hs 108 (265) — — — —

Helium IL 2 4.002 6 0.1664 X 10'^ -269.7 -2 6 8 ,9 5.23


Holmium Ho 67 164.9,30 8.79 1 470 2 330 0.165
Hydiogciit II I 1,007 97 0,083 75 X K) ' -259.19 -252,7 14.4
indium In 49 114.82 7.31 156,6.34 2000 0,233
Iode i 53 126.9П4 4 4.94 113,5 184,35 0,218
Iridium Ir 77 192.2 22.5 2 447 (5 300) 0.130
Krypton Kl 36 83.80 3.488 X 10 •’ -157.37 -152 0,247
Lan! he ne La 57 1.38.91 6.145 918 3464 0.195
Lawrencium Lr 103 (257) — — — —

Lithium Li 3 6,9.39 0,534 180,55 1 300 3.58


Lutécium Lu 71 174.97 9,849 1 663 1 930 0.L55
Magnésium Mg 12 24.312 1,738 650 ! 107 1.0-3
Manganèse Mil 25 54.938 0 7.44 1 244 2 150 0.481
Meitnerium Mt 109 (266) — — — —

Mendélévium Md 101 (256) — — — —

Mercure Hg 80 200.59 13.55 -38.87 .357 0,138


Molybdène Mo 42 95.94 10.22 2 617 5.560 0.251
Ncodynie Nd 60 144.24 7.(X)7 1 016 3 180 0.188
Néon Ne 10 20,183 0.838 7 X 10 ’ -248,597 -246,0 1.03
Neptunium Kp 93 (237) 20.25 6.37 — 1.26
Nickel Ni 28 58.71 8,90 1 45.3 2913 0.444
Niobium Nb 41 92.906 8,57 2 468 4 927 0.264
Nobélium No 102 (255) — — — —

Or Au 79 196.967 19.32 1 064.43 2 970 0.131


fXmium Os 76 190.2 22,59 3 027 5 500 0.130
Oxygène O 8 15.999 4 1..331 8 x 1 0 ^ -218.80 -183.0 0.913
FnlLxlium Pd 46 106,4 12,02 1 552 3 980 0.243
Phosphore P 15 30.973 8 1.83 44,25 280 0.741
Platine Pt 78 195.09 21,45 1 769 4 530 0.134
Ploirh Pb 82 207,19 11.35 327.46 1 749 0.129
Pliiionium Pu 94 (244) 19.8 640 3 235 0,130
Polonium Pu 84 (210) 9.32 254 — —

Potassium K 19 39,102 0,86 63,20 760 0.758


PrascodytiK* Pr 59 140.907 6.773 931 * 3 020 0,1 U7
Proniédiium Pm 61 (145) 7.22 n 027) — —

Protactinium Pa 91 (231) 15.37 (extimaiion) (1 Z30) —


Annexée Propriétés des éléments A-11

Capacité
Élément Symbole Numéro Masse Masse Point Point thermit]uc
atomique. molaire. volumique, de fusitHi, d’ébullition, massique, J/(g • X )
Z g/mol g/cm ' à 20 °Ç X X à 25 C

Radium Ra 88 (226) 5.0 7(X) — —

Radon Ra 86 (222) 9,96 X 10-^(O X ) (-71) -61,8 0,092


Rhénium Re 75 186,2 21.02 3 180 5900 0,1.3-4
Rhrxlium Rh 45 102,905 12,41 1 963 4.500 0,243
Kiibidium Kb yi 85,47 1,5.32 39.49 688 0.364
Ruthénium Ru 44 101,107 12.37 2 2.50 4 900 0.239
Ruthertordium Rf 104 261.11 — — — —

Samarium Sm 62 150.35 7.52 1 072 1630 0.197


Scandium Sc 21 44.956 2.99 1 539 2 730 0..569
Seaborgium Sg 106 263.118 — — —- —

Sélénium Se 34 78.96 4.79 221 685 0.318


Silicium Si 14 28.086 2,33 1412 2 680 0.712
Sodium Na 11 22.989 8 0,971 2 97,85 892 1.23
Strontium Sr 38 87.62 2..54 76X 1 380 0.737
Soufre S 16 32,064 2.07 119,0 444,6 0,707
Tantale Ta 73 180,948 16.6 3 014 5 425 0,138
Teclmétiiim Te 43 (99) 11.46 2 200 — 0,209
Tellure Te 52 127.60 6.24 449.5 990 0 201
Tabium Tb 65 158.924 8.229 1 .357 2 5.V) 0,180
Tlialliuin TI 81 204.37 11,85 304 1 457 0.130
Thorium Th 90 (232) 11,72 1 755 (3 850) 0,117
Thulium Tm 69 168,9-34 9,32 1 -545 1 720 0,159
Titane Ti 22 47.90 4.54 1 670 3 2(50 0,523
Tungstène W 74 183.85 19.3 3 380 5 930 0.134
Lmmbiuni Uub 112 (285) — — — —
Lnunquadium Uuq 114 (289) — — — —

Unununium Uiiii lit (272) — — — —

Uranium U 92 (2.38) 18.95 1 1.32 3 818 0.117


Vanadium V 23 .50.942 6.11 1 902 3 400 0.190
Xénon Xc 54 131..30 5.495 X 10- ^ 111.79 108 0.1 59
Ytterbium Yb 70 173.04 6.965 824 1 5.30 0.1,55
Yttrium Y 39 88,905 4,469 1 526 3 0-30 0.297
Zinc /m 30 65,.37 7.133 419,58 9 0 (5 0..389
Zirconium Zr 40 91,22 6,506 I 852 .( 580 ( 1 .2 7 6

Pour les clcnirnis dont tous les isotopes sont iiustablcs, la colonne des masses molaires iniliqiic entre paromtVKos le nombre de masse <Ie l isoiope
dont la deiiii-vie est la plus longue Les points de lusion et d’ébullition entre parenthèses sont incertains.
1 es données relatives aux gar s'appliguem seulernem quand ceux-ci sont à leur état moléculaire habituel, comme H„ Hc. Oj. Ne, etc.
Les capacités lhei miques massiques des ga/, sont des valeurs à pres.sioti constante
Source: Adapté de J. Emslcy, The t.lemenis. .t'éd.. Oxlord. L'Iaretidou Pie.vs. ISWh. Voyez, égalcmeiil « w« wchdemcnts.tom potii ni appn iidn davaiilagc
SUT les vaU'tns lécx-ntcs et les nniivc.nux éléments.
Av:-,).'»-.
•2V i »’ ' ANNEXE F
Tableau périodique
des éléments
MtWUA
alculiiK CZl B 3 Métaux
IA 0Œ Métalloïdes
' r CS Non-inctaux
H
IIA
3 1
Li tic Métaux de irani<ili<>n
______________________
11 1/ B
Nd Mg VIIIB Al
■g
IllB IVB VB VIB VIIB r IB UB
S » Zl 22 23 24 25 26 27 2% 29 30 31
K Cu Sc Ti V Cr Mn Fe Co Ni Cu Zn Ga

9Ù W 10 U 42 43 44 45 46 47 48 49
Rb Sr Y 7r Nb Mo Te Ru Rh Pd Ag Cd In
s*s SX 72 73 74 is 76 77 7X 79 W) 81
Cs Ra ■ 4 Hf Ta W Re Os Ir Pi Au Tl
Hg
•K7 KK ÿf.:n3 K-lA liW •flf» itn UiK 104 11(1 III 112
Fr Ra Ri Db Bh Hs Mt Ds Uuu U ub
__

Métaux de transition intérieurs

- i r L <8r : 61 ; « ( ,6 5 . 'h ,
Lîimhiiiiidcs * :L a 1 C e - P r : N d ^ Foi Sm ' Eu Gd iH o . ( h r • “ TlD !"Y b ; L u .
Dy . -
■ - 1 - . ic i. ■ 'r t ■7 .• -J 'i
r>é ’1 'W ■ «1 ■ V> -93 «4. ' Û» in? "
Aelinides + A c • Th Pa U Np Pu I Am C m j ^ Bk
1
Fm
I• Md M o. I .r
J. T

Les noms des éléments 104 à 109 (rutherfordium, dubnium, seaborgium, bohrium, has.siiim et meitnerium, respectivement) ont été atloptés
par I International L nion ol J^urc and Applied Chemistry (ICPAC) en 1997 Le nom de l’élément 110 (darmsuuliiuni) a été propo.s«'
par la lUPAC en 20üi. Les elements 111. 112 et 114 ont été découverts, mai.s (du moins en 2004) n’om pas encore été nommés.
Lu découverte des éléments 1lo et I IR a été remise en question. Visite? le site wwu. webelenienfs.com pour les derniers déselt'ppcmenfs
et les nouveaux éléments.

A-12
AÜXSEQIONS
Vérifiez vos connaissances,
Questions, Exercices et problèmes
CHAPITRE 1 S3. 2rr ^mJTk 54. jL ( ; 1/2 55. a) 0..15 Hz b) 0.39 Hz c) 0
VÎRIREZVOSCONNAISSAHCES l. a) b) 4 c) O 2. relation en
56. 14° 57. b) l.a teasionest plus [x;lüe. 58. a) (t JR )Jk]m
a) (elle doit avoir la forme de l'équation 1.10) 3. a) 5 J b) 2 J c) 5 J
b) y/k/t» c) Il n’y a pas d’oscillaiivin. 59.0.39 60.6.0 %
4. l^ s périodes sont toutes égales (Dans l’cquation 1.29, m est
61, a) 14,3 3 b) 5 cxscillations 62. a) 490 N/cm b) I 100 kg/s
incluse clans /). 5 .1 , 2, 3 (I.c rapport in/h est important, mais k ne
63. a) 6) F,„/h 64. De 5.0 cm
l'est pas.) QUESTIONS 1. La relation c) 2. On dérive l'équation et,
par la suite, on effectue la substiiulion 3. a) Le point 2 b) Pcisitive
CHAPITRE2
c) entre 0 et 4. a) et b) 5. a) Vcis b) Vers c) Elle est
VÉRIHE2 VOSCONNAISSANCES 1. a) 2 b) .3 c) 1 (Comparez avec la phase
entre — et 0. d) Elle est entre —.v„, et 0. e) Elle diminue, f) Elle
de l'équation 2.2. pui.s voyez réqiiatinn 2.5.) 2. a) 2, 3, I tvnii
augmente 6. a) —;r rad. —180“ b) —;t/2 rad. —9(f c) +n/2 rad. 4-9(f
l’cquation 2.12) b) 3. 1 et 2 sont ù égalité (trouvez l'amplitude de
7. a) TTrad h) /r rad c) n/2 rad 8. a) 1 b) 3 9. a) Elle est variable.
dy/dt), 3. n) Elle reste In même (indépendante d e /), ht Elle diminue
b) Elle est variable, c) x ±An, d) Il aurait été plus probable.
(/v = v/f). c) Elle augmente, d) Elle augmente 4. a) Il augmente
10. a) Elle e.st plus grande, b) Elle est la mcnic. c) Elle est la même.
b) H augmente, c) L augmente, 5.0,20 et 0,80, qui sont à égalité, 0,60,
dT Elle est plus grande, e) Elle e.st plus grande. 11. b) (période
0,45 6. a) 1 b) 3 c) 2 7. a) 75 Hz b) 525 Hz QUESTIONS 1. Id 2. a.
infinie ; n’oscille pas), c), a) 12. a) l'n c valeur plus grande vers le haut ; b. vers le haut r. vers le bas ; tl. vers le bas ; z. vers le
b) f a même vali:ur c) Une valeur plus grande 13. .Système 1 :
b as, / vers le bas ; g, vers le liant ; /1. vers le haut 3. a) n/2 rad et
i - 1 SOO N/m. m = 500 kg ; système 2 : A = 1 2(X) N/m, 0,25 longueur d ’onde b) n rad et 0,5 longueur d’oncle c) .3;tf2 rad et
m - 4(H) kg : le même rapport k/m - 3 produit la résonance dans
0.75 longueur d'onde d) 2;rrad et 1.0 longueur d'onde c) '¡TI4
les deux systèmes. EXERCICESFî PROBIÉMES 1, a) 0.50 s b) 2,0 Hz
f) TH 4. a) 1,4 , 2, 3 b) 1,4, 2, 3 5. a) 4 h) 4 c) 3 6. l'n e inter­
c) 1« cm 2. a) 0,75 s b) 1,3 Hz c) 8,4 rad/s 3. a) 0,5(H) s b) 2,00 Hz
férence intermédiaire (se rapprochant davantage d'une imcrtcrcncc
c) 12,6 racl/s d) 79.0 N/m e) 4,40 m/s t) 27,6 N 4. 37,8 m/s^
destructive) 7. a) et d) ont la même amplitude, pui.s, h) et c) ont la
5. f > 500 Hz 6. a) 1,23 kN/m b) 76,0 N 7. a) 6,28 x 10^ rad/s
même amplitude. 8. al 8 b) Un ventre c) Elle est plus longue,
b) 1,59 mm 8. a) 10 N b) 120 N/m 9. a) 1,0 mm b) 0,75 m/s d) Elle est plus basse 9. Dans la situation d) 10. a) Un nevud b) Un
c) 570 m/s^ 10. a) 2,8 x 10-’ rad/s b) 2,1 in/s c) 5,7 km/s- ventre 11. a) En diminuant h) Quand l’un disparaît urmplètcnicnl
11. a) 1.29 X lO’’ N/m b) 2.68 Hz 12. a) 3.0 m b) - 4 9 m/s
EXERCICESETPROBLÈMES L a) 3,49 rad/m b) 31,5 m/s 2. a) U x 10 Hz
C) -2 7 0 m/s2 d) 20 rad e) 1,5 Hz f) 0,67 s 13. 7,2 m/s 14. b) 12,47 kg à7_5x I0''‘ Hzh) L 0 m à 2 ,0 x l()^m c)r).0x 10'*lTzà3,Ox I0'” Hz
c) 54.43 kg 15. 2.08 h 16. 22 cm 17.3.1 cm 18. a) 25 cm b) 2,2 Hz
3 .a )0 ,6 8 0 s b ) l,47Hz.c)2,(Kim/s 4. (0,()l0m) stn|/rt.U3.it '
19. a) 5,58 Hz b) 0,325 kg c) 0,400 m 20. a) 0,500 m b) -0,251 m (1.10 X 10-^)f -I- 0,250)1, "ft mètres et 1 est e.ii seiondes
c) 3,06 m/s 21. a) 2,2 Hz, b) .56 cm/s c) 0,1(X) kg d) 20,0 cm sous v;
6. a) 6,00 cm b) KK) cm c) 2.CK) Hz d) 200 cm/s c) La direction
22. 2jt/3 rad 23. a) 0,183/î h) Elle se déplace dans la même négative de Taxe des v H 75,4 cm/s g) —2.55 cru 7. a) (2.00 cm)
direction. 26. a) 1,6 x 10^ m/s’ b) 2.5 m/s c) 7,9 x lO’ m/s'^ rl) 2,2 m/s sinf0.628.v (2,51 X l(>3)t ‘ 4,00]. où .vest en centimètres et (est
28. a) 0,525 m b) 0,686 s 29. a) (n I )k/n b) (n 4- 1 )A- en secondes b) 50,3 m/s c) 4 0 1 1 m/s 8. h) 2,Oem/s c) (4,1) cm)
c) ^/(^^ 4- 1) /n f (I) v ^ n T T / 30. a) I. I Hz b) 5.0 cm 31. 37sin(TO:/lü
mJ —W/5 Æ). où X est en cm cl t est en secondes
32. a) 200 N/m b) 1,39 kg c) 1.91 Hz. 3.3. a) 2,25 I Iz b) 125 J c) 250 J d) 2.5 cm/s vers Taxe des y négatifs 9. a) 11.7 cm b) /Trad 10.3.2
d) cm 34 a) 7,25 x 10* N/m h) 49 400 personnes
ll.l2 9 n i/.s 12.v/2 13. a) 15 m/s b) 0.0.36 N 14. a) .10 m/s b) 17 g/m
35. a) 130 N/m b) 0,62 s c) 1.6 Hz d) 5.0 cm e ) 0 .5 1 m/s 15. (0 120mm) sin[(0.141 mm ')r (678.v” '); 1-7 161
36. a) mv/(ni M) b) mv/-/k{tn 4- M) 37. a) | b) j c) x,„/ J ï 16. .3.0 X 10^ m /s 17. a) h) Non 18. al 0 .64 Hz h) 63 cm
38. a) —(80 N) cos |(2 000 rad/s)r - jt/3 radl b) 3.1 ms c) 4.0 m/s c) \<.r, r) = 5,0 sin(0.1Ür — 4,üt 1. ou X et y sont exprimes en
d) 0,080 J 39. a) 16,6 cm b) 1,23 % 40. a) 0.735 kg - m’ cenliiiièires et t, en secondes d) 0.064 N 19. a) 5,0 cm b) 40 cm
b) 0.024 ON m c) 0.181 racl/s 41. a) .39.5 rad/s b) .34.2 rad/s c) 12 m/s d) 0,0.33 s c) 9,4 m/s D >(jt. t) = (5.0 u n ) siiif(0,16 i.ni“ ')x
c) -124 rad/s^ 47. a) 8,5 s b ) non 43 yc) 44, R 77 s 45. 5.6 cm -t- ( 1. 9x 1 0 -'s-')( + 0.93)] 2 ü .a)2 8 .6 m /sb )2 2 .1 in /8 v ) 188g
46. 27T ^ - P''*“ augmente pour d) .313 g 21. A 2,6.3 m de l’cxtrémitc du til oii la deuxième impiil
d < / / \ / Ï 2 , diminue pour«/ > L j'J Y l c) Elle augmente, sion a été transmi.se 22. a) y è(A /)(/ 1- Al)/m 24. 198 Hz
d) La période reste la même. 48. a) 0.869 s b) z = RH - 6.25 cm 25. a) 3.77 m/s b) 12,3 N c i Od) 4o,4 W e)() J')0g(?O ..'t0ati
49. a) 0,205 kg ■m- b) 47,7 cm c) 1.50 s 50. a) 1,64 s b) Elle est 26- a) 82.8° O'! 1,45 1ad i-) 0,2.30 longiienr d'oncle 27. 1.4 l.v,„
égale à la période initiale. 52. a) 2n \ j b) 0.289 28. a) 0.31 m b) l.Cvl rad c) 2.2 mm 29.5,0 cm 30.1,5 rad

fil
R-2 Reponses aux sections

31. a) 0.83vi b) 0,644 rad 32. a) 2/^ bj 33. a) 144 tn/s b) 60,0 cm 31. a) 57,2 cm b) 42.9 cm 32.0,125 m ; 0,-375 m ; 0,625 m ; 0,875 m
c) 241 Hz 34. 10 cm 35. a) 82,0 iti/.s b) 16,8 ni c) 4,88 Hz ,33. a) 405 m/s b) 596 N c) 44,0 cm d) 37,3 cm 34. a) À 5.0 cm
3ft. aj 66,1 ni/s b) 26.4 11/ 37. 7.9! Hz. 13.« Hz, 23,7 Hz d’une extrémité b) 1,2 c) 1,2 35. a) 1,13 kHz, 1,51 kHz et 1,88 kHz
~ A b 39. a) 105 Hz b) 158 m/s 36. a) ¿(1 - I//) b) 13 cm c) 5/6 .37. À 12.4 m 38. a) 71,5 Hz
40 b) L'cnergic est uniqucnient coinpr)sée de l’éneigie cinénque b) 64,8 N .39. a) Un tneud c) 22 s 40. a) 0,20 m, 0.60 m. 1,0 m
(lev vecrionv de la corde droiie qui se déplacenl iransversalemenl. b) 0.60 m c) 143 Hz 41.45,3 N 42. 2,25 ms 4.3. 387 Hz
41. a) 0.25 cm b) 120 cnVs c) 3.0 cm d) 0 42. a) 0.50 m b) 0,0.25 s. 44. a) 10 b) 4 45. 0.020 I 4 6 .0 47. 17,5 kHz 48. 4,61 m/s
0,5üs 43.a) 50 Hzbi.v - (5.0 x 10 -’ ) sin{7tl.r ± ( l,0 x l()-)ij|. 49. a) 526 Hz b) 555 Hz 50.0.195 .MHz 51. a) 1,02 kHz, h) 1.04 kHz.
en unilés M 44. 36 N 45. a) 1.3 in b) y = (2.0 x 10'-’ ) sin(9.4r) 52. a) 1,58 kHz b) 0,208 m c) 2.16 kHz d) 0,1.52 m S3. 155 1Iz
cos[(3.8 X 10’)f]. en unites SI 46. a) 4.0 m b) 24 m/s c) 1.4 kg 54. 41 kllz 55. a) 485.8 Hz b) 500.0 Hz c) 486,2 Hz dl 500,0 Hz
d) 0.11 s 47. a) 2.0 Hz b) 2,0 in c) 4.0 m/s d) 0.50 m. 1.5 m. 2.5 ni. 56. a) 2 000 Hz b) 2 000 Hz 57. a) 598 Hz b) 608 Hz c) 589 Hz
c l c t OOm 1.0 m. 2,0 m 3 0 m. eic 48. a) 0 ni. 0.20 m. 0.40 m 58. 30,0'' 59. a) 42" b) 11 s 60. 33,0 km
b) 0.030 s cl 8,0 m/s d) 0,020 m e) 0 s, 0.025 s, 0,050 s
50. û) -I 0,0 K) m b) 0 c) 0 d) -0 .1 3 ni/s 51. a) 324 Hz b) 8 nœuds CHAPnREA
52. a) 8.0 cm bl LO cm VÉRIFIEZVOSCONNAISSANCES 1. a) (Servez-vous de la figuie 4.5.)
Sur le côte droit du rectangle. É est dans la direction negative de
CHAPITRES l’axe de.s s ; sur le côté gauche, E 3 dÉ est dans la meme direction
VERIFIEZVOS COHNAISSAHCtS 1. Bile commence à diminuer (exemple : mats possède un plus gnmd module, b) £ est orienté dans la ditection
déplace/ nienialcnieiil les courbes île la figure 3.7 vers la droite en négative de l’axe de.s v. .Sur le côté ilroil, B est dans la direction
observant ce qui se passe au point x = 42 cm). 2. a) 0. une inter­ négative de l’axe des z ; sur le côté gauche, B + dB est dans la même
férence constructive b) 4,()X, une interférence œactructive direction mais possède un plus grand module. 2. Il est dans
3. b ) ! et ? reçoivent la même intensité, puis 3 (voir l’équalion 3.28) la direction positive de l’axe des x. 3. a) Elle reste la même,
b) .3, puis 1 et 2 ont la même aire (voir l équation 3.26) b) 11 diminue 4. Le dessin a) 5. a) Non b) (Xii
4. Le deuxième (voir l’equatton 3.39) 5. Il faudrait la détendre. QUESTIONS 1. a) Dans la direetton positive de Taxe des z
6. a) Plus élevée b) Mnin.s élevée c) Impossible à déterminer h) Parallèlement à l’axe des v 2. En entrant dan.s la page
d) Impossible à déterminer c) Plus élevée f) Moins élevée 3. h. 30" r. 60‘ d. 60“ e. W f. 60’ 4. a. h et c 5. d. b .o c lc
7. (Vfe.sure/, les viies.ses par rapport à l’aii.) a) 222 m/s b) 222 m/s 6. Aucune 7. a) Dans la région du point h b) La bleue
QUESTIONS 1. L’impulsion suivant la trajectoire ?. 2. a) I .a même c) Dans la région < 8. Le rayon 6 9. 1,5
que celle de la première onde b) X c ) d ) (2tn I);rrad, EXERCICES U PROBLÈMES 1. a) 0.500 tres b) 8,4 mtn c) 2,4 h
où /» - 0. ±1, ± 2 .... 3. a) 2.0 longueurs d'onde b) 1.5 longueur d) 5446 av. J C. 2. a) 4,7 x 10 '^ Hz b) 3 min 33 s 3. a) 515 nm,
d’onde c) En a), rinterfétencc est constructive, et en b), elle est 6 IO n m b )5 5 5 n m ,5 ,4 l x lO'” Hz., 1,85 x 10 '•‘'s 4.7,49G H z
dc.struchve. 4. X/2 5. a) Elles sont en opposition de phases, 5. a) Elle augmenterait régulièrement, b) On peut obtenir la période
b) Ellc.s .sont en opixtsiiion de phases, ft. a) Elles sont en pha.se. apparente de revolution de la lune de Jupiter en mesurant l’intervalle
b) Ellc.s .sont en pha.se. 7. a) I h) 9 8. a) 2 b) Im ventre 9. a) Elles de temps cntic deux éclipses de celle-ci (début du pas.sage de la lune
augmentent, h) Elles diminuent. 10. 150 Hz et 450 Hz 11. Tou.s les derrière la planète). On obtient c à l'aide de /* — / = R/c, où rest
haniioiiiqiiev impaü v 12. Dans le tuyau il, le tiwxJe fondamental le temps réel que met la lune à faire les N révolutions entre
1.3. a; .3, puis I et 2 ont la meme valeur de v b) 1, puis 2 et 3 ont la les positions X et V. t* est le temps qu'elle met à cftcctucr
même valeur de uJ c) 3, 2. 1 EXERCKES ETPROBLEMES I. Divisez le les N révolutions mesuré par un observateur sur Tene et B est
temps par 3. 2, L'auditeur éloiaié. le delai étant environ de 0.858 s le rayon de l’orbitc terrestre. 6.4,74 m 7.5,00 x 1 0 H
3. a) 79 ni. 41 m ht 89 m 4, I 72 m 5. 1.9 x 10’ km 8. 1,07 X 10 ‘2 T 9 . B, = 0, /f, = 6,7 X 10 cosItt x l()'’(t - ,v/c)|.
6. a) I( E — v)/( l'r) b) 364 m 7. 40.7 m 8 . 1 ntre 1,7 cm et 17m B. = 0 en unilés SI 11.0.10 MJ 12.4.8 x 10 W/m’
9. a'; 0.076'’ mm h) 0 333 nim 10. a) 6,0 m/s 13. 8.88 V in'* m^ 14. 1,2 MW/m’ 15. a) 1.67 x lO"» T
b) .r ~ (0..î0cm) sint«rx/12 50 nt). où est exprimé en cm et f en .s b) 3 ,31 x K )-’ W /m ' 16. 1,03 kV7m : 3.43 i/T 17. a) 6.7 nT
Il at 1,50 Pah) 15R Hz c) 2.22 nul) 350 m/s 12. a)Xh)K b) 5,3 mW/m^ c) 6.7 W 18. a) 1,4 x 10'^^ W b) 1.1 x 10*' W
13. a) 343(2m — I ) Hz. où ni est un nombre entier cntie 0 et 28 19. a) 8.7 X 10' 2 v /m b ) 2.9 x 10 ">Tc) 1.3 x lO“*W
h) 686m Hz, où m est un nombre entier entre I et 29 14. 4,12 rad 20.3.3 V 10 **Pa 21. 10 MPa 22. a) 6.0 x 10* N
15. a) 143 Hz. 429 Hz. 715 Hz b) 286 Hz. 572 Hz. 858 Hz. b) /p 3.6 X l()-’2 N 2.3. 5.9 X K) * Pa 24. a) 3.97 x 10^ W'/iir’
16. 17,5 cm 17. 15,0 mW 18. a) 0,080 W/in^ b) 0,013 W/m^ h) 13,2 Pa c) 1,67 X 10 '* N d) 3,14 x kP nt/s^ 25. a) 1.0 x 10* Hz
19. 3.68 X lO“ *' m 20. 1.26 21. a) 1.0 x 10-’ b) 32 b) 1.00 X 10 r dans la direction positive de l’axe des r. c) 2,1 rad/m,
22. a) 8.84 x U r ’'W /m -b ) 39.5 dB 23. a) 59.7 b) 2.81 v 10 'f 6.3 X 10* rad/s d) 119 W/m^ e) 8,0 x 10“’ N. 4.0 x K)-’ Pa
24. .Vf,! est proportionnelle à /• 25. h) 5.76 x 10 '^J/m* 26.491 nin 29. l,9m rn/s .30. 0.96 km^ 31. b) 5,8 x IfD^m
2ft. a ) 5 .0 x lo’ ht 71c) 71 27. ht Longueur au carré 32. 180' 33. 1.48 34. a) Oui b) 1.3 35. 1.26 .’ 6. a) 56.9' b) 35,3''
28. a) 5.97 x lO"*' W /m ' b) 4.48 x 10 '>W 29. a) .5.20 x lO’ Hz 37. 1,07 in 38. Voir la (igairc 4.14 bl 40. b) T,e rayon émergent
b) amplifusic .SXAVamplibide .VW> = 2 .M». a) 833 Hz b) 0,418 in est touioufs parallèle au rayon incident. 42. 34“ 43. 1.22
Réponses aux sections R-3

44.4.11 m 45. a) 48,9' b) 29,0° 46. a) Oui b) Non c) Entre 42,9° l’objet, elle est vinuelle, elle est située à 30 cm à la gauclie de la
et 43,3° 47. a) Couvrez le centre de cliaque face du cube lentille divergente; m = i-1,0. 33. a) La position de l'image finale
avec un disque opaque d'un rayon de 4,5 mm. b) Environ 0,64 est la même que celle de l’objet initial et elle est agrandie 5,0 fois,
48. a) 35,6° b) 53,1 ° 49. a) \ / 1 + sin- 6 b) \/2 c) De la lumière c) Elle est virtuelle, d) CXii 35. Une lentille convergente b) À 26.7 cm
émerge dans l’air à la droite du prisme, d) Aucune lumière c) 8.89 cm 37. a) L’image virtuelle ,se situe à 26,3 cm de la lentille
n'emerge dsins l’air à la droite. 50. a) 15 m/s bl 8,7 m/s du côté oppo-sé à l’objet. b> L’image réelle .sc situe à 18,4 cm de la
c) Plus haut d) 72° 51. al 3.15 ra b) Entre 10 m et 12 m c) 2,40 ni lentille, du même côté que l’objet. 38. +3.17 D 39. -0 .6 6 7 D
à rcxiréinilé ouest du tombeau, 1.20 m à l'extrémité est 40. a) À 50,0 cm derrièrc l’a-il b) L’evil est hypennetrope. c) 4 2,00 D
(la base est plus élevée de 1,20 m du côté est) 52. .30 cm 41. a) 5,42 D b) Myopie c) —1,25 D d) 18,5 cm 42. a) Presbytie
b) À 57.1 cm devant .ses yeux c) 44,4 cm 43. a) À 57,1 cm devant
CHAPITRE 5 lui b) +2.25 D c) De 33,3 cm à l'infini 44. a) La personne est
VÉRIFIEZ VOS COHHAiSSANCES 1. 0.2J. 1M 2.2cl 2. a) Elle est rccllc. myope et presbyte b) —I , M D et + 1.1 4 1) c) De 57,3 cm à l'infini,
b) Elle est renversée, c) Du même côté 3. a) La situation c) b) Elle et de 25,0 cm à 44,4 cm 45. a) 2,78 b'i 4,00 46. a) 2,50 b) 9.00 iniii
est vinuellc et du même côté. 4. L’image est virtuelle et droite, 47. 125 48.a) 13,0cm h) À 5.23 cm 0 - 3 , 2 5 d) 3,13 c ) -1 0 .2
et la lentille e.st divergente. QUESTIONS l.L a c o u rb e r 2. a l o b l c 49. a) - 1 3 3 b) -1 4 0 50. a) 3,50 b) .3.13 51. a) (7 = 1 - (25 cm)//
3. a) Tes monstres a et c b) Trois fois c) Sa propre image 4. a) De b) G (25 ciny/c) 3_5, 2,5 52. a) À 8.4('> mm de robjccüf h ) —192
l’infini vers 1e foyer b) Elle diminue continuellement. 5. D’un miroir 53. a) —62,5 b) 152 cm c) —33° 54. a) 2,35 cm b) Ils doivent les
convexe 6. La lentille d) (à l’infini), puis les lentilles a) et b) possè­ diminuer 55. a) 5,3 cm b) 2,6 mm
dent la meme distance focale, e t enfin, la lentille c). 7. a) Elle diminue.
b) Elle augmente, c) Edle augmente. 8. La distance focale du miroir CHAPITRE6
est égale à 3 cm, celle de la lentille est plus grande que 3 cm. VÉRIFIEZVOS CONNAiSSANŒS 1. 6 (plus p etit/;), c, r; 2. a) l.e matériau
9. a) Toutes les situations, sauf la tleiixième b) Dans le cas des du dessus b) Une illumination moyennement brillante (la différence
situations I, 3 et 4, l’image e.st réelle et renversée : dans le cas de phase est de 2.1 longueurs d’onde) 3. a) 3/_ 3 b) 2.5Â. 2,5
des situations 5 et 6, l’image est virtuelle et coaserve la même 4. a) et d) sont à égalité (l’amplitude de l’onde ré.suitante est 4£o),
orientation. 10. a) Elle est plus petite, b) 11 doit être plus petit. puis b) et c) sont à égalité (l'amplitude de l’onde résultante est 2Ey).
EXERCICESETPROSLftlB 1.40 cm 2.9.10 m 3. a) 3 4. a) 7 b) 5 c) I à 3 5. a) I et 4 b) 1 et 4 QUESTIONS 1. n, r. 6 2. a) Il y a une crête.
d) Cela dépend de la position de O et de la per.spective d’où il est vu. b) Il y a un creux. 3. a) 3(X) nm b) Ils sont dcpha.sés de ;rrad.
6. À 1.5 m 7. La nouvelle intensité vaut 10/9 de l’intensité inttiale. 4. a) 2d b) (nombre impair) A/2 c) X/4 5. a) Une inierférenee
8. À 351 cm 9. A 10,5 cm 10. a) f , t 40 cm, —20 cm, + 2,0, Non, intermédiaire plus pirx lie du niaxiiiuiiu, m — 2 b) Une iiiierféreiKe
Non b) Plan, oo, O’, —10 cm. Non c) Concave, +40 cm, ' 60 cm, destructive, rn = 3 c) Une interférence intermediaue plus proche
—2,0. Oui, Oui d) C oncave,+20 cm, f 4 0 c m, ' .30cm,Oui,(X)i du maximum, /« — 2 d) Lnc interférence constructive, m — I
e) Convexe, —20cm , + 20 cm, +0,50, Non, Non 0 Convexe, —, 6. a) Elle augmente It) lA 7. a) à c ) Il diiiiiiiiM' d) l a com|X)same
—40 c m ,- 1 8 c m ,+180 cm. Non, Non g) 20 cm, , , 1-5,0 cm, bleue 8. a) et c) ont la meme inlcnsitc, pins h) cl d) ont lu meme
-I 0,80, Non. Non h) Concave. 1 8,0 cm, I-16 cm, H 12 cm. , Oui intensité (zéro). 9. a) Un maximum h) l'n miriimim c) Des minima
12. b) 0.56 cm/s c) 11 m/s d) 6,7 cni/s 13. a) 2,00 b) Aucun et des maxima en alternance 10. L’onde </ 11. a) ^ longueur
14. a) - 1 8 cm. Non b) - 33 cm. Non 0 + 7 1 cm. Oui d) Toutes tl’unde b) 1 longueur d'onde 12. a) Non b) 0 c) IL
les valeurs de n? possible,s. Non e) t 30 cm. Non f) +10 cm, Non EXERCICES ETPROBIEMES 1. a) 5,0« v lO'^ Hz b) 4XX nm
g) 26 cm, Non h) LO, Oui 16.42 mm 17. 24.4 cm au-dessus c) 1.97 X 10« m/s 2. 4.55 x 10'^ m/s 3.1.56 4. 2.06 x 10» m/s
de robjet 18. a) À 90,0 cm du centre de la splicre du côté o|jposé 5.22°; la réfraction réduit la valeur de 0. 6. a) l/iniinilsion 2
à l’objet b) À 12,9 cm du centre de la sphère du côté itpposé à l’objet b) 0,03L/< 7. a) 3,60 pin h) Dm: ialcrféreiiLe inlermcdiairr, plus
19. Dans le verre, à 9,29 cm de la deuxième surface traversée par priX'lK d’une iiiicrfércncc constructive 8. a) 1,70 b) 1,70 e) 1..30
le.s rayoas lumineux 2 0 .^ —12 cm 21.1.86 mm 22.45 mm. d) Les brillances .sont identiqui's. plus près d'une interfèrem e
90 mm 23. a) + 4 0 cm b) À rinlïni 25.5,0 mm 26. À 15,4 cm constructive, 9. a) 0.833 b) line interférence inleniicdiairc. plus
ou 28.6 cm 27. Iji mention X signifie que la quantité ne peut êlic (liés d’une inierférenee eimsliiiclive 10. a) I 55 p.iii b) 4.65 pui
trouvée à l’aide des dottnées fournies, a) +, X. X, 1-20 cm, X, —LO. i l . a) 0,216 rad b) 12.4" 12. Unt + Vm J3.2.25m m
Oui, Oui b) C, X, X, —10 cm, X, i-2,0, K<hi, Non c) C, + , X, X, 14. a) 0,010 ra.l h) .5,0 mm l5.64X nm 16.0,15 17.16 IX. 0
- 1 0 cm, X, Non. Non d) D, . X. X, - 3.3 cm. X, Non, Non 19.0.072 mm 20. 8.75X 21. 6,64 /ini 22. a) 0.25 i mm
c) C, 1 30 cm. —15 cm, + 1,5, Non. Non f) D. —30em. —7.5cm. bt La (igure est dér aléc de sorie que le minimiim de ’..57 remjilacH
0,75, Non, Non g) D, 120 cm, 9,2 cm, 4 0,92, Non, Non le maximum central. 23.2.65 24. > * 17.4 sin (m f-* 0.23)
h) D, 10 cm, X, X, 5,0 cm, X, 4 , N<m i) C, l-.3,3 cm. X, X. 25. V - 26.8 sin (mr + 0.15) 27. a) 1.17 m. 3,00 m. 7 50 m b) Non
+5.0 cm. X. Oui. Oui 28. a) 4 16,0cm bl 32,0 cm 29. a) ¡7..50cm; 29. / = /(J 11 I 8 cos" (<i>/2)|. - intensité de la lente
lentille convergente b) 3n,n cm , lentille divergente 31. a) À 36 cm la plus étroite quand la donvièmo fente est nb<jmiée .30. n) I 5S nm
derrière la lentille convergente b) 1.2 cm c) I lie est réelle, d'i Elle b) 3I Onm 31. Constructive 3 2 ./ = (m ^¡^)kl2. lorsque
e.st renversée 32, Elle a la même orientation et la même bantenr que w - 0. 1, 2. . . . 33. 0,117 pm . 0.352 p ni 34, A/5 35. 70 0 nm
R-4 Réponses aux sections

36. Dans aucune 37. 120 nm 38. Aux situations a) et c) 40. 523 nm 44. La gamme ira de 470 nm à 560 nm.
39. a) 552 nm b) 442 nni 40.673 nm 42. 338 nm 43. 140 franges 46. 491 fente.s 47.3.65 x 1(P fentes 48. a) 56 pm b) Aucun
44. a) Somltre bj À l 'extremiie bleue 45. 1,80 ;im 46. 840 nm 50. a) 1,0 X 10* nm b) 3,3 mm 52. a) tan Ob) 0,89 53.0,26 nm
47. 2.4 Min 48. 1,0002.5 49. \J(m ■+ ^ )kR, lorsque w = 0. 1,2, ... 54. 25 pm, 38 pm 55. 39.8 pm 56. a) 170 pm b) 130 pm
50. a) .34 anneaux b) 46 anneaux 51. 1,00 m 58.0. 570 nm 59. a) xio/V^, «o/s/s. a u l Æ . «ц/х/Тз,
5 3 ..r =” {D/2a)(m +• i )?u lor.sque m = 0. 1. 2, ... 54. 5.2 nm 60. À À = 130 pm lorsque m — 3.à к ^ 972 pm lorsque m = 4
55. 588 nm 5 6 .1'3c 0,354 mm 57. i,(K1030 58 .1 = /„, co.s^ 2 â’a'/^), 61. À 30.6" et à 15,3' (.sens horaire), à 3,08" et à 37,8°
(Ml /,„ est r intensité maximale 59. a) 0 b) C'onstnictive c) Il augmtailo. (sens antihoraire) 62. a) l.a direction négative de l’axe des y
(1) Différence Posilion Type b) Æj = -c B sin(l:v + wï), £ ï = £',• = 0 c) L’onde est polarisée
(le phase x (/xml et E est orienté le long de l’axe des г. 63. a) 1.9 V/m
0 -* -X' C b) 1.7 X 10 " Pa 6 4 .4 3 x 1 0 -’ % 65.3.1% 66.20° ou 70°
0,5()A 7,88 d 67.4,4W /m ’ 68. l9W /m ’ 69. | 70. a) 0 ,16 b) 0.84
1.(ЮА 3.75 c 71. a) 2 polariseurs Ы 5 pdariscurs 72. a) Environ 53° (53.1’’) b) Oui
1,50X 2.29 d 73.49,0° 74. 55.8°; 55,5° 75. a) 11 la diminuera, b) 1 l"c) 0,23°
2ХЮХ 1.50 c 76. a) 50 m b) Non. сгг la largeui de 10 m est trop étroite poui être
2,50?. ü.9/5 d distinguée, c) Pas durant le jour, mais la pollution lumineuse pourrait
60. f) 0,40 constituer un signe certain de vie intelligente durant la iiiiil.
77. Entre 15 fim ét 220 /xm 78. 1,0
CHAPITRF7
VÙlIftU vos CONflAISSANŒS 1. a) Elle s’étend en s'éloignant du centre CHAPITRES
luillaiil. b) Elle s'étend en s'éloignant du centre brillant. VÉRIFIEZ VOS CONNAISSANCES 1. a) Elle est égale (|xisnilat de la viie,sse
2. a) Du deuxieme maximum secondaire h) 2,5 3. a) Le rouge de la lumière), b) Non (le point de départ et le point d’arrivée
b) Le violet 4. Il diminue. 5. a) Elle augmentera, b) L restera le même. de l’impulsion sont séparés dans l’espace), c) Non (car aucun
6. a) À gauche b) Elles seront plus petites. 7. a), d), b), c) (zéro) des observateurs ne mesure un temps propre), 2. a) La mesure
OUFSHONS 1. a) Elle se contracte, b) Elle sc contracte. de .Sophie b) La mesure de Strphie 3. a) N('n b) Oui c) Non
2. a) Le minimum m 5 b) Le maximum entre m = 4 et m = 5 4. a) Vers la droite b) Elle est supérieure. S. a) L’énergie cinétique
(approximativement) 3. Dans la situation avec le mégaphone est égale, b) L’énergie tiMalc est inférieure, car l’énergie au repos
(une plus grande ouverture et moins de diffraction) d’un électron e.si moins grande que l’éncrgic au repos d’un proton.
4. a) l,es ouvertures I et 3 sont à égalité, puis les ouvertures 2 et 4 QUESTIONS 1. n y a égalité entre les differentes mesures (vites.se de
sont à égalité, h) I^ s ouvertures 1 et 2 sont à égalité, puis l'impulsion - e). 2. a) H\ b) HJ 3. a) H] b) Hj 4. a) Samuel
les ouvertures 3 et 4 sont à égalité. 5 .4 6. a) Il deviendrait plu.s b) Ni l’un ni l’autre S. a) Négatif b) Positif 6, a) Non b) Oui c) Oui
grand, b) Rouge 7. a) La largeur est moins grande, b) La distance 7. Inférieure 8. a) 3, égalité entre 1 et 2. puis 4 b) 4. égalité entre I
est plus grande, c) Le rapport est plu.s grand. 8. a) Elles diminueront, et 2. puis 3 c) 1, 4, 2, 3 9. b), a), c), d) 10. a) 3, puis égalité entre 1
b) Llle restera la même, e) Elles rc.sleronl au même endroit. et 2 b) 2. puis cgalilc entre I et 3 c) 2, 1.3 (1) 2. l, 3
9. a) Elles diminueront, b) Elle diminuera, c) Elles seront d(icalc(s EXERCICES ET PROBLÈMES I. a) 6,72 x I0 ~ s b) 2.20 x 10“ m
ver» la droite. H), a) Le ré.scau A b) Celles de la paire Z a ) 3 x 10-'* b) 9.26 X IO -* c )l,l x l O - ^ d ) 3 , 7 3 x 10-Se)0,10
de gauelic c) À gauche d) À di'oitc 11. a) Elle augmente. 3. 0,990 c 4. a) 0 ,140 b) 0,9905 c) 0,999 950 d) 0,999 999 50
b) Dans le pn-mier ordie 12. a) Elle reste la même, b) Elle augmente 5 . 0. 446 ps 6. a) 0,999 999 5(V 7. 1,32 m 8. 1,53 cm
c) Elle dirtüiMie. 13. a) et b) 4 = 1, n = 4, 0 ~ .30' 14. c 9. 0,626 m 10. a) ü.8C>6c b) 2.(Ю 11. a) 87.4 ni b) 3.94 x 1 0 " s
[XEROCES n PROBLÈMES 1.60,4 /xm 2. a) 0.430'’ h) 0,118 mm 12. h) 0,999 999 I5c 13. a) 26,3 a b) 52.3 a c) 3,71 a
3. a) k„ ■ IXf, b) Des numma coïncident lorsque m/, = 2m„. 14. a) .v' = 0. /' = 2.29 s b) x = 6„54 x 10* m. f' = 3.16 s
4. a) 2.5 mm h) 2,2 x 10"'’ rad 5. a) A 70 cm h) 1,03 min 6. À 41.2 m 15. / = 1 3 8 km. /' = - 374 JUS 16. = 0. li = - 2 ,5 /xs
de l’ûxe central 7 . 1.77 mm 8. 160" 10.a)0.18"b)ü.46rad 17. a) 25,8 /x.s b) I X' petit éclair 18. a) S 'd o it sc déplacer vers l’axe
c)0,93 l l . d ) 52,5", 10,1°, 5.06“ 13. b) 0 rad. 4.493 rad, etc. des X négatifs à une vitesse de 0,480r. b) Ix gros éclair t ) 4,39 /js
c) -0 ,5 0 .0 9 3 . etc. 14. 3! /(ni 15. a) l , 34x 10~'’ radb) lOkm 19. a) 1.25 b) 0.800 /xs 20. 2.40 /is 2 1 .0.81r 22. a) 0.84< dan.s la
16. a) 1,3 X l ( r ‘'ra d b )2 l km 17.50ni 18.30m 19.a) 1,1 x KT^kni direction de l’axe des x positifs b) 0,21c dan,s la direction de l'axe
h) 11 km 20. 53 m 21. 27 cm 22. 4.7 cm 23. a) 0,347° b) 0.97° des A positifs. les prédictions galiléennex sont de 1. 1« et de fl. 1,5c.
24. a) Rouge b) 1.10 /xm 25. a) 8.8 x 10~^ rad b) 8.4 x 10’ km 23. a) 0.35c b) 0.62c 24. 0,588c. en s'éloignant 25. 1,2 /xs
c) 25 /xm 26. Environ 4 x 10“ ’^ 27. 5 28. 3 29. a) 4 b) Une frange 26. a) 1.3 a b) i .60 a c) 4.00 a 27. 22.9 MHz 28. a) 7,0 x IflO m/s
est éliminée toutes les quatre franges 30. à PM 31. a) 9 b) 0.255 b» Elle s’éloigne de la Terre. 29. Elle s’éloigne à 1 x 10^ m/s.
32. a) 5.05 /xm b) 20.2 /xm 33. a) 3.33 «m h) 0, -'-10.2°. '-20.7°. 3 0 .0 . 13( 31. Jaune (550 nm) 32. a) 79 keV b) 3 ,11 MeV
-t32,2", ± 45,0', ±62,2° 34. À toutes les longueurs d ’onde plus c) 10,9 MeV 33. a) 0,062 5, 1,00! % b) 0,941,2.96
petites que 635 nm 35. 3 36. 2.4 /xm 37. a) 6,0 /xm b) 1,5 ixm c) 0,999 999 87. 1,96 x I0-’ 34. a) 0,998 8, 20,6 b) 0.145, 1,01
c) /« - 0 .1 ,2 ,3 , 5 ,6, 7,9 38. a) 3 b) n.f)51° 39.1.1 x I o ’ fentes/inm c) 0,073 l, 1,002 68 35. Ü ,W 987c .36. 8.12 MeV 37. 18 ums/a
Réponses aux sections R-5

38. a) 0,996 keV b) 1,055 MeV 39. a) 0,707c b) 1,41 c) 0,414 mc^ c) 0 3 1 1 M e V d) 938 M e V 44 . 2 ,6 5 f in 4 5 . 3 0 0 %

4(1. a) 0,943c b) 0.866c 41. •J^mc 42. c) 207 (la particule est 4 6 . a) 8.1 X 1 0 “ ’ % b) 4 ,9 x 1 0 “ ' * % c ) 8 . 9 % d ) 6 6 % e) P lu s la

un uiuon) 43. 1,01 X 10^ km, ou envij-on 250 fois la circonférence lo n g u e u r d ’o n d e est courte, p lu s il e.st fa c ile de m e su re r le d é p la c e ­

île la Terre 44. al 0,947 8c b) 226 MeV c) 314 McV/c 45.110 kjn m ent d e C o m p to n . 4 9 . a) 4 1,8 k e V b ) 8,2 k e V 5 0 . a ) 2 ,4 3 p m

46. a) 4,85 mm b) 15,9 mm c) 0,334 ns; non 47.4,00 ii ; il s'agit b ) 4 , 1 1 X I O - ‘’ c ) - 8 . 6 6 X 10 * * c V d ) 2 .43 p m , 9 , 7 9 x I C ' a

probablement d’un noyau d ’hélium 48.6.6 x lO^ m 49.0.33 T - 4 ,4 5 k e V 5 1 . 1 , 1 k e V 53 . À un an g le de 44"^ 5 6 . 1 . 7 x 10 ni

5«. a) 534 b) 0,999 998 25 c) 2,23 T 51. a) 2 MeV b) -1,21 MeV 5 7 . 7 , 7 5 p m 58 . a) 3 8 ,8 p m b ) 1,24 n m e) 9 0 6 fm 5 9 . 4 , 3 2 / ic V

52. a) 1.93 m bl.Cgz = 6.00 m. /^2 = 1.36 x lO'^' s c) I..36 x K)-" s 60 . a) 3 .9 6 X 10*’ m /s b ) 8 1,8 k V 61 . a) 3 8.8 m e V b ) 14 5 p m

d) 0,379 m e) = 3025 m. fv2 = “ 1,01 x 10~^ ,s f) Non 62. a) 3,3 X 1 0 " 2 i k g ■ in/s d a a s cha que ca.s b) 3 8 c V ptiur ré ic c iro n .

g) L’événement 2 i) Monsieur Voilure et monsieur Garage ont tous 6 .2 k e V p o u r le p h o to n 6 3 . a) P lio lo n : 1,24 um, électron : 1.23 n m

les deux raison. 53. a) vi sin 0h ) f[l — (i'/c)eos 0] c) 3,24c b ) 1,24 tm d a n s ch a q u e ca s 64. a) 7 3 p m b ) ,3,4 n m c ) O u i, le ur

lo n g u e u r d ’o n d e de de B r o g lie m o ye n m ; est très inférieure à

CHAPITRER le u r séparation m o y e n n e 65 . a) 1,91 x 1 0 " 2 l k g •m/s b) 3 4,7 fm

VÉRIFIEZVOS(OHNAISSANCES 1. b), a), d), c) 2. a) Lithium, sodium, 66. a) P h o to n : 1,24 keV , électron ; 1,50 e V b ) 1,24 G e V d a n s chaque

potassium, césium b) Elles sont toutes égales 3. a) Elles sont égales. c a s 67 . 2 .4 8 X 10 * n m ; la lo n g u e u r d ’o n d e esl e n v iro n 2(K) fo is

b) à d) Les rayons X 4. a) I x proton h) Les deux sont pareils. in fé rie u re au ra y o n nuclé aire 68. a) 5.2 fm b ) N o n , c a r la lo n g u e u r

c) 1 e proton 5. Elle est égale. OÜESIiONS 1. a) Les ondes du four d 'o n d e de D e B r o g lie est très in fé rie u re à la d istance d 'a p p ro c lie

(micro-ondes) b) Les rayons X c) Ix.s rayons X 2. Parce qu'ils m in im a le 69.11 s ’agit d ’un neutron. 70. a) 15 k e V b ) 120 k e V

émettent dans l'infrarouge 3. Elle est multipliée par 16 4. La sur­ c ) L e m ie i o sc o p e é le c tron iq u e car les é le c tro n s o n t m o in s b e so in

face de Bételgeuse est beaucoup plus grande que celle du Soleil. d ’é n e rg ie 71 . 9 ,7 0 k V (c a lc u l relativiste), 9 ,7 6 k V (c a lc u l n o n

5. a) Vraie b) Eausse c) Fausse d) Vraie e) Vraie f) Fausse re la tiviste ) 7 9. d ) x = n (K/2), o ù « = 0. 1. 2. 3,... 80 . a) N o n

6. Jx potassium 7. Le courant maximum 8. Comme des électrons b ) F ro n ts d ’o n d e s plane.s s ’étendant à l ’in fin i, p e rp e n d ic u la h e m c n t

ont été éjectés, la plaque devient chargée positivement. Cela crée à l ’a xe d e s x 81 . 0 ,1 9 m 82 . 2,1 x 10'"24k|; • fn/s 84. a) 1 2 4 k e V

un champ électrique vers la plaque. 9. e) Le matériau de la plaque b ) 4 0 ,5 k e V c ) O n ne peut p a s o b s e rv e r l'é le c tro n s a n s réjecter.

10. Aucune énergie 11. Dans le cas de la lumière visible, le chan­ 85 . a) f ’roton : 9 ,0 2 x 10 d euteron : 7 ,3 3 x 10 b) 3.0 M e V d a n s

gement relatif de la longueur d'onde est trop petit. 12. a) Elle est c h a q u e c a s c) 3,0 M e V d a n s ch a q u e c a s 8 6. 5,1 e V 8 7 . a) — 2 0 %

supérieure, b) Elle e,st inférieure. 13. a) B b) A c) A d) A b ) - 1 0 % C) + 1 5 % 88. a) lü '**^ a n s b ) 2 x 10 ” s ( L a m a s s e p lu s

14. Aucune différence 15. l.’électron 16. Elle est divisée par -Jl. petite de l ’é le c tron fait u n e é n o rm e d iffé re n ce .) 89 . T = 1 0 “ ^

b) Elle est divisée par 2. 17. Électron, neutron, particule alpha où X 7,2 X ( T a u n e tiè s petite valeur.)
18. a) Elle diminue, b) Elle augmente, c) Elle reste la même.
d) Elle reste la même. 1 9 .1x proton 20. L'action a) CHAPITRE 10
21. 1.’amplitude de l’onde réfléchie est plus petite que l'amplitude VÉRIFIEZVOS COMNAISSANCFS I. b, a, c 2. a) Il y a égalité entre les trois
de l'onde incidente. 22. a) Nul b) Oui 23. Les situations sont puits, b) a, b. c 3. a. b. c. d 4. £’i | S. a) 5 b ) / QUESTIONS
toutes égales. EXEROCES ETPROBIÈMES 1. 4 ,14 cV • fs 2. 2,1 ¡jm. 1. a) Par ^ b) Par le ineme facteur 2. a. c. b 3. a) Non b) Non i ) Oui
infrarouge 4.2,11 cV 5. LO x 10^'’ photons/s 4. a) 18 b) 17 5. a) ( ^ / l / i ) sin (;t/2 /)r h) ( \ /4 /7 ) sin (2n/7,)rc)
6. 1.7 X 10^’ photons/m^ ■s 7 . 5.9 fjeV 8. 8.6 x 10‘’m/s 9. 2.047 eV ( v /2 /Z ) cos (n/Z-)A 6. 12 eV (4 > 2 dans A) correspond à 1 >2
10. 3,6 X 1(1 >'?W 11. 4,7 X 10^6photons 12.3,3 v lO'^photons/s dan.s C , 9 cV (5 —» 4 dims A) uirrcsixmd à I - t 2 dnii.s /) ; 24 cV
13. a) Lri lampe à infrarouge b) 1,41 x lO^' photons/s 14. a) 3.61 kW (5 -> 1 dans A) correspond à 1 —» 3 dans D \ 15 eV (4 —» 1 dans A)
b) 1,00 X 10^ p lu rto n s/s c ) 60,2 s 15. a) 2,96 x 10^’ p h o to n s/s correspond à 1 dan.s E 7. Elle est inferieure. 8. Elle est égale
h) 48 ,5(X) km c) .5.80 x 10** photons/m^ • s 16. 91 K 17. 1,5 in ; à celle de rélc4.trnn. 9. a) En rélargis.vint b) En iuigmcnlant .sa
onde radio 18. a) 1,1 mm; micrtvondcs b) 9,'14 )iin. infrarouges profondeur 10. a) 3 b) 4 11. n Lu 2.n .3 12. a) Elle est
c) 1,6 fjm, infrarouges d) .500 nm ; visible e) 0.3 nm ; rayons X supérieure, b) Il c.st inférieur, c) Elle est inférieure. 13. b, c et d
19.0.580 W 20.0,192 W 21. 1.7 x 10** W/m- 22. a) Non b) 514 14. Il est identique 15. a) La première de Lynwn cl la première
lun, verte 23. Le barvuni et le lithium 24. a) Dans le cé,sium de Balmer h) La limite de la série de E ymmi e,i la limiie de la stj ie
b> Dans les deux cléments 25. 0.17)/m 26. 10 eV 2 7 .676kin/s de Pasehen 16. a) n » 3 b) m — I c) n = 5 EXERCIŒ5 ETPROBLÈMES
28. a) 1.30 V b) 680 km/s 29. a) 2,00 cV h) 0 eV c) 2,00 V d) 295 1. a) 37.7 eV h) 0,020 6 eV 2. tin dnti la imiliipliei pai \ / l
lun 30. 1.07 eV 31.233 nm 32. a) 6,60 x lO“ '’''J s b) 2.27 eV 3. 1.9 GeV 4. 0.85 nm 5. 0.0206 eV 6. 0.65 eV 7. 90 3 rV
c) 545 nm 33. a) 382 nm bi 1,82 cV 34. 6,7 x 10‘ « .l s h) 2,31 eV 8. a) n = 12 et n = |3 b) Il n’y a pas de paire ptissiblc. 10. a) n ~ 10
35. 9,68 X 10 20 A 36. a) 3.10 keV b) 14 keV .37. a) 2.73 pm et n = 11 b) Dans cc tas. il n'y a pus de paire possible. 1 1 .6B.7 nm.
b) 6,05 pm .18. 2,73 x 10 22 kg ■m/s = 0.511 MeV/c b) 2.43 pm 25.8 nm, 13.7 nm et 8.59 nm 12. a) 72,2 cV b) 68,7 nm (2 » 1) ;
ci 1.24 X I()20Hz 39. a) 8.57 x lO'* H? b) 35.5 keV 4 1 .2 n m (.3 2 ) ; 2 9 .1 ru n ( 4 - > 3 ) ; 2 5 ,8 n in (3 •-v 1 ) ; 1 /,2 n m

c) 1.89 X 10 23 kg m/s 35,4 keV/e 40. a) +4,86 pm b) - 4 1 keV (4 > 2 ) ; 13,7 n m ( 4 ■. i ) ; c ) T3,7 n n i .seulement ; JO.â nm ,

t ) 41 keV J) La même c|ue celle des rayons X incidents p u is 25 .8 n m ; 2 9.4 m n. pui.s 41.2 nm . p u is 6 8 .7 n m ; i l . ? nm .
42. a) 2,43 pm b) 4.86 pm c) 0,255 MeV 43. a) 2,43 pm h) | ,32 fm p u is 6 8 .7 nm . 13. a) 1.3 x 1 0 " '' ' c V b ) E n v ir o n 1,2 x lO *'-
R-6 Réponses aox sections

c) 0,% J 6 X 10*** eV d) Oui 14. a) Il la diniinik;. b) Il l’augmenie. c) 19,00(/!^/8ffiL2) 27. 42(/î^/8m L^) 28. a) 45(/i^/8fnL-)
15. b) .Non c) Non dj (Xii Ifi.aKXWI b) 0,091 c) 0.818 b) ‘VIQi^KmL^) c ) 30. S é lé n iu m : 4p. 4 éle c tro n s ;
4 8 (/ jV 8 m L ^ )

17. a) 0,050 b) 0,10 c) 0,009 5 18. b) m 2.5 i9 .5 9 e V 20. 280eV b ro m e ; 4p, 5 é le c tro n s; k r y p t o n : 4p. 6 é le ctron s 31. L ’a rg o n
21. b) k = (2Æ'/()f2ff!( f-o - £■)]''- 22. b) k = ±(2n/h)(2niE)'^ .32. n = l. / = 0. ntl = 0. ffi, = ± \ 33. a) (2. 0, 0. ± i )

24.0,73.5 eV 25. 3.09 eV 26. a) 1,25 b) 2,(K) c) 5.00 d) 1,00 b ) n = 2. / = I , ffif = 3 1 .0 , o u — 1, fHj = ± 5 34. a) 18 ( 3 6 si o u ne

27. 075; 1,00; 1.25; 1.75; 2.(X); 2,25; 3.00; .3.75 2«. a) 3,00 tient pa s co m p te de I ’asitccl distinct) b) 6, les états o ù le s d e u x éle c­

b) 9.00 c> 2.00 d) 3 c) 6 2 9 .1,00.2,00 ; 3,(X); 5,(X) ; 6.00 ; 8,00 ; tro ns partagent Ic.s n o m b re s quantiq ue s (n. I, m), m^) = (2, I, 1,

9.(H) 31. 2,6 eV 32. 1,17 cV 33.4 34. a) -3 .4 0 eV b) 3.40eV (2, 1, 1. - i ) . ( 2 , 1,0 . H ^ ) . ( 2 . 1 ,0 , - i ) . (2, 1. - I , -c^ ).

35. a) 12,1 cV b) 6,45 x 10 kp ■m/s c) 102 nni 36. a) 658 nm (2, I. - I, - ;^) 36. 12,4 k V 39. 4 9 ,6 pm . 9 9 .2 p m 40. a ) 5 ,7 k e V

b ) 366nm 38. a) 291 nni” ^ b) 10.2 nni~* 39. a) 0 b) 10.2 nni~* b ) 8 7 pm , 14 k e V , 2 2 0 pm . 5.7 k e V 42. a) 24 .8 p m b) c l c ) L e s

c) 5..S4 nm ' 40. a) 12,8 eV b) 12,8 eV (4 1). 2,55 cV (4 -♦ 2), v a le u rs reNteroni inchangée-s 43. a) 3 5.4 pm , c o m m e d.ans le Cît.s

0,66 cV (4 >.l), 12,1 e V ( 3 - ^ 1), 1,89 cV ( 3 ^ 2 ) . 10.2 cV d u m o ly b d è n e b ) 5 6 , 4 9 p m c ) 4 9 .6 4 p m 4 4 . 6 .4 2 keV 45. 9/16


(2 » I ) 41. a) 13.6 cV h) 3.40 eV 42.4.1 m/s 43. a) l.a transition 46. 2,2 keV 48. a ) 19.7 k e V , 17,7 k e V b) Z r o u N b 49. a) 6 9 .5 k V

;illant de « - 4 à « 2 b) À la séné de Balnier 44. a) 30.6 nm b) 17.8 p m c ) K „: 21,3 pm , K^: 18,5 p m 50. 80,3 piii
b) 292 nm c) 823 THz, 365 I Hi 45. a) 13,6 cV b) -2 7 .2 eV 51 . a ) (Z - 1 )2 / (Z ' - 1 )^ b ) 5 7 ,5 c ) 2 0 7 0 52. b ) 2 4 .8 % , 15 ,4 % ,

46. a) De n “= 2 à n •“ 1 b) A la série de I .yman 47. a) 2.6 cV 10.9 % . 7.9 % , 6.4 % . 4,7 % . 3-5 % . Z 6 % . 2,0 % . 1,5 %

b) Il s'agit de la liansition allant de n = 4 à n = 2. 49. 0,677 53. a) 6.0 b) 3,2 X lO** a 54. 1,3 x 10*** m o l 55. 9 ,0 x 1 0 "’

50. De n “ 3 à n “ I 52.0,439 54. a) n b) 2 i + l e ) rr 56. a) 2..5S ,s b) 0 .5 0 n s 57. 1,0 x HT* K- 58. - 2 , 7 5 x |0** K

.55 a) 000368 b) 0.00541 56. n « 4 348 58 c) (r^/8r7^X2 - rla^e 59. a) 3 .6 0 m m b ) 5 ,2 4 x 10*^ 60. 7,3 X 1 0 *“* s ~ * 6 1 . 4 .7 km

59. a) I 1^ = {A32nu\->-'^ cos^ « . | \Jf._n i \~ I - 62. 2,1 X lO’ 63- 2.0 X I 0 ‘*’ s - ' 64. 1,8 p m 65. a) 3 ,0 3 x 105
sirr fl b) 1,43 GHz d ) 3,31 X 1 0 " ^ 6 6. a) A p p ro x im a t iv e m e n t au c u n
h ) 6 8 .f 67. a ) N o n b ) 1 4 0 n m 68. a) 7 ,3 3 p m b) 7,0 7 x K y ’ W /m ^

CHAPITRE 11 c) 2,49 X I0‘0 W/m- 69, a) 4,29 //m b) 10.0 pm c) Dans le


ITRIFIE? VOSfOHNAWAHCFS 1.7 2. a) Elle diminue, b) etc) Klle reste domaine des infrarouges 7 0. a) 6,9 p e \ b) Dans le demainc
la iiieiiic 3. Elle c.si inléricuie. 4. 4, C B OUESTIONS 1.0, 2 et 3 des ondes radio
2. Fîllc.s en ont le même noinhre, (10). .3. 6p 4. 1.0 , I et 2
9. a) 2,8 b) 5, 50 6. a) Brtm e h) Rubidium c) Hydrogène 7. a) n CHAPITRE 12
b) « et / 8. Ils sont tous viais. 9. a), e), e), 1) 10. a) Le rubidium VÉRIFIEZVOS CONNAISSANCES 1. '*'’As et '-'»Nd 2. e i c sera un peu
b) I .e kj ypton 11. a) Elle restera inchangée b) Elle diminuera. supérieure à 75 Bq (le temps éct'ulé représente un peu moins que
c) Elle diminuera. 12. a) 2 h) 3 13. a) et b) 14. En plus de l’énergie trois demi-vies). 3 . ^ P b QUESTIONS 1. Il s’immobilisera plus p r«
quuntitiéc, un atome d ’hélium possède de l’énergie cinétique; du centre du noyau. 2. Il ctHitieni plus de protons que de neutrons.
son éiieigie totale peur être égale à 20,66 eV. 3. 24ü L ' 4. A u - d e s s u s 5. Il est inferieur. 6. a) D a n s la case

fXERnCESFl PROBlÈMtS 2. a) 14 b) 6 c) 6 d) 2 3. a ) 3 b) 3 h) N o n 7. a) S u r la dro ite /V - Z b ) Il ém ettrait d e s p o s itro n s


c) E n v ir o n 12 0 8. a) E n d e s s o u s b ) E n d e s s o u s c ) Il s so n t r a d io ­
4. .a) .3,653 X 10“ '^ J s b) 3,165 x 10 ' s 5. a) .12 b) 2 c) 18
d) 8 6 , fl 4 ; / = .1 ; fft/ = "l■3, “‘■2. + 1. 0. “ E 2. —.1, ffig actifs. 9. N o n 10. O u i I I . Oui 12. a ) /1 et C s f 'M é gaux, p u is vient/?,

b) B. p u is vie nne nt A et qui sont égaux. 13. a) L 'a c t iv it é augiixintc.


7.24.1“ 8. ( - 4 . « > 5 , / « , ± 4 9. « . >3,
n q — +3. +2. ‘ I.O. 1, 2. —3.//J, •“ + j . 10. .50 h ) L a constante de d é sin té g ra tio n reste la m êm e. 1 4 .11 n ’a aucun

11. a) s/l7/) b) v T 2/f|3 rapport a v e c le s e c o n d no ya u . 15. 7 h 1 6 . ’ '- A s 17. d)


fl 18. a) Ttxis, sa u l '* ’**Au b ) ” ’ .Sn et -**“ P b
c) ntl l . Mmh. Z
15(f EXERCICFS FI PROBLÈMES 1. 28..3 M e V 2. 15.8 fm 3. a) 0 .3 9 0 M e V
-.3 -3 /i
—7 125“ h) 4,61 M e V 4. 2 7 6. n k m 7. a) S i x b ) H u it 8. a) ' “‘- N d . ' ' ‘^N d .
- 2 fi
lo r i + N d . ' ‘**’N d , •’’« N d b ) **’ R b . ’ “^ S r.‘*^Y. *°**Zr. *'**N h , '"'*T c ,
1 -h
” '*Sn. " * S b , " ’ Te, " ’ l, " ’’X e . " ^ C s e t " ' ’B a c l ^ O z n ,
0 0 0 90.0”
73,2" ^**Cu. *’**Ni. *’**Co, ****Fc. ****Mn. ^ * C r et 10. a) T.'yttriiim et l'iix ic
+1 +ti -itn
.54,7“ b ) .50 d a n s l ’y ttriu m c l 7 4 d a n s l ’icxle c ) 19 11. a) 1,15 G e V b ) 4,81
1-2 -1 2h 2/' b
30,0'’ M e V / n u c Ié o n . 12.2 M e V / p r o t o n 12. a) 2.3 x l O " k g / m ' d a n s
1-3 + Vi -3/fB
c h a c u n d e s c a s b ) 1,0 v 1()25 r / m * d a n s le c a s d u 8,8 x
12. a) 3 b) 5 c ) .2d) 18 0 3 14.a) 3 X 10’‘*l0 6 X lO^“*
Ù 6 x ]0 -* ’*rad 1.5. 5 4 r et 12.5.3’ 16 a) 5 8 /jeV b ) 14 C /m ^ d a n s le c a s d u ^****Bi 14. b) 0 , 0 5 4 % , 0 , 5 0 % , (1.81 , 0 ,8 3 %.

L 1Z.I cm. au donianie des ondes n d io 17. 72,1 km/s- 0 , 8 1 % . 0 ,7 8 % , 0 ,7 4 « ^ . 0 . 7 2 % , 0 . 7 1 % 15. a) 6 , 18 fm b ) O u i

18. a) 1.5 X 10‘ N b) 20 /fill 19. -i.36cm 20. 51 mT 16.4 X 10 ■^’ s 17. A ît .30 \ J e V 18. L O O ilO flO u ( l, 9 0 6 8 3 ii,

21. a) 2,13 meV b) 18 T 22. 19 m l 23. 44(/f’/8m/ ’) 2 3 6 ,2 0 2 5 U 20. a) 19,8 M e V , 6 ,2 6 M e V . 2 .2 4 M e V b ) 28..3 M e V

24. 17.25(/i^/8ffiI/.2) 25. a) -51{h^Kml.-) b) 5'^(h-lV,mL'\ e) 7.0 7 M e V 21. *’'* M g : 7 8 ,9 9 9 6 % , 2‘^ M g 9..303 % , ^'’M g : 11,71 %

c) ü M M in lP ) 26. a) 18,00i/i’/8mL 2) h) 18,25(/)2/8ff7/’) 22. a) ^ 7 ,2 9 M e V b) 1-8.07 M e V e ) 9 1 .1 0 M e V


Réponses ayx sections R-7

23. 1,6 X 10^5 M e V 2 4 . 1 , 0 0 8 6 6 3 6 u 25 . 7 ,9 2 M e V 2 6. a) l b) i 23. a) 8 4 k g b) 1.7 X I t P c) 1.3 x lO^^ 2 4 . 1 .6 x lO'^* 25. 0 , 9 9 9 38


27. 2 8 0 j 28. 3,0 x lO''-* 2 9 . a; 7,5 3 x 10*^ B q h ) 4,91 x l O “ * B q 2 6 .8 030 G W 27. b) 1,0 ; 0,8 9 ; 0 ,28 ; 0 ,0 1 9 e) 8 28. D y a 3,6 x lO'-'a.
3 0 . a) 4,8 x lO “ '*^ B q h) 4 ,6 x lo'^ a 3 1. a) 6 4 ,2 h b ) 0 .1 2 5 29. a) 75 k W b ) 5,8 x 10^ k g 31. I l > a 1,7 x 10^ a 3 Z a) 3 0 M e V
c ) 0 , 0 7 4 9 32 . a) 5 ,0 4 x 10'** b ) 4 .5 9 x !(/> B q 3 3 . 5 ,3 3 x 1Ü-- b) 5 M e V 33. 17 0 k e V 3 5. aj 170 k V 3 6 . 1,4 M e V 37.0 .1 5 1
34. 2 6 5 m g 35 . a) 2 ,0 3 x 10^0 b) 2 ,79 x 10‘' B q 3 6 . a) 59 ,5 j 40. a) 3,1 X 1 0 " proton.s/m^ b ) 1,2 x 1 0 * fo is p lu s g r a n d
b) 1.18 . 3 7 .2 0 9 J . 3 8 . 8 7 . 9 m g 3 9 . 1 , 1 3 x 1 0 " a 4 2 . 0 . 6 5 8 g 4 Z a) 4.3 X 10’ kg/s b ) 3 ,07 x 10 ^ 4.V a) 4 ,0 x 10=^ M e V
4 3 . a) 8 .8 8 x 1 0 " 'B q b l 8 , 8 8 x 1 0 “ * B q c ) 1,19 x 1 0 '^ d ) 0.111 |jLg b ) 5 . 1 X 10^* M e V 44 . a) 1.83 x lO'*** s ' b ) 8 .23 x 10^* s '
44. a) 3 ,6 6 X lO'^ B q b) / » 3 ,g 2 j c) 3 ,6 6 X l o ’ B q d ) 6 ,4 2 n g 46 . a) 4.1 c V / a to n w b) 9 . 0 M J / k g c) 1,5 x 10^ u 4 7 . 5 x 10’ u
45.0,07.3 tx? 47. P u : 3 , 1 x 10 ''', C m : e~'^ 0 48. a) 4.Z5 M e V 48. 1.6 X 1 0 * a 49. a) 6.3 x 1 0 ''’ J/kg b ) 6 ,2 x 1 0 " kg/s
b) - 2 4 . ) M e V c ) 28.3 M e V 49 . 4 .2 6 9 - M e V 50. Q i = - 9 , 5 0 M o V . e) 4,3 X 10’ kg/s d ) 15 x lO ’ a 50. a) 2 4 .9 M e V h) 8.65 M l de T N T
t>4 = 4 ,6 6 M c V , g , - - 1,30 M e V 51. a) 3 1 ,8 M c V , 5 ,9 8 M r V 52. 14.4 k W 53 . A '„ - 3 J i4 1 M c V ; K„ =■ 14,05 M c V
b) 8 6 M c V 5 2 . b ) 4/1 + 3. 4«. 4 « - 2. 4 « + 3, 4«. 4/i + I. 4/i 2.

4/1 *■ !. 4 « 1 53. ’ L i 54 . aj 0 .9 0 p m b ) 6 .4 fm c) N o n d) O u i CHAPITRE 14


55. 1,21 M e V 5 7 . 0 . 7 8 3 M c V 58 . 0 , 6 0 0 M e V 59. b ) 0,961 M e V VÉRIFIEZ VOS COflNAISSANfES 1, a) À la l'ainillc de s m u o n s b ) U n e j>anu ule
60. b ) 2.7 X I 0 ‘3 W 6 1 . 7 8 ,3 e V 62. 1,61 x lO ’ a c) = 4 I 2. Le.s p rin c ip e s b) et e ) 3. c) QUESTIONS 1. d )
6.3. a) U ; 1.06 X 1 0 '^ P b : 0 , 6 2 4 x 1 0 ''^ b ) 1,68 x I0 > ’ 2. Il y entre 3. L e p io n tt'*'. dont la trajectoire est in c u rv é e
c ) 2 .9 7 X 10'^ a 6 4 . 13 2 |jig 6 5 . 1 . 7 m g 66 . 145 R q = 3 ,9 2 n C i v e rs le bas. ù g a u ch e 4. b), t ) et d ) 5. a), h), c ) et d ) 6. f i 7. c ) c i H
67. 1,02 m g 68 . 7,3 m .Sv 69 . 0 .0 1 3 .1 70. a ) 18 m J 8, c) 9. I et d), 2 et e). 3 et a). 4 et b). 5 et c ) 10. a) D ’u n Icpton

b ) 2,9 m S v = 0 . 2 9 rem 7 1 . a) 6.3 x I 0 ‘® b ) 2,5 x 1 0 " c ) 0 ,2 0 J b ) D ’une antipa rticule c ) D ’u n fe rm io n d ) O u i 11. I et h). 2 et c).

d ) 2.3 m G y e ) 3 0 r n S v 72 . a) 6 .6 M c V b i N o n 73 . a) '*< ), 3 et d). 4 et e). 5 et a) I Z b), f). c). d), a), g), e) 13. a) 0 b ) + 1 c) - I
‘■ »^Mo. l'^ S m , «>’ P b b> ‘‘» K . ‘^^Zr. '^ 'S b . '^ ^ N d c) '^ C , d) +1 c )-l REROOSETPRCiliMES 1.6.03 x lo-»Kg 2.2.4 x in-»’
20VJ-) 207pj, 74 , a) 2 5.4 M c V b ) 12,8 M c V c ) 2 5 .0 M e V 7.5. b) 1,00 3. 18.4 fm 4. rr -> p P 5. 1,08 x lO ’^^.I 6. I désinlcgraticin/a

c ) 7 0 .8 d ) 0 . 0 1 0 0 e ) 0 .7 0 8 f) N o n 7 6 . 0 . 4 9 7 7 . 9 , 0 x 10^ B q 7. D e 2.7 cm /s 8. 3 1 n m 9 . 7 6 9 M e V 10. a ) 1,90 x 10 “ k g ■ m/s

78. a ) D é sin té g ra tio n béta m o in s b> 8.2 x 10^ B q c ) 1,2 x 10^ b ) 9 .9 0 m 13. a) L * et le p rin c ip e de c o n s e rv a tio n d u m o m e n t

7 9 .a ) 7 x I0 ’ b ) ( 7 x o ù D e s t l ’année cin é tiq u e b ) et le p rin cip e de c o n s e rv a t io n de la c h a rg e

actuelle 80. 3,2 X 10*^ B q 8 6 C i 81 . 82 . 4 ,2 8 x 10^ a c ) L e p rin c ip e de c o n s e rv a t io n de l ’é n e rg ie et /.^

8 3. 1 0 '^ a to m e s 84 . 1,3 x 10 m 85. 7.31 M c V 86 . 3,2 x !0 ^ a 14. a ) 2 c ' 4 e + 5u + 4ï> h) U n b o so n , un m e so n , fi = 0


15. ^ — 0. /? - I , .S 0 16. b), d ) 17. a) L e p rin cip e
8 7 . 4 . 9 X 1 0 '^ B q
d e c o n s e rv a t io n de l'é n e r g ie b ) L e p rin c ip e de c o n s e rv a tio n
de l ’ctra ngcté c ) L e p rin c ip e de c o n se rv a tio n de la c h iu g c
CHAPITRE 13
VtRiflEZVCRfnHNAlSSANlIS l. c ) e f d ) Z e ) QUESTIONS l. a j Z a ) 18. a) 6 0 5 M e V b ) - 1 8 1 M c V 19. 3 3 8 M c V 2 1 . a) K + b ) î i c ) K ^ ’

3. b) 4. b), c). a), t ) et d ) 5. a) ''•’ S r b ) ' ’*‘1 c) '^ ^ N d 6. c ) 7. c ) 22. a) 3 7,7 M e V h ) 5.3 5 V l c V c ) 3 2,4 M e V ' 23 . a) ïïïïd b) ïïd d

8. c), a), cl) Cl b ) 9. a) 10. d ) U - f ) 1 2 - c) 24. a) n b l 2 ■■ C) 5 “ 25. a) G e la est im p o ssib le , b) uu u


26. a) iid s h ) iis s 29 . S * ’, 7.51 x ICTm /s 30. 1.6 x 1 0 '" a l
B B c n s n im ittiB '•*’ “ 1 ’', " ^ 3 1 . 6 6 6 . 4 n m 3 2. 3.13 X 10*' a I 33. b ) 4 .5 ato m es d ’H/m ^
C ) 2 . f t 0 x K l/ a 2 . l '3 r r a '’e ^ - X ; V; T e ; .1 3.2 .1 x 10 s

4 .4 .5 î x I0 » M oV 6 . . « « № V / . 2 3 .0 M .V 34 . b) 0 ,9 3 4 c) 1.5 X 1 0 " ’ a 1 3 5. o ) 2 5 6 p c V b ) 4,84 m m

36 . 102/V4 3 7 . a) 121 m / s b ) 2 4 8 a 38. K ) ’ .m


« ... 16f o i » , f »■ " ‘ ' " X " - ”' ' ,
I n V v 6 0 M e V p o u r le '^ 'N d c ) 1.60 x l O 'i n / s 39. b) 2 ,3 9 X i n " K 40. a» À 2.6 K b) 2 9 0 nm
h l 1 1 0 M e V n o iir le 6iC. 1 X 2 t
n jim .v ie v p o p , - . „ / . d a n s l e c a s d u ' ‘" N d 12. a ) 10 4 1 . a) 0 .7 8 5 c h) 0 ,9 9 3 c r ) G 2 d ) C I e) 51 n s f) 4 0 ns
d a n sle c a s d u « ^ G e . .
V -»« 1 M c V b ) L c n c rg ie d e fi.ssion ty p iq u e est de 42 . 1 e( A, 2 et J, 3 et /, 4 et I] .5 et G ' 6 et C, 7 et 11, 8 et D, 9 et L
b) 2 2 6 M c V I 3 . a ) 2 -'Î 1 M C /- * ^
. o s a . b) 0 7 o c) - 3 6 % 1.5. 4 6 2 k g 16. 5 7 7 k g 43 . c l r<ï/c f (iii/< Ÿ C/m/i ) ' 4 ... d ) la/i c ) tr //
2 0 0 M e V 1 4 . a) ■ • '2 . 'X U/
¿ t x / M c V . 14. .1/ 20. a) 1,2 M e V h ) \ 2 k g f) 7 ,4 ^6 10® a 1 g ) 7,8 '• 10® a h) 7 4 > 10^ a i) 7 8 x 10® a 1

j) 1,15 X l O ’ a l k ) 1,2 X 10 ’ a 114.1 X 10® a l 44 . 13 ' lü ’ a


INDEX

A lu m iè re d ’u n la s e r e t, 321 é le c triq u e in d u it, e t o n d e s é le c tro ­


A b e rra tio n m o m e n t c in é tiq u e d ’u n . 3 0 6 -3 0 8 m a g n é tiq u e s . 8 5 -8 8
c h ro m a tiq u e , 134 m o m e n ts d ip o la ire s m a g n é tiq u e s q u a d ru p o la ire , 4 1 0 (p ro b lè m p 4 1 )
d e s p h é ric ité , 134 d ’u n , 3 0 6 -3 0 8 C h a u v e s s o u ris , n a v ig a tio n , 7 2
A b o n d a n c e d e s is o to p e s , 3 3 5 n u m é ro ta tio n d e s é lé m e n ts e t, C h lo re . 3 1 7
A b s o rp tio n , 3 2 2 3 1 7 -3 2 1 C h ro m o d y n a m iq u e q u a n tiq u e , 4 0 1
A c c é lé ra tio n p ro p rié té s d e s, 3 0 3 -3 0 5 . Voir a u ssi C o e ffic ie n t d e tra n s m is s io n , 2 6 6
en c h u te lib re É le c tro n (s ) C o h é re n c e d e s o n d e s lu m in e u s e s ,
m e s u ré e p a r u n p e n d u le , 12 ré s o n a n c e m a g n é tiq u e e t, 3 1 1 -3 1 2 1 5 4 -1 5 5
m iix im a le , 4 sp in d e l’é le c tro n d ’u n , 3 0 5 -3 0 6 C o llim a te u r, 189
A c c o m o d a tio n , 124 ta b le a u p é rio d iq u e e t, 3 1 6 -3 1 8 C o m p to n . A rth u r, 2,‘ï4
A c tiv ité ra d io a c tiv e , 3 4 2 A u x lim ite s , c o n d itio n s , 2 9 5 C o n d e n s â t d e B o se E in s te m , 3 8 8
A m p litu d e A x e o p tiq u e , 1 1 0 C o n d itio n d c B ra g g . 195
d e d é p la c e m e n t, 5 8 C ô n e d e M a c h . 73
d e la p o s itio n a n g u la ire , 11 C o n fin e m e n t in e rtie l, 3 7 9
d e p re s s io n , 5 8 B C o n fin e m e n t m a g n é tiq u e , 3 7 9
d u m o u v e m e n t h a rm o n iq u e , 3 B a n d e s s o m b re s , 1 5 1 -1 5 2 C 'o n serv a tio n
d ’u n e o n d e , 2 9 B a n g s u p e rs o n iq u e , 7 3 d e l ’é tra n g e té , 3 9 6
A n d e rs o n , C a ri, 3 8 9 B a rriè re d c l ’é n e rg ie . 2 3 1 -2 3 2
A n g le c o u lo m b ie n n e , 3 7 4 d u n o m b re b a ry o n iq u e , 3 9 4
c ritiq u e , 9 7 d ’é n e rg ie p o te n tie lle . 2 6 5 d u n o m b re le p to n iq u e , 3 9 3 -3 9 4
de p hase, 3 d e p o te n tie l, 3 6 7 C o n s ta n te
ré s o lu tio n d e p ro b lè m e s a v e c , 5 B a ry o n (s ), 3 8 9 d ’a m o rtis s e m e n t, 15
d e ré fle x io n , d 'o n d e s é le c tro ­ q u a rk s e t, 3 9 8 d e d é s in té g ra tio n . 341
m a g n é tiq u e s , 9 3 -9 4 B a tte m e n ts , e t o n d e s s o n o re s , 6 7 -6 8 d e H u b b le . 4 0 3
d e ré fra c tio n , d ’o n d e s é le c tro ­ B e c q u e re l (u n ité ), 3 4 2 de phase, 3, 38
m a g n é tiq u e s , 9 3 -9 4 B e c q u e re l, H e n ri. 3 4 2 d e P la n c k , 2 4 7
d ’in c id e n c e , d 'o n d e s é le c tro ­ B ig B a n g , th é o rie d u , 4 0 5 -4 0 6 d e to rs io n , 9
m a g n é tiq u e s , 9 3 - 9 4 ,9 7 B in n ig , G e rd , 2 6 6 C oordonnées d 'csp a cc tem ps
A n n ih ila tio n p ro to n -a n tip ro to n , 3 9 0 B o h r, m o d è le d e , 2 9 0 -2 9 2 d e l ’c v c n c m c n t. 2 1 2 -2 1 3
A n te n n e , 8 2 B o h r, N ie ls , 2 8 1 .3 5 1 , 3 6 6 C o ra il. 2 8 /
A n tin e u trin o , 3 4 6 B o h r, ra y o n d e , 2 8 3 q iia m iq u e , 2 7 4 , 2 8 6
A n tip a rtic u le , 3 8 9 B o îte , 2 8 7 re c ia n p iila ire , 313
A rc -c n -c ie l re c ta n g u la ire , 3 1 3 C o rre s p o n d a n c e , p rin c i|ie d e , 281
d e M a x w e ll, 81 R o s e , S a ty e n d ra N a th , 3 8 7 C o u c h e s , d 'u n a to m e , 2 % . Voir a u ssi
fo rm a tio n d ’u n , 9 5 B osons, 387 N iv e a u d ’é n e rg ie , d e s é le c tro n s
p rim a ire , 100 B ra g g , W illia m L a w re n c e , 195 C o u ra n t p h o to é le c triq u e , 2 5 1
s e c o n d a ire , 100 C o w a n . C . L ., 3 4 7
A to m c (s ). Voir a u ssi A to m e (s ) C’rayons d e c o m b u s tib ilité , ,)69
d 'h y d ro g è n e C C ritè re
c o m b in a is o n d u sp in e t d u m o m e n t C’a p tu re d ’é le c tro n , 3 4 6 d e L aw son, 379
c in é tiq u e o rb ita l e t, 3 0 8 C e n tre d 'n s c illa tio n , 12 d c R a y le ig h . 183
d ’h y d ro g è n e , 2 8 8 C e rc le d c ré fé re n c e , 14
é ta ls d ’é n e rg ie e t, 2 8 8 2 8 9 C e re n k o v , d é te c te u rs , 4 1 0
fo n c tio n d ’o n d e d e l ’é ta t (p ro h lc m e 41 ) D
fo n d a m e n ta l d e Г , 2 9 3 C h a d w ic k , J a m e s , 3 6 3 .4 0 1 D atatio n p ar rad io a ctiv ité , 349
m o d è le d e B o h r e t, 2 9 0 -3 0 1 C h a în e p ro to n -p ro to n (p -p ), 3 7 6 -3 7 8 D a v isso n , C . J .. 2 5 8
n o m b re s q u a n tiq iie s d e Г , 2 9 3 C ham p D e B ro g lte , L o u is, 258
s p e c tre d e 1’, 2 8 9 d e v is io n , 13 4 D e H aas W J , 304
1-1
1-2 Index

D é c ib e ls , 6 2 E d ’io n is a tio n , 3 0 3
D e m i-v ie , d ’u n ra d io n u c lé id e , 3 4 2 É c h e lle d e s d é c ib e ls , 6 2 -6 3 p o te n tie lle é la s tiq u e
D e n sité E ffe t C o m p to n , 2 5 4 d ’u n e o n d e s in u s o ïd a le d a n s
d e p ro b a b ilité , 2 6 2 E ffe t D o p p le r, 6 9 u n e c o rd e , 3 6
d é te rm in e r la , 2 6 3 a s tro n o m iq u e , 2 2 7 -2 2 8 d ’n n .sy stèm e,
d 'u n é le c tro n . 2 8 0 -2 8 1 é q u a tio n g é n é ra le d e 1’, 6 9 , 71 to ta le , 2 3 2
ra d ia le , 2 9 4 n a v ig a tio n d e s c h a u v e s -s o u ris e t, 7 2 d a n s le m o u v e m e n t h a rm o n iq u e
D c p h a sa ^ 'c s à la ré fle x io n . 1 5 9 -1 6 0 p o u r fa ib le v ite s s e , 2 2 7 s im p le , 7 -8
D é p la c e m e n t p o u r la lu m iè re , et re la tiv ité , 2 2 7 d e l’é ta t fo n d a m e n ta l, 2 8 2
d e C o tn p to n , 2 5 4 so u rc e s o n o re im m o b ile e t, 7 0 d e lia is o n n u c lé a ire , 3 3 8 -3 3 9
v e rs le ro u g e c o s m iq u e , 411 so u rc e so n o re q u i sc d é p la c e e t. re la tiv ité e t, 2 3 1 -2 3 5
(p ro b lè m e 4 3 ) 6 9 ,7 1 to ta le , d ’u n s y s tè m e , 2 3 2 -2 3 3
D é s in lé g ra lio n tra n s v e rs a l, 2 2 8 -2 2 9 É q u a tio n
b ê la , 3 4 6 -3 4 8 E ffe t d e la tra n s fo rm a tio n d e G a lilé e , 2 2 2
d e s ra d io n u c lé id e s , 3 3 5 p h o to é le c triq u e , 2 5 0 -2 5 2 d e la tra n s fo rm a tio n d e L o re n tz ,
d u m u o n , 391 tu n n e l. 2 6 5 -2 6 7 2 2 2 -2 2 3 , 2 2 4
d u p io n . 391 F a n ste in , A lb e rt, 2 1 0 . 3 8 7 d e n o rm a lis a tio n . 281
ra d io a c tiv e , 34(1-344 g ra n d e s th é o rie s u n ifié e s , 4 0 2 d e S h rö d in g e r, 2 6 2 2 6 3
D é te c te u rs Ccrcnko■\^ 4 1 0 F in s te in -d e H aa.s, e x p é rie n c e d ’, d e s d io p tre s s p h é riq u e s , 1 3 5 -1 3 6
(p ro b lè m e 41 ) 3 0 4 -3 0 5 d e s le n tille s m in c e s , 1 3 6
D e u x iè m e h a im o u iq u e , 6 5 É le c tro d y n a m iq u e q u a n liq u e . 4 0 0 .4 0 1 d e s m iro irs s p h é riq u e s , 135
D e u x ie m e lo i d e IN cw ton É lc c tro m a g n c tis m c . 81 p h o to é le c triq u e . 2 5 2
d é riv a tio n d e la v ite sse d ’u n e o n d e É lectro n (.s) É q u ilib re é n e rg é tiq u e , 3 7 4
e t, 4 4 -4 5 c o u c h e s d e s a to m e s . 2 9 5 -2 9 7 E sa k i, L e o , 2 6 6
D ic is, J e a n ( l a n d e , 2 5 8 d e v a le n c e , 3 0 6 E s p a c e -te m p s , c o m m e n c e m e n t d e I’,
D iffé re n c e d e p a rc o u rs , 6 0 n iv e a u d ’é n e rg ie d e s , 2 7 6 -2 7 9 4 0 5 -4 0 6
D iffé re n c e d e p h a s e . 5 d a n s d e s p iè g e s re c ta n g u la ire s , É tra n g e té , 3 9 6
313-315 E vénem ent
d iffé re n c e d e m a rc h e e t, 1 5 2 c o o rd o n n é e s d ’u n , 2 1 2 -2 1 3
e t ré fle x io n d e s o n d e s , 159 p a rtie lle m e n t o c c u p é , 3 1 4
re m p li, 3 1 4 d e p a rtic u le , 3 8 9 -3 9 2
e t ty p e s d ’in te rfé re n c e , 4 0 s im u lta n é s , 2 1 3 2 1 5 . 2 2 4
D iffra c tio n to ta le s . 3 1 4
p ié g é s , 3 1 3 -3 1 5 re la tiv ité e t, 2 1 3 -2 1 5
d e F re s n e l. 175 176 tra n s fo rm a tio n d e I .o re n tz e t, 2 2 4
d e s o n d e s liim in e u s e s , 150, 175 é n e rg ie s d e s , 276-28.3
fo n c tio n s d ’o n d e d e s , 2 8 0 -2 8 3 E x c è s d e m a s s e , 3 4 7 -3 4 8
fe n te s m u ltip le s , 187 193 p rin c ip e d 'e x c lu s io n d e P a u li, 3 1 3 E x p é rie n c e d e Y o u n g s u r l ’in te r­
in te n s ité p io d u itc p a r u n e .seule p ro p rié té s tie s a to m e s e t, 3 0 3 -3 0 5 fé re n c e , 150-1.54
fe n te . 1 7 8 -1 8 2 c o m b in a i.so n d u s p in e t d u
prtK luite p a r d e u x fe n te s, 185 m o m e n t c in é tiq u e o rb ita l e t, 3 0 8
p ro d u ite p a r u n e o u v e rtu re m o m e n t c in é tiq u e e t, 3 0 6 -3 0 8 E
c irc u la ire . 182 m o m e n ts d ip o la ire s m a g n é tiq u e s F a c te u r
rc sc a u x d e , 1 8 7 -1 9 3 . Vnh a itw i e t, 3 0 6 -3 0 8 d ’in te rfé re n c e , 186
R é se a u x d e d iffra c tio n s p in d e F . 3 0 5 d e d iffra c tio n . 186
d e s ra y o n s X . 1 9 3 -1 9 5 É lé m e n ts d e L o re n tz . 2 1 7
D ila ta tio n d u te m p s , 2 1 6 . 2 2 4 p é rio d e s d e s , 3 0 4 F a is c e a u
t e s is d e la, 2 1 8 d ’o m ic s é le c tro m a g n é tiq u e s , 8 4
ra d io a c tifs , 3 3 3
tra n s fo rm a tio n d e L o re n tz e t, 2 2 4 ra y o n s X e t n u m é ro ta tio n d e s, lu m in e u x , 107
D io p tre s s p h é riq u e s , 1 1 4 -1 1 6 3 1 7 -3 2 1 F e m to m è tre , 3 3 7
D io p trie . 124 ta b le a u p é ritx liq iic , .316-317 F er. 3 1 7
D iia c , P a u l, 3 0 6 , 3 8 7 , 3 8 9 F n iis s io n F e rm i (u n ité ), 3 3 7
D iv|3ercion c h ro m a tiq u e , d e lu m iè re s|X )n tan ée, 3 2 2 F e rm i, E n ric o , 3 6 3 ,3 7 2 , 3 8 6 , 3 8 7
b la n c h e . 9 4 -9 6 s tim u lé e , 3 2 2 -3 2 3 F ib re o p tiq u e . 9 8
D ista n c e fo c a le , d u m iro ir, I lO -l 1 1 É n e rg ie F ig u re d ’in te rfé re n c e . 1 5 1
D iv is e u r d e fa is c e a u , 1 6 4 165 a u re p o s , 2 3 1 -2 3 2 F is s io n n u c lé a ire . 3 3 8 , 3 3 9
D o tn a in e d e v is io n d is tin c te , 124 c in é tiq u e . 2 3 3 2 3 5 e t ré a c te u rs n u c lé a ire s , 3 6 8 3 7 2 ,
D o s e a b s o rb é e , d e ra y o n n e m e n ts . q u a n tité d e m o u v e m e n t e t, 2 3 4 -2 3 5 3 7 2 -3 7 4
9 0 - 9 1 .3 5 0 d e d é s in té g ra tio n , .344 m o d è le d e , 3 6 6 -3 6 8
D u ré e d e v ie m o y e n n e , d e s n id io - d e lia is o n m o y e n n e p a r n u c lé o n . p ro c e s s u s d e la . .36 4 -3 6 6
n iic lé id c s , .342 338-3.39 ré a c tio n s e n c h a în e e t, 3 6 8 -3 7 2
Index 1-3

F o n c tio n d ’o n d e . 2 6 ) H In te ra c tio n
d e s é le c tro n s , 2 8 0 -2 8 3 H a d ro n (s ), 3 8 8 -3 8 9 , 3 9 4 -3 9 5 é lc c m o fa ib lc , 401
n o rm a lis a tio n d e la, 2 8 2 H a h n , O tto , 3 6 4 é le c tro m a g n é tiq u e , d e s p a rtic u le s ,
F o rc e (s) H a rm o n iq u e s , d a n s u n e c o rd e , 4 6 4(X)-401
d ’a m o rtis s e m e n t, 15 H e rtz . H e in ric h , 81 fa ib le . 3 8 8 ,4 0 1
é le c tro m a g n é tiq u e H o rlo g e s fo rte , 401
d e s p a rtic u le s . 4 0 0 m a c ro s c o p iq u e s , 2 1 8 In te rfa c e , 9 3
m o u v e m e n t lia rm o n iq u e s im p le e t, m ic ro s c o p iq u e s , 2 1 8 In te rfé re n c e
5 -6 H u b b le , E d w in R . 4 0 3 d e s o n d e s s o n o re s . 5 9 6 0
n u c lé a ire , 3 3 9 H u b b le , lo i d e , 4 0 3 d e s o n d e s , 3 8 -4 0
F o rm u le d e s o p tic ie n s . 1 17 H u y g e n s . C h ris tia a n . 141 d e s tru c tiv e . 4 0 . 146
F o y er, d u m iro ir, 1 1 0 -1 11 H y p e rm e tro p ic , 127 128 d ’o n d e s lu m in e u s e s
F ra n g e (s ) c o h é re n c e e t, 1 5 4 -1 5 5
d a n s u n p a tro n d 'in te r f é r e n c e , 151 1 c o m b in a is o n d e p lu s d e d e u x
s o m b re s , 1 5 1 -1 5 2 Ig m tio n , 3 7 9 o n d e s e t, 158
F ré q u e n c c (s ) Im a g e rie p a r ré s o n a n c e m a g n é tiq u e d iffé re n c e d e p h a s e , e t d iffé re n c e
a n g u la ire (IR M ), 3 1 2 d e m a rc h e e t, 1 5 2 -1 5 4 , 1 5 4 -1 5 5
d ’u n m o u v e m e n t h a rm o n iq u e , 3 Im a g e s d iffra c tio n e t, 1 5 0 , 175.
d ’u n e o n d e , 3 0 fo rm é e s p a r la ré fle x io n , 10 7 Voir a u ssi D iffra c tio n ,
d e ré s o n a n c e . 4 5 g ra n d is s e m e n t tra n s v e rs a l p a r u n d e s o n d e s lu m in e u s e s
d e s c o rd e s , 6 5 m iro ir e t, 112, 114 e x p é rie n c e d e Y tx in g e t, 15 0 -1 5 4
d e se u il, 251 locali.scr d e s , e n fa is a n t d e s tra c é s in c o h é re n te s , 155
d e s o s c illa tio n s , 2 d e s ra y o n s p rin c ip a u x , 112 in d ic e d e ré fra c tio n e t, 147 148
d ’u n e o n d e , 2 8 -3 0 m iro irs p la n s , 1 0 8 -1 0 9 in te n s ité p ro d u ite p a r d e u x fe n te s,
g a m m e d e , p o u r le s in s tru m e n ts m in iirs s p h é riq u e s , 100-1 11, 135 15.5-1.58
lie m u s iq u e , 6 6 -6 7 fo rm é e s p a r la ré fra c tio n , 114 - 116 in te rfé ro m è tre d e M ic h e ls o u e t,
p ro p re . 2 2 7 é q u a tio n d e s d io p tre s s p h é riq u e s , 1 6 4 -1 6 5
F ro n ts d ’o n d e , 5 4 , 8 4 1 3 5 - 136 lo i d e la ré fra c tio n e t. 1 4 6 -1 4 8
F u llrè n e , 2 5 9 é q u a tio n d e s le n tille s m in c e s, lo n g u e u r d ’o n d e e t in d ic e d e
F u s io n . 3 3 9 , 3 7 4 1 3 6 - 137 ré fra c tio n e t, 1 4 8 -1 4 9
p a r la se r, 3 7 9 g ra n d is s e m e n t tra n s v e rs a l p ro d u it p e llic u le s m in c e s e t, 1 5 8 -1 6 4
th e rm o n u c lé a ire , 3 7 4 -3 7 5 p a r d e s le n tille s . 1 14. 120 e x p é rie n c e d e Y ttu n g s u r 1’,
c o n trô lé e , 3 7 8 le n tille s e t, 1 1 6 -1 2 3 150-1.54
d a n s le S o le il e t a u tre s é to ile s . in s tru m e n ts d ’o p tiq u e , 1 3 0 -1 3 4 in te rm é d ia ire , 4 0
3 7 6 -3 7 8 ré e lle s , 107 p ro d u ite p a r le s p e llic u le s m in c e s ,
ré a c te u rs p o u r, 3 7 8 -3 7 9 ré flé c h ie s , 112 1.58-159
ré frîic tio n d e s , 115 d é p h a s a g e à la ré fle x io n et,
v irtu e lle s 159-161
ré flé c h ie s , 112 é q u a tio n s d e , 1 6 0 -1 6 2
G e ig e r, H a n s . 3 3 3 ré fra c té e s . 1 0 7 -1 0 8 . 115 irisa tio n d ’u n e .su rface e t, 162
G c rÎa c h . W a lth e r. 3 0 9 In d ic e d e ré fra c tio n , 9 4 . 1 4 7 -1 4 8 ré s o lu tio n d e p ro b lè m e s a v e c ,
G e rm e r, L . H ., 2 5 8 lo n g u e u r d ’o n d e e t, 1 4 8 -1 4 9 162
G ia e v e r, Iv ar, 2 6 6 In s tru m e n ts d ’o p tiq u e . 130 In te rfé ro m è tre d e M ic h e ls o u ,
G lc ll-M a n n , M u rra y , 3 9 6 , 3 9 7 , 3 9 8 lo u p e . 1 3 0 -1 3 2 1 6 4 -1 6 5
G lu o n (s ), 401 lu n e tte a s tro n o m iq u e , 1 3 3 -1 3 4 In te rv a lle d e te m p s p ro p re , 2 1 6
(io u d s m ith , .S am u el, 3 0 6 m ic ro s c o p e c o m p o s é , 132 In v e rsio n d e p o p u la tio n s . 3 2 4
G ra n d e s th é o rie s u n ifié e s , 4 0 2 In te n s ité Irisa tio n , d ’u n e s u rfa c e , 162
( ira n d is s e m e n t tra n sv e rsa l d e la fig u re d 'in te r f é r e n c e p ro d u ite Is o b a re s , 336>
p ro d u it p a r d e s le n tille s , e t im a g e s p a r d e u x fe n te s, 15.5-158 Is o to p e s . 3 3 5
fo rm é e s p a r la ré fra c tio n . 114, 120 d e s o n d e s é le c tro m a g n é tiq u e s , Is o to p e s d e n é o d y m e . 3 7 3
p ro d u it p a r d e s m in û rs , e t im a g e s 8 9 -9 0
fo rm é e s p a r la ré fle x io n . 1 1 2 , 114 d u n e .so u rce lu m in e u s e , el e ffe t
G ra p h iq u e d e M o s e le y . 3 2 0 - 3 2 1 p h o to é le c triq u e , 2 5 0 -2 5 2
G ra y (u n ité ), 3.50 lu m in e u s e d a n s u n e fig u re d e J a v a n . A li, 3 3 2
G ro s s is s e m e n t a n g u la ire d iffra c tio n p ro d u ite p a r u tie s e u le J e n s e n , H a n s, 3 5 2
p a r d e s in s tru m e n ts d ’o p tiq u e , fe n te , 1 7 8 -1 8 0 J e u d e p a rtic u le s , 411 (p ro h lè m c 4 2 )
1 3 0 -1 3 4 siin o re , 6 1 -6 3 J o s e p h s o n . B ria n , 2 6 6
14 Index

K d iffra c tio n de la , 175 M a g n é to n de B o h r, 3 0 7


K a o n , 386 d iffr a c tio n de Fi-esnel, 1 7 5 -1 7 6 M a ille é lé m e n ta ire , 1 93-19 4
p o s itif, 2 1 9 p ro d u ite p a r d e u x fe n te s, 185-187 M a rs d e n , E rn e s t, 333
K a o n p o s itif, 2 19 p ro d u ite p a r une o u v e rtu re M a ssiv e c o m p a c t lu ilo o b je c ts
c irc u la ire , 1 82-18 4 (M a ch o s), 40 5
p R K iu ite p a r une seule fente, M a tiè re , 38 9
L
1 7 8 -1 8 2 p a rtic u le s et, 2 6 0 -2 6 1 . Voir a i m i
L a i, M in g , 258 réseaux de, 187-19 3. Voir a u ssi P a rtic u le s
L a s c ifs ), 1 5 5 ,3 2 1 R éseaux de d iffra c tio n
a p p lic a tio n des, 322 s o m b re , 4 0 4 -4 0 5
d is p e rs io n c h ro m a tiq u e de la , 9 4 -9 6 M a x im a , 151
fo n c tio n n e m e n t des, 3 2 2 é m is s io n e t a b s o rp tio n p a r les
à h c liiim -n é o n , 3 2 3 -3 2 4 secondaires, 175
ato m e s, 304 M a x im u m c e n tra l de d iffra c tio n , 180
1 m l il le ts ). 116 en ta n t q n 'o n d e de p ro b a b ilité ,
c o n v e rg e n te , 116 M a x w e ll, Jam es C le rk , 81
256 259 M a y e r, M a ria , 352
d 'u n m ic ro s c o p e c o m p o s é , 132-13 3
in c o h é re n te , 155 M e itn e r, Li.sc, 364
d 'u n e lu n e tte a s tro n o m iq u e , 133-13 4
in te n s ité de la , 8 9 -9 0 M ésons, 389
d iv e rg e n te , 116
in te rfé re n c e de la quariks et. 3 9 9
e t o b je ts éte n d u s en tra ç a n t les co h é re n ce et, 1 5 4 -155 M é ta s ta b le s , états, 3 2 2
ra y o n s p rin c ip a u x . 120-121 c o m b in a is o n de p lu s de d e u x M é ta u x a lc a lin s , 317
g ra n d is s e m e n t tra n s v e rs a l d ’ im a g e s ondes e t, 158
p ro d u it p a r des, 114, 120 M è tre , d é fin itio n d u , 85
d iffé re n c e tie phase, e t d iffé re n c e M ic h e ls o n , A lb e r t A ., 164-165
in d ic e de ré fia c lin n el im age p ro d u ite
de m a rch e et, 1.52-154, 154-15 5 M ic h c ls o n . in te rfé ro m è tre de, 164-165
p a r une, 117
d iffra c tio n et, 150. M ic ro s c o p e à e ffe t tu n n e l, 2 6 6 -2 6 7
loupe. 1.30-132 Voir aussi D iffr a c tio n ,
m in ce s, 1 1 6 -1 1 7 M ic ro s c o p e co m p o se . 132 133
des ondes lu m in e u s e s e x p é rie n c e M illik a n , R o b e rt A ., 2 7 1
é q u a tio n des, 136-13 7
de Y o u n g s u r i ’ , 1.50-154 (p ro b lè m e 3 4 )
im age s p ro d u ite s p a r une, 119 in co h é re n te s , 155
ta b le a u des c a ra c té ris tiq u e s M in im a
in d ic e de ré fra c tio n et. 147-14 8 lo c a lis e r les, dans une d iffra c tio n
des im a g e s des, 121 in te n s ité p ro d u ite p a r d e u x fentes,
ré s o lu tio n d e p ro b lè m e avec les, p ro d u ite p a r une seule fe n te ,
155 -1 5 8 176-17 8
120 in te r lë ro m è tre de M ich e l.so n et,
systèm e d e d e u x , 122 M ir o ir ( s ) , 108
164 -1 6 5
L e p to n (s ), 3 8 8 -3 8 9 , 3 9 2 -3 9 4 ce n tre de c o u rb u re , 110
lo i de la ré fra c tio n et. 1 46-14 8
Lotis ) c h a m p de v is io n des, 110
io n g u e u i d ’ on d e e t in d ic e de
de H iib b lc 4 0 3 co n c a v e . 110
ré fra c tio n e t. 148-149
<le la p h y s iq u e , 211 c o n v e x e , 110
p e llic u le s m in c e s et, 1 5 8 -1 6 4
de la ré fra c tio n , 9 3 , 1 4 6-14 8 g ra n d is s e m e n t tra n s v e rs a l des
v e rs io n s de, 2 5 6 -2 5 8
de S n e ll-D e s c a rte s , 9 3 , 1 4 6-14 8 im ages p ro d u it p a r les, 112, 114
p h o to n s et, 2 4 6 -2 6 7
Longueur plans, 1 08-10 9
p re s s io n de ra d ia tio n e t, 9 0 -9 2
p ro p re , 2 2 0 o b je ts étendus dans les, 109
p ro g re s s iv e
re la tiv ité de la , 220-221 ré s o lu tio n de p ro b lè m e s avec, 120
aspect q u a lita tif do la, 8 2 85
L o n g u e u r d 'o n d e sph é riq u e s, 110-111
aspect q u a n tita tif de la , 8 5 -8 8
de De Broglie, 2 5 8 c ré e r des, 110
g é n é re r de la, 8 2 -8 3
de C o m p to n . 2 5 5 é q u a tio n des, 135
vites.se d e la , 8 4 , 85
de s e u il, 2 5 1 , 318 fo y e r des, 110 1 1 1
q u a n tu m de, 2 4 6 -2 6 7
cl fréquence, 28 30 im a g e s p ro d u ite s p a r les, 1 1 1 -1 1 4
ra y o n n e m e n t de la , 82
et in d ic e de r r f i a c tio n , 148-15 0 tableau des c a ra c té ris tiq u e s
ré fle x io n de la, 93-94
et n o m b re d 'o n d e , 29 des im a g e s des. 113
ré fle x io n in te rn e de la , 9 7 -9 8
propre. 2 2 8 M ode
ré fra i'tiu n de la , 9 3 -9 4
l o a 'n l/ . H A , 2 2 2 d ’ o s c illa tio n , 45
th é o rie s u r la , 146
L u m iè re , 8 1 -8 2 fo n d a m e n ta l, 4 6 , 65
tra n s p o rt de l'é n e rg ie p a r la. 88
angle v e c te u r de P o y n tin g , 8 8 -9 0 M o d è le
d r lé fic x in n e t, 9 3 -‘14 c o lle c tif
v ite sse de la , 8 4 , 85, 21 1-212
lie ic ir a c tiu n e t, 9 3 -9 4 L u n e tte a s tro n o m iq u e , 1 33-13 4 du n o y a u a to m iq u e , 351
d ’ in c id e n c e e t, 9 3 -9 4 c< rn tb in é
cha m p s é lc c u iq u e et m a g n é tiq u e et n o y a u a to m iq u e , 353
lie s, 83 M de B o h r, 2 9 0 -2 9 2
c o h é re n te , 1 5 4 -ISS M a c h o s (m a ssive c o m p a ct h a lo en couches, et n o y a u a to m iq u e ,
d ’ un Ia.ser, 3 2 1 -3 2 2 o bjects), 4 05 3 5 1 -3 5 2
Index 1-5

M o d u le d e c o m p re s s ib ilité , 5 5 p ié g é s , 3 1 3 -3 1 5 N u c lé o n s . 3 3 5
M om ent re m p li, 3 1 4 n o m b re s m a g iq u e s d e , 3 5 2
c in é tiq u e v id e , 3 1 4 N u m é ro a to m iq u e , 3 0 8 , 3 1 8 - 3 2 1 ,3 3 6
d e s a to m e s , 31)4, 3 0 6 -3 0 8 n u c lé a ire , 3 3 9
in trin s è q u e , 3 0 5 -3 0 6 N iv e a u s o n o re
n u c lé a ire , 3 3 9 A 62 O
o rb ita l, 3 0 6 in te n s ité d u , 6 1 -6 3 O b je c tif, 132
tlip tila ire m a g n é tiq u e N o m b re (s ) O b je ts é te n d u s
e ffe c tif, 3 0 8 b a ry o n iq u e , c o n s e rv a tio n d u , 3 9 4 d a n s u n m iro ir p la n , 109
o rb ita l, 3 0 6 -3 0 8 d ’o n d e , 2 6 2 lo c a lis e r le s, e n tra ç a n t les ra y o n s
n u c lé a ire , 3 3 9 de M ach , 73 p rin c ip a u x , 120-121
d e s p in , 3 0 7 -3 0 8 d e n e u tro n , 3 3 5 O c u la ire , 132
M o s e le y , g ra p h iq u e d e , 3 2 0 -3 2 1 h a rm o n iq u e , 4 6 . 6 5 -6 6 Πil, 123-1.30
M o se le y , H e n ry G . J ., 3 1 9 -3 2 0 le p to n iq u e , 3 9 3 e m m é tro p e , 125
M ouvem ent c o n s e rv a tio n d u , 3 9 3 -3 9 4 Ü n d e (s)
c irc u la ire u n ifo rm e m a g iq u e s d e n u c lé o n s , 3 5 2 a m p litu d e d ’u n e , 2 9
m o u v e m e n t h a rm o n iq u e s im p le e t, q u a n tiq u e (s ), 2 7 6 d a n s u n e c o rd e te n d u e , 3 3 3 7
1 3 -1 7 d e l ’a to m e d ’h y d ro g è n e , 2 9 3 d e la lu m iè re . 1 4 6 -1 4 7
h a rm o n iq u e s im p le . 2 -4 d e s p in , 3 0 5 d e m a tiè re , 2 5 8 -2 6 1 , 2 7 5 . Voir a u ssi
a e c é lé ra tio n e t, 4 m a g n é tiq u e , 2 9 3 O n d e s d e m a tiè re
a m o rti, 1 5 -1 6 o rb ita l. 2 9 3 d e p ro b a b ilité , 2 5 6 2 5 8
a n g u la ire , 9 p rin c ip a l, 2 9 3 d é fin itio n . 2.57
d é fin itio n d u , 3 N o rm a lis a tio n , é q u a tio n d e , 281 d iffé re n c e d e p h a s e e n tre le s, 3 8
é n e rg ie d a n s le , 7 -8 N o y a u (x ) a to m iq u e (s ), 3 3 3 é le c tro m a g n é tiq u e s . 2 7 , 8 1 -8 2 . Voir
a u ssi O n d e s é le c tro m a g n é tiq u e s
fo rc e e t, 5 -6 c o m p o s é , 351
fré q u e n c e a n g u la ire d u , 3 d a ta tio n p a r ra d io a c tiv ité e t, 3 4 9 e t p a rtic u le s , 2 7
lin é a ire , 5 d é g a g e m e n t d ’é n e rg ie p a r le s, 3 6 3 . fré q u e n c e d e s , 2 8 -3 0
m o u v e m e n t c irc u la ire u n ifo rm e e t. Voir a u ssi F issio n n u c lé a ire ,
in te rfé re n c e d e s , 3 8 -4 0 , 146
3 4 4 -M 5 lo n g itu d in a le s . 2 7 -2 8
13-15
d é s in té g ra tio n a lp h a d ’u n , .344-345 lo n g u e u r d ’, 2 8 -3 0
p e n d u le s e t. 10-1 3
d é s in té g ra tio n b ê ta d ’u n , 3 4 6 -3 4 8 m a g n é tiq u e , 8 1 . Voir a u ssi O n d e s
phase du, 3
d é s in té g ra tio n ra d io a c tiv e d e s, é le c tro m a g n é tiq u e s
re c o n n a ître u n , ré s o lu tio n m é c a n iq u e s , 2 7
d e p ro b lè m e e t, 7 3 4 0 -3 4 4
n e tte , 3 8
s im p le , 3 d o s e lie ra y o n n e m e n t, m e s u re r la, p a rtic u le s e t. 2 6 , 26f)-261
v ite s s e e t, 4 350 p h a se d ’u n e , 2 9
p é rio d iq u e , 2 é n e rg ie d e lia is o n e t, 3 3 8 -3 3 9 p la n e , 8 3
ra d ia l, 2 2 7 fis s io n d e s , 3 3 8 -3 3 9 . Voir a u ssi p ro g re s s iv e s . 2 7 2 8
M uons, 218, 386 F is s io n n u c lé a ire o n d e dan.s u n e c o rd c tendue e t,
M y o p ie . 1 2 5 -1 2 7 fo rc e d e s . 3 3 9 3 3 -3 7
fu s io n d e s , 3 3 9 vites.se tle.s, .30-31
m a g n é tis m e d u , 311 - 3 12 , 3 3 9 q u a n tific a tio n d e s . 2 7 5
N m asses des, 338 ra d io . 81
Is a n o c rts ta llite s , 2 7 5 m ix lè lc s d e , 3 5 1 -3 5 3 ré s u lta n te s . 3 8
N a v ig a tio n d e s c h a u v e s -s o u ris , n iv e a u d ’é n e rg ie d e s . 3 3 9 s in u s o ïd a le s , 2 7 -3 3
e t e ffe t d e D o p p le r, 7 2 p ro p rié té s d e s , 3 3 5 -3 4 0 .siiu iso ïilalcs d a n s u n e c o rd e
N A V S T A R , s y s tè m e d e n a v ig a tio n , ra y o n s d e s , 3 3 7 é n e rg ie c in é tiq u e d ’u n e . 3 6
229 s p in d e s . 3 3 9 é n e rg ie p o te n tie lle é la s tiq u e e t, 3 6
N e ’e m a n , Y u v a l, 3 % N u c lé id e (s ). Voir tatssi in te rfé re n c e d e s , 3 8 4 0
N é o d y m e , is o to p e s d e . 3 7 3 R a d io n u c lé id e fs ) o n d e s s ta tio n n a ire s e t, 4 2 -4 4
N éon. 316 à h a lo . 3 3 7 ré fle x io n d e s . à u n e e x tré m ité , 44
N e u trin o , 3 4 6 , 3 4 7 c la s s e m e n t d e s , 3 3 5 -3 3 6 tra n s m is s io n d e l’é n e rg ie d e s ,
é le c tro n , 3 9 3 iso b a re s d e s . 3 3 6 ta u x d e , 3 6 -3 7
m u on, 393 iso to p e s d e s , 3 3 5 tra n s p o rt d e l ’é n e rg ie e t, 3 6
N ish i jim a, K ., 3 9 6 p ro p rié té s d e s , 3 3 5 v e c te u rs d e F re s n e l e t, 4 3 -4 2
N iv e a u d ’é n e rg ie ta b le a u d e s . 3 3 6 v ite s s e d e s , 3 3 -3 5
d e s é le c tro n s . 2 7 6 -2 7 9 ra d io a c tiv ité e t, 3 4 7 -3 4 8 s o n o re s , 54
p a rtie lle m e n t o c c u p é , 3 1 4 tra n s u ra n ie n s , 3 7 0 a m p litu d e île d é p la c e m e n t d e s . 58
1-6 Index

a m p litu d e d e p re s s io n d e s , 5 8 O s c illa tio n s . 2 . Voir a u ssi O n d e (s ) e x p é rie n c e d e s fe n te s d o u b le s e t.


b a tte m e n ts e t, 6 7 -68 c e n tre d ’, 12 2 5 6 -2 5 8
e ffe t D o p p le r e t, 6 9 - 7 1 c y c le d ’, 2 v irtu e ls , 4(X)
in te n sité d e s, 61 -6 3 fo rc é e s , 17, 18 P h y s iq u e
in te rfé re n c e d e s , 5 9 -6 0 fré q u e n c e d e s , 2 c la s s iq u e v e rs u s m o d e rn e , 3 8 6
o n d e d e c h o c e t, 73 m o u v e m e n t h a rm o n iq u e s im p le , o n d e s d e m a tiè re e t, 2 7 5 . Voir a u ssi
p ro g re s s iv e s . 5 7 -5 8 3 4 7 -3 4 9 . Voir a i m i M o u v e m e n t, O n d e s d e m a tiè re q u a n tiq u e , 2 4 6
v ite s s e d e s , 5 4 -5 5 h a rm o n iq u e sim p le o n d e s d e m a tiè re e i. 2 7 5 . Voir
v ite s s e s s u p e rs o n iq u e s e t, 7 3 p é rio d e d ’, 2 a u ssi O n d e s d e m a tiè re s tru c tu re
s ta tio n n a ire s . 4 2 - 4 6 .6 5 - 6 7 ré s o n a n c e e t, 17-18 d e l ’a to m e e t. 2 7 5
p ro fils d ’o n d e s , e t tu y a u x , t b s tru c tu re d e l’a to m e e t. 2 7 5
ré s o n a n c e e t, 4 4 -4 6 P iè g e s à é le c tro n s
ré fle x io n d ’u n e , à u n e e x tré m ité , a rtif ic ie ls, 2 8 5 -2 8 6
P
44 à u n e d im e n s io n , 2 7 6 -2 8 6 , 3 1 3
s u [ x i|x is itio n d e s , 3 7 -3 8 P a ra m è tre b id im e n sio n n e ls e t trid im e n sio n n e ls,
tra n s v e rs a le s . 2 7 - 2 8 ,5 4 , 8 3 d e d is to rs io n , 3 6 7 2 8 6 -2 8 8
( y jx s < r, 2 6 d e v ite s s e , 2 1 7 n iv e a u tl’é n e rg ie e t, 3 1 4
v e c te u rs d e F re sn e l e t, 4 3 -4 2 P a rtic u le s , 3 8 6 -3 8 9 re c ta n g u la ire s , p lu s ie u rs é le c tro n s
v ite s s e d ’u n e . 3 3 -3 5 in te ra c tio n d e s , 4 0 0 -4 0 4 . Voir a u ssi
d a n s tie s, 3 1 3
Forcefs)
O n d e s d e m a tiè re , 2 5 8 -2 6 1 , 2 7 5 P ié z o c lc c tric ité , 2 6 6
a to m e s d ’h y d ro g è n e e t, 2 8 8 -2 9 7 . in te ra c tio n c ic c tro fa ib ic d e s , 401 P io n . 3 8 6 . 3 8 9 . 3 9 5
Voir a u ssi A to m e (s ), d ’h y d ro g è n e
in te ra c tio n é le c tro m a g n é tiq u e d e s , n e u tre , 2 1 2
électrons e t, 2 5 8 -2 6 1
400 404 P la n s
d c n s ilc d e p ro b a b ilité e t, 2 8 0 -2 8 1 je u d e , 411 (p ro b lè m e 4 2 ) c ris ta llin s , 194
é n e rg ie s q u a n tifié e s e t, 2 7 6 -2 7 9 lib re s, 2 6 2 , 2 7 5 ré flé c h is s a n ts , 194
fonctions d ’o n d e d e s . 2 8 0 -2 8 3
m a tiè re so m b re , 4 0 4 -4 0 5 P la s m a , 3 7 8
o n d e s d a n s le s c o rd e s e t, 2 7 5 m e s s a g è re s , 4(X )-404 P o in ts q u a n tiq u e s , 2 8 6
p iè g e s à é le c tro n s e t o n d e s e t, 2 7 . 2 6 0 -2 6 1 P o s itro n (s ), 3 8 9
a ilifie ie ls , 2 8 5 -2 8 6 ra y o n n e m e n t c o s m iq u e fo s s ile , 4 0 4 P o s tu la t, d e la re la tiv ité . 211
à u n e d im e n s io n , 2 7 6 -2 8 6 th é o rie d u B ig B a n g , 4 0 5 -4 0 6 P o u v o ir
b id im e n s io n n e ls e t tn d im e n P a tro n d e ré s o lu tio n , 134, 191
s io n n e ls . 2 8 6 -2 8 8 d e d iffra c tio n , 175. Voir a u ssi d e s ré s e a u x d e d iffra c tio n . 191
O n d e s é le c tro m a g n é tiq u e s . 2 7 , R é s e a u x d e d iffra c tio n d istin c tio n a v e c le p o u v o ir d isp e isif,
81 -8 2 . Voir a u ssi R a y o n s X d ’in te rfé re n c e . 151 1 9 2 -1 9 3
a n g le d in c id c n e c e t, 9 3 9 4 P a u li, p rin c ip e d ’e x c lu s io n d e, 3 1 3 d is p e rs if, 191
a n g le d e ré fle x io n e t. 9 3 -9 4 fe rm io n , b o s o n e t. 3 8 7 -3 8 8 d is tin c tio n a v e c le p o u v o ir d e
a n g le d e ic ira e tio n e t, 9 3 -9 4 P a u li. W o lfg a n g , 3 1 3 ré s o lu tio n , 1 9 2 -1 9 3
i h a m p s é le c triq u e e t m a g n é tiq u e P e n d u le . 1 0 -1 3 P o y n tin g , 8 8 -9 0
des. 83 a c c é lé ra tio n e n c h u te lib re m e s u ré e P re m ie r h a rm o n iq u e . 6 5
d isiie i'sio n e h ro m a iiq iie e t, 9 4 -9 6 p a r u n , 12 P re sb y tie , 1 2 8 -1 2 9
in te n s ité d e s, 8 0 c o m p o s é . 1 1 -1 2 P re s s io n d e ra d ia tio n , d 'o n d e s
p re s s io n d e ra d ia tio n d e s , 9 0 91 d e to rs io n . 9 é le c tro m a g n é tiq u e s , 90-91
p ro g re s s iv e s sim p le , 10-11 P rin c ip e
a.spect q u a lita tif d e s, 8 2 -8 5 P e n z ia s , A m o , 4 0 4 d e B a b in e t. 2 0 3 (e x e rc ic e 12)
a s p e c t q u a n tita tif d e s , 8 5 -8 8 P é rio d e d e c o n fin e m e n t, 2 7 5
g c n c r c r d c s , 8 2 -8 3 d e s é lé m e n ts . 3 0 4 , 3 1 6 -3 1 7 d e c o rre s p o n d a n c e , 281
v ite s s e d e s , 8 4 , 85 d ’o .se illa tio n . 2 d e H u y g e n s , 146
ra y o n n e m e n t d e s , 8 2 d ’u n e o n d e , 30 d ’e x c lu s io n d e P a u li. 3 1 3
ré fle x i( in d e s , 9 3 -9 4 d u m o u v e m e n t o s c illa to ire , 2 d 'in c e r titu d e d e H e is e n b e rg .
ré fle x io n in te rn e e t. 9 7 -9 8 P e rl, M a rtin , .393 2 6 3 -2 6 5
Infraction d e s , 9 3 -9 4 P h a se P rism e , d isp e rsio n c h n im a tiq u e p ar, 95
spectre d '. 81 8 2 d u m o u v e m e n t h ai m o n iq u e . 3 P ro fils d ’o n d e , 2 7 -2 8
transfiort d e l ’énergie e t. 8 8 -9 0 d ’u n e o n d e . 2 9 P ro fo n d e u r d u p u its, 2 8 3 -2 8 5
^ 'c c té u r d e P o v n tin g e t, 8 8 -9 0 P h o to é le c tro n s , 2 5 1 P ro p rié té s c h im iq u e s , d e s é lé m e n ts ,
U p liq u e g é o m é triq u e , 9 3 -9 4 P h o to m u ltip lic a te u r. 2 5 8 303'
O s e illa tc u r P h o to ii(s ). 2 3 5 -2 3 6 P ro to n (s)
h a rm o n iq u e s im p le lin é a ire , 5 e ffe t C o m p to n e t, 2 6 4 -2 6 7 ré s o n a n c e m a g n é tiq u e d e s, 3 1 1 -3 1 2
L C . 8 2 -8 3 é ta t d ’é n e rg ie d e s é le c tro n s c l, 2 7 8 P ro to n -p ro to n (p -p ), c h a în e . 3 7 6 -3 7 8
Index 17

P u is s a n c e s p e c tie d e s , c a ra c té ris tiq u e s , p o u v o ir d e ré s o lu tio n d e s, 191


d ’a c c o m o d a tio n , 125 3 1 8 -3 1 9 p o u v o ir d is p e rs if d e s , 191
d ’u n e le n tille , 124 J
R é a c te u r R é s o lu tio n , 1 8 3 -1 8 4
P u is s a n c e m o y e n n e , d e la tra n s m is ­ à e a u so u s p re s s io n , 3 7 0 m ic ro s c o p e é le c tro n iq u e e t, 184
s io n d ’é n e rg ie d e s o n d e s , 38 n u c lé a ire , 3 6 8 -3 7 2 p o in tillis m e e t, 1 8 3 -1 8 4
P u its d ’é n e rg ie p o te n tie lle n a tu re l, 3 7 2 -3 7 4 R é s o lu tio n d e p ro b lè m e s
l'iiii, 2 8 3 -2 8 5 R é a c tio n a v e c a n g le s d e p h a s e s , 5
in fin im e n t p ro fo n d , 2 7 6 -2 8 2 c h im iq u e , 231 a v e c h a rm o n iq u e s d a n s u n e COTde, 4 6
P u n c tu m d e fu s io n , 2 3 3 a v e c m iro irs e t le n tille s , 120
p ro x im u m , 124 en c h a în e . 3 6 8 a v e c m o u v e m e n t h a rm o n iq u e
re m o tu m , 124 n u c lé a ire , 231 s im p le , 7
R é fé re n tie l(s ) a v e c m o u v e m e n t h a rm o n iq u e
d ’in e rtie , 2 1 0 s im p le a n g u la ire , 9
p ro p re , 2 1 8 R é s o n a n c e , 1 7 -1 8
Q u a n tific a tio n e t c o n fin e m e n t re la tiv ité e t, 2 1 0 -2 1 1 d é fin itio n , 4 5
d es ondes, 275 R é fle x io n m a g n é tiq u e , 3 1 1 -3 1 2
Q u a n tité d e m o u v e m e n t d ’o n d e s é le c tro m a g n é tiq u e s , 9 3 -9 4 R ic h te r, B u rto n , 4 0 0
é n e rg ie c in é tiq u e e t, 2 3 4 d u ra y o n n e m e n t, 9 0 R o h ie r, H e in ric h , 2 6 6
e n re la tiv ité , 2 3 0 -2 3 1 g ra n d is s e m e n t tra n s v e rs a l p ro d u it R o u g e c o s m iq u e , d é p la c e m e n t v e rs
Q u a n tu m . 2 4 6 p a r u n m iro ir e t. 1 1 2 , 114 le . 411 (p ro b lè m e 4 3 )
d e lu m iè re , 2 4 6 -2 4 7 im a g e fo rm é e p a r la. 107 R u b b ia , C a rlo . 401
Q u a rk (s ), 3 9 8 in te rn e R u b in . V era, 4 0 4
c h a rm é , to p et b o tto m , 3 9 9 to ta le , 9 7 -9 8 R u th e rfo rd , E rn e s t, 3 3 3
e t b a ry o n s , 3 9 8 -3 9 9 lo i d e la , 9 3
e t d é s in té g ra tio n b ê ta , 3 9 9 m iro irs p la n s . 1 0 8 -1 0 9
et m ésons, 399 m iro irs .sp h é riq u e s, 110 114, .S
S a m io s. N ic h o la s , 3 9 7
in te ra c tio n e n tre le s . 401 -402 1 3 5 -1 3 7 S e m i-c o n d u c te u r
Q u a rtz c ris ta llin , 2 6 6 to ta le in te rn e , 97 -98 c o m m e p iè g e à é le c tro n . 2 8 5 -2 8 6
QuasiUT), 4 0 2 R é fra c tio n S é rie
d e s o n d e s é le c tro m a g n é tiq u e s , d e B a lm e r, 2 8 9
9 3 -9 4 h a rm o n iq u e , 4 6
R im a g e s fo rm é e s p ar, 1 1 4 -1 1 6 S h rix lin g e r. E rw in , 2 6 2
R a d ia l, m o u v e m e n t, 2 2 7 é q u a tio n d e s le n tille s m in c e s , S ie v e rt. 3 5 0
R a d io a c tiv ité , e t ta b le a u d e s 1 3 6 -1 3 7 S ig m a , 3 8 6 , 3 9 5
n u c lé id e s , 3 4 7 -3 4 8 é q u a tio n s d e s d io p tre s s p h é riq u e s S n e ll-D e s c a rtc s , loi d e , 9 3
R a d io n u c lé id e (s ), 3 3 5 e t, 1 3 5 -1 3 6 S o n s m u s ic a u x , 6 5 6 7
a c tiv ité d e s , 3 4 2 g ra n d is s e m e n t la té ra l e t, 114 b a tte m e n ts e t. 6 7 -6 8
d e m i-v ie d e s . 3 4 2 le n tille s e t, 1 1 6 -1 2 2 p ro fils d ’o n d e s s ta tio n n a ire s e t. 6 6
d é s in té g ra tio n d e s , 3 3 5 in d ic e d e , 9 4 S o u s -c o u c h e , île s a to m e s . 2 9 6 ,
e x c è s d e m a s s e d e s , 3 4 7 -3 4 8 lo i d e la , 9 3 3 1 6 -3 1 7
R a ie s R e la tiv ité fe rm é e , 3 1 6
d’émission. 190 d e la lo n g u e u r. 2 2 0 -2 2 1 . 224 S p e c tre
d e F ra u n h o fe r, 331 ^ p ro b lè m e 7 0 ) d e la s im u lta n é ité , 2 1 3 2 1 5 , 2 2 4 d e lu m iè re b la n c h e . 9 5
R a y le ig h , c ritè re d e , 183 d u te m p s . 2 15 -2 2 0 d e ré s o n a n c e m a g n é tiq u e . 3 12
R a y o n d e B o h r, 2 9 3 e ffe t d e D o p p le r s u r le s o n d e s é le c tro m a g n é tiq u e , 8 1
R ayonnem ent lu m in e u s e s e t. 2 2 7 -2 3 0 ra y o n s X , 3 1 7 -3 2 1
a b s o rp tio n d e . 9 0 é n e rg ie e n . 2 3 1-2 3 6 S p e c tn rs c o p e à ré s e a u . 18 9
c o s m iq u e fo s s ile , 4 0 4 m e s u re d ’u n é v é n e m e n t e t, 2 1 2 - 2 13 S p in
d 'o n d e s é le c tro m a g n é tiq u e s , 8 2 -8 4 p o s tu la ts d e la , 2 1 1 -2 1 2 d e l’é le c tro n , 3 0 4 -3 0 5 . Voir a u ssi
ré fle x io n d ’u n , 9 0 q u a n tité d e m o u v e m e n t e n , 2 3 0 -2 3 1 É le c tr o n ( s ) , M o m e n t, c in é tiq u e ,
R a y o n s . 54 th é o rie d e la . re.strein te. 2 1 0 in trin sè q u e
R ayons X tra n s fo n n a tio n d e L o re n tz e t, d e p a rtic u le s , 3 8 7 -3 8 8
d it Fraction d e s , I9 3 -J 9 5 2 2 2 -2 2 5 n u c lé a ire . 3 3 9
d is p e rs é s . 194 v ite s s e e t, 2 2 6 S te rn e t (ie rla c 'h . e x p é rie n c e d e ,
e t n u m é ro ta tio n d e s é lé m e n ts , R E M (u n ité ). 3 5 0 .309-311
3 1 7 321 R é s e a u x d e d iffra c tio n , 1 8 7 -0 )0 S te rn , O tto , 3 0 9
s p e c tre c o n tin u d e , 3 1 7 -3 1 8 a p p lic a tio n d e s , 1 8 9 -1 9 0 S tra s s m a n , F r it/. 3 6 4
1-8 Index

S tru c tu re d e l ’a to m e , 2 3 5 V oir a u ssi U d ’u n e p a rtic u le d é c riv a n t u n


A U )n ie(s), d ’h y d ro g è n e ; O n d e s d e U h lc n b e c k , G e o rg e , 3 0 6 m o u v e m e n t h a rm o n iq u e s im p le , 4
m a tiè re U n iv e rs tra n s fo rm a tio n re la tiv is te d e s, 2 2 6
S u p e rp o s itio n e s tim a tio n d e l ’â g e d e l ’, 4 0 3 V o ie o c tu p le , 3 9 6 -3 9 7
o n tle s e t p rin c ip e d e , 37-3X e x p a n s io n d e l ’, 4 0 3 V o n L a n e , M a x , 193
S y s tè m e fo rm a tio n d e f , 4 0 3
d e c o o rd o n n é e s
e s p a c e te m p s , 2 1 2 -2 1 3 W
o p tiq u e , 107 W eakly in tera ctin g m a ssiv e p a rtic le s
V a le n c e , é le c tro n d e . 3 1 6 (W IM P ). 4 0 5
V a le u r e ffic a c e , d u c h a m p é le c triq u e , W h e e le r, J o h n , 3 6 6
T 89 W ils o n , R o b e rt, 4 0 4
T a b le a u V e c te u r W IM P { W eakly in tera ctin g m a ssive
d e s n u c ic id e s , 3 3 6 d e F re s n e l. 4 3 -4 2 p a rtic le s), 405
n id io a c tiv itc e t. 3 47-34H d e P o y n tin g , 8 8 -9 0
p é n o d iq u e , 3 1 6 -3 1 7 V ite sse (s)
T a iio n (s). 3 9 3 d e la lu m iè re , p o s tu la t d e la , 2 1 2
T iiylfir, n I,. 2 5 7 te st. 2 1 2 Y o u n g . T o m a s, 1 5 0 -1 5 1
T em ps d ’o n d e s , 3 0 31
re la tiv ité d u , 2 1 5 -2 1 8 . Voir a u ssi a n a ly s e d im e n s io n n e lle d e la,
Dilatation du temps 3 3 - .34
l'h o m s o n , ( ic o rg e F.. 2 5 8 d a n s u n e c o rd e te n d u e , 3 3 -3 5 Z w e ig , G e o rg e , 3 9 8
T in g , .S am u el, 4 0 0 d é riv a tio n d e la, à l ’a id e d e la
T n lc tm a k , 3 7 0 d e u x iè m e lo i d e N e w to n e t,
Transistor, 321 34- 35
T ra n s itio n q u a n tiq u e . 2 7 7 lim ite , 211
T ra n s m is s io n , c o e ffic ie n t d e , 2 6 6
Troisième harmonique, 6 5

Vous aimerez peut-être aussi